Download as pdf or txt
Download as pdf or txt
You are on page 1of 1568

CFA Institute

CFA-LEVEL-I
Exam Name:
CFA Level I Chartered Financial Analyst

Product: Full
Prepare with PDF and examine your preparation with our multiple testing
modes practice test software.

Download Free Practice Test Demo Here:

https://www.testcollections.com/CFA-LEVEL-I.html
Question: 1

According to the AIMR-PPS, ________ are defined to include all discretionary and nondiscretionary
assets.

A. individually managed assets


B. individual investor's assets
C. total firm assets
D. global assets

Answer: C

Explanation:
Total firm assets are defined to include all discretionary and nondiscretionary assets. Total firm assets
does not refer to assets underlying overlay investment strategies, such as currency overlay, options
and futures overlays, securities lending programs and asset allocation overlay strategies, unless the
firm actually manages the underlying assets.

Question: 2

Standard IV (B.7) deals with ________.

A. Priority of Transactions
B. None of these answers
C. Disclosure of Referral Fees
D. Performance Presentation
E. Prohibition against Misrepresentation
F. Disclosure of Conflicts to Clients and Prospects
G. Preservation of Confidentiality
H. Prohibition against Use of Material Nonpublic Information

Answer: F

Explanation:
Standard IV (B.7) states that members shall disclose to their clients all matters that could become
potential conflicts. These include beneficial ownership of securities or otherinvestments, that
reasonably could be expected to impair the member's ability to make unbiased and objective
recommendations.

Question: 3

Standard III (D) deals with ________.

A. Professional Misconduct
B. Use of Professional Designation
C. Plagiarism
D. Fundamental Responsibilities
E. None of these answers
F. Obligation to Inform Employer of Code and Standards

Page | 2
Answer: E

Explanation:
Standard I deals with Fundamental Responsibilities. Standard II (A) deals with Use of Professional
Designation. Standard II (B) deals with Professional Misconduct. Standard II (C) deals with Plagiarism.
Standard III (A) deals with the Obligation to Inform Employer of Codes and Standards. Standard III (D)
deals with the Disclosure of Additional Compensation Arrangements.

Question: 4

Another name for "access" person is ________.

A. none of these answers


B. supervisor
C. ombudsman
D. guardian
E. fiduciary
F. covered person

Answer: F

Explanation:
Access or covered persons have knowledge of pending or actual investment recommendations or
action. The firm's definition of access (covered) person should be broad enough to cover all people
with that knowledge.

Question: 5

Which of the following is/are true about the Performance Presentation Standards?

I. A member who complies with the mandatory requirements of the PPS but does notfollow the
recommended requirements can publicly claim compliance with the PPS.
II. The PPS are designed to be primarily a performance measurement framework.
III. The Standards are not designed to enhance or detract from the presentation ofhistorical results.

A. III only
B. II and III only
C. I, II and III
D. I and III only

Answer: D

Explanation:
The PPS are designed to be primarily a performance presentation system, not a performance
measurement system. The Standards are not designed to enhance or detract from the usefulness of
the information in historical results, though it does restrict the way they are to be presented. The PPS
are voluntary standards and are not necessarily binding on AIMR members. Some standards are
mandatory while others are recommended. To claim compliance, members must abide by the
mandatory requirements, at the very least.

Question: 6

If a firm uses non-discretionary leverage, it must present performance using:

Page | 3
A. both actual returns and all-cash basis.
B. all-cash basis i.e. removing leverage effects.
C. actual returns.
D. none of these answers.

Answer: B

Explanation:
According to Section B of the PPS standards - "Calculation of Returns" - for non-discretionary
leverage, performance must be presented on an all-cash returns basis.

Question: 7

________ and other hybrid securities must be treated consistently across and within composites.

A. Portfolios
B. Convertibles
C. Assets
D. Bonds

Answer: B

Explanation:
This is one of the requirements which is mandatory in order to be in compliance with the PPS.

Question: 8

Standard IV (B.8), Disclosure of Referral Fees, includes ________.

A. referral fees paid in cash


B. referral fees paid "in kind"
C. soft dollar referral fees
D. all of these answers

Answer: D

Explanation:
Under Standard IV (B.8), appropriate disclosure involves disclosing the nature of the consideration or
benefit given or received for the recommending of services. Consideration includes all fees, whether
paid in cash, in soft dollars, or in kind.

Question: 9

Which of the following relating to compliance procedures for complying with Standard III (E) is false?
The compliance procedures should:

A. none of these answers.


B. outline permissible conduct.
C. delineate procedures for reporting violations and sanctions.
D. designate a team of outside colleagues to form a review board.
E. outline the scope of the procedures.
F. be easy to understand.

Answer: D

Page | 4
Explanation:
The compliance officer should be designated from within the firm.

Question: 10

According to the AIMR-PPS for venture and private placements, ________ internal rate of return
must be presented since inception of the fund and be net of fees, expenses and carry to the limited
partner.

A. limited
B. extended
C. cumulative
D. general

Answer: C

Explanation:
Cumulative internal rate of return must be presented since inception of the fund and be net of fees,
expenses and carry to the limited partner. Irr must be calculated based on cash-on-cash returns plus
residual value.

Question: 11

Which of the following AIMR Standards states that referral fees must be disclosed in writing to clients
or customers?

A. V
B. VI (A)
C. IV (B.8)
D. IV

Answer: C

Explanation:
Standard IV (B.8) - Disclosure of Referral Fees states: "Members shall disclose to clients and prospects
any consideration or benefit received by the member or delivered to others for the recommendation
of any services to the client or prospect."

Question: 12

Omega Prime Securities is a sizable investment bank that undertakes security issuances on behalf of
small and medium-size businesses. Treffil Ellis is the senior vice president of corporate finance at
Omega. Treffil, on one of his golf junkets, made acquaintance with Tralth Trevor, owner of a growing
chain of resort hotels. Tralth invites Treffil to his estate mansion the next day and over drinks, asks
him how fast Omega could issue equity-linked callable notes to finance the $200 million construction
of new hotel businesses in Cairo and Bali. He forthrightly tells him that Omega could receive as much
as 150 basis points above the normal fee if the issuance could be completed within the month. Treffil
knows that this is not enough time to complete a research on Tralth's business and determine the
issue price. However, he does know that his research wing could quickly do a comparison with one of
the other hotel chains and determine an approximate issue price. He instructs his department to do
so. In a month, the public offering is ready for issuance and Omega ends up making almost $15
million more than on other similar business deals. Treffil receives commendation from the CEO for
"going beyond the call of duty for his employer." Treffil has

Page | 5
I. violated Standard IV (B.3) - Fair Dealing.
II. not violated any code of ethics.
III. violated Standard IV (A.1) - Reasonable Basis & Representations.
IV. violated Standard IV (B.1) - Fiduciary Duties.

A. III and IV only


B. I and III only
C. III only
D. II only

Answer: C

Explanation:
If adequate research is not put into determining the fair price of a security issue, investors could end
up paying a price that has no relevance to the intrinsic value of the security. This is in direct violation
of Standard IV (A.1) - Reasonable Basis & Representations.

Question: 13

Which of the following statements clearly conflicts with the recommended procedures for
compliance presented in AIMR's Standards of Practice Handbook?

A. Investment recommendations may be changed by an analyst without prior approval of a


supervisory analyst.
B. Prior approval must be obtained for the personal investment transactions of all employees.
C. For confidentiality reasons, personal transactions should not be compared with those of clients or
the employer unless requested by regulatory organizations.
D. Personal transactions should be defined as including transactions in securities owned by the
employee and member of his or her immediate family and transactions involving securities in which
the employee has beneficial interest.

Answer: C

Explanation:
This question deals with the compliance procedures relating to personal investments of members.
Employees should compare personal transactions of employees with those of clients on a regular
basis, regardless of the existence of a regulatory organization. Such comparisons ensure that
members' personal trades do not conflict with their duty to their clients. All the other statements do
not conflict with the procedures in the Handbook.

Question: 14

Trust Fund is a reasonably successful investment management firm that has as its clients a few
pension plans. Trust Fund executes all of its trades with Prime Brokerage, an average brokerage firm.
Prime Brokerage charges higher commissions than comparable players in the market but in return,
provides investment research on the stocks which are part of the pension plan assets under Trust
Fund's management. Portfolio managers at Trust Fund know about the close relationship on the golf
links between Prime Brokerage's chief broker, Ralph Fiennes, and Trust Fund's CEO, Armis Arvanitis.
They also believe that the research provided by Prime Brokerage, while not superlative, is quite
useful and justifies the excess expense in brokerage. This "soft-dollars" practice is disclosed in Trust
Fund's official documents and contracts but Sisko, a freshly minted CFA charterholder, thinks that
Trust Fund managers are in violation of the AIMR code of Ethics. Which of the following is true?

Page | 6
A. Trust Fund's managers are violating Standard IV (B.8) - Disclosure of Referral Fees by not revealing
the arrangement to pension plan beneficiaries.
B. Trust Fund's managers are violating Standard IV (B.1) - Fiduciary Duties by not executing the trades
at the lowest price available.
C. Sisko is not applying the AIMR code correctly. Trust Fund's managers are not violating any AIMR
standards.
D. Trust Fund's managers are violating Standard IV (B.3) - Fair Dealing by unfairly diverting funds from
the plan assets to Prime Brokerage through higher fees.

Answer: C

Explanation:
The practice of using "soft dollars" (i.e., the usage of brokerage for purchase of research services) is
not forbidden by the AIMR code or securities laws, as long as they are commensurate with the
services received and the practice is disclosed to the clients. In this case, there is no evidence that
Trust Fund is overpaying Prime Brokerage or that it is not seeking best price and execution. Hence,
Sisko is mistaken and Trust Fund managers are not in violation of the AIMR code.

Question: 15

A financial analyst should conduct himself with ________, competence and dignity and act in an
ethical manner in his dealings with the public, clients, customers, employers, employees and fellow
analysts.

A. integrity
B. morality
C. none of these answers
D. honor
E. principalThat answer is correct!

Answer: A

Explanation:
The Code of Ethics: "Members of AIMR shall act with integrity, competence, dignity and in an ethical
manner when dealing with the public, clients, prospects, employers, employees and fellow
members."

Question: 16

Everly Smith has passed Level II of the CFA examination. While studying for Level III, he circulates his
resume stating that he has "completed Charter Financial Analyst II" and his resume lists his name as
"Everly Smith, CFA II." Which of these following statements is correct?

A. Everly Smith is not in compliance with the standards regarding use of the CFA designation. He may
state he "completed Chartered Financial Analyst II," but cannot put "CFA II" after his name.
B. Everly Smith is not in compliance with the standards regarding use of the CFA designation. He can
neither state he "completed Chartered Financial Analyst II" nor "Everly Smith, CFA II."
C. Everly Smith is not in compliance with the standards regarding use of the CFA designation. Since
he is registered for Level III he should state "Everly Smith, CFA III."
D. Everly Smith is in compliance with the standards regarding use of the CFA designation.

Answer: B

Explanation:

Page | 7
Everly Smith has violated Standard II (A), Use of Professional Designation. There is no designation for
someone who has passed Level I, Level II, or Level III, so he may not state "Everly Smith, CFA II." He
also can not state he has "completed Charter Financial Analyst II." However, he may state that he is a
Level III candidate in the CFA Program. If he passed Level II, but was not registered for Level III he
could state "I passed Level II of the CFA Program in the year 19--."

Question: 17

For ________ of five or fewer portfolios, the disclosure "five or fewer portfolios" may be made rather
than a disclosure of the exact number of portfolios.

A. returns
B. investments
C. segments
D. composites

Answer: D

Explanation:
This is an exception allowed by AIMR when disclosing the list of composites.

Question: 18

Standard I deals with ________.

A. Use of Professional Designation


B. Duty to Employer
C. Obligation to Inform Employer of Code and Standards
D. Professional Misconduct
E. Plagiarism
F. Fundamental Responsibilities
G. Disclosure of Conflicts to Employer
H. None of these answers

Answer: F

Explanation:
Standard I deals with Fundamental Responsibilities. Standard II (A) deals with Use of Professional
Designation. Standard II (B) deals with Professional Misconduct. Standard II (C) deals with Plagiarism.
Standard III (A) deals with the Obligation to Inform Employer of Codes and Standards. Standard III (B)
deals with the Duty to Employer. Standard III (C) deals with Disclosure of Conflicts to Employer.

Question: 19

According to the AIMR-PPS, terminated portfolios are included in the composite for how long after
termination?

A. Composites must include terminated portfolios after the last full performance measurement
period the portfolios were under management.
B. Composites should include the terminated portfolio only until the date of termination.
C. Composites should include the terminated portfolio for the last full performance measurement
period under which the portfolios were managed.
D. Composites should include the terminated portfolio for the full ten years required by the
Standards for performance reporting.

Page | 8
Answer: C

Explanation:
Composites must exclude terminated portfolios after the last full performance measurement period
the portfolios were under management, but composites must continue to include terminated
portfolios for all periods prior to termination. This is a requirement for creation and maintenance of
composites.

Question: 20

Edward Witten works for Princeton Investments and has registered to take Level III exam next year.
He had taken the Level III exam 3 years ago but was not successful. In his firm's promotional
material, he has stated that he is a candidate in the CFA program and has successfully passed Level II.
Edward has:

A. violated Standard II (A) - Use of Professional Designation. He cannot claim to be in the CFA
program without having completed Level III exam.
B. violated Standard II (A) - Use of Professional Designation. He cannot claim to be in the CFA
program since he failed the Level III exam.
C. not violated any standards.
D. violated Standard II (A) - Use of Professional Designation. He cannot claim to be CFA - Level II.

Answer: C

Explanation:
Standard II (A) - Use of Professional Designation, states that a person must be registered to take the
next higher level of the exam to be a "candidate" in the CFA program. That Edward failed on his last
attempt does not prevent him from claiming that he's a candidate, as long as he has registered for
the exam.

Question: 21

Standard V (A) deals with ________.

A. Prohibition against Use of Material Nonpublic Information


B. Disclosure of Conflicts to Clients and Prospects
C. Prohibition against Misrepresentation
D. Disclosure of Referral Fees
E. Performance Presentation
F. Priority of Transactions
G. Preservation of ConfidentialityThat answer is correct!

Answer: A

Explanation:
Standard V (A) prohibits members who possess material nonpublic information related to the value
of a security from trading in that security if such trading would breach a duty or if the information
was misappropriated or relates to a tender offer.

Question: 22

Andy Pilling is a bond trading specialist who recently started a special fund, the "Structured Bond
Fund." The strategy behind this fund is quite complex, involving a mix of highly speculative, high-

Page | 9
yield bonds and various tax-free municipal bonds for some stability. Andy has a strong view that the
economy will remain vibrant and bullish over the next two years and hence, is not worried about the
risky bonds. Assuming a falling rate scenario in this case allows the fund to project an expected
return 130 basis points above the S&P 500 return. In his special report, Pilling does not disclose such
assumptions nor does he reveal any details about the bond strategy. He does analyze the state of the
economy and the future outlook in the report. Based on his reputation and his association with some
big name academics, Pilling is able to obtain capital of close to 75 million dollars on this fund alone.
Andy has:

A. not violated any standards in the AIMR Code of Ethics.


B. violated Standard IV (B.6) - Prohibition Against Misrepresentation.
C. violated Standard IV (A.2) - Research Reports - by not revealing the assumptions and details about
the strategy.
D. violated Standard IV (B.1) - Fiduciary Duties - by not disclosing the nature of the strategy.

Answer: C

Explanation:
Standard IV (A.2) - Research Reports requires members to describe the basic characteristics of an
investment, the degree of risks involved and scenario analysis to illustrate possible losses under
different market conditions. By suppressing such relevant details, Pilling has violated the AIMR Code
of Ethics.

Question: 23

According to Standard IV (A.2), members should consider including the following information in
research reports, except:

A. the methodology that drove the investment decisions.


B. yield-to-maturity.
C. annual amount of income expected.
D. degree of uncertainty associated with the cash flows.
E. business, financial, political, sovereign and market risks.
F. none of these answers.
G. degree of marketability / liquidity.
H. expected annual rate of return.That answer is correct!

Answer: A

Explanation:
All the information has to be included in the research reports except the methodology that drove the
investment decision, which is part of the 'maintaining files' compliance procedure for Standard IV
(A.1).

Question: 24

The disclosures for retroactive compliance apply to composites formulated prior to ________.

A. January, 1989
B. January, 1992
C. January, 1991
D. January, 1993
E. January, 1990

Page | 10
Answer: D

Explanation:
The effect date to be in compliance with AIMR-PPS was January 1, 1993. Any composites which
predate the effective date can be brought into compliance retroactively.

Question: 25

With regard to real estate, all properties must be included in at least one ________.

A. account
B. investment type
C. composite
D. sector
E. portfolios

Answer: C

Explanation:
Consistent with the general requirements for all composites, all properties with discretionary fee-
paying investors must be included in at least one account. Because of the unique nature of individual
real estate investments, however, composites containing single properties are appropriate in many
cases.

Question: 26

Each ________ needs to comprise portfolios or asset classes that represent a similar investment
goal.

A. return
B. composite
C. value
D. benchmark
E. mix

Answer: B

Explanation:
All actual fee-paying discretionary portfolios must be included in at least one composite defined
according to similar strategy or investment objective.

Question: 27

ERISA fiduciaries must adhere to the following prudent procedures:

- establish a written investment policy for the plan


- diversify plan assets
- make investment decisions with the skill and care of a ________
- monitor investment performance
- control investment expenses
- avoid prohibited transactions

A. member
B. supervisor

Page | 11
C. prudent expert
D. none of these answers
E. trader

Answer: C

Explanation:
These procedures are stipulated under the detailing of ERISA fiduciary duties, to ensure that the
fiduciary complies with the duty to act with prudence. Under ERISA, the fiduciary is held to a higher
standard than the Prudent Man Rule. The ERISA fiduciary needs to be as prudent as the average
expert, not simply as prudent as the average person.

Question: 28

________ establishes the fiduciary principles for U.S. corporate pension plans.

A. AIMR
B. PSPC
C. ERISA
D. WTO
E. UMIFA
F. None of these answers
G. RELAC

Answer: C

Explanation:
ERISA establishes the fiduciary principles for U.S. corporate pension plans. ERISA is very specific in its
definition of fiduciaries and their responsibilities and investment managers of U.S. corporate pension
plans are subject to ERISA's fiduciary provisions.

Question: 29

Under what conditions is Standard II (B)- Professional Misconduct violated?

I. The member is convicted of a misdemeanor.


II. The member regularly engages in unprofessional but legal behavior.
III. The member engages in dishonest activities that do not result in criminal conviction. IV. The
member is convicted of a felony.

A. II, III & IV.


B. I, II & IV.
C. I, II, III & IV.
D. IV only.That answer is correct!

Answer: A

Explanation:
This question pertains to Standard II (B) - Professional Misconduct. A misdemeanor conviction is only
considered grounds for a violation of II (B) if it occurs repeatedly, or involves "moral turpitude" (lying,
cheating, stealing, or other dishonest conduct). Unprofessional and dishonest behavior or a felony
conviction are violations of Standard II (B).

Question: 30

Page | 12
Complete the following: According to The Code of Ethics, members of AIMR shall: "Act with
________, competence, dignity and in an ethical manner when dealing with the public, clients,
prospects, employers, employees and fellow members."

A. virtue
B. none of these answers
C. honorability
D. integrity
E. morality

Answer: D

Explanation:
According to The Code of Ethics, members of AIMR shall: "Act with integrity, competence, dignity
and in an ethical manner when dealing with the public, clients, prospects, employers, employees and
fellow members."

Question: 31

Standard IV of the Standards of Professional Conduct deals with Relationships with and
Responsibilities to ________.

A. None of these answers


B. Clients and Prospects
C. AIMR
D. Supervisors
E. the Employer

Answer: B

Explanation:
Standard IV of the Standards of Professional Conduct deals with Relationships with and
Responsibilities to Clients and Prospects.

Question: 32

Each of the following is true regarding Standard II (A), except:

A. You must be registered for the next CFA exam in order to call yourself a candidate.
B. This standard relates to business cards and letterheads.
C. This standard does not relate to oral statements.
D. There is no designation for someone who has passed Level I, II, or III.
E. All of these answers.
F. Candidates may state that they have completed Level I, II, or III.

Answer: C

Explanation:
Standard II (A) relates to the responsibility of AIMR members and candidates to use their
professional designation properly and in a non-misleading manner. A person must be registered to
take the next scheduled CFA exam to be a "candidate" in the CFA program. There is no designation
for someone who has passed Level I, II, or III of the CFA exam. Candidates may state, however, that
they have completed Level I, II, or III. The standard applies to all related explanations or descriptions

Page | 13
of the CFA designation, including letterheads and business cards, resumes, directory listings, printed
advertising, brochures and oral statements to clients and prospects.

Question: 33

Standard ________ pertains to fair dealing with customers and clients.

A. III (B)
B. IV (B.3)
C. IV (A)
D. I (D)
E. None of these answers

Answer: B

Explanation:
Standard IV (B.3) states: "Members shall deal fairly and objectively with all clients and prospects
when disseminating investment recommendations, disseminating material changes in prior
investment recommendations and taking investment action."

Question: 34

Social investments:

A. should never be used in pension fund investing.


B. in pensions must be well thought-out, making sure that such investments are legal and do not
impair the integrity of the funds in questions or the financial security of the participants or
beneficiaries.
C. none of these answers.
D. have yet to be used as an investment in pension funds in the U.S.
E. are proper investment vehicles for pensions, since they are a "public good."

Answer: B

Explanation:
In a pension plan, the first and foremost duty of the fiduciary is to the plan participants and their
beneficiaries rather than to the plan sponsor that has the power to hire and fire the investment
manager. Consequently, if urged to make investments that might be of direct benefit to a sponsoring
community or to the community at large, the manager must ensure that such investments are legal
and do not impair the integrity of the funds in question or the financial security of the
participants/beneficiaries.

Question: 35

Which of the following are considered basic characteristics of a security and must be included in
research reports?

A. business risk
B. degree of uncertainty
C. annual expected income
D. yield-to-maturity
E. all of these answers
F. both degree of liquidity and yield-to-maturity
G. expected annual rate of return

Page | 14
Answer: E

Explanation:
Members should include the following information in research reports:

- expected annual rate of return


- annual amount of income expected (current and future)
- current rate of income return of yield to maturity
- degree of uncertainty associated with cash flows
- degree of marketability/liquidity
- business, financial, political, sovereign and market risks

Question: 36

Fundamental Responsibilities is dealt with under:

A. Standard I
B. Standard III
C. None of these answers
D. Standard V
E. Standard II
F. Standard IVThat answer is correct!

Answer: A

Explanation:
Fundamental Responsibilities is dealt with under Standard I.

Question: 37

Corporate directors are governed by the ________. Trustees are governed by the ________.

A. none of these answers


B. "common sense doctrine"; business judgment rule
C. business judgment rule; Prudent Man Rule
D. business judgment rule; "common sense doctrine"
E. "common sense doctrine"; Prudent Man Rule
F. Prudent Man Rule; business judgment rule
G. Prudent Man Rule; "common sense doctrine"

Answer: C

Explanation:
This question is trying to highlight the different standards of business care fiduciaries must uphold in
charitable organizations and trusts respectively. Thus, fiduciaries at charities are bound by the
business judgment rule, since they are comparable to directors of business corporations. In contrast,
trustees are governed by the Prudent Man Rule. This distinction in the standard duties applicable to
fiduciaries in different situations should be recognized.

Question: 38

Maria Golino is a financially savvy client of Hector Gomez, a portfolio manager with a small
investment firm. Maria recently directed Hector to execute all trades on her behalf with Omega

Page | 15
Brokerage. Omega charges higher commissions than most other brokerage firms but in this case, has
agreed to provide research to Hector on behalf of Maria. Hector does not object to this and starts
directing Maria's trades to Omega. Hector has

A. violated Standard IV (B.1) - Fiduciary Duties.


B. violated Standard IV (A.2) - Research Reports.
C. violated Standard IV (B.8) - Disclosure of Referral Fees.
D. not violated any standards.

Answer: D

Explanation:
The practice in which a portfolio manager directs trades to a particular brokerage in exchange for
additional goods or services at the behest of the client is known as "directed brokerage." This
practice is not in violation of any AIMR standards since the portfolio manager is acting on the
instructions of his client and the benefits accrue to the client. Standard IV (B.1) - Fiduciary Duties -
and the Topical Study "Fiduciary Duty."

Question: 39

Another name for "covered" person is ________.

A. guardian
B. none of these answers
C. ombudsman
D. access person
E. fiduciary
F. supervisor

Answer: D

Explanation:
Access or covered persons have knowledge of pending or actual investment recommendations or
action. The firm's definition of access (covered) person should be broad enough to cover all people
with that knowledge.

Question: 40

Which of the following can be found in Standard IV?

A. Members must use the CFA designation in a dignified manner.


B. Members shall make reasonable and diligent efforts to avoid any material misrepresentation in
any research report or investment recommendation.
C. Members shall comply with any prohibitions on activities imposed by their employer if a conflict of
interest exists.
D. Members shall maintain knowledge of AIMR's Code of Ethics.
E. If members receive material nonpublic information in confidence, they shall not breach that
confidence by trading or causing others to trade in securities to which such information relates.

Answer: B

Explanation:
Standard IV states: "Members shall make reasonable and diligent efforts to avoid any material
misrepresentation in any research report or investment recommendation."

Page | 16
Question: 41

If a firm uses discretionary leverage, it must present performance using:

A. all-cash basis i.e. removing leverage effects.


B. both actual returns and all-cash basis.
C. none of these answers.
D. actual returns.

Answer: B

Explanation:
For discretionary leverage, both actual returns and all-cash returns must be presented.

Question: 42

Kruskal Meriwether is a senior research analyst with Bellwether Advisors. He has been following
Crystals & Candles a publicly traded firm which makes high-quality diamond jewelry. Kruskal, after
extensive interviews with senior management at Crystals, has inferred that the firm is about to take
over a diamond- mining firm in South Africa at a rock-bottom price. The Crystal management has
refused to explicitly confirm or deny this but Kruskal firmly believes that such a deal is in the works.
He has not used any inside information; just pieced together information from various avenues to
come to this conclusion. In his reports, he states, "All my research seems to indicate that Crystal &
Candles is likely to buy a South African diamond producer at a bargain price. Clearly, now is the time
to buy Crystal and Candles' stock." 2 weeks after his report is released, Crystal's management
announces that it has no intentions of making any acquisitions in the near future. This leads to a 7%
decline in Crystal's stock, causing a large decline in the account's of Kruskal's clients. Kruskal has

A. violated Standard IV (A.1) - Reasonable Basis & Representation.


B. not violated any AIMR code in this incident.
C. violated Standard IV (B.2) - Portfolio Investment Recommendations and Actions.
D. violated Standard IV (B.1) - Fiduciary Duties.

Answer: B

Explanation:
Kruskal's recommendations were not based on whim, unsubstantiated rumors or inside information.
It is clear that he put in much research behind his recommendation. The fact that his
recommendation turned out to an incorrect choice ex post does not mean he was negligent. An
investment advisor cannot be expected to be correct 100% of the time. What is expected of them is
professional competence and diligence (Code of Ethics). Nothing in this incident indicates that
Kruskal lacked either of these.

Question: 43

Standard II (C) deals with ________.

A. None of these answers


B. Obligation to Inform Employer of Code and Standards
C. Disclosure of Conflicts to Employer
D. Duty to Employer
E. Fundamental Responsibilities
F. Plagiarism

Page | 17
G. Use of Professional Designation
H. Professional Misconduct

Answer: F

Explanation:
Standard I deals with Fundamental Responsibilities. Standard II (A) deals with Use of Professional
Designation. Standard II (B) deals with Professional Misconduct. Standard II (C) deals with Plagiarism.
Standard III (A) deals with the Obligation to Inform Employer of Codes and Standards. Standard III (B)
deals with the Duty to Employer. Standard III (C) deals with Disclosure of Conflicts to Employer.

Question: 44

Which of the following AIMR Standards states that client transactions must have priority over
transactions in which the analyst is a beneficial owner?

A. V
B. None of these answers
C. VII
D. VI

Answer: B

Explanation:
It is Standard IV (B.4) which states that client transactions must have priority over transactions in
which the analyst is a beneficial owner so that such personal transactions do not operate adversely
to their client's or employer's interests.

Question: 45

Cariella is a junior analyst who is currently preparing a report on a diamond producing firm, Dense
Carbon, Inc. Dense Carbon recently announced that the results of a mining survey in its South African
diamond mines were in, which revealed substantial amounts of diamond reserves for the first time.
It has offered to take a few industry analysts for a tour of the facilities and take stock of the situation
first hand. During this tour, all expenses, including air-fare and basic accommodations, were provided
for by Dense Carbon. Since the visit spanned a weekend, Dense Carbon also arranged for a Safari
tour for all the analysts. Cariella did not consider the safari to be an undue entertainment, given the
fact that the analysts had to be in the middle of nowhere for 5 days. She was quite assiduous in her
appraisal of the mining reserves and in the final analysis, the tour proved extremely valuable to her
analysis. However, she did not reveal the fact about the Safari trip to her employer. Cariella has

I. violated Standard III (C) - Disclosure of Conflicts to Employer.


II. violated Standard IV (A.1) - Reasonable Basis and Representations.
III. violated Standard IV (A.3) - Independence and Objectivity.
IV. not violated the AIMR code of ethics.

A. I only
B. I and III only
C. I, II and III only
D. IV only

Answer: B

Explanation:

Page | 18
Standard IV (A.3) - Independence and Objectivity requires members to use reasonable care and
judgment while making investment recommendations. In particular, it requires that members avoid
even appearances of conflict of interest or circumstances that could affect their independence or
objectivity. While Dense Carbons travel arrangements for the analysts might not be considered an
unnecessary "gift" (this is a gray area), the safari definitely is an unacceptable arrangement from the
AIMR code of ethics' perspective. By accepting this gift, Cariella violated Standard IV (A.3). By not
disclosing this fact to her employer, she violated Standard III (C) - Disclosure of Conflicts to Employer.

Question: 46

Arbaaz, an AIMR member, works for an investment advisory firm, Leon Investments. His friend,
Shahzad, recently asked him for some investment recommendations. Arbaaz analyzed Shahzad's
portfolio over a weekend and suggested some changes. While he did not accept any remuneration,
Shahzad promised him some gifts if his portfolio "performed well." Arbaaz did not inform his
employer since he thought he was helping a friend and in any case, the Shahzad's account was
extremely small and there were no financial payments. Arbaaz has:

A. violated Standard III (B), Duty to Employer.


B. has not violated Standard III (B), Duty to Employer, because there was no financial remuneration.
C. has not violated Standard III (B), Duty to Employer, because Shahzad's account was too small to be
deemed lost business for Leon Investments.
D. has not violated Standard III (B), Duty to Employer, because Arbaaz is free to do what he wants on
his time as long as it doesn't affect Leon Investments.That answer is correct!

Answer: A

Explanation:
While Arbaaz did not receive any monetary compensation, there is a possibility of gifts in the future.
Standard III (B) applies even when there is no actual receipt of compensation; what is important is
the possibility of future payments in cash or kind. By not obtaining a written permission from his
employer before advising Shahzad, Arbaaz violated Standard III (B).

Question: 47

"Restricted Periods" are discussed in Standard IV (B.4), Priority of Transactions. Another name for
restricted periods is ________ periods.

A. blackout
B. restrained
C. none of these answers
D. captiveThat answer is correct!

Answer: A

Explanation:
Firm procedures should prevent managers or employees from initiating trades in a security for which
their firms have a pending buy or sell order within a 24-hour period. Firms must determine specific
requirements relating to such blackout, or restricted periods.

Question: 48

Liz Hurley is an investment advisor who has recently started advising a client, Zeta, regarding
investment decisions. Zeta lives in Imphal, where investment laws are quite lax, almost non-existent.
Hurley is domiciled in Britania, where investment laws clearly specify that the laws governing finance

Page | 19
professionals in any given case are the laws that govern their clients. Britania laws, in general, are far
stricter than the AIMR code of ethics. In her dealings with Zeta, Hurley must follow

A. Imphal's laws.
B. Britania's laws.
C. AIMR's code of ethics.
D. a combination of Britania's laws and AIMR code which results in a stricter set of laws.

Answer: C

Explanation:
In her dealings with Zeta, Hurley is governed by Imphal's laws, as specified by Britania's laws. Since
Imphal laws are almost non-existent, the code of ethics sets a stricter standard and hence, as an
AIMR member, Hurley must follow the AIMR code. Standard I.

Question: 49

In order to inform your employer that as a member of AIMR, you must abide by the code of ethics,
you must:

A. inform the legal department in writing.


B. inform senior management in writing.
C. inform your immediate supervisor in writing or by email.
D. inform your supervisor in writing, by email or orally.

Answer: C

Explanation:
Standard III (A)

Question: 50

Spassky was assigned the task of managing the portfolio of Fisher three days ago when Anand, who
was managing Fisher's portfolio, retired. Fisher's portfolio consists of some deep-in-the-money put
options, which will be exercised today, resulting in a cash flow of about $40,000. Spassky has not yet
had a chance to meet Fisher in person to determine his needs, investment objectives and risk
appetite. He did get a briefing from Anand about the portfolio and has a general idea about Fisher's
investment attitude. In fact, over the past two years, Fisher's portfolio has generated handsome
returns due to high-risk investments which Fisher prefers. Spassky's problem is determining what he
should do with the $40,000. According to the AIMR Code of Ethics, he should:
A. keep the money in cash form and not risk it till he can meet Fisher to discuss the situation.
B. "roll over" the put positions for another week or two till he can meet Fisher and discuss the
reinvestment of the funds.
C. invest the funds in a diversified portfolio with a risk profile similar to what Anand and Fisher have
been maintaining over the past 3 months.
D. invest the funds in highly liquid, cash equivalent assets till he can meet Fisher and determine his
needs, investment objectives and risk appetite.

Answer: D

Explanation:
In most cases, a portfolio manager must manage a portfolio based on the investment needs and
objectives of the portfolio owner consistent with the willingness to bear risk. One exception to this
rule is when a new portfolio manager takes over and has the task of reinvesting funds arising from

Page | 20
theexisting portfolio investments. Since these funds should not be kept idle, a prompt investment of
the money in liquid, risk-free securities is prescribed by the AIMR code of Ethics.

Question: 51

Willier is the research analyst responsible for following Company X. All the information he has
accumulated and documented suggest that the outlook for the firm's new products is poor, so the
stock should be rated a weak hold. During lunch, however, Willier overhears a financial analyst from
another firm offer opinions that conflict with Willier's forecasts and expectations. Upon returning to
his office, Willier releases a strong buy recommendation to the public. Willier:

A. violated the Standards by failing to distinguish between facts and opinions in his recommendation.
B. was in full compliance with the Standards.
C. violated the Standards because he did not have a reasonable and adequate basis for his
recommendation.
D. violated the Standards because he did not seek approval of the change from his firm's compliance
department.

Answer: C

Explanation:
The question deals with Standard IV (A.1), Reasonable Basis and Representations. Willier's actions in
changing the recommendation based on the opinion of another financial analyst is not an adequate
basis for the recommendation. This question does not illustrate a violation of the need to distinguish
between facts and opinions. Seeking approval from the firm for a change in a recommendation is a
matter of policy set by the firm, not by the Standards.

Question: 52

Under AIMR Rules of Procedure for the Proceeding Related to Professional Conduct, membership in
AIMR and/or the right to hold and to use the CFA designation may be summarily suspended by
AIMR's Designated Officer for the following misconduct:

I. Conviction for a crime that is defined as a felony or its equivalent.


II. Indefinite bar from registration under the securities laws (even though reapplication may be made
after a specific period of time).
III. Failure to complete and return a professional conduct statement for each of two successive years.

A. I only.
B. I and II only.
C. IV only.
D. II only.
E. III only.
F. II and III only.
G. I, II and III.

Answer: G

Explanation:
This question relates to AIMR's summary suspension powers as defined in the Bylaws under the
Professional Conduct Program. Statements I, II and III describe grounds for summary suspension of
membership in AIMR and/or the right to hold and use the CFA designation.

Question: 53

Page | 21
Grey recommends the purchase of a mutual fund that invests solely in long-term U.S. Treasury
bonds. He makes the following statements to his clients:

I. "The payment of the bond is guaranteed by the U.S. government; therefore, the default risk of the
bonds is virtually zero."
II. "If you invest in the mutual fund, you will earn a 15 percent rate of return each year for the next
several years." Did Grey's statements violated AIMR's Code and Standards?

A. Neither statement violated the Code and Standards.


B. Statement I and II violated the code and Standards.
C. Only statement I violated the Code and Standards.
D. Only statement II violated the Code and Standards.

Answer: D

Explanation:
This question deals with Standard IV (B.6), Prohibition against Misrepresentation. Statement I is a
factual statement that discloses to clients and prospects accurate information about the terms of the
investment instrument. Statement II, which guarantees a specific rate of return, is an opinion stated
as a fact and therefore violates Standard IV (B.6).

Question: 54

Standard III (D) is ________.

A. None of these answers


B. Disclosure of Additional Compensation Arrangements
C. Disclosure of Conflicts to Employer
D. Obligation to Inform Employer of Code and Standards
E. Duty to Employer
F. Responsibilities of Supervisors

Answer: B

Explanation:
Standard III (A) deals with the Obligation to Inform Employer of Code and Standards. Standard III (B)
deals with the Duty to Employer. Standard III (C) deals with Disclosure of Conflicts to Employer.
Standard III (D) deals with Disclosure of Additional Compensation Arrangements. Standard III (E)
deals with Responsibilities of Supervisors.

Question: 55

Level I verification requires independent attestation that the requirements of the AIMR-PPS have
been met on a(n) ________ basis.

A. international
B. attainable
C. nationwide
D. firmwide

Answer: D

Explanation:

Page | 22
This is a requirement under Level I verification procedures.

Question: 56

Carmina Aburana is a sales assistant to Drew Door, a sales manager at Hicost Brokerage. Hicost has a
policy of requiring at least 20% margin on stocks that are deemed illiquid or extremely risky. For
these purposes, it creates and updates a list of such stocks on a weekly basis. Yoddly Yoo, Inc. is an up
and coming internet firm whose stock has been on this list for some time now. One of Carmina's
"blue chip" clients, Amadeus, has been speculating on Yoddly's stock for the past two weeks,
repeatedly going in and out of the market. In this process, he has unfortunately generated significant
losses and his margin on the account has fallen to 12%. To make up for the shortfall, Amadeus calls
up Carmina and requests a "borrowing on the account" of 10% for the next 2 weeks, promising to pay
a hefty interest rate of 38%on an annualized basis. Since Amadeus has never been in default,
Carmina agrees to the arrangement and moves some funds from another client's account. There is
no explicit rule at Hicost that prohibits such an arrangement, though it is clearly an oversight on part
of the Compliance department. Drew notices this transaction and calls Carmina for an explanation.
On hearing the explanation, he tells Carmina that such arrangements are in violation of the company
rules and should not be repeated. After 2 weeks, Amadeus supplies the necessary margin for his
account.

A. Drew has violated Standard II (B) - Professional Misconduct.


B. Carmina has not violated any AIMR code but Drew has violated Standard III (E) - Responsibilities of
Supervisors.
C. None of these answers, since the infraction was too minor and inconsequential.
D. Carmina has violated Standard III (B) - Duty to Employer.

Answer: B

Explanation:
Carmina has shown poor judgment in not recognizing the spirit behind Hicost's 20% margin
requirement. However, in the absence of any explicit rule in this about fund transfers, she can make
a case that she is not in violation of any laws or code of conduct. Drew, on the other hand, has been
negligent in his duties. If he understands the import of the margin rule, then he must take steps to
unwind the effects of the infraction as soon as possible. In this case, he should have directed Carmina
to terminate the arrangement with Amadeus and require that the requisite margin be posted
immediately. Further, verbal admonitions are not enough to satisfy supervisory duties when
handling a supposed violation of rules. Drew must determine how best to prevent such occurrences
in the future and also discuss with the Compliance Department the need for revising the employee
rules handbook.

Question: 57

Susan Jackson, with HRS Investments, is appearing in court as an expert witness. She will have to use
research done at HRS, to which she did not contribute directly, during her testimony. Which of the
following is true, in relation to Jackson's need to comply with Standard II (C)?

A. Jackson is representing herself and may or may not attribute any of HRS's research.
B. Jackson is representing HRS and must attribute any of HRS's research.
C. Jackson is representing herself and must attribute any of HRS's research.
D. There is not enough information given to answer this question.
E. Jackson is representing HRS and may or may not attribute any of HRS's research.

Answer: C

Page | 23
Explanation:
Expert witnesses represent themselves, not organizations. Thus, Susan must attribute any of HRS's
research to HRS and the team that conducted the research.

Question: 58

Which of the following AIMR Standards states that additional compensation agreements must be
disclosed to the employer?

A. IV
B. III (D)
C. V
D. VI (B)

Answer: B

Explanation:
Standard III (D) - Disclosure of Additional Compensation Arrangements states: "Members shall
disclose to their employer in writing all monetary compensation or other benefits that they receive
for their services that are in addition to compensation or benefits conferred by a member's
employer."

Question: 59

When an analyst reaches conclusions about a firm's impending announcements before the actual
release of information, using non-material, non-public information in conjunction with public
information, insider trading charges cannot be leveled against her. This arises from a legal defense
against insider trading charges known as:

A. the Shingles Theory.


B. the question is based on false premise. The analyst can be held responsible for insider trading if
she uses any inside information in her conclusions.
C. the Insider Legal Statute.
D. the Mosaic Theory.

Answer: D

Explanation:
Insider trading charges arise only when the inside information used is material and non-public.
Analysts who use non-material inside information in conjunction with publicly available data to draw
conclusions which happen to be the same as those that would be derived given material inside
information are shielded from insider trading charges under the "Mosaic Theory." The logic behind
the defense is that investment analysts are exposed to a wide variety of public information (i.e., a
"mosaic of information") and already have a good idea about what the information means. Further,
the inside information, when non-material, cannot (by definition) have been the primary shaper of
the final conclusions. Hence, the possession of the inside information makes only marginal
contribution to the conclusions and as such, should not trigger insider trading violations.

Question: 60

Smith, a research analyst with a brokerage firm, decides to change his recommendation on the
common stock of Green company, Inc., from a buy to a sell. He mails this change in investment
advice to all the firm's clients on Wednesday. The day after the mailing, a client calls with a buy for
500 shares of Green Company. In this circumstance, Smith should:

Page | 24
A. accept the order.
B. advise the customer of the change in recommendation before accepting the order.
C. not accept the order because it is contrary to the firm's recommendation.
D. not accept the order until five days have elapsed after the communication of the change in
recommendation.

Answer: B

Explanation:
Under Standard IV (B.3), investment recommendations must be disseminated to all clients at the
same time, so that all clients are treated fairly. The client's request is contrary to Smith's latest
recommendation which he has not yet received. Smith must now make sure that the client receives
the changed recommendation. If the client still wants to place a buy order for the shares, only then is
Smith obligated to comply with the client's directive.

Question: 61

Mirabelle is an experienced analyst who has worked for the investment research arm of Clifford &
Clifford, Inc. for the past 7 years. Recently, TransOmega retained Clifford & Clifford to conduct a study
on possible takeover candidates in the aviation industry. Mirabelle has been given the project and
two assistants to conduct the research. During their review, Mirabelle's assistants located a research
report created recently by Donaldson, a freelance analyst. Mirabelle found the report thorough,
though she did not agree with many of Donaldson's conclusions. She carried out further inquiry
along those lines and modified the report with new conclusions. She showed the completed report,
with proper attributions to Donaldson in places where she had used his results, to a senior partner,
John Cliff of Clifford & Clifford. The report was approved and released to TransOmega. In this case,

A. both Mirabelle and John Cliff violated the standards.


B. only Mirabelle violated the plagiarism standard by using Donaldson's report.
C. only John Cliff violated the Standard of Reasonable Care (IVA.1) by not adequately monitoring
Mirabelle.
D. neither Mirabelle nor John Cliff violated any of the standards.

Answer: D

Explanation:
Since proper care was taken at all steps, no violation has occurred. The use of Donaldson's report is
acceptable as long as proper acknowledgments are given.

Question: 62

According to the AIMR-PPS when presenting results, annual returns for all years must be presented.
Performance for periods of less than one year

A. must be treated as all of the other performance results.


B. must not be included in the presentation.
C. must not be annualized.
D. must be prorated and appropriate disclosures made.

Answer: C

Explanation:
Annual returns for all years must be presented. Performance for periods of less than one year must

Page | 25
not be annualized. This is a requirement for presentation of results.

Question: 63

________ accounting is mandatory for fixed-income securities.

A. Accrual
B. Cash
C. Flexible
D. Equal
E. Risk-free
F. TotalThat answer is correct!

Answer: A

Explanation:
Accrual accounting must be used for fixed-income securities and all other securities that accrue
income. Accrued income must be included in the market value calculation of the denominator and
numerator.

Question: 64

Joseph Silk is a veteran money manager with Aakanksha, Inc., a hedge fund that caters to high net-
worth individuals. His friend, Gribbin, recently incorporated a private business and invited Joseph to
be on its board of directors. Gribbin's business is in the paper industry and does not directly or
indirectly affect Aakanksha's client base. Joseph accepted the board membership with the
understanding that he would participate in Gribbin's business only over the weekends. He
considered this a private venture and did not inform Aakanksha's Compliance officer. In not doing so,
Joseph has

A. violated Standard III (C) - Disclosure of Conflicts to Employer.


B. not violated any standards since there is no conflicts of interest between his two professional
obligations.
C. violated Standard IV (A.3) - Independence and Objectivity.
D. violated Standard IV (B.1) - Fiduciary Duties.That answer is correct!

Answer: A

Explanation:
While Gribbin's business is not in any competition with Aakanksha, Joe's position as a director
creates an appearance of conflicts of interest since he will have a say in many decisions of the
business, including investments in securities. Hence, before accepting any such position, Joe must
discuss the situation with the compliance department at Aakanksha to evaluate the potential
conflicts of interest. While Joe is not precluded from being on an outside Board, not getting this
cleared with his current employer represents a violation of Standard III (C) - Disclosure of Conflicts to
Employer.

Question: 65

Which of the following is/are required by AIMR-PPS with regards to calculation of returns?

I. Total return - realized and unrealized gains plus income - should be used.
II. Returns must be based on arithmetic mean calculations.
III. Accrual accounting must be used for fixed-income securities.

Page | 26
A. I, II and III
B. I only
C. I and III only
D. II and III only

Answer: C

Explanation:
The PPS require that calculations be based on time-weighted (geometric) returns. Accrual accounting
must be used for fixed-income and other securities that accrue income. The performance
measurement must be based on total returns.

Question: 66

What is the effective date for compliance with the AIMR-PPS for U.S. and Canadian investments?

A. January 1, 1995
B. January 1, 1992
C. January 1, 1993
D. January 1, 1994

Answer: C

Explanation:
From January 1, 1993, going forward, all of the firm's actual discretionary fee-paying nontaxable
portfolios solely invested in U.S. and/or Canadian investments ("North American Portfolios") must be
presented in composites that adhere to the Standards.

Question: 67

Standard II (C) - Prohibition against Plagiarism - addresses all of the following forms of
communication, except:

A. internet communications
B. oral presentations
C. electronic data transfer
D. written presentations
E. audio/visual presentations
F. none of these answers
G. group meetings

Answer: F

Explanation:
Standard II (C) does not concern the form of communication of information to clients, prospects,
employees, or the general public. Rather, it concerns the need to acknowledge the source of
information used by the member to avoid the appearance of plagiarism.

Question: 68

________ and other hybrid securities must be treated consistently across and within composites.

A. Bonds

Page | 27
B. Assets
C. Convertibles
D. Portfolios

Answer: C

Explanation:
This is one of the requirements which is mandatory in order to be in compliance with the PPS.

Question: 69

Sterling Drachma is a senior investment consultant currently researching a few high-risk internet
stock companies which recently started trading on NASDAQ. Sterling manages 5 large and private
investment accounts for which he has discretionary investment authority. Sterling is about 3 years
away from retirement and his retirement portfolio is managed by Franc Escudo. Sterling has
concluded from his research that two of the internet stocks he has been following are great buys and
instructs Franc todivert part of the retirement investments into these stocks. Franc executes the
orders as soon as he receives them. Sterling then instructs his brokers to buy the stocks for the two
discretionary accounts that he knows are inclined toward high-risk investments. He does not buy any
for the remaining three accounts since those are income-oriented, low-risk accounts. In this
sequence of events, which of the following is/are true?

I. Franc has violated Standard IV (B.1) - Fiduciary Duties - by investing retirement account funds in the
high-risk stocks.
II. Sterling has violated Standard IV (B.3) - Fair Dealing - by not treating all his accounts equally.
III. Sterling has violated Standard IV (B.4) - Priority of Transactions - by trading for his retirement
account before trading for his client accounts.

A. I, II and III
B. I and III only
C. II and III only
D. III only

Answer: D

Explanation:
Franc is managing a personal portfolio and as such must execute the orders of his client. The fact that
it is a retirement account makes no difference in this situation. Franc would be in violation if he was
managing a pension portfolio or a personal trust portfolio and the investments were deemed in
violation of plan directives. Franc, however, should try to understand Sterling's motives in the
redeployment of funds since this could prevent his client from what could be reckless investment.
Sterling, for his part, as definitely violated Standard IV (B.4) - Priority of Transactions - by trading for
his retirement account before trading for his client accounts. Personal transactions should never take
precedence over client and employer transactions. He has, however, not violated Standard IV (B.3) -
Fair Dealing - by not treating all his accounts equally. Standard IV (B.3) requires a fair treatment of all
clients, not an equal treatment precisely because different accounts have different investment needs
and risk appetites. The internet stocks should only be bought for accounts for which they are a
suitable investment.

Question: 70

Jorgenson is a senior bond analyst with Morgan Co., a large investment banking firm. Over the past
quarter, Morgan's corporate bond department has been betting on the credit spreads in the market
narrowing and in anticipation, has invested a large amount of capital in the bonds of two firms, High

Page | 28
Tech and Amerizone.com. Unfortunately, the credit spreads have not displayed much activity and as
the quarter end is approaching, the department wants to unload the bonds. For this, it puts pressure
on Jorgenson to push the bonds on some of his larger clients. Jorgenson believes that both the bonds
are good investments since High tech and Amerizone have been doing very well and their prospects
look rosy. So he goes ahead and convinces his clients to purchase as much as a third of Morgan's
bond holdings in these companies. Morgan has

A. not violated the AIMR code of ethics.


B. violated Standard IV (A.1) - Reasonable Basis and Representations.
C. violated Standard IV (B.7) - Disclosure of conflicts to Clients and Prospects.
D. violated Standard IV (A.3) - Independence and Objectivity.That answer is correct!

Answer: A

Explanation:
There is no evidence that Jorgenson has bowed to any external pressure in recommending the bonds
to his clients. If his analysis indicates, in his judgment, that a security is a good investment and suits
the needs of a client, then he should recommend it, regardless of whether there is any external
pressure for or against that course of action.

Question: 71

Emmy Noether is a senior division manager at Harding & Harding, a money management firm. Emmy
is quite fastidious about following the rules of the investment industry and has established specific
procedures and guidelines designed to prevent any violations. Recently, it surfaced that one of
theemployees reporting to Emmy, William Bathwater, had been secretly using inside information on
a computer maker to generate profits in his portfolio. William had been extremely clever at hiding
these profits and only a serendipitous audit by the Compliance Department revealed the pattern.
Emmy, in her capacity as William's supervisor, has:

A. has not violated any standard in the AIMR code of ethics.


B. none of these answers.
C. violated Standard III (E) - Responsibilities of Supervisors - by failing to check William's behavior.
D. violated Standard III (B) - Duty to the Employer - by failing to check William's behavior.That answer
is correct!

Answer: A

Explanation:
Under Standard III (E) - Responsibilities of Supervisors, members must take reasonable care to ensure
that their subordinates do not violate any laws or the code of conduct. This includes designing
effective procedures to deter fraudulent activity. However, no amount of scrutiny can prevent the
cleverest of frauds. What is expected of the members is that they be diligent enough in carrying out
their duties. In the present case, given the facts, Emmy cannot be faulted for William's criminal
activity since she hasn't been found to be negligent in any way. Hence, she has not violated Standard
III (E) - Responsibilities of Supervisors nor any other standards.

Question: 72

NL is a country with no securities laws. LS is a country that has securities laws that are less strict than
the AIMR code of ethics while MS has securities laws that are stricter than the code of ethics. Which
of the following is/are true?

I. A member who lives in NL must always follow the AIMR code.

Page | 29
II. A member who lives in MS is governed by the AIMR code.
III. A member lives in NL but does business in MS. If MS laws apply to her business transactions, she
must follow the AIMR code.
IV. A member lives in LS and does business in NL. He must always follow the AIMR code.

A. II and IV only
B. I and IV only
C. I, II, III and IV
D. II and III only

Answer: B

Explanation:
An AIMR member must always adhere to the code of ethics, unless the laws governing his/her
behavior are stricter, in which case, the stricter laws must be followed. A member governed by MS
must always follow the MS laws since they are stricter. Standard I.

Question: 73

Because it is not the preferred method recommended by the PPS, the use of ________ valuation
needs to be disclosed.

A. new-date
B. settlement-date
C. trade-date
D. old-date
E. survivor-date

Answer: B

Explanation:
Trade-date valuation is recommended when calculating performance, although settlement-date
valuation is acceptable if disclosed.

Question: 74

A critical part of Standard IV (A.2) is to distinguish between:

A. research reports and investment memoranda.


B. insider trading and appropriate trading.
C. CFA charterholders and non-CFA charterholders.
D. none of these answers.
E. employees and independent contractors.
F. industry and company analysis.
G. the buy side and the sell side.
H. facts and opinions.

Answer: H

Explanation:
Standard IV (A.2) - Research Reports states the responsibility of AIMR members, CFA charterholders
and candidates to include in each research report those key facts that are instrumental to the
investment recommendation presented in the report. A critical part of this requirement is to
distinguish clearly between opinions and facts.

Page | 30
Question: 75

When a manager is responsible for the portfolios of pension plans or trusts, the duty of loyalty is
owed to the ________.

A. beneficiaries
B. none of these answers
C. stockholders of the firm
D. investing public
E. entity who hires the manager
F. corporation
G. board of directors
H. manager's supervisor(s)That answer is correct!

Answer: A

Explanation:
The first step in fulfilling a fiduciary duty is to determine what the responsibility is and to who it is
owed. Members should take particular care in determining the identity of the "client" to whom the
duty of loyalty is owed. In the context of an investment manager managing the portfolios of pension
plans or trusts, the client is not the person or entity who hires the manager but, rather, the
beneficiaries of the plan or trust. The duty of loyalty is owed to the beneficiaries.

Question: 76

Brokers who knowingly or recklessly engage in excessive trading in customers' accounts are known to
be ________.

A. over-selling
B. mixing
C. churning
D. none of these answers
E. shingling

Answer: C

Explanation:
Brokers and dealers are held to a higher standard of care than the average person. They are liable for
knowingly or recklessly engaging in excessive trading in customers' accounts, (churning); for
accepting funds when they are insolvent, for manipulating the market and for fraud under the
shingle theory.

Question: 77

According to the AIMR-PPS, systems incompatibilities

A. causes distortion of performance presentation and needs to be disclosed.


B. are one reason for which a firm may not claim compliance for all assets.
C. cannot be used as a reason for not claiming compliance for all assets.
D. render a firm unable to claim compliance with the PPS, thus the firm should ensure compatibility.

Answer: C

Page | 31
Explanation:
Systems incompatibilities cannot be used as a reason for not claiming compliance for all assets (i.e., a
firm cannot make the claim of compliance for only those assets that are measured and monitored on
compatible systems).

Question: 78

Argus is a large-accounts money manager with a high-profile hedge fund. He manages all his
accounts in a very efficient manner and all his clients are satisfied with his performance. Argus has
one character flaw, though. He is given to making sexually inappropriate comments in mixed
company, touching female employees in inappropriate ways and cracking adult jokes loudly. This
makes many of the other employees uncomfortable in his presence. Argus

A. none of these answers.


B. has violated AIMR standard on professional incompetence.
C. has violated AIMR standard on professional conduct.
D. has not violated any AIMR standard since he has kept his clients satisfied and the professional
misbehavior is an internal matter to be handled by his superiors.

Answer: C

Explanation:
Standard II (B) is designed to address professional behavior and personal integrity. The behavior
should not put the firm at risk in any way or reflect adversely on the profession. Argus's behavior
leaves his firm open to sexual harassment lawsuits and in general, reduces the productivity and
comfort zone of female employees. Hence, he is in violation of Standard II (B) - Professional
Misconduct.

Question: 79

When dealing with charitable organizations, the fiduciary must consider all of the following, except:

A. general economic conditions.


B. the charity's present and anticipated financial requirements.
C. investigation of supervisory activities once violation is known to have occurred.
D. long and short-term institutional needs in carrying out the charitable purpose.
E. expected total return on its investments.
F. trends in security price levels.

Answer: C

Explanation:
As fiduciaries for charitable organizations, members must consider: long and short-term institutional
needs in carrying out the charitable purpose, the charity's present and anticipated financial
requirements, expected total return on its investments, trends in security price levels and general
economic conditions. Investigation of violations involving supervisors is a compliance procedure
under Standard III (E), not under Standard IV (B.1), which deals with Fiduciary Duties.

Question: 80

According to the Prudent Investor Rule, the trustee must:

- adhere to loyalty, impartiality and ________


- maintain overall portfolio risk at a reasonable level

Page | 32
- provide for reasonable diversification of trust investments
- act with prudence in deciding whether and how to delegate authority to experts and in selecting
and supervising agents
- be cost conscious when investing

A. commitment
B. none of these answers
C. prudence
D. diversification
E. supervisors

Answer: C

Explanation:
Modern Portfolio Theory dictates that trustees consider a portfolio in its entirety and not just on an
investment-by-investment basis. As a fiduciary, therefore, the trustee must:

- adhere to fundamental fiduciary duties of loyalty, impartiality and prudence.


- maintain overall portfolio risk at a reasonable level
- the trade-off between risk and return is the fiduciary's central concern.
- provide for reasonable diversification of trust investments.
- act with prudence in deciding whether and how to delegate authority to experts and in selecting
and supervising agents.
- be cost conscious when investing.

Question: 81

Standard III includes which of the following?

A. Reasonable Basis and Representations


B. Preservation of Confidentiality
C. All of these answers
D. Performance Presentation
E. Use of Professional Designation
F. None of these answers

Answer: F

Explanation:
Standard III deals with Obligation to Inform Employer of Code and Standards, Duty to Employer,
Disclosure of Conflicts to Employer, Disclosure of Additional Compensation Arrangements and
Responsibilities of Supervisors.

Question: 82

The Prudent Man Rule states that the trustee must achieve an equitable balance between current
income and the ________ of principal in ________.

A. prudent management; in money markets


B. preservation; real terms
C. steady growth; nominal terms
D. preservation; nominal terms
E. prudent management; in fixed-income securities
F. steady growth; real terms

Page | 33
Answer: B

Explanation:
Under the Prudent Man Rule, trustees must be impartial between income beneficiaries and
remaindermen and must achieve an equitable balance between current income and the preservation
of principal in real terms.

Question: 83

Standard IV (B.5), Preservation of Confidentiality states:

A. all of these answers.


B. none of these answers.
C. members must release information to AIMR when the Professional Conduct Program is holding an
investigation, unless there is a settlement agreement between the parties that stipulates
confidentiality.
D. if the information is confidential, even illegal activity may not be reported, because there is the
issue of trust between parties.
E. members should avoid disclosing any information received from a client except toauthorized
fellow employees who are also working for the client.

Answer: E

Explanation:
Under Standard IV (B.5), members shall preserve the confidentiality of information communicated by
clients within the scope of client-member or employer-member relationship, except when the
member receives information concerning illegal activity on the part of the client. When there is an
investigation under PCP, members shall provide information about a client in support of the
investigation. Settlement agreements with confidentiality clauses do not prohibit members from
cooperating with a PCP investigation.

Question: 84

Since return results must be calculated on a basis that includes the effect of leverage, return results
must be restated to a(n) ________ basis.

A. margin
B. total asset
C. cash flow
D. all cash
E. multiple cash

Answer: D

Explanation:
Return results must be calculated on a basis that includes the effect of leverage, return results must
be restated to an all-cash basis when the portfolio used leverage and the same securities could have
been purchased at the same prices if the portfolio has the cash to do so. Results should be restated
to an allcash basis only when the necessary restatement can be based entirely on actual transactions
and can be verified in accordance with applicable account standards.

Question: 85

Page | 34
Standard II (A) deals with ________.

A. Obligation to Inform Employer of Code and Standards


B. None of these answers
C. Professional Misconduct
D. Plagiarism
E. Use of Professional Designation
F. Duty to Employer
G. Disclosure of Conflicts to Employer
H. Fundamental Responsibilities

Answer: E

Explanation:
Standard I deals with Fundamental Responsibilities. Standard II (A) deals with Use of Professional
Designation. Standard II (B) deals with Professional Misconduct. Standard II (C) deals with Plagiarism.
Standard III (A) deals with the Obligation to Inform Employer of Codes and Standards. Standard III (B)
deals with the Duty to Employer. Standard III (C) deals with Disclosure of Conflicts to Employer.

Question: 86

Which of the following is/are true about the PPS?

I. Accounts of clients that are not currently under the firm's management should not be included in
the presentation of the historical performance results.
II. Compliance with PPS cannot be met on a composite-by-composite basis, only on a firm-wide
basis.
III. The PPS require that firms report, at a minimum, at least the most recent 5 years (or since
inception, if less than 5 years) of performance results.

A. III only
B. I only
C. I and III only
D. II only
E. I, II and III
F. I and II only
G. II and III only

Answer: D

Explanation:
The PPS require that firms report, at a minimum, at least the most recent 10 years (or since
inception, if less than 10 years) of performance results. If this requirement cannot be met, exact
reasons for this should be disclosed. Historical results should be presented as is to give a fair
representation of past performance. Dropping portfolios selectively can bias the results. Also,
Compliance with PPS cannot be met on a composite-by-composite basis, only on a firm-wide basis.

Question: 87

Which of the following can be found in Standard III?

A. Members must use the CFA designation in a dignified manner.


B. If members receive material nonpublic information in confidence, they shall not breach that
confidence by trading or causing others to trade in securities to which such information relates.

Page | 35
C. Members shall maintain knowledge of AIMR's Code of Ethics.
D. Members shall comply with any prohibitions on activities imposed by their employer if a conflict
of interest exists.
E. Members shall make reasonable and diligent efforts to avoid any material misrepresentation in
any research report or investment recommendation.

Answer: D

Explanation:
Standard III states: "Members shall comply with any prohibitions on activities imposed by their
employer if a conflict of interest exists."

Question: 88

Which of the following can be found in Standard II?

A. Members shall make reasonable and diligent efforts to avoid any material misrepresentation in
any research report or investment recommendation.
B. Members must use the CFA designation in a dignified manner.
C. Members shall comply with any prohibitions on activities imposed by their employer if a conflict of
interest exists.
D. Members shall maintain knowledge of AIMR's Code of Ethics.
E. If members receive material nonpublic information in confidence, they shall not breach that
confidence by trading or causing others to trade in securities to which such information relates.

Answer: B

Explanation:
Standard II states that holders of the CFA designation may use it, but only in a dignified and judicious
manner. The use of the designation may be accompanied by an accurate explanation of the
requirements that have been met to obtain the designation.

Question: 89

Which of the following is/are forms of plagiarism as defined by AIMR code of conduct?

I. Using material from a seminar in research reports without proper acknowledgment.


II. Presenting statistical estimates prepared by others without the associated caveats and qualifiers.
III. Using of information obtained in a teleconference without identifying the original source.
IV. Using Standard & Poor's estimates of stock betas without attribution.

A. I, II and IV only
B. I, II and III only
C. II, III and IV only
D. II and III only

Answer: B

Explanation:
Standard II (C) - Prohibition against Plagiarism - considers a use of factual information published by
recognized financial and statistical services without attribution acceptable. All of the others are forms
of plagiarism and violations of Standard II (C).

Question: 90

Page | 36
Which of the following AIMR standards pertains to the responsibilities of supervisors?

A. IV
B. II C
C. V
D. None of these answers

Answer: D

Explanation:
Responsibilities of Supervisors falls under Standard III (E).

Question: 91

Firms with records or performance calculations for periods prior to the applicable effective date(s)
that are not in conformance with the AIMR Performance Presentation Standards can still claim
compliance with the standards if certain conditions are met. Which of the following is an option
available to such a firm?

A. None of these answers are options available to such a firm.


B. All of these answers are options available to such a firm.
C. The firm can restate its historical performance in accordance with the Relaxed Retroactive
Standards for retroactive compliance.
D. The firm can use its nonconforming historical performance and disclose specifically when and how
the performance is not in compliance.
E. The firm can restate its historical performance numbers in accordance with the Standards.

Answer: B

Explanation:
Firms with records or performance calculations for periods prior to the applicable effective date(s)
that are not in conformance with the Standards can choose any of the options listed.

Question: 92

When formulating an investment policy for a client, all of the following fall under "investor
constraints," except ________.

A. liquidity needs
B. regulatory and legal circumstances
C. risk tolerance
D. tax considerations
E. expected cash flows
F. investable funds
G. investor preferences, circumstances and unique needs
H. none of these answers

Answer: C

Explanation:
Risk tolerance is considered under "investor objectives."

Question: 93

Page | 37
Which of the following can be found in Standard I?

A. Members shall deliver a copy of the Code to their employer.


B. Members shall not participate in any professional conduct involving dishonesty, fraud, deceit, etc.
C. Members shall not knowingly participate or assist in any violation of laws, rules or regulations.
D. Members shall exercise diligence and thoroughness in making investment recommendations or in
taking investment actions.
E. Members shall not misrepresent investment performance.

Answer: C

Explanation:
Standard I states that members shall: "A. Maintain knowledge of and comply with all applicable laws
of any governing agency or professional association governing the member's professional activities,
and B. Not knowingly participate or assist in any violation of laws, rules or regulations."

Question: 94

AIMR Standard ________ prohibits plagiarism.

A. I (D)
B. IV (A)
C. None of these answers
D. II (C)
E. III (B)

Answer: D

Explanation:
Standard II (C) - Prohibition against Plagiarism states: "Members shall not copy or use, in
substantially the same form as the original, material prepared by another without acknowledging
and identifying the name of the author, publisher, or source of such material. Members may use,
without acknowledgment, factual information published by recognized financial and statistical
reporting services or similar sources."

Question: 95

Fiduciaries are obligated to vote proxies:

A. none of these answers.


B. on an "as needed" basis, depending on the issue at hand.
C. only when there is a nonroutine governance issue.
D. only when there is a change in firm capitalization.
E. in an informed and responsible manner.
F. only when management requests the fiduciary to vote, in writing.

Answer: E

Explanation:
As part of a fiduciary's duty of loyalty, they are obligated to vote proxies in an informed and
responsible manner. A fiduciary who fails to vote, casts a vote without considering the impact of the
question or votes blindly with management on nongovernance issues may violate this duty of loyalty.

Page | 38
Question: 96

Standard III (E) is ________.

A. Obligation to Inform Employer of Code and Standards


B. Duty to Employer
C. Responsibilities of Supervisors
D. Disclosure of Conflicts to Employer
E. None of these answers
F. Disclosure of Additional Compensation Arrangements

Answer: C

Explanation:
Standard III (A) deals with the Obligation to Inform Employer of Code and Standards. Standard III (B)
deals with the Duty to Employer. Standard III (C) deals with Disclosure of Conflicts to Employer.
StandardIII (D) deals with Disclosure of Additional Compensation Arrangements. Standard III (E)
deals with Responsibilities of Supervisors.

Question: 97

Each ________ needs to comprise portfolios or asset classes that represent a similar investment
goal.

A. benchmark
B. value
C. return
D. composite
E. mix

Answer: D

Explanation:
All actual fee-paying discretionary portfolios must be included in at least one composite defined
according to similar strategy or investment objective.

Question: 98

Chan and Chung are two of the five Managing Directors of Alfalfa, a mid-size hedge fund. In a recent
court case involving a securities lawsuit, Chan was called on to testify as an expert on securities
research. During his testimony, Chan had to invoke several results from a proprietary research carried
out by his staff at Alfalfa. He did so without specifically attributing the results to them. At around the
same time, Chung had to meet with a few prospective clients for a business presentation. During this
presentation, he showed them some of the results obtained by Chan's team, without specifically
acknowledging the research team. Instead, he referred to them with phrases like, "our studies
indicate that..." In this set of events, as it relates to Standard II(C) - Prohibition against Plagiarism,

A. neither Chan nor Chung has violated the standard.


B. Chan has not violated the standard while Chung has.
C. both Chan and Chung have violated the standard.
D. Chan has violated the standard while Chung has not.

Answer: D

Page | 39
Explanation:
In the interactions with clients, senior management members represent the firm and as such, do not
have to give specific attribution to research results obtained by employees of the firm. Hence, Chung
has committed no violation. On the other hand, in the capacity of an expert, Chan is not representing
the firm but himself. In such a case, he has to properly acknowledge the sources of all results he
quotes. Refer to Standard II(C).

Question: 99

Standard IV (B.3) deals with ________.

A. Duty to Employer
B. Professional Misconduct
C. Interactions with Clients and Prospects
D. None of these answers
E. Prohibition against Use of Material Nonpublic Information
F. Preservation of Confidentiality
G. Fair Dealing
H. Investment Process

Answer: G

Explanation:
Standard IV (A) deals with the Investment Process. Standard III (B) deals with Duty to Employer.
Standard IV (B.3) deals with Fair Dealing. Standard IV (B) deals with Interactions with Clients and
Prospects. Standard V (A) deals with Prohibition against Use of Material Nonpublic Information.
Standard IV (B.5) deals with Preservation of Confidentiality. Standard II (B) deals with Professional
Misconduct.

Question: 100

Brikland Strickowski learned a month ago that one of his clients, Stratmann, had been regularly using
inside information provided by numerous "contacts" in various firms to generate his trading activity
with Brikland. Brikland was surprised at Stratmann's recklessness but did not want to get him into
trouble so he did not inform anyone about this. AIMR has been tipped about Stratmann's activity and
one of the investigators from AIMR's Professional Conduct Program (PCP) recently got in touch with
Brikland about this. Brikland, in his desire not to have his CFA designation voided by AIMR, decided
to divulge everything he knew about Stratmann's investment activity, though this information is
considered confidential even by AIMR. Which of the following is/are true about this sordid mess?

I. Brikland violated Standard IV (B.5) - Preservation of Confidentiality -by revealing confidential


information about Strattman to the PCP investigator.
II. Strattman violated Standard V (A) - Prohibition Against Use of Non-Public Information.
III. Brikland violated Standard I - Fundamental Responsibilities - by not informing his supervisor
and/or the SEC about the insider trading.
IV. Brikland violated Standard I - Fundamental Responsibilities - by not dissociating himself from
Strattman's portfolio.

A. I, II, III and IV


B. II only
C. II and IV
D. I, II and IV only

Answer: C

Page | 40
Explanation:
As an AIMR member, Brikland must cooperate fully with any investigation carried out by the PCP.
Since the information revealed during the investigation is kept in strict confidentiality, such a
cooperation is not considered a violation of Standard IV (B.5) - Preservation of Confidentiality.
Indeed, the Code of Ethics expressly requires members to cooperate with such investigations, failing
which the member can be summarily suspended. Note that the Code does not require members to
report criminal activity to proper authorities, though doing so is most often prudent and a suggested
course of action (Standard I - Fundamental Responsibilities). However, if the member knowingly
participates in illegal activities or does not dissociate himself from such activities, he would be
violating the code.

Question: 101

Xuanchi Zuan is a successful portfolio manager with Up & Away, a money management fund that
operates strongly bullish funds. One of Zuan's clients, Margo Margolis, is about to retire in a couple
of years. Margo has been phasing out her risky investments over the past five years, moving them
into longterm treasury bonds. However, she recently read in a financial magazine the attractiveness
of investing in emerging markets. She was especially enticed by the 30% and 40% annual returns
observed in these markets over the past 2-3 years. She approaches Zuan and instructs him to move
30% of her funds into these markets. Zuan should:

A. none of these answers.


B. evaluate the emerging market investments and move the funds only if he considers them suitable
for investment for Margo.
C. refuse to do so, pointing out the high risks involved in such investments.
D. move the funds into emerging market securities per Margo's request.That answer is correct!

Answer: A

Explanation:
Zuan cannot refuse to do something that his client instructs him to do, as long as it is legal. However,
it is his duty to apprise his clients of the risks and returns associated with different instruments and
advise them about the suitability of including such investments in their portfolios. Once the client is
aware of all the pitfalls and rewards, the final decision must rest with the client. In Margo's case,
Zuan should do the same before acting on her instructions. This question relates most directly to
Standard IV (B.1) - Fiduciary Duties.

Question: 102

If every one of your research reports had a statement stating that every research report on issues by
a corporate client reflects the unbiased opinion of the analyst, you would be complying with
Standard ________.

A. IV (A.3)
B. IV (A.1)
C. IV (A.2)
D. II (A)
E. II (C)
F. II (B)That answer is correct!

Answer: A

Explanation:

Page | 41
To avoid violations of Standard IV (A.3), one of the procedures members should comply with is the
need to protect the integrity of opinions. Members and their firms should establish policies stating
that every research report on issues by a corporate client reflects the unbiased opinion of the
analyst. In this way, firms ensure that the independence and objectivity of their member analysts is
maintained.

Question: 103

Which of the following rules apply for calculations under the Performance Presentation Standards?

A. Accrual accounting must be used for fixed-income securities.


B. All of these answers.
C. Total return, including realized and unrealized gains and losses, must be used.
D. Time-weighted rate of return must be used.

Answer: B

Explanation:
All of these answers are general rules applying to calculation of returns under the PPS.

Question: 104

According to the AIMR-PPS when presenting the performance record of composites containing
portfolios that use leverage, which of the following statements are true?

A. If the use of leverage is discretionary, the performance presented must include the effects of the
leverage.
B. If the use of leverage is discretionary, performance on a restated, all-cash basis must be provided.
C. All of these statements are true.
D. If the use of leverage is nondiscretionary, performance must be presented on an all-cash basis.

Answer: C

Explanation:
For composites containing portfolios that use leverage, all of the statements are true. This is a
requirement for presentation of results.

Question: 105

Which of the following AIMR standards states that a member shall not knowingly participate, or
assist, in any violation of applicable laws, regulations, or the Code and Standards?

A. III
B. VI
C. V
D. I

Answer: D

Explanation:
Standard I: Fundamental Responsibilities states: "Members shall: A. Maintain knowledge of and
comply with all applicable laws, rules and regulations of any government, governmental agency,
regulatory organization, licensing agency, or professional association governing the members'
professional activities, and B. Not knowingly participate or assist in any violation of such laws, rules,

Page | 42
or regulations."

Question: 106

Which of the following is/are true about the definition of "total firm assets" where PPS are
concerned?

I. Total assets include non-discretionary as well as discretionary assets.


II. Unless the firm actually manages the assets underlying investment strategies, they are not
included in the definition of total assets.
III. Assets assigned to subadvisors which are not directly under the firm's management should not be
included as part of "total assets."

A. I, II and III
B. I and III only
C. II and III only
D. I and II onlyThat answer is correct!

Answer: A

Question: 107

Which of the following can be found in Standard I?

A. Members shall maintain knowledge of and comply with all applicable laws.
B. Members shall not undertake any independent practice in competition with employer without
written consent.
C. Members shall maintain appropriate records to support the reasonableness of recommendations.
D. Members shall not participate in plagiarism.
E. Members shall make reasonable efforts to achieve public dissemination of material nonpublic
information disclosed in breach of a duty.That answer is correct!

Answer: A

Explanation:
Standard I states that members shall: "A. Maintain knowledge of and comply with all applicable laws
of any governing agency or professional association governing the member's professional activities,
and B. Not knowingly participate or assist in any violation of laws, rules or regulations."

Question: 108

To claim compliance with the AIMR Performance Presentation Standards,

A. at least 1/2 of the investment managers in the firm must comply with the Standards.
B. the entire firm must comply with the Standards.
C. none of these answers. The Standards apply to individual investment manager's performance only.
D. at least 3/4 of the investment managers in the firm must comply with the Standards.

Answer: B

Explanation:
To claim compliance with the AIMR-PPS, the firm must comply with the Standards on a firmwide
basis. Additionally, the firm must state exactly how it is defining itself for purposes of compliance.

Page | 43
Question: 109

Standard IV (B.5) deals with ________.

A. Disclosure of Conflicts to Clients and Prospects


B. Prohibition against Use of Material Nonpublic Information
C. None of these answers
D. Preservation of Confidentiality
E. Prohibition against Misrepresentation
F. Priority of Transactions
G. Disclosure of Referral Fees
H. Performance Presentation

Answer: D

Explanation:
Standard IV (B.5) deals with the preservation of confidentiality of information communicated by
clients or employers concerning matters within the scope of the client-member, employer-member
relationship.

Question: 110

Which of the following is not a violation of Standard II (C)?

A. Use a small part of an analysts work who also is employed in a completely different (non-
competitive) industry, without acknowledgment.
B. Use a small part of an analysts work who also is employed in the same industry, without
acknowledgment.
C. All of these answers are violations.
D. Using one chart in a presentation that was prepared by another analyst, without acknowledgment.
E. Giving an oral report and citing specific quotations, attributable to "leading analysts," without
specific reference.

Answer: C

Explanation:
Using excerpts from articles or reports prepared by others, either verbatim, or with only a slight
change, without acknowledgment; citing specific quotations, attributable to "leading analysts,"
without specificreference; using charts or graphs without stating their sources; and copying
proprietary computerized spreadsheets or algorithms without seeking the authorization of their
creators - all of these practices are violations of Standard II (C). Only globally recognized sources of
factual material such as that provided by Standard & Poor's, or Moody's Investors Service, can be
used without acknowledgment, since such information is already in the public realm.

Question: 111

Which of the following can be found in Standard II?

A. Members shall not participate in plagiarism.


B. Members shall maintain appropriate records to support the reasonableness of recommendations.
C. Members shall maintain knowledge of and comply with all applicable laws.
D. Members shall not undertake any independent practice in competition with employer without
written consent.
E. Members shall make reasonable efforts to achieve public dissemination of material nonpublic

Page | 44
information disclosed in breach of a duty.That answer is correct!

Answer: A

Explanation:
Standard II states: "Members shall not copy or use, in substantially the same form as the original,
material prepared by another without acknowledging and identifying the name of the author,
publisher or source of such material."

Question: 112

Social factors

A. none of these answers.


B. must be a primary consideration of investments because of the ERISA rule.
C. may never be a primary consideration of investments.
D. may be a primary consideration of investments, depending on the guidelines of the investment
policy.
E. must be a primary consideration of investments because of the prudence rule.
F. must be a primary consideration of investments because of the loyalty rule.

Answer: C

Explanation:
ERISA guidelines issued by the U.S. Department of Labor state that ERISA prudence standard requires
a fiduciary to make investment decisions first on grounds of economic and investment merit.
Consideration of social factors may be an incidental, but never primary, consideration of investments
that are equal in economic and financial terms.

Question: 113

Jackson, an analyst, decides not to change a recommendation from buy to sell because she wants to
sell her holdings first. Is there a standards violation?

A. No
B. Yes, Standard IV (A.2), Research Reports
C. Yes, Standard III (B), Duty to Clients and Prospects
D. Yes, Standard IV (B.4), Priority of Transactions

Answer: D

Explanation:
Jackson violated Standard IV (B.4) - Priority of Transactions Jackson has decided that she did not want
to wait until all her clients have had the opportunity to sell first. Thus, her decision not to change her
recommendation until she accomplished her own financial aims will result in losses to her clients if
the value of the holding subsequently declines.

Question: 114

Standard III includes rules on which of the following?

A. Professional Misconduct
B. All of these answers
C. Prohibition against Plagiarism

Page | 45
D. Obligation to Inform Employer of Code and Standards
E. Use of Professional Designation
F. None of these answers

Answer: D

Explanation:
Standard III deals with Obligation to Inform Employer of Code and Standard, Duty to Employer,
Disclosure of Conflicts to Employer, Disclosure of Additional Compensation Arrangements and
Responsibilities of Supervisors.

Question: 115

Standard II includes rules on which of the following?

A. Professional Misconduct
B. Prohibition against Plagiarism
C. All of these answers
D. Use of Professional Designation

Answer: C

Explanation:
Standard II - Relationships with and Responsibilities to the Profession - includes rules on the use of
Professional Designation, Professional Misconduct and Prohibition against Plagiarism.

Question: 116

Standard III (C) deals with conflicts of interest of a member with ________.

A. colleagues at the same firm


B. the employer
C. none of these answers
D. the client
E. other investment professionals
F. the investing public

Answer: B

Explanation:
Standard III (C) deals with conflicts of interest in any actions or decisions of a member and focuses on
responsibility to the employer. By complying with employer guidelines, members allow their
employers to avoid potentially embarrassing and costly violations.

Question: 117

Which of the following AIMR Standards states that the financial analyst must exercise diligence and
thoroughness in making investment recommendations or taking investment actions?

A. III B
B. IV A.1
C. I
D. None of these answers

Page | 46
Answer: B

Explanation:
Standard IV (A.1) - Reasonable Basis and Representations states: "Members shall exercise diligence
and thoroughness in making investment recommendations or in taking investment actions."

Question: 118

Various countries' securities laws permit a manager to pay up for goods and services without
violating the manager's fiduciary duty, so long as the requirements of the law are followed. Each of
the following are typical requirements, except:

A. the goods or services purchased must be for "brokerage service."


B. the commission paid must be reasonable in relation to the research and execution services
received.
C. none of these answers.
D. at all times, the manager must seek best price and execution.
E. the manager's soft-dollar practice must be disclosed.That answer is correct!

Answer: A

Explanation:
The good and services purchased must be for "research service."

Question: 119

According to Standard IV (B.4), Priority of Transactions, ________ is recommended when a manager


has purchased a security in violation of trading policy.

A. forfeit of wages for one month


B. a fine imposed by the SEC
C. a civil damage
D. public censure
E. disgorgement

Answer: E

Explanation:
This question relates to the compliance procedures for Standard IV (B.4), one of which states that
firms should establish disciplinary procedures to enforce the standard. If a manager has purchased a
security in violation of trading policy, disgorgement is recommended. Managers should disgorge all
profits and assume any losses from the trade.

Question: 120

Pulser Primorak is an investment manager who recently bid in an IPO on behalf of his clients and was
allowed to buy 1,000 shares of the issue. What should Primorak do?

A. He should distribute the IPO shares amongst the client accounts over which he has discretionary
investment powers on a pro rata basis.
B. He should treat all his customers equally and fairly by distributing the IPO shares amongst all his
client accounts on a pro rata basis.
C. He should distribute the IPO shares amongst all his client accounts for which the IPO is an
appropriate investment on a pro rata basis.

Page | 47
D. The question is based on a false premise. Primorak should not have bid on an IPO in the first place
since this violates the AIMR code of Ethics.

Answer: C

Explanation:
Standard IV (B.3) - Fair Dealing. Note that the AIMR code does not prohibit investments by portfolio
managers in IPOs if they are deemed appropriate investments.

Question: 121

Members pursuing the investment profession outside of the U.S. ________ the Code and Standards
when local regulations are less stringent in order to maintain a level playing field.

A. may disregard
B. can modify
C. should ignore
D. must adhere to

Answer: D

Explanation:
The concept of fiduciary duty is not exclusive to the United States. AIMR members are expected to
follow any fiduciary duties imposed on them by their country or province, in addition to being
governed by the Codes and Standards. As always, the primary concern must be to follow principles of
discretion, loyalty and care and to act in the client's best interest.

Question: 122

The AIMR Performance Presentation Standards require that firms report, at a minimum, ________
years of performance to claim compliance with the standards.

A. one
B. fifteen
C. five
D. ten
E. seven
F. two

Answer: D

Explanation:
The AIMR-PPS require that firms report, at a minimum, 10 years of investment performance (or
performance since the inception of the firm if inception is less than 10 years) to claim compliance
with the Standards.

Question: 123

Which of the following AIMR Standards states that the financial analyst shall distinguish between
facts and opinions in research reports?

A. III A
B. III D
C. III (B.1)

Page | 48
D. IV (A.2)

Answer: D

Explanation:
Standard IV (A.2) - Research Reports states: "Members shall distinguish between facts and opinions
in research reports."

Question: 124

Complete the following: According to The Code of Ethics, members of AIMR shall: "Practice and
encourage others to practice in a professional and ethical manner that will ________ members and
their profession."

A. improve the qualifications of


B. reflect credit on
C. none of these answers
D. improve the compensation of
E. improve the access of

Answer: B

Explanation:
According to The Code of Ethics, members of AIMR shall: "Practice and encourage others to practice
in a professional and ethical manner that will reflect credit on members and their profession."

Question: 125

Standard III (A) - Obligation to Inform Employer of Code & Standards - requires all AIMR members,
Charterholders and CFA candidates inform their employers about their personal obligation to abide
by the code. Which of the following is/are true in regard to this requirement?

I. The notification can be in either oral form or in writing, either in print, handwriting or by electronic
means like email.
II. Notification is necessary even if the employer is publicly known to have adopted the Code of
Ethics.
III. For compliance with Standard III (A), notification must be made to the immediate employer and
not necessarily senior management.

A. III only
B. I, II and III
C. I and III only
D. I and II onlyThat answer is correct!

Answer: A

Explanation:
If the employer has publicly adopted the Code of Ethics, no explicit notification is required.
Otherwise, the immediate supervisor must be informed in writing about the obligation to abide by
the Code. "In writing" includes any form of communication that can be documented, including email.
Refer Standard III (A) - Obligation to Inform Employer of Code & Standards

Question: 126

Page | 49
According to Standard IV (B.4), Priority of Transactions: "Transactions for clients and employers shall
have priority over transactions in securities or other investments of which a member is the ________
so that such personal transactions do not operate adversely to their clients' or employer's interests."

A. material agent
B. primary decisionmaker
C. none of these answers
D. proxy voter
E. beneficial owner
F. principal broker
G. sole charterholder
H. registered representative

Answer: E

Explanation:
Standard IV (B.4) deals with the Priority of Transactions. Under this standard, transactions for clients
and employers shall have priority over transactions in securities or other investments of which a
member is the beneficial owner so that such personal transactions do not operate adversely to their
clients' or employer's interests. If members make a recommendation regarding the purchase or sale
of an investment, they shall give their clients and employer adequate opportunity to act on the
recommendation before acting on their own behalf.

Question: 127

Standard III (E) is known as ________.

A. Prohibition against Use of Material Nonpublic Information


B. Preservation of Confidentiality
C. Fair Dealing
D. Professional Misconduct
E. Interactions with Clients and Prospects
F. Investment Process
G. Duty to Employer
H. None of these answers

Answer: H

Explanation:
Standard III (E) is Responsibilities of Supervisors. Be sure you remember that this standard may often
be cited as a violation, along with most other standards (because failure to supervise is often at the
root of many of the standard violations).

Question: 128

According to the AIMR-PPS, total firm assets are defined to include

A. all discretionary and nondiscretionary assets, as well as assets underlying overlay investment
strategies.
B. all discretionary and nondiscretionary assets.
C. any asset deemed eligible by the firm.
D. only assets underlying overlay investments strategies.

Answer: B

Page | 50
Explanation:
Total firm assets are defined to include all discretionary and nondiscretionary assets. Total firm assets
does not refer to assets underlying overlay investment strategies, such as currency overlay, options
and futures overlays, securities lending programs and asset allocation overlay strategies, unless the
firm actually manages the underlying assets.

Question: 129

Standard V (B) addresses any practice that would lead to misrepresentation of a member's record,
whether the practice involves performance ________ or performance ________.

A. presentation; measurement
B. none of these answers
C. presentation; benchmarking
D. assessment; benchmarking
E. measurement; assessment
F. assessment; presentation
G. benchmarking; measurementThat answer is correct!

Answer: A

Explanation:
Standard V (B) addresses any practice that would lead to misrepresentation of a member's record,
whether the practice involves performance presentation or performance measurement.

Question: 130

A(n) ________ is someone who has knowledge of pending or actual investment recommendations or
action.

A. AIMR member
B. access person
C. none of these answers
D. insider

Answer: B

Explanation:
Access or covered persons have knowledge of pending or actual investment recommendations or
action. The firm's definition of access (covered) person should be broad enough to cover all people
with that knowledge.

Question: 131

Although trade-date accounting is recommended, ________ accounting that is disclosed is


acceptable for calculating performance.

A. settlement-date
B. maintenance
C. accrual
D. composite
E. PerformanceThat answer is correct!

Page | 51
Answer: A

Explanation:
Trade-date accounting is recommended when calculating performance, although settlement-date
accounting is acceptable if disclosed. Because of the volatility and lengthy settlement periods of
some markets, trade-date accounting is strongly recommended for calculating the performance of
international portfolios.

Question: 132

Mayberry's Mayor, in consultation with the trustees of Mayberry's Village Investment Fund, has
decided that the fund needs to become more aggressive in its investment objectives to generate
funds for village renovations like a paved road, a new school and an activity center for kids. He
discussed these requirements with the plan manager, Otis, and asked him to suggest a suitable risk
profile and investment instruments to achieve the objectives within a year. Barney, a businessman
from a neighboring town, is trying to decide if he should invest his excess capital in setting up
businesses in Mayberry or another town, Februberry. These businesses promise to generate about
35 new jobs at various levels and stimulate consumer spending in the town in which they are set up.
Barney wants to be sure that the businesses are set up in areas that have high growth promise. So he
calls up Otis to find out about the Mayor's plans for the next two years for Mayberry's development.
Otis sets up a meeting with Barney and outlines the proposed renovation projects. Otis does his best
to convince Barney that Mayberry is the right place for Barney's businesses.

I. Barney has violated the Code of Ethics by discussing the plans with Otis.
II. Otis has violated the Code of Ethics by discussing the plans with Barney.
III. Neither Barney nor Otis violated any standards.

A. II only
B. III only
C. I and II only
D. I onlyThat answer is correct!

Answer: A

Explanation:
Otis has violated Standard IV (B.5) - Preservation of Confidentiality by revealing the Mayor's plans to
Barney. Any information that Otis receives in his capacity as a fund manager is strictly confidential
and should not be revealed unless it is in connection with a criminal investigation or an AIMR
investigation. Barney has not violated any Standards.

Question: 133

Felix Asterix is a shrewd industry analyst working for Obelix, Inc., a brokerage firm of some repute.
Asterix had invested about $50,000 in a few stocks trading on the Australian exchange a year ago.
Obelix itself is not active outside the US and Asterix's transactions do not represent any violations of
the company policies. One of the Australian firms whose shares Asterix holds recently applied for a
dual listing on the NYSE and will begin trading in the US in a month. Asterix has advised in his recent
research reports that investors would be well-off investing in this dual-listed stock. He does not
reveal his holdings in the Australian stock. Felix has:

I. not violated the AIMR standard since his transactions took place long before the dual listing and he
is not obligated to reveal his personal holdings.
II. violated Standard III (C) - Disclosure of Conflicts to Employer.
III. violated Standard IV (A.3) - Independence and Objectivity.

Page | 52
IV. violated Standard IV (B.7) - Disclosure of Conflicts to Clients and Prospects.

A. I only
B. II and III only
C. II and IV only
D. II, III and IV only

Answer: D

Explanation:
This is a classic case of potential conflict and appearance of conflict of interests. Indeed, this kind of
situation is so common that it is covered by multiple AIMR standards of ethics. Asterix, as an
investment advisor and analyst, should be extremely careful about any potential conflicts of interest
that might be perceived as affecting the objectivity of his judgment. Clearly, his substantial Australian
holdings, while not in violation of company policies when purchased, are a source of bias now that
the stock is trading in the US. Felix must discuss the situation with the Compliance Department at
Obelix, Inc. and if he receives the permission to continue holding the stock, should reveal his vested
interests to all his potential clients. This ensures that there is no perception of unfair bias and lack of
objectivity in his investment recommendations.

Question: 134

Which of the following is/are required by AIMR-PPS with regards to calculation of returns?

I. For leveraged portfolios, the stated returns must be on an "all-cash" basis, removing the effects of
debt financing.
II. Performance must be based on "gross" returns i.e. before necessary expenses like brokerage and
SEC fees.
III. Composites must be asset-weighted using beginning-of-period weights.

A. none of them
B. I, II and III
C. I and III only
D. III only

Answer: D

Explanation:
For leveraged portfolios, PPS require a statement of performance using actual returns as well as on
an "all-cash" basis (i.e. without leverage). Performance must be based on returns after trading
expenses like brokerage and SEC fees. Composites must be asset-weighted using beginning-of-period
weights.

Question: 135

Level ________ verification applies to the firm.

A. IV
B. 0
C. I
D. II
E. III

Answer: C

Page | 53
Explanation:
Level I verification applies to all firm composites. Level II verification requires a Level I verification at
least on the specific composites being verified at Level II. (Note: There are only two levels of
verification.)

Question: 136

When an investment manager uses client brokerage to purchase research services that benefit the
investment manager; this is known as ________.

A. under-the-table agreements
B. soft dollars
C. through-research agreements
D. bartering arrangements
E. shifting agreements

Answer: B

Explanation:
An investment manager often has discretion over the selection of brokers executing transactions.
Conflicts arise when an investment manager uses client brokerage to purchase research services that
benefit the investment manager, which is commonly called "soft dollars," "soft pounds," or "soft
commissions." Whenever a manager uses such client brokerage to purchase services that benefit
him, as a fiduciary, he must disclose to clients the methods or policies followed to address the
potential conflict.

Question: 137

ERISA fiduciaries must adhere to the following prudent procedures:

- establish a ________ investment policy for the plan


- diversify plan assets
- make investment decisions with the skill and care of a prudent expert
- monitor investment performance
- control investment expenses
- avoid prohibited transactions

A. supervisory
B. none of these answers
C. written
D. diversified

Answer: C

Explanation:
These procedures are stipulated under the detailing of ERISA fiduciary duties, to ensure that the
fiduciary complies with the duty to act with prudence.

Question: 138

A critical part of Standard IV (A.2) is to distinguish between:

A. buy side and sell side.

Page | 54
B. industry and company analysis.
C. insider trading and appropriate trading.
D. research reports and investment memoranda.
E. employees and independent contractors.
F. CFA charterholders and non-CFA charterholders.
G. none of these answers.

Answer: G

Explanation:
Standard IV (A.2) - Research Reports states the responsibility of AIMR members, CFA charterholders
and candidates to include in each research report those key facts that are instrumental to the
investment recommendation presented in the report. A critical part of this requirement is to
distinguish clearly between opinions and facts.

Question: 139

Members ________ to be in compliance with the Performance Presentation Standards to be in


compliance with Standard V (B).

A. are not required


B. may be required
C. are always required
D. none of these answers
E. will be required, as of 1/1/2001,
F. will be required, as of 1/1/2002,That answer is correct!

Answer: A

Explanation:
The PPS are voluntary standards that a member or a firm may adopt in compiling and presenting
performance figures. AIMR adopted and promotes the PPS as a method of obtaining industry wide
uniformity in performance presentation information so that results are directly comparable among
investment managers. Members are not required to be in compliance with the PPS to be in
compliance with Standard V (B).

Question: 140

Which Standard deals with personal integrity and prohibits individual behavior that reflects badly on
the investment industry?

A. III (C)
B. I (D)
C. II (B)
D. III
E. None of these answers
F. IV (A)

Answer: B

Explanation:
Standard II (B) deals with personal integrity and prohibits individual behavior that reflects adversely
on the entire profession.

Page | 55
Question: 141

The Prudent Man Rule, as it was originally written, was clearly oriented towards ________.

A. charitable organizations
B. public pension plans
C. individual clients
D. personal trust accounts
E. endowments

Answer: D

Explanation:
As enunciated, the concept behind the Prudent Man Rule was clearly oriented towards personal trust
accounts. Under this rule, trustees must be impartial between income beneficiaries and
remaindermen and must achieve an equitable balance between current income and the preservation
of principal in real terms.

Question: 142

Government regulation of charitable organizations and public pension plans:

A. has increased as these organizations have grown in size and importance.


B. none of these answers.
C. has lessened, since there is a growing trend away from charitable organizations since 1989.
D. is still absent, although there is an effort to change this.That answer is correct!

Answer: A

Explanation:
Government regulation of charitable organizations and public pension plans has increased as these
organizations have grown in size and importance. As such, members who are fiduciaries for these
organizations must examine applicable law, client agreement and client guidelines for guidance as to
their duties.

Question: 143

Standard V (A) is known as ________.

A. Prohibition against Use of Material Nonpublic Information


B. Duty to Employer
C. Preservation of Confidentiality
D. Interactions with Clients and Prospects
E. None of these answers
F. Investment Process
G. Fair Dealing
H. Professional MisconductThat answer is correct!

Answer: A

Explanation:
Standard IV (A) deals with the Investment Process. Standard III (B) deals with Duty to Employer.
Standard IV (B.3) deals with Fair Dealing. Standard IV (B) deals with Interactions with Clients and
Prospects. Standard V (A) deals with Prohibition against Use of Material Nonpublic Information.

Page | 56
Standard IV (B.5) deals with Preservation of Confidentiality. Standard II (B) deals with Professional
Misconduct.

Question: 144

Which of the following is/are true about the Performance Presentation Standards?

I. The PPS are voluntary standards and are not required by AIMR to be adopted by a member or a
firm.
II. Members need not be in compliance with the PPS to be in compliance with Standard V (B) -
Performance Presentation.
III. A member can claim compliance with the PPS only if he has complied with all the mandatory
requirements of the PPS.

A. II and III only


B. I and III only
C. I, II and III
D. III only

Answer: C

Explanation:
All of the answers are true of the PPS standards.

Question: 145

Urvashi Kulkarni is an investment manager with Amritrust Bank, a mid-size investment bank. Urvashi
is managing the pension plan assets of Megalith ICs, a maker of cloned integrated chips for PCs.
Megalith is planning a take-over of Microchip Corp., a fast-growing rival which recently patented a
technology which promises to change the face of chip cloning business. The pension plan assets of
Megalith consist of 17% of Microchip's stock. Megalith's management has decided to buy up to 33%
of the stock in a tender offer and transfer the 17% stock holdings of the pension plan to the company
investment account. So it instructs Urvashi to sell the Microchip stock to Megalith's general account
at the current market price and invest the cash in other stock as she feels appropriate. Urvashi knows
that this action and the subsequent takeover of Microchip will substantially elevate Megalith's stock
price, creating shareholder value. Urvashi should

A. refuse to follow the directive since she can be held liable under ERISA if she follows directives
issued by the plan sponsor.
B. refuse to follow the directive since the action is harmful to the pension plan beneficiaries and
Urvashi owes fiduciary loyalty to them, not the plan sponsor.
C. follow the management's directive and sell the stock to the general account.
D. none of these answers.

Answer: B

Explanation:
Urvashi owes fiduciary loyalty to the plan beneficiaries, not the management of Megalith. She must
not sell Microchip's stock prior to the tender offer since in doing so, she will be selling the plan assets
at a much lower price than would be available once the tender is floated. While such an action
benefitsMegalith shareholders, it harms the plan beneficiaries. Urvashi owes absolutely no
allegiance to Megalith shareholders. Standard IV (B.1) - Fiduciary Duties - and the Topical Study,
"Fiduciary Duties."

Page | 57
Question: 146

Standard III includes which of the following?

A. Reasonable Basis and Representations


B. All of these answers
C. Performance Presentation
D. Use of Professional Designation
E. None of these answers
F. Responsibilities of Supervisors

Answer: F

Explanation:
Standard III deals with Obligation to Inform Employer of Code and Standards, Duty to Employer,
Disclosure of Conflicts to Employer, Disclosure of Additional Compensation Arrangements and
Responsibilities of Supervisors.

Question: 147

Andrea, a portfolio manager for XYZ Investment Management Company, a registered investment
organization that advises investment companies and private accounts, was promoted to that position
three years ago. Bates, her supervisor, is responsible for reviewing Andrea's portfolio account
transactions and her required monthly reports of personal stock transactions. Andrea has been using
Jonelli, a broker, almost exclusively for portfolio account brokerage transactions. For securities in
which Jonelli's firm makes a market, Jonelli has been giving Andrea lower prices for personal
purchases and higher prices for personal sales than Jonelli gives to Andrea's portfolio accounts and
other investors.Andrea has been filing monthly reports with Bates only for those months in which
she has no personal transactions, which is about every fourth month. Which of the following
applies/apply?

I. Andrea violated the Code and Standards in that she failed to disclose to her employer her personal
transactions.
II. Andrea violated the Code and Standards by breaching her fiduciary duty to her clients. III. Bates
violated the Code and Standards by failing to enforce reasonable procedures for supervising and
monitoring Andrea in Andrea's trading for her own account.

A. II only.
B. II and III only.
C. III only.
D. I only.
E. I and III only.
F. I and II only.
G. I, II and III.

Answer: G

Explanation:
This question involves three Standards. Andrea, the portfolio manager, has been obtaining lower
prices for her personal securities transactions than she gets for her clients, which is a violation of
Standard IV (B.1), Fiduciary Duties. In addition, she violated Standard II (B), Professional Misconduct,
by failing to adhere to company policy and hiding her personal transactions from her firm. Andrea's
supervisor, Bates, violated Standard III (E), Responsibilities of Supervisors, by failing to enforce the
procedures for reporting personal trading. Therefore, Statements I, II and III are all correct.

Page | 58
Question: 148

Standard II of the Standards of Professional Conduct deals with Relationships with and
Responsibilities to ________.

A. AIMR
B. Clients
C. the Profession
D. Employers
E. None of these answers

Answer: C

Explanation:
Standard II of the Standards of Professional Conduct deals with Relationships with and
Responsibilities to the Profession.

Question: 149

According to Standard IV (B.7), Disclosure of Conflicts, service on a board of directors poses which of
the following as a basic conflict of interest?

A. Fiduciary duties owed to clients and the duties owed to shareholders.


B. None of these answers.
C. All of these answers.
D. Investment personnel who serve as directors may receive securities or the option to purchase
securities of the company as compensation for serving on the board.
E. Opportunity to receive material nonpublic information.

Answer: C

Explanation:
Under Standard IV (B.7), service as a director poses all three statements as basic conflicts of interest.
Hence, when members providing investment service serve also as directors, they should be isolated
from those making investment decisions by the use of Fire Walls or similar restrictions.

Question: 150

Standard IV (B.3) - Fair Dealing requires AIMR members to treat all clients and prospects fairly when
disseminating investment information or recommendation. Which of the following is/are implied by
this?

I. All clients and prospects must be treated equally i.e. the investment recommendation should be
disseminated to all the clients without prejudice.
II. Amongst the eligible clients, no favoritism should be shown i.e. small investors should be treated
the same as large investors.
III. All eligible clients must be informed about new opportunities simultaneously.

A. I and II only
B. II and III only
C. II only
D. I, II and III

Page | 59
Answer: C

Explanation:
Standard IV (B.3) - Fair Dealing requires AIMR members to treat all clients and prospects fairly when
disseminating investment information or recommendation. However, "fairly" does not mean
"equally" for one major reason: each client has different investment needs. A recommendation, if
not suitable for a particular client's risk-reward profile, should not be communicated to that client!
So care must be taken while disseminating investment recommendations to individual clients to
ensure suitability. Therefore, (I) is incorrect. Of the eligible clients, as long as there is no bias shown
favoring one clientover the other, the standard does not require that they be informed
simultaneously; indeed this is almost impossible in most cases because of time lags in
communication channels - mail, phone, fax, etc. So (III) is incorrect.

Question: 151

In the presentation of real estate performance, disclose any changes in ________, including
unrealized gains and losses.

A. accounting
B. investment earnings
C. valuation
D. audit practices
E. cash flow

Answer: C

Explanation:
Questions have arisen about the effects of real estate appraisals on recognizing gains or losses in
value in performance periods. Consistent with industry practices, the AIMR-PPS require that changes
in valuation, including unrealized gains and losses, be recognized in the reporting period that
includes the effective date of the appraisal.

Question: 152

Relationships with and Responsibilities to Clients and Prospects are dealt with under:

A. Standard I
B. Standard III
C. None of these answers
D. Standard II
E. Standard V
F. Standard IV

Answer: F

Explanation:
Relationships with and Responsibilities to Clients and Prospects are dealt with under Standard IV.

Question: 153

Mike Jordan recently met his old friend, Charlie Barklee, who's an accomplished statistician. Charlie
showed him his new model for predicting stock prices and after considerable discussion, Mike was
convinced the model was a viable alternative to his current methodology for picking stocks. To test it,
he downloaded historical data from a well-known statistical data provider's web site. The tests

Page | 60
indicated that some parts of the model needed fine tuning, which Mike implemented himself,
without Charlie's help. Mike:

A. cannot circulate the model since it is not publicly available.


B. can show his clients the model, without any special disclosures, because all of the tests were done
by him with publicly available data.
C. none of these answers.
D. must acknowledge Charlie's contribution. Otherwise, he would be in violation of the code of
ethics and be subject to disciplinary action from AIMR.

Answer: D

Explanation:
Standard II (C) - Prohibition against Plagiarism.

Question: 154

The ________ assumption must be disclosed if results are presented after taxes.

A. composite
B. tax rate
C. settlement-date
D. selection
E. trade

Answer: B

Explanation:
To be in compliance with the PPS, a firm's presentation of its investment performance must disclose
the tax rate information.

Question: 155

Standard IV (B.5), Preservation of Confidentiality specifically states that one may not enter into
________ that may prevent a party from cooperating with the Professional Conduct Program (PCP) in
its investigation of the member's alleged violations of the Code and Standards.

A. settlement agreements
B. confidential conversations
C. priority dealings
D. business relationshipsThat answer is correct!

Answer: A

Explanation:
Under Standard IV (B.5), members are prohibited from executing settlement agreements that may
prevent a customer or other party from providing information, documents, or testimony or
otherwise cooperating with the PCP in its investigation.

Question: 156

When dealing with a trust, according to the Prudent Man Rule, trustees must:

A. apply "all reasonable efforts" to the remaindermen.

Page | 61
B. be impartial between income beneficiaries and remaindermen.
C. take into account income beneficiaries first, then the remaindermen.
D. take into account the remaindermen first, then the income beneficiaries.
E. apply "all reasonable efforts" to the income beneficiaries.

Answer: B

Explanation:
Under the Prudent Man Rule, trustees must be impartial between income beneficiaries and
remaindermen and must achieve an equitable balance between current income and the preservation
of principal in real terms.

Question: 157

Standard IV (A.3) relates to two major components, Independence and Objectivity. Under this
Standard, modest gifts that do not exceed ________ and entertainment are acceptable, without
disclosure.

A. $100
B. one-twentieth of your annual salary and bonus
C. one-tenth of your annual salary and bonus
D. $500
E. none of these answers
F. $1,000That answer is correct!

Answer: A

Explanation:
External sources may try to influence the investment process by offering analysts and portfolio
managers a variety of "perks." The perks may include gifts, invitations to lavish functions, tickets and
so on. Modest gifts that do not exceed $100 and entertainment are acceptable, but special care
should be taken by member analysts and investment managers to resist subtle and not-so-subtle
pressures to act in a manner possibly detrimental to their clients.

Question: 158

Jay Simpson was recently convicted of a felony in the state of California. Jay is a resident of Arizona,
where he conducts all his business. The felony conviction resulted from activities not related to his
primary business. With regard to standard II.B-Professional Misconduct, Jay has:

A. violated the standard since felony convictions are considered professional misconduct.
B. not violated the standard since the conviction occurred in a non-resident state.
C. not violated the standard since AIMR does not impose sanctions for felony convictions.
D. not violated the standard since the conviction is for non-business activities.
E. none of these answers.That answer is correct!

Answer: A

Explanation:
Standard II (B) is violated if convictions for felony or similar offenses occur, regardless of whether the
offenses were related to the member's professional activities or not.

Question: 159

Page | 62
Which of the following will result in an insider trading prosecution?

I. A tippee trades based on inside information, not knowing that the source of the information has
actually received the information illegally and has breached fiduciary duty to the shareholders by
leaking it. He has no reasons to suspect such a behavior.
II. A corporate outsider who has absolutely no connections to any of the insiders misappropriates
inside information and trades for profit based on that information.
III. A tippee trades based on inside information about an impending tender offer after verifying that
the source of the information has not breached any fiduciary duty to the company.

A. I only
B. III only
C. I, II and III
D. II and III only

Answer: D

Explanation:
A "Tippee" is any person who learns inside information from a corporate insider. A tippee can be held
liable under SEC's section 10b and rule 10(b)-5 only if he knows or should know that the insider has
behaved improperly and breached fiduciary duty in revealing the information. Otherwise, the tippee
is not held responsible for insider trading. One exception to this rule is information about tender
offers, in which tippee liability results automatically under SEC Rule 14e-3, even if the insider has not
breached any fiduciary duty. Under the Misappropriation Theory of insider trading, a person who
misappropriates inside information and trades or abets trading in it is committing securities fraud
and can be so charged, even if the person is a complete corporate outsider. The Topical Study,
"Insider Trading," is probably the most important (and the trickiest) section in the handbook and you
can expect at least one question from it.

Question: 160

An AIMR member:

I. must report any criminal activity to the appropriate legal or regulatory authority.
II. may be held responsible for any illegal activity he associates himself with.
III. may be held liable for violations by others when he is unaware of the facts giving rise to the
violations.
IV. can be held liable for violations if he does not take actions to dissociate himself from them.

A. I, II, III and IV


B. II and IV only
C. I, II and IV only
D. II, III and IV only

Answer: B

Explanation:
Standard I does not require a member to report the illegal activity to the appropriate regulatory
authorities, though it may be prudent to do so in many cases if it is evident that the activity is illegal.

Question: 161

Mark Blazewhich, an analyst with an investment advisory firm, changed the recommendation on
Zomega, Inc.'s stock from hold to sell 10 days ago. This recommendation was published in the firm's

Page | 63
newsletter that came out two days after this announcement. Blazewhich has just given his broker
instructions to liquidate Blazewhich's personal holdings of Zomega. Blazewhich has:

A. violated Standard IV (B.3) - Fair Dealing.


B. violated Standard IV (B.7) - Disclosure of Conflicts to Clients and Prospects.
C. violated Standard IV (B.4) - Priority of Transactions.
D. not violated any standard of AIMR code.

Answer: D

Explanation:
Because Blazewhich has allowed sufficient time for dissemination of the change in recommendation,
he is free to carry out personal transactions. Also, since the recommendation is to sell the stock,
there is no personal conflict in the situation. Note that it would be a violation of Standard IV (B.7) -
Disclosure of Conflicts to Clients and Prospects - if the recommendation were to buy the stock and
Crumpet had failed to reveal information about his personal holdings.

Question: 162

Which one of the following actions will not help to ensure the fair treatment of brokerage firm clients
when a new investment recommendation is made?

A. Minimize elapsed time between the decision and the dissemination of a recommendation.
B. Limit the number of people in the firm who are aware in advance that a recommendation is to be
disseminated.
C. Distribute recommendations to institutional clients prior to individual accounts.
D. Monitor the trading activities of firm personnel.

Answer: C

Explanation:
This question deals with Standard IV (B.3), Fair Dealing. All the procedures listed will help members
treat clients fairly, except distributing recommendations to institutional clients prior to individual
accounts. This practice discriminates among clients based on size and class of assets and is a
violation of Standard IV (B.3).

Question: 163

Joan, an investment counselor, states in her firm's written promotional material that she is a CFA
candidate. She has indeed enrolled in the program, but failed Level I five years ago, has not taken any
exams since and is not enrolled in the next exam. Which of the following statements is correct?

A. Joan is in compliance with the correct use of the CFA designation because she states she is a
candidate.
B. Joan is not in compliance with the correct use of the CFA designation. To be in compliance she
must state she is a Level I candidate.
C. None of these answers.
D. Joan is not in compliance with the correct use of the CFA designation. Because she is not
registered for the next exam, she is not a candidate.

Answer: D

Explanation:
Under Standard II (A) - Use of Professional Designation, a person must be registered to take the next

Page | 64
scheduled CFA exam to be a "candidate" in the CFA Program. Therefore, because Joan is not
scheduled to take the next exam, she is not a candidate. There is no designation for someone who
has passed Level I, II, or III. However, candidates may state that they have completed Level I, II, or III,
as the case may be.

Question: 164

Standard V (A), Prohibition against Use of Material Nonpublic Information applies to

A. all members of the investment community.


B. recipients of information who are not directly or indirectly associated with the firm the material
nonpublic information is about.
C. recipients of information who are directly or indirectly associated with the firm the material
nonpublic information is about.
D. none of these answers.

Answer: B

Explanation:
Standard V (A), Prohibition against Use of Material Nonpublic Information applies to recipients of
information who are not directly or indirectly associated with the firm the material nonpublic
information is about. Such persons include research analysts, portfolio managers, venture capitalists,
pension sponsors, investor relations executives, rating agencies, trust officers and others who make
or recommend investment decisions.

Question: 165

Which of the following is/are required by AIMR-PPS with regards to calculation of returns?

I. The return for after-tax composites that hold both taxable and tax-exempt securities should be
stated on an equivalent, "pre-tax" basis.
II. Real Estate must be appraised annually unless client agreements state otherwise.
III. For commingled fund-of-funds structure, segregated Irrs net of trading expenses must be
presented.

A. I, II and III
B. III only
C. I and III only
D. none of them

Answer: D

Explanation:
The return for after-tax composites that hold both taxable and tax-exempt securities should be stated
on an "after-tax" basis and should not be "grossed" up. Real Estate investments must be appraised
every three years and the valuations reviewed quarterly unless client agreements state otherwise.
For separately managed accounts commingled fund-of-funds structure, cumulative Irrs net of trading
expenses must be presented.

Question: 166

Which of the following disclosures must a firm make in order to be in compliance with AIMR-PPS?

I. The effective date of firm compliance.

Page | 65
II. A measure of the dispersion of individual component portfolio returns around the aggregate
composite return.
III. The existence of a minimum asset size below which portfolios are excluded from the composite.

A. I, II and III
B. I and III only
C. I only
D. I and II onlyThat answer is correct!

Answer: A

Explanation:
All of these are mandatory disclosures under AIMR-PPS.

Question: 167

The ultimate responsibility to ensure compliance with code rests with:

A. every member of the firm.


B. the CEO of the firm.
C. all of these answers.
D. the highest ranking AIMR member of the firm.

Answer: D

Explanation:
While every member must always comply with the Code, Standard III (A) stipulates that ensuring
compliance with the Code in an organization ultimately is the responsibility of the senior most AIMR
member of the firm reporting to a non-member. He must make sure that the firm environment is
sympathetic to compliance with the Code.

Question: 168

Arbliss, an AIMR member, has been working for a financial software company, Supersoft Corp. for the
past 3 years. He now wants to leave the firm and start his own software proprietorship. Which of the
following will not constitute a violation of the AIMR code?

I. Making arrangements to set up the proprietorship prior to leaving current employment, without
Supersoft knowing about it.
II. Soliciting Supersoft's clients prior to the termination of employment.
III. Using a confidential software error-checking program he had developed for Supersoft for his own
purposes.

A. I only
B. III only
C. II and III only
D. I and III onlyThat answer is correct!

Answer: A

Explanation:
Standard III (B) - Duty to Employer - does not preclude a member from entering into independent
practice while still employed. It does require the employee to obtain written permission from the
employer as well as the outside entity before doing this. It does not prohibit a departing employee

Page | 66
from making arrangements to enter independent practice prior to leaving current employment, as
long as such preparations do not constitute a breach of loyalty toward the employer. Activities that
can be a breach of loyalty include misappropriation of trade secrets, misuse of confidential
information, solicitation of the firm's clients before the termination of unemployment and in
general, any activity that would destabilize the firm's operations.

Question: 169

Relationships with and Responsibilities to the Investing Public are dealt with under:

A. Standard V
B. None of these answers
C. Standard III
D. Standard II
E. Standard IV
F. Standard IThat answer is correct!

Answer: A

Explanation:
Relationships with and Responsibilities to the Investing Public are dealt with under Standard V.

Question: 170

The ________ states that broker/dealers should be subject to strict standards because they are
advertising themselves as market professionals.

A. cover canon
B. canopy principle
C. posting criterion
D. shingle theory
E. marketing regulation

Answer: D

Explanation:
The shingle theory states that broker/dealers should be subject to strict standards because they are
"hanging out their shingles," or, advertising themselves as market professionals. The theory states
that inherent in the relationship between brokers/dealers and their clients is the representation that
the customer will be dealt with fairly. Brokers/dealers are in a position to exploit their customers'
trust and ignorance and therefore they are held to a higher duty of care.

Question: 171

According to the AIMR PPS, composites must include new portfolios at the start of the next
performance measurement period, ________, after the portfolio comes under management or
according to reasonable and consistently applied firm guidelines.

A. at least annually
B. at least semi-annually
C. at least quarterly
D. at least monthly

Answer: C

Page | 67
Explanation:
Composites must include new portfolios at the start of the next performance measurement period
(at least quarterly) after the portfolio comes under management or according to reasonable and
consistently applied firm guidelines. This is a requirement for creation and maintenance of
composites.

Question: 172

Procedures for compliance with Standard I include ________.

A. activities relating to interns and people under contract


B. keep informed
C. when you suspect illegal activity, determine legality
D. when you suspect illegal activity, dissociate from that activity
E. all of these answers
F. maintain current files
G. review procedures

Answer: E

Explanation:
Members should maintain current reference copies of applicable statutes, rules and regulations in a
readily accessible manner. Members should establish a procedure by which employees are kept
informed about changes in applicable laws. Members should review procedures on a regular basis to
ensure that they reflect current law and provide adequate guidance. When a member suspects illegal
activity is being conducted, the member should consult counsel to determine the conduct's legality
and dissociate from the conduct if there are reasonable grounds to believe that it is illegal. This
applies to the activities of interns or contractors as well.

Question: 173

What is the effective date for compliance with the AIMR-Performance Presentation Standards for
including accrued income in market value performance calculations?

A. January 1, 1997
B. January 1, 1993
C. July 1, 1995
D. January 1, 1992That answer is correct!

Answer: A

Explanation:
From January 1, 1997, going forward, all of the firm's composites and performance presentations
must include accrued income in market value performance calculations. In addition, all of the firm's
performance presentations, including presentations of historical performance, must contain a
measure of composite dispersion.

Question: 174

Wolfram Hitchwalker is a money manager with Armadillo Investments. He currently manages a few
retirement accounts, clients who have a steady current income need and are averse to capital loss.
Wolfram recently read a research report which concluded that the stock of HighFly, Inc. was a great
buy because of a pending expansion plan into Southeast Asia which would double the profits of

Page | 68
HighFly from foreign operations. Wolfram decided that the analysis was sound and that his clients
could gain significantly if he bought the HighFly stock now and sold it once the price run-up occurred.
Accordingly, he sold some of the fixed income securities in his client accounts and bought shares of
HighFly. After two weeks, he sold the shares at a substantial profit and reinvested the funds back in
fixed income securities. Wolfram has

I. not violated any code of ethics since the investment was wise and made his clients better off.
II. has violated Standard IV (A.1) - Reasonable Basis & Representations.
III. has violated Standard IV (B.1) - Fiduciary Duties.
IV. has violated Standard IV (B.2) - Portfolio Investment Recommendations and Actions.

A. II and IV only
B. III and IV only
C. I only
D. II only
E. IV only
F. III onlyThat answer is correct!

Answer: A

Explanation:
Even though the transaction turned out to be profitable for the clients ex post, the decision to invest
in the stock was unwise ex ante. Wolfram should have recognized that his clients do not have the risk
appetite for speculative securities, given their need for current income and preservation of the
principal. Clearly, speculation in stocks is not an appropriate investment for these clients.

Question: 175

According to Standard V (B), firms can claim compliance with the PPS only if their presentation is
________ in compliance with the Performance Presentation Standards in all material respects.

A. none of these answers


B. retroactively
C. fully
D. at least 50 percent
E. partially

Answer: C

Explanation:
If members or firms want to claim compliance with PPS, their presentation must be fully in
compliance with the PPS in all material respects. To claim compliance with the PPS without meeting
the mandatory requirements of the PPS is a violation of Standard V (B).

Question: 176

________ provide a relative measure for the riskiness of a strategy.

A. Volatility measures
B. Benchmarks
C. Indexes
D. Investor universes
E. Disclosures

Page | 69
Answer: B

Explanation:
Benchmarks, including market indexes, manager universes and normal portfolios, provide a relative
measure for the riskiness of a strategy.

Question: 177

Shortin Mart is a quantitative research analyst with Dataminers, an investment advisory firm which
prides itself on finding patterns in past market data. Shortin recently used data on high-yield,
distressed firm corporate convertible bonds and discovered that over the last 3 years, this class has
generated an astounding 76% rate of return (assuming optimal conversion). Realizing that this result
is mainly due to a strong bull market, he recommends to his portfolio managers that if they believe
the market will be bullish over the next year, they should add extreme junk bonds to their portfolios.
Shortin has

I. not violated any code of ethics.


II. has violated Standard IV (A.1) - Reasonable Basis & Representations.
III. has violated Standard IV (A.2) - Research Reports.
IV. has violated Standard IV (B.2) - Portfolio Investment Recommendations and Actions.

A. III and IV only


B. II and III only
C. I only
D. II only

Answer: D

Explanation:
In the present case, Shortin was negligent in that he did not carry out a through analysis of his result.
For one, he should have used a much longer period to determine a pattern. Second, he should have
been careful in laying out all the caveats to his result and even questioned the validity of the result.
By not following such a course, he violated Standard IV (A.1) - Reasonable Basis & Representations.
Note that he has not violated Standard IV (A.2) - Research Reports.

Question: 178

The primary determinant of a fiduciary's powers and duties are to be found in the ________.

A. board of director's meeting minutes


B. governing documents (trust documents & investment management agreements)
C. FDIC General Rules
D. ERISA Statement of Procedures
E. none of these answers
F. ERISA Funding Guidelines

Answer: B

Explanation:
Fiduciaries must manage any pool of assets in their control in accordance with the terms of the
governing documents (such as trust documents and investment management agreements), which
are the primary determinant of a fiduciary's powers and duties. Whenever their actions are contrary
to provisions of those instruments or applicable law, fiduciaries are exposed to liability through
litigation brought by parties at interest.

Page | 70
Question: 179

Which of the following can be found in Standard V?

A. Members shall not undertake any independent practice in competition with employer without
written consent.
B. Members shall not participate in plagiarism.
C. Members shall make reasonable efforts to achieve public dissemination of material nonpublic
information disclosed in breach of a duty.
D. Members shall maintain appropriate records to support the reasonableness of recommendations.
E. Members shall maintain knowledge of and comply with all applicable laws.

Answer: C

Explanation:
Standard V states: "Members shall make reasonable efforts to achieve public dissemination of
material nonpublic information disclosed in breach of a duty."

Question: 180

Which of the following is not a violation of Standard II (C)?

A. Using a chart that was prepared by another analyst in a presentation, without acknowledgment.
B. All of these answers are violations.
C. Giving an oral report and citing specific quotations, attributable to "leading analysts," without
specific reference.
D. Use of statistical information provided by Standard & Poor's, without acknowledgment.
E. Use of a small part of an analyst's work who is employed in a completely different (non-
competitive) industry, without acknowledgment.

Answer: D

Explanation:
Using excerpts from articles or reports prepared by others, either verbatim, or with only a slight
change, without acknowledgment; citing specific quotations, attributable to "leading analysts,"
without specific reference; using charts or graphs without stating their sources; and copying
proprietary computerized spreadsheets or algorithms without seeking the authorization of their
creators - all of these practices are violations of Standard II (C). Only globally recognized sources of
factual material such as that provided by Standard & Poor's, or Moody's Investors Service, can be
used without acknowledgment, since such information is already in the public realm.

Question: 181

Christine Crumbwell and Dorothy Drummond are two portfolio managers with Neptune Funds.
Cristine is managing the personal trust fund of Paul Roker, who created the fund as an income
support for his wife and a legacy for his two sons after his wife's death. Dorothy is in charge of a fund
created by Katey Koric. Katey had started this fund as a long-term investment but recently decided to
shift the asset mix toward municipal and high-income bonds. Her friend pointed out a great
investment opportunity in the newly issued, high-yield-high-income Orange County bonds and Katey
instructed Dorothy to sell off a large chunk of the stock holdings in the fund and reinvest in the
Orange County bonds. Dorothy spoke to Christine about this and they both agreed that the bonds
were an excellent buy. Dorothy carried out Katey's instructions and Christine decided that Paul's
portfolio would be better off if she sold some of the small cap stocks and bought the bonds and

Page | 71
followed Dorothy's suit.

I. Dorothy has violated Standard IV (B.1) - Fiduciary Duties by not discussing the issues further with
Katey.
II. Christine has violated Standard IV (B.1) - Fiduciary Duties by tilting the portfolio mix toward high-
income instruments.
III. Katey has violated Standard V (A) - Prohibition Against Use Of Non-Public Information.

A. I, II and III
B. I and II only
C. I only
D. II only

Answer: D

Explanation:
Christine, as a manager of a personal trust, has to balance the interests of the income beneficiaries
and the remaindermen who need capital appreciation. By tilting the asset mix toward high-income
bonds, Christine has violated her fiduciary duties toward Paul Roker's sons by effectively transferring
some of their wealth to their mother. On the other hand, Dorothy has violated no such duty since she
in charge of an advisory account. The account belongs to Katey alone and she is free to instruct
Dorothy to change investments as she pleases. Finally, the Orange County bonds have already been
issued and there is no misuse of any inside information so Katey cannot be accused of insider
trading. Standard IV (B.1) - Fiduciary Duties

Question: 182

Which of the following relating to procedures for complying with Standard III (E) is false? The
compliance procedures should:

A. outline permissible conduct.


B. delineate procedures for reporting violations and sanctions.
C. be easy to understand.
D. outline the scope of the procedures.
E. designate a compliance officer.
F. none of these answers.

Answer: F

Explanation:
All the statements are true. Adequate compliance procedures should be drafted so that they are easy
to understand. They should designate a compliance officer and clearly define the officer's authority
and responsibility; outline the scope of the procedures; outline permissible conduct and delineate
procedures for reporting violations and sanctions.

Question: 183

By blindly adopting the ideas and works of others without acknowledgment, you have definitely
violated Standard ________. You also may have violated Standard ________ because you may be
making recommendations without a reasonable basis.

A. II (A); IV (A.1)
B. II (A); IV (B.1)
C. II (A); IV (A.2)

Page | 72
D. II (C); IV (B.2)
E. II (C); IV (A.2)
F. II (C); IV (B.1)
G. II (C); IV (A.1)
H. II (A); IV (B.2)

Answer: G

Explanation:
This is explicitly stated under Standard II (C) - Prohibition against Plagiarism.

Question: 184

In reference to AIMR-PPS, which of the following is/are true?

I. If the valuation of an investment asset is not based on current market value, it should not be
included in total assets to which PPS are being applied.
II. Composites cannot not be presented as being in compliance unless all the firm'squalifying
portfolios are accounted for in at least one composite defined according tosimilar strategy or
investment objective.
III. A subsidiary may claim to be in compliance with the PPS even if the parent firm is not.

A. I, II and III
B. I and III only
C. II and III only
D. II onlyThat answer is correct!

Answer: A

Explanation:
Assets to which AIMR-PPS cannot be applied are not to be considered while considering compliance.
For example, investment instruments like the GIC whose valuation is not based on market
conditions, must be excluded.

Question: 185

Standard III (A) states that members notify their ________ of the Code and Standards. This
notification must be done ________.

A. nearest Society secretary; within 45 days of candidacy


B. immediate supervisor; in writing
C. nearest Society secretary; orally or in writing
D. chief operating officer (or equivalent); in writing
E. chief operating officer (or equivalent); orally or in writing
F. immediate supervisor; orally or in writing
G. immediate supervisor; within 45 days of candidacy
H. any of these answers is acceptable; as long as the notification is made

Answer: B

Explanation:
Standard III (A): Relationship with and Responsibilities to the Employer, states that: "Members shall
inform their employer, in writing, through their direct supervisor, that they are obligated to comply
with the Codes and Standards and are subject to disciplinary sanctions for violations thereof."

Page | 73
Question: 186

Level I verification requires independent attestation that the requirements of the AIMR-PPS have
been met on a(n) ________ basis.

A. international
B. nationwide
C. firmwide
D. attainable

Answer: C

Explanation:
This is a requirement under Level I verification procedures.

Question: 187

Arditti manages the pension plan for a publicly traded firm, Eros Computers and in this capacity, has
the right to vote 0.5% of the common shares outstanding. Recently, Eros wanted to expand its
production operations into Latin America and the board had decided to put the matter for open vote
from shareholders. Arditti was on vacation in Fiji at the time and when she was informed about the
vote, instructed that the portfolio proxy votes be voted along the same lines as that favored by the
senior management. Arditti herself has not studied the merits of the proposed expansion plan but
has complete faith in the senior management of Eros, which has always proven to be conscientious
and prudent. Arditti has

I. violated Standard IV (B.1) - Fiduciary Duties by voting the proxy shares in an indiscriminate manner.
II. can be held liable for failing in her duties under ERISA.
III. violated Standard II (B) - Professional Misconduct by behaving in an unprofessional manner.

A. I and II
B. I, II and III
C. I only
D. II and III onlyThat answer is correct!

Answer: A

Explanation:
Corporate pension plan managers are governed by ERISA (Employee Retirement Security Act, 1974).
One of the fiduciary duties expected under these guidelines is the voting of proxy shares. A fiduciary
who fails to vote the proxy votes, votes them without analysis, or votes them blindly with the
management on non-routine decisions will be in violation of ERISA. They can be held liable for trust
violation unless there are explicit provisions in the plan which preclude the manager from voting.
Note, however, that Arditti's voting does not constitute professional misconduct. Standard II (B) -
Professional Misconduct - and Standard IV (B.1) - Fiduciary Duties.

Question: 188

Which of the following is true regarding Standard II (A)?

A. You must be registered for the next CFA exam in order to call yourself a candidate.
B. All of these answers.
C. Candidates may state that they have completed Level I, II, or III.

Page | 74
D. There is no designation for someone who has passed Level I, II, or III.
E. This standard relates to oral statements.
F. This standard relates to business cards and letterheads.

Answer: B

Explanation:
Standard II (A) relates to the responsibility of AIMR members and candidates to use their
professional designation properly and in a non-misleading manner. A person must be registered to
take the next scheduled CFA exam to be a "candidate" in the CFA program. There is no designation
for someone who has passed Level I, II, or III of the CFA exam. Candidates may state, however, that
they have completed Level I, II, or III. The standard applies to all related explanations or descriptions
of the CFA designation, including letterheads and business cards, resumes, directory listings, printed
advertising, brochures and oral statements to clients and prospects.

Question: 189

Which of the following statements is correct regarding Standard II (A) - Use of Professional
Designation?

A. Joe Martin passed Level I and Level II of the CFA exams and is scheduled for the next Level III
exam. He may write "Joe Martin, CFA II."
B. Joe Martin passed Level I and Level II of the CFA exams and is scheduled for the next Level III
exam. He may write "Joe Martin, CFA III."
C. Joe Martin passed Level I and Level II of the CFA exams, but is not scheduled for the next Level III
exam. He may state, "I am a CFA candidate."
D. None of these statements are correct.

Answer: D

Explanation:
"Joe Martin, CFA II" and "Joe Martin, CFA III" are misrepresentations and a violation of Standard II
(A). There is no designation for someone who has passed Level I, Level II, or Level III. He may not
state he is a candidate unless he is registered for the next exam.

Question: 190

Composite ________ measures represent the consistency of a firm's composite performance results
with respect to the individual portfolio returns within a composite.

A. disclosure
B. fee
C. dispersion
D. deviation
E. error

Answer: C

Explanation:
The AIMR-PPS require that managers disclose the dispersion of portfolio returns within each
composite.

Question: 191

Page | 75
Under the Performance Presentation Standards, selected firm composites can be designated as being
in compliance, only if ________ of the firm's composites are in compliance.

A. all
B. many
C. some
D. oneThat answer is correct!

Answer: A

Explanation:
Compliance cannot be met on a per product or per composite basis, but can only be met on a
firmwide basis. Composites may not be presented as being in compliance unless all of the firm's
qualifying portfolios have been accounted for in at least one composite defined according to similar
strategy or investment objective.

Question: 192

Performance Presentation Standards require the disclosure of whether composite returns on


international investments are ________ of foreign withholding taxes.

A. net
B. gross
C. none of these answers
D. net or gross

Answer: D

Explanation:
The performance presentation must disclose whether composites and benchmarks are presented
gross or net of withholding taxes on dividends, interest and capital gains; if net, the assumed tax rate
for both the composite and the benchmark.

Question: 193

According to the AIMR-PPS, ________ accounting must be used for fixed-income and all other
securities that accrue income.

A. time-weighted
B. LIFO
C. accrual
D. fixed income

Answer: C

Explanation:
Accrual accounting must be used for fixed-income and all other securities that accrue income.
Accrued income must be included in the market value calculation of the denominator and the
numerator. This is a requirement for calculation of returns.

Question: 194

Standard III includes which of the following?

Page | 76
A. Performance Presentation
B. All of these answers
C. None of these answers
D. Reasonable Basis and Representations
E. Disclosure of Additional Compensation Arrangements
F. Use of Professional Designation

Answer: E

Explanation:
Standard III deals with Obligation to Inform Employer of Code and Standards, Duty toEmployer,
Disclosure of Conflicts to Employer, Disclosure of Additional CompensationArrangements and
Responsibilities of Supervisors.

Question: 195

According to The Code of Ethics, members of AIMR shall: "Use reasonable care and exercise
________."

A. each and every day


B. none of these answers
C. independent professional judgment
D. optimal diversification
E. open communications with clients

Answer: C

Explanation:
According to The Code of Ethics, members of AIMR shall: "Use reasonable care and exercise
independent professional judgment."

Question: 196

Carlos recently attended a seminar on a new technique in identifying possible relative mispricings
amongst stocks. Impressed by the presentation, he used the technique to suggest that two of his
biggest clients rebalance their portfolios with a heavier emphasis on the stocks the technique
identified as being mispriced. While presenting this suggestion to his clients, Carlos did not mention
anything about the source of the ideas used. Before recommending the changes, he did not
personally check the results nor did he try to find out if there were any significant caveats or
assumptions associated with the methodology. He has:

I. violated Standard II (B) - Professional Misconduct.


II. violated Standard II (C) - Prohibition against Plagiarism.
III. violated Standard IV (A.1) - Reasonable Basis & Representations.
IV. violated Standard IV (B.1) - Fiduciary Duties.

A. I, II and III
B. I and III only
C. I, II, III and IV
D. II and III only

Answer: D

Explanation:

Page | 77
By not mentioning the sources of the results used, Carlos violated AIMR Standard II (C) related to
Plagiarism. By not exercising reasonable care in verifying the soundness of his recommendations, he
violated Standard IV (A.1). However, Standard II (B) - Professional Misconduct - and Standard IV (B.1)
- Fiduciary duties are not directly relevant to this situation.

Question: 197

Tiarera has just landed a big account for her fund and is in a joyous mood. To celebrate, she decides
to go out to a bar with some of her friends and share some drinks. Tiarera, however, has a weak
constitution and cannot handle alcohol very well. In fact, many times in the past, she has gotten into
drunken altercations at bars. Tonight turns out to be no different; Tiarera and her friends are arrested
for disorderly conduct and convicted for misdemeanor. Tiarera has

A. has violated Standard II (B) - Professional Misconduct - because of a pattern of misconduct.


B. not violated any AIMR standard since the arrest was for a misdemeanor, an offense not serious
enough to trigger sanctions on the first occurrence.
C. has violated Standard II (B) - Professional Misconduct - because she has been involved in a
misdemeanor.
D. not violated any AIMR standard since the arrest took place due to activities not related to her
work.That answer is correct!

Answer: A

Explanation:
A single instance of misdemeanor is generally not serious enough to constitute a violation of
Standard II (B). However, Tiarera has shown a pattern of professional misconduct that reflects poorly
on her firm and her profession, thus violating the AIMR standard.

Question: 198

Which of the following is true about Standard III (B) - Duty to Employer?

A. It prohibits a departing employee from making arrangements to enter independent practice prior
to leaving current employment.
B. It precludes a member from entering into an independent business that is in direct competition
with the employer.
C. It requires the employee to obtain written permission from the employer as well as the outside
entity before entering independent practice.
D. It requires departing employee to give all the details about the independent practice before
leaving the current employment.

Answer: C

Explanation:
Standard III (B) - Duty to Employer - does not preclude a member from entering into independent
practice while still employed. It does require the employee to obtain written permission from the
employer as well as the outside entity before doing this. It does not prohibit a departing employee
from making arrangements to enter independent practice prior to leaving current employment, as
long as such preparations do not constitute a breach of loyalty toward the employer.

Question: 199

Standard III (D) - Disclosure of Additional Compensation Arrangements - requires members to


provide complete disclosure to their employer about any additional compensation arrangements. In

Page | 78
order to abide by this, you must:

A. Inform the legal department in writing.


B. Inform your immediate supervisor in writing or by email.
C. Inform senior management orally or in writing.
D. Inform your supervisor in writing, by email or orally.

Answer: B

Explanation:
Standard III (D) requires written notification to the employer. This includes any form of
communication that can be documented.

Question: 200

Institutional Investments is a fund management firm that provides investment advice as well as
investment management to large client accounts. I.I.'s senior portfolio manager, Richard Fineman's
cousin, Alberto Onestone, runs a small brokerage firm. Fineman has directed all of his junior portfolio
managers to execute their trades with Alberto. Alberto's fees are higher than those available from
discount brokers but Alberto provides Fineman and his managers a wealth of investment tips that he
hears on the street. This helps Fineman in producing a successful investment newsletter. This helps
enhance I.I.'s reputation, attracting new clients and allowing lower fees on all the accounts. Since the
client accounts benefit from this arrangement, Fineman's managers do not think the practice is
unethical. The relationship between Fineman and Onestone is not explicitly disclosed to the clients.
You are currently interviewing with them for the position of a portfolio analyst. As a CFA Level I
candidate, they portray the above situation as a hypothetical case and ask you if there are any
violations of the AIMR code of Ethics that they have heard about. Your response should be which of
the following?

I. Fineman and his managers are violating Standard IV (B.1) - Fiduciary Duties since they are not
executing the trades at the lowest price available.
II. Fineman and his managers are violating Standard IV (B.1) - Fiduciary Duties since they are using
client brokerage to pay for services that do not directly benefit them.
III. Fineman and his managers are violating Standard IV (B.7) - Disclosure of Conflicts to Clients and
Prospects.
IV. Fineman and his managers are not violating the Code as far as you can tell from the details given.

A. I, II and III
B. IV only
C. II only
D. II and III only

Answer: D

Explanation:
By not revealing the relationship between Fineman and Onestone - which is clearly a source of
conflict of interest - the managers of Institutional Investors are in violation of Standard IV (B.7) -
Disclosure of Conflicts to Clients and Prospects. Further, while there is a symbiotic relationship
between I.I. and Onestone to the extent that both the parties gain, the clients who pay higher
brokerage lose. Even though the practice leads to lower fees, it is not clear that the clients get
benefits worth the cost incurred on their accounts. Further, there is a clear sense of impropriety in
the setup. Unless the excess brokerage directly benefits the client accounts, this case is a violation of
Standard IV (B.1) - Fiduciary Duties.

Page | 79
Question: 201

"Prohibited transactions" are discussed in Standard IV (B.4), Priority of Transactions. Which of the
following is NOT suggested as a prohibited transaction for investment personnel?

A. equity-based IPOs
B. private placements
C. equity securities
D. none of these answers
E. municipal bonds

Answer: E

Explanation:
Under the compliance procedures for Standard IV (B.4), members and their firms should clearly
define prohibited transactions so that employees completely understand their obligations to clients
and their employer. Participation by investment personnel in equity or equity-based IPOs should be
restricted. Members' firms should, for example, impose strict limits on investment personnel
acquiring securities in private placements. Managers should not be prevented, however, from
purchasing government issues, such as municipal bonds and/or other government securities.

Question: 202

A manager who pays a higher commission than would normally be paid to purchase the goods or
services is said to be ________ services.

A. exploiting
B. undercharged for
C. paying up for
D. fee-bound for
E. none of these answers

Answer: C

Explanation:
A manager who pays a higher commission than would normally be paid to purchase the goods or
services is said to be paying up for services. This practice is a violation of fiduciary duties to the
client. However, various countries' securities laws permit a manager to pay up for goods and services
without violating the manager's fiduciary duty so long as the requirements of the law are followed.

Question: 203

Latham has been following a publicly traded oil firm, Techron Oil. Recently, he read a review about
the oil fields in Venezuela, to the effect that the actual oil content of the fields across the country
might be as much as 35% higher than previously estimated. Techron owns sizable properties in
Venezuela and Latham has concluded that the research report implies that Techron might have as
much as a million gallons a day in excess capacity that has not been factored into the stock price. In
his report, he properly attributed the original review as the source of his information and
recommended that in his opinion, Techron's stock was an immediate BUY. Latham has

I. violated Standard V (A) - Prohibition against Use of Material, Non-Public Information. II. violated
Standard IV (A.2) - Research Reports by failing to distinguishing between fact and opinion.
III. violated Standard IV (A.1) - Reasonable Basis and Representations.
IV. violated Standard II (C) - Prohibition against Plagiarism.

Page | 80
A. II, III and IV only
B. III only
C. none of them
D. I and III only

Answer: B

Explanation:
Since the review was publicly available, Latham has not violated Standard V (A) - Prohibition against
use of Material, Non-Public Information. Also, because he properly attributed his original source, he
has not plagiarized according to Standard II (C). However, by not carrying out an independent study
to verify that his conclusions were indeed warranted, he violated Standard IV (A.1) - Reasonable
Basis andRepresentations. Note that he has not confused fact with opinion and therefore not
violated Standard IV (A.2) - Research Reports.

Question: 204

Which of the following is NOT a violation of Standard III (B) - Duty to Employer?

A. Using the current employer's client list for personal independent practice.
B. Rendering services to independent clients without informing them about your current
employment status but after obtaining written permission from the current employer.
C. Provide free tax assistance to "low-income" people on a volunteer basis, without expecting or
accepting any compensation when some of these people can be potential clients of your current
employer.
D. None of these answers.

Answer: C

Explanation:
Standard III (B) prohibits an employee from going into a competitive business while currently
employed without obtaining written permission. However, this applies only if the independent
practice is for remuneration. Strictly volunteer activities (such as helping low-income people with
free financial advice) which cannot be construed as being done for potential compensation do not
constitute a violation of the standard even if they result in some loss of business to the current
employer. However, the code is not explicit on this point. Obviously, "free tax assistance" to high-
income people will never be considered a volunteer activity.

Question: 205

A(n) ________ is a publicly available independent representation of the market and if used as a
benchmark, should be investable.

A. mutual fund
B. convertible bond
C. portfolio
D. composite
E. index

Answer: E

Explanation:
The most commonly used benchmark for an investment strategy is a market index. Indexes can be

Page | 81
mixed to represent an allocation among markets. (Note: Not all indexes are investable.)

Question: 206

Which of the following statements about the AIMR Performance Presentation Standards is true?

A. All aspects of the AIMR Performance Presentation Standards are mandatory.


B. None of these answers is true.
C. All aspects of the AIMR Performance Presentation Standards are recommended. They are simply
guidelines and don't necessarily need to be followed.
D. Some aspects of the AIMR Performance Presentation Standards are mandatory, while others are
recommended.

Answer: D

Explanation:
Some aspects of the AIMR Performance Presentation Standards are mandatory (i.e., they must be
followed to claim compliance); other aspects are strongly recommended (i.e., they should be
followed). Note that AIMR strongly encourages firms to adopt the recommended Standards in
addition to the required standards.

Question: 207

Julius Christus is an AIMR member and a vice president in the Risk Control department of a major
commercial bank. Julius recently discovered that Jill Chapman had been grossly negligent in her
duties involving monitoring of trading limits and violations by the trading desk. This is the first time
Jill has been found to be remiss in her duties. Which of the following actions is the minimum
necessary for Julius to be in compliance with Standard III (E) - Responsibilities of Supervisors?

A. Initiate an inquiry and take steps, such as placing limits on Jill's activities and increasing the
monitoring of her activities to ensure that future violations do not occur.
B. Determine the extent of the violations and warn Jill in no uncertain terms that a repeat incidence
would lead to her losing the job.
C. Terminate Jill's employment.
D. Report the misconduct to his superiors and warn Jill in writing that future misconduct would lead
to immediate termination of employment.That answer is correct!

Answer: A

Explanation:
"Procedures for Compliance," Standard III (E).

Question: 208

Which of the following can be found in Standard II?

A. Members shall not knowingly participate or assist in any violation of laws, rules, or regulations.
B. Members shall not participate in any professional conduct involving dishonesty, fraud, deceit, etc.
C. Members shall deliver a copy of the Code to their employer.
D. Members shall not misrepresent investment performance.
E. Members shall exercise diligence and thoroughness in making investment recommendations or in
taking investment actions.

Answer: B

Page | 82
Explanation:
Standard II states that members shall: "Not participate in any professional conduct involving
dishonesty, fraud, deceit, or misrepresentation or commit any act that reflects adversely on their
honesty, trustworthiness or professional competence."

Question: 209

To fulfill the basic provisions of Standard IV (B.2), a member should ________.

A. stay free of all conflicts of interest


B. develop an investment policy statement for each client
C. discuss the proxy voting policy with management
D. disclose to clients, all additional compensation agreements
E. disclose all soft dollar arrangements

Answer: B

Explanation:
In formulating an investment policy for the client, the member should take into consideration client
identification, investor objectives, and investor constraints.

Question: 210

Each of the following is true regarding Standard II (A), except:

A. You may not state that you have completed Level I, II, or III until you have received the CFA
designation.
B. This standard relates to oral statements.
C. All of these answers.
D. This standard relates to business cards and letterheads.
E. You must be registered for the next CFA exam in order to call yourself a candidate.
F. There is no designation for someone who has passed Level I, II, or III.That answer is correct!

Answer: A

Explanation:
Standard II (A) relates to the responsibility of AIMR members and candidates to use their
professional designation properly and in a non-misleading manner. A person must be registered to
take the next scheduled CFA exam to be a "candidate" in the CFA program. There is no designation
for someone who has passed Level I, II, or III of the CFA exam. Candidates may state, however, that
they have completed Level I, II, or III. The standard applies to all related explanations or descriptions
of the CFA designation, including letterheads and business cards, resumes, directory listings, printed
advertising, brochures and oral statements to clients and prospects.

Question: 211

Portfolios

A. can be switched from one composite to another at any time at the discretion of the investment
manager.
B. can never be switched from one composite to another, so managers must ensure they consult
their clients and make appropriate choices.
C. must not be switched from one composite to another unless documented changes in client

Page | 83
guidelines make switching appropriate.
D. can only be switched from one composite to another at the end of a performance period and only
if documented changes in client guidelines make switching appropriate.

Answer: C

Explanation:
Portfolios must not be switched from one composite to another unless documented changes in client
guidelines make switching appropriate. This is a requirement for creation and maintenance of
composites.

Question: 212

An AIMR member has violated one of the AIMR code of ethics standards. However, according to the
laws of the country governing his behavior, he has done nothing wrong. Then,

A. AIMR can take disciplinary action against her.


B. AIMR cannot take disciplinary action against her.
C. there is no violation of the code since local laws were adhered to.
D. none of these answers.That answer is correct!

Answer: A

Explanation:
A member must always abide by the AIMR code, unless the local laws in any given case are stricter,
in which case, the stricter standard applies. StandardI. Hence, every violation of the code of ethics is
always subject to disciplinary action.

Question: 213

Convertible and other hybrid securities

A. are never included in composites with conventional securities.


B. are managed separately and treatment must be disclosed in performance reporting.
C. are not assets to which the Standards are applicable.
D. must be treated consistently across and within composites.

Answer: D

Explanation:
Convertible and other hybrid securities must be treated consistently across and within composites.
Portfolios must not be switched from one composite to another unless documented changes in client
guidelines make switching appropriate. This is a requirement for creation and maintenance of
composites.

Question: 214

To claim compliance, firms must meet

A. all composite, calculation, presentation and disclosure requirements of the AIMR Performance
Presentation Standards.
B. all composite, calculation, presentation and disclosure recommendations of the AIMR
Performance Presentation Standards.
C. all composite, calculation, presentation and disclosure requirements and recommendations of the

Page | 84
AIMR Performance Presentation Standards.
D. all composite, calculation, presentation and disclosure requirements and at least half of the
recommendations of the AIMR Performance Presentation Standards.That answer is correct!

Answer: A

Explanation:
To claim compliance, firms must meet all composite, calculation, presentation and disclosure
requirements of the AIMR Performance Presentation Standards. Firms are also strongly encouraged
to follow the recommended standards.

Question: 215

Which of the following statements is NOT correct?

A. All of these statements are correct.


B. Standards II through V contain principles guiding the general activities of members.
C. Members who work in a country where the Code and Standards impose a higher degree of
responsibility than local laws and regulations must adhere to the Code and Standards.
D. Members are not required to report legal violations to the appropriate governmental or regulatory
organizations.
E. Members who work in a country where the Code and Standards impose a lesser degree of
responsibility than local laws and regulations must adhere to the local laws and regulations.

Answer: B

Explanation:
Standards II through V address specific conduct, while Standard I contains principles guiding the
general activities of members. This distinction should be noted.

Question: 216

Which of the following statements is true?

A. Standard I sets forth minimum standards relating to general activities.


B. All of these statements are correct.
C. Members are to be aware of laws and regulations of foreign jurisdictions.
D. Standards II through V address specific conduct.
E. Members who work in a country where the Code and Standards impose a lesser degree of
responsibility than local laws and regulations must adhere to the local laws and regulations.
F. Members are to comply with the laws and regulations of foreign jurisdictions.

Answer: B

Explanation:
All the statements are true. Standard I contains principles that apply to the general activities of
members while Standards II-V address specific conduct. In addition to complying with the
requirements of the Code and Standards, members must be aware of laws and regulations of foreign
jurisdictions. When the Code and Standards impose a higher degree of responsibility than applicable
securities laws, members must adhere to the Code and Standards and vice-versa.

Question: 217

An employee has a duty of ________ to his current employer, even if he has already disclosed plans

Page | 85
to leave within the near future.

A. loyalty
B. judgment
C. none of these answers
D. sagacity
E. prudenceThat answer is correct!

Answer: A

Explanation:
Standard III (B) states that a departing employee is free to make preparations to go into a
competitive business before terminating the relationship with the employee's current employer
provided that such preparations do not breach the employee's duty of loyalty.

Question: 218

When complying with Standard IV (B.3) - Fair Dealing, a change of recommendation from "sell" to
"buy" is considered:

A. none of these answers.


B. material only if so specified prior to the recommendation by the client.
C. not material.
D. material only if so specified prior to the recommendation by the investment manager.
E. generally material.

Answer: E

Explanation:
In general, a material change in a firm's recommendation is one that could be expected to affect the
investor's judgment or motivate an informed buyer or seller to take an investment action. Hence, a
change of recommendation from "sell" to "buy" is generally material.

Question: 219

According to the AIMR-PPS, assets assigned to subadvisors that are not part of the firm

A. are not to be included in total firm assets and their performance record is, therefore, not part of
the firm's performance record.
B. are included in total firm assets at the investment manager's discretion and information regarding
their inclusion should be included in the disclosures.
C. are to be included in total firm assets, however, because they are subadvisors, their performance
record is not part of the firm's performance record.
D. are to be included in total firm assets, hence, their performance record is part of the firm's
performance record.That answer is correct!

Answer: A

Explanation:
Total firm assets are defined to include all discretionary and nondiscretionary assets. Assets assigned
to subadvisors that are not part of the firm are not to be included in total firm assets and their
performance record is, therefore, not part of the firm's record.

Question: 220

Page | 86
Tessmer is a hard-working employee working in the sales department of a large bank. In this
capacity, Tessmer is allowed to have an expense account to be used to entertain clients and cover
business expenses. Over the past 3 months, Tessmer's expenses have almost doubled, a large part
coming from expenses attributed to "business expense." While his manager, Janus Lace, is suspicious
about this and believes that Tessmer is using the account for personal expenses, he has no proof of
this. All of Tessmer's reimbursement bills appear to be in order. Tessmer has:

A. violated the AIMR standard on Professional Misrepresentation.


B. violated the AIMR standard on Reasonable Costs.
C. not violated any AIMR standard.
D. violated the AIMR standard on Professional Misconduct.

Answer: C

Explanation:
There is no evidence that Tessmer is misusing his expense account. The increase may simply be due
to an increased professional activity. There is a chance that Tessmer is altering his bills but that needs
to be investigated in some depth by Janus Lace, with a detailed examination of the results accruing
from increased expenses. If Tessmer is found guilty of falsifying records, he would be in violation of
Standard II (B) - Professional Misconduct. Without such evidence, charges of violation cannot be
leveled against anyone.

Question: 221

The financial analyst may ________.

A. not accept gifts from issuers of securities under any circumstances


B. accept modest gifts
C. accept any gift, as long as it is disclosed
D. none of these answers

Answer: B

Explanation:
According to Standard IV (A.3) - Independence and Objectivity - modest gifts that do not exceed $100
and entertainment are acceptable, but special care should be taken by member analysts and
investment managers to resist lavish gifts such as tickets, favors, job referrals and so on, which may
be tied in with subtle pressures for favors possibly detrimental to clients.

Question: 222

The Performance Presentation Standards maintain that composites should cover a minimum of
________ years, or present the entire record of the firm, if shorter.

A. ten
B. eight
C. seven
D. two
E. fiveThat answer is correct!

Answer: A

Explanation:

Page | 87
A 10 year performance record (or a record for the period since firm inception if inception is less than
10 years) must be presented.

Question: 223

The ________ of portfolio performance over time is a measure of the variability or dispersion of the
historical returns around their central tendency or mean return.

A. variance
B. standard deviation
C. mean variance
D. mode

Answer: B

Explanation:
Standard deviation is also a useful measure of the relative volatility of fund categories.

Question: 224

________ accounting is mandatory for fixed-income securities.

A. Flexible
B. Risky
C. Total
D. Equal
E. Accrual

Answer: E

Explanation:
Accrual accounting must be used for fixed-income securities and all other securities that accrue
income. Accrued income must be included in the market value calculation of the denominator and
numerator.

Question: 225

Bronson provides investment advice to the board of trustees of a private university endowment
fund. The trustees have provided Bronson with the fund's financial information, including planned
expenditures. Bronson receives a phone call on Friday afternoon from Murdock, a prominent
alumnus, requesting that Bronson fax him comprehensive financial information about the fund.
According to Murdock, he has a potential contributor but needs the information that day to close the
deal and cannot contact any of the trustees. Based on AIMR Standards, Bronson should:

A. send Murdock the information because it is not material nonpublic information.


B. not send Murdock the information to preserve confidentiality.
C. send Murdock the information, provided Bronson promptly notifies the trustees.
D. send Murdock the information because disclosure would benefit the client.

Answer: B

Explanation:
This question deals with Standard IV (B.1), Fiduciary Duties and Standard IV (B.5), Preservation of
Confidentiality. Bronson owes a fiduciary duty to the fund trustees. He cannot disclose confidential

Page | 88
financial information to anyone without the permission of the fund, regardless of whether the
disclosure may benefit the fund. The fund must be notified and permission obtained before the
information is publicized. Even if the information were nonmaterial, the member cannot disclose it
because it is confidential.

Question: 226

Taxes must be recognized

A. in the same period as when the taxable event occurred.


B. on a quarterly basis.
C. in the period immediately following the period in which the taxable event occurred.
D. on a yearly basis.That answer is correct!

Answer: A

Explanation:
Taxes must be recognized in the same period as when the taxable event occurred. This is a
requirement for calculation of returns.

Question: 227

Scott works for a regional brokerage firm. He estimates that Walkton Industries will increase its
dividend by $1.50 a share during the next year. He realized that this increase is contingent on
pending legislation that would, if enacted, give Walkton a substantial tax break. The U.S.
representative for Walkton's home district has told Scott that, although he is lobbying hard for the
bill and prospects for passage look good, Congress's concern over the federal deficit could cause the
tax bill to be voted down. Walkton has not made any statements regarding a change in dividend
policy. Scott writes in his research report, "We expect Walkton's stock price to rise by at least $8.00 a
share by the end of the year. Because the dividend will increase by $1.50 a share, the stock price gain
will be fueled, in large part, by the increase in the dividend. Investors buying the stock at the current
time should expect to realize a total return of at least 15 percent on the stock." Which of the
following is/are true?

I. Scott violated the Standards because he used material inside information.


II. Scott violated the Standards because he failed to separate opinion from fact.
III. Scott did not violate the Standards.

A. I and II only.
B. II only.
C. I only.
D. III only.

Answer: B

Explanation:
This question relates to Standard IV (A.2), Research Reports. Scott issued a research report stating
that the stock price of Walkton is expected to rise to $8 a share "because the dividend will increase
by $1.50 a share." Yet, he made this assertion fully aware the dividend will increase only if Congress
enacts certain legislation, which is an uncertain prospect. By stating that the dividend will increase,
Scott failed to separate fact from opinion. The information on the passing of legislation is not
material inside information because it does not come from or pertain to company operations.

Question: 228

Page | 89
Which standard deals with the Prohibition against Use of Material Nonpublic Information?

A. III
B. III (B.4)
C. IV (B.4)
D. IV (B.5)
E. V (A)
F. III (B.2)

Answer: E

Explanation:
Standard V (A) - Prohibition against Use of Material Nonpublic Information, states that "members
who possess material nonpublic information related to the value of a security shall not trade in that
security if such trading would breach a duty or if the information was misappropriated or relates to a
tender offer."

Question: 229

An investment recommendation can be disseminated to clients via each of the following methods,
EXCEPT:

A. through a brief update report.


B. by addition to a recommended list.
C. by oral communication.
D. through an initial detailed research report.
E. by deletion from a recommended list.
F. all of these answers are acceptable methods.

Answer: F

Explanation:
According to Standard IV (B.3) - Fair Dealing, an investment recommendation is any opinion
expressed by a member in regard to purchasing, selling or holding a given security or other
investment. All of the methods listed are acceptable for dissemination.

Question: 230

Parts of the standards that are ________ must be observed.

A. recommended
B. suggested
C. mandatory
D. restraining
E. inhibiting

Answer: C

Explanation:
Some aspects of the Performance Presentation Standards are mandatory and must be observed,
while other aspects are recommended.

Question: 231

Page | 90
A manager who pays a higher commission than would normally be paid to purchase the goods or
services:

A. may be violating the fiduciary duties owed to the client.


B. none of these answers.
C. is violating the fiduciary duties owed to the client.
D. is not violating the fiduciary duties owed to the client.

Answer: C

Explanation:
This practice is commonly referred to as "paying up" for services.

Question: 232

Relationships with and Responsibilities to the Employer are dealt with under:

A. Standard II
B. Standard I
C. Standard V
D. Standard III
E. None of these answers
F. Standard IV

Answer: D

Explanation:
Relationships with and Responsibilities to the Employer are dealt with under StandardIII.

Question: 233

Which of the following statements is/are correct under the Code and Standards?

I. AIMR members are prohibited from undertaking independent practice in competition with their
employer.
II. Written consent from the employer is necessary to permit independent practice that could result
in compensation or other benefit in competition with a member's employer. III. Written consent
from the outside prospective client is necessary to permit independent practice that could result in
compensation or other benefit in competition with a member's employer.
IV. Members are prohibited from making arrangements or preparations to go into a competitive
business before terminating their relationship with their employer.

A. II, III and IV only.


B. I and IV only.
C. IV only.
D. II and III only.

Answer: D

Explanation:
This question pertains to Standard III (B), which states that members may undertake independent
practice that may result in compensation or other benefit in competition with a member's employer,
so long as they obtain written consent from both their employer and those for whom they undertake

Page | 91
the independent practice. Statements II and III are consistent with this Standard. Statement I is
incorrect because the Standards do not completely prohibit independent practice. Also, Statement IV
is incorrect because the Standards allow members to make arrangements or preparations to go into
competitive business so long as those arrangements do not interfere with their duty to their current
employer.

Question: 234

Various countries' securities laws permit a manager to pay up for goods and services without
violating the manager's fiduciary duty, so long as the requirements of the law are followed. Each of
the following are typical requirements, except

A. the manager's soft-dollar practice must be disclosed.


B. the compensation received for engaging in the paying up must be reported in writing to the
manager's employer.
C. the commission paid must be reasonable in relation to the research and execution services
received.
D. the goods or services purchased must be for "research service."
E. at all times, the manager must seek best price and execution.

Answer: B

Explanation:
When paying up for goods and services, the requirements to be met include:

- ensuring that the goods or services purchased must be for "research service;"
- the commission paid must be reasonable in relation to the research and execution services
received;
- the manager's soft-dollar practice must be disclosed, and
- at all times, the manager must seek best price and execution. However, there is no requirement for
any disclosure of compensation to the manager's employer.

Question: 235

Miller heads the research department of a large brokerage firm. The firm has many analysts, some of
whom are subject to the Code and Standards. If Miller delegates some supervisory duties, which
statement best describes her responsibilities under the Code and Standards?

A. Miller's supervisory responsibilities do not apply to those subordinates who are not subject to the
Codes and Standards.
B. Miller no longer has supervisory responsibility for those duties delegated to her subordinates.
C. Miller retains supervisory responsibility for those duties despite her delegation of some duties.
D. AIMR's Standards prevent Miller from delegating supervisory duties to subordinates.

Answer: C

Explanation:
Under Standard III (E), Responsibilities of Supervisors, members may delegate supervisory duties to
subordinates but such delegation does not relieve members of their supervisory responsibilities.
Moreover, whether or not Miller's subordinates are subject to AIMR's Code and Standards is
irrelevant to her supervisory responsibilities. Thus, Miller still retains supervisory responsibilities.

Question: 236

Page | 92
Which of the following can be found in Standard III?

A. Members shall not misrepresent investment performance.


B. Members shall exercise diligence and thoroughness in making investment recommendations or in
taking investment actions.
C. Members shall not participate in any professional conduct involving dishonesty, fraud, deceit, etc.
D. Members shall deliver a copy of the Code to their employer.
E. Members shall not knowingly participate or assist in any violation of laws, rules, or regulations.

Answer: D

Explanation:
Standard III states that members shall deliver a copy of the Codes and Standards to their employer if
the employer does not have a copy.

Question: 237

Level ________ verification assures that all of the firm's actual, discretionary, fee-paying portfolios
are included in at least one composite.

A. V
B. II
C. IV
D. I
E. III

Answer: D

Explanation:
Level I Verification requires that each of the firm's discretionary, fee-paying portfolios is included in
at least one composite and that the firm's procedures for assigning portfolios to composites are
reasonable and have been consistently applied over time.

Question: 238

Level ________ verification applies to specific composites.

A. 0
B. IV
C. II
D. I
E. III

Answer: C

Explanation:
Level I verification applies to all firm composites. Level II verification requires a Level I verification at
least on the specific composites being verified at Level II. (Note: There are only two levels of
verification.)

Question: 239

Standard V (A), Prohibition against Use of Material Nonpublic Information, states that the test for
determining if a tipper is breaching a fiduciary duty

Page | 93
A. is whether the tippee benefits directly or indirectly from the disclosure.
B. is whether the tipper benefits directly or indirectly from the disclosure.
C. is whether the tipper benefits directly from the disclosure.
D. is whether the insider benefits directly from the disclosure.
E. none of these answers.

Answer: B

Explanation:
Standard V (A), Prohibition against Use of Material Nonpublic Information, states that the test for
determining if a tipper is breaching a fiduciary duty is whether the tipper benefits directly or
indirectly from the disclosure. The three types of personal benefits are: 1. pecuniary benefit, 2. a
quid pro quo between the insider and recipient, and 3. a gift of confidential information to a relative.
An insider who selectively discloses material nonpublic information without a legitimate business
purpose may be found to have breached a fiduciary duty.

Question: 240

All of the following statements about a member's use of clients' brokerage commissions are true
except

A. client brokerage commissions may be directed to pay for the investment manager's operating
expenses.
B. client brokerage commissions should be commensurate with the value of the brokerage and
research services received.
C. client brokerage commissions may be used by the member to pay for securities research used in
managing the client's portfolio.
D. client brokerage commissions should be used by the member to ensure that fairness to the client
is maintained.That answer is correct!

Answer: A

Explanation:
This question deals with Standard IV (B.1), Fiduciary Duties and the specific topic of soft dollars. All
the answers deal with the use of client brokerage commissions and the criteria for their use - (1)
fairness to the client and (2) commensurate with the value of the services provided. The only
statement that is not true is that client brokerage commissions may not be directed to pay the
investment manager's operating expenses.

Question: 241

Standard III includes which of the following?

A. None of these answers


B. Use of Professional Designation
C. Disclosure of Conflicts to Employer
D. Fair Dealing
E. Prohibition against Plagiarism
F. All of these answers

Answer: C

Explanation:

Page | 94
Standard III deals with Obligation to Inform Employer of Code and Standards, Duty to Employer,
Disclosure of Conflicts to Employer, Disclosure of Additional Compensation Arrangements and
Responsibilities of Supervisors.

Question: 242

Which of the following persons does AIMR hold most accountable for fiduciary duties owed to
clients?

A. Investment Advisors
B. Brokers
C. None of these answers
D. Dealers

Answer: C

Explanation:
AIMR's Code and Standards do not make distinctions among brokers, dealers and investment
advisors when it comes to fiduciary duties toward customers. All members must comply with the
conduct required of them by the Code and Standards or applicable law, whichever is higher.

Question: 243

When formulating an investment policy for a client, all of the following fall under the category of
"investor constraints," except:

A. risk tolerance
B. liquidity needs
C. expected cash flows
D. regulatory and legal circumstances
E. time horizon
F. tax considerations
G. investable funds
H. investor preferences, circumstances and unique needsThat answer is correct!

Answer: A

Explanation:
Risk tolerance is considered under "investor objectives," not "investor constraints."

Question: 244

When presenting the components of total return for a real estate portfolio, the recognition of income
at ________ level, rather than at the operating level, is preferred.

A. risk
B. purchase
C. realized
D. investment
E. composite

Answer: D

Explanation:

Page | 95
This is recommended under the AIMR-PPS for presenting investment performance results.

Question: 245

The financial analyst is allowed to breach client confidentiality if the information concerns ________
activities that the client is engaged in.

A. personal
B. confidential
C. insider
D. none of these answers
E. illegal

Answer: E

Explanation:
Standard IV (B.5) states that "Members shall preserve the confidentiality of information
communicated by clients, prospects, or employers concerning matters within the scope of the client -
member relationship, unless the member receives information concerning illegal activities on the
part of the client."

Question: 246

When complying with Standard IV (B.3) - Fair Dealing, there are certain points one should be sure to
address when establishing compliance procedures. Which of the following points is NOT mentioned
in the Standards of Practice Handbook?

A. Get as many people involved within the firm that a recommendation is going to be disseminated.
B. Shorten the time frame between decision and dissemination.
C. Disclose levels of service.
D. Establish control over trading activity.
E. Simultaneous dissemination.
F. Publish personnel guidelines for predissemination.
G. Establish systematic account review.That answer is correct!

Answer: A

Explanation:
Members have an obligation to ensure that their firms establish compliance procedures requiring all
employees who disseminate investment recommendations or actions to treat clients fairly. The
number of people privy to an investment recommendation should be limited. The amount of time
that lapses between the decision and the dissemination of the recommendation should be
shortened. Guidelines must be established to prohibit persons who have prior knowledge from
discussing a pending recommendation. Trading activities should be monitored and controlled.
Supervisors should review accounts on a regular basis to ensure that no client is being given
preferential treatment.

Question: 247

According to the AIMR-PPS, performance results of a past firm or affiliation

A. must always be linked to the historical records of the new firm to show a fair and true
representation of results.
B. none of these answers.

Page | 96
C. may or may not be linked to the historical records of new firm or affiliation, it is left to the
discretion of the investment manager and that information should be disclosed.
D. must not be used to represent the historical record of a new affiliation or a new firm identity.

Answer: D

Explanation:
Performance results of a past firm or affiliation must not be used to represent the historical record of
a new affiliation or a new firm entity. This is a requirement for presentation of results.

Question: 248

Various countries' securities laws permit a manager to pay up for goods and services without
violating the manager's fiduciary duty, so long as the requirements of the law are followed. Each of
the following are typical requirements, except:

A. the manager's soft-dollar practice should be always maintained confidentially in his or her files.
B. at all times, the manager must seek best price and execution.
C. the commission paid must be reasonable in relation to the research and execution services
received.
D. the goods or services purchased must be for "research service."
E. none of these answers.That answer is correct!

Answer: A

Explanation:
The soft-dollar practice must be disclosed.

Question: 249

According to the AIMR-PPS, all actual fee-paying discretionary portfolios must be included in at least
one composite defined according to

A. managerial style of investment portfolio.


B. similar strategy or investment objective.
C. size of investment portfolio.
D. date of portfolio inception.

Answer: B

Explanation:
All fee-paying discretionary portfolios must be included in at least one composite defined according
to similar strategy or investment objective. This is a requirement for creation and maintenance of
composites.

Question: 250

According to Standard IV (B.4), Priority of Transactions, a member is a "beneficial owner" if the


member has each of the following, EXCEPT:

A. the power to vote or direct the voting of the shares of the securities or investments.
B. the power to dispose or direct the disposition of the security or investment.
C. the authority to supervise reasonably and adequately the actions of those accountable to him.
D. a direct or indirect pecuniary interest in the securities.

Page | 97
Answer: C

Explanation:
The authority to supervise those below the member has no relation to the concept of "beneficial
owner." Also, supervisory responsibility is covered under Standard III (E).

Question: 251

According to Standard IV (B.7), Disclosure of Conflicts, the most prevalent conflict pertains to
________.

A. members' ownership of stock in companies they recommend


B. broker-sponsored limited partnerships formed to invest venture capital
C. service on a board of directors
D. none of these answersThat answer is correct!

Answer: A

Explanation:
Under Standard IV (B.7), Disclosure of Conflicts, the most prevalent conflict pertains to members'
ownership of stock in companies they recommend. To avoid discrimination against members, sell-
side members should disclose any materially beneficial ownership interest in a security or other
investment that the member is recommending. Buy-side members should disclose their procedures
for reporting requirements for personal transactions.

Question: 252

Various countries' securities laws permit a manager to pay up for goods and services without
violating the manager's fiduciary duty, so long as the requirements of the law are followed. Each of
the following are typical requirements, except:

A. the manager's soft-dollar practice must be disclosed.


B. the manager may seek satisfactory price and execution.
C. the goods or services purchased must be for "research service."
D. none of these answers.
E. the commission paid must be reasonable in relation to the research and execution services
received.

Answer: B

Explanation:
The manager must seek BEST price and execution.

Question: 253

Which of the following AIMR Standards states that the financial analyst shall indicate the basic
characteristics of the investment involved when preparing a research report not directly related to a
specific portfolio or client?

A. III D
B. III C
C. None of these answers
D. IV (A.2)

Page | 98
Answer: D

Explanation:
Standard IV (A.2) - Research Reports states: "Members shall indicate the basic characteristics of the
investment involved when preparing for public distribution a research report that is not directly
related to a specific portfolio or client."

Question: 254

Monserrat Troy is a portfolio analyst at Merryl Flynch Inc., a leading investment bank. Merryl's
corporate policy prohibits its employees from trading in IPOs even if they are not being underwritten
by Merryl. Recently, a fast-growing internet firm, Netblaze, announced that it was going public.
Monserrat's friend, Victor, told her that he could get her bid in for a few shares of the IPO. The size of
the deal was very small, amounting to less than a couple of thousand dollars. Monserrat went ahead
and bought the shares but did not think it was necessary to inform her supervisor about it. She:

A. violated Standard II (B) - Professional Misconduct - by violating company policy.


B. did not violate the AIMR code of conduct even though she violated the company policy since the
size of her transaction was extremely small.
C. violated Standard III (B) - Duty to Employer - by violating company policy.
D. violated Standard III (C) - Disclosure of Conflicts to Employer - by violating company policy.

Answer: D

Explanation:
Merryl's policy is designed to prevent any conflicts or even appearances of conflicts of interest when
its employees trade in capital markets. If Monserrat wants to trade in a particular IPO, she needs to
obtain special permission from Merryl's Compliance department (and such permission may or may
not be granted by Compliance). Without such an exception, Monserrat's behavior is a violation of
Standard III(C) - Disclosure of Conflicts to Employer. Note that Standards II (B) and III (B) do not apply
to this situation.

Question: 255

The investment community's need for a common, accepted set of guidelines to promote fair
representation and full disclosure in every firm's presentation performance results to clients and
prospective clients has guided the development of the

A. AIMR Standards for Calculation of Returns


B. AIMR Standards of Practice
C. AIMR Performance Presentation Standards
D. AIMR Performance and Calculation Investment Board

Answer: C

Explanation:
The Performance Presentation are the manifestation of a set of guiding ethical principles and should
be interpreted as minimum standards for presenting investment performance.

Question: 256

When an investment manager's client instructs the manager to use the client's brokerage to
purchase goods or services for the client, this is known as:

Page | 99
A. horizontal brokerage and does not violate any fiduciary duty.
B. horizontal brokerage and violates the manager's fiduciary duty.
C. directed brokerage and does not violate any fiduciary duty.
D. straight brokerage and does not violate any fiduciary duty.
E. straight brokerage and violates the manager's fiduciary duty.
F. directed brokerage and violates the manager's fiduciary duty.

Answer: C

Explanation:
When an investment manager's client instructs the manager to use the client's brokerage to
purchase goods or services for the client, this is known as directed brokerage. Because brokerage is
an asset of the client and the brokerage is used to benefit that client, not the manager, such practice
does not violate fiduciary duty.

Question: 257

If the use of leverage is ________, the performance presented must include the effects of the
leverage.

A. mandatory
B. nondiscretionary
C. discretionary
D. optional

Answer: C

Explanation:
If the use of leverage is nondiscretionary (i.e. mandated by the client), performance must be
presented on an all-cash basis.

Question: 258

Under the AIMR Code of Ethics, members shall practice and encourage others to practice in a
professional and ethical manner that will reflect credit on members and their ________.

A. families
B. employer
C. public contacts
D. profession
E. none of these answers

Answer: D

Explanation:
In accordance with the Standards, members will also strive to maintain and improve their
competence and the competence of others in the profession.

Question: 259

Under ERISA, fiduciaries must:

-act solely in the interest of and for the exclusive purpose of benefiting, the plan participants and

Page | 100
beneficiaries;
-act with the ________, prudence and diligence of a prudent person acting in like capacity;
-diversify the plan's investments to protect it from the risk of substantial loss;
-act in accordance with the provisions of the plan documents to the extent that the documents
comply with ERISA;
-refrain from engaging in prohibited transactions.

A. caution
B. care, skill
C. none of these answers
D. attention

Answer: B

Explanation:
A fiduciary is required to discharge all duties with respect to a plan "with the care, skill, prudence and
diligence under the circumstances then prevailing that a prudent man acting in a like capacity and
familiar with such matters would use in the conduct of an enterprise of a like character and with like
aims."

Question: 260

The value of a real estate portfolio requires, at minimum, ________ review.

A. weekly
B. quarterly
C. daily
D. monthly
E. annual

Answer: B

Explanation:
The practice of providing quarterly reports satisfies the quarterly review requirements. The issuance
process should include a review of the real estate portfolio, a review of net asset value for financial
and performance purposes, and review and disclosure of any factors that might result in a material
change to net asset value.

Question: 261

Which of the following are mandatory disclosure requirements under AIMR-PPS?

I. The number of portfolios and assets in each composite.


II. The fees deducted or not deducted from the calculations of returns.
III. The volatility of the aggregate composite return.
IV. Benchmarks that the composites are expected to track.

A. I only
B. I and II only
C. I, II and III only
D. III and IV only

Answer: B

Page | 101
Explanation:
(III) & (IV) are recommended disclosures while (I) and (II) are mandatory.

Question: 262

Paul Erdosh is an independent contractor who was hired by Arbitrage Securities to carry out a
research project involving the software industry. The scope of the research was broad and the aim
was to identify relatively undervalued stocks in that sector. During the course of the research, Paul
realized that two of the companies in the sector were prime takeover targets if anyone bothered to
do more research on them. He decided to sell this finding to M&A Consultants and obtain a research
contract from them. He did not inform Arbitrage Securities about this deal, though he did submit a
competent research report on the software industry to them. Paul has

A. not violated Standard III (B) - Duty to Employer.


B. violated Standard III (B) - Duty to Employer - because by selling the information to a rival firm, he
has potentially harmed Arbitrage Securities' business.
C. not violated Standard III (B) - Duty to Employer - because he is an independent contractor, not an
employee of Arbitrage Securities.
D. violated Standard III (B) - Duty to Employer. He needs written permission from Arbitrage Securities
before releasing any of his findings to third parties.That answer is correct!

Answer: A

Explanation:
Arbitrage Securities hired Paul to do a research project that did not involve identifying specific
merger/acquisition candidates. Hence, Paul's discovery is tangential to the original project and as
such, cannot be considered the property of Arbitrage Securities. Standard III (B) does apply to Paul
even though he is a contractor; however, it does not preclude him from selling his findings without
Arbitrage Securities' permission as long as the information is not related to the project he was hired
to conduct.

Question: 263

________ accounting is suggested for calculating performance.

A. Trade-date
B. Performance
C. Accrual
D. Longitudinal
E. CompositeThat answer is correct!

Answer: A

Explanation:
Trade-date accounting is recommended when calculating performance, although settlement-date
accounting is acceptable if disclosed. Because of the volatility and lengthy settlement periods of
some markets, trade-date accounting is strongly recommended for calculating the performance of
international portfolios.

Question: 264

Which of the following AIMR Standards states that a financial analyst must consider the suitability of
an investment for each client or portfolio before undertaking investment action?

Page | 102
A. IV (B.2)
B. III E
C. none of these answers
D. III DThat answer is correct!

Answer: A

Explanation:
Standard IV (B.2) - Portfolio Investment Recommendations and Actions states: "Members shall
consider the appropriateness and suitability of investment recommendations or actions for each
portfolio or client. Members shall not make a recommendation unless they reasonably determine
that the recommendation is suitable to the client's financial situation, investment experience and
investment objectives."

Question: 265

Performance is the record of the ________.

A. individual
B. firm
C. investment manager
D. child
E. person

Answer: B

Explanation:
To be in compliance with the PPS, a firm's presentation of performance must comply on a firmwide
basis and must consist of 10 years of performance (or records since the date of inception if the firm is
younger than 10 years).

Question: 266

John Cochrun is a reputed money manager who is also an AIMR member. John recently discovered
that some employees at the firm where he is employed have been engaging in some questionable
activity which could very well be construed as insider trading by the SEC. John, however, does not
think the activity is egregious enough to be reported to the SEC. In any case, he does not want to be
known as "the rat who blew the whistle." By not reporting the criminal activity, John has:

A. has not violated the AIMR code.


B. violated the AIMR code, Standard I (B) - Fundamental Responsibilities.
C. violated the AIMR code, Standard III (C) - Disclosure of Conflicts to the Employer.
D. violated the AIMR code, Standard III (B) - Duty to the Employer.That answer is correct!

Answer: A

Explanation:
The AIMR code does not require members to report legal violations to the appropriate authorities, as
long as they don't associate themselves with such activities.

Question: 267

Albert and Tye, who recently started their own investment advisory business, have registered to take
the Level III CFA examination. Albert's business card reads, "Judy Albert, CFA Candidate." Tye has not

Page | 103
put anything about the CFA designation on his business card, but promotional material that he
designed for the business describes the CFA requirements and indicates that Tye participates in the
CFA Program and completed Levels I and II. According to the Standards:

A. Albert has violated the Standards but Tye has not.


B. Neither Albert nor Tye had violated the Standards.
C. Tye has violated the Standards and Albert has not.
D. Both Albert and Tye have violated the Standards.That answer is correct!

Answer: A

Explanation:
The question deals with Standard II (A), Use of Professional Designation. Candidates may not use the
designation in any form prior to actually obtaining their charter. Therefore, Albert has violated
Standard II (A). Candidates may communicate that they are participating in the CFA Program,
however and may state the levels that they have completed. Therefore, Tye has not violated
Standard II (A).

Question: 268

An investment management firm has been hired by ETV Corporation to work on an initial public
offering for the company. The firm's brokerage unit now has a sell recommendation on ETV, but the
head of the investment banking department has asked the head of the brokerage unit to change the
recommendation from sell to buy. According to the Standards, the head of the brokerage unit would
be permitted to:

A. Increase the recommendation by no more than one increment (in this case, to a hold
recommendation).
B. Assign a new analyst to decide if the stock deserves a higher rating.
C. Place the company on a restricted list and give only factual information about the firm.
D. Reassign responsibility for rating the stock to the head of the investment banking unit.

Answer: C

Explanation:
This question relates to Standard IV (A.3), Independence and Objectivity. When asked to change a
recommendation on a company stock to gain business for the firm, the head of the brokerage unit
mustrefuse in order to maintain his independence and objectivity in making the recommendation. To
avoid the appearance of a conflict of interest, the firm should discontinue issuing recommendations
about the company. Changing the recommendation in any manner is a violation of the member's
duty to maintain independence and objectivity. Assigning a new analyst would not address the
conflict of interest. Reassigning responsibility for the stock to the head of investment banking would
actually exacerbate the conflict.

Question: 269

Which of the following is/are a correct statement of a member's duty under the Code and Standards?

I. In the absence of specific applicable law or other regulatory requirements, the Code and Standards
govern the member's actions.
II. A member is required to comply only with applicable local laws, rules, regulations, or customs
even though the AIMR Code and Standards may impose a higher degree of responsibility or a higher
duty on the member.
III. A member who trades securities in a foreign securities market where no applicable local laws or

Page | 104
stock exchange rules regulate the use of material nonpublic information may take investment action
based on material nonpublic information.

A. I only.
B. III only.
C. I and II only.
D. II and III only.That answer is correct!

Answer: A

Explanation:
This question pertains to Standard I, Fundamental Responsibilities - specifically, international
application of the Code and Standards. If applicable law is more strict than the requirements of the
Code and Standards, members must adhere to applicable law, otherwise, members must adhere to
the Code and Standards - thus, I is correct. Members must adhere to the higher standard set by the
Code and Standards if local applicable law is less strict- thus, II is incorrect. When no applicable law
exists, the Code applies - thus, III is incorrect.

Question: 270

With regard to calculation of returns for international portfolios, a consistent source of ________
should be used.

A. exchange rates
B. trades
C. settlements
D. disclosures
E. taxesThat answer is correct!

Answer: A

Explanation:
Whenever the currency overlay manager is notified of changes in the underlying currency exposures
as a result of a shift in the underlying assets, the currency overlay portfolios should be revalued to
ensure consistency.

Question: 271

Which of the following is NOT true about Standard IV (A.2) - Research Reports?

A. None of these answers.


B. The analyst must separate fact from statistical conjecture to be in compliance with the standard.
C. As long as the analyst has carried out adequate investigation, she can omit from the report certain
aspects of the investigations that she deems unimportant.
D. The report must contain a basic description of the characteristics of the investment under
consideration.That answer is correct!

Answer: A

Explanation:
Standard IV (A.2) - Research Reports

Question: 272

Page | 105
According to the AIMR-PPS, assets to which the Standards cannot be applied are not to be
considered by firms when claiming compliance with the Standards. Which of the following is an
example of an asset to which the Standards cannot be applied?

A. Guaranteed investment contract assets


B. Convertible securities
C. International assets
D. Assets managed to more than one base currencyThat answer is correct!

Answer: A

Explanation:
Traditional GIC portfolios provide stable results that are not based on a mark-to-market valuation.
The valuation of the traditional GIC is based on book value, not current market value, which are the
valuations required by the total return calculation and reporting requirements of the Standards. GIC
assets would be reported separately.

Question: 273

Under the Performance Presentation Standards, asset-weighting of portfolio returns within a


composite is required. The ________ of period weightings must be used.

A. beginning
B. end
C. none of these answers
D. midpointThat answer is correct!

Answer: A

Explanation:
Composites must be asset weighted using beginning-of-period weightings.

Question: 274

ERISA fiduciaries must adhere to the following prudent procedures: - establish a written investment
policy for the plan - ________ plan assets - make investment decisions with the skill and care of
aprudent expert - monitor investment performance - control investment expenses - avoid prohibited
transactions

A. distribute
B. qualify
C. account for
D. none of these answers
E. diversify

Answer: E

Explanation:
These procedures are stipulated under the detailing of ERISA fiduciary duties, to ensure that the
fiduciary complies with the duty to act with prudence.

Question: 275

Which of the following is/are true about claims of compliance with the AIMR-PPS?

Page | 106
I. If a firm is only in partial compliance with PPS, it must use a disclaimer specifying the exact areas of
non-compliance.
II. If the calculation methodology used by a firm follow standard industry practice, it is allowed to
claim that the methodology is in compliance with the AIMR-PPS.
III. If a member misuses the claim of compliance with the PPS, AIMR can publicly censure him,
suspend his membership and revoke his CFA charter.

A. I, II and III
B. none of them
C. III only
D. I and III only

Answer: C

Explanation:
A firm cannot claim partial compliance with the PPS. It is either fully compliant or non-compliant.
Further, statements referring to the calculation methodologies being in accordance with the PPS are
explicitly prohibited; the PPS are mainly presentation standards, not measurement standards.
Violations are punishable by public censure, membership suspension and revocation of CFA charter.

Question: 276

Regarding beneficiaries and remaindermen, current life-income beneficiaries prefer to receive


________; remaindermen would rather have ________.

A. a minimal current income; a high rate of current income


B. a high rate of current income; growth and stability of principal
C. growth and stability of principal; a high rate of current income
D. small incremental principal repayments; predictable current income
E. none of these answers

Answer: B

Explanation:
Trustees recognize a conflict of interests often exists between the primary or current beneficiaries of
a trust and the remaindermen (secondary or later beneficiaries of the trust). Current life-income
beneficiaries would prefer to receive a high rate of current income; remaindermen would rather
have growth and stability of principal. Trustees thus have a duty to exercise their best skill and care to
serve all beneficiaries impartially and with equal consideration of their needs.

Question: 277

Abra, a vice-president at Mahogany, Inc., recently revealed - quite inadvertently - information about
the tender offer from Mahogany to Kadabra, his friend who works for a rival firm. Kadabra, in turn,
shared this information with his trading buddy, Cosmo. Cosmo immediately recognized that in light
of this information, he was better off not participating in the offer. He went ahead and shorted the
stock of Mahogany and reaped a tidy profit of about a hundred thousand dollars in a month's time
when Mahogany tanked. In this case:

I. Cosmo can be held liable for insider trading under SEC Section 10(b) and Rule 10b-5. II. Cosmo can
be held liable for insider trading under the Misappropriation Theory.
III. Cosmo can be held liable for insider trading under SEC Rule 14e-3 which prohibits insider trading
based on information about tender offers.

Page | 107
IV. Kadabra has breached his fiduciary duty toward Mahogany.

A. I, II, III & IV.


B. I only.
C. none of these answers.
D. III only.
E. I & IV only.
F. IV only.
G. II & III.
H. II only.

Answer: C

Explanation:
This case is similar to the precedent set by United States vs. Chestman. According to the decision in
the Chestman case, Kadabra was a "gratuitous recipient" of inside information and as such, had no
responsibility to maintain its confidentiality. One party - according to this ruling - cannot
unilaterallyimpose a relationship of confidence on another by simply sharing information. Thus,
while Kadabra is a tippee, he has no fiduciary duty toward Mahogany. Now, the traditional theory
applies only when there is a breach of a direct or inherited fiduciary duty or if the insider trading
occurs in a tender offer and the trading is done by an insider or a tippee. So if an outsider - including
Cosmo - received and traded based on this information, he could not be held liable under the
Traditional Theory or the Misappropriation Theory. However, if Kadabra had traded for his own
account, he would be liable under both the theories as well as under Rule 14e-3.

Question: 278

Which of the following statements is NOT true relating to Standard IV (B.3) - Fair Dealing?

A. This standard covers the conduct of two groups: those who prepare recommendations and those
who take investment action.
B. Members shall deal equally and objectively with all clients.
C. Only through the fair treatment of all parties can the investment management profession maintain
the confidence of the investing public.
D. None of these answers.
E. This standard covers investment action such as general purchases, new issues, or secondary
offerings.

Answer: B

Explanation:
The operative word is "fairly," not "equally." The term "fairly" implies that the member must take
care not to discriminate against any customers or clients when disseminating investment
recommendations. Members could not possibly reach all clients "equally" (i.e., simultaneously). In
addition, each client has unique needs, investment criteria and investment objectives.

Question: 279

Grazelle Garbo has decided to change the recommendation on one of the firms she is following from
hold to buy. She has communicated her research report to her senior manager, who is reviewing it. If
the recommendation passes muster, it will be released three days from now. In the meanwhile,
Grazelle has decided to follow her research and purchase the stock for several of her clients' accounts
for which the stock is an appropriate investment and for which she has discretionary investment
powers. In the course of the luncheon meeting that same day, she mentioned that she had bought

Page | 108
the stocks for some client accounts to one of her prospective clients and described her research
which caused her to change her recommendation. Grazelle has

I. violated Standard IV (B.3) - Fair Dealing - by revealing the change in recommendation. II. violated
Standard IV (B.5) - Preservation of Confidentiality - by revealing the information about stock
purchase.
III. did not violate the Ethics code.

A. I and II only
B. II only
C. III only
D. I only

Answer: D

Explanation:
Since Grazelle has not revealed any information about the specific client accounts or any confidential
information, she is not in violation of Standard IV (B.5) - Preservation of Confidentiality. However, by
revealing the information on the change in recommendation prematurely to one of her prospective
clients, she has violated Standard IV (B.3) - Fair Dealing. The information is potentially quite valuable
and giving to one client before another in a partisan fashion is a violation of the AIMR code of Ethics.

Question: 280

Which of the following AIMR Standards pertains to the use of professional designation?

A. II (A)
B. I
C. III (A)
D. III (B)That answer is correct!

Answer: A

Explanation:
Standard II (A) - Use of Professional Designation - states the responsibility of AIMR members, CFA
charterholders and candidates for the CFA designation to use their professional designations, or their
candidacies for the designation, properly and in a manner that does not mislead the investing public
or others.

Question: 281

In reference to AIMR-PPS, which of the following is/are true?

I. The PPS are explicitly incorporated in the AIMR Code of Ethics and Standards of Professional
Conduct.
II. In case of system incompatibilities, the firm can claim compliance for only those assets that are
measured and monitored on compatible systems.
III. Plan sponsors, consultants and software vendors cannot claim compliance with the PPS; they can
only endorse or require their clients to be in compliance.

A. I, II and III
B. III only
C. II and III only
D. I only

Page | 109
Answer: B

Explanation:
The PPS are an adjunct to the AIMR Code of Ethics and Standards of Professional Conduct and not
explicitly incorporated in that framework. Compliance with PPS is voluntary, not required.

Question: 282

Standard IV (B.2) deals with ________.

A. compensation
B. fair dealing
C. soft dollars
D. competition
E. none of these answers
F. suitability
G. proxy voting
H. disclosure of conflicts

Answer: F

Explanation:
Standard IV (B.2) deals with the appropriateness and suitability of investment recommendations for
each portfolio or client.

Question: 283

In order to comply with Standard IV (B.4), Priority of Transactions, firms should prepare and
distribute to firm personnel a code of ethics and compliance procedures. The code and procedures
should do all of the following, EXCEPT:

A. ensure that procedures will be enforced.


B. contain disciplinary procedures.
C. limit the number of access persons.
D. establish reporting and prior-clearance requirements.
E. define personal transactions, investment and prohibited transactions.
F. maintain a list of clients and their holdings.
G. consider special situations.

Answer: F

Explanation:
The question deals with the compliance procedures under Standard IV (B.4). Personal transactions
must be defined to all employees. The number of access persons - those who have knowledge of
pending or actual recommendations or action - should be limited, by implementing Fire Walls. A
reporting system for all employees' personal transactions should be established, as well as
procedures for special situations. Authority should be given to a compliance officer to enforce the
procedures, which should include disciplinary action. Maintaining lists of client holdings is NOT a
compliance procedure under Standard IV (B.4), but Standard IV (B.3).

Question: 284

The procedures for compliance with Standard IV (B.8), Disclosure of Referral Fees, recommend that

Page | 110
you disclose all referral agreements ________.

A. at least orally
B. any of these answers
C. orally or in writing
D. in writing

Answer: D

Explanation:
Under Standard IV (B.8), members should disclose all agreements in order to be in compliance.
Members should disclose, in writing, the existence and terms of any referral agreement to any client
or prospective client as soon as the member learns that the client or prospective client has been
referred by a firm receiving a referral fee.

Question: 285

When taking on a new affiliation while working for his present employer, a member should:

A. notify his employer in writing and obtain written consent.


B. notify any prospective affiliation of his present employer.
C. not render services to a new client until the client gives consent in writing.
D. all of these answers are correct.

Answer: D

Explanation:
The procedures for compliance with Standard III (B) state that members should not render services
until receiving written consent from their employer to all of the terms of the arrangement, after
having provided written statements describing the types of services to be rendered to independent
affiliations. A member should also disclose to prospective clients the identity of the member's
employer and clarify that the member is performing independently of the employer. The member
should not render services until the client gives consent in writing indicating that the client has read
and understood the member's written disclosure statement.

Question: 286

Relationships with and Responsibilities to the Profession are dealt with under:

A. Standard II
B. Standard IV
C. None of these answers
D. Standard I
E. Standard V
F. Standard IIIThat answer is correct!

Answer: A

Explanation:
Relationships with and Responsibilities to the Profession are dealt with under Standard II.

Question: 287

Which AIMR standard states that members must inform employers of their duty to comply with the

Page | 111
Code and Standards and must deliver a copy to their employers?

A. IX
B. IV
C. III A
D. II A

Answer: C

Explanation:
Standard III (A) Obligation to Inform Employer of Code and Standards states: Members shall: "(1)
Inform their employer in writing, through their direct supervisor, that they are obligated to comply
with the Code and Standards and are subject to disciplinary sanctions for violations thereof, and (2)
Deliver a copy of the Code and Standards to their employer if the employer does not have a copy."

Question: 288

Access or covered persons are discussed in detail under which of the following standards?

A. Prohibition against Plagiarism


B. Responsibilities of Supervisors
C. None of these answers
D. Priority of Transactions
E. Fundamental Responsibilities
F. Professional Misconduct
G. Disclosure of Conflicts to Employer
H. Professional Misconduct

Answer: D

Explanation:
Access or covered persons are dealt with under the compliance procedures of Standard IV (B.4),
Priority of Transactions.

Question: 289

When formulating an investment policy for a client, which of the following falls under the category
"client identification?"

A. risk tolerance
B. time horizon
C. expected cash flows
D. the existence of separate beneficiaries
E. none of these answers

Answer: D

Explanation:
"Client identification" requires that the existence of separate beneficiaries be considered. Risk
tolerance is considered under "investor objectives." Time horizon is considered under "investor
constraints."

Question: 290

Page | 112
Hercules Poirot has just received the AIMR letter, informing him that he has passed the Level III exam
in the CFA program. He changes his resume to reflect this information. The header now says,
"Hercules Poirot, CFA," the "CFA" written in a smaller font. With respect to Standard II (A) - Use of
Professional Designation - if Hercules circulates this resume, he will:

A. have violated Standard II (A). He cannot use the CFA designation till AIMR informs him that he has
earned the charter.
B. have violated Standard II (A). He cannot put the CFA designation on his business card immediately
after his name.
C. have violated Standard II (A). He cannot claim to be a CFA till he obtains the final diploma from
AIMR.
D. not have violated Standard II (A).That answer is correct!

Answer: A

Explanation:
A candidate earns a CFA charter only after he has passed all 3 exams and has accumulated sufficient
professional experience to satisfy the requirements of the program. Till the ICFA Board approves his
candidacy, Hercules cannot use the designation, "CFA."

Question: 291

ERISA fiduciaries must adhere to the following prudent procedures: - establish a written investment
policy for the plan - diversify plan assets - make investment decisions with the skill and care of a
prudent expert - monitor investment performance - control investment expenses - avoid ________
transactions

A. insider
B. diversification
C. prohibited
D. none of these answers
E. commitment

Answer: C

Explanation:
These procedures are stipulated under the detailing of ERISA fiduciary duties, to ensure that fiduciary
complies with the duty to act with prudence. Although the ERISA fiduciary duty allows for a fairly
broad range of investments, it does prohibit certain transactions. An ERISA fiduciary can never: - deal
with plan assets in his or her own interest or for his or her own account (self-dealing); - act in a
transaction involving the plan on behalf of a party with interest adverse to the plan (conflict of
interest); - receiveany compensation for his or her own personal account from any party dealing with
the plan in connection with a transaction involving plan assets (kickbacks).

Question: 292

With regard to calculation of returns for international portfolios, a consistent source of ________
should be used.

A. exchange rates
B. settlements
C. trades
D. taxes
E. disclosuresThat answer is correct!

Page | 113
Answer: A

Explanation:
Whenever the currency overlay manager is notified of changes in the underlying currency exposures
as a result of a shift in the underlying assets, the currency overlay portfolios should be revalued to
ensure consistency.

Question: 293

Sanctions that AIMR may impose on its members include:

I. Revocation of the member's registration as an investment advisor.


II. Private censure.
III. A monetary fine.
IV. Suspension of membership.

A. II and IV only.
B. I and III only.
C. II, III and IV only.
D. I, II, III and IV.That answer is correct!

Answer: A

Explanation:
The question deals with the disciplinary sanctions available to AIMR's PCP. The PCP can suspend
membership or impose a private censure on members for violating the Code and Standards.
However, AIMR has no authority to revoke a member's registration as an investment advisor, or to
impose a monetary fine.

Question: 294

When formulating an investment policy for a client, which of the following falls under the category
"client identification?"

A. risk tolerance
B. none of these answers
C. type and nature of clients
D. expected cash flows
E. investable funds

Answer: C

Explanation:
"Client identification" requires that the type and nature of clients be considered. Risk tolerance is
considered under "investor objectives." Investable funds and expected cash flows are considered
under "investor constraints."

Question: 295

Standard IV (B.1) clearly states that ________ owe undivided loyalty to their clients and must place
client interests before their own.

A. AIMR members

Page | 114
B. supervisors
C. none of these answers
D. fiduciaries
E. analysts

Answer: D

Explanation:
Standard IV (B.1) - Fiduciary Duties states that "In relationships with clients, members shall use
particular care in determining fiduciary duty and must act for the benefit of their clients and place
their clients' interests before their own." "Fiduciaries" is the best answer.

Question: 296

The primary beneficiaries of the AIMR-PPS are:

A. current & prospective clients.


B. AIMR members.
C. all investment professionals.
D. investment firms who adopt the standards.That answer is correct!

Answer: A

Explanation:
Since the PPS strive to achieve greater uniformity and compatibility amongst the various
performance presentations, clients who wish to compare investment results across various firms
benefit the most. Note, however, the mere compliance of a firm with the PPS does not mean that
clients should not perform due diligence before making investment decisions.

Question: 297

NL is a country with no securities laws. LS is a country that has securities laws that are less strict than
the AIMR code of ethics while MS has securities laws that are stricter than the code of ethics. A
securities trader lives in MS but conducts his business in LS. His behavior is governed by MS laws.
However, the MS law states that the applicable business laws are the local laws where the business is
conducted. In that case, the trader must adhere to:

A. LS business laws.
B. AIMR code.
C. MS business laws.
D. a combination of MS and LS business law.

Answer: B

Explanation:
An AIMR member must always adhere to the code of ethics, unless the laws governing his/her
behavior are stricter, in which case, the stricter laws must be followed. In this case, since the LS law
applies to the trader's business behavior, the trader must follow the stricter AIMR code of conduct.

Question: 298

Standard III (B) is ________.

A. None of these answers

Page | 115
B. Disclosure of Additional Compensation Arrangements
C. Responsibilities of Supervisors
D. Disclosure of Conflicts to Employer
E. Obligation to Inform Employer of Code and Standards
F. Duty to Employer

Answer: F

Explanation:
Standard III (A) deals with the Obligation to Inform Employer of Code and Standards. Standard III (B)
deals with the Duty to Employer. Standard III (C) deals with Disclosure of Conflicts to Employer.
Standard III (D) deals with Disclosure of Additional Compensation Arrangements. Standard III (E)
deals with Responsibilities of Supervisors.

Question: 299

Charlice is a portfolio manager who manages (with the help of four analysts) the pension plan assets
of Gordon Industries. Charlice received this account four years ago from Gordon Industries Vice
President, Michael Burnbaum, who is also in charge of investment oversight. To whom does Charlice
owe fiduciary duties?

I. Michael Burnbaum, in his capacity as Gordon Industries' Investment Officer.


II. The employees of Gordon Industries.
III. The pension beneficiaries at Gordon Industries.

A. III only
B. I and III only
C. I only
D. II and III onlyThat answer is correct!

Answer: A

Explanation:
A portfolio manager owes fiduciary responsibility to the beneficiaries of the portfolio he is managing.
In case of pension plans, the loyalty is not owed to the person who hired the manager or to the
entity that set up the fund. Rather, the responsibility lies toward the ultimate plan beneficiaries; in
the case of Gordon Industries, it is the pension plan beneficiaries (though not all employees are
pension recipients so II is not a valid choice). Standard IV (B.1) - Fiduciary Duties - and the Topical
Study, "Fiduciary Duty."

Question: 300

Which of the following can be found in Standard III?

A. Members shall make reasonable efforts to achieve public dissemination of material nonpublic
information disclosed in breach of a duty.
B. Members shall maintain knowledge of and comply with all applicable laws.
C. Members shall maintain appropriate records to support the reasonableness of recommendations.
D. Members shall not participate in plagiarism.
E. Members shall not undertake any independent practice in competition with employer without
written consent.

Answer: E

Page | 116
Explanation:
Standard III states: "Members shall not undertake any independent practice that could result in
compensation or other benefit in competition with their employer without written consent from
both the employer and the person for whom they undertake independent practice."

Question: 301

When complying with Standard IV (B.3) - Fair Dealing, there are certain points one should be sure to
address when establishing compliance procedures. Which of the following points is NOT mentioned
in the Standards of Practice Handbook?

A. Disclose levels of service.


B. Limit the number of people involved.
C. Shorten the time frame between decision and dissemination.
D. Establish procedures for determining material change.
E. Simultaneous dissemination.
F. Disclose all corporate relationships.
G. Publish personnel guidelines for predissemination.
H. Establish control over trading activity.

Answer: F

Explanation:
Under Standard IV (B.3), members have an obligation to ensure that their firms establish compliance
procedures requiring all employees who disseminate investment recommendations or actions to
treat clients fairly. The number of people privy to an investment recommendation should be limited.
Reasonable efforts should be made to limit the amount of time that lapses between the decision and
the dissemination of the recommendation. Guidelines must be established to prohibit persons who
have prior knowledge from discussing a pending recommendation. Trading activities should be
monitored and controlled. Procedures should be established to determine whether a change in an
investment recommendation is considered material. The organization should disclose to firms
whether or not it offers two or more levels of service to clients for the same or different fees.

Question: 302

Standard III (C) deals with ________.

A. None of these answers


B. Plagiarism
C. Obligation to Inform Employer of Code and Standards
D. Use of Professional Designation
E. Disclosure of Conflicts to Employer
F. Fundamental Responsibilities
G. Duty to Employer
H. Professional Misconduct

Answer: E

Explanation:
Standard I deals with Fundamental Responsibilities. Standard II (A) deals with Use of Professional
Designation. Standard II (B) deals with Professional Misconduct. Standard II (C) deals with Plagiarism.
Standard III (A) deals with the Obligation to Inform Employer of Codes and Standards. Standard III (B)
deals with the Duty to Employer. Standard III (C) deals with Disclosure of Conflicts to Employer.

Page | 117
Question: 303

The disclosures for retroactive compliance apply to composites formulated prior to ________.

A. January, 1991
B. January, 1992
C. January, 1989
D. January, 1993
E. January, 1990

Answer: D

Explanation:
The effect date to be in compliance with AIMR-PPS was January 1, 1993. Any composites which
predate the effective date can be brought into compliance retroactively.

Question: 304

Which of the following is NOT true about the responsibilities of a member with supervisory duties, as
specified by Standard III (E) - Responsibilities of Supervisors?

A. Compliance activities must be designed to anticipate the activities most likely to result in
misconduct.
B. Compliance procedures must meet industry and regulatory standards.
C. The supervisor is always accountable for the violations committed by those who report to him.
D. If a member cannot discharge supervisory responsibilities due to an absence of compliance
system, he or she should decline to accept the duties in writing till such a system is put into effect.

Answer: C

Explanation:
Under Standard III (E) - Responsibilities of Supervisors, members must take reasonable care to ensure
that their subordinates do not violate any laws or the code of conduct. This includes designing
effective procedures to deter fraudulent activity. If such procedures are reasonable, meet industry
and regulatory standards, are compatible with the AIMR code and are suitable for the firm in
question, then the member cannot be held responsible for criminal activity clever enough to bypass
the compliance system. Occurrence of such activity does indicate a need for a revamping of the
system, though.

Question: 305

Edwin Hubble, who works for a respectable investment advisory firm, recently found out that his
supervisor has been engaging in "front-running," i.e., executing transactions for personal accounts
before client accounts. As an AIMR member, he is required to do all of the following EXCEPT?

A. Report the illegal activity to the appropriate regulatory authorities.


B. Dissociate himself from the activity.
C. Consult with the legal department on the matter.
D. Inform senior management about the activity with a view of stopping it.That answer is correct!

Answer: A

Explanation:
Standard I does not require a member to report the illegal activity to the appropriate regulatory

Page | 118
authorities, though it may be prudent to do so in many cases if it is evident that the activity is illegal.

Question: 306

Complete the following: According to The Code of Ethics, members of AIMR shall: "Act with integrity,
competence, ________ and in an ethical manner when dealing with the public, clients, prospects,
employers, employees and fellow members."

A. honorability
B. dignity
C. morality
D. none of these answers
E. virtue

Answer: B

Explanation:
According to The Code of Ethics, members of AIMR shall: "Act with integrity, competence, dignity
and in an ethical manner when dealing with the public, clients, prospects, employers, employees and
fellow members."

Question: 307

If AIMR Members, CFA Charterholders and Candidates do not follow the AIMR Performance
Presentation Standards and they make material misrepresentations, they will be in violation of what
Standard of Professional Conduct?

A. Standard V (A)
B. None of these answers. The PPS are voluntary, thus failure to follow them will result in no violation
of the Standards of Professional Conduct.
C. Standard I
D. Standard V (B)

Answer: D

Explanation:
Although the AIMR-PPS are not explicitly incorporated in the AIMR Standards of Professional
Conduct, they should be relied on to help ensure no misrepresentation of performance occurs.
Misrepresentation results in violation of Standard V (B) - Performance Presentation.

Question: 308

With regard to international portfolios, if net performance is indicated, managers are required to
disclose the assumed ________ for the benchmark.

A. hedge
B. disclosure
C. tax rate
D. differentials
E. currency effects

Answer: C

Explanation:

Page | 119
To be in compliance with the AIMR-PPS, the performance presentation for international portfolios
must disclose whether composites and benchmarks are presented gross or net of withholding taxes
on dividends, interest and capital gains; if net, the assumed tax rate for both the composite and the
benchmark.

Question: 309

Standard I of the Standards of Professional Conduct deals with ________.

A. None of these answers


B. Fundamental Responsibilities
C. Fiduciary Duties
D. Priority of Transactions
E. Plagiarism

Answer: B

Explanation:
Standard I of the Standards of Professional Conduct deals with Fundamental Responsibilities.

Question: 310

One of the discretionary accounts managed by Farnsworth is the Jones Corporation employee profit-
sharing plan. Jones, the company president, recently asked Farnsworth to vote the shares in the
profit- sharing plan in favor of the company-nominated slate of directors and against the directors
sponsored by a dissident stockholder group. Farnsworth does not want to lose this account because
he directs all the account's trades to a brokerage firm that provides Farnsworth with useful
information about tax-free investments. Although this information is not of value in managing the
Jones Corporation account, it does help managing several other accounts. The brokerage firm
providing this information also offers the lowest commissions for trades and best execution.
Farnsworth investigates the directors' issue, concludes that management's slate is better for the
long-run performance of the firm than the dissident group's slate and votes accordingly. Farnsworth:

A. Did not violate the Standards in voting the share in the manner requested by Jones or in directing
the trades to the brokerage firm.
B. Violated the Standards in directing the trades to the brokerage firm but not in voting the shares as
requested by Jones.
C. Violated the Standard in voting the shares in the manner requested by Jones and in directing
trades to the brokerage firm.
D. Violated the Standards in voting the shares in the manner requested by Jones but not in directing
trades to the brokerage firm.That answer is correct!

Answer: A

Explanation:
This question pertains to Standard IV (B.1) Fiduciary Duties, in particular members' responsibilities
for voting proxies and the use of client brokerage. Because Farnsworth investigated the issue and
concluded independently the best way to vote, he was not in violation of his fiduciary duties even
though the company president asked him to vote the shares of the profit-sharing plan a certain way.
Moreover because the brokerage firm provides the lowest commissions and best executions, he has
met his fiduciary duties to the client in using this brokerage firm. Since he's not paying any extra
money of the client for the information he's receiving from the brokerage firm, it does not matter.

Question: 311

Page | 120
Level ________ verification applies to the firm.

A. II
B. IV
C. I
D. III
E. 0

Answer: C

Explanation:
Level I verification applies to all firm composites. Level II verification requires a Level I verification at
least on the specific composites being verified at Level II. (Note: There are only two levels of
verification.)

Question: 312

________ investing is investing with a view toward the social or political statements made by such an
investment.

A. None of these answers


B. Charitable
C. Political
D. Social

Answer: D

Explanation:
Fiduciaries often feel pressure from various constituencies to engage in social investing - investing
with a view toward the social or political statements made by such an investment. Members should
keep in mind that although such investment may be benign the fiduciary duty of prudence is
paramount and dictates that trustees judge investments according to the criteria of risk and return,
asset diversification and cost- conscious investing.

Question: 313

Andrea Anastasio heads the research wing at HiLo Funds, Ltd., an investment advisory and money
management firm. Andrea was recently informed by one of her junior analysts, Marcus Cambus, that
the investment recommendation on HotPots food chain stock needed an immediate downgrade
from hold to sell. This was based on talks with HotPots' management whom revealed that HotPots
would be launching an ill-advised expansion plan soon (this information has been discussed in the
financial newspapers, too). Andrea checked Marcus' analysis and agreed with the conclusions.
Deciding to include this in her research newsletter, Andrea informed a few of the portfolio managers
about the change. She did warn them that no action on the news should be taken before the
newsletter was out for at least 4 days. However, Cotler, one of her subordinates and an AIMR
member, inadvertently and prematurely sold off a large chunk of the holdings of Hotpots stock in one
of his larger accounts. A month-end review of accounts DID NOT catch the violation. Which of the
following is/are true?

I. Andrea has not violated any AIMR code but Cotler has violated Standard III (B) - Duty to Employer.
II. Andrea has violated Standard III (E) - Responsibilities of Supervisors.
III. Cotler has violated Standard V (A) - Prohibition against Use of Non-Public Information.
IV. Cotler has violated Standard IV (B.3) - Fair Dealing.

Page | 121
A. III and IV only
B. II and IV only
C. I only
D. II, III and IV only

Answer: B

Explanation:
Since the information has been discussed in the financial newspapers, it certainly is not non-public
information and no violation of Standard V (A) - Prohibition against Use of Non-Public Information
has occurred. However, by treating one of his larger accounts differently, Cotler has violated
Standard IV (B.3) - Fair Dealing which requires him to treat all of his clients fairly, without bias. By
ignoring Andrea's admonitions and committing these violations, he has potentially harmed his
employer (this is in breach of Standard II (B) - Professional Misconduct, not Standard III (B) - Duty to
Employer, which is quite specific about its scope of activities covered). Andrea, on the other hand,
has been negligent in not carrying out her supervisory duties. She should have either not revealed
the change in investment recommendation to her employees or if she deemed it necessary to do so,
should have takenappropriate steps to ensure that violations of her instructions did not occur. Simple
verbal or even written warnings are not good enough to absolve her of violating Standard III (E) -
Responsibilities of Supervisors. Further, the fact that a month- end review of accounts did not detect
Cotler's irresponsible actions implies that stricter or more efficient monitoring is required of Andrea.

Question: 314

Under Standard IV (B.6), Prohibition against Misrepresentation, a member may discuss a


"guaranteed" investment, as long as that investment is which of the following?

A. domestic stocks
B. foreign stocks
C. medium-term notes
D. long-term bonds
E. none of these answers
F. short-term Treasury securities

Answer: F

Explanation:
Standard IV (B.6) prohibits statements that an investment is "guaranteed", or that superior returns
can be expected in the future based on the member repeating past success. It does not preclude
truthful statements on investments with guaranteed returns, such as some insurance contracts,
short-term Treasury securities and insured bank deposits.

Question: 315

According to the AIMR-PPS, performance must be calculated

A. before or after the deduction of trading expenses, as long as that information is disclosed.
B. before or after the deduction of trading expenses, without the need for disclosures because there
will be no difference in the numbers.
C. before the deduction of trading expenses.
D. after the deduction of trading expenses.

Answer: D

Page | 122
Explanation:
Performance must be calculated after the deduction of trading expenses (e.g., broker commissions
and SEC fees), if any. This is a requirement for calculation of returns.

Question: 316

What is the effective date for compliance with the AIMR-Performance Presentation Standards for
discretionary fee-paying portfolios meeting the definition of a wrap-fee account?

A. January 1, 1997
B. July 1, 1994
C. July 1, 1995
D. January 1, 1995

Answer: C

Explanation:
From July 1, 1995, going forward, all of the firm's actual discretionary fee-paying portfolios meeting
the definition of a wrap-fee account must be presented in composites that adhere to the Standards.

Question: 317

Connery, a smart professional, passed his Level II CFA exam 2 years ago. He could not take the Level
III exam due to professional time commitments elsewhere. He is thinking of taking Level III a couple
of years from now, once things settle down on his personal front. His resume, which he has sent to a
few prestigious firms, states that he is a candidate in the CFA program and has successfully
completed Level II exam. He makes no exaggerated claims about his abilities on his resume. Connery
has:

A. violated Standard II (A) - Use of Professional Designation. He cannot claim to be CFA - Level II.
B. not violated any standards.
C. violated Standard II (A) - Use of Professional Designation. He cannot claim to be in the CFA
program since he is not registered to take Level III exam.
D. violated Standard II (A) - Use of Professional Designation. He cannot claim to be in the CFA
program without having completed the Level III exam.

Answer: C

Explanation:
Standard II (A) - Use of Professional Designation, states that a person must be registered to take the
next higher level of the exam to be a "candidate" in the CFA program.

Question: 318

Standard IV (B.4) deals with ________.

A. Disclosure of Conflicts to Clients and Prospects


B. Prohibition against Misrepresentation
C. None of these answers
D. Disclosure of Referral Fees
E. Priority of Transactions
F. Prohibition against Use of Material Nonpublic Information
G. Preservation of Confidentiality

Page | 123
H. Performance Presentation

Answer: E

Explanation:
Standard IV (B.4) deals with the priority of transactions of clients and employers over transactions in
investments in which the member is the beneficial owner.

Question: 319

Which of the following is NOT one of the goals of AIMR-PPS?

A. Enforce a single standard on the various performance presentations through a mandatory


adoption of the PPS.
B. None of these answers.
C. Bolster the notion of self-regulation and enhance the professionalism in the industry.
D. Improve the service offered to investment management clients.That answer is correct!

Answer: A

Explanation:
The PPS are voluntary standards and are not necessarily binding on AIMR members. AIMR, of course,
encourages adoption of these standards. The Standards have been designed to meet the following
four goals: - achieve greater uniformity and compatibility amongst the various performance
presentations; - improve the service offered to investment management clients; - enhance the
professionalism in the industry; - bolster the notion of self-regulation.

Question: 320

In allowing a claim of being in compliance with the PPS, AIMR requires that the compliance be on a
"firmwide" basis. Which of the following does not qualify as "a firm" in this requirement?

A. A subset of assets managed in one or more base currencies.


B. An entity registered with the appropriate regulatory authority.
C. A subsidiary or a division of a parent organization.
D. None of these answers.That answer is correct!

Answer: A

Explanation:
The entire subset of a firm's assets which are managed in one or more bases currencies must be
included in the definition of "a firm" as far as PPS is concerned. For example, an organization cannot
claim a proper subset of the assets it manages in Netherlands as "a firm" and claim compliance for
that subset. It must include the entire portfolio managed in the Netherlands Guilders.

Question: 321

According to the Prudent Investor Rule, the trustee must: - adhere to loyalty, impartiality and
prudence - maintain overall portfolio risk at a reasonable level - provide for reasonable ________ of
trust investments - act with prudence in deciding whether and how to delegate authority to experts
and in selecting and supervising agents - be cost conscious when investing

A. supervision
B. commitment

Page | 124
C. diversification
D. none of these answers

Answer: C

Explanation:
Modern Portfolio Theory dictates that trustees consider a portfolio in its entirety and not just on an
investment-by-investment basis. As a fiduciary, therefore, the trustee must: - adhere to fundamental
fiduciary duties of loyalty, impartiality and prudence. - maintain overall portfolio risk at a reasonable
level - the trade-off between risk and return is the fiduciary's central concern. - provide for
reasonable diversification of trust investments. - act with prudence in deciding whether and how to
delegate authority to experts and in selecting and supervising agents. - be cost conscious when
investing.

Question: 322

________ measures, which relate to the total variability of actual returns (i.e. beta), indicate the risk
of having returns different from that particular benchmark or index.

A. Index
B. Absolute
C. Volatility
D. Risk

Answer: C

Explanation:
It is very important not to confuse risk with volatility. The type of risk that volatility measures
represent are only one of many risks.

Question: 323

Marmella is a corporate associate with an investment banking firm, Corpfins. She recently put
together a business proposal which would allow Corpfins to take on White Papers Inc., a producer of
premium printing paper, as a business client for all its underwriting needs. In this proposal, Marmella
has promised that her team will provide full research coverage for White Papers, with the clause that
the team will not be the first on the street to release a "sell" recommendation. Marmella has

A. violated Standard IV (A.3) - Independence and Objectivity.


B. violated Standard IV (A.1) - Reasonable Basis and Representations.
C. not violated the AIMR code of ethics.
D. violated Standard II (B) - Professional Misconduct.That answer is correct!

Answer: A

Explanation:
By agreeing to a clause that prevents her from releasing an objective research report under all
circumstances, Marmella has violated Standard IV (A.3) - Independence and Objectivity - because not
only is there an appearance of impropriety and conflict of interest, but the contract clearly hinders
Marmella's judgment.

Question: 324

Firms with records or performance calculations for periods prior to the effective dates for AIMR-PPS

Page | 125
compliance can still claim compliance with the PPS using which of the following methods?

I. Restate all of its performance numbers in accordance with the standards.


II. Continue to use the non-conforming performance measures with specific disclosures about how
the measures are not in compliance.
III. Use the relaxed standards of AIMR designed specifically for this situation.

A. I, II and III
B. I only
C. I and II only
D. I and III onlyThat answer is correct!

Answer: A

Explanation:
All three are acceptable ways in which firms with records or performance calculations for periods
prior to the effective dates for AIMR-PPS compliance can continue to claim compliance with the PPS.
This is known as "Retroactive Compliance."

Question: 325

According to the AIMR-PPS, composites must be asset weighted using

A. end-of-period weightings.
B. middle-of-period weightings.
C. performance-period weightings.
D. beginning-of-period weightings.

Answer: D

Explanation:
Composites must be asset weighted using beginning-of-period weightings. This is a requirement for
calculation of returns.

Question: 326

Which of the following is not a required disclosure for real estate investments under the Performance
Presentation Standards?

A. amount of leverage used


B. return formulas
C. all of these answers
D. accounting policies for capital expendituresThat answer is correct!

Answer: A

Explanation:
The performance presentation for real estate must disclose the return formulas and accounting
policies for such items as capital expenditures, tenant improvements and leasing commissions. There
is no requirement or recommendation that the amount of leverage used be disclosed under the 1997
standards.

Question: 327

Page | 126
Regarding Standard III (A), which of the following is true?

A. If the employer has publicly acknowledged, in writing, adoption of AIMR's Code and Standards as
part of its firm policies, then the member need not give formal written notification.
B. A member does not have to notify the supervisor, if the supervisor is a CFA charterholder.
C. Notification must be given to the chief operating officer (or equivalent), orally or in writing.
D. Notification must always be given, orally or in writing.
E. None of these answers is true.That answer is correct!

Answer: A

Explanation:
"Members shall inform their employer, in writing, through their direct supervisor, that they are
obligated to comply with the Codes and Standards and are subject to disciplinary sanctions for
violations thereof." However, if the employer has publicly acknowledged, in writing, adoption of
AIMR's Code and Standards as part of its firm policies, then the member need not give formal
written notification. Subordinate employees cannot assume that their supervisors are aware of the
obligations, even if the supervisors are AIMR members.

Question: 328

Misrepresentation of performance is prohibited by section ________ of the Standards of Professional


Conduct.

A. II (E)
B. III (F)
C. V (B)
D. IV (E)
E. III (A)

Answer: C

Explanation:
Standard V (B) states "Members shall not make any statements, orally or in writing, that
misrepresent the investment performance that they or their firm have accomplished or can
reasonably be expected to achieve."

Question: 329

Standard III of the Standards of Professional Conduct deals with Relationships with and
Responsibilities to ________.

A. AIMR
B. Clients and Prospects
C. None of these answers
D. the Employer

Answer: D

Explanation:
Standard III of the Standards of Professional Conduct deals with Relationships with and
Responsibilities to the Employer.

Question: 330

Page | 127
Standard IV (B.8) deals with ________.

A. Prohibition against Use of Material Nonpublic Information


B. Preservation of Confidentiality
C. None of these answers
D. Prohibition against Misrepresentation
E. Disclosure of Referral Fees
F. Disclosure of Conflicts to Clients and Prospects
G. Priority of Transactions

Answer: E

Explanation:
Standard IV (B.8) states that members shall disclose to all clients any referral fees received by the
member for the recommendation of any services to the client or prospect.

Question: 331

Which of the following statements is NOT correct?

A. Members who work in a country where the Code and Standards impose a lesser degree of
responsibility than local laws and regulations must still adhere to the Code and Standards.
B. All of these statements are correct.
C. Standards II through V address specific conduct.
D. Standard I sets forth minimum standards relating to general activities.That answer is correct!

Answer: A

Explanation:
If a member lives in a country, or provides investment services to a client residing in a country, with
more strict securities laws than the Codes and Standards, he/she must comply with the more strict
laws and regulations of the country.

Question: 332

Which of the following is NOT a form of plagiarism as defined by AIMR code of conduct?

A. Copying proprietary software programs without permission.


B. Using data from a well-known statistical service without attribution.
C. Citing specific quotations to "leading analysts," without specific reference.
D. Using publicized research reports prepared by others without attribution.

Answer: B

Explanation:
Standard II(C) - Prohibition against Plagiarism considers a use of factual information published by
recognized financial and statistical services without attribution acceptable. All of the others are forms
of plagiarism and violations of Standard II(C).

Question: 333

Silverio Arvantis is an aggressive and quick-silver analyst with Foresi Associates, an investment
advisory firm. Silverio, a freshly minted Ph.D. from the University of Chicago, recently formulated an

Page | 128
algorithm for selecting the portfolio weights in relatively new high-tech stocks. The algorithm
evaluates 43 different variables to choose the investments and as such, is fairly complex in nature.
Not wishing to drown his clients in esoteric terms like "martingale" and "stochastic transformation,"
Silverio writes up a short report which touches only briefly on the major characteristics of the model.
Instead, he focuses on the performance of the model on past market data, which is absolutely
sterling. Based on these tests, Silverio recommends 4 stocks for immediate purchase. Silverio has:

A. violated Standard V (B) - Performance Presentation by relying on past data.


B. not violated any standards.
C. violated Standard IV (A.2) - Research Reports by failing to disclose sufficient details about the
algorithm.
D. violated Standard IV (A.2) - Research Reports by failing to distinguish opinion from fact.

Answer: C

Explanation:
Standard IV (A.2) requires the member to disclose details of the reasoning and logic behind the
recommendations so that investors can follow and challenge it and analyze the risks inherent in the
recommended strategy. While Silverio does not have to delve into excessive technical details or
reveal any proprietary formulas, the report must contain explanations for the recommendations. By
omitting these, he has violated Standard IV (A.2). Note, however, that there is no evidence that he
has confused opinion with fact.

Question: 334

Parts of the standards that are ________ must be observed.

A. restraining
B. mandatory
C. recommended
D. suggested
E. inhibiting

Answer: B

Explanation:
Some aspects of the Performance Presentation Standards are mandatory and must be observed,
while other aspects are recommended.

Question: 335

Standard III (B) deals with ________.

A. Use of Professional Designation


B. Fundamental Responsibilities
C. Professional Misconduct
D. Disclosure of Conflicts to Employer
E. None of these answers
F. Duty to Employer
G. Obligation to Inform Employer of Code and Standards
H. Plagiarism

Answer: F

Page | 129
Explanation:
Standard I deals with Fundamental Responsibilities. Standard II (A) deals with Use of Professional
Designation. Standard II (B) deals with Professional Misconduct. Standard II (C) deals with Plagiarism.
Standard III (A) deals with the Obligation to Inform Employer of Codes and Standards. Standard III (B)
deals with the Duty to Employer. Standard III (C) deals with Disclosure of Conflicts to Employer.

Question: 336

When a manager is responsible for the portfolios of pension plans or trusts, the duty of loyalty is
owed to the ________.

A. entity who hires the manager


B. corporation
C. none of these answers
D. manager's supervisor(s)
E. stockholders of the firm
F. board of directors
G. investing public

Answer: C

Explanation:
The first step in fulfilling a fiduciary duty is to determine what the responsibility is and to who it is
owed. Members should take particular care in determining the identity of the "client" to whom the
duty of loyalty is owed. In the context of an investment manager managing the portfolios of pension
plans or trusts, the client is not the person or entity who hires the manager but, rather, the
beneficiaries of the plan or trust. The duty of loyalty is owed to the beneficiaries.

Question: 337

According to the AIMR-PPS, subsectors or carve-outs of larger international composites

A. may be used to create stand-alone composites only if the subsectors are actually managed as
separate entities, they need not have their own cash allocations or currency management.
B. may be used to create stand-alone composites only if the subsectors are actually managed as
separate entities with their own cash allocations and currency management.
C. may never be included in the composite with the parent company.
D. may never be used to create stand-alone composites under any circumstances.

Answer: B

Explanation:
Subsectors or carve-outs of larger international composites may be used to create stand-alone
composites only if the subsectors are actually managed as separate entities with their own cash
allocations and currency management. Portfolios must not be switched from one composite to
another unless documented changes in client guidelines make switching appropriate. This is a
requirement for creation and maintenance of composites.

Question: 338

Rebecca Dustin, a freelance research analyst, is currently working on a research study on the
pharmaceutical industry for KLM Brokerage. She has been using KLM's research facilities in this
endeavor. There has been no written agreement between Rebecca and KLM about the project,
though there was a verbal agreement that KLM would provide reasonable support for the research.

Page | 130
In return, KLM would pay for the final report in case she uncovers something worthwhile. Rebecca is
now interviewing with Jacob & Associates, another brokerage firm, for a project and is considering
submitting a rough draft of her KLM study to them. If she does this without KLM's permission, she
would

A. not be violating the AIMR code, because she is the owner of the study.
B. be violating Standard III (B) - Duty to Employer - because KLM has property rights over the study.
C. be violating Standard III (B) - Duty to Employer - because KLM has first rights over the study.
D. not be violating the AIMR code, since she is not under any written obligation to KLM.

Answer: C

Explanation:
Since Rebecca is acting as a freelance analyst under no written contractual agreement with KLM, the
study does not belong to KLM. However, the verbal agreement and the support received from KLM
make the first right to use the report a KLM property. Hence, Rebecca must allow KLM sufficient time
to act upon her study before using it for any other purpose. Without KLM's permission, Rebecca
cannot release the report without violating Standard III (B).

Question: 339

Standard IV (A) is known as ________.

A. Interactions with Clients and Prospects


B. Preservation of Confidentiality
C. Professional Misconduct
D. Prohibition against Use of Material Nonpublic Information
E. Duty to Employer
F. Fair Dealing
G. None of these answers
H. Investment Process

Answer: H

Explanation:
Standard IV (A) deals with the Investment Process. Standard III (B) deals with Duty to Employer.
Standard IV (B.3) deals with Fair Dealing. Standard IV (B) deals with Interactions with Clients and
Prospects. Standard V (A) deals with Prohibition against Use of Material Nonpublic Information.
Standard IV (B.5) deals with Preservation of Confidentiality. Standard II (B) deals with Professional
Misconduct.

Question: 340

If the senior members of an investment advisory firm are unwilling to disseminate negative
information about a firm, the advisory should

A. none of these answers.


B. refrain from making an investment recommendation citing proprietary reasons.
C. disseminate only factual information about the firm and refrain from making any research
comments.
D. release the report without the negative information.

Answer: C

Page | 131
Explanation:
"Procedures for Compliance," Standard IV (A.3)

Question: 341

According to the AIMR-PPS, account portfolios must be grouped into composites

A. based on similar investment strategy or objective.


B. all of these answers are correct.
C. based on the date the portfolios were initiated.
D. based on the individual account manager's clientele.That answer is correct!

Answer: A

Explanation:
For the defined firm, all fee-paying accounts with investment discretion must be included in one or
more composites. The account portfolios must be grouped into composites based on similar
investment strategy or objective. Compliance cannot be met on a per composite or per product basis
but can only be met on a firmwide basis.

Question: 342

Level ________ verification applies to specific composites.

A. II
B. III
C. I
D. 0
E. IVThat answer is correct!

Answer: A

Explanation:
Level I verification applies to all firm composites. Level II verification requires a Level I verification at
least on the specific composites being verified at Level II. (Note: There are only two levels of
verification.)

Question: 343

Which of the following is/are true about corporate pension plans?

I. The plan manager owes a fiduciary duty to the plan sponsor.


II. The plan manager is held to the "Prudent Man Rule."
III. The plan manager is restricted by a legal list of statutes which limit investments to a specified set
of securities.

A. II and III only


B. II only
C. none of them
D. I and II only

Answer: C

Explanation:

Page | 132
Corporate pension plan managers are governed by ERISA (Employee Retirement Security Act, 1974).
Under this, fiduciaries are held to the "Prudent Expert Rule" and are expected to conduct
investments using appropriate analysis and expertise. They owe fiduciary loyalty solely to the plan
beneficiaries and not to the plan sponsor or any other party whom may be involved. ERISA allows the
plan manager to consider investments in a portfolio context rather than on an individual basis. Thus,
it provides far more latitude and is much more sensible than the old rules that relied on legal lists of
securities for investments. Standard IV (B.1) - Fiduciary Duties - and the Topical Study "Fiduciary
Duty."

Question: 344

Misrepresentation of performance is prohibited by section ________ of the Standards of Professional


Conduct.

A. III (A)
B. II (E)
C. III (F)
D. IV (E)
E. V (B)

Answer: E

Explanation:
Standard V (B) states "Members shall not make any statements, orally or in writing, that
misrepresent the investment performance that they or their firm have accomplished or can
reasonably be expected to achieve."

Question: 345

The Performance Presentation Standards ________ require retroactive compliance.

A. never
B. always
C. sometimes

Answer: C

Explanation:
Retroactive compliance is not required, but it is allowed for firms with performance calculations for
periods prior to the applicable dates that are not in conformance with the AIMR-PPS. In this case, the
firm can restate its historical performance in accordance with the Relaxed Retroactive Standards.

Question: 346

Which of the following AIMR Standards maintains that the financial analyst must preserve the
confidentiality of information communicated to him by the client?

A. IV (A)
B. None of these answers
C. II (B)
D. III (C)

Answer: B

Page | 133
Explanation:
Standard IV (B.5) - Preservation of Confidentiality states: "Members shall preserve the confidentiality
of information communicated by clients, prospects, or employers concerning matters within the
scope of the client-member, prospect-member, or employer-member relationship unless the
member receives information concerning illegal activities on the part of the client, prospect, or
employer."

Question: 347

3 months ago, Fred Hoyle received the AIMR letter, informing him that he had passed Level II exam.
He has now registered for Level III exam. Fred can do which of the following?

I. Put "CFA-II" on his business card, as long as it is in a smaller font than his name.
II. Mention on his resume that he has passed Level II exam of the CFA program.
III. Mention that he is a Level III candidate in the CFA program.
IV. Inform his clients of his professional progress and assure them of his quality as an analyst.

A. I, II and III only


B. II and IV only
C. I, II and IV only
D. II and III only

Answer: D

Explanation:
Standard II (A).

Question: 348

Young, a portfolio manager with Northside Bank, has just been given investment authority for a
newly acquired pension account. Client objectives have not yet been established. On the day the
account is received, $2 million in bonds, representing 4 percent of the portfolio, mature. Which of
the following is Young's best course of action on that day?

A. Invest the proceeds in cash equivalents until a meeting can be arranged to establish fund
objectives.
B. Contact the client's former investment advisor and take investment action based on previously
used guidelines.
C. Invest the proceeds in accordance with the bank's current asset allocation strategy.
D. Make no decision until client objectives have been established.That answer is correct!

Answer: A

Explanation:
This question relates to Standard IV (B.2), Portfolio Investment Recommendations and Actions. This
standard requires that members make a reasonable inquiry into a client's financial situation,
investment experience and investment objectives prior to making any investment recommendation.
To rely on outdated investment guidelines, or firmwide strategies, or to hold the funds, would be
inappropriate because it might violate a member's duty to safeguard client's interests. The best
course of action is to invest in cash equivalents until a meeting can be arranged to establish fund
objectives, because it adequately protects the client's interest until investment objectives are
established.

Question: 349

Page | 134
Jamison is a junior research analyst with Howard & Howard, a brokerage and investment banking
firm. Howard & Howard's mergers and acquisitions department has represented the Britland
Company in all of its acquisitions for the past 20 years. Two of Howard & Howard's senior officers are
directors of various Britland subsidiaries. Jamison has been asked to write a research report on
Britland. What is the best course of action for her to follow?

A. Jamison should not write the report because the two Howard officers are constructive insiders.
B. Jamison may write the report but must refrain from expressing any opinions because of the special
relationships between the two companies.
C. Jamison may write the report if she discloses the special relationships with the company in the
report.
D. Jamison may write the report so long as the officers agree not to alter it.

Answer: C

Explanation:
The question involves Standard IV (B.7), Disclosure of Conflicts to Clients and Prospects. There are
two conflicts of interest that must be disclosed: (1) the company is a client of Jamison's employer, (2)
two directors of the company are senior officers of her employer. As long as the relationship the
analyst's employer has with the company is disclosed, the analyst is not prevented from writing a
report. She can or not express opinions, which is irrelevant to her duty to disclose a conflict of
interest, which is the relationship between the two companies.

Question: 350

When formulating an investment policy for a client, all of the following fall under "investor
constraints," except ________.

A. expected cash flows


B. type and nature of clients
C. regulatory and legal circumstances
D. investor preferences, circumstances and unique needs
E. liquidity needs
F. none of these answers
G. proxy voting
H. investable funds

Answer: B

Explanation:
Type and nature of clients is considered under the "client identification" category.

Question: 351

What is the name of the act enacted in 1940 that reflects a Congressional recognition of the
"delicate" fiduciary nature of an investment advisory relationship and the intent to eliminate, or at
least to expose all conflicts of interest?

A. The Investment Company Act


B. The Congressional Investor's Protection Act
C. The U.S. Investment Advisers Act
D. The U.S. Fiduciary-Investor Act

Page | 135
Answer: C

Explanation:
The U.S. Investment Advisers Act of 1940 (Advisers Act) states "that advisors cannot employ any
device or scheme to defraud any client or prospective client; engage in any transaction or course of
business that may operate as a fraud or deceit upon any client or prospective client; engage in
transactions as a principle or as an agent in a client's account without first disclosing the transaction
to the client and receiving the client's consent; or engage in any act or course of business that is
fraudulent, deceptive, or manipulative."

Question: 352

Performance results for a portfolio need to be presented with cash, ________ equivalents, or
substitute assets.

A. multiple
B. composite
C. average
D. cash

Answer: D

Explanation:
Returns from cash and cash equivalents held in portfolios must be included in return calculations and
the cash and cash equivalents must be included in the portfolio amount (total assets) on which the
return is calculated.

Question: 353

Anamese Barrytone is a broker with Caledonia Investments. Caledonia also provides investment
recommendations through its weekly letter. In its last letter released 3 days ago, Caledonia changed
its investment recommendation on Sell-Me-Now, Inc. from "hold" to "sell." Anamese has just
received a call from Sloan who wants to place a market-buy order on 600 shares of Sell-Me-Now.
Anamese should

A. not execute the order since her firm has placed the stock on the sell list.
B. inform her supervisor about the order and then decide if the order should be executed.
C. execute the order as is without trying to change Sloan's mind by telling him about Caledonia's
recommendation since she is required to be a neutral broker by SEC rules.
D. inform Sloan about the change in Caledonia's recommendation before executing the order.

Answer: D

Explanation:
According to the SEC rules, if a person places an order contrary to current firm recommendation, he
or she should first be informed about the firm's position before accepting the order. Standard IV (B.3)
- Fair Dealing.

Question: 354

Standard IV (B.4), Priority of Transactions, suggests implementing "________" in certain


organizations to limit the number of access persons.

A. Gag Orders

Page | 136
B. Limit Orders
C. Poison Pills
D. Token Confines
E. Fences
F. Fire Walls
G. Corrals

Answer: F

Explanation:
Access or covered persons have knowledge of pending or actual investment recommendations or
action. The firm's definition of access (covered) person should be broad enough to cover all people
with that knowledge. Implementing "Fire Walls' - physical and procedural barriers to prevent the
flow of information from one group to another - may be appropriate in certain organizations to limit
the number of persons with such access.

Question: 355

The performance benchmark for a currency overlay portfolio must be calculated in accordance with
the portfolio's mandates, unless the benchmark is the ________ on a published benchmark.

A. tax return
B. cross hedge
C. currency return
D. mean value
E. standard deviation

Answer: C

Explanation:
Two portfolios with the same benchmark may give different mandates to the overlay portfolio
managers. For example, when the mandate is based on the underlying assets, the return of the
overlay performance benchmark will be different unless the underlying assets of two portfolios imply
identical currency exposures. Consequently, the performance benchmark for a currency overlay
portfolio must be calculated in accordance with the portfolio's mandates, unless the benchmark is
the currency return on a published benchmark.

Question: 356

Armina Stump is a junior portfolio manager on the verge of a promotion at Black Stallion, a strongly
bullish fund management firm. Armina reports to Brouster Bragg, who has been with the firm for
about 8 years. Brouster has been busy over the past 6 months and has not been able to keep up with
account reviews but has complete faith in his subordinate managers. Now that it's time for Armina's
promotion, he has been going over her transactions in the past 6 months and has discovered that all
her trades are going through Blackwell Brokers. Blackwell is a well-reputed brokerage firm and is
known to give extremely competitive prices and speed on trade executions. Due to the bulk
transactions that originate with Armina, Blackwell has been providing research reports and
investment tips to Armina which she has found extremely useful. Brouster does not find anything
wrong with this, though he has now instructed Armina to file her monthly reports in more detail and
in a timely fashion. She explained that due to company work and visits to their Los Angeles office,
she had not been able to complete the reports for two months but she would take additional care in
the future in this regard. Black Stallion has no official policy about filing of account reports. Given this
entire sequence of events, your analysis as a student of AIMR ethics code should reveal which of the
following?

Page | 137
I. Armina has violated Standard I - Fundamental Responsibilities by not filing the reports on time.
II. Brouster has violated Standard III (E) - Responsibilities of Supervisors by failing to enforce proper
supervision.
III. Armina has violated Standard IV (B.1) - Fiduciary Duties by routing her orders solely through
Blackwell.

A. I and II only
B. II and III only
C. II only
D. I, II and III

Answer: C

Explanation:
Since Black Stallion does not require a monthly report from its portfolio managers, Armina has not
violated company policy because she is not responsible for monthly reports. However, Brouster still
must develop procedures to monitor the actions of his subordinate portfolio managers since he is
responsible for their actions. Indeed, the absence of such a policy at Black Stallion makes it
incumbent upon Blackwell to be more diligent about monitoring. By failing to regularly review the
accounts for 6 months, Blackwell has left open the possibility of manipulation by his subordinates
and is thus in violation of Standard III (E) - Responsibilities of Supervisors. Finally, since Armina is
getting best price execution from Blackwell, she has not violated any Fiduciary duties. The fact that
she receives free research advice due to bulk trades does not automatically imply that her client
accounts are being hurt or being made to pay for the services. Additionally, it should be noted that if
Blackwell were to wine and dine Armina as a symbol of appreciation or send her lavish gifts, Armina
may be in violation of the Standard IV (A.3) -Independence and Objectivity - because acceptance of
such gifts creates an appearance of impropriety even if none is present.

Question: 357

Which of the following are not required disclosures under the Performance Presentation Standards?

A. Portfolio size range and percentage of total assets in the same class.
B. The existence of a minimum asset size for the inclusion in composites.
C. The inclusion of any non-fee paying portfolios in composites.
D. Whether balanced portfolio segments are included in single-asset composites.That answer is
correct!

Answer: A

Explanation:
The existence of a minimum asset size below which portfolios are excluded from a composite must
be disclosed. Whether balanced portfolio segments are included in single-asset composites and an
explanation of how cash has been allocated among segments must be disclosed. The inclusion of any
non-fee-paying portfolios in composites and included in the definition of total firm assets must be
disclosed. Only the number of portfolios, their description, the amount of assets in a composite, the
percentage of the firm's total assets the composite represents need be disclosed. The portfolio size
range or percentage of total assets in the same class are only recommended, not required
disclosures.

Question: 358

Information is ________ until it has been disseminated to the marketplace in general and investors

Page | 138
have had an opportunity to react to the information.

A. none of these answers


B. walled
C. material
D. inside
E. nonpublic

Answer: E

Explanation:
Standard V (A) deals with the responsibility of members when they are dealing with the receipt and
disclosure of material, nonpublic information. Information is "nonpublic" until it has been
disseminatedto the marketplace in general and investors have had an opportunity to react to the
information. Mere possession of material nonpublic information will trigger trading restrictions
under Standard V (A).

Question: 359

According to the Standards of Practice Handbook, how frequently should managers gather
information on client circumstances, occupation, investment objectives and risk tolerances?

A. at least monthly
B. at least annually
C. at least every two years
D. at least every five years

Answer: B

Explanation:
Under Standard IV (B.2), members have a responsibility to consider carefully the needs,
circumstances and objectives of the client when determining the appropriateness of a given
investment action. In order to provide sound advice, members should gather information on client
circumstances, occupation, investment objectives and risk tolerances at the inception of any client
relationship and such an inquiry should be repeated at least annually. This will ensure that
investment recommendations take into account the diversity and changing nature of portfolio and
client characteristics.

Question: 360

Victor Lazquez is an analyst with Amro Investments and follows the optical instrument
manufacturers. In this capacity, he has learnt a few trade secrets pertaining to a leading edge firm,
Fibroptica. Recently, he was approached by Fibroptica's competitor, Lensmakers Inc., who offered
him a 3 month project on the optical industry practices. Victor realizes that such a project could
reveal Fibroptica's trade secrets indirectly. However, he decides he will be very careful and accepts
Lensmakers' offer. If he does not inform his employer about this project, he will

A. not have violated any AIMR code since the project does not give rise to a conflict of interest with
Amro's business.
B. have violated Standard III (C) - Disclosure of Conflicts to the Employer.
C. have violated Standard III (B) - Duty to the Employer.
D. have violated Standard III (D) - Disclosure of Additional Compensation Arrangements.

Answer: D

Page | 139
Explanation:
Victor's knowledge about the trade secrets have arisen in course of his employment with Amro. As
such, he has an obligation to get written permission from Amro before he can accept any outside
projects. Note that if such a project had been in direct competition with Amro's business, Victor
would also have violated Standard III (B) - Duty to the Employer.

Question: 361

"Prohibited transactions" are discussed in Standard IV (B.4), Priority of Transactions. Which of the
following is NOT suggested as a firm policy?

A. None of these answers.


B. Participation by investment personnel in equity should be restricted.
C. Firms must determine specific requirements relating to blackout period.
D. All individuals who are involved in the investment decision-making process should be subject to
the same restricted period.That answer is correct!

Answer: A

Explanation:
Under the compliance procedures for Standard IV (B.4), members and their firms should clearly
define prohibited transactions so that employees completely understand their obligations to clients
and their employer. Participation by investment personnel in equity or equity-based IPOs should be
restricted. Also, firm procedures should prevent managers or employees from initiating trades in a
security for which their firms have a pending buy or sell order within a 24-hour period. Firms must
determine specific requirements relating to such blackout periods. All individuals who are involved in
the investment decisionmaking process should be subject to the same restricted period.

Question: 362

Anabella Marconi is a junior industry analyst with Trackstar Blue, an investment advisory service firm.
She has been assigned to follow 3 different firms in the telecommunication industry and reports
directly to the senior manager, Constantine Naples. Constantine manages 9 other analysts and has to
frequently visit Trackstar's other offices around the country. Recently, Anabella received a "friendly
tip" from her friend, Socorro Shue, who works for Quicktel Com, one of the firms that Anabella is
following. Socorro informed Anabella about an ultra-fast broadband channel that was about to be
released by Quicktel. The channel had the potential to catapult Quicktel from the minor league to
competing with national telecom giants. Anabella quickly prepared a memo for Constantine advising
him of her change in the recommendation on Quicktel from hold to buy. Constantine briefly
discussed the reasons with her and asked her to circulate the memo amongst the portfolio
managers. Shortly thereafter, he left for San Francisco for a 10-day business trip. Anabella conferred
with the portfolio managers, who then bought a substantial stake in Quicktel in the open market.

I. Anabella has not violated any AIMR code but Constantine has violated Standard III (E) -
Responsibilities of Supervisors.
II. Anabella has violated Standard III (B) - Duty to Employer.
III. Anabella has violated Standard IV (B.3) - Fair Dealing.
IV. Anabella has violated Standard V (A) - Prohibition against Use of Non-Public Information.

A. III and IV only


B. II and IV only
C. IV only
D. I only

Page | 140
Answer: C

Explanation:
By using a "friendly tip" from an insider in her recommendation, Anabella has violated Standard V (A)
- Prohibition against Use of Non-Public Information. Note that due to this, she is also in violation of
Standard I - Fundamental Responsibilities and Standard II (B) - Professional Misconduct (In general,
any violation of the laws of a regulatory or governmental body will automatically trigger an
infringement of these two laws). However, since she does not handle any client accounts, she is not
in a position to violate Standard IV (B.3) - Fair Dealing. The portfolio managers of Trackstar violate
Standard IV (B.3) if they give preferential treatment to some of their clients while using the
investment recommendation. Constantine, in spreading himself too thin to be able to effectively
manage his subordinates and not checking to see if Anabella was violating any securities laws
(insider trading in this case) is in violation of Standard III (E) - Responsibilities of Supervisors. In fact,
he can be held criminally accountable for not curbing insider trading in his group through sheer
negligence.

Question: 363

If senior managers are unwilling to permit dissemination of adverse opinions about a particular
corporate client, according to Standard IV (A.3), you should attempt to put that corporate client on a
________.

A. holding-pattern
B. restricted list
C. moratorium register
D. none of these answers
E. suspension roster
F. captive list

Answer: B

Explanation:
To avoid violations of Standard IV (A.3), one of the procedures members can enact is to create a
restricted list. If senior managers are unwilling to permit dissemination of adverse opinions about a
particular corporate client, then the firm should remove the controversial company from the
research universe and put it on a restricted list so that the firm disseminates only factual information
about the company.

Question: 364

When formulating an investment policy for a client, which of the following falls under "investor
objectives?"

A. investable funds
B. time horizon
C. risk tolerance
D. proxy voting
E. liquidity needs
F. none of these answers

Answer: C

Explanation:

Page | 141
Risk tolerance is considered under "investor objectives." Liquidity needs, time horizon, proxy voting
and investable funds are considered under "investor constraints."

Question: 365

Standard III (F) is ________.

A. Responsibilities of Supervisors
B. Disclosure of Conflicts to Employer
C. Obligation to Inform Employer of Code and Standards
D. Disclosure of Additional Compensation Arrangements
E. None of these answers
F. Duty to Employer

Answer: E

Explanation:
There is no Standard III (F).

Question: 366

An AIMR member who lives in Atlantis, where some laws are stricter than the AIMR Code of Ethics
and others are less strict, should follow:

A. Atlantis' laws, since the member resides in Atlantis.


B. A combination of the two which results in a stricter set of laws.
C. AIMR code, since the code is a strict set of rules.
D. Not clear from the information provided.

Answer: B

Explanation:
A member must always abide by the AIMR code (Standard I), unless the local laws in any given case
are stricter, in which case, the stricter standard applies.

Question: 367

In order to comply with Standard IV (B.4), Priority of Transactions, firms should prepare and
distribute to firm personnel a code of ethics and compliance procedures. The code and procedures
should do all of the following, EXCEPT:

A. establish reporting and prior-clearance requirements.


B. ensure that procedures will be enforced.
C. consider special situations.
D. contain disciplinary procedures.
E. define personal transactions, investment and prohibited transactions.
F. maximize the number of access persons.

Answer: F

Explanation:
The question deals with the compliance procedures under Standard IV (B.4). Personal transactions
must be defined to all employees. The number of access persons - those who have knowledge of
pending or actual recommendations or action - should be limited, by implementing Fire Walls.

Page | 142
Question: 368

When formulating an investment policy for a client, all of the following fall under "investor
constraints," except ________.

A. regulatory and legal circumstances


B. tax considerations
C. investor preferences, circumstances and unique needs
D. none of these answers
E. the existence of separate beneficiaries
F. investable funds
G. proxy voting
H. liquidity needs

Answer: E

Explanation:
The existence of separate beneficiaries is considered under the "client identification" category.

Question: 369

Which of the following AIMR Standards states that the financial analyst must use reasonable
judgment as to the inclusion of relevant factors in research reports?

A. III (C.2)
B. III A
C. III D
D. IV (A.2)

Answer: D

Explanation:
Standard IV (A.2) - Research Reports states: "Members shall use reasonable judgment regarding the
inclusion or exclusion of relevant factors in research reports."

Question: 370

Meriam Mastrani is a senior member of "The Seven Samurai," a high-flying hedge fund. Meriam
supervises a group of 6 portfolio managers and is also involved in advising some independent clients.
To prevent violations of securities laws at the firm, Meriam has instituted some standard procedures
in conjunction with the Compliance Department. However, for the past 6 months, he has been
extremely busy catering to his clients to be able to check on the weekly compliance reports. Meriam
does not have a deputy whom he can delegate his duties to. No violations of any laws have been
detected at The Seven Samurai. Meriam has

A. has not violated any standard in the AIMR code of ethics.


B. violated Standard III (B) - Duty to the Employer.
C. violated Standard III (E) - Responsibilities of Supervisors.
D. none of these answers.

Answer: C

Explanation:

Page | 143
Standard III (E) - Responsibilities of Supervisors - requires that supervisors must take reasonable care
to ensure that their subordinates do not violate any laws or the code of conduct. This includes
designing effective procedures to deter fraudulent activity. However, merely enacting such
procedures does not fulfill the member's duties. If steps are not taken to make sure that the
restrictions are being abided by, the member could be deemed in violation of Standard III (E). By not
keeping track of the activities of his subordinates for an extended period of time and not having a
deputy to whom such a duty could be delegated, Meriam has violated Standard III (E).

Question: 371

Brown works for an investment counseling firm. Green, a new client of the firm, is meeting with
Brown for the first time. Green used another counseling firm for financial advice for years, but she
has switched her account to Brown's firm. After a few minutes of get-acquainted talk, Brown
explained to Green that she has discovered a highly undervalued stock that offers large potential
gains. She recommends that Green purchase the stock. Brown has committed a violation of the
Standards. What should she have done differently?

A. Brown should have determined Green's needs, objectives and tolerance for risk before making a
recommendation for any type of security.
B. Brown should have explained her qualifications, including her education, training, experience and
the meaning of the CFA designation.
C. Brown should have asked Green her reasons for changing counseling firms. If the discovery
process indicated that Green had been treated unfairly at the other firm, Brown should have notified
AIMR of any violation.
D. Brown should have thoroughly explained the characteristics of the company to Green, including
the characteristics of the industry in which the company operates.That answer is correct!

Answer: A

Explanation:
Brown has provided investment recommendation before making inquiries about the client's financial
situation, investment experience or investment objectives. Brown is thus violating Standard IV (B.2),
Portfolio Investment Recommendations and Actions. Why the client changed investment firms is not
useful information to provide suitable recommendations. The other answers indicate information
members should discuss with their clients at the outset of the relationship, but these answers do not
constitute a complete list of those factors.

Question: 372

Performance Presentation Standards state that subsectors of larger international composites may be
used to create stand-alone composites only if the subsectors are managed as ________ entities.

A. independent
B. two
C. unique
D. separate

Answer: D

Explanation:
Subsectors or carve-outs of larger international composites may be used to create stand-alone
composites only if the subsectors are managed as separate entities with their own cash allocations
and currency management.

Page | 144
Question: 373

Standard III (C) deals with conflicts of interest of a member with ________.

A. other investment professionals


B. none of these answers
C. the client
D. colleagues at the same firm
E. the investing public

Answer: B

Explanation:
Standard III (C) deals with conflicts of interest in any actions or decisions of a member with the
employer. Remember, all Standards under Standard III deal with the employer.

Question: 374

Standard IV (A.3) relates to two major components and is titled Independence and ________.

A. Objectivity
B. None of these answers
C. Impartiality
D. Autonomy
E. JusticeThat answer is correct!

Answer: A

Explanation:
Standard IV (A.3) - Independence and Objectivity, states that members shall use reasonable care and
judgment to achieve and maintain independence and objectivity in making investment
recommendations or taking investment action.

Question: 375

Trisdale is a portfolio manager who has consistently outperformed the market on a risk-adjusted
basis for the past 3 years. As an appreciation for his work, one of his clients recently gave him a travel
package to Vancouver worth around $5,000. Trisdale informed his supervisor about this gift and then
took time off from work to enjoy a vacation in Vancouver. Trisdale has

A. violated Standard III (D) - Disclosure of Additional Compensation Arrangements.


B. violated Standard IV (A.3) - Independence and Objectivity.
C. not violated the AIMR code of ethics.
D. violated Standard IV (B.3) - Fair Dealing.

Answer: C

Explanation:
The AIMR code of ethics does not preclude members from accepting gifts in excess of the modest
amount of $100 from clients, as long as they are disclosed to the immediate supervisor. What the
code does prohibit is acceptance of gifts in excess of $100 from parties that have a strong motive to
influence the judgment of the members (e.g., companies that a member might be researching for
investment recommendations). The disclosure of additional compensation arrangements with clients
allows the supervisory authorities to monitor the portfolio activity and ensure that the gift-giving

Page | 145
client's portfolio is not receiving any undue favorable treatment.

Question: 376

Currency overlay portfolios must be valued at least ________.

A. monthly
B. yearly
C. quarterly
D. hourly
E. daily

Answer: C

Explanation:
In accordance with the AIMR-PPS, currency overlay portfolios must be valued at least quarterly, but
because of the volatile nature of these portfolios, firms may need to revalue currency overlay
portfolios more frequently than quarterly to obtain full and fair disclosure.

Question: 377

Standard III (A) deals with ________.

A. None of these answers


B. Obligation to Inform Employer of Code and Standards
C. Professional Misconduct
D. Use of Professional Designation
E. Plagiarism
F. Duty to Employer
G. Disclosure of Conflicts to Employer
H. Fundamental Responsibilities

Answer: B

Explanation:
Standard I deals with Fundamental Responsibilities. Standard II (A) deals with Use of Professional
Designation. Standard II (B) deals with Professional Misconduct. Standard II (C) deals with Plagiarism.
Standard III (A) deals with the Obligation to Inform Employer of Codes and Standards. Standard III (B)
deals with the Duty to Employer. Standard III (C) deals with Disclosure of Conflicts to Employer.

Question: 378

Standard III (D), Disclosure of Additional Compensation Arrangements is important because

A. outside arrangements may affect loyalties and objectivity and create potential conflicts of interest.
B. none of these answers.
C. outside arrangements are not allowed under circumstances.
D. third parties are inherently in competition with the current employer.
E. the verbal disclosure requirement allows for "compensation in kind."
F. all of these answers.That answer is correct!

Answer: A

Explanation:

Page | 146
Under Standard III (D), members must disclose outside compensation/benefits to employers because
outside arrangements may affect loyalties and objectivity and create potential conflicts of interest.
Disclosure thus allows an employer to consider the outside arrangements when evaluating the
actions and motivations of members. Moreover, the employer is entitled to have full knowledge of
compensation/benefit arrangements to assess the true cost of the outside services members are
providing.

Question: 379

Which of the following is/are required by AIMR-PPS with regards to creation and maintenance of
composites?

I. All actual fee-paying discretionary portfolios must be segregated into one or more composites
based on similar investment strategies.
II. Portfolios must not be switched from one composite to another unless documented changes in
client guidelines make the switching appropriate.
III. Composites must include all terminated portfolios while calculating performance for periods prior
to the termination.

A. II only
B. I, II and III
C. I and II only
D. II and III only

Answer: D

Explanation:
All actual fee-paying discretionary portfolios must be included in at least one composite based on
similar investment strategy. Segregation of these accounts is not required. (II) and (III) are true.

Question: 380

Standard II (B) - Professional Misconduct, states which of the following is a violation?

A. Adding inappropriate expenses to expense reports over an extended period, although never
criminally convicted.
B. Using material prepared by another, in the same form as the original, without acknowledging and
identifying the author, publisher or source of such material.
C. All of these answers.
D. Intoxication at work, which reflects poorly on the profession, raises questions about competence
and affects investment decisions.

Answer: D

Explanation:
Standard II (B) addresses personal integrity and prohibits individual behavior that reflects adversely
on the entire profession. Intoxicated behavior falls under Standard II (B) because it reflects poorly on
the member, his employer and the investment industry. Plagiarizing of material falls under Standard
II (C) - Prohibition against Plagiarism.

Question: 381

Which of the following is/are required by AIMR-PPS with regards to creation and maintenance of
composites?

Page | 147
I. Performance must be measured at least quarterly.
II. Combining of assets-only returns with assets-plus-cash returns is prohibited.
III. Convertible and hybrid securities cannot be mixed with money-market securities in performance
calculations.

A. I and III only


B. I, II and III
C. I only
D. I and II only

Answer: D

Explanation:
Section A - "Creation and Maintenance of Composites" - of the PPS requires that convertible and
hybrid securities be treated consistently across all composites.

Question: 382

Standard III includes which of the following?

A. None of these answers


B. Professional Misconduct
C. Duty to Employer
D. Prohibition against Plagiarism
E. All of these answers
F. Use of Professional Designation

Answer: C

Explanation:
Standard III deals with Obligation to Inform Employer of Code and Standards, Duty to Employer,
Disclosure of Conflicts to Employer, Disclosure of Additional Compensation Arrangements and
Responsibilities of Supervisors.

Question: 383

A(n) ________ is someone who has knowledge of pending or actual investment recommendations or
action.

A. covered person
B. none of these answers
C. AIMR member
D. insiderThat answer is correct!

Answer: A

Explanation:
Access or covered persons have knowledge of pending or actual investment recommendations or
action. The firm's definition of access (covered) person should be broad enough to cover all people
with that knowledge.

Question: 384

Page | 148
Neeson Pacino is the senior vice president in the corporate finance arm of Hindenberry Brokerage.
Neeson was recently approached by Curare Creators, a pesticide manufacturer. Curare would like to
offer new equity to raise capital and has provided Neeson with its current balance sheet and details
about the pending projects. Neeson carefully goes over the numbers with a couple of project
managersat Curare and also two of his analysts at Hindenberry. He concludes from these discussions
that the numbers presented by Curare are overly optimistic. The revised numbers would seriously
lower the offering price. Not relishing this prospect, Neeson decides to go ahead with the numbers
as drawn up by Curare and directs the department to prepare the IPO with the offering price. Neeson
has

A. violated Standard IV (A.1) - Reasonable Basis & Representations.


B. violated Standard IV (B.3) - Fair Dealing.
C. violated Standard IV (B.1) - Fiduciary Duties.
D. violated Standard II (B) - Duty to the Employer.That answer is correct!

Answer: A

Explanation:
By knowingly misrepresenting the situation, Neeson has allowed the possibility that investors would
end up paying more than the fair price in the seasoned equity offering. He has thus violated Standard
IV (A.1) - Reasonable Basis & Representations.

Question: 385

Information is ________ if its disclosure would be likely to have an impact on the price of a security
or if reasonable investors would want to know the information before making an investment
decision.

A. inside information
B. none of these answers
C. breached
D. secured
E. material
F. tipable

Answer: E

Explanation:
According to Standard V (A), information is "material" if it would significantly alter the total mix of
information currently available regarding a security.

Question: 386

Amartya is currently managing the investment fund for a charitable institution and has his fiduciary
duties bound by the UMIFA (Uniform Management of Institutional Funds) rules. His colleague,
Bhagwati, is a trustee of a personal trust fund set up by the late R. D. Tata, a famous industrialist.
Which of the following is/are true about this situation?

I. Amartya's behavior as a trustee is governed by the "Prudent Man Rule."


II. Bhagwati is held to a standard of ordinary business care.
III. In general, Amartya is held to a lower standard of professional conduct than Bhagwati.

A. III only
B. I and II only

Page | 149
C. I, II and III
D. I onlyThat answer is correct!

Answer: A

Explanation:
In the US, the investment actions of personal trust fiduciaries is governed by the "Prudent Man
Rule," which requires an exercise of care and judgment which people of "prudence, character and
intelligence" would use in the management of their personal investment affairs. On the other hand,
trustees of charitable funds, when governed by UMIFA, are held to the standard of ordinary business
care. This standard is comparable to that which applies to someone of the rank of a director in a
business corporation and is not as strict as the Prudent Man Rule. Therefore, Amartya, as a UMIFA
fiduciary, is held to a lower standard than Bhagwati. Standard IV (B.1) - Fiduciary Duties - and the
Topical Study "Fiduciary Duty."

Question: 387

Regarding beneficiaries and remaindermen, current life-income beneficiaries prefer to receive


________; remaindermen would rather have ________.

A. a minimal current income; a high rate of current income


B. small incremental principal repayments; predictable current income
C. none of these answers
D. growth and stability of principal; a high rate of current income

Answer: C

Explanation:
None of these choices is the correct answer. Current life-income beneficiaries prefer to receive high
rate of current income, while remaindermen would rather have growth and stability of principal.

Question: 388

Steffanie Graff is an investment advisor with Wimbledon Advisors, Inc. She recently obtained her CFA
charter, reflected on her new business cards. In a meeting, she told a couple of her new clients
something to the effect of the following: "I have a long record of successful investment advises,
which have led my client accounts to have an annual return of 32% over the past 3 years. I am sure
you will be quite satisfied with my performance." In reality, a large part of this return was caused by
a very luckycoincidence wherein one of the penny stocks in a client account registered a 200%
increase over a short time due to a takeover. Without this, her client accounts would have averaged
less than S&P500 on a risk-adjusted basis. Has Stephanie violated the code of ethics?

A. Yes, because she has misused her CFA charter to imply superior performance.
B. Yes, because the statements misrepresent her track record.
C. Yes, since her presentation of her own performance is in violation of the AIMR-PPP.
D. No, because the statements are factual and she has not promised any future returns.

Answer: B

Explanation:
Standard IV (B.6) - Prohibition against Misrepresentation.

Question: 389

Page | 150
Under Standard III (E) - Responsibilities of Supervisors - which of the following are NOT responsible
for maintaining appropriate supervision when they are in a supervisory role?

A. AIMR members
B. CFA charterholders
C. None of these answers
D. Level I CFA candidates
E. CFA candidates

Answer: C

Explanation:
Standard III (E) states the responsibility of AIMR members, CFA charterholders and candidates for the
CFA designation to take steps to prevent persons acting under their supervision from violating the
law or the Code and Standards.

Question: 390

Standard ________ states that the financial analyst must use particular care to maintain
independence and objectivity in relationships with issuers of securities.

A. I (B.2)
B. III (B.1)
C. IV (A.3)
D. II (C.4)
E. None of these answers

Answer: C

Explanation:
Standard IV (A.3) - Independence and Objectivity - ensures that clients have the benefit of work and
opinions unaffected by any potential conflict of interest that may adversely affect their judgment.
Members should maintain their independence by being wary of "perks" offered by external sources
such as corporations, issuers, underwriters and brokers.

Question: 391

Data Droid manages several investment accounts and directs most of the client transactions through
Lore's brokerage firm. Lore provides him with excellent, reliable research, though his commissions
are higher than industry standards. The research is used to manage all the client accounts. Data has
just been approached by the Troy brokerage firm, which is a newcomer in the business. While this
firm does not provide any research, it charges commissions that are significantly lower than those
charged by Lore. If Data decides to stay with Lore's brokerage firm

A. has not violated the code since he has a long-standing relationship with Lore.
B. he has violated the code of ethics, which requires him to keep the expenses to a minimum.
C. has not violated the code if Lore's research justifies the additional expenses.
D. has violated the code by violating the Prudent Man Rule.

Answer: C

Explanation:
Standard IV (B.1) - Fiduciary Duties

Page | 151
Question: 392

Standard IV (A.3) relates to two major components and is titled ________ and Objectivity.

A. None of these answers


B. Impartiality
C. Justice
D. Autonomy
E. Independence

Answer: E

Explanation:
Standard IV (A.3) - Independence and Objectivity, states that members shall use reasonable care and
judgment to achieve and maintain independence and objectivity in making investment
recommendations or taking investment action.

Question: 393

When a composite of international portfolios is created, firms should separate portfolios that are
allowed to use currency ________ from portfolios that may not use ________, unless the use of
currency ________ is considered immaterial.(same answer for all three spaces)

A. selection
B. hedging
C. exchanges
D. swaps
E. strategy

Answer: B

Explanation:
If a firm makes significant use of currency hedging when allowed, it should separate portfolios that
do not allow such hedging into a different composite and use a different benchmark for the highly
hedged and unhedged composites. Similarly, portfolios with materially different risk exposures do
not belong in the same composite.

Question: 394

Standard III (A) is ________.

A. Duty to Employer
B. Disclosure of Conflicts to Employer
C. Obligation to Inform Employer of Code and Standards
D. Disclosure of Additional Compensation Arrangements
E. None of these answers
F. Responsibilities of Supervisors

Answer: C

Explanation:
Standard III (A) deals with the Obligation to Inform Employer of Code and Standards. Standard III (B)
deals with the Duty to Employer. Standard III (C) deals with Disclosure of Conflicts to Employer.
Standard III (D) deals with Disclosure of Additional Compensation Arrangements. Standard III (E)

Page | 152
deals with Responsibilities of Supervisors.

Question: 395

Performance (relating to Performance Presentation Standards) is the record of the ________.

A. person
B. investment manager
C. individual
D. child
E. firm

Answer: E

Explanation:
To be in compliance with the PPS, a firm's presentation of performance must comply on a firmwide
basis and must consist of 10 years of performance (or records since the date of inception if the firm is
younger than 10 years).

Question: 396

Which of the following can be found in Standard IV?

A. Members shall maintain knowledge of and comply with all applicable laws.
B. Members shall make reasonable efforts to achieve public dissemination of material nonpublic
information disclosed in breach of a duty.
C. Members shall maintain appropriate records to support the reasonableness of recommendations.
D. Members shall not undertake any independent practice in competition with employer without
written consent.
E. Members shall not participate in plagiarism.

Answer: C

Explanation:
Standard IV states: "Members shall maintain appropriate records to support the reasonableness of
recommendations or actions."

Question: 397

Carve-outs may not be combined with ________ portfolios.

A. new
B. exchanged
C. stand-alone
D. composite
E. benchmark

Answer: C

Explanation:
Stand-alone composites from subsectors or carve-outs of larger international portfolios can be
created only if the subsectors are actually being managed as separate entities with their own cash
allocations and currency management - for example, a series of currency funds.

Page | 153
Question: 398

Liliana works for Bear Funds, a brokerage firm. Bear has a policy of not allowing its employees to
trade in securities in which it has large holdings. A list of such securities is circulated on a daily basis
to the employees. Liliana knows of this restriction. However, a few days ago, she happened to
purchase a few shares of a firm that are on Bear's restricted list. The total value of the purchase was
less than $500. Liliana should

A. none of these answers.


B. report the transaction to the compliance department and see how best to unwind the transaction.
C. inform her immediate supervisor about it and accept any sanctions.
D. not do anything, because the activity is of negligible size.

Answer: B

Explanation:
Standard III (C) - Disclosure of Conflicts to Employer.

Question: 399

Which of the following can be found in Standard V?

A. Members shall make reasonable and diligent efforts to avoid any material misrepresentation in
any research report or investment recommendation.
B. Members shall maintain knowledge of AIMR's Code of Ethics.
C. If members receive material nonpublic information in confidence, they shall not breach that
confidence by trading or causing others to trade in securities to which such information relates.
D. Members must use the CFA designation in a dignified manner.
E. Members shall comply with any prohibitions on activities imposed by their employer if a conflict of
interest exists.

Answer: C

Explanation:
Standard V details: "Prohibition against use of material nonpublic information. If members receive
material nonpublic information in confidence, they shall not breach that confidence by trading or
causing others to trade in securities to which such information relates."

Question: 400

Arvantis works for Quick-time Brokerage. He has not been happy with his employment for some time
and recently decided to start his own hedge fund. He started making administrative preparations for
setting up the fund while still working for Quick-time Brokerage. He also made contact with a few of
his oldest clients and gave them details about his hedge fund. The clients promised to switch
accounts once the fund was initiated. Arvantis has:

I. violated Standard III (B) - Duty to Employer by making preparations to enter a competitive business
while still being employed with Quick-time Brokerage.
II. violated Standard III (B) - Duty to Employer by soliciting Quick-time Brokerage's clients while still
being employed with Quick-time Brokerage.
III. not violated Standard III (B) - Duty to Employer.

A. I only
B. II only

Page | 154
C. I and II
D. III only

Answer: B

Explanation:
Standard III (B) - Duty to Employer - does not preclude a departing employee from making
arrangements to enter independent practice prior to leaving current employment, as long as such
preparations do not constitute a breach of loyalty toward the employer. However, undertaking any
activity that harms the current employer is a violation of III (B).

Question: 401

To fulfill the duty to inform their employer that they must follow AIMR's Code and Standard,
members must:

A. inform their immediate supervisor in writing.


B. inform their immediate supervisor either orally or in writing.
C. inform the firm's chief executive officer in writing.
D. inform the firm's chief executive officer either orally or in writing.That answer is correct!

Answer: A

Explanation:
This question relates to a member's duty to inform his/her employer of the Code and Standards. The
proper method to do so is in writing and through the member's direct supervisor.

Question: 402

With regard to international portfolios, if net performance is indicated, managers are required to
disclose the assumed ________ for the benchmark.

A. tax rate
B. currency effects
C. hedge
D. disclosure
E. differentialsThat answer is correct!

Answer: A

Explanation:
To be in compliance with the AIMR-PPS, the performance presentation for international portfolios
must disclose whether composites and benchmarks are presented gross or net of withholding taxes
on dividends, interest and capital gains; if net, the assumed tax rate for both the composite and the
benchmark.

Question: 403

Investments-R-Us is a growing investment advisory firm, which recently hired 5 CFA charter-holders
for senior management positions. In an advertisement, the firm promised clients excellent client
service and timely investment advice. To bolster this motto, all 5 CFA charter-holders were praised in
the advertisement for their commitment, professionalism and high intelligence displayed in passing
the CFA program in 3 straight attempts. IRU has

Page | 155
A. violated the AIMR Presentation Standards by selectively presenting its expertise in the field.
B. violated Standard II (A) - Use of Professional Designation - by implying superior results based on
the fact that the CFA charter-holders passed every exam in first attempt.
C. none of these answers.
D. not violated any of standards in the AIMR code.

Answer: B

Explanation:
Implications that CFA Charter-holders who pass their exams in the first attempt can generate
superior results or display a higher level of intelligence are considered inappropriate by Standard II
(A) of the AIMR Code of Ethics.

Question: 404

The Performance Presentation Standards are designed for

A. investment consultants.
B. current clients.
C. prospective clients.
D. all of these are beneficiaries of the AIMR Performance Presentation Standards.

Answer: D

Explanation:
The primary beneficiaries of the PPS are prospective clients who are attempting to compare
performance of different investment firms. However, current clients attempting to evaluate their
investment managers performance also benefit from the PPS. Consultants may also use the
information when evaluating performance data.

Question: 405

Standard IV (B.6) deals with ________.

A. Preservation of Confidentiality
B. Prohibition against Use of Material Nonpublic Information
C. None of these answers
D. Disclosure of Referral Fees
E. Priority of Transactions
F. Disclosure of Conflicts to Clients and Prospects
G. Prohibition against Misrepresentation
H. Performance Presentation

Answer: G

Explanation:
Standard IV (B.6) prohibits members from making statements that misrepresent the services they are
capable of performing, their qualifications and their academic or professional credentials.

Question: 406

What is the effective date for compliance with the AIMR-Performance Presentation Standards for
non-U.S. and/or non-Canadian investments and taxable portfolios?

Page | 156
A. January 1, 1994
B. January 1, 1993
C. January 1, 1992
D. January 1, 1995That answer is correct!

Answer: A

Explanation:
From January 1, 1994, going forward, all of the firm's actual discretionary fee-paying portfolios
invested in non-U.S. and/or non-Canadian investments ("international portfolios") and taxable
portfolios (both North American and international) must be presented in composites that adhere to
the Standards.

Question: 407

Standard II (B) is known as ________.

A. Professional Misconduct
B. Fair Dealing
C. Interactions with Clients and Prospects
D. Preservation of Confidentiality
E. None of these answers
F. Prohibition against Use of Material Nonpublic Information
G. Duty to Employer
H. Investment ProcessThat answer is correct!

Answer: A

Explanation:
Standard IV (A) deals with the Investment Process. Standard III (B) deals with Duty to Employer.
Standard IV (B.3) deals with Fair Dealing. Standard IV (B) deals with Interactions with Clients and
Prospects. Standard V (A) deals with Prohibition against Use of Material Nonpublic Information.
Standard IV (B.5) deals with Preservation of Confidentiality. Standard II (B) deals with Professional
Misconduct.

Question: 408

Under Standard III (E) - Responsibilities of Supervisors - which of the following are NOT employees
that are covered under this standard?

A. Level I CFA candidates


B. CFA charterholders
C. None of these answers
D. Non AIMR members
E. CFA candidates
F. AIMR members

Answer: D

Explanation:
As long as investment professionals have employees subject to their control or influence - whether or
not the employees are AIMR members, CFA charterholders or candidates for the CFA designation -
they exercise supervisory responsibility and hence, are subject to Standard III (E). The non-AIMR
members themselves are not subject to compliance.

Page | 157
Question: 409

Under ERISA, fiduciaries must:

-act solely in the interest of and for the exclusive purpose of benefiting, the plan participants and
beneficiaries;
-act with the care, skill, prudence and diligence of a prudent person acting in like capacity;
-diversify the plan's investments to protect it from the risk of substantial loss;
-act in accordance with the provisions of the plan documents to the extent that the documents
comply with ________;
-refrain from engaging in prohibited transactions.

A. AIMR regulations
B. AIMR code of ethics
C. None of these answers
D. ERISA

Answer: D

Explanation:
ERISA establishes several guidelines for fiduciary conduct with respect to employee benefit plans.
These principles evolved from the common law of trusts, as interpreted by state courts and pre-
ERISA requirements for the tax qualification of plans. Fiduciaries must observe these guidelines in all
aspects of their dealings with a plan or its assets.

Question: 410

Standard III (C) is ________.

A. Responsibilities of Supervisors
B. None of these answers
C. Duty to Employer
D. Disclosure of Conflicts to Employer
E. Disclosure of Additional Compensation Arrangements
F. Obligation to Inform Employer of Code and Standards

Answer: D

Explanation:
Standard III (A) deals with the Obligation to Inform Employer of Code and Standards. Standard III (B)
deals with the Duty to Employer. Standard III (C) deals with Disclosure of Conflicts to Employer.
Standard III (D) deals with Disclosure of Additional Compensation Arrangements. Standard III (E)
deals with Responsibilities of Supervisors.

Question: 411

Jurgens is a portfolio manager with an investment firm based in New York. One of her firm's clients
has told Jurgens that he will compensate her beyond that provided by her firm on the basis of the
capital appreciation of his portfolio each year. Jurgens should:

A. Turn down the additional compensations because it will result in conflicts with the interest of
other client's accounts.
B. Turn down the additional compensation because it will create undue pressure on her to achieve

Page | 158
strong short-term performance.
C. Obtain permission from her employer prior to accepting the compensation arrangement.
D. Receive permission from AIMR for the compensation arrangement.

Answer: C

Explanation:
This question pertains to Standard III (D), Disclosure of Additional Compensation Arrangements. If
Jurgens were to be compensated, based on the account's performance, beyond that provided by her
firm, such a practice is not a violation of the Standards, so long as Jurgens discloses the arrangement
inwriting to her employer and obtains permission from the employer before entering the agreement.
Members are not required to receive permission from AIMR for such arrangements.

Question: 412

Members can comply with Standard IV (A.1) by undertaking the following:

I. Analyzing basic investment characteristics


II. Analyzing portfolio needs of the client
III. Establishing compliance procedures to prevent violations of Code and Standards
IV. Maintaining files to support investment recommendations

A. I, II & IV
B. II & IV
C. I & IV
D. I & II
E. II, III & IV
F. III & IV
G. I, II, III & IV
H. II & IIIThat answer is correct!

Answer: A

Explanation:
Condition III applies to compliance with Standard III (E). For Standard IV (A.1), which deals with the
Investment Process, members should investigate the investment's basic characteristics by looking at
research reports. Also, a member has a continuing responsibility to review a client's needs and
circumstances to determine whether or not the risk exposure of the total portfolio serves the client's
needs. Finally, a member must maintain research files to justify investment decisions later under
scrutiny.

Question: 413

Standard V of the Standards of Professional Conduct deals with Relationships with and
Responsibilities to ________.

A. Supervisors
B. Employers
C. Employees
D. the Investing Public
E. None of these answers

Answer: D

Page | 159
Explanation:
Standard V of the Standards of Professional Conduct deals with Relationships with and
Responsibilities to the Investing Public.

Question: 414

Cornelius is a portfolio manager with Apex Investments, an investment advisory firm. Cornelius has,
over the years, developed a special symbiotic relationship with Mike Milken, the owner of Milk 'em,
Inc., a small brokerage firm. Cornelius puts subtle pressure on the trading desk at Apex to execute its
trades through Milk 'em, thus generating brokerage revenue for Mike. In return, Mike recommends
the services of Apex Investments to many of its clients. This arrangement is not disclosed to either
thesenior management at Apex nor to any of the clients. It has been observed by many at Apex
Investments that the commissions charged by Milk 'em are 10-15% higher than those by other
brokers. However, Cornelius has justified the higher costs by pointing to the extra revenue-flow from
Milk 'em. Cornelius has:

I. not violated any AIMR standards.


II. violated Standard III (C) - Disclosure of Conflicts to Employer.
III. violated Standard IV (B.8) - Disclosure of Referral Fees.
IV. violated Standard IV (B.1) - Fiduciary Duties.

A. II, III and IV only


B. II and IV only
C. I only
D. II and III onlyThat answer is correct!

Answer: A

Explanation:
Cornelius is clearly in a situation representing a conflict of interests. His arrangement and non-
disclosure of the arrangement are unfair to the current clients of Apex and potential clients
recommended by Milk 'em. They represent a violation of his fiduciary duties to the clients. By not
informing senior management at Apex about the arrangement, he has also violated Standard III (B) -
Disclosure of Conflicts to Employer.

Question: 415

According to the Prudent Investor Rule, the trustee must:

- adhere to ________, impartiality and prudence


- maintain overall portfolio risk at a reasonable level
- provide for reasonable diversification of trust investments
- act with prudence in deciding whether and how to delegate authority to experts and in selecting
and supervising agents
- be cost conscious when investing

A. supervisors
B. loyalty
C. none of these answers
D. commitment
E. diversification

Answer: B

Page | 160
Explanation:
Modern Portfolio Theory dictates that trustees consider a portfolio in its entirety and not just on an
investment-by-investment basis. As a fiduciary, therefore, the trustee must:

- adhere to fundamental fiduciary duties of loyalty, impartiality and prudence.


- maintain overall portfolio risk at a reasonable level
- the trade-off between risk and return is the fiduciary's central concern.
- provide for reasonable diversification of trust investments.
- act with prudence in deciding whether and how to delegate authority to experts and in selecting
and supervising agents.
- be cost conscious when investing.

Question: 416

Which of the following statements is correct regarding Standard II (A) Use of Professional
Designation?

A. Joe Martin passed Level I and Level II of the CFA exams and is scheduled for the next Level III
exam. He may write "Joe Martin, CFA II."
B. All of these answers are correct.
C. Joe Martin passed Level I and Level II of the CFA exams and is scheduled for the next Level III
exam. He may write, "I am a Level III candidate in the CFA program."
D. Joe Martin passed Level I and Level II of the CFA exams, but is not scheduled for the next Level III
exam. He may write, "I am a Level III Candidate in the CFA program."

Answer: C

Explanation:
"Joe Martin, CFA II" is a misrepresentation and a violation of Standard II (A). There is no designation
for someone who has passed Level I, Level II, or Level III. He may not state he is a candidate unless
he is registered for the next exam.

Question: 417

Sid Barnes is the senior most partner with Noble & Noble, a well-known brokerage firm. Bruce
Lohmann is a senior partner who reports to Barnes. Lohmann is an AIMR member while Barnes is
not. Recently, N&N's research department identified a sterling investment opportunity and Barnes
decided to allow some of the largest discretionary accounts to benefit from this first. He directed
Lohmann to take the appropriate steps and in turn, Lohmann assigned Doug Jardine, a senior analyst,
to complete the portfolio rebalancing. Doug, an AIMR member, informed Bruce and Barnes that such
a course of action would be in violation of the AIMR Standard IV (B.3) - Fair Dealing. Barnes told him
that he was not aware of any such code and that in any case, the firm had not adopted it. If Doug
refuses to carry out the action but Lohmann does, which of the following is/are true?

I. Barnes has violated the AIMR code.


II. Lohmann has violated the AIMR code.
III. Doug has violated the AIMR code by disobeying his superiors.

A. I, II and III
B. II only
C. I and III only
D. II and III only

Answer: B

Page | 161
Explanation:
Barnes cannot be said to have violated the AIMR code since he's not a member or a CFA candidate
and does not have to abide by those rules. Lohmann, on the other hand, has violated the code in two
ways: first, as the senior most AIMR member in the firm, it was his responsibility to ensure that
Barnes knew not only about the AIMR code but also Lohmann's obligation to uphold it. Barnes'
ignorance about the code implies Lohmann violated Standard III (A) - Obligation to Inform Employer
of Code & Standards. Second, by allowing some of the largest discretionary accounts to benefit from
the research first, he violated AIMR standard IV (B.3) - Fair Dealing. Doug, on the other hand,
dissociated himself from the activity and did not violate any standard.

Question: 418

If the use of leverage is ________, the performance presented must include the effects of the
leverage.

A. discretionary
B. mandatory
C. nondiscretionary
D. optionalThat answer is correct!

Answer: A

Explanation:
If the use of leverage is nondiscretionary (i.e. mandated by the client), performance must be
presented on an all-cash basis.

Question: 419

When creating composites, ________ returns must not be mixed with asset-plus-cash returns.

A. model
B. portfolio
C. cash
D. performance
E. asset-only

Answer: E

Explanation:
To be in compliance with the PPS, a firm creating a composite must meet this requirement.

Question: 420

When complying with Standard IV (B.3) - Fair Dealing, there are certain points one should be sure to
address when establishing compliance procedures. Which of the following points is NOT mentioned
in the Standards of Practice Handbook?

A. Simultaneous dissemination.
B. Establish procedures for determining material change.
C. Disclose levels of service.
D. Publish personnel guidelines for predissemination.
E. Limit the number of people involved.
F. Be flexible on the time frame between decision and dissemination.

Page | 162
G. Establish control over trading activity.

Answer: F

Explanation:
Members have an obligation to ensure that their firms establish compliance procedures requiring all
employees who disseminate investment recommendations or actions to treat clients fairly. The
number of people privy to an investment recommendation should be limited. The amount of time
that lapses between the decision and the dissemination of the recommendation should be
shortened. Guidelines must be established to prohibit persons who have prior knowledge from
discussing a pending recommendation. Trading activities should be monitored and controlled.
Procedures should be established to determine whether a change in an investment recommendation
is considered material. The organization should disclose to firms whether or not it offers two or more
levels of service to clients for the same or different fees.

Question: 421

Michelieu tells a prospective client, "I may not have a long-term track record yet, but I'm sure that
you'll be very pleased with my recommendations and service. In the three years that I've been in the
business, my equity-oriented clients have averaged a total return of more than 26 percent a year."
The statement is true, but Michelieu only has a few clients and one of his clients took a large position
in a penny stock (against Michelieu's advice) and realized a huge gain. This large return caused the
average of all of Michelieu's clients to exceed 26 percent a year. Without this one investment, the
average gain would have been 8 percent a year. Has Michelieu violated the Standards?

A. Yes, because the statement misrepresents Michelieu's track record.


B. Yes, because the statement about return ignores the risk preferences of his clients.
C. No, because the statement is a true and accurate description of Michelieu's track record.
D. No, because Michelieu is not promising that he can earn a 26 percent return in the future.That
answer is correct!

Answer: A

Explanation:
Standard IV (B.6), Prohibition against Misrepresentation. Although Micheliu's statement regarding
the total return of his client's accounts on average may be technically true, it is misleading because
the majority of the gain resulted from one client's large position taken against Micheliu's advice. He
has not taken steps to present a fair, accurate and complete presentation of performance. Even
though he is not guaranteeing future results, his words are still a misrepresentation of performance.
Not disclosing the risk preferences of clients does not make a statement misleading and is not a
violation of the Standards in this context.

Question: 422

Level I verification requires independent attestation that portfolio returns are calculated according to
a(n) ________ weighted return methodology.

A. size
B. price
C. time
D. risk
E. asset

Answer: C

Page | 163
Explanation:
Portfolio returns must be calculated according to a time-weighted return methodology with a
minimum of quarterly valuation and accrual of income for fixed-income securities.

Question: 423

Martha Maris is a qualified investment advisor who has been handling investment accounts of a few
wealthy friends for a fee. She recently interviewed with Capital Management, Inc. and accepted a
position as a money manager with them. She:

A. needs to inform only Capital Management about her old clients.


B. must inform Capital Management as well as all her old clients in writing about the arrangement.
C. does not have to inform her client friends about her employment with Capital Management.
D. does not have to inform Capital Management about her old clients since they were not retained
through Capital Management.

Answer: B

Explanation:
This is required by Standard III (B) - Duty to Employer.

Question: 424

Performance Presentation Standards require that income and capital appreciation be presented in
addition to total return when dealing with ________.

A. anti-linear appreciation
B. real estate
C. none of these answers
D. long-term liabilities

Answer: B

Explanation:
For real estate, PPS require that income and capital appreciation be presented in addition to total
return.

Question: 425

A supervisor complies with Standard III (E) by establishing and enforcing ________ to prevent
violations.

A. legal requirements
B. fiduciary duties
C. none of these answers
D. code of ethics
E. compliance procedures

Answer: E

Explanation:
A supervisor complies with Standard III (E) by identifying situations in which legal violations or
violations of the Code and Standards are likely to occur and establishing and enforcing compliance

Page | 164
procedures to prevent such violations.

Question: 426

Which of the following are characteristics of material nonpublic information?

A. The information may be considered relevant by reasonable investors.


B. The information would have a substantial impact on the market if released.
C. All of these answers.
D. The information has not been generally disclosed.

Answer: C

Explanation:
Information is "material" if its disclosure would be likely to have an impact on the price of a security
or if reasonable investors would want to know the information before making an investment
decision. In other words, information is material if it would significantly alter the total mix of
information currently available regarding a security. Information is "nonpublic" until it has been
disseminated to the marketplace in general and investors have had an opportunity to react to the
information.

Question: 427

According to the AIMR-Performance Presentation Standards, ________ must be used when


calculating investment performance.

A. benchmark
B. asset-weighted return
C. aggregate return
D. total return

Answer: D

Explanation:
Total return, including realized and unrealized gains plus income, must be used when calculating
investment performance. Portfolios must not be switched from one composite to another
unlessdocumented changes in client guidelines make switching appropriate. This is a requirement for
calculation of returns.

Question: 428

Standard V (B) deals with

A. independence and objectivity.


B. performance presentation.
C. preservation of confidentiality.
D. insider information.
E. fair dealing.
F. research report.

Answer: B

Explanation:
The purpose of Standard V (B) is to state the responsibility of AIMR members to avoid

Page | 165
misrepresentation of the investment performance and encourages full disclosure of investment
performance data to clients and client prospects.

Question: 429

The ________ assumption must be disclosed if results are presented after taxes.

A. tax rate
B. trade
C. composite
D. selection
E. settlement-dateThat answer is correct!

Answer: A

Explanation:
To be in compliance with the PPS, a firm's presentation of its investment performance must disclose
the tax rate information.

Question: 430

Performance results for a portfolio need to be presented with cash, ________ equivalents, or
substitute assets.

A. composite
B. multiple
C. cash
D. average

Answer: C

Explanation:
Returns from cash and cash equivalents held in portfolios must be included in return calculations and
the cash and cash equivalents must be included in the portfolio amount (total assets) on which the
return is calculated.

Question: 431

According to the AIMR-PPS, plan sponsors, consultants and software vendors

A. are exempt from any of the Standards.


B. are directly responsible for ensuring the companies they employ are in compliance with the
Standards.
C. are held accountable to the Standards in the same way as AIMR Members, Charterholders and
Candidates.
D. cannot make a claim of compliance unless these entities actually manage the assets for which
they are making the claim of compliance.

Answer: D

Explanation:
Plan sponsors, consultants and software vendors cannot make a claim of compliance unless these
entities actually manage the assets for which they are making the claim of compliance. These groups
can claim to endorse the Standards and/or require that the investment management firms they

Page | 166
employ, from which they solicit information, or to which they sell be in compliance with the
Standards.

Question: 432

Sunil Vaskar is an employee of Glamorgan, a reputable investment banking firm. Glamorgan is a


major underwriter for an equity issued by a private firm, Raindrop Waterworks. Sunil has
accidentally discovered that the preliminary prospectus issued by Raindrop contains material
overstatements about the scope of its businesses and future earning potential. Sunil:

A. must report his findings to his supervisor and consult his legal department.
B. must sever all connections with Glamorgan and dissociate himself from its activities.
C. need not do anything because the prospectus has been distributed and is, in any case, preliminary.
D. must report the case to the SEC since there has been a violation of laws governing equity
issues.That answer is correct!

Answer: A

Explanation:
Standard I requires that suspected violation of laws be reported to appropriate supervisory people in
the firm. It is also advisable to consult the legal department and dissociate oneself from the illegal
activity. Sunil has to dissociate himself from the underwriting process involving Raindrop but he does
not have to sever connections with his employer, Glamorgan.

Question: 433

Stewart has been hired by Goodner Industries, Inc., to manage its pension fund. Stewart's fiduciary
duty is owed to:

A. the management of Goodner.


B. the participants and beneficiaries of Goodner's pension plan.
C. the shareholders of Goodner.
D. each of these answers equally.

Answer: B

Explanation:
Under Standard IV (B.1), Fiduciary Duties, members who manage a company's pension funds owe a
fiduciary duty to the participants and benefits of the plan, not the management of the company or
the company shareholders.

Question: 434

When formulating an investment policy for a client, which of the following falls under "investor
objectives?"

A. proxy voting
B. none of these answers
C. return objectives
D. time horizon
E. liquidity needs
F. investable funds

Answer: C

Page | 167
Explanation:
Return objectives are considered under "investor objectives." Liquidity needs, time horizon and
proxy voting are considered under "investor constraints."

Question: 435

Standard II (B) deals with ________.

A. None of these answers


B. Plagiarism
C. Use of Professional Designation
D. Fundamental Responsibilities
E. Professional Misconduct
F. Obligation to Inform Employer of Code and Standards
G. Duty to Employer
H. Disclosure of Conflicts to Employer

Answer: E

Explanation:
Standard I deals with Fundamental Responsibilities. Standard II (A) deals with Use of Professional
Designation. Standard II (B) deals with Professional Misconduct. Standard II (C) deals with Plagiarism.
Standard III (A) deals with the Obligation to Inform Employer of Codes and Standards. Standard III (B)
deals with the Duty to Employer. Standard III (C) deals with Disclosure of Conflicts to Employer.

Question: 436

Standard III (B) is known as ________.

A. Interactions with Clients and Prospects


B. Preservation of Confidentiality
C. None of these answers
D. Fair Dealing
E. Investment Process
F. Duty to Employer
G. Prohibition against Use of Material Nonpublic Information
H. Professional Misconduct

Answer: F

Explanation:
Standard IV (A) deals with the Investment Process. Standard III (B) deals with Duty to Employer.
Standard IV (B.3) deals with Fair Dealing. Standard IV (B) deals with Interactions with Clients and
Prospects. Standard V (A) deals with Prohibition against Use of Material Nonpublic
Information.Standard IV (B.5) deals with Preservation of Confidentiality. Standard II (B) deals with
Professional Misconduct.

Question: 437

The allocation of shares in oversubscribed IPOs to investment managers for their personal account is
a perk that is most clearly a violation of Standard ________.

A. II (B) - Professional Misconduct

Page | 168
B. IV (A.3) - Independence and Objectivity
C. None of these answers
D. IV (B.6) - Prohibition against Misrepresentation
E. IV (B.8) - Disclosure of Referral Fees

Answer: B

Explanation:
External sources may try to influence the investment process by offering analysts and portfolio
managers a variety of perks. Corporations may be seeking expanded research coverage; issuers and
underwriters may wish to promote new securities offerings; brokers typically want to increase
commission business. The perks may include gifts, invitations to lavish functions, tickets and so on.
One type of perk that has gained particular notoriety is the allocation of shares in oversubscribed
IPOs to investment managers for their personal accounts. This practice is a violation under Standard
IV (A.3).

Question: 438

Tomonaga Olawando is a research analyst currently doing research on Cleebok Shoes, a footwear
manufacturer infamous for its exorbitantly priced shoes. Tomonoga has been interviewing Pollyanna
Givens, a senior vice president with the public relations division. Pollyanna has told him that the
investment community has underestimated the payoffs from Cleebok's plan to outsource shoe
production to Mexico, thus cutting productions costs significantly. She told him her estimate puts
thecost savings close to $600 million, as against the figure of $370 million quoted by a few active
analysts. In his report, Tomonaga states that the extra savings of $230 million a year will raise the
stock price by 26% over the next year and hence, represents a great buy. Tomonaga has

I. not violated any code of ethics.


II. has violated Standard IV (A.1) - Reasonable Basis & Representations.
III. has violated Standard IV (A.2) - Research Reports.
IV. has violated Standard IV (B.2) - Portfolio Investment Recommendations and Actions.

A. III only
B. II and III only
C. II, III and IV only
D. I only

Answer: B

Explanation:
By taking an opinion of a person and representing as a fact in his research report, Tomonaga has
violated Standards IV (A.1) and IV (A.2).

Question: 439

The AIMR-PPS recommend that both ________ and ________ risks be presented in conjunction with
composite returns.

A. absolute; unsystematic
B. total; market
C. external; internal
D. unsystematic; systematic

Answer: B

Page | 169
Explanation:
Total risk relates to the variability of actual (absolute) returns not relative to the market. Market
(systematic) risk relates to the volatility of returns relative to the market or some other benchmark.

Question: 440

Dayon Grimley has been working as a Tech Sector analyst with an advisory firm for the past 3 years.
Recently, he read an analytical report by Sam Swetson about the trends and research developments
in the computer chips industry. The report was an in-depth analysis of how the industry had
developed, where it seemed to be headed and what impact the developments could have on the
growth of the entire industry. In a footnote, Sam had speculated about a secret, ultra-fast chip
rumored to be under development by Chipcorn, Inc. Dayon decided to pursue this line of research
and called up a highly placed official at Chipcorn. While the official did not confirm or deny the
existence of the ultra-fast chip, he did reveal enough for Dayon to conclude that the development
was, in fact, in the final stages and the chip could be out in the market in as little as 3 months. He
quickly modified Sam's report, with a separate section on the super-chip and recommended that his
clients increase the weighting on tech stocks by 20%. The report made no mention about his
interview with Chipcorn official nor Sam Swetson's report. Dayon has:

I. violated Standard II (C) - Prohibition against Plagiarism.


II. violated Standard II (B) - Professional Misconduct.
III. violated Standard IV (A.1) - Reasonable Basis & Representations.
IV. violated Standard V (A) - Prohibition against Use of Material Nonpublic Information.

A. II and IV only
B. II and IV only
C. I and IV only
D. I and III only

Answer: C

Explanation:
By not properly acknowledging Sam's report, Dayon violated Standard II (C) - Prohibition against
Plagiarism. Also, since the information about the super-chip was non-public and obviously material,
Dayon violated Standard V (A) - Prohibition against Use of Material Nonpublic Information.

Question: 441

Standard V (B) deals with ________.

A. None of these answers


B. Priority of Transactions
C. Preservation of Confidentiality
D. Performance Presentation
E. Prohibition against Use of Material Nonpublic Information
F. Disclosure of Conflicts to Clients and Prospects
G. Prohibition against Misrepresentation
H. Disclosure of Referral Fees

Answer: D

Explanation:
Standard V (B) - Performance Presentation - states that members shall not make any statements that

Page | 170
misrepresent the investment performance that they or their firm have accomplished or can expect to
achieve.

Question: 442

Standard III (E) - Responsibilities of Supervisors points out that when violations do occur, ________
procedures may be inadequate.

A. compliance
B. none of these answers
C. legal
D. fiduciary dutyThat answer is correct!

Answer: A

Explanation:
If a member has adopted reasonable procedures and taken steps to institute an effective compliance
program, then the member may not be in violation of Standard III (E). The fact that violations do
occur may indicate, however, that the compliance procedures are inadequate. Furthermore, a
member may be in violation of Standard III (E) if he or she knows that the procedures designed to
detect and prevent violations are not being followed.

Question: 443

In a pension plan, the duty of a fiduciary is to the ________.

A. Board of Directors
B. none of these answers
C. plan sponsor
D. immediate supervisor
E. plan participants and their beneficiaries

Answer: E

Explanation:
In a pension plan, the duty of the fiduciary is to the plan participants and their beneficiaries rather
than to the plan sponsor that has the power to hire and fire the investment manager. If urged to
make investments that might be of direct benefit to a sponsoring community or to the community at
large, the manager must ensure that such investments are legal and do not impair the integrity of
the funds in question or the financial security of the participants/beneficiaries.

Question: 444

The stated purposes of Standard IV (B) (8), Disclosure of Referral Fees, are to:

I. Help the customer or client evaluate the full cost of the services.
II. Help the customer or client evaluate any possible partiality.
III. Help the customer or client evaluate potential conflicts of interest as a result of the participation
of immediate family in transactions.

A. I only.
B. II only.
C. III only.
D. I and II only.

Page | 171
E. II and III only.
F. I and III only.
G. I, II and III.

Answer: D

Explanation:
This question relates to Standard IV (B.8), Disclosure of Referral Fees. Statements I & II give the two
primary reasons listed in the Standards of Practice Handbook for disclosing referral fees to clients.
The purpose given in Statement III is not a primary consideration.

Question: 445

Real estate must be valued through an independent appraisal at least ________ unless client
agreements state otherwise.

A. once every five years


B. once every year
C. once every three years
D. once every two years

Answer: C

Explanation:
Real estate must be valued through an independent appraisal at least once every three years unless
client agreements state otherwise. Real estate valuations must be reviewed at least quarterly. This is
a requirement for calculation of returns.

Question: 446

Which of the following can be found in Standard IV?

A. Members shall not participate in any professional conduct involving dishonesty, fraud, deceit, etc.
B. Members shall exercise diligence and thoroughness in making investment recommendations or in
taking investment actions.
C. Members shall not knowingly participate or assist in any violation of laws, rules, or regulations.
D. Members shall deliver a copy of the Code to their employer.
E. Members shall not misrepresent investment performance.

Answer: B

Explanation:
Standard IV states that members shall exercise diligence and thoroughness in making investment
recommendations or in taking investment actions.

Question: 447

Serena Zaltz is a portfolio manager at Katalina Investments, a small boutique in Connecticut. She
currently manages 3 client accounts, one of which belongs to John Hersham. John recently told
Serena that if his portfolio beat the S&P500 by 75 basis points over the next 3 months, he would give
10 basis points to her as a "superior performance reward." Serena told him this was unnecessary but
John insisted that such an arrangement be made on a handshake. Serena subsequently spoke to her
supervisor, Helena, about it and Helena did not object to the arrangement. Then, Serena has:

Page | 172
A. has violated Standard IV (A.3) - Independence and Objectivity.
B. has violated Standard III (D) - Disclosure of Additional Compensation Arrangements.
C. not violated any code of conduct.
D. has violated Standard IV (B.8) - Disclosure of Referral Fees.

Answer: B

Explanation:
While Serena may have informed her employer orally about her additional compensation
arrangement, Standard III (D) requires written notification to the employer and this includes any
form of communication that can be documented. Such written disclosures act as paper trails of all
such arrangements and act as a deterrent to such arrangements except in the more compelling
cases.

Question: 448

Standard IV (B.5) is known as ________.

A. None of these answers


B. Preservation of Confidentiality
C. Fair Dealing
D. Investment Process
E. Duty to Employer
F. Interactions with Clients and Prospects
G. Prohibition against Use of Material Nonpublic Information
H. Professional Misconduct

Answer: B

Explanation:
Standard IV (A) deals with the Investment Process. Standard III (B) deals with Duty to Employer.
Standard IV (B.3) deals with Fair Dealing. Standard IV (B) deals with Interactions with Clients and
Prospects. Standard V (A) deals with Prohibition against Use of Material Nonpublic Information.
Standard IV (B.5) deals with Preservation of Confidentiality. Standard II (B) deals with Professional
Misconduct.

Question: 449

Carl Pagan lives in Meesopotamia, where insider trading is not considered a crime. His friend, Ann
Dreen, lives in Gondwana, where an investment professional must report any knowledge of criminal
activity to the appropriate authorities. The AIMR code of ethics bars insider trading but does not
require one to report criminal activity to authorities. With regard to insider trading, Carl must follow
________ and with respect to reporting of criminal activity, Ann must follow ________.

A. Meesapotamian law; AIMR code


B. AIMR code; Gondwana law
C. Meesapotamian law; Gondwana law
D. AIMR code; AIMR code

Answer: B

Explanation:
A member must always abide by the AIMR code, unless the local laws in any given case are stricter,
in which case, the stricter standard applies. Standard I.

Page | 173
Question: 450

Which of the following is/are true about insider trading laws?

I. Corporations cannot discriminate amongst recipients without risking insider trading liability.
II. Information provided to a group of analysts remains non-public till it is made available to investors
in general.
III. If a member receives inside information that he deems material, the member must disseminate
the information to the public as soon as possible and not trade on it to avoid insider trading charges.

A. I only.
B. I & II.
C. I, II & III.
D. II & III.

Answer: B

Explanation:
If a member receives inside information that he deems material, the member must: a. refrain from
making decisions based on the information. b. encourage the firm to make the information publicly
available. However, because the information is confidential, the member cannot unilaterally choose
to broadcast it to the public. Indeed, he has an obligation not to personally disclose the information
to an outsider.

Question: 451

Which of the following can be found in Standard V?

A. Members shall not participate in any professional conduct involving dishonesty, fraud, deceit, etc.
B. Members shall deliver a copy of the Code to their employer.
C. Members shall not misrepresent investment performance.
D. Members shall exercise diligence and thoroughness in making investment recommendations or in
taking investment actions.
E. Members shall not knowingly participate or assist in any violation of laws, rules, or regulations.

Answer: C

Explanation:
Standard V states that members shall not make any statements that misrepresent the investment
performance that they or their firms have accomplished or can reasonably be expected to achieve.

Question: 452

The AIMR Performance Presentation Standards are

A. voluntary standards for the industry.


B. mandatory for CFA Charterholders and Candidates.
C. voluntary standards for the industry, but mandatory standards for AIMR members.
D. mandatory standards for the industry.That answer is correct!

Answer: A

Explanation:

Page | 174
The AIMR-PPS are voluntary standards for the industry. Firms are not required to comply with the
Standards when presenting performance, but the Standards are widely recognized as the most
effective guidelines for fair and accurate reporting of investment performance.

Question: 453

The mosaic theory holds that an analyst:

A. Can use material public information or nonmaterial nonpublic information in the analyst's
analysis.
B. Should distinguish between facts and opinions in research reports.
C. Should use all available and relevant information in support of an investment recommendation.
D. Violates the Code and Standards if the analyst fails to notify her or his employer of the Code and
Standards fails to have knowledge of and comply with applicable laws.That answer is correct!

Answer: A

Explanation:
This question pertains to Standard V (A), Prohibition against Use of Material Nonpublic Information.
The mosaic theory states that an analyst may use material public information or nonmaterial
nonpublic information in creating a larger picture than shown by any individual piece of information
and the conclusions drawn become material only after the pieces are assembled. All the other
answers describe violations of the Code and Standards, but are not the mosaic theory.

Question: 454

Portfolios that include more than one asset class are called ________ portfolios.

A. quasi-asset
B. multivariate
C. multiple leverage
D. univariate-asset
E. multiple-asset

Answer: E

Explanation:
If the firm does not have discretion over the asset mix, the segments of the various asset classes,
with their respective cash positions, must be included in composites composed of like assets.

Question: 455

The Standards state that when presenting material to others, members shall not "copy or use, in
substantially the same form as the original, material prepared by another without acknowledging
andidentifying the name of the author, publisher, or source of such material." The analyst may use
information from other sources without acknowledgment, however, if the information:

A. is factual information published in recognized financial and statistical reporting services.


B. does not include a buy or sell recommendation.
C. was originally communicated verbally.
D. is being reported only to the member's employer or associates.That answer is correct!

Answer: A

Page | 175
Explanation:
This question relates to Standard II (C), Prohibition against Plagiarism, which states that members
may use factual information published in recognized financial and statistical reporting services
without attribution. Even if the information is communicated verbally, members must use proper
attribution of the material. Whether or not the plagiarized material is combined with a buy/sell
recommendation or reported only to the member's employer or associates does not change the fact
that the member is copying material without acknowledgment.

Question: 456

Standard I includes rules on which of the following?

A. Professional Misconduct
B. All of these answers
C. Use of Professional Designation
D. None of these answers
E. Prohibition against Plagiarism

Answer: D

Explanation:
Standard I refers to compliance with laws, rules and regulations and their violation. Standard II -
Relationships with and Responsibilities to the Profession - includes rules on the use of Professional
Designation, Professional Misconduct and Prohibition against Plagiarism.

Question: 457

Smithers is a financial analyst with XYZ Brokerage Company. She is preparing a purchase
recommendation on JNI Corporation. Which of the following situations would represent a conflict of
interest for Smithers and, therefore, would have to be disclosed?

I. Smithers is on a retainer as a consultant to JNI.


II. XYZ holds for its own account a substantial common stock position in JNI.
III. Smithers has material beneficial owners of JNI through a family trust.
IV. Smithers' brother-in-law is a supplier to JNI.

A. II and III only.


B. I, II and III only.
C. II only.
D. I and IV only.
E. III only.
F. IV only.
G. I only.
H. I, II and IV only.

Answer: B

Explanation:
This question pertains to Standard IV (B.7), Disclosure of Conflicts to Clients and Standard III (C),
Disclosure of Conflicts to Employer. Statements I, II and III describe conflicts of interest for Smithers
or for her firm that would have to be disclosed. Statement I describes an employment relationship
between the analyst and the company subject to recommendation. Statement II describes the
beneficial interest of the analyst's employer in the company's stock. Statement III describes the
analyst's own beneficial interest in the company stock. In Statement IV, the relationship between the

Page | 176
analyst and the company through a relative is so tangential that it does not create a conflict of
interest necessitating disclosure.

Question: 458

Fiduciary duty includes a duty of loyalty and a reasonable standard of ________.

A. care
B. none of these answers
C. prudence
D. custodyThat answer is correct!

Answer: A

Explanation:
Standard IV (B.1) - Fiduciary Duties - states: Because the fiduciary is in a position of trust, the duty
required in fiduciary relationships exceeds what may be acceptable in other industries. Fiduciaries
owe undivided loyalty to their clients and must place client interests before their own. The
investment manager's fiduciary responsibility to a client includes a duty of loyalty and a duty to
exercise a reasonable standard of care. Investment actions should be carried out for the sole benefit
of the client and in a manner the manager believes to be in the interest of the client, given the
known facts and circumstances.

Question: 459

When a trustee for private investment funds has responsibility toward current beneficiaries who are
income oriented and future beneficiaries ("remaindermen") who are capital growth oriented, she
has primary fiduciary duty toward:

A. both have to be treated on the same footing.


B. the income beneficiaries.
C. the remaindermen.
D. insufficient information.That answer is correct!

Answer: A

Explanation:
Personal trust account trustees are subject to the "Prudent Man Rule" in their investment behavior
and are required to follow the guidelines of the trust. In particular, when there are conflicting
interests involved, the trustee is required to maintain an equitable balance between the interests of
different groups. Standard IV (B.1) - Fiduciary Duties - and the Topical Study, "Fiduciary Duty."

Question: 460

Level II verifiers must obtain and review ________ to ensure proper recording of cash flows.

A. end of period returns


B. accounting methods
C. bank reconciliation
D. cash flow statements

Answer: C

Explanation:

Page | 177
This procedure must be carried out to determine whether capital contributions and withdrawals that
are used in the performance calculations are recorded in the proper accounts, at the right amounts
and on a timely basis.

Question: 461

Standard IV (B) is known as ________.

A. Preservation of Confidentiality
B. Duty to Employer
C. Professional Misconduct
D. Prohibition against Use of Material Nonpublic Information
E. Interactions with Clients and Prospects
F. Investment Process
G. Fair Dealing
H. None of these answers

Answer: E

Explanation:
Standard IV (A) deals with the Investment Process. Standard III (B) deals with Duty to Employer.
Standard IV (B.3) deals with Fair Dealing. Standard IV (B) deals with Interactions with Clients and
Prospects. Standard V (A) deals with Prohibition against Use of Material Nonpublic
Information.Standard IV (B.5) deals with Preservation of Confidentiality. Standard II (B) deals with
Professional Misconduct.

Question: 462

The ERISA fiduciary needs to be as prudent as

A. the average investment advisor.


B. the average person.
C. the average expert.
D. none of these answers.

Answer: C

Explanation:
Under ERISA, the fiduciary is held to a higher standard than the Prudent Man Rule. The ERISA
fiduciary needs to be as prudent as the average expert, not simply as prudent as the average person.
Like all fiduciaries, however, the ERISA fiduciary must act solely in the best interest of the plan
participants and beneficiaries and use all available care, skill, prudence and diligence.

Question: 463

Ward is scheduled to visit the corporate headquarters of Evans Industries. Ward expects to use the
information obtained to complete his research report on Evans stock. Ward learns that Evans plans to
pay all of Ward's expenses for the trip, including the cost of meals, hotel room and air transportation.
Which of the following actions would be the best course for Ward to take under the Code and
Standards?

A. Accept the expense-paid trip and write an objective report.


B. Write the report without taking the trip.
C. Pay for all travel expenses, including costs of meals and incidental items.

Page | 178
D. Accept the expense paid trip but disclose the value of the services accepted in the report.

Answer: C

Explanation:
The best course of action, under Standard IV (A.3), Independence and Objectivity, is to avoid a
conflict of interest whenever possible. Therefore, paying for all expenses is the correct answer.
Merely disclosing the conflict is not as appropriate as avoiding the conflict of interest altogether.
Writing an objective report would not remove the appearance of a conflict of interest. Not taking the
trip is incorrect, because Ward would not be taking advantage of close inspection of the company
and his report would not be using all the information available.

Question: 464

When creating composites, ________ returns must not be mixed with asset-plus-cash returns.

A. model
B. portfolio
C. asset-only
D. cash
E. performance

Answer: C

Explanation:
To be in compliance with the PPS, a firm creating a composite must meet this requirement.

Question: 465

Smirlee Montero is a fixed income analyst at a small investment firm. She has been studying the
subject of "Body Language" in order to gauge the effectiveness of reading into the dispositions of
people. Based on her own empirical tests carried out over the past two years involving 64 different
subjects, she has concluded that during the meetings with analysts, a senior management official is
usually more excited and waves hands quite often when the information like earnings report (which
is released a couple of days later) is very favorable. On the other hand, if the earnings are short of
expectations, the official tends to be sober and appears more introspective. There are other body
signs which she uses to evaluate possible signals about the quality of information that is pending. In
a recent meeting with Chronotron, Inc.'s quarterly analyst meeting, she concluded using her
technique that Chronotron's earnings are very likely much lower than projected earlier. This
information was not actually released in the meeting whose main focus was on the company's plans
for the next quarter. In her report, Smirlee presented a detailed analysis of the publicly available
information which supported her hunch and wrote, "Based on this and certain other observations, it
is my opinion that Chronotron's is not doing as well as we earlier estimated. Therefore, I would
recommend selling this stock before the firm releases its earnings numbers." Smirlee has:

A. violated Standard IV (A.1) - Reasonable Basis and Representations by using a non-conventional


and controversial technique.
B. violated Standard IV (A.2) - Research Reports by failing to distinguishing between fact and opinion.
C. violated Standard V (A) - Prohibition against use of Material, Non-Public Information.
D. has not violated any Standards.

Answer: D

Explanation:

Page | 179
There is no evidence that in her report, Smirlee has tried to pass off an opinion as fact. While her
new technique is unconventional, using it would not be a violation of Standard IV (A.1) only if used in
isolation. Since she has diligently analyzed public information on Chronotron, she cannot be accused
of not having a reasonable basis for recommendation. The AIMR Standard of Ethics do not prohibit a
use of new techniques but rather encourage tempering them with more orthodox analysis.

Topic 2, Quantitative Methods

Question: 466

What simple annual interest rate would cause a $200 deposit to earn $15 interest in one year?

A. 5.00%
B. 12.50%
C. 10.25%
D. 7.5%
E. 6.67%

Answer: D

Explanation:
To solve this question, rearrange the simple interest formula I=PRT to isolate R. On the BAII Plus,
press 15 divide 200 divide 1 = to see the answer. On the HP12C, press 15 ENTER 200 divide 1 divide
to see the answer. Note that the final division by 1 (which is the time T in this question) is not
necessary.

Question: 467

What percent of the observations will lie above the mean plus two standard deviations?

A. 5%
B. 68%
C. 2.5%
D. None of these answers
E. 95%

Answer: C

Explanation:
95% of the observations lie between plus and minus two standard deviations from the mean.
Therefore, 2.5% lie over and above two standard deviations on each side of the mean.

Question: 468

Given the following points: (-2, 0), (-1,0), (0,1), (1, 1) and (2, 3)

What is the critical value necessary to determine a confidence interval for a 90% level of confidence?

A. 1.638
B. 1.533
C. None of these answers
D. 2.132
E. 2.353

Page | 180
Answer: E

Explanation:
Look at the t tables for n-2 degrees of freedom at the 10% level. Here, we look for a two tailed test
with 5-2 = 3 degrees of freedom. This is 2.353.

Question: 469

A coefficient of skewness of +2.48 indicates:

A. all of these answers are correct


B. the mean is larger than the median
C. the tail of the distribution is to the right
D. none of these answers are correct
E. a large positive skewness

Answer: A

Explanation:
In a positively skewed distribution, the mean is greater than the median or mode. The coefficient of
skewness (Sk) generally lies between -3 and +3, therefore an Sk of +2.48 indicates significant positive
skewness.

Question: 470

If you deposit $250 a month, beginning next month, for 20 years into an account paying 7% per year,
compounded monthly, how much is in your account after that last deposit?

A. $58,205.58
B. $308,663.09
C. $148,833.09
D. $147,577.02
E. $130,231.66

Answer: E

Explanation:
On the BAII Plus, press 240 N, 7 divide 12 = I/Y, 0 PV, 250 PMT, CPT FV. On the HP12C, press 240 n, 7
ENTER 12 divide i, 0 PV, 250 PMT, FV. On the BAII Plus, make sure the value of P/Y is set to 1. Note
that the answer is displayed as a negative number.

Question: 471

If you deposit $100 a month, beginning next month, for 8 years into an account paying 6% per year,
compounded monthly, how much is in your account after that last deposit?

A. $13,678.56
B. $12,282.85
C. $9,600.00
D. $12,097.91
E. $23,585,957

Answer: B

Page | 181
Explanation:
On the BAII Plus, press 96 N, 6 divide 12 = I/Y, 0 PV, 100 PMT, CPT FV. On the HP12C, press 96 n, 6
ENTER 12 divide i, 0 PV, 100 PMT, FV. On the BAII Plus, make sure the value of P/Y is set to 1. Note
that the answer is displayed as a negative number.

Question: 472

If you need $25,000 in 10 years, how much must you deposit today, if your money will earn 6% per
year, compounded annually?

A. $25,000
B. $13,959.87
C. $2,320.01
D. $44,771.19
E. $23,200.08

Answer: B

Explanation:
On the BAII Plus, press 10 N, 6 I/Y, 0 PMT, 25000 FV, CPT PV. On the HP12C, press 10 n, 6 i, 0 PMT,
25000 FV, PV. Note that the answer will be shown as a negative number.

Question: 473

How much would you have in a savings account 12 months from now if you start with a balance of
$3,000 today, make a deposit of $1,000 in 6 months and make another deposit 6 months after that of
$500? Assume that interest accrues at 6% per year, compounded monthly.

A. $4,684.42
B. $4,777.55
C. $4,715.41
D. $4,500.00
E. $2,500.00

Answer: C

Explanation:
Find the answer to this question by solving a couple of compound interest problems. Move the
$3,000 to month 12, then move the $1,000 forward 6 months to month 12, then add $500. On the
BAII Plus, press 12 N, 6 divide 12 = I/Y, 3000 PV, 0 PMT, CPT PV. Then press STO 1. Then press 6 N,
1000 PV, CPT FV. Then press + RCL 1 =. Make this number positive by pressing +/- and then press +
500 = to see the answer. On the HP12C, press 12 N, 6 ENTER 12 divide i, 3000 PV, 0 PMT, FV. Then
press STO 1. Then press 6 N, 1000 PV, FV. Then press RCL 1 +. Make this number positive by pressing
CHS and then press 500 + to see the answer. Make sure that the BAII Plus has the value of P/Y set to
1.

Question: 474

Suppose you are modeling long-term interest rates, and you believe that supply of corporate debt is
a major contributing factor. Suppose you believe that the probability that rates will rise if supply of
corporate debt rises is 60%; if the supply of corporate debt stays constant, you believe that there is a
35% chance of increasing interest rates; if the supply of corporate debt falls, you believe that there is
a 5% chance of rates increasing. You think that the likelihood of corporate debt increasing is 50%; of
staying the same is 40%; of dropping is 10%. What is the unconditional probability of interest rates

Page | 182
rising?

A. 55.4%.
B. 44.4%.
C. 44.5%.
D. 55.5%.

Answer: C

Explanation:
We use the total probability rule: P(A), the unconditional probability, = P(A|S_1)*P(S_1) + P(A|S_2)
*P(S_3) + P(A|S_3) *P(S_3), where the S_i represent mutually exclusive and exhaustive events. So
the likelihood of interest rates increasing is 0.60 * 0.50 + 0.35 * 0.40 + 0.05 * 0.10 = 0.30 + 0.14 +
0.005 = 0.445.

Question: 475

If you deposit $1,111 a year, beginning next year, for 20 years into an account paying 7% per year,
compounded annually, how much is in your account after that last deposit?

A. $50,598.15
B. $47,003.98
C. $45,545.99
D. $30,118.37
E. $48,304.12

Answer: C

Explanation:
On the BAII Plus, press 20 N, 7 I/Y, 0 PV, 1111 PMT, CPT FV. On the HP12C, press 20 n, 7 i, 0 PV, 1111
PMT, FV. On the BAII Plus, make sure the value of P/Y is set to 1. Note that the answer is displayed as
a negative number.

Question: 476

The standard normal probability distribution is one which has:

A. a mean of 0 and any standard deviation


B. a mean of 0 and a standard deviation of 1
C. any mean and a standard deviation of 1
D. a mean of 1 and any standard deviation
E. none of these answers are correct

Answer: B

Explanation:
The standard normal probability distribution has a mean of 0 and std. deviation of 1.

Question: 477

A statistician has framed his hypothesis testing problem as:


Ho: mean = 100 H1: mean < 100
Specify the region for the p-statistic which will lead to the rejection of the null at the 80% significance
level.

Page | 183
A. z-statistic < -0.84
B. z-statistic > +1.19
C. z-statistic < -1.35 or z-statistic > +1.35
D. z-statistic < -1.645

Answer: A

Explanation:
Since the alternative is directional and to the left, we use a left-tailed test. For this, the critical value
at 80% level is -0.84. -0.84 is the value above which 80% of the probability mass of the standard
normal distribution lies). The rejection region is then given by z-statistic < -0.84.

Question: 478

If you deposit $400 today into an account paying 8% per year, compounded quarterly, how much will
be in the account after 42 months?

A. $10,135.79
B. $1,174.88
C. $918.90
D. $927.54
E. $527.79

Answer: E

Explanation:
There are 14 quarters in 42 months (42 divide 3 =). On the BAII Plus, press 14 N, 8 divide 4 = I/Y, 400
PV, 0 PMT, CPT FV. On the HP12C, press 14 n, 8 ENTER 4 divide i, 400 PV, 0 PMT, FV. Note that the
answer will be displayed as a negative number. Make sure the BAII Plus has the value of P/Y set to 1.

Question: 479

A market survey was conducted to estimate the proportion of homemakers who could recognize the
brand name of a cleanser based on the shape and color of the container. Of the 1,400 homemakers,
420 were able to identify the brand name. Using the 0.99 degree of confidence, the population
proportion lies within what interval?

A. 0.250 and 0.350


B. 0.268 and 0.332
C. 0.950 and 0.997
D. 0.100 and 0.400
E. None of these answers

Answer: B

Explanation:
Interval estimate can be found from p +/- z[p(1-p)/n]^0.5. Here we have n = 1400, p = 414/1400 = 0.3
and z = 2.58 (for 99%).

Therefore 0.3 +/- 2.58*0.01225 and we get 0.268 and 0.332.

Question: 480

Page | 184
What annual interest rate, compounded annually, would cause a series of 10 deposits of $500 to
accumulate to $9,000, if the first deposit is made one year from today?

A. 12.52%
B. 15.38%
C. 11.12%
D. 8.04%
E. 10.09%

Answer: A

Explanation:
On the BAII Plus, press 10 N, 0 PV, 500 PMT, 9000 +/- FV, CPT I/Y. On the HP12C, press 10 n, 0 PV, 500
PMT, 9000 CHS FV, i. Make sure the BAII Plus has the P/Y value set to 1.

Question: 481

How much would you need to deposit today in order to be able to withdraw $500 in 2 years and $700
in 4 years, if the account has nothing in it today and interest is 6% per year, compounded annually?

A. $999.46
B. $1,007.54
C. $445.00
D. $1,200.00
E. $950.51

Answer: A

Explanation:
Work this question as two compound interest problems. On the BAII Plus, press 2 N, 6 I/Y, 0 PMT, 500
FV, CPT PV, which yields $445.00. Then press STO 1. Then press 4 N, 700 FV, CPT PV, which yields
$554.46. Then press + RCL 1 = to see the answer. On the HP12C, press 2 n, 6 i, 0 PMT, 500 FV, PV.
Then press STO 1. Then press 4 n, 700 FV, PV. Then press RCL 1 + to see the answer. Note that the
answer will be displayed as a negative number.

Question: 482

You have invested in a long-term, fixed deposit account earning 4% per year for 20 years,
compounded annually. Your friend has invested in a similar account but one that earns 4.25% per
year, compounded semi-annually. If each of you deposited $5,000, by what amount is your friend
wealthier than you due to this deposit?

A. $538
B. $601
C. $639
D. $521

Answer: C

Explanation:
The friend's account has 5,000*(1+0425/2)^40 = 11,595 at the end of 20 years. Your account has
5,000*(1.04^20) = 10,956. The difference is $639.

Question: 483

Page | 185
You are examining a group of 6 companies. Their average profit margins have been 49%, 10%, 5%,
35%, 30%, and 30%. What is the range of profit margins?

A. 49.0%.
B. 30.0%.
C. 44.0%.
D. 5.0%.

Answer: C

Explanation:
The range = the maximum value - the minimum value. Here, we have 49% - 5% = 44%.

Question: 484

If the hypothesized value of a parameter under the null hypothesis lies outside the confidence
interval, the null hypothesis:

A. none of these answers.


B. cannot be rejected at the corresponding significance level.
C. should be rejected at the corresponding significance level.
D. is ill-specified.

Answer: C

Explanation:
The confidence interval specifies the range over which the true value of the estimated parameter can
lie without rejecting the null hypothesis at the given level of significance.

Question: 485

What sample statistic is used to estimate a population value?

A. None of these answers


B. Point estimate
C. Parameter
D. Interval estimate
E. Sampling error

Answer: B

Explanation:
The point estimate is the estimate of a particular value in the population.

Question: 486

What is the area under the normal curve for z > 1.79?

A. None of these answers


B. 0.4633
C. 0.0401
D. 0.9599
E. 0.0367

Page | 186
Answer: E

Explanation:
From the z-tables, z = 1.79 is 0.4633. So 1 - 0.4633*2 = 0.0734. Since it is on each side of the curve,
0.0734/2 = 0.0367.

Question: 487

What does it mean if r = -1.00?

A. High values of one variable are associated with low values of the other variable
B. Dependent variable can be perfectly predicted by the independent variable
C. All of the variation in the dependent variable can be accounted for by the independent variable
D. Coefficient of nondetermination equals zero
E. All of these answers are correct

Answer: E

Explanation:
All the above are properties or explanations of the coefficient of correlation being equal to -1.0.

Question: 488

Type I error refers to the event that we will:

A. Accept the alternative when it is true.


B. none of these answers.
C. Reject the null when it is true.
D. Fail to reject the null when it is false.
E. Reject the alternative when it is true.

Answer: C

Explanation:
Remember that the null hypothesis is the one that you maintain to be true unless there is sufficient
evidence to prove otherwise. Therefore, the first type of mistake that can happen is that you reject
the maintained hypothesis when in fact, it is true. This error is referred to as "Type I" error. On the
other hand, you may not have sufficient evidence to disprove the null when in fact, it is false. This
failure to appropriately reject the null is referred to as "Type II" error.

Question: 489

You are faced with a counting problem in which you must choose k objects from n total objects. The
order of choosing does not matter. The counting method you should use is:

A. None of these answers is correct.


B. The binomial formula.
C. The multinomial formula.
D. The multiplication rule.

Answer: B

Explanation:

Page | 187
The combination, or binomial formula, gives the number of ways that k objects can be chosen from n
items, without regard to the order of choosing. The formula is n_C_k = (n choose k) = n! / [k! *(n-k)!].

Question: 490

In a statistical regression estimation, the R-square is found to be 39% and the slope coefficient is -0.3.
The percentage of variance of the dependent variable not explained equals ________.

A. 0.30
B. 0.61
C. 0.09
D. 0.39

Answer: B

Explanation:
The R-square of the regression measures the amount of variance of the dependent variable
explained by the independent variable. This is given to be 39%. Hence, the amount not explained
equals 100% - 39% = 61%.

Question: 491

A bell-shaped, symmetrical frequency distribution has a mean of 10. If 16% of the observations in the
distribution are negative, what is the coefficient of variation of X?

A. 1.0
B. 0.32
C. 10.0
D. 0.1

Answer: A

Explanation:
The fraction of observations which are less than zero equals 16% i.e. the fraction of observations
which are less than (mean - 10) equals 16% (given). Since the distribution is symmetrical about the
mean, this implies that the fraction of observations which are more than (mean + 10) also equals
16%. Thus, the fraction of the observations lying between 0 and 20 equals 1-0.16-0.16 = 0.68. For a
bellshaped, symmetrical frequency distribution, 68% of the observations lie within one standard
deviation of the mean. Hence, the standard deviation of the distribution equals 10. The coefficient of
variation is then equal to standard deviation/mean = 10/10 = 1.

Question: 492

A population consists of all the weights of all defensive tackles on Sociable University's football team.
They are: Johnson, 204 pounds; Patrick, 215 pounds; Junior, 207 pounds; Kendron, 212 pounds;
Nicko, 214 pounds; and Cochran, 208 pounds. What is the population standard deviation (in
pounds)?

A. About 4
B. None of these answers
C. About 40
D. About 100
E. About 16

Page | 188
Answer: A

Explanation:
Population variance = (Sum of squared deviation from the mean)/N. The mean is 210. Population
variance = (36 + 25 + 9 + 4 + 16 + 4)/6 = 94/6 = 15.67. Population standard deviation is the square
root of the population variance = 3.958.
xx-mean(x-mean)^2
204-636
215525
207-39
21224
214416
208-24

Question: 493

Which measure of dispersion disregards the algebraic signs (plus and minus) of each difference
between X and the mean?

A. Standard deviation
B. Variance
C. Mean deviation
D. Mean
E. None of these answers

Answer: C

Explanation:
The mean deviation is the mean of the absolute values of the deviations from the mean.

Question: 494

Sweetwater & Associates write weekend trip insurance at a very nominal charge. Records show that
the probability that a motorist will have an accident during the weekend and file a claim is 0.0005.
Suppose they wrote 400 policies for the coming weekend, what is the probability that exactly two
claims will be filed?

A. 0.0164
B. 0.0001
C. 0.8187
D. 0.2500
E. None of these answers

Answer: A

Explanation:
This is a binomial probability. The probability of getting r successes out of n trials where the
probability of success each trial is p and probability of failure each trial is q (where q = 1-p) is given
by: n!(p^r)[q^(n-r)]/r!(n-r)!. Here n = 400, r = 2,p = 0.0005 and q = 0.9995. Therefore we have
400!(0.0005^2)(0.9995^398)/2!398! = 0.0164.

Question: 495

The semiannually compounded rate is 10% quoted on an annualized basis. The equivalent annually

Page | 189
compounded rate is:

A. 10.25%
B. 10.5%
C. 9.65%
D. 10.1%

Answer: A

Explanation:
To solve such problems, think about investing a dollar for a year. The final amount should be the
same under both the quotations. Under annually compounded rate, r, $1 grows to 1+r in 1 year.
Under semiannual compounding, it grows to (1+0.1/2)^2 = 1.1025. Since these two should be equal,
we get 1+r = 1.1025, giving r = 10.25%. Note that the annually compounded rate must be larger than
the semiannually rate, ruling out 9.65 automatically.

Question: 496

In hypotheses testing, what is the level of significance?

A. Symbolized by the Greek letter "alpha"


B. All of these answers are true
C. Selected before a decision rule can be formulated
D. Value between 0 and 1
E. Risk of rejecting the null hypothesis when it is true

Answer: B

Explanation:
All the above a correct descriptions of the level of significance.

Question: 497

A random variable can take on four values: 12, 36, 78, 111, each with probability 0.25. The standard
deviation of the random variable equals ________.

A. 152.4
B. 49.3
C. 1,451
D. 38.1

Answer: D

Explanation:
Since all the observations occur with equal probability, the mean of the observations is (12 + 36 + 78
+ 111)*0.25 = 59.25. The variance is then given by 0.25*( [12-59.25]^2 + [36-59.25]^2 + [78-59.25]^2
+ [111-59.25]^2) = 1451. The standard deviation is equal to sqrt(1451) = 38.1.

Question: 498

If you deposit $10,000 into an account paying 6% per year, compounded semiannually, how much do
you have in the account in 8 years?

A. $15,938.48

Page | 190
B. $16,047.06
C. $12,667.70
D. $14,800.00
E. $25,403.52

Answer: B

Explanation:
On the BAII Plus, press 16 N, 6 divide 2 = I/Y, 10000 PV, 0 PMT, CPT FV. On the HP12C, press 16 n, 6
ENTER 2 divide i, 10000 PV, 0 PMT, FV. Note that the answer will be displayed as a negative number.
Make sure the BAII Plus has the value of P/Y set to 1.

Question: 499

A cumulative frequency distribution on days absent during a calendar year by employees of a


manufacturing company is shown below.
Days AbsentCumulative Number of Employees
0 - 260
3 - 531
6 - 814
9 - 116
12 - 142
How many employees were absent more than five days?

A. 14
B. 8
C. None of these answers
D. 22
E. 31

Answer: A

Explanation:
The cumulative number for class 6-8 will give us the number of people absent more than five days;
14.

Question: 500

Tran Holten, a quantitative analyst with Smith, Kleen & Beetchnutty Brokerage, has just been
informed of an important error in one of his recent statistical endeavors. Specifically, in one
hypothesis test, Mr. Holten rejected a null hypothesis that later was proven to be true. Which of the
following best describes this type of error in hypothesis testing? Further, if the confidence level of
the test were increased, would the probability of this error increase, decrease, or remain
unchanged?

A. Type I error; decrease


B. Type I error; increase
C. Type II error; decrease
D. Type I error; remain unchanged
E. Type II error; remain unchanged
F. Type II error; increase

Answer: A

Page | 191
Explanation:
In this example, Tran Holten has incorrectly rejected a null hypothesis. This type of error in
hypothesis testing is called a Type I error. In hypothesis testing, the Type I error is given much more
attention than the Type II error. In most hypothesis tests, the probability of a null hypothesis is equal
to the significance level of the test. A significance level of 0.01, for example, indicates that a 1%
chance exists that the null hypothesis will be rejected when it is indeed true. Another way to think of
the probability of a Type I error is to observe the following relationship:
{Probability of a Type I error = (1 - confidence level)}.
For example, a confidence level of 95% leaves a 5% probability of a Type I error occurring. If this
confidence level were to increase to say, 98%, then the probability of a Type I error would reduce to
2%. As you can see, there is a relationship between the confidence level of a test and the probability
of a Type I error. If the confidence level of the test in this example were to increase, then the
probability of a Type I error would decrease.

Question: 501

You believe the stock of Microsquish reflects a 90% chance of a dismissal of its contractual liability
suit. If the suit is dismissed, you expect the stock of Microsquish and its partner, AWOL, to rise. If the
suit is not dismissed, both will fall in price. However, you think that AWOL's stock only reflects a 60%
chance of dismissal of this suit. What strategy would be optimal?

A. Sell both short


B. Buy Microsquish
C. Sell AWOL
D. Buy AWOL

Answer: D

Explanation:
This is an example of the investment consequences of inconsistent probabilities. If the true
probability of a lawsuit dismissal is only 60%, then AWOL's stock is fairly valued, and Microsquish is
overvalued. If the true probability is 90%, then Microsquish is fairly valued, and AWOL is
undervalued. Your can thus construct a profitable risk-free strategy. You should buy AWOL, because
at worst, the stock will be fairly valued. At best, it is undervalued. You could also simultaneously
short Microsquish, because at best, it is fairly valued; there is a chance that it is overvalued. Note
that this example ignores all other factors other than the lawsuit.

Question: 502

If the correlation coefficient between two variables equals zero, what can be said of the variables X
and Y?

A. Highly correlated
B. Not correlated
C. Dependent on each other
D. None of these answers are correct
E. All of these answers are correct

Answer: B

Explanation:
r ranges from -1 to +1. The closer it is to -1 or +1, the stronger the relationship. Closer it is to zero, the
weaker.

Page | 192
Question: 503

How much must you deposit today in order to withdraw $500 in 1 year, $700 in 3 years and $500 in 4
years, assuming your money earns 6.5% per year, compounded annually?

A. $1,200.00
B. $1,503.27
C. $1,240.80
D. $1,421.95
E. $1,437.64

Answer: E

Explanation:
Solve this question by working 3 compound interest problems. On the BAII Plus, press 1 N, 6.5 I/Y, 0
PMT, 500 FV, CPT PV, which yields $469.48. Then press STO 1. Then press 3 N, 700 FV, CPT PV, which
yields $579.49. Then press + RCL 1 = STO 1. Then press 4 N, 500 FV, CPT PV, which yields $388.66.
Then press + RCL 1 = to see the answer. On the HP12C, press 1 n, 6.5 i, 0 PMT, 500 FV, PV. Then press
STO 1. Then press 3 N, 700 FV, PV. Then press RCL 1 + STO 1. Then press 4 n, 500 FV, PV. Then press
RCL 1 + to see the answer. Note that the answer will be displayed as a negative number. Make sure
the BAII Plus has the P/Y value set to 1.

Question: 504

A study of a company's practice regarding the payment of invoices revealed that on the average an
invoice was paid 20 days after it was received. The standard deviation equaled five days. Assuming
that the distribution is normal, what percent of the invoices were paid within 15 days of receipt?

A. 37.91%
B. 86.74%
C. 15.87%
D. 34.13%
E. None of these answers

Answer: C

Explanation:
z = (x-u)/sigma = 15 - 20/5 = -1.0. z = 1 is 0.3413. 1.0 - 0.8413 = 0.1587.

Question: 505

Which of the following is/are discrete random variables? I. The height of a student selected at
random from 217 students. II. The slot number where the ball comes to rest in a roulette wheel. III.
The color of a playing card drawn at random from a pack. IV. The speed of a car tracked by the radar
gun of highway patrol.

A. IV only
B. II & III
C. I, II & IV.
D. II only
E. III only
F. I, II & III
G. I only
H. I & II

Page | 193
Answer: B

Explanation:
A student's height and the speed of a car can take values over a continuum and hence, are not
discrete variables.

Question: 506

At a local burger joint, 3 workers receive $5.3 per hour, 4 workers receive $4.9 per hour and 2
workers receive $8.1 per hour. The average pay of a worker at the joint is ________.

A. $5.18
B. none of these answers
C. $5.74
D. $6.10

Answer: C

Explanation:
There are 3 + 4 + 2 = 9 workers. The total wages paid equal 3*5.3 + 4*4.9 + 2*8.1 = $51.7. The
average pay equals 51.7/9 = $5.74

Question: 507

Which of the following measures of dispersion are based on deviations from the mean?

A. None of these answers


B. Standard deviation
C. Mean deviation
D. All of these answers
E. Variance

Answer: D

Explanation:
The formulas for all three measures are based on the deviation of the mean. Note: variance is the
standard deviation squared.

Question: 508

What semiannual deposit is needed to accumulate $8,000 in 5 years if the account pays 7% per year,
compounded semiannually, assuming that the first deposit is made in 6 months and also assuming
that the account already has $1,250 in it today?

A. $336.15
B. $677.44
C. $559.52
D. $278.01
E. $531.63

Answer: E

Explanation:

Page | 194
On the BAII Plus, press 10 N, 7 divide 2 = I/Y, 1250 PV, 8000 +/- FV, CPT PMT. On the HP12C, press 10
n, 7 ENTER 2 divide i, 1250 PV, 8000 CHS FV, PMT. Note that the answer this time is a positive
number. This means that the $531.63 is a deposit in addition to the $1,250 original deposit. The
$5,000 is entered as a negative number, because the $1,250 and the $531.63 are deposits and the
$8,000 is a withdrawal. Make sure the BAII Plus has the value of P/Y set to 1.

Question: 509

A variable, W, is normally distributed with a mean of 12. 68% of its observations lie between 8 and
16. The coefficient of variation of W is ________.

A. 0.66
B. 1.5
C. 3.0
D. 0.33

Answer: D

Explanation:
For a normal distribution, 68% of the observations lie within one standard deviation of the mean.
Hence, the standard deviation of X equals 12-8 = 4. The coefficient of variation is then equal to
standard deviation/mean = 4/12 = 0.33.

Question: 510

If you buy an item and pay for it with 48 monthly payments of $75, beginning next month, what was
the cash price today, if interest accrues at 13% per year, compounded monthly?

A. $575.29
B. $44.71
C. $2,795.64
D. $2,904.12
E. $2,598.42

Answer: C

Explanation:
The cash price would be the present value of the payments made using the specified interest rate.
On the BAII Plus, press 48 N, 13 divide 12 = I/Y, 75 PMT, 0 FV, CPT PV. On the HP12C, press 48 n, 13
ENTER 12 divide i, 75 PMT, 0 FV, PV. Note that the answer will be shown as a negative number. Make
sure the BAII Plus has the P/Y value set to 1.

Question: 511

A histogram is best described as:

A. a collection of frequency classes in which the raw data are arranged.


B. a bar chart of relative frequencies.
C. a scatter plot of the raw data on the X-Y plane.
D. a visual representation of a frequency distribution.

Answer: D

Explanation:

Page | 195
A histogram is best described as a visual representation of a frequency distribution.

Question: 512

Samples of the wires coming off the production line were tested for tensile strength. The statistical
results (in PSI) were:
Mean = 500
Median = 500
Mode = 500
Standard deviation = 40
Mean deviation = 32
Quartile deviation = 25
Range = 240
Number is sample = 100
The middle 95 percent of the wires tested between approximately what two values?

A. 450 and 550


B. 420 and 580
C. 380 and 620
D. 460 and 540
E. None of these answers

Answer: B

Explanation:
95% of the observations lie between plus and minus two standard deviations from the mean, so it is
500 +/- 2(40).

Question: 513

Theta Corp.'s stock is expected to appreciate 6% per year. If no dividends are paid out over the next 5
year and the current stock price is $28, what's the expected price in 5 years?

A. $39.6
B. $37.5
C. $41.2
D. $34.9

Answer: B

Explanation:
The expected price in 5 years is 28*(1.06^5) = 37.5

Question: 514

The lowest significance level at which the null hypothesis can be rejected is called the:

A. F-statistic of the test.


B. t-value of the test.
C. p-value of the test.
D. critical value of the test.
E. none of these answers.

Answer: C

Page | 196
Explanation:
The lowest significance level at which the null hypothesis can be rejected is called the p-value of the
test. Thus, if the p-value is less than the significance level, the null hypothesis can be rejected at that
significance level.

Question: 515

The sponsors of a well-known charity came up with a unique idea to attract wealthy patrons to the
$500 a plate dinner. After the dinner, it was announced that each patron attending could buy a set of
20 tickets for the gaming tables. The chance of winning a prize for each of the 20 plays is 50-50. If you
bought a set of 20 tickets, what is the chance that you will win 15 or more prizes?

A. 0.021
B. 0.250
C. 0.006
D. None of these answers
E. 0.750

Answer: A

Explanation:
This is a binomial probability. The probability of getting r successes out of n trials where the
probability of success each trial is p and probability of failure each trial is q (where q = 1-p) is given
by: n!(p^r)[q^(n-r)]/r!(n-r)!. Here n = 20, p = 0.5 and q = 0.5 and r = 15,16,17,18,19,20. Therefore we
have
P(15) = 20!(0.5^15)(0.5^5)/15!5! = 0.0148
P(16) = 20!(0.5^16)(0.5^4)/16!4! = 0.0046
P(17) = 20!(0.5^17)(0.5^3)/17!3! = 0.0011
P(18) = 20!(0.5^18)(0.5^2)/18!2! = 0.0002
P(19) = 20!(0.5^19)(0.5^1)/19!1! = 0.00002
P(20) = 20!(0.5^20)(0.5^0)/20!0! = 0.000001
The sum adds up to 0.207.

Question: 516

David's gasoline station offers 4 cents off per gallon if the customer pays in cash and does not use a
credit card. Past evidence indicates that 40% of all customers pay in cash. During a one hour period
twenty-five customers buy gasoline at this station.
What is the probability that more than ten and less than fifteen customers pay in cash?

A. .401
B. .562
C. .541
D. None of these answers
E. .380

Answer: E

Explanation:
Binomial distribution: n!(p^r)(q^(n-r))/r!(n-r)!. n = 25, r = 10, p = 0.4 q = 0.6
P(11) = 25!(0.4^11)(0.6^14)/11!14! = 0.1465
P(12) = 25!(0.4^12)(0.6^13)/12!13! = 0.1140
P(13) = 25!(0.4^13)(0.6^12)/13!12! = 0.0760

Page | 197
P(14) = 25!(0.4^14)(0.6^11)/14!11! = 0.0434
Summing up: 0.3799.

Question: 517

What monthly payment, beginning next month, would pay off a debt of $1,000 over 15 months, if
the interest rate on the debt is 18% per year, compounded monthly?

A. $5.12
B. $66.67
C. $74.94
D. $68.92
E. $76.51

Answer: C

Explanation:
On the BAII Plus, press 15 N, 18 divide 12 = I/Y, 1000 PV, 0 FV, CPT PMT. On the HP12C, press 15 N, 18
ENTER 12 divide i, 1000 PV, 0 FV, PMT. Note that the answer will be displayed as a negative number.
Make sure the BAII Plus has the value of P/Y set to 1.

Question: 518

Suppose 1,600 of 2,000 registered voters sampled said they planned to vote for the Republican
candidate for president. Using the 0.95 degree of confidence, what is the interval estimate for the
population proportion (to the nearest tenth of a percent)?

A. 77.7% to 82.3%
B. 78.2% to 81.8%
C. 69.2% to 86.4%
D. None of these answers
E. 76.5% to 83.5%

Answer: B

Explanation:
Interval estimate can be found from p +/- z[p(1-p)/n]^0.5. Here we have n = 2000, p = 1600/2000 =
0.8 and z = 1.96 (for 95%).
Therefore 0.8 +/- 1.96*0.008944 and we get 0.7825 and 0.8175.

Question: 519

What is the present value today of these annual cash flows: $5,000, $4,000, $3,000, $2,000, $1,000?
Assume the first cash flow occurs 1 year from today and an interest rate of 8% per year, compounded
annually.

A. $12,854.32
B. $15,000
C. $13,598.41
D. $12,591.14
E. $5,000

Answer: D

Page | 198
Explanation:
You could solve this question using 5 different compound interest problems, but it is easier to solve
them using the calculator's cash flow functions. On the BAII Plus, press CF 2nd CLRWork 0 ENTER
DownArrow 5000 ENTER DownArrow DownArrow 4000 ENTER DownArrow DownArrow 3000 ENTER
DownArrow DownArrow 2000 ENTER DownArrow DownArrow 1000 ENTER DownArrow DownArrow
2nd Quit. Then press NPV 8 ENTER DownArrow CPT. On the HP12C, press these keys: 0 BlueShift CFo
5000 BlueShift CFj 4000 BlueShift CFj 3000 BlueShift CFj 2000 BlueShift CFj 1000 BlueShift CFj. Then
press 8 i, YellowShift NPV. The "DownArrow" represents the downward-pointing arrow on the top
row of the BAII Plus keyboard.

Question: 520

Which of the following are true assumptions underlying linear regression?

(1) For each value of X, there is a group of Y values which are normally distributed
(2) The means of these normal distributions of Y values all lie on the straight line of regression
(3) The standard deviations of these normal distributions are equal

A. All of them
B. None of them
C. Only (2) and (3)
D. Only (1) and (3)
E. Only (1) and (2)

Answer: A

Explanation:
All are part of the assumptions of the linear regression. Make sure you know all the assumptions.

Question: 521

The average cost of tuition, room and board at a small private liberal arts college is reported to be
$8,500 per term, but a financial administrator at Moravian College believes that the average cost is
much higher. A study was conducted using 150 small liberal arts colleges and findings showed that
the average cost per term is $9,000 with a standard deviation of $1,200. Let alpha = 0.05. The Ho: Mu
less than or equal to $8,500. H1: Mu is bigger than $8,500.
What is the critical z-value for this test?

A. -1.645
B. None of these answers
C. +1.645
D. -1.96
E. +1.96

Answer: C

Explanation:
From the hypothesis, this is a one tailed test. Since alpha is 0.05. We have 0.05, which gives us a z
value of +/1.645. From the Ho we see that the rejection region lies to the right of the mean, so the
critical value is +1.645.

Question: 522

Which of the following is/are true?

Page | 199
I. There are as many values above the mode as below it.
II. The sum of the differences between the observations in a sample and the mode of the sample
equals zero.
III. The mode is not affected by "outliers."
IV. A sample has a unique mode.

A. I only
B. II, III & IV
C. II only
D. I, III & IV
E. III only
F. III & IV

Answer: E

Explanation:
A sample can have multiple modes, corresponding to all the numbers which appear with the
maximum frequency. Note that the mode is not affected by outliers unless an extreme value occurs
frequently. In such a case, it is likely that the observed data contain large errors and hence, it must be
reexamined. (I) holds for the median not the mode i.e. there are as many values above the median as
below it. (II) holds for the mean i.e. the sum of the differences between the observations in a sample
and the mean of the sample equals zero.

Question: 523

If the level of confidence is lowered from 95% to 90%, but the allowable error and the standard
deviation remain the same, what happens to the required sample size?

A. None of these answers


B. Decreases
C. Increases
D. Remain unchanged

Answer: B

Explanation:
This is related to the degree or confidence and the maximum error allowed. If you want to have a
lower degree of confidence, you can have a smaller sample size (given that the standard deviation is
constant). If you want 100% confidence, you want to use the whole population as your sample.

Question: 524

Affirmative action commitments by industrial organizations have led to an increase in the number of
women in executive positions. Satellite Office Systems has vacancies for two executives, which it will
fill from among four women and six men. What is the probability that no woman is selected?

A. None of these answers


B. 2/15
C. 1/3
D. 8/15
E. 1/5

Answer: C

Page | 200
Explanation:
Probability of both positions being filled by men = 6/10*5/9 = 1/3.

Question: 525

Which measure of central tendency is found by arranging the data from low to high and selecting the
middle value?

A. Geometric mean
B. Median
C. None of these answers
D. Mean
E. Mode

Answer: B

Explanation:
This is exactly how the median is found. Make sure you know how to find the median in a frequency
distribution also.

Question: 526

How much would an original deposit of $500 grow to be after 8 and a half years, if the deposit earns
interest at 6.5% per year, compounded quarterly?

A. $853.97
B. $573.42
C. $4,254.58
D. $837.51
E. $864.95

Answer: E

Explanation:
On the BAII Plus, press 500 PV, 0 PMT, 34 N, 6.5 divide 4 = I/Y, then press CPT FV. On the HP12C, press
500 PV, 0 PMT, 34 n, 6.5 ENTER 4 divide i, then press FV. Note that N = 34 quarters and the answer is
displayed as a negative number. Make sure that the BAII Plus has the P/Y value set to 1.

Question: 527

The National Center for Health Statistics reported that of every 883 deaths in recent years, 24
resulted from an automobile accident, 182 from cancer and 333 from heart disease. Using the
relative frequency approach, what is the probability that a particular death is due to an automobile
accident?

A. 182/883 or 0.206
B. None of these answers
C. 24/883 or 0.027
D. 24/333 or 0.072
E. 539/883 or 0.610

Answer: C

Page | 201
Explanation:
The probability is the number of automobile deaths divided by the total number of deaths.

Question: 528

You are examining a special group of 5 stock market indices. Of these 5, the returns were 4%, 8%,
12%, 16%, and 10%. What is the population standard deviation of this group of stock market indices?

A. 10%.
B. 4%.
C. 0%.
D. 16%.

Answer: B

Explanation:
The population standard deviation is the positive square root of the population variance. The
population variance is equal to the sum of the squared differences between each population
member and the population mean, divided by the number of items in the population. In this case,
we have a mean of 10%. The first squared difference will be (4% - 10%)^2 = 0.0036. The others will be
0.0004, 0.0004, 0.0036, and 0. The sum of these squared differences is 0.008, and divided by 5, we
get 0.0016 = 16%%. The square root of this gives us our population standard deviation of 4%.

Question: 529

Each salesperson in a large department store chain is rated either below average, average, or above
average with respect to sales ability. Each salesperson is also rated with respect to his or her
potential for advancement either fair, good, or excellent. These traits for the 500 salespeople were
cross classified into the following table.

Sales AbilityPotential for Advancement


FairGood Excellent
Below Average161222
Average456045
Above Average9372135

What is the probability that a salesperson selected at random will have above average sales ability
and excellent potential for advancement?

A. 0.20
B. 0.50
C. None of these answers
D. 0.75
E. 0.27

Answer: E

Explanation:
We need the prob(above average) = (93 + 72 + 135)/500 and prob(excellent given above average) =
135/300. Therefore 300/500*135/300 = 0.27

Question: 530

What is the Net Present Value of this series of annual cash flows at an interest rate of 12% per year:

Page | 202
Year 0: <$14,000>, Year 1: $5,000, Year 2: $4,000, Year 3: $5,000, Year 4 are used to indicate a
negative number).

A. <$379.31>
B. $114.37
C. <$77.49>
D. $51.21
E. <$245.97>

Answer: E

Explanation:
On the BAII Plus, press CF 2nd CLRWork 14000 +/- ENTER DownArrow 5000 ENTER DownArrow
DownArrow 4000 ENTER DownArrow DownArrow 5000 ENTER DownArrow DownArrow 4000 ENTER
DownArrow DownArrow 2nd Quit. Then press NPV 12 ENTER DownArrow CPT. On the HP12C, press
these keys: 14000 CHS BlueShift CFo 5000 BlueShift CFj 4000 BlueShift CFj 5000 BlueShift CFj 4000
BlueShift CFj. Then press 12 i, YellowShift NPV. The "DownArrow" represents the downward-pointing
arrow on the top row of the BAII Plus keyboard. Make sure the BAII Plus has the value of P/Y set to 1.

Question: 531

Your investment rate of return is 6% per year and you invest according to the following schedule:

Beginning of year 1: $100


Beginning of year 2: $200
Beginning of year 3: $300
Beginning of year 4: $400

The amount you will have at the end of year 4 is:

A. $1,125
B. $640
C. $808
D. $845

Answer: A

Explanation:
This is a future value question. The amount at the end of year 4 = 100*(1.06^4) + 200*(1.06^3) +
300*(1.06^2) + 400*1.06 = 1,125

Question: 532

What quarterly payment is required beginning one period from today to pay off a $3,000 debt, if
interest accrues at 6% per year, compounded quarterly, and the debt is to be retired in 10 years?

A. $107.41
B. $88.63
C. $114.29
D. $87.83
E. $100.28

Answer: E

Page | 203
Explanation:
On the BAII Plus, press 40 N, 6 divide 4 = I/Y, 3000 PV, 0 FV, CPT PMT. On the HP12C, press 40 n, 6
ENTER 4 divide i, 3000 PV, 0 FV, PMT. Note that the answer will be displayed as a negative number.
Make sure the BAII Plus has the P/Y value set to 1.

Question: 533

Which of the following statements is false regarding confidence levels and/or tests of significance?

A. In most hypothesis testing, the confidence level is equal to 0.10, 0.05, and 0.01.
B. All else equal, the confidence interval of a hypothesis test with a 5% level of significance is less
than the confidence interval for a hypothesis test with a 1% level of significance.
C. In most hypothesis tests, the power of the test is equal to (1 - alpha).
D. More than one of these answers is incorrect.
E. The significance level of a test is equal to (1 - confidence level).
F. The power of a test is equal to the confidence level.

Answer: D

Explanation:
More than one of these answers is incorrect. Specifically, in most hypothesis tests, the significance
level is equal to the probability of a Type I error (denoted by the Greek letter alpha) and the power of
the test can be found by (1 - beta), where beta represents the probability of a Type II error.
Remember that statisticians are much more concerned with the probability of a Type I error, which is
defined as the act of incorrectly rejecting the null hypothesis. A Type II error, which is defined as the
act of incorrectly failing to reject the null hypothesis, is much more difficult to predict. Second, power
of a test is not equal to the confidence level of the test.

Question: 534

Which of the following is not true regarding the normal distribution?

A. It is symmetrical
B. It has a single peak
C. None of these answers
D. Mean, median and mode are all equal
E. The points of the curve meet the X-axis at z = -3 and z = 3

Answer: E

Explanation:
Nothing is stated about the points where the curve meets the X axis (does it meet at all?)

Question: 535

You wish to count the number of ways in which n objects can be assigned to k different categories,
with n_i members in the ith category. The counting method you should use is:

A. The permutation rule.


B. The binomial formula.
C. None of these answers is correct.
D. The multinomial formula.

Answer: D

Page | 204
Explanation:
The number of ways that you can arrange n objects so that there are n_1 of one kind, n_2 of another
kind, and so on, up to n_k of a kth kind, is found by using the multinomial formula: n! / [(n_1)! *
(n_2)! * ... * (n_k)!].

Question: 536

If you owe 3 debts ($800 due 3 months from now, $900 due 7 months from now, and $1,200 due 11
months from now), what single payment can you make today to settle them, if interest is assessed at
10% per year, compounded monthly?

A. $2,387.29
B. $2,600.00
C. $3,552.39
D. $2,724.84
E. $2,504.88

Answer: D

Explanation:
Find the answer to this question by solving 3 compound interest problems. On the BAII Plus, press 3
N, 10 divide 12 = I/Y, 0 PMT, 800 FV, CPT PV which yields $849.21. Then press STO 1. Then press 7 N,
900 FV, CPT PV, which yields $780.33. Then press + RCL 1 = STO 1. Then press 11 N, 1200 FV, CPT PV,
which yields $1,095.31. Press + RCL 1 = to see the answer. On the HP12C, press 3 n, 10 ENTER 12
divide i, 0 PMT, 800 FV, PV. Then press STO 1. Then press 7 n, 900 FV, PV. Then press RCL 1 + STO 1.
Then press 11 n, 1200 FV, PV. Press RCL 1 + to see the answer. Make sure the BAII Plus has the P/Y
value set to 1.

Question: 537

If an investor who has a required rate of return of 7% per year pays $1,000 for a five-year ordinary
annuity, the annuity pays ________ per year.

A. $244
B. $256
C. $271
D. $263

Answer: A

Explanation:
If the annuity pays C per year, we have 1,000 = C/0.07*(1-1/(1.07^5)) => C = 1,000*0.07/0.287 = 244.

Question: 538

For a standard normal distribution what is the probability that z is greater than 1.75?

A. 0.4599
B. 0.0401
C. None of these answers
D. 0.9599
E. 0.0459

Page | 205
Answer: B

Explanation:
The area under the curve for z = 1.75 is 0.4599. Therefore, 0.4599*2 = 0.9198. We want z >1.75. So
we want (1 - 0.9198)/2 = 0.0401.

Question: 539

An automatic machine inserts mixed vegetables into a plastic bag. Past experience revealed that
some packages were underweight and some were overweight, but most of them had satisfactory
weight.

Weight% of Total
Underweight2.5
Satisfactory90.0
Overweight7.5

What is the probability of selecting and finding that all three of them are overweight?

A. 0.0004218
B. 0.0000156
C. 0.075
D. 0.0000001
E. None of these answers

Answer: A

Explanation:
P(all three overweight)=0.075*0.075*0.075 = 0.0004218.

Question: 540

Variables X and Y are perfectly negatively correlated. Given only this information, if you run a
regression of Y against X, which of the following is/are true?
I. The intercept term is zero.
II. The slope equals 1.
III. R-square equals 1.
IV. The percentage of unexplained variance equals 100%.

A. I only
B. III & IV
C. IV only
D. I & II
E. III only
F. I & III
G. II only

Answer: E

Explanation:
If Y = aX + b, then X and Y are perfectly negatively correlated for any value of b and any negative
value of a. Hence, given the information in the question, the slope must be negative but not
necessarily equal to 1 and the intercept term cannot be determined. Since the correlation coefficient
is 1, R-square is also 1, as expected, since all the variation in Y can be explained by variation in X.

Page | 206
Note that the percentage of unexplained variance equals zero.

Question: 541

If you make an initial deposit $500 now into an account, an additional deposit of $800 in 2 years, and
a final deposit of $300 in 4 years, how much is in your account in 5 years? Assume the account earns
interest at 8% per year, compounded annually.

A. $2,120.04
B. $2,408.29
C. $2,066.43
D. $1,905.51
E. $2,331.88

Answer: C

Explanation:
Solve this question by working several compound interest problems, moving each dollar amount
over to year 5. On the BAII Plus, press 5 N, 8 I/Y, 500 PV, 0 PMT, CPT FV, which yields $734.66. Then
press STO 1, 3 N, 800 PV, CPT FV, which yields $1,007.77. Then press + RCL 1 = STO 1. Then press 1 N,
300 PV, CPT FV, which yields $324.00. Then press + RCL 1 = to see the answer. On the HP12C, press 5
n, 8 i, 500 PV, 0 PMT, FV. Then press STO 1, 3 n, 800 PV, FV. Then press RCL 1 + STO 1. Then press 1 n,
300 PV, FV. Then press RCL 1 + to see the answer. Note that the answer will be displayed as a
negative number. Also note that the value of "N" for each dollar amount is based on the distance
from year 5. Make sure the BAII Plus has the P/Y value set to 1.

Question: 542

What simple annual interest rate would cause a $110 deposit to grow to $120 in 2 years?

A. 4.55%
B. 9.09%
C. 8.33%
D. 54.55%
E. 4.17%

Answer: A

Explanation:
To solve this question, we have to rearrange the simple interest formula, I=PRT so that the R is by
itself. Note that the I (interest) is equal to 10 (120 - 100). On the BAII Plus, press 10 divide 110 divide
2 = to see the answer. On the HP12C, press 10 ENTER 110 divide 2 divide to see the answer.

Question: 543

The probability that events A and B do not occur simultaneously equals 0.77. The probability of
neither A nor B occurring equals 0.38. If P(A) equals 0.26, the probability of B occurring equals
________.

A. 0.43
B. 0.59
C. 0.62
D. 0.21

Page | 207
Answer: B

Explanation:
The probability that events A and B do not occur simultaneously equals one minus the probability
that events A and B occur simultaneously. Thus,

P(A and B) = 1 - 0.77 = 0.23.

The probability of neither A nor B occurring equals one minus the probability of either A or B
occurring. Thus, P(A or B) = 1 - 0.38 = 0.62.

Now, P(A or B) = P(A) + P(B) - P(A and B). Therefore, P(B) = P(A or B) - P(A) + P(A and B) = 0.62 - 0.26 +
0.23 = 0.59.

Question: 544

Given the following points: (-2, 0), (-1,0), (0,1), (1, 1) and (2, 3) What is the slope of the regression
line?

A. None of these answers


B. 0.6
C. 0.0
D. 0.5
E. 0.7

Answer: E

Explanation:
For this, you need to create tables with columns for X,Y,XY,X^2,Y' and Y-Y'. This will be as follows.

XYXY X^2 Y'Y-Y'


-2004-0.40.4
-10010.3-0.3
010010
11111.7-0.7
23642.40.6
Sum057100

N=5.
The regression equation is Y' = a + bX.
Then to find the slope, which is b, use the formula
b = [n(SXY)-(SX)(SY)]/[n(SX^2)-(SX)^2] where here S stands for "Sum of." b = [5(0)-(0)(5)]/[5(10)-0] =
0.7

Question: 545

Which of the following statements regarding confidence levels and/or tests of significance is/are
false? Choose the best answer.

A. In most hypothesis tests, the significance level is set equal to 0.10, 0.05, or 0.01.
B. All else equal, the probability of a Type I error decreases as the level of confidence increases.
C. The probability of a Type I error is denoted by the Greek letter alpha.
D. The confidence level of a hypothesis test can be found by subtracting the level of significance from
the number one.

Page | 208
E. More than one of these answers is false.
F. The significance level of a test is equal to the power of a test.

Answer: F

Explanation:
Remember that the power of the test is equal to (1 - Type II error probability). This is because in most
hypothesis tests, the level of significance, denoted by alpha, is set equal to the probability of a Type I
error. Statisticians are primarily concerned with the probability of a Type I error, and rarely specify
the probability of a Type II error. Determining the probability of a Type II error, which is defined as
the act of incorrectly failing to reject the null hypothesis, is inherently difficult to determine. The
probability of a Type I error, however, can be determined with relative ease. A Type I error is defined
as the act of incorrectly rejecting the null hypothesis. The remaining answers are all correct.

Question: 546

In an investment environment, an initial outlay of $1 grows to $1.23 in 3 years. If you are expecting a
cash inflow of $500 in 3 years, what's the present value of the cash flow to you?

A. $431.2
B. $452.25
C. $406.5
D. $615

Answer: C

Explanation:
The discount factor for 3 years is 1/1.23. Hence, the PV of $500 in 3 years is 500/1.23 = 406.5.

Question: 547

The median and the mode of the sample: 3, 2, 4, 5, 6, 99, 99, 100 equal:

A. 99, 6
B. 6, 99
C. 5, 99
D. 5.5, 99

Answer: D

Explanation:
The mode equals the most common observation; in this case, it is 99.
If the number of observations is even, the median is the mean of the two middle observations. In
this dataset, arrange the data in ascending order and select the 4th and 5th observations. These
happen to be 5 and 6. The median is then 5.5.

Question: 548

What is the proportion of the total area under the normal curve within plus and minus two standard
deviation?

A. 68%
B. 95%
C. 99.7%

Page | 209
D. None of these answers are correct
E. 34%

Answer: B

Explanation:
95% of the area under the curve lies within plus and minus one standard deviation of the mean.

Question: 549

Suppose the probability that oil prices will rise any given quarter is 0.51, and the probability that oil
prices will stay level or decline is 0.49. If oil prices rise, GNP will contract by 1% with 80% probability,
and expand by 0.5% with 20% probability. If oil prices decline or stay level, GNP will expand 3% with
a 75% probability and contract 0.5% with a 25% probability. What is the expected change in GNP in
the next quarter?

A. +0.68%
B. -0.86%
C. +0.86%
D. -0.68%

Answer: A

Explanation:
We need the total probability rule for expected value, for which the formula is E(X) = E(X | S_1) *
P(S_1) + E(X |S_2) * P(S_2) + ... + E(X |S_n) * P(S_n). Here, E(X) is the expected change in GNP. S_1 is
the event that oil prices rise, and S_2 is the event that oil prices fall. Therefore, E(X) = 0.51 * (-
1%*80% + 0.5%*20%) + 0.49 * (3%*75% - 0.5%*25%) = 0.68%, an expansion.

Question: 550

If you already have $500 in a savings account, how much must you deposit 4 years from now in order
to have $5,000 in 8 years, assuming the account earns interest at 10% per year, compounded
annually?

A. $4,500.00
B. $732.05
C. $2,683.02
D. $3,415.07
E. $2,332.54

Answer: C

Explanation:
The way to approach this question is to find the PV today of the $5,000 amount, determine the
difference between this PV and the existing $500 amount in the account, and then move this
difference out to the 4 year from now point. On the BAII Plus, press 8 N, 10 I/Y, 0 PMT, 5000 FV, CPT
PV. Then press + 500 =. Then press PV, 4 N, CPT FV to see the answer. On the HP12C, press 8 n, 10 i, 0
PMT, 5000 FV, PV. Then press 500 +. Then press PV, 4 n, FV to see the answer. Make sure the BAII Plus
has the P/Y value set to 1.

Question: 551

The joint probability of events A and B occurring equals 0.11. The probability of neither A nor B

Page | 210
occurring equals 0.64. If P(A) equals 0.24, the probability of B occurring equals ________.

A. 0.46
B. 0.29
C. 0.51
D. 0.23

Answer: D

Explanation:
We are given that P(neither A nor B) = 0.64, P(A and B) = 0.11 and P(A) = 0.24. The probability of
neither A nor B occurring equals one minus the probability of either A or B occurring i.e. P(neither A
nor B) = 1 - P(A or B).

Thus, P(A or B) = 1 - P(neither A nor B) = 1 - 0.64 = 0.36.

Now, P(A or B) = P(A) + P(B) - P(A and B)

Therefore, P(B) = P(A or B) - P(A) + P(A and B) = 0.36 - 0.24 + 0.11 = 0.23.

Question: 552

Which of the following is/are true?


I. There are as many values above the median as below it.
II. The sum of the differences between the observations in a sample and the median of the sample
equals zero.
III. The median is greatly affected by "outliers."
IV. A sample has a unique median.

A. II & IV
B. I, II & IV
C. I only
D. II only
E. I & IV
F. III only
G. IV only
H. I, III & IV

Answer: E

Explanation:
By definition, there are as many values above the median as below it. Each sample has a unique
median. (II) and (III) hold for the mean, not for the median.

Question: 553

If you deposit $150 a month, beginning next month, for 20 years into an account paying 6% per year,
compounded monthly, how much is in your account after the last deposit?

A. $70,343.82
B. $48,833.09
C. $49,904.67
D. $143,582.01
E. $69,306.13

Page | 211
Answer: E

Explanation:
On the BAII Plus, press 240 N, 6 divide 12 = I/Y, 0 PV, 150 PMT, CPT FV. On the HP12C, press 240 n, 6
ENTER 12 divide i, 0 PV, 150 PMT, FV. On the BAII Plus, make sure the value of P/Y is set to 1. Note
that the answer is displayed as a negative number.

Question: 554

A cumulative frequency distribution on days absent during a calendar year by employees of a


manufacturing company is shown below.

Days AbsentCumulative Number of Employees


0 - 260
3 - 531
6 - 814
9 - 116
12 - 142

How many employees were absent fewer than six days?

A. 60
B. 91
C. 46
D. 31
E. None of these answers

Answer: C

Explanation:
This is the difference between the cumulative numbers for the 0-2 group and the 6-8 group, 60 - 14 =
46. There are 46 people between those two groups and they were absent fewer than six days.

Question: 555

A perpetuity of $5,000 a year is priced at $40,000. The annual discount rate is:

A. 12.5%
B. 13.1%
C. 11.8%
D. 12.75%

Answer: A

Explanation:
If r is the discount rate, then 40,000 = 5,000/r, giving r = 12.5%

Question: 556

The formula for conditional probability is given by:

A. P(A | B) = P(AB) / P(B)


B. P(A | B) = P(AB) / P(A)

Page | 212
C. P(A | B) = P(AB) * P(A)
D. P(A | B) = P(AB) * P(B)

Answer: A

Explanation:
A conditional probability takes the form of P(A | B) = P(AB) / P(B), where P(B) does not equal 0. Note
that this is just a rearranged form of the formula for joint probability.

Question: 557

The closing prices of a common stock have been 61 1/2, 62, 61 1/4, 60 7/8, and 61 1/2 for the past
week. What is the range?

A. None of these answers


B. $1.750
C. $1.875
D. $1.250
E. $1.125

Answer: E

Explanation:
62 - 60.875 = 1.125

Question: 558

You run a mutual fund that holds 24 stocks. Each week, you intend to comprehensively review 3 of
them. Assuming this is the first week you are doing this, how many ways can you choose 3 from the
24?

A. 2,024.
B. 1,492.
C. 1,024.
D. 2,048.

Answer: A

Explanation:
The combination, or binomial formula, gives the number of ways that k objects can be chosen from n
items, without regard to the order of choosing. The formula is n_C_k = (n choose k) = n! / [k! *(n-k)!].
In this case, we have (24 choose 3) = 24! / [3! * 21!] = (24 * 23 * 22 ) / (6!) = 2,024.

Question: 559

An empirical finance professor estimates the following regression between the return on a stock, R,
and the return on S&P 500 index, Rsp:

R = 5% + 1.1 Rsp + error term

If the regression R-square is 0.25, estimate the change in the return on the stock when the return on
the S&P 500 index changes from 12% to 15%.

A. 19.5%

Page | 213
B. 8.8%
C. 18.2%
D. 3.3%

Answer: D

Explanation:
With the given regression, the change in the return on the stock when the return on the S&P 500
index changes by one unit equals the slope coefficient, 1.1. Hence, when the return on the S&P 500
index changes by 3% from 12% to 15%, the return on the stock will change by 1.1*3% = 3.3%

Question: 560

The lengths of time (in minutes) several underwriters took to review applications for similar
insurance coverage are: 50, 230, 52 and 57. What is the median length of time required to review an
application?

A. 141.0
B. 54.5
C. None of these answers
D. 109.0
E. 97.25

Answer: B

Explanation:
Order the numbers: 50,52,57,230. The median is (52 + 57)/2 = 54.5

Question: 561

A statistician has framed his hypothesis testing problem as:


Ho: mean = 0H1: mean > 0
For the given sample, he calculates the z-statistic. Then, the region of rejection at the 99% level is
given by:

A. z-statistic > +2.32


B. z-statistic < 1.96
C. z-statistic < -2.32 or z-statistic > +2.32
D. z-statistic > +1.96

Answer: A

Explanation:
Since the alternative is directional and to the right, we use a right-tailed test. For this, the critical
value at 99% level is +2.32. +2.32 is the value above which only 1% of the probability mass of the
standard normal distribution lies). The rejection region is then given by z-statistic > +2.32.

Question: 562

A cumulative frequency distribution on days absent during a calendar year by employees of a


manufacturing company is shown below.

Days AbsentCumulative ## of Employees


0 - 260

Page | 214
3 - 531
6 - 814
9 - 116
12 - 142

How many employees were absent between 6 and 11 days?

A. 12
B. 17
C. 20
D. None of these answers
E. 8

Answer: A

Explanation:
This is found by the difference between the 6-8 class and the 12-14 class. In this case: 14 - 2 = 12.

Question: 563

Which of the following is the formula for the correlation between X and Y?

A. Cov(X,Y)/[(sigma_X)*(sigma_Y)]^2.
B. [Cov(X,Y)]^0.5
C. (sigma_X)*(sigma_Y)/Cov(X,Y).
D. Cov(X,Y)/[(sigma_X)*(sigma_Y)].

Answer: D

Explanation:
The correlation between two random variables X and Y is Cov(X,Y)/[(sigma_X)*(sigma_Y)].

Question: 564

The coefficient of variation of a distribution X is twice that of Y. If X and Y have the same means, the
variance of Y is:

A. half that of X.
B. twice that of X.
C. none of these answers.
D. same as that of X.

Answer: C

Explanation:
The coefficient of variation equals the standard deviation divided by mean. Since X and Y have the
same mean, X must have a standard deviation which is twice that of Y for its coefficient of variation
to be twice that of Y. Then, the variance of X is 2^2 = 4 times that of Y.

Question: 565

Milton Samuel, a quantitative analyst with Middle Road Brokerage, is examining a data sample and
has amassed the following information:

Page | 215
Standard deviation of the sample: 12.37
Number of observations: 100
Sample mean: 231

Assume that Mr. Samuel formulates a null hypothesis stating that the population mean is equal to
212. Additionally, assume that the population standard deviation is unknown. Given this information,
what is the standard error of the estimate? Further, what is the test statistic? Choose the best
answer.

A. 1.530; 10.19
B. 1.250; 15.20
C. 1.250; 16.91
D. 1.237; 5.40
E. None of these answers is correct.
F. 1.530; 12.42
G. 1.237; 15.36

Answer: G

Explanation:
If the population standard deviation is unknown, as in this example, the standard error of the
estimate is found by using the following equation:

{Standard error = s / square root of n} where s = the sample standard deviation and n = the number
of observations in the sample.

In this example, all of the necessary information has been provided, and the determination of the
standard error of the estimate is found as:

{Standard error = [12.37 / 10] = 1.237}

Now that the standard error of the estimate has been calculated, the test statistic can be found by
using the following equation:

{Test statistic = [sample statistic - value of the population parameter under the null hypothesis] /
standard error of the sample statistic].

Again, all of the necessary information has been provided, and the calculation of the test statistic is
found as follows:

{Test statistic = [231 - 212] / 1.237 = 15.360}

Question: 566

If you deposit $123 into an account paying 6% per year simple interest, what is the balance in your
account 8 months later?

A. $182.04
B. $246.00
C. $123.92
D. $127.92
E. $131.51

Answer: D

Page | 216
Explanation:
Calculate the simple interest earned and add it to the original deposit. On the BAII Plus, press 123 x
0.06 x 8 divide 12 = + 123 = to see the answer. On the HP12C, press 123 ENTER 0.06 x 8 x 12 divide
123 + to see the answer.

Question: 567

Suppose you were given $10,000 today and deposited it into an account paying 10% per year,
compounded monthly. If you know that you will need $5,000 in the account 5 years from now, what
monthly withdrawal can you make from the account, beginning one month from now, that will leave
the account with exactly $5,000 in it in 5 years?

A. $147.90
B. $83.33
C. $1,066.81
D. $140.04
E. $1,001.65

Answer: A

Explanation:
On the BAII Plus, press 60 N, 10 divide 12 = I/Y, 10000 PV, 5000 +/- FV, CPT PMT. On the HP12C, press
60 n, 10 ENTER 12 divide i, 10000 PV, 5000 CHS FV, PMT. Note that the answer is a negative number.
This is because it is a withdrawal from the account balance, just as the $5,000 is. Make sure the BAII
Plus has the value of P/Y set to 1.

Question: 568

How many deposits of $450 a month, beginning next month, are needed before an account has
$200,000 in it, assuming it had a balance of $0 at the beginning and that interest is 6% per year,
compounded monthly?

A. 166.79
B. 234.6
C. 119.48
D. No solution/Error
E. 444.44

Answer: B

Explanation:
One of the two dollar amounts must be entered as a negative number and the other as a positive
number or an error will result. On the BAII Plus, press 6 divide 12 = I/Y, 0 PV, 450 PMT, 200000 +/- FV,
CPT N. On the HP12C, press 6 ENTER 12 divide i, 0 PV, 450 PMT, 200000 CHS FV, n. Note that the
HP12C returns a result of 235. Make sure the BAII Plus has the P/Y value set to 1.

Question: 569

Suppose you need $1,500 in 15 months. How much must you deposit today, if the deposit will earn
interest at 8% per year, compounded monthly?

A. $472.86
B. $1,249.93

Page | 217
C. $1,357.71
D. $291.77
E. $341.55

Answer: C

Explanation:
On the BAII Plus, press 15 N, 8 divide 12 = I/Y, 0 PMT, 1500 FV, CPT PV. On the HP12C, press 15 n, 8
ENTER 12 divide i, 0 PMT, 1500 FV, PV. Make sure the BAII Plus has the P/Y value set to 1.

Question: 570

Where does the coefficient of variation (CV) generally lie between?

A. -1 and +1
B. -3 and +3
C. None of these answers
D. 0% and infinity
E. Unlimited values

Answer: D

Explanation:
CV always lies between 0% and infinity. The larger the CV, the larger the dispersion.

Question: 571

What deposit would you need to make today in order to withdraw $200 a month, beginning next
month, for the next 5 years, if the deposit will accrue interest at 8% per year, compounded monthly?

A. $7,237.66
B. $9,863.69
C. $11,492.59
D. $8,399.27
E. $9,141.23

Answer: B

Explanation:
On the BAII Plus, press 60 N, 8 divide 12 = I/Y, 200 PMT, 0 FV, CPT PV. On the HP12C, press 60 n, 8
ENTER 12 divide i, 200 PMT, 0 FV, PV. The answer will be displayed as a negative number. Make sure
the BAII Plus has the value of P/Y set to 1.

Question: 572

A survey of 144 retail stores revealed that a particular brand and model of a VCR retails for $375 with
a standard deviation of $20.

If 95% and 98% confidence intervals are developed to estimate the true cost of the VCR, what
difference would they have?

A. Interval widths
B. Z-variates
C. None of these answers

Page | 218
D. Both interval widths and z-variates
E. Standard errors

Answer: D

Explanation:
The interval widths and the z variates differ according to the rule chosen. The standard error is the
same regardless.

Question: 573

What is a Type II error?

A. Rejecting a false alternative hypothesis


B. Accepting a false null hypothesis
C. None of these answers
D. Accepting a false alternative hypothesis
E. Rejecting a false null hypothesis

Answer: B

Explanation:
The type II error is accepting the null hypothesis when it is actually false.

Question: 574

Which of the following statements regarding hypothesis testing is false?

A. If the population standard deviation is known, then the standard deviation of the sample is found
by dividing the population standard deviation by the square root of "n."
B. The test statistic is also known as the "critical value."
C. A Type II error is defined as the act of incorrectly failing to reject the null hypothesis.
D. The level of significance is denoted by the Greek letter alpha.
E. Despite the different ways to formulate a hypothesis, the null hypothesis is always tested at the
point of equality.
F. More than one of these answers is incorrect.

Answer: B

Explanation:
The test statistic is measured AGAINST the critical value. The test statistic is defined as "...a quantity,
calculated on the basis of sample, whose value is the basis for deciding whether to reject or not
reject the null hypothesis." The critical value is found by referencing the probability distribution for
the series (z-distribution, t-distribution, Chi-squared distribution, F-distribution, etc.).
The remaining answers are correct.

Question: 575

What is the Net Present Value of this series of annual cash flows using an interest rate of 20% per
year: Year 0: <$6,000>, Year 1: $4,000, Year 2: $3,000, Year 3: $2,000, Year 4: $1,000? (Note that the
<> are used to indicate a negative number).

A. $1,077.49
B. $1,056.33

Page | 219
C. $589.11
D. $1,134.39
E. $981.21

Answer: B

Explanation:
On the BAII Plus, press CF 2nd CLRWork 6000 +/- ENTER DownArrow 4000 ENTER DownArrow
DownArrow 3000 ENTER DownArrow DownArrow 2000 ENTER DownArrow DownArrow 1000 ENTER
DownArrow DownArrow 2nd Quit. Then press NPV 20 ENTER DownArrow CPT. On the HP12C, press
these keys: 6000 CHS BlueShift CFo 4000 BlueShift CFj 3000 BlueShift CFj 2000 BlueShift CFj 1000
BlueShift CFj. Then press 20 i, YellowShift NPV. The "DownArrow" represents the downward-pointing
arrow on the top row of the BAII Plus keyboard. Make sure that the BAII Plus has the P/Y value set to
1.

Question: 576

What is the variable used to predict another variable called?

A. Important
B. Causal
C. Independent
D. None of these answers
E. Dependent

Answer: C

Explanation:
The dependent variable is the variable Y which is being predicted by the X variable, the independent
variable. The regression is written as Y' = a + bX. The letter "a" is the Y intercept and b is the slope of
the line. Y' is the predicted value of Y given a specific value of X.

Question: 577

If you deposit $345 into an account paying 5.45% per year simple interest, how much interest will be
earned after 10 months?

A. $15.67
B. $360.67
C. $188.03
D. $$359.59
E. $14.59

Answer: A

Explanation:
The question only asks for the earned interest, not the total amount in the account. On the BAII Plus,
press 345 x 0.0545 x 10 divide 12 = to see the answer. On the HP12C, press 345 ENTER 0.0545 x 10 x
12 divide to see the answer.

Question: 578

If two events A and B are mutually exclusive, what does the special rule of addition state?

Page | 220
A. P(A and/or B) = P(A) +P(B)
B. P(A and B) = P(A) + P(B)
C. None of these answers
D. P(A or B) = P(A) - P(B)
E. P(A or B) = P(A) + P(B)

Answer: E

Explanation:
The key word is that these events are mutually exclusive. So P(A or B) = P(A) + P(B). The probabilities
stand alone.

Question: 579

Assume you deposit $300 now into an account that had nothing in it previously, make an additional
deposit of $400 in 1 year, and a final deposit of $200 in 4 years. How much is in your account in 6
years, if the account earns interest at 7% per year, compounded annually?

A. $418.29
B. $1,431.88
C. $1,120.04
D. $925.51
E. $1,240.22

Answer: E

Explanation:
Solve this question by working several compound interest problems, moving each dollar amount
over to year 5. On the BAII Plus, press 6 N, 7 I/Y, 300 PV, 0 PMT, CPT FV, which yields $450.22. Then
press STO 1, 5 N, 400 PV, CPT FV, which yields $561.02. Then press + RCL 1 = STO 1. Then press 2 N,
200 PV, CPT FV, which yields $228.98. Then press + RCL 1 = to see the answer. On the HP12C, press 6
n, 7 i, 300 PV, 0 PMT, FV. Then press STO 1, 5 n, 400 PV, FV. Then press RCL 1 + STO 1. Then press 2 n,
200 PV, FV. Then press RCL 1 + to see the answer. Note that the answer will be displayed as a
negative number. Also note that the value of "N" for each dollar amount is based on the distance
from year 6.

Question: 580

If you were to receive $100 per year for the next 10 years, starting today, and the discount rate
applicable is 9% per year, what's the value of this stream of cash flows to you?

A. $700
B. $600
C. $678
D. $642

Answer: A

Explanation:
Since you receive one payment today, the stream of cash flow equals $100 plus a 9-year ordinary
annuity. The value of the annuity today = (100/0.09)*(1 - 1/(1.09^9)) = 600. Thus, the total value to
you is 600+100 = 700.

Question: 581

Page | 221
You have set up a test of a suitably phrased null hypothesis. After analyzing the statistical
underpinnings of the test and the stringency requirements you have imposed, you have determined
that the probability of rejecting the null hypothesis equals 30%. The probability that you will reject
the null hypothesis when it is true is equal to 13% and the probability that you will reject the null
hypothesis when it is false is equal to 17%. The significance level of your test equals ________.

A. 17%
B. 6%
C. 30%
D. 13%

Answer: D

Explanation:
The significance level equals the probability of making a Type I error, which is the error of rejecting
the null when it is, in fact, true. In this case, this has been given to be equal to 13%.

Question: 582

A study of business faculty at state supported institutions in Ohio revealed that the mean salary for
nine months is $52,000 and the standard deviation of the sample is $3,000. The study also showed
that the faculty had been employed an average (mean) of 15 years with a standard deviation of 4
years. How does the relative dispersion in the distribution of salaries compare with that of the
lengths of service?

A. Salaries about 6%, service about 27%


B. Salaries about 100%, service about 50%
C. Salaries about 2%, service about 6%
D. None of these answers
E. Salaries about 42%, service about 81%

Answer: A

Explanation:
The coefficient of variation = (s*100)/mean. Salary: 3,000*100/52,000 = 5.7%. Service: 4*100/15 =
26.7%.

Question: 583

Johnny deposited a check he received from his Aunt when he was 10 years old into an account paying
10% per year, compounded monthly. If the original check was for $100, how much does Johnny have
in his account today, 50 years later?

A. $151.43
B. $5,905.33
C. $58,304.28
D. $100.00
E. $14,536.99

Answer: E

Explanation:
There are 600 months in 50 years (50 x 12). On the BAII Plus, press 600 N, 10 divide 12 = I/Y, 100 PV, 0

Page | 222
PMT, CPT FV. On the HP12C, press 600 n, 10 ENTER 12 divide i, 100 PV, 0 PMT, FV. Note that the
answer will be displayed as a negative number. Make sure the BAII Plus has the value of P/Y set to 1.

Question: 584

Past experience of a large manufacturing firm with administering a test to recent college graduates
who had applied for a job revealed that the mean test score was 500, and the standard deviation was
50. The distribution of the test scores was normal. Based on this experience, management is
considering placing a person whose scores is in the upper 6 percent of the distribution directly into a
responsible position. What is the lowest score a college graduate must earn to qualify for a
responsible position?

A. None of these answers


B. 460
C. 625
D. 578
E. 50

Answer: D

Explanation:
Find the z value representing 44% of the area under the curve. From the z tables, z = 1.55. Using z =
(x-u)/sigma. 1.55 = (x-500)/50. x = 577.5

Question: 585

You are examining a portfolio composed of 33% money-market investments, 9.5% bonds, and 57.5%
stocks. Last year, the return on the money-market investments was 4%; the return on bonds was 9%,
and the return on stocks was -11%. What is the contribution of stocks toward the portfolio weighted
average return?

A. -57.50%.
B. -11.00%.
C. -4.15%.
D. -6.325%.

Answer: D

Explanation:
The portfolio weighted-average mean return is equal to the sum (as i goes from 1 to n) of w_i * X_i,
where w_i is the percentage weight in the portfolio of the ith asset, and X_i is the investment return
of the ith asset. The contribution of any asset will equal its weight in the portfolio times its return.
Here, we get 0.575 * -0.11 = -6.325%.

Question: 586

When comparing the riskiness of investments with different expected returns, one must use
________.

A. none of these answers


B. skewness
C. the coefficient of variation
D. the standard deviation
E. kurtosis

Page | 223
Answer: C

Explanation:
The coefficient of variation equals the ratio of the standard deviation to the mean. Thus, it
standardizes the variation in the returns in terms of the expected values.

Question: 587

What annual interest rate, compounded annually, would cause a series of 20 deposits of $500 to
accumulate to $18,000, if the first deposit is made one year from today?

A. 6.15%
B. 4.94%
C. 7.25%
D. 2.78%
E. 5.80%

Answer: E

Explanation:
On the BAII Plus, press 20 N, 0 PV, 500 PMT, 18000 +/- FV, CPT I/Y. On the HP12C, press 20 n, 0 PV,
500 PMT, 18000 CHS FV, i.

Question: 588

The heights of the five members in a family have been measured and found to be (in centimeters):
146, 162, 165, 187, 152
The range and the mean deviation of the heights are:

A. 41, 46.9
B. 41, 54.4
C. 41, 10.9
D. 49, 61.8

Answer: C

Explanation:
range = maximum value - minimum value = 187 - 146 = 41 centimeters. To calculate the mean
deviation, you have to first calculate the mean. The mean equals (146 + 162 + 165 + 187 + 152)/5 =
162.4. The total deviation is abs(146-162.4) + abs(162-162.4) + abs(165-162.4) + abs(187-162.4) +
abs(152-162.4) = 16.4 + 0.4 + 2.6 + 24.6 + 10.4 = 54.4. Therefore, MD = 54.4/5 = 10.88.

Question: 589

How long will it take for an initial deposit of $2,500 to grow to be $4,000, if the interest rate is 5% per
year, compounded annually?

A. 32 years
B. 9.63 years
C. 7.72 years
D. 6.93 years
E. 14.48 years

Page | 224
Answer: B

Explanation:
Either the $2,500 or the $4,000 must be entered as a negative number - it won't matter which. On
the BAII Plus, press 5 I/Y, 2500 PV, 0 PMT, 4000 +/- FV, CPT N. On the HP12C, press 5 i, 2500 PV, 0
PMT, 4000 CHS FV, n. Note that the HP12C will indicate 10 years for the answer.

Question: 590

Which of the following is/are true?


I. Multiplying a variable by a positive constant will cause its standard deviation to be multiplied by
the square of that constant.
II. Multiplying a variable by a constant will cause its mean to be multiplied by that constant.
III. Adding a constant leaves the mean of a variable unchanged.
IV. Adding a positive constant to a variable causes the standard deviation of the variable to increase
by the same amount.

A. IV only
B. I, II and IV
C. II only
D. I only
E. I & II
F. III only
G. II only
H. II and IV

Answer: C

Explanation:
Adding a constant changes the mean by the same amount and leaves the variance and standard
deviation unchanged. Multiplying with a constant causes the mean and the standard deviation to be
multiplied by the same constant. The variance gets multiplied by the square of the constant.

Question: 591

If the alternate hypothesis states that u (Mu) does not equal 4,000, what is the rejection region for
the hypothesis test?

A. Center
B. None of these answers
C. Lower or left tail
D. Upper or right tail
E. Both tails

Answer: E

Explanation:
The alternate hypothesis says does not equal, which means it could be less or more than 4000. So we
must consider both tails.

Question: 592

An investor faces the following investment scenarios:

Page | 225
ScenarioProbabilityReturn
Bull market60%30%
Neutral market 30%7%
Market crash10%-25%

The variance of the investor's rate of return is ________.

A. 17.6%%
B. 4.17%%
C. 307.4%%
D. 6.17%%

Answer: C

Explanation:
The expected return equals 0.6 * 30% + 0.3 * 7% + 0.1*(-25%) = 17.6%. To calculate the variance, an
easy way is to first calculate the second moment, which is the expected value of the square of the
return. Thus, the second moment equals 0.6*[(30%)^2] + 0.3*[(7%)^2)] + 0.1*[(-25%)^2] = 617.2%%.
The variance of a random variable equals the second moment minus the square of the mean. In this
case, the variance equals 617.2%% - 17.6%^2 = 307.4%%

Question: 593

According to Chebyshev's Theorem, what percent of the observations lie within plus and minus 1.75
standard deviations of the mean?

A. Cannot compute because it depends on the shape of the distribution


B. 56%
C. 95%
D. 67%
E. None of these answers

Answer: D

Explanation:
Chebyshev's theorem applies regardless of the shape of the distribution. The minimum proportion
that lie within k standard deviations of the mean is at least 1-[1/(k^2)]. In this case k = 1.75. k^2 =
3.0625. So we get 67%.

Question: 594

Which of the following is/are true?


I. The mean of a sample represents its average value.
II. The median can only be calculated if the sample contains an odd number of values.
III. The mode is not affected by extreme outliers in the data.
IV. For samples with an even number of data points, the mode is preferable to the median as a
measure of central tendency.

A. III only
B. IV only
C. II only
D. I only
E. I & II
F. I, III & IV

Page | 226
G. II & III
H. I & III

Answer: H

Explanation:
Note that the mode is not affected by outliers unless an extreme value occurs frequently. In such a
case, it is likely that the observed data contain large errors and hence, it must be reexamined. The
median of a set with odd number of observations equals the "middle" observation. If the number of
observations is even, the median is the mean of the two middle observations. Thus, II is false.
Finally, there is no reason for a rule like (IV) to hold.

Question: 595

Consider the following three investments with annual compounding:

Present valueyearsinterest rate


1.$22,50056% per year
2.$10,00075% per year
3.$15,00038% per year

The future values of the 3 investments, at the ends of their investment periods, are:

A. $26,454, $12,067, $17,181


B. $23,850, $10,500, $16,200
C. $30,110, $14,071, $18,896
D. $16,813, $7,107, $11,907

Answer: C

Explanation:
Future value = Present value*(1+r)^N for annual compounding. Therefore,

Present valueyearsrateFuture value


1.22,50056%22,500*(1.06)^5 = 30,110
2.10,00075%10,000*(1.05)^7 = 14,071
3.15,00038%15,000*(1.08)^3 = 18,896

Note that the future value must always be greater than the present value.

Question: 596

________ states the range over which a population parameter likely lies.

A. A point estimate
B. A normal deviate
C. A confidence interval
D. A stratified range estimate

Answer: C

Explanation:
A confidence interval gives a range of values within which there is a high probability that the
population parameter occurs.

Page | 227
Question: 597

In graphs of return performance, which of the following scales is commonly used?

A. Exponential scale.
B. Semi-logarithmic scale.
C. Arithmetic scale.
D. Log-normal scale.

Answer: B

Explanation:
Historical returns are often graphed using a semi-logarithmic scale, where equally-large y-axis
movements correspond to equally-large percentage changes. An arithmetic scale, on the other hand,
has equally-large y-axis movements corresponding to equally-large arithmetic changes. So on an
arithmetic scale, the vertical movement for a price change from 100 to 110 is just as large as the
vertical movement from 10,000 to 10,010. This is widely considered a distortion (if you are
considering rates of return), so the semi-logarithmic scale is used instead.

Question: 598

A researcher has a sample of 900 observations from a population whose standard deviation is known
to be 3,381. The mean of the sample is calculated to be 465.2. The null hypothesis is stated as Ho:
mean < 200. The p-value in this case equals ________.

A. 1.54%
B. 2.26%
C. 1.13%
D. 0.94%

Answer: D

Explanation:
To test the hypothesis, you need to calculate the smallest z-statistic since the null hypothesis is
unidirectional and to the left. This makes it the hardest to reject the null and you should always use
the most stringent criterion for rejecting the null. After all, the null is the hypothesis maintained to
be true by default and only a sufficient weight of evidence should be used to reject that view. The
smallest z-statistic under the null is calculated to be (465.2 - 200)/(3381/(900^.5)) = 2.35. The right-
tailed probability of observing a z-statistic which is at least as big as 2.35 equals 1.0 - 0.9906 = 0.0094
= 0.94%. This is the p-value of the right-tailed test in this sample.

Question: 599

The weights (in kilograms) of a group of crates being shipped to Panama are 95, 103, 110, 104, 105,
112 and 92. What is the mean deviation?

A. 52.50 kg
B. 5.43 kg
C. 0.53 kg
D. None of these answers
E. 6.25 kg

Answer: B

Page | 228
Explanation:
The mean is 103. The mean deviation is the absolute values of the deviation from the mean: (8 + 0 +
7 + 1 + 2 + 9 + 11)/7 = 38/7 = 5.43

Question: 600

A distribution has a mean of 7 and a range of -100 to +50. The distribution is ________.

A. incomplete
B. right skewed
C. defective
D. left skewed

Answer: D

Explanation:
The range of values is larger to the left of the mean. Hence, the distribution is left-skewed.

Question: 601

The distribution of a sample of the outside diameters of PVC gas pipes approximates a symmetrical,
bell-shaped distribution. The mean outside diameter of the sample is 14.0 inches, and the standard
deviation is 0.1 inches. About 68 percent of the outside diameters lie between what two amounts?

A. 13.9 and 14.1 inches


B. 13.8 and 14.2 inches
C. 13.0 and 15.0 inches
D. None of these answers
E. 13.5 and 14.5 inches

Answer: A

Explanation:
Approximately 68% of observations will lie within plus and minus one standard deviation of the
mean.

Question: 602

You borrow $100,000 at a mortgage rate of 8% per year. What's the annual payment you must make
to repay the loan in 25 years?

A. $8,349
B. $5,982
C. $9,368
D. $7,493

Answer: C

Explanation:
If P is the annual payment, then using the annuity formula,
100,000 = P/0.08*[1 - 1/1.08^25] = 10.67P
This gives P = $9,368

Page | 229
Question: 603

What are half of the observations always greater than?

A. Median
B. Geometric mean
C. None of these answers
D. Mode
E. Mean

Answer: A

Explanation:
By definition, the median is the middle point. It divides the observations into two. The mean is NOT
the answer because it may be affected by extreme values.

Question: 604

A random sample of 85 group leaders, supervisors, and similar personnel revealed that on the
average a person spent 6.5 years on the job before being promoted. The standard deviation of the
sample was 1.7 years. Using the 0.95 degree of confidence, what is the confidence interval within
which the population mean lies?

A. 6.99 and 7.99


B. 6.49 and 7.49
C. None of these answers
D. 4.15 and 7.15
E. 6.14 and 6.86

Answer: E

Explanation:
Interval estimate can be found from x_bar +/-z*s/(n^0.5). Here we have n = 85, x_bar = 6.5 and z =
1.96 (for 95%) and s = 1.7. Therefore 6.5+/-1.96*1.7/9.22 and we get 6.14 and 6.86.

Question: 605

You are examining a group of 20 mutual funds. You find that 10 have a 5% cash position, 4 have a 10%
cash position, and 6 have a 2% cash position. What is the weighted average cash position of these 20
mutual funds?

A. 4.80%.
B. 5.10%.
C. 5.00%.
D. 4.90%.

Answer: B

Explanation:
A weighted-average is equal to the sum (as i goes from 1 to n) of w_i * X_i, where w_i is the
percentage weight of the ith item, and X_i is the value of the ith item. Here, we get a weighted mean
of 10/20 * 0.05 + 4/20 * 0.10 + 6/20 * 0.02 = 5.10%.

Question: 606

Page | 230
What is the critical z-value for a one tailed lower test at the 1% level of risk?

A. + 2.58
B. +2.33
C. -2.58
D. None of these answers
E. -2.33

Answer: E

Explanation:
This is found by subtracting the 0.01 from 0.5 and then finding the z-value for 0.49.

Question: 607

What is the annual Internal Rate of Return of this series of annual cash flows: Year 0: <$15,000>, Year
1: $2,000, Year 2: $0, Year 3: $15,000, Year 4 to indicate a negative number).

A. 25.29%
B. 18.91%
C. 30.04%
D. 22.49%
E. 25.42%

Answer: E

Explanation:
On the BAII Plus, press CF 2nd CLRWork 15000 +/- ENTER DownArrow 2000 ENTER DownArrow
DownArrow 0 ENTER DownArrow DownArrow 15000 ENTER DownArrow DownArrow 0 ENTER
DownArrow DownArrow 18000 ENTER DownArrow DownArrow 2nd Quit. Then press Irr CPT. On the
HP12C, press these keys: 15000 CHS BlueShift CFo 2000 BlueShift CFj 0 BlueShift CFj 15000
BlueShift CFj 0 BlueShift CFj 18000 BlueShift CFj. Then press YellowShift Irr. The "DownArrow"
represents the downward-pointing arrow on the top row of the BAII Plus keyboard. Make sure the
BAII Plus has the P/Y value set to 1.

Question: 608

Suppose you are modeling GNP, and you believe that the probability that GNP will expand if interest
rates fall is 70%; if interest rates stay constant, you believe that there is a 29% chance of expanding
GNP; if interest rates increase, you believe that there is a 1% chance of GNP expanding. You think
that the likelihood of interest rates falling is 60%; of staying the same is 30%; of increasing is 10%.
What is the unconditional probability of GNP expanding?

A. 49.8%.
B. 50.0%.
C. 50.8%.
D. 50.9%.

Answer: C

Explanation:
We use the total probability rule: P(A), the unconditional probability, = P(A|S_1)*P(S_1) + P(A|S_2)
*P(S_3) + P(A|S_3) *P(S_3), where the S_i represent mutually exclusive and exhaustive events. So

Page | 231
the likelihood of interest rates increasing is 0.70 * 0.60 + 0.29 * 0.30 + 0.01 * 0.10 = 0.42 + 0.087 +
0.001 = 0.508.

Question: 609

The marketing department of a nationally known cereal maker plans to conduct a national survey to
find out whether or not consumers of flake cereals can distinguish one of their favorite flake cereals.
To test the questionnaire and procedure to be used, eight persons were asked to cooperate in an
experiment. Five very small bowls of flake cereals were placed in front of a person. The bowls were
labeled A, B, C, D, and E. The person was informed that only one bowl contained his or her favorite
flake cereal. Suppose that the eight persons in the experiment were unable to identify their favorite
cereal and just guessed which bowl it was in. What is the probability that none of the eight guessed
correctly?

A. 0.168
B. 0.788
C. 0.125
D. None of these answers
E. 0.009

Answer: A

Explanation:
This is a binomial probability. The probability of getting r successes out of n trials where the
probability of success each trial is p and probability of failure each trial is q (where q = 1-p) is given
by: n!(p^r)[q^(n-r)]/r!(n-r)!. Here n = 8, r = 0,p = 0.2 and q = 0.80. Therefore we have
8!(0.2^0)(0.8^8)/0!8! = 0.168.

Question: 610

Which of the following is true?

A. If the geometric return is zero, the arithmetic return must be negative.


B. All of these answers are false.
C. If the annual returns have zero volatility, the geometric return is equal to the arithmetic return.
D. The geometric return is always less than the arithmetic return.

Answer: C

Explanation:
The mean and the geometric mean are equal when volatility in the rate of return is zero. For a non-
zero volatility, the mean exceeds the geometric mean and the difference is larger the higher the
volatility.

Question: 611

If you deposit $900 a year, beginning next year, for 20 years into an account paying 8% per year,
compounded annually, how much is in your account after that last deposit?

A. $40,598.15
B. $50,003.98
C. $530,118.37
D. $48,304.12
E. $41,185.77

Page | 232
Answer: E

Explanation:
On the BAII Plus, press 20 N, 8 I/Y, 0 PV, 900 PMT, CPT FV. On the HP12C, press 20 n, 8 i, 0 PV, 900
PMT, FV. On the BAII Plus, make sure the value of P/Y is set to 1. Note that the answer is displayed as
a negative number.

Question: 612

In computing skewness, which of the following is true?


I. It uses the cubed deviation from the mean.
II. It raises the deviation from the mean to the third power.
III. The formula preserves the direction of the deviation.

A. I and III
B. II and III
C. I, II, and III
D. None of these answers is correct.

Answer: C

Explanation:
I and II are true and equivalent. III is true because the cube of a negative number is still a negative
number: -2 * -2 * -2 = -8.

Question: 613

How much must you deposit today if you wish to have $30,000 in 20 years, assuming that interest
accumulates at 8% per year, compounded annually?

A. $6,436.45
B. $7,046.51
C. $3,000.00
D. $5,045.65
E. $6,604.73

Answer: A

Explanation:
On the BAII Plus, press 20 N, 8 I/Y, 0 PMT, 30000 FV, CPT PV. On the HP12C, press 20 n, 8 i, 0 PMT,
30000 FV, PV. Note that the answer is displayed as a negative number. Make sure that the BAII Plus
has the P/Y value set to 1.

Question: 614

An investment of $2,300 grows to $2,904 in 4 years. The annually compounded rate of return is:

A. 6.00%
B. 6.25%
C. 5.90%
D. 6.37%

Answer: A

Page | 233
Explanation:
Let r be the annually compounded rate. Then, 2904 = 2300*(1+r)^4. Hence, r = (2904/2300)^(1/4) - 1
= 6%

Question: 615

Assume the least squares equation is Y' = 10 + 20X. What does the value of 10 in the equation
indicate?

A. None of these answers


B. For each unit increased in X, Y increases by 10
C. Y intercept
D. For each unit increased in Y, X increases by 10

Answer: C

Explanation:
The regression is written as Y' = a + bX. The letter "a" is the Y intercept and b is the slope of the line.
Y' is the predicted value of Y given a specific value of X. Here a=10.

Question: 616

What quarterly payment would you have to make to pay off a $5,000 debt in 7 years, assuming the
first payment is made 3 months from today and interest accrues at 6% per year, compounded
quarterly?

A. $220.01
B. $72.96
C. $145.01
D. $372.96
E. $757.78

Answer: A

Explanation:
On the BAII Plus, press 28 N, 6 divide 4 = I/Y, 5000 PV, 0 FV, CPT PMT. On the HP12C, press 28 n, 6
ENTER 4 divide i, 5000 PV, 0 FV, PMT. Note that the answer will be displayed as a negative number.
Make sure the BAII Plus has the P/Y value set to 1. The value of "N" is set to 28 since there are 28
quarters in 7 years (7 x 4 = 28).

Question: 617

If you deposit $1,202.50 into an account paying 6% per year simple interest, how much interest will
you have earned in 2 years?

A. $124.00
B. $126.80
C. $120.50
D. $112.50
E. $144.30

Answer: E

Page | 234
Explanation:
Since this is a simple interest question, the formula is I=PRT, with T here being 2, since the timeframe
is 2 years. On the BAII Plus, press 1202.50 x 0.06 x 2 = to see the answer. On the HP12C, press
1202.50 ENTER 0.06 x 2 x to see the answer. Since the question asks for the amount of interest
earned, the original deposit should not be added to this value.

Question: 618

In a large metropolitan area, past records revealed that 30 percent of all the high school graduates go
to college. From 20 graduates selected at random, what is the probability that exactly 8 will go to
college?

A. 0.114
B. 0.400
C. 0.231
D. 0.887
E. None of these answers

Answer: A

Explanation:
This is a binomial probability. The probability of getting r successes out of n trials where the
probability of success each trial is p and probability of failure each trial is q (where q = 1-p) is given
by: n!(p^r)[q^(n-r)]/r!(n-r)!. Here n = 20, r = 8,p = 0.3 and q = 0.7. Therefore we have
20!(0.3^8)(0.7^12)/8!12! = 0.114.

Question: 619

The following is a distribution of monthly commissions:

Monthly CommissionsClass Frequencies


$600 - $7993
$800 - $9997
$1,000 - $1,19911
$1,200 - $1,39922
$1,400 - $1,59940
$1,600 - $1,79924
$1,800 - $1,9999
$2,000 - $2,1994

Referring to the table above, what is the relative frequency of those salespersons that earn more
than $1,599?

A. 25.5%
B. None of these answers
C. 29.5%
D. 27.5%
E. 30.8%

Answer: E

Explanation:
This is found by adding up all the frequencies of the classes above $1599. In this case 24 + 9 + 4 = 37.
Then we divide this by the total frequencies, which is 120. Therefore, 37/120 = 30.8%

Page | 235
Question: 620

The probability of the occurrence of an airplane crash and a successful resolution to a labor strike is
called:

A. a conditional probability.
B. an unrelated probability.
C. a joint probability.
D. an unconditional probability.

Answer: C

Explanation:
A joint probability takes the form of P(AB), the probability that an event A (an airplane crash) and an
event B (the resolution of a strike) both happen.

Question: 621

How much interest would an original $500 deposit earn at 5% per year simple interest after 14
months?

A. $29.17
B. $28.14
C. $583.33
D. $350.00
E. $30.04

Answer: A

Explanation:
The question only asks for the interest, not the ending balance. On the BAII Plus, press 500 x 0.05 x
14 divide 12 = to see the answer. On the HP12C, press 500 ENTER 0.05 x 14 x 12 divide to see the
answer.

Question: 622

In the regression equation, what does the letter "a" represent?

A. Slope of the line


B. Y intercept
C. Any value of the independent variable that is selected
D. None of these answers

Answer: B

Explanation:
The regression is written as Y' = a + bX. The letter "a" is the Y intercept and b is the slope of the line.

Question: 623

Suppose you have two assets, A and B. Over the past 3 periods, A has returned 8%, 2%, and 6%,
while B has returned 11%, -5%, and 20%. What is the return covariance between assets A and B?

Page | 236
A. 0.0%%
B. 19.79%%
C. 10.21%%
D. 31.38%%

Answer: B

Explanation:
First we must find the expected returns for A and B. These are 5.33% and 8.67%. Second, we find the
difference between each observation and the average: (8% - 5.33%), (2% - 5.33%), and (6% - 5.33%)
for A, and (11% - 8.67%), (-5% - 8.67%), and (20% - 8.67%) for B. Next, we multiply these together
and sum them: (8% - 5.33%)*(11% - 8.67%) + (2% - 5.33%)*(-5% - 8.67%) + (6% - 5.33%)*(20% -
8.67%). The sum of these is 59.33%. The covariance is the probability weighted average of these
cross products, so we divide by 3 to get 19.78%%. Note, we could have divided each cross product by
3 rather than the sum of the cross products. If the observations did not have the same probability or
frequency, we would need to treat each cross product separately rather than divide at the end.

Question: 624

The mean amount of gasoline and services charged by Key Refining Company credit customers is $70
per month. The distribution of amounts spent is approximately normal with a standard deviation of
$10. What is the probability of selecting a credit card customer at random and finding the customer
charged between $70 and $83?

A. 0.4032
B. None of these answers
C. 0.1962
D. 0.3413
E. 0.4750

Answer: A

Explanation:
z = (x-u)/sigma. z1 = 70 - 70/10 = 0 and z2 = 83 - 70/10 = 1.3. For z = 1.3, the area under the curve is
0.4032.

Question: 625

Two events, A and B, are independent if:

A. P(A and B) = 1
B. P(A and B) = P(A) + P(B)
C. P(A and B) = 0
D. P(A and B) = P(A) P(B)

Answer: D

Explanation:
This is the definition of independence.

Question: 626

If you deposit $4,250 today into a savings account paying 6% per year, compounded semiannually,
how much is in your account in 4 years?

Page | 237
A. $5,365.53
B. $6,773.85
C. $6,000.00
D. $4,783.41
E. $5,383.77

Answer: E

Explanation:
On the BAII Plus, press 8 N, 6 divide 2 = I/Y, 4250 PV, 0 PMT, CPT FV. On the HP12C, press 8 n, 6 ENTER
2 divide i, 4250 PV, 0 PMT, FV. Note that the answer will be displayed as a negative number. Make
sure the BAII Plus has the P/Y value set to 1.

Question: 627

A stock has the following returns over 4 years: +5%, -6%, +8%, +3%. The annual arithmetic and
geometric rates of return over the 4 years are:

A. 5%; 5.57%
B. 2.5%; 9.79%
C. 10%; 9.79%
D. 2.5%; 2.36%

Answer: D

Explanation:
The mean equals (5% + (-6%) + 8% + 3%)/4 = 2.5%
The geometric mean equals (1.05 * 0.94 * 1.08 * 1.03)^(1/4) = 2.36%
Note that the geometric mean is less than the mean.

Question: 628

What is the annual Internal Rate of Return of this series of annual cash flows: Year 0: <$15,000>, Year
1: $5,000, Year 2: $8,000, Year 3: $11,000? (Note that the <> are used to indicate a negative number).

A. 27.12%
B. 23.74%
C. 25.55%
D. 26.61%
E. 24.02%

Answer: E

Explanation:
On the BAII Plus, press CF 2nd CLRWork 15000 +/- ENTER DownArrow 5000 ENTER DownArrow
DownArrow 8000 ENTER DownArrow DownArrow 11000 ENTER DownArrow DownArrow 2nd Quit.
Then press Irr CPT. On the HP12C, press these keys: 15000 CHS BlueShift CFo 5000 BlueShift CFj 8000
BlueShift CFj 11000 BlueShift CFj Then press YellowShift Irr. The "DownArrow" represents the
downward-pointing arrow on the top row of the BAII Plus keyboard. Make sure the BAII Plus has the
P/Y value set to 1.

Question: 629

Page | 238
You are examining a group of 4 stock analysts within an industry. The average stock analyst correctly
predicts the direction of a given stock 51% of the time. For these analysts, their accuracy has been
40%, 50%, 60%, and 70%. What is the mean absolute deviation of their accuracy from the average?

A. 12%.
B. 10%.
C. 15%.
D. 9%.

Answer: B

Explanation:
The mean absolute deviation = the sum of (as i goes from 1 to n) of the absolute value of (X_i - Xbar),
divided by n. Here, we have [|40% - 51%| + |50% - 51%| + |60% - 51%| + |70% - 51%|]/4 = [11% +
1% + 9% + 19%] / 4 = 40%/4 = 10%.

Question: 630

Assume the least squares equation is Y' = 10 + 20X. What does the value of 20 indicate?

A. For each unit increased in Y, X increases by 20


B. Y intercept
C. None of these answers
D. For each unit increased in X, Y increases by 20

Answer: D

Explanation:
The regression is written as Y' = a + bX. The letter "a" is the Y intercept and b is the slope of the line.
Y' is the predicted value of Y given a specific value of X. Here b = 20.

Question: 631

How much would you need to deposit today in order to be able to withdraw $2,500 in 2 years and
$1,500 in 5 years, if the account has nothing in it today and interest is 8% per year, compounded
annually?

A. $2,722.33
B. $3,429.36
C. $3,164.22
D. $4,000.00
E. $3,378.18

Answer: C

Explanation:
Work this question as two compound interest problems. On the BAII Plus, press 2 N, 8 I/Y, 0 PMT,
2500 FV, CPT PV, which yields $2,143.35. Then press STO 1. Then press 5 N, 1500 FV, CPT PV, which
yields $1,020.87. Then press + RCL 1 = to see the answer. On the HP12C, press 2 n, 8 i, 0 PMT, 2500
FV, PV. Then press STO 1. Then press 5 n, 1500 FV, PV. Then press RCL 1 + to see the answer. Note that
the answer will be displayed as a negative number.

Question: 632

Page | 239
How many monthly deposits of $50, beginning next month, will you need to make into an account
that pays 6% per year, compounded monthly, before you will have $20,000?

A. 110.59
B. 201.49
C. 400.00
D. 220.27
E. 287.71

Answer: D

Explanation:
On the BAII Plus, press 6 divide 12 = I/Y, 0 PV, 50 PMT, 20000 +/- FV, CPT N. On the HP12C, press 6
ENTER 12 divide i, 0 PV, 50 PMT, 20000 CHS FV, n. Note that the HP12C will display 221 as the answer.
Make sure the BAII Plus has the value of P/Y set to 1.

Question: 633

What annual interest rate, compounded annually, would cause a series of 10 deposits of $1,000 to
accumulate to $18,000, if the first deposit is made one year from today?

A. 14.49%
B. 12.52%
C. 12.69%
D. 11.84%
E. 35.82%

Answer: B

Explanation:
On the BAII Plus, press 10 N, 0 PV, 1000 PMT, 18000 +/- FV, CPT I/Y. On the HP12C, press 10 n, 0 PV,
1000 PMT, 18000 CHS FV, i. Make sure that the BAII Plus has the P/Y value set to 1.

Question: 634

What is the remaining balance on a $90,000 loan after making monthly payments of $660.39 for 10
years, if the interest rate is 8% per year, compounded monthly?

A. $51,948.80
B. $81,343.92
C. $0
D. $76,204.11
E. $78,951.89

Answer: E

Explanation:
On the BAII Plus, press 120 N, 8 divide 12 = I/Y, 90000 PV, 660.39 +/- PMT, CPT FV. On the HP12C,
press 120 n, 8 ENTER 12 divide i, 90000 PV, 660.39 CHS PMT, FV. The remaining balance will be
displayed as a negative number. Make sure the BAII Plus has the P/Y value set to 1.

Question: 635

What is the Net Present Value of this series of annual cash flows at an interest rate of 14% per year:

Page | 240
Year 0: <$4,000>, Year 1: $2,000, Year 2: $0, Year 3: $0, Year 4 to indicate a negative number).

A. $214.37
B. <$67.49>
C. $122.71
D. $1.21
E. <$179.41>

Answer: C

Explanation:
On the BAII Plus, press CF 2nd CLRWork 4000 +/- ENTER DownArrow 2000 ENTER DownArrow
DownArrow 0 ENTER DownArrow DownArrow 0 ENTER DownArrow DownArrow 4000 ENTER
DownArrow DownArrow 2nd Quit. Then press NPV 14 ENTER DownArrow CPT. On the HP12C, press
these keys: 4000 CHS BlueShift CFo 2000 BlueShift CFj 0 BlueShift CFj 0 BlueShift CFj 4000 BlueShift
CFj. Then press 14 i, YellowShift NPV. The "DownArrow" represents the downward-pointing arrow on
the top row of the BAII Plus keyboard. Make sure the BAII Plus has the P/Y value set to 1.

Question: 636

You are examining a group of 20 mutual funds. You find that 5 have an 8% position in foreign stocks, 7
have a 9% position in foreign stocks, and 8 have no position in foreign stocks. What is the weighted
average position in foreign stocks of these 20 mutual funds?

A. 5.10%.
B. 5.20%.
C. 5.25%.
D. 5.15%.

Answer: D

Explanation:
A weighted-average is equal to the sum (as i goes from 1 to n) of w_i * X_i, where w_i is the
percentage weight of the ith item, and X_i is the value of the ith item. Here, we get a weighted mean
of 5/20 * 0.08 + 7/20 * 0.09 + 8/20 * 0 = 5.15%.

Question: 637

If you deposit $300 a month, beginning next month, for 10 years into an account paying 8% per year,
compounded monthly, how much is in your account after that last deposit?

A. $38,444,973.53
B. $30,000.00
C. $54,883.81
D. $3,091.62
E. $58,402.98

Answer: C

Explanation:
On the BAII Plus, press 120 N, 8 divide 12 = I/Y, 0 PV, 300 PMT, CPT FV. On the HP12C, press 120 n, 8
ENTER 12 divide i, 0 PV, 300 PMT, FV. On the BAII Plus, make sure the value of P/Y is set to 1. Note
that the answer is displayed as a negative number.

Page | 241
Question: 638

The sum of the squares of 1,200 observations equals 9,830. The sum of the observations equals
1,510. The population standard deviation of the observations equals ________.

A. 4.22
B. 2.57
C. 5.31
D. 6.61

Answer: B

Explanation:
For N observations, it is easy to show that

population variance*N = (sum of squares) - N*(mean^2)

The mean equals 1,510/1,200 = 1.258. Hence, population variance = (9,830 - 1,200*1.258^2 )/1,200=
6.608. The standard deviation then equals sqrt(6.608) = 2.57

Note: You should be careful about the difference between population variance and sample variance.
The formula for sample variance is:

sample variance*(N-1) = (sum of squares) - N*(mean^2)

You can expect an exam question which asks for population variance, with the choices given
containing both the population and the sample variances or vice versa.

Question: 639

If you deposit $10,000 into an account paying 6% per year, compounded semiannually, how much do
you have in the account in 10 years?

A. $15,403.52
B. $18,061.11
C. $18,938.48
D. $21,667.70
E. $17,800.00

Answer: B

Explanation:
On the BAII Plus, press 20 N, 6 divide 2 = I/Y, 10000 PV, 0 PMT, CPT FV. On the HP12C, press 20 n, 6
ENTER 2 divide i, 10000 PV, 0 PMT, FV. Note that the answer will be displayed as a negative number.
Make sure the BAII Plus has the value of P/Y set to 1.

Question: 640

An automatic machine inserts mixed vegetables into a plastic bag. Past experience revealed that
some packages were underweight and some were overweight, but most of them had satisfactory
weight.

Weight% of Total
Underweight2.5

Page | 242
Satisfactory90.0
Overweight7.5

Three packages are selected from the food processing line. What is the probability of selecting and
finding that all three of them are satisfactory?

A. 0.900
B. 0.075
C. 0.729
D. 0.810
E. None of these answers

Answer: C

Explanation:
P(all three satisfactory) = 0.9*0.9*0.9 = 0.729.

Question: 641

The p-value in hypothesis testing:

A. equals the significance level.


B. is less than the probability of type I error.
C. equals the probability of type II error.
D. none of these answers.

Answer: D

Explanation:
The lowest significance level at which the null hypothesis can be rejected is called the p-value of the
test. Thus, if the p-value is less than the significance level, the null hypothesis can be rejected at that
significance level. None of the choices fit this definition of the p-value.

Question: 642

You are running a year-end bonus pool for your top credit analysts. You have 10 analysts, and you
wish to recognize the top 3. How many different possible outcomes are there, if all three get the
same bonus? How many different outcomes are there if the top analyst gets 50% of the bonus pool,
the second best analyst gets 30%, and the third best gets 20%?

A. 720; 720.
B. 720; 120.
C. 120; 720.
D. 120; 120.

Answer: C

Explanation:
The combination, or binomial formula, gives the number of ways that k objects can be chosen from n
items, without regard to the order of choosing. The formula is n_C_k = (n choose k) = n! / [k! *(n-k)!].
In this case, we have (10 choose 3) = 10! / [3! * 7!] = (10
* 9 * 8) / (6) = 120. In the second case, where the order chosen does matter, we need the general
permutation formula, which gives the number of ways that k objects can be chosen from n items,
with regard to order. The formula is n_P_k = n! / (n- k)!. Here, it is 10! / (10-3)! = 720. For

Page | 243
permutations, we don't consider choosing analysts 3, 7, and 6 the same as choosing analysts 7, 3,
and 6, so there are more combinations than under the combination formula.

Question: 643

If the cost of a hamburger is $1.99 today, what would it cost in 30 years, assuming the price increases
at a rate of 8% per year, compounded annually?

A. $2.22
B. $17.60
C. $6.51
D. $20.02
E. $31.48

Answer: D

Explanation:
Consider the $1.99 as a PV and solve for the FV. On the BAII Plus, press 30 N, 8 I/Y, 1.99 PV, 0 PMT,
CPT FV. On the HP12C, press 30 n, 8 i, 1.99 PV, 0 PMT, FV. Make sure the BAII Plus has the P/Y value
set to 1.

Question: 644

The sampling method in which a sample is selected by first dividing the population into groups and
then selecting members from each group is known as:

A. simple random sampling.


B. stratified random sampling.
C. cluster sampling.
D. systematic random sampling.

Answer: B

Explanation:
The population is divided into strata or subcategories and a sample is randomly selected from each
strata.

Question: 645

What monthly payment is required over the next 48 months to pay off a $10,000 debt today, if
interest is charged at 14% per year, compounded monthly?

A. $366.67
B. $250.54
C. $104.54
D. $273.26
E. $116.02

Answer: D

Explanation:
On the BAII Plus, press 48 N, 14 divide 12 = I/Y, 10000 PV, 0 FV, CPT PMT. On the HP12C, press 48 n,
14 ENTER 12 divide i, 10000 PV, 0 FV, PMT. Make sure the BAII Plus has the P/Y value set to 1.

Page | 244
Question: 646

If alpha = .05, what is the probability of making a Type I error?

A. None of these answers


B. 20/20
C. 0
D. 19/20
E. 1/20

Answer: E

Explanation:
Alpha is the probability of committing a type I error. So 0.05 = 1/20.

Question: 647

What is the annual Internal Rate of Return of this series of annual cash flows: Year 0: <$15,000>, Year
1: $8,000, Year 2: $8,000, Year 3: $1,000, Year 4: $4,000 (Note that the <> are used to indicate a
negative number).

A. 16.61%
B. 17.12%
C. 18.83%
D. 15.55%
E. 13.74%

Answer: C

Explanation:
On the BAII Plus, press CF 2nd CLRWork 15000 +/- ENTER DownArrow 8000 ENTER DownArrow
DownArrow 8000 ENTER DownArrow DownArrow 1000 ENTER DownArrow DownArrow 4000 ENTER
DownArrow DownArrow 2nd Quit. Then press Irr CPT. On the HP12C, press these keys: 15000 CHS
BlueShift CFo 8000 BlueShift CFj 8000 BlueShift CFj 1000 BlueShift CFj 4000 BlueShift CFj. Then press
YellowShift Irr. The "DownArrow" represents the downward-pointing arrow on the top row of the
BAII Plus keyboard. Make sure the BAII Plus has the P/Y value set to 1.

Question: 648

The weights (in grams) of the contents of several small bottles are 4, 2, 5, 4, 5, 2 and 6. What is the
sample variance?

A. 6.92
B. None of these answers
C. 1.96
D. 2.33
E. 4.80

Answer: D

Explanation:
Sample variance is given by: (Sum of squared deviation from the mean)/(n-1). Mean is 4. Sample
variance = 14/6 = 2.33

Page | 245
xx-mean(x-mean)^2
400
2-24
511
400
511
2-24
624

Question: 649

What semiannual deposits are needed to accumulate $7,000 in 5 years if the account pays 6% per
year, compounded semiannually, assuming that the first deposit is made in 6 months and also
assuming that the account already has $1,750 in it today?

A. $405.46
B. $377.44
C. $436.15
D. $459.52
E. $578.01

Answer: A

Explanation:
On the BAII Plus, press 10 N, 6 divide 2 = I/Y, 1750 PV, 7000 +/- FV, CPT PMT. On the HP12C, press 10
n, 6 ENTER 2 divide i, 1750 PV, 7000 CHS FV, PMT. Note that the answer this time is a positive
number. This means that the $405.46 is a deposit in addition to the $1,750 original deposit. The
$7,000 is entered as a negative number, because the $1,750 and the $405.46 are deposits and the
$7,000 is a withdrawal. Make sure the BAII Plus has the value of P/Y set to 1.

Question: 650

A hypothesis test is conducted at the .05 level of significance to test whether or not the population
correlation is zero. If the sample consists of 25 observations and the correlation coefficient is 0.60,
then what is the computed value of the test statistic?

A. 1.96
B. 3.60
C. 2.94
D. 2.07
E. None of these answers

Answer: B

Explanation:
Using the t statistics, t = r* [sq. root of ((n-2)/(1-r_squared))]. t = r*[n-2/(1-r^2)]^0.5. So t =
0.6*[23/0.64]^0.5 = 3.60.

Question: 651

You can enter a derivative contract that will pay $100 at the end of a year if the price of oil exceeds
$25 per barrel, or $50 if it is equal to $25 or lower. The probability that oil will exceed $25 by the end
of one year is 60%. If interest is 4% for one year, what should the fair price of the contract be?

Page | 246
A. $80.00
B. $76.92
C. $60.00
D. $83.20

Answer: B

Explanation:
The expected payoff for the contract is $100 * 0.60 + $50 * 0.40 = $80. At 4% interest, the present
value of the expected payoff is $80/1.04 = $76.92. A deviation from this value would represent an
example of the investment consequences of inconsistent probabilities.

Question: 652

What monthly payment, beginning next month, would repay a $25,000 car loan over 60 months,
assuming your loan has an interest rate of 6.9% per year, compounded monthly?

A. $1,757.07
B. $493.85
C. $843.95
D. $1,789.14
E. $650.58

Answer: B

Explanation:
On the BAII Plus, press 60 N, 6.9 divide 12 = I/Y, 25000 PV, 0 FV, CPT PMT. On the HP12C, press 60 n,
6.9 ENTER 12 divide i, 25000 PV, 0 FV, PMT. Note that the answer will be displayed as a negative
number. Make sure the BAII Plus has the value of P/Y set to 1.

Question: 653

A new extended life light bulb has an average service life of 750 hours, with a standard deviation of
50 hours. If the service life of these light bulbs approximates a normal distribution, about what
percent of the distribution will be between 600 hours and 900 hours?

A. 95%
B. 68%
C. None of these answers
D. 34%
E. 99.7%

Answer: E

Explanation:
z = (X-u)/sigma. z1 = (600 - 750)/50 = -3.0. z2 = (900 - 750)/50 = 3.0. from the z tables, z = 3 is 0.4987.
Therefore the area between z1 and z2 is 0.4987*2 = 0.9974

Question: 654

An employee had the following percentage increases in salary over the last 5 years: 4%, 7%, 10%,
15%, 12%. The geometric mean of his salary increases equals ________.

A. 9.60%

Page | 247
B. 9.22%
C. 9.53%
D. 8.72%

Answer: D

Explanation:
The straight geometric mean of the increases is (0.04*0.07*0.1*0.15*0.12)^(1/5) = 8.72%. Note that
this is different from geometric rate of change, which in this case would be
(1.04*1.07*1.1*1.15*1.12)^(1/5) - 1 = 9.53%. You should be very careful about this point since the
Mason & Lind textbook is quite ambiguous on this point. Finally, note that the geometric mean may
not be defined if some of the salary changes are negative.

Question: 655

Suppose that stocks A, B, C, and D are independent with respect to their price movement, and have
probabilities of increasing of 0.25, 0.50, 0.40, and 0.30. What is the probability that stocks A and C
will increase in price? In other words, find P(AC).

A. 3.5%.
B. 11%.
C. 10%.
D. 1.0%.

Answer: C

Explanation:
If events are independent, then the joint probability of them occurring together is just the product of
the individual probabilities. So P(AC) = 0.25 * 0.40 = 10%. Note that we did not state whether the
other stocks, B and D, increased in value or not.

Question: 656

A stock has the following returns over 3 years: +2%, +15%, +25%. The annual geometric rate of return
over the three years is ________.

A. 7.42%
B. 19.36%
C. 11.31%
D. 9.34%
E. 12.21%
F. 10.15%
G. 14.64%
H. 13.61%

Answer: H

Explanation:
The annual geometric rate of return equals [(1+2%)(1+15%)(1+25%)]^(1/3) - 1 = (1.02 * 1.15 *
1.25)^0.33 - 1 = 0.1361 = 13.61%

Question: 657

What annual interest rate, compounded annually, would cause a series of 30 deposits of $500 to

Page | 248
accumulate to $50,000, if the first deposit is made one year from today?

A. 9.04%
B. 10.12%
C. 7.32%
D. 5.38%
E. 10.09%

Answer: C

Explanation:
On the BAII Plus, press 30 N, 0 PV, 500 PMT, 50000 +/- FV, CPT I/Y. On the HP12C, press 30 n, 0 PV,
500 PMT, 50000 CHS FV, i. Make sure the BAII Plus has the P/Y value set to 1.

Question: 658

A true-false test consists of six questions. If you guess the answer to each question, what is the
probability of getting all six questions correct?

A. None of these answers


B. 0
C. 0.06250
D. 0.03125
E. 0.0156

Answer: E

Explanation:
This is binomial distribution with p = 0.5, q = 0.5, n = 6, r = 6. Therefore 6!(0.5^6)(0.5^0)/6!0! =
0.0156.

Question: 659

If the decision in the hypothesis test of the population correlation is to reject the null hypothesis,
what can we conclude about the population correlation?

A. None of these answers is correct


B. Is not zero
C. Equals the computed sample correlation
D. Is zero
E. Could be zero

Answer: B

Explanation:
The Ho is always stated as Ho: Population parameter=0. So if we reject Ho, we say that it is not zero.

Question: 660

As the head of a trading desk at a major bank, it is your job to evaluate whether the superior
performance of a trader is due to skill or luck. To test this, you set up the following hypothesis:
Ho: Expected excess returns = 0
H1: Expected excess returns > 0
The excess returns are returns adjusted for risk using a proprietary factor model. In this set-up, which

Page | 249
of the following is/are true?
I. You must employ a one-tailed test.
II. H1 is a directional alternative.
III. Your critical z-statistics will be larger than the z-statistics in the case where the alternative is
specified as H1: Excess returns are non-zero.

A. I only
B. II & III
C. I & III
D. I & II
E. III only
F. II only
G. I, II & III

Answer: D

Explanation:
The alternative hypothesis, H1, does not assign a specific value to the expected excess return but
specifies a directional region. Since it specifies the region to the right of 0 as an alternative, he must
employ a right-tailed test i.e. a one-sided test. Note that the critical z-statistics in one-tailed
regressions are always lower than the z-statistics in the corresponding two-tailed test.

Question: 661

If you buy a computer for $950 and agree to pay for it with 24 payments of $35 with the first
payment occurring next month, what is the size of the final payment needed at month 24 to
completely pay off the computer? Assume the interest rate you are being charged is 16% per year,
compounded monthly.

A. $294.39
B. $0.00
C. $207.88
D. $87.14
E. $323.18

Answer: E

Explanation:
This question describes a situation often called a balloon payment. On the BAII Plus, press 24 N, 16
divide 12 = I/Y, 950 PV, 35 +/- PMT, CPT FV. On the HP12C, press 24 n, 16 ENTER 12 divide i, 950 PV,
35 CHS PMT, FV. The answer is shown as a negative number. Make sure the BAII Plus has the value of
P/Y set to 1.

Question: 662

What is the range of values for the coefficient of determination?

A. None of these answers


B. 0 to +1 inclusive
C. -1 to +1 inclusive
D. -3 to +3 inclusive
E. #AI -1 to 0 inclusive

Answer: B

Page | 250
Explanation:
R-square ranges from 0-1 inclusive.

Question: 663

A cola dispensing machine is set to dispense a mean of 2.02 liters into a container labeled 2 liters.
Actual quantities dispensed vary and the amounts are normally distributed with a standard deviation
of 0.015 liter. What is the probability a container will have less than 2 liters?

A. 0.8741
B. 0.0918
C. 0.3413
D. 0.1926
E. None of these answers

Answer: B

Explanation:
z = (x-u)/sigma = 2 - 2.02/0.015 = -1.33 (negative since it is below the mean). The area between -
infinity and z = -1.33 is 0.0918.

Question: 664

How long will it take for an initial deposit of $1,500 to grow to be $4,000, if the interest rate is 5% per
year, compounded annually?

A. 16.93 years
B. 20.10 years
C. 17.72 years
D. 14.48 years
E. 24.03 years

Answer: B

Explanation:
Either the $1,500 or the $4,000 must be entered as a negative number - it won't matter which. On
the BAII Plus, press 5 I/Y, 1500 PV, 0 PMT, 4000 +/- FV, CPT N. On the HP12C, press 5 i, 2500 PV, 0
PMT, 4000 CHS FV, n. Note that the HP12C will indicate 21 years for the answer. Make sure the BAII
Plus has the P/Y value set to 1.

Question: 665

Ball-Bearing, Inc. produces ball bearings automatically on a Kronar BBX machine. For one of the ball
bearings, the mean diameter is set at 20mm. the standard deviation of the production over a long
period of time was computer to be 0.150 mm. What percent of the ball bearings will have a diameter
of 20.27 mm or more?

A. 41.00%
B. 85.00%
C. 12.62%
D. None of these answers
E. 3.59%

Page | 251
Answer: E

Explanation:
z = (x-u)/sigma = 20.27 - 20/0.15 = 1.8. from the z-table, z = 1.8 is 0.4641. So 1.0 - 0.9641 = 0.0359.

Question: 666

In an investment environment, an initial outlay of $100 grows to $156 in 7 years. The quarterly
compounded rate of annual interest implicit in this is:

A. 6.56%
B. 6.40%
C. 6.12%
D. 6.73%

Answer: B

Explanation:
There are 28 quarters in 7 years. If the quarterly compounded rate is r, then we have 100*(1+r/4)^28
= 156, giving r = 6.4%

Question: 667

David's gasoline station offers 4 cents off per gallon if the customer pays in cash and does not use a
credit card. Past evidence indicates that 40% of all customers pay in cash. During a one-hour period
twenty-five customers buy gasoline at this station. What is the probability that at least ten pay in
cash?

A. None of these answers


B. .575
C. .425
D. .416
E. .586

Answer: B

Explanation:
This is a binomial distribution: n!(p^r)(q^(n-r))/r!(n-r)!. n = 25, r = 10, p = 0.4 q = 0.6

P(10) = 25!(0.4^10)(0.6^15)/10!15! = 0.1612


P(11) = 25!(0.4^11)(0.6^14)/11!14! = 0.1465
P(12) = 25!(0.4^12)(0.6^13)/12!13! = 0.1140
P(13) = 25!(0.4^13)(0.6^12)/13!12! = 0.0760
P(14) = 25!(0.4^14)(0.6^11)/14!11! = 0.0434
P(15) = 25!(0.4^15)(0.6^10)/15!10! = 0.0212
P(16) = 25!(0.4^16)(0.6^9)/16!9! = 0.0088
P(17) = 25!(0.4^17)(0.6^8)/17!8! = 0.0031

Summing up we get close to 0.574.

We can continue until r = 25 but the probability gets smaller and converges to 0.575.

Question: 668

Page | 252
Consider the following three investments:

Future valueyearsinterest rate


1.$50,00089% per year
2.$20,000612% per year
3.$35,00037% per year

The present values of the 3 investments are:

A. $28,193, $12,145, $31,422


B. $99,628, $39,476, $42,877
C. $25,093, $10,133, $28,570
D. $24,192, $11,876, $27,864

Answer: C

Explanation:
Future value = Present value*(1+r)^N for annual compounding. Therefore,

Future valueyearsratePresent Value


1.50,00089%50,000/(1.09)^8 = 25,093
2.20,000612%20,000/(1.12)^6 = 10,133
3.35,00037%35,000/(1.07)^3 = 28,570

Note that the future value must always be greater than the present value.

Question: 669

What is the relationship among the mean, median and mode in a symmetric distribution?

A. Mean is always the smallest value


B. Mean is always the largest value
C. None of these answers
D. Mode is the largest value
E. All equal

Answer: E

Explanation:
Since the distribution is symmetric, all three are equal.

Question: 670

If you deposit $5,000 today and make an additional deposit of $3,000 in 3 years, how much is in the
account 6 years from today, if the money earns interest at 8% per year, compounded annually?

A. $10,077.70
B. $13,775.16
C. $17,335.61
D. $11,713.51
E. $12,695.00

Answer: D

Page | 253
Explanation:
This question requires the solution of two problems. Find the FV of the original $5,000 deposit which
equals $7,934.37, then find the FV of the $3,000 deposit 3 years from now which equals $3779.14,
and add the two together. On the BAII Plus, press 6 N, 8 I/Y, 5000 PV, 0 PMT, CPT FV. Then press STO
1. Then press 3 N, 3000 PV, CPT FV. Then press + RCL 1 = for the answer. On the HP12C, press 6 n, 8 i,
5000 PV, 0 PMT, FV. Then press STO 1. Then press 3 n, 3000 PV, FV. Then press RCL 1 + for the answer.

Question: 671

A study by the National Park Service revealed that 50% of the vacationers going to the Rocky
Mountain region visit Yellowstone Park, 40% visit the Tetons and 35% visit both. What is the
probability that a vacationer will visit at least one of these magnificent attractions?

A. None of these answers


B. 0.55
C. 0.95
D. 0.05
E. 0.35

Answer: B

Explanation:
0.5 + 0.4 - 0.35 = 0.55

Question: 672

Assume you buy a computer for $1,450 and agree to pay for it with 36 monthly payments of $55,
beginning next month. What is the size of the final payment needed at month 36 to completely pay
off the computer, if the interest rate you are being charged is 16% per year, compounded monthly?

A. $0.00
B. $184.31
C. $217.88
D. $187.14
E. $174.39

Answer: B

Explanation:
This question describes a situation often called a balloon payment. On the BAII Plus, press 36 N, 16
divide 12 = I/Y, 1450 PV, 55 +/- PMT, CPT FV. On the HP12C, press 36 n, 16 ENTER 12 divide i, 1450 PV,
55 CHS PMT, FV. The answer is shown as a negative number. Make sure the BAII Plus has the value of
P/Y set to 1.

Question: 673

A positively skewed distribution:

A. has fat tails.


B. is skewed to the right.
C. has a large variance.
D. is skewed to the left.

Answer: B

Page | 254
Explanation:
In a positively skewed distribution, large values are more common than correspondingly small
values. This skews the distribution to the right, moving the mean to the right of the median.

Question: 674

Which of the following is/are true?


I. Type I error is the event in which we reject the null when it is false.
II. Type II error occurs when we accept the null when it is false
III. Type I error occurs if we accept the alternative when it is false.
IV. Type II error is the event where we reject the alternative when it is true.

A. II, III & IV


B. II only
C. IV only
D. I only
E. II & IV
F. I, II & IV
G. I, II & III
H. III only

Answer: A

Explanation:
Type I error is the event in which we reject the null when it is true. This is the same as accepting the
alternative when it is false. Type II error is the event in which we fail to reject the null when it is false.
This is the same as rejecting the alternative when it is true.

Question: 675

If you owe a debt of $1,000 today and also owe $2,000 in 24 months, what single payment could you
make 15 months from today that would pay off both of these debts, if interest is assessed at 8% per
year, compounded monthly?

A. $2,988.71
B. $2,751.62
C. $3,041.93
D. $3,000.00
E. $1,980.86

Answer: A

Explanation:
To solve this question, set the problem up as the sum of two compound interest calculations. Move
the $1,000 from today over to month 15 and add it to the $2,000 brought back from month 24 to
month 15. On the BAII Plus, press 15 N, 8 divide 12 = I/Y, 1000 PV, 0 PMT, CPT FV which yields
$1,104.80. Then press STO 1. Then press 9 N, 2000 FV, CPT PV, which yields $1,883.91. Finally press +
RCL 1 = to see the answer. On the HP12C, press 15 n, 8 ENTER 12 divide i, 1000 PV, 0 PMT, FV. Then
press STO 1. Then press 9 n, 2000 FV, PV. Finally press RCL 1 + to see the answer. Make sure the BAII
Plus has the value of P/Y set to 1.

Question: 676

Page | 255
You recently purchased a twin-engine plane after landing an ultra-lucrative job on Wall Street. The
annual payments on the plane are $8,000 per year and the installment plan extends over 5 years. The
payments start today. If your discount rate is 8.5% per year, how much would it have cost you to
purchase the plane on an all-cash basis?

A. $34,205
B. $40,611
C. $41,232
D. $36,429

Answer: A

Explanation:
The present value of the installment payments equals 8,000 + (8,000/0.085)*(1-1/(1.085^4)) =
$34,205

Question: 677

A normal distribution has a mean of 39 and a variance of 625. The area under the distribution
between 50 and 75 equals ________.

A. 0.745
B. 0.319
C. 0.212
D. 0.255

Answer: D

Explanation:
First note that the area between two numbers under a probability distribution is the same as the
probability of getting a number between those two numbers. The standard deviation of the
distribution equals sqrt(625) = 25. The z-value of 50 equals (50-39)/25 = 0.44. The z-value of 75
equals (75-39)/25 = 1.44. Using the Normal probability tables, we get P(z < 0.44) = 0.67 and P(z <
1.44) = 0.9251. The area between 50 and 75 is then given by P(0.44 < z < 1.44) = P(z < 1.44) - P(z <
0.44) = 0.9251 - 0.67 = 0.255.

Question: 678

How much would an original deposit of $900 grow to be after 10 and a half years, if the deposit earns
interest at 6.5% per year, compounded quarterly?

A. $1,853.97
B. $1,254.58
C. $1,573.42
D. $1,771.19
E. $1,837.51

Answer: D

Explanation:
On the BAII Plus, press 42 N, 6.5 divide 4 = I/Y, 900 PV, 0 PMT, then press CPT FV. On the HP12C, press
42 n, 6.5 ENTER 4 divide i, 900 PV, 0 PMT, then press FV. Note that N = 42 quarters (10.5 x 4 =) and
the answer is displayed as a negative number. Make sure that the BAII Plus has the P/Y value set to 1.

Page | 256
Question: 679

If you deposit $6,000 into an account paying 4% per year, compounded semiannually, how much do
you have in the account in 20 years?

A. $13,248.24
B. $12,667.70
C. $14,667.76
D. $7,403.52
E. $12,204.65

Answer: A

Explanation:
On the BAII Plus, press 40 N, 4 divide 2 = I/Y, 6000 PV, 0 PMT, CPT FV. On the HP12C, press 40 n, 4
ENTER 2 divide i, 6000 PV, 0 PMT, FV. Note that the answer will be displayed as a negative number.
Make sure the BAII Plus has the value of P/Y set to 1.

Question: 680

What disadvantage(s) are there of the mean deviation?

A. None of these answers.


B. It is based on only two observations.
C. It is based on deviations from the mean.
D. All of these answers.
E. It uses absolute values, which are difficult to manipulate.

Answer: E

Explanation:
Using absolute values can cause problems since they are difficult to work with.

Question: 681

In a stratified random sampling method,

A. a sample is selected by randomly generating an integer, N, and selecting every Nth member of the
population.
B. a sample is selected by first dividing the population into groups and then selecting members from
each group.
C. a sample is selected by drawing numbers from a normal probability distribution.
D. a sample selected such that every member of the population has the same chance of being
selected.

Answer: B

Explanation:
The population is divided into strata or subcategories and a sample is randomly selected from each
strata.

Question: 682

Ten experts rated a newly developed chocolate chip cookie on a scale of 1 to 50. Their ratings were:

Page | 257
34, 35, 41, 28, 26, 29, 32, 36, 38 and 40. What is the mean deviation?

A. 12.67
B. 4.12
C. 0.75
D. 8.00
E. None of these answers

Answer: B

Explanation:
The mean is 33.9. The mean deviation is the absolute values of the deviation from the mean. (0.1 +
1.1 + 7.1 + 5.9 + 7.9 + 4.9 + 1.9 + 2.1 + 4.1 + 6.1)/10 = 41.2/10 = 4.12

Question: 683

Which of the following are formulas for sample variance?


I. [1/(n-1)] * sum (as i goes from 1 to n) of (X_i - Xbar)^2.
II. [1/n] * sum (as i goes from 1 to n) of (X_i - Xbar)^2.
III. [1/(n-1)] * {sum (as i goes from 1 to n) of (X_i)^2 - (1/n) * [sum (as i goes from 1 to n) of (X_i)]^2}.

A. I only.
B. None of these answers is correct.
C. III only.
D. II only.

Answer: B

Explanation:
I and III are equivalent and correct; III is the computational formula for sample variance.

Question: 684

You are examining a group of 6 stocks. Their average annual standard deviations have been 25%,
20%, 15%, 25%, 16%, and 45%. What is the range of annual standard deviations?

A. 25.0%.
B. 15.0%.
C. 30.0%.
D. 29.0%.

Answer: C

Explanation:
The range = the maximum value - the minimum value. Here, we have 45% - 15% = 30%.

Question: 685

Routine physical examinations are conducted annually as part of a health service program for the
employees. It was discovered that 8% of the employees needed corrective shoes, 15% needed major
dental work and 3% needed both corrective shoes and major dental work. What is the probability
that an employee selected at random will need either corrective shoes or major dental work?

A. 0.20

Page | 258
B. 1.00
C. 0.25
D. 0.50
E. None of these answers

Answer: A

Explanation:
8% + 15% - 3% = 20%

Question: 686

Eternal Life Insurance has a policy that will pay $50,000 per year starting 20 years from now, to you
and all your progeny. If the appropriate discount rate is 10% per year and you have to pay premiums
every year for the next 15 years starting at the end of this year, what's the annual premium payment
that you and your heirs must make for this to be a fair deal?

A. $10,749
B. $14,495
C. $9,771
D. $12,189

Answer: A

Explanation:
You are entitled to a perpetuity which start paying 20 years from now. Therefore, at the end of year
19, you will have an asset worth $50,000/0.1 = $500,000. The present value of this equals
$500,000/1.1^19 = $81,754. Your premium payments are a 15-year annuity. If the annual premium is
P, then this annuity has a present value of P/0.1*[1 - 1/1.1^15] = 7.606P. This must equal the value of
the perpetuity for this to be a fair deal. This gives P = $81,754/7.606 = $10,749.

Question: 687

Delroy McWilliams, a quantitative analyst with Churn Brothers Brokerage, is examining a data sample
and has amassed the following information:

Standard deviation of the sample: 70


Number of observations: 600
Sample mean: 812

Assume that Mr. McWilliams formulates a null hypothesis that states that the value of the population
mean is equal to 800. Additionally, assume that the population standard deviation is unknown. Given
this information, what is the standard error of the estimate? Further, what is the test statistic?
Choose the best answer.

A. 8.370; 1.434
B. 8.370; 4.148
C. None of these answers is completely correct.
D. 0.014; 11.834
E. 0.014; 857.143
F. 2.858; 1.40
G. 2.858; 11.60

Answer: C

Page | 259
Explanation:
The standard error and test statistic for this example is 2.858 and 4.199, respectively. Therefore, none
of these answers is correct.
If the population standard deviation is unknown, as in this example, the standard error of the
estimate is found by using the following equation:
{Standard error = s / square root of n} where s = the sample standard deviation and n = the number
of observations in the sample.
In this example, all of the necessary information has been provided, and the determination of the
standard error of the estimate is found as:
{Standard error = [70 / 24.495] = 2.858}
Now that the standard error of the estimate has been calculated, the test statistic can be found by
using the following equation:
{Test statistic = [sample statistic - value of the population parameter under the null hypothesis] /
standard error of the sample statistic].
Again, all of the necessary information has been provided, and the calculation of the test statistic is
found as follows:
{Test statistic = [812 - 800] / 2.858 = 4.199}

Question: 688

Which of the following is/are true?


I. If a test always rejects the null, its significance level is zero.
II. If a test always accepts the null, the probability of type II error equals zero.
III. If the p-value exceeds the significance level, you cannot reject the null.

A. II only
B. I only
C. III only
D. II & III

Answer: C

Explanation:
The lowest significance level at which the null hypothesis can be rejected is called the p-value of the
test. Thus, if the p-value is less than the significance level, the null hypothesis can be rejected at that
significance level; otherwise, you must fail to reject the null. The significance level in hypothesis
testing refers to the probability that we will reject the null when it is true. Hence, if we always reject
the null, the probability of rejecting it when it is true equals 1. A Type II error occurs when we fail to
reject the null when in fact it is false. Therefore, if we always accept the null, we will never reject it
even when it is false. So the probability of Type II error will equal 1.

Question: 689

You are given n = 12, sum of the (X_i) = 2%, sum of the (X_i)^2 = 0.5%%. Find the sample standard
deviation.

A. 1.27%.
B. 1.73%.
C. 1.23%.
D. 1.11%.

Answer: C

Page | 260
Explanation:
The sample standard deviation will be the positive square root of the sample variance. The sample
variance will be determined using the computational formula [1/(n-1)] * {sum (as i goes from 1 to n)
of (X_i)^2 - (1/n) * [sum (as i goes from 1 to n) of (X_i)]^2}. So we get 1/11 * (0.5 - 1/12*(2^2)) =
1.51%%. The positive square root of 1.51%% is 1.23%.

Question: 690

Sharleef Nettleton, a quantitative analyst with Churn Brothers Brokerage, is examining a data sample
and has amassed the following information:
Standard deviation of the sample: 2.90
Number of observations: 68
Degrees of freedom: 2
Sample mean: 114
Assume that Ms. Nettleton formulates a null hypothesis that states that the value of the population
mean is zero. Additionally, assume that the population standard deviation is unknown. Given this
information, what is the standard error of the estimate? Further, what is the test statistic? Choose
the best answer.

A. 1.0199; 111.78
B. 0.3517; 84.34
C. 1.0199; 56.44
D. 0.3517; 324.14
E. None of these answers is correct.
F. 0.3570; 29.91
G. 0.3570; 319.38

Answer: D

Explanation:
If the population standard deviation is unknown, as in this example, the standard error of the
estimate is found by using the following equation:
{Standard error = s / square root of n} where s = the sample standard deviation and n = the number
of observations in the sample.
In this example, all of the necessary information has been provided, and the determination of the
standard error of the estimate is found as:
{Standard error = [2.90 / 8.2462] = 0.3517}
Now that the standard error of the estimate has been calculated, the test statistic can be found by
using the following equation:
{Test statistic = [sample statistic - value of the population parameter under the null hypothesis] /
standard error of the sample statistic].
Again, all of the necessary information has been provided, and the calculation of the test statistic is
found as follows:
{Test statistic = [114 - 0] / 0.3517 = 324.14}
This is a very large test statistic and the null hypothesis will likely be rejected unless a very low level
of confidence is employed.

Question: 691

If all the 10,000 data points in a regression lie exactly on a straight line, which of the following is/are
true?
I. The intercept term is zero.
II. The percentage of unexplained variance equals zero
III. The slope coefficient is positive.

Page | 261
IV. The correlation coefficient between the dependent and independent variables is 1.

A. III only
B. IV only
C. II & IV
D. III & IV
E. II only
F. I only

Answer: E

Explanation:
Since there is no error in the regression, the percentage of unexplained variance will be zero. The
intercept and slope terms can be any real numbers and the correlation coefficient will be either + 1
or - 1.

Question: 692

Which is true of a leptokurtic distribution?


I. It will be more peaked than the normal distribution.
II. It will have thinner tails than the normal distribution.
III. It will be less peaked than the normal distribution.
IV. It will have fatter tails than the normal distribution.

A. I and II.
B. I and IV.
C. III and II.
D. III and IV.

Answer: B

Explanation:
I and IV are true of leptokurtic distributions. This is also called having "excess kurtosis." II and III are
characteristic of platykurtic distributions.

Question: 693

An experiment may have:

A. Two or more results


B. None of these answers
C. Only two results
D. Only one result

Answer: A

Explanation:
Experiments may have many outcomes.

Question: 694

A sample of the paramedical fees charged by clinics revealed these amounts: $55, $49, $50, $45, $52
and $55. What is the median charge?

Page | 262
A. None of these answers
B. $51.00
C. $47.50
D. $52.00
E. $55.00

Answer: B

Explanation:
Order them: 45,49,50,52,55,55. The median is therefore (50 + 52)/2 = 51

Question: 695

A sample of 10 observations is drawn from a population with mean 17. The mean of the observations
equals 14.3 and the sample standard deviation equals 4.8. The sampling error in mean equals
________.

A. 9.6
B. -2.7
C. 14.3
D. 4.8

Answer: B

Explanation:
"Sampling error" refers to the difference between a sample statistic and the corresponding
population parameter. In this case, the sampling error equals 14.3 - 17 = -2.7.

Question: 696

What is the present value today of these annual cash flows: $3,000, $2,000, $1,000? Assume the first
cash flow occurs 1 year from today and an interest rate of 10% per year, compounded annually.

A. $5,205.67
B. $5,131.48
C. $6,089.92
D. $5,067.65
E. $6,000.00

Answer: B

Explanation:
You could solve this question using 3 different compound interest problems, but it is easier to solve
them using the calculator's cash flow functions. On the BAII Plus, press CF 2nd CLRWork 0 ENTER
DownArrow 3000 ENTER DownArrow DownArrow 2000 ENTER DownArrow DownArrow 1000 ENTER
DownArrow DownArrow 2nd Quit. Then press NPV 10 ENTER DownArrow CPT. On the HP12C, press
these keys: 0 BlueShift CFo 3000 BlueShift CFj 2000 BlueShift CFj 1000 BlueShift CFj. Then press 10 i,
YellowShift NPV. The "DownArrow" represents the downward-pointing arrow on the top row of the
BAII Plus keyboard. Make sure that the BAII Plus has the P/Y value set to 1.

Question: 697

A stock has the following returns over 3 years: -10%, +15%, +25%. The annual geometric rate of
return over the 3 years is ________.

Page | 263
A. 19.36%
B. 9.34%
C. 10.15%
D. 7.42%
E. 13.31%
F. 14.64%
G. 12.21%
H. 8.96%

Answer: H

Explanation:
The annual geometric rate of return equals [(1-10%)(1+15%)(1+25%)]^(1/3) - 1 = (0.9 * 1.15 *
1.25)^0.33 - 1 = 0.0896 = 8.96%

Question: 698

Monthly commissions of first-year insurance brokers are $1,270, $1,310, $1,680, $1,380, $1,410,
$1,570, $1,180 and $1,420. These figures are referred to as:

A. None of these answers


B. Raw data
C. Frequency distribution
D. Ogive
E. Histogram

Answer: B

Explanation:
Data or observations that have not been organized in any form are called raw data.

Question: 699

Which of the following is/are true?


I. It is harder to reject the null under a two-tailed test than under a one-tailed test.
II. To test the hypothesis: Ho: X = 0, H1: X < 0, you have to employ a left-tailed test.
III. The acceptance region under a right tailed test extends from zero to positive infinity. IV. The
critical z-statistics in one-tailed tests are always lower than the z-statistics in the corresponding two-
tailed test.

A. I, II, III & IV


B. none of these answers
C. I, II & III
D. I, II & IV
E. II, III & IV

Answer: D

Explanation:
Under a two-tailed test, values on either side of the null can contribute to a rejection. Hence, the
critical values under a two-tailed test have to be higher than under a one-tailed test for a given level
of significance. This makes it harder to reject the null under a two-tailed test. This makes intuitive
sense. In a one-tailed test, you are specifying a stricter alternative than under a completely general,

Page | 264
two-tailed alternative (in the current example, you are claiming that X is less than zero as an
alternative which is a stronger statement than the claim that X is non-zero). The rejection region in a
right-tailed regression extends from the critical t-value associated with the given significance level
and positive infinity. This critical value is greater than zero.

Question: 700

What is the Net Present Value of this series of annual cash flows using an interest rate of 15% per
year: Year 0: <$15,000>, Year 1: $5,000, Year 2: $8,000, Year 3: $11,000? (Note that the <> are used to
indicate a negative number).

A. $2,589.11
B. $3,104.37
C. $2,981.21
D. $5,077.49
E. $2,629.65

Answer: E

Explanation:
On the BAII Plus, press CF 2nd CLRWork 15000 +/- ENTER DownArrow 5000 ENTER DownArrow
DownArrow 8000 ENTER DownArrow DownArrow 11000 ENTER DownArrow DownArrow 2nd Quit.
Then press NPV 15 ENTER DownArrow CPT. On the HP12C, press these keys: 15000 CHS BlueShift CFo
5000 BlueShift CFj 8000 BlueShift CFj 11000 BlueShift CFj Then press 15 i, YellowShift NPV. The
"DownArrow" represents the downward-pointing arrow on the top row of the BAII Plus keyboard.
Make sure the BAII Plus has the P/Y value set to 1.

Question: 701

How many monthly payments of $30, beginning next month, are needed to pay off a debt of $1,000,
if interest accrues at 10% per year, compounded monthly?

A. No solution/Error
B. 32.80
C. 51.48
D. 39.21
E. 27.60

Answer: D

Explanation:
On the BAII Plus, press 10 divide 12 = I/Y, 1000 PV, 30 +/- PMT, 0 FV, CPT N. On the HP12C, press 10
ENTER 12 divide i, 1000 PV, 30 CHS PMT, 0 FV, n. Note that the HP12C will display 40 as the answer.
Make sure the BAII Plus has the value of P/Y set to 1.

Question: 702

Management is considering adopting a bonus system to increase production. One suggestion is to


pay a bonus on the highest 5 percent of production based on past experience. Past records indicate
that, on the average, 4,000 units of a small assembly are produced during a week. The distribution of
the weekly production is approximately normally distributed with a standard deviation of 60 units. If
the bonus is paid on the upper 5 percent of production, the bonus will be paid on how many units or
more?

Page | 265
A. None of these answers
B. 3196
C. 5120
D. 4099
E. 6255

Answer: D

Explanation:
We want the area 1.0 - 0.55 = 0.45. For this we have a z value of 1.645. Now, using z = (x-u)/sigma,
we have 1.645 = (x-4000)/60. x = 4099.

Question: 703

A selected group of employees of Unique Buying Services is to be surveyed with respect to a new
pension plan. In depth interviews are to be conducted with each employee selected in the sample.
The employees are classified as follows.

ClassificationEventNumber of Employees
SupervisorsA120
MaintenanceB50
ProductionC1,460
ManagementD302
SecretarialE68

What is the probability that the first person selected is classified as a maintenance employee?

A. 0.025
B. 0.50
C. None of these answers
D. 0.20
E. 1.00

Answer: A

Explanation:
There are 2000 employees of which 50 are from maintenance. So 50/2000 = 0.025.

Question: 704

Which measure of central tendency is used to determine the average annual percent increase?

A. Mode
B. Mean
C. Weighted mean
D. Median
E. Geometric mean

Answer: E

Explanation:
The formulaic presentation of the geometric mean is suited for calculating percentage changes. The
geometric mean of a set of n positive numbers and is defined as the nth root of the product of the n
numbers The two main uses of the geometric mean (GM) are:

Page | 266
1. To average percents, indexes, and relatives, and

2. To determine the average percent increase in sales, production or other business or economic
series from one time period to another.

Question: 705

Given the following four points: (-2, 0), (-1,0), (0,1), (1, 1) and (2, 3)

What is the standard error of the regression line estimate?

A. None of these answers


B. 0.135
C. 0
D. 0.606
E. 0.367

Answer: C

Explanation:
For this, you need to create tables with columns for X,Y,XY,X^2,Y' and Y-Y'. This will be as follows.

XYXY X^2 Y'Y-Y'


-2004-0.40.4
-10010.3-0.3
010010
11111.7-0.7
23642.40.6
Sum057100

N=5.

The regression equation is Y' = a + bX.

The standard error of estimate is

s = sq. root of [S(Y-Y')^2/n-2] where S stands for "Sum of." S = [0^2/3] = 0.

Question: 706

A study of the opinion of designers with respect to the primary color most desirable for use in
executive offices showed that:

Primary ColorNumber of Opinions


Red92
Orange86
Yellow46
Green91
Blue37
Indigo46
Violet2

What is the probability that a designer does not prefer yellow?

Page | 267
A. 1.000
B. 0.885
C. None of these answers
D. 0.765
E. 0.000

Answer: B

Explanation:
There are 400 opinions. 46/400 prefer yellow. So 1 - 46/400 = 177/200 = 0.885 does not prefer
yellow.

Question: 707

What is the median of 26, 30, 24, 32, 32, 31, 27 and 29?

A. 32
B. 30
C. None of these answers
D. 29
E. 29.5

Answer: E

Explanation:
Ordering the numbers we have 24,26,27,29,30,31,32,32. Since there is an even number of
observations (8), the median is the average of the two middle observations (29 + 30)/2 = 29.5.

Question: 708

If the significance level of a test is the probability of incorrectly rejecting the null hypothesis, then the
"power of a test" is defined as which of the following? Choose the best answer.

A. The probability of a Type II error.


B. More than one of these answers is correct.
C. The "power of a test" is a nonsensical term.
D. The probability of incorrectly accepting the alternate hypothesis.
E. The probability of correctly rejecting the null.
F. The probability of a Type I error.

Answer: E

Explanation:
If the significance level of a test is the probability of incorrectly rejecting the null hypothesis (a Type I
error), then the power of the test is defined as the probability of correctly rejecting the null
hypothesis. In other words, the power of a test is equal to (1 - the probability of a Type II error). The
standard method of hypothesis testing involves stating only the significance level, which is equal to
the probability of a Type I error.

Question: 709

There are 2,000 eligible voters in a precinct. Despite protests from knowledgeable persons that a
sample size of 500 was too large in relation to the total, the 500 selected at random were asked to

Page | 268
indicate whether they planned to vote for the Democratic incumbent or the Republican challenger.
Of the 500 surveyed, 350 said they were going to vote for the Democratic incumbent. Using the 0.99
confidence coefficient, what is the confidence limits for the proportion who plan to vote for the
Democratic incumbent?

A. 0.060 and 0.700


B. 0.826 and 0.926
C. None of these answers
D. 0.397 and 0.797
E. 0.612 and 0.712

Answer: C

Explanation:
Interval estimate can be found from p +/- z[p(1-p)/n]^0.5. Here we have n = 500, p = 350/500 = 0.7
and z = 2.58 (for 99%). Therefore 0.7 +/- 2.58*0.02049 and we get 0.647 and 0.7529.

Question: 710

Three defective electric toothbrushes were accidentally shipped to a drugstore by the manufacturer
along with 17 nondefective ones. What is the probability that the first two electric toothbrushes sold
will be returned to the drugstore because they are defective?

A. 1/4 or 0.25
B. 3/20 or 0.15
C. None of these answers
D. 3/17 or 0.176
E. 3/190 or 0.01579

Answer: E

Explanation:
3/20*2/19 = 3/190

Question: 711

How many quarters will it take for an original $1,000 deposit to grow to be $2,000, if the deposit
earns interest at 6% per year, compounded quarterly?

A. 22.15
B. 11.90
C. 41.12
D. 51.52
E. 46.56

Answer: E

Explanation:
On the BAII Plus, press 1000 PV, 2000 +/- FV, 0 PMT, 6 divided by 4 = I/Y, then CPT N. On the HP12C,
press 1000 PV, 2000 CHS FV, 0 PMT, 6 ENTER 4 divide i, then press n. HP12C answer is shown as 47.
Answer should be in quarters, not years. Make sure the BAII Plus has the P/Y value set to 1.

Question: 712

Page | 269
What does a regression equation do?

A. Predicts the value of the independent variable based on the dependent variable
B. All of these answers
C. None of these answers
D. Measures the association between two variables
E. Predicts the value of the dependent variable based on the independent variable

Answer: E

Explanation:
The dependent variable is the variable Y which is being predicted by the X variable, the independent
variable. The regression is written as Y' = a + bX. The letter "a" is the Y intercept and b is the slope of
the line. Y' is the predicted value of Y given a specific value of X.

Question: 713

Under the null hypothesis, Ho, x = y. Under the alternative hypothesis, x does not equal y. If the
critical z-statistic for the desired significance level is 1.68 and you find the z-statistic to be -3.2, you
should:

A. Reject the alternative hypothesis.


B. Reject the null hypothesis.
C. Fail to reject the null hypothesis.
D. Accept the null hypothesis.

Answer: B

Explanation:
Since the z-statistic is larger in magnitude than the critical value, you should reject the null
hypothesis. Note that all the other alternatives loosely mean the same thing.

Question: 714

Kevin wants to retire a millionaire. If he is 45 years old today and already has $80,000 in the bank,
what monthly deposit would he need to make beginning one month from today and continuing until
he retires on his 65th birthday, if the money will earn 8% per year, compounded monthly?

A. $1,028.58
B. $1,760.14
C. $407.81
D. $1,138.45
E. $2,119.81

Answer: A

Explanation:
The total number of deposits will be 240 (20 years x 12 months/year). On the BAII Plus, press 240 N,
8 divide 12 = I/Y, 80000 PV, 1000000 +/- FV, CPT PMT. On the HP12C, press 240 n, 8 ENTER 12 divide i,
80000 PV, 1000000 CHS FV, PMT. Note that the answer will be displayed as a negative number. Make
sure the BAII Plus has the P/Y value set to 1.

Question: 715

Page | 270
The employees of Cartwright Manufacturing are awarded efficiency ratings. The distribution of the
ratings approximates a normal distribution. The mean is 400, the standard deviation 50. What is the
area under the normal curve between 400 and 482?

A. 0.4750
B. 0.3413
C. 0.5000
D. 0.4495
E. None of these answers

Answer: D

Explanation:
z = (x-u)/sigma. z1 = 400 - 400/50 = 0 and z2 = 482 - 400/50 = 1.64. Therefore for z = 1.64, the area
under the curve is 0.4495.

Question: 716

Which of the following is/are true about a standard normal distribution?


I. It has zero skewness.
II. It is characterized completely by exactly one parameter.
III. It ranges from negative infinity to positive infinity.
IV. It has a non-zero mean and variance.

A. I, II, III and IV


B. I & III
C. II only
D. III only
E. I only
F. IV only
G. II and III
H. III and IV

Answer: B

Explanation:
A normal distribution is characterized by two parameters, the mean and the variance. The standard
normal distribution, though, has a zero mean and a variance of 1 and hence, has no free parameters.

Question: 717

What annual interest rate, compounded annually, is equivalent to 7% per year, compounded
quarterly?

A. 6.88%
B. 7.19%
C. 7%
D. 7.03%
E. 7.09%

Answer: B

Explanation:
Questions of this type are illustrating the concept of an Effective Interest Rate, which is a rate

Page | 271
compounded annually that has the same effect as a rate compounded more often than one time a
year. As such, a depositor or a creditor is indifferent between them, since they have the same effect.
To solve this question, make any deposit and see how much is in the account after one year. The ratio
of the ending FV to the beginning PV will indicate the annual rate earned. On the BAII Plus, press 4 N,
7 divide 4 = I/Y, 100 PV, 0 PMT, CPT FV. On the HP12C, press 4 n, 7 ENTER 4 divide i, 100 PV, 0 PMT, FV.
The number displayed will be 107.19. In other words, after one year, $100 has become $107.19. An
interest rate of 7.19%, compounded annually, would cause a $100 deposit to become $107.19 in one
year. Choosing an initial deposit of $100 helps a great deal in these situations. Make sure the BAII
Plus has the value of P/Y set to 1.

Question: 718

The following stock market return data were observed last year:

Returns# of stocks
less than -5%129
between -5% and 0%301
between 0% and 10%636
between 10% and 20%274
more than 20%173

The probability that a randomly selected stock had either a negative return or a return more than
20% equals ________.

A. 0.27
B. 0.53
C. 0.60
D. 0.40

Answer: D

Explanation:
For a frequency distribution, the estimated probability of a particular class/event occurring equals
the relative frequency of that class. Now, the number of stocks that had either a negative return or a
return more than 20% equals 129 + 301 + 173 = 603. The total number of stocks equals 129 + 301 +
636 + 274 + 173 = 1513. Therefore, the estimated probability of a randomly selected stock having
either a negative return or a return more than 20% equals 603/1513 = 0.40.

Question: 719

What annual interest rate, compounded annually, is equivalent to 6% per year, compounded
monthly?

A. 6.01%
B. 6.17%
C. 6.09%
D. 6%
E. 5.87%

Answer: B

Explanation:
Questions of this type are illustrating the concept of an Effective Interest Rate, which is a rate
compounded annually that has the same effect as a rate compounded more often than one time a

Page | 272
year. As such, a depositor or a creditor is indifferent between them, since they have the same effect.
To solve this question, make any deposit and see how much is in the account after one year. The ratio
of the ending FV to the beginning PV will indicate the annual rate earned. On the BAII Plus, press 12
N, 6 divide 12 = I/Y, 100 PV, 0 PMT, CPT FV. On the HP12C, press 12 n, 6 ENTER 12 divide i, 100 PV, 0
PMT, FV. The number displayed will be 106.17. In other words, after one year, $100 has become
$106.17. An interest rate of 6.17%, compounded annually, would cause a $100 deposit to become
$106.17 in one year. Choosing an initial deposit of $100 helps a great deal in these situations. Make
sure the BAII Plus has the value of P/Y set to 1.

Question: 720

What is the Net Present Value of this series of annual cash flows: Year 0: <$20,000>, Year 1: $15,000,
Year 2: $5,000, Year 3: $7,500, using an interest rate of 10% per year? (Note that the <> are used to
indicate a negative number).

A. $2,851.93
B. $3,512.81
C. $48.12
D. $4,116.55
E. $3,403.46

Answer: E

Explanation:
On the BAII Plus, press CF 2nd CLRWork 20000 +/- ENTER DownArrow 15000 ENTER DownArrow
DownArrow 5000 ENTER DownArrow DownArrow 7500 ENTER DownArrow DownArrow 2nd Quit.
Then press NPV 10 ENTER DownArrow CPT. On the HP12C, press these keys: 20000 CHS BlueShift CFo
15000 BlueShift CFj 5000 BlueShift CFj 7500 BlueShift CFj Then press 10 i, YellowShift NPV. The
"DownArrow" represents the downward-pointing arrow on the top row of the BAII Plus keyboard.

Question: 721

Three young children approach a gum ball machine, each with a nickel to spend. The machine has
just been filled with 50 black, 150 white, 100 red and 100 yellow balls that have been thoroughly
mixed. Sue and Jim approached the machine first. They both said they wanted red gum balls. What is
the likelihood they both will get their wish?

A. None of these answers


B. 0.062
C. 0.33
D. 0.75
E. 0.50

Answer: B

Explanation:
100/400*99/399 = 0.062

Question: 722

Which of the following is not a characteristic of the normal probability distribution?

A. Bell shaped
B. Symmetrical

Page | 273
C. All of these answers
D. Positively skewed
E. Asymptotic

Answer: D

Explanation:
The normal probability distribution is not skewed but bell shaped and symmetric.

Question: 723

In a regression, the independent variable explains 79% of the variance in the dependent variable,
leaving 21% unexplained. The slope coefficient is 0.67 and the intercept equals -10.2 The correlation
coefficient between the two variables is

A. 0.21
B. 0.89
C. 0.31
D. -0.79

Answer: B

Explanation:
In a univariate regression, the correlation coefficient between the dependent and the independent
variables equals the square root of R-square. However, you should also be careful about the sign,
which is not given by the R-square. Rather, you have to look at the sign of the slope coefficient. In
this case, the slope coefficient is positive, implying that the dependent and the independent
variables are positively correlated. The R-square equals 79% so the correlation coefficient equals
+sqrt(0.79) = 0.89.

Question: 724

RipOff Dealers, Inc. offers a financing plan for car purchases, charging 2% per month. If a car costs
$30,000, what's the monthly payment you will have to make - starting a month from now - if you
need 5 years to pay off the debt on the car?

A. $824
B. $513
C. $863
D. $798

Answer: C

Explanation:
The payments constitute an ordinary annuity. The present value of an n-period annuity that starts
paying at the end of the current period equals PV = (C/r)*[1 - 1/(1+r)^n] where C is the payment per
period and r is the one-period interest rate. In this example, the annuity is over 5*12 = 60 periods
and the per period rate equals 2%. So the present value of the payments equals (C/0.02)*[1 -
1/1.02^60] = 30,000 (given). Solving for C gives the per month payment equal to $863.

Question: 725

If you buy an item for $500 and agree to pay for it with 24 monthly payments of $24.50, beginning
next month, what annual interest rate, compounded monthly, are you being charged?

Page | 274
A. 1.34%
B. 26.53%
C. 16.08%
D. 11.15%
E. 14.92%

Answer: C

Explanation:
The interest rate returned by the calculator will be the periodic interest rate. It must be multiplied by
the number of periods per year to have the correct answer. On the BAII Plus, press 24 N, 500 PV,
24.50 +/- PMT, 0 FV, CPT I/Y. Then press x 12 = to see the answer. On the HP12C, press 24 n, 500 PV,
24.50 CHS PMT, 0 FV, i. Then press 12 x to see the answer. Make sure the BAII Plus has the P/Y value
set to 1.

Question: 726

A bank savings account offers a savings rate of 3.5% per year, compounded annually. How long would
it take for an investment of $710 to grow to $1,000?

A. 9 years
B. 8 years
C. 11 years
D. 10 years

Answer: D

Explanation:
If it takes n years for the growth, then 1000 = 710*(1+3.5%)^n. Solving for n gives 9.96 years.

Question: 727

Assume you buy a computer for $1,350 and agree to pay for it with 24 payments of $45, beginning
next month. What is the size of the final payment needed at month 24 to completely pay off the
computer, if the interest rate you are being charged is 16% per year, compounded monthly?

A. $707.88
B. $387.14
C. $494.39
D. $0.00
E. $592.21

Answer: E

Explanation:
This question describes a situation often called a balloon payment. On the BAII Plus, press 24 N, 16
divide 12 = I/Y, 1350 PV, 45 +/- PMT, CPT FV. On the HP12C, press 24 n, 16 ENTER 12 divide i, 1350 PV,
45 CHS PMT, FV. The answer is shown as a negative number. Make sure the BAII Plus has the value of
P/Y set to 1.

Question: 728

What amount would you have in your savings account in 5 years, if it has a balance of $1,234 today

Page | 275
and you deposit an additional $1,234 two years from today? Assume that the savings account earns
5% per year, compounded annually.

A. $10,048.91
B. $3,003.44
C. $2,468.00
D. $2,999.86
E. $3,149.86

Answer: B

Explanation:
Solve this question as two compound interest problems. The first $1,234 will be in the account for 5
years. The second $1,234 will be in the account for 3 years. Find the FV of both values 5 years from
today and add them together. On the BAII Plus, press 5 N, 5 I/Y, 1234 PV, 0 PMT, CPT FV, which yields
$1,574.93. Then press STO 1. Then press 3 N, CPT FV, which yields $1,428.51. Press + RCL 1 = to see
the answer. On the HP12C, press 5 n, 5 i, 1234 PV, 0 PMT, FV. Then press STO 1. Then press 3 n, FV.
Press RCL 1 + to see the answer. Note that the answer will be displayed as a negative number. Make
sure the BAII Plus has the P/Y value set to 1.

Question: 729

The probability that SUV sales will decline by over 10% by June 1 if gasoline prices increase by 50%
by May 1, is called:

A. a lagged likelihood.
B. an unconditional probability.
C. a joint probability.
D. a conditional probability.

Answer: D

Explanation:
A conditional probability takes the form of P(A|B), the probability that a an event (A) will happen,
given the occurrence of another event, B. It is conditional because it is conditioned on another event,
B.

Question: 730

You are examining a group of 6 stocks within an industry. The industry average profit margin is
expected to be 30%. For these stocks, the average profit margins have been 50%, 25%, 15%, 5%,
45%, and 30%. What is the mean absolute deviation of profit margins from the industry average?

A. 13.3%.
B. 13.7%.
C. 13.5%.
D. 13.0%.

Answer: A

Explanation:
The mean absolute deviation = the sum of (as i goes from 1 to n) of the absolute value of (X_i - Xbar),
divided by n. Here, we have [|50% - 30%| + |25% - 30%| + |15% - 30%| + |5% - 30%| + |45% - 30%|
+ |30% - 30%|]/6 = 13.33%

Page | 276
Question: 731

A large group of inductees was given a mechanical aptitude and a finger dexterity test. The mean
score on the mechanical aptitude test was 200, with a standard deviation of 10. The mean and
standard deviation for the finger dexterity test were 30 and 6 respectively. What is the relative
dispersion in the two groups?

A. Mechanical 5 percent, finger 20 percent


B. Mechanical 500 percent, finger 200 percent
C. Mechanical 20 percent, finger 10 percent
D. Mechanical 50 percent, finger 200 percent
E. None of these answers

Answer: A

Explanation:
The respective CVs are found from (s*100)/mean. Mechanical: 10*100/200 = 5% and Finger:
6*100/30 = 20%.

Question: 732

A sample of the personnel files of eight male employees revealed that during a six month period,
they lost the following number of days due to illness: 2, 0, 6, 3, 10, 4, 1 and 2. What is the mean
deviation (in days)?

A. 2 3/8
B. None of these answers
C. 1
D. 0
E. 3 1/8

Answer: A

Explanation:
The mean is 3.5. The mean deviation is the absolute values of the deviation from the mean. (1.5 +
3.5 + 2.5 + 0.5 + 6.5 + 0.5 + 2.5 + 1.5)/8 = 19/8 = 2.375

Question: 733

The slope coefficient in a regression measures:

A. The significance of the regression line.


B. The percentage change in the dependent variable caused by a 1% change in the independent
variable.
C. The rate at which the dependent variable changes with respect to the independent variable.
D. The change in the independent variable caused by a unit change in the dependent variable.

Answer: C

Explanation:
In a univariate regression, the slope coefficient gives the change in the dependent variable caused by
a unit change in the independent variable. It thus measures the rate at which the dependent variable
changes with respect to the independent variable.

Page | 277
Question: 734

Which of the following is/are true?


I. A sample is a subset of the population.
II. The population mean is used to estimate the sample mean.
III. The population mean equals the sum of all the available observations divided by the number of
observations.
IV. A sample has a unique mean.

A. I, II, III & IV


B. I, III & IV
C. IV only
D. I only
E. I & III
F. I & IV
G. II only
H. III only

Answer: F

Explanation:
The sample mean is used to estimate the population mean, not the other way around. The sum of all
the available observations divided by the number of observations is the sample mean, not the
population mean (unless the observation set equals the entire population, which is almost never the
case).

Question: 735

The R-square in a univariate regression measures:

A. the significance of the regression.


B. the variance of the error term.
C. the amount of variance in the dependent variable caused by the variance in the independent
variable.
D. the correlation between the dependent and the independent variables.

Answer: C

Explanation:
It should be noted that in a univariate regression, the square root of R-square equals the correlation
coefficient. The significance of a univariate or a multivariate regression is measured by the F-statistic.

Question: 736

John buys a house that costs $175,000 and agrees to pay for it with a 15 year mortgage at 7% per
year, compounded monthly. What is John's monthly payment on the loan?

A. $972.22
B. $552.12
C. $12,218.50
D. $12,250.06
E. $1,572.95

Page | 278
Answer: E

Explanation:
On the BAII Plus, press 180 N, 7 divide 12 = I/Y, 175000 PV, 0 FV, CPT PMT. On the HP12C, press 180 n,
7 ENTER 12 divide i, 175000 PV, 0 FV, PMT. Make sure the BAII Plus has the P/Y value set to 1.

Question: 737

Each salesperson in a large department store chain is rated either below average, average, or above
average with respect to sales ability. Each salesperson is also rated with respect to his or her
potential for advancement either fair, good, or excellent. These traits are the 500 salespeople were
cross classified into the following table.

Sales AbilityPotential for Advancement


FairGood Excellent
Below Average161222
Average456045
Above Average9372135

What is the probability that a salesperson selected at random will have average sales ability and
good potential for advancement?

A. 0.09
B. None of these answers
C. 0.12
D. 0.525
E. 0.30

Answer: C

Explanation:
Prob. of selecting a person with average sales ability: (45 + 60 + 45)/500 = 3/10. Prob. of selecting
good potential amongst those with average sales ability: 60/150. Therefore: 3/10*6/15 = 0.12.

Question: 738

You are given a risk-free rate of 3% per year, a portfolio return of 8% per year, and a standard
deviation of portfolio return of 22% per year. What is the Sharpe measure of risk-adjusted
performance?

A. 0.250.
B. 0.234.
C. 0.227.
D. 0.222.

Answer: C

Explanation:
The Sharpe measure of risk-adjusted performance is equal to (rbar_p - rbar_f)/sigma_p, where
rbar_p is the mean portfolio return, rbar_f is the mean risk-free return, and sigma_p is the standard
deviation of portfolio return. In our case, we have (8% - 3%) / 22% = 5/22 = 0.227.

Question: 739

Page | 279
A random variable, X, has a mean of 10 and a standard deviation of 15. Another random variable, Y, is
defined by Y = 2X + 3. Then, which of the following is/are true? I. The mean of Y is 33. II. The variance
of Y is 30. III. The standard deviation of Y is 33. IV. X and Y are perfectly correlated.

A. II and IV
B. I, II and IV
C. II only
D. III and IV
E. IV only
F. III only
G. I only

Answer: E

Explanation:
The mean of Y is 2*10 + 3 = 23. The standard deviation of Y is 2*15 = 30. Its variance is then equal to
30^2 = 900. Finally, since Y is a linear multiple of X, X and Y are perfectly correlated (positively, in this
case)

Question: 740

A systematic sampling method should not be used when:

A. the data are commingled.


B. the data are systematically biased.
C. the data are already arranged in the form of a predetermined pattern.
D. the data are divided into open-ended classes.

Answer: C

Explanation:
If the data are already arranged in the form of a predetermined pattern, using systematic sampling
can introduce a systematic pattern in the sample itself, which would lead to erroneous inferences
about the entire population.

Question: 741

What is the relationship between the variance and the standard deviation?

A. Variance is the square root of the standard deviation


B. None of these answers
C. No constant relationship between the variance and the standard deviation
D. Variance is twice the standard deviation
E. Variance is the square of the standard deviation

Answer: E

Explanation:
The variance is the mean of the squared deviations from the mean. The standard deviation is the
positive square root of the variance.

Question: 742

A bell-shaped, symmetrical frequency distribution has a mean of 5 and a standard deviation of 2.5.

Page | 280
The percentage of observations less than zero is about:

A. cannot be calculated
B. 5%
C. 10%
D. 2.5%

Answer: D

Explanation:
Note that the mean is 2 standard deviations above zero. The fraction of observations in a bell-
shaped, symmetrical frequency distribution which lie outside the 2-sigma range around the mean is
about 5%. Since the distribution is symmetrical about the mean, the percentage of observations less
than zero is approximately equal to 5%/2 = 2.5%.

Question: 743

What is the area under the normal curve between z = 1.0 and z = 2.0?

A. 1.0000
B. 0.1359
C. None of these answers
D. 0.7408
E. 0.4772

Answer: B

Explanation:
From the z-tables, z = 1 is 0.3413 and z = 2 is 0.4772. So the area in between is 0.4772 - 0.3413 =
0.1359.

Question: 744

For a one-tailed hypothesis test, the critical value of the test statistic is -2.33. Which of the following
is true about the hypothesis test?

A. alpha = .05 for a lower-tailed test


B. alpha = .01 for an upper-tailed test
C. alpha = .05 for an upper-tailed test
D. alpha = .01 for a lower-tailed test
E. none of these answers

Answer: D

Explanation:
This is a one tailed test and the z-value of -2.33 suggests that it is a lower tailed test and that it is for a
1% level of significance.

Question: 745

To test whether small-cap stocks perform worse than large stocks under the Fama-French three-
factor model, you set up the following hypothesis:
Ho: Expected excess returns of small stocks = 0
H1: Expected excess returns of small stocks < 0

Page | 281
The excess returns are returns adjusted for risk using the Fama-French three-factor model. In this
setup, which of the following is/are true?
I. You must employ a right-tailed test.
II. The rejection region for the z-statistic on the excess return extends from negative infinity to the
critical value associated with the significance level.
III. It is harder to reject the null than in the case where the alternative is specified as H1: Excess
returns are non-zero.

A. II only
B. I & III
C. II & III
D. none of these answers
E. I only

Answer: A

Explanation:
Since the rejection region for the null is to the left of the maintained null value of zero, you must
employ a left-tailed test. In that case, the rejection region for the z-statistic on the excess return
extends from negative infinity to the critical value associated with the significance level. Also note
that it is harder to reject the null under a two-tailed test than under a one-tailed test since the critical
value for any given significance level is higher (in magnitude) for a two-tailed test.

Question: 746

How many months will it take for an original $1,000 deposit to grow to be $2,000, if the deposit
earns interest at 7% per year, compounded monthly?

A. 9.93
B. 10.24
C. 117.19
D. 125.42
E. 119.17

Answer: E

Explanation:
On the BAII Plus, press 1000 PV, 2000 +/- FV, 0 PMT, 7 divide 12 = I/Y, then CPT N. On the HP12C,
press 1000 PV, 2000 CHS FV, 0 PMT, 7 ENTER 12 divide i, then press n. HP12C answer is shown as 120.
Answer should be in months, not years. Make sure the BAII Plus has the P/Y value set to 1.

Question: 747

You are examining the return on equity ratios of the nation's commercial airlines. You have calculated
the mean ROE to be 8%, and the sum of the squared ROEs is 0.27. Assuming there are 30 commercial
airlines, what is the population variance of ROEs in this industry?

A. 8%%.
B. 12%%.
C. 20%%.
D. 26%%.

Answer: D

Page | 282
Explanation:
The population variance is equal to the sum of the squared differences between each population
member and the population mean, divided by the number of items in the population. There is,
however, an equivalent alternate formula that is more often used for large amounts of calculation. It
is equal to the average squared observation less the squared mean. In this case, we have the average
squared observation will be 0.27 / 30 = 0.009, less the squared mean of 8%. We have 0.009 - 0.0064
= 0.0026 = 26%%.

Question: 748

Suppose a research firm conducted a survey to determine the average amount of money steady
smokers spend on cigarettes during a week. A sample of 500 steady smokers revealed that the
sample mean is $20 and the sample standard deviation is $5. What is the point estimate?

A. $20
B. None of these answers
C. $25
D. $0.04
E. $5

Answer: A

Explanation:
The sample mean is a good estimate for the population mean.

Question: 749

A survey of passengers on domestic flights revealed these miles:

Miles FlownNumber of Passengers


100 - 49916
500 - 89941
900 - 129981
1300 - 169911
1700 - 20999
2100 - 24996

What is the range (in miles)?

A. 1,100
B. 1,999
C. 2,499
D. 2,399
E. None of these answers

Answer: D

Explanation:
2499 - 100 = 2399

Question: 750

The coefficient of variation is useful when:


I. The frequency distribution contains open-ended classes or overlapping classes.

Page | 283
II. The data sets being compared contain data in different measurement units.
III. The data sets being compared contain data with similar measurement units.

A. I only
B. I & II
C. III only
D. II only
E. I & III

Answer: D

Explanation:
The coefficient of variation is useful when the means of the data sets are widely different or when
the observations are in different measurement units.

Question: 751

If you buy an item for $475 and agree to pay for it with 24 monthly payments of $22.50, beginning
next month, what annual interest rate, compounded monthly, are you being charged?

A. 17.53%
B. 14.15%
C. 12.63%
D. 12.92%
E. 1.05%

Answer: C

Explanation:
The interest rate returned by the calculator will be the periodic interest rate. It must be multiplied by
the number of periods per year to have the correct answer. On the BAII Plus, press 24 N, 475 PV,
22.50 +/- PMT, 0 FV, CPT I/Y. Then press x 12 = to see the answer. On the HP12C, press 24 n, 475 PV,
22.50 CHS PMT, 0 FV, i. Then press 12 x to see the answer. Make sure the BAII Plus has the P/Y value
set to 1.

Question: 752

The semiannually compounded rate is 6% quoted on an annualized basis. The equivalent quarterly
compounded rate is:

A. 6.12%
B. 5.91%
C. 5.96%
D. 5.76%

Answer: C

Explanation:
To solve such problems, think about investing a dollar for 1 year. The final amount should be the
same under both the quotations. Under quarterly compounded rate, r, $1 grows to (1+r/4)^4 in 1
year. Under semiannual compounding, it grows to (1+0.06/2)^2 = 1.0609. Since these two should be
equal, we get (1+r/4)^4 = 1.0609, giving r = 5.96%. Note that the quarterly compounded rate must
be smaller than the semiannually compounded rate, ruling out 6.12 automatically.

Page | 284
Question: 753

A mortgage holding company has found that 1% of its mortgage holders default on their mortgage
and lose the property. Furthermore, 90% of those who default are late on at least two monthly
payments over the life of their mortgage as compared to 45% of those who do not default. What is
the probability that a mortgagee with two or more late monthly payments will default on the
mortgage and lose the property?

A. None of these answers


B. 0.019
C. 0.009
D. 0.020
E. 0.018

Answer: D

Explanation:
We have P(def) = 0.01. P(not def) = 0.99. P(two late payments/def) = 0.90. P(two late payments/not
def) = 0.45. Using Bayes formula: p(def/two late payments) = (0.01*0.9)/(0.01*0.9 + 0.99*0.45) =
0.0198 = 0.020.

Question: 754

During the past six months, the purchasing agent bought:

Tons of Coal1,2003,000500
Price Per Ton$28.50$87.25$88.00

What is the weighted mean price per ton?

A. $89.18
B. $68.47
C. None of these answers
D. $87.25
E. $72.33

Answer: E

Explanation:
(1200*28.5)+(3000*87.25)+(500*88) = 339950. Mean = 339950/4700 = 72.33

Question: 755

Sixty percent of the customers of a fast food chain order the Whopper, fries and a drink. If a random
sample of 15 cash register receipts is selected, what is the probability that 10 or more will show that
the above three food items were ordered?

A. 0.186
B. None of these answers
C. 1,000
D. 0.403
E. 0.000

Answer: D

Page | 285
Explanation:
This is a binomial probability. The probability of getting r successes out of n trials where the
probability of success each trial is p and probability of failure each trial is q (where q = 1-p) is given
by: n!(p^r)[q^(n-r)]/r!(n-r)!. Therefore, we need to find out the probability of getting 10,
11,12,13,14,15 successes and add them up. Here n=15, p=0.6 and q=0.4. r changes from 10 to 15.

P(10 successes) = 15!(0.6^10)(0.4^5)/10!(15-10)! = 0.1859


P(11 successes) = 15!(0.6^11)(0.4^4)/11!(15-11)! = 0.1268
P(12 successes) = 15!(0.6^12)(0.4^3)/12!(15-12)! = 0.0634
P(13 successes) = 15!(0.6^13)(0.4^2)/13!(15-13)! = 0.0219
P(14 successes) = 15!(0.6^14)(0.4^1)/14!(15-14)! = 0.0047
P(15 successes) = 15!(0.6^15)(0.4^0)/15!(15-15)! = 0.00047

The sum of all the probabilities is 0.403.

Question: 756

What is the present value today of these annual cash flows: <$10,000>, $5,000, $4,000, $3,000,
$2,000? Assume the first cash flow occurs today and an interest rate of 8% per year, compounded
annually. (Note that the <> are used to indicate a negative number).

A. $1,910.54
B. $2,045.10
C. $2,110.45
D. $2,380.01
E. $4,000.00

Answer: A

Explanation:
You could solve this question using several different compound interest problems, but it is easier to
solve them using the calculator's cash flow functions. On the BAII Plus, press CF 2nd CLRWork 10000
+/- ENTER DownArrow 5000 ENTER DownArrow DownArrow 4000 ENTER DownArrow DownArrow
3000 ENTER DownArrow DownArrow 2000 ENTER DownArrow DownArrow 2nd Quit. Then press
NPV 8 ENTER DownArrow CPT. On the HP12C, press these keys: 10000 CHS BlueShift CFo 5000
BlueShift CFj 4000 BlueShift CFj 3000 BlueShift CFj 2000 BlueShift CFj. Then press 8 i, YellowShift NPV.
The "DownArrow" represents the downward-pointing arrow on the top row of the BAII Plus
keyboard.

Question: 757

The number of work stoppages in a highly industrialized region for selected months are: 6, 0, 10, 14,
8 and 0. What is the median number of stoppages?

A. 7
B. 8
C. 0
D. 6
E. None of these answers

Answer: A

Explanation:

Page | 286
Order the observations: 0,0,6,8,10,14. The median is (6 + 8)/2 = 7

Question: 758

Which of the following is not one of the five steps in the hypothesis testing procedure?

A. Identify the test statistic


B. All of these answers are part of the five steps
C. Select a level for beta
D. State the null and alternate hypothesis
E. Formulate a decision rule

Answer: C

Explanation:
The beta is determined by our choice of the decision rule and significance level. It is not determine
on the outset.

Question: 759

How much must you deposit today if you wish to have $60,000 in 10 years, assuming that interest
accumulates at 10%, compounded annually?

A. $20,153.37
B. $21,500.00
C. $9,143.39
D. $26,474.73
E. $23,132.60

Answer: E

Explanation:
On the BAII Plus, press 10 N, 10 I/Y, 0 PMT, 60000 FV, CPT PV. On the HP12C, press 10 n, 10 i, 0 PMT,
60000 FV, PV. Note that the answer is displayed as a negative number.

Question: 760

The results of the regressions using 200 observation on a variable Y against X are as follows:

CoefficientStandard error
intercept3.62.1
slope-8.11.3
R square = 49%

The correlation coefficient between X and Y is ________.

A. -0.49
B. -0.70
C. + 0.70
D. -8.10

Answer: B

Explanation:

Page | 287
In a univariate regression, the correlation coefficient between the dependent and the independent
variables equals the square root of R-square. However, you should also be careful about the sign,
which is not given by the R-square. Rather, you have to look at the sign of the slope coefficient. In
this case, the slope coefficient is negative, implying that the dependent and the independent
variables are negatively correlated and the correlation coefficient equals -sqrt(0.49) = -0.7.

Question: 761

A researcher has a sample of 400 observations from a population whose standard deviation is known
to be 136. The mean of the sample is calculated to be 17.2. The null hypothesis is stated as Ho: mean
= 4. The p-value under the alternative hypothesis H1: mean > 4 equals ________.

A. 3.92%
B. 5.2%
C. 1.96%
D. 2.6%

Answer: D

Explanation:
The z-statistic under the null is calculated to be (17.2 - 4)/(136/(400^.5)) = 1.94. The right-tailed
probability of observing a z-statistic at least as big as 1.94 equals 1.0 - 0.9738 = 0.026 = 2.6%. This is
the p-value of the right-tailed test in this sample.

Question: 762

You are given a portfolio mean return of 15%, and a standard deviation of portfolio return is 20%, and
a Sharpe ratio of 0.51. What is the risk-free rate?

A. None of these answers is correct.


B. 4.0%.
C. 5.0%.
D. 4.2%.

Answer: A

Explanation:
The Sharpe measure of risk-adjusted performance is equal to (rbar_p - rbar_f)/sigma_p, where
rbar_p is the mean portfolio return, rbar_f is the mean risk-free return, and sigma_p is the standard
deviation of portfolio return. In our case, we have (15% - x%) / 20% = 0.51. Solving for x, we obtain x
= 0.15 - 0.51
* 0.20 = 0.15 - 0.102 = 0.048 = 4.8%.

Question: 763

If you deposit $500 today into an account paying 8% per year, compounded quarterly, how much will
be in the account after 60 months?

A. $717.54
B. $702.48
C. $751.59
D. $724.46
E. $742.97

Page | 288
Answer: E

Explanation:
There are 20 quarters in 60 months (60 divide 3 =). On the BAII Plus, press 20 N, 8 divide 4 = I/Y, 500
PV, 0 PMT, CPT FV. On the HP12C, press 20 n, 8 ENTER 4 divide i, 500 PV, 0 PMT, FV. Note that the
answer will be displayed as a negative number. Make sure the BAII Plus has the value of P/Y set to 1.

Question: 764

An experiment involves selecting a random sample of 256 middle managers at random for study. One
item of interest is their mean annual income. The sample mean is computed to be $35,420 and the
sample standard deviation is $2,050. What is the point estimate?

A. $138.36
B. None of these answers
C. $2,050
D. $34,520
E. $35,420

Answer: E

Explanation:
The sample mean is a good estimate for the population mean.

Question: 765

Which of the following statements is false in reference to confidence intervals and/or tests of
significance? Choose the best answer.

A. The three conventional level of confidence are 0.10, 0.05, and 0.01.
B. All else equal, the confidence interval for a 1% significance level is larger than the confidence
interval for a 5% significance level.
C. The confidence level is typically equal to (1 - the probability of a Type II error).
D. The significance level is denoted by the Greek letter alpha.
E. The confidence level is equal to the significance level.
F. More than one of these answers is incorrect.

Answer: F

Explanation:
More than one of these answers is correct. First, the confidence level is not equal to the significance
level. Rather the confidence level is equal to (1 - the significance level). Remember that the
significance level of a test is used to quantify the probability of a Type I error, which is defined as the
act of incorrectly rejecting the null hypothesis. For example, a confidence level of 95% implies a 5%
probability of incorrectly rejecting the null hypothesis (i.e. a Type I error). For example, a hypothesis
test associated with a 0.01 significance level indicates a 0.99 level of confidence. The second
incorrect statement in this example is that the confidence level of a hypothesis test is found by (1 -
alpha), where "alpha" is equal to the probability of a Type I error. Subtracting the probability of a
Type II error from one will yield the power of the test. The remaining answers are all correct.

Question: 766

Which of the following are true regarding covariance?


I. Covariance of returns on two assets is 0 when the returns are unrelated.

Page | 289
II. Covariance will be negative if, when the return on one asset is below expected value, the other
return will also be below its expected value.
III. The covariance of returns of an asset with itself is the asset's return variance.
IV. Covariance is the probability weighted average of the cross product of the deviation of each
random variable from its expected value.

A. All but II.


B. All but I.
C. All but IV.
D. All but III.

Answer: A

Explanation:
I, III, and IV are all true. II is false. To make II true, we would have to restate it as "Covariance will be
negative if, when the return on one asset is below expected value, the other return will be above its
expected value."

Question: 767

The U.S. Federal Aviation Administration reported that passenger revenues on international flights
increased from $528 million in 1972 to $5,100 million in 1995. What is the geometric mean annual
percent increase in international passenger revenues?

A. None of these answers


B. 27.9
C. 9.96
D. 103.6
E. 10.4

Answer: E

Explanation:
There are 24 years involved. The geometric mean is = [(1 + 5100/528)^1/23]-1. In words, it is the
23rd square root of (1 + 5100/528) minus 1. So we have GM = (1 + 9.66)^(1/23) - 1 = 0.1036 = 10.4%

Question: 768

Which of the following is/are true?


I. There are as many values above the mean as below it.
II. The sum of the differences between the observations in a sample and the mean of the sample
equals zero.
III. The mean is greatly affected by "outliers."
IV. The mean is harder to estimate with reliability for open-ended data.

A. I, II & III
B. III only
C. II & III
D. II only
E. I only
F. IV only
G. I & III
H. II, III & IV

Page | 290
Answer: H

Explanation:
I would be correct if "mean" is replaced by "median" i.e. there are as many values above the median
as below it. Note that one of the advantages of using the median instead of the mean is that the
mean is greatly affected by outliers since it represents an average of all the observations while the
median is not affected in this fashion.

Question: 769

What degree of relationship do values of the correlation coefficient (r) with magnitudes close to 1.00
indicate?

A. Weak
B. Strong
C. Moderate
D. None

Answer: B

Explanation:
The closer to 1, the stronger the relationship.

Question: 770

A researcher has a sample of 700 observations from a population whose standard deviation is known
to be 1,235.6. The mean of the sample is calculated to be 219.2. The null hypothesis is stated as Ho:
mean = 150 and the alternative is non-directional. The p-value in this case equals ________.

A. 10.16%
B. 13.88%
C. 6.94%
D. 12.82%

Answer: B

Explanation:
The z-statistic under the null equals (219.2 - 150)/(1235.6/(700^.5)) = 1.48. Since the alternative is
non-directional, you must use a two-tailed test and ask the question, "What is the probability of
observing a z-statistic at least as large in magnitude as 1.48?" The answer to this will give you the p-
value for this sample under the two-tailed test. The right-tailed probability of observing a z-statistic
which is at least as big as 1.48 equals 1.0 - 0.9306 = 0.0694 = 6.94%. The left-tailed probability of
observing a z-statistic which is at least as small as - 1.48 also equals 6.94% since the standard normal
distribution is symmetrical about zero. Therefore, the p-value of this two-tailed test in this sample
equals 6.94%*2 = 13.88%.

Question: 771

In a sample of 2,000 persons, 1,600 favored more strict environmental protection measures. What is
the estimated population proportion?

A. 0.82 or 82%
B. 0.20 or 20%
C. 0.16 or 16%

Page | 291
D. None of these answers
E. 0.80 or 80%

Answer: E

Explanation:
Population proportion is given by 1600/2000 = 0.80

Question: 772

What is the Net Present Value of this series of annual cash flows using an interest rate of 15% per
year: Year 0: <$10,000>, Year 1: $5,000, Year 2: $5,000, Year 3: $7,500? (Note that the <> are used to
indicate a negative number).

A. $2,981.21
B. $5,077.49
C. $3,059.92
D. $3,224.98
E. $3,104.37

Answer: C

Explanation:
On the BAII Plus, press CF 2nd CLRWork 10000 +/- ENTER DownArrow 5000 ENTER DownArrow
DownArrow 5000 ENTER DownArrow DownArrow 7500 ENTER DownArrow DownArrow 2nd Quit.
Then press NPV 15 ENTER DownArrow CPT. On the HP12C, press these keys: 10000 CHS BlueShift CFo
5000 BlueShift CFj 5000 BlueShift CFj 7500 BlueShift CFj Then press 15 i, YellowShift NPV. The
"DownArrow" represents the downward-pointing arrow on the top row of the BAII Plus keyboard.

Question: 773

The following are the weekly amounts of welfare payments made by the federal government to a
sample of six families: $139, $136, $130, $136, $147 and $136. What is the range?

A. $52
B. None of these answers
C. $14
D. $17
E. $0

Answer: D

Explanation:
Range = 147 - 130 = 17

Question: 774

A board of directors consists of eight men and four women. A four member search committee is to
be chosen at random to recommend a new company president. What is the probability that all four
members of the search committee will be women?

A. None of these answers


B. 1/16 or 0.0625
C. 1/495 or 0.002

Page | 292
D. 1/8 or 0.125
E. 1/120 or 0.00083

Answer: C

Explanation:
4/12*3/11*2/10*1/9 = 1/495

Question: 775

You are evaluating 5 portfolio managers (A, B, C, D, and E) whose Sharpe ratios are 0.25, 0.41, 0.92,
0.78, and 0.51, respectively. Which manager would most risk-averse investors prefer?

A. C.
B. A.
C. E.
D. None of these answers is correct.

Answer: A

Explanation:
The Sharpe measure of risk-adjusted performance is equal to (rbar_p - rbar_f)/sigma_p, where
rbar_p is the mean portfolio return, rbar_f is the mean risk-free return, and sigma_p is the standard
deviation of portfolio return. Thus, the manager with the highest Sharpe ratio is generating the
largest return in excess of the risk free rate, per unit of risk assumed. This is manager C, with a
Sharpe ratio of 0.92.

Question: 776

The mean of a normal distribution is 400 pounds. The standard deviation is 10 pounds. What is the
area between 415 pounds and the mean of 400 pounds?

A. None of these answers


B. 0.4332
C. 0.3413
D. 0.1932
E. 0.5000

Answer: B

Explanation:
z = (x-u)/sigma. z = 415 - 400/10 = 1.5. From the z-tables, z = 1.5 is 0.4332. Since we are considering
just one side of the distribution and the area from the mean, then the area between the points is
0.4332.

Question: 777

Four of your firm's analysts have been chosen as finalists for the Wealth Magazine All-Star Analysts
team. They will get different bonuses depending on the order in which they finish among
themselves. How many different ways could they finish?

A. 10.
B. 24.
C. 16.

Page | 293
D. 36.

Answer: B

Explanation:
The factorial method allows you to determine the number of ways of arranging n things. Initially,
there are n choices for the first item, then n-1 for the second, and so on. The number of
combinations is therefore n! = n * (n-1) * (n-2) * (n-3) * ... * (1). Here, we wish to find out how many
different orders can be made with 4 persons. The answer is 4! = 4 * 3 * 2 * 1 = 24.

Question: 778

If you deposit $2,000 today, how much will you have in 3 years, if interest is earned at a rate of 8%
per year, compounded annually?

A. $2,519.42
B. $2,576.14
C. $2,400.00
D. $3,121.88
E. $2,614.09

Answer: A

Explanation:
On the BAII Plus, press 3 N, 8 I/Y, 2000 PV, 0 PMT, CPT FV. On the HP12C, press 3 n, 8 i, 2000 PV, FV.
Note that the answer will be displayed as a negative number. Make sure the BAII Plus has the P/Y
value set to 1.

Question: 779

What are the critical values for a two-tailed test of hypothesis test if alpha = .01?

A. +/- 1.645
B. +/- 2.58
C. +/- 1.96
D. None of these answers
E. +/- 2.33

Answer: B

Explanation:
For a level of significance of 0.01, we can find the z value by 1.0 - 0.505 = 0.495 which is 2.58.

Question: 780

A test statistic is:

A. a value calculated from a sample and used to accept or reject the null hypothesis.
B. all of these answers.
C. a sample value that is used to measure a population parameter.
D. a parameter characterizing the null hypothesis.

Answer: A

Page | 294
Explanation:
A test statistic is a value calculated from a sample and used to accept or reject the null hypothesis
concerning a population's parameter.

Question: 781

If you deposit $250 a year, beginning next year, for 30 years into an account paying 6% per year,
compounded annually, how much is in your account after that last deposit?

A. $21,149.52
B. $12,334.91
C. $19,764.55
D. $18,304.12
E. $3,441.21

Answer: C

Explanation:
On the BAII Plus, press 30 N, 6 I/Y, 0 PV, 250 PMT, CPT FV. On the HP12C, press 30 n, 6 i, 0 PV, 250
PMT, FV. Note that the answer is displayed as a negative number. Make sure the BAII Plus has the P/Y
value set to 1.

Question: 782

What is the present value of $250 per year for 6 years, with the first cash flow occurring today, rather
than 1 year from now? Assume interest is 7% per year, compounded annually.

A. $1,209.48
B. $1,500.00
C. $1,191.63
D. $1,275.05
E. $1,293.55

Answer: D

Explanation:
Recognize that this question is an annuity due situation, since the first cash flow occurs immediately
or at the beginning of each period. Annuities where the first payment occurs 1 period from today (or
at the end of each period) are called "ordinary" annuities. This requires placing the calculator into
"Begin" mode prior to solving the question. NOTE: Be sure to place the calculator OUT OF annuity
due mode after this question before going on to subsequent questions, or you will get the wrong
answers! On the BAII Plus, press 2nd BGN. If the display shows END, then press 2nd SET and then 2nd
Quit. This will place the BAII Plus into annuity due mode (you can tell this because the BAII Plus will
display BGN in small letters). Now press 6 N, 7 I/Y, 250 PMT, 0 FV, CPT PV. Place the calculator back
into End mode (for ordinary annuities) by pressing 2nd BGN and then if the calculator is displaying
BGN, press 2nd SET and 2nd Quit. The BGN letters should disappear from the display. On the HP12C,
press BlueShift END, which is the blue function on the front of the 8 digit key. This places the HP12C
into Begin mode (the HP12C shows the word BEGIN in the display when in this mode). Then press 6
n, 7 i, 250 PMT, 0 FV, PV. To place the HP12C back into ordinary annuity mode (or END mode), press
BlueShift BEG (the blue function written on the front of the 7 digit key). Note that the answer is
displayed as a negative number. Make sure the BAII Plus has the value of P/Y set to 1.

Question: 783

Page | 295
Assume you buy a car for $20,000 today and agree to pay $200 a month, beginning next month, for 4
years with a final payment also due 4 years from today to pay off any remaining balance. How large
will that final payment be, if interest accrues at 5.9% per year, compounded monthly?

A. $19,590.32
B. $24,280.77
C. $19,204.41
D. $10,400.00
E. $14,511.19

Answer: E

Explanation:
There are 48 months in 4 years. This is a problem known as a balloon payment. On the BAII Plus,
press 48 N, 5.9 divide 12 = I/Y, 20000 PV, 200 +/- PMT, CPT FV. On the HP12C, press 48 n, 5.9 ENTER
12 divide i, 20000 PV, 200 CHS PMT, FV. Note that the answer will be displayed as a negative number.
Make sure the BAII Plus has the value of P/Y set to 1.

Question: 784

You are reviewing a list of 8 recommended securities and wish to invest in 4. You will put 40% of your
capital in one, 30% in another, 20% in the third, and 10% in the last one. How many different ways
can you choose among the 8 securities and invest according to your design?

A. 1,980.
B. 1,680.
C. 1,774.
D. 1,860.

Answer: B

Explanation:
The combination, or binomial formula, gives the number of ways that k objects can be chosen from n
items, without regard to the order of choosing. The formula is (n choose k) = n! / [k! *(n-k)!].
However, in this case, the order does matter. If we choose stocks 3, 1, 7, and 5, we don't consider
that the same as choosing 5, 7, 1 and 3, because the portfolio weights would differ. We need the
general permutation formula, which gives the number of ways that k objects can be chosen from n
items, with regard to order. The formula is n_P_k = n! / (n-k)!. Here, it is 8! / (8-4)! = 8 * 7 * 6 * 5 =
1,680.

Question: 785

The significance level in hypothesis testing refers to the:

A. probability of Type II error.


B. 1-probability of Type I error.
C. joint significance of the associated regression, indicated by the F-statistic.
D. probability of Type I error.
E. none of these answers.

Answer: D

Explanation:
Type I error refers to the event that we will reject the null when, in fact, it is true. The probability of

Page | 296
this error is called the "significance level" of the test used to evaluate the hypothesis. Clearly, lower
significance levels are better, all else equal.

Question: 786

What is the area under the normal curve between z = -1.0 and z = -2.0?

A. 0.0228
B. 0.4772
C. 0.3413
D. 0.1359
E. None of these answers

Answer: D

Explanation:
From the z-tables, z = 1 is 0.3413 and z = 2 is 0.4772. So the area in between is 0.4772 - 0.3413 =
0.1359.

Question: 787

What is the annual Internal Rate of Return of this series of annual cash flows: Year 0: <$14,000>, Year
1: $5,000, Year 2: $4,000, Year 3: $5,000, Year 4 a negative number).

A. 10.69%
B. 11.14%
C. 12.04%
D. 45.29%
E. 10.49%

Answer: B

Explanation:
On the BAII Plus, press CF 2nd CLRWork 14000 +/- ENTER DownArrow 5000 ENTER DownArrow
DownArrow 4000 ENTER DownArrow DownArrow 5000 ENTER DownArrow DownArrow 4000 ENTER
DownArrow DownArrow 2nd Quit. Then press Irr CPT. On the HP12C, press these keys: 14000
CHSBlueShift CFo 5000 BlueShift CFj 4000 BlueShift CFj 5000 BlueShift CFj 4000 BlueShift CFj. Then
press YellowShift Irr. The "DownArrow" represents the downward-pointing arrow on the top
row of the BAII Plus keyboard. Make sure the BAII Plus has the value of P/Y set to 1.

Question: 788

If the sample variance for a frequency distribution consisting of hourly wages was computed to be
10, what is the sample standard deviation?

A. $3.16
B. None of these answers
C. $1.96
D. $4.67
E. $10.00

Answer: A

Explanation:

Page | 297
Sample standard deviation is the square root of the sample variance.

Question: 789

Type II error refers to the event that we will:

A. Fail to reject the null when it is false.


B. Accept the alternative when it is false.
C. Reject the null when it is true.
D. Reject the alternative when it is true.

Answer: A

Explanation:
Remember that the null hypothesis is the one that you maintain to be true unless there is sufficient
evidence to prove otherwise. Therefore, the first type of mistake that can happen is that you reject
the maintained hypothesis when in fact, it is true. This error is referred to as "Type I" error. On the
other hand, you may not have sufficient evidence to disprove the null when in fact, it is false. This
failure to appropriately reject the null is referred to as "Type II" error.

Question: 790

Of 900 consumers surveyed, 414 said they were very enthusiastic about a new home decor scheme.
What is the 99% confidence interval for the population proportion (in percent)?

A. 42 and 50
B. None of these answers
C. 30 and 40
D. 30 and 60
E. 31 and 51

Answer: A

Explanation:
Interval estimate can be found from p +/- z[p(1-p)/n]^0.5. Here we have n = 900, p = 414/900 = 0.46
and z = 2.58 (for 99%). Therefore 0.46 +/- 2.58*0.01661 and we get 0.42 and 0.50.

Question: 791

What is the present value today of these annual cash flows: $300, $1,200, $2,500? Assume the first
cash flow occurs 1 year from today and an interest rate of 9% per year, compounded annually.

A. $3,215.70
B. $2,989.06
C. $3,244.09
D. $3,298.82
E. $3,308.86

Answer: A

Explanation:
You could solve this question using 3 different compound interest problems, but it is easier to solve
them using the calculator's cash flow functions. On the BAII Plus, press CF 2nd CLRWork 0 ENTER
DownArrow 300 ENTER DownArrow DownArrow 1200 ENTER DownArrow DownArrow 2500 ENTER

Page | 298
DownArrow DownArrow 2nd Quit. Then press NPV 9 ENTER DownArrow CPT. On the HP12C, press
these keys: 0 BlueShift CFo 300 BlueShift CFj 1200 BlueShift CFj 2500 BlueShift CFj. Then press 9 i,
YellowShift NPV. The "DownArrow" represents the downward-pointing arrow on the top row of the
BAII Plus keyboard. Make sure that the BAII Plus has the P/Y value set to 1.

Question: 792

A perpetual preferred stock has a face value of $1,000 and a coupon rate of 6% per year. If it is issued
at $850, what's the implicit annual discount rate?

A. 6.94%
B. 7.00%
C. 7.19%
D. 7.06%

Answer: D

Explanation:
A preferred stock is valued like a perpetuity. The price of a preferred stock with a face value F and a
coupon rate c equals F*c/r, where r is the implicit discount rate. In this case, we have 850 =
1,000*0.06/r, giving r = 7.06%.

Question: 793

If you deposit $2,250 today into a savings account paying 8% per year, compounded semiannually,
how much is in your account in 10 years?

A. $4,587.85
B. $10,487.15
C. $3,330.55
D. $4,857.58
E. $4,930.03

Answer: E

Explanation:
On the BAII Plus, press 20 N, 8 divide 2 = I/Y, 2250 PV, 0 PMT, CPT FV. On the HP12C, press 20 n, 8
ENTER 2 divide i, 2250 PV, 0 PMT, FV. Note that the answer will be displayed as a negative number.
Make sure the BAII Plus has the P/Y value set to 1.

Question: 794

The ages of all the patients in the isolation ward of the hospital are 38, 26, 13, 41 and 22. What is the
population variance?

A. 91.4
B. 106.8
C. None of these answers
D. 240.3
E. 42.4

Answer: B

Explanation:

Page | 299
Population variance = (Sum of squared deviation from the mean)/N. the mean is 28. Population
variance = (100 + 4 + 225 + 169 + 36)/5 = 534/5 = 106.8

xx-mean(x-mean)^2
3810100
26-24
13-15225
4113169
22-636

Question: 795

If you deposit $100 a month, beginning next month, for 20 years into an account paying 8% per year,
compounded monthly, how much is in your account after that last deposit?

A. $2,131.88
B. $58,902.04
C. $4,576.20
D. $61,549.18
E. $24,000.00

Answer: B

Explanation:
On the BAII Plus, press 240 N, 8 divide 12 = I/Y, 0 PV, 100 PMT, CPT FV. On the HP12C, press 240 n, 8
ENTER 12 divide i, 0 PV, 100 PMT, FV. On the BAII Plus, make sure the value of P/Y is set to 1. Note
that the answer is displayed as a negative number.

Question: 796

Rhonda McLeavon, a quantitative analyst with Churn Brothers Brokerage, has been instructed to
perform a statistical analysis to test whether increases in marketing expenses amongst drug
companies is positively related to institutional buying of these same companies' common stock.
Rhonda begins her analysis and performs the following steps, in succession:

Step 1: Formulating and stating the hypothesis


Step 2: Identifying the appropriate test statistic and probability distribution
Step 3: Specifying the significance level
Step 4: Stating the decision rule

Now that these first four steps have been completed, Ms. McLeavon should proceed with which of
the following? Choose the best answer.

A. None of these answers is correct


B. Collecting the data and performing the calculations
C. Determining the z-statistic
D. Determining the p-value
E. Performing and autocorrelation test to identify any heterskedasticity problems
F. Evaluating the goodness of fit

Answer: B

Explanation:
Hypothesis testing involves a series of seven explicit steps:

Page | 300
Step 1: Formulating and stating the hypothesis
Step 2: Identifying the appropriate test statistic and its probability distribution
Step 3: Specifying the significance level
Step 4: Stating the decision rule
Step 5: Collecting the data and performing the necessary calculations
Step 6: Making the statistical decision
Step 7: Making the economic/investment decision.

Question: 797

As the size of the sample increases, what happens to the shape of the sampling mean?

A. Negatively skewed
B. Cannot be predicted in advance
C. Positively skewed
D. None of these answers
E. Approaches a normal distribution

Answer: E

Explanation:
The central limit theorem states that as the sample size gets larger, the sampling distribution of the
sample means becomes approximately normal.

Question: 798

The coefficient of variation (CV) for a set of annual incomes is 18%; the coefficient of variation for the
length of service with the company is 29%. What does this indicate?

A. More dispersion in the distribution of the incomes compared with the dispersion of their length of
service
B. Dispersions are equal
C. Dispersions in the two distributions (income and service) cannot be compared using percents
D. None of these answers
E. More dispersion in the lengths of service compared with incomes

Answer: E

Explanation:
The CV is the ratio of the standard deviation to the mean, express as a percent. The higher the
coefficient, the greater the dispersion.

Question: 799

Carlos Johanson, a quantitative analyst with Eastern Rhodium Institutional Brokerage, has been
instructed to perform a regression analysis to test whether the proliferation of neural networks is
positively related to the growth of domestic research grants. Mr. Johanson begins the process by
formulating and stating a hypothesis. Now that the hypothesis has been explicitly stated, Mr.
Johanson should proceed to which of the following steps? Choose the best answer.

A. Specifying the significance level


B. Stating the decision rule
C. Collecting the data and performing the calculations

Page | 301
D. Identifying the appropriate test statistic and probability distribution
E. Performing an autocorrelation test

Answer: D

Explanation:
Hypothesis testing involves a series of seven explicit steps:
Step 1: Formulating and stating the hypothesis
Step 2: Identifying the appropriate test statistic and its probability distribution
Step 3: Specifying the significance level
Step 4: Stating the decision rule
Step 5: Collecting the data and performing the necessary calculations
Step 6: Making the statistical decision
Step 7: Making the economic/investment decision.
In this example, Carlos Johanson, a quantitative analyst, has been instructed to perform a regression
analysis analyzing the relationship between two sets of variables. Mr. Johanson began his task with
the appropriate first step - formulating and stating the hypothesis. The next step in the hypothesis
testing process is to identify the appropriate test statistic and its probability distribution.

Question: 800

What simple annual interest rate would cause a $120 deposit to grow to $125 in 6 months?

A. 25%
B. 7.89%
C. 4.17%
D. 3.14%
E. 8.33%

Answer: E

Explanation:
To solve this question, we have to rearrange the simple interest formula, I=PRT so that the R is by
itself. Note that the I (interest) is equal to 5 (125 - 120). Note also that the time component T of the
formula is 6/12. On the BAII Plus, press 5 divide 120 divide 6 x 12 = to see the answer. On the HP12C,
press 5 ENTER 120 divide 6 divide 12 x to see the answer.

Question: 801

What is the annual Internal Rate of Return of this series of annual cash flows: Year 0: <$20,000>, Year
1: $15,000, Year 2: $5,000, Year 3: $7,500? (Note that the <> are used to indicate a negative number).

A. 20.49%
B. 21.17%
C. 23.12%
D. 18.32%
E. 25.51%

Answer: B

Explanation:
On the BAII Plus, press CF 2nd CLRWork 20000 +/- ENTER DownArrow 15000 ENTER DownArrow
DownArrow 5000 ENTER DownArrow DownArrow 7500 ENTER DownArrow DownArrow 2nd Quit.
Then press Irr CPT. On the HP12C, press these keys: 20000 CHS BlueShift CFo 15000 BlueShift CFj

Page | 302
5000BlueShift CFj 7500 BlueShift CFj Then press YellowShift Irr. The "DownArrow" represents the
downward-pointing arrow on the top row of the BAII Plus keyboard.

Question: 802

Stock A is highly volatile while stock B has low volatility. The difference between the arithmetic and
geometric rates of return will be:

A. same for both stocks, since the difference is not dependent on volatility.
B. larger for stock A or stock B, depending on their riskiness.
C. larger for stock A.
D. larger for stock B.

Answer: C

Explanation:
The mean and the geometric mean are equal when volatility in the rate of return is zero. For a non-
zero volatility, the mean exceeds the geometric mean and the difference is larger the higher the
volatility.

Question: 803

There are 10 rolls of film in a box and 3 are defective. Two rolls are to be selected, one after the
other. What is the probability of selecting a defective roll followed by another defective roll?

A. 1/120, or about 0.0083


B. 1/15, or about 0.07
C. None of these answers
D. 1/4, or 0.25
E. 1/2, or 0.50

Answer: B

Explanation:
The probability of selecting a defective roll in the first selection is 3/10. The probability of choosing
another defective roll is 2/9. So 3/10*2/9 = 6/90 = 1/15.

Question: 804

The results of the regressions using 200 observation on a variable Y against X are as follows:

CoefficientStandard error
intercept3.62.1
slope8.11.3

R square = 49%

The percentage of variance in the dependent variable not explained by the regression equals
________.

A. 36.4%
B. 49%
C. 18%
D. 51%

Page | 303
Answer: D

Explanation:
The R-square of the regression measures the amount of variance of the dependent variable
explained by the independent variable. This is given to be 49%. Hence, the amount not explained
equals 100% - 49% = 51%.

Question: 805

What is the probability of making a Type II error if the null hypothesis is actually true?

A. .05
B. alpha
C. 1
D. 0
E. none of these answers

Answer: D

Explanation:
The type II error is accepting a null hypothesis when it is actually false. The probability of committing
this error is called beta. Therefore the probability of making a type II error when the null is true is
zero.

Question: 806

Which of the following is not a valid reason for selecting a sample instead of studying the whole
population?

A. All of these answers reasons are valid


B. Sample results have more accuracy than population results
C. Cost of studying an entire population can be too high
D. Population can be destroyed in the process of studying it
E. Study of the population can be too time consuming

Answer: B

Explanation:
Sample results cannot be more accurate since it is based on a subset of the population.

Question: 807

What annual interest rate, compounded annually, will cause an original deposit of $500 to grow to
$625, after 3 years?

A. 7.27%
B. 9.14%
C. 6.78%
D. 7.72%
E. 2.57%

Answer: D

Page | 304
Explanation:
On the BAII Plus, press 500 PV, 625 +/- FV, 0 PMT, 3 N, then CPT I/Y. On the HP12C, press 500 PV, 625
CHS FV, 0 PMT, 3 n, then press i.

Question: 808

A cumulative frequency distribution on days absent during a calendar year by employees of a


manufacturing company is shown below.

Days AbsentCumulative Number of Employees


0 - 260
3 - 531
6 - 814
9 - 116
12 - 142

How many employees were never absent?

A. None of these answers


B. 29
C. Cannot be determined from the given data
D. 31
E. 60

Answer: C

Explanation:
We were given only the number of employees in the 0-2 group. So we do not know how many were
never absent since this group includes those who were absent 1 and 2 days.

Question: 809

Judging from recent experience, 5 percent of the worm gears produced by an automatic, high speed
machine are defective. What is the probability that out of six gears selected at random, exactly zero
gears will be defective?

A. 0.500
B. 0.167
C. 0.735
D. 0.001
E. None of these answers

Answer: C

Explanation:
Using Binomial probability with n = 6, p = 0.05 and q = 0.95 and r = 0 we get [6!(0.05^0)(0.95^6)]/0!(6
- 0)! = 0.735.

Question: 810

Armando Delrio, a quantitative analyst with Brown Brothers Brokerage, has been instructed to create
a regression analysis comparing the relationship between same store sales figures for a batch of
retail stocks and marketing expenses for the same series of stocks. In order to adhere to the
traditional seven-step method of hypothesis testing, Armando should begin his analysis by

Page | 305
performing which of the following actions? Choose the best answer.

A. Formulating the hypothesis


B. Collecting the data
C. Stating the significance level
D. None of these answers is correct
E. Identifying the test statistic
F. Identifying the probability distribution

Answer: A

Explanation:
Hypothesis testing involves a series of seven explicit steps:

Step 1: Formulating and stating the hypothesis


Step 2: Identifying the appropriate test statistic and its probability distribution
Step 3: Specifying the significance level
Step 4: Stating the decision rule
Step 5: Collecting the data and performing the necessary calculations
Step 6: Making the statistical decision
Step 7: Making the economic/investment decision.

In this example, Armando Delrio, a quantitative analyst, has been instructed to perform a regression
analysis in which a hypothesis is tested. The first step in this process is the formulation of the
hypothesis.

Question: 811

The sampling distribution of the sampling mean is:

A. the distribution of the means of all possible samples of a given size from a given population.
B. the distribution of all possible samples of a given population.
C. the distribution of all possible means of a given population.
D. the distribution of the means of all possible samples of a given population.

Answer: A

Explanation:
The sampling distribution of the sample means is a probability distribution of all possible sample
means of a specific sample size.

Question: 812

All possible samples of size n are selected from a population and the mean of each sample is
determined. What is the mean of the sample mean?

A. Cannot be estimated in advance


B. None of these answers
C. Exactly the same as the population mean
D. Larger than the population mean
E. Smaller than the population mean

Answer: C

Page | 306
Explanation:
If we selected all possible samples from the population, then the sample mean will be equal to the
population mean. Only when we cannot select all possible samples will be a difference.

Question: 813

What deposit today is needed to have $2,000 in 4 years, assuming the money will earn interest at 6%
per year, compounded monthly?

A. $1,934.02
B. $1,290.81
C. $1,574.20
D. $1,384.57
E. $1,900.00

Answer: C

Explanation:
On the BAII Plus, press 48 N, 6 divide 12 = I/Y, 0 PMT, 2000 FV, CPT PV. On the HP12C, press 48 n, 6
ENTER 12 divide i, 0 PMT, 2000 FV, PV. Make sure the BAII Plus has the P/Y value set to 1.

Question: 814

A bell-shaped, symmetrical frequency distribution has a mean of 50. If 68% of the observations in the
distribution fall between 40 and 60, what's the range within which 99% of the observations in the
distribution fall?

A. [19.79; 80.21]
B. [23.1; 76.9]
C. [30; 70]
D. [24.2; 75.8]

Answer: D

Explanation:
For a bell-shaped, symmetrical frequency distribution, approximately 68% of the observations lie
within one standard deviation of the mean. The mean equals (40 + 60)/2 = 50. Therefore, the
standard deviation equals 50 - 40 = 10. Now, by the normal rule, 99% of the observations lie within
2.58 standard deviations of the mean. Hence, the requisite range equals [50 - 2.58*10, 50 + 2.58*10]
= [24.2, 75.8].

Question: 815

The significance level of a test is usually equal to which of the following:

A. The power of a test.


B. The probability of a Type I error.
C. None of these answers is correct.
D. The probability of a Type II error.
E. More than one of these answers is correct.
F. (1 - the probability of a Type I error).

Answer: B

Page | 307
Explanation:
The standard method of hypothesis testing involves stating the significance level as equal to the
probability of incorrectly rejecting the null hypothesis. This action is defined as a Type I error. The
converse of a Type I error, in which the null hypothesis is incorrectly rejected, is a Type II error, in
which a false null hypothesis is mistakenly accepted.

Question: 816

The first card selected from a standard 52 card deck was a king. If it is returned to the deck, what is
the probability that a king will be drawn on the second selection?

A. 1/4 or 0.25
B. 1/13, or 0.077
C. None of these answers
D. 1/3 or 0.33
E. 12/13, or 0.923

Answer: B

Explanation:
There are still 4 kings out of 52 in the pack. So 4/52 = 1/13. The first selection does not affect the
second selection.

Question: 817

If your discount rate is 8% per year, calculate the present value of the following cash flows:

End of year 1: $2,200


End of year 2: $3,000
End of year 3: $7,300

A. $11,239
B. $14,155
C. $9,876
D. $10,404

Answer: D

Explanation:
The present value = 2,200/1.08 + 3,000/(1.08^2) + 7,300/(1.08^3) = 10,404

Question: 818

A firm's treasurer estimates that the firm will need about $15 million in 3 years' time to allow the
acquisition of a growing software firm. If the firm can invest in the capital market at the risk-free rate
of7% per year and wants to make sure it has the necessary funds available in 3 years, how much does
it need to invest today?

A. $12.244 million
B. $11.964 million
C. $12.113 million
D. $13.642 million

Answer: A

Page | 308
Explanation:
The amount needed today equals 15/(1.07^3) = $12.244 million

Question: 819

A tire manufacturer advertises that "one-half of our new all-season radial tire last at least 50,000
miles. An immediate adjustment will be made on any tire that does not last 50,000 miles." You
purchased four of these tires. What is the probability that all four tires will wear out before traveling
50,000 miles?

A. None of these answers


B. 1/10, or 0.10
C. 1/4, or 0.25
D. 1/16, or 0.0625
E. 1/64, or 0.0156

Answer: D

Explanation:
1/2*1/2*1/2*1/2 = 1/16 where 1/2 is the probability that a tire will wear our before 50,000 miles.

Question: 820

The U.S. Department of Education reported that for the past six years 23, 19, 15, 30, 27 and 25
women received doctorate degrees in computer and information sciences. What is the mean
arithmetic annual number of women receiving this degree?

A. 15.1
B. None of these answers
C. 22.9
D. 37.9
E. 23.2

Answer: E

Explanation:
(23 + 19 + 15 + 30 + 27 + 25)/6 = 23.2

Question: 821

If you have $4,000 in an account today and withdraw $2,000 in 3 years, how much can you withdraw
from the account in 5 years, if the account earns interest at 8% per year, compounded annually?

A. $3,544.51
B. $3,407.98
C. $3,000.00
D. $2,000.00
E. $7,714.93

Answer: A

Explanation:
The way to approach this problem is to find the PV today of the $2,000 amount, determine the

Page | 309
difference between this amount and the original $4,000 amount, and then move this remaining
amount to the 5 year point. On the BAII Plus, press 3 N, 8 I/Y, 2000 FV, 0 PMT, CPT PV, which yields
$1,587.66. Then press + 4000 =. Then press PV, 5 N, CPT FV to see the answer. On the HP12C, press 3
n, 8 i, 2000 FV, 0 PMT, CPT PV. Then press 4000 +. Then press PV, 5 n, CPT FV to see the answer. Make
sure the BAII Plus has the P/Y value set to 1.

Question: 822

If you buy a television for $650 and agree to pay for it with 24 payments of $30 (with the first
payment occurring next month), what is the size of the final payment needed at month 24 to
completely pay off the television? Assume the interest rate you are being charged is 16% per year,
compounded monthly.

A. $47.54
B. $0.00
C. $203.98
D. $30.00
E. $51.25

Answer: E

Explanation:
This question describes a situation often called a balloon payment. On the BAII Plus, press 24 N, 16
divide 12 = I/Y, 650 PV, 30 +/- PMT, CPT FV. On the HP12C, press 24 n, 16 ENTER 12 divide i, 650 PV,
30 CHS PMT, FV. The answer is shown as a negative number. Make sure the BAII Plus has the P/Y
value set to 1.

Question: 823

How many annual deposits of $1,500, beginning next year, would you need to make before you had
accumulated $30,000, if the money earns 9% per year, compounded annually? Assume the account
begins with a $0 balance.

A. 14.01
B. 20.00
C. 11.95
D. 25.51
E. 5.19

Answer: C

Explanation:
On the BAII Plus, press 9 I/Y, 0 PV, 1500 PMT, 30000 +/- FV, CPT N. On the HP12C, press 9 i, 0 PV, 1500
PMT, 30000 CHS FV, n. Note that the HP12C will display 12 as the answer.

Question: 824

The weekly mean income of a group of executives is $1,000 and the standard deviation of this group
is $100. The distribution is normal. What percent of the executive have an income of $925 or less?

A. About 23%
B. About 15%
C. About 85%
D. About 50%

Page | 310
E. None of these answers

Answer: A

Explanation:
z = (x-u)/sigma = 925 - 1000/100 = -0.75. From z-table z = 0.75 is 0.2734. So 1.0 - 0.7734 = 0.2266.

Question: 825

A stock's return has a mean of 6% and a coefficient of variation of 2. Its variance equals

A. 144%%
B. 3%%
C. 6%%
D. 12%%

Answer: A

Explanation:
Coefficient of variation = standard deviation/mean. Therefore, standard deviation = 2*6 = 12.
Variance = 12^2 = 144

Question: 826

The coefficient of determination measures:

A. The percentage change in the dependent variable caused by a 1% change in the independent
variable.
B. The degree of linear association between the dependent and the independent variables.
C. the slope of the regression line.
D. The amount of variance of the dependent variable explained by the independent variable.

Answer: D

Explanation:
The coefficient of determination, also known as the R-square of the regression, measures the
amount of variance of the dependent variable explained by the independent variable.

Question: 827

Which of the following statements regarding the coefficient of correlation is true?

A. It ranges from -1.0 to +1.0 inclusive


B. A value of 0.00 indicates two variables are not related
C. None of these answers
D. All of these answers
E. It measures the strength of the relationship between two variables

Answer: D

Explanation:
All of these answers are properties of the coefficient of correlation. Make sure you know the formula
for the coefficient of correlation.

Page | 311
Question: 828

What is the chart called when the paired date (the dependent and independent variables) are
plotted?

A. Linear regression
B. Pie
C. Bar
D. Dotted-Swiss
E. None of these answers

Answer: E

Explanation:
This chart is the scatter diagram.

Question: 829

What annual interest rate, compounded annually, is equivalent to 8% per year, compounded
semiannually?

A. 8.04%
B. 8.16%
C. 7.82%
D. 8.09%
E. 8%

Answer: B

Explanation:
Questions of this type are illustrating the concept of an Effective Interest Rate, which is a rate
compounded annually that has the same effect as a rate compounded more often than one time a
year. As such, a depositor or a creditor is indifferent between them, since they have the same effect.
To solve this question, make any deposit and see how much is in the account after one year. The ratio
of theending FV to the beginning PV will indicate the annual rate earned. On the BAII Plus, press 2 N,
8 divide 2 = I/Y, 100 PV, 0 PMT, CPT FV. On the HP12C, press 2 n, 8 ENTER 2 divide i, 100 PV, 0 PMT, FV.
The number displayed will be 108.16. In other words, after one year, $100 has become $108.16. An
interest rate of 8.16%, compounded annually, would cause a $100 deposit to become $108.16 in one
year. Choosing an initial deposit of $100 helps a great deal in these situations. Make sure the BAII
Plus has the value of P/Y set to 1.

Question: 830

An insurance agent has appointments with four prospective clients tomorrow. From past experience
the agent knows that the probability of making a sale on any appointment is 1 out of 5. Using the
rules of probability, what is the likelihood that the agent will sell a policy to 3 of the 4 prospective
clients?

A. 0.410
B. None of these answers
C. 0.250
D. 0.026
E. 0.500

Page | 312
Answer: D

Explanation:
This is a binomial probability. The probability of getting r successes out of n trials where the
probability of success each trial is p and probability of failure each trial is q (where q = 1-p) is given
by: n!(p^r)[q^(n-r)]/r!(n-r)!. Here n = 4, r = 3,p = 0.20 and q = 0.80. Therefore we have
4!(0.2^3)(0.8^1)/3!1! = 0.026.

Question: 831

An investor has a quarterly compounded required rate of return of 9% per year. How much will he
pay for a nine-year ordinary annuity that pays $100 per year?

A. $599.5
B. $574.3
C. $592.6
D. $594.6

Answer: C

Explanation:
The annually compounded rate equals (1 + 9%/4)^4 - 1 = 9.3%. Therefore, the present value of the
annuity equals 100/0.093*[1 - 1/1.093^9] = $592.6.

Question: 832

A relative frequency distribution is

A. all of these answers.


B. a collection of relative frequency classes with the relative frequencies.
C. a grouping of data into non-overlapping categories.
D. an aggregation of raw data in the form of a probability distribution.

Answer: A

Explanation:
A relative frequency distribution is an aggregation of raw data in the form of a probability
distribution, a collection of relative frequency classes with the relative frequencies, and a grouping of
data into nonoverlapping categories.

Question: 833

Suppose the probability that electricity prices will rise any given quarter is 0.60, and the probability
that oil prices will stay level or decline is 0.40. If electricity prices rise, corporate profits will decline
by 2% with 90% probability, and increase by 0.5% with 10% probability. If electricity prices decline or
stay level, corporate profits will increase 3% with a 65% probability and decline 0.5% with a 35%
probability. What is the expected change in corporate profits in the next quarter?

A. +0.34%.
B. -0.34%.
C. +0.43%.
D. -0.43%.

Answer: B

Page | 313
Explanation:
We need the total probability rule for expected value, for which the formula is E(X) = E(X | S_1) *
P(S_1) + E(X |S_2) * P(S_2) + ... + E(X |S_n) * P(S_n). Here, E(X) is the expected change in corporate
profits. S_1 is the event that electricity prices rise, and S_2 is the event that electricity prices fall.
Therefore, E(X) = 0.60 * (-2%*90% + 0.5%*10%) + 0.40 * (3%*65% - 0.5%*35%)= -0.34%, a decline.

Question: 834

What is the variable used to predict the value of another called?

A. Correlation
B. Determination
C. Dependent
D. None of these answers
E. Independent

Answer: E

Explanation:
The dependent variable is the variable Y which is being predicted by the X variable, the independent
variable. The regression is written as Y = a + bX. The letter "a" is the Y intercept and b is the slope of
the line.

Question: 835

A mortgage holding company has found that 3% of its mortgage holders default on their mortgage
and lose the property. Furthermore, 90% of those who default are late on at least two monthly
payments over the life of their mortgage as compared to 45% of those who do not default.
What is the probability that a mortgagee with two or more late monthly payments will default on the
mortgage and lose the property?

A. 0.039
B. 0.058
C. 0.436
D. None of these answers
E. 0.027

Answer: B

Explanation:
We have P(def)=0.03. P(not def) = 0.97. P(two late payments/def) = 0.90. P(two late payments/not
def) = 0.45. Using Bayes formula: p(def/two late payments) = (0.03*0.9)/(0.03*0.9 + 0.97*0.45) =
0.058.

Question: 836

Which of the following are true regarding correlation?


I. Correlation is a number between 0 and 1.
II. Correlation is a measure of the strength of the linear relationship between two variables.
III. The higher the correlation, the higher the covariance.

A. All but I.
B. All are true.

Page | 314
C. All but III.
D. All but II.

Answer: A

Explanation:
II is true. Because the correlation between two random variables X and Y is
Cov(X,Y)/[(sigma_X)*(sigma_Y)], III is true. I is false, because correlation is a number that can take on
values between -1 and +1, inclusive. A value of -1 indicates perfectly negative linear correlation.

Question: 837

You are examining a portfolio composed of 10% money-market investments, 30% bonds, and 60%
stocks. Last year, the return on the money-market investments was 4%; the return on bonds was 6%,
and the return on stocks was 7%. What is the portfolio weighted average return?

A. 6.50%.
B. 6.30%.
C. 6.40%.
D. None of these answers is correct.

Answer: C

Explanation:
The portfolio weighted-average mean return is equal to the sum (as i goes from 1 to n) of w_i * X_i,
where w_i is the percentage weight in the portfolio of the ith asset, and X_i is the investment return
of the ith asset. Here, we get a weighted mean of 0.10 * 0.04 + 0.30 * 0.06 + 0.60 * 0.07 = 6.40%.

Question: 838

If you owe $4,500 today and will repay it with 8 annual payments of $900, beginning next year, what
is the interest rate per year, compounded annually, that you are paying?

A. 10.41%
B. 19.80%
C. 11.81%
D. 4.4%
E. 14.4%

Answer: C

Explanation:
On the BAII Plus, press 8 N, 4500 PV, 900 +/- PMT, 0 FV, CPT I/Y. On the HP12C, press 8 n, 4500 PV,
900 CHS PMT, 0 FV, i. Note that the PMT must be entered with a sign that is opposite that given the
PV. Make sure the BAII Plus has the P/Y value set to 1.

Question: 839

How much will you have in an account two years from now if you start with $100 in it today and add
$300 to it one year from now? Assume that interest is 6% per year, compounded annually.

A. $430.36
B. $424.00
C. $449.44

Page | 315
D. $372.02
E. $400.00

Answer: A

Explanation:
Solve this question by working 2 compound interest problems. On the BAII Plus, press 2 N, 6 I/Y, 100
PV, 0 PMT, CPT FV, which yields $112.36. Then press STO 1. Then press 1 N, 300 PV, CPT FV, which
yields $318.00. To find the answer, press + RCL 1 =. On the HP12C, press 2 n, 6 i, 100 PV, 0 PMT, FV.
Then press STO 1. Then press 1 n, 300 PV, FV. Then press RCL 1 + to show the answer.

Question: 840

A sample of single persons receiving social security payments revealed these monthly benefits: $826,
$699, $1,087, $880, $839 and $965. How many observations are below the median?

A. 1
B. 3
C. 2
D. 0
E. None of these answers

Answer: B

Explanation:
Order the sample: 699, 826, 839, 880, 965, 1087. The median is somewhere between 839 and 880.
So there are three observations below the median.

Question: 841

What annual interest rate, compounded annually, is equivalent to 5% per year, compounded
quarterly?

A. 5.09%
B. 4.97%
C. 5.25%
D. 5%
E. 5.01%

Answer: A

Explanation:
Questions of this type are illustrating the concept of an Effective Interest Rate, which is a rate
compounded annually that has the same effect as a rate compounded more often than one time a
year. As such, a depositor or a creditor is indifferent between them, since they have the same effect.
To solve this question, make any deposit and see how much is in the account after one year. The ratio
of the ending FV to the beginning PV will indicate the annual rate earned. On the BAII Plus, press 4 N,
5 divide 4 = I/Y, 100 PV, 0 PMT, CPT FV. On the HP12C, press 4 n, 5 ENTER 4 divide i, 100 PV, 0 PMT, FV.
The number displayed will be 105.09. In other words, after one year, $100 has become $105.09. An
interest rate of 5.09%, compounded annually, would cause a $100 deposit to become $105.09 in one
year. Choosing an initial deposit of $100 helps a great deal in these situations. Make sure the BAII
Plus has the value of P/Y set to 1.

Question: 842

Page | 316
If you deposit $204.50 into an account paying 3.15% per year simple interest, how much interest will
be earned after 3 years?

A. $$19.59
B. $19.33
C. $24.59
D. $16.67
E. $18.03

Answer: B

Explanation:
The question only asks for the earned interest, not the total amount in the account. On the BAII Plus,
press 204.50 x 0.0315 x 3 = to see the answer. On the HP12C, press 204.50 ENTER 0.0315 x 3 x to see
the answer.

Question: 843

In a management trainee program, 80 percent of the trainees are female, 20 percent male. Ninety
percent of the females attended college, 78 percent of the males attended college. A management
trainee is selected at random. What is the probability that the person selected is a female who did
not attend college?

A. 0.8
B. 0.25
C. 0.20
D. 0.08
E. None of these answers

Answer: D

Explanation:
Prob. of choosing a female=0.8. Prob. of a female not attending college = 0.1. So 0.8*0.1 = 0.08.

Question: 844

Melissa Hart, a quantitative analyst with the Kansas City Federal Reserve, has been involved with
accumulating data for an essay regarding economic conditions in her area. Specifically, Melissa has
been gathering data related to the levels of consumer debt in the state of Missouri. The following is a
description of the data:
Sample mean: $3,451.00
Standard deviation of the sample: $819
Number of observations: 441
Assume that Ms. Hart formulates a hypothesis test in which the null hypothesis specifies that the
population mean is equal to $4,600. Additionally, assume that the population standard deviation is
unknown. Given this information, what is the standard error of the estimate? Further, what is the
test statistic? Choose the best answer.

A. The answer cannot be determined from the information provided.


B. $39.09; 1.17
C. $1.86; (617.74)
D. $39.00; 0.52
E. $39.00; (29.46)

Page | 317
F. $1.86; 862.00
G. $39.09; (29.39)

Answer: E

Explanation:
If the population standard deviation is unknown, as in this example, the standard error of the
estimate is found by using the following equation:
{Standard error = s / square root of n} where s = the sample standard deviation and n = the number
of observations in the sample.
In this example, all of the necessary information has been provided, and the determination of the
standard error of the estimate is found as:
{Standard error = [$819 / 21] = $39}
Now that the standard error of the estimate has been calculated, the test statistic can be found by
using the following equation: {Test statistic = [sample statistic - value of the population parameter
under the null hypothesis] / standard error of the sample statistic]. Again, all of the necessary
information has been provided, and the calculation of the test statistic is found as follows:
{Test statistic = [$3,451 - $4,600] / $39 = (29.46)}
This is a rather large test statistic, and will likely result in the rejection of the null hypothesis unless a
very low level of confidence is employed.

Question: 845

How many years would it take for a deposit of $10,000 to become $200,000, if interest of 9% per year
is compounded annually?

A. 400.93
B. 28.89
C. 134.64
D. 34.76
E. 8.04

Answer: D

Explanation:
On the BAII Plus, press 10000 PV, 0 PMT, 200000 +/- FV, 9 I/Y, CPT N. On the HP12C, press 10000 PV,
200000 CHS FV, 9 i, then press n. Note that the HP12C shows 35 as the answer.

Question: 846

What is the Net Present Value of this series of annual cash flows at an interest rate of 12% per year:
Year 0: <$30,000>, Year 1: $5,000, Year 2: $8,000, Year 3: $11,000, Year 4 <> are used to indicate a
negative number).

A. $951.21
B. <$1,795.81>
C. $1,104.37
D. <$1,077.49>
E. <$1,579.31>

Answer: B

Explanation:
On the BAII Plus, press CF 2nd CLRWork 30000 +/- ENTER DownArrow 5000 ENTER DownArrow

Page | 318
DownArrow 8000 ENTER DownArrow DownArrow 11000 ENTER DownArrow DownArrow 15000
ENTER DownArrow DownArrow 2nd Quit. Then press NPV 12 ENTER DownArrow CPT. On the HP12C,
press these keys: 30000 CHS BlueShift CFo 5000 BlueShift CFj 8000 BlueShift CFj 11000 BlueShift CFj
15000 BlueShift CFj. Then press 12 i, YellowShift NPV. The "DownArrow" represents
the downward-pointing arrow on the top row of the BAII Plus keyboard. Make sure the BAII Plus has
the P/Y value set to 1.

Question: 847

What is a listing of all possible outcomes of an experiment and their corresponding probability of
occurrence called?

A. Random variable
B. Frequency distribution
C. Subjective probability
D. None of these answers
E. Probability distribution

Answer: E

Explanation:
The probability distribution shows the outcomes with corresponding probabilities.

Question: 848

A cumulative frequency distribution on days absent during a calendar year by employees of a


manufacturing company is shown below.

Days AbsentCumulative Number of Employees


0 - 260
3 - 531
6 - 814
9 - 116
12 - 142

How many employees were absent between 3 and 5 days?

A. 29
B. 17
C. 14
D. 31
E. 2

Answer: B

Explanation:
To find the number of people absent between 3-5 days, we need to get the difference in the
cumulative number of employees between the 3-5 days group and the group after it, the 6-8 group.
So we have 31 - 14 = 17.

Question: 849

What annual interest rate, compounded quarterly, would cause a series of 20 quarterly deposits of
$800 to accumulate to $25,000, if the first deposit is made three months from today?

Page | 319
A. 12.31%
B. 19.94%
C. 19.78%
D. 17.85%
E. 18.15%

Answer: D

Explanation:
The value returned by the calculator will be the periodic interest rate which must be multiplied by
the number of periods per year to have the correct answer. On the BAII Plus, press 20 N, 0 PV, 800
PMT, 25000 +/- FV, CPT I/Y. Then press x 4 = to see the answer. On the HP12C, press 20 n, 0 PV, 800
PMT, 25000 CHS FV, i. Then press 4 x to see the answer. Make sure the BAII Plus has the P/Y value set
to 1.

Question: 850

Bert wants to retire a millionaire. If he is 40 years old today and already has $40,000 in the bank,
what monthly deposit would he need to make beginning one month from today and continuing until
he retires on his 65th birthday, if the money will earn 8% per year, compounded monthly?

A. $703.26
B. $739.04
C. $738.95
D. $770.98
E. $742.77

Answer: E

Explanation:
The total number of deposits will be 300 (25 years x 12 months/year). On the BAII Plus, press 300 N,
8 divide 12 = I/Y, 40000 PV, 1000000 +/- FV, CPT PMT. On the HP12C, press 300 n, 8 ENTER 12 divide i,
40000 PV, 1000000 CHS FV, PMT. Note that the answer will be displayed as a negative number. Make
sure the BAII Plus has the P/Y value set to 1.

Question: 851

What annual interest rate, compounded annually, is equivalent to 9% per year, compounded
quarterly?

A. 9%
B. 9.25%
C. 9.01%
D. 9.30%
E. 8.95%

Answer: D

Explanation:
Questions of this type are illustrating the concept of an Effective Interest Rate, which is a rate
compounded annually that has the same effect as a rate compounded more often than one time a
year. As such, a depositor or a creditor is indifferent between them, since they have the same effect.
To solve this question, make any deposit and see how much is in the account after one year. The ratio

Page | 320
of the ending FV to the beginning PV will indicate the annual rate earned. On the BAII Plus, press 4 N,
9 divide 4 = I/Y, 100 PV, 0 PMT, CPT FV. On the HP12C, press 4 n, 9 ENTER 4 divide i, 100 PV, 0 PMT, FV.
The number displayed will be 109.30. In other words, after one year, $100 has become $109.30. An
interest rate of 9.30%, compounded annually, would cause a $100 deposit to become $109.30 in one
year. Choosing an initial deposit of $100 helps a great deal in these situations. Make sure the BAII
Plus has the value of P/Y set to 1.

Question: 852

Suppose you have two assets, A and B. A has returns of 10%, 2%, and 15%, with probabilities 40%,
50%, and 10% respectively. B has returns of 5%, 10%, and -20%, with probabilities 40%, 50%, and
10% respectively. What is the return covariance between assets A and B?

A. -50.23%%
B. -60.65%%
C. -10.78%%.
D. -32.45%%

Answer: D

Explanation:
First we must find the expected returns for A and B. These are 6.50% and 5.00%, respectively.
Second, we find the difference between each observation and the average: (10% - 6.5%), (2% - 6.5%),
and (15% - 6.5%) for A, and (5% - 5%), (10% - 5%), and (-20% - 5%) for B. Next, we take the probability
weighted cross product of these differences and sum them. So we get 3.5% * 0% * 40% + (-4.5%) *
5% * 50% + 8.5% * (-25%) * 10% = -32.45%%.

Question: 853

The probability that GNP will expand in the next 12 months is called:

A. an unconditional probability.
B. a conditional probability.
C. a joint probability.
D. a guess.

Answer: A

Explanation:
An unconditional probability takes the form of P(A), the probability that an event (A) will happen. It is
unconditional because it is not conditioned on any other event.

Question: 854

"Sampling error" refers to:

A. an incorrect selection of a sampling method.


B. the standard error of the normal deviates of a sample drawn from a population.
C. the difference between a sample statistic and the corresponding population parameter.
D. an incorrect usage of a sampling method.

Answer: C

Explanation:

Page | 321
Sampling error is the difference between the population's actual parameter and the parameter
estimate that results from analysis of the sample.

Question: 855

A perpetuity pays $1,000 a year. You have a discount rate of 10% per year. If you are indifferent
between buying this perpetuity and a 7-year annuity, how much does the annuity pay per year?

A. $2,054
B. $3,199
C. $1,867
D. $1,429

Answer: A

Explanation:
The value of the perpetuity is 1,000/0.1 = $10,000. If the annuity pays $C per year, then its value is
C/0.1*[1 - 1/(1.1)^7] = 4.868C. For you to be indifferent between these two, the two values must be
equal, giving C = 10,000/4.868 = $2,054.

Question: 856

What annual deposit would you need to make, beginning one year from today, into an account
paying 11% per year, compounded annually, in order to have $1,000,000 in 30 years?

A. $6,129.83
B. $33,333.33
C. $418.67
D. $5,024.60
E. $1,374.48

Answer: D

Explanation:
On the BAII Plus, press 30 N, 11 I/Y, 0 PV, 1000000 FV, CPT PMT. On the HP12C, press 30 n, 11 i, 0 PV,
1000000 FV, PMT. Note that the answer will be displayed as a negative number. Make sure the BAII
Plus has the P/Y value set to 1.

Question: 857

The probability of getting a value less than 1.3 from a standard normal distribution equals ________.

A. 0.4032
B. 0.6847
C. 0.9032
D. 0.8192

Answer: C

Explanation:
The z-value of a selected observation, X, from a normal distribution with mean M and standard
deviation S equals z = (X-M)/S. For a standard normal distribution, M = 0 and S = 1. Therefore, for X =
1.3, z = 1.3. Using the Normal probability tables, we get P(Y < 1.3) = 0.9032.

Page | 322
Question: 858

What is the present value of $50 per year for 10 years, with the first cash flow occurring today, rather
than 1 year from now? Assume interest is 6% per year, compounded annually.

A. $368.00
B. $402.55
C. $390.08
D. $357.61
E. $398.00

Answer: C

Explanation:
Recognize that this question is an annuity due situation, since the first cash flow occurs immediately
or at the beginning of each period. Annuities where the first payment occurs 1 period from today (or
at the end of each period) are called "ordinary" annuities. This requires placing the calculator into
"Begin" mode prior to solving the question. NOTE: Be sure to place the calculator OUT OF annuity
due mode after this question before going on to subsequent questions, or you will get the wrong
answers! On the BAII Plus, press 2nd BGN. If the display shows END, then press 2nd SET and then 2nd
Quit. This will place the BAII Plus into annuity due mode (you can tell this because the BAII Plus will
display BGN in small letters). Now press 10 N, 6 I/Y, 50 PMT, 0 FV, CPT PV. Place the calculator back
into End mode (for ordinary annuities) by pressing 2nd BGN and then if the calculator is displaying
BGN, press 2nd SET and 2nd Quit. The BGN letters should disappear from the display. On the HP12C,
press BlueShift END, which is the blue function on the front of the 8 digit key. This places the HP12C
into Begin mode (the HP12C shows the word BEGIN in the display when in this mode). Then press 10
n, 6 i, 50 PMT, 0 FV, PV. To place the HP12C back into ordinary annuity mode (or END mode), press
BlueShift BEG (the blue function written on the front of the 7 digit key). Note that the answer is
displayed as a negative number. Make sure the BAII Plus has the value of P/Y set to 1.

Question: 859

The Investment Company Institute reported in its Mutual Fund Fact Book that the number of mutual
funds increased from 410 in 1985 to 857 in 1995. What is the geometric mean annual percent
increase in the number of funds?

A. 48.66
B. None of these answers
C. 1.12
D. 19.41
E. 7.65

Answer: E

Explanation:
There are 11 years involved. The geometric mean = [(1 + 857/410)^1/10]-1. In words, it is the 10th
square root of (1 + 857/410) minus 1. So we have GM = (1 + 2.09)^(1/10) - 1 = 0.0765 = 7.65%

Question: 860

You are faced with a counting problem in which you must choose k objects from n total objects. The
order of choosing matters. The counting method you should use is:

A. The binomial formula.

Page | 323
B. The multinomial formula.
C. None of these answers is correct.
D. The permutation rule.

Answer: D

Explanation:
We need the general permutation formula, which gives the number of ways that k objects can be
chosen from n items, with regard to order. The formula is n_P_k = n! / (n-k)!

Question: 861

How much would an original deposit of $2,500 grow to be after 6 and a half years, if the deposit
earns interest at 8.5% per year, compounded monthly?

A. $4,335.53
B. $5,541.02
C. $3,601.34
D. $10,183.49
E. $3,570.42

Answer: A

Explanation:
On the BAII Plus, press 2500 PV, 0 PMT, 78 N, 8.5 divided by 12 = I/Y, then press CPT FV. On the
HP12C, press 2500 PV, 0 PMT, 78 n, 8.5 ENTER 12 divide i, then press FV. The answer will be displayed
as a negative number. Make sure that the BAII Plus has the P/Y value set to 1.

Question: 862

If you buy an item and pay for it with 60 monthly payments of $75 that begin next month, what was
the cash price today, if interest accrues at 11% per year, compounded monthly?

A. $4,598.42
B. $3,544.71
C. $2,545.29
D. $3,904.12
E. $3,449.48

Answer: E

Explanation:
The cash price would be the present value of the payments made using the specified interest rate.
On the BAII Plus, press 60 N, 11 divide 12 = I/Y, 75 PMT, 0 FV, CPT PV. On the HP12C, press 60 n, 11
ENTER 12 divide i, 75 PMT, 0 FV, PV. Note that the answer will be shown as a negative number. Make
sure the BAII Plus has the P/Y value set to 1.

Question: 863

Suppose that stocks A, B, C, and D are independent with respect to their price movement, and have
probabilities of increasing of 0.25, 0.50, 0.40, and 0.30. What is the probability that stocks A and C
will increase in price, while stocks B and D fail to increase?

A. 11%.

Page | 324
B. 2.5%.
C. 3.5%.
D. 10%.

Answer: C

Explanation:
If events are independent, then the joint probability of them occurring together is just the product of
the individual probabilities. So P(AC) = 0.25 * 0.40 = 10%. To this, we multiply the probabilities of B
and D failing to increase: 10% * (1 - 0.50)*(1 - 0.30) = 3.5%.

Question: 864

In the equation Y' - a + bX, what is Y'?

A. Slop of the line


B. Value of Y when X=0
C. None of these answers
D. Predicted value of Y, given a specific X value
E. Y intercept

Answer: D

Explanation:
The regression is written as Y' = a + bX. The letter "a" is the Y intercept and b is the slope of the line.
Y' is the predicted value of Y given a specific value of X.

Question: 865

What deposit would you need to make today in order to withdraw $100 a month for the next 5 years,
beginning next month, if the deposit will accrue interest at 8% per year, compounded monthly?

A. $5,141.23
B. $492.59
C. $1,237.66
D. $399.27
E. $4,931.84

Answer: E

Explanation:
On the BAII Plus, press 60 N, 8 divide 12 = I/Y, 100 PMT, 0 FV, CPT PV. On the HP12C, press 60 n, 8
ENTER 12 divide i, 100 PMT, 0 FV, PV. The answer will be displayed as a negative number. Make sure
the BAII Plus has the value of P/Y set to 1.

Question: 866

If the 1% level of significance is used and the computed value of z is +6.00, what is our decision?

A. Do not reject Ho
B. None of these answers
C. Reject Ho
D. This cannot be answered

Page | 325
Answer: C

Explanation:
This falls beyond the critical value of 1.645. Therefore it falls in the rejection region.

Question: 867

A stock's return is normally distributed with a mean of 16% and a standard deviation of 8%. The
probability that an investment in the stock will result in a loss in one year is approximately:

A. 5%
B. 2.5%
C. cannot be calculated
D. 10%

Answer: B

Explanation:
Note that the mean is 2 standard deviations above zero. The probability that the stock's return will lie
outside the 2-sigma range around the mean equals 5% for a standard normal distribution. This
implies that the stock return will be either less than 0% or more than 32% with a 5% probability.
Since the normal distribution is symmetrical about the mean, the probability that the stock return
will be less than zero equals 5/2 = 2.5%. Instead of the above, you could also solve the problem using
the z-score and the normal probability distribution table. You should, however, be aware of short-
cuts like the above.

Question: 868

Which of the following is not a characteristic of a binomial probability distribution?

A. Probability of success remains constant from trial to trial


B. Each outcome results from two trials
C. Each trial is independent
D. All of these answers are characteristics of the binomial distribution?
E. Each outcome is mutually exclusive

Answer: B

Explanation:
The binomial distribution does not state that outcomes have to come from a certain number of trials.

Question: 869

The probability that a technology stock will fall in price over the next 12 months, given a decline in
consumer confidence over the same period, is called:

A. an unconditional probability.
B. a conditional probability.
C. a joint probability.
D. a likelihood.

Answer: B

Explanation:

Page | 326
A conditional probability takes the form of P(A|B), the probability that a an event (A) will happen,
given the occurrence of another event, B. It is conditional because it is conditioned on another event,
B.

Question: 870

The sampling method commonly employed when the population is scattered over a large geographic
area is known as:

A. systematic random sampling.


B. simple random sampling.
C. stratified random sampling.
D. cluster sampling.

Answer: D

Explanation:
Cluster sampling is a modified form of stratified random sampling used to reduce sampling costs
when the population is scattered over a large geographic area.

Question: 871

Giorgio offers the person who purchases an 8 ounce bottle of Allure two free gifts, either an
umbrella, a 1 ounce bottle of Midnight, a feminine shaving kit, a raincoat or a pair of rain boots. If
you purchased Allure what is the probability you selected at random an umbrella and a shaving kit in
that order?

A. None of these answers


B. 0.00
C. 0.20
D. 0.05
E. 1.00

Answer: D

Explanation:
There are 5 types of gifts. The probability of choosing an umbrella first is 1/5. Then the probability of
choosing a shaving kit from the remaining four gifts is 1/4. So 1/5*1/4 = 1/20 = 0.05.

Question: 872

The quarterly compounded rate is 8% quoted on an annualized basis. The equivalent annually
compounded rate is:

A. 8.00%
B. 8.24%
C. 8.16%
D. 7.95%

Answer: B

Explanation:
To solve such problems, think about investing a dollar for a year. The final amount should be the
same under both the quotations. Under annually compounded rate, r, $1 grows to 1+r in 1 year.

Page | 327
Under quarterly compounding, it grows to (1+0.08/4)^4 = 1.0824. Since these two should be equal,
we get 1+r = 1.0824, giving r = 8.24%. Note that the annually compounded rate must be larger than
the quarterly rate, ruling out (d) automatically.

Question: 873

The least squared regression minimizes the:

A. adjusted R-square.
B. absolute value of the distance between the observed points and the regression line.
C. explained variance.
D. the square of the distance between the observed points and the regression line.

Answer: D

Explanation:
The least squared regression minimizes the square of the distance between the observed points and
the regression line.

Question: 874

You are faced with a counting problem in which the number of outcomes is infinite. The counting
method you should use is:

A. The multinomial formula.


B. The multiplication rule.
C. The binomial formula.
D. None of these answers is correct.

Answer: D

Explanation:
If the number of outcomes is infinite, a counting method should not be used at all, and the question
cannot be answered with counting methods. None of the above is the correct response.

Question: 875

A joint probability is expressed symbolically as:

A. P(A | B).
B. P(B | AB).
C. P(AB).
D. P(A or B).

Answer: C

Explanation:
A joint probability takes the form of P(AB), the probability that a an event A and an event B will both
happen.

Question: 876

A bar owner in Wisconsin has just won the Illinois Big Lotto jackpot that pays $200 million dollars in
25 equal installments paid annually. The bar owner can either accept the annual payments starting

Page | 328
today or receive a lump sum today, using a discount rate of 9% per year. If the bar owner chooses to
accept the lump sum, how much will he receive today?

A. $85.65 million
B. $91.33 million
C. $101.31 million
D. $105.13 million

Answer: A

Explanation:
The annual lotto payments equal 200/25 = 8 million dollars per year. Since the payments start today,
the cash stream equals a payment of 8 million dollars today and an ordinary annuity with 24
payments. Thus, the present value of the lotto equals 8 + 8/0.09*(1-1/(1.09^24)) = 85.65 million
dollars.

Question: 877

A sample of size 600 is drawn from a population. The sample mean equals 329. The total width of the
99% confidence interval for the population mean is 893. The estimated population variance equals
________.

A. 6.7
B. 4.9
C. 6.3
D. 7.3

Answer: C

Explanation:
If z is the z-value corresponding to the specified confidence level, the sample mean is M and the
standard deviation is D in a sample size N, the confidence interval is specified as [M - z*D/sqrt(N), M
+ z*D/sqrt(N)]. Thus, the width of the interval equals 2*z*D/sqrt(N). In the present case, for the 99%
confidence interval, the normal probability table gives z = 2.58. The sample standard deviation
equals sqrt(893) = 29.88. Therefore, the width of the 99% interval is 2*2.58*29.88/sqrt(600) = 6.30.

Question: 878

The monthly compounded rate is 15% quoted on an annualized basis. The equivalent quarterly
compounded rate is:

A. 15.19%
B. 15.05%
C. 15.31%
D. 14.8%

Answer: A

Explanation:
To solve such problems, think about investing a dollar for 1 year. The final amount should be the
same under both the quotations. Under quarterly compounded rate, r, $1 grows to (1+r/4)^4 in 1
year. Under monthly compounding, it grows to (1+0.15/12)^12 = 1.1608. Since these two should be
equal, we get (1+r/4)^4 = 1.1608, giving r = 15.19%. Note that the quarterly compounded rate must
be larger than the monthly compounded rate, ruling out 14.8% automatically.

Page | 329
Question: 879

An accelerated life test on a large number of type-D alkaline batteries revealed that the mean life for
a particular use before they failed is 19.0 hours. The distribution of the lives approximated a normal
distribution. The standard deviation of the distribution was 1.2 hours. About 95.44 percent of the
batteries failed between what two values?

A. 16.6 and 21.4


B. 14.1 and 22.1
C. 12.2 and 14.2
D. None of these answers
E. 8.9 and 18.9

Answer: A

Explanation:
9544/2 = 0.4772. For an area under the curve of 0.4772, the z value is (+/-)2. We know that z = (x-
u)/sigma. Therefore, +/-2 = (x-19)/1.2. Rearranging we get x = u +/- z*sigma. So x = 19 +/- 2*1.2 =
19+/-2.4. x is 16.6 and 21.4.

Question: 880

If the p-value is ________ the significance level, the null hypothesis ________.

A. less than; can be rejected


B. none of these answers
C. more than; can be rejected
D. less than; cannot be rejected

Answer: A

Explanation:
The lowest significance level at which the null hypothesis can be rejected is called the p-value of the
test. Thus, if the p-value is less than the significance level, the null hypothesis can be rejected at that
significance level. To fix this in your mind, think about the p-value as the maximum probability that
the null hypothesis is true despite observing the value of the test statistic that you have in the sample
at hand. For e.g., suppose the p-value for a given sample turns out to be 3%. Then, if you reject the
null, the probability that you have made an error in rejecting it is at most 3%. Clearly, if a critic asked
you, "Is the probability that you have made an error less than 5%?" your answer would be, "yes." The
critic will then conclude that your test is significant at the 5% level and perhaps even at a lower level.

Question: 881

At the end of every year for the next 3 years, you deposit $400 in an account that pays 5% per year,
annually compounded. After that, you do not make any more deposits. The amount that you can
withdraw after 7 years is:

A. $1,533
B. $1,492
C. $1,917
D. $1,261

Answer: A

Page | 330
Explanation:
For the first 3 years, your regular deposits constitute an annuity and after that, a simple term deposit.
The future value of an n-period annuity that starts paying at the end of the current period equals FV =
(C/r)*[(1+r)^n - 1] where C is the payment per period and r is the one-period interest rate. In this
example, the annuity is over 3 periods, C = 400 and the per period rate equals 5%. So the future
value of the deposits after 3 years equals (400/0.05)*[1.05^3 - 1] = 1,261. Starting in year 3, this
amount grows at an annual rate of 5% for a period of 7-3 = 4 years. So in 7 years, the amount in the
account equals 1,261*1.05^4 = $1,533.

Question: 882

The coefficient of variation of X is three times that of Y. If X and Y have the same means, the variance
of Y is:

A. none of these answers.


B. same as that of X.
C. three times that of X.
D. one-third that of X.

Answer: A

Explanation:
The coefficient of variation equals the ratio of the standard deviation to the mean. Hence, if X and Y
have the same means, then the standard deviation of X must be 3 times the standard deviation of Y.

Question: 883

A variable, Z, is regressed against variables X and Y and the following equation is estimated: Z = -3.0 +
1.2X - 2.5Y + error. If X = 5 and Y = 3, the estimated value of Z is ________.

A. -3.0
B. cannot be estimated since the error term is unknown
C. -1.5
D. -4.5

Answer: D

Explanation:
With the given data, Z = -3 + 1.2*5 - 2.5*3 = -4.5

Question: 884

If the alternate hypothesis specifies a particular direction for the population parameter, it is
appropriate to use:

A. a z-statistic test.
B. a stratified sampling test.
C. a one-tailed test.
D. a two tailed test.

Answer: C

Explanation:

Page | 331
A one-tailed test is used when the alternate hypothesis specifies a particular direction for the
population parameter.

Question: 885

If you buy a house costing $120,000 and pay for it over 30 years, what is your monthly payment, if
your loan's interest rate is 7% per year, compounded monthly and the first payment is due next
month?

A. $4,371.83
B. $8,400.00
C. $798.36
D. $665.30
E. $804.39

Answer: C

Explanation:
On the BAII Plus, press 360 N, 7 divide 12 = I/Y, 120000 PV, 0 FV, CPT PMT. On the HP12C, press 360 n,
7 ENTER 12 divide i, 120000 PV, 0 FV, PMT. Note that the answer will be displayed as a negative
number. Make sure the BAII Plus has the value of P/Y set to 1.

Question: 886

Suppose a tire manufacturer wants to set a mileage guarantee on its new XB 70 tire. Life test
revealed that the mean mileage is 47,900 and the standard deviation of the normally distributed
distribution of mileage is 2,050 miles. The manufacturer wants to set the guaranteed mileage so that
no more than 5 percent of the tires will have to be replaced. What guaranteed mileage should the
manufacturer announce?

A. 32,960
B. 44,528
C. 49,621
D. 40,922
E. None of these answers

Answer: B

Explanation:
Find the z-value for an area under the curve of 1.0 - 0.55 = 0.45. For an area of 0.45, z = -1.645. Using
z = (x-u)/sigma. -1.645 = x - 47900/2050. x = 44528.

Question: 887

If you have $5,000 in an account today and withdraw $2,000 in 3 years, how much can you withdraw
from the account in 5 years, if the account earns interest at 8% per year, compounded annually?

A. $3,000.00
B. $5,013.84
C. $4,407.98
D. $5,714.93
E. $2,000.00

Answer: B

Page | 332
Explanation:
The way to approach this problem is to find the PV today of the $2,000 amount, determine the
difference between this amount and the original $5,000 amount, and then move this remaining
amount to the 5 year point. On the BAII Plus, press 3 N, 8 I/Y, 2000 FV, 0 PMT, CPT PV. Then press +
5000 =. Then press PV, 5 N, CPT FV to see the answer. On the HP12C, press 3 n, 8 i, 2000 FV, 0 PMT,
CPT PV. Then press 5000 +. Then press PV, 5 n, FV to see the answer. Make sure the BAII Plus has the
P/Y value set to 1.

Question: 888

The monthly compounded rate is 4% quoted on an annualized basis. The equivalent semiannually
compounded rate is:

A. 4.25%
B. 4.03%
C. 4.12%
D. 3.96%

Answer: B

Explanation:
To solve such problems, think about investing a dollar for 1 year. The final amount should be the
same under both the quotations. Under semiannually compounded rate, r, $1 grows to (1+r/2)^2 in 1
year. Under monthly compounding, it grows to (1+0.04/12)^12 = 1.0407. Since these two should be
equal, we get (1+r/2)^2 = 1.0407, giving r = 4.03%. Note that the semi-annually compounded rate
must be larger than the monthly compounded rate, ruling out 3.96 automatically.

Question: 889

You are examining a portfolio composed of 33% money-market investments, 9.5% bonds, and 57.5%
stocks. Last year, the return on the money-market investments was 4%; the return on bonds was 9%,
and the return on stocks was -11%. What is the portfolio weighted average return?

A. -4.05%.
B. -4.15%.
C. None of these answers is correct.
D. -3.90%.

Answer: B

Explanation:
The portfolio weighted-average mean return is equal to the sum (as i goes from 1 to n) of w_i * X_i,
where w_i is the percentage weight in the portfolio of the ith asset, and X_i is the investment return
of the ith asset. Here, we get a weighted mean of 0.33 * 0.04 + 0.095 * 0.09 + 0.575 * -0.11 = -4.15%.

Question: 890

You are examining the return on equity ratios of the nation's publicly owned companies. You wish to
calculate a typical deviation from the average ROE, but you do not have time to gather data on all the
firms. What measure should you use?

A. Population variance.
B. Sample variance.

Page | 333
C. Population standard deviation.
D. Sample standard deviation.

Answer: D

Explanation:
A typical deviation is going to be a standard deviation, not a variance. When using a sample, instead
of the entire population, you have sample standard deviation, not population standard deviation.

Question: 891

Three banks have quoted interest rates as follows:

Bank A: 10% per year, compounded quarterly.


Bank B: 11% per year, compounded annually.
Bank C: 10.5% per year, compounded semi-annually.

Which bank should you choose to invest with for a period of one year and what's the effective annual
rate?

A. Bank B, 11%
B. Bank A, 10.38%
C. Bank C, 12.01%
D. Bank A, 11.19%

Answer: A

Explanation:
The annual yield for Bank A is (1+10%/4)^4 - 1 = 10.38%, that for Bank B is 11% and that for Bank C is
(1+10.5%/2)^2 - 1 = 10.78%. Therefore, you should invest with Bank B.

Question: 892

Which is true for a binomial distribution?

A. All of these answers are correct


B. Value of p is equal to 1.50
C. Probability of success remains the same from trial to trial
D. There are three or more possible outcomes
E. None of these answers are correct

Answer: C

Explanation:
The probability of success is a constant in a binomial distribution.

Question: 893

What annual interest rate, compounded annually, will cause an original deposit of $400 to grow to
$625, after 7 years?

A. 8.57%
B. 6.58%
C. 7.27%

Page | 334
D. 6.14%
E. 5.78%

Answer: B

Explanation:
On the BAII Plus, press 7 N, 400 PV, 0 PMT, 625 +/- FV, then CPT I/Y. On the HP12C, press 7 n, 400 PV,
0 PMT, 625 CHS FV, then press i. Make sure the BAII Plus has the P/Y value set to 1.

Question: 894

Given the following points: (-2, 0), (-1,0), (0,1), (1, 1) and (2, 3)
What is the critical value necessary to determine a confidence interval for a 95% level of confidence?

A. 2.776
B. None of these answers
C. 3.182
D. 2.132
E. 2.353

Answer: C

Explanation:
Look at the t tables for n-2 degrees of freedom at the 5% level. Here, we look for a two tailed test
with 5-2 = 3 degrees of freedom. This is 3.182.

Question: 895

The net incomes of a sample of importers were organized into the following table:

Net Income (in $ millions)Number of Importers


2 - 41
5 - 74
8 - 1010
11 - 133
14 - 162

What is the mean net income?

A. None of these answers


B. $7.65 million
C. $14.2 million
D. $9.15 million
E. $9.00 million

Answer: D

Explanation:
(3*1)+(6*4)+(9*10)+(12*3)+(15*2) = 183. Mean is 183/20 = 9.15

Question: 896

Tom wants to retire a millionaire. If he is 30 years old today, what monthly payment would he need
to make beginning one month from today and continuing until he retires on his 65th birthday, if the

Page | 335
money will earn 8% per year, compounded monthly?

A. $670.98
B. $435.94
C. $2,380.95
D. $5,803.26
E. $439.04

Answer: B

Explanation:
The total number of deposits will be 420 (35 years x 12 months/year). On the BAII Plus, press 420 N,
8 divide 12 = I/Y, 0 PV, 1000000 FV, CPT PMT. On the HP12C, press 420 n, 8 ENTER 12 divide i, 0 PV,
1000000 FV, PMT. Note that the answer will be displayed as a negative number. Make sure the BAII
Plus has the P/Y value set to 1.

Question: 897

What deposit today is needed to have $4,000 in 4 years, assuming the money will earn interest at 5%
per year, compounded monthly?

A. $3,290.81
B. $3,276.28
C. $4,000.00
D. $384.57
E. $3,934.02

Answer: B

Explanation:
On the BAII Plus, press 48 N, 5 divide 12 = I/Y, 0 PMT, 4000 FV, CPT PV. On the HP12C, press 48 n, 5
ENTER 12 divide i, 0 PMT, 4000 FV, PV. Make sure the BAII Plus has the P/Y value set to 1.

Question: 898

What is the difference between a sample mean and the population mean called?

A. Sampling error
B. Point estimate
C. Standard error of the mean
D. None of these answers
E. Interval estimate

Answer: A

Explanation:
This is the error incurred when sampling from a population.

Question: 899

What annual interest rate, compounded annually, will cause an original deposit of $400 to grow to
$725, after 7 years?

A. 21.45%

Page | 336
B. 9.14%
C. 6.94%
D. 9.01%
E. 8.87%

Answer: E

Explanation:
On the BAII Plus, press 400 PV, 725 +/- FV, 0 PMT, 7 N, then CPT I/Y. On the HP12C, press 400 PV, 725
CHS FV, 0 PMT, 7 n, then press i. Make sure the BAII Plus has the P/Y value set to 1.

Question: 900

The weight of an offensive lineman may be 210 pounds, 210.1 pounds, 210.13 pounds or 210.137
pounds depending on the accuracy of the scale. What is this an illustration of?

A. none of these answers


B. discrete random variable
C. continuous random variable
D. all of these answers
E. complement rule

Answer: C

Explanation:
The weight changes are in a random fashion and without a separate interval. Therefore it is
continuous and random.

Question: 901

Suppose the least squares regression equation is Y' = 1202 + 1,133X. When X=3, what does Y' equal?

A. None of these answers


B. 5,734
C. 4,601
D. 8,000
E. 4,050

Answer: C

Explanation:
Y' = 1202 + 1133(3) = 4601.

Question: 902

The following is a distribution of monthly commissions:

Monthly Commissions Class Frequencies


$600 - $7993
$800 - $9997
$1,000 - $1,19911
$1,200 - $1,39922
$1,400 - $1,59940
$1,600 - $1,79924

Page | 337
$1,800 - $1,9999
$2,000 - $2,1994

Referring to the table above, what is the relative frequency for those salespersons that earn between
$1,600 and $1,799?

A. 24%
B. 2.4%
C. None of these answers
D. 20%
E. 2%

Answer: D

Explanation:
The sum of all the class frequencies is 120. The relative frequency for the $1600-$1799 class is
therefore 24/120 = 20%

Question: 903

You are examining a group of 3 stocks within an industry. The industry average debt-to-equity ratio is
expected to be 40%. For these stocks, the average debt-to-equity ratios have been 50%, 40%, and
30%. What is the mean absolute deviation of debt-to-equity ratios from the industry average?

A. 13.3%.
B. 6.67%.
C. 20.0%.
D. 0%.

Answer: B

Explanation:
The mean absolute deviation = the sum of (as i goes from 1 to n) of the absolute value of (X_i - Xbar),
divided by n. Here, we have [|50% - 40%| + |40% - 40%| + |30% - 40%|]/3 = [10% + 0% + 10%] / 3 =
20%/3 = 6.67%.

Question: 904

If you buy a television for $650 and agree to pay for it with 24 monthly payments of $33, beginning
next month, what annual interest rate, compounded monthly, are you being charged?

A. 15.93%
B. 21.00%
C. 5.08%
D. 18.38%
E. 19.74%

Answer: E

Explanation:
On the BAII Plus, press 24 N, 650 PV, 33 +/- PMT, 0 FV, CPT I/Y. Then press x 12 = to see the answer.
On the HP12C, press 24 n, 650 PV, 33 CHS PMT, 0 FV, i. Then press 12 x to see the answer. The last
multiplication by 12 is needed since the calculator returns the periodic interest rate. Make sure the
BAII Plus has the P/Y value set to 1.

Page | 338
Question: 905

Suppose we select every fifth invoice in a file. What type of sampling is this?

A. Random
B. None of these answers
C. Cluster
D. Systematic
E. Stratified

Answer: D

Explanation:
One of the ways to conduct systematic sampling is to have a rule for picking the sample in an
unbiased way. Selecting items at specified intervals can achieve this.

Question: 906

What single deposit could you make today in order to have $500,000 in 30 years, assuming it earns
interest at 9% per year, compounded monthly?

A. $19,423.23
B. $215,024.60
C. $23,682.82
D. $33,943.00
E. $53,512.59

Answer: D

Explanation:
On the BAII Plus, press 360 N, 9 divide 12 = I/Y, 0 PMT, 500000 FV, CPT PV. On the HP12C, press 360 n,
9 ENTER 12 divide i, 0 PMT, 500000 FV, PV. Note that the answer is displayed as a negative number.
Make sure the BAII Plus has the value of P/Y set to 1.

Question: 907

Suppose you were given $4,000 today and deposited it into an account paying 8% per year,
compounded monthly. If you know that you will need $5,000 in the account 6 years from now, what
monthly withdrawal can you make from the account, beginning one month from now, that will leave
the account with exactly $5,000 in it in 6 years?

A. $204.45
B. $15.80
C. Can't be done
D. $123.45
E. $1.23

Answer: B

Explanation:
On the BAII Plus, press 72 N, 8 divide 12 = I/Y, 4000 PV, 5000 +/- FV, CPT PMT. On the HP12C, press 72
n, 8 ENTER 12 divide i, 4000 PV, 5000 CHS FV, PMT. Note that the answer is a negative number. This is
because it is a withdrawal from the account balance, just as the $5,000 is. It is a very small value

Page | 339
because most of the $4,000 is needed to ensure there is $5,000 in the account in 6 years! Make sure
the BAII Plus has the value of P/Y set to 1.

Question: 908

The mean amount spent by a family of four on food per month is $500 with a standard deviation of
$75. Assuming that the food costs are normally distributed, what is the probability that a family
spends less than $410 per month?

A. 0.1151
B. 0.0362
C. None of these answers
D. 0.2158
E. 0.8750

Answer: A

Explanation:
z = (X - u)/sigma = (410 - 500)/75=-1.2. z = 1.2 is 0.3849. Since 410 is below the mean, 1.0 - 0.8849 =
0.1151.

Question: 909

The results of the regressions using 200 observation on a variable Y against X are as follows:

CoefficientStandard error
intercept3.62.1
slope8.11.3

R square = 49%

The regression equation can be expressed as:

A. Y = 3.6 + 8.1 X + error


B. X = 8.1 + 3.6 Y + error
C. Y = 8.1 + 3.6 X + error
D. X = 3.6 + 8.1 Y + error

Answer: A

Explanation:
Note that Y is regressed against X, implying that Y is the dependent (left-hand side) variable and X is
the independent (right-hand side) variable. The intercept in a regression equals the constant term
and the slope coefficient is the multiplier on the independent variable.

Question: 910

Suppose we set the criterion for the rejection of the null that is extremely lax, assuring us that the
null will not be rejected. Then, which of the following is/are true?
I. The probability of a Type I error is zero.
II. The probability of Type II error is zero.
III. The significance level of the test is 1.

A. I & III

Page | 340
B. II only
C. II & III
D. none of these answers
E. I only

Answer: E

Explanation:
A Type I error refers to the event that we will reject the null when, in fact, it is true. If the criterion is
so loose that you never reject the null, then the probability of type I error is zero. A Type II error
refers to the event that we will fail to reject the null when, in fact, it is false. If you never reject the
null, then the probability of type II error is clearly non-zero. Finally, the significance level is the same
as the probability of making a Type I error.

Question: 911

A stock has the following returns over 3 years: -5%, +15%, -4%. The annual geometric rate of return
over the three years is ________.

A. 9.36%
B. 1.60%
C. 7.42%
D. 0.15%
E. -2.21%
F. 4.64%
G. 3.31%
H. -1.34%

Answer: B

Explanation:
The annual geometric rate of return equals [(1-5%)(1+15%)(1-4%)]^(1/3) - 1 = (0.95 * 1.15 *
.96)^0.33 - 1 = 0.016 = 1.60%

Question: 912

What is the annual Internal Rate of Return of this series of annual cash flows: Year 0: <$25,000>, Year
1: $2,000, Year 2: $0, Year 3: $15,000, Year 4 to indicate a negative number).

A. 10.04%
B. 15.29%
C. 11.59%
D. 8.61%
E. 9.13%

Answer: E

Explanation:
On the BAII Plus, press CF 2nd CLRWork 25000 +/- ENTER DownArrow 2000 ENTER DownArrow
DownArrow 0 ENTER DownArrow DownArrow 15000 ENTER DownArrow DownArrow 0
ENTERDownArrow DownArrow 18000 ENTER DownArrow DownArrow 2nd Quit. Then press Irr CPT.
On the HP12C, press these keys: 25000 CHS BlueShift CFo 2000 BlueShift CFj 0 BlueShift CFj 15000
BlueShift CFj 0 BlueShift CFj 18000 BlueShift CFj. Then press YellowShift Irr. The "DownArrow"
represents the downward-pointing arrow on the top row of the BAII Plus keyboard. Make sure the

Page | 341
BAII Plus has the P/Y value set to 1.

Question: 913

What is your monthly payment, beginning next month, on a $15,000 loan, if you pay it off over 48
months and the interest rate is 2.9% per year, compounded monthly?

A. $582.76
B. $3,772.68
C. $331.35
D. $324.51
E. $306.88

Answer: C

Explanation:
On the BAII Plus, press 48 N, 2.9 divide 12 = I/Y, 15000 PV, 0 FV, CPT PMT. On the HP12c, press 48 n,
2.9 ENTER 12 divide i, 15000 PV, 0 FV, PMT. Note that the answer will be displayed as a negative
number. Make sure the BAII Plus has the value of P/Y set to 1.

Question: 914

Which of the following is/are true about the Central Limit Theorem?
I. It cannot be applied if the population distribution is non-normal.
II. It cannot be applied if the population distribution is highly skewed.
III. It implies that the mean of the population equals the mean of the means of all possible samples.

A. II only
B. I only
C. I & III
D. III only
E. I & II.
F. I, II, & III

Answer: D

Explanation:
The Central Limit Theorem states that for a population with mean M and variance S, a sample of
large size n has a sampling distribution of mean which is approximately normal with mean M and
variance S/n.

Question: 915

Which of the following is the formula for the covariance between X and Y?

A. (X - E(X))*(Y - E(Y)).
B. E[(X - E(X))*(Y - E(Y))].
C. E[(X + E(X))*(Y + E(Y))].
D. E[XY - E(XY)].

Answer: B

Explanation:
E[(X - E(X))*(Y - E(Y))] is the covariance between X and Y.

Page | 342
Question: 916

A sample of the amounts spent to heat all-electric homes of similar sizes in March revealed these
amounts (to the nearest dollar): $212, $191, $176, $129, $106, $92, $108, $109, $103, $121, $175
and $194. What is the range?

A. $130
B. $100
C. $120
D. None of these answers
E. $112

Answer: C

Explanation:
Range = 212 - 92 = 120

Question: 917

A dataset of 105 observations is organized in a relative frequency distribution into 9 classes. The sum
of the relative frequencies across all the classes equals ________.

A. 9
B. 1
C. 105
D. 100

Answer: B

Explanation:
Relative class frequencies are percentages and across all classes, they must sum up to 100%, which is
the same as summing up to 1.00.

Question: 918

A portfolio manager has estimated the beta of a stock at 0.8, with a z-statistic of 1.7 in a sample size
of 300 observations. At 5% significance level in a two-sided test the critical value is 1.96, therefore,
the manager should conclude that:

A. the null hypothesis of beta = 0 cannot be rejected.


B. the null hypothesis of beta = 0 cannot be accepted.
C. the alternative hypothesis that beta is nonzero should be rejected.
D. the alternative hypothesis that beta is not zero should be accepted.

Answer: A

Explanation:
Since the z-statistic is less than the critical value of 1.96 in a two-sided test, the null hypothesis that
beta = 0 cannot be rejected. Note, however, that this does not imply that beta equals zero. It just says
that with the given observations, one cannot conclude that beta is statistically significantly different
from zero. This is always the interpretation of the null hypothesis (that's the reason statisticians
emphasize that "failing to reject the null hypothesis" is not the same as "accepting the null
hypothesis." You should always remember this.) Also note that the nature of the alternative

Page | 343
hypothesis is important. If the alternative had been onesided (beta > 0), then the null would be
rejected in this case at the 5% level.

Question: 919

Which of the following is NOT a measure of dispersion?

A. MAD
B. variance
C. mode
D. range

Answer: C

Explanation:
The mode is a measure of central tendency.

Question: 920

How much would an original deposit of $1,500 grow to be after 8 and a half years, if the deposit
earns interest at 6.5% per year, compounded quarterly?

A. $870,183.49
B. $3,601.34
C. $2,541.02
D. $2,594.84
E. $1,570.42

Answer: D

Explanation:
On the BAII Plus, press 1500 PV, 0 PMT, 34 N, 6.5 divide 4 = I/Y, then press CPT FV. On the HP12C,
press 1500 PV, 0 PMT, 34 n, 6.5 ENTER 4 divide i, then press FV. Note that N = 34 quarters and the
answer is displayed as a negative number. Make sure that the BAII Plus has the P/Y value set to 1.

Question: 921

A survey of 144 retail stores revealed that a particular brand and model of a VCR retails for $375 with
a standard deviation of $20. What is the 95% confidence interval to estimate the true cost of the
VCR?

A. $328.40 to $421.60
B. $323.40 to $426.60
C. $335.80 to $414.20
D. $335.00 to $415.00
E. None of these answers

Answer: C

Explanation:
Interval estimates can be found from the empirical rule where 95% will lie between plus and minus
1.96 standard deviations of the mean.

Question: 922

Page | 344
You are examining a group of 5 mutual funds. Their average annual returns have been 10%, 8%, 13%,
19%, and 15%. What is the range of annual returns?

A. 12.0%.
B. 10.0%.
C. 11.0%.
D. 13.0%.

Answer: C

Explanation:
The range = the maximum value - the minimum value. Here, we have 19% - 8% = 11%.

Question: 923

What is it called when all the items in the population have the same chance of being selected for the
sample?

A. Cluster sampling
B. Simple Random sampling
C. Judgment sampling
D. Nonprobability sampling
E. None of these answers

Answer: B

Explanation:
When every item in a population is selected at random, each have the same probability of being
selected.

Question: 924

If a distribution is heavily right or left skewed, which of the following statistics should not be used as
a measure of central tendency?

A. skewness
B. mean
C. median
D. mode

Answer: B

Explanation:
The mean is affected severely by outliers and hence, should not be used as a measure of central
tendency with skewed distributions.

Question: 925

What is the annual Internal Rate of Return of this series of annual cash flows: Year 0: <$6,000>, Year
1: $4,000, Year 2: $3,000, Year 3: $2,000, Year 4: $1,000 (Note that the <> are used to indicate a
negative number).

A. 25.55%

Page | 345
B. 27.12%
C. 24.61%
D. 31.38%
E. 33.74%

Answer: D

Explanation:
On the BAII Plus, press CF 2nd CLRWork 6000 +/- ENTER DownArrow 4000 ENTER DownArrow
DownArrow 3000 ENTER DownArrow DownArrow 2000 ENTER DownArrow DownArrow 1000 ENTER
DownArrow DownArrow 2nd Quit. Then press Irr CPT. On the HP12C, press these keys: 6000 CHS
BlueShift CFo 4000 BlueShift CFj 3000 BlueShift CFj 2000 BlueShift CFj 1000 BlueShift CFj. Then press
YellowShift Irr. The "DownArrow" represents the downward-pointing arrow on the top row of the
BAII Plus keyboard. Make sure that the BAII Plus has the P/Y value set to 1.

Question: 926

John buys a house that costs $275,000 and agrees to pay for it with a 15 year mortgage at 6% per
year, compounded monthly. What is John's monthly payment on the loan?

A. $2,250.06
B. $2,320.61
C. $1,552.12
D. $1,972.22
E. $2,218.50

Answer: B

Explanation:
On the BAII Plus, press 180 N, 6 divide 12 = I/Y, 275000 PV, 0 FV, CPT PMT. On the HP12C, press 180 n,
6 ENTER 12 divide i, 275000 PV, 0 FV, PMT. Make sure the BAII Plus has the P/Y value set to 1.

Question: 927

Assume you deposit $800 now into an account that had nothing in it previously, make an additional
deposit of $800 in 2 years, and a final deposit of $800 in 4 years. How much is in your account in 5
years, if the account earns interest at 8% per year, compounded annually?

A. $3,047.23
B. $2,708.29
C. $4,331.88
D. $3,120.04
E. $2,905.51

Answer: A

Explanation:
Solve this question by working several compound interest problems, moving each dollar amount
over to year 5. On the BAII Plus, press 5 N, 8 I/Y, 800 PV, 0 PMT, CPT FV, which yields $1,175.46. Then
press STO 1, 3 N, CPT FV, which yields $1,007.77. Then press + RCL 1 = STO 1. Then press 1 N, CPT FV,
which yields $864.00. Then press + RCL 1 = to see the answer. On the HP12C, press 5 n, 8 i, 800 PV, 0
PMT, FV. Then press STO 1, 3 n, FV. Then press RCL 1 + STO 1. Then press 1 n, FV. Then press RCL 1 + to
see the answer. Note that the answer will be displayed as a negative number. Also note that the
value of "N" for each dollar amount is based on the distance from year 5. Make sure that the BAII

Page | 346
Plus has the P/Y value set to 1.

Question: 928

You can enter a derivative contract that will pay $100 at the end of a year if the price of corn exceeds
$3 per bushel, or $50 if it is equal to $3 per bushel or lower. The probability that corn will exceed $x
by the end of one year is 50%. The current price of the contract is $60, and interest is 5% per year.
What is the optimal strategy?

A. Sell the derivative contract short if corn prices rise.


B. Invest $60 at 5% until the end of the year.
C. Buy $3 per bushel worth of corn futures.
D. Enter into the derivative contract for a cost of $60.

Answer: D

Explanation:
Enter into the derivative contract for a cost of $60, for the expected payoff is 0.50 * $100 + 0.50 * $50
= $75. That is a 25% return on your investment in one year, greater than the 5% that could be made
by investing the $60 at interest. This is an example of the investment consequences of inconsistent
probabilities. The present value of the contract should be $75/1.05 = $71.43. Thus, an arbitrage
opportunity is present. On an expected value basis, you can buy an asset worth $71.43 for only $60.

Question: 929

An investment of $100 grows in four years to $345. The investor observes that the annual arithmetic
rate of return and the geometric rate of return were the same over this period. The annual
arithmetic rate of return must be ________.

A. 40.33%
B. 33.93%
C. 34.32%
D. 37.84%
E. 36.29%

Answer: E

Explanation:
If the annual geometric rate of return is r, then 100 * (1 + r)^4 = 345. This gives r = 36.29%. Note that
the only way the mean will be equal to the geometric mean if every year, the stock experienced a
return of 36.29% per year.

Question: 930

A sample of the monthly amounts spent for food by families of four receiving food stamps
approximates asymmetrical distribution. The sample mean is $150 and the standard deviation is $20.
About 95 percent of the monthly food expenditures are between what two amounts?

A. None of these answers


B. $110 and $190
C. $205 and $220
D. $85 and $105
E. $100 and $200

Page | 347
Answer: B

Explanation:
About 95% of the observations lie between plus and minus two standard deviations from the mean.

Question: 931

A manufacturer of headache medicine claims it is 70 percent effective within a few minutes. That is,
out of every 100 users 70 get relief within a few minutes. A group of 12 patients are given the
medicine. If the claim is true, what is the probability that 8 have relief within a few minutes?

A. None of these answers


B. 0.168
C. 0.667
D. 0.001
E. 0.231

Answer: E

Explanation:
This is a binomial probability. The probability of getting r successes out of n trials where the
probability of success each trial is p and probability of failure each trial is q (where q = 1-p) is given
by: n!(p^r)[q^(n-r)]/r!(n-r)!. Here n = 12, r = 8,p = 0.7 and q = 0.3. Therefore we have
12!(0.7^8)(0.3^4)/8!4! = 0.231.

Question: 932

If you deposit $400 a month, beginning next month, for 10 years into an account paying 8% per year,
compounded monthly, how much is in your account after that last deposit?

A. $18,121,973.53
B. $73,178.41
C. $48,000.00
D. $13,091.62
E. $88,402.98

Answer: B

Explanation:
On the BAII Plus, press 120 N, 8 divide 12 = I/Y, 0 PV, 400 PMT, CPT FV. On the HP12C, press 120 n, 8
ENTER 12 divide i, 0 PV, 400 PMT, FV. On the BAII Plus, make sure the value of P/Y is set to 1. Note
that the answer is displayed as a negative number.

Question: 933

If you deposit $245 into an account paying 5.5% per year simple interest, how much interest will be
earned after 15 months?

A. $259.59
B. $15.59
C. $202.13
D. $261.84
E. $16.84

Page | 348
Answer: E

Explanation:
The question only asks for the earned interest, not the total amount in the account. On the BAII Plus,
press 245 x 0.055 x 15 divide 12 = to see the answer. On the HP12C, press 245 ENTER 0.055 x 15 x 12
divide to see the answer.

Question: 934

A survey of top executives revealed that 35% of them regularly read Time magazine, 20% real
Newsweek and 40% read U.S. News & World Report. Ten percent read both Time and U.S. News &
World Report. What is the probability that a particular top executive reads either Time or U.S. News
& World Report regularly?

A. 0.85
B. 0.06
C. None of these answers
D. 0.65
E. 1.00

Answer: D

Explanation:
35% + 40% - 10% = 65%

Question: 935

What is the annual Internal Rate of Return of this series of annual cash flows: Year 0: <$10,000>, Year
1: $5,000, Year 2: $5,000, Year 3: $7,500? (Note that the <> are used to indicate a negative number).

A. 31.45%
B. 21.16%
C. 34.45%
D. 20.03%
E. 28.89%

Answer: A

Explanation:
On the BAII Plus, press CF 2nd CLRWork 10000 +/- ENTER DownArrow 5000 ENTER DownArrow
DownArrow 5000 ENTER DownArrow DownArrow 7500 ENTER DownArrow DownArrow 2nd Quit.
Then press Irr CPT. On the HP12C, press these keys: 10000 CHS BlueShift CFo 5000 BlueShift CFj 5000
BlueShift CFj 7500 BlueShift CFj Then press YellowShift Irr. The "DownArrow" represents the
downward-pointing arrow on the top row of the BAII Plus keyboard.

Question: 936

If your estate were to receive $100 a year, beginning next year, for 100 years, what is the equivalent
single amount today that this series of cash flows is equal to, assuming interest is 5% per year,
compounded annually?

A. $11,302.37
B. $10,000.00
C. $2,041.54

Page | 349
D. $58.42
E. $1,984.79

Answer: E

Explanation:
On the BAII Plus, press 100 N, 5 I/Y, 100 PMT, 0 FV, CPT PV. On the HP12C, press 100 n, 5 i, 100 PMT, 0
FV, PV. Note that the answer will be displayed as a negative number. As a side note, continuing the
$100 payments for 10,000 years only increases the PV today by a little over $15. This is because the
$100 payments beyond 100 years from now would add very little to the PV today. Make sure the BAII
Plus has the value of P/Y set to 1.

Question: 937

What monthly payment would repay a $28,500 car loan over 60 months, assuming your loan has an
interest rate of 3.9% per year, compounded monthly?

A. $492.29
B. $764.58
C. $423.75
D. $550.58
E. $523.59

Answer: E

Explanation:
On the BAII Plus, press 60 N, 3.9 divide 12 = I/Y, 28500 PV, 0 FV, CPT PMT. On the HP12C, press 60 n,
3.9 ENTER 12 divide i, 28500 PV, 0 FV, PMT. Note that the answer will be displayed as a negative
number. Make sure the BAII Plus has the value of P/Y set to 1.

Question: 938

If you deposit $5,000 into an account paying 9% per year, compounded monthly, how much do you
have in the account in 10 years?

A. $5,403.52
B. $12,001.66
C. $9,116.87
D. $2,039.69
E. $12,256.79

Answer: E

Explanation:
On the BAII Plus, press 120 N, 9 divide 12 = I/Y, 5000 PV, 0 PMT, CPT FV. On the HP12C, press 120 n, 9
ENTER 12 divide i, 5000 PV, 0 PMT, FV. Note that the answer will be displayed as a negative number.
Make sure the BAII Plus has the value of P/Y set to 1.

Question: 939

Suppose you need $500 in 9 months. How much must you deposit today, if the deposit will earn
interest at 6% per year, compounded monthly?

A. $522.96

Page | 350
B. $478.05
C. $381.95
D. $295.95
E. $291.77

Answer: B

Explanation:
On the BAII Plus, press 9 N, 6 divide 12 = I/Y, 0 PMT, 500 FV, CPT PV. On the HP12C, press 9 n, 6 ENTER
12 divide i, 0 PMT, 500 FV, PV. Make sure the BAII Plus has the P/Y value set to 1.

Question: 940

What does the complement rule state?

A. P(A) = 1 - P(not A)
B. None of these answers
C. P(A) = P(A) - P(B)
D. P(A) = P(A) x P(B)
E. P(A) = P(A) + P(B)

Answer: A

Explanation:
The sum of probability must equal 1. So P(A) + P(not A) = 1.

Question: 941

Two events, A and B, are mutually exclusive if:

A. P(A and B) = P(A) + P(B)


B. P(A and B) = 0
C. P(A and B) = P(A) P(B)
D. P(A and B) = 1

Answer: B

Explanation:
By definition, if A and B are mutually exclusive, they cannot occur simultaneously. Therefore, P(A and
B) = 0 for mutually exclusive events A and B. Note that P(A and B) = P(A).P(B) if and only if A and B
are independent. Clearly, if A and B are mutually exclusive, they cannot be independent unless at
least one of the two has a zero probability of occurring (i.e. is an impossible event). Finally, the only
way "P(A and B) = P(A) + P(B)" will be true is if P(A) = P(B) = 0. To see this, recall that P(A or B) = P(A) +
P(B) - P(A and B). The only way for "P(A and B) = P(A) + P(B)" to hold would be to have P(A or B) = 0.
This is possible only when P(A) = P(B) = 0.

Question: 942

Affirmative action commitments by industrial organizations have led to an increase in the number of
women in executive positions. Satellite Office Systems has vacancies for two executives, which it will
fill from among four women and six men. What is the probability that two woman are selected?

A. 3/5
B. 8/15

Page | 351
C. 3/4
D. 2/15
E. None of these answers

Answer: D

Explanation:
Probability of both positions being filled by women = 4/10*3/9 = 12/90.

Question: 943

Which one of the following is referred to as the population mean?

A. Sample
B. X-Bar
C. The Greek letter "Mu"
D. Statistic
E. None of these answers

Answer: C

Explanation:
The normal symbol for the population mean is the Greek letter "Mu."

Question: 944

Mileage tests were conducted on a randomly selected sample of 100 newly developed automobile
tires. The average tread wear was found to be 50,000 miles with a standard deviation of 3,500 miles.
What is the best estimate of the average tread life in miles for the entire population of these tires?

A. None of these answers


B. (3,500/100)
C. 3,500
D. 50,000
E. (50,000/100)

Answer: D

Explanation:
The sample mean is a good estimate of the population mean.

Question: 945

You have a portfolio of two assets, X and Y. X has an expected return of 15% with a variance of
900%%, and Y has an expected return of 9% with a variance of 400%%. Assume the covariance
between X and Y is 225%%. For a portfolio composed of 50% X and 50% Y, find the expected return
and variance.

A. 10%, 437.5%%.
B. 12%, 437.5%%.
C. 12%, 367.5%%.
D. 14%, 512.5%%.

Answer: B

Page | 352
Explanation:
The expected return is 0.5*15% + 0.5*9% = 7.5% + 4.5% = 12%. The variance will be trickier. The
variance will be equal to (w_X)^2 * (sigma_X)^2 + (w_Y)^2 * (sigma_Y)^2 + 2 * w_X * w_Y * cov(X,Y)
= (0.5)^2 * 900%% + (0.5)^2 * 400%% + 2 * (0.5) * (0.5) * 225%% = 437.5%%.

Question: 946

How much must you deposit today if you wish to have $20,000 in 10 years, assuming that interest
accumulates at 9% per year, compounded annually?

A. $2,000.00
B. $9,104.36
C. $8,448.22
D. $7,236.99
E. $7,103.49

Answer: C

Explanation:
On the BAII Plus, press 10 N, 9 I/Y, 0 PMT, 20000 FV, CPT PV. On the HP12C, press 10 n, 9 i, 0 PMT,
20000 FV, PV. Note that the answer is displayed as a negative number. Make sure that the BAII Plus
has the P/Y value set to 1.

Question: 947

What is another name for the alternate hypothesis?

A. None of these answers


B. Rejected hypothesis
C. Research hypothesis
D. Null hypothesis

Answer: C

Explanation:
The alternate is also referred to as the research hypothesis.

Question: 948

William Henry, a quantitative analyst with Smith, Kleen & Beetchnutty Brokerage, is examining a data
sample and has amassed the following information:
Standard deviation of the sample: 0.97
Number of observations: 109
Degrees of freedom: 2
Sample mean: 11.03
Assume that Mr. Henry formulates a null hypothesis that states that the value of the population
mean is equal to zero. Additionally, assume that the population standard deviation is unknown.
Given this information, what is the standard error of the estimate? Further, what is the test statistic?
Choose the best answer.

A. 0.0929; 10.90
B. 0.0929; 118.73
C. 0.0938; 117.59

Page | 353
D. None of these answers is correct.
E. 0.1858; 59.36
F. 0.1858; 11.41
G. 0.0938; 58.80

Answer: B

Explanation:
If the population standard deviation is unknown, as in this example, the standard error of the
estimate is found by using the following equation:
{Standard error = s / square root of n} where s = the sample standard deviation and n = the number
of observations in the sample.
In this example, all of the necessary information has been provided, and the determination of the
standard error of the estimate is found as:
{Standard error = [0.97 / 10.44] = 0.0929}
Now that the standard error of the estimate has been calculated, the test statistic can be found by
using the following equation:
{Test statistic = [sample statistic - value of the population parameter under the null hypothesis] /
standard error of the sample statistic].
Again, all of the necessary information has been provided, and the calculation of the test statistic is
found as follows:
{Test statistic = [11.03 - 0.00] / 0.0929 = 118.73}
This is a very large value for the test statistic. In this example, the null hypothesis would likely be
rejected unless a very low confidence level is assumed.

Question: 949

If a frequency distribution has open-ended intervals at the extremes, which measure of central
tendency is the most difficult to estimate?

A. Mean
B. None of these answers
C. Median
D. All of these answers
E. Mode

Answer: A

Explanation:
This is so since in a frequency distribution we do not necessarily know what the extreme values are
especially in an open ended class. Therefore, the mean, which is very sensitive to extreme values will
be difficult to determine.

Question: 950

What is the following table called?

Number of HeadsProbability of Outcome


01/8 = 0.125
13/8 = 0.375
23/8 = 0.375
31/8 = 0.125
Total8/8 = 1.000

Page | 354
A. Ogive
B. Standard deviation
C. Probability distribution
D. None of these answers
E. Frequency table

Answer: C

Explanation:
The table gives both the outcomes and probability of each outcome. Therefore it is a probability
distribution table.

Question: 951

What is a disadvantage of the range as a measure of dispersion?

A. It is not in the same units as the original data.


B. It can be distorted by a large mean.
C. It is based on only two observations.
D. None of these answers
E. It has no disadvantage.

Answer: C

Explanation:
The range uses only the highest and the lowest observations and ignores all the observations in the
middle. This may not be indicative of the dispersion if the two extreme observations are outliers.

Question: 952

The expected value of Y is 9 and the expected value of X is 3. If X and Y are uncorrelated and you run
a regression of Y against X, the intercept term will equal ________.

A. 9
B. 0
C. none of these answers
D. 3

Answer: A

Explanation:
Since the correlation is given to be zero, the slope will be zero and hence, R-square will be zero i.e.
the variance of the ranking of a trader in one year is not explained to any extent by his ranking in the
previous year. Thus, regardless of his ranking in previous year, he is likely to be either "Great" or
"Not- great" in this year. His expected score is then equal to 0.5*1 + 0.5*0 = 0.5. The expected value
of the dependent variable is equal to the intercept term when slope equals zero (remember this!!)

Question: 953

Given the following points: (-2, 0), (-1,0), (0,1), (1, 1) and (2, 3) What is the Y intercept of the
regression line?

A. 1.0
B. None of these answers

Page | 355
C. 1.5
D. 0.7
E. 0.0

Answer: A

Explanation:
For this, you need to create tables with columns for X,Y,XY,X^2,Y' and Y-Y'. This will be as follows.
XYXY X^2 Y'Y-Y'
-2004-0.40.4
-10010.3-0.3
010010
11111.7-0.7
23642.40.6
Sum057100
N=5.
The regression equation is Y' = a + bX.
First, find the slope, which is b, use the formula
b = [n(SXY)-(SX)(SY)]/[n(SX^2)-(SX)^2] where S stands for "Sum of." b = [5(0)-(0)(5)]/[5(10)-0]=0.7
The Y intercept is a.
a = [S(Y)/n]-b[SX/n]
a = 5/5 - 0.7[0/5] = 1.

Question: 954

What is the remaining balance on a $100,000 loan after making monthly payments of $733.76 for 10
years, if the interest rate is 8% per year, compounded monthly?

A. $86,204.11
B. $11,948.80
C. $0
D. $87,725.54
E. $91,343.92

Answer: D

Explanation:
On the BAII Plus, press 120 N, 8 divide 12 = I/Y, 100000 PV, 733.76 +/- PMT, CPT FV. On the HP12C,
press 120 n, 8 ENTER 12 divide i, 100000 PV, 733.76 CHS PMT, FV. The remaining balance will be
displayed as a negative number. Make sure the BAII Plus has the P/Y value set to 1.

Question: 955

What are the two rejection areas in using a two-tailed test and the 0.01 level of significance?

A. None of these answers


B. Above 1.96 and below -1.96
C. Above 1.645 and below -1.645
D. Above 1.00 and below -1.00
E. Above 2.58 and below -2.58

Answer: E

Explanation:

Page | 356
0.01 means that we are considering a 99% level of significance. For this, we can find the z-value to be
+/- 2.58.

Question: 956

If the critical value for a test statistic equals 2.45, what value of the test statistic would guarantee no
chance of making a Type I error?

A. None of these answers


B. 3.74
C. 2.14
D. 4.56
E. 2.46

Answer: C

Explanation:
The type I error is rejecting the null hypothesis when it is actually true. So if we accept the null
hypothesis, there is no possibility of committing the type I error. Therefore, we can only accept the
null if our test statistic falls inside the acceptance region between -2.45 and +2.45.

Question: 957

John's great-grandfather left him $50 when he died 80 years ago in an account paying 8% per year,
compounded annually. How much would the account hold for John now?

A. $407.76
B. $24,229.78
C. $24,113.90
D. $22,504.51
E. $23,597.74

Answer: E

Explanation:
On the BAII Plus, press 80 N, 8 I/Y, 50 PV, 0 PMT, CPT FV. On the HP12C, press 80 n, 8 i, 50 PV, 0 PMT,
FV. Note that the answer will be displayed as a negative number. Make sure that the BAII Plus has the
P/Y value set to 1.

Question: 958

What quarterly payment is required beginning one period from today to pay off a $4,000 debt, if
interest accrues at 8% per year, compounded quarterly, and the debt is to be retired in 5 years?

A. $1,001.83
B. $244.63
C. $848.63
D. $407.41
E. $214.29

Answer: B

Explanation:
On the BAII Plus, press 20 N, 8 divide 4 = I/Y, 4000 PV, 0 FV, CPT PMT. On the HP12C, press 20 n, 8

Page | 357
ENTER 4 divide i, 4000 PV, 0 FV, PMT. Note that the answer will be displayed as a negative number.
Make sure the BAII Plus has the P/Y value set to 1.

Question: 959

What monthly payment, beginning next month, would repay a $17,500 car loan over 48 months,
assuming your loan has an interest rate of 4.9% per year, compounded monthly?

A. $364.58
B. $650.58
C. $953.46
D. $402.22
E. $4,923.75

Answer: D

Explanation:
On the BAII Plus, press 48 N, 4.9 divide 12 = I/Y, 17500 PV, 0 FV, CPT PMT. On the HP12C, press 48 n,
4.9 ENTER 12 divide i, 17500 PV, 0 FV, PMT. Note that the answer will be displayed as a negative
number. Make sure the BAII Plus has the value of P/Y set to 1.

Question: 960

A random variable with a mean equal to 6.0 and a standard deviation of 1.5 has a coefficient of
variation equal to ________.

A. 0.25
B. zero
C. 4.0
D. 1.5
E. none of these answers

Answer: A

Explanation:
The coefficient of variation equals the ratio of the standard deviation to the mean.

Question: 961

A researcher has organized 310 observations into 9 classes. The frequency of the 50-75 frequency
class is 34. The relative frequency of the class equals ________.

A. 0.14
B. 34
C. 0.11
D. 0.27

Answer: C

Explanation:
relative frequency = class frequency/total # of observations
= 34/310 = 0.11
Remember that relative frequency is always a number between zero and one.

Page | 358
Question: 962

Variable X is distributed normally and has a mean of 10. If the probability that an observation of X
will be negative is 0.16, what is the coefficient of variation of X?

A. 10.0
B. 1.0
C. 0.1
D. 0.32

Answer: B

Explanation:
The probability that X lies a distance 10 below the mean is given to be 0.16. Since the normal
distribution is symmetric about the mean, this implies that the probability that X will be 10 greater
than the mean is also 0.16. Thus, the probability that X lies between 0 and 20 is 1-0.16-0.16 = 0.68.
For a normal distribution, 68% of the observations lie within one standard deviation of the mean.
Hence, the standard deviation of X equals 10. The coefficient of variation is then equal to standard
deviation/mean = 10/10 = 1.

Question: 963

You wish to determine the number of unique combinations that can result from a process that
involves n_1 options in one respect, n_2 in another respect, and so on. The counting method you
should use is:

A. The binomial formula.


B. The multiplication rule.
C. The multinomial formula.
D. The permutation rule.

Answer: B

Explanation:
The multiplication rule of counting states that the number of combinations available when there are
n_1 options in one aspect, n_2 in another, and so on, up to n_k, is n_1 * n_2 * ... * n_k.

Question: 964

A sample of assistant professors on the business faculty at state supported institutions in Ohio
revealed the mean income to be $32,000 for 9 months with a standard deviation of $3,000. Using
Chebyshev's Theorem, what is the proportion of faculty that earn more than $26,000 but less than
$38,000?

A. At least 25%
B. None of these answers
C. At least 75%
D. At least 100%
E. At least 50%

Answer: C

Explanation:
26,000 and 38,000 are two standard deviations away from the mean. Using 1-(1/k^2) where k = 2, we

Page | 359
get 0.75

Question: 965

If you deposit $7,000 into an account paying 6% per year, compounded annually, how much do you
have in the account in 20 years?

A. $15,938.48
B. $23,403.52
C. $20,800.00
D. $22,470.62
E. $22,449.95

Answer: E

Explanation:
On the BAII Plus, press 20 N, 6 I/Y, 7000 PV, 0 PMT, CPT FV. On the HP12C, press 20 n, 6 i, 7000 PV, 0
PMT, FV. Note that the answer will be displayed as a negative number. Make sure that the BAII Plus
has the P/Y value set to 1.

Question: 966

With a discount rate of 12% per year, the value of a perpetuity that pays $500 every year is:

A. $5,231
B. $6,000
C. $4,167
D. $4,756

Answer: C

Explanation:
The value of the perpetuity = 500/0.12 = 4,167

Question: 967

David's gasoline station offers 4 cents off per gallon if the customer pays in cash and does not use a
credit card. Past evidence indicates that 40% of all customers pay in cash. During a one hour period
twenty-five customers buy gasoline at this station. What is the probability that no more than twenty
pay in cash?

A. 0.1
B. 0.9
C. None of these answers
D. 0.0
E. 1.0

Answer: E

Explanation:
P(20) is almost zero. Probabilities of higher than 20 are also close to zero. So the probability of not
more than 20 is almost 1.

Question: 968

Page | 360
You have invested in a stock with a expected return of 14% and a standard deviation of 7%. Your
target rate of return is 7%. What is the probability that you will not meet your objective, assuming
stock returns are normally distributed?

A. 32%
B. 34%
C. 16%
D. 68%

Answer: C

Explanation:
There is a 68% chance that the stock returns will be within one standard deviation of the mean i.e.
with 68% chance, the stock returns will be between 7% and 21%. Hence, the probability that stock
returnswill lie outside this range is 100% - 68% = 32%. Since the normal distribution is symmetrically
distributed about the mean, the probability that the returns will be less than 7% equals 32/2 = 16%
Instead of the above, you could also solve the problem using the z-score and the normal probability
distribution table. You should, however, be aware of short-cuts like the above.

Question: 969

In a systematic random sampling method,

A. a sample selected such that every member of the population has the same chance of being
selected.
B. a sample is selected by randomly from within the first N members of the population and then
selecting every Nth member of the population.
C. a sample is selected by drawing numbers from a normal probability distribution.
D. a sample is selected by first dividing the population into groups and then selecting members from
each group.

Answer: B

Explanation:
The population members are arranges in a systematic way, such as alphabetically or by date
received. A random starting point is selected and then every Nth item is selected for inclusion in the
random sample.

Question: 970

The mean, as a measure of central tendency would be inappropriate for which one of the following?

A. Incomes of lawyers
B. Number of pages in textbooks on statistics
C. Ages of adults at a senior citizen center
D. Marital status of college students at a particular university
E. None of these answers

Answer: D

Explanation:
The answer to the question what is your marital status is usually yes or no, or some other status -
divorced, widow etc. So we cannot take an average of non-ordinal observations.

Page | 361
Question: 971

Which is true of positively skewed distributions?


I. They have a limited, but frequent, upside.
II. Their downside is less frequent but more unlimited.
III. They are attractive to investors because their mean is larger than their median.

A. III
B. I and II
C. II
D. I and III
E. II and III

Answer: A

Explanation:
I and II are false. The correct statements for them would be: 1) positively skewed distributions have a
limited, but frequent, downside, and 2) they have a less frequent, but more unlimited, upside. III is
true.

Question: 972

What annual interest rate, compounded annually, will cause a deposit of $850 to become $1,475 in
10 years?

A. 5.67%
B. 7.60%
C. 7.88%
D. 4.65%
E. 5.50%

Answer: A

Explanation:
On the BAII Plus, press 10 N, 850 PV, 1475 +/- FV, 0 PMT, CPT I/Y. On the HP12C, press 10 n, 850 PV,
1475 CHS FV, 0 PMT, i. Make sure that the BAII Plus has the P/Y value set to 1.

Question: 973

A researcher has decided to create a frequency distribution using the following classes:
30-45, 45-60, 60-75, 76 & over.
The selection of this set involves which of the following?
I. non-overlapping classes.
II. open-ended classes.
III. equal class intervals

A. III only
B. I only
C. I & III only
D. II & III only
E. II only

Answer: E

Page | 362
Explanation:
Note that "76 & over" is an open-ended class. Also, the classes are overlapping (for e.g., the classes
"30-45" and "45-60" have the point "45" in common).

Question: 974

The mean of a normally distributed group of weekly incomes of a large group of executive is $1,000
and the standard deviation is $100. What is the z-score for an income of $1,100?

A. -0.90
B. 2.00
C. None of these answers
D. 1.683
E. 1.00

Answer: E

Explanation:
z = (x-u)/sigma = 1100 - 1000/100 = 1.

Question: 975

A mining firm has purchased a derivative security to partially hedge itself against the losses caused
by fluctuations in a base metal price. The security pays a million dollars if the metal price falls below
$2 per pound. Otherwise, it pays $100,000. If the expected rate of return on the security is 21% and
the security costs $400,000, what is the probability that the metal price will remain above $2 per
pound?

A. 42.66%
B. 61.1%
C. 57.33%
D. 46.87%

Answer: C

Explanation:
If p is the probability of the metal price remaining above $2/pound, then the expected total return
on the derivative security is (p * 100,000 + (1-p)*1,000,000)/400,000 = (10-9p)/4. Since the expected
return is given to be 21%, we get (10-9p)/4 = 1.21. Therefore, p = 57.33%.

Question: 976

Walter Jennings, a quantitative analyst with Smith, Kleen & Beetchnutty Brokerage, has just been
informed of an important error in one of his recent statistical endeavors. Specifically, in one
hypothesis test, Mr. Jennings failed to reject a null hypothesis that later was proven to be false.
Which of the following best describes this type of error in hypothesis testing? Further, if the
confidence level of the test were increased, would the probability of this error increase, decrease, or
is this probability difficult to determine?

A. Type II error; difficult to determine


B. Type I error; difficult to determine
C. Type I error; decrease
D. Type II error; decrease

Page | 363
E. Type I error; increase
F. Type II error; increase

Answer: A

Explanation:
In this example, Mr. Jennings has incorrectly failed to reject a null hypothesis. This type of error in
hypothesis testing is called a Type II error. In hypothesis testing, the Type I error is given more
attention than the Type II error. A Type I error is defined as the act of incorrectly rejecting the null
hypothesis. In most hypothesis tests, the probability of a null hypothesis is equal to the significance
level of the test. A significance level of 0.01, for example, indicates that a 1% chance exists that the
null hypothesis will be rejected when it is indeed true. Another way to think of the probability of a
Type I error is to observe the following relationship:
{Probability of a Type I error = (1 - confidence level)}.
For example, a confidence level of 95% leaves a 5% probability of a Type I error occurring. If this
confidence level were to increase to say, 98%, then the probability of a Type I error would reduce to
2%. While a relationship exists between the confidence level for a hypothesis test and the probability
of a Type I error, the relationship between confidence levels and Type II errors is not as explicit. The
probability of a Type II error is inherently difficult to quantify, and as such, few hypothesis tests call
for a determination of the probability of a Type II error. If the confidence level of a hypothesis test to
be increased, it would be difficult to determine what effect, if any, this would have on the probability
of a Type II error.

Question: 977

If you owe a debt of $1,000 today and also owe $500 in 24 months, what single payment could you
make 12 months from today that would pay off both of these debts, if interest is assessed at 8% per
year, compounded monthly?

A. $751.62
B. $1,426.30
C. $1,500.00
D. $1,544.68
E. $2,041.93

Answer: D

Explanation:
To solve this question, set the problem up as the sum of two compound interest calculations. Move
the $1,000 from today over to month 12 and add it to the $500 brought back from month 24 to
month 12. On the BAII Plus, press 12 N, 8 divide 12 = I/Y, 1000 PV, 0 PMT, CPT FV, which yields
$1,083.00. Then press STO 1. Then press 500 FV, CPT PV which yields $461.68. Finally press + RCL 1 =
to see the answer. On the HP12C, press 12 n, 8 ENTER 12 divide i, 1000 PV, 0 PMT, FV. Then press STO
1. Then press 500 FV, PV. Finally press RCL 1 + to see the answer.

Question: 978

On a very hot summer day, 5 percent of the production employees at Midland States Steel are absent
from work. The production employees are to be selected at random for a special in depth study on
absenteeism. What is the probability of selecting 10 production employees at random on a hot
summer day and finding that none of them are absent?

A. 0.599
B. 0.100

Page | 364
C. 0.002
D. 0.344
E. None of these answers

Answer: A

Explanation:
This is a binomial probability. The probability of getting r successes out of n trials where the
probability of success each trial is p and probability of failure each trial is q (where q = 1-p) is given
by: n!(p^r)[q^(n-r)]/r!(n-r)!. Here n = 10, r = 0,p = 0.05 and q = 0.95. Therefore we have
10!(0.05^0)(0.95^10)/0!10! = 0.599.

Question: 979

If an analyst is trying to estimate the value of a stock, using the formula E(Y | E) = [y_1 * P(y_1 |E) +
y_2 * P(y_2|E) + ... y_n * P(y_n|E), the analyst is making use of:

A. subjective probability.
B. the multiplication rule for independent events.
C. unconditional expectation.
D. conditional expectation.

Answer: D

Explanation:
Since the stock value is conditioned on an uncertain event and the likelihood of certain stock price
outcomes given the occurrence of that uncertain event, the analyst is using conditional expectation.
The expected value is conditioned upon the occurrence of the event.

Question: 980

What is the present value of $350 per year for 10 years, with the first cash flow occurring today,
rather than 1 year from now? Assume interest is 8% per year, compounded annually.

A. $2,408.90
B. $48.49
C. $2,536.41
D. $3,500.00
E. $2,348.53

Answer: C

Explanation:
Recognize that this question is an annuity due situation, since the first cash flow occurs immediately
or at the beginning of each period. Annuities where the first payment occurs 1 period from today (or
at the end of each period) are called "ordinary" annuities. This requires placing the calculator into
"Begin" mode prior to solving the question. NOTE: Be sure to place the calculator OUT OF annuity
due mode after this question before going on to subsequent questions, or you will get the wrong
answers! On the BAII Plus, press 2nd BGN. If the display shows END, then press 2nd SET and then 2nd
Quit. This will place the BAII Plus into annuity due mode (you can tell this because the BAII Plus will
display BGN in small letters). Now press 10 N, 8 I/Y, 350 PMT, 0 FV, CPT PV. Place the calculator back
into End mode (for ordinary annuities) by pressing 2nd BGN and then if the calculator is displaying
BGN, press 2nd SET and 2nd Quit. The BGN letters should disappear from the display. On the HP12C,
press BlueShift END, which is the blue function on the front of the 8 digit key. This places the HP12C

Page | 365
into Begin mode (the HP12C shows the word BEGIN in the display when in this mode). Then press 10
n, 8 i, 350 PMT, 0 FV, PV. To place the HP12C back into ordinary annuity mode (or END mode), press
BlueShift BEG (the blue function written on the front of the 7 digit key). Note that the answer is
displayed as a negative number. Make sure the BAII Plus has the value of P/Y set to 1.

Question: 981

For which measure of central tendency will the sum of the deviations of each value from that
average always be zero?

A. Median
B. Geometric mean
C. Mean
D. Mode
E. None of these answers

Answer: C

Explanation:
For the mean, the sum of the deviations from the average is always zero because the mean is the
average.

Question: 982

Which of the following statements is false in reference to confidence intervals and/or tests of
significance?

A. As the confidence level increases, the area between the upper confidence limit and the lower
confidence limit increases.
B. More than one of these answers is false.
C. The significance level is denoted by the Greek letter alpha.
D. The significance level is typically measured by (1 - the probability of a Type II error).
E. As the confidence level increases, the power of the does not necessarily increase.
F. Significance levels are typically set equal to 0.10, 0.05, and 0.01.

Answer: D

Explanation:
In most statistical tests, the significance level is equal to one minus the probability of a Type I error.
The Type I error is given much more attention in hypothesis testing than is the Type II error, and this
is partially because of the inherent difficulty in quantifying the probability of a Type II error.
Remember that a Type II error occurs when one incorrectly fails to reject the null hypothesis. Reliably
predicting this occurrence is difficult. Conversely, determining the probability of a Type I error
requires much less effort and is subject to much less estimation bias. The remaining answers are
correct.

Question: 983

The mean gross annual income of certified tack welders are normally distributed with the mean of
$20,000 and a standard deviation of $2,000. The ship building association wishes to find out whether
their tack welders earn more or less than $20,000 annually. The alternate hypothesis is that the
mean is not $20,000. The 0.10 level of significance is to be used. What is the decision rule?

A. Do not reject the null hypothesis if computed z lies between -1.96; otherwise, reject it

Page | 366
B. None of these answers
C. Reject the null hypothesis if computed z is below -1.96; otherwise, reject it
D. Do not reject the null hypothesis if computed z lies greater than 1.645; otherwise, reject it
E. Do no reject the null hypothesis if computed z lies between -1.645 and +1.645; otherwise, reject it

Answer: E

Explanation:
Since it is at the 10% significance level we must use z=+/-1.645. The null hypothesis Ho will not be
rejected if the z-value falls between the critical value. We reject Ho if it falls beyond the critical level.

Question: 984

What is the future value of $450 per year for 10 years, with the first cash flow occurring today, rather
than 1 year from now? Assume interest is 7% per year, compounded annually.

A. $6,652.62
B. $6,217.40
C. $3,381.85
D. $3,160.61
E. $4,951.85

Answer: A

Explanation:
Recognize that this question is an annuity due situation, since the first cash flow occurs immediately
or at the beginning of each period. Annuities where the first payment occurs 1 period from today (or
at the end of each period) are called "ordinary" annuities. This requires placing the calculator into
"Begin" mode prior to solving the question. NOTE: Be sure to place the calculator OUT OF annuity
due mode after this question before going on to subsequent questions, or you will get the wrong
answers! On the BAII Plus, press 2nd BGN. If the display shows END, then press 2nd SET and then 2nd
Quit. This will place the BAII Plus into annuity due mode (you can tell this because the BAII Plus will
display BGN in small letters). Now press 10 N, 7 I/Y, 450 PMT, 0 PV, CPT FV. Place the calculator back
into End mode (for ordinary annuities) by pressing 2nd BGN and then if the calculator is displaying
BGN, press 2nd SET and 2nd Quit. The BGN letters should disappear from the display. On the HP12C,
press BlueShift END, which is the blue function on the front of the 8 digit key. This places the HP12C
into Begin mode (the HP12C shows the word BEGIN in the display when in this mode). Then press 10
n, 7 i, 450 PMT, 0 PV, FV. To place the HP12C back into ordinary annuity mode (or END mode), press
BlueShift BEG (the blue function written on the front of the 7 digit key). Note that the answer is
displayed as a negative number. Make sure the BAII Plus has the P/Y value set to 1.

Question: 985

If you deposit $10,000 into an account paying 8% per year, compounded semiannually, how much do
you have in the account in 10 years?

A. $15,403.52
B. $21,911.23
C. $24,800.00
D. $22,667.70
E. $25,938.48

Answer: B

Page | 367
Explanation:
On the BAII Plus, press 20 N, 8 divide 2 = I/Y, 10000 PV, 0 PMT, CPT FV. On the HP12C, press 20 n, 8
ENTER 2 divide i, 10000 PV, 0 PMT, FV. Note that the answer will be displayed as a negative number.
Make sure the BAII Plus has the value of P/Y set to 1.

Question: 986

How many deposits of $250 a month, beginning next month, are needed before an account has
$200,000 in it, assuming it had a balance of $0 at the beginning and that interest is 6% per year,
compounded monthly?

A. 319.48
B. No solution/Error
C. 322.69
D. 800.00
E. 66.79

Answer: C

Explanation:
One of the two dollar amounts must be entered as a negative number and the other as a positive
number or an error will result. On the BAII Plus, press 6 divide 12 = I/Y, 0 PV, 250 PMT, 200000 +/- FV,
CPT N. On the HP12C, press 6 ENTER 12 divide i, 0 PV, 250 PMT, 200000 CHS FV, n. Note that the
HP12C returns a result of 323. Make sure the BAII Plus has the P/Y value set to 1.

Question: 987

If you deposit $2,500 into an account paying 6% per year, simple interest, how much is in the account
in 18 months?

A. $2,651.04
B. $2,725.00
C. $2,750.19
D. $2,349.38
E. $225.00

Answer: B

Explanation:
Since the question asks for how much is in the account, the simple interest earned should be added
to the original deposit to find the answer. On the BAII Plus, press 2500 x 0.06 x 18 divide 12 = + 2500
=. On the HP12C, press 2500 ENTER 0.06 x 18 x 12 divide 2500 + to see the answer.

Question: 988

You are examining a special group of 5 stock market indices. Of these 5, the returns were 4%, 8%,
12%, 16%, and 10%. What is the population variance of this group of stock market indices?

A. 10%.
B. 16%%.
C. 10%%.
D. 16%.

Answer: B

Page | 368
Explanation:
The population variance is equal to the sum of the squared differences between each population
member and the population mean, divided by the number of items in the population. In this case,
wehave a mean of 10%. The first squared difference will be (4% - 10%)^2 = 0.0036. The others will be
0.0004, 0.0004, 0.0036, and 0. The sum of these squared differences is 0.008, and divided by 5, we
get 0.0016 = 16%%.

Question: 989

An investment of $100 grows in three years to $221. The investor observes that the annual
arithmetic rate of return and the geometric rate of return were the same over this period. The
annual arithmetic rate of return must be ________.

A. 30.26%
B. 23.93%
C. 40.33%
D. 24.32%
E. 27.84%

Answer: A

Explanation:
If the annual geometric rate of return is r, then 100 * (1 + r)^3 = 221. This gives r = 30.26%. Note that
the only way the mean will be equal to the geometric mean if every year, the stock experienced a
return of 30.26% per year.

Question: 990

If all the data points in a regression lie exactly on a straight line, which of the following is/are true?
I. The observed values of the dependent variable will equal the predicted values.
II. The R-square will equal 100%.
III. The slope coefficient will be 1.
IV. The residual error will be 100%.

A. I & II
B. III only
C. II only
D. IV only
E. I only
F. I & III
G. I, II & III
H. III & IV

Answer: A

Explanation:
Since there is no error in the regression, the R-square equals 1 (100%). The slope coefficient can be
any real number, not necessarily 1. The residual error will be zero.

Question: 991

Which of the following answers is false in reference to confidence levels and/or tests of significance?
Choose the best answer.

Page | 369
A. All else equal, the confidence interval for a test with a 5% significance level is larger than the
confidence interval for a test with a 1% significance level.
B. The Greek letter alpha is used to denote the probability of a Type I error.
C. The significance level used in hypothesis testing is typically 0.10, 0.05, or 0.01.
D. In most hypothesis testing, the power of a test is equal to (1 - the significance level).
E. The confidence level can be found by (1 - alpha).
F. More than one of these answers is false.

Answer: A

Explanation:
The confidence interval for a test with a 5% level of significance is smaller than the confidence
interval for a 1% significance level. Remember that the significance level is typically set equal to the
probability of a Type I error, which is defined as the act of incorrectly rejecting the null hypothesis. In
hypothesis testing, the significance level is denoted by the Greek letter alpha. As the level of
confidence increases, then the confidence interval will increase. This will be mirrored by a decrease
in the alpha coefficient (i.e. the probability of a Type I error) for the hypothesis test. The remaining
answers are correct.

Question: 992

If 68% of the observations of a bell-shaped, symmetrical frequency distribution fall between -20 and
+ 60, the mean and the standard deviation of the distribution are:

A. 40, 20
B. 40, 40
C. 20, 20
D. 20, 40

Answer: D

Explanation:
Since the distribution is symmetrical about the mean, the mean is equal to (-20 + 60)/2 = 20.
According to the empirical/Normal rule, for a bell-shaped, symmetrical frequency distribution, 68%
of the observations lie within one standard deviation of the mean. Therefore, the standard deviation
equals 60-20 = 40.

Question: 993

What is the range of values for a coefficient of correlation?

A. -1.0 to +1.0 inclusive


B. Unlimited range
C. -3 to +3 inclusive
D. None of these answers
E. 0 to +1.0

Answer: A

Explanation:
The closer the value to 1 or -1, the stronger the correlation. The sign +/- just shows the direction, not
the strength.

Page | 370
Question: 994

What is your monthly payment, beginning next month, on a $19,000 loan, if you pay it off over 48
months and the interest rate is 5.9% per year, compounded monthly?

A. $398.68
B. $1,411.86
C. $448.60
D. $397.26
E. $445.34

Answer: E

Explanation:
On the BAII Plus, press 48 N, 5.9 divide 12 = I/Y, 19000 PV, 0 FV, CPT PMT. On the HP12c, press 48 n,
5.9 ENTER 12 divide i, 19000 PV, 0 FV, PMT. Note that the answer will be displayed as a negative
number. Make sure the BAII Plus has the value of P/Y set to 1.

Question: 995

A selected group of employees of Unique Buying Services is to be surveyed with respect to a new
pension plan. In depth interviews are to be conducted with each employee selected in the sample.
The employees are classified as follows.

ClassificationEventNumber of Employees
SupervisorsA120
MaintenanceB50
ProductionC1,460
ManagementD302
SecretarialE68

What is the probability that the first person selected is either in maintenance OR in secretarial?

A. 0.001
B. 0.015
C. 0.200
D. None of these answers
E. 0.059

Answer: E

Explanation:
There are 2000 employees of which 50 are from maintenance and 68 in secretarial. Therefore (50 +
68)/2000 = 0.059.

Question: 996

A researcher has rejected the null hypothesis. The p-value in the test must be:

A. less than the significance level.


B. more than 1-significance level.
C. less than 1-significance level.
D. more than the significance level.

Page | 371
Answer: A

Explanation:
The lowest significance level at which the null hypothesis can be rejected is called the p-value of the
test. Thus, if the p-value is less than the significance level, the null hypothesis can be rejected at that
significance level. To fix this in your mind, think about the p-value as the maximum probability that
the null hypothesis is true despite observing the value of the test statistic that you have in the sample
at hand. For e.g., suppose the p-value for a given sample turns out to be 3%. Then, if you reject the
null, the probability that you have made an error in rejecting it is at most 3%. Clearly, if a critic asked
you, "Is the probability that you have made an error less than 5%?" your answer would be, "yes." The
critic will then conclude that your test is significant at the 5% level and perhaps even at a lower level.

Question: 997

What deposit today is needed to have $2,000 in 4 years, assuming the money will earn interest at 5%
per year, compounded monthly?

A. $2,000.00
B. $1,290.81
C. $584.57
D. $1,638.14
E. $1,934.02

Answer: D

Explanation:
On the BAII Plus, press 48 N, 5 divide 12 = I/Y, 0 PMT, 2000 FV, CPT PV. On the HP12C, press 48 n, 5
ENTER 12 divide i, 0 PMT, 2000 FV, PV. Make sure the BAII Plus has the P/Y value set to 1.

Question: 998

An economic researcher estimates the following regression between the annual income, I, of a
family and the amount it spends on consumption goods, C:
C = 2,000 + 0.65 I + error term
If the R-square of the regression is just 0.3, estimate the amount that a family with an income of
50,000 will spend on consumption.

A. 34,500
B. none of these answers
C. 32,500
D. 2,000

Answer: A

Explanation:
With the given regression, the amount that a family with an income of 50,000 will spend on
consumption equals 2,000 + 0.65*50,000 = 34,500.

Question: 999

Which of the following is/are true?


I. For a given level of significance, it becomes harder to reject the null hypothesis as the sample size
decreases.
II. For a given sample size, it becomes harder to reject the null hypothesis as the significance level

Page | 372
decreases.
III. It is easier to reject the null hypothesis the lower the R-square.

A. I only
B. II only
C. I, II & III
D. III only
E. I & III
F. II & III
G. I & II

Answer: G

Explanation:
As the sample size increases, it becomes easier to test whether the null hypothesis can be rejected at
the specified significance level (Intuitively, it is harder for a false hypothesis to hide its falsity when
there are a larger number of observations testing its veracity). In fact, the critical t-statistic required
to reject the null decreases as the sample size increases for a given level of significance.
A higher significance level makes it easier to reject the null hypothesis. This is because you are
enforcing a looser standard on rejecting the null (recall that the significance level represents the
probability thatyou have rejected the null when, in fact, it is true). In fact, significance level and
critical t-statistic are inversely related.
R-square is not used in hypothesis testing but in regression analysis, though it does not measure the
significance of a regression. A regression with a very low R-square can be highly significant; the low
R-square only implies that the behavior of the dependent variable is governed largely by random
noise.

Question: 1000

How much will $3,250 become after 6 years at 8% per year, compounded annually?

A. $5,298.66
B. $2,039.09
C. $2,048.05
D. $5,157.34
E. $5,180.00

Answer: D

Explanation:
On the BAII Plus, press 6 N, 8 I/Y, 3250 PV, 0 PMT, CPT FV. On the HP12C, press 6 n, 8 i, 3250 PV, 0
PMT, FV. Note that the answer will be displayed as a negative number. Make sure the BAII Plus has
the P/Y value set to 1.

Question: 1001

What is the standard error of estimate?

A. Based on squared vertical deviations between Y and Y'


B. None of these answers
C. Measure of the accuracy of the prediction
D. Cannot be negative
E. All of these answers

Page | 373
Answer: E

Explanation:
All the descriptions are correct for the standard error of estimate.

Question: 1002

What does a coefficient of correlation of 0.70 mean?

A. Coefficient of determination is 0.49


B. Almost no correlation because 0.70 is close to 1.0
C. 70% of the variation in one variable is explained by the other
D. Coefficient of nondetermination is 0.30
E. None of these answers

Answer: A

Explanation:
The coefficient of correlation, r, is just the square root of the coefficient of determination r-squared.

Question: 1003

A bell-shaped, symmetrical frequency distribution has a mean of 45. If 95% of the observations on
the variable fall between 30 and 60, the standard deviation of the variable is:

A. 5.00
B. 7.50
C. Insufficient information
D. 15.0

Answer: B

Explanation:
95% of the observations in a bell-shaped, symmetrical frequency distribution lie within
approximately 2 standard deviations of the mean. The standard deviation is then (45-30)/2 = 7.50.

Question: 1004

What monthly payment, beginning next month, is required over the next 60 months to pay off a
$10,000 debt today, if interest is charged at 14% per year, compounded monthly?

A. $116.02
B. $166.67
C. $2070.54
D. $1,400.54
E. $232.68

Answer: E

Explanation:
On the BAII Plus, press 60 N, 14 divide 12 = I/Y, 10000 PV, 0 FV, CPT PMT. On the HP12C, press 60 n,
14 ENTER 12 divide i, 10000 PV, 0 FV, PMT. Make sure the BAII Plus has the P/Y value set to 1.

Question: 1005

Page | 374
What value does the null hypothesis make a claim about?

A. None of these answers


B. Population parameter
C. Sample mean
D. Type II error
E. Sample statistic

Answer: B

Explanation:
The null hypothesis makes claims regarding the values that the population parameter may take.

Question: 1006

For a negatively skewed, unimodal distribution, which of the following relationships holds?

A. mode < median


B. mean > median
C. mean < median
D. mean > mode

Answer: C

Explanation:
In a negatively skewed distribution, very low values are more common than correspondingly large
values. This skews the distribution to the left, moving the mean to the left of the median.

Question: 1007

The research analyst for a stock brokerage firm wants to compare the dispersion in the price-earnings
ratios for a group of common stock with their return on investment. For the price-earnings ratios, the
mean is 10.9 and the standard deviation is 1.8. The mean return on investment is 25 percent and the
standard deviation is 5.2 percent. What is the relative dispersion for the price earnings ratios and
return on investment?

A. None of these answers


B. Ratios 50.0 percent, investment 10.0 percent
C. Ratios 132.0 percent, investment 190.0 percent
D. Ratios 32.0 percent, investment 19.0 percent
E. Ratios 16.5 percent, investment 20.8 percent

Answer: E

Explanation:
The respective CVs are found from (s*100)/mean. Ratios: 1.8*100/10.9 = 16.5% and investment:
5.2*100/25 = 20.8%.

Question: 1008

What semiannual deposit is needed to accumulate $5,000 in 5 years if the account pays 6% per year,
compounded semiannually, assuming that the first deposit is made in 6 months and also assuming
that the account already has $750 in it today?

Page | 375
A. $277.44
B. $436.15
C. $359.52
D. $348.23
E. $778.01

Answer: D

Explanation:
On the BAII Plus, press 10 N, 6 divide 2 = I/Y, 750 PV, 5000 +/- FV, CPT PMT. On the HP12C, press 10 n,
6 ENTER 2 divide i, 750 PV, 5000 CHS FV, PMT. Note that the answer this time is a positive number.
This means that the $348.23 is a deposit in addition to the $750 original deposit. The $5,000 is
entered as a negative number, because the $750 and the $348.23 are deposits and the $5,000 is a
withdrawal. Make sure the BAII Plus has the value of P/Y set to 1.

Question: 1009

A normal distribution has a mean of -10.8 and a variance of 723.6. The probability that a value from
this distribution will be between -25 and + 25 equals ________.

A. 0.91
B. 0.30
C. 0.61
D. 0.39

Answer: C

Explanation:
The standard deviation of the distribution equals sqrt(723.6) = 26.9. The z-value of -25 equals (-25 +
10.8)/26.9 = -0.53. The z-value of + 25 equals (25 + 10.8)/26.9 = + 1.33. Therefore, the probability
that a number Y lies between -25 and + 25 is the same as the z-score lying between -0.53 and + 1.33.
Using the Normal probability tables, we get P(z < 1.33) = 0.9082. Getting P(z < -0.53) can be tricky
when the table given is only for positive values. To calculate the probability, use the fact that the
standard normal distribution is symmetrical about zero. Hence, P(z < - 0.53) = P(z > 0.53) = 1.0 -
0.7019 = 0.2981. The probability that a number Y lies between -25 and + 25 is then given by P(-0.53 <
z < 1.33) = P(z < 1.33) - P(z < -0.53) = 0.9082 - 0.2981 = 0.61.

Question: 1010

If you buy an item for $1,500 and agree to pay for it with 36 monthly payments of $55.50, beginning
next month, what annual interest rate, compounded monthly, are you being charged?

A. 24.92%
B. 16.53%
C. 18.84%
D. 11.15%
E. 19.68%

Answer: E

Explanation:
The interest rate returned by the calculator will be the periodic interest rate. It must be multiplied by
the number of periods per year to have the correct answer. On the BAII Plus, press 36 N, 1500 PV,

Page | 376
55.50 +/- PMT, 0 FV, CPT I/Y. Then press x 12 = to see the answer. On the HP12C, press 36 n, 1500 PV,
55.50 CHS PMT, 0 FV, i. Then press 12 x to see the answer. Make sure the BAII Plus has the P/Y value
set to 1.

Question: 1011

The mean weight of trucks traveling on a particular section of I-475 is not known. A state highway
inspector needs an estimate of the mean. He selects a random sample of 49 trucks passing the
weighing station and finds the mean is 15.8 tons, with a standard deviation of the sample of 3.8 tons.
What is the 95 percent interval for the population mean?

A. 14.7 and 16.9


B. 16.1 and 18.1
C. 10.0 and 20.0
D. None of these answers
E. 13.2 and 17.6

Answer: A

Explanation:
Interval estimate can be found from x_bar +/-z*s/(n^0.5). Here we have n = 49, x_bar = 15.8 and z =
1.96 (for 95%) and s = 3.8. Therefore 15.8 +/- 1.96*3.8/7 and we get 14.76 and 16.84.

Question: 1012

A sample of size 100 is drawn from a population. The sample mean equals 35.2 and the variance of
the sample equals 47.8. The 95% confidence interval for the population mean is given by:

A. [33.13, 37.27]
B. [35.13, 35.27]
C. [35.06, 35.34]
D. [33.85, 36.56]

Answer: D

Explanation:
If z is the z-value corresponding to the specified confidence level, the sample mean is M and the
standard deviation is D in a sample size N, the confidence interval is specified as [M - z*D/sqrt(N), M
+ z*D/sqrt(N)]. In the present case, for the 95% confidence interval, z = 1.96. The sample standard
deviation equals sqrt(47.8) = 6.91. Therefore, the confidence interval equals [35.2 -
1.96*6.91/sqrt(100), 35.2 - 1.96*6.91/sqrt(100)] = [33.85, 36.56]

Question: 1013

Which of the following is correct about a probability distribution?

A. None of these answers


B. Sum of the probabilities of all possible outcomes must equal 1
C. Probability of each outcome must be between 0 and 1 inclusive
D. Outcomes must be mutually exclusive
E. All of these answers

Answer: E

Page | 377
Explanation:
All describes properties of a probability distribution.

Question: 1014

Suppose that events A, B, C, and D are independent, and have probabilities of 0.25, 0.50, 0.40, and
0.30, respectively. What is P(ABCD)?

A. 1.0%.
B. 15.0%.
C. 1.5%.
D. 0.15.

Answer: C

Explanation:
If events are independent, then the joint probability of them occurring together is just the product of
the individual probabilities. So P(ABCD) = 0.25 * 0.50 * 0.40 * 0.30 = 1.5%.

Question: 1015

If r = .65, what does the coefficient of determination equal?

A. 0.423
B. None of these answers
C. 0.577
D. 0.806
E. 0.194

Answer: A

Explanation:
The coefficient of determination is r-squared.

Question: 1016

A financial economist runs the following regression: Demand for cars = alpha + beta*income level +
error In this regression, the demand for cars is ________ variable and the income level is ________
variable.

A. dependent; dependent.
B. dependent; independent.
C. independent; independent.
D. independent; dependent.

Answer: B

Explanation:
The left-hand side variable is the dependent variable since its behavior in the equation is being
explained by the right-hand side variable, which is the independent variable. Note, however, that
this does not imply any kind of causal relationship in general.

Question: 1017

Page | 378
What is the measure that indicates how precise a prediction of Y is based on X or, conversely, how
inaccurate the prediction might be?

A. Least squares principle


B. Slope of the line
C. None of these answers
D. Y intercept
E. Regression equation

Answer: C

Explanation:
This refers to the standard error of estimates.

Question: 1018

You are examining a portfolio composed of 1/3 money-market investments, 1/3 bonds, and 1/3
stocks. Last year, the return on the money-market investments was 5%; the return on bonds was
15%, and the return on stocks was -2%. What is the portfolio's weighted-average return?

A. 6.66%.
B. 6.33%.
C. 7.00%.
D. 6.00%.

Answer: D

Explanation:
The portfolio weighted-average return is equal to the sum (as i goes from 1 to n) of w_i * X_i, where
w_i is the percentage weight in the portfolio of the ith asset, and X_i is the investment return of the
ith asset. Here, we get a weighted mean of 1/3 * 0.05 + 1/3 * 0.15 + 1/3 * -0.02 = 6.00%. Note that
because of the equal weighting, this is the same as the arithmetic return: (5% + 15% - 2%) / 3 = 18% /
3 = 6%.

Question: 1019

Which of the following is true in a normal distribution?

A. None of the answers


B. All of these answers are correct
C. Mean divides the distribution into two equal parts
D. Mean equals the mode and the median
E. Mode equals the median

Answer: B

Explanation:
All the above holds for a normal distribution.

Question: 1020

Events X and Y are mutually exclusive. P(X) = 0.15, P(Y) = 0.32. The probability of either X or Y
occurring equals ________.

Page | 379
A. 0.048
B. 0.53
C. 0.47
D. 0.17

Answer: C

Explanation:
Note the relationship, P(X or Y) = P(X) + P(Y) - P(X and Y) Also remember that for mutually exclusive
events, by definition, P(X and Y) = 0. Therefore, P(X or Y) = 0.15 + 0.32 = 0.47.

Question: 1021

What is the general form of the regression equation?

A. Y' = a x bX
B. Y' = a - bX
C. Y' = a + bX
D. Y' = ab
E. None of these answers

Answer: C

Explanation:
The regression is written as Y' = a + bX. The letter "a" is the Y intercept and b is the slope of the line.

Question: 1022

Which of the following is true regarding probability?


I. 0 < P(E) < 1: the probability of an event E is a number between 0 and 1, exclusive.
II. The sum of the probabilities of any group of mutually exclusive events equals 1.
III. 0 <= P(E) <= 1: the probability of an event E is a number between 0 and 1, inclusive.

A. III only.
B. I and II.
C. None of these answers is correct.
D. II and III.

Answer: A

Explanation:
III only is true: 0 <= P(E) <= 1: the probability of an event E is a number between 0 and 1, inclusive. A
probability can equal 1 or 0; thus, statement I is false. Statement II would only be true if it read: The
sum of the probabilities of any group of mutually exclusive and exhaustive events equals 1. Throwing
a 6-sided die can lead to 6 different outcomes, each of which has probability of 1/6. However, If your
group of mutually exclusive outcomes only included results 1 and 2, then the sum of those
probabilities is not 1. Such a group would not be exhaustive because it excludes the outcomes 3, 4, 5,
6.

Question: 1023

What annual interest rate, compounded quarterly, would cause a series of 30 quarterly deposits of
$500 to accumulate to $25,000, if the first deposit is made three months from today?

Page | 380
A. 3.31%
B. 12.78%
C. 9.94%
D. 13.22%
E. 8.15%

Answer: D

Explanation:
The value returned by the calculator will be the periodic interest rate which must be multiplied by
the number of periods per year to have the correct answer. On the BAII Plus, press 30 N, 0 PV, 500
PMT, 25000 +/- FV, CPT I/Y. Then press x 4 = to see the answer. On the HP12C, press 30 n, 0 PV, 500
PMT, 25000 CHS FV, i. Then press 4 x to see the answer. Make sure the BAII Plus has the P/Y value set
to 1.

Question: 1024

Production of passenger cars in Japan increased from 3.94 million in 1986 to 6.74 million in 1994.
What is the geometric mean annual percent increase?

A. 16.6
B. 1.9
C. 4.0
D. 47.3
E. 6.9

Answer: E

Explanation:
There are 9 years involved. The geometric mean = [(1 + 6.74/3.94)^1/8] - 1. In words, it is the 8th
square root of (1 + 6.74/3.94) minus 1. So we have GM = (1 + 1.7107)^(1/8) - 1 = 0.069 = 6.9%

Question: 1025

A very small p-value calculated in a sample implies that:

A. the null hypothesis should be rejected.


B. the null hypothesis is very likely true.
C. the alternate hypothesis is very likely false.
D. the null hypothesis should be accepted.

Answer: A

Explanation:
The lowest significance level at which the null hypothesis can be rejected is called the p-value of the
test. Thus, if the p-value is less than the significance level, the null hypothesis can be rejected at that
significance level.

Question: 1026

The results of the regressions using 200 observation on a variable Y against X are as follows:

CoefficientStandard error
intercept3.62.1

Page | 381
slope8.11.3

R square = 49%

If an observation on X indicates X = 5 the estimated value of Y would be ________.

A. 44.1
B. none of these answers
C. 26.1
D. 40.5

Answer: A

Explanation:
From the information given, the regression equation is Y = 3.6 + 8.1X + error term. The expected
value of the error is zero. Hence, when X = 5, the estimated value of Y equals 3.6 + 8.1*5 = 44.1.

Question: 1027

How much must you deposit today if you wish to have $50,000 in 20 years, assuming that interest
accumulates at 6% per year, compounded annually?

A. $2,500.00
B. $160,356.77
C. $12,299.08
D. $16,604.73
E. $15,590.24

Answer: E

Explanation:
On the BAII Plus, press 20 N, 6 I/Y, 0 PMT, 50000 FV, CPT PV. On the HP12C, press 20 n, 6 i, 0 PMT,
50000 FV, PV. Note that the answer is displayed as a negative number. Make sure the BAII Plus has
the P/Y value set to 1.

Question: 1028

You have just received a sales brochure from a reputable investment firm which describes a financial
investment vehicle which pays $300 at the end of year 1, $500 at the end of year 2 and $100 at the
end of year 3. If you are looking for an annual return of 6%, what's the maximum amount you should
be willing to purchase this investment certificate for?

A. $797
B. $827
C. $900
D. $812

Answer: D

Explanation:
The maximum amount you should pay equals the present value of the payments (you would like to
pay less!). This equals 300/1.06 + 500/(1.06^2) + 100/(1.06^3) = $812.

Question: 1029

Page | 382
You are given a risk-free rate of 3% per year, a portfolio return of -11% per year, and a standard
deviation of portfolio return is 20% per year. What is the Sharpe measure of risk-adjusted
performance?

A. -0.70.
B. -0.15.
C. -0.67.
D. -0.55.

Answer: A

Explanation:
The Sharpe measure of risk-adjusted performance is equal to (rbar_p - rbar_f)/sigma_p, where
rbar_p is the mean portfolio return, rbar_f is the mean risk-free return, and sigma_p is the standard
deviation of portfolio return. In our case, we have (-11% - 3%) / 20% = -14/20 = -0.7.

Question: 1030

If the underlying theory suggests that the value of an estimated variable is less than a particular
number, it is appropriate to use:

A. a right-tailed test.
B. a left-tailed test.
C. a two-tailed test.
D. either a right-tailed or left-tailed test.

Answer: B

Explanation:
When an observation is made and we see a value less than the threshold suggested by the theory,
we would like to known the probability of getting the smaller value due simply to chance rather than
to the event being driven by the theory. For this, we need to known the left-tail probabilities under
the null hypothesis.

Question: 1031

If you owe 3 debts ($100 due 6 months from now, $500 due 8 months from now, and $1,000 due 11
months from now), what single payment can you make today to settle them, if interest is assessed at
10% per year, compounded monthly?

A. $1,600.00
B. $1,475.78
C. $1,552.39
D. $1,387.29
E. $1,504.88

Answer: B

Explanation:
Find the answer to this question by solving 3 compound interest problems. On the BAII Plus, press 6
N, 10 divide 12 = I/Y, 0 PMT, 100 FV, CPT PV, which yields $95.14. Then press STO 1. Then press 8 N,
500 FV, CPT PV, which yields $467.88. Then press + RCL 1 = STO 1. Then press 11 N, 1000 FV, CPT PV,
which yields $912.76. Press + RCL 1 = to see the answer. On the HP12C, press 6 n, 10 ENTER 12 divide

Page | 383
i, 0 PMT, 100 FV, PV. Then press STO 1. Then press 8 n, 500 FV, PV. Then press RCL 1 + STO 1. Then
press 11 n, 1000 FV, PV. Press RCL 1 + to see the answer. Make sure the BAII Plus has the P/Y value set
to 1.

Question: 1032

What is the level of significance?

A. None of these answers


B. Probability of a Type II error
C. Beta error
D. Z-vale of 1.96
E. Probability of a Type I error

Answer: E

Explanation:
The level of significance and the type I error are both the probability of rejecting the null hypothesis
when it is actually true.

Question: 1033

What can we say if the coefficient of determination is 0.94?

A. Direction of relationship is positive


B. 94% of total variation in one variable is explained by variation in another variable
C. All of these answers are correct
D. Strength of relationship is 0.94
E. None of these answers is correct

Answer: B

Explanation:
We cannot tell anything about the direction of the relationship from the coefficient of determination
or r- squared.

Question: 1034

What is the Net Present Value of this series of annual cash flows using an interest rate of 15% per
year: Year 0: <$25,000>, Year 1: $5,000, Year 2: $0, Year 3: $30,000? (Note that the <> are used to
indicate a negative number).

A. $14.37
B. <$926.69>
C. $258.11
D. <$291.21>
E. <$204.58>

Answer: B

Explanation:
On the BAII Plus, press CF 2nd CLRWork 25000 +/- ENTER DownArrow 5000 ENTER DownArrow
DownArrow 0 ENTER DownArrow DownArrow 30000 ENTER DownArrow DownArrow 2nd Quit. Then
press NPV 15 ENTER DownArrow CPT. On the HP12C, press these keys: 25000 CHS BlueShift CFo 5000

Page | 384
BlueShift CFj 0 BlueShift CFj 30000 BlueShift CFj Then press 15 i, YellowShift NPV. The answer is
negative, indicating that the PV of the cash flows to be received is less than the original cash outflow.
The "DownArrow" represents the downward-pointing arrow on the top row of the BAII Plus
keyboard. Make sure the BAII Plus has the P/Y value set to 1.

Question: 1035

A researcher has a sample of 600 observations from a population whose standard deviation is known
to be 51.44. The mean of the sample is calculated to be 3.6. The null hypothesis is stated as Ho:
mean > 8.4. The researcher should

A. accept the null at 95% significance.


B. reject the null at 99% significance.
C. fail to reject the null at 95% significance.
D. none of these answers.

Answer: B

Explanation:
To test the hypothesis, you need to calculate the largest z-statistic since the null hypothesis is
unidirectional and to the right. This makes it the hardest to reject the null and you should always use
the most stringent criterion for rejecting the null. After all, the null is the hypothesis maintained to
be true by default and only a sufficient weight of evidence should be used to reject that view. The
largest z-statistic calculated under the null hypothesis is equal to (3.6 - 8.4)/2.1 = -2.4. Now, the
alternative hypothesis is H1: mean <= 8.4. Since the alternative is directional, we use a one-tailed
test. For this, the critical value at 95% level is -1.645 and for 99%, it is -2.32. -2.32 is the value below
which only 1% of the probability mass of the standard normal distribution lies. Similarly, only 5% of
probability lies below -1.645). Since even the calculated z-statistic that is most favorable to the null
lies in the rejection region, we can reject the null at the 99% level in a one-sided test.

Question: 1036

If you make 22 deposits of $33 with the first deposit occurring next month, how much do you have
immediately after the last deposit, if the account earns 4.44% per year, compounded annually?

A. $666.00
B. $1,207.81
C. $$457.44
D. $1,313.51
E. $1,189.62

Answer: E

Explanation:
On the BAII Plus, press 22 N, 4.44 I/Y, 0 PV, 33 PMT, CPT FV. On the HP12C, press 22 n, 4.44 i, 0 PV, 33
PMT, FV. Note that the answer will be displayed as a negative number. Make sure the BAII Plus has
the P/Y value set to 1.

Question: 1037

As the alternative mean approaches the hypothesized mean, what can we say about the risk?

A. Smaller risk of a Type II error


B. Greater risk of a Type II error

Page | 385
C. Smaller risk of a Type I error
D. None of these answers
E. Greater risk of a Type I error

Answer: B

Explanation:
When the alternative mean approaches the hypothesized mean, there is a greater possibility of
accepting the null when it is actually false since they are close together.

Question: 1038

Chances are 50-50 that a newborn baby will be a girl. For families with five children, what is the
probability that all the children are girls?

A. 0.100
B. None of these answers
C. 0.250
D. 0.001
E. 0.031

Answer: E

Explanation:
This is a binomial probability. The probability of getting r successes out of n trials where the
probability of success each trial is p and probability of failure each trial is q (where q = 1-p) is given
by: n!(p^r)[q^(n-r)]/r!(n-r)!. Here n = 5, r = 5,p = 0.5 and q = 0.5. Therefore we have
5!(0.5^5)(0.5^0)/5!0! = 0.03125.

Question: 1039

You are given a risk-free rate of 5% per year, a portfolio return of 15% per year, and a standard
deviation of portfolio return of 16% per year. What is the Sharpe measure of risk-adjusted
performance?

A. 0.625.
B. 0.675.
C. 0.500.
D. 0.750.

Answer: A

Explanation:
The Sharpe measure of risk-adjusted performance is equal to (rbar_p - rbar_f)/sigma_p, where
rbar_p is the mean portfolio return, rbar_f is the mean risk-free return, and sigma_p is the standard
deviation of portfolio return. In our case, we have (15% - 5%) / 16% = 10/16 = 5/8 = 0.625.

Question: 1040

An analysis of the grades on the first test in History 101 revealed that they approximate a normal
curve with a mean of 75 and a standard deviation of 8. The instructor wants to award the grade of A
to the upper 10 percent of the test grades. What is the dividing point between an A and a B grade?

A. 80

Page | 386
B. 90
C. 85
D. 95
E. None of these answers

Answer: C

Explanation:
First we need to find the z value for an area under the curve representing 40%. (This is because we
are looking for the top 10%). From the table z = 1.282. Therefore using the z formula we have 1.282 =
X-75/8. We find that x = 85.25

Question: 1041

If you owe $5,000 today and will repay it with 8 annual payments of $900 beginning next year, what
is the annual interest rate, compounded annually, that you are paying?

A. 9.80%
B. 9.98%
C. 18%
D. 8.90%
E. 44%

Answer: D

Explanation:
On the BAII Plus, press 8 N, 5000 PV, 900 +/- PMT, 0 FV, CPT I/Y. On the HP12C, press 8 n, 5000 PV,
900 CHS PMT, 0 FV, i. Note that the PMT must be entered with a sign that is opposite that given the
PV.

Question: 1042

Distributions X and Y have the same coefficients of variation. If the mean of X is twice the mean of Y
and the standard deviation of X is 5, the standard deviation of Y is ________.

A. 5
B. 10
C. 2.5
D. 2

Answer: C

Explanation:
coefficient of variation = standard deviation/mean. If the mean of Y is M, then the coefficient of
variation of X is 5/(2M) and that of Y is (standard deviation of Y)/M. Given that these two are equal,
5/(2M) = (standard deviation of Y)/M. Therefore, standard deviation of Y = 5/2 = 2.5.

Question: 1043

Consider the following list of numbers:


6, 4, 3, 8, 3, 3, 5, 9, 11, 4
For this list, the mean, the median and the mode are given respectively by:

A. 5.6, 4.5, 3

Page | 387
B. 5.6, 4, 3
C. 5, 3, 5
D. 5.6, 6, 3

Answer: A

Explanation:
The sum of the 10 numbers is 56. The mean is 56/10 = 5.6. The most common number is 3, which is
the mode. Since there is an even number (10) of numbers in the list, the median is the average of the
5th and 6th numbers when the list is arranged in an ascending order. The median is therefore equal
to (4 + 5)/2 = 4.5.

Question: 1044

Suppose you were given $4,000 today and deposited it into an account paying 8% per year,
compounded monthly. If you know that you will need $5,000 in the account 10 years from now, what
monthly withdrawal can you make from the account, beginning one month from now, that will leave
the account with exactly $5,000 in it in 10 years?

A. $24.45
B. $21.20
C. $23.45
D. Can't be done
E. $11.23

Answer: B

Explanation:
On the BAII Plus, press 120 N, 8 divide 12 = I/Y, 4000 PV, 5000 +/- FV, CPT PMT. On the HP12C, press
120 n, 8 ENTER 12 divide i, 4000 PV, 5000 CHS FV, PMT. Note that the answer is a negative number,
indicating that this is a monthly withdrawal that can be made from the account in addition to the
$5,000 in 10 years. Make sure the BAII Plus has the value of P/Y set to 1.

Question: 1045

Which of the following is/are true?


I. Doubling every single value in a sample doubles the sample median.
II. Adding a constant to each value in a sample causes the median to increase by the same constant.
III. The median of a sample decreases as the number of observations increases.

A. II only
B. I only
C. I only
D. I & III
E. II only
F. III only
G. I & II

Answer: G

Explanation:
The median of a set with odd number of observations equals the "middle" observation. If the
number of observations is even, the median is the mean of the two middle observations. From this, I
& II follow. (III) is not necessarily true.

Page | 388
Question: 1046

A discrete probability distribution is the same as:

A. a collection of all possible outcomes of an experiment.


B. an absolute frequency distribution.
C. a relative frequency distribution.
D. none of these answers.

Answer: C

Explanation:
The relative probability distribution of a set shows the relative frequency in each category in which
the observation set is subdivided i.e. it shows the percentage of the observation set lying in each
class.
This also represents the probability distribution of the observation set.

Question: 1047

In the regression equation, what does the letter "b" represent?

A. Slope of the line


B. Value of Y when X=0
C. Any value of the independent variable that is selected
D. Y concept
E. None of these answers

Answer: A

Explanation:
The regression is written as Y' = a + bX. The letter "a" is the Y intercept and b is the slope of the line.

Question: 1048

How many years will it take money to double at 7.25% per year, compounded annually?

A. 7.15 years
B. 12.14 years
C. 7.29 years
D. 8.03 years
E. 9.90 years

Answer: E

Explanation:
To solve this question, choose a future value that is twice the size of the present value. The actual
number will not matter. On the BAII Plus, press 7.25 I/Y, 1000 PV, 0 PMT, 2000 +/- FV, CPT N. On the
HP12C, press 7.25 i, 1000 PV, 0 PMT, 2000 CHS FV, n. Note that the HP12C will show 10 for the
answer. The answer will be the same as long as the FV is twice as large as the PV. Make sure that the
BAII Plus has the P/Y value set to 1.

Question: 1049

Page | 389
Which of the following is/are true about a normal distribution?
I. It has only one mode.
II. It has a positive third moment.
III. It is symmetrical about zero.

A. III only
B. I, II and III
C. II only
D. I & IV
E. I only
F. I & III

Answer: E

Explanation:
A normal distribution is symmetrical about the mean, which can be non-zero. It has a zero third
moment (zero skewness).

Question: 1050

An automobile manufacturer makes 2 different chassis, each of which can have 3 different body
types and 8 colors. How many different looking cars can be made with these choices?

A. 40.
B. 24.
C. 13.
D. 48.

Answer: D

Explanation:
The multiplication rule of counting states that the number of combinations available when there are
n_1 options in one aspect, n_2 in another, and so on, up to n_k, is n_1 * n_2 * ... * n_k. In this case,
the number of combinations is 2*3*8 = 48.

Question: 1051

A linear regression is ________ relationship.

A. a causal
B. an acausal
C. a functional
D. none of these answers

Answer: C

Explanation:
It is important to remember that a regression equation does not imply a causal relationship between
the dependent and the independent variables. It simply postulates a functional i.e. an associative
relationship between them.

Question: 1052

John was offered a salary of $25,000 when he graduated from school 15 years ago. He was somewhat

Page | 390
depressed to read that the average starting salary for someone with his degree today was $45,000.
He then decided to determine the annual, compound rate of increase implied in the increase of the
starting salary from $25,000 to $45,000 in 15 years. What is this annually compounded growth rate?

A. 4.00%
B. 7.15%
C. 1.03%
D. 8.42%
E. 12.00%

Answer: A

Explanation:
Essentially, this question is a compound interest problem with the interest rate as the unknown
value. On the BAII Plus, press 15 N, 25000 PV, 0 PMT, 45000 +/- FV, CPT I/Y. On the HP12C, press 15 n,
25000 PV, 0 PMT, 45000 CHS FV, i. John would seem to have no reason to be depressed, since the
starting salary today has increased by only 4% per year over the last 15 years, which might be
somewhat near the past rate of inflation. Make sure the BAII Plus has the P/Y value set to 1.

Question: 1053

Which of the following is NOT a reason for sampling the population?

A. Some tests are destructive in nature and hence, should be carried out only on small samples.
B. Inferences from a sample are the same that would be obtained by studying the entire population.
C. It is almost impossible to work with the entire population in most cases.
D. Samples are far easier to work with than the entire population.

Answer: B

Explanation:
Remember, the entire population contains more information than the sample. With the population,
you would get exact parameter values characterizing the population whereas with a sample, you only
get estimates.

Question: 1054

A statistician has framed his hypothesis testing problem as:


Ho: mean >= 1 H1: mean < 1
For the given sample, he calculates the z-statistic. Then, the region of rejection at the 95% level is
given by:

A. z-statistic > +1.96


B. z-statistic < -1.645
C. -1.96 < z-statistic < +1.96
D. z-statistic < -1.96

Answer: B

Explanation:
Since the alternative hypothesis is directional and to the left, the rejection region for the null is also
to the left. Hence, you should use a one-tailed test. 5% of the probability mass in a Normal
distribution lies in the left tail with values which are less than -1.645. Therefore, the rejection region
comprises of z-values less than -1.645.

Page | 391
Question: 1055

On a survey questionnaire, students were asked to indicate their class rank in college. If there were
only four choices from which to choose, which measure(s) of central tendency would be appropriate
to use for the data generated by that questionnaire item?

A. Mean and mode


B. Mode only
C. Mode and median
D. Median only
E. Mean and median

Answer: C

Explanation:
We can discard the mean since it is not appropriate here. If we want to find out which class has the
most students or where the students are most frequently ranked, then the mode and median should
be used.

Question: 1056

What annual deposit would you need to make, beginning in one year, into an account paying 7% per
year, compounded annually, in order to have $50,000 in it after 20 years, assuming that the account
has nothing in it today?

A. $1,129.56
B. $1,219.65
C. $4,719.65
D. $1,291.56
E. $2,191.65

Answer: B

Explanation:
On the BAII Plus, press 20 N, 7 I/Y, 0 PV, 50000 FV, CPT PMT. On the HP12C, press 20 n, 7 i, 0 PV,
50000 FV, PMT. Note that the answer will be displayed as a negative number.

Question: 1057

An investment of $100 grows in five years to $205. The investor observes that the annual arithmetic
rate of return and the geometric rate of return were the same over this period. The annual
arithmetic rate of return must be ________.

A. 4.32%
B. 15.44%
C. 17.84%
D. 13.93%
E. 8.33%

Answer: B

Explanation:
If the annual geometric rate of return is r, then 100 * (1 + r)^5 = 205. This gives r = 15.44%. Note that

Page | 392
the only way the mean will be equal to the geometric mean if every year, the stock experienced a
return of 15.44% per year.

Question: 1058

What is the present value today of these annual cash flows: $1,000, $2,000, $3,000? Assume the first
cash flow occurs 1 year from today and an interest rate of 10% per year, compounded annually.

A. $2,304.67
B. $2,754.32
C. $4,815.93
D. $4,000.00
E. $4,104.98

Answer: C

Explanation:
You could solve this question using 3 different compound interest problems, but it is easier to solve
them using the calculator's cash flow functions. On the BAII Plus, press CF 2nd CLRWork 0 ENTER
DownArrow 1000 ENTER DownArrow DownArrow 2000 ENTER DownArrow DownArrow 3000 ENTER
DownArrow DownArrow 2nd Quit. Then press NPV 10 ENTER DownArrow CPT. On the HP12C, press
these keys: 0 BlueShift CFo 1000 BlueShift CFj 2000 BlueShift CFj 3000 BlueShift CFj. Then press 10 i,
YellowShift NPV. The "DownArrow" represents the downward-pointing arrow on the top row of the
BAII Plus keyboard. Make sure that the BAII Plus has the P/Y value set to 1.

Question: 1059

You are given a risk-free rate of 7% per year, a portfolio return of 19% per year, and a standard
deviation of portfolio return of 12% per year. What is the Sharpe measure of risk-adjusted
performance?

A. 1.111.
B. 0.583.
C. 1.000.
D. 1.583.

Answer: C

Explanation:
The Sharpe measure of risk-adjusted performance is equal to (rbar_p - rbar_f)/sigma_p, where
rbar_p is the mean portfolio return, rbar_f is the mean risk-free return, and sigma_p is the standard
deviation of portfolio return. In our case, we have (19% - 7%) / 12% = 12/12 = 1.000.

Question: 1060

A normal distribution has a mean of 4.1 and a standard deviation of 2.3. The probability that a value
from this distribution will be negative equals ________.

A. 0.0206
B. 0.0375
C. 0.0121
D. 0.4625

Answer: B

Page | 393
Explanation:
The z-value of a selected observation, X, from a normal distribution with mean M and standard
deviation S equals z = (X-M)/S. in the present case, the z-value of X = 0 equals -4.1/2.3 = -1.78. Then,
the probability that a number Y is negative is the same as the probability of a z-value less than -1.78.
Using the Normal probability tables to get P(z < -1.78) is tricky because the table given in the
assigned text is only for positive values. To calculate the probability, use the fact that the standard
normal distribution is symmetrical about zero. Hence,
P(z < -1.78) = P(z > + 1.78) = 1.0 - 0.9625 = 0.0375.

Question: 1061

A study of the opinion of designers with respect to the primary color most desirable for use in
executive offices showed that:

Primary ColorNumber of Opinions


Red92
Orange86
Yellow46
Green91
Blue37
Indigo46
Violet2

What is the probability that a designer does not prefer blue?

A. None of these answers


B. 0.9075
C. 1.0000
D. 0.8850
E. 0.7725

Answer: B

Explanation:
P(Blue) = 37/400. P(not blue) = 1 - 37/400 = 0.9075.

Question: 1062

If you deposit $2,345 into an account paying 6.5% per year simple interest, how much interest will be
earned after 8 months?

A. $101.62
B. $14.09
C. $309.68
D. $94.64
E. $122.65

Answer: A

Explanation:
The question only asks for the earned interest, not the total amount in the account. On the BAII Plus,
press 2345 x 0.065 x 8 divide 12 = to see the answer. On the HP12C, press 2345 ENTER 0.065 x 8 x 12
divide to see the answer.

Page | 394
Question: 1063

If you deposit $1,400 today into a savings account paying 8% per year, compounded semiannually,
how much is in your account in 5 years?

A. $2,072.34
B. $1,587.85
C. $2,487.15
D. $1,330.55
E. $1,857.58

Answer: A

Explanation:
On the BAII Plus, press 10 N, 8 divide 2 = I/Y, 1400 PV, 0 PMT, CPT FV. On the HP12C, press 10 n, 8
ENTER 2 divide i, 1400 PV, 0 PMT, FV. Note that the answer will be displayed as a negative number.
Make sure the BAII Plus has the P/Y value set to 1.

Question: 1064

The sum of the squares of 65 observations equals 862. The sum of the observations equals 93. The
sample standard deviation of the observations equals ________.

A. 11.21
B. 1.43
C. 3.37
D. 4.96

Answer: C

Explanation:
For N observations, it is easy to show that
sample variance*(N-1) = (sum of squares) - N*(mean^2)
Hence, in this case, sample variance = (862 - 65*(93/65)^2)/64 = 11.39. The standard deviation then
equals sqrt(11.39) = 3.37
Note: You should be careful about the difference between population variance and sample variance.
The formula for population variance is:
population variance*N = (sum of squares) - N*(mean^2)
You can expect an exam question which asks for population variance, with the choices given
containing both the population and the sample variances or vice versa.

Question: 1065

Which of the is/are true?


I. The probability of type II error equals 1 - significance level.
II. A higher significance level is makes it easier to reject a null hypothesis.
III. Minimizing the chance of a Type I error minimizes the probability of Type II error. IV. The higher
the probability of Type II error, the higher is the chance that the alternative will be accepted when it
is true.

A. II & IV
B. IV only
C. I & IV

Page | 395
D. III & IV
E. II only
F. I only
G. I, II & IV
H. III only

Answer: A

Explanation:
The probability of Type I (not type II) error equals 1 - the significance level.
The significance level represents an upper bound on the probability that the null hypothesis is true
given the observed sample. The higher this level is set, the easier it is to say that the null is false
(though the probability that you are making a mistake in rejecting the null also becomes higher!).
Type I and Type II errors represent two different types of errors and are not directly related. A
relationship like (III) appears tempting but is not true.
For the purposes of CFA Level I exam, (IV) can be taken to be true, though technically, it is not
entirely accurate. It holds only if the alternative hypothesis is exactly complementary to the null
hypothesis i.e. the null hypothesis and the alternative hypothesis span the entire range of values that
the variable being tested can take. If you set up the alternative hypothesis incorrectly, then rejection
of the null does not necessarily imply that the alternative is true; it could also imply that you have
not taken all the possibilities into consideration. For e.g., suppose a theory does not rule out the
possibility that a variable X can be negative but you mistakenly set up the hypotheses as Ho: X = 0,
H1: X > 0. Then clearly, even if you reject the null hypothesis, it does not imply that X can take only
positive values. Recognizing such mistakes in setting up a hypothesis test is crucial.

Question: 1066

What single deposit could you make today in order to have $1,000,000 in 30 years, assuming it earns
interest at 11% per year, compounded monthly?

A. $403,512.59
B. $115,024.60
C. $9,523.23
D. $43,682.82
E. $37,441.83

Answer: E

Explanation:
On the BAII Plus, press 360 N, 11 divide 12 = I/Y, 0 PMT, 1000000 FV, CPT PV. On the HP12C, press 360
n, 11 ENTER 12 divide i, 0 PMT, 1000000 FV, PV. Note that the answer is displayed as a negative
number. Make sure the BAII Plus has the value of P/Y set to 1.

Question: 1067

The probabilities and the number of automobiles lined up at a Lakeside Olds at opening time (7:30
a.m.) for service are:

NumberProbability
10.05
20.30
30.40
40.25

Page | 396
On average, how many automobiles should Lakeside Olds expect to be lined up at opening?

A. None of these answers


B. 1.
C. 1.96
D. 10.
E. 2.85

Answer: E

Explanation:
1*0.05 + 2*0.30 + 3*0.40 + 4*0.25 = 2.85.

Question: 1068

A 95% confidence interval for a population parameter signifies which of the following?
I. 95% of similarly constructed intervals will contain the population parameter.
II. For a given sample size, 95% of the samples will have the sample statistic for the population
parameter lie within the specified confidence interval around the actual population parameter.
III. The confidence interval will correctly estimate the population parameter with a probability of
95%

A. III only
B. I, II & III
C. I only
D. II only
E. I & II

Answer: E

Explanation:
Note that III is incorrect since the confidence interval is a range estimate and hence, cannot
represent a point estimate of the population parameter itself.

Question: 1069

The probability that a mutual fund will generate a positive return in the next 12 months is called:

A. a conditional probability.
B. an unconditional probability.
C. a joint probability.
D. a fantasy.

Answer: B

Explanation:
An unconditional probability takes the form of P(A), the probability that an event (A) will happen. It is
unconditional because it is not conditioned on any other event.

Question: 1070

For a positively skewed, unimodal distribution, which of the following relationships holds?

A. mean < mode

Page | 397
B. mode < median
C. mean > median
D. mean < median

Answer: C

Explanation:
In a positively skewed distribution, very large values are more common than correspondingly small
values. This skews the distribution to the right, moving the mean to the right of the median.

Question: 1071

If alpha = .05 for a two-tailed test, how large is the acceptance area?

A. .975
B. None of these answers
C. .950
D. .025
E. .050

Answer: C

Explanation:
The acceptance region is the area in between the two critical values. In this case it is 1 - 0.05 = 0.95.

Question: 1072

Which of the is/are false?


I. A high p-value is necessary to reject a null hypothesis.
II The higher the significance level, the greater is the chance that the null will be rejected when it is
false.
III. The higher the probability of Type II error, the higher is the chance of rejecting the null when it is
false.

A. II & III
B. I, II & III
C. I only
D. II only
E. III only
F. I & II

Answer: B

Explanation:
You can think of the p-value as the maximum probability that the null hypothesis is true despite
observing the value of the test statistic that you have in the sample at hand. Thus, the lower the p-
value, the greater is your confidence in rejecting the null hypothesis. The significance level
represents an upper bound on the probability that the null hypothesis is true given the observed
sample and the testing procedure. Hence, if you reject the null at the 5% significance level, for e.g.,
then the probability that the null is true despite your statistical evidence to the contrary could be as
high as 5% (but no more, under the assumptions of the test). A Type II error refers to the event that
we will fail to reject the null when it is false. The higher the probability of a Type II error, the lower
the chance of rejecting the null when it is false.

Page | 398
Question: 1073

Mr. and Mrs. Jones live in a neighborhood where the mean family income is $45,000 with a standard
deviation of $9,000. Mr. and Mrs. Smith live in a neighborhood where the mean is $100,000 and the
standard deviation is $30,000. What is the relative dispersion of the family incomes in the two
neighborhoods?

A. Jones 20%, Smith 30%


B. Jones 50%, Smith 33%
C. None of these answers
D. Jones 40%, Smith 20%
E. Jones 30%, Smith 20%

Answer: A

Explanation:
The coefficient of variation = (s*100)/mean. Jones: 9000*100/45,000 = 20%. Smith:
30,000*100/100,000 = 30%.

Question: 1074

You have a portfolio of two assets, X and Y. The returns of X and Y follow a joint probability function
as follows: There is a 25% chance that X will return 16% and Y will return 10%; there is a 60% chance
that X will return 9% and Y will return 7%; and there is a 15% chance that Y will return 15% and X will
return 5%. Find the covariance of X and Y.

A. 3.46%%
B. 1.79%%
C. -3.46%%
D. -1.79%%

Answer: D

Explanation:
For a joint probability function, the covariance of X and Y is given by the formula: double summation
(over all X, and over all Y) of P(X,Y)*[X - E(X)]*[Y - E(Y)]. E(X) will be 25% * 16% + 60% * 9% + 15% *
5% = 4% + 5.4% + 0.75% = 10.15%. Similarly, we will find that E(Y) = 8.95%. Next, we plug these
values into our formula and obtain 25% * (16% - 10.15%) *(10% - 8.95%) + 60% (9% - 10.15%) * (7% -
8.95%) + 15% (5% - 10.15%) * (15% - 8.95%) = -1.79%%.

Question: 1075

Which is true of positively skewed distributions?


I. They are not symmetrical.
II. Their mean is larger than their median.
III. They are characterized by many small values and a few extreme values.

A. I and II
B. I and III
C. None of these answers is correct.
D. II and III

Answer: C

Page | 399
Explanation:
All are true of positively skewed distributions, so the answer is I, II, and III: None of the above is the
correct choice.

Question: 1076

If you want $1,000,000 when you retire in 25 years, how much must you deposit each month,
beginning one month from today, if your funds will earn 9% per year, compounded monthly?

A. $7,336.35
B. $903.59
C. $$29,848.16
D. $891.96
E. $29,848.16

Answer: D

Explanation:
Note that since the deposits are made monthly, there are 300 of them (25 x 12 = 300). On the BAII
Plus, press 300 N, 9 divide 12 = I/Y, 0 PV, 1000000 FV, CPT PMT. On the HP12C, press 300 n, 9 ENTER
12 divide i, 0 PV, 1000000 FV, PMT. Note that the answer will be displayed as a negative number.
Make sure the BAII Plus has the value of P/Y set to 1.

Question: 1077

If alpha equals 0.05 for a two-tailed hypothesis test, how large is the area of rejection in the upper or
right tail?

A. 0.975
B. 0.025
C. 0.05
D. 0.95
E. None of these answers

Answer: B

Explanation:
If it is a two tailed test, then there is a similar size region in each tail. So 0.05/2 = 0.025.

Question: 1078

Two sets of observations, A and B, each contain 160 observations. The variance of data in A equals
198.4 and that of the data in B equals 231.9. The two sets have the same mean. Then, A has a
________ dispersion and a ________ coefficient of variation.

A. higher; higher
B. lower; higher
C. lower; lower
D. higher; lower

Answer: C

Explanation:
B has a higher variance and hence, a higher dispersion. The coefficient of variation equals the

Page | 400
standard deviation divided by the mean. Since A and B have the same mean, B has a higher
coefficient of variation.

Question: 1079

If the null hypothesis that two means are equal is in fact true, where will 97% of the computed z-
value lie between?

A. +/- 2.07
B. +/- 2.33
C. +/- 2.17
D. none of these answers
E. +/- 2.58

Answer: C

Explanation:
From the z tables, we can find 0.97/2 = 0.485, which corresponds to a z-value of +/-2.17.

Question: 1080

The distribution of the annual incomes of a group of middle management employees approximated a
normal distribution with a mean of $37,200 and a standard deviation of $800. About 68 percent of
the incomes lie between what two incomes?

A. $36,400 and $38,000


B. None of these answers
C. $34,800 and $39,600
D. $35,600 and $38,800
E. $30,000 and $40,000

Answer: A

Explanation:
68/2 = 0.34. The z value for 0.34 is 1. x = u +/- z*sigma. So x = 37200 +/- 1*800. e get x = 38,000 and
36,400.

Question: 1081

What is the future value of $750 per year for 6 years, with the first cash flow occurring today, rather
than 1 year from now? Assume interest is 7% per year, compounded annually.

A. $5,364.97
B. $4,500.00
C. $5,740.52
D. $6,209.48
E. $5,293.55

Answer: C

Explanation:
Recognize that this question is an annuity due situation, since the first cash flow occurs immediately
or at the beginning of each period. Annuities where the first payment occurs 1 period from today (or
at the end of each period) are called "ordinary" annuities. This requires placing the calculator into

Page | 401
"Begin" mode prior to solving the question. NOTE: Be sure to place the calculator OUT OF annuity
due mode after this question before going on to subsequent questions, or you will get the wrong
answers! On the BAII Plus, press 2nd BGN. If the display shows END, then press 2nd SET and then 2nd
Quit. This will place the BAII Plus into annuity due mode (you can tell this because the BAII Plus will
display BGN in small letters). Now press 6 N, 7 I/Y, 0 PV, 750 PMT, CPT FV. Place the calculator back
into End mode (for ordinary annuities) by pressing 2nd BGN and then if the calculator is displaying
BGN, press 2nd SET and 2nd Quit. The BGN letters should disappear from the display. On the HP12C,
press BlueShift END, which is the blue function on the front of the 8 digit key. This places the HP12C
into Begin mode (the HP12C shows the word BEGIN in the display when in this mode). Then press 6
n, 7 i, 0 PV, 750 PMT, FV. To place the HP12C back into ordinary annuity mode (or END mode), press
BlueShift BEG (the blue function written on the front of the 7 digit key). Note that the answer is
displayed as a negative number. Make sure the BAII Plus has the value of P/Y set to 1.

Question: 1082

In computing kurtosis, which of the following is true?


I. It raises the deviation from the mean to the fourth power.
II. The formula preserves the direction of the deviation.
III. Kurtosis measures how clustered the values in a distribution are to the mean.

A. I, II, and III


B. None of these answers is correct.
C. II and III
D. I and III

Answer: D

Explanation:
I is true. II is false because the fourth power of a negative number is not a negative number: -2 * -2 *
-2 * -2 = +16. III is true.

Question: 1083

An investor has a required rate of return of 5% per year. How much will he pay for a seven-year
ordinary annuity that pays $200 per year?

A. $1,093
B. $1,157
C. $1,215
D. $1,213

Answer: B

Explanation:
The value of the annuity = (200/0.05)*(1- 1/(1.05^7)) = 1,157

Question: 1084

If the underlying theory does not suggest a particular direction for the value of an estimated variable,
it is appropriate to use a:

A. a right-tailed test.
B. a left-tailed test.
C. a two-tailed test.

Page | 402
D. either a right-tailed or left-tailed test.

Answer: C

Explanation:
If there is no suggested directionality to the underlying variable, then you are interested in finding
the probability under the null hypothesis that you will find a value beyond a given distance from the
null value. For this, you need to use a two-tailed test.

Question: 1085

In a binomial probability distribution,

A. there are 2 distinct modes.


B. the probability graph is bell-shaped and symmetrical.
C. each trial can only take values between 0 and 1.
D. each trial can take only two values.

Answer: D

Explanation:
A binomial distribution with "N trials" arises when the same experiment is repeated N times. Each of
the trails is known as a "Bernoulli" trial. It is the Bernoulli trial which can only take two distinct
values, "success" and "failure." Each outcome of a binomial distribution with N trials is the string of N
values generated by the individual Bernoulli trials. Clearly, the binomial distribution with N trials can
take 2^N values, even though each individual trial can have only two outcomes. It is important that
you understand the difference between the binomial distribution and the underlying Bernoulli trials
themselves.

Question: 1086

Which of the following are notations for the covariance between X and Y?
I. Cov(X,Y).
II. sigma_(X,Y)
III. [sigma_(X,Y)]^2.

A. I only.
B. II only.
C. I and II.
D. I and III.

Answer: C

Explanation:
I and II are both notations for covariance between X and Y.

Question: 1087

A sample of size 225 is drawn from a population. The sample mean equals 876 and the variance of
the sample equals 5,924. The 85% confidence interval for the population mean is given by:

A. [866.2; 884.6]
B. [870.1; 882.2]
C. [868.6; 883.4]

Page | 403
D. [869.3; 882.7]

Answer: C

Explanation:
If z is the z-value corresponding to the specified confidence level, the sample mean is M and the
standard deviation is D in a sample size N, the confidence interval is specified as [M - z*D/sqrt(N), M
+ z*D/sqrt(N)]. In the present case, for the 85% confidence interval, the normal probability table
gives z = 1.44. The sample standard deviation equals sqrt(5924) = 76.97. Therefore, the confidence
interval equals [876 - 1.44*76.97/sqrt(225), 876 + 1.44*76.97/sqrt(225)] = [868.6, 883.4]

Question: 1088

The seasonal output of a new experimental strain of pepper plants was carefully weighed. The mean
weight per plant is 15.0 pounds, and the standard deviation of the normally distributed weights is
1.75 pounds. Of the 200 plants in the experiment, how many produced peppers weighing between
13 and 16 pounds?

A. 100
B. None of these answers
C. 118
D. 53
E. 197

Answer: C

Explanation:
z = (x-u)/sigma. z1 = 13 - 15/1.75 = -1.1429 and z2 = 16 - 15/1.75 = 0.5714. The respective areas for
those z-values are 0.3729 and 0.2157. Since they are on opposite sides of the mean, we add them up
to find the area in between which is 0.5886. Therefore, 0.5886*200 = 118.

Question: 1089

If you deposit $1,500 into an account paying 6% per year, compounded quarterly, how much is in the
account after 60 months?

A. $2,224.46
B. $2,117.54
C. $2,020.28
D. $1,702.48
E. $1,751.59

Answer: C

Explanation:
There are 20 quarters in 60 months (60 divide 3 =). On the BAII Plus, press 20 N, 6 divide 4 = I/Y, 1500
PV, 0 PMT, CPT FV. On the HP12C, press 20 n, 6 ENTER 4 divide i, 1500 PV, 0 PMT, FV. Note that the
answer will be displayed as a negative number. Make sure the BAII Plus has the value of P/Y set to 1.

Question: 1090

BeigeBox, a computer manufacturer, can construct computer systems with any one of 4 processors, 2
memory configurations, 3 hard drives, 2 monitors, 2 keyboards, and 2 CD drives. How many unique
configurations of computer systems can BeigeBox construct?

Page | 404
A. 192.
B. 96.
C. 384.
D. 64.

Answer: A

Explanation:
The multiplication rule of counting states that the number of combinations available when there are
n_1 options in one aspect, n_2 in another, and so on, up to n_k, is n_1
* n_2 * ... * n_k. In this case, the number of combinations is 4*2*3*2*2*2 = 192.

Question: 1091

In a beer taste study involving the ExtraFerm lager beer, the following responses were recorded:
ResponseFrequency
Great taste103
Okay taste46
Bad taste64
The probability that a randomly selected beer drinker will NOT find ExtraFerm to be bad tasting is
calculated from these data to be:

A. 0.22
B. 0.30
C. 0.70
D. 0.78

Answer: C

Explanation:
The probability that a randomly selected beer drinker will find ExtraFerm to be bad tasting is
estimated from the above data as 64/(64 + 46 + 103) = 0.3. Therefore, the probability that a
randomly selected beer drinker will NOT find ExtraFerm to be bad tasting equals 1 - 0.3 = 0.7. We
could also have calculated this probability as (103 + 46)/(64 + 46 + 103) = 0.7 Note that given a
frequency distribution, the estimated probability of a particular class/event occurring equals the
relative frequency of that class.

Question: 1092

A derivative security pays $100 if the Dow Jones Industrial Average shows a 15% return over the next
year and does not pay anything of the return is lower. If the security costs $45 and its expected return
is 30%, what's the probability that the Dow Jones return will exceed 15%?

A. 39.87%
B. 58.51%
C. 41.49%
D. 61.22%

Answer: B

Explanation:
If p is the probability of the Dow Jones return exceeding 15%, then the expected total return on the
derivative security is (p * 100 + (1-p)*0)/45 = 2.222p. Since the expected return is given to be 30%,

Page | 405
we get 2.22p = 1.3. Therefore, p = 1.3/2.222 = 58.51%

Question: 1093

A bottling company offers three kinds of delivery services-instant, same day and within five days. The
profit per delivery varies according to the kind of delivery. The profit for an instant delivery is less
than the other kinds because the driver has to go directly to a grocery store with a small load and
return to the bottling plan. To find out what effect each type of delivery has on the profit picture, the
company has made the following tabulation based on deliveries for the previous quarter.

Type ofNumber of DeliveriesProfit per Delivery


DeliveryDuring the Quarter
Instant100$70
Same day60$100
Within five days40$160

What is the weighted mean profit per delivery?

A. $97
B. $72
C. $142
D. None of these answers
E. $100

Answer: A

Explanation:
(100*70)+(60*100)+(40*160) = 1940. Mean is 1940/200 = 97

Question: 1094

How much must you deposit today in order to withdraw $1,500 in 1 year, $400 in 2 years and $1,500
in 4 years, assuming your money earns 6% per year, compounded annually?

A. $2,771.03
B. $2,821.95
C. $2,959.23
D. $3,029.49
E. $404.92

Answer: C

Explanation:
Solve this question by working 3 compound interest problems. On the BAII Plus, press 1 N, 6 I/Y, 0
PMT, 1500 FV, CPT PV, which yields $1,415.09. Then press STO 1. Then press 2 N, 400 FV, CPT PV,
which yields $356.00. Then press + RCL 1 = STO 1. Then press 4 N, 1500 FV, CPT PV, which yields
$1,188.14. Then press + RCL 1 = to see the answer. On the HP12C, press 1 n, 6 i, 0 PMT, 1500 FV, PV.
Then press STO 1. Then press 2 N, 400 FV, PV. Then press RCL 1 + STO 1. Then press 4 n, 1500 FV, PV.
Then press RCL 1 + to see the answer. Note that the answer will be displayed as a negative number.
Make sure the BAII Plus has the P/Y value set to 1.

Question: 1095

A normally distributed variable has a mean of 45. If 95% of the observations on the variable fall

Page | 406
between 30 and 60, the standard deviation of the variable is approximately ________.

A. 7.50
B. 30.3
C. insufficient information
D. 15.0

Answer: A

Explanation:
95% of the observations on a normally distributed variable lie within about two standard deviations
of the mean. The standard deviation is then (45-30)/2 = 7.5.

Question: 1096

Given the following distribution regarding the cost of textbooks:

Cost of TextbooksNumber
$25 - $342
$35 - $445
$45 - $547
$55 - $6420
$65 - $7416

What is the relative class frequency for the $25 - $34 class?

A. 10%
B. 2%
C. 5%
D. None of these answers
E. 4%

Answer: E

Explanation:
The relative frequency is given by 2/50 = 0.04 = 4%

Question: 1097

If the underlying theory suggests that the value of an estimated variable is greater than a particular
number, it is appropriate to use:

A. either a right-tailed or left-tailed test.


B. a two-tailed test.
C. none of these answers.
D. a right-tailed test.
E. a left-tailed test.

Answer: D

Explanation:
When an observation is made and we see a value greater than the threshold suggested by the
theory, we would like to known the probability of getting the larger value due simply to chance
rather than to the event being driven by the theory. For this, we need to known the right-tail

Page | 407
probabilities under the null hypothesis.

Question: 1098

What monthly payment, beginning next month, is required over the next 48 months to pay off a
$10,000 debt today, if interest is charged at 10% per year, compounded monthly?

A. $253.63
B. $266.67
C. $216.02
D. $270.54
E. $400.54

Answer: A

Explanation:
On the BAII Plus, press 48 N, 10 divide 12 = I/Y, 10000 PV, 0 FV, CPT PMT. On the HP12C, press 48 n,
10 ENTER 12 divide i, 10000 PV, 0 FV, PMT. Make sure the BAII Plus has the P/Y value set to 1.

Question: 1099

Which of the following is/are true?


I. Doubling every single value in a sample doubles the sample mean.
II. Adding a constant to each value in a sample leaves the mean unchanged.
III. The mean of a sample decreases as the number of observations increases.

A. I only
B. I & III
C. I, II & III
D. II only
E. III only
F. II & III

Answer: A

Explanation:
Adding a constant to each value in a sample increases the mean by the same constant. III is incorrect.
As the number of observations increases, it is the expected difference between the sample mean and
the population mean that decreases.

Question: 1100

An automatic machine inserts mixed vegetables into a plastic bag. Past experience revealed that
some packages were underweight and some were overweight, but most of them had satisfactory
weight.

Weight% of Total
Underweight2.5
Satisfactory90.0
Overweight7.5

Three packages are selected from the food processing line. What is the probability of selecting and
finding that all three of them are underweight?

Page | 408
A. None of these answers
B. 0.0004218
C. 0.075
D. 0.0000156
E. 0.0000001

Answer: D

Explanation:
P(all three underweight) = 0.025*0.025*0.025 = 0.0000156.

Question: 1101

Jack wants to retire a millionaire. If he is 30 years old today and already has $30,000 in the bank,
what monthly payment would he need to make beginning one month from today and continuing
until he retires on his 65th birthday, if the money will earn 8% per year, compounded monthly?

A. $222.86
B. $139.04
C. $503.26
D. $470.98
E. $238.95

Answer: A

Explanation:
The total number of deposits will be 420 (35 years x 12 months/year). On the BAII Plus, press 420 N,
8 divide 12 = I/Y, 30000 PV, 1000000 +/- FV, CPT PMT. On the HP12C, press 420 n, 8 ENTER 12 divide i,
30000 PV, 1000000 CHS FV, PMT. Note that the answer will be displayed as a negative number. Make
sure the BAII Plus has the P/Y value set to 1.

Question: 1102

A national manufacturer of unattached garages discovered that the distribution of the lengths of
time it takes two construction workers to erect the Red Barn model is approximately normally
distributed with a mean of 32 hours and a standard deviation of 2 hours. What percent of the garages
take between 32 and 34 hours to erect?

A. 76.71%
B. None of these answers
C. 16.29%
D. 34.13%
E. 3.14%

Answer: D

Explanation:
z = (x-u)/sigma. z1 = 32 - 32/2 = 0 and z2 = 34 - 32/2 = 1. From the z tables, z = 0 and z = 1 are 0 and
0.3413 respectively. Therefore the area under the curve is 0.3413.

Question: 1103

An investor faces the following investment scenarios:

Page | 409
ScenarioProbabilityReturn
Bull market60%30%
Neutral market30%7%
Market crash10%-25%

The investor's expected rate of return is ________.

A. 12.45%
B. 17.6%
C. 16.2%
D. 19.3%

Answer: B

Explanation:
The expected return equals 0.6 * 30% + 0.3 * 7% + 0.1 * (-25%) = 17.6%.

Question: 1104

"Statistical inference" refers to the process of:

A. drawing conclusions about the entire population based on the statistical characteristics of a
sample.
B. drawing inferences about the characteristics of a distribution based on population data.
C. predicting the statistical characteristics of a randomly drawn sample based on the knowledge
about the entire population.
D. selecting a probability sample and the statistical method used to analyze it.

Answer: A

Explanation:
Note that statistical inference is always carried out with sample data, since you never have access to
the entire population (if you did, you would no longer call it "inference"!).

Question: 1105

A study of the scores on an in-plant course in management principles and the years of service of the
employees enrolled in the course resulted in these statistics:

Mean test score was 200 with a standard deviation of 40

Mean number of years of service was 20 years with a standard deviation of 2 years

In comparing the relative dispersion of the two distributions, what are the coefficients of variation
(CV)?

A. Test 50%, service 60%


B. Test 35%, service 45%
C. Test 100%, service 400%
D. Test 20%, service 10%
E. None of these answers

Answer: D

Page | 410
Explanation:
The respective CVs are found from (s*100)/mean. Test: 40*100/200 = 20% and Service: 2*100/20 =
10%.

Question: 1106

How long will it take money to double at 9.5% per year, compounded annually?

A. 28.03 years
B. 1.29 years
C. 10.14 years
D. 7.64 years
E. 4.1 years

Answer: D

Explanation:
To solve this question, choose a future value that is twice the size of the present value. The actual
number will not matter. On the BAII Plus, press 9.5 I/Y, 1000 PV, 0 PMT, 2000 +/- FV, CPT N. On the
HP12C, press 9.5 i, 1000 PV, 0 PMT, 2000 CHS FV, n. Note that the HP12C will show 8 for the answer.
The answer will be the same as long as the FV is twice as large as the PV.

Question: 1107

Which measures of central tendency are not affected by extremely low or extremely high values?

A. None of these answers


B. Geometric mean and mean
C. Mean and mode
D. Mode and median
E. Mean and median

Answer: D

Explanation:
Both are concepts which are based on the position of the observations when ordered (median) and
the number of most frequent occurrences (mode). They are measures which are not affected by
extreme values.

Question: 1108

In a distribution that is right skewed:

A. the median is larger than the mean.


B. large negative deviations dominate large positive deviations.
C. the mean is positive.
D. large positive deviations dominate large negative deviations.

Answer: D

Explanation:
large positive deviations dominate large negative deviations.

Question: 1109

Page | 411
The mean of a normal probability distribution is 500 and the standard deviation is 10. About 95
percent of the observations lie between what two values?

A. None of these answers


B. 475 and 525
C. 480 and 520
D. 350 and 650
E. 400 and 600

Answer: C

Explanation:
95% lie between plus and minus two standard deviations of the mean.

Question: 1110

Which of the following is/are true?


I. If P(A or B) = P(A) + P(B), A and B are independent
II. If P(A and B) = P(A).P(B), A and B are mutually exclusive
III. If P(A and B) = P(A) + P(B), P(A) = P(B) = 0

A. III only
B. II & III
C. I only
D. II only
E. I & II

Answer: A

Explanation:
If A and B are independent, P(A and B) = P(A).P(B) and vice versa. If A and B are mutually exclusive,
P(A and B) = 0. Note that P(A or B) = P(A) + P(B) - P(A and B). Hence, if P(A and B) = P(A) + P(B), the
P(A or B) = 0. This implies that neither A nor B can occur, giving P(A) = P(B) = 0.

Question: 1111

Manufacturers were subdivided into groups by volume of sales. Those with more than $100 million
in sales were classified as Class A large; those from $50 to $100 million as Class A medium size; and
those between $25 and $50 million, and so on. Samples were then selected from each of these
groups. What is this type of sampling called?

A. Stratified random
B. Simple random
C. Convenience
D. None of these answers
E. Panel

Answer: A

Explanation:
This is stratified random sampling. It is a method of selecting a random item from each section of the
population. The randomness comes from the random selection from a particular section. Stratified
refers to the division of the sections.

Page | 412
Question: 1112

A study of 200 stamping firms revealed these incomes after taxes:

Income After TaxesNumber of Firms


Under $1 million102
$1 million up to $20 million61
$20 million and more37

What is the probability that a particular firm selected has $1 million or more in income after taxes?

A. 0.25
B. 0.49
C. 0.00
D. 0.51
E. None of these answers

Answer: B

Explanation:
There are 200 firms altogether. 98 firms have more than a million in income. So 98/200 = 0.49

Question: 1113

Fifteen accounting majors had an average grade of 90 on a finance exam. Seven marketing majors
averaged 85, while ten finance majors averaged 93 on the same exam. What is the weighted mean
for the 32 students taking the exam?

A. None of these answers


B. 89.33
C. 89.48
D. Impossible to determine without more information
E. 89.84

Answer: E

Explanation:
(15*90)+(7*85)+(10*93) = 2875. Therefore, the weighted mean will be 2875/32 = 89.84.

Question: 1114

The probability that a value from a normal distribution will be less than 4 equals 83%. If the
distribution has a mean of 2.2, the standard deviation of the distribution must be:

A. 2.21
B. 0.96
C. 1.89
D. 3.77

Answer: C

Explanation:
Using the Normal probability tables, the z-value of the observation, X, which can be exceeded with a

Page | 413
probability of 17% equals 0.955. We are given that X = 4. Now, the z-value of a selected observation,
X, from a normal distribution with mean M and standard deviation S equals z = (X-M)/S. Therefore,
0.955 = (4 - 2.2)/S. This gives S = 1.885.

Question: 1115

The significance level in hypothesis testing refers to the probability that we will:

A. Reject the alternative when it is true.


B. Reject the null when it is true.
C. Accept the alternative when it is true.
D. Fail to reject the null when it is false.

Answer: B

Explanation:
Note that the significance level is the same as the probability of making a Type I error. A Type I error
refers to the event that we will reject the null when, in fact, it is true. Clearly, lower significance
levels are better, all else equal.

Question: 1116

What annual simple interest rate would cause a $105 deposit to grow to $125 in 15 months?

A. 7.89%
B. 15.24%
C. 20%
D. 13.14%
E. 14.17%

Answer: B

Explanation:
To solve this question, we have to rearrange the simple interest formula, I=PRT so that the R is by
itself. Note that the I (interest) is equal to 20 (125 - 105). Note also that the time component T of the
formula is 15/12. On the BAII Plus, press 20 divide 105 divide 15 x 12 = to see the answer. On the
HP12C, press 20 ENTER 105 divide 15 divide 12 x to see the answer.

Question: 1117

How many monthly payments of $50, beginning next month, are needed to pay off a debt of $700, if
interest accrues at 8% per year, compounded monthly?

A. 18.60
B. No solution/Error
C. 11.48
D. 12.80
E. 14.75

Answer: E

Explanation:
On the BAII Plus, press 8 divide 12 = I/Y, 700 PV, 50 +/- PMT, 0 FV, CPT N. On the HP12C, press 8
ENTER 12 divide i, 700 PV, 50 CHS PMT, 0 FV, n. Note that the HP12C will display 15 as the answer.

Page | 414
Make sure the BAII Plus has the P/Y value set to 1.

Question: 1118

What quarterly deposit do you need to make each quarter, starting next quarter, over the next 10
years in order to have a balance of $15,000 in your account, assuming the account starts with $1,000
in it and that the interest rate is 6% per year, compounded quarterly?

A. $242.98
B. $1.23
C. $1,293.08
D. $203.41
E. $30.46

Answer: A

Explanation:
There are 40 quarters in 10 years (10 x 4 =). The payment to be solved for as well as the original
$1,000 balance can be viewed as deposits into the account and hence given the same sign. The
$15,000 amount at the end of the timeframe should be of the opposite sign. On the BAII Plus, press
40 N, 6 divide 4 = I/Y, 1000 PV, 15000 +/- FV, CPT PMT. On the HP12C, press 40 n, 6 ENTER 4 divide i,
1000 PV, 15000 CHS FV, PMT. Note that the answer will be displayed as a negative number. Make
sure the BAII Plus has the value of P/Y set to 1.

Question: 1119

What is the Net Present Value of this series of annual cash flows at an interest rate of 20% per year:
Year 0: <$15,000>, Year 1: $2,000, Year 2: $0, Year 3: $15,000, Year 4 $0, Year 5 $18,000? (Note that
the <> are used to indicate a negative number).

A. $2,947.49>
B. $2,581.02
C. <$77.41>
D. $2,442.21
E. $2,414.37

Answer: B

Explanation:
On the BAII Plus, press CF 2nd CLRWork 15000 +/- ENTER DownArrow 2000 ENTER DownArrow
DownArrow 0 ENTER DownArrow DownArrow 15000 ENTER DownArrow DownArrow 0 ENTER
DownArrow DownArrow 18000 ENTER DownArrow DownArrow 2nd Quit. Then press NPV 20 ENTER
DownArrow CPT. On the HP12C, press these keys: 15000 CHS BlueShift CFo 2000 BlueShift CFj 0
BlueShift CFj 15000 BlueShift CFj 0 BlueShift CFj 18000 BlueShift CFj. Then press 20 i, YellowShift NPV.
The "DownArrow" represents the downward-pointing arrow on the top row of the BAII Plus
keyboard. Make sure the BAII Plus has the P/Y value set to 1.

Question: 1120

For the normal distribution, the mean plus and minus 1.96 standard deviations will include about
what percent of the observations?

A. None of these answers


B. 50%

Page | 415
C. 68%
D. 95%
E. 99.7%

Answer: D

Explanation:
95% of the are under the curve will lie within plus and minus 2 standard deviations of the mean.

Question: 1121

Assume you buy a car for $18,000 today and agree to pay $250 a month, beginning next month, for 5
years with a final payment also due 5 years from today to pay off any remaining balance. How large
will that final payment be, if interest accrues at 2.9% per year, compounded monthly?

A. $3,000.00
B. $4,683.97
C. $4,280.77
D. $9,690.32
E. $1,204.41

Answer: B

Explanation:
There are 60 months in 5 years. This is a problem known as a balloon payment. On the BAII Plus,
press 60 N, 2.9 divide 12 = I/Y, 18000 PV, 250 +/- PMT, CPT FV. On the HP12C, press 60 n, 2.9 ENTER
12 divide i, 18000 PV, 250 CHS PMT, FV. Note that the answer will be displayed as a negative number.
Make sure the BAII Plus has the value of P/Y set to 1.

Question: 1122

After a sterling year, your boss has decided to reward you. She has offered you three alternatives:
Take $28,000 right now.
Take $2,500 bonus per month for the next 12 months, starting next month.
Take $16,000 at the end of six months and another $15,000 at the end of the year.
If your discount rate is 6% per year on a monthly compounded basis, how much is the present value
of your bonus?

A. $29,656
B. $29,047
C. $28,000
D. $30,176

Answer: A

Explanation:
The discount rate per period (1 month) = 6%/12 = 0.5%. The present value of alternative B is (using
the annuity formula) 2,500/0.005*[1 - 1/1.005^12] = $29,047. The present value of alternative C is
trickier, since the rate you have been given is compounded monthly. Therefore, the PV of C is
16,000/1.005^6 + 15,000/1.005^12 = $29,656. So you should take alternative C and your bonus is
worth $29,656.

Question: 1123

Page | 416
Excess kurtosis is a problem for investment researchers using normal distributions because:

A. The likelihood of extreme outcomes will be overestimated.


B. Historical returns are better modeled with platykurtic distributions.
C. It is difficult to calculate.
D. The likelihood of extreme outcomes will be underestimated.

Answer: D

Explanation:
Historical returns are better modeled with fat-tailed (or leptokurtic) distributions, not platykurtic
ones. If a researcher uses a normal distribution to model a fat-tailed distribution, the estimated
probability of an extreme outcome will be underestimated. Thus, the frequency of market crashes
will be underestimated, an underestimation of the volatility risk.

Question: 1124

Which of the following best describes a type II error?

A. The probability of incorrectly failing to reject the null hypothesis.


B. (1 - the significance level).
C. The probability of incorrectly rejecting the null hypothesis.
D. The power of a test.
E. More than one of these answers is correct.

Answer: A

Explanation:
A type II error occurs when one incorrectly fails to reject the null hypothesis. In most cases, the
probability of a type II error is not expressly stated because the determination of this probability is
inherently difficult. A type II error is contrasted by a type I error, which is defined as the act of
incorrectly rejecting the null hypothesis.

Question: 1125

Which of the following is/are true?


I. The mean deviation can never be negative.
II. The range of a data set can never be negative.
III. The mean deviation is always less than the mean absolute deviation (MAD).

A. I only
B. I & II
C. I, II & III
D. III only
E. II & III
F. II only

Answer: B

Explanation:
Range of a dataset = maximum value - minimum value. Therefore, the range can never be negative.
Also, since the mean deviation is the average of the absolute deviations from the mean, the MD can
never be negative, either. Finally, Mean deviation and mean absolute deviation are one and the
same thing.

Page | 417
Question: 1126

What is the following table called?

AgesNumber of Ages
20 - 2916
30 - 3925
40 - 4951
50 - 5980
60 - 6920
70 - 798

A. None of these answers


B. Ogive
C. Histogram
D. Frequency distribution
E. Frequency polygon

Answer: D

Explanation:
The table above tallies the number of people within each age group. The tallies are then reported in
the second column and form the frequency of occurrences in each group. By doing so for every age
group, we have a distribution of the frequencies.

Question: 1127

Which of the following is/are assumptions of a linear regression?


I. For every given value of the independent variable, the dependent variable is normally distributed.
II. The error terms are normally distributed with mean zero.
III. The error terms are statistically uncorrelated.

A. I & III
B. II & III
C. III only
D. I & II
E. II only
F. I only
G. I, II & III

Answer: G

Explanation:
For every given value of the independent variable, the dependent variable is normally distributed,
the error terms are normally distributed with mean zero, and the error terms are statistically
uncorrelated.

Question: 1128

Which value of r indicates the strongest correlation than 0.40?

A. +0.48
B. -0.50

Page | 418
C. 0
D. -0.30
E. None of these answers

Answer: B

Explanation:
The bigger the absolute value of r, the stronger the correlation. The sign +/- just shows the direction,
not the strength.

Question: 1129

If you deposit $202.50 today into an account paying 6% per year simple interest, how much interest
will you have earned in 2 years?

A. $2.50
B. $20.50
C. $24.30
D. $24.00
E. 226.80

Answer: C

Explanation:
Since this is a simple interest question, the formula is I=PRT, with T here being 2, since the timeframe
is 2 years. On the BAII Plus, press 202.50 x 0.06 x 2 = to see the answer. On the HP12C, press 202.50
ENTER 0.06 x 2 x to see the answer. Since the question asks for the amount of interest earned, the
original deposit should not be added to this value.

Question: 1130

If 68% of the observations on a normal distribution fall between -20 and + 60, the mean and the
standard deviation of the distribution are:

A. 20,40
B. 40,20
C. 40,40
D. 20,20

Answer: A

Explanation:
Since the normal distribution is symmetrical about the mean, the mean is equal to (-20 + 60)/2 = 20.
Further, for a normal distribution, 68% of the observations lie within one standard deviation of the
mean. Therefore, the standard deviation equals 60-20 = 40.

Question: 1131

If you owe a debt of $3,000 today and also owe $5,000 in 24 months, what single payment could you
make 15 months from today that would pay off both of these debts, if interest is assessed at 8% per
year, compounded monthly?

A. $8,024.17
B. $7,751.62

Page | 419
C. $7,980.86
D. $8,000.00
E. $9,041.93

Answer: A

Explanation:
To solve this question, set the problem up as the sum of two compound interest calculations. Move
the $3,000 from today over to month 15 and add it to the $5,000 brought back from month 24 to
month 15. On the BAII Plus, press 15 N, 8 divide 12 = I/Y, 3000 PV, 0 PMT, CPT FV, which yields
$3,314.41. Then press STO 1. Then press 9 N, 5000 FV, CPT PV, which yields $4,709.76. Finally press +
RCL 1 = to see the answer. On the HP12C, press 15 n, 8 ENTER 12 divide i, 3000 PV, 0 PMT, FV. Then
press STO 1. Then press 9 n, 5000 FV, PV. Finally press RCL 1 + to see the answer. Make sure the BAII
Plus has the value of P/Y set to 1.

Question: 1132

What monthly payment is required over the next 60 months to pay off a $12,500 debt today, if
interest is charged at 12% per year, compounded monthly?

A. $316.02
B. $270.54
C. $296.67
D. $3,467.62
E. $278.06

Answer: E

Explanation:
On the BAII Plus, press 60 N, 12 divide 12 = I/Y, 12500 PV, 0 FV, CPT PMT. On the HP12C, press 60 n,
12 ENTER 12 divide i, 12500 PV, 0 FV, PMT. Make sure the BAII Plus has the P/Y value set to 1. Note
that the answer will be displayed as a negative number.

Question: 1133

What is the Net Present Value of this series of annual cash flows using an interest rate of 12% per
year: Year 0: <$15,000>, Year 1: $8,000, Year 2: $8,000, Year 3: $1,000, Year 4: $4,000? (Note that the
<> are used to indicate a negative number).

A. $2,077.49
B. $1,589.11
C. $1,774.26
D. $1,981.21
E. $1,104.37

Answer: C

Explanation:
On the BAII Plus, press CF 2nd CLRWork 15000 +/- ENTER DownArrow 8000 ENTER DownArrow
DownArrow 8000 ENTER DownArrow DownArrow 1000 ENTER DownArrow DownArrow 4000 ENTER
DownArrow DownArrow 2nd Quit. Then press NPV 12 ENTER DownArrow CPT. On the HP12C, press
these keys: 15000 CHS BlueShift CFo 8000 BlueShift CFj 8000 BlueShift CFj 1000 BlueShift CFj 4000
BlueShift CFj. Then press 12 i, YellowShift NPV. The "DownArrow" represents the downward-pointing
arrow on the top row of the BAII Plus keyboard. Make sure the BAII Plus has the P/Y value set to 1.

Page | 420
Question: 1134

The wildlife department has been feeding a special food to rainbow trout fingerlings in a pond. A
sample of the weights of 40 trout revealed that the mean weight is 402.7 grams and the standard
deviation 8.8 grams. What is the point estimate?

A. None of these answers


B. 10.0675
C. 8.8
D. 40
E. 402.7

Answer: E

Explanation:
The sample mean is a good estimate for the population mean.

Question: 1135

Given the following points: (-2, 0), (-1,0), (1, 1) and (2, 3)
If the regression equation is Y' = 2 - .4X, what is the value of Y when X = -3?

A. None of these answers


B. 3.2
C. -10.0
D. 0.8
E. 14.0

Answer: B

Explanation:
Y' = 2 - 0.4*(-3) = 2 + 1.2 = 3.2

Question: 1136

What must you know to develop a binomial probability distribution?

A. Probability of success
B. Probability of success and number of trials
C. Probability of success and number of successes
D. Number of trials
E. Number of successes

Answer: B

Explanation:
Binomial distribution: n!(p^r)(q^(n-r))/r!(n-r)!. We need to know n and p. q = 1-p. r is the number of
successes which we determine ourselves.

Question: 1137

The weights of a sample of crates ready for shipment to Laos are (in kilograms): 103, 97, 101, 106
and 103. What is the mean deviation?

Page | 421
A. 2.4 kg
B. 102.0 kg
C. 0 kg
D. None of these answers
E. 6.9 kg

Answer: A

Explanation:
The mean is 102. The mean deviation is the absolute values of the deviation from the mean. (1 + 5 +
1 + 4 + 1)/5 = 12/5 = 2.4

Question: 1138

What is the distribution with a mean of 0 and a standard deviation of 1 called?

A. Binomial probability distribution


B. None of these answers
C. Frequency distribution
D. Z-score
E. Standard normal distribution

Answer: E

Explanation:
These values describe the standard normal distribution.

Question: 1139

Suppose we set the criterion for the rejection of the null that is extremely stringent, assuring us that
the null will be definitely rejected. Then, which of the following is/are true?
I. The probability of a Type I error is zero.
II. The probability of Type II error is zero.
III. The significance level of the test is 1.

A. II only
B. none of these answers
C. I only
D. II & III
E. I & III

Answer: D

Explanation:
A Type I error refers to the event that we will reject the null when, in fact, it is true. If the criterion is
so stringent that you always reject the null, then the probability of type I error is one. A Type II error
refers to the event that we will fail to reject the null when, in fact, it is false. If you always reject the
null, then the probability of type II error is clearly zero. Also remember that the significance level is
the same as the probability of making a Type I error. Hence, in this case, the significance level equals
100%.

Question: 1140

Page | 422
Which of the following is/are true about a normal distribution?
I. It is a bimodal distribution.
II. It can be characterized completely by a single parameter.
III. It ranges from negative infinity to positive infinity.
IV. It is positively skewed.

A. I only
B. III only
C. IV only
D. II only
E. II, III and IV
F. II and III
G. III and IV

Answer: B

Explanation:
A normal distribution is completely characterized by 2 parameters, mean and variance. It ranges
from negative infinity to positive infinity, has a single mode or peak (occurring at the mean), is
symmetrical about the mean and has zero skewness.

Question: 1141

Assume a student received the following grades for the semester: History, B; Statistics, A; Spanish, C;
and English, C. History and English are 5 credit hour courses, Statistics a 4 credit hour course and
Spanish a 3 credit hour course. If 4 grade points are assigned for an A, 3 for a B and 2 for a C, what is
the weighted mean for the semester grade?

A. 1.96
B. 2.88
C. 4.00
D. 2.76
E. 3.01

Answer: D

Explanation:
The student has taken 17 credit hours. The weighted grades for each are: History:5*3 = 15. English:
5*2 = 10. Statistics: 4*4 = 16. Spanish: 3*2 = 6. Adding we have 15 + 10 + 16 + 6 = 47. Mean = 47/17 =
2.76

Question: 1142

Suppose that you write an investment newsletter. You track 12 stocks, and you classify these stocks
into equal-sized categories of buy, sell, and hold. How many different ways can you classify these
stocks into those categories?

A. 34,860.
B. 36,540.
C. 34,680.
D. 34,650.

Answer: D

Page | 423
Explanation:
To answer this question, we need the formula for permutations that takes into account different
ways to label things. The number of ways that you can arrange n objects so that there are n_1 of one
kind, n_2 of another kind, and so on, up to n_k of a kth kind, is found by using the multinomial
formula: n! /[(n_1)! * (n_2)! * ... * (n_k)!]. In this case, your equally-sized categories will be 4 of each
kind, 12 in all. 12! / (4!*4!*4!) = 34,650.

Question: 1143

A random variable with a mean equal to 2.5 and a standard deviation of 2.0 has a coefficient of
variation equal to ________.

A. zero
B. -2.0
C. 1.25
D. none of these answers
E. 0.8

Answer: E

Explanation:
The coefficient of variation equals the ratio of the standard deviation to the mean.

Question: 1144

If you deposit $100 a month, beginning next month, for 20 years into an account paying 6% per year,
compounded monthly, how much is in your account after that last deposit?

A. $46,204.09
B. $1,973,585,957
C. $24,000.00
D. $2,097.91
E. $3,678.56

Answer: A

Explanation:
On the BAII Plus, press 240 N, 6 divide 12 = I/Y, 0 PV, 100 PMT, CPT FV. On the HP12C, press 240 n, 6
ENTER 12 divide i, 0 PV, 100 PMT, FV. On the BAII Plus, make sure the value of P/Y is set to 1. Note
that the answer is displayed as a negative number.

Question: 1145

How many annual deposits of $1,000, beginning next year, would you need to make before you had
accumulated $30,000, if the money earns 8% per year, compounded annually? Assume the account
begins with a $0 balance.

A. 15.90
B. 25.51
C. 5.58
D. 5.19
E. 21.41

Answer: A

Page | 424
Explanation:
On the BAII Plus, press 8 I/Y, 0 PV, 1000 PMT, 30000 +/- FV, CPT N. On the HP12C, press 8 i, 0 PV, 1000
PMT, 30000 CHS FV, n. Note that the HP12C will display 16 as the answer.

Question: 1146

You are examining a portfolio composed of 10% money-market investments, 30% bonds, and 60%
stocks. Last year, the return on the money-market investments was 4%; the return on bonds was 9%,
and the return on stocks was -11%. What is the portfolio weighted average return?

A. -3.00%.
B. -4.50%.
C. None of these answers is correct.
D. -3.25%.

Answer: C

Explanation:
The portfolio weighted-average mean return is equal to the sum (as i goes from 1 to n) of w_i * X_i,
where w_i is the percentage weight in the portfolio of the ith asset, and X_i is the investment return
of the ith asset. Here, we get a weighted mean of 0.10 * 0.04 + 0.30
* 0.09 + 0.60 * -0.11 = -3.50%. None of these answers is correct.

Question: 1147

Which of the following statements regarding hypothesis testing is false?

A. The F-statistic is used in multivariate quantitative analysis.


B. More than one of these answers is incorrect.
C. If the null hypothesis is rejected, then it is said the result is "not statistically significant."
D. If the population standard deviation is unknown, then the standard error of the estimate is found
by dividing the sample standard deviation by the square root of "n."
E. The power of a test is usually equal to (1 - the probability of a Type II error).
F. In quantitative analysis, the Type I error is defined as the act of incorrectly rejecting the null
hypothesis.

Answer: C

Explanation:
Remember that when the null hypothesis is rejected, the results of the regression are said to be
"statistically significant." In other words, the analyst has enough reason to assume that the results of
the analysis are valid at the given level of significance. When the analyst fails to reject the null
hypothesis, then the result is said to "not be statistically significant." While this specification may
seem excessive, it is nevertheless important. The remaining answers are all correct.

Question: 1148

What annual interest rate, compounded annually, would cause a series of 20 deposits of $1,000 to
accumulate to $40,000, if the first deposit is made one year from today?

A. 6.77%
B. 8.03%
C. 7.23%

Page | 425
D. 5.09%
E. 14.98%

Answer: A

Explanation:
On the BAII Plus, press 20 N, 0 PV, 1000 PMT, 40000 +/- FV, CPT I/Y. On the HP12C, press 20 n, 0 PV,
1000 PMT, 40000 CHS FV, i. Make sure that the BAII Plus has the P/Y value set to 1.

Question: 1149

In a statistical regression estimation, the R-square is found to be 63% and the slope coefficient is 0.6.
The correlation coefficient between the dependent and the independent variables is ________.

A. 0.60
B. 0.63
C. 0.24
D. 0.79

Answer: D

Explanation:
In a univariate regression, the correlation coefficient is the square root of the R-square. Hence,
correlation coefficient = sqrt(0.63) = 0.79

Question: 1150

Which approach to probability is exemplified by the following formula?


Prob. of Event happening = No. of times event occurred in the past/Total no. of observations.

A. A priori approach
B. None of these answers
C. Relative frequency approach
D. Subjective approach

Answer: C

Explanation:
When we based the probability on the number of times the events has occurred in the past over the
total number of observations, we are using the relative frequency approach.

Question: 1151

Richard Eisenberg, a quantitative analyst with Middle Road Brokerage, has been instructed to
perform a statistical analysis to determine the relationship between the volatility of smaller-cap
stocks and the quarterly performance of a series of smaller-cap mutual funds. Mr. Eisenberg begins
his assignment by formulating and stating a hypothesis. Next, he identifies the appropriate test
statistic and the probability distribution for the series. Mr. Eisenberg then progresses to specifying
the significance level for the hypothesis test. Now that the first three steps of a hypothesis test have
been completed, Mr. Eisenberg should proceed to which of the following? Choose the best answer.

A. Evaluating the goodness of fit


B. Collecting the data and performing the calculations
C. Determining the p-value

Page | 426
D. None of these answers is correct
E. Stating the decision rule
F. Testing for multicolinearity

Answer: E

Explanation:
Hypothesis testing involves a series of seven explicit steps:

Step 1: Formulating and stating the hypothesis


Step 2: Identifying the appropriate test statistic and its probability distribution
Step 3: Specifying the significance level
Step 4: Stating the decision rule
Step 5: Collecting the data and performing the necessary calculations
Step 6: Making the statistical decision
Step 7: Making the economic/investment decision.

Question: 1152

If you owe $2,000 today and pay for it by making 20 monthly payments with the first payment made
immediately (today), what is the size of this monthly payment, if interest accrues at 7% per year,
compounded monthly?

A. $193.55
B. $191.63
C. $209.48
D. $105.62
E. $106.24

Answer: D

Explanation:
Recognize that this question is an annuity due situation, since the first cash flow occurs immediately
or at the beginning of each period. Annuities where the first payment occurs 1 period from today (or
at the end of each period) are called "ordinary" annuities. This requires placing the calculator into
"Begin" mode prior to solving the question. NOTE: Be sure to place the calculator OUT OF annuity
due mode after this question before going on to subsequent questions, or you will get the wrong
answers! On the BAII Plus, press 2nd BGN. If the display shows END, then press 2nd SET and then 2nd
Quit. This will place the BAII Plus into annuity due mode (you can tell this because the BAII Plus will
display BGN in small letters). Now press 20 N, 7 divide 12 = I/Y, 2000 PV, 0 FV, CPT PMT. Place the
calculator back into End mode (for ordinary annuities) by pressing 2nd BGN and then if the calculator
is displaying BGN, press 2nd SET and 2nd Quit. The BGN letters should disappear from the display. On
the HP12C, press BlueShift END, which is the blue function on the front of the 8 digit key. This places
the HP12C into Begin mode (the HP12C shows the word BEGIN in the display when in this mode).
Then press 20 n, 7 ENTER 12 divide i, 2000 PV, 0 FV, PMT. To place the HP12C back into ordinary
annuity mode (or END mode), press BlueShift BEG (the blue function written on the front of the 7
digit key). Note that the answer is displayed as a negative number. Make sure the BAII Plus has the
value of P/Y set to 1.

Question: 1153

How much would you have in a savings account 12 months from now if you start with a balance of
$1,000 today, make a deposit of $2,000 in 6 months and make another deposit 6 months after that of
$3,000? Assume that interest accrues at 6% per year, compounded monthly.

Page | 427
A. $6,000.00
B. $5,500.00
C. $6,122.43
D. $7,684.42
E. $5,777.55

Answer: C

Explanation:
Find the answer to this question by solving a couple of compound interest problems. Move the
$1,000 to month 12, then move the $2,000 forward 6 months to month 12, then add $3,000. On the
BAII Plus, press 12 N, 6 divide 12 = I/Y, 1000 PV, 0 PMT, CPT FV, which yields $1,061.68. Then press
STO 1. Then press 6 N, 2000 PV, CPT FV, which yields $2,060.76. Then press + RCL 1 =. Make this
number positive by pressing +/- and then press + 3000 = to see the answer. On the HP12C, press 12
N, 6 ENTER 12 divide i, 1000 PV, 0 PMT, FV. Then press STO 1. Then press 6 N, 2000 PV, FV. Then press
RCL 1 +. Make this number positive by pressing CHS and then press 3000 + to see the answer.

Question: 1154

A portfolio consists of a six-year annuity paying $250 a year and a perpetuity that pays $300 a year.
The payments start at the end of the year. With a discount rate of 9% per year, the portfolio is worth:

A. $5,210
B. $3,333
C. $4,454
D. $4,768

Answer: C

Explanation:
The value of the perpetuity is 300/0.09 = 3,333. The annuity is worth (250/0.09)*(1-1/(1.09^6)) =
1,121 The portfolio is thus worth 1,121 + 3,333 = 4,454

Question: 1155

If all the plots on a scatter diagram lie on a straight line, what is the standard error of estimate?

A. Infinity
B. None of these answers
C. 0
D. +1
E. -1

Answer: C

Explanation:
The standard error of estimate basically measures the difference between the points and the straight
line.

Question: 1156

You wish to rank n different securities according to the deviation of the market price from your
estimate of their intrinsic value. You are curious how many different ways this can come out. The

Page | 428
counting method you should use is:

A. The permutation rule.


B. The multinomial formula.
C. The binomial formula.
D. None of these answers is correct.

Answer: D

Explanation:
The factorial method allows you to determine the number of ways of arranging n things. Initially,
there are n choices for the first item, then n-1 for the second, and so on. The number of
combinations is therefore n! = n * (n-1) * (n-2) * (n-3) * ... * (1).

Question: 1157

What annual interest rate, compounded annually, is equivalent to 7% per year, compounded
semiannually?

A. 6.87%
B. 7.05%
C. 7.12%
D. 7.02%
E. 7%

Answer: C

Explanation:
Questions of this type are illustrating the concept of an Effective Interest Rate, which is a rate
compounded annually that has the same effect as a rate compounded more often than one time a
year. As such, a depositor or a creditor is indifferent between them, since they have the same effect.
To solve this question, make any deposit and see how much is in the account after one year. The ratio
of the ending FV to the beginning PV will indicate the annual rate earned. On the BAII Plus, press 2 N,
7 divide 2 = I/Y, 100 PV, 0 PMT, CPT FV. On the HP12C, press 2 n, 7 ENTER 2 divide i, 100 PV, 0 PMT, FV.
The number displayed will be 107.12. In other words, after one year, $100 has become $107.12. An
interest rate of 7.12%, compounded annually, would cause a $100 deposit to become $107.12 in one
year. Choosing an initial deposit of $100 helps a great deal in these situations. Make sure the BAII
Plus has the value of P/Y set to 1.

Question: 1158

A random variable, X, has a mean of 12 and a standard deviation of 14. If another variable, Y, is
defined by Y = 2X - 3, the coefficient of variation of Y is ________.

A. 0.75
B. 1.33
C. 1.19
D. 1.17

Answer: B

Explanation:
You should remember two important points:

Page | 429
1. Multiplying a random variable by a constant multiplies its mean and standard deviation by the
same constant.
2. Adding a constant to a random variable increases the mean by the same constant but leaves the
standard deviation unchanged.

Using these two rules, the mean of Y equals 2 * 12 - 3 = 21. The standard deviation of Y equals 2 * 14
= 28. The coefficient of variation equals the ratio of the standard deviation to the mean. Thus, the
coefficient of variation of Y equals 28 / 21 = 1.33.

Question: 1159

What annual interest rate, compounded annually, will cause a deposit of $550 to become $1,475 in
15 years?

A. 0.55%
B. 5.50%
C. 8.60%
D. 17.88%
E. 6.80%

Answer: E

Explanation:
On the BAII Plus, press 15 N, 550 PV, 1475 +/- FV, 0 PMT, CPT I/Y. On the HP12C, press 15 n, 550 PV,
1475 CHS FV, 0 PMT, i.

Question: 1160

A trial generates only two results, "success" and "failure." The variance of the number of failures in
20 trials equals 2.35. The probability of success on a given trial equals ________.

A. 0.66
B. 0.136
C. 0.231
D. 0.452

Answer: B

Explanation:
For a binomial distribution with N trials, with the probability of success = p in each trial, the variance
equals Np(1-p). Hence, 20*p*(1-p) = 2.35. Solving this gives quadratic equation gives p = 0.136.

Question: 1161

Suppose you need $800 in 20 months. How much must you deposit today, if the deposit will earn
interest at 8% per year, compounded monthly?

A. $607.27
B. $700.45
C. $1,330.48
D. $949.93
E. $571.06

Answer: B

Page | 430
Explanation:
On the BAII Plus, press 20 N, 8 divide 12 = I/Y, 0 PMT, 800 FV, CPT PV. On the HP12C, press 20 n, 8
ENTER 12 divide i, 0 PMT, 800 FV, PV. Make sure the BAII Plus has the P/Y value set to 1.

Question: 1162

A statistician is working with the following dataset:


129, 162, 159, 192, 142, 195, 203, 177, 138, 187, 149, 123, 146, 169, 162, 191, 155, 146, 176, 182,
174, 190, 168.
The relative frequency in the frequency class 150-175 equals ________.

A. 0.30
B. 0.29
C. 7.00
D. 0.34

Answer: A

Explanation:
There are 23 observations. Of these, 7 lie in the 150-175 class.

relative frequency = class frequency/total # of observations

= 7/23 = 0.30

Remember that relative frequency is always a number between zero and one.

Question: 1163

An investment of $275 grows to $400 in 3 years. The holding period return is:

A. 1.13
B. 0.6875
C. 1.455
D. 1.455

Answer: C

Explanation:
The holding period return (HPR) is 400 / 275 = 1.455

Question: 1164

If the variance of a probability distribution was computed to be 3.6, what is the standard deviation?

A. 6.0
B. 1.9
C. None of these answers
D. 0.6
E. 12.96

Answer: B

Page | 431
Explanation:
Standard deviation is the square root of the variance.

Question: 1165

A price-linked derivative security pays $300 if the oil price over the next year increases by more than
5%, an event that can happen with a 60% probability. Otherwise, it pays $50. If the expected return
on the security is 15%, how much does the security cost?

A. $174
B. $180
C. $168
D. $191

Answer: A

Explanation:
The expected payoff on the security equals 0.6 * 300 + 0.4 * 50 = 200. Since the expected return is
15%, the security must cost 200/1.15 = $173.9

Question: 1166

If you buy an item for $500 and agree to pay for it with 36 monthly payments of $22.22, beginning
next month, what annual interest rate, compounded monthly, are you being charged?

A. 27.84%
B. 24.92%
C. 41.15%
D. 33.58%
E. 26.43%

Answer: D

Explanation:
The interest rate returned by the calculator will be the periodic interest rate. It must be multiplied by
the number of periods per year to have the correct answer. On the BAII Plus, press 36 N, 500 PV,
22.22+/- PMT, 0 FV, CPT I/Y. Then press x 12 = to see the answer. On the HP12C, press 36 n, 500 PV,
22.22 CHS PMT, 0 FV, i. Then press 12 x to see the answer. Make sure the BAII Plus has the P/Y value
set to 1.

Question: 1167

The formula for joint probability is given by:

A. P(AB) = P(A | B) * P(A)


B. P(AB) = P(A | B) / P(B)
C. P(AB) = P(A | B) * P(B)
D. P(AB) = P(A | B) / [P(B) * P(A)]

Answer: C

Explanation:
A joint probability takes the form of P(AB) = P(A | B) * P(B). Note that this is just a rearranged form of
the formula for conditional probability.

Page | 432
Question: 1168

What does the coefficient of determination equal if r = 0.89?

A. 0.94
B. None of these answers
C. 0.89
D. 0.06
E. 0.79

Answer: E

Explanation:
The coefficient of determination is the r-squared.

Question: 1169

The semi-logarithmic scale is so named because:

A. Values on the x-axis are equally spaced if their logarithms are the same.
B. Values on the y-axis are equally spaced if their logarithms are the same.
C. Values on the x-axis are equally spaced if there are equal differences in their logarithms.
D. Values on the y-axis are equally spaced if there are equal differences in their logarithms.

Answer: D

Explanation:
Values on the y-axis are equally spaced if there are equal differences in their logarithms. Thus, 100 is
as close to 10, as 10 is to 1. (In base 10, log 100 - log 10 = 1, log 10 - log 1 = 1; in base e, the
differences are also the same, each equal to 2.302.)

Question: 1170

The annual commissions per salesperson employed by a manufacturer of light machinery averaged
$40,000 with a standard deviation of $5,000. What percent of the sales persons earn between
$32,000 and $42,000?

A. 60.06%
B. 34.13%
C. 39.94%
D. None of these answers
E. 81.66%

Answer: A

Explanation:
z = (x - u)/sigma. z1 = (32000 - 40000)/5000 = -1.6. z2 = (42000 - 40000)/5000 = 0.4. The area under
the curve for the respective z values are 0.4452 and 0.1554. Since the earnings are on the opposite
sides of the mean, we add them up 0.4452 + 0.1554 = 0.6006.

Question: 1171

The weights of cans of fruit are normally distributed with a mean of 1,000 grams and a standard

Page | 433
deviation of 50 grams. What portion of the cans weigh 860 grams or less?

A. None of these answers


B. 0.8400
C. 0.0001
D. 0.0100
E. 0.0026

Answer: E

Explanation:
z = (x-u)/sigma = 860 - 1000/50 = -2.8. For z = 2.8, the area is 0.4974. So 1.0 - 0.9974 = 0.0026.

Question: 1172

You are examining the return on equity ratios of the nation's commercial airlines. You have calculated
the mean ROE to be 8%, and the sum of the squared ROEs is 0.27. Assuming there are 30 commercial
airlines, what is the population standard deviation of ROEs in this industry?

A. 5.1%.
B. 7.9%.
C. 8.0%.
D. 5.0%.

Answer: A

Explanation:
The population standard deviation is the positive square root of the population variance. The
population variance is equal to the sum of the squared differences between each population
member and the population mean, divided by the number of items in the population. There is,
however, an equivalent alternate formula that is more often used for large amounts of calculation. It
is equal to the average squared observation less the squared mean. In this case, we have the average
squared observation will be 0.27 / 30 = 0.009, less the squared mean of 8%. We have 0.009 - 0.0064
= 0.0026 = 26%%. The square root of this number, 5.1%, will be the population standard deviation.

Question: 1173

The U.S. Department of Education reported that for the past seven years 4,033, 5,652, 6,407, 7,201,
8,719, 11,154, and 15,121 people received bachelor's degrees in computer and information sciences.
What is the mean annual number receiving this degree?

A. None of these answers


B. About 6,217
C. About 8,327
D. About 15,962
E. About 12,240

Answer: C

Explanation:
(4033 + 5652 + 6407 + 7201 + 8719 + 11154 + 15121)/7 = 8327

Question: 1174

Page | 434
What do we call the conditions under which the null hypothesis is not rejected?

A. Critical value
B. Decision rule
C. Alternate hypothesis
D. None of these answers
E. Test statistic

Answer: B

Explanation:
The decision rule lays out the conditions under which we may compare the z values and decide
whether or not to reject the hypothesis.

Question: 1175

If the alternative hypothesis states that u(Mu) > 6,700, what is the rejection region for the hypothesis
test?

A. Center
B. None of these answers
C. Lower tails
D. Upper tail
E. Both tails

Answer: D

Explanation:
Since the alternative is that Mu is bigger than the mean, then it will be located at the upper tail.

Question: 1176

A light bulb manufacturer would like to supply only those bulbs which have a life of 2,000 hours or
more. Otherwise, it wants the bulbs to be rejected at the assembly line. The statistician hired by the
bulb manufacturer is trying to frame the situation as a hypothesis testing problem. He should specify
the null and alternate hypothesis as:

A. Ho: average life < 2,000; H1: average life > 2,000
B. Ho: average life = 2,000; H1: average life > 2,000
C. Ho: average life = 2,000; H1: average life < 2,000
D. Ho: average life >= 2,000; H1: average life < 2,000

Answer: D

Explanation:
In any experiment, you have a natural candidate for a default hypothesis that is maintained to be
true until there is sufficient evidence to reject it. This is the hypothesis that must be selected as the
null. Inthe present example, the manufacturer is in the business of supplying bulbs which have an
average life of more than 2000 hours. So it is logical to assume that the processes usually produce
such bulbs and occasionally produce faulty bulbs which have a shorter average life. Hence, the null
hypothesis must be that any given bulb has an average life of more than 2000 hours and the
alternative hypothesis must be the complement (i.e. average life is less than 2000 hours).

Question: 1177

Page | 435
The larger the critical value on the z-statistic,

A. all of these answers can happen, depending on the R-square.


B. the harder it is to reject the null hypothesis.
C. the easier it is to accept the null hypothesis.
D. the easier it is to reject the null hypothesis.
E. none of these answers.

Answer: B

Explanation:
To reject a null hypothesis, the z-statistic must be larger in magnitude than the critical value for a
given level of significance.

Question: 1178

If a distribution is right skewed, which of the following is/are true?


I. It cannot be normally distributed.
II. Its skewness is negative.
III. It has a positive mean.
IV. Large positive deviations dominate large negative deviations.

A. I only
B. I & IV
C. II only
D. I & III
E. III only
F. I, II, III & IV

Answer: B

Explanation:
A normal distribution is symmetrical and hence, has zero skewness i.e. it is neither positively nor
negatively skewed. If the distribution is right skewed, its skewness is positive but it does not
necessarily have a positive mean.

Question: 1179

Which of the following statements is false in reference to confidence intervals and/or tests of
significance? Choose the best answer.

A. More than one of these answers is correct.


B. The specification of a confidence level is the third step in the hypothesis testing process.
C. Determining the significance level of a hypothesis test after calculating the test statistic can lead to
an erosion of objectivity.
D. The significance level is denoted by the Greek letter theta.
E. The confidence level can be found by (1 - the probability of a Type I error).
F. The significance level is equal to the probability of a Type I error.

Answer: D

Explanation:
The significance level is denoted by the Greek letter alpha. The significance level is equal to the

Page | 436
probability of a Type I error, and can be found by subtracting the confidence level from the letter
one. The relationship between the confidence level and the significance level is illustrated as follows:

Confidence level = (1 - significance level)

which can also be expressed as:

Confidence level = (1 - probability of a Type I error).

The Greek letter theta is used to represent the population parameter in the null and alternate
hypotheses.

The remaining answers are all correct. It is important to remember that the level of significance
should be determined prior to the calculation of the test statistic. Specifying the significance level
after the calculation of the test statistic can detract from the objectivity of the test.

Question: 1180

A mortgage holding company has found that 2% of its mortgage holders default on their mortgage
and lose the property. Furthermore, 90% of those who default are late on at least two monthly
payments over the life of their mortgage as compared to 45% of those who do not default. What is
the probability that a mortgagee with two or more late monthly payments will default on the
mortgage and lose the property?

A. 0.018
B. None of these answers
C. 0.441
D. 0.039
E. 0.459

Answer: D

Explanation:
We have P(def) = 0.02. P(not def) = 0.98. P(two late payments/def) = 0.90. P(two late payments/not
def) = 0.45. Using Bayes formula: p(def/two late payments) = (0.02*0.9)/(0.02*0.9 + 0.98*0.45) =
0.039.

Question: 1181

How many monthly payments of $40 are needed to pay off a debt of $1,000, if interest accrues at
10% per year, compounded monthly?

A. No solution/Error
B. 28.15
C. 31.48
D. 17.60
E. 22.80

Answer: B

Explanation:
On the BAII Plus, press 10 divide 12 = I/Y, 1000 PV, 40 +/- PMT, 0 FV, CPT N. On the HP12C, press 10
ENTER 12 divide i, 1000 PV, 40 CHS PMT, 0 FV, n. Note that the HP12C will display 29 as the answer.
Make sure the BAII Plus has the value of P/Y set to 1.

Page | 437
Question: 1182

What is the present value today of these annual cash flows: $2,500, $1,200, $300? Assume the first
cash flow occurs 1 year from today and an interest rate of 9% per year, compounded annually.

A. $3,535.25
B. $4,000.00
C. $3,976.69
D. $3,608.82
E. $3,114.09

Answer: A

Explanation:
You could solve this question using 3 different compound interest problems, but it is easier to solve
them using the calculator's cash flow functions. On the BAII Plus, press CF 2nd CLRWork 0 ENTER
DownArrow 2500 ENTER DownArrow DownArrow 1200 ENTER DownArrow DownArrow 300 ENTER
DownArrow DownArrow 2nd Quit. Then press NPV 9 ENTER DownArrow CPT. On the HP12C, press
these keys: 0 BlueShift CFo 2500 BlueShift CFj 1200 BlueShift CFj 300 BlueShift CFj. Then press 9 i,
YellowShift NPV. The "DownArrow" represents the downward-pointing arrow on the top row of the
BAII Plus keyboard. Make sure that the BAII Plus has the P/Y value set to 1.

Question: 1183

How much is in your account in 5 years, if you start with a balance of $1,000 and add $3,500 in 18
months and add an additional $1,000 in 48 months, if the money earns interest at 6% per year,
compounded monthly?

A. $5,500.00
B. $6,514.27
C. $7,131.50
D. $6,726.14
E. $6,892.81

Answer: D

Explanation:
Solve this question by working three compound interest problems. On the BAII Plus, press 60 N, 6
divide 12 = I/Y, 1000 PV, 0 PMT, CPT FV, which yields $1,348.85. Then press STO 1. Then press 42 N,
3500 PV, CPT FV, which yields $4,315.61. Then press + RCL 1 = STO 1. Then press 12 N, 1000 PV, CPT
FV, which yields $1,061.68. Then press + RCL 1 = for the final answer. On the HP12C, press 60 n, 6
ENTER 12 divide i, 1000 PV, 0 PMT, FV. Then press STO 1. Then press 42 n, 3500 PV, FV. The press RCL
1 + STO 1. Then press 12 n, 1000 PV, FV. Then press RCL 1 + for the final answer. Note that the answer
will be a negative number. Make sure the BAII Plus has the value of P/Y set to 1. Note that the $3,500
deposit is 42 months away from the end of the 60-month time period.

Question: 1184

The z-value of a selected observation, X, from a normal distribution, also known as the normal
deviate or the z-score of X, represents:

A. the probability that another observation from the distribution will be less than or equal to X.
B. the probability that another observation from the distribution will be more than or equal to X.

Page | 438
C. the distance of X from the mean, in terms of standard deviation.
D. the distance of X from zero.

Answer: C

Explanation:
The z-value of a selected observation, X, from a normal distribution with mean M and standard
deviation S equals z = (X-M)/S. The z-value thus represents the distance of X from the mean, in terms
ofstandard deviation. Note that the z-value is a distance measure. It does not measure probabilities
directly.

Question: 1185

How much will $1,250 become after 7 years at 10% per year, compounded annually?

A. $2,298.66
B. $1,324.77
C. $2,435.90
D. $2,500.00
E. $2,388.14

Answer: C

Explanation:
On the BAII Plus, press 7 N, 10 I/Y, 1250 PV, 0 PMT, CPT FV. On the HP12C, press 7 n, 10 i, 1250 PV, 0
PMT, FV. Note that the answer will be displayed as a negative number. Make sure the BAII Plus has
the P/Y value set to 1.

Question: 1186

Melinda May, a quantitative analyst with Smith, Kleen, & Beetchnutty Brokerage, has been instructed
to perform a statistical analysis to test whether the earnings of biotechnology companies are
negatively related to the growth of generic drug consumption. Ms. May begins her analysis by
formulating and stating a hypothesis. Next, Melinda proceeds with an identification of the
appropriate test statistic and its probability distribution. Now that the first two steps in the
hypothesis testing process have been completed, Ms. May should proceed with which of the
following? Choose the best answer.

A. Evaluating the goodness of fit


B. None of these answers is correct
C. Specifying the significance level
D. Stating the decision rule
E. Collecting the data and performing the calculations
F. Determining the p-value

Answer: C

Explanation:
Hypothesis testing involves a series of seven explicit steps:

Step 1: Formulating and stating the hypothesis


Step 2: Identifying the appropriate test statistic and its probability distribution
Step 3: Specifying the significance level
Step 4: Stating the decision rule

Page | 439
Step 5: Collecting the data and performing the necessary calculations
Step 6: Making the statistical decision
Step 7: Making the economic/investment decision.

In this example, Melinda May, a quantitative analyst, has begun her analysis by completing the first
two steps in the hypothesis testing process. Now that the hypothesis has been stated and the
appropriate test statistic has been calculated, Ms. May should proceed with a determination of the
significance level, which is denoted by the Greek letter alpha. Remember that the significance level is
equal to the probability of a Type I error, which is defined as incorrectly rejecting the null hypothesis.
Subtracting the significance level from the number one will yield the confidence interval.

Question: 1187

A stockbroker placed the following order:


50 shares of Kaiser Aluminum preferred at $104 a share
100 shares of GTE preferred at $25 1/4 a share
20 shares of Boston Edison preferred at $9 1/8 a share
What is the weighted mean price per share?

A. $79.75
B. $25.25
C. $103.5
D. None of these answers
E. $42.75

Answer: D

Explanation:
(50*104)+(100*25.25)+(20*9.125) = 7907.5. Weighted mean is 7907.5/170 = 46.51

Question: 1188

Which one of the following is NOT a condition of the binomial distribution?

A. Independent trials
B. Only two outcomes
C. Probability of success remains constant from trial to trial
D. At least 10 observations
E. None of these answers

Answer: D

Explanation:
There no requirement regarding the number of observations.

Question: 1189

Suppose you need $500 in 20 months. How much must you deposit today, if the deposit will earn
interest at 8% per year, compounded monthly?

A. $2,330.48
B. $437.78
C. $571.06
D. $249.93

Page | 440
E. $107.27

Answer: B

Explanation:
On the BAII Plus, press 20 N, 8 divide 12 = I/Y, 0 PMT, 500 FV, CPT PV. On the HP12C, press 20 n, 8
ENTER 12 divide i, 0 PMT, 500 FV, PV. Make sure the BAII Plus has the P/Y value set to 1.

Question: 1190

A study of the opinion of designers with respect to the primary color most desirable for us in
executive offices showed that:

Primary ColorNumber of Opinions


Red92
Orange86
Yellow46
Green91
Blue37
Indigo46
Violet2

What is the probability that a designer does not prefer red?

A. 0.73
B. 1.00
C. 0.77
D. 0.23
E. None of these answers

Answer: C

Explanation:
There are 400 opinions of which 92 prefer red. The other 308 do not prefer red. Therefore the
probability of not preferring red is 308/400 = 0.77.

Question: 1191

Events A and B are mutually exclusive. P(A) = 0.45, P(B) = 0.27. The probability of neither A nor B
occurring equals ________.

A. 0.28
B. 0.88
C. 0.72
D. 0.12

Answer: A

Explanation:
The probability of neither A nor B occurring equals one minus the probability of either A or B
occurring i.e. P(neither A nor B) = 1 - P(A or B). Now,
P(A or B) = P(A) + P(B) - P(A and B).
Also, for mutually exclusive events, P(A and B) = 0 by definition. Therefore,
P(A or B) = 0.45 + 0.27 = 0.72. Finally, P(neither A nor B) = 1 - P(A or B) = 1 - 0.72 = 0.28.

Page | 441
Question: 1192

A selected group of employees of Unique Buying Services is to be surveyed with respect to a new
pension plan. In depth interviews are to be conducted with each employee selected in the sample.
The employees are classified as follows.
ClassificationEventNumber of Employees
SupervisorsA120
MaintenanceB50
ProductionC1,460
ManagementD302
SecretarialE68
What is the probability that the first person selected is in management AND in supervision?

A. 0.00
B. None of these answers
C. 0.15
D. 0.21
E. 0.06

Answer: A

Explanation:
There are 2000 employees of which 302 are in management and 120 in supervision. However, there
are no employees who are in both management and supervision. Therefore the probability is 0.

Question: 1193

What is the annual Internal Rate of Return of this series of annual cash flows: Year 0: <$25,000>, Year
1: $5,000, Year 2: $0, Year 3: $30,000? (Note that the <> are used to indicate a negative number).

A. 6.21%
B. 15.15%
C. 14.34%
D. 13.37%
E. 12.12%

Answer: D

Explanation:
On the BAII Plus, press CF 2nd CLRWork 25000 +/- ENTER DownArrow 5000 ENTER DownArrow
DownArrow 0 ENTER DownArrow DownArrow 30000 ENTER DownArrow DownArrow 2nd Quit. Then
press Irr CPT. On the HP12C, press these keys: 25000 CHS BlueShift CFo 5000 BlueShift CFj 0 BlueShift
CFj 30000 BlueShift CFj Then press YellowShift Irr. The "DownArrow" represents the downward-
pointing arrow on the top row of the BAII Plus keyboard. Make sure the BAII Plus has the P/Y value
set to 1.

Question: 1194

When a researcher uses the classes 129-147, 147-165, 165-183 to create a distribution, he is violating
which of the following suggested practices?

A. Avoid a use of non-standard class marks.


B. Avoid a use of open-ended classes.

Page | 442
C. Avoid a use of uneven classes.
D. Avoid a use of overlapping classes.

Answer: D

Explanation:
The classes are overlapping (for e.g., the classes "147-165" and "165-183" have the point "165" in
common). There are no open-ended classes and the classes have the same class interval. Note that
"Avoid a use of non-standard class marks" is a bogus choice; there is no such suggested practice.

Question: 1195

Your distant uncle's will specifies that you will receive a 20-year annuity of $10,000 a year, the
payments starting seven years from now. How much is the annuity worth to you today if your
discount rate is 4% per year?

A. $107,406
B. $135,903
C. $122,321
D. $110,911

Answer: A

Explanation:
The payments start at the end of 7 years. Thus, at the end of year 6, the annuity will be worth
10,000/0.04*(1-1/(1.04^20)) = 135,903. The present value of this amount is 135,903/(1.04^6) =
107,406.

Question: 1196

What do we call the value called z that is computed using sample data?

A. Decision rule
B. Test statistic
C. Parameter
D. None of these answers

Answer: B

Explanation:
The z-values are statistics generated to provide a measure of testing the hypothesis.

Question: 1197

Carlson Jewelers permits the return of their diamond wedding rings, provided the return occurs
within two weeks of the purchase date. Their records reveal that 10 percent of the diamond wedding
rinds are returned. Five rings are bought by five different customers. What is the probability that
none will be returned?

A. 0.372
B. 0.590
C. 0.500
D. 0.073
E. 0.250

Page | 443
Answer: B

Explanation:
This is a binomial probability. The probability of getting r successes out of n trials where the
probability of success each trial is p and probability of failure each trial is q (where q = 1-p) is given
by: n!(p^r)[q^(n-r)]/r!(n-r)!. Here n = 5, r = 0,p = 0.10 and q = 0.90. Therefore we have
5!(0.1^0)(0.9^5)/0!5! = 0.590.

Question: 1198

What is the annual Internal Rate of Return of this series of annual cash flows: Year 0: <$4,000>, Year
1: $2,000, Year 2: $0, Year 3: $0, Year 4 number).

A. 14.04%
B. 15.28%
C. 11.61%
D. 25.29%
E. 12.59%

Answer: B

Explanation:
On the BAII Plus, press CF 2nd CLRWork 4000 +/- ENTER DownArrow 2000 ENTER DownArrow
DownArrow 0 ENTER DownArrow DownArrow 0 ENTER DownArrow DownArrow 4000 ENTER
DownArrow DownArrow 2nd Quit. Then press Irr CPT. On the HP12C, press these keys: 4000 CHS
BlueShift CFo 2000 BlueShift CFj 0 BlueShift CFj 0 BlueShift CFj 4000 BlueShift CFj. Then press
YellowShift Irr. The "DownArrow" represents the downward-pointing arrow on the top row of the
BAII Plus keyboard. Make sure the BAII Plus has the P/Y value set to 1.

Question: 1199

Which of the following is/are true?


I. Multiplying every value in a distribution by 2 will double the sample variance.
II. Multiplying every value in a distribution by 2 will double the sample mean.
III. Adding a constant to every value in a distribution increase the sample variance by the square of
the constant.

A. I, II & III
B. I only
C. II only
D. III only
E. I & III

Answer: C

Explanation:
Multiplying every value in a distribution by 2 will double the sample mean and standard deviation
and quadruples the variance. Adding a constant to every value in a distribution leaves the sample
variance unchanged.

Question: 1200

If one or more numbers in a dataset are negative, which of the following is not always defined?

Page | 444
A. mean
B. median
C. geometric mean
D. mode

Answer: C

Explanation:
The geometric mean involves taking mathematical roots. These operations are not always defined to
give real numbers when you are dealing with negative numbers.

Question: 1201

A survey of 144 retail stores revealed that a particular brand and model of a VCR retails for $375 with
a standard deviation of $20. What is the 99% confidence interval to estimate the true cost of the
VCR?

A. $328.40 to $421.60
B. $323.40 to $426.60
C. $335.80 to $414.20 #AI None of these answers
D. $335.80 to $415.20

Answer: B

Explanation:
For 99%, the z value will be 2.575. Therefore we have 375 +/- 2.575*20.

Question: 1202

What monthly deposit would you need to make, beginning one month from today, into an account
paying 11% per year, compounded monthly, in order to have $500,000 in 30 years?

A. $14,555.70
B. $178.28
C. $218.67
D. $129.83
E. $314.48

Answer: B

Explanation:
On the BAII Plus, press 360 N, 11 divide 12 = I/Y, 0 PV, 500000 FV, CPT PMT. On the HP12C, press 360
n, 11 ENTER 12 divide i, 0 PV, 500000 FV, PMT. Note that the answer will be displayed as a negative
number. Make sure that the BAII Plus has the value of P/Y set to 1.

Question: 1203

Each salesperson in a large department store chain is rated either below average, average, or above
average with respect to sales ability. Each salesperson is also rated with respect to his or her
potential for advancement either fair, good, or excellent. These traits are the 500 salespeople were
cross classified into the following table.

Sales AbilityPotential for Advancement

Page | 445
FairGood Excellent
Below Average161222
Average456045
Above Average9372135

What is the probability that a sales person selected at random will have below average sales ability
and fair potential for advancement?

A. 0.32
B. None of these answers
C. 0.10
D. 0.16
E. 0.032

Answer: E

Explanation:
prob. of selecting a below average person: (16 + 12 + 22)/500 = 1/10. Prob. of selecting fair potential
amongst those in the below average group: 16/50 = 8/25. Therefore 1/10*8/25 = 4/125.

Question: 1204

A distribution has a mean equal to 12 and a standard deviation of 36. It has a coefficient of variation
equal to:

A. zero
B. none of these answers
C. 0.335
D. 3.00

Answer: D

Explanation:
The coefficient of variation equals the standard deviation divided by mean.

Question: 1205

John's great-grandfather left him $100 when he died 100 years ago in an account paying 5% per year,
compounded annually. How much would the account hold for John now?

A. $12,504.51
B. $13,150.13
C. $14,229.78
D. $8,407.76
E. $14,113.90

Answer: B

Explanation:
On the BAII Plus, press 100 N, 5 I/Y, 100 PV, 0 PMT, CPT FV. On the HP12C, press 100 n, 5 i, 100 PV, 0
PMT, FV. Note that the answer will be displayed as a negative number. Make sure the BAII Plus has
the P/Y value set to 1.

Question: 1206

Page | 446
If we reject the null hypothesis what can we conclude subject to the alpha risk?

A. Alternative hypothesis is false


B. Null hypothesis is false
C. Null hypothesis is true
D. Both the null hypothesis and the alternative hypothesis are false
E. Both the null hypothesis and the alternative hypothesis are true

Answer: B

Explanation:
The alpha risk is the probability of rejecting the null when it is actually true. So if we had rejected the
null, we can conclude, given what we know of the alpha, that the null is false.

Question: 1207

Which is true of all skewed distributions?


I. They are is not symmetrical.
II. Their mean and median are not equal.
III. They are characterized by many small values and a few extreme values.

A. I and III
B. None of these answers is correct.
C. I and II
D. II and III

Answer: C

Explanation:
A skewed distribution is not symmetrical and its mean and median are not equal. While a positively
skewed distribution is characterized by many small values and a few extreme values, the opposite is
true of negatively skewed distributions. So III is false as far as "all skewed distributions" are
concerned.

Question: 1208

A stock had the following percentage increases in its value over the last 5 years: 7%, 4%, 12%, 10%,
15%. The geometric rate of return on the stock equals ________.

A. 8.72%
B. 9.22%
C. 9.60%
D. 9.53%

Answer: D

Explanation:
The geometric rate of change equals (1.04*1.07*1.1*1.15*1.12)^(1/5) - 1 = 9.53%. Note this is
different from the straight geometric mean, which in this case would be
(0.04*0.07*0.1*0.15*0.12)^(1/5) = 8.72%. You should be very careful about this point since the
Mason & Lind textbook is quite ambiguous on this point.

Question: 1209

Page | 447
What is the Net Present Value of this series of annual cash flows at an interest rate of 10% per year:
Year 0: <$25,000>, Year 1: $2,000, Year 2: $0, Year 3: $15,000, Year 4 $0, Year 5 $18,000? (Note that
the <> are used to indicate a negative number).

A. <$735.51>
B. $314.37
C. <$279.41>
D. $42.21
E. <$967.49>

Answer: A

Explanation:
On the BAII Plus, press CF 2nd CLRWork 25000 +/- ENTER DownArrow 2000 ENTER DownArrow
DownArrow 0 ENTER DownArrow DownArrow 15000 ENTER DownArrow DownArrow 0
ENTERDownArrow DownArrow 18000 ENTER DownArrow DownArrow 2nd Quit. Then press NPV 10
ENTER DownArrow CPT. On the HP12C, press these keys: 25000 CHS BlueShift CFo 2000 BlueShift CFj
0 BlueShift CFj 15000 BlueShift CFj 0 BlueShift CFj 18000 BlueShift CFj. Then press 10 i, YellowShift
NPV. The "DownArrow" represents the downward-pointing arrow on the top row of the BAII Plus
keyboard. Make sure the BAII Plus has the P/Y value set to 1.

Topic 3, Economics

Question: 1210

The current exchange rate for French francs is $0.20. For a U.S. bank this is an example of:

A. An indirect quote in European terms


B. An indirect quote in American terms
C. A direct quote in American terms
D. A direct quote in European terms

Answer: C

Explanation:
A direct quote provides the home currency price of a specific quantity of foreign currency. An indirect
quote states the foreign currency price of one unit of home currency. A quote in American terms
gives the number of US dollars per units of foreign currency, and in European terms gives the
number of foreign currency units per U.S. dollar.

Question: 1211

If a nation is running a "trade deficit," it is

A. spending more on public services than it is raising in tax revenues.


B. exporting more goods and services than it imports.
C. importing more goods and services than it exports.
D. encountering a balance of payments disequilibrium.
E. worse off as the result of its trade with foreign countries.

Answer: C

Explanation:

Page | 448
A trade deficit implies that a country is importing more value in terms of goods and services than it is
exporting to foreign countries.

Question: 1212

Which of the following transactions belong to the Current account?

I. Balance of services.
II. Unrequited transfers.
III. Purchase of a foreign company.
IV. Income from foreign investments.

A. I, II, III & IV


B. I, II & III
C. I only
D. I, II & IV
E. II only
F. I & II
G. III only
H. IV only

Answer: D

Explanation:
The current account consists of Trade and Services balance, Net income from all foreign investments
and Unrequited transfers, which include aid to foreign countries, losses due to expropriation of
property by foreign governments, etc. Hence, I, II and IV are directly specified as part of the Current
account. A purchase of a foreign firm is a capital investment and as such belongs to the Capital
account.

Question: 1213

John makes T-shirts and sells them only to American citizens and he purchases one bottle of French
wine with his income. Which of the following will be true?

A. John will not be affecting the U.S. trade deficit.


B. John will be reducing the U.S. capital account surplus.
C. John will be adding to the U.S. trade deficit.
D. John will be reducing the U.S. trade deficit.

Answer: C

Explanation:
John adds to the U.S. trade deficit because he purchases imports why contributing no exports to the
trade balance.

Question: 1214

________ permits the producers of each nation to concentrate on the things they do best while
trading for those things they do least well.

A. None of these answers


B. A tariff
C. A quota

Page | 449
D. Trade

Answer: D

Explanation:
Trade leads to mutual gains because it allows each country to specialize more fully in the production
of those things that it does best.

Question: 1215

What would the bid-ask spread be for pound sterling quoted at $14419-28?

A. 6.4%
B. 0.64%
C. 6.2%
D. 0.62%
E. 0.63%

Answer: D

Explanation:
Percent spread = [(1.4428 - 1.4419)/1.4428] x 100 = 0.062%

Question: 1216

An appreciation of the U.S. dollar

A. is the same thing as an increase in the domestic price level.


B. increases the purchasing power of the U.S. dollar in foreign markets for goods and services.
C. decreases the purchasing power of the U.S. dollar in foreign markets for goods and services.
D. is the same thing as a decrease in the consumer price level.

Answer: B

Explanation:
An appreciation of the U.S. dollar implies that it takes less U.S. dollars to purchase one unit of foreign
currency. Thus, a U.S. dollar goes further than it did before. This is an increase in the purchasing
power of the U.S. dollar.

Question: 1217

Relative to the French franc, the dollar has over the period 1973 to 1993:

A. fallen from a high of 25.5 francs to the dollar in 1973 to a low of 3.4 francs to the dollar in 1985
and risen again to 12.6 francs to the dollar in 1993.
B. none of these answers.
C. fallen from a high of 22.5 francs to the dollar in 1973 to a low of 11.1 francs to the dollar in 1985 to
17.6 francs to the dollar in 1993.
D. risen from 22.5 francs to the dollar in 1973 to 35.2 francs to the dollar in 1985 to 40.5 francs to the
dollar in 1993.
E. has remained relatively stable around 15 francs to the dollar.

Answer: C

Page | 450
Explanation:
The U.S. dollar has moved greatly over this period relative to the French franc. The dollar fell from a
high of 22.5 francs to the dollar in 1973 to a low of 11.1 francs to the dollar in 1985 and then rose
again to 17.6 francs to the dollar in 1993.

Question: 1218

Which of the following is/are true?

I. All Central bank transactions are excluded from the BOP account.
II. All cash flows associated with foreign investments belong to the Capital account.
III. Repatriation of domestic currency to a foreign country by foreign nationals living in the country
are treated as part of the Current account.

A. I & II
B. III only
C. II only
D. I only

Answer: B

Explanation:
All Central Bank transactions are excluded from the Current and Capital accounts and charged to the
Official Reserve account. However, since the Reserve account is part of the BOP account, I is false.
Direct foreign investments are Capital account items. However, income associated with these is
treated as part of the Current account. Hence, II is false.
III is a direct outflow of domestic currency and treated as a debit entry on the Current account.

Question: 1219

A ________, which is between a bank and a customer (or another bank), specifies delivery at a fixed
future date, of a fixed amount of one currency against dollar payment.

A. foreign exchange contract


B. forward contract
C. futures contract
D. currency arbitrage
E. cross rate

Answer: B

Explanation:
A U.S. firm buys goods from Germany with payment of DM 500,000 due in 90 days. The current price
of the DM is $0.4490, yet may rise against the dollar which increases the dollar cost of the goods. The
U.S. firm can protect itself against this exchange risk by entering into a 90-day forward contract with
a bank at a price of $0.4511. Thus, according to the forward contract, the bank will give the U.S. firm
DM 500,000 in 90 days to pay for the goods and the U.S. firm will give the bank $225,550 million (DM
500,000 x 0.4511). If the spot rate in 90 days is less than $0.4511, the U.S. firm will experience a loss
on the forward contract because it is buying marks for more than the current rate. On the other
hand, if the spot rate is higher than $0.4511, the importer will implicitly profit.

Question: 1220

The simple or linear annualized short-term interest rate is calculated according to:

Page | 451
A. multiply the annual rate by the length of the period in months.
B. divide the annual rate by the length of the period in months.
C. multiply the annual rate by the length of the period, as a proportion of a year.
D. multiply the annual rate by the length of the period as a percentage of six months.
E. divide the annual rate by the length of the period as a proportion of the year.

Answer: C

Explanation:
The rate to be paid over the period is equal to the annual rate multiplied by the length of the period,
as a proportion of a year.

Question: 1221

________ exchange-rates are determined by the market forces of supply and demand.

A. None of these answers


B. Fixed
C. Mixed
D. Flexible

Answer: D

Explanation:
A system of flexible exchange rates is a system where the exchange rate is determined by the market
forces. Floating exchange rate is synonymous with flexible exchange rate.

Question: 1222

Exports of goods (merchandise) and services minus the imports of goods and services equals
________.

A. current account
B. balance of merchandise trade
C. balance on current accounts
D. balance on goods and services

Answer: D

Explanation:
When the balance of service exports and imports are added to the balance of merchandise trade the
balance on goods and services is obtained.

Question: 1223

A country cannot maintain currency convertibility if:

A. none of these answers.


B. it allows the exchange rate value of its currency to fluctuate and follows an independent monetary
policy.
C. it fixes the exchange rate value of its currency and has a dependent monetary policy.
D. it fixes the exchange rate value of its currency and follows an independent monetary policy.

Page | 452
Answer: D

Explanation:
A country can either follow an independent monetary policy and allow its exchange rate to fluctuate
or tie its monetary policy to the maintenance of the fixed exchange rate.

Question: 1224

When a domestic resident purchases a foreign stock directly on the foreign stock exchange, the
transaction is entered as a ________ in the BOP account.

A. deficit
B. surplus
C. credit
D. debit

Answer: D

Explanation:
In BOP accounting, by convention:

1. Any inflow of domestic currency represents a credit and any outflow of domestic currency
represents a debit on the BOP account.

2. Any inflow of foreign currency represents a debit and any outflow of foreign currency represents a
credit on the BOP account.

Thence, direct purchase of a foreign stock, which represent an outflow of domestic currency, is
treated as a debit in the BOP account.

Question: 1225

If high-yield investment opportunities attract capital from abroad and lead to a capital account
surplus, then the

A. nation will also experience a balance of trade surplus.


B. nation must run a current account surplus under a pure flexible exchange rate system.
C. nation's currency must depreciate.
D. nation must run a current account deficit under a pure flexible exchange rate system.
E. nation's currency must appreciate.

Answer: D

Explanation:
If a nation is experiencing a surplus on its capital account balance, it must experience an offsetting
deficit on its current account and vice versa. By definition, the balance of payments must be in
balance so that the capital account offsets the current account.

Question: 1226

If a restrictive fiscal policy places downward pressure on real interest rates, we would expect

A. all of these answers are correct.


B. inflationary pressures to increase.

Page | 453
C. an increase in the dollar.
D. an outflow of capital.
E. an increase in the budget deficit.

Answer: D

Explanation:
Investors facing a declining real interest rate would liquidate their assets and move their capital
elsewhere in search of a higher yield. The consequence would be a capital outflow from the country.

Question: 1227

The flow of money for the purpose of taking advantage of a covered interest differential is known as
________.

A. an outright swap
B. covered interest differential
C. the swap rate
D. covered interest arbitrage
E. interest rate parity

Answer: D

Explanation:
The flow of money for the purpose of taking advantage of a covered interest differential is known as
covered interest arbitrage'

Question: 1228

Supplementary reserves in the form of accounting entries maintained with the IMF by various
governments are known as:

A. Official Reserve Accounts.


B. BOP funds.
C. Special Depository Receipts.
D. Special Drawing Rights.

Answer: D

Explanation:
Special Drawing Rights are accounting entries established with the International Monetary Fund
which can be used to satisfy debits or credits on the BOP account. Of course, these are simply
accounting devices used to keep track of the assets or debts of the different nations arising from
international exchange. Ultimately, the entries must come to fruition through a supply of goods,
services or unrequited transfers. So if country A uses the SDR to make a short-term payment to
country B on its BOP account, it is not considered to be in default but must ultimately extinguish this
debt by supplying goods or services to the B.
The SDRs are more significant for economies trying to artificially maintain a fixed exchange rate
system.
For floating currencies, the BOPs can be at least partially offset through changes in exchange rates.

Question: 1229

________ risk is the risk that a bank will deliver currency on one side of a foreign exchange deal

Page | 454
while the counterparty does not send any money in return.

A. Trading
B. Settlement (Herstatt)
C. Foreign Exchange
D. Exchange
E. Credit

Answer: B

Explanation:
In order to minimize credit risk, most banks will transact large amounts only with blue chip
customers.

Question: 1230

Which of the following would most likely cause a nation's currency to appreciate?

A. an increase in real interest rates abroad


B. an increase in inflation of the nation's trading partners
C. an increase in the nation's domestic inflation rate
D. a decrease in domestic real interest rates

Answer: B

Explanation:
Inflation in a nation's trading partners implies that prices are rising abroad. This would increase the
demand for the nation's output because consumers will prefer imports to domestic output because it
will be relatively less expensive since its prices are not rising. Increased demand for output increases
the demand for the nation's currency and therefore the currency will appreciate.

Question: 1231

If restrictive monetary policy results in a deceleration in the domestic inflation rate and higher real
interest rates, other things constant, the

A. nation will run a current account surplus.


B. nation's currency will appreciate.
C. nation will run a capital account deficit.
D. nation will run a balance of trade surplus.
E. nation's currency will depreciate.

Answer: B

Explanation:
Contractionary monetary policy will lead to a deceleration in inflation and higher real interest rates.
As a result, demand for the nation's exports and assets will increase as will the demand for the
nation's currency. This in turn will cause the currency to appreciate.

Question: 1232

Which of the following would most likely cause a nation's currency to depreciate?

A. an increase in domestic real interest rates

Page | 455
B. higher inflation than one's trading partners and a reduction in domestic real interest rates
C. a reduction in domestic real interest rates
D. higher inflation than one's trading partners
E. an increase in exports coupled with a decline in imports

Answer: B

Explanation:
Higher relative inflation and a reduction in domestic real interest rates causes the demand for the
nation's exports and assets to decline. This in turn causes the demand for the nation's currency
todecline. Once the demand for the currency falls, the "price" of the currency (or the exchange rate)
falls. This is a depreciation in the currency.

Question: 1233

The ______of services from foreigners ________ the supply of dollars to the exchange market.

A. import; reduces
B. export; expands
C. none of these answers
D. import; expands

Answer: D

Explanation:
The import of services from foreigners expands the supply of dollars to the exchange market; service
imports are entered on the balance of payments accounts as debit items.

Question: 1234

Both a fixed exchange rate and ________ can be maintained if a country is willing to use its monetary
policy to maintain the fixed exchange rate.

A. current account deficit


B. currency convertibility
C. capital account surplus
D. capital account deficit
E. current account surplus

Answer: B

Explanation:
A country can either follow an independent monetary policy and allow its exchange rate to fluctuate
or tie its monetary policy to the maintenance of the fixed exchange rate.

Question: 1235

If prices in two countries are rising at the same annual rate, then the prices of imports and exports
will:

A. increase in the country with the more valuable currency and decrease in the country with the less
valuable currency.
B. increase relative to domestically produced goods.
C. decrease relative to domestically produced goods.

Page | 456
D. none of these answers.
E. remain unchanged relative to domestically produced goods.

Answer: E

Explanation:
If prices in two countries are rising at the same annual rate, then the prices of imports and exports
will remain unchanged relative to domestically produced goods. Equal rates of inflation in each of
the countries will not cause the value of exports to change relative to imports.

Question: 1236

The opportunity cost of producing a good:

A. generally decreases as the country produces more and more of the good.
B. is not dependent on the marginal cost of producing the good.
C. generally increases as the country produces more and more of the good.
D. is usually uniform across countries.
E. none of these answers.

Answer: C

Explanation:
The opportunity cost of producing a good beyond some level of production will often increase. This is
because of rising marginal costs as the level of output increases. Thus, the degree to which a country
will specialize in the production of a good is limited.

Question: 1237

American textile manufacturers and union members have often lobbied successfully for restrictive
quotas limiting the importation of textile products. The major impact of these quotas is

A. lower prices for American consumers and an improvement in the quality of textile products
available.
B. a permanent reduction in unemployment in the U.S.
C. higher prices for American consumers, a narrower selection of products and less competition in
the U.S. textile industry.
D. long-run profits in the U.S. textile industry that are substantially above market equilibrium.

Answer: C

Explanation:
Import quotas cause the market price for the imported good to rise domestically. This hurts
consumers but helps domestic producers who in addition to gaining market share also enjoy a higher
price.

Question: 1238

In a given year, Gondolpha had total imports of 976 and total exports of 734. It also made direct
foreign investments of 297. There were no other transactions on the BOP account. The Official
Reserves account:

A. decreases by 1,413
B. increases by 539

Page | 457
C. decreases by 539
D. decreases by 55

Answer: C

Explanation:
Remember the convention:

1. Any inflow of domestic currency represents a credit and any outflow of domestic currency
represents a debit on the BOP account.

2. Any inflow of foreign currency represents a debit and any outflow of foreign currency represents a
credit on the BOP account.

Therefore, the credits on the combined current and capital account equal 734 and the debits equal
976 + 297 = 1,273.

Since debits must equal credits at the end of the accounting period, there must be a credit entry in
the amount of (1,273 - 734) = 539 in the Official Reserves account, since the sum of the changes in
the current, capital and official reserves accounts equals zero over an accounting period. Thus, the
Official Reserves account decreases by 539.

If you do not like the above rule-based treatment of the problem, try the intuition: The total net
outflow of cash from the current and capital accounts of Gondolpha equals 976 + 297 - 734 = 539.
This amount has to come from somewhere. That "somewhere" is represented by the Official
Reserves Account, which loses 539 in order to satisfy the liabilities of the current and capital
accounts.

Caution: One of the bizarre quirks in international accounting is that decreases in Official Reserves
Account are represented by a "+."

Question: 1239

If the U.S. is viewed by foreigners as a great nation in which to invest, generating a large inflow of
foreign investment, this will cause the U.S. to run a

A. deficit on the official reserve account.


B. deficit on the capital account.
C. deficit on the current account.
D. surplus on the current account.

Answer: C

Explanation:
If foreigners invest often in the U.S. the U.S. will tend to run a capital account surplus. By definition of
the balance of payments, the U.S. must also tend to run a current account deficit since these two
accounts must counteract each other and sum to zero.

Question: 1240

Another name for flexible exchange rates is:

A. none of these answers.


B. all of these answers.

Page | 458
C. floating exchange rates.
D. moveable exchange.
E. freely determined exchange.

Answer: C

Explanation:
Flexible exchange rates, also called floating exchange rates, are determined by market forces.

Question: 1241

Suppose the lowest-wage state in the U.S. is West Virginia and the highest-wage state is New York.
Which of the following would be true?

A. If New York trades with West Virginia, wages in New York will fall until they equal the wages in
West Virginia.
B. New York would be better of if its state government imposed restrictions on the importation of
goods made in West Virginia.
C. Both New York and West Virginia will be better of if they are allowed to trade freely.
D. If New York trades with West Virginia, consumers in New York will be worse off.

Answer: C

Explanation:
Low-wage and high-wage states (like countries) are better off if they are allowed to freely trade with
each other. The comparative advantage of low wage states will be in the production of labor
intensive goods while the advantage for high wage states will be in the production of capital
intensive goods. By specializing in the area of their comparative advantage, total production for each
state will increase as will real income.

Question: 1242

The domestic demand Q for a good A at a price P is given by Q = 500 - 5P while the supply function is
given by 300 + 3P. The world price for good X is 19.
If the government imposes a 10% tariff on imports, the revenues of the domestic producers will

A. decrease by 233.
B. increase by 579.
C. increase by 797.
D. decrease by 485.

Answer: C

Explanation:
First note that without imports, the price prevailing in the domestic market will satisfy 500 - 5P = 300
+ 3P, giving P = 25. The world price is 19 and with a 5% import tariff, it becomes 19 * 1.1 = 20.9. Since
this price is lower than 25, there will continue to be imports and the price prevailing in the domestic
market after the tariffs will equal 20.9. Before the tariffs, the producers supply a quantity equal to
300 + 3 * 19 = 357 and have revenues of 357 * 19 = 6,783. With the tariff in place, the producers
produce 300 + 3 * 20.9 = 362.7 units and have revenues of 362.7 * 20.9 = 7,580. Thus, the revenues
increase by 7,580 - 6,783 = 797.
Note that we have implicitly used the fact that the domestic producers do not have to pay the tariff
and pocket the entire higher price.

Page | 459
Question: 1243

What gives the home currency price of a certain quantity of the foreign currency quoted?

A. bid-ask spread
B. spot rate
C. cross rate
D. direct quotation
E. indirect quotation

Answer: D

Explanation:
For example, the price of foreign currency is expressed in French francs in France and in Deutsche
marks in Germany. Thus, in France, the Deutsche mark might be quoted at FF 4, whereas in Germany,
the franc would be quoted at DM 0.25.

Question: 1244

Given the following data, what will an arbitrageur earn after one year on a covered interest arbitrage
if he begins with $1 million?

In New York todayInterest rate = 7%


In London todayspot rate = $1.75/L
Interest rate = 12%
In London one yearforward rate = $1.68/L

A. $7,560
B. $5,200
C. $4,300
D. $3,580
E. $6,800

Answer: B

Explanation:
Begin by borrowing $1,000,000 at 7%, owing $1,070,000 at year end.
Convert the $1,000,000 to pounds at $1.75/L = L571,428.57 and invest this in London at 12% yielding
L640,000 at year end. At the same time, sell the L640,000 forward at a rate of $1.68/L for delivery in
one year yielding $1,075,200 at year end. At the end of the year, collect the L640,000 and deliver to
the bank's foreign exchange dept. in return for $1,075,200. Repay the $1,070,000 from the
$1,075,200 leaving a profit of $5,200.

Question: 1245

Which of the following is correct?

A. Without tariff protection the number of jobs available to domestic workers would decline.
B. Less than 5 percent of world output is sold in a country different from the one in which it is
produced.
C. Exports represent about 30 percent of U.S. GDP.
D. Regarding international trade, if one country gains, another country must lose.
E. The volume of international trade has grown rapidly in recent decades.

Page | 460
Answer: E

Explanation:
The volume of international trade, enhanced by improved transportation and communication has
grown rapidly in recent years. Approximately 21 percent of the world's total output is now sold in a
country other than that in which it was produced. This is double the figure of three decades ago.

Question: 1246

If the one month FF/$ exchange rate is 8.0200-50 this implies:

A. the bank is willing to commit itself today to sell dollars in one month for 8.0200 francs or to buy
them for 8.0250 francs.
B. the bank is willing to commit itself today to buy dollars in one month for 8.0200 francs or to sell
them for 8.0250 francs.
C. the bank is willing to commit itself today to buy dollars for 8.0200 francs or to sell them for 8.0250
francs.
D. the bank is willing to commit itself today to buy dollars in one month for 8.0200 francs or to sell
them for 8.0150 francs.
E. none of these answers

Answer: B

Explanation:
In a forward, or futures, contract, a commitment is irrevocably made on the transaction date, but
delivery takes place later, here a month later. The bid-ask quotation here implies that the bank is
willing to commit itself today to buy dollars in one month for 8.0200 francs or to sell them for 8.0250
francs.

Question: 1247

If the French franc moves from 8.00 to 7.10 francs to the dollar:

A. the dollar now trades for 0.141 francs.


B. the French franc has depreciated.
C. the franc now trades for 7.10 dollars.
D. the French franc has appreciated.
E. the U.S. dollar has appreciated.

Answer: D

Explanation:
Since the U.S. dollar now purchases less French francs the U.S. dollar has depreciated; symmetrically,
the French franc has appreciated. Each franc now buys .141 dollars whereas before each franc
purchased .125 dollars.

Question: 1248

The exchange rate of a country's currency will ________ if the income of the country and its trading
partners rises.

A. neither of these answers


B. increase
C. This answer depends on which income rises the fastest: for countries that are similar in size and

Page | 461
propensity to import, the country that is growing the fastest will increase its demand for imports
relatively more than its trading partner, resulting in a decrease in the value of the more rapidly
growing nation's currency.
D. decrease

Answer: C

Explanation:
Fore countries are similar in size and propensity to import, the country that is growing the fastest will
increase its demand for imports relatively more than its trading partner, resulting in a decrease in the
value of the more rapidly growing nation's currency. Sluggish growth of income relative to one's
trading partners tends to cause the slow-growth nation's currency to appreciate.

Question: 1249

The supply curve for British pounds in the foreign exchange market:

A. is independent on the purchases of British goods by foreigners.


B. none of these answers.
C. is dependent on the purchases of foreign goods by the British.
D. is dependent on the purchases of British goods by foreigners.

Answer: C

Explanation:
The supply curve for pounds is dependent on the purchases of foreign goods by British citizens. An
increase in the foreign price of the pound means that a pound will purchase more foreign currency
and more goods priced in terms of foreign currency. The British buy more from abroad and therefore
supply more pounds to the market as the foreign price of the pound rises.

Question: 1250

When both exports and imports are considered, the major advantage of international trade is that it
allows us to

A. share our technology and efficiency with less-developed countries that would otherwise never
have the opportunity to observe modern goods and services.
B. sample foreign products that many of us would otherwise never see.
C. consume a larger, more diverse quantity of goods and services at lower prices than would
otherwise prevail.
D. maintain jobs for workers who would otherwise have little to do.

Answer: C

Explanation:
In the absence of trade the consumption of each country is constrained by the country's production
possibilities. Trade expands the consumption possibilities of both countries. Specialization and
exchange permits two countries to expand their joint output and as a result, both countries can
increase their consumption of both all goods.

Question: 1251

A tax levied on imported goods is called a(n)

Page | 462
A. excise tax.
B. foreign profits tax.
C. tariff.
D. quota.

Answer: C

Explanation:
A tariff is a tax levied on goods imported into a country.

Question: 1252

Under a flexible exchange rate system, a nation that offers more attractive investment opportunities
than its trading partners can expect to run a

A. balance of merchandise trade surplus.


B. deficit on its capital account transactions.
C. surplus on current account transactions.
D. deficit on current account transactions.

Answer: D

Explanation:
Since a high yield on investments will attract capital, the nation will run a capital account surplus
which must be offset by a current account deficit.

Question: 1253

According to the method for calculating short-term interest rates from annualized rates, a three-
month interest rate of 4% means that the interest rate paid after three months will be ________.

A. .25%
B. 1%
C. 2%
D. 4%
E. none of these answers

Answer: B

Explanation:
The rate to be paid over the period is equal to the annual rate multiplied by the length of the period,
as a proportion of a year. Thus: 4%(3/12) = 1%.

Question: 1254

If you are about to go to Brazil and would like to obtain Brazilian currency, this acquisition would be
accomplished in the

A. internal affairs office.


B. Office of the U.S. Treasury.
C. Federal Reserve market.
D. export/import market.
E. foreign exchange market.

Page | 463
Answer: E

Explanation:
The foreign exchange market is the market in which the currencies of different countries are bought
and sold. Thus, in order to exchange your dollars for cruzeiros you must access the foreign exchange
market.

Question: 1255

The inevitable consequence of price controls is:

A. increased currency convertibility.


B. none of these answers.
C. the development of black markets.
D. trade war.

Answer: C

Explanation:
Since price controls, of which exchange rate control are a type, lead to shortages, black markets
develop.

Question: 1256

An unanticipated shift to a more expansionary monetary policy will most likely cause

A. depreciation in the exchange rate of the nation's currency, higher real interest rates and an inflow
of capital.
B. appreciation in the exchange rate of the nation's currency, higher real interest rates and an inflow
of capital.
C. appreciation in the exchange rate of the nation's currency, lower real interest rates and an outflow
of capital.
D. depreciation in the exchange rate of the nation's currency, lower real interest rates and an outflow
of capital.

Answer: D

Explanation:
An unanticipated expansionary monetary policy will increase economic growth, accelerate in the
inflation rate and lower real interest rates. These factors lead to an decrease in the demand for the
nation's exports and assets and consequently a decrease in the demand for the nation's currency.
This serves to cause the currency to depreciate. Lower real interest rates cause a capital outflow
because investors liquidate their assets in search of a higher yield abroad.

Question: 1257

The dollar would have appreciated if

A. the U.S. had a balance of trade surplus under a system of flexible exchange-rates.
B. under fixed exchange-rates, the number of dollars demanded in the foreign exchange market
increased.
C. it had been exchanging for one mark but can now be exchanged for two.
D. it had been exchanging for three marks but can now be exchanged for two.

Page | 464
Answer: C

Explanation:
An increase in the value of a domestic currency relative to foreign currencies defines a currency
appreciation. Thus, the value of the dollar in this example has increased since one dollar now buys
two marks whereas before it only purchased one mark. The U.S. dollar now "buys" more marks.

Question: 1258

Under a system of fixed exchange rates, which of the following will most likely increase a balance of
payments deficit?

A. an increase in foreign-aid grants


B. an increase in foreign-aid grants and expansionary monetary policy, which will drive prices up and
interest rates down
C. expansionary monetary policy, which will drive prices up and interest rates down
D. a reduction in income from investments abroad
E. restrictive monetary policy, which will keep prices down and interest rates up

Answer: C

Explanation:
Expansionary monetary policy will create inflation and cause interest rates to fall. Both of these
consequences will cause the demand for U.S. exports and assets to fall which will increase the
balance of payments deficit.

Question: 1259

A Japanese investor purchasing a U.S. government bond

A. causes the yen to appreciate.


B. creates a demand for yen and a supply of dollars in the foreign exchange market.
C. causes the dollar to depreciate.
D. creates a demand for dollars and a supply of yen in the foreign exchange market.

Answer: D

Explanation:
A foreigner must pay for a U.S. bond using U.S. dollars. Therefore, the investor must exchange
Japanese yen on the foreign exchange market for U.S. dollars. This transaction increases the supply
of Japanese yen and the demand for U.S. dollars.

Question: 1260

When referring to foreign exchange trades, "American terms" refers to:

A. a direct quote outside the U.S.


B. trades by Americans living abroad
C. the number of foreign currency units per U.S. dollar
D. an indirect quote in the U.S.
E. the number of U.S. dollars per unit of foreign currency

Answer: E

Page | 465
Explanation:
"European terms" refers to the number of foreign currency units per U.S. dollar.

Question: 1261

In a purely floating exchange rate economy, a shift toward a more expansionary fiscal policy will
move the capital account toward a ________. The current account will move toward a ________.

A. surplus, surplus too


B. surplus, deficit
C. deficit, surplus
D. deficit, deficit too

Answer: B

Explanation:
An expansionary fiscal policy serves to raise interest rates and increase capital inflow. This moves the
capital account toward a surplus. Since the changes in the capital and current accounts must balance
out in a purely floating exchange rate economy, the current account moves toward a deficit.

Question: 1262

In an open economy, the market demand for domestic products is

A. overstated by the horizontal sum of the domestic and foreign demand.


B. the horizontal sum of domestic and foreign demand.
C. understated by the horizontal sum of the domestic and foreign demand.
D. the vertical sum of domestic and foreign demand.

Answer: B

Explanation:
The domestic producer can supply both the domestic consumer and consumers abroad. Thus, market
demand, which is the addition of domestic and foreign demand, can be determined by finding the
domestic demand (at the intersection of the domestic demand curve with the international price)
and adding it to foreign demand (found in the same way with the demand curve for foreigners).

Question: 1263

________ utilize forward contracts to safeguard the value (in home currency terms) of assets
denominated in a foreign currency on its balance sheet.

A. All of these answers


B. Arbitrageurs
C. Traders
D. Hedgers
E. Speculators

Answer: D

Explanation:
Speculators choose to be exposed to currency risk (unlike the three above who attempt to
minimize/eliminate risk) by using forward contracts to benefit from exchange rate fluctuations.
Arbitrageurs utilize forward contracts to rid themselves of exchange risk in the currency markets.

Page | 466
Traders utilize forward contracts to end (or cover) the chance of loss on export or import orders that
are denominated in foreign currencies.

Question: 1264

The supply curve for American dollars in the foreign exchange market:

A. none of these answers.


B. is dependent on the purchases of American goods by foreigners.
C. is independent on the purchases of American goods by foreigners.
D. is dependent on the purchases of foreign goods by Americans.

Answer: D

Explanation:
The supply curve for dollars is dependent on the purchases of foreign goods by Americans. An
increase in the foreign price of the dollar means that a dollar will purchase more foreign currency
and more goods priced in terms of foreign currency. The Americans buy more from abroad and
therefore supply more dollars to the market as the foreign price of the dollar rises.

Question: 1265

International trade can benefit domestic firms because:

A. the market contracts and allows economies of scale in production.


B. consumers purchase more from local firms.
C. none of these answers.
D. competition is promoted in domestic markets and consumers can purchase a wide diversity of
goods at economic prices.
E. opportunity costs for all goods increase.

Answer: D

Explanation:
Competition from abroad helps keep domestic producers on their toes. Domestic producers that
otherwise might have few rivals will have to constantly be seeking ways to improve quality and keep
costs low.
Simultaneously, the diversity of goods that is available from abroad provides consumers with a
broader array of choices than would be available in the absence of international trade.

Question: 1266

Inflation contributes to the depreciation of a nation's currency only when a country's rate of inflation
is ________ that of its trading partners.

A. less rapid than


B. equal to
C. more rapid than
D. none of these answers, inflation rate is irrelevant

Answer: C

Explanation:
If the rate of inflation in a nation is greater than that of its trading partners prices are rising faster at

Page | 467
home than abroad. As a result, foreign consumers demand less of the nation's exports because they
are now relatively more expensive. Falling demand for exports causes the demand for the nation's
currency to decline and thus the currency depreciates.

Question: 1267

Which one of the following will most likely cause a depreciation in the U.S. dollar on the foreign
exchange market under a system of flexible exchange rates?

A. a sudden increase in the demand for California cabernet wines by Europeans


B. an increase in real interest rates among European nations
C. a switch from French stocks and bonds to U.S. stocks and bonds by U.S. speculators
D. rampant inflation in Japan, a major trading partner of the U.S.

Answer: B

Explanation:
An increase in real interest rates abroad will cause the demand for high yield assets in those
countries to increase. As a result the demand for assets in the U.S. will fall, as will the demand for
U.S. currency. The U.S. dollar will depreciate under falling demand.

Question: 1268

State the formula for approximating interest rate parity.

A. rh - rf = f1 - e0 * e0
B. rh + rf = f1 - e0/e0
C. rh + rf = f1 + e0/e0
D. rh - rf = f1 + e0/e0
E. rh - rf = f1 - e0/e0

Answer: E

Explanation:
Interest rate parity will hold when there are no covered interest arbitrage opportunities.

Question: 1269

Capital account transactions include:

A. direct investments by Americans in real assets abroad and loans from foreigners.
B. direct investments by Americans in real domestic assets and loans to and from foreigners.
C. unilateral transfers to and from foreigners and direct investments by Americans in real assets
abroad.
D. direct investments by Americans in real assets abroad and loans to and from foreigners.
E. unilateral transfers to and from foreigners and loans to and from foreigners.

Answer: D

Explanation:
Capital account transactions are composed of direct investment by Americans in real assets abroad
(or by foreigners in the U.S.) and loans to and from foreigners.

Question: 1270

Page | 468
The domestic demand Q for a good X at a price P is given by Q = 400 - 2P while the supply function is
given by 100 + 4P. The world price for good X is 45 in international markets. All quantities are in
thousands of units. If the government imposes a tariff of 5% on imports, the imports will fall by:

A. 9.75
B. 13.25
C. 16.5
D. 19

Answer: C

Explanation:
First note that without imports, the price prevailing in the domestic market will satisfy 400 - 2P = 100
+ 4P, giving P = 50. The world price is 45 and with a 5% import tariff, it becomes 45 * 1.05 = 47.25.
Since this price is lower than 50, there will continue to be imports and the price prevailing in the
domestic market after the tariffs will equal 47.25.

Before the tariffs, the producers supply a quantity equal to 100 + 4 * 45 = 280 and domestic
consumers demand 400 - 2 * 45 = 310 units. Thus, imports without the tariffs equal 300 - 280 = 30
units. With the tariff in place, the producers produce 100 + 4 * 47.25 = 289 units. The consumers
demand 400 - 2 * 47.25 = 305.5 units. Thus, the imports now equal 305.5 - 289 = 16.5 units.

Question: 1271

The ________ elasticity of domestic demand for imports and foreign demand for exports is ______.

A. short-run; elastic
B. long-run; inelastic
C. short-run; unit-elastic
D. none of these answers
E. short-run; inelastic

Answer: E

Explanation:
The short run elasticity of domestic demand for imports and foreign demand for exports is inelastic.
So a depreciation initially increases import expenditures and export sales. In the long run, the
demand for both imports and exports is elastic.

Question: 1272

Which of the following would be a debit in the U.S. balance of payments?

A. a short-term loan extended to a Japanese manufacturer by a U.S. bank


B. the purchase of a Japanese car by an American
C. the purchase of air service from a U.S. airline by a Japanese traveler
D. the purchase of U.S. grain by a Japanese bakery

Answer: B

Explanation:
Since the purchase of a car from a Japanese firm by an American increases the supply of U.S. dollars
on the foreign exchange market, this transaction is accounted for as a debit on the U.S. current

Page | 469
account.

Question: 1273

A(n) ________ in a nation's currency in the foreign exchange market compensates for the nation's
high ________ rate.

A. depreciation; exchange
B. appreciation; exchange
C. depreciation; inflation
D. appreciation; inflation
E. none of these answers

Answer: C

Explanation:
Exchange rate adjustments permit nations with even high rates of inflation to engage in trade with
countries experiencing relatively stable prices. A depreciation in a nation's currency in the exchange
market compensates for the nation's inflation rate.

Question: 1274

Compared to the no-trade situation, when a country exports a good

A. domestic consumers gain, domestic producers lose and the losses outweigh the gains.
B. domestic consumers gain, domestic producers lose and the gains outweigh the losses.
C. domestic producers gain, but domestic consumers lose an equal amount.
D. domestic producers gain, domestic consumers lose and the gains outweigh the losses.

Answer: D

Explanation:
Domestic producers gain under export because only goods for which the international price is higher
than the domestic price will be exported. Thus, the producer will receive a higher price abroad than
it is receiving at home. Domestic consumers lose because producers will reduce their output for
domestic consumption which will cause the price to rise. The gains outweigh the losses because the
country will divert resources away from inefficient production to the efficient production of the
exported goods.
Producers will be motivated to divert these resources by higher prices abroad.

Question: 1275

Which of the following is the best example of a quota?

A. a tax placed on all small cars sold in the domestic market


B. a limit imposed on the number of men's suits that can be imported from a foreign country
C. a $100-per-car excise tax levied on all small cars sold in the domestic market
D. a subsidy from the U.S. government to domestic manufacturers of small cars so they can compete
more effectively with foreign producers of small cars
E. a $100-per-car fee imposed on all small cars imported

Answer: B

Explanation:

Page | 470
An import quota is a specific quantity (or value) of a good permitted to be imported into a country
during a given year.

Question: 1276

For an upcoming sight-seeing visit to India, a U.S. resident recently purchased a hundred thousand
Indian Rupees. His action

A. created a credit balance in the U.S. BOP account.


B. created a debit balance in the U.S. BOP account.
C. created a deficit in the U.S. trade account.
D. none of these answers.

Answer: B

Explanation:
This is a point of convention where much confusion exists. Before you answer it, you should
remember two things:

In International Finance, a country's reserve of foreign currency and foreign- produced goods is
treated as an asset.

In Accounting, by convention, increases in assets are recorded as "debit" entries and reductions are
treated as "credit" entries.

In the above example, there is an inflow of Indian Rupees and is hence recorded as a debit to the U.S.
BOP account.

Of course, it is also quite important to remember that the BOP account must balance out, just as
surely as you must have debits = credits in accounting. The subtle part in BOP accounting is to
recognize the offsetting credit entry that arises. Consider how this happens in the above example:

The dollars used to buy the Indian rupees represent a liability for the U.S. This debt will be
extinguished during the year through a supply of goods by U.S. residents in the form of exports or
end up as a recognition of liability on the Official Reserve account when the accounts are balanced at
the end of an accounting period (quarter end or year end). When this happens, the offsetting credit
entry shows up on the accounts and the national balance sheet balances out. Note that exports are
thus treated as credits and imports are treated as debits on the BOP account.

In summary,

1. Any inflow of domestic currency represents a credit and any outflow of domestic currency
represents a debit on the BOP account.

2. Any inflow of foreign currency represents a debit and any outflow of foreign currency represents a
credit on the BOP account.

Question: 1277

The price for French francs is $0.1945-67. What is the percent spread for this currency?

A. 0.99%
B. 1.12%
C. 11.3%

Page | 471
D. 1.13%
E. 11.2%

Answer: B

Explanation:
The percent spread is the (ask price - bid price)/ask price times 100. In this case, (0.1967 -
0.1945)/0.1967 x 100 = 1.12%

Question: 1278

Which of the following factors will cause country A's currency to appreciate relative to the currency
of country B, all else equal?

I. A has higher income growth.


II. A has higher inflation.
III. B has higher real interest rate.

A. II & III
B. None of them
C. I, II & III
D. I only
E. II only
F. III only
G. I & III

Answer: B

Explanation:
A country's currency will appreciate relative to those of its trading partners if it has lower income
growth (which will cause the increase in imports to lag behind the increase in exports), has a lower
inflation rate or offers higher real interest rates.

Question: 1279

The government currently has no debt. To alleviate an economic slow down, Congress proposes an
increase in spending partially offset by an increase in the highest marginal tax rate. This will force the
Treasury to issue new debt. According to which of the following economic theories would this
stimulate the economy?

I. fiscal policy
II. supply-side
III. crowding out

A. none of these answers is correct


B. I only
C. I, II, III
D. II only
E. II, III

Answer: B

Explanation:
Anytime the government runs a deficit, it can be considered a fiscal stimulus. However, since

Page | 472
marginal tax rates were increased, the supply-side effect would be negative. The crowding out effect
suggests that government borrowing will increase interest rates and partially offset the stimulate
effects of the new spending.

Question: 1280

If the quality of education in the U.S. improved, the direct effect would be a(n)

A. reduction in prices.
B. increase in both output and prices.
C. increase in both long-run and short-run aggregate supply.
D. increase in both short-run aggregate supply and aggregate demand.

Answer: C

Explanation:
Increases in education, training and skill-enhancing experience can improve the quality of the labor
force and thereby expand the supply of human resources, This could cause the cost of resources to
decline (permanently) and expand both short run output and long run sustainable output.

Question: 1281

The crowding-out effect refers to the possibility that an

A. increase in consumption spending will crowd out government spending.


B. increase in private savings will crowd out the taxable income of households.
C. increase in the money supply will result in a decline in taxes.
D. increase in the federal deficit will result in higher interest rates, which will crowd out private
investment and consumption.

Answer: D

Explanation:
The crowding out theory implies that government borrowing drives up real interest rates and thus
crowds out" private investment. Private investment falls under higher interest rates because the cost
of investment (the real interest rate) rises if the government borrows heavily. Under the usual law of
supply and demand, the government causes the interest rate to rise under deficit spending because
there is a limited supply of loanable funds. The government competes with the private sector for
these resources and thus drives up the price (i.e., the interest rate).

Question: 1282

An acceleration in the growth rate of the money supply is known as ________.

A. expansionary monetary policy


B. restrictive monetary policy
C. the quantity of money
D. the velocity of money
E. excess supply of money

Answer: A

Explanation:
The Fed generally pumps additional money into the economy via open market operations - the

Page | 473
purchase of bonds in the open market. The consequence of this action is usually a rise in the real
interest rate. The Fed can also implement expansionary monetary policy through manipulation of the
discount rate. A higher discount rate will serve to contract the money supply; a lower rate will
expand the money supply.

Question: 1283

Which of the following would be counted in the calculation of GDP?

A. the sale of cocaine in the black market


B. the broker's fees from the sale of 100 shares of IBM stock
C. damage to a house caused by a hurricane
D. the sale of a rare coin to a coin collector

Answer: B

Explanation:
GDP is the total market value of all final goods and services produced domestically during a specific
period. Non market activities are not counted; the sale of used goods are also not counted. A
broker's fee is counted because it represents a service that was produced domestically.

Question: 1284

Under a ________ income tax system, as income grows, the average personal income tax liability of
individuals and families increases.

A. corporate
B. regressive
C. discretionary
D. automatic
E. progressive

Answer: E

Explanation:
With rising incomes, more people find themselves above the "no tax due" cutoff. Others are pushed
into a higher tax bracket. Therefore, during an economic expansion, revenue from the personal
income tax increases more rapidly than income.

Question: 1285

The inflation rate in an economy currently in equilibrium jumps up unexpectedly. According to the
Adaptive Expectations hypothesis, unemployment will in the short run.

A. all of these answers can happen.


B. decrease
C. increase
D. remain unaffected.

Answer: B

Explanation:
According to the Adaptive Expectations hypothesis, people consider the recent past as the best
predictor of the immediate future. Therefore, when the inflation rate jumps up unexpectedly,

Page | 474
workers using adaptive expectations will lag behind in their estimates and underestimate the future
inflation. Hence, they will get into wage contracts that will under-compensate for the higher
inflation. Over a short run, this will lead to improved profit margins for businesses, who will expand
the output by utilizing more resources. In the short run, unemployment rate will fall.
Over the long run, this mistake gets corrected, wages increase and unemployment rate falls back to
the natural rate.

Question: 1286

The Keynesian range (the horizontal range) of a short-run aggregate supply curve illustrates that
changes in aggregate demand exert little impact on prices and a great deal of impact on output when

A. resources in the economy are fully employed.


B. aggregate demand exceeds aggregate supply.
C. natural unemployment is present.
D. substantial excess capacity is present.

Answer: D

Explanation:
Below the full employment capacity of the economy, increases in aggregate supply have little effect
on the price level. This is the result of the Keynesian assumption that at less than full employment
output levels, prices and wages are fixed since they are inflexible in a downward direction.

Question: 1287

Which of the following will most likely increase aggregate supply in the long run?

A. advances in technology
B. a low rate of capital formation
C. higher real interest rates due to the federal government's large budget deficits
D. unfavorable weather conditions in agricultural areas
E. an increase in the expected inflation rate

Answer: A

Explanation:
Improvements in technology can cause the aggregate supply curve to permanently shift to the right.
This is because such advances make it possible to produce and sustain a larger rate of output from a
specific resource supply.

Question: 1288

Cyclical unemployment is a result of

A. inaccurate and/or costly information about job opportunities.


B. an inadequate matching of qualified workers and available jobs.
C. not enough employees to fill available jobs.
D. insufficient employment in the building trades.
E. a downturn in economic activity.

Answer: E

Explanation:

Page | 475
Cyclical unemployment occurs due to recessionary business conditions and inadequate aggregate
demand for labor.

Question: 1289

On a certain date, the banking system had $2 billion in excess reserves. The legally required reserve
ratio was 12.5 percent. Potentially, the banking system as a whole could increase its loans a
maximum of

A. more than $25 billion.


B. $2 billion.
C. $12.5 billion.
D. $16 billion.

Answer: D

Explanation:
The deposit expansion multiplier in this economy is 8 (found by taking the inverse of
12.5%=1/(.125)). Since banks hold an excess $2 billion in reserves they could extend loans in this
amount and increase the money supply by 8 x $2 billion = $16 billion.

Question: 1290

Structural unemployment is a result of

A. not enough employees to fill available jobs.


B. inaccurate and/or costly information about job opportunities.
C. insufficient employment in the building trades.
D. an inadequate matching of qualified workers and available jobs.

Answer: D

Explanation:
Structural unemployment occurs because there are structural characteristics of the economy that
make it difficult for job seekers to find employment and employers to hire workers. Basic structural
characteristics of the economy make it difficult for employees and employers to "match up."
Unemployment due to this structure of the economy, means that unemployed workers do not
possess the skills necessary to fill available jobs.

Question: 1291

Which of the following is a false statement?

A. CPI is calculated using a market basket of consumer goods.


B. CPI is a broad measure of prices throughout the economy.
C. The GDP deflator is used to calculate real GDP from nominal GDP.
D. Both CPI and the GDP deflator suffer from substitution bias.
E. The CPI and the GDP deflator tend to move in the same direction.

Answer: B

Explanation:
CPI is designed to measure the changes in the price level of consumer goods only. Since GDP includes
productions in all sectors, not just the consumer sector, consumer price inflation is inadequate to

Page | 476
calculate real GDP from nominal GDP.

Question: 1292

Which of the following best describes the relationship between the velocity of money and the
demand for money?

A. When the demand for money increases, the velocity of money increases.
B. The demand for money is not related to the velocity of money.
C. The demand for money must be stable for the velocity of money to increase.
D. When the demand for money declines, the velocity of money increases.

Answer: D

Explanation:
When decision makers conduct a specific amount of business with a smaller amount of money, their
demand for money balances is reduced. Each dollar, though, is now being used more often.
Therefore, the velocity of money is increasing. Thus, for a given income level, when the demand for
money declines, the velocity of money increases.

Question: 1293

Which of the following lend support to supply-side economics?

I. High marginal tax rates discourage additional work effort and reduce productivity.
II. High tax rates increase the costs of supplier resources, leading to inflation.
III. High tax rates affect adversely the efficient channeling of capital.
IV. High marginal tax rates skew the demand preferences of consumers geared toward non-
productive activities.

A. I, III & IV
B. I & III
C. I & IV
D. I, II, III & IV

Answer: C

Explanation:
Marginal tax rates are central to the effects postulated by Supply-side economists. According to this
theory, an increase in marginal tax rates decreases the incentive to work. People to shift away from
productive work and toward more leisure. Non-productive activities explicitly directed toward
avoiding taxes increase and projects are selected at least partly due to their tax benefits and not
necessarily due to their intrinsic merits. This leads to inefficient channeling of capital resources.

Question: 1294

Uncertainty may cause banks to hold larger excess reserves. Other things constant, this will

A. tend to reduce the money supply during a period of inflation and increase it during a recession.
B. tend to reduce the money supply.
C. tend to increase the money supply.
D. have no effect on the money supply.

Answer: B

Page | 477
Explanation:
The money supply is reduced when banks hold larger excess reserves through the work of the
deposit expansion multiplier. Since banks extend fewer loans when they hold larger excess reserves,
the money supply falls.

Question: 1295

Which of the following is a definition of inflation:

I. a pervasive increase in prices


II. too many dollars chasing too few goods
III. a general decline in the value of money
IV. a general increase in wages

A. II, IV
B. I, II, III, IV
C. I, II, III
D. II only
E. I, III
F. I, II

Answer: C

Explanation:
Inflation is a pervasive increase in prices which is caused by too many dollars chasing too few goods.
A price increase that has some other cause (such as an improvement in a product) is not inflationary.
Therefore these two ideas are inseparable from the definition of inflation. Since prices are increasing,
each unit of currency can buy fewer goods. Hence, a general decline in the value of money.

Question: 1296

A new payroll tax on businesses results in an increase in cost for each unit of production. Assuming
no change in income or in the production mix, would this cause inflation?

A. Yes, because producers would pass new costs onto consumers.


B. No, because firms would choose to use more capital and less labor.
C. Yes, because government spending would increase aggregate demand.
D. Yes, because higher costs result in less production and lead to higher prices.
E. No, because firms are "price takers" and will be unable to increase prices.

Answer: D

Explanation:
When firms face generally higher unit costs they decrease production. Assuming no change in
income, there would be too many dollars (i.e. income) chasing too few goods (i.e. production).
Although it may be tempting to say that costs are being "passed on" to consumers, this explanation is
not economically viable. If demand for the product would support a higher price, then firms would
have been charging a higher price prior to the oil shock and reaping higher profits. Even if we assume
firms are "price takers," the new tax will take the firm's zero economic profit and turn it into an
economic loss. The result will be fewer firms in the industry and less production overall.

Question: 1297

Page | 478
Which of the following determine(s) the demand curve for a good?

I. Disposable income.
II. Price of the good demanded.
III. Availability of substitutes.
IV. Consumer tastes and preferences.

A. I & III
B. I, III & IV
C. I, II & III
D. II only

Answer: B

Explanation:
You should be careful about distinguishing between a movement of the demand curve and a
movement along the demand curve. When the demand curve moves, the price for every quantity
that may be demanded changes. Such a move can come about when people's disposable income
changes. An increase in income moves the demand curve to the right, since for every given price, the
quantity demanded is higher at higher incomes. Similarly, an increase in the availability of
substitutes moves the demand curve to the left, since for every given price, the quantity demanded
is now lower. Further, as the consumers' tastes and preferences evolve over time, so will the demand
curve for a good. However, a simple change in the price of the good demanded will only lead to an
adjustment in the quantity demanded, without moving the demand curve. After all, that is the
definition of a demand curve!

Question: 1298

The actual rate of unemployment could fall below the natural rate of unemployment during which of
the following?

I. An economic boom.
II. An economic expansion, where the economy is trying to extricate itself from a recession.
III. A war.
IV. Never. The natural rate of unemployment is a lower limit on unemployment rate in a given
economy, depending on its structural features.

A. II & III
B. I & III
C. I, II & III
D. IV only

Answer: B

Explanation:
If the economy is in a recession, that means the unemployment rate is quite high, far above the
natural rate. Hence, II is incorrect. IV is also incorrect since the natural rate of unemployment
prevails in a healthy economy operating at maximum sustainable efficiency. However, in times of
high demand like economic booms or war, it is possible that the unemployment rate temporarily dips
below the natural rate, causing the output to rise above the sustainable GDP.

Question: 1299

Suppose the Fed sells $25 million worth of Treasury bonds in the open market. This action will serve

Page | 479
to ________ the consumption of interest-rate sensitive goods like home mortgages. The U.S. dollar
will ________ against the other currencies.

A. increase; appreciate
B. increase; depreciate
C. decrease; appreciate
D. decrease; depreciate

Answer: C

Explanation:
The sale of bonds implies that the Fed takes out money from the economy. This reduces the
monetary base and the total money supply in the economy, causing the real interest rates to rise in
the short run. This makes the opportunity cost of consuming interest rate sensitive goods like homes
and cars more expensive, leading to a decrease in the consumption of these goods. The high real
rates will attractforeign funds into the U.S., causing the demand for U.S. dollars to increase. The U.S.
dollar will thus appreciate in the short run.

Question: 1300

Which of the following statements is false?

A. real GDP excludes earnings by domestic citizens abroad


B. the change in real GDP plus the inflation rate equals growth in nominal GDP
C. changes in nominal GDP include the impact of changes in the money supply
D. real GDP measures strictly changes in national output
E. change in nominal GDP is always greater than the change in real GDP

Answer: E

Explanation:
The change in nominal GDP usually exceeds that of real GDP because inflation is usually positive.
However, in a flat or declining price environment real GDP would exceed nominal GDP change.

Question: 1301

Which of the following is not an example of an inflationary change in prices?

I. A car dealer decides to charge more for a new model year of cars because of new features
II. A university decides to increase tuition to pay for a new athletic field
III. An electronics retailer charges more for new DVD players than for outdated VCRs

A. I, II
B. I, III
C. I only
D. II, III
E. II only
F. None of these answers is correct

Answer: F

Explanation:
Prices for goods can increase for reasons other than inflation. Inflationary price increases are the
result of too many dollars chasing too few goods. These examples are price increases that are the

Page | 480
result of improvements in the good being purchased.

Question: 1302

During periods of rapid growth in output

A. there has been no consistent relationship between output growth and employment or
unemployment.
B. both employment and unemployment generally rise.
C. employment generally decreases and unemployment increases.
D. employment generally increases, while unemployment decreases.
E. both employment and unemployment generally fall.

Answer: D

Explanation:
During a business expansion, business sales tend to rise, GDP grows rapidly and the rate of
unemployment declines.

Question: 1303

Which of the following would be considered an automatic stabilizer?

I. Federal Reserve policy


II. government payrolls
III. public works projects

A. I, III
B. none of these answers is correct
C. II, III
D. I, II, III
E. I, II
F. III only

Answer: B

Explanation:
An automatic stabilizer is anything that would decrease the government budget surplus during slow
economies and increase the surplus during strong economic periods. Federal Reserve policy is not
automatic, its directed. Government payrolls do not inherently increase or decrease during economic
periods. Similarly, public works projects would not automatically increase due to an economic
slowdown.

Question: 1304

Which of the following would appear on the liability side of the balance sheet of a commercial bank?

A. loans outstanding
B. demand and other transaction deposits
C. vault cash
D. U.S. government securities

Answer: B

Page | 481
Explanation:
Demand and other transaction deposits constitute liabilities for commercial banks because they
"owe" depositors the amount of their deposits. Outstanding loans represent assets for commercial
banks because the borrower "owes" the bank the principal of the loan plus accrued interest.

Question: 1305

A passive budget deficit is a deficit that

A. merely reflects a decline in tax revenue during an economic boom.


B. results from legislative action that reduces tax rates.
C. results from legislative action that increases government expenditures.
D. merely reflects a decline in tax revenue during a recession.

Answer: D

Explanation:
A passive deficit is a good example of an automatic stabilizer. During a recession, government tax
revenues fall and expenditures rise. Thus, the preferred and natural state for the government budget
during a recession is to be in a deficit in order to stimulate aggregate demand.

Question: 1306

Which one of the following will most likely reduce aggregate supply?

A. a technological advance that reduces the cost of energy


B. an increase in the nation's net investment rate
C. an increase in the labor force participation rate
D. regulatory action that generates more benefits than costs
E. a substantial increase in the minimum wage

Answer: E

Explanation:
An increase in the minimum wage reflects and increase in the cost of a resource, labor. This implies
that the SRAS will shift inward because the costs of production are now higher. If the change is
permanent, then LRAS will also shift inward.

Question: 1307

If the effects of an expansionary macroeconomic policy are accurately anticipated, then

A. wages will rise more rapidly than product prices.


B. market participants will quickly adjust to the expected inflation.
C. profit margins will improve.
D. resource prices will remain constant.

Answer: B

Explanation:
Anticipated macroeconomic policies are ineffective in stimulating aggregate output and employment
because market participants create labor contracts which automatically account for rising prices.
Thus, employment is not increased because the real wage is constant.

Page | 482
Question: 1308

An economy is currently in a state of equilibrium, at full employment. If a sudden supply shock were
to decrease aggregate supply, which of the following effects will occur in the short run?

I. Real interest rates will increase.


II. Prices will rise.
III. Aggregate demand will remain unaffected.
IV. The SRAS will shift to the left.

A. II & III
B. I & III
C. I, II & III
D. I, II & IV

Answer: D

Explanation:
The decrease in the aggregate supply curve will be represented by a movement of the short-run
supply curve to the left. In the short run, this will cause an increase in prices since the demand curve
does not move. Aggregate demand will fall, unemployment will rise above the natural rate and
aggregate output will fall. The total disposable income in the economy will decrease and consumers
will liquidate part of their savings to maintain stable consumption. This will decrease the supply of
loanable funds, raising interest rates in the short run.

Question: 1309

According to supply-side theory, an increase in marginal tax rates will

A. all of these answers.


B. encourage individuals to substitute less-desired, tax-deductible goods for more-desired,
nondeductible goods.
C. decrease the supply of capital and reduce its productive efficiency.
D. decrease the supply of labor and reduce its productive efficiency.

Answer: A

Explanation:
Increasing marginal tax rates reduces the incentive to work since individuals keep a smaller
percentage of their earnings. Additionally, increased marginal tax rates divert resources away from
productive roles to roles designed to "shelter" capital owners from taxation. The final consequence is
that efficiency is reduced since individuals choose deductible (and less desirable) goods over
nondeductible goods. Efficiency is compromised because although the non-deductible goods are
affordable and preferable to deductible goods, they are not consumed.

Question: 1310

Which of the following statements about expected inflation are true?

A. Interest rate volatility will increase


B. Price increases will typically outpace increases in wages
C. All of these answers are correct
D. Lenders are not compensated for the loss of purchasing power when funds are repaid
E. None of these answers is correct

Page | 483
Answer: E

Explanation:
Market participants will compensate for expected inflation. For example, when lenders make loans,
they will price a certain inflation rate into the interest rate. If inflation is as expected, then the rate
will adequately compensate the lender for lost purchasing power. Inflation is generally assumed to
impact all prices, including wages, equally, therefore workers will demand wage increases in line
with inflation.

Question: 1311

Which of the following is an integral part of the Keynesian view of the business cycle?

A. Equilibrium is a state of full employment without inflation.


B. Multiplier effects magnify the impact of changes in aggregate demand (especially in investment)
and thereby promote economic instability.
C. Changes in private consumption are the major source of economic instability.
D. Supply creates its own demand.
E. Falling wage rates eventually lead the economy back to full employment.

Answer: B

Explanation:
The multiplier effect causes small changes in aggregate expenditures to result in much larger
changes in income, leading to economic instability. This effect applies to all components of the
aggregate expenditure model, and therefore none of the four components can be called "the major
source of economic stability."

Question: 1312

The period between the trough of the business cycle and the next peak is called the

A. cyclic phase.
B. expansionary phase.
C. contractionary phase.
D. recessionary phase.

Answer: B

Explanation:
After a downturn reaches bottom and economic conditions begin to improve, the economy enters
the expansion phase of the cycle. Here business sales rise, GDP grows rapidly and the rate of
unemployment declines. The expansion eventually blossoms into another business peak.

Question: 1313

An unanticipated decrease in short-run aggregate supply will lead to a

A. higher price level and a lower real interest rate.


B. lower price level and a lower real interest rate.
C. lower price level and a higher real interest rate.
D. higher price level and a higher real interest rate.

Page | 484
Answer: D

Explanation:
An unanticipated reduction in short run aggregate supply will cause the price level to rise as the
short run aggregate supply curve shifts left. Since people will believe that their lower incomes are
temporary, households will reduce their current saving level to maintain current consumption at a
level more consistent with their longer term perceived opportunities. This reduction in the supply of
loanable funds causes the interest rate to increase.

Question: 1314

If decision makers fail to anticipate the inflationary effects of demand stimulus policies, in the short
run the stimulus will

A. increase both inflation and the nominal interest rate but exert little impact on real output and
employment.
B. increase the real interest rate.
C. reduce the real interest rate.
D. lead primarily to larger output and high employment.
E. both lead primarily to larger output and high employment and reduce the real interest rate.

Answer: E

Explanation:
Unanticipated inflation, as the result of demand stimulus policies, increases output and employment
since the real wage is eroded and the cost of labor declines.

Question: 1315

The primary cause of frictional unemployment is

A. fluctuations in aggregate demand.


B. the lack of training and marketable qualifications in job seekers.
C. inaccurate and costly information about job opportunities.
D. high unemployment benefits that reduce the incentive of unemployed workers to seek work.

Answer: C

Explanation:
Frictional unemployment is the result of a scarcity of information and the search activities of both
employers and employees for information that will help them make better employment choices.

Question: 1316

What incentives does inflation create in consumers and producers?

A. Consumers will save less and producers will invest more.


B. Consumers will spend more and producers will produce more.
C. Consumers will save more and producers will produce more.
D. Consumers will spend more and producers will sell off inventories.
E. Consumers will spend less and producers will lay off workers.

Answer: B

Page | 485
Explanation:
Inflation causes consumers to spend now, because goods will be more expensive in the future.
Conversely, producers will produce goods now, because wages and raw materials prices are expected
to rise. Producers will also build up inventories in anticipation of higher future prices.

Question: 1317

"Rapid growth in the money supply is the primary cause of inflation. The time period between
acceleration in the growth rate of the money supply and an acceleration in inflation is often lengthy
(for example, from 12 to 36 months) and difficult to predict." These two statements are

A. basic tenets of the Keynesian doctrine.


B. normative economic statements.
C. basic tenets of the Monetarist doctrine.
D. internally inconsistent.

Answer: C

Explanation:
Monetarists believe that monetary instability is the major cause of fluctuations in real GDP and rapid
growth of the money supply is the major cause of inflation.
Monetarists cite lengthy and unpredictable time lags between the implementation of a monetary
policy change and the observation of its primary effects as a justification for not using discretionary
monetary policy as a stabilization tool.

Question: 1318

The Central Bank increases the money supply by 5%. This was not anticipated by economic
participants. The economy is operating below potential. In the long-run, this will cause the aggregate
supply curve to _______, the aggregate demand curve to ________, and the price level to ________.

A. shift left, shift right, increase


B. not change, shift right, increase
C. not change, shift left, increase
D. not shift, not shift, not change
E. shift right, shift left, increase
F. shift right, shift right, not change

Answer: A

Explanation:
In order to determine how each curve changes, think of the effects independently. First the demand
curve. The new money supply is effectively new income for consumers, it is irrelevant for this
exercise whether this is real or nominal income. An increase in income shifts the demand curve to
the right.
Next the supply curve. If producers anticipate inflation, this is effectively an increase in their costs.
Again, whether this is a nominal or real cost increase is irrelevant. Higher costs cause the supply
curve to shift to the left.
The directional shift in both curves indicates a higher price level. Note that the quantity
demanded/produced is the same. This is because while the increase in income shifts the demand
curve, the increase in prices offsets the effect. Similarly, the increase in revenues (due to higher
prices) offsets the higher costs for suppliers.

Question: 1319

Page | 486
According to the Keynesian model, if the marginal propensity to consume were 0.80, an independent
increase in investment expenditures of $20 billion would cause the equilibrium aggregate nominal
income to rise

A. $80 billion.
B. $16 billion.
C. $100 billion.
D. $20 billion.
E. $50 billion.

Answer: C

Explanation:
The expenditure multiplier is found by M = 1/(1-MPC). Thus, here M = 1/(1-.8) = 5. Therefore the $20
billion increase in aggregate expenditures is magnified five times to $100 billion.

Question: 1320

First National Bank currently does not have the legally required cash reserves on hand. The bank
expects this situation will only last a day or two. In order to rectify the reserve problem, the bank
treasurer borrows cash on the intra-bank loan market. The loan was actually granted by the Central
Bank. This is the only action by the Central Bank that day. How has the Central Bank's actions
impacted the demand curve for the purchase of intra-bank loans and the interest rate paid on those
loans?

A. Demand curve shifts upward, interest rate falls


B. Demand curve shifts upward, interest rate increases
C. Quantity demanded increases, interest rate falls
D. Demand curve does not change, interest rate increases
E. Quantity demanded decreases, interest rates rise

Answer: A

Explanation:
When a bank lacks reserves it has essentially loaned too much money, i.e. it has put too much money
into circulation. If the Central Bank lends money to the bank to cover the shortfall, then the cash
from the excess loans can stay in circulation.
If the reserve loan had come from another bank, then this would have simply transferred excess
reserves from one bank to another. Since the Central Bank stepped in, banks with excess reserves
will now choose to make more traditional loans and further expand the money supply.
Remember that the borrower has sold a loan and the lender has bought a loan. Therefore, the
Central Bank's actions have increased demand for intra-bank loans. This would cause the interest
rate paid to fall.

Question: 1321

According to the New Classical View, ________ is unaffected by deficits since people will save more
money in order to pay the higher future taxes.

A. the real interest rate


B. the marginal tax rate
C. price equalization
D. the supply-side

Page | 487
E. fiscal policy

Answer: A

Explanation:
New classical economies stress that debt financing merely affects the timing of taxes, not their
magnitude. Debt financing substitutes higher future taxes for lower current taxes. As a result,
households reduce their current consumption in response to additional government debt as surely as
would be the case if an equivalent amount of current taxes were levied.

Question: 1322

If the Central Bank wishes to diminish unemployment, it would attempt to ________ the money
supply by ________ short-term interest rates.

A. decrease, increasing
B. stabilize, decreasing
C. stabilize, increasing
D. increase, decreasing
E. increase, increasing
F. stabilize, stabilizing
G. decrease, decreasing

Answer: D

Explanation:
The central bank would increase the money supply in an attempt to decrease unemployment. This
can be accomplished by decreasing short-term rates.

Question: 1323

An individual who is employed part time but who is looking for a full-time job is classified as

A. frictionally unemployed.
B. employed.
C. structurally unemployed.
D. cyclically unemployed.

Answer: B

Explanation:
Since the individual is currently employed he or she is considered employed, technically. Such a
person is underemployed but not technically unemployed.

Question: 1324

When the output of an economy exceeds the economy's full-employment capacity, then

A. aggregate supply will increase until the economy can produce the output at the existing price
level.
B. aggregate demand will decline until full-employment equilibrium is restored at a lower price level.
C. unemployment will exceed the economy's natural unemployment.
D. wages and prices will rise until full employment is restored at a higher price level.

Page | 488
Answer: D

Explanation:
Higher than full employment output will cause resource prices to rise (i.e. the price of labor and
other prices in the economy) which will result in decreased demand for labor and a contraction of
aggregate supply. The economy will toward its long run equilibrium output.

Question: 1325

Under adaptive expectations, a shift to a more expansionary macroeconomic policy will

A. reduce unemployment in both the short and long run.


B. fail to systematically reduce unemployment in either the short run or long run.
C. reduce unemployment in the short run but not in the long run.
D. reduce unemployment in the long run but not in the short run.

Answer: C

Explanation:
The effects of a more expansionary macroeconomic policy under adaptive expectations are to reduce
unemployment only in the short run. This is because the inflationary effects of the demand-stimulus
policy will be underestimated and thus the real wage would be eroded. The consequence of this
erosion is an increase in employment. However, in the long run, unemployment is unchanged since
the higher rate of inflation is anticipated.

Question: 1326

Which of the following is true?

A. During a recession, cyclical unemployment will be low.


B. Frictional unemployment implies a lack of available jobs.
C. When an economy is at full employment, actual unemployment will be less than the natural
unemployment.
D. When actual GDP equals potential GDP, the actual unemployment will equal the economy's
natural unemployment.

Answer: D

Explanation:
Under full employment, the rate of unemployment will be equal to the natural rate of
unemployment given the economy's frictional and structural characteristics. Potential GDP is defined
as the amount of output that could be expected at full employment. Therefore, at potential GDP, the
rate of unemployment is equal to the natural rate of unemployment.

Question: 1327

The unemployment rate equals the number of persons

A. unemployed divided by the population age 16 and over.


B. unemployed divided by the number in the labor force.
C. not working divided by the population age 16 and over.
D. unemployed divided by the number employed.

Answer: B

Page | 489
Explanation:
The rate of unemployment is the percent of persons in the labor force who are unemployed. It is
calculated as: number of persons unemployed divided by the number of persons in the labor force.

Question: 1328

Within the simple Keynesian model, when an economy operates below its long-run, full-employment
output constraint, an increase in aggregate demand will lead to an increase in

A. real income.
B. unemployment.
C. prices.
D. employment, output and prices, but real income will remain constant.
E. interest rates and money income, but employment and real income will remain constant.

Answer: A

Explanation:
Keynes considered aggregate supply to be accommodative of aggregate demand. Thus, an increase
in aggregate demand will stimulate aggregate output. The equivalence between output and income
also suggests that real income will rise. Below the full employment capacity of the economy,
increases in aggregate supply have little effect on the price level. This is the result of the Keynesian
assumption that at less than full employment output levels, prices and wages are fixed since they are
inflexible in a downward direction.

Question: 1329

In the simple Keynesian model, if equilibrium output is less than the level required for full
employment, what must happen for full employment to be achieved?

A. Aggregate demand must fall.


B. Interest rates must rise.
C. Aggregate supply must increase.
D. Prices must rise.
E. Aggregate demand must increase.

Answer: E

Explanation:
When equilibrium output is less than the economy's capacity, only an increase in expenditures will
lead to full employment. This is because under the Keynesian model, aggregate expenditures are
considered the catalyst the change in output; that is, aggregate output is accommodative of
aggregate expenditures.

Question: 1330

In the Keynesian model, if the marginal propensity to consume were 0.75, an independent decline in
investment of $10 billion would cause equilibrium income to decline ________.

A. $40 billion
B. $75 billion
C. $10 billion
D. $50 billion

Page | 490
Answer: C

Explanation:
The expenditure multiplier is found by M = 1/(1-MPC). Thus, here M = 1/(1-3/4) = 4. Therefore the
$10 billion decrease in aggregate expenditures is magnified four times to $40 billion.

Question: 1331

In year 1, the nation of Economica has no government debt, production is at potential, the nominal
interest rate is 8.6% and the real rate is 5.2%. In year 2, the nominal rate is 11.1% and the real rate is
6.7%. Which of the following would be most likely to cause such a situation?

A. Federal budget deficit


B. Monetary expansion
C. Recession
D. Trade surplus
E. Increase in aggregate supply

Answer: A

Explanation:
In this case, the real rate has increased, as well as the rate of inflation. This is most likely to be
caused by a budget deficit. Deficit spending causes the real rate to rise due to government demand
for debt. Inflation would also increase because government spending amounts to an increase in
aggregate demand, which would shift the price level higher.

Question: 1332

If the money velocity is 5, the amount of money in circulation $200 million and real GDP $10 million,
then prices are ________.

A. 4
B. 400
C. 42
D. 100

Answer: D

Explanation:
According to the equation of exchange which is defined as MV = PY we must solve for P in the
following way: ($200 million x 5) = ($10 million x P). This implies that P=100.

Question: 1333

In the 1980s, the government cut back on the tax rates in an effort to spur the economy. This led to a
significant decrease in real tax revenues. A breakdown of this decrease indicated that most of the
decrease came from the lower tax brackets. Tax revenues from top tax bracket actually increased
despite the cut. There was a modest growth in the GDP during this period, though amongst all the
industrialized nations, only Japan matched this rate. This empirical evidence indicates that:

A. The supply side effects are not as potent as hypothesized by economists.


B. The supply-side response of tax payers can have a strong effect on a nation's growth.
C. The Rational Expectations model is a fair description of how people respond to fiscal policy.

Page | 491
D. The tax avoidance behavior governing the demand side has a significant impact on the economy.

Answer: B

Explanation:
The important part in this empirical observation is the effect of a change in marginal tax revenues.
Marginal tax rates are central to the effects postulated by Supply-side economists. According to this
theory, a reduction in marginal tax rates increases the incentive to work and save more by increasing
the disposable income. This causes people to shift away from leisure and toward more productive
work, enlarging the effective resource base and improving the efficiency with which it is utilized. The
empirical evidence in the study indicate that supply-side effects can be quite important in directing
the growth of an economy.

Question: 1334

How does inflation impact interest rates, all else equal?

A. increases the real rate; increases the nominal rate


B. has no direct impact on interest rates
C. has no effect on the real rate; increases the nominal rate
D. increases the real rate; has no effect on the nominal rate
E. impossible to determine

Answer: C

Explanation:
The real rate of interest is simply the nominal rate less inflation. Therefore, the real rate is not
directly effected by inflation, while the nominal rate would rise as inflation rises.

Question: 1335

If the economy is producing less than the full-employment output level, which of the following
would most likely direct the economy back to long-run equilibrium?

A. a decrease in resource prices


B. an increase in the real rate of interest
C. an increase in resource prices
D. an increase in technology

Answer: A

Explanation:
To understand this question, note that the price of resources is simply a function of supply and
demand. This question is implying that demand for resources (i.e. labor) is weak. Holding supply
constant, if demand for a good falls, prices must fall as well. Eventually some equilibrium will be
reached.

Question: 1336

The goal of an ideal inflation policy would be

A. focusing on unemployment first


B. minimizing average inflation
C. keeping inflation stable

Page | 492
D. growing the money supply at a constant rate
E. driving the price level higher

Answer: C

Explanation:
Anticipated inflation, even at relatively high levels, is generally not problematic. Therefore an ideal
inflation policy would attempt to keep inflation as predictable and as stable as possible. Of course,
theideal inflation rate would be zero, but minimizing average inflation, with no attention paid to the
variability, would not be helpful. In the U.S., for example, the Federal Reserve and the Treasury
department are careful to broadcast the money supply, so that market participants can make as
accurate an estimate of inflation as possible.

Question: 1337

Recent data reveal that the ratio of inventories to sales has risen sharply, suggesting that businesses
are experiencing an unplanned build-up in inventories. According to the Keynesian model, these
findings indicate that

A. the economy is operating at full-employment equilibrium, which will probably be sustained in the
foreseeable future.
B. aggregate expenditures exceed output and, as a result, inflation is likely to accelerate.
C. aggregate expenditures exceed output and, as a result, national income will rise.
D. aggregate expenditures are less than output and, as a result, national income will decline.

Answer: D

Explanation:
Actual inventories exceed planned inventories only when actual expenditures fall short of expected
expenditures. Thus, producers anticipated a certain level of expenditure and produced output
accordingly. Thus, output exceeded expenditure and national output (i.e., income) will decline.

Question: 1338

The multiplier effect refers to the fact that an autonomous change in spending (aggregate demand)
will

A. cause prices to rise by some multiple of the initial increase in spending.


B. reduce prices by some multiple of the increase in spending.
C. increase the money supply.
D. increase unemployment.
E. cause nominal output to rise by some multiple of the initial increase in spending.

Answer: E

Explanation:
The multiplier explains why small changes in investment, government or consumption spending
triggers much larger changes in output.

Question: 1339

"Fractional Reserve Banking" refers to:

A. The Federal Reserve System.

Page | 493
B. All of these answers.
C. The banking system where part of the reserves are derived in the form of loans from the Central
Bank.
D. The banking system where banks hold less than 100% reserves against deposits.

Answer: D

Explanation:
In a fractional reserve system, a bank can loan out almost all of the deposits it receives, maintaining
only a small part as reserves to meet regular withdrawals and as a provision for loan defaults.

Question: 1340

The phrase, "GDP equals what you eat plus what you invest plus what you export" characterizes
which of the following methods?

A. Expenditure approach.
B. Income-cost approach.
C. Consumption-investment approach.
D. Resource cost-income approach.

Answer: A

Explanation:
The Expenditure approach has four components:

1. Personal consumption expenditure.


2. Gross private domestic investment (investments by people residing outside the country are
ignored, even if they happen to be citizens of the country).
3. Government consumption & investment.
4. Net exports.

These can be summarized as "GDP equals what you eat plus what you invest plus what you export."

Question: 1341

The current account trade balance + the capital account trade balance =

A. the currency balance at the World Bank


B. GDP
C. the foreign exchange rate
D. GNP
E. the trade deficit/surplus
F. zero

Answer: F

Explanation:
By accounting identity, the capital account and current account must balance to zero. If there is a
deficit in one, there must be a surplus in the other. This is one reason why if a nation has a
government budget deficit, which amounts to negative domestic investment, the shortfall could be
made up by foreigners, causing a capital account surplus. Conversely there must be a current account
deficit in such a situation. This is one reason why budget deficits and trade deficits are linked. Note
that the term "trade balance" usually refers to the current account.

Page | 494
Question: 1342

Mathematically, the marginal propensity to consume is

A. income divided by consumption.


B. additional consumption divided by additional income.
C. consumption divided by income.
D. additional income divided by additional consumption.

Answer: B

Explanation:
The marginal propensity to consume is found according to the following equation:
MPC = total change in consumption / total change in income

Question: 1343

The rate of unemployment is equal to:

A. the number of unemployed people as a fraction of the total labor force.


B. the number of unemployed people as a fraction of the total population.
C. none of these answers.
D. the number of unemployed people as a fraction of the total population eligible to work.

Answer: D

Explanation:
The labor force of an economy consists of adults (defined as being of age 16 and above) who are
either employed or actively seeking employment. The unemployment rate is then the fraction of the
labor force that is not employed.

Question: 1344

Mrs. Davis was in the habit of keeping $500 in currency in her jewelry chest in case she ever needed
cash and the bank was closed. However, she recently got a combination debit/ATM card that gives
her access to her checking account 24 hours a day. She deposits her $500 in cash into her checking
account.
How does this immediately impact the monetary base and the effective amount money available for
transactions?

A. decrease, increase
B. increase, decrease
C. no change, decrease
D. both increase
E. no change for either
F. decrease, no change
G. both decrease

Answer: F

Explanation:
Effectively, there is no difference in how Mrs. Davis is using money, simply a different vehicle for
spending it. Therefore the effective level of money in circulation has not changed. However, where

Page | 495
the monetary base previously had including her $500 in currency, it would now only include the
percentage of this money held in reserve by her bank. The monetary base equals currency plus bank
reserves. Therefore the monetary base would decrease.
Innovations like debit cards have decreased the need to hold currency. Therefore certain measures of
the money supply may decrease despite no real change in the money level in circulation.

Question: 1345

The primary cause of frictional unemployment is

A. the presence of legislated high minimum wages that price unskilled workers out of the market.
B. discouraged workers who quit looking for a job after extended periods of unsuccessful job search.
C. inaccurate and costly information about job opportunities.
D. high unemployment benefits that reduce the incentive of unemployed workers to search for
employment.

Answer: C

Explanation:
Frictional unemployment results from a scarcity of information and the search activities of both
employers and employees for information that will help them make better employment choices.

Question: 1346

The change in aggregate income/output for a given change in aggregate expenditure is known as the:

A. demand multiplier.
B. expenditure multiplier.
C. marginal propensity to consume.
D. income elasticity.

Answer: B

Explanation:
The expenditure multiplier is directly linked to the "marginal propensity to consume" (MPC), which
reflects the amount of each additional dollar in income that is spent on current consumption. In
particular, the ideal expenditure multiplier equals 1/(1-MPC).

Question: 1347

Assuming that an expansionary fiscal policy package could be enacted and implemented
immediately, approximately how far in advance would be problem of a lagging economy need to be
recognized?

A. 1-2 years
B. 6 months to 1 year
C. impossible to determine
D. if policy could be enacted immediately, timing would not be a problem
E. 1 year

Answer: C

Explanation:
One of the primary problems with fiscal policy is one of timing. Any stimulus package would take

Page | 496
time to work its way through the economy. For example, if unemployment were high and the
government passed fiscal policy legislation involving many new hires, it would take time for the new
employees to be hired, then they would go through training, and probably not begin productive work
for many months. Furthermore, the new income the employees now receive would be spent on
goods, which would become income for other individuals, etc. This creates a multiplier effect over
time. How long this will take to be fully realized is impossible to determine.

Question: 1348

When the number of discouraged workers increases during a severe recession,

A. the measured unemployment tends to overstate the severity of the unemployment problem.
B. the measured unemployment tends to understate the severity of the unemployment problem.
C. frictional unemployment will rise.
D. the natural unemployment will decline.

Answer: B

Explanation:
Persons who have given up searching for employment because they believe additional job search
would be fruitless are termed discouraged workers. Because they are not current searching for work,
they are not counted among the unemployed. As a result, the number of unemployed workers will
be understated since unemployed individuals will drop out of the labor force and thus not be
counted as unemployed.

Question: 1349

The chairman of the House Ways and Means committee, who has major sway on spending programs,
has been researching the economy and believes that the nation is heading for an inflationary spiral.
Inflation is currently not considered a problem. He proposes a fiscal policy solution. Which of the
following would not impede his proposition from alleviating the problem?

I.congressional opposition to spending cuts


II.difficulty determining the degree of spending cuts required
III.correctly choosing between tax hikes and spending cuts
IV.impact lag
V.recognition lag

A. III, IV
B. I, II, IV, V
C. II, IV, V
D. II, IV
E. I, II, III, IV, V
F. I, II, V

Answer: B

Explanation:
I, II, IV, and V are classic problems with fiscal policy. III is incorrect because in terms of fiscal policy tax
or spending measures are interchangeable.

Question: 1350

An economy is currently in equilibrium at full employment. If there is an unanticipated decrease in

Page | 497
demand, the employment rate in the short run:

A. will increase.
B. will remain unaffected but wages will decrease.
C. will rise above the natural rate.
D. none of these answers.

Answer: D

Explanation:
If there is an unanticipated decrease in demand, the demand curve will move to the left. The supply
curve will not move in the short run since the demand cut-back is unanticipated. This will lead to a
decrease in the employment rate and a fall in real GDP. Prices will also fall as a result of decreased
demand.

Question: 1351

Seeing that the unemployment rate is reaching new highs each month, several Senators group
together to propose major tax cuts and spending increases to stimulate the economy. Both the
increased spending and tax cuts will be implemented in the following January, more than six months
away. Due to major public outcry, these bills pass through legislature very quickly. During the six
months in between passage and implementation, the unemployment rate begins to fall. Another
year passes, now 18 months after the bill was first passed, and the economy is suffering from
significant inflation. What problem(s) with fiscal policy does this illustrate?

I. recognition lag
II. impact lag
III. crowding out effect
IV. rational expectations

A. I, II, III, IV
B. I, II, III
C. I only
D. II only
E. I, II
F. III, IV

Answer: E

Explanation:
In this case, tax cuts and spending increases took approximately six months to enact. However,
illustrating the recognition lag, by the time six months had passed, the economy was recovering. One
year later, the economy is suffering from inflation. This is due to the fact that fiscal policy takes time
to have its full impact, or impact lag. By the time the full impact was realized, the economy had
recovered, and the fiscal stimulus caused inflation.

Question: 1352

Since World War II the labor force participation rate (LPR)

A. of women has steadily decreased while the LPR for men has increased.
B. of women has steadily increased while the LPR for men has declined.
C. of both women and men has increased.
D. of both women and men has decreased.

Page | 498
Answer: B

Explanation:
Since 1948 the labor force participation rate for women has steadily increased while the rate for men
has been declining. This suggests that the composition of work force participation within the family
has changed substantially during the last five decades.

Question: 1353

To the extent that people eventually adjust to demand stimulus policies, persistent expansionary
macropolicy will lead to

A. lower money interest rates.


B. a reduction in the long-run unemployment.
C. inflation with no permanent reduction in unemployment.
D. more rapid economic growth but no change in prices.

Answer: C

Explanation:
Correctly anticipated inflation will result in persistent, though consistent, inflation with no effect on
unemployment since there will be no effect on the real wage.

Question: 1354

Which of the following is true?

A. GDP = GNP + (income of citizens received for factors of production supplied abroad) - (income paid
to foreigners for their contribution to domestic output)
B. The GDP is always larger than GNP since it includes the output produced by foreign nationals living
in the country.
C. The GNP is always larger than GDP since it includes the income by the country's citizens living
abroad.
D. GNP = GDP + (income of citizens received for factors of production supplied abroad) - (income paid
to foreigners for their contribution to domestic output).

Answer: D

Explanation:
GNP measures the productive value generated by the labor and capital owned by the citizens of a
country, regardless of whether those citizens and assets reside within the country or out of it. Hence,
while calculating GNP, the income generated by foreign nationals residing in the country as well as
by assets owned by foreign nationals is excluded. On the other hand, GDP measures the productive
activity within the country, regardless of who produces it.

Question: 1355

When the economy is in short-run equilibrium below potential output, then

A. actual unemployment will be greater than the natural unemployment.


B. the natural unemployment will decline.
C. the economy will be unable to sustain output in the long run.
D. actual unemployment will be less than the natural unemployment.

Page | 499
E. a strong demand for resources will place upward pressure on resource prices.

Answer: A

Explanation:
An equilibrium below potential output implies that demand for labor and other resources is weak.
This implies that the actual rate of unemployment will be greater than the natural rate since the
natural rate prevails when the economy is operating at full employment.

Question: 1356

Full employment means that which of the following is zero?

A. frictional unemployment
B. aggregate unemployment
C. cyclical unemployment
D. structural unemployment
E. total unemployment

Answer: C

Explanation:
Full employment is the level of employment that results from the efficient use of the labor force
after allowance is made for the normal rate of unemployment due to information cost, dynamic
changes and the structural conditions of the economy. Cyclical unemployment is short run and
therefore must be zero in order to attain full employment.

Question: 1357

Keynesian analysis implies that

A. flexible wages and prices will quickly direct a market economy to full employment.
B. the federal budget should be balanced annually.
C. fluctuations in aggregate demand are an important source of economic instability.
D. fluctuations in the money supply are the major source of economic instability.

Answer: C

Explanation:
According to Keynes, the major source of business instability is fluctuations in aggregate demand.
Thus, policies that effectively stabilize aggregate demand substantially reduce economic instability.

Question: 1358

An economy is currently in equilibrium at full employment. If there is an anticipated governmental


demand-stimulus policy and people correctly anticipate the effects, which of the following effects
can be seen in the short run?

I. The demand curve moves to the right.


II. Real GDP increases.
III. Prices increase.
IV. The supply curve shifts to the left.

A. I, II, III & IV

Page | 500
B. I, III & IV
C. I & III
D. III & IV

Answer: B

Explanation:
If the buyers and sellers in the resource market completely anticipate the effects of an increase in
demand, then they will correctly forecast the higher future inflation. This will prompt buyers to try
and buy the goods today, at lower prices. On the other hand, suppliers would prefer to sell in the
future, at higher prices. Thence, the demand curve will quickly move to the right and the supply
curve will move tothe left. An equilibrium will be reached at a higher price which leaves the real
quantities unaffected, changing only the nominal variables.

Question: 1359

Which of the following factors would reduce aggregate demand in the goods and services market?

A. a decrease in the real interest rate


B. an increase in the expected inflation rate
C. increased pessimism concerning the expected strength of future business conditions
D. an increase in stock prices

Answer: C

Explanation:
What people think will happen in the future influences current purchasing decisions. Pessimism
about the future state of the economy causes consumers and investors to cut back on their current
spending for fear of becoming over extended. This pessimism leads to a decline in aggregate
demand.

Question: 1360

According to Keynesians, which of the following is/are true?

I. Wages and prices are flexible and automatically direct an economy toward full employment.
II. Changes in output rather than changes in prices direct an economy toward equilibrium. III. An
economy can be in equilibrium even if there isn't full employment prevailing.

A. I, II & III
B. III only
C. I & II
D. II & III

Answer: D

Explanation:
Keynesian economics maintains that businesses produce only the quantities consistent with
anticipated demand. Thus, expected aggregate expenditures are crucial to determining the state of
the economy. If the planned expenditures exceed the anticipated demand, then the economy will
expand and vice versa. Equilibrium will be attained where output equals the spending level. Hence,
in Keynesian economics, equilibrium can occur at any level of output and employment rate, even if
that level is less than the potential GDP. Further, only changes in demand will fuel changes in output;
changes in prices will not be capable of moving an economy toward equilibrium.

Page | 501
Question: 1361

If the Fed starts following a contractionary monetary policy:

I. the U.S. trade deficit with other countries rises.


II. the unemployment rate increases.
III. real interest rates increase.

A. I, II & III
B. I & III
C. II only
D. II & III

Answer: B

Explanation:
When the Fed institutes a contractionary monetary policy, the real interest rates rise because money
supply falls. This makes the opportunity cost of current consumption higher, reducing consumer
demand and spending. This lower demand leads to lower production and higher unemployment.
Further, the U.S. dollar tends to strengthen because foreign investments in U.S. increase due to
higher real interest rates. The exports fall due to this. The effect on imports is more ambiguous since
reduced consumer spending decreases imports but a stronger dollar increases them. The net effect is
usually seen to be an increase in trade deficit due to a larger fall in exports.

Question: 1362

In the aggregate demand/aggregate supply model, an increase in a country's sustainable potential


output is represented by an increase in

A. actual unemployment.
B. prices.
C. aggregate demand.
D. long-run aggregate supply.

Answer: D

Explanation:
Changes in long run aggregate supply affect the economy's long run potential output. Changes in
short run aggregate supply do not affect the long run potential of the economy.

Question: 1363

An increase in the discount rate makes it more expensive for banking institutions to fall below the
________ ratio.

A. none of these answers


B. discount
C. reserve
D. federal funds
E. required reserve

Answer: E

Page | 502
Explanation:
The required reserve ratio is the ratio of reserves to deposits that the Fed requires all banks to meet.
If a bank fails to meet its required reserve ratio is must borrow funds from the Fed or from the
federal funds market to meet the ratio. The bank can borrow funds from the Fed at the discount rate;
thus if this rate increases, it becomes more expensive for banks to fail to meet the required reserve
ratio.

Question: 1364

An improvement in technology would shift which of the following curves?

A. aggregate demand and long-run aggregate supply


B. short-run and long-run aggregate supply
C. only short-run aggregate supply
D. aggregate demand and short-run aggregate supply
E. only aggregate demand

Answer: B

Explanation:
Improvements in technology permit the economy to squeeze a larger output from a specific resource
supply. Thus, such improvements enhance productivity and thereby shift the LRAS curve to the
right.Before such technology shifts the long run aggregate supply however, the short run aggregate
supply is affected. Thus, both the SRAS and LRAS curves shift outward under a technology
improvement.

Question: 1365

Which of the following determine(s) the supply curve?

I. Supply curves of factor resources.


II. Total money supply flowing in the economy.
III. Cost of running operating capacity at a given level. IV. Prices of related goods.

A. I & III
B. III & IV
C. I, II, III & IV
D. I, II & III

Answer: D

Explanation:
Note that the total money supply in the economy determines the values plotted on the price-axis of
a supply curve.

Question: 1366

Under a monetary expansion, aggregate demand can be expected to ________.

A. stay the same


B. rise
C. fall, then rise
D. fall
E. rise, then fall

Page | 503
Answer: B

Explanation:
Under a monetary expansion, individuals posses more money balances. Since they invest/save more
and consume more, aggregate demand increases due to increased consumption.

Question: 1367

If you were asked to study the economy over the past five years, you would use the real GDP series
rather than the nominal GDP series because

A. the nominal GDP series reflects changes in both output and prices, whereas the real GDP series,
roughly speaking, merely reflects changes in output.
B. the nominal GDP series fails to account for transfer payments, whereas the real GDP series
includes these payments.
C. the real GDP series accounts for imports, making it more precise than the nominal GDP series.
D. exports are excluded from the real GDP series, making it less complicated than the nominal GDP
series.

Answer: C

Explanation:
It is almost always true that economists are interested in real changes rather than nominal changes
because nominal changes look at both changes in the amount of goods and services produced and
prices. Real changes focus only on changes in the amount of goods and services produced.

Question: 1368

If the Fed wanted a more expansionary monetary policy, which of the following would be most
appropriate?

A. reduce taxes
B. increase government expenditures
C. buy government bonds from the public
D. raise the discount rate

Answer: C

Explanation:
To expand the money supply the Fed may buy government bonds from the public. This action injects
money into the economy because individuals who formerly held U.S. security notes now hold money
balances.

Question: 1369

Country A's workforce consists in large part of seasoned workers, with an average age of around 45.
Country B is relatively young, with an average workforce age of just 31. A is also a mature country,
having implemented many modern advances in technology. B is a growing country with a dynamic
leadership which is bringing about a technological revolution in the country. Given this, country A
has ________ structural unemployment and ________ frictional unemployment than country B.

A. lower, lower
B. higher, lower

Page | 504
C. lower, higher
D. higher, higher

Answer: D

Explanation:
Frictional employment arises because of the fact that employers do not known about all the
available workers and their qualifications while job-seekers are not fully aware of the available job
opportunities matching their interests and skills. This leads to a longer job placement time and also
causes a higher employee turnover due to job mismatches. The resulting contribution to the
unemployment rate is called "frictional unemployment." Since younger workers tend to have a
higher job mobility, a younger workforce will also have a higher frictional unemployment.
Changes due to technology, public policy or demands in the market place lead to people with
outdated skills or skills which do not match the jobs being offered. The resultant contribution to
unemployment is referred to as "structural unemployment." Hence, a rapidly growing economy will
tend to generate a higher level of structural unemployment since in such an economy, skills tend to
get outdated faster.

Question: 1370

Which of the following is not true about monetary policy?

A. The money supply is neutral in the long-run.


B. Monetary policy can only serve to decrease economic volatility.
C. In the long-run, the price level is a function of production and money supply.
D. Without interference, the money supply would remain constant.
E. The money supply will have no impact on real income if changes in the price level are anticipated.

Answer: D

Explanation:
Usually we think of the Central Bank being in control of the money supply, but several factors outside
of the baker's control can impact the money supply. Most notably is the currency deposit ratio, which
measures how much currency consumers actually hold. If consumers hold more physical currency,
this decreases banks' ability to create more money by lending funds.

Question: 1371

Rich owns a shoe factory. He believes that a recent monetary policy announcement will cause 5%
inflation over the next year, equally effecting the price of shoes, the cost of inputs, and wages. How
will Rich change his production plans considering this forecast?

A. He will try to hide this information from his laborers.


B. Cannot determine, this depends on how much prices actually inflate over the next year.
C. He will increase shoe production because the expansionary monetary policy should boost
consumer confidence and hence shoe demand.
D. No changes.
E. He will increase shoe production to take advantage of higher prices.

Answer: D

Explanation:
Anticipated inflation is always a wash for both consumers and producers, therefore Rich would not
change his production plans. Remember that possible secondary effects, like an increase in

Page | 505
consumer confidence, cannot be assumed. How high actual inflation is during the year is irrelevant
since planning is based on forecasted inflation. This is an important concept for understanding how
policy effects decision making. Note that if the entire economy thought exactly as Rich did, the
expansionary monetary policy will have no real impact on the economy.

Question: 1372

A shoe manufacturer believes inflation will increase dramatically over the next year. What incentives
does this create for this firm?

A. The firm will manufacture less, knowing that prices will be higher in the future.
B. The firm will respond to demand from retailers, regardless of inflation.
C. If inflation is widely expected, it will have no impact on the firm's strategy.
D. The firm will want to decrease inventories now because future inventories will be more costly and
therefore less profitable.
E. The firm will attempt to store shoes in inventory, knowing that in the future these shoes will be
worth more.

Answer: E

Explanation:
Inflation will cause the manufacturer's costs to increase in terms of raw materials and wages. In
addition, the price of shoes should also increase. Therefore the firm has an incentive to produce now,
and store these goods in inventory until prices rise.

Question: 1373

Which of the following will most likely occur in the short-run if there is an unanticipated increase in
aggregate demand?

A. an increase in output and a higher price level


B. a decrease in prices, while output remains unchanged
C. a decrease in output and a higher price level
D. a decrease in output and a lower price level
E. an increase in output, while prices remain unchanged

Answer: A

Explanation:
In response to an unanticipated increase in aggregate demand prices will rise and output will
temporarily exceed full employment capacity. In the long run, output will return to its long run
potential.

Question: 1374

An economy is currently in equilibrium at full employment. If there is an anticipated decrease in


demand, which of the following effects can be seen in the short run?

I. Real GDP decreases.


II. The supply curve shifts to the right.
III. The demand curve moves to the left.
IV. Prices decrease.

A. IV only

Page | 506
B. I, II, III & IV
C. III & IV
D. II, III & IV

Answer: D

Explanation:
If the buyers and sellers in the resource market completely anticipate the effects of a decrease in
demand, then they will correctly forecast the lower future inflation. This will prompt buyers to
postpone current consumption and wait till prices drop. On the other hand, suppliers would prefer to
sell today,while prices are high, instead of waiting till the prices fall. The demand curve will therefore
quickly move to the left and the supply curve will move to the right. An equilibrium will be reached
at a lower price which leaves the real quantities unaffected, changing only the nominal variables.

Question: 1375

Employment is the number of persons employed divided by the number of persons

A. In the civilian population age 16 and over.


B. Employed plus the number unemployed.
C. Unemployed.
D. In the labor force.

Answer: A

Explanation:
The employment/population ratio is the number of persons aged 16 years and over employed as
civilians divided by the total civilian population 16 years of age and over. The ratio is expressed as a
percentage. This rate does not make a subjective judgment as to whether a person is actually
"available for work" or "actively seeking employment."

Question: 1376

Which economic model supports the theory that budget deficits cause inflation?

A. The supply-side model


B. The crowding-out model
C. None of these models support the theory
D. The new classical model
E. The Keynesian model

Answer: B

Explanation:
The crowding out model indicates that, when the supply of money is constant, budget deficits will
simply lead to higher real interest rates and a fall in net exports, which will crowd out private
spending and thereby dampen the stimulus effect of the deficit.

Question: 1377

An unfortunate consequence of ________ and monetary instability is that people spend less time
producing and more time trying to protect their wealth.

A. unemployment

Page | 507
B. all of these answers
C. monetary contraction
D. inflation
E. none of these answers

Answer: D

Explanation:
Since failure to anticipate the rate of inflation has substantial effects on one's wealth, individuals
divert scarce resources away from the production of goods and services and apply them into the
acquisition of business decision makers to forecast the price changes. Thus, speculative practices are
encouraged while productive activities are discouraged.

Question: 1378

An economy is currently operating at full employment, with an inflation rate of 6%. The Central Bank
adopts an inflationary measure consistent with an inflation rate of 8% but people anticipate an
inflation of 7%. Then, the unemployment in the short run will be ________ the natural rate, as
predicted by the Rational Expectations Model.

A. could be above or below.


B. same as
C. below
D. above

Answer: C

Explanation:

The short-run equilibrium is affected by the accuracy of the predictions made by decision-makers.
Since workers under-estimate the future inflation resulting from the changed policy, they will settle
for lower wages than those consistent with the actual inflation. Consequently, the Rational
Expectations Phillips curve predicts that unemployment will decrease in the short run. With the
economy currently at full employment, the unemployment rate will fall below the natural rate,
temporarily expanding the real GDP beyond the potential level. Over the long run, people will
correct their erroneous predictions and wages will rise to a level where full employment will prevail
once again.

Question: 1379

Which of the following would increase GDP?

A. Mercedes-Benz begins to produce and sell cars in Alabama.


B. An American investor buys 100 shares of Ford stock.
C. Ford Motor Company begins to produce and sell cars in Japan.
D. An American investor purchases 100 shares of Mercedes-Benz stock.

Answer: A

Explanation:
Since GDP represents the total market value of all final goods and services produced domestically
during a specific period, GDP would rise if a foreign company starts to produce cars domestically.

Question: 1380

Page | 508
If the marginal propensity to consume (MPC) is .75 or 3/4, what is the expenditure multiplier?

A. 0.25
B. 4.0
C. 3.0
D. 2.0

Answer: B

Explanation:

The expenditure multiplier is found according to the equation M= 1/(1-MPC). Thus, M = 1/(1-3/4) =
4.

Question: 1381

Which of the following would be an example of non-activist monetary policy?

A. The Central Bank attempts to counter-act negative developments in the economy.


B. Inflation is the only target for the Central Bank.
C. The Central Bank tries to keep the money supply constant.
D. The government keeps spending constant and allows tax revenues to rise or fall to compensate for
changes in aggregate income.
E. The Central Bank increases the money supply by 5% every year.

Answer: E

Explanation:

The classic non-activist monetary policy example is to increase the money supply by a particular level
every year. Activist or discretionary monetary policy involves changing the supply of money to
counteract negative developments in the economy.

Question: 1382

"Counter-cyclical macroeconomic policy will be ineffective as a stabilization tool because people will
undermine the policy by adjusting their choices once they expect a systematic policy response to
recessions and booms." This statement most clearly reflects the

A. Keynesian view.
B. rational expectations view.
C. supply-side view.
D. 1960 view of the Phillips curve.

Answer: B

Explanation:

Rational agents will weigh all of the likely economic policies in their estimation of future inflation
rates. Therefore, once a systematic policy response to recessions and booms is established, counter-
cyclical macroeconomic policy will be ineffective since it will be fully anticipated.

Question: 1383

Page | 509
An economy which is experiencing substantial inflation and slow economic growth is said to be in:

A. a contraction.
B. a stagflation.
C. a hyperinflation.
D. a recession.

Answer: B

Explanation:

High and variable inflation rates have severe negative impact on an economy. At times, the economy
spirals into a cycle of extremely slow growth and very high inflation. This stagnant state battered by
rising prices is called "stagflation."

Question: 1384

Congress proposes to stimulate the economy by cutting taxes for middle income families but raising
taxes for wealthier tax payers. The effect will be the same net taxes collected, but most tax payers
would pay less in taxes. According to which of the following economic theories would this stimulate
the economy?
I. fiscal policy
II. supply-side
III. monetary policy

A. I, III
B. none of these answers is correct
C. III only
D. I, II, III
E. I only
F. II only

Answer: B

Explanation:

Since there was no net reduction in taxes, the tax plan would not create a fiscal stimulus. Marginal
tax rates have effectively increased under this plan, implying a negative supply-side impact.
Monetary policy involves changing the money supply, which is not applicable here.

Question: 1385

An increase in the nominal interest rate would

A. encourage people to hold larger money balances.


B. cause households to increase consumption.
C. encourage people to hold smaller money balances.
D. force the Fed to reduce the money supply.

Answer: C

Explanation:

Page | 510
The nominal interest rate represents the opportunity cost of holding money as cash-money held as
cash earns no interest. Money held in some checking accounting or other interest bearing accounts
earns a positive interest rate. Thus, money held as cash could be earning this interest rate. An
increase in the nominal interest rate encourages people to hold less money as cash and to hold more
money in interest bearing accounts since they can now earn a greater return by doing so.

Question: 1386

An increase in the long-run aggregate supply curve indicates that

A. unemployment has increased.


B. employment has increased.
C. natural unemployment has increased.
D. potential real GDP has increased.

Answer: D

Explanation:

Short run increases in aggregate supply do not shift the long run potential of the economy; only
increases in the long run aggregate supply curve will effectively increase potential GDP. Short run
aggregate supply may exceed the long run potential of the economy but only temporarily.

Question: 1387

Which of the following will most likely occur in the short run when the long-run equilibrium of an
economy is disturbed by an unanticipated decrease in aggregate demand?

A. an increase in output and a lower price level


B. a decrease in output and a higher price level
C. an increase in output while prices remain unchanged
D. a decrease in output and a lower price level

Answer: D

Explanation:

In response to a decline in aggregate demand, resources may be inflexible; that is, they may not
decline sufficiently to adjust to the reduction in aggregate demand. As a result, there will be a
recession in which output declines and prices in other markets decline.

Question: 1388

In year 0, $10 could purchase a certain basket of goods. In year 20, the identical basket of goods cost
$36. What was the average annualized inflation rate during this period?

A. 4.51%
B. 7.88%
C. 12.21%
D. 30.00%
E. 6.61%

Answer: E

Page | 511
Explanation:

The calculation is as follows: (36/10)^(1/20)-1=0.0661

Question: 1389

Which of the following would increase GDP?

A. buying a 10-year-old house


B. giving $100 to a charity
C. buying a new automobile made in Indiana by a Japanese owned firm
D. buying hamburger buns by McDonald's

Answer: C

Explanation:

Since GDP is the total market value of all final goods and services produced domestically during a
specific period, the purchase of any good that was produced within the U.S. will positively contribute
to the calculation of GDP. Despite the fact that the transaction profits a foreign company, the good
was produced within the U.S. and therefore is counted in GDP.
The hamburger buns do not contribute to GDP because they are an intermediate good. The house is
a re-sold item, which would have been counted in GDP when it was produced, therefore is not
recounted when sold.

Question: 1390

According to the quantity theory of money, which one of the following economic variables would
change in response to an increase in the money supply?

A. prices
B. velocity
C. real income
D. employment

Answer: A

Explanation:
The quantity theory of money implies that the existing money stock M multiplied by velocity V
equals the nominal GDP (output times the price level). In order to maintain the equality, if M
increases, the price level P must also increase.

Question: 1391

Higher unemployment insurance benefits tend to increase unemployment because they

A. reduce the opportunity cost of job search and hence decrease the search time.
B. increase the opportunity cost of job search and hence increase the search time.
C. reduce the opportunity cost of job search and hence increase the search time.
D. increase the opportunity cost of job search and hence decrease the search time.

Answer: C

Explanation:

Page | 512
A change that reduced the job seeker's cost of continued search would lead to more lengthy periods
of search. If unemployment benefits increase, it is less costly to continue looking for a preferred job.
This reduction in the cost of job search would induce job seekers to expand their search time.
Unemployment would be pushed upward.

Question: 1392

Which one of the following would be classified as employed?

A. an auto worker vacationing in Florida during a layoff at a General Motors plant due to an annual
change-over in models
B. a parent who works 50-60 hours per week caring for family members
C. a 21-year-old full-time college student
D. a 17-year-old high school student who works six hours per week as a route person for the local
newspaper

Answer: D

Explanation:

A person who is not actively looking for a job is not a member of the labor force and therefore is not
employed. An individual who is not a member of the formal market and works at home without
wages is not a member of the formal labor force. An auto worker who is waiting to be re-hired or
who was laid off is considered unemployed.

Question: 1393

If unemployment was deemed too high by policy makers, which of the following policy tools might
be utilized?

A. decrease the money supply


B. borrow to finance new military spending
C. reduce government debt
D. increase target interest rates
E. reduce both taxes and government spending
F. raise tariffs to help domestic workers

Answer: B

Explanation:

One method to reduce unemployment is to engage in expansionary fiscal policy. This requires the
government to spend more than it collects in taxes. The result is a net increase in aggregate demand,
which will increase the quantity supplied, and therefore reduce unemployment.

Question: 1394

An increase in the expected future inflation rate will:

A. Move the short-run supply curve to the left.


B. Move the short-run supply curve to the right.
C. Move the long-run supply curve to the right.
D. Move the long-run supply curve to the left.

Page | 513
Answer: A

Explanation:
An increase in the expected future inflation rate will have two impacts. First, sellers will have reduced
incentive to sell at current prices; they would rather store the current production for future sales at
higher prices. Secondly, resource suppliers, to the extent that they anticipate the higher inflation, will
increase the resource prices in their contracts. Both these factors will serve to reduce the quantity
the producers will be ready to supply at any given price, moving the short run supply curve to the
left. The long-run supply curve will not be affected since over that period, all adjustments to the
expected future conditions will have been made, restoring the equilibrium.

Question: 1395

Countries A and B have the same monetary base and reserve requirement. People in A tend to hold
more currency than people in B. The money supply will be:

A. higher in B
B. same in the two countries.
C. insufficient information.
D. higher in A

Answer: A

Explanation:
When people hold currency instead of bank deposits, the money goes out of circulation temporarily
and the full impact of the deposit expansion multiplier is not felt. The higher this tendency to hold
currency, the lower will be the money supply, even though the monetary base has not been affected.

Question: 1396

According to Say's law, there cannot be overproduction of goods and services because

A. overproduction will lead to higher unemployment, which will reduce production.


B. demand creates its own supply.
C. less fortunate countries will always buy the excess output.
D. producing goods generates enough income to buy the total output.

Answer: D

Explanation:

Say's Law is the view that production creates its own demand. Demand will always be sufficient to
purchase the goods produced because the income payments to the resource suppliers will equal the
value of the goods produced.

Question: 1397

The crowding-out model implies that a

A. budget surplus will be highly effective against inflation.


B. budget deficit is likely to stimulate aggregate demand and trigger a multiplier effect that will lead
to inflation.
C. budget deficit will increase the real interest rate and thereby retard private spending.

Page | 514
D. budget surplus will retard aggregate demand and throw the economy into a downward spiral.

Answer: C

Explanation:

The crowding out theory implies that government borrowing drives up real interest rates and thus
crowds out" private investment. Private investment falls under higher interest rates because the cost
of investment (the real interest rate) rises if the government borrows heavily. Under the usual law of
supply and demand, the government causes the interest rate to rise under deficit spending because
there is a limited supply of loanable funds. The government competes with the private sector for
these resources and thus drives up the price (i.e., the interest rate).

Question: 1398

Within the Keynesian model, equilibrium output takes place ________.

A. when actual and expected rates of inflation are equal


B. when the nominal interest rate and real interest rate are equal
C. when spending equals output
D. at full employment

Answer: C

Explanation:

Equilibrium is defined in this model as when aggregate expenditures are equal to output. Thus, the
sum of planned consumption, investment government purchases and the difference between
exports and imports must equal GDP.

Question: 1399

If a broad increase in the price of stocks causes an increase in the real wealth of individuals, then the

A. aggregate demand curve will shift to the left.


B. aggregate demand curve will shift to the right.
C. general price level will fall.
D. aggregate quantity demanded must rise.

Answer: B

Explanation:

As the real wealth of households increases, people demand more goods and services. This causes the
entire aggregate demand curve to shift to the right.

Question: 1400

A client tells you that he currently earns $100,000 per year and is comfortable with his lifestyle at
that income level. He says he is planning on retiring in 5 years. If inflation averages 8% over the next
5 years, approximately what income level will this client require to maintain his current lifestyle?

A. $147,000
B. Not enough information

Page | 515
C. $169,000
D. $122,000
E. $140,000

Answer: B

Explanation:
The calculation is as follows: (1.08)^(5)*100000=$146,933

Question: 1401

Use the table below to choose the correct answer.

Time PeriodActual Inflation


14 percent
24 percent
36 percent
48 percent

According to the adaptive expectations hypothesis, at the beginning of period 3, decision makers
would expect inflation during period 3 to be ________.

A. 6 percent
B. 5 percent
C. 8 percent
D. 4 percent

Answer: D

Explanation:

Under the adaptive expectations hypothesis economic agents base their future expectations on
actual outcomes observed during recent periods. Thus, the most recent periods suggest an inflation
rate of 4 percent will persist in the future.

Question: 1402

Congress passes a law requiring the government to pay certain debts of companies that have
declared bankruptcy. Which of the following terms most accurately describes this program?

A. supply-side
B. automatic stabilizer
C. expansionary fiscal policy
D. moral hazard
E. none of these answers is correct
F. monetary policy

Answer: B

Explanation:

An automatic stabilizer is anything that would decrease the government budget surplus during slow
economies and increase the surplus during strong economic periods. During slow economic periods,
bankruptcies are likely to rise, and by paying a portion of the defunct firms' debts, the government is

Page | 516
injecting demand into the economy. This should be distinguished from an expansionary fiscal policy,
because the program is not designed to expand national income, but to stabilize a slowdown without
the need for further government action.

Question: 1403

The "extra" money you take on a trip in case your car breaks down is an example of the

A. transactions demand for money.


B. inflationary demand for money.
C. restrictive demand for money.
D. speculative demand for money.
E. precautionary demand for money.

Answer: E

Explanation:

Precautionary demand for money is defined as a demand for cash balances in case of unforeseen
circumstances which require money.

Question: 1404

Mr. Taylor states in his will that his $50,000 portfolio of bank CD's will go to his only son. Upon Mr.
Taylor's death, the son sells the CD's and invests the proceeds in a stock mutual fund.
How does this immediately impact M2 and the effective amount money available for transactions?

A. both decrease
B. decrease, increase
C. no change, decrease
D. both increase
E. no change for either
F. decrease, no change
G. increase, decrease

Answer: F

Explanation:

Prior to Mr. Taylor's death, the CD's were counted as part of M2. In addition, this money was not
used for current consumption. After the sale of the CD's, the money is still not being used for current
consumption, but it is now not counted as part of M2. The real level of money in circulation has not
changed here, but the measure of the money supply has decreased. This is an example of a distortion
of money supply measures.

Question: 1405

Given the information below, during which quarters does real income decline?
Nominal GDP GDP Deflator
Q 0 (base) 981100
Q 1993101
Q 21,001106
Q 31,042111
Q 41,040107

Page | 517
A. all four quarters
B. none of these answers is correct
C. 2,3
D. 4 only
E. 2,3,and 4
F. not enough information given

Answer: C

Explanation:

Real income is measure by real GDP. The first step in this problem is to calculate real GDP in each
period by multiplying nominal GDP by the base GDP deflator and then dividing by the current GDP
deflator. The result is as follows:
Q1 983
Q2 944
Q3 939
Q4 972
Therefore real income declines in the second and third quarters.

Question: 1406

Within the Keynesian model, when planned aggregate demand equals total output,

A. the employment rate will equal the labor force participation rate.
B. government expenditures will equal revenues.
C. the output level will tend to persist into the future.
D. income in the future will tend to rise.

Answer: C

Explanation:

Because Keynesian equilibrium is dependent on equality between planned aggregate expenditures


and output, it need not take place at full employment. An economy in Keynesian equilibrium has no
tendency for output to change even if output is well below full employment capacity.

Question: 1407

Suppose that the Central Bank announces that it will increase the money supply by 5%. Producers
collectively determine that this will cause a 5% increase in both the price of their products and the
cost of their inputs. How will this impact real GDP?

A. increase by more than 5% in the short-run


B. increase by an indeterminate degree in the short-run
C. cannot determine without knowing whether GDP is below potential
D. increase by less than 5% in the short-run
E. increase by 5%
F. no impact

Answer: F

Explanation:

Page | 518
In order for an increase in the money supply to cause a temporary increase in production, firms must
essentially be fooled by inflationary price increases that are misread as an increase in demand. In the
scenario described, producers know of the change in money and expect inflation, therefore the
increase in the money supply has no impact.

Question: 1408

If the amount of money in circulation is $200 million and the nominal GDP is $950 million, then the
money velocity is ________.

A. 1.90
B. 4.75
C. 0.21
D. 19.0

Answer: B

Explanation:

Velocity of money is defined as GDP divided by the stock of money. It is simply the average number
of times a dollar is used to purchase a final product or service during a year. Thus, $950 million
divided by $200 million equals 4.75.

Question: 1409

Each month, the Bureau of Labor Statistics calculates unemployment by

A. making projections based on census data.


B. surveying large business and government employers.
C. surveying all workers in the economy.
D. surveying a random sample of households.

Answer: D

Explanation:

The BLS does not contact each person in the U.S. to determine his or her employment status.
Instead, it randomly samples 59,500 households drawn from 729 locations in the U.S. The survey is
designed to reflect geographic and demographic groups in proportion to their representation in the
nation as a whole.

Question: 1410

If the Fed conducts a demand stimulus policy, the rational expectations theory implies that

A. the slope of the Phillips curve will increase.


B. unemployment will temporarily fall below the natural rate.
C. the Phillips curve immediately shifts upward.
D. the natural unemployment will rise.

Answer: C

Explanation:

Page | 519
Under rational expectations people quickly anticipate the effect of policy changes and adjust their
actions accordingly. Thus, under a demand stimulus policy the Phillips curve immediately shifts
upward because individuals anticipate an acceleration in the inflation rate due to the more
expansionary macropolicy.

Question: 1411

If the resources of an economy were fully employed and the marginal propensity to consume were
0.75, a $10 billion increase in investment would cause

A. real income to rise $40 billion.


B. real income to rise $80 billion.
C. real income to rise $20 billion.
D. inflation to increase.

Answer: D

Explanation:

When the economy is operating at full employment capacity, increases in aggregate demand cannot
be accommodated by increases in output. Instead, increases in aggregate demand are met simply by
price increases due to the heightened competition for output.

Question: 1412

The consumer price index (CPI) is calculated

A. using a fixed basket of goods and will tend to understate inflation.


B. using a constantly changing basket of goods and will tend to understate inflation.
C. using a constantly changing basket of goods and will tend to overstate inflation.
D. using a fixed basket of goods and will tend to overstate inflation.

Answer: D

Explanation:

The consumer price index is calculated using a market basket based on the Consumer Expenditure
Survey of urban consumers. There are 364 items in the market basket. The CPI compares the price of
the fixed market basket in the base year with the price of the fixed market basket in the current year
to determine how prices have changed over time.

Question: 1413

Which of the following will most likely result in an increase in the per capita GDP of a country?

A. an increase in population
B. improvement in the average skill level of the labor force
C. an increase in youthful workers as a proportion of the labor force
D. an increase in the number of retired workers
E. an increase in the GDP deflator

Answer: B

Page | 520
Explanation:

Education, training and skill enhancing experience can improve the quality of the labor force and
expand the supply of human resources. Such a change causes the potential output of the country to
increase so that it is possible to produce and sustain a larger rate of output (in nominal and per capita
terms).

Question: 1414

In a situation where information is costly to acquire and individuals must spend time searching for
jobs, which type of unemployment will result?

A. structural
B. seasonal
C. frictional
D. cyclical

Answer: C

Explanation:

Frictional unemployment is due to constant changes in the economy that prevent qualified
unemployed workers from being immediately matched up with existing job opening. It results from
the scarcity of information and the search activities of both employers and employees for
information that will help them make better employment choices.

Question: 1415

Which of the following would be considered non-activist monetary policy?

A. The government allows tax revenues to rise and fall with the business cycle, while keeping
spending constant.
B. The Central Bank acts to smooth the business cycle.
C. The Central Bank attempts to keep inflation at a minimum, ignoring unemployment.
D. The Central Bank prints new currency each year in order to increase the money supply by 3%.
E. The Central Bank targets short-term interest rates.

Answer: D

Explanation:

An activist monetary policy involves a changing policy in reaction to economic events. A classic
example of non-activist policy would be one that increases the money supply by a certain amount
each year.

Question: 1416

In the Keynesian aggregate expenditure model, the larger the marginal propensity to consume,

A. more consumption expands as the result of a decline in investment.


B. greater the marginal propensity to save.
C. the smaller the multiplier.
D. greater the change in income derived from a given change in private investment.

Page | 521
Answer: D

Explanation:

The expenditure multiplier is found by M = 1/(1-MPC). Thus, the larger the MPC the larger the
multiplier. The multiplier magnifies changes in aggregate expenditure into larger changes in
aggregate output. the larger the multiplier then, the greater the change in income derived from a
given change in private investment.

Question: 1417

When financing a budget deficit, the government might borrow money from ________.

A. a central bank
B. individuals
C. all of these answers
D. businesses
E. foreigners

Answer: C

Explanation:

The government can borrow money from the central bank, from private businesses, private citizens
and foreigners.

Question: 1418

If consumption equals 1,100 when disposable income is 1,200 and increases to 1,400 when
disposable income goes to 1,600, what are the marginal propensities to consume and to save?

A. MPC = 1/5; MPS = 4/5


B. MPC = 2/3; MPS = 1/3
C. MPC = 3/4; MPS = 1/4
D. MPC = 1/3; MPS = 2/3

Answer: C

Explanation:

Since the MPC is determined according to the ratio: MPC = additional consumption divided by
additional income, the MPC here equals: 300/400 = .75. MPS = 1-MPC so MPS = .25.

Question: 1419

Keynesians believe that a discretionary fiscal policy:

A. is destabilizing due to difficulty in timing and should not be used actively.


B. is ineffective since people rationally anticipate its future effects.
C. none of these answers.
D. is not as effective as the monetary policy in controlling unemployment.

Answer: C

Page | 522
Explanation:

Keynesians believe that aggregate demand has a major influence. They maintain that suppliers will
produce at a level consistent with anticipated aggregate demand. Hence, to increase economic
production when the economy is operating below capacity, Keynesians advocate spurring the
demand in the market through the use of an expansionary fiscal policy.

Question: 1420

Which of the following is/are true about monetary policy?

I. The money supply is neutral in the long-run.


II. Monetary policy can only serve to decrease economic volatility.
III. Of the monetary aggregates, Central Bankers only have direct control over the monetary base.

A. I, II, III
B. III only
C. I only
D. I, III
E. none of these answers is correct
F. II only

Answer: C

Explanation:
In the long-run, relative prices are important, not the price level itself. To understand why, imagine if
your salary increased by ten times but so did the price of all the goods you buy. This would be a
neutral event.
If money is neutral, then monetary policy can have no long-term effect. However, in the short-term,
changes in the price level can cause producers to change their production choices, ostensibly helping
to smooth out the business cycle.
Central Bankers generally have control over the bank reserve requirement and the level of currency.
They do not have control over the amount of currency individuals put into deposit accounts which
could later be lent out by banks. Hence, Bankers control the monetary base, but not other money
aggregates such as M2 and M3.

Question: 1421

Keynesian analysis suggests that a planned budget surplus

A. will affect aggregate demand only if the money supply decreases by the size of the surplus.
B. is proper during periods of inflation but may increase unemployment if timed improperly.
C. will stimulate output and employment.
D. will stimulate both consumption and income.

Answer: B

Explanation:

Keynesians support "counter-cyclical" polices: that is, under an inflationary/full capacity economy,
the government should reduce spending (operate under a budget surplus) to contract aggregate
demand and control economic growth so as to avoid high levels of inflation. Thus, a budget surplus is
appropriate during periods of inflation. If such a policy is enacted when the economy is below full
capacity, the proper counter-cyclical policy for the government is to deficit spend. Thus, if the

Page | 523
government operates under a budget surplus in this situation, aggregate demand will further
contract and unemployment will increase.

Question: 1422

Within the Keynesian aggregate expenditure model, the central catalyst that leads to changes in
output and employment is changes in

A. prices.
B. aggregate supply.
C. wage rates.
D. aggregate expenditures.

Answer: D

Explanation:

Aggregate expenditures are the catalyst of the Keynesian model. Until full employment is attained,
supply is always accommodative. Increases in aggregate expenditures thus lead to an expansion in
both output and employment as long as the economy is operating below potential capacity.

Question: 1423

Built-in features that tend to promote a budget deficit during a recession and a budget surplus during
an inflationary boom are known as ________.

A. automatic stabilizers
B. counter-cyclical policies
C. active budget deficits
D. restrictive fiscal policies

Answer: A

Explanation:
Such built-in stabilizers exist independently of any policy change: tax revenues necessarily fall during
a recession (as the economy contracts, there is less income to tax) and government expenditures
necessarily rise (more individuals receive government transfers such as unemployment benefits or
welfare payments). The converse is true for an inflationary environment: tax revenues are in excess
of government expenditures creating a budget surplus. The consequence of these automatic
reactions is that during a recession, the economy is stimulated by higher government spending and
during inflation economic activity is reduced by smaller government spending.

Question: 1424

If business decision makers suddenly become more optimistic about future sales and profits, which
of the following will most likely occur?

A. The interest rate will fall, causing both investment and GDP to rise.
B. Income will increase and actual saving will remain constant.
C. Actual investment will increase but consumption will decrease, leaving GDP unchanged.
D. Investment will increase, causing both aggregate demand and GDP to increase.

Answer: D

Page | 524
Explanation:

If business decision makers are optimistic about the future, they will increase their private
investment and stimulate other parts of the economy. Thus, aggregate demand and aggregate
output will increase.

Question: 1425

Which one of the following will most likely cause a future increase in a country's sustainable level of
real output?

A. an increase in income redistribution payments from high- to low-income recipients


B. a decline in the nation's oil reserves
C. higher marginal tax rates
D. an increase in the national debt
E. an increase in the economy's net investment rate

Answer: E

Explanation:

Net investment can increase the supply of physical capital which makes it feasible for the economy to
produce a higher level of output permanently into the future. Investment in physical capital can
expand the supply of buildings, machines and other physical assets. This allows the economy to
produce and sustain a large rate of output, shifting both LRAS and SRAS to the right.

Question: 1426

Assume that Polaroid decides to build a camera-research facility in Twin Falls, Idaho. The plant will
employ 1,000 workers who will be guaranteed full-time jobs at annual earnings of $25,000. If the
marginal propensity to consume in Twin Falls is 3/4, what change in income will result from
operation of the plant for one year?

A. $75 million
B. $50 million
C. $25 million
D. $100 million

Answer: D

Explanation:

The expenditure multiplier is found by M = 1/(1-MPC). Thus, here M = 1/(1-3/4) = 4. Therefore the
1,000 x $25,000 = $25 million increase in aggregate expenditures is magnified four times to $100
million.

Question: 1427

In the basic Keynesian model of national income determination, aggregate expenditures refer to

A. the amount of demand for consumer goods that would arise if all citizens had all the income they
wanted.
B. the combined expenditures of consumers and businesses minus government spending.
C. spending for consumption, investment and exports less imports plus government purchases of

Page | 525
goods and services.
D. consumer spending measured in constant prices.
E. the amount of GDP that could be produced if unemployment were zero.

Answer: C

Explanation:

Aggregate expenditures are the sum of planned consumption, investment, government expenditures
and net exports.

Question: 1428

Fiscal policy designed to increase aggregate demand during economic downturns and decrease
aggregate demand during economic booms is called

A. expansionary fiscal policy.


B. supply-side fiscal policy.
C. new classical fiscal policy.
D. counter-cyclical fiscal policy.

Answer: D

Explanation:

Counter-cyclical fiscal policy promotes the economy's use of its "automatic stabilizers": government
expenditures should rise during recessions (increasing aggregate demand) and should fall during
economic booms (decreasing aggregate demand).

Question: 1429

________ policy may be used by government officials to deliberately generate a budget deficit.

A. Discretionary fiscal
B. Active fiscal
C. Discretionary monetary
D. Active monetary

Answer: A

Explanation:
Discretionary fiscal policy is defined as policymakers instituting deliberate changes in tax laws or
spending on government programs that are designed to generate a budget deficit. Deficits
emanating from this source are referred to as active budget deficits.

Question: 1430

Deposits denominated in U.S. dollars at banks and other financial institutions outside the U.S. are
known as ________.

A. Federal reserves
B. Money market deposit accounts
C. Foreign bank reserves
D. Fiat money

Page | 526
E. Eurodollar deposits

Answer: E

Explanation:

Although this name originated because of the large amounts of such deposits held at banks in
Western Europe, similar deposits in other parts of the world are also called Eurodollars.

Question: 1431

The prevailing budget philosophy prior to Keynes called for a balanced budget. Keynes argued that
the government's tax and spending policies should be determined by the

A. demand requirements necessary to attain full employment of resources.


B. public's willingness to accept or reject tax changes.
C. size and quality of the labor force.
D. demand for public goods.

Answer: D

Explanation:
Keynesian economists suggest that the government should operate under budget deficits when the
economy is in a recession and under a surplus when the economy is growing very quickly. The
consequence of this "counter-cyclical" policy will be a stabilization of the economy orchestrated
through the government's contribution to aggregate demand. The government should attempt to
stabilize the economy through its contribution to aggregate demand.

Question: 1432

A reserve requirement of 5 percent implies a potential money deposit multiplier of ________.

A. 20
B. 5
C. 10
D. 25
E. 100

Answer: A

Explanation:
The potential money deposit multiplier is the reciprocal (inverse) of the required reserve ratio. Thus,
a reserve requirement of .5 implies a potential money deposit multiplier of 1/.5 = 20.

Question: 1433

Inflation is problematic for which of the following reasons


I. Purchasing power declines at a faster rate than incomes rise
II. Inflation causes nominal interest rates to rise
III. Inflation creates a disincentive to enter into otherwise beneficial long-term agreements

A. I, III
B. I only
C. II, III

Page | 527
D. III only
E. III only
F. I, II, III

Answer: D

Explanation:

Inflation generally implies that all prices, including the price of labor, increase at the same rate.
Therefore inflation does not cause a real decline in income. Although nominal interest rates would
rise with increased inflation, this does not cause a fundamental economic problem. However,
because inflation tends to create long-term uncertainty, many long-term agreements are foregone
that would otherwise be beneficial.

Question: 1434

The unemployment rate of teen-agers is substantially higher than the unemployment rate of prime-
age workers primarily because teen-agers

A. are more likely to be in the armed forces and therefore they end up being counted as
unemployed.
B. are less likely to be in the labor force than older workers since many teenagers are still attending
school.
C. are less likely to switch jobs and move into and out of the labor force than older workers.
D. are more likely to switch jobs and move into and out of the labor force than older workers.
E. if they search more diligently, can generally find high-paying jobs that are available only to
experienced, highly skilled workers.

Answer: D

Explanation:

Youthful workers experience more unemployment because they change jobs and move in and out of
the labor force often.

Question: 1435

Frictional unemployment means

A. there is a decline in the demand for labor in the aggregate, due to recessionary tendencies in the
economy.
B. there are not enough jobs to go around.
C. jobs are plentiful but workers scarce.
D. imperfect information prevents qualified workers from matching up with the available jobs.

Answer: D

Explanation:

Frictional unemployment refers to unemployment due to constant changes in the economy that
prevent qualified unemployed workers from being immediately matched up with existing job
openings. It results from the scarcity of information and the search activities of both employers and
employees for information that will help them make better employment choices.

Page | 528
Question: 1436

Mr. Brown was in the habit of cashing his weekly paycheck and using this cash for his daily expenses.
However, he now gets his paycheck direct deposited into his checking account and uses a debit card
exclusively instead of cash. This represents an increase in which of the following types of money
demand for Mr. Brown?

A. transactions demand
B. electronic demand
C. speculative demand
D. store-of-wealth demand
E. all of these answers are correct
F. none of these answers is correct

Answer: F

Explanation:

Mr. Brown money demand has not changed. He is still using his weekly paycheck to satisfy normal
expenses, or transactions demand. Although he is "storing" his cash in a checking account until it is
to be used, this does not represent store-of-wealth demand. Store-of-wealth implies money that is
either invested or accumulated for some future need.
Note that although Mr. Brown's money spending habits have not changed, the way his money is
measured by economists has changed. Previously his money was held entirely in currency, whereas
now a portion of his money is held in reserve by his bank. Although effectively there has been no
change in the use of money, certain monetary aggregates will change.

Question: 1437

Inflation has been about 5% for the last several years. There is widespread fear that oil and natural
gas prices are about to spike at the same time there is unusually high unemployment. If inflation
were actually 6% next year, and this causes no change in real GDP, what can be said about the
general expectation for inflation?

A. producers were forming inflation forecasts based on rational expectations


B. the general consensus inflation forecast must have been less than 6%
C. producers were basing their inflation views on adaptive expectations
D. consumers must have foreseen inflation of 6% and increased savings accordingly
E. the oil scare held down GDP
F. none of these answers is correct

Answer: D

Explanation:
If actual inflation were 6%, and this caused no change in real GDP, we know that inflation
expectations were probably higher than 6%. There are two major theories as to how inflation
expectations are formed. One is adaptive expectations theory. This states that economic participants
will expect inflation to be about what it was in the past. The rational expectations hypothesis states
that economic participants will consider all available information and make an estimation based on
this knowledge.
In this case, we are told that inflation has been about 5% the last several years. Therefore if
producers are generally following the adaptive expectations hypothesis, they would have expected
5% inflation. However, we are also told that there may be inflationary pressure from the
commodities market.

Page | 529
Therefore under the rational expectations hypothesis, market participants would expect inflation to
be something higher than 5%. Since we know that expectations must be higher than 6%, we also
know that market participants must be forming their expectations rationally.

Question: 1438

Which of the following correctly describes how expansionary fiscal policy through tax cuts eventually
impacts employment?

A. Tax cuts result in more income for consumers, which shifts the aggregate demand curve upward,
which increases the price level. This inflationary effect encourages firms to use more flexible labor
and less inflexible capital.
B. A tax cut shifts the aggregate demand curve out. This causes upward movement along the
aggregate supply curve, which increases resource utilization.
C. Tax cuts result in more government debt which causes the interest rate to rise and therefore
unemployment to fall.
D. Tax cuts result in a multiplier effect, as politicians showing confidence in the economy encourages
businesses to spend on expansions.
E. The tax cut causes an increase in quantity demand. This causes an outward shift in the aggregate
supply curve and hence more employment.

Answer: B

Explanation:

Consumers react to tax cuts as if it were an increase in income. This causes more demand for goods
and services at every price level. The demand curve would therefore shift to the right. This causes
movement along the supply curve, expanding output. This expansion in output should reduce slack
resources, i.e. unemployment.

Question: 1439

If the money supply is held constant, an increase in nominal GDP leads to ________ in the velocity of
money.

A. all of these answers are possible.


B. no change
C. a decrease
D. an increase

Answer: D

Explanation:

The velocity of money, V, satisfies the equation, MV = GDP, where M is the money supply. If M is held
constant while GDP increases, then V must increases. Intuitively, this says that if the money supply is
not changed, then for production and consumption of a higher GDP, each dollar must change hands
more frequently.

Question: 1440

In the Keynesian model, if the multiplier is 3, then ________.

A. MPS must be approximately .30

Page | 530
B. MPC must be approximately 2/3
C. MPC must be approximately .30
D. MPC must be approximately 1/3
E. MPS must be approximately 2/3

Answer: D

Explanation:

The expenditure multiplier is found by M = 1/(1-MPC). Therefore, 3= 1/(1-MPC) implies that MPC is
.333)

Question: 1441

If the government moves the entire tax schedule upward by 5%, the primary effect on the economy
will be through:

A. all of these answers.


B. the demand side, wherein disposable income and private demand goes down.
C. a decrease in the budget deficit due to increase in tax revenues.
D. the supply side, wherein the relative attractiveness of productive activity goes down in relation to
leisure and tax avoidance activities.

Answer: B

Explanation:

It is important to realize that "the supply side answer" is not entirely accurate. The supply side effects
become important when the marginal tax rates are skewed in such a way that additional
productiveactivity is discouraged due to higher taxes. When the entire tax schedule moves, the effect
through the demand side is much bigger.

Question: 1442

An economy is currently experiencing high inflation. A Keynesian would suggest which of the
following to combat this:
I. Increasing interest rates.
II. Increasing tax rates.
III. Decreasing government expenditure.
IV. Raising reserve requirements.

A. I & III
B. I, II, III & IV
C. I, II & III
D. II & III

Answer: D

Explanation:

To control high inflation, Keynesians suggest cutting back on aggregate demand by either raising
taxes or decreasing the size of the budget deficit by decreasing government expenditures. They thus
recommend a restrictive fiscal policy to combat high inflation. Note that changing the reserve
requirement is not an option available to the Federal government in the U.S. because the reserve

Page | 531
requirement is set by an independent body, The Federal Reserve.

Question: 1443

According to monetarists, ________ is a primary source of economic instability:

A. fluctuations in aggregate demand for money


B. erratic monetary policy
C. fluctuations in aggregate demand for money and fluctuations in aggregate consumption demands
D. fluctuations in aggregate consumption demands

Answer: B

Explanation:

Monetarists believe that monetary policy has a powerful influence on the economy but also realize
that there are lengthy and unpredictable time lags between the implementation of a monetary policy
and the realization of its primary effects. Hence, they reject the active use of discretionary monetary
policy for controlling the economy, prescribing a steady growth in money supply to track the real
growth rate.

Question: 1444

"Decision makers systematically err in their forecasts of economic variables." This is implied by which
of the following:

A. Rational Expectations hypothesis.


B. Adaptive Expectations hypothesis.
C. Random Expectations hypothesis.
D. Constant Expectations hypothesis.

Answer: B

Explanation:

According to the Adaptive Expectations hypothesis, people consider the recent past as the best
predictor of the immediate future. Hence, when an economic variable is increasing, they will
consistently tend to under-estimate the future value of that variable and vice versa. Thus, they
systematically err in their decision-making, without learning from past mistakes.

Question: 1445

Within the simple Keynesian model, when an economy operates below its long-run, full employment
output constraint, an increase in aggregate demand will lead to an

A. increase in unemployment.
B. increase in prices.
C. increase in interest rates and money income, but employment and real income will remain
constant.
D. increase in employment, output and real income.

Answer: D

Explanation:

Page | 532
Since according to the Keynesian model aggregate demand determines the level of output in the
economy, an increase in the demand will cause output to increase. Since output is equivalent to
income, real income also increases. The increase in output stimulates increases in labor demand and
thus unemployment declines. Increases in aggregate demand below the full employment capacity of
the economy has no effect on prices.

Question: 1446

An economic researcher publishes evidence that the consensus inflation estimation for the following
year has no correlation with the actual inflation level the year before. The estimation error is often
very large, but does not display a pattern. Which of the following theories would this evidence
support?

A. rational ignorance
B. mean reversion
C. heteroskedasticity
D. monetarist theory
E. rational expectations hypothesis
F. adaptive expectations hypothesis

Answer: E

Explanation:

Under the rational expectations hypothesis, individuals will form inflation expectations based on all
relevant information available. Therefore a comparison of expected and realized inflation would
show a random estimation error and no correlation with past estimates. The adaptive expectations
hypothesis suggests that individuals will base their views of the future on their recent experience.
Under this theory, estimations would show a pattern in the error. Expected inflation would tend to
be too high following a period of high inflation, and then too low follow a period of limited inflation.

Question: 1447

During an economic boom, the AD-AS model indicates that

A. both the real interest rate and real wage rates will decline.
B. real interest rate will decline and real wage rates will rise.
C. real interest rate will rise and real wage rates will decline.
D. both the real interest rate and real wage rates will increase.
E. real wage rates will remain constant while the real interest rate will rise.

Answer: D

Explanation:

High demand for resources (like labor) during an economic boom will increase the price of labor (the
real wage rate). Similarly high demand for loanable funds during a boom will cause the real interest
rate to rise.

Question: 1448

An economy is currently operating at full employment. If the Fed unexpectedly decreases the reserve
requirement, in the short run, the aggregate output will ________. The unemployment will

Page | 533
________ the natural rate.

A. fall, rise above


B. rise, fall below
C. rise, rise above
D. fall, fall below

Answer: B

Explanation:

The unexpected decrease in the reserve requirement causes an unexpected expansion in money
supply. In the short run, this leads to lower real interest rates. The increased availability of credit
coupled with the rise in demand leads to an increase in real output and employment. Temporarily,
the unemployment rate falls below the natural rate.

Question: 1449

The population of Tunisia is 100 million; 5 million are unemployed and 90 million hold jobs. The
employment rate in Tunisia is

A. 90 percent.
B. 5 percent.
C. 95 percent.
D. 85 percent.

Answer: A

Explanation:
The employment rate is equal to the number employed/total population or 90 million/100 million =
90%.

Question: 1450

When individuals are unemployed because they lack the qualifications to fill available jobs, we call
this a form of

A. natural unemployment.
B. frictional unemployment.
C. structural unemployment.
D. cyclical unemployment.

Answer: C

Explanation:

Structural unemployment implies that changes in the basic characteristics of the economy prevent
the "matching up" of available jobs with available workers. Thus, workers possess skills which are not
demanded by employers and employers demand skills that unemployed workers do not have.

Question: 1451

An expansionary fiscal policy causes which of the following effects?

Page | 534
I. The domestic currency depreciates.
II. Exports decrease.
III. Real interest rates increase.
IV. Capital flows in from abroad.

A. II, III & IV


B. I, II & III
C. I, II, III & IV
D. I & III

Answer: B

Explanation:
When the government adopts an expansionary fiscal policy, the aggregate demand increases,
causing an increase in prices. Since the demand for loanable funds increases due to the
governmental presence in the market, the real interest rate increases. This attracts foreign
investment, raising the demand for domestic currency, which appreciates in response. The
appreciation of the domestic currency makes imports cheaper and exports costlier. As a result,
exports decline and imports increase over time.

Question: 1452

Holding income constant, if consumers and investors decide to spend more on goods and services,
then

A. additional spending will require increased borrowing (or less saving), which will drive up the real
rate of interest.
B. producers will expand output without increasing prices to accommodate the stronger demand.
C. additional spending will increase the supply of loanable funds, which will reduce the real interest
rate and thereby trigger additional spending.
D. the economy's long-run capacity (LRAS) will expand to accommodate the stronger demand.

Answer: A

Explanation:
Strong aggregate demand will put pressure on the market for loanable funds and will result in higher
interest rates (as the demand for funds increases). This will cause aggregate demand to contract from
its heightened state. This adjustment represents one of the economy's self-correcting mechanisms.

Question: 1453

The Keynesian model indicates that when individuals plan to save more (and spend less), the result
may be a(n)

A. increase in investment because investment always equals saving.


B. decline in the equilibrium level of income.
C. increase in the marginal propensity to consume.
D. increase in equilibrium income by some multiple of the increase in saving.

Answer: B

Explanation:

An increase in savings implies a decrease in disposable income and thus a decline in consumption. As

Page | 535
a result aggregate demand will fall and output will follow. Output is equivalent to income and thus
the equilibrium level of income also declines.

Question: 1454

Given:

Population50 million
Number in the labor force30 million
Number employed full time20 million
Number employed part time8 million
Number unemployed2 million

What is the unemployment rate of the economy?

A. 10.0 percent
B. 4.0 percent
C. 7.1 percent
D. 6.7 percent

Answer: D

Explanation:

The unemployment rate is the number of unemployed people divided by the number of people in
the labor force or: 2/20 = 6.7%.

Question: 1455

Which of the following about the multiplier is false?

A. Idle resources are necessary before the multiplier can bring about an increase in real income.
B. The size of the multiplier relates directly to the size of the marginal propensity to consume.
C. It is defined as 1 / (1 - the marginal propensity to save).
D. It takes time for the multiplier to work.

Answer: C

Explanation:

The multiplier is defined as M = 1/(1-MPC) rather than 1 / (1 - MPS).

Question: 1456

In Keynesian view, the best macroeconomic policy is to:

A. increase government investment expenditures during economic booms to offset effects of


reduced capacity during the recession to follow.
B. regulate aggregate expenditures to match output capacity.
C. regulate wages to control inflation and promote full employment.
D. control government expenditures to control inflation.

Answer: B

Page | 536
Explanation:

In Keynesian economics, fluctuations in aggregate demand are a major source of movements in the
economy. Equilibrium occurs at any output level which equals the spending level. To maintain an
equilibrium at the maximum sustainable real GDP, Keynesians prescribe regulating the planned
aggregate expenditures to equal the potential real GDP.

Question: 1457

If the Fed introduces an expansionary monetary policy:


I. real interest rates fall.
II. the U.S. dollar appreciates.
III. the U.S. exports increase relative to imports.

A. II & III
B. I, II & III
C. I & II
D. I & III

Answer: D

Explanation:

When the Fed introduces an expansionary monetary policy, the money supply increases, causing the
real interest rate to fall. This leads to a flow of funds out of the U.S. and into economies with higher
real rates. The decreased demand for the U.S. dollar causes it to depreciate. This, in turn, makes the
U.S. goods cheaper relative to foreign goods, increasing the U.S. exports and decreasing its imports.
Note that this offsetting increase in the demand for U.S. dollar works more slowly than the initial
depreciation caused by the outflow of monetary funds.

Question: 1458

The nation of Myopia is having a massive inflation problem. To stabilize prices, the Myopian Central
Bank decides to acquire large numbers of Capitalian Dollars (a very stable currency) and offer to
exchange five Myopian Pesos for one Capitalian Dollar on demand. How does this impact the
monetary base and the effective money supply for the nation of Capitalia?

A. both decrease
B. no change, decrease
C. decrease, increase
D. no change, increase
E. no change for either
F. both increase
G. increase, decrease

Answer: B

Explanation:

The monetary base is made up of currency and bank reserves. In this case, Myopia has taken large
amounts of currency out of circulation in Capitalia, but the expatriated currency is technically still
part of the monetary base. Therefore the base has not changed, but the effective money in
circulation has decreased. This is one of the problems economists face in attempting to measure the
real money supply.

Page | 537
Question: 1459

Which one of the following will most likely cause a future increase in the growth rate of real output?

A. an increase in income redistribution payments from high- to low-income recipients


B. discovery of a new low-cost method of converting oil shale into petroleum
C. higher marginal tax rates
D. a decrease in the economy's net investment rate

Answer: B

Explanation:

Improvements in technology that permit us to squeeze a larger output from a specific resource
supply enhance our productivity and thereby shift the long run aggregate supply curve to the right.

Question: 1460

Within the AD/AS model, an unanticipated increase in short-run aggregate supply will cause real
output to

A. decline and prices fall.


B. decline and prices rise.
C. expand and prices rise.
D. expand and prices fall.

Answer: D

Explanation:

An unanticipated increase in aggregate supply decreases the price level and increases current
output. The long run aggregate supply of the country is not affected.

Question: 1461

Within the Keynesian model, if an economy operates below full employment,

A. reducing wage rates and resource prices will quickly restore full-employment equilibrium.
B. reducing government expenditures will direct the economy back to full- employment equilibrium.
C. an increase in the real interest rate will soon restore full-employment equilibrium.
D. output will tend to remain below full-employment capacity unless aggregate expenditures
increase.
E. reducing the real interest rate will soon restore full-employment equilibrium.

Answer: D

Explanation:

An economy operating below its full employment level can only reach its full employment capacity
by increasing aggregate expenditures. This can be achieved only by inducing consumers, investors,
governments and foreigners to increase their expenditures.

Question: 1462

Page | 538
When an economy operates well below its full-employment capacity and the marginal propensity to
consume is 3/4, a $20 billion increase in autonomous investment will cause the equilibrium income
to rise

A. $80 billion.
B. $40 billion.
C. $20 billion.
D. $15 billion.

Answer: A

Explanation:
The expenditure multiplier is found by M = 1/(1-MPC). Thus, here M = 1/(1-3/4) = 4. Therefore $20
billion increase in aggregate expenditures is magnified four times to $80 billion.

Question: 1463

Government borrowing to fund current spending tends to cause _________ to rise. Subsequently,
the local currency will _________ causing the trade deficit to rise.

A. savings rate, depreciate


B. national income, depreciate
C. inflation, depreciate
D. marginal propensity to consume, appreciate
E. interest rates, appreciate

Answer: E

Explanation:

Government borrowing creates demand for loanable funds, and therefore increases the price of such
funds, i.e. the interest rate. When interest rates rise versus foreign rates, the value of the local
currency rises on the foreign exchange market. This causes imports to be less expensive locally and
exports to be more expensive abroad, and therefore tends to cause the current account trade deficit
to widen.

Question: 1464

The relationship between inflation and unemployment rate is known as:

A. Phillips curve.
B. Equation of exchange
C. Lucas curve.
D. Inflation-unemployment parity

Answer: A

Explanation:
Following earlier work by the British economist,

A.Phillips, economists noted an inverse relationship between the rate of unemployment and the rate
of inflation. This curve is widely known as the Phillips curve.

Page | 539
Question: 1465

The GDP measures:

A. the total value of all goods, services and financial transactions during a period.
B. the total value of all the earnings in the economy.
C. the total value of all goods and services produced in the economy during a period.
D. the total value of all final goods and services produced in the economy during a period.

Answer: D

Explanation:

Note that the phrase "final goods" is critical. Intermediate goods are ignored in the calculation of
GDP, as are all financial transactions. Further, since earnings look at net profits on goods and services
but subtract out the employee salaries, for one, a sum of the earnings in an economy will
significantly underestimate GDP.

Question: 1466

A bank has deposits of $23 billion and reserves of $3.1 billion. If the excess reserves equal $400
million, the deposit expansion multiplier equals ________.

A. 9.11
B. 8.33
C. 8.52
D. 7.42

Answer: C

Explanation:

The required reserves equal 3.1 - 0.4 = $2.7 billion on deposits worth $23 billion. Thus, the required
reserve ratio equals 2.7/23 = 11.74%. So the deposit expansion multiplier equals 1/0.1174 = 8.52.

Question: 1467

Which of the following would not be an expected impact of a debt pay down program?

A. a decrease in aggregate demand


B. a shift to the left in the aggregate supply curve
C. an increase in unemployment
D. a decrease in the price level
E. falling interest rates

Answer: B

Explanation:

Theoretically, fiscal policy should cause a shift in the aggregate demand curve, which will cause
movement along the supply curve. Debt pay down implies that net government spending (spending
less taxes) is negative, and therefore the demand curve has shifted to the left, while the supply curve
has not moved.

Page | 540
Question: 1468

A reserve requirement of 12.5 percent implies a potential money deposit expansion multiplier of
________.

A. 8
B. 12.5
C. 25
D. 5

Answer: A

Explanation:
The potential money deposit expansion multiplier is calculated by taking the inverse of the reserve
requirement ratio. This is: 1/.125 = 8.

Question: 1469

James Morrison is a profit-seeking banker. His bank has $25 million in excess reserves. Mr. Morrison

A. can probably increase his profits by increasing his excess reserves.


B. cannot affect his profits by changing the amount of excess reserves held by his bank.
C. cannot change excess reserves held by his bank because this level is set and strictly enforced by
the Fed.
D. can probably increase his profits by reducing his excess reserves.

Answer: D

Explanation:

A bank holding excess reserves can increase its profits by extending more loans and holding fewer
excess reserves. This is because the excess reserves held by the bank do not earn any interest.
However, excess reserves extended as loans earn a positive interest rate and therefore will allow the
bank to make a positive profit. Excess reserves represent the portion of reserves held by the bank in
excess of the required reserve ratio set by the Fed.

Question: 1470

Inflation

A. causes the purchasing power of a dollar to rise.


B. generally benefits the poor at the expense of the rich.
C. is measured by changes in the cost of a typical market basket of goods between time periods.
D. almost always benefits creditors at the expense of debtors.

Answer: C

Explanation:

Inflation can be measured by using price indexes. In general, these indexes are constructed by
calculating the difference between the price of a basket of consumer goods in earlier and later years
and dividing by the price of the basket in the first year.

Question: 1471

Page | 541
Which of the following is/are true about aggregate demand?

I. An increase in the real rates of interest increases current demand.


II. An increased optimism about the future direction of the economy will increase current demand.
III. An increase in the expected future inflation rate increases current demand.

A. II & III
B. I & II
C. I & III
D. II only

Answer: A

Explanation:
An increase in real interest rates makes current consumption more expensive in terms of opportunity
cost. Hence, when real rates rise, current demand falls. On the other hand, if future inflation is
expected to be high, then consumers want to buy goods in the current period, raising aggregate
demand. Similarly, if they expect the future to be prosperous, they will spend some of that expected
future income on current consumption. Hence, both higher expected future inflation and future
wealth will increase current demand.

Question: 1472

"An erratic monetary policy is the primary sources of business instability and inflation." This view is
held by

A. Monetarists.
B. Keynesians.
C. Classical economists.
D. Neoclassical economists.

Answer: A

Explanation:
Monetarists believe that monetary policy has a powerful influence on the economy but also realize
that there are lengthy and unpredictable time lags between the implementation of a monetary policy
and the realization of its primary effects. Hence, an erratic monetary policy can lead to big
instabilities in the economy.

Question: 1473

How will an unanticipated increase in aggregate demand emanating from an increase in business
and consumer optimism influence equilibrium output in the goods and services market?

A. Output will decrease and prices rise.


B. Output will decrease and prices fall.
C. Output will increase and prices rise.
D. Output will increase and prices decline.

Answer: C

Explanation:

Page | 542
Optimism concerning the future direction of the economy will stimulate investment. Investment
today may be necessary in order to benefit fully from future opportunities. This will cause aggregate
demand to rise; in response, output will rise as will prices (due to increased competition for output
and resources).

Topic 4
4,Financial Reporting and Analysis

Question: 1474

Cash outflows for payment of cash dividends is an example of:

A. cash flows from operating activities


B. cash flows from financing activities
C. cash flows from noncash investing and financing activities
D. cash flows from investing activities

Answer: C

Explanation:
Providing stockholders with a return on their investment in the form of a cash dividend is a financing
activity.

Question: 1475

Basic earnings per share is calculated as:

A. [Net Income- Dividends]/Weighted Avg # of shares outstanding.


B. [Net Income-Preferred Dividends]/Weighted Avg # of common shares outstanding.
C. [Net Income]/[Common shares outstanding]
D. [Net Income-Preferred Dividends]/[Weighted Avg # of common shares outstanding] - EPS impact
of diluted convertibles.

Answer: B

Explanation:
The basis EPS formula aims to derive earnings per share of common stock for the amount of the year
that the common shares were outstanding. It is for this reason that preferred dividends are not
included and the weighted average number of common shares outstanding are used in the
denominator.

Question: 1476

Which of the following about Free Cash Flows (FCFs) is TRUE?

A. FCF = Cash from operations - outlays for replacement of used capacity.


B. FCF = Cash from operations - outlays for maintenance of current capacity - dividends declared.
C. FCF = Cash from operations - outlays for dividend payments and scheduled debt retirement
D. FCF = Net Income + Non-cash expenses.

Answer: B

Explanation:
FCF = Cash flows from operations - Net capital expenditures required to maintain productive capacity

Page | 543
- Dividends on preferred stock and common stock (assuming a payout policy).

Question: 1477

The opinion paragraph of an independent auditor's report begins, "In our opinion, the financial
statements referred to above present fairly, in all material respects, the financial position..." This
language states ________.

A. an unqualified opinion
B. a disclaimer of opinion
C. none of these answers
D. a qualified opinion
E. an "except for" opinionThat answer is correct!

Answer: A

Explanation:
These references do not constitute a disclaimer or a qualified or adverse opinion, therefore, the
language refers to an unqualified opinion.

Question: 1478

A firm has a net income of 150, an increase in accounts receivables of 30, depreciation of 55 and a
decrease in accounts payable of 25. Its operating cash flow is ________.

A. 200
B. 150
C. 95
D. 210

Answer: B

Explanation:
operating cash flow = net income + noncash expenses - non-cash revenues = 150 + 55 - 30 - 25 = 150.

Question: 1479

Stock dividends and stock splits differ in that

A. stock splits are paid in additional shares of common stock, whereas a stock dividend results in
replacement of all outstanding shares with a new issue of shares.
B. a stock dividend results in a decline in the par value per share.
C. in a stock split a larger number of new shares replaces the outstanding shares.
D. stock splits involve a bookkeeping transfer from retained earnings to the capital stock account.
E. none of these answers.

Answer: C

Explanation:
A stock split does not involve any accounting entries. Instead, a larger number of new shares are
issued to replace and retire all outstanding shares.

Question: 1480

Page | 544
Which of the following would be classified a cash inflow from investing activities?

A. Cash paid to retire bonds


B. Proceeds from issuing stock
C. Cash paid for dividends
D. Proceeds from selling investments in the equity securities of other companies

Answer: D

Explanation:
All other responses qualify as cash flows from financing activities.

Question: 1481

Which of the following would be classified a cash inflow from investing activities?

A. proceeds from selling investments in the debt securities of other entities, except cash equivalents
B. all responses are correct
C. proceeds from collecting the principal amount of loans
D. proceeds from selling investments in the equity securities of other companies

Answer: B

Explanation:
All responses qualify as cash inflows from investing activities.

Question: 1482

Under the treasury stock method, primary earnings per share data are computed as if options and
warrants (outstanding for the entire year) were exercised at the

A. none of these answers.


B. beginning of the period and as if the funds obtained thereby were used to purchase common stock
at the current market price in effect at the end of the period.
C. end of the period and as if the funds obtained thereby were used to purchase common stock at
the current market price in effect at the end of the period.
D. end of the period and as if the funds obtained thereby were used to purchase common stock at
the average market price during the period.
E. beginning of the period and as if the funds obtained thereby were used to purchase common stock
at the average market price during the period.

Answer: E

Explanation:
The treasury stock method assumes the exercise of outstanding options and warrants at the
beginning of the period or at time of issuance, if later. It also assumes that the proceeds from the
exercise were used to purchase common stock at the average market price during the period.

Question: 1483

Retained earnings represent:

A. none of these answers.


B. undistributed net income in the last accounting period.

Page | 545
C. accumulated, undistributed earnings since inception.
D. earnings inclusive of any paid-in capital since inception.

Answer: C

Explanation:
Any undistributed net income during a period is swept into the "retained earnings" account. Thus,
retained earnings at any point represent the cumulative undistributed earnings since the company
was formed. This account is the chief source of distribution of dividends and other payments to
shareholders.

Question: 1484

Below is an example of an incorrectly prepared statement of cash flows. The descriptions of activities
are correct.

Cash from operating activities$60,000


Net Income(4,000)
Depreciation(2,000)
Increase in accounts receivable(1,000)
Increase in deferred tax liability$53,000

Cash from investing activities($48,000)


Purchase of marketable securities2,500
Dividends received1,500
Dividends paid($44,000)

Cash from financing activities(500)


Increase in Short-term debt(2,500)
Increase in Long-term debt($3,000)

Increase in cash$6,000

The correct change in cash for the year is ________

A. None of these answers


B. $19,000
C. $15,000
D. $4,000

Answer: B

Explanation:
$60,000 + $4,000 - $2,000 + $1,000 - $48,000 + $2,500 - $1,500 + $500 + $2,500 = $19,000.

Question: 1485

Which of the following is/are true about potentially dilutive securities?

I. Options and warrants are assumed to be exercised at the beginning of the period or at the time of
issuance, whichever is later.
II. Warrants enter into the Diluted EPS calculations only if the maximum stock price during the period
exceeds the exercise price.
III. In Diluted EPS calculations, convertible bonds considered potentially dilutive only if their yield is

Page | 546
less than two-thirds the average yield on Aa bonds.

A. I & II
B. I only
C. I, II & III
D. II & III

Answer: B

Explanation:
In Diluted EPS, all potentially dilutive securities are assumed converted to common stock and
appropriate adjustments made to the income number. If this process serves to increase Diluted EPS,
then the anti-dilutive security is left out of the calculations.

The conversion assumptions for options and warrants are as follows:

1. They are assumed to be exercised at the beginning of the period or at the time of issuance,
whichever is later.
2. They enter into the Diluted EPS calculations only if the average stock price during the period
exceeds the exercise price.

Convertible bonds, regardless of their historical or current yield, are included in Diluted EPS
calculations unless they are anti-dilutive.

Question: 1486

Which of the following are operating cash flows?

I. Interest received
II. Interest paid
III. Dividends received
IV. Dividends paid

A. I, II, III & IV


B. I, II & IV
C. I & III
D. I, II & III

Answer: D

Explanation:
It is very important to remember the following points about dividends and interests:

1. Dividends received from stock investments are considered operating cash flows.
2. Dividends paid on equity are considered financing cash flows.
3. Interest payments on debt are considered operating cash flows.
4. Interest received from debt investments are considered operating cash flows.

Question: 1487

Which of the following is not a current asset?

A. Accumulated Depreciation
B. Allowance for Uncollectible Accounts

Page | 547
C. Notes Receivable
D. Prepaid InsuranceThat answer is correct!

Answer: A

Explanation:
Accumulated Depreciation is a contra account to the fixed asset account(s), which are reported in the
ling-term asset section of the balance sheet.

Question: 1488

Which of the following is not an example of a cash flow from operations?

A. cash paid for merchandise


B. none of these answers
C. cash received from customers
D. cash paid for taxes

Answer: B

Explanation:
Operating activities include the cash effects of transactions and other events that enter into the
determination of net income. All of the responses apply.

Question: 1489

Companies report accounts receivable at:

A. their net realizable value.


B. their liquidation value.
C. the lower-of-cost-or-market value.
D. their fair market value.That answer is correct!

Answer: A

Explanation:
The net realizable value of the accounts receivable is equal to the total amount receivable less an
allowance for uncollectible items. In practice, companies report accounts receivable at their net
realizable value.

Question: 1490

A firm is purchased for more than the fair market value of its assets. The excess is:

A. written off against the retained earnings on the balance sheet.


B. treated as an extraordinary loss & presented net of taxes on the income statement.
C. considered as "Goodwill."
D. considered a "premium paid" and amortized over the life of the acquired assets.

Answer: C

Explanation:
Goodwill is defined as the price paid in excess of the fair market value of the assets of the target firm.
Under US GAAP, only Goodwill generated during acquisitions is capitalized; it does not allow

Page | 548
capitalization of internally generated Goodwill. The capitalized Goodwill is amortized over a period
which can range from a few years to as long as 40 years.

Question: 1491

Which type of revenue recognition method(s) is used when there is uncertainty regarding the
amount or collectibility of future cash flows?

A. percentage-of-completion and installment methods


B. completed contract method
C. installment and cost recovery methods
D. installment method
E. completed contract and cost recovery methods

Answer: C

Explanation:
The installment method recognizes gross profit in proportion to cash collections. The cost recovery
method requires that all cash receipts be first accounted for as a recovery of costs. Only after all costs
are recovered can profit be recognized.

Question: 1492

A loss contingency should be accrued on a company's records only if it is

A. reasonably possible that a liability has been incurred and the amount of the loss is known.
B. remotely probable that a liability has been incurred but the amount of the loss can be reasonably
estimated.
C. probable that a liability has been incurred and the amount of the loss is unknown.
D. none of these answers.
E. probable that a liability has been incurred and the amount of the loss can be reasonably
estimated.

Answer: E

Explanation:
Loss contingencies should be accrued when information available prior to the issuance of financial
statements indicates that it is probable that an asset has been impaired or a liability has been
incurred, and the amount of the loss can be reasonably estimated.

Question: 1493

If an auditor issues an "adverse opinion" qualification in her opinion, she is referring to the fact that:

A. the firm's financial statements do not fairly represent the company's financial performance and
position.
B. there is considerable uncertainty in the firm's asset-liability valuation, thus causing a concern
about its operational health.
C. the firm has inadequate controls in place and needs an on-going, frequent audit.
D. the firm's liquidity is in jeopardy and the firm may not survive for long.That answer is correct!

Answer: A

Explanation:

Page | 549
An adverse opinion is rendered in cases where financial statements are not prepared in accordance
with accepted accounting principles, and this has a material effect on the fair presentation of the
statements.

Question: 1494

As a general rule, revenue is normally recognized when it is ________.

A. realizable
B. measurable and received
C. measurable and earned
D. realizable and earned

Answer: D

Explanation:
Revenue is generally recognized when it is realizable and earned.

Question: 1495

Cricketers, Inc. recently purchased Baseball Corp. for $3 million using debt financing. Analysts are in
agreement that the fair value for Baseball's assets was around $2.7 million and that the overpayment
was for strategic reasons. When Cricketers' accountants record the transaction, they must record:

A. an asset of $2.7 million an equity of $0.3 million and a debt of $2.7 million.
B. an asset of $3.3 million an equity of $0.3 million and a debt of $3 million.
C. an asset of $2.7 million, a goodwill of $0.3 million and a debt of $3 million.
D. an asset of $2.7 million, a premium of $0.3 million and a debt of $3 million.

Answer: C

Explanation:
Goodwill is defined as the price paid in excess of the fair market value of the assets of the target firm.
Under US GAAP, Goodwill can and should be capitalized only when purchased externally (i.e.
internally developed goodwill cannot be capitalized). Therefore, the over-payment of $0.3 million
must be recognized separately as goodwill on the balance sheet of Cricketers, Inc.

Question: 1496

Which of the following would be considered a liability that arises from financing activities?

A. Accounts payable
B. Accrued wages
C. Notes payable
D. Taxes payable
E. None of these answers

Answer: C

Explanation:
Notes payable represents a liability that originates from financing activities. Liabilities that arise from
financing activities typically require compensation in the form of interest. This can be contrasted with
liabilities that arise from operating activities where interest bearing credit is not being extended but
the liability arises from the "normal course of business."

Page | 550
Question: 1497

Termite Terminators has a debt-to-equity ratio of 50%. The equity consists of 80,000 common stock,
valued at the year-end at 65. The preferred equity consists of 30,000 shares of par value 25 and
coupon of 6%. The debt carries a coupon rate of 8%. During the year, Termite earned 1.2 million and
had an average stock price of 75. It also has 40,000 warrants attached to the debt, each exercisable at
68. Termite's reported Diluted EPS should be:

A. 14.64
B. 13.79
C. 15.08
D. 15.00

Answer: B

Explanation:
For Diluted EPS, the average price is used in the treasury stock method. Since the average price of 75
is more than the strike price of 68 of the warrants, the warrants are dilutive for DEPS calculations.
The preferred dividend is 30,000*25*6% = 45,000. The additional shares using the treasury stock
method equal 40,000 - 40,000*68/75 = 3,733. The DEPS then equals (1,200,000-
45,000)/(80,000+3,733) = 13.79.

Question: 1498

Which of the following is/are problems with comparing firm ratios with industry ratios?

I. The firm under consideration may not be a typical firm in the industry.
II. The firm may be located in a homogeneous industry.
III. The firm may be multi-product, multi-industry in nature.
IV. Industry ratios are difficult to calculate.

A. I, II, III & IV


B. I, III & IV
C. I, II & III
D. I & III

Answer: D

Explanation:
II is not a problem, but rather the relationship among firms in an industry with homogenous products
is significant.

Question: 1499

The peaks and valleys of the business cycle tend to be smoothed out using which inventory method?

A. gross profit method


B. LIFO
C. weighted average
D. FIFO

Answer: B

Page | 551
Explanation:
When prices are moving either upward or downward, the cost of goods sold (under LIFO) will show
costs closer to the price level at the time the goods were sold. Therefore the LIFO method tends to
show a smaller net income during inflationary times and a larger net income during deflationary
times than other methods of inventory valuation.

Question: 1500

Which of the following investments in debt securities would not normally be classified as long-term?

A. Held-to-maturity securities
B. Trading securities
C. Available-for-sale securities
D. All of these answers
E. None of these answers

Answer: B

Explanation:
Only held-to-maturity and available-for-sale securities are potential non-current investments.
Trading securities are always considered to be current in nature, whereas held-to-maturity and
available-for- sale securities may be classified as long-term should their maturity, or management's
intent dictate.

Question: 1501

Which of the following best describes an income statement?

A. An income statement reports changes over a period of time in component accounts that comprise
the ownership of a firm.
B. An income statement details the cash inflows and outflows that are related to a company's
operating, investing, and financing activities over a period of time.
C. None of these answers.
D. An income statement summarizes the financial position of a company at a given point in time.
E. An income statement measures a company's financial performance over a specified period of
time.

Answer: E

Explanation:
An income statement measures a company's financial performance between balance sheet dates
and, hence, reflects a period of time. It lists revenues, expenses, gains, and losses of a company over
a time period.

Question: 1502

Which of the following factors would affect the authenticity of a firm's receivables?

A. Industry practices
B. All of these answers
C. A firm's policy for returns and allowances
D. None of these answers
E. A firm's credit policy

Page | 552
Answer: B

Explanation:
Current industry practices will provide clues as to the collectibility of receivables; e.g., a rapidly
deteriorating industry could cause a large percentage of receivables to be uncollectible. A firm could
extend credit very liberally, in which case the quality of the receivables would be suspect.
Additionally, a liberal policy for returns and allowances might cause the value reported on a firm's
balance sheet to be overstated.

Question: 1503

Which of the following best describes an balance sheet?

A. None of these answers.


B. It reports cash receipts and cash disbursements for a specific accounting period.
C. It reports investment activities for a specified accounting period.
D. It reports the amount and composition of assets and liabilities for a specific accounting period.
E. It reports revenues and expenses for a specific accounting period.

Answer: D

Explanation:
A balance sheet provides a detailed listing of a company's assets, liabilities, and equity at a point in
time. This provides a glimpse at a company's financial condition.

Question: 1504

When the FIFO method is used, ending inventory units are priced at the

A. most recent price


B. the average price
C. earliest price
D. none of these answers is correctThat answer is correct!

Answer: A

Explanation:
The first-in-first-out (FIFO) method is based on the assumption that the costs of the first items
acquired should be assigned to the first items sold, therefore ending inventory on hand is based on
the most recent prices.

Question: 1505

Disclosure of information about the extent, nature, and terms of financial instruments with off-
balance- sheet credit or market risk and about concentrations of credit risk is required for all financial
instruments. Which of the following is defined as a financial instrument?

A. Merchandise inventory.
B. A note payable in U.S. Treasury bonds.
C. None of these answers.
D. Deferred subscriptions revenue.
E. A warranty payable.

Answer: B

Page | 553
Explanation:
A financial instrument is cash, evidence of an ownership interest in an entity, or a contract that both:

1. imposes on one entity a contractual obligation to deliver cash or another financial instrument to a
second entity or to exchange financial instruments on potentially unfavorable terms with the second
entity, and
2. conveys to that second entity a contractual right (A) to receive cash or another financial
instrument from the first entity or (B) to exchange other financial instruments on potentially
favorable terms with the first entity. Therefore, a note payable in U.S. Treasury bonds gives the
holder the contractual right to receive and imposes on the issuer the contractual obligation to deliver
bonds that are themselves financial instruments.

Question: 1506

Firm A capitalized an expense and an otherwise identical Firm B expensed it. Then,

A. A shows higher operating cash flow and investing cash flow.


B. A shows higher operating cash flow and lower investing cash flow.
C. A shows lower operating cash flow and investing cash flow.
D. A shows lower operating cash flow and higher investing cash flow.

Answer: B

Explanation:
Since the capitalized expense represents an investing cash outflow, the investing cash flow is lower
for A. Firm B charges all the expenditure to operating cash flow and has a lower operating cash flow
than A.

Question: 1507

Which of the following is/are true about "compensating balances?"

I. They are reserve balances maintained for emergency spending requirements.


II. If compensating balances are legally restricted, they must be segregated on the balance sheet.
III. Compensating balances are overstated if "floats" are included as part of the cash.

A. II only
B. I & III
C. I, II & III
D. II & III

Answer: D

Explanation:
Compensating Balances are cash balances in the form of demand deposits which are used to satisfy
existing borrowing arrangements and assure a good credit standing. They are not reserves
maintained for emergencies. Unrestricted balances are disclosed in footnotes; however, if they are
legally restricted for specific purposes, they must be separately presented in the balance sheet.

Question: 1508

The following asset is subject to the least amount of depreciation or amortization during its useful
life:

Page | 554
A. land
B. warehouse
C. patents
D. computer-based equipmentThat answer is correct!

Answer: A

Explanation:
Land is not subject to any depreciation. The useful life of land does not diminish over time as its
economic value does not diminish based on use.

Question: 1509

Which of the following represents an investing activity in the statement of cash flows ________.

A. stock dividend
B. purchase of inventory
C. depreciation of plant assets
D. sale of plant assets at a loss

Answer: D

Explanation:
The sale of a fixed asset at a loss is classified under investing activities.

Question: 1510

When a plant asset is sold for less than its book value

A. cash received plus accumulated depreciation plus loss on disposal equals the original cost
B. original cost minus accumulated depreciation equals cash received minus loss on disposal
C. book value of the asset plus loss on disposal equals cash received
D. cash received plus accumulated depreciation minus loss on disposal equals the original costThat
answer is correct!

Answer: A

Explanation:
Gain or loss on disposal of a fixed asset is calculated by subtracting the book value (Original cost -
Accumulated Depreciation) from the cash received.

Question: 1511

One way for a company to increase its book value per share is to ________.

A. issue long-term debt


B. increase dividend payout ratio
C. buy back shares at market prices below their book value
D. retire long-term debt

Answer: C

Explanation:

Page | 555
This will of course decrease the shares outstanding, but the $ value at a lower rate, thus increasing
the book value per share.

Question: 1512

An acceptable method for recognizing profit when the collection of cash is in doubt is the ________.

A. installment method
B. percentage-of-completion method
C. consignment method
D. production method
E. completed-contract methodThat answer is correct!

Answer: A

Explanation:
Revenue is recognized when the conditions of "realized" or "realizable" and earned are met. If the
collectibility of assets to be received for products or services is in doubt, revenues may be recognized
when cash is received, as in the installment method.

Question: 1513

Use the following financial data on Enterprise:

a. Sale of equipment$32,000
b. Loss on equipment sale$9,000
c. Dividends paid$12,500
d. Purchase of an office suite$104,000
e. Common stock repurchase$45,000
f. Dividends received from investments$8,500
g. Interest received on Treasury bonds$1,200
h. Supplier accounts paid$3,700
i. Cash collection from customers$14,200
j. Ending cash balance$98,000

In the above question, the investing cash flow is ________.

A. -$145,000
B. -$72,000
C. $12,000
D. -$136,000

Answer: B

Explanation:
Items a and d are the investing cash flows. Note that item b is a non-cash event.

Question: 1514

All of the following are sources of creditor financing except:

A. banks
B. accounts payable.
C. employees who receive pay in arrears.

Page | 556
D. shareholders who receive no dividends.

Answer: D

Explanation:
Shareholders, whether they receive dividends or not, are not considered creditors. Creditors do have
a stake in the success of a company, but they are not owners of the company.

Question: 1515

The allowance for uncollectible accounts is based on all of the following except:

A. Experience.
B. Profitability expectancy.
C. Customer fortunes.
D. Industry expectations.

Answer: B

Explanation:
The allowance should be based on expectations both internal and external to the corporation. The
effect on the bottom line should not be used as a method of determining the uncollectible amounts.

Question: 1516

Which of the following is/are true about Treasury stock?

I. They represent registered shares that have been reacquired from shareholders.
II. Ownership of treasury stock represents part of a firm's assets.
III. Purchase of treasury stock reduces both assets and equity.

A. I & II
B. I, II & III
C. I only
D. I & III

Answer: D

Explanation:
Treasury stock represents the equity that has been repurchased by the company. It should be
remembered that treasury stock is not an asset; rather it is a contra equity account, serving to reduce
the outstanding equity. A compensating reduction in assets also occurs when the firm has to pay cash
to buy-back the shares.

Question: 1517

A Consolidated Statement of Stockholders' Equity could have all of the following, except

A. number of shares of cumulative, callable preferred shares.


B. unrealized gains and losses on securities.
C. the cumulative effect of exchange rate changes.
D. the book value of redeemable preferred stock.
E. the change in reinvested earnings.

Page | 557
Answer: D

Explanation:
Redeemable preferred stock is reported after liabilities but before the equity section of the balance
sheet.

Question: 1518

Discrepancies between the changes in accounts reported on the balance sheet and those reported in
the cash flow statement are due to

A. investment in majority interests.


B. foreign exchange translations.
C. unfunded pension liabilities.
D. prior period adjustments and accounting rule changes.
E. acquisitions and divestitures, and foreign subsidiaries.

Answer: E

Explanation:
Acquisitions, divestitures, and continuing corporate reorganizations can distort trends in both cash
flows from operations and investing cash flows.

Question: 1519

After the initial feasibility studies, all software development costs can be capitalized under US GAAP.
Ultrasoft and Littlesoft are rival firms which are similar in size and scope of operations. Ultrasoft has
decided not to capitalize but expense software development costs in Year

1. Littlesoft, on the other hand, has decided to capitalize a similar amount of development costs, to
be amortized over 5 years. Which of the following is/are true over the next 5 years?

I. Littlesoft will show higher equity than Ultrasoft


II. The difference in Littlesoft's assets and Ultrasoft's assets will be lower in Year 3 than in Year 2.
III. The total tax deductions due to the development costs are equal for the two firms.

A. III only
B. II & III
C. I & II
D. I, II & III

Answer: D

Explanation:
When Littlesoft capitalizes the expenses, its assets increase, net of depreciation and tax expenses.
Hence, it shows higher assets than Ultrasoft (but this difference reduces steadily to zero over the
period over which the expense is amortized). This difference also translates into an equal difference
between the retained earnings as well as equity (Recall assets = liabilities + equity). Thus, over the
next 5 years, Littlesoft will show higher equity than Ultrasoft. The difference will reduce to zero by
the end of the period over which the capitalized expense is amortized. Over 5 years, the total tax
deductions taken by the two firms are equal to the initial expense. Hence, the taxes paid are equal;
the difference is that Ultrasoft delays the payment of taxes by expensing. Lesson: Expensing delays
tax payments, capitalizing hastens them.

Page | 558
Question: 1520

The Jones company had a net receivable balance at the beginning of 1998 of $350,000 and ended
1998 with a receivable balance of $400,000. During the year the company had credit sales of
$1,000,000 and cash sales of $200,000. How much cash did the company collect during 1998?

A. $1,050,000
B. $950,000
C. $1,150,000
D. $1,250,000

Answer: C

Explanation:
During the year the company sold $1,200,000 of goods or services. However, receivables increased
by $50,000, which decreases the change in cash during the year. Therefore the change in cash, or the
cash collected during the year was $1,050,000.

Question: 1521

Which of the following items that are reported on an income statement will directly affect the
retained earnings from one accounting period to another?

A. The sale of new common stock.


B. The payment of cash dividends.
C. The purchase of new equipment with the proceeds of a loan.
D. None of these answers.
E. Accounts payable.

Answer: B

Explanation:
Only the payment of dividends will directly affect the change in a company's retained earnings from
one accounting period to another. The change in retained earnings consists of a company's beginning
retained earnings, plus net income, minus cash dividends paid and the repurchase (not sale) of stock.

Question: 1522

Which of the following is/are true about Goodwill?

I. It represents the amount paid for an acquired firm that cannot be identified with tangible assets.
II. US GAAP prohibits capitalization of Goodwill.
III. IRS does not allow amortization of Goodwill for tax purposes.

A. I & III
B. I & II
C. II & III
D. I, II & IIIThat answer is correct!

Answer: A

Explanation:
When the sum of the fair values of all identifiable assets and liabilities involved in the purchase of
another entity is less than the amount paid, the excess is attributed to "intangibles." This excess is

Page | 559
referred to as "Goodwill" and is capitalized as an asset. Under US GAAP, only Goodwill generated
during acquisitions is capitalized; it does not allow capitalization of internally generated Goodwill

Question: 1523

The formula for calculating profit margin is:

A. net sales minus total expenses divided by net income


B. none of these answers
C. net income divided by net sales
D. total sales divided by total expenses

Answer: C

Explanation:
Profit margin, also called return on net sales, is calculated by dividing net income by net sales. This
ratio measures the average portion of each dollar of revenue that ends up as profit.

Question: 1524

Warranty liabilities:

A. are estimated liabilities


B. must also result in an expense during the same period as the revenue from the sale of the product
C. result when a company sells a product
D. all of these answers

Answer: D

Explanation:
All responses meet the definition of a warranty liability.

Question: 1525

An auditor will express an adverse opinion if

A. the firm' s ability to continue as a going concern is subject to substantial doubt.


B. none of these answers.
C. a qualified opinion cannot be expressed because the auditor lacks independence.
D. a violation of GAAP is sufficiently material that a qualified opinion is not justified.
E. a severe scope limitation has been imposed by the client.

Answer: D

Explanation:
The auditor will express a qualified opinion when the statements are fairly presented in accordance
with GAAP except for the effects of the matter to which the qualification applies. When the
statements as a whole are not fairly presented in conformity with GAAP, a qualified opinion will be
inappropriate, and an adverse opinion must be expressed.

Question: 1526

A firm has a high debt-to-asset ratio. In an inflationary environment, in order to improve this ratio in
earlier years, it will prefer:

Page | 560
A. FIFO inventory valuation and accelerated depreciation.
B. FIFO inventory valuation and straight-line depreciation.
C. LIFO inventory valuation and straight-line depreciation.
D. LIFO inventory valuation and accelerated depreciation.

Answer: B

Explanation:
To improve the ratio, it uses FIFO and straight-line depreciation to increase income and hence,
retained earnings. FIFO also increases current and total assets in inflationary conditions, improving
the debt-to-asset ratio.

Question: 1527

The statement of cash flows includes sections on all of the following except ________.

A. financing activities
B. purchasing activities
C. investing activities
D. operating activities

Answer: B

Explanation:
Operating, investing and financing are the 3 activities depicted on that statement of cash flows.

Question: 1528

Which of the following revenue recognition methods would generally be considered the most
conservative?

A. order receipt method


B. installment method
C. completion of production
D. cost recovery method

Answer: D

Explanation:
Revenue is generally recognized when it is realizable and earned.

Question: 1529

Treasury stock is

A. assets held for safekeeping in company's vaults.


B. a company's own stock that it has repurchased.
C. retained earnings that have been appropriated to make equity investments.
D. investments in government securities.

Answer: B

Explanation:

Page | 561
Treasury stock is not an asset. It represents shares of a company's stock reacquired after having been
previously issued and fully paid.

Question: 1530

Which of the following best qualifies as a "cash equivalent?"

A. A firm's investment in "held to maturity" U.S. treasury bonds that mature in 5 years.
B. All of these answers.
C. A firm's equity investment in an unconsolidated subsidiary of a privately held firm.
D. A firm's investment in U.S. treasury bills.
E. None of these answers.

Answer: D

Explanation:
Only a firm's investment in U.S. treasury bills is correct because it is readily converted to cash. U.S.
treasury bills have a maximum maturity of six months.

Question: 1531

A long-term asset currently has a book value of $45,000 and a salvage value of $5,000. It was
acquired 3 years ago at a cost of $75,000. If the firm uses straight-line depreciation, how many years
is the asset expected to be in service?

A. 5 years
B. 4 years
C. 6 years
D. 7 years

Answer: B

Explanation:
In Straight-line method, depreciation = (1/n)*(acquisition cost - salvage value). The accumulated
depreciation in 3 years = 75,000 - 45,000 = $30,000. Hence, per year depreciation equals $10,000 =
(1/n)*(75,000-5,000). Thus, n = 7 years, implying the asset will be in service for another 4 years.

Question: 1532

Unearned Revenue is classified as

A. a liability
B. a contra account
C. an operating expense
D. revenueThat answer is correct!

Answer: A

Explanation:
Unearned revenue represents "services or product owed" and is reported as a liability on the balance
sheet.

Question: 1533

Page | 562
In 1998, firm A's financial statements showed the following:

Net income = $23,000


Operating cash flows = $10,000
Financing cash flows = $24,000
Non-cash expenses = $19,000
Change in cash accounts = $6,500
Cash spent on acquiring operating capacity = $19,000

Then, the investing cash flow of the firm in 1998 was ________.

A. -$27,500
B. $42,000
C. -$19,000
D. $69,500That answer is correct!

Answer: A

Explanation:
Operating + Financing + Investing cash flows = change in cash accounts. Hence, Investing cash flow =
6,500-10,000-24,000 = -$27,500.

Question: 1534

Under accrual accounting, revenues are recognized when:

I. Sales are made.


II. The earnings process is complete.
III. The realization of payment is reasonably assured.
IV. Cash is received from outside sources.

A. I, II, & III


B. I & IV
C. I & III
D. II & III.

Answer: D

Explanation:
Both are required for earnings to be recognized. IV is not true under accrual accounting and I is
incorrect since even after a sale is made, realization of payment may be doubtful or there may be a
high probability of merchandise return.

Question: 1535

Swiss Inc. had the following activities during 1996:

Acquired 2,000 shares of stock in Molly Inc. for $26,000.

Sold an investment in General Motors for $35,000 when the carrying value was $33,000.

Acquired a $50,000, 4-year certificate of deposit from a bank. (During the year, interest of $3,750 was
paid to Swiss.)

Page | 563
Collected dividends of $1,200 on stock investments.

In Swiss's 1996 statement of cash flows, net cash used in investing activities should be ________.

A. $37,250
B. $76,000
C. $41,000
D. $39,800
E. $38,050

Answer: C

Explanation:
Investing activities include the acquisition and disposal of all long-term assets. Thus, the purchase of
shares, sale of shares held as an investment, acquisition of long-term certificate of deposit (not a
cash equivalent since long-term) are classified as investing activities and total $41,000 used (-
$26,000 +35,000 - 50,000). The receipts of dividends and interest are cash flows from operating
activities.

Question: 1536

The equity turnover of a firm equals:

A. asset turnover times financial leverage.


B. asset turnover times financial leverage and sales to equity ratio.
C. net income to equity ratio.
D. sales to equity ratio.

Answer: B

Explanation:
Equity Turnover = sales/equity = (sales/total assets)*(total assets/equity)= (asset turnover)*(financial
leverage)

Question: 1537

Warranty liabilities:

A. result when a company sells a product


B. must also result in an expense during the same period as the revenue from the sale of the product
C. all of these answers are correct
D. are estimated liabilities

Answer: C

Explanation:
All responses meet the definition of a warranty liability.

Question: 1538

Which of the following is not a theoretical basis for the allocation of expenses?

A. Profit maximization
B. Systematic allocation

Page | 564
C. None of these answers
D. Cause and effect
E. Immediate recognitionThat answer is correct!

Answer: A

Explanation:
Profit maximization means that the goal is not to report costs as expenses by hiding them on the
balance sheet. It is not an acceptable basis as it goes against the purpose of generally accepted
accounting methods which is to fairly present the results of operations and financial position.

Question: 1539

Below is an example of an incorrectly prepared statement of cash flows. The descriptions of activities
are correct.

Cash from operating activities$60,000


Net Income(4,000)
Depreciation(2,000)
Increase in accounts receivable(1,000)
Increase in deferred tax liability$53,000

Cash from investing activities($48,000)


Purchase of marketable securities2,500
Dividends received1,500
Dividends paid($44,000)

Cash from financing activities(500)


Increase in Short-term debt(2,500)
Increase in Long-term debt($3,000)

Increase in cash$6,000

The correct Cash flows from investing activities is ________.

A. ($45,500)
B. ($41,000)
C. None of these answers
D. ($48,000)

Answer: D

Explanation:
The dividends received is an operating activity and the dividend paid is a financing activity.

Question: 1540

In 1994, Butler Inc. issued $10 par value common stock for $25 per share. No other common stock
transactions occurred until March 31, 1996, when Butler acquired some of the issued shares for $20
per share and retired them. Which of the following statements correctly states an effect of this
acquisition and retirement?

A. Retained earnings is increased.


B. Additional paid-in capital is decreased.

Page | 565
C. Additional paid-in capital is increased.
D. 1996 net income is increased.
E. 1996 net income is decreased.

Answer: B

Explanation:
When common shares are reacquired and retired, common stock par value is decreased by $10 per
share and additional paid-in capital is decreased by $15 per share, cash is decreased by $20 per
share, and $5 per share increases additional paid-in capital. Accounting rules do not allow a firm to
record a gain or loss when it buys back treasury stock because the transaction is viewed as an equity
transaction. Therefore, the effect is to decrease additional paid-in capital.

Question: 1541

A liability can be recognized when

A. an obligation exists to make a future payment based on a past event


B. all of these answers
C. only when the amount is certain
D. any time a future payment is dueThat answer is correct!

Answer: A

Explanation:
The amount need not be certain and some future payments such as wages to be paid in the future
may not be reported as liabilities.

Question: 1542

A firm had an operating cash flow of 4,500, financing cash flow of 1,200 and an investing cash flow of
-900. The firm has announced common stock dividends of 45 and preferred stock dividends of 23. If
the outlays for maintenance of current capacity is 310, the firm's free cash flow equals ________.

A. 4,445
B. 5,345
C. 4,122
D. 4,145

Answer: C

Explanation:
FCF = Cash from operations - outlays for maintenance of current capacity - dividends declared. In this
case, FCF = 4,500 - 310 - 45 - 23 = 4,122.

Question: 1543

A firm using LIFO accounting has a LIFO reserve of 500, with a LIFO ending inventory of 2,300. It is
currently in the 35% tax bracket. If it switches to FIFO accounting, its ending inventory would be
reported at ________.

A. 1,305
B. 2,800
C. 1,800

Page | 566
D. 1,805

Answer: B

Explanation:
LIFO Reserve = FIFO Ending inventory value - LIFO Ending inventory value Therefore, 500 = FIFO
Ending inventory value - 2,300, giving the value of the ending inventory under FIFO = 2,800.

Question: 1544

The following financial data on CashCow, Inc. have been taken from its financial statements for 1996:

a. Dividends paid$25,000
b. Sale of land$64,000
c. Inventory purchases$29,000
d. Purchase of a warehouse$208,000
e. Bonds issued$90,000
f. Dividends received from investments$17,000
g. Interest paid on bonds$2,400
h. Salaries paid$107,400
i. Cash collection from customers$28,400
j. Loss on land sale$18,000
k. Beginning cash balance$312,000

In the above question, the financing cash flow is ________.

A. $79,600
B. $90,000
C. $82,000
D. $65,000

Answer: D

Explanation:
Items a and e are the financing cash flows.

Question: 1545

A defined contribution pension plan:

A. disburses benefits based on the returns on the fund's investments.


B. none of these answers.
C. promises to pay retirees a specific income stream.
D. pays defined benefits for a certain period after retirement.That answer is correct!

Answer: A

Explanation:
In a defined benefit contribution plan, the employee makes definite contributions to the pension
plan from the regular salary. Investment of these contributions is at the discretion of the employee
and the size of the benefits paid out depends upon the total contributions made and the fund
performance. Thus, in this plan, the risk of pension plan performance is borne by the employee and
not the employer.

Page | 567
Question: 1546

Which of the following would not normally be found in the footnotes that accompany financial
statements?

A. significant legal proceedings


B. list of officers and directors of company
C. details of business acquisitions
D. significant accounting principles used

Answer: B

Question: 1547

Proctor Ltd. sells major household appliance service contracts for cash. The service contracts are for a
1-year, 2-year, or 3-year period. Cash receipts from contracts are recorded as unearned contract
revenues. This account had a balance of $720,000 at December 31, 1996. Service contract costs are
charged as incurred to the service contract expense account, which had a balance of $180,000 at
December 31, 1996. Outstanding service contracts at December 31, 1996 expire as follows:

During 1997$150,000
During 1998$225,000
During 1999$100,000

What amount should be reported as unearned service contract revenues in Proctor's December 31,
1996 balance sheet?

A. $475,000
B. None of these answers
C. $295,000
D. $245,000
E. $540,000That answer is correct!

Answer: A

Explanation:
Unearned service contract revenues relate to outstanding contracts for which the agreed service has
not yet been provided. Thus, the amount to be reported as unearned service contract revenues
covers all contracts that are outstanding at December 31, 1996 which totals $475,000 (150,000 +
225,000 + 100,000)

Question: 1548

A firm follows LIFO accounting. Its inventory accounts for a period showed the following sequential
accounts:

Beginning inventory of 50 units, purchased at $5


50 units purchased at $10
35 units purchased at $9
25 units sold at $15
70 units sold at $12
Tax rate = 40%
Beginning LIFO reserve = $300

Page | 568
Then, the net income of the firm using LIFO is ________.

A. $285
B. $210
C. $198
D. $350

Answer: B

Explanation:
The total revenue equals 25*15 + 70*12 = 1,215, coming from a sale of 95 units. Under LIFO, the
goods purchased last are assumed to have been sold first. Therefore, 35 of the sold units have a cost
of $9 each, 50 units have a cost of $10 each and the remaining 10 have a cost of $5 each. Therefore,
LIFO COGS = 35*9 + 50*10 + 10*5 = $865. Therefore, assuming no other costs, the pre-tax income
equals 1,215 - 865 = $350 and the post tax income equals 350*(1-40%) = $210.

Question: 1549

Which of the following is considered a characteristic of a liability?

A. A liability represents a claim against a company's present and future assets and resources.
B. A liability requires the rendering of future services.
C. All of these answers.
D. A liability requires the payment of money.
E. A liability represents a claim that is usually senior to the holders of equity securities.

Answer: C

Explanation:
By definition, a liability is a financial obligation that requires the payment of money, or the rendering
of future services. It is a claim against the business that must be satisfied in one way or another.
Lastly, liabilities are claims that have a higher level of priority than equity as it relates to the
distribution of income, or assets.

Question: 1550

The formula for calculating profit margin is

A. total sales divided by total expenses


B. net sales minus total expenses divided by net income
C. net income divided by net sales
D. none of these answers is correct

Answer: C

Explanation:
Profit margin, also called return on net sales, is calculated by dividing net income by net sales. This
ratio measures the average portion of each dollar of revenue that ends up as profit.

Question: 1551

"Deferral" refers to:

A. a postponement of recognition of income until the earnings process is actually completed.

Page | 569
B. none of these answers.
C. recognition of an asset for all payments made for goods and services not yet delivered.
D. postponement of tax payments through the use of tax credits.That answer is correct!

Answer: A

Explanation:
"Deferral" is the accounting process of recognizing a liability when payments are made in advance of
the earnings process. Over time, as the earnings process proceeds, income is recognized and
liabilities are decreased.

Question: 1552

The following information was taken from the accounting records of Boston Corp. for the year ended
December 31, 1996 (in $000):

Proceeds from issuance of preferred stock F$4,000


Dividends paid on preferred stock F400
Bonds payable converted to common stock NC2,000
Payment for purchase of machinery I500
Proceeds from sale of plant building I1,200
2% stock dividend on common stock NC300
Gain on sale of plant building I200

The net cash flows from investing and financing activities that should be presented on Boston's
statement of cash flows for the year ended December 31, 1996 are, respectively (in $000) ________.

A. $700 and $3,600


B. $700 and $3,900
C. $700 and $4,000
D. $900 and $3,900
E. $900 and $3,600That answer is correct!

Answer: A

Explanation:
Investing activities include the acquisition and disposal of all long-term assets. Thus, the purchase of
machinery and sale of plant building for a total of $700 inflow from investing (-$500 + 1,200).
Financing activities include the issuance and repurchase of shares, dividends and changes in long-
term liabilities. In this case, the issuance of preferred stock and payment of dividends for a total of
$3,600 inflow from financing ($4,000-400). The conversion of bonds into common stock and the stock
dividend do not affect cash.

Question: 1553

Under GAAP, which of the following costs can be capitalized?

A. drilling costs for oil wells


B. public relations expense to develop goodwill
C. research and development
D. none of these answersThat answer is correct!

Answer: A

Page | 570
Explanation:
Drilling costs can be either expensed under Successful Efforts method or capitalized under the Full
Cost method. R&D costs cannot be capitalized unless the costs are incurred on assets that have
alternative uses. Similarly, internally developed Goodwill cannot be capitalized in the form of an
asset. Only if it is purchased during an acquisition can it appear on the balance sheet in a capitalized
form.

Question: 1554

A piece of equipment costs $250,000, has a useful life of 10 years and an estimated salvage value of
$50,000. How much depreciation expense should the company recognize in year 7 if it is using the
straight-line method of depreciation?

A. $10,000
B. $25,000
C. $140,000
D. $20,000

Answer: D

Explanation:
Using the straight line method an equal amount of the cost of the asset less the salvage value is
charged to each year of the assets useful life. In this example the value of the equipment being used
during its life is $200,000 divided evenly among 10 years.

Question: 1555

"Book value per share" refers to:

A. the per share value of the company's assets less the liabilities.
B. all of these answers.
C. the per share value of total equity of the firm.
D. the per share value of the common stock accounts of a firm.

Answer: D

Explanation:
The book value represents the company's liquidation or residual value at the amounts reported on
the balance sheet. As such, it represents the "balance sheet" value of the common equity. Since total
equity can include senior securities like preferred equity, "the per share value of the company's
assets less the liabilities" and "the per share value of total equity of the firm" are incorrect.

Question: 1556

Which accounting principle is consistent with reporting financial results that can be compared with
previous periods?

A. matching
B. adequate disclosure
C. materiality
D. consistency

Answer: D

Page | 571
Explanation:
The consistency principle requires that an accounting procedure, once adopted by a company,
remain in use from one period to the next unless users are informed of the change.

Question: 1557

Contingent losses are generally recognized when they are ________.

A. measurable
B. probable and measurable
C. measurable and realized
D. probable

Answer: B

Explanation:
Contingent losses are generally recognized when they are both probable and measurable.

Question: 1558

Which of the following is/are true about available-for-sale securities?

I. They are current assets.


II. They are reported at fair market value.
III. Changes in their reported value are allocated directly to retained earnings.

A. I & II
B. II only
C. II & III
D. I, II & III

Answer: D

Explanation:
Marketable securities classified as "Available-for-sale" are financial securities which management
does not intend to hold to maturity and may or may not sell in the near future. They may be
categorized as either current or non-current assets, depending on the management's projected
horizon over which they will be sold. They are reported on the balance sheet at the fair market value
and changes in their values are reported as a separate component of shareholder equity.

Question: 1559

Peterborough's capital structure was as follows:

Dec. 31, 1995 Dec. 31, 1996


Outstanding shares of stock:
Common$110,000$110,000
Convertible preferred$10,000$10,000
8% convertible bonds$1,000,000$1,000,000

During 1996, Peterborough paid dividends of $3.00 per share on its preferred stock. The preferred
shares are convertible into 20,000 common shares and are considered common stock equivalents.
The 8% bonds are convertible into 30,000 common shares but are not considered common stock
equivalents. Net income for 1996 was $850,000. Assume that the income tax rate is 30%. The

Page | 572
primary earnings per share for 1996 is ________.

A. $7.45
B. $6.31
C. $6.54
D. $7.73
E. $7.08

Answer: C

Explanation:
As common stock equivalent (CSE), the preferred stock must be considered in the calculation of
primary earnings per share (PEPS) and assumed to have no preferred dividends paid. PEPS is then
$6.54 ($850,000/ (110,000 + 20,000)).

Question: 1560

Which of the following would not be included as an asset on a corporate balance sheet?

A. Inventory
B. Marketable securities
C. Buildings
D. Accounts receivable
E. Common stock

Answer: E

Explanation:
Common stock. Common stock is not as asset; it represents funds received from the sale of stock to
owners of the firm. It is ownership equity, not an asset.

Question: 1561

Which of the following groups are least likely to be creditors of a company?

A. suppliers
B. investors
C. banks
D. customers

Answer: D

Explanation:
The customers that buy on credit owe the company; they are not creditors of the company unless the
company owes them.

Question: 1562

A useful tool in financial statement analysis is the common-size financial statement. What does this
tool enable the financial analyst to do?

A. Determine which companies in the same industry are at approximately the same stage of
development.
B. Evaluate financial statements of companies within a given industry of approximately the same

Page | 573
value.
C. Ascertain the relative potential of companies of similar size in different industries.
D. Compare the mix of assets, liabilities, capital, revenue, and expenses within a given industry
without respect to relative size.
E. None of these answers.

Answer: D

Explanation:
Common-size financial statements present items in a financial statement as percentages of a
common base. Comparison among firms in the same industry is made possible despite differences in
size. Comparison of firms in different industries has drawbacks because the optimum mix of assets,
liabilities, etc. will vary from industry to industry.

Question: 1563

The following data have been extracted from the financial statements of a firm for two years, 1995
and 1996:

19951996
Assets8,1748,439
Net Sales9,4128,949
Receivables1,135986
Inventory2,1192,464
COGS6,9487,089

The receivables turnover ratio and the average receivables collection period for 1996 equal
________.

A. 8.44, 43.25 days


B. 9.07, 40.22 days
C. 7.88, 46.29 days
D. none of these answersThat answer is correct!

Answer: A

Explanation:
Receivables turnover ratio = Net annual sales/average receivables Average receivables collection
period = 365/receivables turnover. For 1996, the average receivables equal (986+1135)/2 = 1061.
Receivables turnover ratio = 8949/1061 = 8.44. Average receivables collection period = 365/8.44 =
43.25 days.

Question: 1564

Purchases of marketable securities are:

A. operating cash flows.


B. investing cash flows.
C. financing cash flows.
D. either financing cash flows or investing cash flows.

Answer: B

Explanation:

Page | 574
The purchase of investment securities is clearly an investing cash flow.

Question: 1565

Which of the following is NOT involved in calculating the operating cash flow from net income from
operations?

A. Increase in current assets in the form of receivables.


B. Increase in current liabilities in the form of payables.
C. Gain from retirement of debt.
D. None of these answers.

Answer: C

Explanation:
Gain from retirement of debt is an extraordinary item under GAAP and is not a part of operating cash
flow. Indeed, the gain is a non-cash item.

Question: 1566

The following financial data on CashCow, Inc. have been taken from its financial statements for 1996:

a. Dividends paid$25,000
b. Sale of land$64,000
c. Inventory purchases$29,000
d. Purchase of a warehouse$208,000
e. Bonds issued$90,000
f. Dividends received from investments$17,000
g. Interest paid on bonds$2,400
h. Salaries paid$107,400
i. Cash collection from customers$28,400
j. Loss on land sale$18,000
k. Beginning cash balance$312,000

The operating cash flow for 1998 was ________.

A. -$101,200
B. -$75,400
C. -$111,400
D. -$93,400

Answer: D

Explanation:
Items c, f, g, h, i are operating events.

Question: 1567

Marine Corp. uses the direct method to prepare its statement of cash flows. Marine's trial balance at
December 31, 1996 and 1995 are as follows:

Dec. 31, 1996 Dec. 31, 1995


Debits:

Page | 575
Cash$35,000$32,000
Accounts receivable33,00030,000
Inventory31,00047,000
Property, plant & equipment100,00095,000
Unamortized bond discount4,5005,000
Cost of goods sold250,000380,000
Selling expenses141,500172,000
General and administrative expenses137,000151,300
Interest expense4,3002,600
Income tax expense20,40061,200
Total debits$756,700$976,100
Credits:
Allowance for doubtful accounts$1,300$1,100
Accumulated depreciation16,50015,000
Trade accounts payable25,00017,500
Income taxes payable21,00027,100
Deferred income taxes5,3004,600
8% callable bonds payable45,00020,000
Common stock50,00040,000
Additional paid-in capital9,1007,500
Retained earnings44,70064,600
Sales538,800778,700
Total credits$756,700$976,100

Marine purchased $5,000 in equipment during 1996. Marine allocated one-third of its depreciation
expense to selling expenses and the remainder to general and administrative expenses. What
amount should Marine report in its statement of cash flows for the year ended December 31, 1996
for cash collected from customers?

A. $535,800
B. $601,800
C. $536,000
D. $541,600
E. $541,800That answer is correct!

Answer: A

Explanation:
To calculate cash collected from customers we do a T-account analysis of the gross accounts
receivable. The beginning balance was $30,000 debit, we add the sales for 1996 (by debiting the
accounts receivable) which were $538,800 and we subtract cash collections (the unknown here) by
crediting accounts receivable to derive the ending balance of $33,000. Therefore, cash collections are
$535,800.

Question: 1568

At the end of the fiscal period, the account debited to show the estimated amount of uncollectible
accounts is:

A. Bad Debt Expense


B. Accounts Receivable
C. None of these answers
D. Unearned Revenue
E. Allowance for Uncollectible AccountsThat answer is correct!

Page | 576
Answer: A

Explanation:
The adjusting entry should recognize an expense and increase the allowance account.

Question: 1569

A firm's tax rate is 40%. If its pretax income is overstated by 100 due to errors in recording cash
transactions, then the operating cash flow due to these errors is:

A. overstated by 40.
B. overstated by 100.
C. overstated by 60.
D. understated by 60.

Answer: C

Explanation:
operating cash flow = net income + noncash expenses - non-cash revenues - cash reductions in
operating accounts Since the pretax income is overstated by 100, net income is overstated by 100*(1-
0.4) = 60. Since the errors are in the cash transaction accounts, not non-cash accounts, this implies
that operating cash flow is overstated by 60 due to these errors.

Question: 1570

Beginning inventory of 50 units, purchased at $5


50 units purchased at $10
35 units purchased at $9
25 units sold at $15
70 units sold at $12
Tax rate = 40%.
Beginning LIFO reserve = $300

The net income using FIFO is ________.

A. $210
B. $427
C. $309
D. $515

Answer: C

Explanation:
The total revenue equals 25*15 + 70*12 = 1,215, coming from a sale of 95 units. Under FIFO, the
goods purchased earliest are assumed to have been sold first. Therefore, 50 of the sold units have a
cost of $5 each and 45 units have a cost of $10 each. Therefore, FIFO COGS = 50*5 + 45*10 = $700.
Therefore, assuming no other costs, the pre-tax income equals 1,215 - 700 = $515 and the post tax
income equals 515*(1-40%) = $309.

Question: 1571

Examples of Common Stock Equivalents (CSE's) include all of the following except:

Page | 577
A. stock options.
B. treasury stock.
C. warrants.
D. convertible preferred stocks that have an effective yield of less than two-thirds of the average Aa
corporate bond yield at the time of issuance.

Answer: B

Explanation:
Treasury stock is stock held by the company and therefore is not included as a CSE. The earnings of
the shares held by investors would not be affected by the treasury stock.

Question: 1572

A firm decides to capitalize the interest expenditure on a large, building construction project, instead
of expensing it. This ________ its debt-to-equity ratio.

A. has an ambiguous effect on


B. leaves unaffected
C. decreases
D. increases

Answer: C

Explanation:
Interest capitalization results in higher earnings compared to the case where it is expensed. This
results in higher retained earnings and higher equity, thus reducing the debt-to-equity ratio.

Question: 1573

A firm's gross profit margin equals 35.2% and the operating profit margin equals 26.7%. If its net
sales equal 6,128, the firm's SGA expenses equal ________.

A. 448
B. 521
C. 496
D. 357

Answer: B

Explanation:
Gross Profit Margin = Gross Profit/Net sales and Operating Profit Margin = Operating Profit/Net sales.
Therefore, Gross Profit = 6,128*0.352 = 2,157 and Operating Profit = 6,128*0.267 = 1,636. Now,
Operating profit = income before interest, depreciation and taxes (EBDIT)= Gross Profit - Sales &
General Expenses, so 2,157 - 1,636 = 521.

Question: 1574

When the FIFO method is used, ending inventory units are priced at the:

A. none of these answers


B. most recent price
C. the average price
D. earliest price

Page | 578
Answer: B

Explanation:
The first-in-first-out (FIFO) method is based on the assumption that the costs of the first items
acquired should be assigned to the first items sold, therefore ending inventory on hand is based on
the most recent prices.

Question: 1575

The following account is affected when recording a return of inventory to the vendor under a
perpetual inventory system:

A. merchandise inventory
B. cash
C. accounts receivable
D. purchase returns and allowancesThat answer is correct!

Answer: A

Explanation:
When inventory is purchased, Merchandise Inventory is debited, and credited when returned. The
periodic system records purchases in an account called Purchases and returns are booked to
Purchases Returns and Allowances.

Question: 1576

Property, Plant & Equipment has all of the following characteristics except:

A. They are intended for use in operating activities, and are not acquired for sale in the ordinary
course of business.
B. They are classified as noncurrent tangible assets
C. Their service potential diminishes with use.
D. They don't typically make up a large part of a corporation's operating assets.

Answer: D

Explanation:
They are typically the largest of the operating assets. Included in this category would be heavy
equipment and machinery as well as real estate and buildings used in the business.

Question: 1577

Which of the following is/are true about turnover ratios?

I. A high receivables turnover ratio might imply overly lax credit policy.
II. A low inventory ratio implies that capital might be locked up for a long time in excess inventory.
III. A high payables turnover ratio implies that the firm is lagging behind in its credit payments.

A. II only
B. I, II & III
C. I & II
D. I & IIIThat answer is correct!

Page | 579
Answer: A

Explanation:
The receivables turnover is the ratio of net sales to average receivables. If this ratio is high, then it
indicates that the receivables are relatively low compared to sales. This could happen if the credit
policy is stringent, forcing fewer sales on credit. The inventory turnover is the ratio of net sales to
average inventory. A low value for this ratio indicates that the firm is maintaining a high level of
inventory to maintain the sales. This might be an indicator (though not necessarily) that capital is
being locked up in excess inventory. The payables turnover ratio is the ratio of the cost of goods sold
to average trade payables. A high ratio implies that the payables are not very high compared to the
COGS. This will happen when the firm is paying off its payables debt at a quick pace.

Question: 1578

The "Management Discussion and Analysis" section of the stockholder's report need not contain
which of the following?

I. Analysis of cash flows and liquidity.


II. Discussion of unusual events like discontinued operations.
III. Future outlook based on current and past events.
IV. Firm's ability to maintain its dividend policy while sustaining growth.

A. III & IV
B. III only
C. I & II
D. IV onlyThat answer is correct!

Answer: A

Explanation:
Companies are encouraged, but not required, to provide forward-looking information i.e. they need
not extrapolate past trends into the future to draw conclusions about the firm's prospects.

Question: 1579

Which of the following is/are true about straight-line depreciation?

I. With constant pre-depreciation income, it leads to an increasing rate of return over time.
II. It ignores loss of productivity and increased maintenance costs over time.
III. It leads to higher taxes in later years compared to accelerated depreciation methods.

A. I, II & III
B. I & III
C. II & III
D. I & II

Answer: D

Explanation:
With accelerated depreciation methods, the depreciation expense gets lower than that in the case of
straight line depreciation, leading to higher taxes in the later years. One of the mathematical facts
about straight-line and accelerated depreciation methods is that even when the pre-depreciation
income stays constant, the rate of return steadily increases over time purely because of the decrease
in asset value.

Page | 580
Question: 1580

Which of the following statements reflects the basic purposes of financial reporting?

A. The best indication of a firm's ability to generate favorable cash flow is information based on
previous cash receipts and payments.
B. Financial accounting is expressly designed to measure directly the value of a business enterprise.
C. None of these answers.
D. The primary focus of financial reporting is information about an enterprise's resources.
E. Investments and credit decisions often are based, at least in part, on evaluations of the past
performance of an enterprise.

Answer: E

Explanation:
Although investment and credit decisions reflect investors' and creditors' expectations about future
enterprise performance, those expectations are commonly based, at least in part, on evaluations of
past enterprise performance.

Question: 1581

Metals Inc. had the following liabilities at December 31, 1996:

Accounts payable$55,000
Unsecured notes, 8%, due 7/1/97$400,000
Accrued expenses$35,000
Contingent liability$450,000
Deferred income tax liability$25,000
Senior bonds, 7%, due 3/31/97$1,000,000

The contingent liability is an accrual for possible losses on a $1,000,000 lawsuit filed against Metals
Inc. Metals' legal counsel expects the suit to be settled in 1998 and has estimated that Metals will be
liable for damages in the range of $450,000 to $750,000. The deferred income tax liability is not
related to an asset for financial reporting and is expected to reverse in 1998. What amount should
Metals report in its December 31, 1996 balance sheet for current liabilities?

A. None of these answers


B. $1,490,000
C. $940,000
D. $515,000
E. $1,515,000

Answer: B

Explanation:
A current liability is an obligation that will be liquidated within 1 year. Therefore, we need to include
accounts payable, unsecured notes, accrued expenses, and the senior debt which total all together
$1,490,000.

Question: 1582

The accrual basis of accounting

Page | 581
A. does not pertain to revenue recognition-only expense recognition.
B. recognizes the development of assets and liabilities externally.
C. is used only for taxes reporting purposes.
D. begins with the cash basis of accounting and proceeds to make the necessary adjustments.
E. allocates many transactions that produce cash flows to time periods other than those in which the
cash flows occur.

Answer: E

Explanation:
Accrual accounting principles are the decision rules that state when to recognize the revenue and
expense consequences of cash flows and other events.

Question: 1583

The primary purpose of the statement of cash flows is to

A. state the company's financial position at period-end


B. measure the change in the company's assets
C. provide information about a company's cash receipts and cash payments during the accounting
period
D. analyze net income during the accounting period

Answer: C

Explanation:
Other financial statements are used to analyze revenues, expenses, assets, liabilities, and
stockholders' equity. The statement of cash flows summarizes transactions affecting cash; it is the
balance sheet that states the company's financial position at a point in time.

Question: 1584

Which of the following best describes a firm's book value of its common stock?

A. The amount of retained earnings of a firm minus any authorized but unissued preferred stock.
B. None of these answers.
C. All of these answers.
D. The total number of common shares of a firm that are outstanding multiplied by the present price
that the firm's stock is selling for.
E. The total assets of a firm less liabilities and any claims senior to common stock that is reported on
a firm's balance sheet.

Answer: E

Explanation:
The book value of a firm's common stock is represented by the common shareholders ownership
interest in the firm. This consists of the value that the firm's assets are reported on the firm's balance
sheet minus the liabilities of the firm and any "equity" claims that are senior to the common
stockholders, such as preferred stock.

Question: 1585

The allowance for uncollectible accounts is

Page | 582
A. a reduction in revenue
B. a liability
C. long-term asset
D. a contra account

Answer: D

Explanation:
Allowance for Uncollectible Accounts, sometimes referred to as Bed Debt Reserve, is a contra
account to Accounts Receivable.

Question: 1586

Float refers to:

A. the number of days that a bank will allow a corporation to hold a negative balance in its checking
account before charging fees for the negative balance.
B. the companies bank balance in excess of its working capital needs.
C. the receivable balance on the books of the corporation.
D. checks issued but not yet paid by a bank.

Answer: D

Explanation:
Companies, specifically those that are strapped for cash, will try and play the float by having a
smaller balance in the bank than the level of outstanding checks that they have written. While it can
certainly ease cash shortfalls, it can be troublesome when more checks clear than expected.

Question: 1587

Each of the following transactions has the effect of reducing retained earnings except ________.

A. cash dividends
B. stock splits
C. stock dividends
D. dividends in kind

Answer: B

Explanation:
Stock splits have no effect on retained earnings. They are simply a redistribution of the same equity
of the company but with a different number of outstanding shares.

Question: 1588

Trading securities are carried on the books at ________.

A. current market value


B. acquisition cost or current market value
C. acquisition cost
D. lower-of-cost-or-marketThat answer is correct!

Answer: A

Page | 583
Explanation:
Marketable securities are classified into various categories, one of which is "Trading securities." As
the name implies, these financial securities are purchased with the intent to sell in the near future.
They are therefore categorized as current assets and reported at the fair market value on the date of
the balance sheet

Question: 1589

The following information should be used according to the provisions of SFAS 95 (Statement of Cash
flows) and using the following data.

Net Income$50,000
Provision for bad debts$2,000
Increase in Inventory$1,000
Increase in accounts payable$2,000
Purchase of new equipment$15,000
Sale of equipment for $10,000 gain$20,000
Depreciation expense$5,000
Repurchase of common stock$10,000
Payment of dividend$4,000
Interest payment$3,000

What is net cash flow from operations?

A. $55,000
B. $48,000
C. $54,000
D. $58,000

Answer: B

Explanation:
$50,000 + $2,000 - $1,000 + $5,000 + $2,000 - $10,000

Question: 1590

In a period of rising prices, the inventory method that gives the highest possible value for ending
inventory is:

A. weighted average
B. FIFO
C. LIFO
D. gross profit

Answer: B

Explanation:
The ending inventory under FIFO is priced at the most recent, and thus the highest prices (in a period
of rising prices) than any of the other methods.

Question: 1591

When a marketable security's classification is changed, which of the following apply?

Page | 584
I. The transfer is accounted for at the amortized historical cost of the security.
II. The change in the fair value of the security is reported as part of the income statement.
III. The switch in classification is at management's discretion.

A. I & II
B. I, II & III
C. III only
D. I & III

Answer: C

Explanation:
When the classification of a marketable security is changed, the assignation of the security to the
new account is carried out at the fair market value on the transfer date, with any gain or loss
reflected separately on the income statement (in the case of trading and held-to-maturity securities)
or as a separate component of shareholder equity (in the case of available-for-sale securities). The
classification and reclassification of securities is largely based on management intent. This allows the
firm to manipulate current income by timing the reclassifications. Consequently, any financial
statement analysis must take into account the effects of such artificial effects on income.

Question: 1592

Accounting rules differentiate research and development activities from activities not considered
research and development. Which one of the following is not considered a research and
development activity?

A. Periodic design changes to existing products.


B. Testing in search of product processing alternatives.
C. Modification of the design of a process.
D. None of these answers.
E. Laboratory research intended for the discovery of a new product.That answer is correct!

Answer: A

Explanation:
R&D costs must be expensed as incurred. Research is planned search or critical investigation aimed at
discovery of new knowledge with the hope that such knowledge will be useful in developing a new
product or service or process or in bringing a significant improvement in an existing product or
process. Development is the translation of research findings into a plan or design. Seasonal or other
periodic design changes in existing products do not meet either of these definitions.

Question: 1593

An asset is estimated to have a total life of 8 years. Its acquisition cost is 16,000 and a salvage value
of 4,000. The firm follows the double declining method of depreciation. In the second year, the rate
of depreciation for the firm's asset is closest to ________.

A. 25%
B. 17%
C. 9.5%
D. 12.5%That answer is correct!

Answer: A

Page | 585
Explanation:
In DDM depreciation, depreciation expense = (2/n)*book value. Thus, depreciation rate = 2/n = 2/8 =
25%. Remember that in the later years, DDM has to switch to a different depreciation method to
avoid depreciating below the salvage value.

Question: 1594

Under inflationary conditions and stable inventories, the COGS under Average Cost method:

A. lower than FIFO COGS.


B. could be greater or less than FIFO COGS.
C. higher than FIFO COGS.
D. is higher than LIFO COGS.

Answer: C

Explanation:
When prices are rising, the Average cost during the period is higher than the FIFO costs. Therefore,
COGS under Average Cost method will be higher than the FIFO COGS (but lower than the LIFO COGS).

Question: 1595

Why do the corporation's directors declare stock dividends?

A. all of these answers


B. to increase the number of shares outstanding
C. to provide tangible evidence of management's confidence in the company's strong performance
D. to keep the market value of the company's stock affordableThat answer is correct!

Answer: A

Explanation:
Declaring a stock dividend will increase the number of shares outstanding and thereby keep the per
share price low enough to be an attractive investment. Stock dividends may also show the company
management's confidence in the present and future performance of the company.

Question: 1596

A firm has convertible bonds, preferred equity, common equity and straight bonds in its capital
structure. Its net income equals 7,800. The interest payments on straight debt equal 423 and that on
convertible debt equals 680. The preferred equity holders received 220 in dividends. If the weighted
number of shares for Diluted EPS equals 1,300 and the firm is in the 40% tax bracket, the Diluted EPS
equals _________. Assume the convertibles are dilutive.

A. 5.31
B. 6.14
C. 5.52
D. 6.35

Answer: B

Explanation:
The earnings available for distribution amongst common stock holders and convertible bond holders
equals Net income - preferred dividends + interest payments on convertible net of taxes. Therefore,

Page | 586
earnings in the numerator = 7,800 - 220 +680*(1-40%) = 7,988. The Diluted EPS then equals
7,988/1,300 = 6.14.

Question: 1597

Which of the following is usually not a feature of cumulative preferred stock?

A. Has priority over common stock with regard to assets.


B. Has priority over common stock with regard to earnings.
C. None of these answers.
D. Has the right to receive dividends in arrears before common stock dividends can be paid.
E. Has voting rights.

Answer: E

Explanation:
Preferred stock does not usually have voting rights. Preferred shareholders are usually given the right
to vote for directors only if the company has not paid the preferred dividend for a specified period of
time, such as 10 quarters.

Question: 1598

A firm's ROE equals 47%. Its financial leverage equals 1.8 and its asset turnover is 0.8. If the firm's
total net sales equal $1.35 million, its net income equals ________.

A. 1,427,625
B. 440,625
C. 352,500
D. 282,000

Answer: B

Explanation:
ROE = net income/equity = (net income/sales)*(sales/assets)*(assets/equity) = (net
income/sales)*(asset turnover)*(financial leverage) Therefore, net income = 0.47*1.35/(0.8*1.8)
million = 440,625.

Question: 1599

An increase in the LIFO reserves implies which of the following?

I. Prices may have risen.


II. More goods might have been sold than purchased.
III. Firm may have changed inventory accounting from FIFO to LIFO.
IV. The firm may have indulged in excess purchasing to reduce taxes.

A. II, III & IV


B. I & II
C. I & IV
D. none of them

Answer: C

Explanation:

Page | 587
The LIFO Reserve equals the difference in the inventory value under FIFO and under LIFO. This
reserve will increase if either prices have risen sufficiently to cause a high ending inventory value or
if the firm purchased more goods than it sold in times of rising prices. Note that II represents a LIFO
liquidation, not an increase in the reserves. III does not lead to a change in LIFO reserves. Finally,
remember that IV is one way firms using LIFO can manipulate earnings and taxes, by changing the
purchasing behavior at year-end.

Question: 1600

Which of the following is a valid reason for an analyst to question the value of a deferred asset or
justify capitalizing it?

A. Rapid product obsolescence.


B. Over estimation of future sales.
C. All of these answers.
D. The proper amortization might be substantially shorter that the one selected by management.
E. Non-existence of future benefits.

Answer: C

Explanation:
The analyst needs to scrutinize all factors related to the value of deferred assets. Conditions such as
rapid obsolescence can considerably shorten the time horizon that a future project's value is based
upon. Further, management can be overly optimistic in the estimation of future sales; these sales
may fall far short, or they may never materialize. It is the analysts' responsibility to research all facts
available and come to an independent conclusion as to the validity of such claims.

Question: 1601

Excerpts from the balance sheet of Milton Corporation as of April 30, 1997 are presented as follows:

Cash$725,000
Accounts receivable (net)$1,640,000
Inventories$2,945,000
Total current assets$5,310,000
Accounts payable$1,236,000
Accrued liabilities$831,000
Total current liabilities$2,067,000

The board of directors of Milton met on May 5, 1997 and declared a quarterly cash dividend in the
amount of $200,000 ($0.50 per share). The dividend was paid on May 28, 1997 to shareholders of
record as of May 15, 1997. Assume that the only transactions that affected Milton during May 1997
were the dividend transactions. Milton's total shareholders' equity would be

A. unchanged by either the dividend declaration or the dividend payment.


B. decreased by the dividend declaration and unchanged by the dividend payment.
C. decreased by the dividend payment and unchanged by the dividend declaration.
D. none of these answers.
E. increased by the dividend declaration and unchanged by the dividend payment.

Answer: B

Explanation:
The declaration of the dividend results in an increase in dividends payable (a current liability

Page | 588
account) and a corresponding decrease in retained earnings (a shareholders' equity account).
Therefore, the declaration of a dividend reduces shareholders' equity. The subsequent payment of
the dividend has no effect on shareholders' equity because that transaction reduces cash and
reduces the previously recorded dividends payable.

Question: 1602

Below is an example of an incorrectly prepared statement of cash flows. The descriptions of activities
are correct.

Cash from operating activities$60,000

Net Income(4,000)
Depreciation(2,000)
Increase in accounts receivable(1,000)
Increase in deferred tax liability$53,000

Cash from investing activities($48,000)


Purchase of marketable securities2,500
Dividends received1,500
Dividends paid($44,000)
Cash from financing activities(500)
Increase in Short-term debt(2,500)
Increase in Long-term debt($3,000)
Increase in cash$6,000

The correct Cash flows from financing activities is ________.

A. None of these answers


B. $1,500
C. $3,000
D. ($4,500)

Answer: B

Explanation:
The increase in short-term and long-term debt are cash inflows, not outflows. The dividend paid is
also a financing activity. $500 + $2,500 - $1,500 = $1,500.

Question: 1603

Stockholders' Equity is

A. the rights to the assets of the business once the liabilities have been met
B. assets plus liabilities
C. all of these answers are correct
D. the financial obligations of the companyThat answer is correct!

Answer: A

Explanation:
The accounting equation shows us that the Assets (Own) are equal to the Liabilities (obligations) plus
the Stockholders' Equity.

Page | 589
Question: 1604

Tracy company reports the following in its statement of cash flows:

Net Income$1,000
Depreciation and Amortization350
Decrease (Increase) in Accounts receivable(10)
Decrease (increase) in inventory200
Decrease (increase) in prepaid expenses80
Increase (decrease) in trade payables(300)
Increase (decrease) in taxes payable75
Cash Flow from operations1,395

Tracy used the indirect method of determining cash flow from operations (CFO), had they used the
direct method

A. CFO would have been higher as gains are not deducted in arriving at CFO.
B. CFO would have been the same.
C. it is not possible to determine what CFO would have been without more information.
D. CFO would have been lower as losses and depreciation are not added back in arriving at CFO.

Answer: B

Explanation:
The method does not affect the CFO.

Question: 1605

Financial accounting data has some inherent limitations. Which of the following are limitations?

I. not all economic events are easily quantifiable


II. many accounting entries rely heavily on estimates
III. historical cost can distort statements
IV. inflation can distort accounting data

A. II, III and IV


B. I, II and III
C. I, III and IV
D. I, II, III and IV

Answer: D

Explanation:
All of the responses can be considered limitations of financial accounting data.

Question: 1606

Which of the following is/are component(s) of Shareholder's equity?

I. Preferred equity
II. Retained earnings
III. Treasury shares

A. II & III

Page | 590
B. I only
C. I, II & III
D. I & III

Answer: C

Explanation:
All of the selections appear in the Shareholders' Equity section of the balance sheet.

Question: 1607

Which inventory costing method is most useful in estimating the amount of inventory lost or
destroyed by theft, fire, or other hazards?

A. FIFO
B. None of these answers
C. average cost method
D. gross profit method
E. LIFO

Answer: D

Explanation:
The historical relationship between cost of goods sold and sales is applied to sales of the current
period as a way of estimating COGS during the current period. COGS is then subtracted from the cost
of goods available for sale to get the estimated cost of the ending inventory.

Question: 1608

Firm A uses the completed contract method and an otherwise identical Firm B uses percentage-of-
completion method for revenue recognition in a project. Then, during the project,

A. Firm A shows lower assets than B.


B. Firm B shows higher inventory than A.
C. Firm A shows higher equity than B.
D. Firm A shows higher inventory than B.

Answer: D

Explanation:
Under the completed contract method, revenues and expenses are recognized only after the contract
has been fulfilled. During all intermediate periods, costs are reported as inventory while revenues
are treated as advances from customers. On the other hand, in the percentage-of-completion
method, revenues are recognized in proportion to the percentage of the project completed, as
measured by the costs incurred or engineering estimates available. Therefore, a firm shows higher
assets and liabilities under completed contract method but lower equity than under percentage-of-
completion.

Question: 1609

Which of the following is not an example of a cash flow from operations?

A. none of these answers is correct


B. cash received from customers

Page | 591
C. cash paid for merchandise
D. cash paid for taxesThat answer is correct!

Answer: A

Explanation:
Operating activities include the cash effects of transactions and other events that enter into the
determination of net income. All of the responses apply.

Question: 1610

All of the following are financing cash flows EXCEPT?

A. cash payments on preferred dividends


B. cash received from new debt
C. cash changes in contributed capital from equity issue
D. cash changes in retained earnings

Answer: D

Explanation:
Cash changes in retained earnings arise from income and hence, are part of operating cash flows.

Question: 1611

Accounting rules specify that interest must be capitalized for assets that are

A. not being used in the earning activities of the enterprise and not undergoing the activities
necessary to get them ready for use.
B. routinely produced.
C. being constructed or otherwise being produced as discrete projects for an enterprise's own use.
D. none of these answers.
E. in use or ready for their intended use in the earning activities of the enterprise.

Answer: C

Explanation:
Interest costs must be capitalized only for assets constructed for internal use, or for sale or lease as
discrete projects.

Question: 1612

The deferred income tax account

A. is where the difference between income tax expense and income tax payable is reconciled
B. is always reported as a long-term liability since the tax is not due until the next fiscal year
C. is reported as an other asset even though it has a credit balance
D. none of these answers is correctThat answer is correct!

Answer: A

Explanation:
The difference between income tax expense (based on accounting income) and the actual income
taxes payable (based on taxable income) is reconciled in an account called deferred income taxes.

Page | 592
Question: 1613

An operating cycle is:

A. the time frame from when a company purchases goods and or services to the point at which the
company generates a receivable.
B. the amount of time that a company allots to any individual product line.
C. always one full fiscal year.
D. the circle of time from the commitment of cash for purchases until the collection of receivables
resulting from the sale of goods or services.

Answer: D

Explanation:
An operating cycle begins and ends with cash. It goes through the full cycle of buying inventory,
converting the inventory, generating a receivable and finally turning the receivable into cash.

Question: 1614

The matching principle requires that

A. non-operating gains and losses should be netted against each other.


B. a proportion of each dollar collected will be assumed to be a recovery of cost.
C. revenues earned and expenses incurred in generating those revenues should be reported in the
same income statement.
D. assets will be matched to the liabilities incurred to purchase them.

Answer: C

Explanation:
Performance can be measured only if the related revenues and cost are accounted for in the same
period.

Question: 1615

One method management can "manipulate" earnings is by

A. reporting a gain on the sale of subsidiary when the operating income of the firm is showing a loss.
B. changing from LIFO to any other inventory valuation method.
C. reporting the full amount of anticipated restructuring costs.
D. changing from the percentage-of-completion method.
E. reporting additional losses in bad years, assuming future reported profits will increase.

Answer: E

Explanation:
Known as "Big Bath" accounting, by taking all available losses at one time, management believes it
can clear all the losses from the profit and loss statement and begin to increase profits.

Question: 1616

When preparing a direct method statement of cash flows, the principal component of cash flow from
operating activities is

Page | 593
A. the decrease in accounts receivable.
B. cash collections.
C. cash received from short- and long-term borrowings.
D. the increase in accounts payable.
E. net income plus depreciation expense.

Answer: B

Explanation:
Cash collections is the beginning point and principal component of cash flow from operating
activities.

Question: 1617

Cash received from the sale of fixed assets is an example of:

A. cash flows from investing activities


B. cash flows from operating activities
C. cash flows from financing activities
D. cash flows from noncash investing and financing activitiesThat answer is correct!

Answer: A

Explanation:
Investing activities usually involve activities that affect long-term assets.

Question: 1618

Which of the following is/are FALSE under GAAP?

I. The change in cash balances in consecutive Balance sheets equals the operating cash flow of the
firm over that period.
II. The investing, financing and operating cash flows must together balance out to zero, to preserve
the relationship, cash in = cash out.
III. Dividends received from investments in the stocks of other firms are classified under investing
cash flows.
IV. Interest paid on debt financing is classified as operating cash flow.

A. I, II & III
B. II & IV
C. I, II & IV
D. I, II, III & IVThat answer is correct!

Answer: A

Explanation:
The change in cash balances of a firm can come about due to investing and financing cash flows, in
addition to operating cash flows. Therefore, (I) is false. The relationship cash in = cash out is incorrect
since the difference between the two goes into changing the cash balance of the firm. Hence, the
operating, investing and financing cash flows need not balance out. It is very important to remember
the following points about dividends and interests:

1. Dividends received from stock investments are considered operating cash flows.

Page | 594
2. Dividends paid on equity are considered financing cash flows.
3. Interest payments on debt are considered operating cash flows.
4. Interest received from debt investments are considered operating cash flows.

Question: 1619

All of the following would be considered financing activities except for:

A. dividends payable.
B. notes payable to bank.
C. current portion of long-term debt.
D. loans receivable (ex. from officers).

Answer: D

Explanation:
Investing activities include lending funds and collecting the principal on these loans.

Question: 1620

In a classified balance sheet the following is always true:

A. Current assets include only items that will be converted into cash within 1 year.
B. Assets and liabilities are broken down as current and non-current.
C. Current assets divided by current liabilities will equal working capital.
D. All of these answers are true.

Answer: B

Explanation:
A classified balance sheet always breaks down current and non-current assets and liabilities. Items
defined as current assets are those that will be converted to cash or used in operations within one
year or the operating cycle, whichever is longer. Current liabilities are obligations due during the
same period. Current assets minus current liabilities is equal to working capital.

Question: 1621

Which of the following is a valuable investigation tool for analysis of the collectibility of a firm's
receivables?

A. All of these answers.


B. Determining patterns of receivables for peers as a percent of sales.
C. An examination of any customer concentrations.
D. An analysis of the adequacy of allowances for trade discounts, and returns and allowances.
E. An aging schedule.That answer is correct!

Answer: A

Explanation:
Each of the above mentioned tools are valuable in assessing the collectibility of a firm's accounts
receivables. An aging schedule will pinpoint which customers are not paying within terms. An analyst
should investigate if a firm is understating allowances for discounts, returns and allowances, as this
would overstate receivables. If too much of a firm's sales are made to one or more firm's that are
experiencing financial distress, receivables are at risk.

Page | 595
Question: 1622

Which of the following is/are true under GAAP?

I. Inventories can only be "written down" but not "written up."


II. Marketable securities are carried at acquisition cost.
III. For income tax purposes, LIFO is preferable to FIFO during inflation.

A. II only
B. I, II & III
C. III only
D. I & III

Answer: C

Explanation:
(I) is not true since after a write-down, inventories can be written back up to the original cost if the
prices of goods increase sufficiently. (II) is also not true since marketable securities are carried at
current market value. Finally, when prices are rising, the COGS under LIFO is higher than that under
FIFO, leading to lower tax payments.

Question: 1623

Which of the following is/are FALSE?

I. Over the life of the firm, income and cash flow are the same.
II. Cash accounting follows the Matching Principle of matching cash flows to appropriate periods.
III. Accrual accounting matches revenues with the associated costs.
IV. Accrual accounting allocates many cash flows to time periods other than those in which they
occur.

A. II only
B. I, III & IV
C. III & IV
D. I, II, III & IVThat answer is correct!

Answer: A

Explanation:
Unlike Accrual accounting, Cash accounting recognizes revenues and expenses when cash flows
occur, not necessarily when the earnings process is complete or goods and services consumed.

Question: 1624

The balance sheet of Firm A shows the following:

Cash & Cash equivalents432


Receivables98
Inventories143
Marketable securities329
Short-term loans732
Current portion of long-term debt210

Page | 596
The quick ratio of the firm equals ________.

A. 0.91
B. 4.09
C. 1.17
D. 0.83That answer is correct!

Answer: A

Explanation:
The quick ratio is the current ratio calculated using only relatively liquid securities. These current
assets are cash, marketable securities and receivables. Thus, Quick ratio = (cash + marketable
securities + receivables)/current liabilities In this case, Quick ratio = (432+98+329)/(732+210) = 0.91

Question: 1625

Why is book value per share important?

A. Because it is frequently used in assessing merger terms.


B. All of these answers.
C. It is the best method of determining the market value of a firm's common stock.
D. None of these answers.
E. Book value per share is the "true" value of a firm's common stock.That answer is correct!

Answer: A

Explanation:
Book value per share is often times used by firms in determining merger terms. Book value per share
plays an important role in the analysis of financial statements since one method of valuation begins
with the book value per share figure, to which an estimate of future earnings is added. An estimate
of the future profitability of a firm can be based directly on the earnings capacity of its asset base.

Question: 1626

With respect to depreciation methods, which of the following is true?

I. Accelerated depreciation methods lead to higher depreciation expense over time.


II. The Straight-line method causes higher taxes in later years.
III. Accelerated methods are preferred for tax reasons.

A. III only
B. II & III
C. I & III
D. II onlyThat answer is correct!

Answer: A

Explanation:
Accelerated methods cause depreciation expense to decrease over time. In later years, the straight-
line method leads to higher depreciation compared to accelerated methods and hence, leads to
lower taxes.

Question: 1627

Page | 597
Which accounting principle is consistent with reporting financial results that can be compared with
previous periods?

A. adequate disclosure
B. matching
C. consistency
D. materiality

Answer: C

Explanation:
The consistency principle requires that an accounting procedure, once adopted by a company,
remain in use from one period to the next unless users are informed of the change.

Question: 1628

Which of the following best highlights a company's financial condition and overview of the results of
operations?

A. The security and exchange commissions 401-k report


B. The auditor report
C. The statement of cash flows
D. None of these answers
E. Management's discussion and analysis

Answer: E

Explanation:
Management's discussion and analysis provide a great deal of detail regarding the company's
financial condition, the results of operations, favorable, or unfavorable trends, and significant events
that have affected or will affect a firm.

Question: 1629

The portion of the insurance premiums that has expired during the fiscal period is classified as:

A. an expense
B. an asset
C. an increase in retained earnings
D. a liabilityThat answer is correct!

Answer: A

Explanation:
In accordance with the matching rule, the amount of an insurance policy that has expired during the
accounting period must be recorded and classified as an expense for the period.

Question: 1630

Which of the following is/are true about the cash conversion cycle?

I. It increases as the inventory turnover ratio decreases.


II. It decreases as inventory processing period decreases.
III. It is directly proportional to the payables payment period.

Page | 598
IV. It increases as the cash ratio decreases.

A. II, III & IV


B. I & IV
C. II & III
D. I & II

Answer: D

Explanation:
The cash conversion cycle is a measure of how long the cash is tied up in short term loans and
credits. These short-term financing items include receivables, inventory and accounts payable.
Therefore cash conversion cycle is defined as: CCC = (Average receivables collection period) plus
(Average inventory processing time) minus (Average payables payment period). Now, an increase in
inventory ratio decreases inventory processing time, lowering the CCC. Also, an increase in the
payables payment period decreases the CCC. However, CCC is independent of the cash ratio.

Question: 1631

When a firm experiences LIFO liquidation under rising prices, it recognizes:

A. lower COGS and lower cash flows.


B. lower COGS and higher cash flows.
C. higher COGS and lower cash flows.
D. higher COGS and higher cash flows.That answer is correct!

Answer: A

Explanation:
In LIFO liquidation under rising prices, the quantity of goods purchased during a period is less than
the quantity sold. This leads to a sale of goods purchased in prior periods at lower prices (since prices
are rising). This understates the cost-of-goods-sold (COGS), thus overstating the income and causing
higher taxes than in the absence of the LIFO liquidation. The higher taxes cause a higher cash
outflow. (This is an extremely important topic with which you should be completely familiar.)

Question: 1632

An analysis of a firm's financial statements indicates that the average age of its assets is declining.
This could be due to which of the following?

I. The firm is acquiring new assets with longer depreciable lives.


II. The firm's capital expenditures are outpacing depreciation.
III. The firm is not using its assets as intensively as it should.
IV. The firm is operating in its maturity phase.

A. I & IV
B. I & II
C. I, II, III & IV
D. III & IV

Answer: B

Explanation:
Note that the firm is likely not in a maturity or declining phase but in a growth phase, which is

Page | 599
characterized by increasing capital investments.

Question: 1633

A cash flow statement using the indirect method

A. is as informative as using the direct method.


B. begins with a firm's cash collections.
C. uses major categories of gross cash receipts and payments.
D. provides little new information into a firm's cash-generating ability.

Answer: D

Explanation:
Because of the indirect format, it is not possible to compare operating cash inflows and outflows by
function with the revenue and expense activities that generated them.

Question: 1634

The following are all true concerning transactional analysis, except,

A. it can be used to convert indirect method cash flow from operations to the direct method.
B. it is a technique that can be used to create a cash flow statement for firms that do not prepare
such statements in accordance with SFAS 95 and IAS 7.
C. it helps to understand the relationship between the accrual of revenues, expenses, assets, and
liabilities and their cash flow consequences.
D. it is a method that groups changes in the current assets and current liabilities and reconciles them
to the relevant income statement items.
E. it is a method that classifies cash flows among operating, financing, and investing activities.

Answer: D

Explanation:
This method reconciles line-item changes in the balance sheet with their related income statement
components to derive the cash flow consequences of the reported transactions and events.

Question: 1635

The statement of cash flows cannot be used to

A. consider the interrelationship between cash flow components over time.


B. examine the trend of different cash flow components over time.
C. examine the cash flow components and their relationship to related income statement items.
D. review individual cash flow items for analytic significance.
E. examine the firm's ability to realize assets and settle liabilities.

Answer: E

Explanation:
Income statement and balance sheet data must be combined with cash flows for insights into the
firm's ability to realize assets based on reported revenues and settle liabilities resulting from accrued
expenses.

Question: 1636

Page | 600
Which of the following is/are true?

I. An increase in inventories has a positive impact on cash flows.


II. An increase in receivables has a negative impact on cash flows.
III. Deferred taxes increase current cash balance.
IV. Utilization of tax loss carry-forwards has a positive impact on cash flows.

A. I, II & IV
B. II & III
C. I & IV
D. II, III & IV

Answer: D

Explanation:
An increase in inventories implies cash was spent to obtain more goods than were sold. An increase
in receivables implies part of the sales were made on credit, not a cash basis. Deferring taxes implies
some of the cash expense was delayed into the future, increasing the current cash balance.
Utilization of taxloss carry-forwards implies a reduction in the taxable income by the amount of the
carry-forward. This reduces cash expense on taxes.

Question: 1637

A benefit period for accounting amortization can not exceed how many years ________.

A. 15
B. 40
C. 30
D. 25

Answer: B

Explanation:
In accounting no benefit can be amortized over more than 40 years. The difficulties of going further
do not provide sufficient benefit to justify the cost.

Question: 1638

Suppose the average stock price in the above example were 40. How many shares will be used in the
calculation of Diluted EPS now?

A. none of these answers


B. 1,250,000
C. 1,208,333
D. 1,300,000

Answer: C

Explanation:
Since the average stock price now exceeds the strike price, the warrants are assumed converted. The
proceeds from the exercise equal 800,000*35 = 28 million. The Treasury Stock method assumes a
purchase of shares at 40 using these proceeds. Thus, the number of shares repurchased =
28,000,000/40 = 700,000. Thus, the additional shares arising from warrant exercise equals 800,000 -

Page | 601
700,000 = 100,000. Now, in the Treasury Stock method, options and warrants are assumed to be
exercised at the beginning of the period or at the time of issuance, whichever is later. Thus, in this
case, the addition 100,000 shares have been outstanding since May 31, 1998. So the weighted
number of shares for Diluted EPS = 1 million + 200,000*9/12 + 100,000*7/12 = 1,208,333 shares.

Question: 1639

For accounting purposes, which of the following is/are TRUE about assets?

I. Assets are future benefits whose values are known today.


II. Assets are generated by past transactions or events.
III. Assets include the value of residual ownership of the firm.
IV. Assets are best viewed as sources of future cash flows.

A. I, II, III & IV


B. I, II & IV
C. II only
D. I & II

Answer: C

Explanation:
Assets are defined as "probable future benefits obtained or controlled by a particular entity as a
result of past transactions or events." III refers to "Equity" while IV is not correct since the benefits
need not be solely in the form of cash.

Question: 1640

The financial statements of Multiverse for 1997 showed the following:

a. Dividends paid60
b. Provision for bad debt15
c. Depreciation90
d. Interest paid35
e. Proceeds from new bonds issued225
f. Bonds retired400
g. Gain on bonds retired25
h. Shares repurchased195
i. Net Income350
j. Tax rate50%

What was Multiverse's financing cash flow in 1997?

A. -405
B. -430
C. -370
D. -380

Answer: D

Explanation:
Items a, e, f, g and h are financing items. However, remember that item g, gain on bonds retired, is
an extraordinary item under SFAS 4 and presented after-tax. Hence, the total cash spent on retiring
bonds = 400 - 25/(1-0.5) = 350.Therefore, financing cash flow = -60 + 225 - (400 - 25/0.5) - 195 = - 380.

Page | 602
Question: 1641

When a business is acquired, the purchasing company calculates goodwill associated with the
acquisition as the difference between the purchase price and the ________.

A. book value of the identifiable net assets acquired


B. book value of the net tangible assets acquired
C. fair value of the net tangible assets acquired
D. book value of the total assets acquired
E. fair value of the identifiable net assets acquired

Answer: E

Explanation:
Goodwill is the difference between the amount paid for the business and the fair value of this
business. The fair value can be calculated using net assets (total assets minus total liabilities), or
shareholders' equity.

Question: 1642

A firm has legal cash expenses of 300 and a tax rate of 40. If it chooses to capitalize the costs and
depreciated them over 3 years, the operating cash flow under capitalization, compared to that under
expensing, will be ________.

A. smaller by 220
B. larger by 180
C. larger by 220
D. smaller by 180

Answer: C

Explanation:
Under expensing, the operating cash flow will be reduced due to this expense by 300*(1-0.4) = 180
(there is an outflow of 300 but since the expenses are tax deductible, taxes on total income get
reduced by 120). Instead, if the firm capitalized the expense, an asset value of 300 will be generated.
The entire expense will be charged against investing cash flow. Depreciation of capitalized expense
leads to a depreciation expense of 300/3 = 100. This will reduce taxes paid by 100*0.4 = 40. Thus,
capitalization leads to an increase in the operating cash flow of 40. In summary, expensing reduces
the expense by 180 while capitalization increases operating cash flow by 40. Thus, operating cash
flow under capitalization is larger by 40 - (-180) = 220.

Question: 1643

Which of the following would not be included as equity in a corporate balance sheet?

A. Common stock
B. Preferred stock
C. Cash
D. Retained earnings

Answer: C

Explanation:

Page | 603
Cash does not represent equity; equity consists of the book value of a company; i.e., assets, minus
liabilities.

Question: 1644

All of the following are attributes of depreciation except

A. depreciation stays on the balance sheet as long as the asset is owned by the corporation.
B. depreciation provides funds for replacement of an asset.
C. depreciation is allocation of the cost of an asset over its useful life.
D. the purpose of depreciation is to charge against operations, by means of allocation, the cost of an
asset.

Answer: B

Explanation:
Depreciation is a non-cash expense. It shows up as an increase to cash on the statement of cash
flows.

Question: 1645

The balance sheet

A. reports all of the cash inflows and shows specifically how that cash was used.
B. is a report that shows the change in the financial position of a firm.
C. reports the firm resources and how those resources were specifically used.
D. reports the firm resources, claims on those resources, and stockholders' equity at specific points in
time.
E. reports the assets and liabilities of the firm for the accounting period.

Answer: D

Explanation:
The balance sheet reports the categories and amounts of assets, liabilities, and stockholders' equity
at specific points in time.

Question: 1646

How should holdings of equity securities of less than a 20% interest generally be classified on a firm's
balance sheet?

A. As either trading or available-for-sale securities


B. Using the equity method
C. All of these answers
D. As controlling interest securities
E. None of these answersThat answer is correct!

Answer: A

Explanation:
So long as it is management intent, these securities should be classified as either trading or
availablefor-sale securities. The equity method is used to account for an equity interest where
management has an ability to exert significant influence over an investee's business activities.

Page | 604
Question: 1647

Which of the following is/are true?

I. An increase in inventories has a positive impact on cash flows.


II. An increase in receivables has a positive impact on cash flows.
III. Deferred taxes increase current cash balance.
IV. Utilization of tax loss carry-forwards has a positive impact on cash flows.

A. III & IV
B. II & III
C. I & II
D. I, II, III & IVThat answer is correct!

Answer: A

Explanation:
An increase in inventories implies cash was spent to obtain more goods than were sold. An increase
in receivables implies part of the sales were made on credit, not a cash basis. Both of these have a
negative impact on cash flows. Deferring taxes implies some of the cash expense was delayed into
the future, increasing the current cash balance. Utilization of tax loss carry-forwards implies a
reduction in the taxable income by the amount of the carry-forward. This reduces cash expense on
taxes.

Question: 1648

Firms report payments for capital leases in the cash flow statement

A. only as financing cash flows.


B. partly as operating cash flows and partly as investing cash flows.
C. only as investing cash flows.
D. partly as operating cash flows and partly as financing cash flows.

Answer: D

Explanation:
Interest expense is classified as CFO and amortization of the lease obligation is reported as cash from
financing activities.

Question: 1649

Which of the following is NOT a deferred credit item?

A. Deferred gain on installment sales


B. None of these answers
C. Deferred tax credits
D. Advance rental payments

Answer: B

Explanation:
All of the responses qualify as deferred credit items.

Question: 1650

Page | 605
The primary current source of generally accepted accounting principles rests with the ________.

A. Securities and Exchange Commission


B. Institute of Management Accountants
C. American Institute of Certified Public Accountants
D. New York Stock Exchange
E. Financial Accounting Standards Board

Answer: E

Explanation:
FASB was created as an autonomous body with the responsibility of establishing financial accounting
standards. It is responsible to hear all viewpoints from the entire economic community and be
unbiased.

Question: 1651

Which of the following statements is true?

A. Recognition is concerned with recognizing a liability from a current cash.


B. Deferral is the process of formally recording or incorporating an item in the financial statements of
an entity.
C. Realization would reduce a liability that is recorded as a result of a cash receipt by recognizing
revenues.
D. Allocation is the process of converting non-cash resources and rights into money.
E. Accrual is the process of recognizing assets, liabilities, and components of comprehensive income.

Answer: E

Explanation:
Accrual is the process of recognizing assets, liabilities, and components of comprehensive income.

Question: 1652

Preferred and common stock differ in that

A. failure to pay dividends on common stock will not force the firm into bankruptcy, while failure to
pay dividends on preferred stock will force the firm into bankruptcy.
B. common stock dividends are a fixed amount, while preferred stock dividends are not.
C. preferred stock dividends are deductible as an expense for tax purposes, while common stock
dividends are not.
D. preferred stock has a higher priority than common stock with regard to earnings and assets in the
event of bankruptcy.
E. none of these answers.

Answer: D

Explanation:
In the event of bankruptcy, the claims of preferred shareholders must be satisfied before common
shareholders receive anything. The interests of common shareholders are secondary to those of
other claimants.

Question: 1653

Page | 606
Which of the following statements is true?

A. Recognition is concerned with recognizing a liability from a current cash receipt.


B. The Accrual process would reduce an asset that is recorded as a result of a cash payment.
C. Realization is the process of converting non-cash resources and rights into money.
D. All of these are true.
E. Allocation is the process of formally recording or incorporating an item in the financial statements
of an entity.

Answer: C

Explanation:
Realization is the process of converting non-cash resources and rights into money and is most
precisely used in accounting and financial reporting to refer to sales of assets for cash or claims to
cash.

Question: 1654

When valuing inventories using the lower-of-cost-or market (LCM), which of the following is/are
true?

I. The inventory value cannot exceed the net realizable value.


II. If the inventory is written down from cost, the value of the inventory cannot fall below the net
realizable value.
III. Inventories can only be written back up to the original cost.
IV. Write-downs are charged directly to retained earnings account.

A. I & III
B. I, II & III
C. II only
D. II & IIIThat answer is correct!

Answer: A

Explanation:
US GAAP requires that inventory be valued at the lower of cost or market value. This means that if
the inventory value falls below the cost (FIFO, LIFO, average cost, etc.) for reasons which include
price changes, obsolescence, damage or theft, the loss in value must be recognized immediately.
Specifically, the inventory must be "written-down" in value and the loss recognized on the income
statement. In the application of this principle, "market value" is defined as current replacement cost,
except that this value cannot exceed the net realizable value (NRV) or fall below NRV minus the
normal profit margin.

Question: 1655

In the full cost method, oil firms:

A. none of these answers.


B. are required to expense all oil-drilling costs resulting in dry holes.
C. must expense drilling costs which result in productive oil wells.
D. can capitalize all oil-drilling costs.

Answer: D

Page | 607
Explanation:
In extractive industries, firms are allowed to use either the full-cost method, in which all search and
development costs can be capitalized, or the successful-efforts method, where all such costs are
expensed unless they result in revenue-generating assets, in which case, they are capitalized.

Question: 1656

Which of the following will most affect the valuation of a firm's receivables?

A. The rate of sales growth


B. None of these answers
C. The type of business that a firm is engaged in
D. The allowance for uncollectible accounts
E. The seasonality of a company's products

Answer: D

Explanation:
The most important factor that will affect the valuation of receivables, of the choices given, is the
allowance for uncollectible accounts. This is so, because receivables are reported on the balance
sheet as "net realizable value;" i.e., total amount of receivables, less an allowance for uncollectible
accounts. If management understates this allowance, the realizable value of accounts receivable will
be overstated and the firm's liquidity position will not be as strong as it would appear.

Question: 1657

Bay Co. purchased a 3-month U.S. Treasury bill. In preparing Bay's statement of cash flows, this
purchase would

A. have no effect.
B. be treated as an outflow from lending activities.
C. be treated as an outflow from financing activities.
D. be treated as an outflow from operating activities.
E. be treated as an outflow from investing activities.That answer is correct!

Answer: A

Explanation:
The T-bill is a cash equivalent and has no effect on the statement of cash flows. Cash equivalents are
short-term, highly liquid investments that are both readily convertible to known amounts of cash and
so near their maturities that they present insignificant risk of changes in value because of the
changes in interest rates. They only include investments with original maturities to the holder of 3
months or less.

Question: 1658

The following data have been extracted from the financial statements of a firm for two years, 1985
and 1986:

19851986
Assets4,3765,214
Sales8,3127,845
Receivables985756

Page | 608
Inventory1,1081,243
COGS5,4944,971

The inventory turnover ratio and the average inventory processing period for 1986 equal ________.

A. 4.23 86.31 days


B. 5.19 70.33 days
C. 4.87 74.95 days
D. 5.62 64.95 daysThat answer is correct!

Answer: A

Explanation:
One of the properties of the inventory that is of interest is the rate at which it turns over i.e. the
average processing time it takes for a given inventory item to be sold. This can be estimated using
either net sales (as is the case with receivables) or the cost-of-goods-sold. COGS is preferable since it
does not include the profit margins involved in net sales. Therefore, two relevant ratios are: a.
Inventory turnover ratio = COGS/average inventory. b. Average inventory processing period =
365/inventory turnover. For 1986, the average inventory equals (1243+1108)/2 = 1,176. Inventory
turnover ratio = 4,971/1,176 = 4.23. Average inventory processing period = 365/4.23 = 86.31 days.

Question: 1659

If a company converted a short-term note payable into a long-term note payable, this transaction
would

A. none of these answers.


B. increase working capital.
C. decrease working capital and the current ratio.
D. increase both working capital and the current ratio.
E. decrease only working capital.

Answer: D

Explanation:
This transaction reduces current liabilities, but does not change current assets and, therefore,
increases working capital and increases the current ratio.

Question: 1660

A firm needs to purchase 20 warehouses in various states. To finance the purchases, it issues new
shares in a seasoned equity offering. It also issues high coupon bonds on which interest must be paid
semi-annually. If the firm purchases the warehouses at the beginning of the year, which of the cash
flows will be affected in the year-end statements?

InvestingOperatingFinancing

I.affectedaffectedaffected
II.affectedaffectedunaffected
III.affectedunaffectedaffected
IVunaffectedunaffectedunaffected

A. I.
B. II.

Page | 609
C. IV.
D. III.That answer is correct!

Answer: A

Explanation:
The purchase of the warehouses affects the investing cash flows. The bonds and equity offering affect
the financing cash flows. These transactions do not affect operating cash flow immediately. The
interest payments on the issued debt, however, will affect it. In addition, the dividends paid on the
new equity will affect financing cash flows.

Question: 1661

The __________ is the most conservative of the following liquidity ratios.

A. current ratio
B. quick ratio
C. cash ratio
D. liquid ratio

Answer: C

Explanation:
The cash ratio is the most conservative of the 3 liquidity ratios since it considers only cash and
marketable securities relative to the current liabilities.

Question: 1662

The promulgation of GAAP is a responsibility that rests with:

A. The Securities and Exchange Commission (SEC).


B. The Committee on Accounting Procedures (CAP).
C. None of these answers.
D. The American Accounting Association (AAA).That answer is correct!

Answer: A

Explanation:
The SEC has the primary responsibility for creation of accounting standards in the US. The AAA does
issue its own statements about accounting standards and these are quite influential in shaping the
official framework. However, its statements are not binding on the industry unless the SEC adopts
them.Currently, the SEC has relegated most of the duties of addressing needs for various accounting
standards to the Financial Accounting Standards Board (FASB), though it remains the final authority in
these matters.

Question: 1663

A firm had retained earnings of $100,000 at the beginning of the year. During the year, the firm had a
net income of $55,000 and paid dividends worth $20,000. Then, the change in the firm's retained
earnings equals ________.

A. $155,000
B. $35,000
C. $55,000

Page | 610
D. $135,000

Answer: B

Explanation:
Ending retained earnings = Beginning retained earnings + Net Income - Dividends paid. Hence,
change in retained earnings = $55,000-$20,000 = $35,000.

Question: 1664

Firm A issues convertible bonds to raise capital in the amount of a million dollars. An identical Firm B
issues debt with warrants attached to raise the same amount. Which of the following statements
is/are true in this situation?

I. Firm A ignores the convertibility feature completely while recording the bonds.
II. Firm B's interest expense is higher than that of Firm A.
III. Both the firms recognize the dilutive effects of the debt while calculating EPS.

A. I, II & III
B. II & III
C. I only
D. II onlyThat answer is correct!

Answer: A

Explanation:
While both convertible bonds and bonds with attached warrants derive significant value from the
possibility of conversion to common stock, the accounting treatments for the two are quite different.
With convertible bonds, the conversion feature is completely ignored and the bond is recorded as an
ordinary bond. On the other hand, the proceeds from a bond-with-warrant issue are divided into two
groups: those related to the bond and those related to the warrant. The former is recorded as a
liability while the latter is directly added to equity, without any income statement effects. This leads
to a bigger amortization of a discount (or a smaller amortization of premium) with a warrant-debt
than with a convertible bond. This effect offsets the fact that a higher liability is recognized with a
convertible bond. Hence, the interest expense is higher in general when debt-with-warrant is issued.

Question: 1665

Birch Ltd. had net income for the year of $101,504 and a simple capital structure consisting of the
following common shares outstanding:

Months OutstandingNumber of Shares

January - February24,000
March - June29,400
July - November36,000
December35,040

Birch's earnings per share (rounded to the nearest cent) were ________.

A. $2.90
B. $3.45
C. $4.23
D. $3.20

Page | 611
E. $3.25

Answer: D

Explanation:
EPS equals net income divided by the weighted average number of shares outstanding and is $3.20
($101,504 / 31,720). Following is the calculation for the weighted average number of shares
outstanding:

24,000 X (2 / 12) = 4,000


29,400 X (4 / 12) = 9,800
36,000 X (5 / 12) = 15,000
35,040 X (1 /12) = 2,920
weighted average 31,720

Question: 1666

Stock splits have the following effects on the financial statements:

A. contributed capital and retained earnings are unchanged


B. all of these answers are correct
C. the account title for common stock changes to reflect the change in the par value of stock
D. disclosures about the stock on the balance sheet are changed to reflect the additional outstanding
shares and the revised par value per share

Answer: B

Explanation:
All of the responses represent the effect of stock splits on the financial statements.

Question: 1667

The statement of cash flows provides us with important clues on all of the following except:

A. Financial practices of management.


B. Projected changes for the following year in current liability accounts.
C. Feasibility of financing capital expenditures.
D. Ability to meet debt service requirements.

Answer: B

Explanation:
The statement of cash flows is very informative and can tell you a lot about a company. However, it
does not tell you the direction the company is heading as far as its near term liquidity. It can not be
inferred that because a company has a lot of cash flow that its payable balances will decrease in the
followingyear. There could be many reasons for holding excess cash including business seasonality
and potential acquisitions.

Question: 1668

Which of the following expenses can be capitalized?

I. Research costs incurred in developing a new medicine.


II. Purchase of intangibles for R&D activities which have alternative future uses.

Page | 612
III. Salaries of research personnel.

A. II only
B. I & II
C. III only
D. none of themThat answer is correct!

Answer: A

Explanation:
US GAAP requires that all R&D costs be expensed immediately (unless they generate tangible or
intangible assets which have alternative future uses which are measurable). R&D costs include
salaries for research personnel.

Question: 1669

Which of the following does not affect a firm's cash flows:

A. warranty expenses.
B. sale of an "impaired" asset.
C. purchase of a trademark financed by stock issuance.
D. change of depreciation method.

Answer: C

Explanation:
When the depreciation method is changed, the taxes paid are affected, thus affecting the cash flows
indirectly. Sales or purchases of assets and expenses like warranty costs have a direct impact on cash
flows. However, when stock is issued to finance an acquisition, there is no net cash involved in the
transaction.

Question: 1670

Which of the following is/are true under accrual accounting?

I. Revenues are recognized when goods are delivered.


II. Revenues are recognized when cash is received.
III. Revenues are recognized in proportion to expenditures incurred.
IV. Revenues are matched with the associated costs.

A. II & IV
B. I, III & IV
C. II & III
D. I & IV

Answer: D

Explanation:
III is not necessarily true. It is used only by a few specific firms under a method called "Percentage
completion of contract."

Question: 1671

Which of the following is/are revenue recognition methods?

Page | 613
I. Cost Recovery Method
II. Installment Method
III. Sales Cost Method
IV. Successful Efforts Method

A. II, III & IV


B. I & II
C. I, II & III
D. III & IV

Answer: B

Explanation:
IV refers to a method under which costs in drilling oil wells can be capitalized and is not a revenue
recognition method.

Question: 1672

Current liabilities are recorded at ________.

A. their market value


B. their present value
C. their maturity value
D. the lower of cost or market value

Answer: C

Explanation:
While all assets and liabilities, for analysis purposes, should be considered at their fair present value,
the short horizon of current liabilities means that in practice, they are recorded at their face value.
This means that the stated principal value of the liability is recorded as is, without factoring in
interests expenses. This practice covers current liabilities arising from both operating activities (like
advance payments and taxes payable) and financing activities (like accounts payable and short-term
loans).

Question: 1673

In its 1996 income statement, Coopers Co. reported income before income taxes of $300,000.
Coopers estimated that, because of permanent differences, taxable income for 1996 would be
$280,000. During 1996, Coopers made estimated tax payments of $50,000, which were charged to
income tax expense. Coopers is subject to a 30% tax rate. What amount should Coopers report as
income tax expense?

A. $84,000
B. $40,000
C. $34,000
D. $90,000
E. $50,000That answer is correct!

Answer: A

Explanation:
Income tax expense is the sum of current income tax and deferred tax expense. There is no timing

Page | 614
difference in this case and, therefore, no deferred tax expense. Thus, the income tax expense equals
current income tax expense which is the amount of taxes payable and equal to $84,000 ($280,000 X
30%).

Question: 1674

A common size balance sheet shows all quantities as a percentage of ________ while an income
statement uses ___________ as the base amount.

A. sales, assets
B. assets, sales
C. equity, sales
D. assets, equity

Answer: B

Explanation:
A common size balance sheet shows all quantities as a percentage of assets while an income
statement uses sales as the base amount.

Question: 1675

Which of the following are considered when applying generally accepted accounting principles.

A. When income is recognized as earned.


B. How assets are measured.
C. All of these answers are correct.
D. When expenses and losses accrue.
E. When liabilities are incurred.

Answer: C

Explanation:
The rules or standards of accounting determine such matters as how assets are measured, when
liabilities are incurred, when income is recognized as earned, and when expenses and losses accrue.

Question: 1676

At the end of a fiscal period, any revenue that has been earned but not received should be credited
to an appropriate:

A. expense account
B. revenue account
C. liability account
D. asset account

Answer: B

Explanation:
Revenue should be recognized in the period in which it is earned and is credited to the appropriate
revenue account.

Question: 1677

Page | 615
A firm uses LIFO for inventory costing. The beginning inventory balance of the firm was 700. During
the period, it purchased inventory worth 320 and sold goods worth 270. The market value of the
inventory at the end of the period was 650. The total inventory expense during this period was
________.

A. 370
B. 270
C. 320
D. 340That answer is correct!

Answer: A

Explanation:
In this problem, you have to recognize the fact that inventory is valued at the end of an accounting
period at the lower of cost or market (LCM). The write-down that occurs if the market value is lower
than cost is charged to the income. In this case, the COGS equals 270 (given). If the LCM were
ignored, the ending inventory would have been valued at 700 + 320 - 270 = 750. However, due to the
lower market value of 650, inventory gets written down to 650 and the loss of 100 in inventory is
charged to income. Thus, the total inventory expense equals 270 + 100 = 370.

Question: 1678

Tasty Food, Inc.'s statements have overstated accounts payable by 400 and understated the accounts
receivables by 225. Then, which of the following is/are true?

I. Its income is correctly stated.


II. Its income is overstated by 175.
III. Its current assets are overstated by 225.
IV. Its operating cash flows are overstated by 175.

A. II, III and IV


B. I & III
C. I & IV
D. II & IV

Answer: C

Explanation:
Current assets are understated by 225. The reported income is not affected since the errors are in the
balance sheet items. The operating cash flow is overstated by 400-225=175. Note that current
liability is overstated by 400.

Question: 1679

On Jan 1, a firm had 700 stocks outstanding. On May 1, it issued 300 more shares. On October 1, it
repurchased 100 shares. If the firm's net income was 12,000 and it paid a preferred dividend of
1,400, its reported EPS is ________.

A. 12
B. 10.6
C. 11.77
D. 12.1

Answer: D

Page | 616
Explanation:
For a simple capital structure, EPS = (Net Income - Preferred stock dividends)/weighted # of common
shares To calculate the weighted # of shares, note that 600 shares remained outstanding for 12
months, 100 shares for 9 months and 300 shares for 8 months. Therefore, weighted shares =
12/12*600 + 9/12*100 + 8/12*300 = 875. EPS = (12,000 - 1,400)/875 = 12.11.

Question: 1680

Which of the following is/are true about pension plans?

I. In a defined benefit plan, the investment risks are borne by the employer.
II. In a defined contribution plan, the investment risks are borne by the employee.
III. The employee has investment flexibility with defined contribution plan.

A. I & II
B. II only
C. I, II & III
D. I & III

Answer: C

Explanation:
In a defined benefit contribution plan, the employee makes definite contributions to the pension
plan from the regular salary. Investment of these contributions is at the discretion of the employee
and the size of the benefits paid out depends upon the total contributions made and the fund
performance. Thus, in this plan, the risk of pension plan performance is borne by the employee and
not the employer.

Question: 1681

Regarding financial accounting for public companies, the role of the SEC as currently practiced is to

A. make rules and regulations regarding filings with the SEC but not to regulate annual or quarterly
reports to shareholders.
B. develop and promulgate most generally accepted accounting principles.
C. make rules and regulations pertaining more to disclosure of financial information than to the
establishment of accounting recognition and measurement principles.
D. none of these answers.
E. regulate financial disclosures for corporate, state, and municipal reporting.

Answer: C

Explanation:
The SEC has authority to regulate external financial reporting. However, its role is to promote
disclosure rather than to exercise its power to establish accounting recognition and measurement
principles. Its objective is to allow the accounting profession (through FASB) to establish accounting
principles.

Question: 1682

Which of the following methods is the best way of conducting a comparative financial statement
analysis?

Page | 617
A. Comparing of a company's balance sheet to its income statement.
B. All of these methods are equally good.
C. Break-even analysis.
D. An index-number trend series analysis.
E. A cash flow forecast.

Answer: D

Explanation:
An index-number trend series analysis is quite useful when comparing financial statements that
cover more than two or three periods with a year-to-year change analysis. The year-to-year change
analysis method can be cumbersome when attempting to compare three or more periods.

Question: 1683

A firm's financial statements reveal the following data:

operating profit margin41%


interest expense ratio4.2%
debt-to-equity ratio0.45
total asset turnover1.6
tax rate45%

The firm's ROE equals ________.

A. 28.91%
B. 48.97%
C. 19.68%
D. 33.23%

Answer: B

Explanation:
ROE = Net income/Equity
Now, Net Income = (Operating profits - interest expense)*(1 - tax rate)
Therefore, ROE = (Operating profits/equity - interest expense/equity) *(1 - tax rate) interest
expense/equity = (interest expense/assets)*(assets/equity) = interest expense ratio*(1+debt/equity)
= 4.2%*(1+0.45) = 6.09%
Operating profits/equity = (Operating profits/sales)*(sales/assets)*(assets/equity) = 41%*1.6*1.45 =
95.12%
Thus, ROE = (95.12% - 6.09%)*(1-45%) = 48.97%

Question: 1684

Which of the following temporary differences will result in a deferred tax asset?

A. Installment sale profits accounted for on the accrual basis for financial statement purposes and on
a cash basis for income tax purposes.
B. Prepaid expenses accounted for on the accrual basis for financial statement purposes and on a
cash basis for income tax purposes.
C. Use of the straight-line depreciation method for financial statement purposes and the Modified
Accelerated Cost Recovery System (MACRS) for income tax purposes.
D. None of these answers.
E. Advance rental receipts accounted for on the accrual basis for financial statement purposes and on

Page | 618
a cash basis for tax purposes.

Answer: E

Explanation:
A deferred tax asset records deferred tax consequences attributable to deductible temporary
differences. Advance rental receipts accounted for on the accrual basis for accounting purposes and
on a cash basis for tax purposes would give rise to a deferred tax asset. The income statement would
show no revenue and no related tax expense. However, on the tax return, the cash received is
reported as revenue and the tax would be due now, resulting in a prepaid tax on the balance sheet
called a deferred tax asset.

Question: 1685

Jumbotron's accountant has mistakenly understated depreciation by 125 and understated accounts
payables by 65. Jumbotron's tax rate is 40%. Which of the following is/are true?

I. Assets are overstated by 125


II. Income is overstated by 125
III. Current liabilities are understated by 65
IV. Operating cash flow is understated by 125

A. III only
B. I & III
C. none of them
D. II, III & IV

Answer: B

Explanation:
If depreciation is understated by 125, the income is actually overstated by 125*(1-tax rate) = 75. The
understatement of accounts payables does not affect the income statement; it understates the
reported current liabilities. It also affects the operating cash flow. Note that operating cash flow = net
income + noncash expenses - non-cash revenues - cash reductions in operating accounts Since
income is overstated by 75, a non-cash expense (depreciation) is understated by 125 and another
non-cash expense (accounts payable) is understated by 65, operating cash flow is understated by -75
+ 125 + 65 = 115.

Question: 1686

When merchandise inventory is purchased under a periodic system, which account is debited?

A. Accounts Payable
B. Cash
C. Merchandise Inventory
D. Purchases

Answer: D

Explanation:
The periodic inventory system records purchases of merchandise inventory in an income statement
account called Purchases.

Question: 1687

Page | 619
The section of a firm's annual report entitled "Management's Discussion and Analysis" (MD&A)

A. is a technical analysis of past results and a defense of those results of management.


B. includes the company president's letter.
C. covers three financial aspects of a firm's business: liquidity, capital resources, and results of
operations.
D. covers marketing and product line issues.
E. none of these answers.

Answer: C

Explanation:
The MD&A is included in SEC filings. It addresses in a nonquantified manner the prospects of a
company. The SEC examines it with care to determine that management has disclosed material
information affecting the company's future results.

Question: 1688

Which of the following is/are FALSE?

I. Depreciation does not affect cash flows since it is a non-cash expense.


II. Depreciation does not affect the cash flow statement since it is an allocation of past investing
expense.
III. Under US GAAP, companies can use different methods of depreciation for financial reporting and
tax purposes.

A. III only
B. I & II
C. II & III
D. I, II & III

Answer: B

Explanation:
Depreciation does affect cash flows since it reduces the amount of income tax paid. Further, US firms
have the latitude of choosing an accelerated method of depreciation for tax purposes and a different
method (like straight-line method) for financial reporting. This flexibility has an important effect on
the reported balance sheet, which has to recognize an explicit liability in the form of deferred taxes
payable.

Question: 1689

In a period of falling prices, the inventory method that gives the lowest possible value for ending
inventory is

A. FIFO
B. LIFO
C. gross profit
D. weighted averageThat answer is correct!

Answer: A

Explanation:

Page | 620
The first-in-first-out (FIFO) method is based on the assumption that the costs of the first items
acquired should be assigned to the first items sold, therefore ending inventory on hand is based on
the most recent prices.

Question: 1690

The main purpose of the statement of financial position is to reflect

A. the market value of the firm's assets at some point in time.


B. the status of the firm's assets in case of forced liquidation of the firm.
C. items of value, debts and net worth.
D. none of these answers.
E. the firm's potential for growth in stock values in the stock market.

Answer: C

Explanation:
The Statement of Financial position or balance sheet presents 3 major elements: assets (items of
value), liabilities (debts)), and shareholders' equity (net worth).

Question: 1691

The primary purpose of the statement of cash flows is to

A. provide information about a company's cash receipts and cash payments during the accounting
period
B. measure the change in the company's assets
C. state the company's financial position at period-end
D. analyze net income during the accounting periodThat answer is correct!

Answer: A

Explanation:
Other financial statements are used to analyze revenues, expenses, assets, liabilities, and
stockholders' equity. The statement of cash flows summarizes transactions affecting cash; it is the
balance sheet that states the company's financial position at a point in time.

Question: 1692

A firm has preferred equity outstanding but has not declared payment of dividend on it. If the firm's
net income is 1000 and there are 100 common shares outstanding, the firm's Basic EPS is ________.

A. equal to 10
B. insufficient information
C. less than 10
D. more than 10

Answer: C

Explanation:
Even when preferred dividends have not been declared, they must be subtracted from net income
while calculating Basic EPS since no dividends can be paid on common share unless the preferred
dividends are paid. Basic EPS includes only the amount that is available for distribution on common
shares.

Page | 621
Question: 1693

In a period of falling prices, the inventory method that gives the lowest possible value for ending
inventory is:

A. gross profit
B. FIFO
C. LIFO
D. weighted average

Answer: B

Explanation:
The first-in-first-out (FIFO) method is based on the assumption that the costs of the first items
acquired should be assigned to the first items sold, therefore ending inventory on hand is based on
the most recent prices.

Question: 1694

According to SFAS 95, cash and cash equivalents includes

A. bank accounts, U.S. Treasury Bills and fixed income securities.


B. risk-free assets with original maturities of 90 days or less.
C. risk-free assets with original maturities of 30 days or less.
D. bank accounts, U.S. Treasury Bills and marketable securities.
E. only bank accounts.

Answer: B

Explanation:
Bank accounts, U.S. Treasury Bills, and similar assets qualify.

Question: 1695

The following is true about the price earnings ratio

A. it is the ratio between the company's current market value and its earnings per share
B. none of these answers
C. the ratio is calculated based on the earnings per share from the past year
D. stocks with price earnings ratios of 25 or less are considered to be underpricedThat answer is
correct!

Answer: A

Explanation:
PE ratio is calculated based on expected earnings per share and a ratio of less than 5 to 8 makes a
stock appear underpriced.

Question: 1696

According to U.S. GAAP, the format of the income statement

A. must include footnotes which detail particular income statement sections.

Page | 622
B. must not vary across different industries.
C. must include an "operating expenses" section.
D. must be identical in the reporting of equity in earnings of affiliates.
E. is not specified.

Answer: E

Explanation:
U.S. GAAP do not specify the format of the income statement. Actual formats vary across firms,
especially in the reporting of equity in earnings of affiliates, and nonoperating income and expense.

Question: 1697

If a firm's net income after dividends is greater than zero, then

I. its equity increases.


II. its retained earnings increase.
III. its liabilities decrease.
IV. its stock price increases.

A. I, II & IV
B. I & IV
C. I & II
D. II only

Answer: C

Explanation:
The income not paid out as dividends becomes part of retained earnings, which is a component of
equity. A firm's market value is not determined solely by its current book value but expected future
earnings. If the current period's income falls short of expectations, then the stock price will fall even
if nothing else changes simply because the unrealized part of the income was factored into the
original price. A further complication arises if the market interprets the fall in income as an
unfavorable signal about future earnings. Thus, while book value of the stock increases, the market
value could increase or decrease.

Question: 1698

The following information should be used according to the provisions of SFAS 95 (Statement of Cash
flows) and using the following data.

Net Income$50,000
Provision for bad debts$2,000
Increase in Inventory$1,000
Increase in accounts payable$2,000
Purchase of new equipment$15,000
Sale of equipment for $10,000 gain$20,000
Depreciation expense$5,000
Repurchase of common stock$10,000
Payment of dividend$4,000
Interest payment$3,000

What is net cash flow from financing?

Page | 623
A. $6,000
B. ($17,000)
C. $3,000
D. ($14,000)

Answer: D

Explanation:
($14,000) = ($10,000) for repurchase of stock and ($4,000) for payment of dividend

Question: 1699

Which of the following could cause a firm's equity position to be weaker than is reflected in the
balance sheet.

A. All of these answers.


B. Holding held-to-maturity securities in a portfolio with non-amortized discounts.
C. Holding available-for-sale securities in a portfolio that have unrealized losses.
D. None of these answers.
E. Holding trading securities in a portfolio with unrealized gains.

Answer: C

Explanation:
Evaluating available-for-sale securities in a portfolio can be challenging to the analyst. A firm might
hold a variety of securities in a portfolio for which the unrealized gains of one or more securities
offsets the unrealized losses of other securities. While these values offset each other, the security
that has an unrealized loss might be a high-grade investment that is not subject to much fluctuation.
On the contrary, the securities that have unrealized gains might be highly volatile securities that
were at all time highs at the close of the firm's fiscal period but will these gains be sustainable? It is
conceivable then that the firm's equity position might be a bit weaker that the balance sheet
suggests.

Question: 1700

In a statement of cash flows done using the indirect method all of the following would be included as
cash flows from investing activities except for:

A. purchase of fixed assets.


B. sale of fixed assets.
C. all would be included as cash flows from investing activities.
D. gain on sale of equipment.

Answer: D

Explanation:
Gains and losses on the sale of fixed assets are included in cash flows from operating activities. The
amount included in the investing activities area is the amount of the asset on the books of the
company.

Question: 1701

A firm has net total sales of 16,000 and administrative cash expense of 4,500. It paid taxes in the
amount of 2,300 and had an increase in accounts payable of 400. The depreciation expense was

Page | 624
estimated at 1,200. Then, the firm's operating cash flow was ________.

A. 9,600
B. 8,000
C. 9,200
D. none of these answersThat answer is correct!

Answer: A

Explanation:
The increase in accounts payable implies that some of the operating expenses were on credit and did
not cause a cash outflow. Therefore, this increase must be added to the net sales. Further,
depreciation should be ignored since it is a non-cash expense and its effect on taxes is already
impounded in the taxes of 2,300. Hence, the total operating cash flow = 16,000 + 400 - 4,500 - 2,300
= 9,600

Question: 1702

Which of the following could increase outstanding capital stock?

A. Conversion of bonds.
B. All of these answers.
C. The exercise of stock options.
D. The exercise of warrants.
E. The payment of stock dividends.

Answer: B

Explanation:
All of these answers would cause a firm's capital stock to increase. While the exercise of stock
options or warrants offers a price to the buyer that is more favorable than the firm's current market
price, these still result in the issuance of new shares. Likewise, the conversion of bonds to stock
swaps debt for equity and the payment of stock dividends means that the firm is issuing new shares.

Question: 1703

A firm's operating cash flow is overstated by 90. Its non-cash expenses were correctly stated,
noncash revenues were overstated by 55 and a total of 35 went into reducing outstanding current
debt. The firm's tax rate is 40%. Then, the firm's net income is ________.

A. overstated by 55
B. overstated by 20
C. understated by 55
D. correctly stated

Answer: D

Explanation:
operating cash flow = net income + noncash expenses - non-cash revenues - cash reductions in
operating accounts

Question: 1704

Which of the following statements about the Securities and Exchange Commission (SEC) is true?

Page | 625
A. The SEC must audit all financial statements of publicly traded corporations.
B. The SEC has issued specific rules and regulations concerning the preparation of financial
statements and the degree of detail that they contain.
C. None of these statements are true.
D. The SEC functions as the standard-setting body of the accounting profession.

Answer: B

Explanation:
The Securities and Exchange Commission has issued specific rules and regulations concerning the
preparation of financial statements and the degree of detail that they contain.

Question: 1705

The following data have been extracted from the financial statements of a firm for two years, 1993
and 1994:

19931994
Assets10,89512,444
Sales8,4659,275
Inventory3,1263,549
COGS7,1207,387
Receivables2,1541,768

The receivables turnover ratio and the average receivables collection period for 1994 equal
________.

A. 4.31, 84.77 days


B. 5.25, 69.58 days
C. none of these answers
D. 4.73, 77.17 days

Answer: D

Explanation:
Receivables turnover ratio = Net annual sales/average receivables Average receivables collection
period = 365/receivables turnover. Typically, average receivables for a given year are taken to be the
average of the ending values of the receivables for this year and the last year. For 1994, the average
receivables equal (2154+1768)/2 = 1,961. Receivables turnover ratio = 9,275/1,961 = 4.73. Average
receivables collection period = 365/4.73 = 77.17 days.

Question: 1706

Which of the following is/are advantages of accelerated methods of depreciation?

I. They implicitly recognize the loss of productivity and increased maintenance costs over time.
II. They allow deferral of taxes compared to the straight-line method, thus making more cash
available for current operations.
III. The lower depreciation charges in later years compensate for the greater uncertainty in future
revenues.

A. I & III
B. II only

Page | 626
C. I, II & III
D. I & II

Answer: C

Explanation:
All three responses are correct as advantages of accelerated methods of depreciation.

Question: 1707

A firm has convertible bonds, preferred equity, common equity and straight bonds in its capital
structure. In calculating Diluted EPS, which of the following is true about the earnings number used,
assuming the convertible bonds are dilutive?

A. Earnings used = Net income - preferred dividends + interest payments on convertible


B. Earnings used = Net income - preferred dividends - interest payments on convertible net of taxes.
C. Earnings used = Net income - preferred dividends - interest payments on convertible before taxes.
D. Earnings used = Net income - preferred dividends + interest payments on convertible net of taxes.

Answer: D

Explanation:
Think of the earnings used in EPS calculations as the earnings that are available for distribution
amongst common shareholders. Under Diluted EPS, convertible bonds are assumed converted if
dilutive. Hence, for Diluted EPS purposes, convertible bond holders are assumed to be a part of the
residual claimants of the earnings available after the liabilities to debt holders and preferred equity
holders are satisfied. The earnings available for distribution amongst these security holders then
equal Net income - preferred dividends + interest payments on convertible net of taxes.

Question: 1708

Redeemable preferred stock

A. is reported in stockholders' equity of the balance sheet.


B. is reported after liabilities but before the equity section of the balance sheet.
C. must be computed at its market value on the balance sheet.
D. has higher priority for liquidation and dividends than preferred stock.
E. is required to be listed only as a footnote in the balance sheet.

Answer: B

Explanation:
The liquidation preference or redemption price should be used in the computation of book value per
share, leverage, and capital ratios.

Question: 1709

Simon Steel Inc. had the following unusual financial events occur this year: Bonds payable were
retired 5 years before their scheduled maturity, resulting in a $260,000 gain, Simon has frequently
retired bonds early when interest rates declined significantly. A steel forming segment suffered
$255,000 in losses from hurricane damage. This was the fourth similar loss sustained in a 5-year
period at that location. A segment of Simon's operations, steel transportation, was sold at a net loss
of $350,000. This was Simon's first divestiture of one of its operating segments. Before income taxes,
what amount should be disclosed as the gain (loss) from extraordinary times for this year?

Page | 627
A. $260,000
B. $(345,000)
C. $5,000
D. $(90,000)
E. $(350,000)That answer is correct!

Answer: A

Explanation:
Accounting rules require that any gain or loss from early retirement of debt must be recognized as an
extraordinary item. For all other transactions to be classified as extraordinary items, they must be
both unusual and infrequent. In this case, none of the other gains or losses qualify.

Question: 1710

A credit entry to Allowance for Uncollectible Accounts

A. increases the balance


B. neither of these answers is correct
C. both of these answers are correct
D. increases net receivablesThat answer is correct!

Answer: A

Explanation:
Allowance for Uncollectible Accounts, sometimes referred to as Bed Debt Reserve, is a contra
account to Accounts Receivable. A credit entry increases its balance and the balance reduces the
amount of net Accounts Receivable.

Question: 1711

The following is a sign that a company is a "growth company":

A. It has no earnings and does not pay a dividend.


B. It has very strong negative cash flows and borrows heavily to make up for the negative cash flows.
C. It has earnings but does not pay dividends and its employees are not overly compensated.
D. It has earnings and pays out large dividends.

Answer: C

Explanation:
Companies characterized as "growth companies" are those that do make money, but they keep the
money within the company so as to grow the company. Companies that pay dividends are not
maximizing the funds available to fund future growth. Companies that limit earnings by paying
employees exorbitant salaries are not growth companies. Companies like this are giving funds to
employees rather than holding them within the corporation to fund future growth.

Question: 1712

New Gestalt, Inc., a software firm had a net income of 1.7 million last year. It has 200,000 common
shares and 300,000 convertible bonds with face value of 100 outstanding. The convertible bonds
carry a coupon of 4% and can be converted one-for-one. The average stock price last year was 39 and
the maximum price was 57. The effective interest rate on the convertible debt is 8%. New Gestalt

Page | 628
issued 100,000 preferred shares with face value 100 and a coupon of 5% on March 31st of last year.
Assume the convertible bonds are dilutive and that New Gestalt faces a 30% tax rate. Given the
above, New Gestalt's Basic EPS equals ________.

A. 8.5
B. none of these answers
C. 6.62
D. 10.4

Answer: C

Explanation:
The preferred dividends = 100,000*5%*100*9/12 = 375,000 (they were outstanding for 9 months).
Therefore, Basic EPS = (Net Income - Preferred dividends)/weighted # of common shares =
(1,700,000 - 375,000)/200,000 = 6.62

Question: 1713

Dilutive warrants are ________ included in the calculation of Diluted EPS.

A. never
B. often
C. always
D. sometimes

Answer: C

Explanation:
Diluted EPS reflects all securities that are potential shares and which decrease earnings per share.
Also remember that all anti-dilutive securities are ignored. A dilutive security, by definition, is one
that decreases EPS. Therefore, dilutive warrants are always included in the calculation of Diluted EPS.

Question: 1714

Two otherwise identical firms, A and B, have one difference. A has a higher interest expense than B,
arising from its higher reliance on debt financing. All other accounting statistics, including net sales
and expenses are equal for the two firms. The return on total capital for firm A will be __________
that for firm B.

A. same as
B. insufficient information to answer the question
C. lower than
D. higher than

Answer: D

Explanation:
The return on total capital equals the net income before interest expense as a fraction of the average
total capital invested. Therefore, if interest expenses were not tax deductible, the two firms would
have exactly the same return on total capital. However, due to tax advantages of interest expenses,
firm A will show a higher net income and hence, a higher return on capital.

Question: 1715

Page | 629
Where is Unearned Revenue reported in the financial statements?

A. liability section of the balance sheet


B. asset section of the balance sheet
C. operating expense section of the income statement
D. revenue section of the income statementThat answer is correct!

Answer: A

Explanation:
Unearned revenue represents "services owed" and is reported in the liability section of the balance
sheet.

Question: 1716

The balance sheet of firm A shows the following:

Cash & Cash equivalents386


Receivables119
Inventories204
Marketable securities143
Short-term loans649
Current portion of long-term debt312

The cash ratio of the firm equals ________.

A. 0.55
B. 0.82
C. 1.82
D. 0.63That answer is correct!

Answer: A

Explanation:
The cash ratio is the most conservative liquidity ratio calculated using only cash and marketable
securities relative to current liabilities. Thus, Cash ratio = (cash + marketable securities)/current
liabilities In this case, Cash ratio = (386+143)/(649+312) = 0.55

Question: 1717

Which of the following are considered when applying generally accepted accounting principles.

A. When income is recognized as earned.


B. When expenses and losses accrue.
C. How assets are measured.
D. All of these answers are correct.
E. When liabilities are incurred.

Answer: D

Explanation:
The rules or standards of accounting determine such matters as how assets are measured, when
liabilities are incurred, when income is recognized as earned, and when expenses and losses accrue.

Page | 630
Question: 1718

A capital lease is recorded as ________.

A. an asset and a liability


B. a temporary asset
C. an off-balance-sheet item
D. a liabilityThat answer is correct!

Answer: A

Explanation:
According to the GAAP, leases that transfer substantially all of the benefits and risks of the property
to the lessee be treated as an asset acquisition and an assumption of liability. Hence, capital leases
generate an asset and a liability.

Question: 1719

Which of the following best describes a statement of cash flows?

A. A statement of cash flows details the cash inflows and outflows that are related to a company's
operating, investing, and financing activities over a period of time.
B. A statement of cash flows summarizes the financial position of a company at a given point in time.
C. A statement of cash flows measures a company's financial performance over a specified period of
time.
D. A statement of cash flows reports changes over a period of time in component accounts that
comprise the ownership of a firm.
E. None of these answers.That answer is correct!

Answer: A

Explanation:
Under accrual accounting, net income does not typically equal net cash flow. While net income is
important to identify, accrued income does not pay wages, payables, or dividends. Cash flows are
important to identify since it is cash that repays loans, etc.

Question: 1720

Which of the following is NOT an operating efficiency ratio?

A. Asset turnover
B. Equity turnover
C. Sales-to-assets ratio
D. Operating profit margin

Answer: D

Explanation:
Operating efficiency ratios are a measure of how well the firm is utilizing its assets and capital. This
utilization has a direct yardstick in terms of total sales generated. Hence, operating efficiency ratios
express total sales as a fraction of assets or equity. On the other hand, Operating profit margin
belongs to the group of operating profitability ratios.

Question: 1721

Page | 631
Which of the following is/are TRUE?

I. Accrual accounting follows the matching principle.


II. In accrual accounting, revenues and cash flows are distinct concepts.
III. In cash accounting, net income equals cash inflow.
IV. In cash accounting, earnings equal net operating cash inflow.

A. I, II & IV
B. I, II,
C. III & IV
D. I, II & IIIThat answer is correct!

Answer: A

Explanation:
In cash accounting, net income equals net operating cash inflow (total operating cash inflow - total
operating cash outflow) and is the same as earnings.

Question: 1722

An end-of-period adjustment for depreciation of fixed assets is necessary

A. proper statement of net income


B. to be consistent with the matching principle
C. all of these answers are correct
D. to recognize the expense of using fixed assets

Answer: C

Explanation:
An adjustment for depreciation expense matches expenses with revenues for the period , thus
contributing to correct statement of net income for the period.

Question: 1723

Under the Percentage-of-completion method of recognizing contract revenue, common methods of


estimation of completion include all of the following except:

A. Ratio of units completed to total units expected to be completed.


B. Ratio of costs incurred by expected total costs.
C. Ratio of profits of the company year to date to expected profits for the year of the company.
D. Ratio of units delivered to total units expected to be delivered.

Answer: C

Explanation:
This method attempts to match expenses with revenues and profits. Revenues recognized would not
be based on the expected profits for the company during the year.

Question: 1724

The following data are available for a firm for a given year:

Page | 632
Net Sales21,896
Sales & marketing expenses4,346
Administrative expenses2,143
COGS10,084
Depreciation967
Interest expense573
Tax rate35%
Dividends paid3,445
Preferred Dividends897
Average total equity37,432
Average common equity26,782
Average total liabilities18,583

The firm's gross profit margin equals ________.

A. 0.45
B. 0.39
C. 0.24
D. 0.54

Answer: D

Explanation:
Gross Profit = 21,896 - 10,084 = 11,812 and
Gross Profit margin = 11,812/21,896 = 0.54 = 54%.

Question: 1725

Under what condition would a firm report marketable securities as a long-term asset?

A. None of these answers.


B. When funds are set aside for a specific long-term purpose such as plant expansion.
C. When the value of a firm's investment in marketable securities is less than cost.
D. All of these answers.
E. When a firm maintains excess trading securities.

Answer: B

Explanation:
A firm may maintain funds invested in marketable securities that are earmarked for plant expansion,
sinking fund payments, or other long-term uses. In these situations many firms would report this
portion of its cash, or cash equivalents as long-term because these invested funds are not available in
the daily operation of the business.

Question: 1726

A firm's financial statements reveal the following data:

interest coverage ratio3.6X


interest expense rate7.2%
depreciation678
total assets10,946
total asset turnover1.46

Page | 633
The firm's operating margin equals ________.

A. 13.84%
B. 23.18%
C. 21.99%
D. 25.93%

Answer: C

Explanation:
Interest Coverage = EBIT/interest expense
Operating Profit Margin = EBDIT/Net sales
Interest expense rate = interest expense/total assets
Total Asset turnover = sales/total assets
Therefore, interest expense = 7.2%*10,946 = 788 and EBIT = 3.6*788 = 2,837.
So EBDIT = EBIT + depreciation = 2,837 + 678 = 3,515. Also, net sales = 1.46*10,946 = 15,981. This
gives Operating Profit Margin = 3,515/15,981 = 21.99%.

Question: 1727

The following data are available for a firm for a given year:

Net Sales21,896
Sales & marketing expenses4,346
Administrative expenses2,143
COGS10,084
Depreciation967
Interest expense573
Tax rate35%
Dividends paid3,445
Preferred Dividends897
Average total equity37,432
Average common equity26,782
Average total liabilities18,583

In the above example, the firm's operating profit margin equals ________.

A. 0.24
B. 0.35
C. 0.18
D. 0.54That answer is correct!

Answer: A

Explanation:
Operating Profit Margin = 21,896 - 10,084 - 4,346 - 2,143 = 5,323. (Earnings before depreciation,
interest, and taxes as a % of sales; EBDIT). Therefore, Operating Profit Margin = 5,323/21,896 = 0.24.

Question: 1728

The Income Summary account

A. is used to facilitate the closing process


B. all of these answers are correct

Page | 634
C. is a temporary account
D. should have a zero balance at period-end

Answer: B

Explanation:
The Income Summary account is a temporary account created especially for the closing process and
is used only for flowing adjustments and closing entries through.

Question: 1729

Depreciation expense for fixed assets is recorded

A. for each period the asset is in use


B. as a liability until the asset is sold
C. at the end of each fiscal period during the asset's useful life
D. when the asset is sold

Answer: C

Explanation:
Depreciation is the periodic allocation of the cost of a tangible, long-term asset over its estimated
useful life, and therefore a period expense. This expense is recorded at the end of each fiscal period
until the asset is fully depreciated or disposed of.

Question: 1730

A firm has long-term assets worth 3,812 and total assets worth 5,937. It has a total equity of 3,934
and long term debt of 879. The firm's current ratio equals ________.

A. 1.32
B. 0.92
C. 1.03
D. 1.89

Answer: D

Explanation:
The firm's current assets equal 5,937 - 3,812 = 2,125. The total liabilities equal total assets - total
equity = 5,937 - 3,934 = 2,003. Thus, the current liabilities equal 2,003 - 879 = 1,124. The current ratio
is then equal to current assets/current liabilities = 2125/1124 = 1.89

Question: 1731

Marine Corp. uses the direct method to prepare its statement of cash flows. Marine's trial balance at
December 31, 1996 and 1995 are as follows:

Dec. 31, 1996 Dec. 31, 1995

Debits:

Cash$35,000$32,000
Accounts receivable33,00030,000
Inventory31,00047,000

Page | 635
Property, plant & equipment100,00095,000
Unamortized bond discount4,5005,000
Cost of goods sold250,000380,000
Selling expenses141,500172,000
General and administrative expenses137,000151,300
Interest expense4,3002,600
Income tax expense20,40061,200

Total debits$756,700$976,100

Credits:

Allowance for doubtful accounts$1,300$1,100


Accumulated depreciation16,50015,000
Trade accounts payable25,00017,500
Income taxes payable21,00027,100
Deferred income taxes5,3004,600
8% callable bonds payable45,00020,000
Common stock50,00040,000
Additional paid-in capital9,1007,500
Retained earnings44,70064,600
Sales538,800778,700

Total credits$756,700$976,100

Marine purchased $5,000 in equipment during 1996. Marine allocated one-third of its depreciation
expense to selling expenses and the remainder to general and administrative expenses.

What amount should Marine report in its statement of cash flows for the year ended December 31,
1996 for cash paid for interest?

A. $1,700
B. $3,800
C. $3,600
D. $4,300
E. $4,800

Answer: B

Explanation:
Interest expense is $4,300 and was all paid since there is no interest payable account. However, since
there is an account for unamortized bond discount, the reduction of $500 between the two years
represents discount amortization, a noncash item. Therefore $4,300 less $500 (or $3,800) was paid in
cash for interest.

Question: 1732

Why are certain costs of doing business capitalized when incurred and then depreciated or amortized
over subsequent accounting periods.

A. To reduce the federal income tax liability.


B. To adhere to the concept of conservatism.
C. To aid management in the decision-making process.
D. To match the costs of production with the revenues as earned.

Page | 636
E. To maximize accounting income.

Answer: D

Explanation:
If costs benefit more than one accounting period, they should be systematically and rationally
allocated to all periods benefited. Matching refers to the process of recognizing expenses over the
periods where revenues have benefited from these expenses.

Question: 1733

Which of the following is/are TRUE?

I. Losses due to union strike at a plant are classified as extraordinary items.


II. Unusual and infrequent items appear as part of income from continuing operations.
III. Gains from debt retirement are classified as extraordinary items.
IV. The loss from the sale of a portion of business segment is included in income from continuing
operations.

A. II & III
B. I & IV
C. I & II
D. III & IV

Answer: D

Explanation:
Extraordinary items are those items are those which are unusual in nature and infrequent in
occurrence. Extraordinary items appear segregated from the income from continuing operations on
the Income Statement. Therefore, II is false. III is true under current GAAP. Losses due to a strike or
the sale of a portion of a business segment are considered unusual but not infrequent and hence, do
not qualify as extraordinary items.

Question: 1734

Which of the following is an operating cash flow?

A. Dividends paid out to shareholders.


B. Dividends received on investments in stocks.
C. All of these answers.
D. Cash used to replace the machine tool used in operational activities.

Answer: B

Explanation:
Dividends received on investments in stocks are classified as operating cash flows while dividends
paid out to shareholders are financing cash flows. Cash used to replace the machine tool used in
operational activities is an investing cash flow.

Question: 1735

When a plant asset is sold for more than its book value:

A. cash received plus accumulated depreciation plus gain on disposal equals the original cost.

Page | 637
B. cash received plus accumulated depreciation minus gain on disposal equals the original cost.
C. book value of the asset minus gain on disposal equals cash received.
D. original cost minus accumulated depreciation equals cash received plus gain on disposal.

Answer: B

Explanation:
Gain or loss on disposal of a fixed asset is calculated by subtracting the book value (Original cost -
Accumulated Depreciation) from the cash received.

Question: 1736

A firm's quick ratio equals 1.31 and its cash ratio equals 1.19. If the firm has receivables of 693, its
current liabilities equal ________.

A. 8,129
B. 4,693
C. 5,775
D. 7,926

Answer: C

Explanation:
The definitions of the two ratios are:
Quick ratio = (cash + marketable securities +receivables)/current liabilities
Cash ratio = (cash + marketable securities)/current liabilities
Thus, (Quick ratio - Cash ratio) = receivables/current liabilities.
Therefore, 1.31 - 1.19 = 0.12 = 693/current liabilities. Solving, we get current liabilities = 693/0.12 =
5,775.

Question: 1737

The following information is from the financial statements of Complex Capitalists for 1997 and 1998:

Dec. 31, 1997 - 1 million common shares outstanding, capital structure all-equity
March 31, 1998 - issued 200,000 common shares.
May 31, 1998 - issued 800,000 warrants exercisable at a strike of 35.

The stock price on Dec. 31, 1998 was 37. The average price during 1998 was 34 and the maximum
price was 39. How many shares will the basic EPS computation for 1998 use?

A. 2.0 million
B. 1.15 million
C. 1.2 million
D. 1.617 million

Answer: B

Explanation:
Basic EPS will ignore the warrants and calculate the weighted number of common stock outstanding
for 1998. The newly issued share were outstanding for 9 months and hence weighted # of shares
used = 1 million + 3/4*200,000 = 1.15 million.

Question: 1738

Page | 638
Miller Mining, a calendar-year corporation, purchased the rights to a copper mine on July 1, 1996. Of
the total purchase price, $2,800,000 was appropriately allocated to the copper. Estimated reserves
were 800,000 tons of copper. Miller expects to extract and sell 10,000 tons of copper per month.
Production began immediately. The selling price is $25 per ton. Miller uses percentage depletion
(15%) for tax purposes. To aid production, Miller also purchased some new equipment on July 1,
1996. The equipment cost $76,000 and had an estimated useful life of 8 years. After all the copper is
removed from the mine, however, the equipment will be of no use to Miller and will be sold for an
estimated $4,000. If sales and production conform to expectations, what is Miller's depletion
expense on this mine for financial accounting purposes for the calendar year 1996?

A. $420,000
B. $215,400
C. $105,000
D. $210,000
E. $430,800

Answer: D

Explanation:
Depletion expense is based on the units-of-production method. Based on the estimates, the
depletion charge per ton is $3.50 ($2,800,000 / 800,000 tons). Since 10,000 tons are extracted per
month and there are 6 months of operations for 1996, the depletion expense is $210,000 ($3.50 per
ton X 10,000 tons X 6 months).

Question: 1739

In preparing its cash flow statement for the year ended December 31, 1998, Roman Co. collected the
following data:

Gain on sale of equipment$6,000


Proceeds from sale of equipment10,000
Purchase of A.S. Inc. bonds (par value $200,000)180,000
Amortization of bond discount2,000
Dividends declared45,000
Dividends paid38,000
Proceeds from sale of treasury stock
(carrying amount of $65,000)75,000

In its December 31, 1998 statement of cash flows, what amount should Roman report as net cash
used in investing activities?

A. $194,000
B. $188,000
C. $170,000
D. $174,000
E. $176,000

Answer: C

Explanation:
Investing activities include the acquisition and disposal of all long-term assets. Thus, the proceeds
from sale of equipment of $10,000 and purchase of bonds of $180,000 result in a net use of cash of
$170,000 from investing ($10,000-180,000).

Page | 639
Question: 1740

The completed contract method

A. may understate gross profit.


B. generally must be used when no contract exists.
C. cannot be used for short-term contracts.
D. may be used for long-term contracts at the option of the company.
E. recognizes revenues and costs in proportion to the work completed.

Answer: B

Explanation:
The completed contract method recognizes revenues and expenses only at the end of the contract. It
must be used when any one of the conditions required for use of the percentage-of-completion
method is not met.

Question: 1741

All of the following are typically fixed expenses except for ________.

A. raw materials
B. rent
C. insurance
D. property taxesThat answer is correct!

Answer: A

Explanation:
Fixed expenses are costs that are relatively constant and costs that would be difficult to match
directly with units being sold.

Question: 1742

An end-of-period adjustment for depreciation of fixed assets is necessary:

A. to recognize the expense of using fixed assets


B. all of these answers
C. proper statement of net income
D. to be consistent with the matching principle

Answer: B

Explanation:
An adjustment for depreciation expense matches expenses with revenues for the period, thus
contributing to correct statement of net income for the period.

Question: 1743

A ratio that measures the average portion of each dollar of revenue that results in profit is

A. profit margin
B. both of these answers are correct

Page | 640
C. neither of these answers is correct
D. return on sales

Answer: B

Explanation:
Profit margin, also called return on net sales, is calculated by dividing net income by net sales. This
ratio measures the average portion of each dollar of revenue that ends up as profit.

Question: 1744

The following information should be used according to the provisions of SFAS 95 (Statement of Cash
flows) and using the following data.

Net Income$50,000
Provision for bad debts$2,000
Increase in Inventory$1,000
Increase in accounts payable$2,000
Purchase of new equipment$15,000
Sale of equipment for $10,000 gain$20,000
Depreciation expense$5,000
Repurchase of common stock$10,000
Payment of dividend$4,000
Interest payment$3,000

What is net cash flow from investing?

A. $10,000
B. ($15,000)
C. ($5,000)
D. $5,000

Answer: D

Explanation:
Outflow of $15,000 for new equipment and $20,000 inflow from sale.

Question: 1745

The portion of the supplies inventory that has been consumed during the fiscal period is classified as

A. an expense
B. an increase in retained earnings
C. a liability
D. an assetThat answer is correct!

Answer: A

Explanation:
In accordance with the matching rule, the amount of an asset such as supplies that has been
consumed during the accounting period must be recorded and classified as an expense for the
period.

Question: 1746

Page | 641
New Gestalt, Inc., a software firm had a net income of 1.7 million last year. It has 200,000 common
shares and 300,000 convertible bonds with face value of 100 outstanding. The convertible bonds
carry a coupon of 4% and can be converted one-for-one. The average stock price last year was 39 and
the maximum price was 57. The effective interest rate on the convertible debt is 8%. New Gestalt
issued 100,000 preferred shares with face value 100 and a coupon of 5% on March 31st of last year.
Assume the convertible bonds are dilutive and that New Gestalt faces a 30% tax rate. Given the
above, if New Gestalt had 300,000 warrants with a strike of 36 outstanding instead of the convertible
bonds, the number of shares used in Diluted EPS would equal ________.

A. 200,000
B. 223,077
C. 312,648
D. 500,000

Answer: B

Explanation:
Since the strike price of 36 on the warrants is less than the average stock price of 39, the warrants are
dilutive. Using the Treasury stock method, the additional shares due to warrant exercise =
300,000*(1 - 36/39) = 23,077. Total number of shares used in Diluted EPS = 200,000 + 23,077 =
223,077.

Question: 1747

Which of the following best describes an income statement?

A. It reports revenues and expenses for a specific accounting period.


B. It reports the amount and composition of assets and liabilities for a specific accounting period.
C. None of these answers.
D. It reports investment activities for a specified accounting period.
E. It reports cash receipts and cash disbursements for a specific accounting period.That answer is
correct!

Answer: A

Explanation:
An income statement measures a company's financial performance between balance sheet dates
and, hence, reflects a period of time. It lists revenues, expenses, gains, and losses of a company over
a time period.

Question: 1748

All of the following are components of Shareholder's equity EXCEPT ________.

A. par value
B. dividends paid
C. retained earnings
D. capital contributed in excess

Answer: B

Explanation:
Dividends declared are part of equity.

Page | 642
Question: 1749

Most components of the balance sheet are reported at

A. historical cost plus allowance for inflation.


B. fair market value.
C. historical cost.
D. replacement value.

Answer: C

Explanation:
Most components of the balance sheet are reported at historical cost, that is, the exchange price at
their acquisition date.

Question: 1750

Holiday Corp. holds 10,000 shares of its $10 par value common stock as treasury stock reacquired in
1994 for $120,000. On December 12, 1996, Holiday reissued all 10,000 shares for $190,000. Under
the cost method of accounting for treasury stock, the reissuance resulted in increasing

A. additional paid-in capital by $70,000.


B. treasury stock by $100,000.
C. capital stock by $100,000.
D. retained earnings by $70,000.
E. gain on sale of investments by $70,000.That answer is correct!

Answer: A

Explanation:
Under the cost method, treasury stock is decreased by $120,000, cash is increased by $190,000 and
the difference (which is like a gain) is reported as an increase in additional paid-in capital. Accounting
rules do not allow a firm to record a gain or loss when it buys back treasury stock because the
transaction is viewed as an equity transaction.

Question: 1751

Which of the following would occur prior to the completion of a long-term contract when using the
completed-contract method as opposed to the percentage-of-completion method?

A. A firm's current period income tax liability would be lowered.


B. A firm's current period profits would be greater.
C. None of these answers.
D. All of these answers.
E. A firm's current period revenues would be greater.That answer is correct!

Answer: A

Explanation:
A firm's income tax liability is lower under the competed-contract method because revenues and
related income taxes on profits are deferred until the contract is completed. The percentage-of-
completion method would recognize revenues earlier and would generate higher current period
income tax liabilities.

Page | 643
Question: 1752

A liability can be recognized when

A. an obligation exists to make a future payment based on a past event


B. any time a future payment is due
C. only when the amount is certain
D. all of these answers are correctThat answer is correct!

Answer: A

Explanation:
The amount need not be certain and some future payments such as wages to be paid in the future
may not be reported as liabilities.

Question: 1753

Profit margin is a ratio that:

A. shows the return on net sales


B. is calculated as net sales divided by operating expenses
C. yields the company's financial position at a point in time
D. compares total assets to net salesThat answer is correct!

Answer: A

Explanation:
Profit margin, also called return on net sales, is calculated by dividing net income by net sales. This
ratio measures the average portion of each dollar of revenue that ends up as profit.

Question: 1754

The Income Statement:

I. reflects the current operating performance of the firm.


II. indicates whether the firm is healthy and growing or not.
III. explains the changes in assets, liabilities and equity of the firm.
IV. is a snapshot of a firm's operations at a given time.

A. I, II & IV
B. II & III
C. I, II, III & IV
D. I only

Answer: D

Explanation:
To assess the health and growth of a firm, one needs the cashflow statement in addition to the
income statement to evaluate the liquidity and solvency of the firm. III is not true since the Income
statement does not contain all the details which pertain to changes in assets and liabilities. Finally,
(IV) is true of a balance sheet; an income statement displays some aspects of a firm's operations over
a selected period.

Page | 644
Question: 1755

When a firm records a lease as a capital lease:

A. Its working capital increases.


B. Its current ratio decreases.
C. Its debt-to-asset ratio increases.
D. Its financial leverage ratio decreases.

Answer: B

Explanation:
A capital lease is recorded as an asset as well as a liability. The next principal payment to be made on
the lease is recorded as current liability (since it occurs within an operating cycle), thus decreasing
the current ratio and working capital. The remaining portion is recorded as a long-term liability. Since
total assets increase, the financial leverage ratio (assets/equity) increases.

Question: 1756

Under the indirect method, when reporting the cash flow from operations the starting point is

A. the net changes of the operating asset and liability accounts.


B. cost of goods sold.
C. net income.
D. all noncash expenses.
E. cash collections.

Answer: C

Explanation:
Under the indirect method the starting point is the period's net income.

Question: 1757

The Management Discussion and Analysis Section of the annual report ________.

A. is optional but normally included in the annual report


B. is required by the SEC only if the company has suffered from unfavorable trends or there are
significant uncertainty concerning liquidity of the company
C. is required by the SEC only if they have a qualified audit opinion
D. is required by the SEC

Answer: D

Explanation:
Companies with publicly traded securities have been required since 1968 to provide a discussion of
earnings in the MD&A section.

Question: 1758

Which of the following is/are TRUE?

I. The cash flow from operations are higher when expenses are capitalized.
II. Total cash flows are higher when the expenses are capitalized.

Page | 645
III. Capitalization of interest costs leads to higher net income.
IV. Capitalization of interest costs is not allowed under GAAP.

A. II, III & IV


B. I, II, III & IV
C. III & IV
D. I & III

Answer: D

Explanation:
While CFO is lower under expensing since the entire cash expense is charged against it, expensing
leads to a lower total income, causing lower tax payments. Hence, total cash flows are higher under
expensing.

Question: 1759

In a given period, the firm's beginning gross investment is 4,000 and ending gross investment is
12,000. The accumulated depreciation at the beginning was 800 and the ending balance in this
account was 900. The firm uses straight-line depreciation. The average age of the firm's assets at the
end of the period is ________.

A. 5 years
B. 13.3 years
C. 9 years
D. 15 years

Answer: C

Explanation:
Note that: Average age = Accumulated depreciation/Depreciation expense. Depreciation expense =
900-800 = 100. Hence, Avg. age = 900/100 = 9 years.

Question: 1760

Which of the following transactions will impact a company's balance sheet?

A. The payment of principal on a long-term loan


B. The sale of new common stock
C. Net income earned by a company
D. All of these answers
E. The purchase of equipment for cash

Answer: D

Explanation:
All of these answers are correct. Net income earned by a company and the sale of common stock will
affect the equity of a company; the purchase of equipment for cash will affect two assets accounts
(cash and long-term assets); the payment of principal of a long-term loan will affect the liabilities of a
company.

Question: 1761

When preparing a statement of cash flow, one of the relationships between balance sheet changes

Page | 646
and cash flows can be summarized as

A. decreases in liabilities represent net cash inflows.


B. increases in liabilities represent net cash outflows.
C. increases in assets represent net cash outflows.
D. decreases in assets represent net cash outflows.

Answer: C

Explanation:
Increases (decreases) in assets represent net cash outflows (inflows). If an asset increases, the firm
must have paid cash in exchange. On the other hand, when a liability increases, the firm must have
received cash in exchange.

Question: 1762

Which of the following is/are true about Goodwill?

I. It represents the amount paid for an acquired firm that cannot be identified with tangible assets.
II. US GAAP prohibits capitalization of Goodwill.
III. IRS does not allow amortization of Goodwill for tax purposes.

A. I & II
B. II only
C. I, II & III
D. I & III

Answer: D

Explanation:
Under US GAAP, Goodwill generated during acquisitions are capitalized; it does not allow
capitalization of internally generated Goodwill.

Question: 1763

Each of the following are conditions that must be met if revenue is going to be recognized at the time
of sale except

A. seller has no significant obligations for future performance to directly effect resale of product.
B. product return periods are less than or equal to 30 days from the time of sale.
C. price is substantially fixed or determinable at the sale date.
D. buyer pays the seller or is obligated to pay the seller (not contingent on resale).

Answer: B

Explanation:
Revenue can be recognized at the time of sale as long as the product returns are reasonably
estimated as well as several other conditions existing. There is no definitive number of days that
make the recognition possible.

Question: 1764

Under the indirect method, cash flow from operating activities is computed by adjusting net income
for all of the following, except

Page | 647
A. noncash changes in fixed assets.
B. nonoperating items included in net income.
C. noncash revenues and expenses.
D. noncash changes in operating assets.
E. noncash changes in operating liabilities.That answer is correct!

Answer: A

Explanation:
Only operating assets are considered, not fixed assets.

Question: 1765

Depreciation expense for fixed assets is recorded:

A. for each period the asset is in use


B. when the asset is sold
C. as a liability until the asset is sold
D. at the end of each fiscal period during the asset's useful life

Answer: D

Explanation:
Depreciation is the periodic allocation of the cost of a tangible, long-term asset over its estimated
useful life, and therefore a period expense. This expense is recorded at the end of each fiscal period
until the asset is fully depreciated or disposed of.

Question: 1766

Which of the following is true with respect to extraordinary items?

I. Extraordinary items are recorded net of tax in income statement.


II. Extraordinary items, by definition, are probable and unusual in nature.
III. By definition, gains and losses from strikes are always extraordinary. IV By definition, gains and
losses from sale of plant property and equipment are never extraordinary.

A. I and IV
B. I, III and IV
C. II and IV
D. I, II and IIIThat answer is correct!

Answer: A

Explanation:
Extraordinary items are infrequent and unusual in nature. Strikes are not considered extraordinary.

Question: 1767

The Statement of Stockholders' Equity does not report

A. any minimum pension liability.


B. the investment of the owners in the firm.
C. the various accounting adjustments that reflect selected market value changes in noncurrent

Page | 648
assets.
D. any cumulative impact on prior period earnings.
E. the effect of exchange rate changes on certain foreign subsidiaries.

Answer: D

Explanation:
Any cumulative impact on prior period earnings is reported net of tax after extraordinary items and
discontinued operations on the income statement.

Question: 1768

The contents of the section of the annual report entitled "Management's Discussion and Analysis"
(MD&A) are

A. mandated by regulations of the SEC.


B. mandated by pronouncements of the Financial Accounting Standards Board (FASB).
C. reviewed by independent auditors.
D. mandated by regulations of the Internal Revenue Service.
E. none of these answers.That answer is correct!

Answer: A

Explanation:
The contents of the MD&A section are mandated by the SEC and not FASB.

Question: 1769

An employee's pension payments are based on his salary and length of service. He does not bear the
risk of the pension size being affected by market performance. He is enrolled in the:

A. defined benefits plan.


B. defined contribution plan.
C. defined pension plan.
D. employee pension benefits plan.That answer is correct!

Answer: A

Explanation:
In a defined benefit pension plan, the retirement benefits are "predefined." The employer commits
to providing the benefits regardless of the performance of the pension plan. Thus, in this plan, the
risk of pension plan performance is borne by the employer and not the employee.

Question: 1770

In computing EPS, the equivalent number of shares of convertible preferred stock is added as an
adjustment to the denominator (number of shares outstanding). If the preferred stock is preferred as
to dividends, which amount should then be added as an adjustment to the numerator (earnings
available to common shareholders)?

A. Annual preferred dividend.


B. Annual preferred dividend times (100% minus the income tax rate).
C. Annual preferred dividend divided by the income tax rate.
D. No adjustment should be made.

Page | 649
E. Annual preferred dividend times the income tax rate.That answer is correct!

Answer: A

Explanation:
If a capital structure has convertible preferred stock with a dilutive effect on EPS, the "if-converted"
method is used. This method assumes the conversion of the preferred stock occurred at the
beginning of the accounting period or at issuance, if later. The annual preferred dividend is added
back to earnings available to common shareholders.

Question: 1771

If a company estimates that its expected return on pension plan assets will increase to 9.5% from
9.0% this would be considered ________.

A. an extraordinary gain
B. a change in accounting estimate
C. a prior period adjustment
D. a change in accounting principle

Answer: B

Explanation:
Such a change is a change in accounting estimate and disclosure is required of the effect on both net
income and income before extraordinary items for the current period.

Question: 1772

Anti-dilutive securities are those that:

A. are always included in the computation of the Diluted EPS.


B. differentiate between a simple capital structure and complex capital structure.
C. none of these answers.
D. increase the earnings per share when assumed exercised.

Answer: D

Explanation:
Anti-dilutive securities are always excluded from the computation of the Diluted EPS.

Question: 1773

Jones company earned $100,000 of pre-tax income in 1998, had $50,000 of pre-tax income in 1997,
pays a 40% tax rate and has a dividend payout of 50%. What is the dividend amount declared by
Jones Company in 1998?

A. $45,000
B. $40,000
C. $30,000
D. $50,000

Answer: C

Explanation:

Page | 650
1998 Pre-tax income was $100,000 in 1998. The pre-tax income from the prior year has no relevance
to the question. If the company pays a 40% tax rate, they are left with net income of $60,000 for the
year. If the dividend payout rate is 50%, the company will declare $30,000 of dividends for 1998.

Question: 1774

All of the following statements are true except for:

A. When inventory balances decrease, cash flows increase.


B. When short-term debt increases, cash flows increase.
C. When accounts receivable balances increase, cash flows increase.
D. When accounts payable balances increase, cash flows increase.

Answer: C

Explanation:
As accounts receivable balances increase, there is less income received as cash, therefore cash
balances on the statement of cash flows will decrease.

Question: 1775

Beginning inventory of 50 units, purchased at $5


50 units purchased at $10
35 units purchased at $9
25 units sold at $15
70 units sold at $12
Tax rate = 40%
Beginning LIFO reserve = $300

Given the above, the net income using Average Cost method is ________.

A. $279
B. $224
C. $381
D. $249That answer is correct!

Answer: A

Explanation:
The Average Cost method first calculates the average value of each item in the inventory, including
the purchases made during the period. In this case, the total inventory value at cost is 50*5 + 50*10 +
35*9 = $1,065. This is the value of (50 + 50 + 35) = 135 units. Therefore, average cost per unit =
1,065/135 = $7.89. Total COGS under Average Cost method is then $7.89*95 = $749.44. The total
revenue equals 25*15 + 70*12 = 1,215, coming from a sale of 95 units. Thus, pretax income = 1,215 -
749.44 = $465.56. The after-tax income equals 465.56*0.6 = $279.

Question: 1776

Which of the following is used to calculate the actual adjustment for bad debt expense for the
period?

A. percent of accounts receivable


B. percent of net sales
C. aging

Page | 651
D. all of these answers

Answer: B

Explanation:
The aging and percent of Accounts Receivable methods calculate what the balance of the allowance
account should be, however, percent of net sales method calculates the actual expense for the
period.

Question: 1777

MacDonald Inc. reported net income of $300,000 for 1996. Changes occurred in several balance
sheet accounts as follows:

Equipment$25,000 increase
Accumulated depreciation$40,000 increase
Note payable$30,000 increase

Additional information:

During 1996, MacDonald sold equipment costing $25,000, with accumulated depreciation of
$12,000, for a gain of $5,000.

In December 1996, MacDonald purchased equipment costing $50,000 with $20,000 cash and a 12%
note payable of $30,000.

Depreciation expense for the year was $52,000.

In MacDonald's 1996 statement of cash flows, net cash provided by operating activities is ________.

A. $347,000
B. $357,000
C. $243,000
D. $340,000
E. $352,000That answer is correct!

Answer: A

Explanation:
To calculate cash flows from operating activities, net income will be adjusted for noncash items and
working capital. Depreciation of $52,000 is added back to net income of $300,000 and the gain on
sale of equipment of $5,000 is subtracted from net income. Therefore, the net cash inflow from
operating activities is $347,000.

Question: 1778

The following data are available for a firm for a given year:

Net Sales21,896
Sales & marketing expenses4,346
Administrative expenses2,143
COGS10,084
Depreciation967
Interest expense573

Page | 652
Tax rate35%
Dividends paid3,445
Preferred Dividends897
Average total equity37,432
Average common equity26,782
Average total liabilities18,583

In the above example, the firm's return on total equity equals ________.

A. 8.31%
B. 4.22%
C. 6.57%
D. 5.91%

Answer: C

Explanation:
Return on total equity = Net income/average total equity. In the above example, Net Income =
Earnings after depreciation, interest expense and taxes = (21,896 - 4,346 - 2,143 - 10,084 - 967 -
573)*(1 - 0.35) = 2,459. Therefore, Return on total equity = 2,459/37,432 = 6.57%.

Question: 1779

In the country of Merlenia, authorities require that tax reports and financial reports be identical. In
Merlenia, there will be:

A. no recognition of deferred taxes.


B. a prevalence of LIFO accounting.
C. no advantage to capitalizing a lease.
D. no off-Balance-sheet financing.That answer is correct!

Answer: A

Explanation:
Deferred taxes arise out of temporary differences between tax reports and financial statements. To
account for such differences under accrual accounting, deferred tax assets or liabilities are generated
in accordance with the Matching Principle.

Question: 1780

The average age of a firm's depreciable assets is 9 years. The firm, subject to a tax rate of 40%, saves
20,000 each year due to depreciation expense. The total depreciation that the firm has accumulated
is _______.

A. 300,000
B. 180,000
C. 450,000
D. 72,000

Answer: C

Explanation:
Since tax savings from depreciation = (depreciation expense)*tax rate, one obtains depreciation
expense = 20,000/0.4 = 50,000. Also, Average age = Accumulated depreciation/Depreciation

Page | 653
expense. Hence, total accumulated depreciation = 9*50,000 = 450,000

Question: 1781

Which of the following is not a financing activity in the statement of cash flows?

A. repurchase of common stock


B. issuance of new debt
C. payment of interest on debt
D. cash dividend

Answer: C

Explanation:
The payment of interest on debt is not a financing activity.

Question: 1782

A firm has net sales of $6,000 in fiscal 1998, cash expenses (including taxes) of $2,800, and
depreciation of $1,000. If accounts receivable increased in the period by $800, cash flows from
operations equal ________.

A. $2,400
B. $3,200
C. $3,400
D. $4,200That answer is correct!

Answer: A

Explanation:
(6,000 - 2,800 - 800)

Question: 1783

The following is referred to as the critical event in income determination:

A. recording of revenues
B. recording of changes in cash
C. recording of changes in retained earnings during the period
D. recording of expensesThat answer is correct!

Answer: A

Explanation:
Recording revenues at the proper time is the critical event. Expenses are typically easier for
companies to allocate to the proper periods. Expenses are more certain than revenues for most
companies.

Question: 1784

Tracy company reports the following in its statement of cash flows:

Net Income$1,000
Depreciation and Amortization350

Page | 654
Decrease (Increase) in Accounts receivable(10)
Decrease (increase) in inventory200
Decrease (increase) in prepaid expenses80
Increase (decrease) in trade payables(300)
Increase (decrease) in taxes payable75
Cash Flow from operations1,395

If Tracy shows cost of goods sold of $2,050 on its income statement, cash paid to suppliers is
________.

A. $2,650
B. $1,550
C. $1,950
D. $2,150

Answer: D

Explanation:
$2,050 - $200 (purchased $200 less than used) + 300 (paid more to suppliers.

Question: 1785

Which of the following statements about the cash flow statement is/are true?

I. The cash flow statement provides information on the liquidity of the firm.
II. The income statement is more susceptible to management manipulation than the cash flow
statement.
III. The cash flow statement serves as a check on the inherent assumptions of the income statement.
IV. The cash flow statement is based on actual events while the income statement is based on the
allocation of the effects of these events over time.

A. I & III
B. I, II, III & IV
C. I, II & III
D. II & IV

Answer: B

Explanation:
The cash flow statement cannot be manipulated as easily as the income statement since it is based
on actual cash flows while income statement has to follow the accrual principle, which allocates non-
cash expenses and revenues to the current and future periods, leaving open the avenue for earnings
manipulation. For this reason, the cash flow statement acts as a check on the income statement.

Question: 1786

On Jan. 1, DataCom prepays $3,000 rent for Mar. 1, June 1, and Sept.

1. Under accrual basis accounting, what is the impact on the Feb. financial statements?

A. $3,000 expense
B. $1,000 decrease in net income
C. no impact
D. an increase to current assets of $1,000

Page | 655
Answer: B

Explanation:
In order to match expenses with revenues for the period, the cost of prepaid insurance should be
allocated to each accounting period. The asset value is decreased (credit), and an expense is
recognized (debit), thus decreasing net income.

Question: 1787

Stockholders' Equity is

A. all of these answers


B. the financial obligations of the company
C. the rights to the assets of the business once the liabilities have been met
D. assets plus liabilities

Answer: C

Explanation:
The accounting equation shows us that the Assets (Own) are equal to the Liabilities (obligations) plus
the Stockholders' Equity.

Question: 1788

If a firm recognizes expenses before that dictated by accrual accounting, which of the following best
describes the effects on income, total assets and retained earnings?

IncomeTotal Assets Retained Earnings

I.UnderstatedUnderstated Understated
II.UnderstatedOverstatedUnderstated
III.OverstatedUnderstated Overstated
IV.UnderstatedOverstatedOverstated

A. III.
B. IV.
C. I.
D. II.

Answer: C

Explanation:
Income is understated. Hence, retained earnings and equity are understated. Hence, assets are
understated.

Question: 1789

At the beginning of fiscal 1998 Jones Company had retained earnings of $1,000,000 and at the
beginning of fiscal 1999 Jones Company had retained earnings of $1,200,000. During fiscal 1998
Jones Company declared a dividend of $300,000 and declared a 3 for 1 stock split. How much did
Jones Company earn during fiscal 1998?

A. $200,000

Page | 656
B. $166,667
C. $600,000
D. $500,000

Answer: D

Explanation:
The company's retained earnings increased by $200,000 during 1998 and dividends were paid of
$300,000, therefore earnings had to be $500,000. The stock split has no effect on retained earnings.

Question: 1790

Under what condition would a firm report cash, or cash equivalents as a long-term asset?

A. When a firm maintains less than its' minimum target cash balance.
B. When funds are set aside for a specific long-term purpose such as plant expansion.
C. All of these answers.
D. When a firm has an under-funded pension obligation.
E. None of these answers.

Answer: B

Explanation:
A firm may maintain cash equivalents that are earmarked for plant expansion, sinking fund
payments, or other long-term uses. In these situations many firms would report this portion of its
cash, or cash equivalents as long-term because these funds are not available in the daily operation of
the business.

Question: 1791

The amount by which a plant asset depreciated is classified as ________.

A. revenue
B. an expense
C. a liability
D. an asset

Answer: B

Explanation:
Depreciation is the periodic allocation of the cost of a tangible, long-term asset over its estimated
useful life, and therefore a period expense.

Question: 1792

The relationship of the total debt to the total equity of a firm is a measure of ________.

A. liquidity
B. creditor risk
C. none of these answers
D. profitability
E. solvency

Answer: B

Page | 657
Explanation:
The ratio of total debt to total equity is a measure of risk to creditors because it helps in evaluating a
company's relative reliance on debt and equity financing.

Question: 1793

What entity is presently responsible for establishing accounting standards?

A. The Financial Accounting Standards Board.


B. The FDA.
C. The Securities and Exchange Commission.
D. The Internal Revenue Service.
E. The Federal Accounting Society Board of DirectorsThat answer is correct!

Answer: A

Explanation:
The Financial Accounting Standards Board.

Question: 1794

Materiality is one of the pervasive concepts in financial accounting. Which of the following
statements is true with regard to materiality?

A. Relevant items are very material.


B. Materiality judgments generally may be made without consideration of the magnitude of the item
involved.
C. None of these answers.
D. The nature and magnitude of an item as well as the circumstances in which the judgment has to
be made are integral aspects of a materiality judgment.
E. Materiality judgments generally may be based solely on the magnitude of the item.

Answer: D

Explanation:
The basis for a materiality judgment is generally not sufficient unless the nature of the item, the
circumstances in which the judgment has to be made, and the magnitude of the item are all
considered.

Question: 1795

At December 31, 1995, Del Monte Co. had the following balances in the accounts it maintains at
Royal Street Bank:

Checking account #101$175,000


Checking account #102$(10,000)
Money market account$25,000
90-day certificate of deposit due 2/28/96$50,000
180-day certificate of deposit due 3/15/96$80,000

Del Monte classifies investments with original maturities of 3 months or less as cash equivalents. In
its December 31, 1995 balance sheet, what amount should Del Monte report as cash and cash
equivalents?

Page | 658
A. $240,000
B. $190,000
C. $320,000
D. $165,000
E. $200,000That answer is correct!

Answer: A

Explanation:
Cash consists of coins, currency on hand, checking accounts (demand deposits), certificates of
deposit (time deposits) and near-cash assets (money market accounts). Cash equivalents are limited
to investments with original maturities to the holding entity of 3 months or less. As a result, cash and
cash equivalents will be $240,000 ($175,000 - 10,000 + 25,000 + 50,000).

Question: 1796

Which of the following statements is true?

A. Allocation of plant asset costs, through depreciation, if consistently applied, does not lend itself to
serious distortion.
B. None of these answers.
C. All of these answers.
D. Determination of the useful life of an asset, if consistently applied, does not lend itself to serious
distortion.
E. Historical cost valuation of plant assets, if consistently applied, does not lend itself to serious
distortion.

Answer: E

Explanation:
Determining the useful life of an asset is central to the allocation of its cost and its period-to-period
valuation; the same is true with depreciation schedules that are selected. These pose problems in
determining the fair value of plant and equipment.

Question: 1797

Which of the following is/are true in an inflationary environment?

I. For income statement purposes, LIFO is preferable to FIFO.


II. For balance sheet purposes, LIFO is preferable to FIFO.
III. The weighted average cost method provides a better balance sheet representation than LIFO.
IV. The weighted average cost method provides a better income statement representation than FIFO.

A. II, III & IV


B. II & IV
C. III only
D. I, III & IV

Answer: D

Explanation:
Remember that for income statement, it is desirable to have the COGS as close to true, current cost,
as possible. For balance sheet, the assets must be as close to current, market value, as possible.

Page | 659
Hence, for income statement, LIFO preferred to average cost preferred to FIFO. For balance sheet,
FIFO preferred to average cost preferred to LIFO.

Question: 1798

Deferred credits arise because of:

A. an over-estimation of assets or earnings, which must now be reversed.


B. none of these answers.
C. the concept of accrual accounting.
D. the use of cash accounting.

Answer: C

Explanation:
Deferred credits can be liabilities incurred in the course of current business which are not yet
recognized in the income statement or are unearned assets or revenues which have been recognized
prematurely, as determined by the matching principle. Examples of these include deferred income
taxes and unearned royalties received. In either case, accrual accounting dictates that such
"mistakes" be corrected for through the use of deferred credits.

Question: 1799

The consistency principle

A. requires that the company use a selected accounting method period after period
B. all of these answers are correct
C. is limited to inventory pricing methods
D. requires that a company use one inventory method exclusivelyThat answer is correct!

Answer: A

Explanation:
The consistency principle requires that an accounting procedure, once adopted by a company,
remain in use from one period to the next unless users are informed of the change. It is not limited
to inventory valuation methods.

Question: 1800

Which of the following is used to calculate the actual adjustment for bad debt expense for the
period?

A. aging
B. percent of accounts receivable
C. percent of net sales
D. all of these answers are correct

Answer: C

Explanation:
The aging and percent of Accounts Receivable methods calculate what the balance of the allowance
account should be, however, percent of net sales method calculates the actual expense for the
period.

Page | 660
Question: 1801

Ratios are best interpreted when compared to which of the following?

A. Predetermined standards.
B. The ratios of prior periods.
C. All of these answers are correct.
D. Ratios of competitors.

Answer: C

Explanation:
Ratios are best interpreted when compared to, the ratios of prior periods, predetermined standards,
and the ratios of competitors.

Question: 1802

New Gestalt, Inc., a software firm had a net income of 1.7 million last year. It has 200,000 common
shares and 300,000 convertible bonds with face value of 100 outstanding. The convertible bonds
carry a coupon of 4% and can be converted one-for-one. The average stock price last year was 39 and
the maximum price was 57. The effective interest rate on the convertible debt is 8%. New Gestalt
issued 100,000 preferred shares with face value 100 and a coupon of 5% on March 31st of last year.
Assume the convertible bonds are dilutive and that New Gestalt faces a 30% tax rate. Given the
above, the number of shares used in Diluted EPS equals ________.

A. 575,000
B. 500,000
C. 600,000
D. 200,000

Answer: B

Explanation:
With dilution, there are 300,000 additional shares from the convertible bonds. Preferred equity does
not count. Therefore, the number of shares used in Diluted EPS equals 200,000 + 300,000 = 500,000.

Question: 1803

The ROE equals:

I. profit margin times equity turnover.


II. profit margin times asset turnover times financial leverage.
III. financial leverage times sales-to-debt ratio.

A. none of these answers


B. II & III
C. I & II
D. I, II & III

Answer: C

Explanation:
ROE = Net income/Equity
= (Net income/sales)*(sales/equity)

Page | 661
= (profit margin)*(equity turnover).
You can also express this as:
ROE = Net income/Equity
= (Net income/sales)*(sales/total assets)*(total assets/equity)
= (profit margin)*(asset turnover)*(financial leverage)

Question: 1804

GAAP requires that:

A. cash flows from interest related inflows and outflows on debt be considered investing cash flows.
B. cash flows from dividend receipts be considered investing cash flows.
C. all of these answers.
D. none of these answers.

Answer: D

Explanation:
It is very important to remember the following points about dividends and interests:

1. Dividends received from stock investments are considered operating cash flows.
2. Dividends paid on equity are considered financing cash flows.
3. Interest payments on debt are considered operating cash flows.
4. Interest received from debt investments are considered operating cash flows.

Question: 1805

A firm has purchased heavy machinery with a useful life of 7 years. It cost $28,000 and its salvage
value is estimated at $4,000. If the firm uses double declining method, what's the depreciation
expense recognized in Year 1?

A. $3,429
B. $8,000
C. $6,857
D. $4,000

Answer: B

Explanation:
In Double Declining method, depreciation = (2/n)*book value = (2/7)*28,000.

Question: 1806

Employers' income tax expense consists of the following obligations except:

A. Medicare tax
B. Social Security tax
C. Federal income tax
D. State unemployment tax

Answer: C

Explanation:
Federal income tax is withheld from employee paychecks; it is not an expense to the company.

Page | 662
Question: 1807

Which of the following is an essential characteristic of a liability?

A. The identity of the recipient entity must be known to the obligated entity before the time of
settlement.
B. Liabilities represent an obligation that has arisen as the result of a previous transaction.
C. None of these answers.
D. Liabilities must be legally enforceable.
E. Liabilities must require the obligated enterprise to pay cash to a recipient entity.

Answer: B

Explanation:
A liability has 3 essential characteristics:

1. It represents an obligation that requires settlement by probable future transfer or use of assets.
2. The firm has little or no discretion to avoid the obligation.
3. The transaction or other event, giving rise to the obligation has already occurred.

Question: 1808

Quasar, Inc., currently shows assets worth 5,000 and a debt of 1,500. During the year, it capitalized
interest expense worth 300, of which 60 was amortized. Quasar's tax rate is 50%. If it had expensed
the interest paid, which of the following would be true?

A. Its operating cash flow would be lower by 150.


B. Its tax expense would be lower by 150.
C. Its equity would be higher by 300.
D. Its operating cash flow would be lower by 300.That answer is correct!

Answer: A

Explanation:
If Quasar had expensed the interest, its tax savings would have been 300*50% = 150. By capitalizing,
it amortized 60, leading to a tax savings of 60*50%=30. Thus, if it had expensed the interest, the tax
expense would have been lower by 150-30=120 (What would the effect on net income be? The net
income would be lower by (300 - 60 - 120 = 120). When capitalizing, all the expense gets charged to
investing cash flow while expensing charges the entire cash flow to operating cash flow. After tax,
this results in an operating cash flow of -300*(1-0.5) = -150. Thus, expensing would have resulted in
an operating cash flow lower by 150.

Question: 1809

On January 1, 1992, Carter Inc. purchased equipment having an estimated salvage value equal to
20% of its original cost at the end of a 10 year life. The equipment was sold December 31, 1996 for
50% of itsoriginal cost. If the equipment's disposition resulted in a reported loss, which of the
following depreciation methods did Carter use?

A. Straight-line.
B. Double-declining-balance.
C. Composite.
D. Sum-of-the-years'-digits

Page | 663
E. Accelerated.That answer is correct!

Answer: A

Explanation:
The straight-line method yields the lowest amount of depreciation for the early part of the
depreciable life of the asset. Because only 50% of the original cost was received and straight-line
accumulated depreciation equaled 40% of cost ((100%-20%) / 10 years X 5 years( at time of sale, a
10% loss (50%-(100%-40%)) occurs.

Question: 1810

The accounting equation reads:

A. Revenue minus expenses equals profit


B. Assets equal Liabilities plus Stockholders' Equity
C. Assets plus Liabilities is equal to Stockholders' Equity
D. None of these answers is correct

Answer: B

Explanation:
The accounting equation shows us that the Assets (Own) are equal to the Liabilities (obligations) plus
the Stockholders' Equity.

Question: 1811

Which of the following is NOT an objective of the conceptual framework devised by FASB?

A. Establish a logical and coherent system of accounting thought.


B. Establish objectives of financial reporting by businesses.
C. Identify the characteristics of accounting information that make its disclosure useful.
D. Enforce a single standard of accounting disclosures across the various industries.

Answer: D

Explanation:
The conceptual framework devised by FASB is not concerned with enforcing any rigid standards of
accounting disclosures. Rather, it is a system of laying out the goals and objectives of accounting
disclosure and formulating standards which could best serve the needs of the industry. In such a
framework, a fair amount of flexibility is built-in so that the differing characteristics of various
industries can be accommodated. The various SFACs which comprise the conceptual framework are
the foundation on which the detailed principles and pronouncements of the FASB are based.

Question: 1812

In a common-size balance-sheet statement, a firm's current assets are reported at 37% and the long-
term liabilities are at 12%. The equity is shown to be 64%. The firm's current ratio equals ________.

A. 1.12
B. 1.54
C. 0.65
D. 1.37

Page | 664
Answer: B

Explanation:
In a common-size balance-sheet statement, all quantities are expressed as a fraction of the total
assets. Therefore, since equity equals 64%, total liabilities equal 36%, of which 12% equals long-term
liabilities. Thus, current liabilities equal 24%, giving the current ratio equal to 37%/24% = 1.54.

Question: 1813

Under an inflationary environment with stable inventories, a firm may change to FIFO from LIFO due
to which of the following reason(s)?

I. To allow earnings manipulation.


II. To improve the reported working capital.
III. To reduce tax drain on cash.
IV. Show a more accurate representation of reported assets than LIFO.

A. II & IV
B. III only
C. I & IV
D. I & IIIThat answer is correct!

Answer: A

Explanation:
Under FIFO, inventory is overstated and hence, working capital is overstated under normal,
inflationary conditions. At the same time, COGS is lower than under LIFO due to rising prices,
increasing the taxable income and leading to higher taxes. However, firms cannot manipulate
earnings under FIFO by changing purchasing patterns at the end of an accounting period, as it is able
to do under LIFO, because FIFO prices are determined by the earliest goods purchased in the
inventory.

Question: 1814

The classification guidelines of SFAS 95 have not created problems for analysts of the cash flow
statement in the following area

A. differences due to some accounting methods.


B. noncash transactions.
C. long-term borrowings.
D. interest and dividends received.
E. interest paid.

Answer: C

Explanation:
The classification guidelines of SFAS 95 have created problems for analysts of the cash flow
statement in cash flows involving property, plant, and equipment, interest and dividends received,
interest paid, noncash transactions, and differences due to some accounting methods.

Question: 1815

The GAAP require inventory to be valued using ________.

Page | 665
A. Lower of market or cost
B. LIFO or FIFO
C. LIFO
D. FIFOThat answer is correct!

Answer: A

Explanation:
Inventory valuation is a balance sheet consideration and is distinct from the calculation of COGS,
which is an income statement item. US GAAP requires that inventory valuation be carried out using
the lower of cost or market. The "cost" is determined by the inventory costing method chosen, like
LIFO or FIFO. If the market value of the inventory falls below that obtained under the inventory
costing method, then the firm must take a charge against the current income and "write the
inventory down." Under this scenario, the basic inventory equation, Ending inventory = Beginning
inventory + Purchases - COGS no longer holds. If the market price rises in the subsequent period, the
firm can only write-up to the original cost of the inventory under the costing method chosen. In all
periods in which there is no write-up or write-down, the basic inventory equation holds true.

Question: 1816

Which of the following best describes a statement of shareholders equity?

A. A statement of shareholders equity measures a company's financial performance over a specified


period of time.
B. A statement of shareholders equity details the cash inflows and outflows that are related to a
company's operating, investing, and financing activities over a period of time.
C. None of these answers.
D. A statement of shareholders equity reports changes over a period of time in component accounts
that comprise the ownership of a firm.
E. A statement of shareholders equity summarizes the financial position of a company at a given
point in time.

Answer: D

Explanation:
The statement of shareholders equity reports changes in component accounts that comprising equity
of a firm. This statement is useful in identifying reasons for changes in shareholders' claims on the
assets of a company.

Question: 1817

New Gestalt, Inc., a software firm had a net income of 1.7 million last year. It has 200,000 common
shares and 300,000 convertible bonds with face value of 100 outstanding. The convertible bonds
carry a coupon of 4% and can be converted one-for-one. The average stock price last year was 39 and
the maximum price was 57. The effective interest rate on the convertible debt is 8%. New Gestalt
issued 100,000 preferred shares with face value 100 and a coupon of 5% on March 31st of last year.
Assume the convertible bonds are dilutive and that New Gestalt faces a 30% tax rate. The number of
shares used in Basic EPS equals ________.

A. 600,000
B. 575,000
C. 500,000
D. 200,000

Page | 666
Answer: D

Explanation:
Basic EPS considers only the weighted number of common stock outstanding. Therefore, number of
shares used in Basic EPS = 200,000.

Question: 1818

According to GAAP classification of cash flows, all of the following are financing cash flows except:

A. share repurchase.
B. interest paid on new debt.
C. losses on debt retirement.
D. dividends paid out.

Answer: B

Explanation:
Interest payments and receipts are classified as operating cash flows.

Question: 1819

When the weighted average method is used, ending inventory units are priced at the:

A. average price
B. none of these answers
C. lower of cost or market
D. earliest priceThat answer is correct!

Answer: A

Explanation:
Under the weighted average method, inventory is priced at the average cost of the goods available
for sale (Beginning inventory plus purchases during the period).

Question: 1820

The weighted average method is based on the assumption that the cost of merchandise sold should
be calculated using the:

A. average price of beginning inventory plus purchases during the period


B. average price per unit of ending inventory
C. average price of ending inventory plus purchases during the period
D. average price per unit of beginning inventoryThat answer is correct!

Answer: A

Explanation:
Under the weighted average method, inventory is priced at the average cost of the goods available
for sale (Beginning inventory plus purchases during the period).

Question: 1821

An entry made to recognize the cost of using an asset during the period is

Page | 667
A. a closing entry
B. a reversing entry
C. an opening entry
D. an adjusting entry

Answer: D

Explanation:
Expenses should be recognized in the period in which they are incurred. In order to be consistent
with the matching principle, an adjusting entry must be made to record the expense of using certain
assets.

Question: 1822

If a firm's asset turnover were to increase by 10% and the tax rate were to increase from 35% to 40%,
leaving all else constant, the resultant change in the firm's ROE equals ________.

A. -2.3%
B. -1.9%
C. +1.5%
D. -1.1%

Answer: C

Explanation:
ROE = Net income/Equity. Using the Extended duPont System, = (EBT/sales)*(sales/total
assets)*(total assets/equity)*(1-tax rate) The asset turnover ratio equals sales/total assets.
Therefore, ROE(after)/ROE(before) = [asset turnover(after)/asset turnover(before)*]*[0.6/0.65] =
1.1*0.6/0.65 = 1.015. Thus, the ROE increases by 1.5%.

Question: 1823

New Gestalt, Inc., a software firm had a net income of 1.7 million last year. It has 200,000 common
shares and 300,000 convertible bonds with face value of 100 outstanding. The convertible bonds
carry a coupon of 4% and can be converted one-for-one. The average stock price last year was 39 and
the maximum price was 57. The effective interest rate on the convertible debt is 8%. New Gestalt
issued 100,000 preferred shares with face value 100 and a coupon of 5% on March 31st of last year.
Assume the convertible bonds are dilutive and that New Gestalt faces a 30% tax rate. Given the
above, if New Gestalt had 300,000 warrants with a strike of 36 outstanding instead of the convertible
bonds, its Diluted EPS would equal ________.

A. 5.94
B. 5.24
C. 6.62
D. 4.91That answer is correct!

Answer: A

Explanation:
For Diluted EPS with warrants, numerator = net income - preferred dividends = 1.7 million - 375,000
= 1,325,000. Using the Treasury stock method, the additional shares due to warrant exercise =
300,000*(1 - 36/39) = 23,077. Total number of shares used in Diluted EPS = 200,000 + 23,077 =
223,077. Therefore, Diluted EPS = 1,325,000/223,077 = 5.94.

Page | 668
Question: 1824

Firm A currently shows assets worth 2,000 and equity of 1,500. During the year, it capitalized interest
expense worth 200, of which 50 was depreciated/amortized. The firm faces a tax rate of 50%. If the
firm had expensed the interest paid, which of the following would be true?

A. Its assets would be stated at 1,750.


B. Its assets would be stated at 1,850.
C. Its assets would be stated at 2,000.
D. Its assets would be stated at 1,800.

Answer: B

Explanation:
If the firm's assets without capitalization equal A, then we have A+200-50 = 2,000. Hence, A = 1,850.

Question: 1825

Contingent losses are recognized:

I. when they actually occur during a contingency.


II. only if it is probable that a loss will occur.
III. only if the amount of the loss can be reasonably estimated.
IV. at the management's discretion.

A. II, III & IV


B. I only
C. II & III
D. II & IVThat answer is correct!

Answer: A

Explanation:
II and III must hold together for a recognition of contingent losses to take place. GAAP defines
"probable" as "greater than 50% chance." Since this is subjective, the management has considerable
discretion on the timing of recognition.

Question: 1826

A firm has a cash conversion cycle of 31.6 days. It turns over its inventory on average in 43.1 days and
pays off its payables in an average of 23.2 days. Its receivables turnover ratio equals ________.

A. 31.2
B. 42.3
C. 14.9
D. 25.1That answer is correct!

Answer: A

Explanation:
CCC = (Average receivables collection period) plus (Average inventory processing time) minus
(Average payables payment period).
Hence, 31.6 = Average receivables collection period + 43.1 - 23.2, giving

Page | 669
Average receivables collection period = 31.6 - 43.1 + 23.2 = 11.7 days. Since Average receivables
collection period = 365/receivables turnover, the receivables turnover ratio equals 365/11.7 = 31.2.

Question: 1827

The cash conversion cycle equals:

A. receivables days - inventory processing days + payables payment period.


B. receivables days - inventory processing days - payables payment period.
C. receivables days + inventory processing days + payables payment period.
D. receivables days - payables payment period + inventory processing days.

Answer: D

Explanation:
The cash conversion cycle is a measure of how long the cash is tied up in short term loans and
credits. These short-term financing items include receivables, inventory and accounts payable. The
first two of these represent sources of cash while accounts payable represent a future drain of cash.
So the cash conversion cycle is defined as: CCC = (Average receivables collection period) plus
(Average inventory processing time) minus (Average payables payment period). A low cash
conversion cycle could indicate that you are either collecting your debts or churning inventory faster
or you are paying your bills slower.

Question: 1828

Investments in debt securities classified as held-to-maturity securities should be valued at

A. fair value.
B. lower of cost or market.
C. present value.
D. amortized cost.
E. acquisition cost.

Answer: D

Explanation:
Accounting rules specify that debt securities classified as held-to-maturity must be valued at
amortized cost.

Question: 1829

All of the following are often characteristics of intangible assets except:

A. they are often non-separable from a company.


B. the value of the assets portrayed in the financial statements tends to increase for companies
performing a lot of R&D.
C. they have indefinite benefit periods.
D. they experience large valuation changes based on competitive circumstances.

Answer: B

Explanation:
The value of intangible assets on the balance sheet can only increase when the assets are purchased
from another company. This is to encourage conservatism on the balance sheet.

Page | 670
Question: 1830

When the weighted average method is used, ending inventory units are priced at the

A. none of these answers is correct


B. earliest price
C. average price
D. lower of cost or market

Answer: C

Explanation:
Under the weighted average method, inventory is priced at the average cost of the goods available
for sale (Beginning inventory plus purchases during the period).

Question: 1831

The main purpose of a statement of cash flows of a firm is to provide relevant information about

A. differences between net income and associated cash receipts and disbursements.
B. the cash receipts and cash disbursements of an enterprise during a period.
C. a firm's ability to generate future positive net cash flows.
D. a firm's ability to meet cash operating needs.
E. none of these answers.

Answer: B

Explanation:
The statement cash flows helps investors, creditors and other users to assess the firm's ability to
generate net cash inflows, meet its obligations, pay dividends and obtain external financing.

Question: 1832

Which inventory costing method is most useful in estimating the amount of inventory lost or
destroyed by theft, fire, or other hazards?

A. average cost method


B. LIFO
C. none of these answers is correct
D. FIFO
E. gross profit method

Answer: E

Explanation:
The historical relationship between cost of goods sold and sales is applied to sales of the current
period as a way of estimating COGS during the current period. COGS is then subtracted from the cost
of goods available for sale to get the estimated cost of the ending inventory.

Question: 1833

If a company receives an unqualified audit opinion it means the auditors

Page | 671
A. are providing assurance that the company's financial statements are free from misstatement,
fraudulent accounting and fairly indicate future performance.
B. are providing assurance that the company will remain financially viable for at least the next year.
C. are providing assurance that the company's financial statements fairly present company's financial
performance and position.
D. did not complete a full audit and therefore do not feel qualified to give and opinion on financial
statements.

Answer: C

Question: 1834

The following is true about the price earnings ratio

A. all of these answers are correct


B. the ratio is calculated based on the expected earnings per share for the next period
C. stocks have low PE ratios if they are less than five to eight and are considered underpriced
D. it is the ratio between the company's current market value and its earnings per shareThat answer
is correct!

Answer: A

Explanation:
PE ratio is calculated based on expected earnings per share and a ratio of less than 5 to 8 makes a
stock appear underpriced.

Question: 1835

At the end of the fiscal period, the account debited to show the estimated amount of uncollectible
accounts is

A. Allowance for Uncollectible Accounts


B. None of these answers is correct.
C. Accounts Receivable
D. Bad Debt Expense
E. Unearned Revenue

Answer: D

Explanation:
The adjusting entry should recognize an expense and increase the allowance account.

Question: 1836

Under US GAAP accounting, R&D expenses cannot be capitalized. They must be expensed as and
when incurred. Which of the following is NOT a reason to support this practice?

A. Most R&D activities represent intangible benefits which are hard to evaluate objectively from an
accounting perspective.
B. In most cases, there is a significant lag between R&D expenses and the determination of the
potential success of the research.
C. There remains a high uncertainty about the ultimate benefits to be derived from the R&D activities
on which the expenses are incurred.
D. R&D expenses are difficult to measure and hence, must be expensed in accordance with accrual

Page | 672
accounting principles.

Answer: D

Explanation:
Note that R&D expenses are, for the most part, are clearly ear-marked as such and hence, are not
difficult to measure. Of course, some of these expenses go into generating tangible assets like
laboratories and flasks but these are easily measurable and do get capitalized under US GAAP. The
other three reasons imply that it is difficult to measure the present value of the assets being
generated (if at all) by the research. This prevents a consistent and reliable application of the accrual
principles, which require allocation of expenses in proportion to the current benefits generated.
Hence, US GAAP takes the easiest solution of expensing all the R&D costs.

Question: 1837

Firm A uses Sum-of-digits and Firm B uses straight-line method of depreciation. In the first year,
which of the following is/are TRUE?

I. A shows higher equity than B.


II. A shows lower assets than B.
III. A shows a higher debt-to-equity ratio than B.
IV. A shows a higher debt-to-asset ratio.

A. III only
B. I & III
C. II, III & IV
D. I & IV

Answer: C

Explanation:
Sum-of-digits results in a higher depreciation in the first year. Hence, Firm A shows lower assets,
lower income and hence lower equity. Thus, its debt-to-equity and debt-to-asset ratios are higher
than B's.

Question: 1838

Goodwill should:

A. be expensed in the period in which it is purchased from another company or organization.


B. not be amortized as it will be a benefit to the company over its entire life. Companies are assumed
to have an indefinite life.
C. written off over a 40 year period even if the superior earning power justifying its existence
disappears.
D. be amortized to income over a period not exceeding 40 years.

Answer: D

Explanation:
Goodwill can be amortized over a maximum of 40 years. However, goodwill should be amortized
over a number of periods equal to its ability to deliver superior earning power unless that period is
over 40 years.

Question: 1839

Page | 673
The primary purpose of the statement of cash flows is to:

A. measure the change in the company's assets


B. state the company's financial position at period-end
C. provide information about a company's cash receipts and cash payments during the accounting
period
D. analyze net income during the accounting period

Answer: C

Explanation:
Other financial statements are used to analyze revenues, expenses, assets, liabilities, and
stockholders' equity. The statement of cash flows summarizes transactions affecting cash; it is the
balance sheet that states the company's financial position at a point in time.

Question: 1840

Which of the following statements about ratios is false?

A. Analysis of ratios can identify areas that require further investigation.


B. All of these answers are true.
C. A ratio expresses a mathematical relation between two, or more, economically important
quantities.
D. The advantage of ratio analysis is that very little skill is required in interpreting the meaning of the
ratios.
E. Ratios must be interpreted with care since factors affecting the numerator can correlate with those
affecting the denominator.

Answer: D

Explanation:
The usefulness of ratio analysis is highly dependent upon skillful interpretation by the analyst.

Question: 1841

An increase in accounts payable would be considered ________.

A. a non-cash charge to income


B. an adjusting entry
C. a use of cash
D. a source of cash

Answer: D

Explanation:
An increase in accounts payable shows a decrease in the outflow of cash.

Question: 1842

The following financial statement depicts only one point in time:

A. Income Statement
B. Statement of Cash Flows

Page | 674
C. Statement of Retained Earnings
D. Balance Sheet

Answer: D

Explanation:
Only the balance sheet depicts the company's financial position at one point in time. All others show
what happens over a certain period of time.

Question: 1843

According to SFAC 5, the condition(s) that must be met for revenue recognition to take place is

A. the ability to measure the revenue amount.


B. there must be no remaining significant contingent obligation that pertains to the sale.
C. the verbal or written agreement of the sale.
D. the ability to recognized when the revenue actually takes place.
E. the completion of the earnings process and assurance of payment.

Answer: E

Explanation:
The firm must have provided all or virtually all the goods/services for which it is to be paid and it
must be possible to measure the total expected cost of providing the these goods/services. Also, the
seller must reasonably estimate the probability of nonpayment; if this cannot be assured, assurance
of payment is not satisfied.

Question: 1844

A firm's tax rate is 30%. If the beginning inventory was overstated by 50, the purchases understated
by 30 and the ending inventory overstated by 10, the income is ________.

A. overstated by 12
B. understated by 10
C. overstated by 21
D. understated by 7

Answer: D

Explanation:
Ending inventory = beginning inventory + Purchases - COGS Hence, in this case, COGS is overstated
by 50 + (-30) - 10 = 10. Therefore, income is understated by 10*(1-tax rate) = 7. It should be
remembered that implicit in the use of the above inventory equation is the assumption that there
have been no write-downs or write-ups in the inventory.

Question: 1845

Oils Galore is a large firm that prefers to use the Successful Efforts method of accounting. A similar
firm, Rival Oil & Gas, Inc., uses the Full Cost method. An inspection of their financial statements
would reveal that:

I. Oils Galore has a higher debt-to-asset ratio.


II. Rival O&G has higher equity.
III. Oils Galore is burdened with higher taxes.

Page | 675
IV. Rival O&G shows higher cash flows.

A. I & III
B. II & IV
C. III & IV
D. I & II

Answer: D

Explanation:
The Successful-efforts method requires expensing of all dry-hole costs while Full Cost method allows
these to be capitalized. Therefore, Rival shows higher assets, higher income and hence, higher taxes.
It also shows higher equity. Since Rival pays higher taxes, it has lower cash flows.

Question: 1846

A firm currently has a negative working capital. Which of the following is true?

A. The firm is close to bankruptcy.


B. The firm's accounting profits are low.
C. The firm's current ratio is less than 1.
D. The firm has a high debt-to-asset ratio.

Answer: C

Explanation:
Working capital = Current assets - Current liabilities. If working capital is negative, then current
liabilities exceed current assets, implying a current ratio of less than

1.

Question: 1847

Companies reporting consolidated cash flows using the direct method must report all of the
following cash receipts except for:

A. other operating cash receipts, if any.


B. cash from customers, including lessees, licensees and similar.
C. repayments of funds from foreign subsidiaries.
D. interest and dividend payments received.

Answer: C

Explanation:
Subsidiaries are part of the same company. Since the cash flows are consolidated there is no effect
on the statement for repayments of funds from foreign subsidiaries.

Question: 1848

When reporting contingencies

A. disclosure of a loss contingency is to be made if there is a remote possibility that the loss has been
incurred.
B. guarantees of others' indebtedness are reported as a loss contingency only if the loss is considered

Page | 676
imminent or highly probable.
C. disclosure of a loss contingency must include a dollar estimate of the loss.
D. none of these answers.
E. a loss that is probable but not estimable must be disclosed with a notation that the amount of the
loss cannot be estimated.

Answer: E

Explanation:
Contingent losses should be accrued if both probable and reasonably estimable. In this case, the
contingent loss is not estimable and, therefore should only be disclosed in the notes since it is
probable.

Question: 1849

The following data are taken from the Ultimate Omega's financial statements ending Dec 31, 1985:

Increase in inventories220
Stock's repurchased135
Provision for warranty expenses15
Depreciation90
Interest paid35
Outstanding bonds retired225
After-tax gain on bond retirement25
Net Income430
Tax rate40%

What was Ultimate Omega's operating cash flow in 1985?

A. 240
B. 225
C. 305
D. 290

Answer: D

Explanation:
In this case, OCF = 430 - 220 + 15 + 90 - 25 = 290. Note a few important things: Warranty provision is
a non-cash expense and must be added back to net income. Gain on bond retirement is an
extraordinary item and is part of financing cash flow. Hence, it must be subtracted from net income
to obtain income from continuing operations. Stock repurchase and bond retirement are financing
cash flows. Note that tax rate is not relevant in the indirect method since it has already been factored
in the net income figure.

Question: 1850

According to the hierarchy of accounting qualities, which of the following is considered a pervasive
constraint?

A. Timeliness
B. Benefits versus costs
C. Verifiability
D. Conservatism
E. Relevance

Page | 677
Answer: B

Explanation:
All accounting information is subject to two quantitative constraints: materiality and cost/benefit.
The constraint of benefit/cost states that the benefits of information must exceed the cost of
obtaining it.

Question: 1851

Which of the following statements about directors of a company is true?

A. Directors only get paid if the company increases its profitability that year.
B. All directors of a company are senior managers in that company.
C. Directors are shareholders' representatives.
D. Directors are elected by management of a company.

Answer: C

Explanation:
The stockholders elect the board of directors, which in turn decide on the major business policies of
the corporation.

Question: 1852

Which of the following is/are true about marketable securities?

I. They are carried on the books at the market value.


II. Transfers between different classifications are carried out at fair market value.
III. Companies are not required to disclose the classification of the specific investment securities in
the balance sheet.

A. I & III
B. I, II & III
C. II & III
D. II only

Answer: C

Explanation:
Marketable securities are classified into various categories. Securities that are designated as "held-
to- maturity" are reported at historical cost, not fair market value. Other categories prescribe a
reporting at fair market value. Thus, (I) is not necessarily true. Despite this, when the classification of
a security is changed, the assignation of the security to the new account is carried out at the fair
market value on the transfer date, with any gain or loss reflected separately on the income
statement (in the case of trading and held-to-maturity securities) or as a separate component of
shareholder equity (in the case of available-for-sale securities). Companies are not required to
disclose securities classification on the balance sheet; disclosure can occur in the footnotes.

Question: 1853

Which of the following relationship is true about the liquidity ratios?

A. Cash ratio > Quick ratio > Current ratio.

Page | 678
B. Quick ratio > Cash ratio > Current ratio.
C. Cash ratio < Quick ratio < Current ratio.
D. Cash ratio < Current ratio < Quick ratio.

Answer: C

Explanation:
The cash ratio is the most conservative of the 3 liquidity ratios since it considers only cash and
marketable securities relative to the current liabilities. The quick ratio considers receivables in
addition to cash and marketable securities. The current ratio takes all current assets into account.

Question: 1854

Which of the following is/are true about expenses?

I. R&D expenses must always be capitalized.


II. Software expenses may sometimes be capitalized.
III. In extractive industries like oil and gas, companies have a choice of either expensing or
capitalizing extractive costs.

A. I, II & III
B. II & III
C. III only
D. I only

Answer: B

Explanation:
Difficulties in measuring the current value of assets being generated by R&D prevent a reliable
application of the accrual principles, which require allocation of expenses in proportion to the
current benefits generated. Hence, US GAAP takes the easiest solution and dictates that all R&D costs
must be expensed immediately (unless they generate tangible or intangible assets which have
alternative future uses which are measurable). On the other hand, software development costs
which are similar to R&D costs are capitalized after feasibility of the product is determined. In
extractive industries, firms are allowed to use either the full-cost method, in which all search and
development costs can be capitalized, or the successful-efforts method, where all such costs are
expensed unless they result in revenue-generating assets, in which case, they are capitalized.

Question: 1855

In periods of rising prices, which inventory costing method results in the highest net income?

A. FIFO
B. Average cost
C. Perpetual
D. LIFOThat answer is correct!

Answer: A

Explanation:
FIFO assigns the lowest dollar value amounts to COGS when purchase prices are rising, thus resulting
in higher net income.

Question: 1856

Page | 679
Which of the following is/are true about an auditor's opinion?

I. If an auditor finds that the firm's financial statements fairly represent the company's financial
performance and position, he will issue an unqualified opinion.
II. If an auditor finds that some parts of the firm's financial statements are questionable or do not
have sufficient verifiable corroboration, he will issue a qualified opinion.
III. If an auditor finds that the audit is insufficient in scope to render an opinion, he will not issue an
opinion.

A. I only
B. I & III
C. I & II
D. III only

Answer: C

Explanation:
If an auditor finds that the audit is insufficient in scope to render an opinion, he will issue a
Disclaimer of opinion.

Question: 1857

Which of the following is NOT a financing cash flow?

A. Both "cash used for share repurchase" and "cash received from issuing bonds."
B. Cash used for share repurchase.
C. Cash received from issuing bonds.
D. Cash acquired from the sale of a liquid asset.

Answer: D

Explanation:
You should read the question carefully. It asks for the cash flow that is not a financing cash flow. Cash
used for share repurchase and cash received from debt issue are both financing cash flows while cash
received from asset sales is an investing cash flow.

Question: 1858

The following data pertain to Cosmotron Company's financial state as of Dec 31, 1994:

Net Income$400
Increase in inventory$120
Decrease in accounts receivable$45
Increase in payables$165
Depreciation$100
Dividends paid$85
Tax rate30%

What was Cosmotron's operating cash flow in 1994?

A. $590
B. $490
C. $675

Page | 680
D. $505That answer is correct!

Answer: A

Explanation:
In this case, OCF = 400 - 120 + 45 + 165 + 100 = $590. Dividends paid are investing cash flows and not
part of OCF. Note that tax rate is not relevant in the indirect method since it has already been
factored into in the net income figure.

Question: 1859

The accumulated depreciation account should show:

A. total depreciation for fixed assets since the business was formed
B. only the depreciation expense recognized during the current year
C. total depreciation for fixed assets still in use
D. current depreciation expense plus estimated depreciation for next year

Answer: C

Explanation:
Depreciation recognized and accumulated on fixed assets still on the books is reflected in the
Accumulated Depreciation account(s).

Question: 1860

While comparing a firm that expenses costs with one that capitalizes them:

I. The capitalizing firm always shows higher cash flow from operations.
II. The cumulative difference between the cash flow from operations increases over time.
III. The capitalizing firm pays lower taxes in the first year.
IV. The capitalizing firm recognizes higher deferred tax liabilities.

A. II & III
B. I, II & IV
C. II, III & IV
D. I & IV

Answer: B

Explanation:
Expensing charges the entire cost to CFO while capitalization leads to a charge against the investing
cash flow. Over time, depreciation reduces the tax payments. Hence, the cumulative difference in the
CFOs increases over time. The expensing firm reports lower income in the first year and hence, pays
lower taxes.

Question: 1861

On June 30, 1996, Union Inc. purchased goodwill for $125,000 when it acquired the net assets of
Apex Corp. During 1996, Union incurred additional costs of developing goodwill by training Apex
employees ($50,000) and hiring additional Apex employees ($25,000). Before amortization of
goodwill, Union's December 31, 1996 balance sheet should report goodwill of ________.

A. $150,000

Page | 681
B. $200,000
C. $75,000
D. $175,000
E. $125,000

Answer: E

Explanation:
Goodwill is recorded only when it is purchased. In this case, only $125,000 was purchased goodwill.
The costs incurred to maintain, develop, or restore goodwill must be expensed. In this case, $75,000
will be expensed.

Question: 1862

Which of the following is/are true under accrual accounting?

I. Revenues are recognized when goods are delivered.


II. Revenues are recognized when cash is received.
III. Matching principle accounting is followed under accrual accounting.
IV. Cash outflows determine expense recognition.

A. I, III & IV
B. III only
C. I & III
D. I & IV

Answer: C

Explanation:
Accrual accounting follows the Matching Principle, which requires that revenues and gains be
recognized when earned and expenses and losses be recognized when incurred. Hence, revenue
recognition is completely separate from cash flows. In the sales process, the earnings process is
completed when goods are delivered and that's when revenue is recognized under accrual
accounting.

Question: 1863

Revenues of an entity are usually measured by the exchange values of the assets or liabilities
involved. Recognition of revenue does not occur until

A. the entity has substantially accomplished what it is agreed to do.


B. the revenue is realizable.
C. none of these answers.
D. products or services are exchanged for cash or claims to cash.
E. the revenue is realized and earned.

Answer: E

Explanation:
Revenues are only recognized when they have been earned. Simply exchanging assets or liabilities
does not result in revenue recognition.

Question: 1864

Page | 682
To examine the trends over time within a single firm or compare firms of different sizes, you should
use:

A. balance-sheet analysis.
B. common industry analysis.
C. common-size statements.
D. cash flow analysis.

Answer: C

Explanation:
Common size statements normalize the financial data to allow easier comparison of different-size
firms.

Question: 1865

Stock dividends have the greatest impact on which section of the balance sheet?

A. total assets
B. total stockholders' equity
C. total assets and total stockholders' equity
D. retained earnings
E. the components of stockholders' equity

Answer: E

Explanation:
The effect of stock dividends is recorded by transferring part of the retained earnings to the
contributed capital accounts. Because of the increase in contributed, it is often described as
capitalizing retained earnings.

Question: 1866

The following data are available for a firm for a given year:

Net Sales21,896
Sales & marketing expenses4,346
Administrative expenses2,143
COGS10,084
Depreciation967
Interest expense573
Tax rate35%
Dividends paid3,445
Preferred Dividends897
Average total equity37,432
Average common equity26,782
Average total liabilities18,583

In the above example, the firm's net profit margin equals ________.

A. 0.23
B. 0.17
C. 0.20
D. 0.11

Page | 683
Answer: D

Explanation:
Net Profit Margin = Net Income/Net Sales Net Income = Earnings after depreciation, interest expense
and taxes
= (21,896 - 4,346 - 2,143 - 10,084 - 967 - 573)*(1 - 0.35) = 2,459.
Therefore, Net Profit Margin = 2,459/21,896 = 0.11.

Question: 1867

Capital stock (common stock)

A. is a higher risk investment vehicle for the buyer than would be the purchase of bonds or preferred
stock.
B. represents the residual ownership of a firm.
C. none of these answers.
D. is typically the only type of stock that maintains voting privileges.

Answer: B

Explanation:
Capital (common) stock is a class of stock that represents ownership interest in a firm. Common
stockholders are typically the only shareholders that have voting privileges because they are the true
owners of the firm. Investors bear the greatest risk when purchasing common stock because
common stockholders stand last in line when it comes to the payment of income, or assets in the
event of liquidation. By like token, common stockholders reap the greatest rewards because all
earnings of a firm in excess of all obligations belong to the common stockholders.

Question: 1868

On a statement of cash flows that uses the indirect approach, calculation of cash flow from
operations treats depreciation as an adjustment to reported net income because

A. depreciation is a direct source of cash.


B. depreciation is an outflow of cash to a reserve account for the replacement of assets.
C. depreciation reduces net income but does not involve an outflow of cash.
D. depreciation reduces net income and involves an outflow of cash.

Answer: C

Explanation:
Depreciation is an expense that reduces net income, but it is a non-cash item.

Question: 1869

Use the following financial data on Enterprise:

a. Sale of equipment$32,000
b. Loss on equipment sale$9,000
c. Dividends paid$12,500
d. Purchase of an office suite$104,000
e. Common stock repurchase$45,000
f. Dividends received from investments$8,500

Page | 684
g. Interest received on Treasury bonds$1,200
h. Supplier accounts paid$3,700
i. Cash collection from customers$14,200
j. Ending cash balance$98,000

In the above question, the financing cash flow is ________.

A. -$12,500
B. -$45,000
C. -$57,500
D. -49,500

Answer: C

Explanation:
Items c and e are the financing cash flows.

Question: 1870

Which of the following would affect the comparison of financial statements across two different
firms?

I. different accounting principles


II. different accounting estimates
III. different reporting periods
IV. different industries

A. I and IV
B. I, III and IV
C. I, II, III and IV
D. I and II

Answer: C

Explanation:
All of these responses would affect the comparison of financial statements across two different firms.

Question: 1871

Which of the following statements about stock dividends is true?

A. Stock dividends are dividends given in the form of stock from another company.
B. Stock dividends are more valuable than stock splits.
C. Stock dividends are recorded as a reduction in cash.
D. Stock dividends increase the number of shares outstanding.

Answer: D

Explanation:
Stock dividends increase the number of shares outstanding of the issuing company and do not
reduce cash.

Question: 1872

Page | 685
Under an inflationary environment with stable inventories, a firm may change to LIFO from FIFO due
to which of the following reason(s)?

I. To allow earnings manipulation.


II. To improve the reported current ratio.
III. To reduce tax drain on cash.
IV. Show a more accurate representation of reported assets than FIFO.

A. I, II, III & IV


B. I & II
C. I & III
D. I, III & IV

Answer: C

Explanation:
Under LIFO, inventory is understated and hence, current ratio is understated under normal,
inflationary conditions. At the same time, COGS is larger than under FIFO due to rising prices,
reducing the taxable income and leading to lower taxes. Further, firms can manipulate earnings
under LIFO by changing purchasing patterns at the end of an accounting period. This happens
because the LIFO COGS is determined by the prices of the latest goods purchased.

Question: 1873

Which of the following information would not be filed with the SEC by a publicly company?

A. proxy statement
B. tax return
C. 10K report
D. prospectus

Answer: B

Explanation:
Tax returns are filed with the IRS.

Question: 1874

The following data have been extracted from the financial statements of a firm for two years, 1991
and 1992:

19911992
Assets9,9479,682
Sales11,03510,377
Current liabilities4,1933,920
Non-current liabilities3,8273,476

The asset turnover and the equity turnover for 1992 equal ________.

A. 0.98, 2.56
B. 1.06, 4.93
C. 2.11, 3.92
D. 1.34, 6.38

Page | 686
Answer: B

Explanation:
Total asset turnover = net sales/average total assets
Total equity turnover = net sales/average equity

The total liabilities in 1991 = 4,193 + 3,827 = 8,020 and total liabilities in 1992 = 3,920 + 3,476 =
7,396. Therefore, using the equation,
Total Assets = Total Liabilities + Total Equity,
the equity in 1991 was 9,947 - 8020 = 1,927 and in 1992, it was 9,682 - 7,396 = 2,286. Thus, the
average equity for 1992 equals (1,927 + 2,286)/2 = 2,107 and average assets for 1992 equal (9,682 +
9,947)/2 = 9,815. Therefore, asset turnover for 1992 = 10,377/9,815 = 1.06 and equity turnover =
10,377/2,107 = 4.93.

Question: 1875

The two primary qualities of accounting information to make it useful for decision-making are
________.

A. materiality and comparability


B. full disclosure and relevance
C. reliability and comparability
D. relevance and reliability

Answer: D

Explanation:
The financial information used to make decisions should be both reliable and relevant.

Question: 1876

A firm has entered into a long-term purchase obligation. This will allow the firm to obtain raw
materials for operations at a known price over the next 6 years. This obligation must be reflected on
the balance sheet as:

A. a current liability until the first purchase occurs.


B. a footnote disclosure.
C. none of these answers.
D. a long-term liability.

Answer: B

Explanation:
It is important to recognize that liabilities arise and are recognized on the balance-sheet only if the
firm has consumed at least a part of the services or goods under the contract. Until then, a liability is
not admitted since there is no corresponding asset or equity that has been generated (a similar
principle holds for the recognition of assets). However, due to the bonding nature of the contract, it
must be disclosed in footnotes along with the relevant details so that analysts can figure in the
impact of all such agreements while evaluating the future health of the firm.

Question: 1877

Sparten, Inc., a plumbing contractor, received a check for $3,000 on June 29 for services to be
performed in the following fiscal month. During the July accounting period, Sparten completed all

Page | 687
but $500 of the job. What adjusting entry needs to be made at the end of July?

A. increase Unearned revenue and decrease cash for $500


B. debit Unearned Revenue and credit Revenue for $2,500
C. increase cash and revenue for $3,000
D. decrease accounts receivable and increase revenue for $2,500

Answer: B

Explanation:
When the check for was received on June 30, a liability was established for $3,000 because revenue
could not be recognized until it was earned. During July, $2,500 of revenue was earned and should be
recognized, along with a reduction (debit) to the Unearned Revenue account.

Question: 1878

The following data are taken from the AlphaBeta Company's financial statements ending Dec 31,
1990:

Increase in salaries payable120


Decrease in accounts payable45
Provision for bad debt15
Depreciation90
Interest paid35
Proceeds from new bonds issued225
Net Income350
Tax rate35%

What was AlphaBeta's operating cash flow in 1990?

A. 450
B. 405
C. 720
D. 530

Answer: D

Explanation:
In this case, OCF = 350 + 120 - 45 + 15 + 90 = 530. Proceeds from bond issue are part of financing cash
flow, not of OCF. Note that tax rate is not relevant in the indirect method since it has already been
factored into in the net income figure. Also, only non-cash items need to be adjusted.

Question: 1879

New Gestalt, Inc., a software firm had a net income of 1.7 million last year. It has 200,000 common
shares and 300,000 convertible bonds with face value of 100 outstanding. The convertible bonds
carry a coupon of 4% and can be converted one-for-one. The average stock price last year was 39 and
the maximum price was 57. The effective interest rate on the convertible debt is 8%. New Gestalt
issued 100,000 preferred shares with face value 100 and a coupon of 5% on March 31st of last year.
Assume the convertible bonds are dilutive and that New Gestalt faces a 30% tax rate. Given the
above, New Gestalt's Diluted EPS equals ________.

A. 7.45
B. 2.65

Page | 688
C. 6.01
D. 7.51

Answer: C

Explanation:
Diluted EPS = (Net Income - Preferred dividends + interest on convertibles net of taxes)/(weighted #
of common shares assuming convertible conversion) In this case, numerator = 1.7 million - 375,000 +
300,000*8%*100*(1-30%) = 3,005,000. The denominator equals 200,000 + 300,000 = 500,000.
Therefore, Diluted EPS = 6.01. Note that the interest expense on the convertible bond is calculated
using the effective interest rate, not the coupon rate.

Question: 1880

Which of the following are changes in accounting principle?

I. a change from LIFO to FIFO


II. a change in estimated salvage value of depreciable asset
III. a change from an accelerated depreciation method to straight line depreciation
IV. recording depreciation for first time on machinery purchased five years ago

A. I, II, III and IV


B. I, II and III
C. I, III and IV
D. I and III

Answer: D

Explanation:
Both I and III involve changes in the method of accounting.

Question: 1881

Under accrual accounting, which of the following is/are correct?

I. Revenues are recognized when cash is received.


II. Expenses do not always involve cash flows.
III. Revenues and the related costs are matched in the same period.
IV. The reported income is a good indicator of the firm's current performance.

A. II, III & IV


B. I & III
C. I, II, III & IV
D. I & IIThat answer is correct!

Answer: A

Explanation:
Accrual accounting requires that revenues and gains be recognized when earned and expenses and
losses be recognized when incurred. Hence, revenue recognition is completely separate from cash
flows. In the sales process, the earnings process is completed when goods are delivered and that's
when revenue is recognized under accrual accounting.

Question: 1882

Page | 689
Stockholders' equity may contain all of the following components, except:

A. Minimum pension liability


B. Minority interests in consolidated subsidiaries
C. Employee Stock Option Plan accruals
D. Treasury stock
E. Reinvested earnings

Answer: B

Explanation:
Classified as a liability, minority interests in consolidated subsidiaries represent the equity held by
investors other than the subject firm in those subsidiaries.

Question: 1883

The "Treasury Stock" method of calculating EPS involves:

A. considering call option exercise when the average stock price is below strike price.
B. considering call option exercise when the average stock price is above the strike price.
C. using options as stock equivalents only if they are anti-dilutive.
D. considering repurchase of shares at the highest market price.

Answer: B

Explanation:
In the Treasury stock method, it is assumed that the proceeds from the exercise of warrants and
employee call options are used to repurchase shares at the average market price during the period.
Considering call option exercise when the average stock price is below strike price is incorrect since it
is an irrational strategy and leads to anti-dilution of EPS.

Question: 1884

Firms X and Y, similar in all respects, recently bought identical securities. However, X has classified
the securities as "held-to-maturity" securities while Y has categorized them as "available-for-sale"
securities. Which of the following statements is/are true as a result of this difference?

I. X and Y will show same assets on their balance sheets over time.
II. X will have a higher income volatility than Y.
III. X and Y will show different total equity values.

A. I & II
B. I & III
C. I, II & III
D. III only

Answer: D

Explanation:
"Available-for-sale" securities are reported at their fair market value on the balance sheet while
"held- to-maturity" securities are carried at amortized historical cost. Therefore, the reported assets
for X and Y will be different over time. Hence, they will also show different equities because liabilities
for the two firms are the same. However, changes in the market value of available-for-sale securities

Page | 690
are charged directly to the retained earnings account, without going through the income statement.
On the other hand, changes in the market value of "held-to-maturity" securities are not even
recognized on financial statements due to historical cost accounting. For both the classifications,
interest and dividend payments are attributed to the income. So the income volatility is not affected
due to the classification.

Question: 1885

The basic asset-liability equation, assets = liabilities + shareholder equity implies:

A. operating activities = financing activities


B. investing activities = financing activities
C. investing activities = operating activities
D. none of these answers

Answer: B

Explanation:
The asset-liability equation has the results of investing activities on the left-hand side as assets. On
the right hand side are results of creditor financing (debt) and equity financing.

Question: 1886

Under GAAP, when interest rates rise, the carrying value of a long-term bond ________.

A. remains unchanged
B. increases
C. depends on the characteristics of the bond
D. decreasesThat answer is correct!

Answer: A

Explanation:
Long-term debt is recorded at acquisition cost and historical cost accounting does not allow
revaluation. Hence, even though the market value of the debt changes due to a change in interest
rates, the book value of the debt changes only at the regular amortization schedule, independent of
the market changes.

Question: 1887

The following is true about liquidating dividends:

A. all of these answers


B. these dividends return part of the original investment to the stockholders
C. the cash dividend account is closed to retained earnings
D. they only occur when the company is making a major acquisition

Answer: B

Explanation:
The cash dividend account is not affected, and they would not likely occur when making an
acquisition.

Question: 1888

Page | 691
The best indication of a firm's present and continuing ability to generate favorable cash flows is
information about enterprise earnings based on which of the following basis?

A. Tax accounting.
B. Cash accounting.
C. Modified cash accounting.
D. Accrual accounting.
E. Modified accrual accounting.

Answer: D

Explanation:
Information about an enterprise's earnings based on accrual accounting generally provides a better
indication of the firm's present and continuing ability to generate favorable cash flows than would
information limited to the financial effects of cash receipts and payments.

Question: 1889

When prices are falling, which of the following is/are true?

I. FIFO results in higher current assets.


II. LIFO results in higher taxes.
III. LIFO results in higher income.
IV. FIFO allows earnings manipulation through purchasing behavior.

A. II & III
B. II, III & IV
C. I & IV
D. II onlyThat answer is correct!

Answer: A

Explanation:
When prices are falling, the units purchased later have a lower purchase price. Therefore, under the
FIFO cost-flow assumption, the higher cost goods will be assumed to have been sold first. This will
raise COGS, lower the ending inventory value (and therefore lower current assets), lead to lower
income and hence lower income taxes compared to LIFO. However, earnings manipulation is not
possible under FIFO.

Question: 1890

If an overstatement of depreciation by 100 and an understatement of accounts receivables by 40


causes a firm's net income to be understated by 70, the firm's tax rate is ________.

A. 35%
B. 50%
C. 40%
D. 30%

Answer: D

Explanation:
An overstatement of depreciation by 100 causes the income to be understated by 100*(1-tax rate).

Page | 692
Hence, tax rate = 30%.

Question: 1891

Excerpts from the balance sheet of Milton Corporation as of April 30, 1997 are presented as follows:

Cash $725,000 Accounts receivable (net) $1,640,000 Inventories $2,945,000 Total current assets
$5,310,000 Accounts payable $1,236,000 Accrued liabilities $831,000 Total current liabilities
$2,067,000

Cash$725,000
Accounts receivable (net)$1,640,000
Inventories$2,945,000
Total current assets$5,310,000
Accounts payable$1,236,000
Accrued liabilities$831,000
Total current liabilities$2,067,000

The board of directors of Milton met on May 5, 1997 and declared a quarterly cash dividend in the
amount of $200,000 ($0.50 per share). The dividend was paid on May 28, 1997 to shareholders of
record as of May 15, 1997. Assume that the only transactions that affected Milton during May 1997
were the dividend transactions. If the dividend declared by Milton had been a 10% stock dividend
instead of a cash dividend, Milton's current liabilities would have been

A. unchanged by either the dividend declaration or the dividend distribution.


B. unchanged by the dividend declaration and decreased by the dividend distribution.
C. decreased by the dividend declaration and increased by the dividend distribution.
D. increased by the dividend declaration and decreased by the dividend distribution.
E. increased by the dividend declaration and unchanged by the dividend distribution.That answer is
correct!

Answer: A

Explanation:
The declaration and distribution of a stock dividend involves transferring an amount from retained
earnings to common stock. However, there is no impact on current liabilities as there is no future
obligation created to distribute cash.

Question: 1892

When preparing a direct method statement of cash flows, the net cash flow must equal

A. the cash collections minus the cash expenses.


B. the cash collections plus the net noncash revenues and expenses.
C. the cash flow from operations plus the cash flow from investments.
D. the change in cash.
E. the cash collections plus the changes in the operating accounts.

Answer: D

Explanation:
Using the direct method, the net cash flow must equal the change in cash.

Question: 1893

Page | 693
If a company changes the useful life of its assets from 10 years to 12 years, this will be recorded as
________.

A. a non recurring gain


B. a change in accounting principle
C. an extraordinary item
D. none of these answers

Answer: D

Explanation:
The depreciation schedule for each asset affected is simply recalculated for the remainder of the life.

Question: 1894

Which of the following is a current liability?

A. Bonds
B. Depreciation
C. Preferred stock
D. Accounts payable
E. None of these answers

Answer: D

Explanation:
Only accounts payable would be a current liability. Current liabilities must be settled over the longer
of one year or a firm's operating cycle; thus, the maximum time frame for settlement is one year.
Preferred stock is generally not repaid, bonds are repaid over many years and depreciation is an
expense not a liability.

Question: 1895

Which of the following is/are revenue recognition methods?

I. Completed Contract Method


II. Successful Efforts Method
III. Deferred Collection Method
IV. Full Cost Method

A. I, II, III & IV


B. I only
C. I & III
D. I, II & IV

Answer: B

Explanation:
II and IV refer to the methods under which costs in drilling oil wells can be capitalized. There is no
such thing as
III.

Question: 1896

Page | 694
When using transactional analysis, the following changes in the balance sheet account and the
corresponding cash flow description would be correct except,

A. long-term debt; increase or decrease in debt


B. rent payable; cash paid for inputs
C. accounts payable; cash paid for inputs
D. short-term debt; increase or decrease in debt
E. retained earnings; dividends paid

Answer: B

Explanation:
The cash flow of rent payable would be cash expenses.

Question: 1897

Over a given year, a firm's total assets increased by $7,000 and total liabilities decreased by $4,000. If
the firm did not issue any new equity and paid out $1,500 in dividends, then its net income during
the year was ________.

A. $1,500
B. $9,500
C. $4,500
D. $12,500

Answer: D

Explanation:
Total assets - Total liabilities = Equity. Hence,
Change in Assets - Change in Liabilities = Change in Equity.
Thus, change in equity = 7,000-(-4,000) = $11,000
All of the net income not paid out as dividends goes into increasing the equity. Since no new shares
were issued, Net Income = change in equity + dividends paid. This gives net income =
$11,000+$1,500 = $12,500.

Question: 1898

An EPS amount is always shown for:

A. all of these answers


B. income before extraordinary items and the cumulative effect of accounting changes
C. income from continuous operations
D. cumulative effect of accounting changesThat answer is correct!

Answer: A

Explanation:
All of these responses are true.

Question: 1899

A firm currently has a financial leverage ratio of


2. After a thorough review, the firm's management has concluded that they have to write-down

Page | 695
assets worth 200,000. This will cause the firm's financial leverage to ________.

A. decrease
B. insufficient information given
C. increase
D. stay constant

Answer: C

Explanation:
The write-down of assets effectively decreases assets and equity by the same amount, once tax
effects are correctly figured. Since A > E, a decrease in both quantities by the same amount increases
the financial leverage (A/E) ratio.

Question: 1900

According to GAAP classification of cash flows, all of the following are investing cash flows except:

A. purchase of an office building.


B. gains on asset sales.
C. dividends received from investments in stocks.
D. takeovers financed partly with cash.

Answer: C

Explanation:
Dividends received from stock investments are considered operating cash flows while dividends paid
on equity are considered financing cash flows.

Question: 1901

Stock splits have the following effects on the financial statements except:

A. the account title for common stock changes to reflect the change in the par value of stock
B. contributed capital and retained earnings are unchanged
C. disclosures about the stock on the balance sheet are changed to reflect the additional outstanding
shares and the revised par value per share
D. the shareholder's percentage interest in the corporation is changed by the percentage change in
the market value of the stock

Answer: D

Explanation:
The shareholder's percentage interest in the corporation is unchanged.

Question: 1902

Hupta Corporation reports cost of goods sold for the year ended December 31, 1998 of $3,500. Other
information as of December 31 is as follows:

19971998

Accounts Receivable$500$550
Inventory$400$380

Page | 696
Accounts Payable$250$290

Cash paid to suppliers for year ended December 31, 1998 is ________.

A. $3,520
B. $3,480
C. $3,440
D. $3,560

Answer: C

Explanation:
An increase in accounts payable shows that $40 less than the $3,480 of purchases ($3,500 COGS +
(20) change in inventory) was paid: $3,480 - $40 = $3,440.

Question: 1903

A firm using LIFO accounting has a LIFO reserve of 600, with a LIFO ending inventory of 4,900. It is
currently in the 40% tax bracket. If it switches to FIFO accounting, it's equity ________.

A. decreases by 240
B. decreases by 360
C. increases by 360
D. increases by 240

Answer: C

Explanation:
The presence of a LIFO reserve implies that by following LIFO, the firm has been able to deduct
higher COGS from gross income than under FIFO (remember that LIFO reserve equals inventory value
under FIFO minus inventory value under LIFO and this will be positive when the COGS under FIFO is
lower than that under LIFO). thus, the retained earnings (and hence equity) is understated by LIFO
Reserve*(1 - tax rate). Therefore, when the firm switches from LIFO to FIFO, the equity is increased
by 600*(1 - 0.4) = 360.

Question: 1904

A firm with a simple capital structure had a net income of 7,700 last year. It had 1,000 common
shares outstanding and its reported EPS was 6.6. What was the firm's payment to its preferred stock
holders?

A. 1,300
B. 900
C. 1,100
D. 1,200

Answer: C

Explanation:
For a simple capital structure, EPS = (Net Income - Preferred stock dividends)/weighted # of common
shares This gives 6.6 = (7700- Preferred stock dividends)/1000. Hence, payment to preferred equity
holders = 1,100.

Question: 1905

Page | 697
A firm's accountant has mistakenly overstated depreciation by 50 and understated accounts
receivables by 35. The firm's tax rate is 40%. Then, which of the following is/are true?

I. Income is understated by 30.


II. Income is understated by 50.
III. Operating cash flow is overstated by 55.
IV. Operating cash flow is overstated by 85.

A. I, II & III
B. II, III & IV
C. I & III
D. II & IV

Answer: C

Explanation:
If depreciation is overstated by 50, the income is actually understated by 50*(1-tax rate) = 30. The
understatement of accounts receivables does not affect the income statement; it understates the
reported current assets. It also affects the operating cash flow. Note that operating cash flow = net
income + noncash expenses - non-cash revenues - cash reductions in operating accounts Since
income is understated by 30, non-cash expense (depreciation) is overstated by 50 and noncash
revenue (accounts receivable) is understated by 35, operating cash flow is overstated by -30 + 50 - (-
35) = 55.

Question: 1906

Prepaid expenses are:

A. classified as current liabilities on the balance sheet.


B. advance payments for goods not yet received which extend beyond the current accounting period.
C. not included in the calculation of a company's working capital.
D. advance payments for work that the company has not yet performed. The work will be completed
within the current accounting period.

Answer: B

Explanation:
Prepaid expenses occur when you pay for a service before you actually receive the benefits of the
service.

Question: 1907

When prices are rising, which of the following inventory valuation methods produces a lower ending
inventory value?

A. LIFO
B. Average cost
C. None of these answers
D. FIFOThat answer is correct!

Answer: A

Explanation:

Page | 698
When prices are rising, LIFO will produce the lowest ending inventory value because the last in
(highest cost) inventory is the first out. This leaves the oldest inventory on hand and since it was
purchased at lower prices, it has the lowest cost and ending inventory will have the lowest value.

Question: 1908

Under the current US GAAP, the treatment of R&D expenses:

I. underestimates current income and leads to lower taxes.


II. leads to a more conservative balance sheet.
III. is tantamount to asserting that R&D outlays have no current benefits.

A. I only
B. none of these are correct
C. I, II & III
D. I & II

Answer: C

Explanation:
Difficulties in measuring the current value of assets being generated by R&D prevent a reliable
application of the accrual principles, which require allocation of expenses in proportion to the
current benefits generated. Hence, US GAAP takes the easiest solution of expensing all the R&D costs
(unless they generate tangible or intangible assets which have alternative future uses which are
measurable). Since R&D does generate future benefits on an expected basis, the full expensing of the
costs when incurred tends to under-estimate current income and leads to lower taxes. Further, when
the R&D activities do lead to profitable results, income gets overstated because of non-recognition of
a portion of it in previous periods. Thus, R&D expensing adds volatility to income. On the other hand,
expensing leads to a more conservative balance sheet since no assets or equity is recognized until
the benefits actually materialize. Thus, by assuming that current R&D has no current benefits, the
GAAP accounting tries to prevent "bad" surprises.

Question: 1909

Which of the following could decrease outstanding capital stock?

A. The exercise of warrants.


B. All of these answers.
C. The retention of earnings.
D. The payment of a cash dividend.
E. The purchase of treasury stock.

Answer: E

Explanation:
Only the purchase of treasury stock would cause a firm's capital stock account to decrease because
the firm acquires outstanding shares and immediately retires these shares.

Question: 1910

Which of the following is/are not extraordinary items?

I. Write-downs of inventory
II. Expropriation by a foreign government

Page | 699
III. Losses due to hurricanes
IV. Gains from disposal of a business segment

A. I & III
B. I & IV
C. II, III & IV
D. I & II

Answer: B

Explanation:
Gains from disposal of a business segment are part of "Discontinued Operations," not extraordinary
items. They appear segregated from the income from continuing operations as well as extraordinary
items and are reported net of tax effects. Inventory write-downs and other such asset revaluations
are considered unusual but not extraordinary items. These appear as part of income from continuing
operations.

Question: 1911

A liability can be recognized when

A. when the amount is uncertain


B. when the identity of the creditor is uncertain
C. an obligation exists to make a future payment based on a past event
D. all of these answers are correct

Answer: D

Explanation:
The amount or the creditor need not be certain and some future payments such as wages to be paid
in the future may not be reported as liabilities.

Question: 1912

Over a given year, a firm's total assets increased by $8,000 and total liabilities decreased by $6,000. If
the firm did not issue any new equity and paid out $1,800 in dividends, then its net income during
the year was ________.

A. $12,200
B. $200
C. $3,800
D. $15,800

Answer: D

Explanation:
Total assets - Total liabilities = Equity. Hence,
Change in Assets - Change in Liabilities = Change in Equity.
Thus, change in equity = 8,000-(-6,000) = $14,000 All of the net income not paid out as dividends
goes into increasing the equity. Since no new shares were issued, Net Income = change in equity +
dividends paid. This gives net income = $14,000 + $1,800 = $15,800.

Question: 1913

Page | 700
The reason that the translation of foreign subsidiaries will generate discrepancies between the
changes in accounts reported on the balance sheet and those reported in the cash flow statement is

A. the change in the exchange rate does not appear on the balance sheet but will appear as a
component of cash collections it is not a change resulting from operations.
B. the change in the exchange rate appears as part of the balance of accounts receivable on the
balance sheet but will not appear as a component of cash collections because it is not a change
resulting from operations.
C. the change in the exchange rate does not appear on the balance sheet but will appear as a
component of cash from financing activities.
D. the change in the exchange rate appears as part of the stockholders' equity on the balance sheet
but will not appear as a component of cash collections because it is not a change resulting from
operations.

Answer: B

Explanation:
Thus, cash flow from operations does not include the effects of the translation process.

Question: 1914

Which of the following is/are true under accrual accounting?

I. Expenses are recognized as services are used.


II. Revenues are recognized when service is performed.
III. Revenues are recognized in proportion to expenditures incurred.
IV. Revenues are matched with the associated costs.

A. I & III
B. I, II, III & IV
C. I, II & IV
D. II, III & IV

Answer: C

Explanation:
Accrual accounting follows the Matching Principle, which requires that revenues and gains be
recognized when earned and expenses and losses be recognized when incurred. Under this, (I), (II)
and (IV) are correct. However, (III) is not necessarily true.

Question: 1915

The following data have been extracted from the financial statements of a firm for two years, 1993
and 1994:

19931994
Assets10,89512,444
Sales8,4659,275
Inventory3,1263,549
COGS7,1207,387

The average inventory processing period for 1994 equals ________.

A. 164.9 days

Page | 701
B. 223.6 days
C. 114.7 days
D. 198.2 daysThat answer is correct!

Answer: A

Explanation:
This can be estimated using either net sales (as is the case with receivables) or the cost-of-goods-
sold. COGS is preferable since it does not include the profit margins involved in net sales.

Therefore, two relevant ratios are:

a. Inventory turnover ratio = COGS/average inventory.


b. Average inventory processing period = 365/inventory turnover.

Typically, average inventory for a given year is taken to be the average of the ending values of the
inventory for this year and the last year. For 1994, the average inventory equals (3549+3126)/2 =
3,338. Inventory turnover ratio = 7,387/3,338 = 2.21. Average inventory processing period =
365/2.21 = 164.9 days.

Question: 1916

Which of the following groups will have seniority in the event of bankruptcy of a company?

A. all have same seniority


B. preferred stockholders
C. common stockholders
D. debt holders

Answer: D

Explanation:
Debt holders are paid first.

Question: 1917

What does FASB refer to?

A. The Financial Analysts Standards Board.


B. The Federal Accounting Society Board of Directors.
C. The Financial Accounting Standards Board.
D. None of these answers.

Answer: C

Explanation:
The Financial Accounting Standards Board.

Question: 1918

A product financing arrangement exists when

A. the party who manufactures the inventory gives extended terms to certain preferred customers.
B. a party transfers ownership of inventory to a purchaser yet retains the inventory in their own

Page | 702
warehouse.
C. all of the answers would be accounted for as product financing arrangements.
D. a party bearing the risks and rewards of ownership transfers inventory to a purchaser, and in a
related transaction agrees to repurchase the product at a specified price over a specified time.

Answer: D

Explanation:
The key to product financing is that the purchaser retains very little of the risk because typically
whatever does not sell, the seller agrees to buy back at a specified price.

Question: 1919

Which of the following qualifies as a disposal of a segment of a business?

A. Perrier Inc., producer of bottled spring water and soft drinks, discontinues production of its
bottled spring water. This operation is self-contained.
B. Superpharm Drugs, operator of chain of discount drug stores, reaches an agreement to sell its
Boston location to a competitor.
C. Irving Inc. supplies its own service stations and independent operators from its refineries. The firm
decides to discontinue marketing to independents.
D. None of these answers.
E. Treadrite, a shoe manufacturer, has decided to discontinue its line of work boots. The firm's
operations are concentrated in single plant.That answer is correct!

Answer: A

Explanation:
A segment is a component of an entity whose activities represent a separate major product line and
activities can be physically and operationally distinguished from the remainder of the entity for
financial reporting purposes. The discontinued spring water operation thus qualifies as disposal of a
segment.

Question: 1920

Analysis, Inc. had gross sales of 5,000 last year. Its operating expenses amounted to 339 and cost of
goods sold equaled 2,386. Analysis faces a corporate tax rate of 37% and had no other revenues or
costs last year. Its net income after taxes was ________.

A. 1,647
B. 1,433
C. 2,275
D. 1,308

Answer: B

Explanation:
Analysis' gross profit = gross sales - cost of goods sold = 5,000 - 2,386 = 2,614. The operating
expenses are deductible for tax purposes. Hence, pre-tax income equaled 2,614 - 339 = 2,275.
Therefore, its after-tax income equals 2,275*(1-0.37) = 1,433.

Question: 1921

Extraordinary items are presented on the income statement:

Page | 703
A. net of taxes, separate from income from continuing operations.
B. in the footnotes, with the required disclosures.
C. as part of income from continuing operations, before taxes.
D. none of these answers.That answer is correct!

Answer: A

Explanation:
The present GAAP applied to extraordinary items is a refined form of the "Current Operating
Performance." It mandates that income effects of extraordinary items are to be separately disclosed
net of taxes after the presentation of the net income from continuing operations.

Question: 1922

Which of the following methods does a company use to determine what the balance of the
allowance for uncollectible accounts should be?

A. percent of accounts receivable


B. neither of these answers are correct
C. both of these answers are correct
D. aging

Answer: C

Explanation:
The aging and percent of Accounts Receivable methods calculate what the balance of the allowance
account should be, however, percent of net sales method calculates the actual expense for the
period.

Question: 1923

A firm using LIFO accounting has a LIFO reserve of 4,700, with a FIFO ending inventory of 34,600. It is
currently in the 40% tax bracket. If it switches to FIFO accounting for reporting purposes, it's deferred
taxes ________.

A. decrease by 2,820
B. increase by 2,820
C. increase by 1,880
D. decrease by 1,880

Answer: C

Explanation:
When the firm switches from LIFO to FIFO, it must recognize the additional tax liability that arises
because the taxes not paid in the past will now have to be paid. This liability equals the LIFO reserve
times the current tax rate and must be added to the deferred taxes account (to be completely
accurate, you would have to go back all the way to the inception of the firm and recalculate historical
taxes. This is never done).

Question: 1924

A common-size balance sheet consists of which of the following?

Page | 704
A. Expressing all items on the balance sheet as a percentage of total capital.
B. Expressing all items on the balance sheet as a percentage of total sales.
C. Expressing all items on the balance sheet as a percentage of total assets.
D. None of these answers.
E. Expressing all items on the balance sheet as a percentage of total liabilities.

Answer: C

Explanation:
Expressing all items on the balance sheet as a percentage of total assets allows the analyst to review
several years of balance sheets at a glance and to determine the relative changes that have taken
place within balance sheet accounts.

Question: 1925

Which of the following is not a current liability?

A. Notes Payable
B. Wages Payable
C. Allowance for Uncollectible Accounts
D. Unearned Revenue

Answer: C

Explanation:
Allowance for Uncollectible Accounts is a contra account to Accounts Receivable, both of which are
reported in the current asset section of the balance sheet.

Question: 1926

Stock dividends have the greatest impact on which section of the balance sheet?

A. the components of stockholders' equity


B. total stockholders' equity
C. total assets
D. total assets and total stockholders' equity
E. retained earningsThat answer is correct!

Answer: A

Explanation:
The effect of stock dividends is recorded by transferring part of the retained earnings to the
contributed capital accounts. Because of the increase in contributed, it is often described as
capitalizing retained earnings.

Question: 1927

Morgan Inc. was organized on January 2, 1997 with the following capital structure: 10% cumulative
preferred stock, par value $100 and liquidation value $105; authorized, issued and outstanding
1,000shares, $100,000 Common stock, par value $25; authorized 100,000 shares; Issued and
outstanding 10,000 shares, $250,000 Morgan had net income of $450,000 for its first year, but no
dividends were declared. How much was Morgan's book value per preferred share at December 31,
1997?

Page | 705
A. $100
B. $145
C. $110
D. $115
E. $105

Answer: D

Explanation:
The book value per share of cumulative preferred stock is its liquidation value plus any dividends in
arrears. Thus, the book value per preferred share is the $105 liquidation value plus $10 ($100 X 10%)
of dividends in arrears, or $115.

Question: 1928

Which of the following types of receivables is generally the most liquid?

A. Trade receivables
B. Receivables due from affiliated companies
C. None of these answers
D. Notes receivable
E. Receivables due from corporate officersThat answer is correct!

Answer: A

Explanation:
Trade receivables are generally the most liquid type of receivable because they are generated from
the sale of the firm's goods or services. Other forms of receivables are less liquid because the terms
of repayment may vary such as the case with those due from corporate officers and/or affiliated
companies. Of particular interest is notes receivable. The analyst should take care to understand the
terms of these arrangements.

Question: 1929

Typical features attached to preferred stock include all of the following except:

A. Demand redemption rights.


B. Dividend distribution preferences.
C. Liquidation priorities.
D. Non-voting rights.That answer is correct!

Answer: A

Explanation:
Preferred shareholders hold a security that looks very similar to a bond, but they are very rarely
allowed to demand redemption of their shares. On the other hand, many bonds are callable.

Question: 1930

In a period of rising prices, the inventory method that gives the lowest possible value for ending
inventory is:

A. gross profit
B. LIFO

Page | 706
C. FIFO
D. weighted average

Answer: B

Explanation:
The ending inventory under LIFO is priced at the earliest, and thus the lowest prices (in a period of
rising prices) than any of the other methods.

Question: 1931

The cash spent on replacing operating capacity used up in the normal course of business is classified
as:

A. investing cash flow.


B. operating cash flow.
C. either operating cash flow and investing cash flow.
D. financing cash flow.That answer is correct!

Answer: A

Explanation:
Operating cash flow does not include cash outlays for replacement of depreciation of capacity. The
cash used for this purpose is classified as an investment cash flow.

Question: 1932

The current economic environment is one of steady inflation. A firm uses historical cost accounting.
Which of the following is/are true?

I. The firm's book value is overstated.


II. The firm's depreciation expense is understated.
III. The firm's tax expense is overstated.
IV. The firm's net income is understated.

A. II & III
B. I & IV
C. I & III
D. I, II & IVThat answer is correct!

Answer: A

Explanation:
Since the assets are carried at historical values which are lower than the current asset values due to
inflation, the depreciation expense is understated. Therefore, income is overstated, leading to an
overstatement of taxes. Historical cost accounting in inflationary environment implies that the asset
values are understated.

Question: 1933

Which of the following is/are true?

I. A simple capital structure is one that does not include any common stock equivalents or dilutive
contingent securities.

Page | 707
II. A firm with common stock, preferred equity and debt can be considered to have a simple capital
structure.
III. Any security that is dilutive must be included in the calculation of Diluted EPS.

A. I & II
B. II & III
C. I, II & III
D. I only

Answer: C

Explanation:
A simple capital structure consists of only common stock and non-convertible senior securities while
a complex structure is one which contains potentially dilutive securities. Since there is no contingent
conversion associated with straight, preferred equity, it is considered to be a part of a simple capital
structure. Finally, only dilutive securities i.e. securities that do not increase earnings or decrease
losses are included in the calculation of DEPS. Anti-dilutive securities are ignored in the calculations.

Question: 1934

Which of the following would affect both net income and cash flow from operations?

A. credit sale to a customer


B. sale of marketable security for a gain
C. interest payment on capitalized interest
D. none of these answers

Answer: D

Explanation:
The sale of a marketable security falls under an investing activity. Interest payments fall under
financing activities. Credit sale to a customer doesn't apply.

Question: 1935

A firm has 400 preferred stocks outstanding. It started the year with 900 common stocks and over the
year, had the following transactions:

Feb 15Issued 300 common shares.


May 1Redeemed 100 preferred shares.
Aug 15Repurchased 150 common shares.
Nov. 30Issued 400 common shares.

In the calculation of EPS, the number in the denominator would be closest to:

A. 1440
B. 1140
C. 1220
D. 1540

Answer: B

Explanation:
The number of preferred stocks is not relevant in the calculation of the denominator of simple EPS. It

Page | 708
had 900-150 = 750 shares for 12 months, 150 shares for 7.5 months (repurchased in Aug), 300 shares
for 10.5 months (issued in Feb) and 400 shares for 1 month (issued in Nov). Hence, weighted # of
shares = 750 + (7.5/12*150) + (10.5/12*300) + (1/12*400) = 1,139.6

Question: 1936

What should an analyst look for when evaluating a firm's cash and cash equivalent position that
would cause the firm's cash position to be less liquid?

A. The amount of stock dividends required during the coming year.


B. None of these answers.
C. All of these answers.
D. Potential limitations on the use of cash such as restricted balances.
E. The amount of deposit collection float.

Answer: D

Explanation:
Companies may have restrictions place upon the disposition of the cash position. For example, a firm
may maintain cash equivalents that are earmarked for plant expansion, sinking fund payments, or
legally restricted compensation balance requirements. A careful review of the firm's operations must
be undertaken to uncover any cash or cash equivalents that are restricted from use in the general
operation of the business.

Question: 1937

The correction of an error in the financial statements of a prior-period should be reported, net of
applicable income taxes, in the current

A. none of these answers.


B. income statement after income from continuing operations and after extraordinary items.
C. income statement after income from continuing operations and before extraordinary items.
D. retained earnings statement after net income but before dividends.
E. retained earnings statement as an adjustment of the opening balance.

Answer: E

Explanation:
Accounting rules require that prior-period adjustments due to a correction of an error cannot be
reported on the current year income statement because they do not affect this year's operations.
They are shown as an adjustment to the opening balance of retained earnings.

Question: 1938

The deferred income tax account

A. none of these answers


B. is always reported as a long-term liability since the tax is not due until the next fiscal year
C. is reported as an other asset even though it has a credit balance
D. is where the difference between income tax expense and income tax payable is reconciled

Answer: D

Explanation:

Page | 709
The difference between income tax expense (based on accounting income) and the actual income
taxes payable (based on taxable income) is reconciled in an account called deferred income taxes.

Question: 1939

The following data are available for a firm for a given year:

Net Sales21,896
Sales & marketing expenses4,346
Administrative expenses2,143
COGS10,084
Depreciation967
Interest expense573
Tax rate35%
Dividends paid3,445
Preferred Dividends897
Average total equity37,432
Average common equity26,782
Average total liabilities18,583

In the above example, the firm's return on total capital equals ________.

A. 11.3%
B. 3.9%
C. 5.4%
D. 6.2%

Answer: C

Explanation:
Return on total capital = (Net income + interest expense)/average total capital. The average total
capital includes debt, common equity and preferred stock. Note that since Total assets = Total
liabilities + Total Equity, the denominator is also equal to total assets. In the above example, Net
Income = Earnings after depreciation, interest expense and taxes = (21,896 - 4,346 - 2,143 - 10,084 -
967 - 573)*(1 - 0.35) = 2,459. Therefore, Return on total capital = (2,459 + 573)/(37,432 + 18,583) =
5.41%.

Question: 1940

The construction of cash flow statement using the indirect method requires which of the following?

I. Income Statement.
II. Balance Sheet.
III. Statement of Retained Earnings.
IV. Statement of Shareholders' Equity.

A. I & II
B. I, II & III
C. I, II & IV
D. I & IIIThat answer is correct!

Answer: A

Explanation:

Page | 710
In the indirect method of construction of the cash flow statement, you start with the net income and
start adding back non-cash expenses, adjusting for tax effects. In addition, one needs to look at
changes in assets and liabilities accounts to back out investing and financing cash flows. Thus, you
need both income statement and the balance sheet for the indirect method of cash flow
construction.

Question: 1941

Which of the following is a long-term liability?

A. Cost of goods sold


B. Bonds
C. Commercial paper
D. Accounts payable
E. None of these answers

Answer: B

Explanation:
Only bonds would be a long-term liability. Long-term liabilities are repaid over a time frame of
greater than one year. Bonds are repaid over many years, whereas accounts payable and commercial
paper are current liabilities (repaid over less than one year); cost of goods sold is an expense not a
liability.

Question: 1942

According to FASB, initial franchise fees should be recognized as income when

A. the franchisee shows the ability to pay the fee.


B. the franchisor has substantially performed or satisfied all material services and conditions.
C. the franchisor has collected the majority of fee in cash.
D. the franchisor bills the franchisee.

Answer: B

Explanation:
Accounting and reporting standards for franchisors require that revenue be recognized when the
franchisor has substantially performed or satisfied all material services and conditions.

Question: 1943

The retail industry has __________ asset turnover ratio compared to that of a capital intensive
industry like the auto industry.

A. a higher
B. about the same
C. a higher and lower are both possible
D. a lowerThat answer is correct!

Answer: A

Explanation:
Total asset turnover = net sales/average total assets Since the auto industry is highly capital intensive
compared to the retail industry, it require far more capital assets to generate the same revenues as

Page | 711
the retail industry.

Question: 1944

When financial statements are presented that are not in conformity with generally accepted
accounting principles, an auditor may express a Qualified Opinion Disclaimer of an Opinion

Qualified OpinionDisclaimer of an Opinion


I.YesNo
II.YesYes
III.NoYes
IV.NoNo

A. II
B. III
C. I
D. IV

Answer: C

Explanation:
Departures from GAAP may result in either a qualified or an adverse opinion. The auditor must
exercise judgment as to the materiality of the departure. If the departure from GAAP is not
sufficiently material to require an adverse opinion, the auditor should express a qualified opinion.

Question: 1945

Companies A and B, similar in all respects, recently bought identical securities. However, using the
"Management intent" rule, A has classified the securities as "trading" securities while B has
categorized them as "available-for-sale" securities. Which of the following statements is/are true as a
result of this difference?

I. A and B will show same assets on their balance sheets.


II. A will have a higher income volatility than B.
III. A will have a higher cash flow volatility than B.

A. II & III
B. III only
C. I & III
D. I, II & III

Answer: D

Explanation:
Both "trading" and "available-for-sale" securities are reported at their fair market value on the
balance sheet. Hence, the reported assets for A and B are not affected by the different classifications.
However, changes in the market value of trading securities are considered part of the Income
statement while changes in the market value of available-for-sale securities are charged directly to
the retained earnings account. This will cause a higher volatility in the income of A. Further, this
higher volatility in income leads to a higher volatility in tax payments, thus causing a higher volatility
in the cash flows.

Question: 1946

Page | 712
Which would not be categorized as an unusual or infrequent item?

A. provisions for environmental remediation


B. gains or losses from disposal of a portion of a business segment
C. restructuring costs
D. gains or losses on qualifying early retirement of debt
E. employee separation costs

Answer: D

Explanation:
Gains or losses on qualifying early retirement of debt is classified as an extraordinary item, not an
unusual or infrequent item.

Question: 1947

Compared with firms with capital leases, firms with operating leases generally report

A. lower cash flow from operations.


B. lower or higher cash flow from operations depending upon market interest rates.
C. higher cash flow from operations.
D. identical cash flow from operations.That answer is correct!

Answer: A

Explanation:
The lease payment is an outflow in the CFO section.

Question: 1948

Which of the following is not subject to depreciation?

A. automobiles
B. machinery
C. land
D. land improvements

Answer: C

Explanation:
Land has unlimited useful life and is not consumed when it is used, and therefore not subject to
depreciation.

Question: 1949

When preparing a direct method statement of cash flows, the calculation of equity financing cash
flows requires analysis of the change in stockholders' equity, separating all of the following except

A. investments in affiliates.
B. dividends declared.
C. net income.
D. shares issued/repurchased.
E. changes in valuation accounts.That answer is correct!

Page | 713
Answer: A

Explanation:
Cash flows from investments in joint ventures and affiliates and long-term investments in securities
are components of cash flows from investing activities.

Question: 1950

The Basic EPS represents which of the following?

I. The net income divided by number of shares outstanding.


II. The amount of net income available for distribution to common stock holders, per share.
III. The net income after preferred dividends, expressed per common share.
IV. Net income after preferred dividends divided by total equity.

A. II & III
B. III & IV
C. I, II & IV
D. I, II, III & IVThat answer is correct!

Answer: A

Explanation:
For a simple capital structure, Basic EPS = (Net Income - Preferred dividends)/weighted # of common
shares = The amount of net income available for distribution to common stock holders, per share.
Net income after preferred dividends divided by total equity is not true since total equity includes
preferred equity whereas the denominator of Basic EPS consists only of common shares.

Question: 1951

Which of the following would affect the comparability of accounting information for a given company
from one accounting period to the next?

I. Change in accounting principles


II. Disposition of segment of business
III. Acquisition of company accounted for using purchase accounting
IV. Change in auditors

A. I, III and IV
B. I and III
C. I, II and III
D. I and II

Answer: C

Explanation:
A change in the representation by an accounting firm has no material effect on the accounting
information.

Question: 1952

A firm has purchased a building with a useful life of 7 years. It cost $35,000 and its salvage value is
estimated at $5,000. If the firm uses double declining method, what's the depreciation expense
recognized in Year 2?

Page | 714
A. $9,125
B. $7,143
C. $10,000
D. $6,857

Answer: B

Explanation:
In Double Declining method, depreciation = (2/n)*book value. In Year 1, depreciation = (2/7)*35,000
= 10,000. Hence, in Year 2, depreciation = (2/7)*(35,000-10,000) = $7,143.

Question: 1953

Book value of a company is equal to all of the following except

A. the amount resulting if the company were to liquidate at amounts reported on the balance sheet.
B. total assets reduced by claims against them.
C. the market value of the net assets.
D. net asset value.

Answer: C

Explanation:
The book value of a company is its value based on the balance sheet. The balance sheet is historical
and after day 1 of operations the book value of a company is almost never equal to the market value
of the company.

Question: 1954

Beginning inventory of 50 units, purchased at $5


50 units purchased at $10
35 units purchased at $9
25 units sold at $15
70 units sold at $12
Tax rate = 40%
Beginning LIFO reserve = $300

Given the above, the LIFO reserve at the end of the period is ________.

A. $225
B. $185
C. $135
D. $165

Answer: D

Explanation:
COGS = Beginning inventory - Ending inventory + Purchases.
FIFO COGS = 50*5 + 45*10 = $700. LIFO COGS = 35*9 + 50*10 + 10*5 = $865. Beginning inventory =
50*5 = $250. Purchases = 50*10 + 35*9 = $815.
Therefore, FIFO ending inventory = 250 + 815 - 700 = 365 and LIFO ending inventory = 250 + 815 - 865
= 200.
The LIFO Reserve is defined as the difference between the values of the inventory under FIFO and

Page | 715
under LIFO. Therefore, LIFO reserve = 365 - 200 = 165.

Question: 1955

Investments in available-for-sale securities should be valued on the balance sheet at ________.

A. amortized cost
B. lower of cost or market for the portfolio
C. lower of cost or market for individual securities
D. fair value
E. acquisition cost

Answer: D

Explanation:
Available-for-sale securities are investments in debt securities that are not classified as held-to-
maturity or trading securities and in equity securities with readily determinable fair values that are
not classified as trading securities. They are measured at fair value on the balance sheet.

Question: 1956

A firm has 1,000 common shares valued at 30 each outstanding. It has also issued 500 preferred
shares of face value 100 and a coupon of 5% and a debt of 50,000 with a coupon rate of 6%. The
firm's net income after taxes is 14,300. Then, the EPS reported by the firm is ________.

A. 7.87
B. 11.8
C. 14.3
D. 8.8

Answer: B

Explanation:
For a simple capital structure, Basic EPS = (Net Income - Preferred dividends)/weighted # of common
shares Therefore, Basic EPS = (14,300 - 500*100*5%)/1,000 = 11.8.

Question: 1957

Use the following financial data on Enterprise:

a. Sale of equipment$32,000
b. Loss on equipment sale$9,000
c. Dividends paid$12,500
d. Purchase of an office suite$104,000
e. Common stock repurchase$45,000
f. Dividends received from investments$8,500
g. Interest received on Treasury bonds$1,200
h. Supplier accounts paid$3,700
i. Cash collection from customers$14,200
j. Ending cash balance$98,000

The operating cash flow for 1998 was ________.

A. $32,700

Page | 716
B. $29,200
C. $20,200
D. $7,700

Answer: C

Explanation:
Items f, g, h and i are operating events.

Question: 1958

Which of the following best describes the distinction between expenses and losses?

A. Losses result from peripheral or incidental transactions, whereas expenses result from ongoing
major or central operations of the entity.
B. Losses are extraordinary charges, whereas expenses are ordinary charges.
C. Losses are reported net of related tax effect, whereas expenses are not reported net of tax.
D. Losses are material items, whereas expenses are immaterial items.
E. None of these answers.That answer is correct!

Answer: A

Explanation:
Expenses are outflows or other using up of assets or incurrence of liabilities from delivering or
producing goods, services or key operations. Losses are defined as decreases in equity (net assets)
except those that result from expenses or distributions to owners.

Question: 1959

In preparing its cash flow statement for the year ended December 31, 1998, Roman Co. collected the
following data:

Gain on sale of equipment$6,000


Proceeds from sale of equipment10,000
Purchase of A.S. Inc. bonds (par value $200,000)180,000
Amortization of bond discount2,000
Dividends declared45,000
Dividends paid38,000
Proceeds from sale of treasury stock
(carrying amount of $65,000)75,000

In its December 31, 1998 statement of cash flows, what amount should Roman report as net cash
provided by financing activities?

A. $20,000
B. $37,000
C. $75,000
D. $30,000
E. $27,000

Answer: B

Explanation:
Financing activities include the issuance and repurchase of shares, dividends paid and changes in

Page | 717
long-term liabilities. In this case, there were dividends paid of $38,000 and proceeds from sale of
treasury stock of $75,000. Therefore, the net cash provided from financing activities is $37,000
($75,000-38,000).

Question: 1960

Under a periodic inventory system, how is COGS determined?

A. the cost of ending inventory is subtracted from costs of goods available for sale
B. cost of goods sold is accumulated as sales are made
C. ending inventory is counted and subtracted from beginning inventory
D. none of these answersThat answer is correct!

Answer: A

Explanation:
Under the periodic inventory system, only the ending inventory is counted and priced. Cost of goods
sold is determined by deducting the cost of the ending inventory from the cost of goods available for
sale.

Question: 1961

The accrual basis of accounting:

A. is used only for taxes reporting purposes.


B. recognizes the development of assets and liabilities externally.
C. does not pertain to revenue recognition-only expense recognition.
D. begins with the cash basis of accounting and proceeds to make the necessary adjustments.
E. allocates many transactions that produce cash flows to time periods other than those in which the
cash flows occur.

Answer: E

Explanation:
Accrual accounting principles are the decision rules that state when to recognize the revenue and
expense consequences of cash flows and other events.

Question: 1962

The following account is affected when recording a return of inventory to the vendor under a
perpetual inventory system.

A. purchase returns and allowances


B. accounts receivable
C. cash
D. merchandise inventory

Answer: D

Explanation:
When inventory is purchased, Merchandise Inventory is debited, and credited when returned. The
periodic system records purchases in an account called Purchases and returns are booked to
Purchases Returns and Allowances.

Page | 718
Question: 1963

Which of the following would not be reported as an extraordinary item?

A. gain or loss on sale of fixed assets


B. gain or loss from passing of a new law
C. gain or loss from early retirement of debt
D. uninsured loss from a floodThat answer is correct!

Answer: A

Explanation:
An item must be both unusual and infrequent (and material in amount) to be classified as
extraordinary.

Question: 1964

Calculate the book value per share of General Industries common stock, given the following
information. Par value of common stock, $1 per share; total assets, $12,565,000; retained earnings,
$5,550,000; total liabilities, $5,012,500; number of common shares outstanding, 475,000; number of
preferred shares outstanding, 0. Market value of common stock, $96.25.

A. $26.45
B. $96.25
C. None of these answers
D. $15.90
E. $10.55

Answer: D

Explanation:
The book value of the firm's common stock is $15.90 which is calculated as follows: total assets of
$12,565,000, minus total liabilities of $5,012,500, minus, preferred stock outstanding of $0, divided
by the number of common shares outstanding of 475,000.

Question: 1965

The XYZ Company operates a catering service specializing in business luncheons for large businesses.
XYZ requires customers to place their orders two weeks in advance of the scheduled events. XYZ bills
its customers on the tenth day of the month following the date of service and requires that payment
be made within 30 days of the billing date. Conceptually, XYZ should recognize revenue from its
catering services at the date when a ________.

A. billing is mailed
B. customer's payment is received
C. customer places an order
D. luncheon is served
E. none of these answers

Answer: D

Explanation:
Revenues should be recognized when they are realized or realizable and earned. The most common
time at which these two conditions are met is when the product or service is delivered.

Page | 719
Question: 1966

An EPS amount is always shown for:

A. income before extraordinary items and the cumulative effect of accounting changes
B. income from continuous operations
C. cumulative effect of accounting changes
D. all of these answers are correct

Answer: D

Explanation:
All of the responses are true.

Question: 1967

In a period of rising prices, the inventory method that gives the lowest possible value for ending
inventory is:

A. weighted average
B. LIFO
C. FIFO
D. gross profit

Answer: B

Explanation:
The ending inventory under LIFO is priced at the earliest, and thus the lowest prices (in a period of
rising prices) than any of the other methods.

Question: 1968

Employers' income tax expense consists of the following obligations except:

A. Federal Income Tax


B. Social Security Tax
C. State Unemployment Tax
D. Medicare TaxThat answer is correct!

Answer: A

Explanation:
Federal income tax is withheld from employee paychecks; it is not an expense to the company.

Question: 1969

If a firm recognizes expenses before that dictated by accrual accounting, which of the following best
describes the effects on income, total assets and retained earnings?

IncomeTotal assetsRetained earnings

I.UnderstatedUnderstated Understated
II.UnderstatedOverstatedUnderstated

Page | 720
III.OverstatedUnderstated Overstated
IV.UnderstatedOverstatedOverstated

A. I.
B. II.
C. IV.
D. III.That answer is correct!

Answer: A

Explanation:
Total assets Retained earnings Understated Understated Overstated Understated Understated
Overstated Overstated Overstated

If expenses are recognized before they should be, then income gets understated since you subtract
more from the revenues than you should. Hence, retained earnings get understated, leading to an
understatement of total equity. Since assets equal liabilities plus equity and liabilities have not been
affected, the understatement of equity leads to an understatement of assets.

Question: 1970

Which of the following is/are TRUE under GAAP?

I. The change in the assets of a firm net of the change in liabilities represents a change in the total
equity of the firm.
II. If the assets of the firm increase by $100 and liabilities remain unchanged, the common stock
equity increases by $100.
III. A firm can increase its assets by borrowing money and keeping it as cash reserve.
IV. A firm can increase its equity by borrowing money and keeping it as cash reserve.

A. I & II
B. I, II & III
C. II only
D. I & III

Answer: D

Explanation:
Use the basic equation, Assets = Liabilities + Total Equity.
II is false since you need "Total equity" comprises of common stock as well as other forms of equity
like preferred issues. IV is false since the borrowing increases only the liabilities while the cash
increases only the assets, leaving equity unchanged.

Question: 1971

A firm has just experienced a LIFO liquidation under inflation. Which of the following could be true?

I. The firm's purchases were lower than the number of units sold.
II. The firm experienced a cash drain due to taxes.
III. The firm's income was understated.

A. II only
B. I & III
C. II & III

Page | 721
D. I, II & III

Answer: D

Explanation:
The LIFO Reserve is defined as the difference between the values of the inventory under FIFO and
under LIFO. A decrease in this reserve is referred to as LIFO liquidation. Under the usual case of rising
prices, this occurs when units sold exceed units purchased, leading to a "dipping" into the LIFO
layers. Since some of the goods sold were purchased at lower prices in previous periods as accounted
for by LIFO, COGS gets understated compared to the case where the firm purchases the sold goods in
the current period instead of dipping into the LIFO layers. Hence, the firm's income is overstated,
leading to higher taxes paid.

Question: 1972

Firms A and B have simple capital structure with the same number of common stock outstanding. A
finances its operations relying on debt financing while B prefers issuing preferred equity. If both the
firms had the same net income last year, Firm A will have a:

A. higher EPS than B.


B. lower EPS than B.
C. all of these answers can happen.
D. the same EPS as B.That answer is correct!

Answer: A

Explanation:
For a simple capital structure, EPS = (Net Income - Preferred stock dividends)/weighted # of common
shares Since both the firms have the same net income and common shares outstanding, Firm B will
show a lower EPS due to the payment of preferred stock dividends.

Question: 1973

The peaks and valleys of the business cycle tend to be magnified using which inventory method?

A. LIFO
B. FIFO
C. gross profit method
D. weighted average

Answer: B

Explanation:
Businesses tend to increase selling prices as costs rise, regardless of the fact that inventories may
have been purchased before the price rise. The reverse effect occurs in periods of price decreases.

Question: 1974

When merchandise inventory is purchased under a periodic system, which account is debited?

A. Accounts Payable
B. Cash
C. Merchandise Inventory
D. Purchases

Page | 722
Answer: D

Explanation:
The periodic inventory system records purchases of merchandise inventory in an income statement
account called Purchases.

Question: 1975

Patterson Company has the following information of one of its vehicles purchased on January 1,
1992:

Vehicle cost$50,000
Useful life, years, estimated5
Useful life, miles, estimated100,000
Salvage value, estimated$10,000

Actual miles driven:

199230,000
199320,000
199415,000
199525,000
199612,000

No estimates were changed during the life of the asset. The 1996 depreciation expense using the
units-of-production method was ________.

A. $4,800
B. $4,000
C. $6,000
D. $10,000
E. $5,000

Answer: B

Explanation:
Under the units-of-production method, periodic depreciation is based on the proportion of expected
total production that occurred. For the years 1992 to 1995, the total accumulated depreciation was
$36,000 (($50,000-10,000) X (30,000 + 20,000 + 15,000 + 25,000) / 100,000(. Therefore, the
remaining depreciation for the last year, 1996, is only $4,000 ($40,000-36,000). Given that the 12,000
miles driven in 1996 exceeded the remaining estimated production of 10,000 miles, only $4,000 can
be taken as depreciation in 1996.

Question: 1976

Form 10-K is filed with the SEC to update the information a company supplied when filing a
registration statement under the Securities Exchange Act of 1934. Form 10-K is a report that is filed

A. semiannually within 30 days of the end of a company's second and fourth fiscal quarters.
B. quarterly within 45 days of the end of each quarter.
C. within 15 days of the occurrence of significant events.
D. none of these answers.
E. annually within 90 days of the end of a company's fiscal year.

Page | 723
Answer: E

Explanation:
Form 10-K is the annual report to the SEC. It must be filed within 90 days after the corporation's
yearend. It must contain audited statements.

Question: 1977

A thorough analysis of financial statements should include an examination of which of the following
sources of information?

A. All of these are correct.


B. The report of the independent external auditor.
C. Management's discussion and analysis of its financial condition and results of operation.
D. Form 10-K reports.
E. Explanatory footnotes that accompany financial statements.That answer is correct!

Answer: A

Explanation:
All of these statements are correct. Management's discussion and analysis, footnotes and Form 10-K
reports provide important details as to the company's balance sheet, income statement and overall
financial condition; auditors reports will indicate if the financial information that is presented
complies with generally accepted accounting principles.

Question: 1978

When a firm's inventory undergoes LIFO liquidation under rising prices, it experiences ________.

A. higher income and higher cash flows


B. higher income and lower cash flows
C. lower income and lower cash flows
D. lower income and higher cash flows

Answer: B

Explanation:
In LIFO liquidation under rising prices, the quantity of goods purchased during a period is less than
the quantity sold. This leads to a sale of goods purchased in prior periods at lower prices (since prices
are rising). This understates the cost-of-goods-sold (COGS), thus overstating the income and causing
higher taxes than in the absence of the LIFO liquidation. The higher taxes cause a higher cash
outflow. (This is an extremely important topic that you should be completely familiar with)

Question: 1979

Which of the following is considered a characteristic of equity?

A. None of these answers.


B. All forms of equity must ultimately be repaid by a business.
C. Compensation to the holders of equity takes precedence over that required to the holders of
liabilities.
D. Equity represents the residual ownership of a business.
E. Equity represents a claim against a business that is senior to liabilities.

Page | 724
Answer: D

Explanation:
Equity refers to the claims of the owners of a business. Equity investors expose themselves to greater
risk than the holders of liability claims because owner's claims are junior to liability instruments.
Equity claims against income and assets are last in line; thus, they are residual.

Question: 1980

The following information was obtained from the financial statements of Firm A:

Current assets = $4,500


Current liabilities = $3,000

Long-range assets = $150,000


Long-term liabilities = $42,000
Contributed capital = $31,000

Then, the retained earnings of the firm equal ________.

A. $39,000
B. $31,000
C. $78,500
D. $123,500

Answer: C

Explanation:
Total assets = Total liabilities + Total Equity
Total Equity = Retained earnings + contributed capital.
Total assets = $4,500+$150,000 = $154,500.
Total liabilities = $42,000+$3,000 = $45,000, hence,
Total equity = $154,500-$45,000 = $109,500
Retained earnings = $109,500-$31,000 = $78,500

Question: 1981

The following financial data on CashCow, Inc. have been taken from its financial statements for 1996:

a. Dividends paid$25,000
b. Sale of land$64,000
c. Inventory purchases$29,000
d. Purchase of a warehouse$208,000
e. Bonds issued$90,000
f. Dividends received from investments$17,000
g. Interest paid on bonds$2,400
h. Salaries paid$107,400
i. Cash collection from customers$28,400
j. Loss on land sale$18,000
k. Beginning cash balance$312,000

In the above question, the investing cash flow is ________.

Page | 725
A. -$208,000
B. -$126,000
C. -$144,000
D. -$162,000

Answer: C

Explanation:
Items b and d are the investing cash flows. Note that item j is a non-cash event.

Question: 1982

Intraperiod income tax allocation arises because

A. certain revenues and expenses appear in the financial statements but are excluded from taxable
income.
B. items included in the determination of taxable income may be presented in different sections of
the financial statements.
C. income taxes must be allocated between current and future periods.
D. certain revenues and expenses appear in the financial statements either before or after they are
included in taxable income.
E. none of these answers.

Answer: B

Explanation:
To provide a fair presentation of the results of operations, accounting rules require that income tax
expense for the period be allocated among income from continuing operations, discontinued
operations, extraordinary items, and items charged or credited to shareholders' equity.

Question: 1983

Which of the following would not be considered an intangible asset?

A. Customer lists
B. All of these answers represent intangible assets
C. Mineral rights
D. Patents, copyrights, and trademarks
E. Goodwill

Answer: C

Explanation:
Intangibles represent rights, privileges, and benefits of possession. Two common characteristics are
high uncertainty regarding future benefits and lack of physical existence. All of the above are
examples thatmeet this definition except mineral rights which are not an intangible but a natural
resource that has physical properties.

Question: 1984

Which of the following is NOT a measure of a firm's liquidity?

A. interest coverage
B. inventory turnover

Page | 726
C. quick ratio
D. receivables turnoverThat answer is correct!

Answer: A

Explanation:
Interest coverage ratio is a measure of the financial risk.

Question: 1985

When an auditor finds that the audit is insufficient in scope to render an opinion, he will issue
________.

A. an unqualified opinion
B. an adverse opinion
C. a qualified opinion
D. a disclaimer of opinion

Answer: D

Explanation:
A disclaimer of opinion is a statement of inability to express an opinion.

Question: 1986

When a plant asset is sold for more than its book value

A. book value of the asset minus gain on disposal equals cash received
B. cash received plus accumulated depreciation plus gain on disposal equals the original cost
C. original cost minus accumulated depreciation equals cash received plus gain on disposal
D. cash received plus accumulated depreciation minus gain on disposal equals the original cost

Answer: D

Explanation:
Gain or loss on disposal of a fixed asset is calculated by subtracting the book value (Original cost -
Accumulated Depreciation) from the cash received.

Question: 1987

In practice, companies who use the direct method in presenting cash flows from operations:

A. must also disclose cash flows from operations using the indirect method.
B. must classify interest paid as an investing activity.
C. must be privately held or closely held corporations.
D. are not required to report cash flows not incurred during the ordinary course of business.That
answer is correct!

Answer: A

Explanation:
The advantage of the indirect method is it reconciles differences between net income and cash flows
from operating activities.

Page | 727
Question: 1988

The amount by which a plant asset depreciated is classified as

A. a liability
B. an asset
C. an expense
D. revenue

Answer: C

Explanation:
Depreciation is the periodic allocation of the cost of a tangible, long-term asset over its estimated
useful life, and therefore a period expense.

Question: 1989

Under an inflationary environment with a constant inventory quantity, the LIFO Reserve tends to
________.

A. increase over time.


B. decrease over time.
C. stays constant over time.
D. fluctuate with COGS.That answer is correct!

Answer: A

Explanation:
As prices increase and LIFO layers are maintained, the inventory value at FIFO keeps increasing while
that at LIFO remains constant. Hence, the reserve keeps increasing.

Question: 1990

At December 31, 1996, Eaton Corp. reported $1,750,000 of appropriated retained earnings for the
construction of a new office building, which was completed in 1997 at a cost of $1,500,000. In 1997,
Eaton appropriated $1,200,000 of retained earnings for the construction of a new plant. Also,
$2,000,000 of cash was restricted for the retirement of bonds due in 1998. In its 1997 balance sheet,
what amount should Eaton report as appropriated retained earnings?

A. $2,950,000
B. $3,200,000
C. $1,450,000
D. $1,750,000
E. $1,200,000

Answer: E

Explanation:
Appropriating retained earnings is a formal way of marking a portion of retained earnings for other
uses. This is done by reducing retained earnings and transferring the money to appropriated retained
earnings. When the appropriation is no longer necessary, the money is moved back to retained
earnings.

Question: 1991

Page | 728
Which of the following would not be a limitation of the balance sheet?

A. Long-lived assets may not reflect their market value because they are stated at historical cost.
B. Significant assets and liabilities may be omitted because GAAP does not require their inclusion.
C. GAAP permits companies to delay recognition of value changes, such as employee benefit plans.
D. Some assets and liabilities are carried at historical cost bearing little relationship to their real
market value.
E. The firm's market value would not be able to be estimated due to historical costing.

Answer: E

Explanation:
The balance sheet does not report the market value of a firm's assets, liabilities, or equity, although
the information provided can be useful when estimating the market value of the firm or its
securities.

Question: 1992

Which combination leads to the lowest income tax paid in earlier years of a firm?

Inventory valuationDepreciation

I.FIFOStraight-line
II.LIFOStraight-line
III.FIFODouble Declining
IV.LIFODouble Declining

A. IV.
B. II.
C. I.
D. III.That answer is correct!

Answer: A

Explanation:
In the earlier years, this combination results in the lowest income reported and hence, the lowest
taxes.

Question: 1993

A firm has a constant pre-depreciation income using assets that last 5 years. The straight-line method
results in a rate of return that ________ over the life of the assets. The accelerated methods result in
a rate of return that ________ over the life of the assets.

A. increases, increases
B. decreases, increases
C. decreases, decreases
D. increases, decreasesThat answer is correct!

Answer: A

Explanation:
The straight line and all accelerated depreciation methods giving an increasing rate of return on

Page | 729
assets over the asset life (the pre-depreciation income is held constant to isolate the effect). This is a
mathematical fact which you can easily prove.

Question: 1994

An asset with a 10-year life has an acquisition cost of 5,000. The firm, using double declining method
of depreciation has an accumulated depreciation of 2,440 at the end of year 3 (verify this!). The
depreciation expense in year 4 is equal to ________.

A. 467
B. 630
C. 512
D. 731

Answer: C

Explanation:
The book value at the beginning of year 4 = 5,000-2,440 = 2,560. Then, depreciation expense in year
4 = 2/10*book value = 512.

Question: 1995

The weighted average method is based on the assumption that the cost of merchandise sold should
be calculated using the :

A. lower of cost or market (LCM)


B. FIFO
C. LIFO
D. weighted average cost

Answer: D

Explanation:
Under the weighted average method, inventory is priced at the average cost of the goods available
for sale (Beginning inventory plus purchases during the period). The cost for the ending inventory
under this method is influenced by all the prices paid during the period.

Question: 1996

If a company fails to record a material amount of depreciation in a previous year, this is considered

A. an accounting error.
B. a change in estimate.
C. an unusual item.
D. a change in accounting principle.That answer is correct!

Answer: A

Explanation:
This is neither unusual or a change, but simply an error.

Question: 1997

Which of the following is/are current liabilities?

Page | 730
I. Accounts receivable
II. Cash advances received
III. Advance magazine subscription payments received
IV. Revenues from credit card fees

A. II, III & IV


B. II & III
C. III & IV
D. I, II, III & IVThat answer is correct!

Answer: A

Explanation:
Accounts receivable represent current assets.

Question: 1998

Which of the following marketable securities would provide the greatest liquidity and stability?

A. None of these answers


B. All of these answers
C. AA corporate bonds that mature in 5 years
D. Common stock of a publicly held firm
E. U.S. treasury bills

Answer: E

Explanation:
U.S. treasury bills would be the most liquid because the maximum maturity is 182 days (although
most treasury bills purchased by firm's have a shorter maturity). In addition, the value of treasury
bills would fluctuate less than the value of the common stock of another firm or long-term corporate
bonds.

Question: 1999

In a given period, the firm's beginning gross investment is 6,000 and ending gross investment is
10,000. The accumulated depreciation at the beginning was 500 and the ending balance in this
account was 1,500. The firm uses straight-line depreciation. Then, the average depreciable life of the
firm's assets is ________.

A. 6.67 years
B. 6 years
C. 10 years
D. 12 years

Answer: C

Explanation:
Note that: Average depreciable life = Ending Gross Investment/Depreciation expense. Depreciation
expense = 1,500-500 = 1,000. Hence, ADL = 10,000/1,000 = 10 years.

Question: 2000

Page | 731
Firm A uses Double Declining and Firm B uses Sum-of-digits method of depreciation. In the first year,
which of the following is/are TRUE?

I. A shows lower assets than B


II. A shows higher retained earnings than B
III. A shows a higher debt-to-equity ratio than B
IV. A shows a lower debt-to-asset ratio

A. II & IV
B. I & III
C. I, III & IV
D. II, III & IV

Answer: B

Explanation:
Double Declining results in a higher depreciation in the first year. Therefore, Firm A shows lower
assets, lower income and hence lower equity in the first year. Thus, its debt-to-equity and debt-to-
asset ratios are higher than B's.

Question: 2001

If the net cash inflow = $12,000, Investing cash flow = $(4,000) and operating cash flow = $7,000, the
financing cash flow equals ________.

A. $9,000
B. $23,000
C. $15,000
D. $12,000That answer is correct!

Answer: A

Explanation:
financing cash flow = 12,000 - (-4,000) - 7,000 = $9,000

Question: 2002

An end-of-period adjustment for depreciation of fixed assets involves an entry to an expense and

A. the increase of a contra account


B. the decrease of a contra account
C. the reduction of a liability
D. the increase of a liabilityThat answer is correct!

Answer: A

Explanation:
An adjustment for depreciation expense matches expenses with revenues for the period. An expense
is incurred and the asset is reduced by increasing the contra account.

Question: 2003

Morgan Inc. was organized on January 2, 1997 with the following capital structure:

Page | 732
10% cumulative preferred stock, par value $100 and liquidation value $105; authorized, issued and
outstanding 1,000 shares, $100,000

Common stock, par value $25; authorized 100,000 shares;


Issued and outstanding 10,000 shares, $250,000

Morgan had net income of $450,000 for its first year, but no dividends were declared. How much was
Morgan's book value per common share at December 31, 1997?

A. $68.50
B. $70
C. $25
D. $45.00
E. $69.50That answer is correct!

Answer: A

Explanation:
The book value per share of cumulative preferred stock is its liquidation value plus any dividends in
arrears. Thus, the book value per preferred share is the $105 liquidation value plus $10 ($100 X 10%)
of dividends in arrears, or $115. The total book value for preferred shares is $115,000 ($115 X 1000).
The total book value of the company is $800,000 ($100,000 par value of preferred stock + $250,000
par value of common stock + retained earnings equal to $450,000 of net income). Hence, $685,000
($800,000-115,000) is the book value of the common stock, and book value per common share is
$68.50 ($685,000 / 10,000 shares).

Question: 2004

When purchased, plant assets are recorded at ________.

A. future value
B. market value
C. lower of cost or market
D. cost

Answer: D

Explanation:
Fixed assets are recorded at cost, which includes all normal and reasonable expenditures necessary
to get the asset in place and ready to use.

Question: 2005

From tax perspective, which of the following depreciation methods is usually the best?

A. Double declining
B. Modified accelerated
C. Straight line
D. Sum-of-digitsThat answer is correct!

Answer: A

Explanation:
Since Double Declining results in the biggest depreciation expense of the four in the initial years, it

Page | 733
allows the biggest deferral of tax payments. These deferrals can sometimes be for an indefinite
period, thus avoiding them almost altogether.

Question: 2006

Gamma-Theta is a financial software firm with gross sales of 176,000 last year. Its cost of goods sold
equaled 25,685. Gamma-Theta's corporate tax rate is 35%. If its net income after taxes was 70,279,
its operating expenses were ________.

A. 31,889
B. 42,193
C. 29,495
D. 51,722

Answer: B

Explanation:
Gamma-Theta's pre-tax income equals 70,279/0.65 = 108,122. Now, pre-tax income = gross sales -
cost of goods sold - operating expenses. Therefore, 108,122 = 176,000 - 25,685 - operating expenses.
This gives operating expenses = 42,193.

Question: 2007

Restricted cash balances are ________.

A. non-current assets
B. not considered assets
C. none of these answers
D. regarded as part of long-term equityThat answer is correct!

Answer: A

Explanation:
At times, firms earmark cash deposits and balances for meeting longer term obligations like sinking
funds or plant expansions. Such restricted cash balances are reported as non-current assets even
though they are in a liquid form, since they are not available to satisfy current liabilities.

Question: 2008

Which of the following is/are true?

I. Straight preferred equity holders to do not have a right to demand redemption.


II. Dividend payments on preferred equity are cumulative.
III. Dividends on common stock cannot be paid unless preferred dividends are paid.
IV. Preferred dividends which are in arrears are a non-recorded liability.

A. I & IV
B. I, II, III & IV
C. I, II & III
D. II, III & IV

Answer: B

Explanation:

Page | 734
Preferred equity is usually not redeemable, implying that preferred equity holders do not have a
right to demand from the company a retirement of their equity through redemption payment.
Preferred dividends that are not paid when due remain a liability, though they are not recorded as
such. This leads to an understatement of liabilities since someday, these dividends must definitely be
paid. In particular, the dividends in arrears are cumulative and must be paid in full before any
ordinary dividends can be paid to common stockholders.

Question: 2009

The accumulated amount by which a plant asset has been depreciated over its useful life is classified
as

A. a liability
B. an asset
C. an expense
D. revenue

Answer: B

Explanation:
Accumulated Depreciation reports the total amount of depreciation expense recognized in all prior
periods since the asset(s) were put into service.

Question: 2010

Under accrual accounting, which of the following is/are Incorrect?

I. Revenues are recognized when cash is received.


II. The reported income is a good indicator of the firm's current performance.
III. Expenses do not always involve cash flows.
IV. Revenues and the related costs are matched in the same period.

A. I only
B. II & III
C. III only
D. All of these answers are correctThat answer is correct!

Answer: A

Explanation:
Under accrual accounting, revenues are recognized when the earnings process behind the revenues
is complete and expenses are recognized in the period the goods and services are used for
generating income. Therefore, revenues and expenses do not always involve actual cash flows,
though revenues and costs that are related do get matched in the same period whenever such an
identification is possible.

Question: 2011

The Income Summary account is

A. a permanent account
B. an asset
C. a temporary account
D. a liability

Page | 735
Answer: C

Explanation:
The Income Summary account is a temporary account created especially for the closing process and
is used only for flowing adjustments and closing entries through.

Question: 2012

Use the following financial data on Enterprise:

a. Sale of equipment$32,000
b. Loss on equipment sale$9,000
c. Dividends paid$12,500
d. Purchase of an office suite$104,000
e. Common stock repurchase$45,000
f. Dividends received from investments$8,500
g. Interest received on Treasury bonds$1,200
h. Supplier accounts paid$3,700
i. Cash collection from customers$14,200
j. Ending cash balance$98,000

In the above question, the beginning cash balance of the firm was ________.

A. $216,300
B. $207,300
C. $198,300
D. $109,300

Answer: B

Explanation:
Note that the loss on equipment is a non-cash event. Using the direct method, beginning cash
balance + net cash inflow = ending cash balance Hence, beginning balance = 98,000 - (32,000 -12,500
-104,000 - 45,000 + 8,500 +1,200 - 3,700 + 14,200) = $207,300.

Question: 2013

The following information is from the financial statements of Complex Capitalists for 1997 and 1998:

Dec. 31, 1997 - 1 million common shares outstanding, capital structure all-equity
March 31, 1998 - issued 200,000 common shares.
May 31, 1998 - issued 800,000 warrants exercisable at a strike of 35.
The average price during 1998 was 34 and the maximum price was 39.
Given the above example, the Diluted EPS will use how many shares?

A. 1.2 million
B. 2.0 million
C. 1.617 million
D. 1.15 million

Answer: D

Explanation:

Page | 736
Options and warrants enter into the Diluted EPS calculations only if the average stock price during
the period exceeds the exercise price. In this case, since the average price is less than the strike price,
the warrants are anti-dilutive and ignored in the calculation of Diluted EPS. Hence, weighted number
of shares used = 1 million + 9/12*200,000 = 1.15 million.

Question: 2014

Which of the following conditions would allow a firm to classify a short-term liability as a long-term
debt?

I. The firm has issued a long-term note with the stated purpose of extinguishing the short-term debt
when it matures. The note is cancelable if there are violations of certain operating provisions.
II. The firm has entered into a binding agreement with a bank to refinance the short-term debt with a
long-term liability.
III. The firm has announced that it will continue to refinance the debt with available credit for the
next 2 years.

A. III only
B. I & III
C. II only
D. II & III

Answer: C

Explanation:
If these agreements have any provisions for cancellation which are either ambiguous or which have a
good probability of being violated, then the short-term debt cannot be classified as a long-term debt.
That's why (I) is not a valid choice. (III) is not acceptable since there is no demonstration of credible
intent or ability to be able to refinance the debt.

Question: 2015

At the end of a fiscal period, any revenue that has been earned but not received should be credited
to an appropriate

A. revenue account
B. asset account
C. liability account
D. expense accountThat answer is correct!

Answer: A

Explanation:
Revenue should be recognized in the period in which it is earned and is credited to the appropriate
revenue account.

Question: 2016

Current liabilities are defined as those that come due:

A. within a year.
B. the shorter of "within an operating cycle" or "within a year."
C. within an operating cycle.
D. the longer of "within an operating cycle" or "within a year."

Page | 737
Answer: D

Explanation:
The classification of liabilities into short and long-term is somewhat subjective but for balance-sheet
purposes, all obligations that are coming due within a year or an operating cycle, whichever is longer,
are considered to be current

Question: 2017

At the end of an accounting period, each expense that has been incurred but not yet paid should be
recorded as ________.

A. a closing entry
B. an opening entry
C. an adjusting entry
D. a reversing entry

Answer: C

Explanation:
Expenses should be recognized in the period in which they are incurred. In order to be consistent
with the matching principle, an adjusting entry must be made to record the expense, regardless of
whether it has been paid.

Question: 2018

Deferred taxes would be classified as

A. a long-term borrowing.
B. a current liability.
C. a current asset.
D. a prepaid expense.
E. stockholders' equity.

Answer: C

Explanation:
Deferred income taxes represent deferred tax assets.

Question: 2019

A firm issued 2 million warrants with an exercise price of 102 on June 15, 1996. The maximum price
of the stock in 1996 was 129, the average price was 110. The stock closed out the year at 99. How
many additional shares must be used in the computation of Diluted EPS as a result of these
warrants?

A. 111,782
B. 145,454
C. 78,788
D. zero

Answer: C

Page | 738
Explanation:
The conversion assumptions for options and warrants are as follows:

1. They are assumed to be exercised at the beginning of the period or at the time of issuance,
whichever is later.
2. They enter into the Diluted EPS calculations only if the average stock price during the period
exceeds the exercise price. With the Treasury stock method, the total shares repurchased using
exercise proceeds = 2*102/110 = 1,854,546. Thus, additional shares issued = 2 million - 1,854,546 =
145,454. These are outstanding for 6.5 months. Therefore, additional weighted shares =
145,454*6.5/12 = 78,788.

Question: 2020

Which of the following correctly defines an element directly related to measuring the performance
and status of a business entity?

A. Gains are increases in equity from transactions and other events and circumstances that result
from revenues or investments by owners.
B. Equity is a residual interest.
C. Investments by owners are limited to receipts of assets and satisfaction or conversion of liabilities.
D. Revenues are inflows from peripheral or incidental transactions as well as the entity's ongoing
major operations.
E. None of these answers.

Answer: B

Explanation:
Equity in a business is considered a residual interest and represents the ownership (total assets
minus total liabilities) in the firm.

Question: 2021

A piece of equipment costs $200,000, has a useful life of 5 years and an estimated salvage value of
$50,000. How much depreciation expense should the company recognize in year 4 if it is using the
sum-of-the- years' digits method of depreciation?

A. $30,000
B. $80,000
C. $26,666.67
D. $20,000

Answer: D

Explanation:
Using the sum-of-the-years' digits the sum of the years is 15. In year 4 you would take 2/15 of the
cost less the salvage value. 2/15 of $150,000 is equal to $20,000.

Question: 2022

In the successful effort method, oil firms:

I. are required to expense all oil-drilling costs resulting in dry holes.


II. can capitalize drilling costs which result in productive oil wells.
III. are required to capitalize all oil-drilling costs.

Page | 739
IV. are required to expense all oil-drilling costs.

A. I only
B. I & II
C. III & IV
D. II only

Answer: B

Explanation:
In extractive industries, firms are allowed to use either the full-cost method, in which all search and
development costs can be capitalized, or the successful-efforts method, where all such costs are
expensed unless they result in revenue-generating assets, in which case, they are capitalized. If an oil
drilling firm uses the successful-efforts method, it expenses costs incurred on dry holes and
capitalizes those resulting in oil-producing wells.

Question: 2023

MacDonald Inc. reported net income of $300,000 for 1996. Changes occurred in several balance
sheet accounts as follows:

Equipment$25,000 increase
Accumulated depreciation40,000 increase
Note payable30,000 increase

Additional information:

During 1996, MacDonald sold equipment costing $25,000, with accumulated depreciation of
$12,000, for a gain of $5,000.

In December 1996, MacDonald purchased equipment costing $50,000 with $20,000 cash and a 12%
note payable of $30,000.

Depreciation expense for the year was $52,000.

In MacDonald's 1996 statement of cash flows, net cash used in investing activities is ________.

A. $12,000
B. $18,000
C. $20,000
D. $32,000
E. $2,000

Answer: E

Explanation:
The purchase and sale of equipment are investing activities. The firm spent $20,000 cash to buy
equipment and received $18,000 cash (net book value was $13,000 plus $5,000 gain) by selling its
old equipment. Therefore, the net cash used in investing is $2,000. The issuance of a note payable is
part of the acquisition price of equipment and is classified as a noncash financing and investing
activity.

Question: 2024

Page | 740
"Horizontal analysis" refers to:

A. comparison of various quantities in a common-sized financial statement.


B. comparison of a firm's financial statements with those of a similar firm.
C. comparative analysis of financial statements as they evolve over time.
D. comparative study of a firm's dependencies on businesses in the different industries in which it is
active.

Answer: C

Explanation:
In comparative financial analysis, one of the important parts is detecting trends in the changes that
occur over time in the different financial statements. This trend analysis is quirkily referred to as
"horizontal analysis," probably because comparison implies a side-by-side analysis of the financial
items in the statements. Two popular techniques in horizontal analysis are "year-to-year change"
analysis and "Index number trend series" analysis.

Question: 2025

Which of the following is NOT a disclosure requirement for financial instruments?

A. Firms must present fair valuations of financial instruments in the financial statements or
accompanying notes.
B. Firm must present reasons for not meeting any of the disclosure requirements.
C. Firms must disclose the purposes of holding any speculative financial instruments.
D. Firms must disclose significant assumptions used to value financial instruments.

Answer: C

Explanation:
Firms must disclose the purposes of holding any speculative financial instruments.

Question: 2026

The purpose of the Management Report is to:

I. analyze favorable and unfavorable trends affecting the firm.


II. reinforce senior management's responsibilities for the company's financial and internal control
systems.
III. review the firm's financial condition in light of short-term liquidity requirements and proposed
investment activities.
IV. emphasize the roles of management, directors and auditors in the preparation of financial
statements.

A. I, II & III
B. II & III
C. II, III & IV
D. II & IV

Answer: D

Explanation:
I & III are part of Management's Discussion & Analysis (MD&A)

Page | 741
Question: 2027

The Statement of Stockholders' Equity does not report

A. any minimum pension liability.


B. any cumulative impact on prior period earnings.
C. the investment of the owners in the firm.
D. the various accounting adjustments that reflect selected market value changes in noncurrent
assets.
E. the effect of exchange rate changes on certain foreign subsidiaries.

Answer: B

Explanation:
Any cumulative impact on prior period earnings is reported net of tax after extraordinary items and
discontinued operations on the income statement.

Question: 2028

A test to determine whether a convertible security is dilutive or antidilutive is to calculate EPS

A. for the security alone and compare it to the EPS without inclusion of the security.
B. for the security alone and compare it to the EPS with inclusion of the security.
C. for the security alone.
D. without inclusion of the security.
E. with inclusion of the security.That answer is correct!

Answer: A

Explanation:
By comparing the EPS of the security alone with the EPS without inclusion of the security, if the
former is smaller than the latter, then the security is dilutive which means it causes EPS to decline.

Question: 2029

At the end of an accounting period, each expense that has been incurred but not yet paid should be
recorded as

A. a reversing entry
B. an opening entry
C. an adjusting entry
D. a closing entry

Answer: C

Explanation:
Expenses should be recognized in the period in which they are incurred. In order to be consistent
with the matching principle, an adjusting entry must be made to record the expense, regardless of
whether it has been paid.

Question: 2030

Which of the following is/are true about held-to-maturity securities?

Page | 742
I. They are mostly non-current assets.
II. They are reported at fair market value.
III. Income on these securities is included in operating income.

A. II only
B. II & III
C. I & III
D. I, II & III

Answer: C

Explanation:
Marketable securities classified as "Held-to-maturity" are financial securities purchased with the
intent (and ability) to hold until maturity. They are therefore categorized as non-current assets in all
periods except the one in which they are about to mature. They are reported on the balance sheet at
theamortized historical cost and income on the securities, including amortization of premium or
discount, is part of the operating income.

Question: 2031

Which of the following would be classified a cash inflow from investing activities?

A. Proceeds from collecting the principal amount of loans.


B. Proceeds from selling investments in the equity securities of other companies.
C. All of these answers are correct.
D. Proceeds from selling investments in the debt securities of other entities, except cash equivalents.

Answer: C

Explanation:
All responses qualify as cash inflows from investing activities.

Question: 2032

Under an inflationary, LIFO environment with stable inventories, which of the following is true
compared to FIFO?

Working CapitalTaxesCurrent Ratio

I.HigherLowerLower
II.LowerLowerLower
III.HigherLowerLower
IVLowerHigherHigher

A. II
B. I
C. IV
D. IIIThat answer is correct!

Answer: A

Explanation:
With rising prices and stable inventories, the COGS is higher under LIFO than under FIFO, thus lower
net income and lower taxes. Ending inventory is valued lower under LIFO than under FIFO and hence,

Page | 743
a lower current asset value and lower working capital and current ratio.

Question: 2033

Long-term obligations that are or will become callable by the creditor because of the debtor's
violation of a provision of the debt agreement at the balance sheet date should be classified as

A. current liabilities unless the creditor has waived the right to demand repayment for more than 1
year from the balance sheet date.
B. current liabilities unless it is reasonably possible that the violation will be corrected within the
grace period.
C. contingent liabilities until the violation is corrected.
D. none of these answers.
E. long-term liabilitiesThat answer is correct!

Answer: A

Explanation:
Current liabilities include obligations that must be liquidated within 1 year as well as obligations that
are or will be callable by the creditor within 1 year because of a violation of a debt covenant. An
exception exists, however, if the creditor has waived or subsequently lost the right to demand
repayment for more than 1 year from the balance sheet date.

Question: 2034

Why do the corporation's directors declare stock dividends?

A. to increase the market value of the company's stock


B. to provide tangible evidence of management's confidence in the company's strong performance
C. to decrease the number of shares outstanding
D. none of these answers is correct

Answer: B

Explanation:
Declaring a stock dividend will increase the number of shares outstanding and thereby keep the per
share price low enough to be an attractive investment. Stock dividends may also show the company
management's confidence in the present and future performance of the company.

Question: 2035

Which of the following is the proper method of reporting the value of financial instruments on a
firm's balance sheet?

A. All of these answers


B. Higher of cost or market
C. None of these answers
D. Cost
E. Fair market value

Answer: D

Explanation:
Company's are required by accounting regulations to record and report the value of financial

Page | 744
instruments such as derivatives at cost.

Question: 2036

A financial auditor's report is required to:

I. make sure that no fraudulent activity is occurring with the accounting systems.
II. provide reasonable assurance that there are no material errors in the statements.
III. attest to the fact that the auditor has performed adequate testing on the company's accounting
system to ensure that the financial reporting is accurate.
IV. state an opinion about the controls and checks present in the firm's reporting procedures.

A. I & IV
B. I, III & IV
C. I, II & III
D. II & III

Answer: D

Explanation:
IV is usually reported to management but not in the audit report. Also, a financial auditor, would, in
the course of her audit, check for material misstatements but is not responsible for unearthing
fraudulent activities.

Question: 2037

Accounting Standards are best described as

A. the state-of-the-art presentation of the science of accounting.


B. presentation standards mandated by the Securities and Exchange Commission.
C. the result of a political process among groups with diverse interests.
D. measuring the quality of stewardship.

Answer: C

Explanation:
Accounting standards have been through a long development process and are subject to continual
innovation, modification, and change. Currently, the Financial Accounting Standards Board (FASB),
composed of seven full-time paid members, functions as the standard-setting body of the accounting
profession.

Question: 2038

A firm using LIFO accounting has a LIFO reserve of 900, with a LIFO ending inventory of 8,100. It is
currently in the 40% tax bracket. If it switches to FIFO accounting, which of the following is true?

I. Its ending FIFO inventory equals 7,200


II. Its deferred taxes decrease by 360
III. Its equity increases by 540

A. III only
B. I & III
C. II & III
D. I, II & IIIThat answer is correct!

Page | 745
Answer: A

Explanation:
LIFO Reserve = FIFO Ending inventory value - LIFO Ending inventory value Therefore, the ending
inventory under FIFO = 8,100 + 900 = 9,000. The deferred taxes increase by 900*0.4 = 360 and the
equity increases by 900*(1-0.4) = 540.

Question: 2039

Which of the following is/are true under accrual accounting?

I. Expenses are recognized as services are used.


II. Revenues are recognized when service is performed.
III. Cash outflows determine expense recognition.

A. II & III
B. I & II
C. I & III
D. I, II & III

Answer: B

Explanation:
With accrual accounting expenses are recognized when they are incurred, not when the cash is
disbursed.

Question: 2040

In January 1996, Weber Co. purchased a mineral mine for $2,640,000 with removable ore estimated
at 1,200,000 tons. After it has extracted all the ore, Weber will be required by law to restore the land
to its original condition at an estimated cost of $180,000. Weber believes it will be able to sell the
property afterwards for $300,000. During 1996, Weber incurred $360,000 of development costs
preparing the mine for production and removed and sold 60,000 tons of ore. In its 1996 income
statement, what amount should Weber report as depletion?

A. $150,000
B. $132,000
C. $135,000
D. $159,000
E. $144,000

Answer: E

Explanation:
The depletion base is the purchase price of the land ($2,640,000), minus the value of the land after
restoration $120,000 ($300,000 -180,000), plus any costs necessary to prepare the property for the
extraction of ore ($360,000). The depletion charge is $2.40 per ton ($2,640,000 -120,000 + 360,000) /
1,200,000 tons). Therefore, the firm should report $144,000 ($2.40 X 60,000 tons) as 1996 depletion
expense.

Question: 2041

If a firm's profit margin increases by 8%, the debt-to-equity ratio increases from 35% to 55% and its

Page | 746
asset turnover falls by 20%, the effect on its ROE is ________.

A. +1.3%
B. +0.24%
C. -0.8%
D. +1.6%

Answer: C

Explanation:
ROE = net income/equity = (net income/sales)*(sales/assets)*(assets/equity) = (profit margin)*(asset
turnover)*(1+debt/equity) The original assets/equity = 1 + 0.35 = 1.35 and the changed assets/equity
= 1.55. Therefore, the change in ROE equals (1+8%)*(1-20%)*1.55/1.35 = 0.992. Thus, ROE falls by
0.8%.

Question: 2042

Below is an example of an incorrectly prepared statement of cash flows. The descriptions of activities
are correct.

Cash from operating activities$60,000


Net Income(4,000)
Depreciation(2,000)
Increase in accounts receivable(1,000)
Increase in deferred tax liability$53,000

Cash from investing activities($48,000)


Purchase of marketable securities2,500
Dividends received1,500
Dividends paid($44,000)

Cash from financing activities(500)

Increase in Short-term debt(2,500)


Increase in Long-term debt($3,000)
Increase in cash$6,000

The correct cash flows from operating activities is ________.

A. $65,500
B. $63,500
C. $53,500
D. None of these answersThat answer is correct!

Answer: A

Explanation:
60,000 + 4000(depreciation) - 2,000 (receivables) + 1,000 (deferred taxes) +2,500 (dividends
received)

Question: 2043

Which of the following would not be reported as an extraordinary item?

Page | 747
A. gain or loss on sale of fixed assets
B. gain or loss from passing of a new law
C. gain or loss from early retirement of debt
D. uninsured loss from a floodThat answer is correct!

Answer: A

Explanation:
An item must be both unusual and infrequent (and material in amount) to be classified as
extraordinary.

Question: 2044

Sparten, Inc., a plumbing contractor, received a check for $3,000 on June 30 for services to be
performed in the following fiscal month. During the July accounting period, Sparten completed all
but $500 of the job. What adjusting entry needs to be made at the end of July?

A. increase Cash and Revenue for $3,000


B. decrease Accounts Receivable and increase Revenue for $2,500
C. increase Unearned Revenue and decrease Cash for $500
D. debit Unearned Revenue and credit Revenue for $2,500

Answer: D

Explanation:
When the check was received on June 30, a liability was established for $3,000 because revenue
could not be recognized until it was earned. During July, $2,500 of revenue was earned and should be
recognized, along with a reduction (debit) to the Unearned Revenue account.

Question: 2045

Extraordinary items are placed on the income statement:

A. after net income from continuing operations and net of tax.


B. as a footnote to the statement as they are 1 time items and are not a part of the normal course of
business for a corporation.
C. within income from operations net of tax.
D. after net income from continuing operations and before taxes.That answer is correct!

Answer: A

Explanation:
Extraordinary items are always included net of tax after the net income from continuing operations.
These items are unusual items for the company and occur infrequently.

Question: 2046

What is the primary function of the Securities and Exchange Commission as it relates to a company's
financial statements?

A. None of these answers.


B. To function as the standard-setting body of the accounting profession.
C. To provide the investment public with completely independent and unbiased advice regarding the
purchase of good quality public securities.

Page | 748
D. To ensure that a public company makes full and accurate disclosure of all pertinent information
relating to a company's business.

Answer: D

Explanation:
To ensure that a public company makes full and accurate disclosure in the company's registration
statement of all pertinent information relating to a company's business, its securities, its financial
position and earnings, and the underwriting arrangements.

Question: 2047

Minority interest on the consolidated balance sheet is listed ________.

A. none of these answers


B. as a liability
C. between liabilities and shareholders' equity
D. as a component of shareholders' equity
E. as an asset

Answer: C

Explanation:
Minority interest represents the proportionate stake of minority shareholders in a company's
majority- owned subsidiary that is consolidated. Minority interest is neither a liability nor equity but
it is a financing component and, therefore, is presented between liabilities and shareholders' equity.

Question: 2048

When prices are rising, which of the following inventory valuation methods produces a higher profit?

A. None of these answers


B. FIFO
C. LIFO
D. Average cost

Answer: B

Explanation:
When prices are rising, FIFO will produce the highest profit because the first in (lowest cost)
inventory is the first out. This leaves the most recently purchased inventory on hand and since it has
the highest cost, ending inventory will have the greatest value which means that cost of goods sold
will be the lowest and profits will be the highest (ending inventory = beginning inventory + net
purchases - cost of goods sold).

Question: 2049

Calculate the book value per share of Quality, Inc.'s common stock, given the following information.
Par value of common stock, $2.5 per share; total assets, $19,100,000; retained earnings, $7,375,000;
total liabilities, $6,975,000; number of common shares outstanding, 1,250,000; number of preferred
sharesoutstanding, 15,000 at $100 par value. Market value of common stock, $24.25. Market value
of preferred stock, $106.50.

A. $8.50

Page | 749
B. $15.28
C. $3.80
D. $9.38
E. None of these answersThat answer is correct!

Answer: A

Explanation:
The book value of the firm's common stock is $8.50 which is calculated as follows: total assets of
$19,100,000, minus total liabilities of $6,975,000, minus, preferred stock outstanding of $1,500,000
(15,000 X $100), divided by the number of common shares outstanding of 1,250,000.

Question: 2050

An organization purchased a computer on January 1, 1996 for $108,000. It was estimated to have a 4-
year useful life and a salvage value of $18,000. The double-declining-balance method is to be used.
The amount of depreciation to be reported for the year ending December 31, 1996 is ________.

A. ($108,000 - 18,000) (25% X 2)


B. ($108,000) (25% X 2)
C. ($108,000 - 18,000) (25% X 1/2)
D. ($108,000) (25%)
E. ($108,000) (25% X 1/2)

Answer: B

Explanation:
When using a declining-balance method, a constant rate is applied to the changing carrying value of
the asset. The constant rate for the double-declining-balance method is twice the straight-line rate
((100% / 4 years) X 2)

Question: 2051

Which of the following is/are true about depreciation?

I. Depreciation allocates non-cash expenses to period in which long-lived assets are used.
II. Depreciation provides funds for the replacement of an asset.
III. Depreciation charges arise due to an adherence to accrual method of accounting.

A. I only
B. III only
C. I & III
D. I, II & III

Answer: C

Explanation:
Depreciation is the accounting device which facilitates a charging of the costs incurred in prior
periods on assets that have been used for productive activities in the current period. Note that this is
a non- cash expense since the cash left the firm's coffers at the time the asset was purchased (either
in part or in full). Remember, however, that depreciation has cash flow consequences indirectly
through its effect on taxes. Depreciation expense owes its existence to the accrual method of
revenue recognition whichrequires that expenses be matched with the revenues generated. No
depreciation charges would exist under Cash accounting, which would allocate expenses as and

Page | 750
when cash was used to make payments on asset purchases. Clearly, depreciation does not provide
funds for anything for it is an accounting entry.

Question: 2052

Which of the following is/are true about trading securities?

I. They are current assets.


II. They are reported at fair market value.
III. Changes in their reported value are allocated directly to retained earnings.

A. I only
B. II & III
C. I, II & III
D. I & II

Answer: D

Explanation:
Marketable securities classified as "Trading securities" are financial securities purchased with the
intent to sell in the near future. They are therefore categorized as current assets and reported at the
fair market value on the date of the balance sheet. When the classification of a trading security is
changed, the assignation of the security to the new account is carried out at the fair market value on
the transfer date, with any gain or loss reflected separately on the income statement.

Question: 2053

Where is Unearned Revenue reported in the financial statements?

A. liability section of the balance sheet


B. revenue section of the income statement
C. asset section of the balance sheet
D. operating expense section of the income statementThat answer is correct!

Answer: A

Explanation:
Unearned revenue represents "services owed" and is reported in the liability section of the balance
sheet.

Question: 2054

Under a periodic inventory system, how is COGS determined?

A. Ending inventory is counted and subtracted from beginning inventory.


B. Cost of goods sold is accumulated as sales are made.
C. None of these answers is correct.
D. The cost of ending inventory is subtracted from costs of goods available for sale.

Answer: D

Explanation:
Under the periodic inventory system, only the ending inventory is counted and priced. Cost of goods
sold is determined by deducting the cost of the ending inventory from the cost of goods available for

Page | 751
sale.

Question: 2055

The portion of the insurance premiums that has expired during the fiscal period is classified as

A. an increase in retained earnings


B. an asset
C. an expense
D. a liability

Answer: C

Explanation:
In accordance with the matching rule, the amount of an insurance policy that has expired during the
accounting period must be recorded and classified as an expense for the period.

Question: 2056

How should the effect of a change in accounting estimate be accounted for?

A. None of these answers.


B. By reporting pro forma amounts for prior periods.
C. In the period of change and future periods if the change affects both.
D. By restating amounts reported in financial statements of prior periods.
E. As prior-period adjustments to beginning retained earnings.

Answer: C

Explanation:
The effect of a change in accounting estimate must be accounted for as a component of income from
continuing operations in the period of change and in future periods.

Question: 2057

In a period of falling prices, the FIFO inventory method

A. neither of these answers is correct


B. both of these answers are correct
C. magnifies the effects of the business cycle on income
D. gives the lowest possible value for ending inventory

Answer: B

Explanation:
The first-in-first-out (FIFO) method is based on the assumption that the costs of the first items
acquired should be assigned to the first items sold, therefore ending inventory on hand is based on
the most recent prices.

Question: 2058

Realized gains and losses from sales of assets are measured as

A. fair value of asset less net book value of asset.

Page | 752
B. sales price of asset less net book value of asset.
C. net book value less fair value of asset.
D. sales price of asset less fair value of asset.

Answer: B

Question: 2059

A firm has total assets of 5,320, net sales of 8,395, average receivables of 894, current assets of 2,393
and a current ratio of 1.2. Its average receivables collection period equals ________.

A. 41.73 days
B. 29.92 days
C. 23.19 days
D. 38.87 days

Answer: D

Explanation:
This is measured by two ratios in slightly different ways:

a. Receivables turnover ratio = Net annual sales/average receivables


b. Average receivables collection period = 365/receivables turnover

In this case, receivables turnover = 8,395/894 = 9.39. Therefore, average receivables collection
period = 365/9.39 = 38.87 days. Notice that the collection period is measured in "days."

Question: 2060

Calculating COGS under a periodic inventory system relies on which of the following?

A. a physical count of the ending inventory


B. an analysis of the inventory value of each sale
C. both of these answers are correct
D. neither of these answers is correctThat answer is correct!

Answer: A

Explanation:
Under the periodic inventory system, only the ending inventory is counted and priced. Cost of goods
sold is determined by deducting the cost of the ending inventory from the cost of goods available for
sale.

Question: 2061

When prices are rising, which of the following inventory valuation methods produces a higher ending
inventory value?

A. LIFO
B. FIFO
C. None of these answers
D. Average cost

Answer: B

Page | 753
Explanation:
When prices are rising, FIFO will produce the highest ending inventory value because the first in
(lowest cost) inventory is the first out. This leaves the most recently purchased inventory on hand
and since it was purchased at higher prices, it has the highest cost and ending inventory will have the
greatest value.

Question: 2062

Which of the following is/are true?

I. Assets = Equity + Liabilities


II. Equity = Retained earnings + Dividends Payable
III. Assets + Liabilities = Equity
IV. Assets = Liabilities - Equity

A. II & IV
B. I only
C. I & II
D. III only

Answer: B

Explanation:
The basic accounting equation states that Assets = Liabilities + Owners' Equity.

Question: 2063

Which of the following would have an effect on cash flow from operations?

I. Sale of machinery for $50,000 with a net book value of $35,000.


II. Purchase of supplies on credit.
III. Remittance by customer in payment of goods purchased last accounting period.
IV. Lease payment on machinery, which is accounted for as a capital lease.

A. I, II and III
B. I, III and IV
C. III and IV
D. I and III

Answer: C

Question: 2064

Which of the following is/are true about liquidity ratios, all else equal?

I. The cash ratio increases as average receivables increase.


II. The quick ratio is a more conservative liquidity ratio than the current ratio.
III. Liquidity ratios decrease as total liabilities decrease.

A. III only
B. II only
C. I, II & III
D. I & III

Page | 754
Answer: B

Explanation:
Receivables are not part of the cash ratio and hence, changes in receivables do not directly affect the
cash ratio. The quick ratio does consider receivables in addition to cash and marketable securities but
ignores all other current assets. Thus, it is more conservative than the current ratio.

Question: 2065

Which of the following is/are true about stock dividends?

I. stock dividends lead to a decrease in the retained earnings account.


II. large stock dividends are valued at their par value.
III. small stock dividends are valued at their fair market value at the time of issuance.

A. II & III
B. I & III
C. I, II & III
D. I only

Answer: C

Explanation:
Stock dividends are given to move retained earnings into capital accounts and hence represent a
permanent capitalization of earnings. In that sense, stock dividends do not have any economic
significance but are an accounting bookkeeping device. The recapitalization does have legal
implicationsfor future distributions since the retained earnings account is the primary source of cash
dividends and other accounts are more difficult to dip into for distributions. If the stock dividend is
less than 20-25%, the additional stocks are valued at fair market value on the declaration date and
the resultant amount moved from retained earnings account to the "par-value" account and the
"additional paid-in capital" account. Larger stock dividends are valued at par.

Question: 2066

Which of the following items is deducted from net income to arrive at cash flow from operations
when using the indirect method?

A. depreciation expense
B. decrease in accounts receivable
C. decrease in accounts payable
D. amortization expense

Answer: C

Explanation:
A decrease in accounts payable is an outflow.

Question: 2067

The peaks and valleys of the business cycle tend to be smoothed out using which inventory method?

A. weighted average
B. LIFO

Page | 755
C. gross profit method
D. FIFO

Answer: B

Explanation:
When prices are moving either upward or downward, the cost of goods sold (under LIFO) will show
costs closer to the price level at the time the goods were sold. Therefore the LIFO method tends to
show a smaller net income during inflationary times and a larger net income during deflationary
times than other methods of inventory valuation.

Question: 2068

Profit margin is a ratio that:

A. shows the return on net sales


B. is calculated as net sales divided by operating expenses
C. yields the company's financial position at a point in time
D. compares total assets to net salesThat answer is correct!

Answer: A

Explanation:
Profit margin, also called return on net sales, is calculated by dividing net income by net sales. This
ratio measures the average portion of each dollar of revenue that ends up as profit.

Question: 2069

Tracy company reports the following in its statement of cash flows:

Net Income$1,000
Depreciation and Amortization350
Decrease (Increase) in Accounts receivable(10)
Decrease (increase) in inventory200
Decrease (increase) in prepaid expenses80
Increase (decrease) in trade payables(300)
Increase (decrease) in taxes payable75
Cash Flow from operations1,395

If Tracy shows depreciation expense of $275 in its income statement, cash paid for amortization is
________.

A. $75
B. $525
C. not determinable
D. $0

Answer: D

Explanation:
No cash outflow.

Question: 2070

Page | 756
The following information should be used according to the provisions of SFAS 95 (Statement of Cash
flows) and using the following data.

Net Income$50,000
Provision for bad debts$2,000
Increase in Inventory$1,000
Increase in accounts payable$2,000
Purchase of new equipment$15,000
Sale of equipment for $10,000 gain$20,000
Depreciation expense$5,000
Repurchase of common stock$10,000
Payment of dividend$4,000
Interest payment$3,000

What is change in cash?

A. $39,000
B. $45,000
C. $46,000
D. $49,000That answer is correct!

Answer: A

Explanation:
$50,000 + (-$1,000 + $2,000 - $15,000 + $20,000 - $10,000 - $4,000 - $3,000)

Question: 2071

Excerpts from the balance sheet of Milton Corporation as of April 30, 1997 are presented as follows:

Cash$725,000
Accounts receivable (net)$1,640,000
Inventories$2,945,000
Total current assets$5,310,000
Accounts payable$1,236,000
Accrued liabilities$831,000
Total current liabilities$2,067,000

The board of directors of Milton met on May 5, 1997 and declared a quarterly cash dividend in the
amount of $200,000 ($0.50 per share). The dividend was paid on May 28, 1997 to shareholders of
record as of May 15, 1997.

Assume that the only transactions that affected Milton during May 1997 were the dividend
transactions. If the dividend declared by Milton had been a 10% stock dividend instead of a cash
dividend, Milton's total shareholders' equity would have been

A. unchanged by either the dividend declaration or the dividend distribution.


B. increased by the dividend declaration and unchanged by the dividend distribution.
C. decreased by the dividend declaration and increased by the dividend distribution.
D. unchanged by the dividend declaration and decreased by the dividend distribution.
E. increased by the dividend declaration and decreased by the dividend distribution.That answer is
correct!

Answer: A

Page | 757
Explanation:
The declaration and distribution of a stock dividend involves transferring an amount from retained
earnings to common stock. However, the total shareholders' equity remains the same.

Question: 2072

When purchased, plant assets are recorded at:

A. cost
B. market value
C. future value
D. lower of cost or marketThat answer is correct!

Answer: A

Explanation:
Fixed assets are recorded at cost, which includes all normal and reasonable expenditures necessary
to get the asset in place and ready to use.

Question: 2073

If an independent auditor has not become satisfied by means of other auditing procedures with
respect to opening inventories, they should

A. either disclaim an opinion on the statement of income or qualify the opinion thereon, regardless
of the degree of materiality of the amounts involved.
B. either disclaim an opinion on the statement of income or qualify the opinion thereon, depending
on the degree of materiality of the amounts involved.
C. disclaim an opinion or qualify the opinion on the statements as a whole.
D. express an adverse opinion on the statements taken as a whole when the amount in question is
material.

Answer: B

Explanation:
If the auditor has become satisfied as to current inventory, the auditor may use alternative
procedures to become satisfied as to opening inventories. But if they cannot become satisfied
regarding inventories, a qualified opinion or a disclaimer of opinion may be expressed, depending on
materiality. Because cost of goods sold is dependent on opening inventories, an unqualified opinion
on the income statement is not possible.

Question: 2074

The cumulative effect of changing to a new accounting principle on the amount of retained earnings
at the beginning of the period in which the change is made should be included in net income of
________.

A. the period of change and future periods


B. the period of change
C. future periods
D. none of these answers
E. neither future periods nor the period of change

Page | 758
Answer: B

Explanation:
A change from one acceptable method of accounting to another should be accounted for as the
cumulative effect of a change in accounting principle. The cumulative effect should be recognized as
a component of net income, net of tax effect, in the period of change.

Question: 2075

A company is currently being sued by a customer. A reasonable estimate can be made of the costs
that would result from a ruling unfavorable to the company, and the amount involved is material.
The company's managers, lawyers, and auditors agree that there is a remote likelihood of an
unfavorable ruling. This contingency

A. none of these answers.


B. should be disclosed as a parenthetical comment in the balance sheet.
C. should be disclosed as an appropriation of retained earnings.
D. should be disclosed in a footnote.
E. need not be disclosed in a footnote.

Answer: E

Explanation:
Losses arising from litigation should be accrued if both probable and reasonably estimable, and
should be disclosed if reasonably possible. In this case, the likelihood is remote. Therefore, no
disclosure is required.

Question: 2076

Which of the following is not a current liability?

A. Allowance for Uncollectible Accounts


B. Wages Payable
C. Unearned Revenue
D. Notes PayableThat answer is correct!

Answer: A

Explanation:
Allowance for Uncollectible Accounts is a contra account to Accounts Receivable, both of which are
reported in the current asset section of the balance sheet.

Question: 2077

Which of the following is/are true about computation of Basic EPS?

I. With stock splits and stock dividends, previously reported EPS numbers are retroactively
readjusted.
II. Shares issued for the acquisition of another business are included from the date of issuance.
III. Shares issued in pooling of interests are assumed to have been outstanding at the beginning of all
the periods reported.

A. II & III
B. I, II & III

Page | 759
C. I & III
D. I only

Answer: B

Explanation:
In the computation of basic EPS, one must be careful about the time period for which shares are
assumed to be outstanding. This becomes an issue when the firm issues new shares or reacquires
outstanding shares during the accounting period. Further, the purpose for which the shares are
issued are also of concern since the computation of the weighted number of shares outstanding must
be consistent with the logic behind the accounting treatment of the newly issued shares. Hence,
when shares are issued for the acquisition of another business, they are considered as equity issued
to raise new capital for investment and therefore included from the date of issuance. On the other
hand, under pooling of interests method, the merged companies are assumed to have been a
combined entity since their respective inceptions. Therefore, when shares are issued under pooling
of interests, they are assumed to have been outstanding at the beginning of all the periods reported.
Finally, stock splits and stock dividends change the number of shares without any direct impact on
earnings. Hence, for consistency, previously reported EPS numbers are retroactively readjusted.

Question: 2078

When prices are rising, which of the following inventory valuation methods produces the lowest
income tax liability?

A. None of these answers


B. LIFO
C. Average cost
D. FIFO

Answer: B

Explanation:
When prices are rising, LIFO will produce the lowest profit, and the lowest income tax liability,
because the last in (highest cost) inventory is the first out. This leaves the oldest inventory on hand
and since it has the lowest cost, ending inventory will have the lowest value which means that cost of
goods sold will be the highest and profits will be the lowest (ending inventory = beginning inventory
+ net purchases - cost of goods sold).

Question: 2079

What impact will the amortization of an intangible asset over a longer than appropriate period have
on a firm?

A. It will result in higher than otherwise sales revenues.


B. None of these answers.
C. All of these answers.
D. It will reduce a firm's current income tax liability.
E. It will improve reported earnings.

Answer: E

Explanation:
Earnings, not revenues will be improved because amortizing an intangible asset over a longer than
appropriate period reduces the annual amortization expense; this improves pre-tax and after-tax

Page | 760
profits.

Question: 2080

The interest expense on a premium bond _______ over time.

A. decreases
B. remains constant
C. can increase or decrease, depending on interest rate movements
D. increasesThat answer is correct!

Answer: A

Explanation:
Remember that the book value of the liability of any straight bond equals the face value at maturity.
Hence, when the bond is issued at a premium, the premium amount is amortized over the life of the
bond. The outstanding liability thus decreases steadily toward the face value. The decreasing liability
decreases the interest expense over time.

Question: 2081

According to the FASB conceptual framework, which of the following situations violates the concept
of reliability?

A. Financial statements include property with a carrying amount increased to management's


estimate of market value.
B. None of these answers.
C. Financial statements are issued 9 months late.
D. Data on segments having the same expected risks and growth rates are reported to analysts
estimating future profits.
E. Management reports to stockholders regularly refer to new projects undertaken, but that financial
statements never report project results.That answer is correct!

Answer: A

Explanation:
Reliability has 3 primary qualities: verifiability, neutrality and representational faithfulness.
Neutrality is violated here as information should not be prepared or reported to obtain a
predetermined result, and should be free from bias.

Question: 2082

The accumulated depreciation account should show

A. total depreciation for fixed assets since the business was formed
B. current depreciation expense plus estimated depreciation for next year
C. total depreciation for fixed assets still in use
D. only the depreciation expense recognized during the current year

Answer: C

Explanation:
Depreciation recognized and accumulated on fixed assets still on the books is reflected in the
Accumulated Depreciation account(s).

Page | 761
Question: 2083

Costs that can be reasonably associated with specific revenues but not with specific products should
be

A. capitalized and then amortized over a period not to exceed 60 months.


B. expensed in the period in which the related revenue is recognized.
C. capitalized and then amortized over a period not to exceed 40 years.
D. allocated to specific products based on the best estimate of the production processing time.
E. charged to expense in the period incurred.

Answer: B

Explanation:
The expense recognition principle of "associating cause and effect" or "matching" applies when a
direct cause and effect relationship can be demonstrated between costs and particular revenues.

Question: 2084

Which of the following relationships is true?

A. Gross profit margin < Net profit margin < Operating profit margin
B. Gross profit margin < Operating profit margin < Net profit margin
C. Gross profit margin > Operating profit margin > Net profit margin
D. Net profit margin > Gross profit margin > Operating profit margin

Answer: C

Explanation:
Gross profit = Net sales - COGS
Operating profit = Gross Profit - Sales & General Expenses = EBDIT
Net Income = Earnings after depreciation, interest expense and taxes
= Operating profit - depreciation - interest expense - taxes
Since gross profit > operating profit > net income, "Gross profit margin > Operating profit margin >
Net profit margin" is the correct choice.

Question: 2085

A firm's financial data show:

Taxable income1,500
Taxes paid500
Non-cash operating expenses780
Bonds retired700
Loss on retired bonds140

Then, the financing cash flow equals ________.

A. -700
B. -840
C. 160
D. -900

Page | 762
Answer: D

Explanation:
The firm's tax rate equals 500/1,500 = 30%. Note that loss on bonds retired is an extraordinary item
under US GAAP and presented after-tax. Hence, the total cash spent on retiring bonds = 700 +
140/(1-0.3) = 900. So financing cash flow = -900.

Question: 2086

Which of the following is/are FALSE?

I. Interest expenses that are capitalized are charged against investing cash flows.
II. Firms that expense interest costs incurred on debt must treat them as financing cash flows.
III. Firms that expense costs show lower equity than comparable firms that capitalize the costs.
IV. Capitalization of expenses leads to lower tax payments in the first year.

A. I, II & IV
B. II & IV
C. II & III
D. III & IV

Answer: B

Explanation:
Interest expenses are treated as operating cash flows under US GAAP. Capitalization leads to higher
net income since the entire expenses are not charged against it. Due to this, the tax deductions are
lower in the first year, leading to higher taxes. The higher income leads to the capitalizing firm having
a higher equity. The difference in equity reduces over time to zero.

Question: 2087

The deferred income tax account

A. can be reported as an asset


B. is reported as a liability if it has a credit balance
C. is where the difference between income tax expense and income tax payable is reconciled
D. all of these answers are correct

Answer: D

Explanation:
The difference between income tax expense (based on accounting income) and the actual income
taxes payable (based on taxable income) is reconciled in an account called deferred income taxes. It
can be reported as an asset or a liability, depending on its balance and the circumstances.

Question: 2088

In periods of rising prices, which inventory costing method results in the smallest income tax
expense?

A. Average cost
B. Perpetual
C. FIFO
D. LIFO

Page | 763
Answer: D

Explanation:
LIFO assigns the largest dollar value amounts to COGS when purchase prices are rising, thus resulting
in lower net income and lower income taxes.

Question: 2089

Patterson Company has the following information of one of its vehicles purchased on January 1,
1992:

Vehicle cost$50,000
Useful life, years, estimated5
Useful life, miles, estimated100,000
Salvage value, estimated$10,000

Actual miles driven:

199230,000
199310,000
199415,000
199525,000
199612,000

No estimates were changed during the life of the asset. The 1994 depreciation expense using the
sum-of-years'-digits (SYD) method was ________.

A. $13,333
B. $8,000
C. $10,667
D. $10,000
E. $6,000

Answer: B

Explanation:
SYD depreciation is calculated on a constant depreciable base equal to $40,000 ($50,000 original cost
-10,000 salvage value), multiplied by the SYD fraction. The denominator is the sum of the digits of
the total years of the expected useful life (1+2+3+4+5=15). The SYD fraction's numerator equals the
remaining years. For 1994, the fraction is 3/15, and depreciation expense is $8,000 ($40,000 X 3/15).

Question: 2090

Under the accrual basis of accounting, which of the following statements is true?

I. Reported net income provides a measure of operating performance.


II. Revenue is recognized when cash is received, and expenses are recognized when payment is
made.
III. Cash inflows are recognized when they are received, and cash outflows are recognized when they
are made.

A. I only
B. I, II and III

Page | 764
C. III only
D. I and III

Answer: D

Question: 2091

Valuation of PP&E and natural resources emphasizes all of the following accounting objectives except
________.

A. accounting for monies invested in assets


B. historical cost
C. conservatism principle
D. reality

Answer: D

Explanation:
The financials portray historical information; they do not paint a picture of today.

Question: 2092

A firm has a net profit margin of 25%. Its total asset turnover equals 1.3 and equity turnover equals
2.1. The firm's ROE and financial leverage equal ________.

A. 28.6%, 1.47
B. 43.2%, 1.33
C. 52.5%, 1.62
D. 39.8%, 0.62

Answer: C

Explanation:
ROE = net income/equity = (net income/sales)*(sales/equity) = net profit margin * equity turnover =
25%*2.1 = 52.5% Financial Leverage = Total assets/equity = (total assets/sales)*(sales/equity) =
(1/asset turnover)*(equity turnover) = 2.1/1.3 = 1.62.

Question: 2093

When compared to the percentage-of-completion method, the completed contract method

A. reports larger total assets.


B. reports higher net assets.
C. reports higher cash flows.
D. reports income earlier.
E. uses higher estimates of selling prices.That answer is correct!

Answer: A

Explanation:
The completed contract method reports larger total assets because it accumulates inventory.

Question: 2094

Page | 765
In the direct method statement of cash flows, the primary component of investing cash flow is
typically

A. dividend payments.
B. long-term investments in securities.
C. capital expenditures for long-term assets.
D. repayment of debt.
E. investments in affiliates.

Answer: C

Explanation:
Capital expenditures for long-term assets such as plant and machinery are usually the primary
component of investing cash flow.

Question: 2095

Items reported as prior-period adjustments

A. do not include the effect of a mistake in the application of accounting principles as this is
accounted for as a change in accounting principle rather than a prior-period adjustment.
B. do not affect the presentation of prior-period comparative financial statements.
C. are reflected as adjustments of the opening balance of the retained earnings of the earliest period
presented.
D. none of these answers.
E. do not require further disclosure in the body of financial statements.

Answer: C

Explanation:
Prior-period adjustments are made for the correction of errors. All the effects of errors on prior-
period financial statements are reported as adjustments to beginning retained earnings for the
earliest period presented in the retained earnings statement.

Question: 2096

Which of the following is/are true about computation of Basic EPS?

I. Reacquired shares are excluded from the date of acquisition.


II. Shares issued in the purchase of assets are assumed to have been outstanding for the entire
period.
III. Shares issued in mergers are assumed to have been outstanding from the date of issuance.

A. I only
B. II & III
C. I & II
D. I, II & IIIThat answer is correct!

Answer: A

Explanation:
Weighted average shares used in EPS computations are calculated using the following rules:

1. Reacquired shares are excluded from the date of acquisition.

Page | 766
2. Shares issued in the purchase of assets are included from the date of issuance.
3. Shares issued in mergers are assumed to have been outstanding for all periods for which EPS
numbers are presented.

Question: 2097

At December 31, 1996, Davis Inc. awaits judgment on a lawsuit for a competitor's infringement of
Davis' patent. Legal counsel believes it is probable that Davis will win the suit and indicated the most
likely award together with a range of possible awards. How should the lawsuit be reported in Davis'
1996 financial statements?

A. None of these answers.


B. Neither in note disclosure nor by accrual.
C. In note disclosure only.
D. By accrual for the lowest amount of the range of possible awards.
E. By accrual for the most likely award.

Answer: C

Explanation:
A loss contingency is recognized but not a gain contingency following the principle of conservatism. A
gain contingency is only disclosed in the notes until the settlement is realized.

Question: 2098

A firm is purchased for more than the fair market value of its assets. The excess is:

A. considered a "premium paid" and amortized over the life of the acquired assets.
B. considered as "Goodwill."
C. written off against the retained earnings on the balance sheet.
D. treated as an extraordinary loss & presented net of taxes on the income statement.

Answer: B

Explanation:
Goodwill is defined as the price paid in excess of the fair market value of the assets of the target firm.

Question: 2099

When analyzing the balance sheet, which of the following is an argument against using LIFO in times
of rising prices?

A. Neither of these answers is correct.


B. Both of these answers are correct.
C. Under LIFO, ending inventory will be overstated.
D. Under LIFO, ending inventory is valued at the oldest prices, an unrealistic valuation.

Answer: D

Explanation:
LIFO values ending inventory at the oldest prices, thus in times of rising prices, inventory will be
understated. This results in reporting an unrealistic valuation of the company's inventory.

Question: 2100

Page | 767
Cash outflows for payment of cash dividends is an example of:

A. cash flows from financing activities


B. cash flows from investing activities
C. cash flows from noncash investing and financing activities
D. cash flows from operating activitiesThat answer is correct!

Answer: A

Explanation:
Providing stockholders with a return on their investment in the form of a cash dividend is a financing
activity.

Question: 2101

Why do the corporation's directors declare stock dividends?

A. to increase the number of shares outstanding


B. all of these answers are correct
C. to keep the market value of the company's stock affordable
D. to provide tangible evidence of management's confidence in the company's strong performance

Answer: B

Explanation:
Declaring a stock dividend will increase the number of shares outstanding and thereby keep the per
share price low enough to be an attractive investment. Stock dividends may also show the company
management's confidence in the present and future performance of the company.

Question: 2102

The cash flow statement provides more objective information about all of the following, except

A. trends in cash flow components.


B. management decisions regarding financial policy, dividend policy, and investment for growth.
C. a firm's ability to generate cash flows from operations.
D. cash consequences of investing and financing decisions.
E. the amount a firm can be leveraged.

Answer: E

Explanation:
In this case balance sheet data must be used to determine the answer.

Question: 2103

Which of the following would not be included as a liability on a corporate balance sheet?

A. Accrued liabilities
B. Current portion of long-term debt
C. Accounts payable
D. Marketable securities
E. Notes payable

Page | 768
Answer: D

Explanation:
Marketable securities are not a liability; it represents the value of a company's investment in stocks
bonds and money market instruments.

Question: 2104

The Income Statement:

I. reflects the current operating performance of the firm.


II. indicates whether the firm is healthy and growing or not.
III. explains the changes in assets, liabilities and Equity of the firm.
IV. is a snapshot of a firm's operations at a given time.

A. I, II, III & IV


B. II & III
C. I only
D. I & IV

Answer: C

Explanation:
II needs a cash flow statement, in addition. III is not true since the Income statement does not
contain all the details which pertain to changes in assets and liabilities. Finally, an income statement
shows the performance over a time period and is hence, not a "snapshot" of operations.

Question: 2105

Which of the following is not a common tool used in financial statement analysis?

A. trend series analysis


B. random walk analysis
C. common size statement analysis
D. ratio analysis

Answer: B

Question: 2106

Irwin Inc. has a self-insurance plan. Each year, retained earnings is appropriated for contingencies in
an amount equal to insurance premiums saved minus recognized losses from lawsuits and other
claims. As a result of a 1996 accident, Irwin is a defendant in a lawsuit in which it will probably have
to pay damages of $190,000. What are the effects of this lawsuit's probable outcome on Irwin's 1996
financial statements?

A. An increase in expenses and no effect on liabilities.


B. No effect on expenses and an increase in liabilities.
C. An increase in both expenses and liabilities.
D. None of these answers.
E. No effect on either expenses nor liabilities.

Answer: C

Page | 769
Explanation:
Since the outcome of the lawsuit is probable and estimable, the amount of $190,000 will be recorded
in the financial statements as a contingent liability and an expense.

Question: 2107

Which of the following best describes a balance sheet?

A. None of these answers.


B. A balance sheet reports changes over a period of time in component accounts that comprise the
ownership of a firm.
C. A balance sheet summarizes the financial position of a company at a given point in time.
D. A balance sheet details the cash inflows and outflows that are related to a company's operating,
investing, and financing activities over a period of time.
E. A balance sheet measures a company's financial performance over a specified period of time.

Answer: C

Explanation:
A balance sheet provides a detailed listing of a company's assets, liabilities, and equity at a point in
time. This provides a glimpse at a company's financial condition.

Question: 2108

A dividend that distributes more than 25% of the outstanding shares before the dividend is called:

A. a liquidating dividend
B. illegal
C. a large stock dividend
D. a capitalizing dividend

Answer: C

Explanation:
Large stock dividends distribute greater than 25% of outstanding shares before the dividend and
small stock dividends distribute less than or equal to 25% of previously outstanding shares.

Question: 2109

The loss from an uncollectible account is

A. an asset
B. a regular expense of doing business
C. a liability
D. a reduction in revenue

Answer: B

Explanation:
The benefit from selling on credit to customers far outweighs the cost of losses from uncollectible
accounts. These losses are a regular expense of doing business.

Question: 2110

Page | 770
If the estimated life of a long-term asset is increased, which of the following is true?

I. The depreciation expense increases


II. Taxes decrease
III. Income increases
IV. Cashflow decreases

A. I & II
B. III & IV
C. I & III
D. I, III & IV

Answer: B

Explanation:
The increase in the asset's life estimate decreases the depreciation expense. Hence, income
increases, taxes increase and cashflow decreases (due to higher taxes).

Question: 2111

When merchandise inventory is purchased under a perpetual system, which account is debited?

A. Cash
B. Merchandise Inventory
C. Accounts Payable
D. Purchases

Answer: B

Explanation:
The perpetual inventory system records purchases of merchandise inventory directly into a balance
sheet account called Merchandise Inventory.

Question: 2112

The weighted average method is based on the assumption that the cost of merchandise sold should
be calculated using the:

A. average price of beginning inventory plus purchases during the period


B. average price per unit of ending inventory
C. average price per unit of beginning inventory
D. average price of ending inventory plus purchases during the periodThat answer is correct!

Answer: A

Explanation:
Under the weighted average method, inventory is priced at the average cost of the goods available
for sale (Beginning inventory plus purchases during the period).

Question: 2113

A transaction that is unusual in nature and infrequent in occurrence should be reported separately as
a component of income

Page | 771
A. after cumulative effect of accounting changes and after discontinued operations of a segment of a
business.
B. before cumulative effect of accounting changes and before discontinued operations of a segment
of a business.
C. after cumulative effect of accounting changes and before discontinued operations of a segment of
a business.
D. before income from continuing operations.
E. before cumulative effect of accounting changes and after discontinued operations of a segment of
a business.

Answer: E

Explanation:
A transaction that is unusual and infrequent is classified as an extraordinary item. The following is
the order of items to be reported separately in the income statement: income from continuing
operations, discontinued operations, extraordinary items, cumulative effect of changes in accounting
principle, and net income.

Question: 2114

Which of the following is not subject to depreciation?

A. automobiles
B. land
C. machinery
D. land improvements

Answer: B

Explanation:
Land has unlimited useful life and is not consumed when it is used, and therefore not subject to
depreciation.

Question: 2115

Which of the following statements are correct?

I. A company's choice of accounting principles for financial reporting purposes does not affect net
cash flow for the accounting period.
II. A company's choice of accounting principles for financial reporting purposes does not affect
operating cash flow.
III. If a company sells its receivables this will increase operating cash flow.
IV. If a company sells its receivables this will increase financing cash flow.

A. I and III
B. I, II and III
C. II and IV
D. I and IVThat answer is correct!

Answer: A

Explanation:
The choice of accounting principles affects only the classification and not the net cash flow, and

Page | 772
reducing receivables in considered an increase in operating cash flow.

Question: 2116

A firm pays out half its earnings as dividends. If its net income is $50, then

I. Its assets increase by $25


II. Its equity increases by $25
III. Its equity increases by $50
IV. The book value of the firm increases by $50

A. III & IV
B. I & II
C. I & IV
D. II & IV

Answer: B

Explanation:
Since half of $50 is paid out as dividends and the other half retained, the book value and equity
increase by $25 and so do assets. Remember the basic equation, Assets = Liabilities + Equity.

Question: 2117

Birch Ltd. had net income for the year of $101,504 and a simple capital structure consisting of the
following common shares outstanding:

Months OutstandingNumber of Shares

January - February24,000
March - June29,400
July - November36,000
December35,040

Assume Birch issued a 20% stock dividend on August 1st. In this case, earnings per share (rounded to
the nearest cent) were

A. $2.72
B. $2.67
C. $2.88
D. $4.23
E. $2.41

Answer: B

Explanation:
Following is the calculation for the weighted average number of shares outstanding (before the 20%
stock dividend):

24,000 X (2 / 12) = 4,000


29,400 X (4 / 12) = 9,800
36,000 X (5 / 12) = 15,000
35,040 X (1 /12) = 2,920
weighted average 31,720

Page | 773
Stock dividends are assumed to have occurred at the beginning of the year. Thus, the weighted
average number of shares equals the amount before the stock dividend plus 20% more which is
equal to 38,064 (31,720 + 20% X 31,720). EPS equals net income divided by the weighted average
number of shares outstanding which is $2.67 ($101,504 / 38,064).

Question: 2118

A firm has a high debt-to-equity ratio. In order to improve this ratio in earlier years, it will prefer:

A. LIFO accounting and accelerated depreciation.


B. LIFO accounting and straight-line depreciation.
C. FIFO accounting and straight-line depreciation.
D. FIFO accounting and accelerated depreciation.

Answer: C

Explanation:
The company will use FIFO (assuming prices are increasing) and straight line depreciation to increase
income and hence, retained earnings. This improves the debt-to-equity ratio.

Question: 2119

Bungling, Inc., is currently facing a lawsuit as a defendant. Its honest lawyers estimate that there is a
60-40 chance that Bungling will lose the lawsuit. The jury could award the plaintiff punitive damages
ranging from $10,000 to $50,000. Bungling should

A. not recognize any liability but disclose the details in footnotes.


B. not do anything since the lawsuit is still not decided and considerable uncertainty exists.
C. recognize a liability of $50,000 on its balance sheet as a conservative estimate.
D. recognize a liability of $10,000 on its balance sheet.That answer is correct!

Answer: A

Explanation:
If considerable uncertainty exists in measurement of an asset or a liability arising from events like a
lawsuit, disclosure must occur in footnotes. The disclosure includes the nature of the contingency
and an estimate or a range of estimates about the possible losses.

Question: 2120

Firms A and B are identical. In one year, Firm A's statements have the beginning inventory
understated and the ending inventory overstated. Then,

I. A's tax payment is higher.


II. B's tax payment is higher.
III. B shows a higher income.
IV. A shows a higher income.

A. I & IV
B. II & III
C. I & III
D. II & IVThat answer is correct!

Page | 774
Answer: A

Explanation:
COGS = Beginning inventory - Ending inventory + Purchases. Hence, if BI is understated and EI
overstated, COGS is understated, implying income is overstated. Hence, Firm A will show higher
income and pay higher taxes. It should be remembered that implicit in the use of the above
inventory equation is the assumption that there have been no write-downs or write-ups in the
inventory.

Question: 2121

The main use of the balance sheet for creditors would be

A. to forecast future cash flow needs.


B. to review the short-term liquidity of the firm.
C. to forecast cash collections.
D. to forecast changes in fixed assets thereby assisting in the firm's profitability.
E. to provide information about the nature of assets that the firm uses as debt collateral.

Answer: E

Explanation:
The balance sheet provides information about a firm's resources and obligations, including liquidity
and solvency.

Question: 2122

Which of the following describes a change in reporting entity?

A. A company acquires a subsidiary that is to be accounted for as a purchase.


B. None of these answers.
C. A business combination is made using the pooling-of-interests method.
D. A manufacturing company expands its market from regional to nationwide.
E. A company acquires additional shares of an investee and changes from the equity method of
accounting to consolidation of the subsidiary.

Answer: C

Explanation:
A change in reporting entity can occur in the following ways: initial publication of consolidated
financial statements; change in consolidation policy regarding subsidiaries; and pooling of interests.

Question: 2123

Free cash flow is not

A. cash flow that includes discretionary uses such as debt reduction.


B. the measure of cash available to the firm for discretionary uses after making all required cash
outlays.
C. better to have a large sum of.
D. defined as cash flow from operations minus all capital expenditures.
E. a cash flow with one definition because of its several discretionary uses.

Answer: E

Page | 775
Explanation:
The definition of free cash flow varies widely, depending on how one defines required and
discretionary uses.

Question: 2124

The following data have been obtained from a firm's financial statements:

operating profit margin34%


interest expenses465
depreciation expenses123
debt-to-equity ratio0.60
total assets2,375
total sales4,109
tax rate37%

The firm's ROE equals ________.

A. 19.17%
B. 22.88%
C. 56.32%
D. 34.44%

Answer: D

Explanation:
The answer to this requires you to know the extended duPont system. However, you should NOT rely
on your memory for complex formulas. Rather, start from the primary quantity needed (ROE here)
and use some basic definitions and common sense. This approach will ensure that you are not caught
by surprise during the exam. You need to calculate net income and total equity to obtain ROE, since
ROE = net income/total equity. Operating profits equal earnings before depreciation, interest and
taxes (EBDIT). The operating profit margin expresses EBDIT as a fraction of total sales and indicates
the level of profitability of the firm. In this case, EBDIT = 4109*34% = 1,397. Since interest expenses
are taxdeductible, EBT = 1,397 - interest expense - depreciation = 1,397 - 465 - 123 = 809. Therefore,
net income after taxes = 809*(1 - 37%) = 510. Also, debt + equity = total assets and debt = 0.6*equity
(given). So total assets = 2,375 = 1.6*equity, giving equity = 2,375/1.6 = 1,484. Finally, ROE =
510/1484 = 34.34%.

Question: 2125

A defined benefit pension plan:

A. pays defined benefits for a certain period after retirement.


B. disburses benefits based on the returns on the fund's investments.
C. promises to pay retirees a specific income stream.
D. none of these answers.

Answer: C

Explanation:
In a defined benefit pension plan, the retirement benefits are "predefined." The employer commits
to providing the benefits regardless of the performance of the pension plan. Thus, in this plan, the
risk of pension plan performance is borne by the employer and not the employee.

Page | 776
Question: 2126

At the end of a fiscal period, any revenue that has been earned but the company has not received
payment for should be debited to an appropriate

A. asset account
B. expense account
C. liability account
D. revenue accountThat answer is correct!

Answer: A

Explanation:
Revenue should be recognized in the period in which it is earned and is credited to the appropriate
revenue account. If payment has not been received, then a debit entry to Accounts Receivable must
be booked.

Question: 2127

Which of the following is/are FALSE about Basic EPS and Diluted EPS?

I. Basic EPS excludes anti-dilutive securities but Diluted EPS must include these.
II. Basic EPS ignores instruments like convertible bonds and warrants but Diluted EPS does not.
III. The Treasury stock method, when applied to Basic EPS, compares the average stock price during
the period to the strike price in determining conversion.

A. III only
B. I & III
C. I, II & III
D. II only

Answer: B

Explanation:
Note that the questions asks for false statements. (I) is false since both methods exclude anti-dilutive
securities from computations. Further, since the Basic EPS ignores all potentially dilutive securities
and takes into account only simple capital structure instruments (stocks, bonds, preferred equity), it
has no need for the Treasury Stock method i.e. the Treasury stock method is never used in the
computation of Basic EPS.

Question: 2128

Which of the following relationships is/are correct:

I. change in assets + change in liabilities = change in equity.


II. change in retained earnings = net income + dividends paid.
III. assets - liabilities = retained earnings + contributed capital.
IV. assets = liabilities + revenues - expenses.

A. I, II & III
B. III only
C. I, II, III & IV
D. II & III

Page | 777
Answer: B

Explanation:
The basic accounting equation is Total assets = Total liabilities + Total Equity. In I & II, the "plus"
should be "minus."

Question: 2129

When the Percentage of Sales method is used, the estimated bad debt expense is calculated by:

A. multiplying net sales on account times the percentage


B. multiplying total sales on account times the percentage
C. subtracting the percentage of net sales on account from the balance of Allowance for Uncollectible
Accounts
D. dividing total sales on account by the percentageThat answer is correct!

Answer: A

Explanation:
This method bases the period adjustment on a percent of net sales on account for the period.

Question: 2130

The following data are available for a firm for a given year:

Net Sales21,896
Sales & marketing expenses4,346
Administrative expenses2,143
COGS10,084
Depreciation967
Interest expense573
Tax rate35%
Dividends paid3,445
Preferred Dividends897
Average total equity37,432
Average common equity26,782
Average total liabilities18,583

In the above example, the firm's return on common equity equals ________.

A. 5.83%
B. 4.77%
C. 6.12%
D. 4.38%That answer is correct!

Answer: A

Explanation:
Return on common equity = (Net income - preferred dividends)/average common equity. In the
above example, Net Income = Earnings after depreciation, interest expense and taxes = (21,896 -
4,346 - 2,143 - 10,084 - 967 - 573)*(1 - 0.35) = 2,459. Therefore, Return on common equity = (2,459 -
897)/26,782 = 5.83%.

Page | 778
Question: 2131

A firm has just acquired a long-term asset with a useful life of 5 years. Its acquisition cost was
$65,000 and its salvage value is estimated at $10,000. If the firm uses straight-line depreciation
method, what's the depreciation expense recognized in Year 3?

A. $10,600
B. $14,500
C. $11,000
D. $13,000

Answer: C

Explanation:
In Straight-line method, depreciation is constant at (1/n)*(acquisition cost - salvage value). In this
case, depreciation = (1/5)*(65,000-10,000)=$11,000.

Question: 2132

Which of the following is/are differences between depreciation and depletion?

I. Depletion can be applied only to natural resources while depreciation can be applied to most
production resources.
II. The amount of depletion depends upon total production but the amount of depreciation need not
be so dependent.
III. Depreciation expenses conform with accrual accounting while depletion expenses conform with
cost recovery accounting.

A. II & III
B. I & II
C. I, II & III
D. none of these answers

Answer: B

Explanation:
Depletion expenses also conform with accrual accounting.

Question: 2133

Why should the analyst place special emphasis in their assessment of cash balances?

A. Cash represents the point in a firm's operating cycle where management has maximum discretion
with the deployment and use of its resources.
B. Cash is a company's most liquid asset.
C. All of these answers.
D. Cash is typically the only asset that is available to meet a firm's financial obligations.
E. None of these answers.

Answer: C

Explanation:
All of these answers are correct because any assessment of a firm's liquidity should be focused on a
firm's ability to meet its near-term liabilities. While inventory may be sold, it is subject to shrinkage

Page | 779
through loss of value, theft, etc. and while accounts receivable a current assets, not all firms collect
the full balance of what they are owed, let alone collecting it when it is due. Cash is king in the world
of finance because it is cash and cash alone that is used to meet financial commitments.

Question: 2134

A firm wants to decrease its debt-to-asset ratio without affecting its current ratio. Which of the
following actions can it undertake?

I. Retire some of its outstanding bonds by using proceeds from the sale of old assets.
II. Increase sales on credit.
III. Pay off a part of the "salaries payable" account using cash.
IV. Issue new stocks and invest the proceeds to purchase a production plant.

A. I, II & III
B. I & IV
C. IV only
D. II & III

Answer: C

Explanation:
Both "accounts receivable," which represents sales on credit and "salaries payable" are current
accounts. Therefore, if you do not want to affect current ratio, II and III are not acceptable strategies.
When old assets are sold and the proceeds used to retire outstanding bonds, the debt-to-asset ratio
decreases. This is because debt/asset ratio is almost always less than 1 (we will ignore abnormal
cases where book equity can go negative; e.g. the case where the firm keeps borrowing and paying
out the proceeds as dividends). Hence, when both numerator and denominator are decreased, the
ratio decreases. However, the current portion of the long-term debt also gets retired, increasing the
current ratio. So I is also not an acceptable strategy, though at first glance it appears so. With IV, debt
is unaffected but assets increase, decreasing the debt/asset ratio.

Question: 2135

When the Percentage of Sales method is used, the estimated bad debt expense is calculated by

A. dividing total sales on account by the percentage


B. subtracting the percentage of net sales on account from the balance of allowance for uncollectible
accounts
C. multiplying total sales on account times the percentage
D. multiplying net sales on account times the percentage

Answer: D

Explanation:
This method bases the period adjustment on a percent of net sales on account for the period.

Question: 2136

The interest expense on a discount bond _______ over time.

A. decreases
B. remains constant
C. can increase or decrease, depending on interest rate movements

Page | 780
D. increases

Answer: D

Explanation:
Remember that the book value of the liability of any straight bond equals the face value at maturity.
Hence, when the bond is issued at a discount, the discount amount is amortized over the life of the
bond. The outstanding liability thus increases steadily toward the face value. The increasing liability
increases the interest expense over time.

Question: 2137

A firm has to pay 1,000 to its suppliers in one year. The interest rate on this liability is 9%. The current
liability recorded by the firm is ________.

A. 1,090
B. 90
C. 1,000
D. 917

Answer: C

Explanation:
Current liabilities, in general, are recorded at the principal portion of the payment. Hence, even
though the firm in this case will have to pay 1,090 in 1 year, the current liability equals the principal
portion = 1,000.

Question: 2138

What is the purpose of information presented in notes to the financial statements?

A. To provide disclosures required by generally accepted accounting principles.


B. To present management's responses to auditor comments.
C. To provide recognition of amounts not included in the totals of the financial statements.
D. To correct improper presentation in the financial statements.
E. None of these answers.That answer is correct!

Answer: A

Explanation:
Users of the financial information are the focus of financial reporting. GAAP requires disclosures in
the notes to facilitate the users' understanding of the financial statements.

Question: 2139

In a common size statement, a firm shows average receivables of 9.3%. Its receivables turnover
equals 1.23. What's the average receivables collection period?

A. 296.8 days
B. insufficient information
C. 321.6 days
D. 184.3 daysThat answer is correct!

Answer: A

Page | 781
Explanation:
Average receivables collection period = 365/receivables turnover. In this case, the collection period =
365/1.23 = 296.8 days.

Question: 2140

Which of the following appear(s) on the Income Statement?

I. Interest revenue.
II. Depreciation of a machinery.
III. Employee payroll taxes payable.
IV. Commissions earned by sales staff.

A. III only
B. I, II & IV
C. II & IV
D. I & III

Answer: B

Explanation:
Employee payroll taxes payable are a current liability.

Question: 2141

To carry out a comparative analysis of a firm's financial statements over a period of more than 3-4
years, one should use ________.

A. year-to-year change analysis


B. horizon analysis
C. ratio analysis
D. index-number trend analysis

Answer: D

Explanation:
To carry out a comparative statement analysis over a long period of time, you can express the
financial statement numbers in terms of a common base. This base could be one of the years for
which financial statements are available or can be arbitrarily set to a number like 100. This process
then converts the financial statement quantities into indices and these can be analyzed more
conveniently.

Question: 2142

A firm's financial audit ensures all of the following EXCEPT:

A. The company is managing its risks within the guidelines specified in its charter.
B. There are adequate controls and checks in place in its accounting systems.
C. There are no material errors or omissions in its financial reports.
D. The management is not misstating its performance.That answer is correct!

Answer: A

Page | 782
Explanation:
It is not a financial audit function to check on the reasonableness of the risks undertaken by the
institution.

Question: 2143

The loss from an uncollectible account is:

A. an asset
B. a liability
C. a regular expense of doing business
D. a reduction in revenue

Answer: C

Explanation:
The benefit from selling on credit to customers far outweighs the cost of losses from uncollectible
accounts. These losses are a regular expense of doing business.

Question: 2144

Topaz Metals Inc. produces precious metals from its mining operations. The selling price for its
product is reasonably assured, the units are interchangeable, and the costs of selling and distributing
the product are insignificant. In order for Topaz to recognize revenue as early in the revenue cycle as
is permitted under generally accepted accounting principles, the revenue recognition method that
Topaz should use is the ________.

A. percentage-of-completion method
B. production method
C. cash method
D. completed-contract method
E. cost recovery method

Answer: B

Explanation:
Revenue is recognized when the conditions of "realized" or "realizable" and earned are met. If
products are readily realizable because they are salable at reliably determinable prices without
significant effort, revenues may be recognized at completion of production.

Question: 2145

The following financial data on CashCow, Inc. have been taken from its financial statements for 1996:

a. Dividends paid$25,000
b. Sale of land$64,000
c. Inventory purchases$29,000
d. Purchase of a warehouse$208,000
e. Bonds issued$90,000
f. Dividends received from investments$17,000
g. Interest paid on bonds$2,400
h. Salaries paid$107,400
i. Cash collection from customers$28,400
j. Loss on land sale$18,000

Page | 783
k. Beginning cash balance$312,000

In the above question, the ending cash balance of the firm was ________.

A. $157,600
B. $121,600
C. $139,600
D. $172,400

Answer: C

Explanation:
Note that the loss on equipment is a non-cash event. Using the direct method, beginning cash
balance + net cash inflow = ending cash balance Hence, ending balance = 312,000 + (-25,000+64,000-
29,000 - 208,000 + 90,000 + 17,000 - 2,400 - 107,400 + 28,400) = $139,600.

Question: 2146

The par value of a common stock represents

A. the estimated market value of the stock when it was issued.


B. none of these answers.
C. the liability ceiling of a shareholder when a company undergoes bankruptcy proceedings.
D. the total value of the stock that must be entered in the issuing corporation's records.
E. the amount that must be recorded on the issuing corporation's record as paid-in capital.

Answer: C

Explanation:
Par value represents a stock's legal capital. It is an arbitrary value assigned to stock before it is issued.
Par value represents a shareholder's liability ceiling because, as long as the par value has been paid
in to the corporation, the shareholder obtains the benefits.

Question: 2147

Which of the following is NOT a revenue recognition method when there is uncertainty surrounding
the realizability of income?

A. Completed Contract Method


B. Cost Recovery Method
C. Installment Sales Method
D. Real Estate MethodThat answer is correct!

Answer: A

Explanation:
The completed contract method is used for revenue recognition in the case of long-term contracts. In
particular, it is chosen instead of the percentage-of-completion method if there are no reliable
measures of the degree to which the contract obligations have been fulfilled.

Question: 2148

Which of the following would be considered a liability that arises from operating activities?

Page | 784
A. Taxes payable
B. All of these answers
C. Unearned income
D. Accounts payable
E. Accrued salaries

Answer: B

Explanation:
All of these answers represent liabilities that originate from the operation of a business; i.e., from
the normal course of operating a business. Liabilities that arise from "operations" typically do not
require compensation in the form of interest. This can be contrasted with liabilities that arise from
financing activities where the lender requires the payment of interest to compensate for the
extension of credit.

Question: 2149

Which of the following best describes retained earnings?

A. All of these answers.


B. None of these answers.
C. Retained earnings represent the cumulative net profits a firm since inception.
D. Retained earnings represent the sum total of cash obtained from the sale of common stock to
investors.
E. Retained earnings represent the cumulative net profits of a firm since inception, minus cumulative
dividends paid to common stockholders.

Answer: E

Explanation:
Retained earnings represent the earned capital of a firm. It consists of the accumulated undistributed
earnings (net profits, less cash dividends) of a firm since inception.

Question: 2150

Which of the following would be reported as an extraordinary item?

A. uninsured loss from a flood


B. gain or loss from passing of a new law
C. all of these answers are correct
D. gain or loss from early retirement of debt

Answer: C

Explanation:
An item must be both unusual and infrequent (and material in amount) to be classified as
extraordinary.

Question: 2151

Permanent earnings refers to

A. the net cash flow plus the change in market value of the firm's net assets.
B. the amount of earnings that can be paid out as dividends without changing the value of the firm.

Page | 785
C. the level of earnings that can be maintained in the future given the firm's capital investment.
D. the amount that can be normally earned and equals the market value of the firm's assets times
the firm's required rate of return.
E. the average earnings the firm generates over a specified period.
F. none of these answers.

Answer: D

Explanation:
Permanent earnings is similar to economic earnings; it is the base to which a multiple is applied to
arrive at a "fair price."

Question: 2152

If the beginning inventory is overstated and the ending inventory is understated, which of the
following is/are true:

I. COGS is understated
II. COGS is overstated
III. Income is overstated
IV. Income is understated

A. I & III
B. IV only
C. II only
D. II & IV

Answer: D

Explanation:
COGS = Beginning inventory - Ending inventory + Purchases Hence, if BI is overstated and EI
understated, COGS is overstated, implying income is understated. It should be remembered that
implicit in the use of the above inventory equation is the assumption that there have been no write-
downs or write-ups in the inventory.

Question: 2153

Which of the following is NOT a change in accounting?

A. Change in accounting principle


B. Change in past accounting error
C. Changing in accounts reporting entity
D. Changing in accounting estimate

Answer: B

Explanation:
Correction to an accounting error is not considered a change in accounting. It is treated as a "prior
period adjustment" to the beginning balance of retained earnings. All other are considered
accounting changes.

Topic 5, Corporate Finance

Question: 2154

Page | 786
Rollins Corporation is constructing its MCC schedule. Its target capital structure is 20 percent debt, 20
percent preferred stock, and 60 percent common equity. Its bonds have a 12 percent coupon, paid
semiannually, a current maturity of 20 years, and sell for $1,000. The firm could sell, at par, $100
preferred stock, which pays a 12 percent annual dividend, but flotation costs of 5 percent would be
incurred. Rollins' beta is 1.2, the risk-free rate is 10 percent, and the market risk premium is 5
percent. Rollins is a constant growth firm, which just paid a dividend of $2.00, sells for $27.00 per
share, and has a growth rate of 8 percent. The firm's policy is to use a risk premium of 4 percentage
points when using the bond-yield-plus-risk-premium method to find k(s) (component cost of retained
earnings). The firm's net income is expected to be $1 million, and its dividend payout ratio is 40
percent. Flotation costs on new common stock total 10 percent, and the firm's marginal tax rate is 40
percent. What is Rollins' cost of preferred stock?

A. 12.6%
B. 13.2%
C. 11.0%
D. 12.0%
E. 10.0%

Answer: A

Explanation:
K(ps) (cost of preferred stock) = $12/$100 (0.95) = 12.6%.

Question: 2155

Despite relative congruence in their ranking methods, NPV and MIRR will sometimes produce
conflicting answers. Which of the following correctly illustrates an example in which the two
methods would likely produce conflicting rankings?
I. When examining projects with non-normal cash flows
II. When examining projects that differ substantially in scale
III. When examining independent projects
IV. When examining projects that differ substantially in their lifespan

A. I and III
B. I and II
C. II and IV
D. II and IV

Answer: C

Explanation:
While the MIRR method is designed to tackle many of the problems associated with the traditional
IRR calculation, there exist situations in which the MIRR will produce rankings which conflict with
those produced by the NPV method. Specifically, when mutually-exclusive projects whose lifespans
or scale differ substantially are being examined. In these situations, the NPV calculation should be
relied on, as this method is considered to produce the correct results.

Question: 2156

Monte Carlo simulation

A. All of the answers are correct.


B. Is capable of using probability distributions for variables as input data instead of a single numerical

Page | 787
estimate for each variable.
C. Produces both an expected NPV (or IRR) and a measure of the riskiness of the NPV or IRR.
D. None of the answers are correct.
E. Can be useful for estimating a project's stand-alone risk.

Answer: A

Explanation:
These are all true.

Question: 2157

Proponents of which of the following theories would claim that companies seek to balance the
taxshelter benefits of debt financing with the increased interest rates and risk of bankruptcy that
come with increased debt levels?

A. Modigliani & Miller's "with-taxes" Theory of Capital Structure


B. Bird-in-the-Hand Theory
C. Modigliani & Miller's Theory of Capital Structure
D. Tax Preference Theory
E. Signaling Theory
F. Trade-off Theory of Leverage

Answer: F

Explanation:
Trade-off Theory of Leverage
The Trade-off Theory of Leverage claims that firms will seek to balance the tax-shelter benefits of
debt financing with the increased interest costs and risk of bankruptcy that come with increased debt
levels. The Trade-off Theory of Leverage came about largely from criticisms raised against the
Modigliani andMiller Theory of Capital Structure under the "with-taxes" assumption. M&M claimed
that, under a restrictive set of assumptions, the value of firms would be maximized only when their
capital structure is comprised of 100% debt. The Trade-off Theory of Leverage proposed a more
realistic and moderate answer to the Capital Structure debate, and remains an important milestone
in the field of Pure Finance.

Question: 2158

Photon Corporation has a target capital structure of 60 percent equity and 40 percent debt. The firm
can raise an unlimited amount of debt at a before-tax cost of 9 percent. The company expects to
retain earnings of $300,000 in the coming year and to face a tax rate of 35 percent. The last dividend
was $2 per share and the growth rate of the company is constant at 6 percent. If the company needs
to issue new equity, then the flotation cost will be $5 per share. The current stock price is $30.
Photon has the following investment opportunities:
ProjectCostIRR
1$100,00010.5%
2$200,00013.0
3$100,00012.0
4$150,00014.0
5$75,0009.0

A. $150,000
B. $450,000
C. $350,000

Page | 788
D. $550,000
E. $625,000

Answer: B

Explanation:
Calculate the retained earnings break point (BPRE) as $300,000/0.6 = $500,000. Calculate ks as
D1/P0 + g = $2(1.06)/$30 + 6% = 13.07%. Calculate ke as D1/(P0 - F) + g = $2(1.06)/($30 - $5) + 6% =
14.48%. Find WACC below BPRE as: WACC = 0.6(13.07%)+ 0.4(9%)(1 - 0.35) = 10.18%. Thus, up to
$500,000 can be financed at 10.18%. Find WACC above BPRE as: WACC = 0.6(14.48%) + 0.4 (9%) (1 -
0.35) = 11.03%. Thus, financing in excess of $500,000 costs 11.03%. Projects 2, 3, and 4 all have IRRs
exceeding either WACC and should be accepted. These projects require $450,000 in financing.
Project 1 is the next most profitable project.
Given its cost of $100,000, half or $50,000 can be financed at 10.18% and the other half must be
financed at 11.03%. The relevant cost of capital for Project 1 is then 0.5(10.18%) + 0.5(11.03%) =
10.61%. Since Project 1's IRR is less than the cost of capital, it should not be accepted. The firm's
optimal capital budget is $450,000.

Question: 2159

A stock has an expected dividend growth rate of 4.9%. The firm has just paid a dividend of $2.5 per
share. With a required rate of return of 10%, the stock is trading at $42.8. The stock is:

A. overpriced.
B. insufficient information.
C. fairly priced.
D. under-priced.

Answer: D

Explanation:
The fair price of the stock with a required rate of return, r and a dividend growth rate, g, is given by P
= D1/(r-g), where D1 = Do*(1-g) = dividend to be paid next year. In this case, the fair price of the stock
equals 2.5*1.049/(10% - 4.9%) = $51.42. Thus, the stock is under-priced by $(51.42 - 42.8) = $8.62.

Question: 2160

Which of the following is not considered a capital component?

A. All of these are considered capital components


B. Preferred stock
C. Common stock
D. Long-term debt
E. Retained earnings

Answer: A

Explanation:
The four major capital components are debt, preferred stock, retained earnings, and new issues of
common stock.

Question: 2161

Sun State Mining Inc., an all-equity firm, is considering the formation of a new division, which will

Page | 789
increase the assets of the firm by 50 percent. Sun State currently has a required rate of return of 18
percent, U.S. Treasury bonds yield 7 percent, and the market risk premium is 5 percent. If Sun State
wants to reduce its required rate of return to 16 percent, what is the maximum beta coefficient the
new division could have?

A. 2.0
B. 1.0
C. 2.2
D. 1.6
E. 1.8

Answer: B

Explanation:
Old assets = 1.0.New assets = 0.5.Total assets = 1.5.
Old required rate:New required rate:
18% = 7% + (5%)b16% = 7% + (5%)b
beta = 2.2.beta = 1.8.
New b must not be greater than 1.8, therefore
0.3333(b) = 0.3333
b = 1.0.
Therefore, beta of the new division cannot exceed 1.0.

Question: 2162

Which of the following is not considered a relevant concern in determining incremental cash flows
for a new product?

A. The cost of a product analysis completed in the previous tax year and specific to the new product.
B. All of these are relevant.
C. The use of factory floor space which is currently unused but available for production of any
product.
D. Shipping and installation costs associated with preparing the machine to be used to produce the
new product.
E. Revenues from the existing product that would be lost as a result of some customers switching to
the new product.That answer is correct!

Answer: A

Explanation:
The product analysis cost is considered a sunk cost and is not relevant.

Question: 2163

Which of the following statements is correct?

A. The primary advantage of simulation analysis over scenario analysis is that scenario analysis
requires a relatively powerful computer, coupled with an efficient financial planning software
package, whereas simulation analysis can be done using a PC with a spreadsheet program or even a
calculator.
B. All of these answers are correct.
C. Sensitivity analysis is incomplete because it fails to consider the range of likely values of key
variables as reflected in their probability distributions.
D. In comparing two projects using sensitivity analysis, the one with the steeper lines would be

Page | 790
considered less risky, because a small error in estimating a variable, such as unit sales, would
produce only a small error in the project's NPV.
E. Sensitivity analysis is a risk analysis technique that considers both the sensitivity of NPV to
changes in key variables and the likely range of variable values.

Answer: C

Explanation:
A project's stand-alone risk depends on (1) the sensitivity of NPV to changes in key variables and (2)
the range of likely values of these variables as reflected in their probability distribution. Sensitivity
analysis considers only the first factor.

Question: 2164

The Clientele Effect theory implies that investors in the low tax brackets will prefer:

A. none of these answers.


B. high capital gains.
C. high dividend payouts.
D. low dividend payouts.

Answer: C

Explanation:
The Clientele Effect is based on the presumption that different groups of investors will prefer
different dividend policies based on their tax status and their need for current versus future income
requirements. Hence, investors who face high taxes on current income will tend to avoid stocks with
high pay-out ratios. This lowering of demand for such stocks will tend to depress their prices and to
take advantage of this; investors in low tax brackets would gravitate toward them. To this, add the
fact that usually, investors in low tax brackets with sufficient capital to invest tend to be either people
who are old and retired or institutions like pension funds. Both these groups have a higher need for
current income but are sensitive to liquidation of capital. They therefore prefer their income from
stocks to be in the form of dividends rather than from the sale of their stock holdings.

Question: 2165

Alyeska Salmon Inc., a large salmon canning firm operating out of Valdez, Alaska, has a new
automated production line project it is considering. The project has a cost of $275,000 and is
expected to provide after-tax annual cash flows of $73,306 for eight years. The firm's management is
uncomfortable with the IRR reinvestment assumption and prefers the modified IRR approach. You
have calculated a cost of capital for the firm of 12 percent. What is the project's MIRR?

A. 17.0%
B. 15.0%
C. 12.0%
D. 14.0%
E. 16.0%

Answer: E

Explanation:
TV = $73,306(FVIFA(12%,8)) = $73,306(12.300) = $901,663.80.
$275,000 = $901,663.80 / (1 + MIRR)^8
(1 + MIRR)^8 = (FVIF(Irr,8)) = 3.27869.

Page | 791
Look in table: Periods = 8, I = 16%. MIRR = 16%.
Alternate method
3.27869^1/8 = 1 + MIRR
MIRR = 16%.

Question: 2166

Consider the following argument: "The cost of common stock should decrease as the dividend payout
is increased because investors are more certain of receiving these dividends than the capital gains
which are supposed to be derived from retained earnings." This statement applies best to which of
the following financial theories? Choose the best answer.

A. Tax Preference Theory


B. Dividend Irrelevance Theory
C. Tax Irrelevance Theory
D. Dividend Relevance Theory
E. Bird-in-hand Theory

Answer: E

Explanation:
The Bird-in-the-Hand Theory came about as a refutation of Modigliani and Miller's Dividend
Irrelevance Theory. The founders of the Bird-in-the-Hand Theory, Myron Gordon and John Lintner,
stated that investors are more confident in the fact that they will receive dividends versus capital
gains. So said, the cost of common stock should decrease as the payout ratio is increased.
The Tax Preference Theory states that investors prefer capital gains to dividends, and this is due to
the structure of tax rates. Specifically, dividends are typically taxed at a higher rate than capital gains,
and are in this respect less attractive.

Question: 2167

An investment project has an initial cost, and then generates inflows of $50 a year for the next five
years. The project has a payback period of 3.6 years. What is the project's internal rate of return
(IRR)?

A. 12.05%
B. 13.47%
C. 15.89%
D. 14.66%
E. 11.18%

Answer: A

Explanation:
Investment cost = $180.
CF(0) = -180
CF(1-5) = 50
Solve for IRR = 12.05%.

Question: 2168

According to the signaling theory, if a firm issues debt capital to finance a project, the firm's
management must consider the project to be ________.

Page | 792
A. none of these answers
B. likely to raise the probability of bankruptcy
C. very desirable
D. not very profitable

Answer: C

Explanation:
According to the signaling theory of capital structure, a firm will try to raise debt capital when the
project's returns are deemed very favorable and vice versa. The firm is signaling that the project has
sufficient cash flows to pay back the debt.

Question: 2169

The return on the best alternative use of an asset, or the highest return that will not be earned if
funds are invested in a particular project is known as which of the following terms?

A. Sunk Cost
B. Cannibalization
C. Opportunity Cost
D. Externality
E. Incremental Cash Flow

Answer: C

Explanation:
Opportunity cost is defined as the return on the best alternative use of an asset, or the highest return
that will not be earned if funds are invested in a particular project

Question: 2170

Suppose Congress votes to raise the personal tax rate on interest and dividend income. However, it
does not change the capital gains tax or the corporate tax rates. This will have the effect of:

A. increasing the reliance on debt financing.


B. increasing the reliance on retained earnings as capital.
C. decreasing the reliance on equity capital.
D. decreasing the sizes of seasoned equity offerings.

Answer: B

Explanation:
As personal tax rates increase, firms have to modify their reliance on different capital markets so as
to minimize the costs imposed on debt and equity investors. An increase in interest and dividend
income makes debt and dividend payouts costlier. On the other hand, since capital gains are not
affected, firms will tend to decrease their dividend pay-out ratios and bank on retained earnings to
finance their capital requirements.

Question: 2171

The initial investment outlay consists of which of the following?


I. Up-front costs of the project's fixed assets.
II. Flotation costs associated with raising the necessary capital.
III. Increases in net working capital.

Page | 793
IV. Present value of all interest expenses associated with the project capital.

A. II only
B. I, II & IV
C. I, II & III
D. I, II, III & IV
E. I & III
F. III only
G. I only
H. IV only

Answer: E

Explanation:
The initial investment outlay consists of up-front costs of the project's fixed assets and any increases
in net working capital. Costs involved in raising the finances are not part of the initial outlay.

Question: 2172

Which of the following statements is most correct?

A. The CAPM approach is typically used to estimate a firm's flotation cost adjustment factor, and this
factor is added to the DCF cost estimate.
B. These statements are all incorrect.
C. In practice (as opposed to in theory), the DCF method and the CAPM method usually produce
exactly the same estimate for k(s).
D. The risk premium used in the bond-yield-plus-risk-premium method is the same as the one used
in the CAPM method.
E. Under normal conditions, the CAPM (Capital Asset Pricing Model) approach to estimating a firm's
cost of retained earnings gives a higher estimate than the DCF (Discounted Cash Flow) approach.

Answer: B

Explanation:
All are incorrect. Under the CAPM approach, it is at best difficult to obtain correct estimates of the
inputs require to make it operational. The same could be said about the growth rate input under the
DCF approach. The risk premium under the bond-yield-plus-risk premium approach is purely
judgmental and results in a ballpark estimate.

Question: 2173

Which of the following statements is most correct?

A. If a company does a 2-for-1 stock split, its stock price will roughly double.
B. An open-market dividend reinvestment plan is likely to be attractive to companies that are looking
to issue additional shares of common stock.
C. All of these answers are correct.
D. None of the answers are correct.
E. Stock repurchases have the effect of reducing financial leverage.

Answer: D

Explanation:
A new stock type of DRIP would result in raising new capital for the firm. Stock repurchases increase

Page | 794
financial leverage. In a 2-for-1 stock split, the stock price will be halved.

Question: 2174

A financial analyst with Smith, Kleen, & Beetchnutty is examining shares of Claypool Manufacturing
for possible investment. Assume the following information:
Sales: $50,000,000
Fixed costs: $33,000,000
Variable costs: $8,500,000
Interest expense: $900,000
Tax rate: 35%
Weighted Average Cost of Capital: 11.50%
Beta coefficient: 0.96
Common shares outstanding: 4,000,000
Using this information, what are the earnings per share (EPS) for Claypool Manufacturing?

A. $1.34
B. $1.40
C. The answer cannot be determined from the information provided.
D. $1.70
E. $1.24
F. $1.11

Answer: E

Explanation:
The EPS figure is perhaps the single most popular term in the field of conventional equity
investments, along with the Price-to-Earnings Ratio (P/E). Any glance into financial media and
business periodicals will undoubtedly uncover numerous instances in which the EPS figure is cited or
discussed. While quite popular and useful, most investors, and many business professionals,
undoubtedly do not understand the mechanics behind the EPS calculation, and an investigation into
the components of EPS is a valuable learning experience. The EPS calculation is found by the
following equation:
{EPS = [(Sales - Fixed Costs - Variable Costs - Interest Expense)(1 - Tax Rate)] / [# of Common Shares
Outstanding]}
Additionally, the EPS figure can be found by:
{EPS = [(EBIT - Interest Expense)(1 - Tax Rate) / # of Common Shares Outstanding]} Incorporating the
given information into the first EPS equation will yield the following: {EPS = [($50,000,000 -
$33,000,000 - $8,500,000 - $900,000)(1 - 0.35)] / 4,000,000} = $1.24.

Question: 2175

The net cash flow attributable to an investment project is known as which of the following terms?

A. Sunk Cost
B. Opportunity Cost
C. Incremental Cash Flow
D. Cannibalization
E. Externality

Answer: C

Explanation:
Incremental cash flow is defined as the net cash flow attributable to an investment project.

Page | 795
Question: 2176

Following an internal investigation into her professional business activities, a financial analyst with
Smith, Kleen & Beetchnutty admits that in her NPV and IRR calculations, she has failed to index all
cash flows for the effects of anticipated inflation. However, the analyst claims that the discount rate
she has used in her calculations does take into effect anticipated inflation.
Which of the following correctly describes the effects this will have on the NPV and IRR calculations?

A. NPV will be biased downward, IRR will be biased upward


B. NPV will be biased upward, IRR will be biased downward
C. Both NPV and IRR will be biased downward
D. Both NPV and IRR will remain unaffected
E. NPV will be biased downward, IRR will be unaffected
F. Both NPV and IRR will be biased upward

Answer: C

Explanation:
By failing to index the cash flows of projects in her NPV analysis, while at the same time including an
adjustment for inflation into the discount rate, this analyst has biased the NPV calculation
downward. This is because the cash inflows are being understated by the inflation-adjusted
discounting. This phenomenon will skew the NPV figure downward. Remember that while the
Internal Rate of Return calculation does not specify an explicit discount rate, rather calculates the
discount rate that equates the cash inflows of a project with its cash outflows, the fact remains that
the cash flows in the calculation have not been indexed for the effects of positive inflation. What has
happened here is that cash flows have been understated, and this will bias the IRR calculation
downward.

Question: 2177

Your company has decided that its capital budget during the coming year will be $20 million. Its
optimal capital structure is 60 percent equity and 40 percent debt. Its earnings before interest and
taxes (EBIT) are projected to be $34.667 million for the year. The company has $200 million of assets;
its average interest rate on outstanding debt is 10 percent; and its tax rate is 40 percent. If the
company follows the residual dividend policy and maintains the same capital structure, what will its
dividend payout ratio be?

A. 35%
B. 30%
C. 15%
D. 20%
E. 25%

Answer: E

Explanation:
Capital budget = $20 million.
Optimal capital structure: 60% equity, 40% debt.
EBIT = $34.667 million.
Assets = $200 million.
k(d) = 10%; T = 40%. Dividend Payout = ?
Debt = 0.40($200 million) = $80 million.
Interest = 0.10($80 million) = $8 million.

Page | 796
EBIT$34.667
-INT8.000
EBT$26.667
Taxes (40%)10.667
NI$16.000
Equity needed = 0.60($20 million) = $12 million.
Net Income$16
-Equity needed12
Amount left for dividend$4
Dividend Payout = $4/$16 = 25%.

Question: 2178

Modigliani and Miller (MM) argued that dividend policy is irrelevant. On the other hand, Gordon and
Lintner (GL) argued that dividend policy does matter. GL's argument rests on the contention that

A. most investors will reinvest rather than spend dividends, so it would save investors money (taxes)
if corporations simply reinvested earnings rather than paid them out as dividends.
B. k(s) = D1/P0 + g is constant for any dividend policy.
C. none of the answers are correct.
D. investors, because of tax differentials, value a dollar of expected capital gains more highly than a
dollar of dividends.
E. because of perceived differences in risk, investors value a dollar of dividends more highly than a
dollar of expected capital gains.

Answer: E

Explanation:
The main conclusion of MM's irrelevance theory is that dividend policy does not affect the required
rate of return on equity. Gordon-Lintner disagreed stating that k(s) decreases as the dividend payout
is increased because investors are less certain of receiving the capital gains which should result from
retaining earnings than they are of receiving dividends. They said that investors value expected
dividends more highly than expected capital gains because the dividend yield is less risky than the
growth component in the total expected return equation, k(s) = D1/Po + g. MM disagreed and
theorized that k(s) is independent of dividend policy, implying that investors are indifferent between
dividends and capital gains.

Question: 2179

Gulf Electric Company (GEC) uses only debt and equity in its capital structure. It can borrow
unlimited amounts at an interest rate of 10 percent so long as it finances at its target capital
structure, which calls for 55 percent debt and 45 percent common equity. Its last dividend was $2.20;
its expected constant growth rate is 6 percent; its stock sells on the NYSE at a price of $35; and new
stock would net the company $30 per share after flotation costs. GEC's tax rate is 40 percent, and it
expects to have $100 million of retained earnings this year. GEC has two projects available: Project A
has a cost of $200 million and a rate of return of 13 percent, while Project B has a cost of $125 million
and a rate of return of 10 percent. All of the company's potential projects are equally risky. What is
GEC's cost of equity from newly issued stock?

A. 13.77%
B. 13.33%
C. 10.00%
D. 12.66%
E. 12.29%

Page | 797
Answer: A

Explanation:
k(d) (interest rate on the firm's new debt) = 10%
k(d)(1 - T) (after-tax-component cost of the debt) = 10%(0.6) = 6%.
D/A = 55%; D0 = $2.20; g = 6%; P0 = $35; PN = $35; T = 40%.
Retained earnings = $100M; BP(RE) = $100M/ .45 = $222.22M.
k(s) (component cost of retained earnings) = $2.33/$35 + 6% = 12.66%.
k(e) (component cost of external equity) = $2.33/$30 + 6% = 13.77%.

Question: 2180

Suppose changes in corporate law make it more difficult for debt holders to force companies into
bankruptcies. This will cause firms to:

A. raise more equity capital through retained earnings.


B. either increase or decrease their debt levels.
C. increase their debt-to-equity ratios.
D. decrease their debt-to-equity ratios.

Answer: B

Explanation:
It is tempting to assume that lower probability of bankruptcy will entice firms into borrowing more.
However, remember that debt holders are not entirely stupid! They will factor in this change in the
law and demand a higher yield and stricter covenants on corporate debt to compensate for the lower
recourse they have against the firms. Depending on whether they actually underestimate or
overestimate the effect and also depending on whether the firms perceive that the debt holders
have under- or overestimated the effect, companies could increase or decrease their debt levels.

Question: 2181

As the capital budgeting director for Chapel Hill Coffins Company, you are evaluating construction of
a new plant. The plant has a net cost of $5 million in Year 0 (today), and it will provide net cash
inflows of $1 million at the end of Year 1, $1.5 million at the end of Year 2, and $2 million at the end
of Years 3 through 5. Within what range is the plant's IRR?

A. 17 - 18%
B. 15 - 16%
C. 18 - 19%
D. 14 - 15%
E. 16 - 17%

Answer: C

Explanation:
Time line: (In millions)
012345 Years
-511222
Financial calculator solution: (In millions)
Inputs: CF(0) = -5; CF(1) = 1.0; CF(2) = 1.5; CF(3) = 2.0; N(j) = 3.
Output: IRR% = 18.37%.

Page | 798
Question: 2182

A project that is intended to increase income is known as ________.

A. Externality
B. Replacement Project
C. Cannibalization
D. Opportunity Cost
E. Expansion Project
F. Low Cost Provider

Answer: E

Explanation:
An expansion project is defined as a project that is intended to increase income.

Question: 2183

A mutual fund manager is examining the financial and operating condition of a Questron Media
Corporation, and has discovered the following information.
Sales: $3,000,000
Fixed costs: $1,000,000
Variable costs: $300,000
Interest expense: $150,000
Tax rate: 35%
Weighted Average Cost of Capital: 14.75%
Beta coefficient: 1.66
Common shares outstanding: 1,321,000
Using this information, what are the earnings per share (EPS) for Questron Media?

A. $1.26
B. $1.47
C. $0.66
D. $0.76
E. $0.78
F. $0.89

Answer: D

Explanation:
The EPS figure is perhaps the single most popular term in the field of conventional equity
investments. Any glance into financial media and business periodicals will undoubtedly uncover
numerous instances in which the EPS figure is cited. While quite popular and useful, many investors
and business professionals do not truly understand the mechanics behind the EPS calculation, and an
investigation into the components of EPS figure is a valuable learning experience. The EPS calculation
is found by the following equation: {EPS = [(Sales - Fixed Costs - Variable Costs - Interest Expense)(1 -
Tax Rate)] / [# of Common Shares Outstanding]}
Additionally, the EPS figure can be found by:
{EPS = [(EBIT - Interest Expense)(1 - Tax Rate) / # of Common Shares Outstanding]} Incorporating the
given information into the first EPS equation will yield the following: {EPS = [($3,000,000 -
$1,000,000 - $300,000 - $150,000)(1 - 0.35)] / 1,321,000} = $0.7627

Question: 2184

Page | 799
Dick Boe Enterprises, an all-equity firm, has a corporate beta coefficient of 1.5. The financial
manager is evaluating a project with an IRR of 21 percent, before any risk adjustment. The risk-free
rate is 10 percent, and the required rate of return on the market is 16 percent. The project being
evaluated is riskier than Boe's average project, in terms of both beta risk and total risk. Which of the
following statements is most correct?

A. Riskier-than-average projects should have their IRRs increased to reflect their added riskiness.
Clearly, this would make the project acceptable regardless of the amount of the adjustment.
B. The accept/reject decision depends on the risk-adjustment policy of the firm. If the firm's policy
were to reduce a riskier-than-average project's IRR by 1 percentage point, then the project should be
accepted.
C. The project should be accepted since its IRR (before risk adjustment) is greater than its required
return.
D. The project should be rejected since its IRR (before risk adjustment) is less than it's required
return.
E. Projects should be evaluated on the basis of their total risk alone. Thus, there is insufficient
information in the problem to make an accept/reject decision.

Answer: B

Explanation:
k(s) = 10% + (16% - 10%)1.5 = 10% + 9% = 19%.
Original IRR = 21%. 21% - Risk adjustment 1% = 20%.
Risk adjusted IRR = 20% > k(s) = 19%.

Question: 2185

Consider the following information:


30-day treasury rate (Risk Free rate) 7.2%
Company XYZ Bond yield 10.2%
Beta 0.8
Risk Premium 4.0%
Credit Rating B-
Calculate Company XYZ's cost of retained earnings using the Bond-Yield-plus-Risk-Premium
approach.

A. 15.2%
B. 12.2%
C. 11.36%
D. 17.4%
E. 5.2%
F. 14.2%

Answer: F

Explanation:
To estimate a firm's cost of retained earnings using the Bond-Yield-plus-Risk-Premium approach,
simply take the company's bond yield and add the risk premium. In this case the cost of retained
earnings = 10.2% + 4.0% = 14.2%.

Question: 2186

J. Ross and Sons Inc. has a target capital structure that calls for 40 percent debt, 10 percent preferred
stock, and 50 percent common equity. The firm's current after-tax cost of debt is 6 percent, and it can

Page | 800
sell as much debt as it wishes at this rate. The firm's preferred stock currently sells for $90 a share
and pays a dividend of $10 per share; however, the firm will net only $80 per share from the sale of
new preferred stock. Ross expects to retain $15,000 in earnings over the next year. Ross' common
stock currently sells for $40 per share, but the firm will net only $34 per share from the sale of new
common stock. The firm recently paid a dividend of $2 per share on its common stock, and investors
expect the dividend to grow indefinitely at a constant rate of 10 percent per year. What is the firm's
cost of newly issued common stock?

A. 16.5%
B. 18.0%
C. 10.0%
D. 12.5%
E. 15.5%

Answer: A

Explanation:
Cost of new common equity:
k(e) = ($2.20/$34) + 0.10 = 0.1647 = 16.5%.

Question: 2187

Clay Industries, a large industrial firm, is in the process of developing a coal refining system which
greatly increases the efficiency of coal as an energy source. However, the new system has been
criticized as leading to a tremendous increase in emissions of CFTA, a dangerous carbon-based
pollutant believed to be linked to thyroid cancer. While the firm is concerned about the possible risk
to the public posed by the new system, the management of Clay Industries decides that the sales
potential for the product outweighs both the risk to society and the liability exposure of the firm.
Which of the following choices best describes this situation faced by Clay Industries?

A. Opportunity cost problem


B. Diminishing returns problem
C. Cannibalization problem
D. Positive externality
E. Negative externality
F. Principal/agent problem

Answer: E

Explanation:
In this situation, a negative externality exists for Clay Industries and the development of the new coal
refining system. A negative externality is some detrimental effect which is to result on stakeholders
from the acceptance of a project. While often difficult to quantify, externalities are an important
consideration in the accept/reject decision in capital projects. Externalities can be both positive and
negative.

Question: 2188

Which of the following figures is not expressly incorporated into the Degree of Operating Leverage,
as based on the "unit sales" calculation.

A. Average sales price


B. Total variable operating costs per unit
C. Price

Page | 801
D. Common shares outstanding
E. Total fixed operating costs
F. Sales in units

Answer: D

Explanation:
The Degree of Operating Leverage (DOL) measures the percentage change in EBIT that results from a
given change in sales. The DOL can be calculated using several methods, including one that is based
on unit sales. This version of the DOL equation is as follows: {DOL = [(Sales in units(average sales
price - variable cost per unit) / (sales in units(average sales price - variable cost per unit) - total fixed
operating costs]}. Of the choices listed, only the number of common shares outstanding is not
incorporated into the DTL equation. In fact, the number of common shares outstanding is not
factored into any of the equations used to calculate DOL.

Question: 2189

The additional risk associated with a firm's earnings when it uses debt capital is known as

A. business risk.
B. systematic risk.
C. capital market risk.
D. financial risk.

Answer: D

Explanation:
Financial risk is the additional risk associated with a firm's earnings when it uses debt capital.

Question: 2190

Which of the following statements is most incorrect?

A. All of these answers are correct.


B. If the after-tax cost of equity financing exceeds the after-tax cost of debt financing, firms are
always able to reduce their WACC by increasing the amount of debt in their capital structure.
C. The optimal capital structure minimizes the WACC.
D. None of these answers are correct.
E. Increasing the amount of debt in a firm's capital structure is likely to increase the cost of both debt
and equity financing.

Answer: B

Explanation:
This statement is not always true.

Question: 2191

Returns on the market and Company Y's stock during the last 3 years are shown below:
YearMarketCompany Y
1995-24%-22%
19961013
19972236
The risk-free rate is 5 percent, and the required return on the market is 11 percent. You are

Page | 802
considering a low-risk project whose market beta is 0.5 less than the company's overall corporate
beta. You finance only with equity, all of which comes from retained earnings. The project has a cost
of $500 million, and it is expected to provide cash flows of $100 million per year at the end of Years 1
through 5 and then $50 million per year at the end of Years 6 through 10. What is the project's NPV
(in millions of dollars)?

A. $7.10
B. $12.10
C. $9.26
D. $15.75
E. $10.42

Answer: E

Explanation:
Step 1 Run a regression to find the corporate beta. Market returns are the X-input values, while Y's
returns are the Y-input values. Beta is 1.2102.
Step 2 Find the project's estimated beta by subtracting 0.5 from the corporate beta. The project beta
is thus 1.2102 - 0.5 = 0.7102.
Step 3 Find the company's cost of equity, which is its WACC because it uses no debt: k(s) = WACC =
5% + (11% - 5%)0.7102 = 9.26%.
Step 4 Now find the project's NPV (inputs are in millions):
CF(0) = -500
CF(1-5) = 100
CF(6-10) = 50
I = 9.26%
Solve for NPV = $10.42 million.

Question: 2192

The management of Clay Industries have adhered to the following capital structure: 50% debt, 45%
common equity, and 5% perpetual preferred equity. The following information applies to the firm:
Before-tax cost of debt = 7.5%
Combined state/federal tax rate = 35%
Expected return on the market = 14.5%
Annual risk-free rate of return = 5.25%
Historical Beta coefficient of Clay Industries Common Stock = 1.15
Annual preferred dividend = $1.35
Preferred stock net offering price = $17.70
Expected annual common dividend = $0.45
Common stock price = $30.90
Expected growth rate = 11.75%
Subjective risk premium = 3.3%
Given this information, and using the Bond-Yield-plus-Risk-Premium approach to calculate the
component cost of common equity, what is the Weighted Average Cost of Capital for Clay Industries?

A. 15.03%
B. 9.97%
C. 8.762%
D. 7.70%
E. 7.30%
F. The WACC for Clay Industries cannot be calculated from the information.

Answer: D

Page | 803
Explanation:
The calculation of the Weighted Average Cost of Capital is as follows: {fraction of debt * [yield to
maturity on outstanding long-term debt][1-combined state/federal income tax rate]} + {fraction of
preferred stock * [annual dividend/net offering price]} + {fraction of common stock * cost of equity}.
The cost of common equity can be calculated using three methods, the Capital Asset Pricing Model
(CAPM), the Dividend-Yield-plus-Growth-Rate (or Discounted Cash Flow) approach, and the Bond-
Yield-plus-Risk-Premium approach. In this example, you are asked to calculate the cost of common
equity using the Bond-Yield-plus-Risk-Premium approach. To calculate the cost of equity using this
approach, take the yield to maturity on the firm's outstanding debt (7.5%) and add a subjective risk
premium (3.3%), which gives a cost of common equity of 10.8%. The after-tax cost of debt can be
found by multiplying the yield to maturity on the firm's outstanding long-term debt (7.5%) by (1-tax
rate). Using this method, the after-tax cost of debt is found as 4.875%. The calculation of the cost of
perpetual preferred stock is relatively straightforward, simply divide the annual preferred dividend
by the net offering price. Using this method, the cost of preferred stock is found as 7.627%.
Incorporating these figures into the WACC equation gives the answer of 7.679%.

Question: 2193

Which of the following statements is most correct?

A. All of these statements are false.


B. A break point is based on the dollar value used of a specific type of capital, and occurs at the point
where the cost of that capital type increases. Thus, if a firm has $100,000 in earnings, and
stockholders want $50,000 of those earnings paid as dividends, then retained earnings will have two
break points.
C. A firm facing a steep demand curve (that is, high flotation costs) for new equity would likely also
face, at some point, a steeply upward sloping WACC curve.
D. All of these statements are correct.
E. One purpose of calculating the WACC (Weighted Average Cost of Capital) is to have a singular cost
of capital measure that can be applied to evaluate all of the firm's projects, including those of greater
than and lesser than average risks.

Answer: C

Explanation:
Because of high flotation costs, dollars raised by selling new stock must work harder than dollars
raised by retaining earnings. Steep demand would result in a steeply upward sloping WACC curve.
Note, however, that for most firms, new equity issues are rare.

Question: 2194

Scott Corporation's new project calls for an investment of $10,000. It has an estimated life of 10
years. The IRR has been calculated to be 15 percent. If cash flows are evenly distributed and the tax
rate is 40 percent, what is the annual before-tax cash flow each year? (Assume depreciation is a
negligible amount.)

A. $1,500
B. $3,321
C. $5,019
D. $1,993
E. $4,983

Answer: B

Page | 804
Explanation:
X = after-tax cash flow.
Y = before-tax cash flow.
X = Y(1 - T).
$10,000 = X(PVIFA(15%,10))
$10,000 = X(5.0188)
X = $1,992.51.
$1,992.51 = Y(1 - 0.40)
Y = $3,320.85 = $3,321.

Question: 2195

Suppose the firm's WACC is stated in nominal terms, but the project's expected cash flows are
expressed in real dollars. In this situation, other things held constant, the calculated NPV would

A. possibly have a bias, but it could be upward or downward.


B. more information is needed; otherwise, we can make no reasonable statement.
C. be biased upward.
D. be biased downward.
E. be correct.

Answer: D

Explanation:
Given the fact that there is inflation, a cost of capital stated in nominal terms would understate the
calculated NPV. If inflation is expected, but this expectation is not built into the forecasted cash flows,
then the calculated NPV will be downward biased.

Question: 2196

Which of the following types of dividends, are never paid out in the form of cash?

A. All of these are paid in the form of cash.


B. Stock dividends.
C. Regular dividends.
D. Extra dividends.
E. Liquidating dividends.

Answer: B

Explanation:
Stock dividends are dividends paid in the form of additional shares of stock rather than in cash. The
total number of shares is increased, so earnings, dividends, and price per share all decline. Stock
dividends that are used on a regular basis will keep the stock price more or less constrained, that is,
within the optimal trading range.

Question: 2197

The post-audit is used to

A. eliminate potentially profitable but risky projects.


B. all of these answers are correct.
C. review cash flow forecasts.

Page | 805
D. stimulate management to improve operations and bring results into line with forecasts.
E. none of these answers are correct.

Answer: D

Explanation:
The two main purposes of the post-audit are to improve forecasts and improve operations.
Management is putting their reputations on the line when forecasting an investment, and they will
strive to improve operations to bring results into line with forecasts.

Question: 2198

Which of the following is most correct?

A. Conflicts between NPV and IRR rules arise in choosing between two mutually exclusive projects
(that each have normal cash flows) when the cost of capital exceeds the crossover point (that is, the
point at which the NPV profiles cross).
B. None of the statements are correct.
C. The discounted payback method overcomes the problems that the payback method has with cash
flows occurring after the payback period.
D. The NPV and IRR rules will always lead to the same decision in choosing between mutually
exclusive projects, unless one or both of the projects are "non-normal" in the sense of having only
one change of sign in the cash flow stream.
E. The Modified Internal Rate of Return (MIRR) compounds cash outflows at the cost of capital.

Answer: B

Explanation:
IRR can lead to conflicting decisions with NPV even with normal cash flows if the projects are
mutually exclusive. Cash outflows are discounted at the cost of capital with the MIRR method, while
cash inflows are compounded at the cost of capital. Conflicts between NPV and IRR arise when the
cost of capital is below the crossover point. The discounted payback method does correct the
problem of ignoring the time value of money, but it still does not account for cash flows beyond the
payback period.

Question: 2199

Quick Launch Rocket Company, a satellite launching firm, expects its sales to increase by 50 percent
in the coming year as a result of NASA's recent problems with the space shuttle. The firm's current
EPS is $3.25. Its degree of operating leverage is 1.6, while its degree of financial leverage is 2.1. What
is the firm's projected EPS for the coming year using the DTL approach?

A. $3.25
B. $5.46
C. $19.63
D. $10.92
E. $8.71

Answer: E

Explanation:
EPS(1) = EPS(0) + EPS(0) [DTL x (percent change in sales)]
= $3.25 [1 + (1.6)(2.1)(0.5)] = $3.25 [2.68]
EPS(1) = $8.71.

Page | 806
Question: 2200

Calculate the cost of debt for the following firm:


Borrowing Rate 10%
Marginal Tax Rate 40%
Project IRR 12.5%
Owner's Equity 15%

A. 1.5%
B. 6%
C. 60%
D. 27.5%
E. 10%

Answer: B

Explanation:
The cost of debt is simply the rate of borrowing less the tax savings. Due to the fact that interest
expense is tax deductible, the cost of debt in this case is 10%(1 - .4) = 10%(.6) = 6%.

Question: 2201

Firms A and B have the same fixed costs in producing widgets. However, firm A charges 15% more
than firm B for a widget while its variable costs per widget are 12% lower than those of B. If firm A
sells a widget for 35% above its variable costs, the break-even point for B is ________ times higher
than that for A.

A. 2.0
B. 9.2
C. 3.3
D. 2.5

Answer: B

Explanation:
The break-even quantity, Q, is given by Q = FC/(P - V), where FC = total fixed costs, P = average sale
price per unit and V = average variable cost per unit.
You're given that FCA = FCB, PA = 1.35VA, PA = 1.15PB and VA = 0.88VB. Therefore, PB =
(1.35/1.15)VA = 1.174VA and VB = (1/0.88)VA = 1.136V
A.
This gives QA/QB = FC(PA-VA)/FC(PB-VB) = (PB - VB)/(PA - VA) = (1.174 - 1.136)/(1.35 - 1) = 0.109.
Thus, the break-even point for B is (1/0.109) = 9.21 times that for
A.

Question: 2202

Assume you are the director of capital budgeting for an all-equity firm. The firm's current cost of
equity is 16 percent; the risk-free rate is 10 percent; and the market risk premium is 5 percent. You
areconsidering a new project that has 50 percent more beta risk than your firm's assets currently
have, i.e., its beta is 50 percent larger than the firm's existing beta. The expected return (IRR) on the
new project is 18 percent. Should the project be accepted if beta risk is the appropriate risk
measure?

Page | 807
A. Yes; its IRR is greater than the firm's cost of capital.
B. No; a 50 percent increase in beta risk gives a risk-adjusted required return of 24 percent.
C. No; the project's risk-adjusted required return is 1 percentage point above its IRR.
D. Yes; the project's risk-adjusted required return is less than its IRR.
E. No; the project's risk-adjusted required return is 2 percentage points above its IRR.

Answer: C

Explanation:
Calculate the beta of the firm, and use to calculate project beta:
k(s) = 0.16 = 0.10 + (0.05)b. b = 1.2.
b(Project) = b(Firm)1.5 b(Project) is 50% greater than current b(Firm)
b(Project) = (1.2)1.5 = 1.8.
Calculate required return on project, k(Project), and compare to IRR.
Project: k(Project) = 0.10 + (0.05)1.8 = 0.19 = 19%. IRR = 0.18 = 18%.
Since the required return is one percentage point greater than the expected IRR, the firm should not
accept the new project.

Question: 2203

Ace Consulting, a multinational corporate finance consulting firm, is analyzing the profitability of a
new line of superconductors designed by Clay Industries, a large industrial firm. In their analysis, Ace
Consulting has developed a detailed statistical model that generates random values for key variables,
and these random numbers are incorporated into the analysis. Using this proprietary statistical
software, Ace Consulting is allowed to formulate a computer-based model of the
superconductor'sexpected cash flows and NPV, given any randomly selected value for seven essential
variables. Which of the following choices best describes this technique for measuring stand-alone
risk?

A. Relational computation analysis


B. Monte Carlo simulation
C. Sensitivity analysis
D. Scenario analysis
E. Regression analysis
F. Marco Polo simulation

Answer: B

Explanation:
In this example, Ace Consulting has developed a statistical model which generates random values for
seven key variables. Using this information, Ace can provide Clay Industries with an expected range
of NPV and IRR for any assumed variable values. This process is referred to as Monte Carlo
simulation, and is so named because the first Monte Carlo models were incorporated into the
mathematical analysis of Casino gambling.

Question: 2204

Cepeda Corporation requires a computer system for the next ten years, and is in the process of
choosing among two mutually exclusive alternatives. System A costs $50,000 today, and will produce
positive net cash flows of $12,000 a year for the next ten years (t = 1 through t = 10). System B costs
$30,000 today and will produce positive net cash flows of $11,000 a year for the next five years. After
five years, System B can be replaced under the same terms. The company's cost of capital is 10
percent. What is the equivalent annual annuity (EAA) of the best system?

Page | 808
A. $6,261.18
B. $3,862.73
C. $5,002.39
D. $3,086.07
E. $2,373.48

Answer: B

Explanation:
First find the NPV's of each system over its initial life.
System A: CF(0) = -50,000; CF(1-10)= 12,000; I = 10; solve for NPV = $23,734.81. System B: CF(0) = -
30,000; CF(1-5)= 11,000; I = 10, solve for NPV = $11,698.65. Second, find the value of the EAA of each
system.
System A: N = 10; I = 10; PV = -23,734.81; FV = 0; solve for PMT = EAA = $3,862.73. System B: N = 5; I
= 10; PV = -11,698.65; FV = 0; solve for PMT = EAA = $3,086.07. Given System A has a higher EAA, it is
the better of the two systems.

Question: 2205

Rollins Corporation is constructing its MCC schedule. Its target capital structure is 20 percent debt, 20
percent preferred stock, and 60 percent common equity. Its bonds have a 12 percent coupon, paid
semiannually, a current maturity of 20 years, and sell for $1,000. The firm could sell, at par, $100
preferred stock, which pays a 12 percent annual dividend, but flotation costs of 5 percent would be
incurred. Rollins' beta is 1.2, the risk-free rate is 10 percent, and the market risk premium is 5
percent. Rollins is a constant growth firm, which just paid a dividend of $2.00, sells for $27.00 per
share, and has a growth rate of 8 percent.
The firm's policy is to use a risk premium of 4 percentage points when using the bond-yield-plus-risk-
premium method to find k(s). The firm's net income is expected to be $1 million, and its dividend
payout ratio is 40 percent. Flotation costs on new common stock total 10 percent, and the firm's
marginal tax rate is 40 percent.
What is Rollins' retained earnings break point?

A. $800,000
B. $1,000,000
C. $1,200,000
D. $1,400,000
E. $600,000

Answer: B

Explanation:
Retained earnings = 0.6($1,000,000) = $600,000.
BP(RE) = $600,000/0.6 = $1,000,000.

Question: 2206

A firm's dividend growth rate is 3.2% when the dividend payout ratio equals 37%. It is expected to
pay a dividend of $2.2 next year. If the cost of external equity for the firm equals 19.2% and the firm's
stock is currently priced at $14.1, the flotation cost of equity equals ________.

A. 1.78%
B. 0.89%
C. 2.50%
D. 1.91%

Page | 809
Answer: C

Explanation:
IF F is the percentage flotation cost and P is the amount of new equity raised per new share, then Ke
= D1/[P(1-F)] + g, where Ke is the cost of external equity. Here, g = 3.2%, D1 = $2.2, P = $14.1 and Ke
= 19.2%. Therefore, 19.2% = 2.2/(14.1*(1-F)) + 3.2%. Solving for F gives F = 2.5%.

Question: 2207

The management of Clay Industries have adhered to the following capital structure: 50% debt, 35%
common equity, and 15% perpetual preferred equity. The following information applies to the firm:
Before-tax cost of debt, i.e. yield to maturity of the outstanding senior long-term debt = 9.5%
Combined State/Federal tax rate = 35%
Cost of common equity = 14.45%
Annual preferred dividend = $2.75
Preferred stock net offering price = $28.50
Given this information, what is the Weighted Average Cost of Capital for Clay Industries?

A. 9.60%
B. 10.45%
C. The WACC for Clay Industries cannot be calculated from the information given.
D. 11.27%
E. 6.52%
F. 8.67%That answer is correct!

Answer: A

Explanation:
The calculation of the Weighted Average Cost of Capital is as follows: {fraction of debt * [yield to
maturity of outstanding long-term debt][1-combined state/federal income tax rate]} + {fraction of
preferred stock * [annual dividend/net offering price]} + {fraction of common stock * cost of equity}.
The cost of common equity can be calculated using three methods, the Capital Asset Pricing Model
(CAPM), the Dividend-Yield-plus-Growth-Rate (or Discounted Cash Flow) approach, and the Bond-
Yield-plus-Risk-Premium approach. In this example, the cost of equity is given, so none of the three
approaches is necessary. However, the cost of debt and preferred stock must be calculated. The cost
of debt is found by multiplying the before tax cost of debt (9.5%) by (1-tax rate). Incorporating the
given figures into this equation will yield a cost of debt at 6.175%. Determining the cost of perpetual
preferred stock is relatively straightforward, simply divide the annual preferred dividend ($2.75) by
the net price of preferred stock ($28.50), which yields a cost of preferred stock of 9.65%. These
figures can now be incorporated into the WACC equation, which is provided below: {[50% debt *
9.5% * (1- 35%)] + [15% * ($2.75/$28.50)] + [35% * 14.45%]} = 9.60%

Question: 2208

Which of the following is not expressly incorporated into the Degree of Total Leverage (DTL)
calculation?

A. None of these answers


B. Discount rate
C. Interest expense
D. Fixed costs
E. Sales
F. Variable costs

Page | 810
Answer: B

Explanation:
The Degree of Total Leverage (DTL) calculation measures the percentage change in EPS from a given
percentage change in sales. The equation used to produce DTL is as follows: {DTL = [(Sales - Variable
Costs) / (Sales - Variable Costs - Fixed Costs - Interest Expense)]. As you can see, the DTL calculation
does not involve the use of an explicit discount rate.

Question: 2209

Company D has a 50 percent debt ratio, whereas Company E has no debt financing. The two
companies have the same level of sales, and the same degree of operating leverage. Which of the
following statements is most correct?

A. None of these answers are correct.


B. If sales increase 10 percent for both companies, then Company D will have a larger percentage
increase in its operating income (EBIT).
C. All of these answers are correct.
D. If EBIT increases 10 percent for both companies, then Company D's net income will rise by more
than 10 percent, while Company E's net income will rise by less than 10 percent.
E. If sales increase 10 percent for both companies, then Company D will have a larger percentage
increase in its net income.

Answer: E

Explanation:
After the sales increase, the percentage increase in EBIT will be the same for both companies.
Company E's net income will rise by exactly 10%.

Question: 2210

Which of the following statements is most correct?

A. The optimal capital structure is the one that maximizes EBIT, and this always calls for a debt ratio,
which is lower than the one that maximizes expected EPS.
B. When financial leverage is used, the graphical probability distribution of net income would tend to
be more peaked than a distribution where no leverage is present, other things held constant.
C. From an operational standpoint the goal of maintaining financial flexibility translates into
maintaining adequate reserve borrowing capacity.
D. While business risk varies from one industry to another and can change over time, it affects all
firms equally within a particular industry.
E. All of these statements are false.

Answer: C

Explanation:
Even in normal times, a firm should maintain a reserve borrowing capacity, which is the ability to
borrow money at a reasonable cost when good investment opportunities arise.

Question: 2211

A firm is considering the purchase of an asset whose risk is greater than the current risk of the firm,
based on any method for assessing risk. In evaluating this asset, the decision-maker should

Page | 811
A. Increase the NPV of the asset to reflect the greater risk.
B. Ignore the risk differential if the asset to be accepted would comprise only a small fraction of the
total assets of the firm.
C. Reject the asset, since its acceptance would increase the risk of the firm.
D. Increase the IRR of the asset to reflect the greater risk.
E. Increase the cost of capital used to evaluate the project to reflect the higher risk of the project.

Answer: E

Explanation:
An increase in a project's beta will cause its stock price to decrease unless the increased beta were
offset by a higher expected rate of return. Therefore, high-risk investments require higher rates of
return, whereas low-risk investments require lower rates of return.

Question: 2212

Lascheid Enterprises is an all-equity firm with 175,000 shares outstanding. The company's stock price
is currently $80 a share. The company's EBIT is $2,000,000 and is expected to remain constant over
time. The company pays out all of its earning each year--so its earnings per share equals its dividends
per share. The firm's tax rate is 30 percent. The company is considering issuing $800,000 worth of
bonds and using the proceeds for a stock repurchase. If issued the bonds would have an estimated
yield to maturity of 8 percent. The risk-free rate is 5 percent and the market risk premium is also 5
percent. The company's beta is currently 1.0, but its investment banker's estimate that the
company's beta would rise to 1.2 if they proceeded with the recapitalization. What would be the
company's stock price following the repurchase transaction.

A. $102.63
B. $106.67
C. $77.14
D. $74.67
E. $70.40

Answer: D

Explanation:
The bonds used in the repurchase will create a new interest expense for the company. This will
change net income. Dividends per share will change because net income changes and the number of
shares outstanding changes.
New interest expense: $800,000 x 8% = $64,000.
New net income: ($2,000,000 - $64,000)(1 - 0.3) = $1,355,200.
Shares repurchased: $800,000/80 = 10,000 shares.
New shares outstanding: 175,000 - 10,000 = 165,000 shares.
New dividends per share: $1,355,200/165,000 = $8.21.
We must also calculate a new cost of equity: 5% + (5%)1.2 = 11%.
New stock price: $8.21/11% = $74.67.

Question: 2213

Driver Corporation faces an IOS schedule calling for a capital budget of $60 million. Its optimal capital
structure is 60 percent equity and 40 percent debt. Its earnings before interest and taxes (EBIT) were
$98 million for the year. The firm has $200 million in assets, pays an average of 10 percent on all its
debt, and faces a marginal tax rate of 35 percent. If the firm maintains a residual dividend policy and
will keep its optimal capital structure intact, what will be the amount of the dividends it pays out

Page | 812
after financing its capital budget?

A. $30.0 million
B. $59.4 million
C. $22.5 million
D. $60.0 million
E. $0

Answer: C

Explanation:
Calculate interest cost:
Total assets = $200M; 40% debt x $200M = $80 million in debt.
Interest cost = $80M x 0.10 = $8.0 million.
Calculate net income (in millions):
EBIT$98.0
less: Interest- 8.0
EBT$90.0
less: Taxes (35%) 31.5
Net income$58.5
Calculate portion of projects financed with retained earnings:
IOS contains $60 million in positive NPV projects.
Retained earnings portion:$60M x 0.60 = $36.0 million
Debt portion:$60M x 0.40 = $24.0 million
Calculate residual available for dividends:
$58.5M - $36.0M = $22.5 million in dividends.

Question: 2214

Texas Products Inc. has a division, which makes burlap bags for the citrus industry. The unit has fixed
costs of $10,000 per month, and it expects to sell 42,000 bags per month. If the variable cost per bag
is $2.00, what price must the division charge in order to break even?

A. $2.47
B. $3.15
C. $2.24
D. $2.00
E. $2.82

Answer: C

Explanation:
Total costs = $10,000 + $2(42,000) = $94,000.
Price = $94,000/42,000 = $2.24.

Question: 2215

Market risk in a revenue-producing project can best be adjusted for by

A. Ignoring it.
B. Adjusting the discount rate downward for increasing risk.
C. Picking a risk factor equal to the average discount rate.
D. Adjusting the discount rate upward for increasing risk.
E. Reducing the NPV by 10 percent for risky projects.

Page | 813
Answer: D

Explanation:
An increase in a project's beta will cause its stock price to decrease unless the increased beta were
offset by a higher expected rate of return. Therefore, high-risk investments require higher rates of
return, whereas low-risk investments require lower rates of return.

Question: 2216

Normal projects C and D are mutually exclusive. Project C has a higher net present value if the WACC
is less than 12 percent, whereas Project D has a higher net present value if the WACC exceeds 12
percent. Which of the following statements is most correct?

A. None of the answers are correct.


B. Project C probably has a faster payback.
C. Project D is probably larger in scale than Project C.
D. Project D has a higher internal rate of return.
E. All of the statements are correct.

Answer: D

Explanation:
In this situation, D has a flatter NPV profile and a higher IRR. Project acceptance depends on what is
the correct discount rate. If the discount rate is less than 12%, accept C. If it is greater than 12%,
accept D. Projects that return their cash flows early (faster payback) and have lower initial
investments tend to have higher IRRs.

Question: 2217

Which of the following statements is correct?

A. If you are choosing between two projects which have the same life, and if their NPV profiles cross,
then the smaller project will probably be the one with the steeper NPV profile.
B. If the cost of capital is relatively high, this will favor larger, longer-term projects over smaller,
shorterterm alternatives because it is good to earn high rates on larger amounts over longer periods.
C. If the cost of capital is less than the crossover rate for two mutually exclusive projects' NPV
profiles, a NPV/IRR conflict will not occur.
D. Because discounted payback takes account of the cost of capital, a project's discounted payback is
normally shorter than its regular payback.
E. The NPV and IRR methods use the same basic equation, but in the NPV method the discount rate
is specified and the equation is solved for NPV, while in the IRR method the NPV is set equal to zero
and the discount rate is found.

Answer: E

Explanation:
This statement reflects exactly the difference between the NPV and IRR methods.

Question: 2218

When a mature firm raises the dividend, signaling theory implies that its stock price ________. When
a growth firm cuts the dividend, signaling theory implies that its stock price ________.

Page | 814
A. will fall; may rise or fall
B. will rise; will fall
C. will fall; will rise
D. will rise; may rise or fall

Answer: D

Explanation:
The signaling theory is properly applicable only to mature firms which have had stable dividend
policies. In its pure form, the theory regards dividend changes as signals of management's forecasts
of future earnings. Such an assumption is not fully justifiable for young, growth firms, which may cut
dividendssimply to supply retained earnings capital for expansion projects, without any signaling
about the firm's future earnings prospects. Indeed, many growth firms pay no dividends at all for
quite some time without an adverse effect on their stock prices.
Hence, the increase in the dividend of a mature firm is taken as a signal by investors - under the
signaling hypothesis - that the management's forecasts of future earnings are quite favorable,
leading to a rise in the stock price. On the other hand, for a growth firm, such a signaling conclusion
does not necessarily hold.

Question: 2219

Which of the following projects is likely to have multiple Modified Internal Rates of Return. Assume a
14.5% cost of capital.
Project A
Initial investment outlay: ($1,000,000)
t1: $0.00
t2: $0.00
t3: $0.00
t4: $0.00
t5: $0.00
t6: $10,000,000
Project B
Initial investment outlay: ($1,000,000)
t1: $500,000
t2: $500,000
t3: $500,000
t4: $0.01
Project C
Initial investment outlay: ($1,000,000)
t1: $800,000
t2: ($100,000)
t3: $550,000
Project D
Initial investment outlay: ($500,000)
t1: $400,000
t2: ($1,000)
t3: $230,000
t4: ($50,000)

A. Project D
B. More than one of these answers are correct
C. None of these answers is correct
D. Project A
E. Project B

Page | 815
F. Project C

Answer: C

Explanation:
Remember that the Modified Internal Rate of Return method will not produce multiple answers for
nonnormal projects. The fact that MIRR will not produce multiple answers for non-normal projects is
one of the reasons that this method should be considered as superior to the traditional Internal Rate
of Return method.

Question: 2220

Shelby Inc. is considering two projects which have the following cash flows:
Project 1Project 2
Time Cash Flows Cash Flows
0-$2,000-$1,900
15001,100
2700900
3800800
41,000600
51,100400
At what cost of capital would the two projects have the same net present value?

A. 5.98%
B. 5.85%
C. 6.40%
D. 6.70%
E. 4.73%

Answer: B

Explanation:
Subtract Project 2 cash flows from Project 1 cash flows:
CF(0) = -100
CF(1) = -600
CF(2) = -200
CF(3) = 0
CF(4) = 400
CF(5) = 700
Put these in the cash flow register and solve for the IRR = 5.85%.

Question: 2221

Which of the following factors directly influence capital structure decisions?


I. Business risk
II. Availability of various sources of capital under attractive terms
III. Expropriation risk
IV. The firm's tax position
V. Management's subjective attitudes toward risk
VI. Country risk

A. II, III, IV, V


B. I, II, III, IV, V, VI
C. I, III, IV

Page | 816
D. I, II, IV, V
E. I, II, II, IV, VI

Answer: D

Explanation:
There are four primary factors which influence capital structure decisions: business risk, financial
flexibility, the firm's tax position, and management's subjective attitudes toward risk. Business risk is
defined as the riskiness of a firm if it uses no debt. Financial flexibility refers to the ability of a
company to easily raise various sources of capital under favorable terms. "Expropriation risk," and
"country risk," while legitimate forms of risk, are not directly applicable to the capital structure
decision. Expropriation risk is defined as the risk that a firm's existing assets and facilities will be
seized, or "expropriated" by a governmental, social, or military entity. This risk is frequently
incorporated into discussions of international operations.

Question: 2222

Which of the following statements is correct?

A. The bond-yield-plus-risk-premium approach to estimating the cost of equity is not always accurate
but its advantages are that it is a standardized and objective model.
B. Although some methods of estimating the cost of capital encounter severe difficulties, the CAPM
(Capital Asset Pricing Model) is a simple and reliable model that provides great accuracy and
consistency in estimating the cost of capital.
C. Depreciation-generated funds are an additional source of capital and, in fact, represent the largest
single source of funds for some firms.
D. The DCF (Discounted Cash Flow) model is preferred over other models to estimate the cost of
equity because of the ease with which a firm's growth rate is obtained.

Answer: C

Explanation:
Since depreciation is a significant non-cash expense, it is added back to net income when calculating
cash flow.

Question: 2223

The Global Advertising Company had net income after interest but before taxes of $40,000 this year.
The marginal tax rate is 40 percent, and the dividend payout ratio is 30 percent. The company can
raise debt at a 12 percent interest rate. The last dividend paid by Global was $0.90. Global's common
stock is selling for $8.59 per share, and its expected growth rate in earnings and dividends is 5
percent. If Global issues new common stock, the flotation cost incurred will be 10 percent. Global
plans to finance all capital expenditures with 30 percent debt and 70 percent equity. What is the cost
of common equity raised by selling new stock?

A. 10.33%
B. 12.22%
C. 16.00%
D. 17.22%
E. 9.66%

Answer: D

Explanation:

Page | 817
k(e) (component cost of external equity) = $.945/$8.59 (1-.10) + 0.05 = 0.1722 = 17.22%.

Question: 2224

Which of the following statements is most correct?

A. If Congress cuts the capital gains rate, but leaves the personal tax rate unchanged, then this would
provide an incentive for companies to increase their dividend payouts.
B. If a firm follows a residual dividend policy, then a sudden increase in the number of profitable
projects is likely to reduce the firm's dividend payout.
C. None of these answers are correct.
D. Despite its drawbacks, a residual dividend policy is an effective way to stabilize dividend payouts,
which makes it easier for firms to attract a clientele which prefers high dividends.
E. All of these answers are correct.

Answer: B

Explanation:
The residual dividend model is a model in which the dividend paid is set equal to the actual earnings
minus the amount of retained earnings necessary to finance the firm's optimal capital budget.
Theresidual dividend policy minimizes the costs to the company of raising outside funds, but it does
not provide a stable cash flow to the investors and most investors prefer stable dividends.

Question: 2225

Which of the following statements is most correct?

A. If a company undertakes a 3-for-1 stock split, then the number of shares outstanding should fall,
and the stock price should rise.
B. All of these answers are correct.
C. None of these answers are correct.
D. If a company wants to issue new shares of common stock and also wants to implement a dividend
reinvestment plan, then it should implement a new-stock dividend reinvestment plan, rather than an
open-market purchase plan.
E. If a company wants to reduce its debt ratio, then it should repurchase some of its common stock.

Answer: D

Explanation:
The new stock type of dividend reinvestment plan invests the dividends in newly issued stock, hence
these plans raise new capital for the firm.

Question: 2226

McCarver Inc. is considering the following mutually exclusive projects:


Project AProject B
TimeCash FlowCash Flow
0-$5,000-$5,000
12003,000
28003,000
33,000800
45,000200
At what cost of capital will the net present value of the two projects be the same?

Page | 818
A. 16.15%
B. 17.72%
C. 17.80%
D. 15.68%
E. 16.25%

Answer: A

Explanation:
Find the differences between the two projects' respective cash flows as follows:
(Project A CF - Project B CF).
CF(0) = -5,000 - (-5,000) = 0.
CF(1) = 200 - 3,000 = -2,800.
CF(2) = -2,200.
CF(3) = 2,200.
CF(4) = 4,800.
Enter these CFs and find the IRR = 16.15% which is the crossover rate.

Question: 2227

Clay Industries, a large industrial firm, is evaluating the sales of its existing line of coiled machine
tubing. In their analysis, the operating managers of Clay Industries have identified the following
information related to the coiled machine tubing division and its product:
Annual fixed operating expenses of $925,000
Average selling price of $90
Average variable cost of $44
Which of the following best describes the breakeven quantity for this product?

A. 20,109 units
B. The breakeven quantity cannot be determined from the information provided.
C. None of these answers is correct.
D. 20,468 units
E. 10,278 units
F. 21,023 units

Answer: A

Explanation:
To calculate the breakeven quantity for a product, use the following equation: {Fixed operating
costs/[avg. sales price per unit - variable cost per unit]}. The determination of the breakeven quantity
for this product is relatively straightforward, and does not involve any algebraic manipulation of the
original equation. Incorporating the given information into the equation yields the following:
{$925,000/[$90 - $44]} = 20,109 units.

Question: 2228

From the information below, select the optimal capital structure for Minnow Entertainment
Company.

A. Debt = 80%; Equity = 20%; EPS = $3.42; Stock price = $30.40.


B. Debt = 60%; Equity = 40%; EPS = $3.18; Stock price = $31.20.
C. Debt = 70%; Equity = 30%; EPS = $3.31; Stock price = $30.00.
D. Debt = 50%; Equity = 50%; EPS = $3.05; Stock price = $28.90.
E. Debt = 40%; Equity = 60%; EPS = $2.95; Stock price = $26.50.

Page | 819
Answer: B

Explanation:
Since the optimal capital structure is the one that maximizes the price of the firm's stock, and
minimizes the firm's WACC, this would be the optimal capital structure.

Question: 2229

Tapley Acquisition Inc. is considering the purchase of Target Company. The acquisition would require
an initial investment of $190,000, but Tapley's after-tax net cash flows would increase by $30,000 per
year and remain at this new level forever. Assume a cost of capital of 15 percent. Should Tapley buy
Target?

A. Yes, because the IRR < the cost of capital.


B. Yes, because the NPV = $10,000.
C. No, because NPV < 0.
D. No, because k > IRR.
E. Yes, because the NPV = $30,000.

Answer: B

Explanation:
NPV = $30,000/0.15 - $190,000 = $200,000 - $190,000 = $10,000.

Question: 2230

When Modigliani and Miller relaxed their assumption of zero taxes, they concluded which of the
following?

A. Firm's will wish to issue preferred stock.


B. A firm's value is maximized only when it uses no debt.
C. Business risk will become the main determinant of a firm's value.
D. Dividend policy will dominate the investment decision.
E. A firm's capital structure is irrelevant.
F. A firm's value is maximized only when it uses 100% debt.

Answer: F

Explanation:
When M&M relaxed their "no-taxes" assumption, they concluded that a firm's value was maximized
only when its capital structure is composed of 100% debt. This is due to the tax-deductibility of debt.
Due to these tax-shelter benefits, companies who are financed by 100% debt pass the maximum
amount of their operating income to investors, and this should increase the attractiveness of the firm
as an investment. M&M would be criticized on this finding, particularly because the risk of
bankruptcy was completely ignored. This criticism would give birth to the "Trade-off Theory of
Leverage," which states that companies will balance the tax-shelter benefits of debt with the
increased interest rates and risk of bankruptcy that are associated with increased debt levels..

Question: 2231

Which of the following statements is most correct?

A. All of the answers above are correct.

Page | 820
B. An increase in the personal tax rate is likely to increase the debt ratio of the average corporation.
C. A reduction in the corporate tax rate is likely to increase the debt ratio of the average corporation.
D. None of these answers are correct.
E. If changes in the bankruptcy code make bankruptcy less costly to corporations, then this would
likely reduce the debt ratio of the average corporation.

Answer: D

Explanation:
A major reason for using debt is that interest is deductible, which lowers the effective cost of debt.
Lowering the corporate tax rate reduces the tax advantages of debt leading firms to use less debt
financing. If the personal tax rate were to increase, individuals would now find interest received on
corporate debt less attractive, causing firms to utilize less debt financing. An increase in the costs of
bankruptcy might lead firms to use less debt in order to reduce the probability of having to incur
these higher costs.

Question: 2232

A stock has an expected dividend growth rate of 4.4%. The firm has just announced a dividend of
$1.9 per share, with an ex dividend date 3 days from now. Investors expect a rate of return of 9%
from the stock and the stock is trading at $31.84. Ignoring stock price uncertainty between now and
the ex dividend date and expecting the same growth, the stock is:

A. fairly priced.
B. under-priced.
C. insufficient information.
D. overpriced.

Answer: B

Explanation:
Ignoring stock price uncertainty between now and the ex dividend date, the stock would have a fair
price of Po on the ex dividend date, given by Po = D1/(r-g) = 1.9*1.044/(9% - 4.4%) = $43.12.
Therefore,the current fair price of the stock is $43.12 + $1.9 = $45.02, which is much greater than the
actual trading price of $31.84. Thus, the stock is under-priced.
Note that the operative phrase in this example is "ignoring all uncertainty between now and the ex
dividend date and expecting the same growth." If the market is expecting some negative information
about the firm to be released some time in the near future (not necessarily before ex dividend date)
or does not believe that the firm will continue to grow at or more than 4.4%, the actual trading price
could be a fair price. Hence, you should never blindly apply the DDM.

Question: 2233

J. Ross and Sons Inc. has a target capital structure that calls for 40 percent debt, 10 percent preferred
stock, and 50 percent common equity. The firm's current after-tax cost of debt is 6 percent, and it can
sell as much debt as it wishes at this rate. The firm's preferred stock currently sells for $90 a share
and pays a dividend of $10 per share; however, the firm will net only $80 per share from the sale of
new preferred stock. Ross expects to retain $15,000 in earnings over the next year. Ross' common
stock currently sells for $40 per share, but the firm will net only $34 per share from the sale of new
common stock. The firm recently paid a dividend of $2 per share on its common stock, and investors
expect the dividend to grow indefinitely at a constant rate of 10 percent per year. Where will a break
in the MCC schedule occur?

A. $20,000

Page | 821
B. $10,000
C. $42,000
D. There will be no breaks in the MCC schedule.
E. $30,000

Answer: E

Explanation:
Break point(RE) = $15,000/.50 = $30,000.

Question: 2234

A financial analyst with Mally, Feasance & Company is examining shares of Intelligent Semiconductor.
Assume the following information:
Retention rate: 80%
EPS: $3.98
Discount rate: 12.35%
Tax rate: 35%
Beta coefficient: 1.5
Expected return on the market: 12.5%
Using this information, what is the expected growth rate of Intelligent Semiconductor? Choose the
best answer.

A. 65.00%
B. The answer cannot be determined from the information provided.
C. 61.75%
D. None of these answers is correct.
E. 43.33%
F. 9.88%

Answer: B

Explanation:
In order to determine the dividend growth rate of Intelligent Semiconductor, the following equation
should be used:
{g = ROE(1 - Dividend Payout Ratio)}
While the Retention Rate of Dividends (which equals one minus the Dividend Payout Ratio) is
provided, the ROE figure is not. Further, the ROE figure cannot be determined from this information
provided. Therefore, the growth figure cannot be calculated. Remember that in determining
dividend growth using the formula outlined above, neither the tax rate nor the discount rate is
incorporated into theequation. Furthermore, the Beta Coefficient and the expected return on the
market are largely irrelevant in this example.

Question: 2235

The modified IRR (MIRR) is normally ________.

A. all of these answers are correct


B. none of these answers are correct
C. greater than the regular IRR if IRR > k
D. equal to the regular IRR if IRR < k
E. less than the regular IRR if IRR > k

Answer: E

Page | 822
Explanation:
MIRR assumes that cash flows from all projects are reinvested at the cost of capital, while the regular
IRR assumes that the cash flows from each project are reinvested at the project's own IRR. Therefore
the IRR will be greater than the MIRR if the IRR is greater than the cost of capital.

Question: 2236

An increase in the dividend payout ratio ________ the retained earnings break-point.

A. decreases
B. increases or decreases, depending on the tax rate
C. increases
D. does not affect

Answer: A

Explanation:
As the dividend payout ratio is increased, the amount of earnings retained decreases, decreasing the
amount of new debt that can be issued without changing the capital structure. Hence, the retained
earnings breakpoint will decrease.

Question: 2237

The management of Clay Industries have adhered to the following capital structure: 50% debt, 35%
common equity, and 15% preferred equity. The following information applies to the firm:
Before-tax cost of debt = 9.5%
Combined state/federal tax rate = 35%
Expected return on the market = 14.5%
Annual risk-free rate of return = 6.25%
Historical Beta coefficient of Clay Industries Common Stock = 1.24
Annual preferred dividend = $1.55
Preferred stock net offering price = $24.50
Expected annual common dividend = $0.80
Common stock price = $30.90
Expected growth rate = 9.75%
Subjective risk premium = 3.3%
Given this information, and using the Bond-Yield-plus-Risk-Premium approach to calculate the
component cost of common equity, what is the Weighted Average Cost of Capital for Clay Industries?

A. 9.79%
B. 8.36%
C. 9.82%
D. 6.93%
E. 8.52%
F. The WACC for Clay Industries cannot be calculated from the information.

Answer: E

Explanation:
The calculation of the Weighted Average Cost of Capital is as follows: {fraction of debt * [yield to
maturity of outstanding long-term debt][1-combined state/federal income tax rate]} + {fraction of
preferred stock * [annual dividend/net offering price]} + {fraction of common stock * cost of equity}.
The cost of common equity can be calculated using three methods, the Capital Asset Pricing Model

Page | 823
(CAPM), the Dividend-Yield-plus-Growth-Rate (or Discounted Cash Flow) approach, and the Bond-
Yield-plus-Risk-Premium approach. In this example, you are asked to calculate the cost of common
equity using the Bond-Yield-plus-Risk-Premium approach. To calculate the cost of equity using this
approach, take the yield to maturity on the firm's outstanding debt (9.5%) and add a subjective risk
premium (3.3%), which gives a cost of common equity of 12.8%. The after-tax cost of debt can be
found by multiplying the yield to maturity on the firm's outstanding long-term debt (9.5%) by (1-tax
rate). Using this method, the after-tax cost of debt is found as 6.175%. The calculation of the cost of
perpetual preferred stock is relatively straightforward, simply divide the annual preferred dividend
by the net offering price. Using this method, the cost of preferred stock is found as 6.327%.
Incorporating these figures into the WACC equation gives the answer of 8.516%.

Question: 2238

Consider the following information for Company ABC:


Current Price of Stock $40.25
Expected dividend in 1 Year $1.10
Growth rate 9.2%
Beta 1.2
Risk Free Rate 4.5%
Expected Market Return 10%
Calculate this company's cost of retained earnings using the Discounted Cash Flow (DCF) method.

A. 13.70%
B. 11.93%
C. 12.0%
D. 9.20%
E. 13.30%
F. 11.04%

Answer: B

Explanation:
The DCF method for estimating the cost of retained earnings states: Cost of Retained Earnings =
(Dividend for period 1 / Current Price) + Growth Rate. In this case the estimated Cost of Retained
Earnings = (1.1 / 40.25) + 9.2% = 2.73 + 9.2 = 11.93%

Question: 2239

Project A has a higher IRR than project B. Both projects have normal cash flows. If the projects have
the same cost of capital which is greater than the crossover rate,

A. Project A has a higher NPV.


B. Both projects have the same NPV.
C. Project B has a higher NPV.
D. Insufficient information.

Answer: A

Explanation:
The crossover rate is the discount rate at which the graphs of NPV versus discount rate for the two
projects cross. Since the projects have normal cash flows, they will have a single crossover rate.
Further, the project with the higher IRR has a "flatter" NPV profile. Therefore, if the cost of capital is
larger than the crossover rate, the project with the flatter profile will have a larger NPV.

Page | 824
Question: 2240

Steadybeta currently operates 3 projects, resulting in a beta of 1.27. It is considering a risky


expansion project whose cash flow analysis indicates a beta of 2.3. The project requires a capital
commitment of $4.8 million and has an NPV of $2 million. The current risk-free rate is 5.6% and the
market risk premium is 8.9%. Steadybeta's current market capitalization is $17.2 million. If
Steadybeta undertook the project, the required rate of return expected by its shareholders will be:

A. 14.8%
B. 13.9%
C. 19.5%
D. 16.2%

Answer: C

Explanation:
19.5% The firm can be considered a portfolio of 4 projects. The beta of a portfolio equals the
weighted average of the betas of the individual components. The weight of a component equals the
fraction of the market value it comprises. Since the project has an NPV of $2 million, its market value
equals $6.8 million and the market value after the project is undertaken will be $(17.2+6.8) = $24
million. Therefore, the beta of the firm after it undertakes the project equals 17.2/24*1.27 +
6.8/24*2.3 = 1.56. The required rate of return then equals 5.6% + 1.56*8.9% = 19.5%.

Question: 2241

Which of the following is the correct chronological order in dividend payment procedures?

A. Declaration date, record date, ex dividend date, dividend payment date.


B. Declaration date, dividend payment date, record date, ex dividend date.
C. Declaration date, ex dividend date, record date, dividend payment date.
D. Declaration date, ex dividend date, dividend payment date, record date.

Answer: C

Explanation:
In the U.S., the ex dividend date, i.e. the date after which the stock does not carry with it the right to
receive the declared dividend, is 4 business days before the record date. The record date is the last
day for registering the ownership of the stock with the firm so that the dividend check is mailed to
you and not someone else.

Question: 2242

Which of the following statements is most correct?

A. All of these statements are correct.


B. Stockholders pay no income tax on dividends reinvested in a dividend reinvestment plan.
C. Investors receiving stock dividends must pay taxes on the new shares at the time the stock
dividends are received.
D. None of these statements are correct.
E. "New-stock" dividend reinvestment plans are similar to stock dividends because they both
increase the number of shares outstanding but don't change the total equity of a firm.

Answer: D

Page | 825
Explanation:
Stock dividends are dividends paid in the form of additional shares of stock rather than in cash. The
total number of shares is increased, so earnings, dividends, and price per share all decline. In a
dividend reinvestment plan, the stockholder must pay taxes on the dividend amount, even though
stock and not cash has been received.

Question: 2243

The Residual Dividend Model is characterized as which of the following?

A. Dividend paid = EBIT (1 - combined state/federal tax rate) - preferred dividends - interest expense
B. Dividend paid = EBIT - Interest expense (1 - combined state/federal tax rate)
C. Dividend paid = EBIT - capital expenditures
D. None of these answers
E. Dividend paid = EBITA - proposed capital expenditures - interest expense
F. Dividend paid = EBIT - Retained earnings which are necessary to maintain the firm's optimal capital
budget

Answer: E

Explanation:
The Residual Dividend Model is characterized by a firm paying out all earnings to common
stockholders beyond that which is necessary to maintain the firm's optimal capital budget. Answer C
correctly illustrates the calculation of the dividend paid according to the Residual Dividend Model.

Question: 2244

10 months ago, a firm had leased a downtown office for $5,000 per month. The lease runs for the
next 2.5 years. The current office space of similar size rents at $4,000 per month. If the firm uses the
space exclusively for a project over the next 6 months, the opportunity cost related to this equals
________.

A. $5,000 per month


B. insufficient information
C. $4,000 per month
D. zero

Answer: C

Explanation:
Opportunity cost is based on current market cost. It does not matter what the firm originally paid or
agreed to pay for the property in question. Of course, since taxes are based on actual costs, the
aftertax opportunity cost is definitely affected by historical contracts currently in force.

Question: 2245

A firm's capital structure consists of 35% debt with an after-tax cost of 5.2%. Common equity makes
up 55% of the structure and the rest is made up of preferred equity. The preferred stock has a coupon
of 7% and the required rate of return on the common stock is 13.7%. The firm's WACC is ________.

A. 10.06%
B. 15.61%
C. none of these answers
D. 13.24%

Page | 826
E. 12.19%
F. 11.95%
G. 14.39%

Answer: C

Explanation:
To get the WACC in this case, you need to have information on the cost of preferred stock. This is not
necessarily equal to the coupon rate on the preferred equity. Rather, it is the discount rate that
equates the present value of the perpetual payments on the preferred equity to its current price.
Without the price information, you cannot get the cost of preferred equity and hence, WACC cannot
be calculated.

Question: 2246

The date on which a firm's directors issue a statement declaring a dividend is known as ________.

A. Ex-Dividend Date
B. Declaration Date
C. Payment Date
D. Dividend Date
E. Holder-of-Record Date

Answer: B

Explanation:
The date on which a statement is issued by a firm's directors declaring a dividend is known as the
"Declaration Date."

Question: 2247

The Target Copy Company is contemplating the replacement of its old printing machine with a new
model costing $60,000. The old machine, which originally cost $40,000, has 6 years of expected life
remaining and a current book value of $30,000 versus a current market value of $24,000. Target's
corporate tax rate is 40 percent. If Target sells the old machine at market value, what is the initial
aftertax outlay for the new printing machine?

A. -$22,180
B. -$36,000
C. -$30,000
D. -$33,600
E. -$40,000

Answer: D

Explanation:
Initial outlay
Cost of new machine-$60,000
Salvage value (old)+ 24,000
Tax effect of sale = $6,000(0.4) = + 2,400
After-tax outlay =-$33,600

Question: 2248

Page | 827
A project requires an initial outlay of 650. It also needs capital spending of 700 at the end of year 1
and 900 at the end of year 2. It has no revenues for the first 2 years but receives 1,200 in year 3,
1,600 in year 4 and 2,300 in year 5. The project's cost of capital is 10%. The project's NPV equals
________.

A. $2,043
B. $1,938
C. $1,428
D. $1,393

Answer: D

Explanation:
The discounted cash flow at the end of year N is obtained by dividing that year's cash flow by 1.1N,
since the project's cost of capital is 10%. Using this, the discounted cash flows are:
-636, -744, +902, +1,093, +1,428.
The Present value of the cash flows is = -636 - 744 + 902 + 1,093 + 1,428 = $2,043. The net present
value of the project = $(2,043 - 650) = $1,393.

Question: 2249

A project's break-even point is 1,235 units when the average sale price per unit is $35 and the
average variable cost equals $17.5 per unit. The fixed costs of the project are closest to ________.

A. $613
B. $21,612.5
C. none of these answers
D. $43,225

Answer: B

Explanation:
The break-even sales revenue equal 1,235*$35 = $43,225. The total variable costs equal $17.5*1,235
= $21,612.5. The fixed costs are therefore equal to $43,225 - $21,612.5 = $21,612.5.

Question: 2250

Which of the following companies has the highest degree of financial leverage? Choose the best
answer.
Firm A
EBIT: $10,000,000
Interest Paid: $750,000
Total Operating Expenses: $25,000,000
Fixed Operating Expenses: $19,750,000
Firm B
EBIT: $8,970,000
Interest Paid: $88,000
Total Operating Expenses: $20,050,000
Fixed Operating Expenses: $17,000,000
Firm C
EBIT: $10,500,000
Interest Paid: $1,050,000
Total Operating Expenses: $50,000,000
Fixed Operating Expenses: $35,000,000

Page | 828
Firm D
EBIT: $10,000,000
Interest Paid: $750,000
Total Operating Expenses: $50,000,000
Fixed Operating Expenses: $41,000,000
Firm E
EBIT: $5,195,000
Interest Paid: $400,000
Total Operating Expenses: $35,000,000
Fixed Operating Expenses: $9,875,000

A. Firm A
B. Firm E
C. Firm B
D. Firm C
E. Firm D
F. Firm A and D have identical DFL's

Answer: D

Explanation:
The Degree of Financial Leverage (DFL) measures the percentage change in EPS that results from a
given percentage change in EBIT. Financial Leverage is the second component of total leverage, along
with Operating Leverage. The equation used to calculate the Degree of Financial Leverage is as
follows: {DFL = [EBIT/(EBIT - Interest Paid)]}.
In this example, Firm C has the highest DFL, with a figure of 1.11. While Firm A and D do have
identical Degree of Financial Leverage calculations, the question asks which firm has the highest DFL,
which is firm C. When calculating the DFL figure, remember that the answer can never be less than
one, and can never be negative. In a situation where the company under examination has zero
interest expense, the DFL would be equal to one; i.e. the EBIT is equal to the EBIT minus the interest
expense. Another important note to remember is that in calculating the Degree of Financial
Leverage, dividend payments to preferred stockholders should be included in the interest expense
figure. Operating expenses are not factored into the DFL calculation, rather are used in the
determination of Operating Leverage.

Question: 2251

A stock's P/E ratio is 10.4, with an expected return on equity of 14% and a dividend growth rate of
5.7%. The firm's dividend payout ratio equals ________.

A. 24.19%
B. 56.17%
C. 86.32%
D. 13.68%

Answer: C

Explanation:
Po/E1 = dividend payout/(k - g)
Therefore, 10.4 = dividend payout /(0.14 - 0.057), giving dividend payout = 86.32%.

Question: 2252

Kulwicki Corporation wants to determine the effect of an expansion of its sales on its operating

Page | 829
income (EBIT). The firm's current degree of operating leverage is 2.5. It projects new unit sales to be
170,000, an increase of 45,000 over last year's level of 125,000 units. Last year's EBIT was $60,000.
Based on a degree of operating leverage of 2.5, what is this year's expected EBIT with the increase in
sales?

A. $175,000
B. $60,000
C. $114,000
D. $90,000
E. $100,000

Answer: C

Explanation:
Set up the DOL equation, letting x be the unknown new EBIT:
Let x = New EBIT.
DOL(Q) = % change EBIT/ % change Sales
% change in sales = 45000/125000 = 36%
2.5 = (x - $60,000/$60,000) / .36
2.5 (0.36) = ((x - $60,000)/$60,000)
0.90 = (x - $60,000/$60,000)
$54,000 = x - $60,000
x = $114,000.
New EBIT = $114,000.

Question: 2253

Which of the following statements is most correct?

A. All of these answers.


B. All else equal, an increase in a company's stock price will increase the marginal cost of issuing new
common equity.
C. None of these answers.
D. If a company's tax rate increases but the yield to maturity of its noncallable bonds remains the
same, the company's marginal cost of debt capital used to calculate its weighted average cost of
capital will fall.
E. All else equal, an increase in a company's stock price will increase the marginal cost of retained
earnings.

Answer: D

Explanation:
The debt cost used to calculate a firm's WACC is k(d)(1 - T). If k(d) (interest rate on the firm's new
debt) remains constant but T increases, then the term (1 - T) decreases and the value of the entire
equation, k(d)(1 - T), decreases.

Question: 2254

Which of the following figures are incorporated into the Degree of Operating Leverage equation as
based on total dollar sales?
I. Sales in dollars
II. Total variable costs
III. Average sales price
IV. Total fixed operating costs

Page | 830
V. Average variable cost per unit
VI. Number of common shares outstanding
VII. Sales in units
VIII. Discount rate

A. I, II, IV, VIII


B. II, IV, VII
C. I, II, IV, VIII
D. III, V, VII
E. I, II, IV
F. I, II, IV, VI

Answer: E

Explanation:
The Degree of Operating Leverage (DOL) measures the percentage change in EBIT that results from a
given percentage change in sales. Degree of Operating Leverage can be calculated using several
methods, including equations based upon unit sales and dollar sales. The DOL equation based on
dollar sales is illustrated as follows:
{DOL = [(sales in dollars - total variable costs) / (sales in dollars - total variable costs - total fixed
operating costs)]. Of the answers provided, only I, II, and IV are incorporated into this equation.

Question: 2255

Taylor Technologies has a target capital structure, which is 40 percent debt and 60 percent equity.
The equity will be financed with retained earnings. The company's bonds have a yield to maturity of
10 percent. The company's stock has a beta = 1.1. The risk-free rate is 6 percent, the market risk
premium is 5 percent, and the tax rate is 30 percent. The company is considering a project with the
following cash flows:
TimeCash flow ($)
0-50,000
135,000
243,000
360,000
4-40,000
What is the project's modified internal rate of return (MIRR)?

A. 6.76%
B. 16.14%
C. 20.52%
D. 10.78%
E. 9.26%

Answer: C

Explanation:
First, find the company's weighted average cost of capital:
We're given the before-tax cost of debt, k(d) = 10%. We can find the cost of equity as follows:
K(s) = 0.06 + 0.05(1.1) = 0.115 or 11.5%.
Thus, the WACC is:
k = 0.4(0.10)(1 - 0.3) + 0.6(0.115) = 0.097 or 9.7%.
Second, the PV of all cash outflows can be calculated as follows:
PV of CF(4): N = 4, I = 9.7, PMT = 0, FV = 40,000 and solve for PV. Total PV(Costs) = -$50,000 -
$27,620.62 = -$77,620.62.

Page | 831
Third, find the terminal value of the project at t = 4:
FV of CF(1) at t = 4 is calculated as follows: N = 3, I = 9.7, PV = -35,000, PMT = 0, and solve for FV =
$46,204,89. Similarly, the FVs of CF(2) and CF(3) are found to be $51,746.59 and $65,820,
respectively. Summing these FVs gives a terminal value of $46,204.89 + $51,746.59 + $65,820.00 =
$163,771.48.
Finally, the MIRR can be calculated as N = 4, PV = -77,620.62, PMT = 0, FV = 163,771.48, and solve for
I = MIRR = 20.52%.

Question: 2256

True or false. Firms with higher proportions of fixed costs will have an EBIT figure that is more
sensitive to changes in sales, all else equal. Additionally, companies that have low Degree of
Financial Leverage figures will have more aggressive depreciation and amortization schedules.

A. False, true
B. The answer cannot completely be determined from the information provided.
C. True, false
D. True, true
E. False, false

Answer: B

Explanation:
A somewhat tricky question as the answer cannot be fully determined from the information
provided. While it is true that firms whose cost structure is weighted heavily toward fixed costs, i.e.
have high relative DOL figures, will be more sensitive to changes in sales, the second question cannot
be answered from the information provided. The Degree of Financial Leverage is defined as the
percentage change in EPS that results from a given percentage change in EBIT. For purposes of
general discussion, and the Level 1 CFA exam, the DFL calculation does not have an explicit bearing
on the depreciation and amortization schedules used by companies. Had the second question asked
"...companies that have low Degree of Financial Leverage figures will have EPS figures which are LESS
sensitive to changes in EBIT," then both answers would be true.

Question: 2257

Which of the following statements is correct?

A. Market risk is important but does not have a direct effect on stock price because it only affects
beta.
B. Well diversified stockholders do not consider corporate risk when determining required rates of
return.
C. Undiversified stockholders, including the owners of small businesses, are more concerned about
corporate risk than market risk.
D. Empirical studies of the determinants of required rates of return (k) have found that only market
risk affects stock prices.

Answer: C

Explanation:
Corporate risk is the project's risk to the corporation giving consideration to the fact that the project
represents only one of the firm's portfolio of assets, hence that some of its risk effects on the firm's
profits will be diversified away.

Question: 2258

Page | 832
Calculate the cost of debt for the following firm:
Borrowing Rate 9.5%
Marginal Tax Rate 34%
Credit Rating BB+
Owner's Equity 15%

A. 1.5%
B. 8.075%
C. 1.43%
D. 9.5%
E. 6.27%

Answer: E

Explanation:
The cost of debt is simply the rate of borrowing less the tax savings. Due to the fact that interest
expense is tax deductible, the cost of debt in this case is 9.5%(1 - .34) = 9.5%(.66) = 6.27%.

Question: 2259

A firm's preferred equity has a face value of 100 and a 5.5% coupon. The equity is trading at $87.29
per share. The firm is in the 40% tax bracket. Its cost of preferred stock equals ________.

A. 2.52%
B. 3.30%
C. 6.30%
D. 3.78%

Answer: C

Explanation:
Preferred dividends are not tax-deductible. Hence, no tax adjustment is made while calculating the
cost of preferred equity. The price of a perpetuity that pays C per year, at a discount rate of R, equals
C/R. Hence, 87.29 = 5.5/R, giving R = 6.3%.

Question: 2260

The common stock of Anthony Steel has a beta of 1.20. The risk-free rate is 5 percent, and the market
risk premium is 6 percent. This year's addition to retained earnings is $3,000,000. The company's
capital budget is $4,000,000 and its target capital structure is 50 percent debt and 50 percent equity.
What is the company's cost of equity financing?

A. 12.4%
B. 7.0%
C. 11.0%
D. 12.2%
E. 7.2%

Answer: D

Explanation:
Anthony Steel will use retained earnings to fund the equity portion of its capital budget. We can see
this because the retained earnings break point is $3,000,000/0.5 = $6,000,000, which is greater than

Page | 833
the capital budget.
The cost of equity from the CAPM (Capital Asset Pricing Model) is:
k(s) = krf) + (k(m) - k(rf))b(i) = 5% + (6%)1.2 = 12.2%.

Question: 2261

Assume that a firm has a degree of financial leverage of 1.25. If sales increase by 20 percent, the firm
will experience a 60 percent increase in EPS, and it will have an EBIT of $100,000. What will be the
EBIT for this firm if sales do not increase?

A. $42,115
B. $84,375
C. $67,568
D. $100,000
E. $113,412

Answer: C

Explanation:
DTL = % change EPS/% change Sales = 60%/20% = 3.0.
DOL = DTL/DFL = 3.0/1.25 = 2.40.
Old EBIT = $100,000/[1 + (0.20)(2.40)] = $100,000/1.48 = $67,568.
Alternate solution:
Use DFL expression to calculate change in EBIT and previous EBIT:
DFL = 1.25 = %change EPS/%change EBIT
= 0.60/[change EBIT/($100,000 - change EBIT)]
= [0.60($100,000) - 0.60(change EBIT)]/change EBIT
1.25 change EBIT = $60,000 - 0.60(change EBIT)
1.85 change EBIT = $60,000
change EBIT = $32,432.
Old EBIT = $100,000 - $32,432 = $67,568.

Question: 2262

Returns on the market and Takeda Company's stock during the last 3 years are shown below:
YearMarketTakeda
1995-12%-14%
19962331
19971610
The risk-free rate is 7 percent, and the required return on the market is 12 percent. Takeda is
considering a project whose market beta was found by adding 0.2 to the company's overall corporate
beta. Takeda finances only with equity, all of which comes from retained earnings. The project has a
cost of $100 million, and it is expected to provide cash flows of $20 million per year at the end of
Years 1 through 5 and then $30 million per year at the end of Years 6 through 10. What is the
project's NPV (in millions of dollars)?

A. $23.11
B. $22.55
C. $28.12
D. $20.89
E. $25.76

Answer: A

Page | 834
Explanation:
1. Run a regression to find the corporate beta. It is 1.1633.
2. Find the project's estimated beta by adding 0.2 to the corporate beta. The project beta is thus
1.3633.
3. Find the company's cost of equity, which is its WACC because it uses no debt: k(s) = WACC = 7% +
(12% - 7%)1.3633 = 13.8165%.
4. Now find NPV (in millions):
CF(0) = -100
CF(1-5) = 20
CF(6-10) = 30
I = 13.82 Solve for NPV = $23.11 million.

Question: 2263

Which of the following statements is correct?

A. Only if one attempts to calculate MIRRs does one have to worry about multiple IRRs.
B. The discounted payback is generally shorter than the regular payback.
C. The NPV and IRR methods can lead to conflicting accept/reject decisions only if (1) mutually
exclusive projects are being evaluated and (2) if the projects' NPV profiles cross at a rate less than the
firm's cost of capital.
D. The NPV and IRR methods can lead to conflicting accept/reject decisions only if (1) mutually
exclusive projects are being evaluated and (2) if the projects' NPV profiles cross at a rate greater than
the firm's cost of capital.
E. Any type of project might have multiple rates of return if the IRR is sufficiently high.

Answer: C

Explanation:
The two conditions which can cause NPV profiles to cross, and thus conflicts to arise between NPV
and IRR: 1) when project size differences exists, or 2) when timing differences exist.

Question: 2264

A company has determined that its optimal capital structure consists of 40 percent debt and 60
percent equity. Given the following information, calculate the marginal weighted average cost of
capital when the capital budget is $40,000.
k(d) (interest rate on the firm's new date) = 10%
Net income = $40,000
Payout ratio = 50%
Tax rate = 40%
P(0) = $25
Growth = 0%
Shares outstanding = 10,000 Flotation cost on additional equity = 15%

A. 13.69%
B. 11.81%
C. 8.05%
D. 14.28%
E. 7.20%

Answer: C

Explanation:

Page | 835
First, find the amount of equity and debt needed for a $40,000 budget:
Debt = 0.4 x $40,000 = $16,000; Equity = 0.6 x $40,000 = $24,000.
We can find the amount of retained earnings = Net Income(1 - Payout ratio), or RE = $40,000 x 0.5 =
$20,000.
We will need to find the cost of new common equity, because we have only $20,000 of equity on
hand, and we need $4,000 more!
Find the dividend, Do = [(0.5) $40,000]/# of Shares = $20,000/10,000 = $2.00.
Then, find the cost of new equity: k(e) = D1/[P0(1 - F)] + g = $2.00/[$25(1 - 0.15)] + 0% = 0.0941 =
9.41%.
Finally, calculate WACC, using k(e) = 0.0941, and k(d) = 0.10, so
WACC = (D/A)(1 - Tax rate)k(d) + (E/A)k(e)
WACC = 0.4(1 - 0.4)(0.10) + 0.6(0.0941) = 0.0805, or 8.05%.

Question: 2265

Which of the following statements is most correct?

A. None of the statements are correct.


B. When choosing between mutually exclusive projects, managers should accept all projects with
IRRs greater than the weighted average cost of capital.
C. Multiple IRRs can occur in cases when project cash flows are normal, but they are more common
in cases where project cash flows are nonnormal.
D. All of the statements are correct.
E. One of the disadvantages of choosing between mutually exclusive projects on the basis of the
discounted payback method is that you might choose the project with the faster payback period but
with the lower total return.

Answer: E

Explanation:
The payback and discounted payback methods both ignore cash flows that are paid or received after
the payback period. Concerning the other statements: Multiple IRRs can occur only for projects
withnonnormal cash flows. Mutually exclusive projects implies that only one project should be
chosen. The project with the highest NPV should be chosen.

Question: 2266

Which of the following equations correctly illustrates the calculation of the cost of equity using the
Bond-Yield-plus-Risk-Premium approach?

A. Required rate of return on outstanding debt + subjective risk premium


B. Before-tax yield on outstanding debt + subjective risk premium
C. None of these answers
D. After-tax cost of debt + subjective risk premium
E. Annual dividend/current stock price + subjective risk premium
F. Yield to maturity on outstanding long-term debt + subjective risk premium

Answer: F

Explanation:
The Bond-Yield-plus-Risk-Premium approach is a rather ad hoc method used by financial managers to
determine the cost of common equity. Under this approach, a subjective risk premium is added to
the yield to maturity of the firm's outstanding long-term debt. Typically, senior debt is used when
possible, however, due to the ad hoc nature of this approach, there is abounding room for flexibility.

Page | 836
This large degree of flexibility both adds to and detracts from the attractiveness of Bond-Yield-plus-
Risk-Premium approach.

Question: 2267

Ace Consulting, a multinational corporate finance consulting firm, is examining the data storage
division of Intelligent Semiconductor Company. In order to evaluate the proposed expansion of this
division, Ace Consulting is trying to determine its beta. In their analysis, Ace Consulting regresses the
monthly returnon assets for the data storage division against the average return on assets for the
data storage index, a division of the S&P 100. Which of the following techniques most correctly
describes this method of identifying individual project betas?

A. Regression analysis
B. Scenario analysis
C. Pure-play method
D. Situation analysis
E. Monte Carlo simulation
F. Accounting Beta method

Answer: F

Explanation:
In this example, Ace Consulting is employing the Accounting Beta method to determine the beta of
the data storage division of Intelligent Semiconductor. This technique is often used when "pure play"
firms cannot be found, or when a more empirical, "firm-specific" analysis is desired. "Monte Carlo
simulation, situation analysis," and "scenario analysis" are all techniques for measuring stand-alone
risk. While "regression analysis" is an attractive choice, it does not represent the best possible
answer.

Question: 2268

Which of the following statements is most correct?

A. When comparing two projects, the project with the higher IRR will also have the higher MIRR.
B. Both IRR and MIRR can produce multiple rates of return.
C. The modified internal rate of return (MIRR) of a project increases as the cost of capital increases.
D. All of these statements are correct.
E. The internal rate of return (IRR) of a project increases as the cost of capital increases.

Answer: C

Explanation:
The MIRR is dependent on the cost of capital. As the cost of capital increases, so does the terminal
value. Because the MIRR is the rate, which equates the PV with the terminal value, the MIRR
increases as the terminal value increases.

Question: 2269

Copybold Corporation is a start-up firm considering two alternative capital structures--one is


conservative and the other aggressive. The conservative capital structure calls for a D/A ratio = 0.25,
while the aggressive strategy call for D/A = 0.75. Once the firm selects its target capital structure it
envisions two possible scenarios for its operations: Feast or Famine. The Feast scenario has a 60
percent probability of occurring and forecast EBIT in this state is $60,000. The Famine state has a 40
percent chance of occurring and the EBIT is expected to be $20,000. Further, if the firm selects the

Page | 837
conservative capital structure its cost of debt will be 10 percent, while with the aggressive capital
structure its debt cost will be 12 percent. The firm will have $400,000 in total assets, it will face a 40
percent marginal tax rate, and the book value of equity per share under either scenario is $10.00 per
share. What is the difference between the EPS forecasts for Feast and Famine under the aggressive
capital structure?

A. $0
B. $2.20
C. $2.40
D. $1.00
E. $1.80

Answer: C

Explanation:
Debt = 75% = $300,000; Equity = 25% = $100,000; Total assets = $400,000.
FeastFamine
Probability0.60.4
EBIT$60,000$20,000
Less: Interest36,00036,000
EBT$24,000-$16,000
Less: Taxes9,600-6,400
NI$14,400-$9,600
# shares10,00010,000
EPS$1.44-$0.96
Difference in EPS for conservative capital structure:
EPS(Feast) - EPS(Famine) = $1.44 - (-$0.96) = $2.40

Question: 2270

The Mike & Laurie Consulting Group Inc. is trying to decide which computer system to purchase.
They can purchase state-of-the-art equipment for $20,000, which they expect to generate cash flows
of $6,000 at the end of each of the next 6 years. Alternatively, they can spend $12,000 for equipment
that can be used for 3 years and generates cash flows of $6,000 at the end of each year. If the
company'scost of capital is 10 percent and both "projects" can be repeated indefinitely, then what is
the equivalent annual annuity (EAA) of the more profitable strategy?

A. $2,423.74
B. $1,407.85
C. $1,666.67
D. $6,131.56
E. $1,174.62

Answer: B

Explanation:
First, compute the NPV of each project over its initial life. The relevant cash flows for the state-of-
the- art equipment are CF(0) = -$20,000 and CF(1-6) = $6,000. Discounting at 10 percent yields NPV =
$6,131.56.
The relevant CFs for the less advanced equipment are CF(0) = -$12,000 and CF(1-3) = $6,000.
Discounting again at 10 percent yields NPV = $2,921.11. Next, find the EAA of each project as follows:
EAA for the state-of-the-art equipment: N = 6; I/YR = 10; PV = -6,131.56, FV = 0; solve for PMT = EAA
= $1,407.85. EAA less advanced: N = 3; I/YR = 10; PV = $2,921.11; FV = 0; solve for PMT = EAA =
$1,174.62.

Page | 838
Thus, the state-of-the-art project is more profitable.

Question: 2271

The "degree of leverage" concept is designed to show how changes in sales will affect EBIT and EPS.
If a 10 percent increase in sales causes EPS to increase from $1.00 to $1.50, and if the firm uses no
debt, then what is its degree of operating leverage?

A. 4.2
B. 3.6
C. 4.7
D. 5.0
E. 5.5

Answer: D

Explanation:
These two equations could be used:
DTL = (DOL)(DFL).
EPS(1) = EPS(0)[1 + (DTL)(%Change Sales)].
Note that EPS rises by 50 percent, from $1.00 to $1.50, on a 10 percent increase in sales, so
1.50 = 1.00[1 + (DTL)(0.1)]
1.50 = 1 + 0.1 DTL
0.1 DTL = 0.50
DTL = 5.00.
Now DTL = 5 = (DOL)(DFL)
But if Debt = 0, then DFL = 1, so DOL = DTL = 5.0.

Question: 2272

Clay Industries, a large industrial firm, is considering the development of an underwater drilling
system which will greatly increase the productivity of deep-sea petroleum extraction. However, the
development of the system involves substantial setup and implementation costs. If Clay Industries
chooses to begin developing the new system, the firm will be forced to decline several other
promising projects, due to a lack of available investment capital. Which of the following terms most
correctly describes the problem faced by Clay Industries?

A. Diminishing returns problem


B. Marginal cost problem
C. Principal/agent problem
D. Opportunity cost problem
E. Externality problem

Answer: D

Explanation:
In this example, Clay Industries is faced with several mutually-exclusive projects. If the firm begins to
develop the underwater drilling system, it will be forced to decline the acceptance of other projects.
This is an opportunity cost problem.

Question: 2273

Which of the following types of risk can be reduced through diversification? Choose the best answer.
I. Stand-alone risk

Page | 839
II. Unsystematic risk
III. Systematic risk
IV. Market risk
V. Beta risk
VI. Diversifiable risk

A. I, III, VI
B. I, VI
C. II, III, V
D. I, II, VI
E. II, III, V, VI

Answer: D

Explanation:
Of the risks listed, only unsystematic and stand-alone risk are diversifiable. Unsystematic risk is also
referred to as "diversifiable risk," therefore answer VI is correct. Stand-alone risk is defined as the
variability of an asset's expected returns if it were the only asset of a firm and the stock of that firm
isthe only security in an investor's portfolio. This type of risk is definitively reduced through
diversification, and is commonly referred to as "unsystematic risk." Systematic risk measures that
part of an assets risk that is inherent regardless of the level of diversification, and is measured by the
Beta coefficient. Systematic risk is also referred to as "market risk" and "beta risk." Corporate risk is
defined as the variability of an asset's expected returns without taking into consideration the effects
of shareholder diversification. This is one step away from Stand-alone Risk, which measures the risk
of an asset not only without taking into consideration the effect of shareholder diversification, but of
company diversification as well. Stand-alone risk assumes that the asset in question is the only asset
of the firm and that the securities of the firm are the only asset in investors' portfolios. Corporate risk
takes into consideration that firms will diversify their asset bases.

Question: 2274

Martin Corporation's common stock is currently selling for $50 per share. The current dividend is
$2.00 per share. If dividends are expected to grow at 6 percent per year and if flotation costs are 10
percent, then what is the firm's cost of retained earnings and what is its cost of new common stock?

A. 10.71%; 10.24%
B. 10.24%; 10.71%
C. 11.38%; 10.71%
D. 10.24%; 11.38%
E. 9.31%; 9.86%

Answer: B

Explanation:
Cost of retained earnings:
k(s) = $2.12/$50 + 0.06 = 10.24%.
Cost of new common equity:
k(e) = $2.12/($50(1-.10)) + 0.06 = 10.71%.

Question: 2275

A stock has a beta of 0.44 and the market risk premium is 7.9%. Its dividend growth rate is 4.25% and
its P/E ratio is 8.7. If the firm has a dividend payout ratio of 70%, the risk-free rate equals ________.

Page | 840
A. 6.81%
B. 7.12%
C. 8.82%
D. 4.56%

Answer: C

Explanation:
Po/E1 = dividend payout/(k - g) Therefore, 8.7 = 0.7/(k - 0.0425), giving expected return = k = 12.3%.
Now, the CAPM expected return on the stock is given by k = Rf + beta*(Rm - Rf). Therefore, 12.3% =
Rf + 0.44*7.9%, giving risk-free rate = 8.82%.

Question: 2276

Which of the following may be used as mechanisms to motivate managers to act in the best interest
of the stockholders?
I. Managerial compensation
II. Direct stockholder intervention
III. Threat of firing
IV. Threat of takeover

A. IV only
B. I only
C. I, II, III & IV
D. III only
E. I, II & III
F. II only

Answer: C

Explanation:
Managerial compensation may be designed to not only attract and retain the best managerial talent
for a firm, but also to align the management's action with the interest of the shareholders. Direct
intervention is another mechanism that may be used to motivate management into acting in the
owner's best interest. This in practice is executed with a wide degree of success. The threat of firing
may also be used as well as the threat of hostile takeovers. The threat of a hostile takeover is
strongest when a company is under performing and/or its stock is under valued.

Question: 2277

The following facts apply to your company:


Target capital structure: 50% debt; 50% equity.
EBIT:$200 million
Assets:$500 million
Tax rate:40%
Cost of new and old debt:8%
Based on the residual dividend policy, the payout ratio is 60 percent. How large (in millions of dollars)
will the capital budget be?

A. $43.2
B. $50.0
C. $108.0
D. $86.4
E. $64.8

Page | 841
Answer: D

Explanation:
Debt = 0.5(Assets) = 0.5($500) = $250 million.
Interest = 0.08($250) = $20 million.
EBT = EBIT - I = $200 - $20 = $180.
NI = $180 - Taxes = $180 - $180(0.4) = 0.6($180) = $108 million.
Dividends = $108(0.6) = $64.80 million.
Retained earnings = NI - D = $108.00 - $64.80 = $43.20 million.
Half of the capital budget will be debt, half will be common equity from retained earnings, so the
capital budget will = $86.40 million.

Question: 2278

Clay Industries, a large industrial firm, is evaluating the sales of its existing line of coiled machine
tubing. In their analysis, the operating managers of Clay Industries have identified the following
information related to the coiled machine tubing division and its product: Annual fixed operating
expenses of $925,000 Average variable cost of $90 Breakeven quantity of 20,109 units Which of the
following best describes the average variable cost for this product?

A. $44
B. None of these answers is correct.
C. $46
D. $41.70
E. $38
F. The average selling price of this product cannot be determined from the information provided.That
answer is correct!

Answer: A

Explanation:
To calculate the breakeven quantity for a product, use the following equation: {Fixed operating
costs/[avg. sales price per unit - variable cost per unit]}. To determine the average selling price of this
product, we must rearrange the standard equation using algebra, in a manner such that the resulting
equation resembles the following: {[$925,000/20,109] + X = $90}. This equation is further rearranged
into the following: {$46 + X = $90}. Finally, the ending equation becomes: {X = $90 - $46}. Solving for
X yields an average variable cost per unit of $44.

Question: 2279

Stromburg Corporation makes surveillance equipment for intelligence organizations. Its sales are
$75,000,000. Fixed costs, including research and development, are $40,000,000, while variable costs
amount to 30 percent of sales. Stromburg plans an expansion which will generate additional fixed
costs of $15,000,000, decrease variable costs to 25 percent of sales, and also permit sales to increase
to $100,000,000. What is Stromburg's degree of operating leverage at the new projected sales level?

A. 3.50
B. 3.33
C. 4.67
D. 4.20
E. 3.75

Answer: E

Page | 842
Explanation:
Calculate DOL using new sales, new variable cost percentage, and new fixed costs: (In millions)
DOL(S) = S-VC/S-VC-FC = $100-$25/($100-$25-$55) = 3.75.

Question: 2280

Which of the following is not expressly incorporated into the Degree of Total Leverage (DTL)
calculation?

A. Sales
B. Fixed costs
C. Variable costs
D. Interest expense
E. None of these answers
F. Common shares outstanding

Answer: F

Explanation:
The Degree of Total Leverage (DTL) calculation measures the percentage change in EPS from a given
percentage change in sales. The equation used to produce DTL is as follows:
{DTL = [(Sales - Variable Costs) / (Sales - Variable Costs - Fixed Costs - Interest Expense)]. Of all the
choices listed, "only the number of common shares outstanding" is not expressly incorporated into
the DTL calculation.

Question: 2281

Ameriscam, Inc. is considering the issuance of some perpetual preferred stock. The Company's
corporate tax rate is 30%, and the yield on its outstanding senior debt is 7.55%. Additionally,
Ameriscam has been told by a leading investment bank that if issued, its preferred stock would merit
a price of $40 net of flotation costs and other charges. If issued, the firm plans to dedicate preferred
annual dividends of $2.35 per share. What is the cost of the proposed preferred stock for this firm?

A. The cost of preferred stock cannot be calculated from the information supplied.
B. 2.265%
C. 7.98%
D. 0.0875%
E. 5.875%

Answer: E

Explanation:
The cost of preferred stock can be found by dividing the annual dividend by the issuing price, which is
net of any underwriting, or "flotation," charges. The cost of preferred stock in this example is very
low. Typically, the cost of preferred stock is greater than the cost of debt but less than the cost of
common equity.

Question: 2282

Petersen Co. has a capital budget of $1,200,000. The company wants to maintain a target capital
structure, which is 60 percent debt and 40 percent equity. The company forecasts that its net income
this year will be $600,000. If the company follows a residual dividend policy, what will be its payout
ratio?

Page | 843
A. 20%
B. 80%
C. 60%
D. 40%
E. 0%That answer is correct!

Answer: A

Explanation:
The amount of new investment, which must be financed with equity, is: $1,200,000 x 40% =
$480,000. Since the firm has $600,000 of net income only $120,000 will be left for dividends. This
means the payout ratio is $120,000/$600,000 = 20%.

Question: 2283

In theory, the decision-maker should view market risk as being of primary importance. However,
within-firm, or corporate, risk is relevant to a firm's

A. All of the answers are correct.


B. None of the answers are correct.
C. Creditors, because it affects the firm's credit worthiness.
D. Management, because it affects job stability.
E. Well-diversified stockholders, because it may affect debt capacity and operating income.That
answer is correct!

Answer: A

Explanation:
These are all relevant to a firm's corporate risk, which is measured by the project's impact on
uncertainty about the firm's future earnings.

Question: 2284

Arizona Rock, an all-equity firm, currently has a beta of 1.25, and k(RF) = 7 percent and k(M) = 14
percent. Suppose the firm sells 10 percent of its assets (beta = 1.25) and purchases the same
proportionof new assets with a beta of 1.1. What will be the firm's new overall required rate of
return, and what rate of return must the new assets produce in order to leave the stock price
unchanged?

A. 15.750%; 15.645%
B. 14.750%; 15.750%
C. 15.645%; 14.700%
D. 15.750%; 14.700%
E. 15.645%; 15.645%

Answer: C

Explanation:
b(old, firm) = 1.25.
k(old, firm) = 0.07 + (14 - 7)1.25 = 15.75%.
b(new, firm) = 0.9(1.25) + 0.1(1.1) = 1.235.
k(new, firm) = 0.07 + 1.235(0.07) = 15.645%.
k(new, assets) = 0.07 + 1.1(0.07) = 14.7%.

Page | 844
Question: 2285

In order for the NPV and MIRR methods to consistently produce similar rankings, the projects being
examined must possess which of the following characteristics? Choose the best answer.

A. Projects must be independent and equal in size


B. Projects must equal in scale and be mutually exclusive
C. Projects must be profitable and have normal cash flows
D. Projects must equal in scale and have the same life
E. Projects must equal in scale and have identical cash flows
F. Projects must have equal lifespans and normal cash flows

Answer: D

Explanation:
When examining mutually-exclusive projects with normal cash flows, the MIRR and NPV methods
will ALWAYS produce similar results as long as the projects being examined are equal in size and have
the same life.

Question: 2286

Which of the following projects would likely produce multiple Internal Rates of Return? Assume a
14% discount rate.
Project A
Initial investment outlay: ($500,000)
t1: $900,000
t2: ($100,000)
t3: ($100,000)
t4: ($10,000)
Project B
Initial investment outlay: ($500,000)
t1: $0.00
t2: $650,000
Project C
Initial investment outlay: ($50,000)
t1: $0.00
t2: $0.00
t3: $65,000
t4: $0.00
t5: $0.00
t6: $65,000
Project D
Initial investment outlay: ($1,000,000)
t1: $675,000
t2: $675,000
t3: ($1,500)
t4: $1,500
Project E
Initial investment outlay: ($1,000,000)
t1: $0.00
t2: $0.00
t3: $0.00
t4: $0.00

Page | 845
t5: $2,000,000

A. Project A, D, and E
B. Project A and D
C. Project C
D. All of these projects will likely result in multiple Internal Rates of Return.
E. Project B
F. Project A

Answer: B

Explanation:
First of all, the cost of capital is irrelevant in Internal Rate of Return calculations. What is being
examined in this example is the determination of "normal" versus "non-normal" projects. Non-
normal projects are classified as projects that possess non-normal cash flows. In evaluating projects
with "non-normal cash flows" the Internal Rate of Return method will often produce multiple IRRs
which leads to an incorrect accept/reject decision. Non-normal cash flows are defined as cash flows
in which the sign changes more than once. Projects A and D involve cash outflows superimposed
within their cash inflows, resulting in a sign change from positive to negative and negative to
positive. In examining projects such as this, it is advisable to use either the NPV or MIRR methods,
which are not subject to the problem of multiple IRRs. From observation alone, we can determine
that project A and D are non-normal projects, and are thus likely to result in multiple IRR
calculations. While project B, C and E have periods of zero cash flows, each only has one change of
sign in the overall cash flow process, and therefore all three projects should be characterized as
"normal" for purposes of examination. While the cost of capital has been provided, it is not
necessary for the determination of the correct answer in this case. What you should look for are
projects with non-normal cash flows, and this should not involve any computational analysis.
Besides, the cost of capital is not incorporated into the Internal Rate of Return calculation, rather, it is
a component of the NPV and MIRR computational methods.

Question: 2287

Cochran Corporation has a weighted average cost of capital of 11 percent for projects of average risk.
Projects of below-average risk have a cost of capital of 9 percent, while projects of above-average
risk have a cost of capital equal to 13 percent. Projects A and B are mutually exclusive, whereas all
other projects are independent. None of the projects will be repeated. The following table
summarizes the cash flows, internal rate of return (IRR), and risk of each of the projects.
Year (t)Project AProject BProject CProject DProject E
O-200,000-100,000-100,000-100,000-100,000
166,00030,00030,00030,00040,000
266,00030,00030,00030,00025,000
366,00040,00030,00040,00030,000
466,00040,00040,00050,00035,000
IRR12.1114.03810.84816.63611.630
ProjectBelowBelowAverageAboveAbove
RiskAverageAverageAverageAverage
Which projects will the firm select for investment?

A. Projects: A, D
B. Projects: B, C, D, and E
C. Projects: B, D
D. Projects: B, C, and D
E. Projects: A, B, and DThat answer is correct!

Page | 846
Answer: A

Explanation:
Look at the NPV, IRR, and hurdle rate for each project:
ProjectABCDE
Hurdle9.00%9.00%11.00%13.00%13.00%
NPV$13,822$11,998
IRR12.11%14.04%10.85%16.64%11.63% Projects A and B are mutually exclusive, so we pick project A
because it has the largest NPV. Projects C, D, and E are independent so we pick the ones whose IRR
exceeds the cost of capital, in this case, just D. Therefore, the projects undertaken are A and D.

Question: 2288

Ace Consulting, a corporate finance consulting firm, is examining the operating performance of
Microscam Incorporated. In their analysis, Ace Consulting has identified the following information:
Year 1 interest paid $28,000
Year 2 interest paid $35,000
Year 1 sales $1,675,000
Year 2 sales $1,895,000
Year 1 EBIT $750,000
Year 2 EBIT $987,500
Cost of debt 7.70%
Given this information, what is the Degree of Operating Leverage for this firm during the time period
in question?

A. 1.531
B. 2.431
C. The Degree of Operating Leverage cannot be calculated due to the fact that an appropriate
discount rate has not been provided.
D. 2.412
E. 2.618
F. 0.415

Answer: D

Explanation:
To calculate the degree of operating leverage, use the following equation: {% change in EBIT/%
change in sales}. In this example, neither the percentage change in sales, neither the percentage
change in EBIT are provided, and must be calculated manually. To calculate the percentage change in
sales, use the following equation: {[sales in year 2 - sales in year 1]/sales in year 1}. Incorporating the
given information into this calculation yields a % change in sales of 13.13%. To calculate the
percentage change in EBIT, use the same equation as follows: {[EBIT in year 2 - EBIT in year 1]/EBIT in
year 1}. Incorporating the given information into this calculation yields a percentage change in the
EBIT of 31.67%. Finally, to calculate the Degree of Operating Leverage, divide the percentage change
in EBIT by the percentage change in sales, which derives a DOL of 2.412. The "interest paid"
information is irrelevant in the calculation of the DOL, rather is incorporated into a determination of
the Degree of Financial Leverage. This information has been provided to trick you. Additionally, the
Degree ofOperating Leverage can be calculated regardless of whether an appropriate discount rate
has been provided or not.

Question: 2289

If the expected return on the market portfolio increases, the price of a firm's share ______, all else
equal.

Page | 847
A. can be all of these answers.
B. is not affected
C. decreases
D. increases

Answer: C

Explanation:
In the usual notation, Po = D1/(k-g). Therefore, if the expected return on the stock, k, increases and
all else remains constant, the price will fall. The expected return on the stock, k, increases if the
market's expected rate of return increases.

Question: 2290

Lugar Industries is considering an investment in a proposed project which requires an initial


expenditure of $100,000 at t = 0. This expenditure can be depreciated at the following annual rates:
tDepreciation Rate
120%
232%
319%
412%
511%
66%
The project has an economic life of six years. The project's revenues are forecasted to be $90,000 a
year. The project's operating costs (not including depreciation) are forecasted to be $50,000 a year.
After six years, the project's estimated pre-tax salvage value is $10,000. The company's WACC is 10
percent, and its corporate tax rate is 40 percent. What is the project's net present value (NPV)?

A. $31,684
B. $34,667
C. $45,453
D. $33,843
E. $38,840

Answer: E

Explanation:
The cash flows for each of the years are as follows:
0-100,000
1 [90,000 - 50,000 -(100,000)(0.20)](1-0.4)+(100,000)(0.20)= 32,000
2 [90,000 - 50,000 -(100,000)(0.32)](1-0.4)+(100,000)(0.32)= 36,800
3 [90,000 - 50,000 -(100,000)(0.19)](1-0.4)+(100,000)(0.19)= 31,600
4 [90,000 - 50,000 -(100,000)(0.12)](1-0.4)+(100,000)(0.12)= 28,800
5 [90,000 - 50,000 -(100,000)(0.11)](1-0.4)+(100,000)(0.11)= 28,400
6 [90,000 - 50,000 -(100,000)(0.06)](1-0.4)+(100,000)(0.06) + (10,000)(1 - 0.4) = 32,400 Enter the cash
flows and solve for the NPV = $38,839.56.

Question: 2291

If the IRS lowers the tax rate applicable to firms in a particular category, the optimal debt ratio for
that category will ________.

A. not be affected

Page | 848
B. increase
C. decrease
D. insufficient information

Answer: C

Explanation:
If the tax rate is lower, the tax-deductibility of the interest payments on debt becomes less attractive.
The after-tax cost of debt rises, lowering the optimal debt ratio.

Question: 2292

A company has an EBIT of $4 million, and its degree of total leverage is 2.4. The firm's debt consists
of $20 million in bonds with a 10 percent yield to maturity. The company is considering a new
production process that will require an increase in fixed costs but a decrease in variable costs. If
adopted, the new process will result in a degree of operating leverage of 1.4. The president wants to
keep the degree of total leverage at 2.4. If EBIT remains at $4 million, what amount of bonds must be
outstanding to accomplish this (assuming the yield to maturity remains at 10 percent)?

A. $16.7 million
B. $18.5 million
C. $20.1 million
D. $19.2 million
E. $19.8 millionThat answer is correct!

Answer: A

Explanation:
First, find the new DFL:
DTL = (DOL)(DFL)
2.4 = (1.4)(DFL)
DFL = 1.7143.
Then, find the new interest payments in a year:
DFL = (EBIT)/(EBIT - I)
1.7143 = ($4,000,000)/($4,000,000 - I)
I = $1,666,686.11.
Finally, solve for the new debt level, knowing that the yield to maturity remains at 10%. Debt
Value(YTM) = Interest Payment
Debt(0.10) = $1,666,686.11
Debt = $16,666,861.11 = $16.7 million.

Question: 2293

Clay Industries, a large industrial firm, has just released a new process system allowing mining
companies to automate much of their copper extraction procedures. While the sales of this process
system are expected to be hugely successful, analysts predict that sales of Clay Industries existing
products will decline as a result, as customers substitute the new process system for much of the Clay
Industries' older drilling components and non-automated process systems. Which of the following
terms most correctly describes the problem faced by Clay Industries?

A. Externality problem
B. Diminishing returns problem
C. Cannibalization
D. Opportunity cost problem

Page | 849
E. Incremental sales deterioration

Answer: C

Explanation:
When a new product or service takes sales away from existing products or services, this is often
referred to as cannibalization (or erosion). While firms naturally do not wish to cannibalize existing
products, often if they do not, other firms will begin to erode their market share. The cannibalization
problem is frequently considered in the analysis of new releases of products and services.

Question: 2294

As the debt level rises, the cost of equity increases because:

A. the probability of default increases.


B. all of these answers.
C. the variability of EPS increases.
D. the financial risk increases.

Answer: B

Explanation:
All of the above are reasons why the cost of equity increases as the debt level rises.

Question: 2295

Maxvill Motors has annual sales of $15,000. Its variable costs equal 60 percent of its sales, and its
fixed costs equal $1,000. If the company's sales increase 10 percent, what will be the percentage
increase in the company's earnings before interest and taxes (EBIT)?

A. 18%
B. 20%
C. 16%
D. 12%
E. 14%

Answer: D

Explanation:
First, find EBIT before sales increase:
EBIT = Sales - (Sales x VC%) - FC
= $15,000 - ($15,000 x 0.60) - $1,000
= $5,000.
Now, assuming sales increase by 10% or to $15,000 x 1.10 = $16,500, calculate the new EBIT. EBIT =
$16,500 - ($16,500 x 0.60) - $1,000 = $5,600.
So, the percentage increase is [($5,600 - $5,000)/$5,000] x 100 = 12%.

Question: 2296

Louisiana Enterprises, an all-equity firm, is considering a new capital investment. Analysis has
indicated that the proposed investment has a beta of 0.5 and will generate an expected return of 7
percent. The firm currently has a required return of 10.75 percent and a beta of 1.25. The
investment, if undertaken, will double the firm's total assets. If k(RF) = 7 percent and the market
return is 10 percent, should the firm undertake the investment?

Page | 850
A. No; the expected return of the asset (7%) is less than the required return (8.5%).
B. Yes; the expected return of the asset (7%) exceeds the required return (6.5%).
C. Yes; the beta of the asset will reduce the risk of the firm.
D. No; the risk of the asset (beta) will increase the firm's beta.
E. No; the expected return of the asset is less than the firm's required return, which is 10.75%.That
answer is correct!

Answer: A

Explanation:
Calculate the required return and compare to the expected return.
k(s) = k(RF) + (k(M) - k(RF))b = 0.07 + (0.10 - 0.07)0.5 = 0.085 = 8.5%.
The expected return of the asset (7%) is less than the required return (8.5%) so the investment
should not be made.

Question: 2297

The process of planning expenditures on assets whose cash flows are expected to extend beyond one
year is known as ________.

A. Net Present Valuing


B. Capital Budgeting
C. Optimal Capital Structure
D. Payback Period
E. Weighted Average Cost of Capital (WACC)

Answer: B

Explanation:
Capital Budgeting is defined as the process of planning expenditures on assets whose cash flows are
expected to extend beyond one year.

Question: 2298

A portfolio manager with Mally, Feasance, & Company is examining shares of Melton Industries, a
large industrial firm. Assume the following information:
Annual Dividend: $0.70
EPS: $1.65
Tax Rate: 35%
Discount Rate: 13.15%
ROE: 16%
Using this information, what is the expected growth rate of this firm? Assume that the discount rate,
tax rate, and ROE are expected to remain stable.

A. 11.59%
B. 9.21%
C. None of these answers
D. 6.79%
E. 6.00%
F. 10.00%

Answer: B

Page | 851
Explanation:
To calculate the dividend growth rate, assuming a stable ROE figure, use the following equation: {g =
ROE(1 - Dividend Payout Ratio)}. While the ROE figure has been provided, the Dividend Payout
Ratiomust be calculated manually. To find the Dividend Payout Ratio, divide the annual dividend by
the EPS figure, giving the following: {Dividend Payout Ratio = ($0.70/$1.65)}. From this equation, we
determine that the Dividend Payout Ratio for this firm is 42.42%. Imputing this figure into the
Dividend Growth Rate Equation will yield a growth rate of 9.21% for this firm. As you can see, neither
tax rates nor discount rates are incorporated into the calculation.

Question: 2299

All else equal, which of the following is/are true about break-even point?
I. An increase in the sale price per unit increases the break-even quantity.
II. An increase in the variable cost per unit increases the break-even quantity.
III. An increase in the fixed costs increases the break-even quantity.

A. III only
B. I & II
C. I only
D. II & III
E. I, II & III
F. II only
G. I & III

Answer: D

Explanation:
The break-even quantity is the number of units that must be sold to just cover the fixed and variable
costs. An increase in revenues per unit will decrease the break-even quantity while an increase in
costs per unit will increase the break-even quantity.

Question: 2300

Flood Motors is an all-equity firm with 200,000 shares outstanding. The company's EBIT is $2,000,000
and is expected to remain constant over time. The company pays out all of its earnings each year, so
its earnings per share equals its dividends per share. The company's tax rate is 40 percent. The
company is considering issuing $2 million worth of bonds (at par) and using the proceeds for a stock
repurchase. If issued, the bonds would have an estimated yield to maturity of 10 percent. The risk-
free rate in the economy is 6.6 percent, and the market risk premium is 6 percent. The company's
beta is currently 0.9, but its investment banker's estimate that the company's beta would rise to 1.1
if they proceed with the recapitalization. Assume that the shares are repurchased at a price equal to
the stock market price prior to the recapitalization. What would be the company's stock price
following the recapitalization?

A. $53.85
B. $51.14
C. $76.03
D. $56.02
E. $68.97

Answer: B

Explanation:
First, find the company's current cost of capital, dividends per share, and stock price: k = 0.066 +

Page | 852
(0.06)0.9 = 12%. To find the stock price, you still need the dividends per share or DPS = ($2,000,000 (1
- 0.4))/200,000 = $6.00. Thus, the stock price is Po = $6.00/0.12 = $50.00. Thus, by issuing $2,000,000
in new debt the company can repurchase $2,000,000/$50.00 = 40,000 shares.
Now after recapitalization, the new cost of capital, DPS, and stock price can be found: k = 0.066 +
(0.06)1.1 = 13.20%. DPS for the remaining (200,000 - 40,000) = 160,000 shares are thus [($2,000,000 -
($2,000,000 x 0.10))(1 - 0.4)]/160,000 = $6.75. And, finally, Po = $6.75/0.132 = $51.14.

Question: 2301

Which of the following is/are disadvantages of stock repurchases?


I. If investors are not indifferent between dividends and capital gains, regular repurchase programs
could drive them away.
II. The IRS could tax the firm for improper accumulation of capital gains if it felt regular repurchase
programs had taken the place of dividends.
III. The firm might end up paying a higher than fair price if it commits to a repurchase program.

A. II & III
B. I & III
C. III only
D. II only
E. I only
F. I, II & III

Answer: F

Explanation:
If shareholders are not indifferent between dividends and capital gains, the stock price might
increase more with dividends. This is because cash dividends are generally made very regularly,
while stock repurchases are irregularly made. Although this has been rarely done for public
corporations, the IRS can impose a tax if it believes regular repurchases are being made to avoid
paying dividends. If a company's shares are thinly traded, and the firm wishes to repurchase a large
number of shares, it could bid up the stock price above the equilibrium price and overpay for the
shares.

Question: 2302

As the director of capital budgeting for Denver Corporation, you are evaluating two mutually
exclusive projects with the following net cash flows:
YearProject XProject Z
0-$100,000-$100,000
150,00010,000
240,00030,000
330,00040,000
410,00060,000
If Denver's cost of capital is 15 percent, which project would you choose?

A. Project Z, since it has the higher NPV.


B. Project X, since it has the higher NPV.
C. Neither project.
D. Project X, since it has the higher IRR.
E. Project Z, since it has the higher IRR.

Answer: C

Page | 853
Explanation:
(In thousands)
NPV(X) = -100 + 50(PVIF(15%,1)) + 40(PVIF(15%,2)) + 30(PVIF(15%,3)) + 10(PVIF(15%,4)) = -100 +
50(0.8696) + 40(0.7561) + 30(0.6575) + 10(0.5718) = -0.833 = -$833.
NPVZ = -100 + 10(PVIF(15%,1)) + 30(PVIF(15%,2)) + 40(PVIF(15%,3)) + 60(PVIF(15%,4)) = -100 +
10(0.8696) + 30(0.7561) + 40(0.6575) + 60(0.5718) = -8.013 = -$8,013.
At a cost of capital of 15%, both projects have negative NPVs and, thus, both would be rejected.

Question: 2303

Which of the following statements is most correct?

A. Suppose a firm is losing money and thus, is not paying taxes, and that this situation is expected to
persist for a few years whether or not the firm uses debt financing. Thus the firm's after-tax cost of
debt will equal its before-tax cost of debt.
B. The bond-yield-plus-risk-premium approach to estimating a firm's cost of common equity involves
adding a subjectively determined risk-premium to the market risk-free bond rate.
C. The reason that a cost of capital is assigned to retained earnings is because these funds are already
earning a return in the business, the reason does not involve the opportunity cost principle.
D. The component cost of preferred stock is expressed as k(ps)(1 - T), because preferred stock
dividends are treated as fixed charges, similar to the treatment of debt interest.
E. All of these statements are false.That answer is correct!

Answer: A

Explanation:
Obviously if the firm is paying no taxes, its after-tax cost of debt will equal its before-tax cost of debt.

Question: 2304

Firms with higher operating leverage tend to have _______ financial leverage.

A. lower
B. same
C. higher or lower (since the two are not related).
D. higherThat answer is correct!

Answer: A

Explanation:
Firms with higher operating leverage have a higher fraction of costs in the form of fixed costs and
hence, have a higher business risk. This makes them more averse to debt. Consequently, firms with
higher operating leverage tend to have lower D/E ratios i.e. lower financial leverage.

Question: 2305

The percentage mix of debt, preferred stock and common equity that maximizes a firm's stock price
is known as:

A. Composite Cost of Capital (CCC)


B. Security Market Line
C. Weighted Average Cost of Capital (WACC)
D. Target (Optimal) Capital Structure
E. Least Cost Structure

Page | 854
F. Efficient Frontier

Answer: D

Explanation:
The Target (Optimal) Capital Structure is defined as the percentages of debt, preferred stock, and
common equity that will maximize the firm's stock price.

Question: 2306

Stock dividends

A. must be accompanied by cash dividends.


B. are similar to stock splits in that they do not change the fundamental position of current
shareholders.
C. have the same effects on financial statements as cash dividends.
D. are viewed unfavorably by investors and thus should not be used.
E. have no effect on a firm's balance sheet.

Answer: B

Explanation:
Stock dividends are dividends paid in the form of additional shares of stock rather than in cash. The
total number of shares is increased, so earnings, dividends, and price per share all decline. Stock
dividends that are used on a regular basis will keep the stock price more or less constrained, that is,
within a desired trading range.

Question: 2307

Given the following net cash flows, determine the IRR of the project:
TimeNet cash flow
0$1,520
1-1,000
2-1,500
3500

A. 36%
B. 28%
C. 32%
D. 24%
E. 20%

Answer: D

Explanation:
Time line:
0123 Periods
1,520-1,000-1,500500
Financial calculator solution: Using cash flows,
Inputs: CF(0) = 1,520; CF(1) = -1,000; CF(2) = -1,500; CF(3) = 500.
Output: IRR% = 23.98%.

Question: 2308

Page | 855
International Transport Company is considering building a new facility in Seattle. If the company goes
ahead with the project, it will spend $2 million immediately (at t = 0) and another $2 million at the
end of Year 1 (t = 1). It will then receive net cash flows of $1 million at the end of Years 2 - 5, and it
expects to sell the property for $2 million at the end of Year 6. The company's cost of capital is 12
percent, and it uses the modified IRR criterion for capital budgeting decisions. Which of the following
statements is most correct?

A. The regular IRR is less than the cost of capital. Under this condition, the modified IRR will also be
less than the regular IRR.
B. The project should be accepted because the modified IRR is greater than the cost of capital.
C. If the regular IRR is less than the cost of capital, then the modified IRR will be greater than the
regular IRR. That situation applies in this case.
D. The project should be rejected because the modified IRR is less than the regular IRR.
E. Given the data in the problem, the modified IRR criterion indicates that the project should be
accepted. However, the NPV is negative. This demonstrates that the modified IRR criterion is not
always a valid decision method for projects such as this one.

Answer: C

Explanation:
PV(Outflows) = -$2,000,000 - $2,000,000/1.12 = -$3,785,714.
TV(Inflows) = $1,000,000(FVIFA(12%,4))(1.12) + $2,000,000
= $1,000,000(4.7793)(1.12) + $2,000,000 = $7,352,816.
1 + MIRR = [7,352,816/3,785,714]^1/6; MIRR = 11.7%.
Since the MIRR is less than the cost of capital, the IRR is less than the MIRR.

Question: 2309

An investment of $1,000 will return $60 annually forever. What is its internal rate of return?

A. 6.00%
B. 60.00%
C. 16.67%
D. cannot be determined
E. 0.60%That answer is correct!

Answer: A

Explanation:
$1,000 = $60/Irr; IRR = 0.06 = 6%.

Question: 2310

Which of the following statements is most correct?

A. We ideally would like to use historical measures of the component costs from prior financing in
estimating the appropriate weighted average cost of capital.
B. The cost of a new equity issuance could possibly be lower than the cost of retained earnings if the
market risk premium and risk-free rate decline by a substantial amount.
C. None of these statements.
D. In the weighted average cost of capital calculation, we must adjust the cost of preferred stock for
the tax exclusion of 70% of dividend income.
E. All of these statements.

Page | 856
Answer: C

Explanation:
Unlike interest expense on debt, preferred dividends are not deductible, hence there are no tax
savings associated with the use of preferred stock. The component costs of WACC should reflect the
costs of new financing not historical measures. The cost of issuing new equity is always greater than
the cost of retained earnings because of the existence of flotation costs.

Question: 2311

Rapacity Consultants has just finished a project feasibility study for a cash-rich firm at a cost of $3
million. The consultants have concluded after much analysis that the project's cash flows have a net
present value of $1.3 million and a payback period of 5.3 years. The firm should:

A. reject the project since it has a long payback period.


B. reject the project since it has a negative NPV.
C. none of these answers.
D. accept the project since it has a positive NPV.

Answer: D

Explanation:
The $3 million spent on consultants represent sunk costs and must be ignored while looking toward
the future. In that direction, the project has a positive NPV and should be accepted.

Question: 2312

The firm's target capital structure is consistent with which of the following?

A. Minimum cost of equity.


B. Maximum earnings per share (EPS).
C. Minimum cost of debt.
D. Minimum risk.
E. Minimum weighted average cost of capital (WACC).

Answer: E

Explanation:
The target capital structure is the mix of debt, preferred stock, and common equity with which the
firm plans to raise capital.

Question: 2313

Consider the following three projects:


Project A
Initial cash outflow: $1,000,000
Cash inflows as follows
t1: $500,000
t2: $450,000
t3: $150,000
t4: $150,000
t5: $150,000
Project B
Initial cash outflow: $1,000,000

Page | 857
Cash inflows as follows
t1: $150,000
t2: $150,000
t3: $150,000
t4: $450,000
t5: $500,000
Project C
Initial cash outflow $1,000,000
Cash inflows as follows
t1: $280,000
t2: $280,000
t3: $280,000
t4: $280,000
t5: $280,000
Assuming no taxes, an 8.5% cost of capital, along with a $0.00 salvage value at the end of the fifth
year, what is the NPV of each project? Additionally, which of the three projects has the steepest NPV
profile?

A. Project A NPV: $276,837; Project B NPV: $40,334; Project C NPV: $103,380; Project A has a
steepest NPV profile
B. Project A NPV: $ 267,837; Project B NPV: $44,330, Project C NPV: $135,820; Project A has a
steepest NPV profile
C. Project A NPV: $168,513.54 Project B NPV: $40,334; Project C NPV: $103,380; Project B has a
steepest NPV profile
D. Project A NPV: $168,531.54; Project B NPV: $40,334; Project C NPV: $103,380; Project C has a
steepest NPV profile
E. Project A NPV: $168,513.54, Project B NPV: $14,550; Project C NPV: $103,380; Project B has the
steepest NPV profile
F. Project A NPV: $276,837; Project B NPV: $114,550; Project C NPV: $135,820; Project A has a
steepest NPV profile

Answer: C

Explanation:
Due to the fact that project B is characterized by having the majority of its cash inflows occurring
during later time periods, it is more sensitive to changes in the cost of capital. This fact is exemplified
by a steeper NPV profile.

Question: 2314

Intelligent Semiconductor, a diversified technology company, is considering two mutually-exclusive


projects. Assume the following information:
Project A
Initial cash outlay: ($500,000)
t1: $125,000
t2: $125,000
t3: $155,000
t4: $285,000
Cost of capital 11.35%
Project B
Initial cash outlay ($395,000)
t1: $170,000
t2: $160,000
t3: $175,000

Page | 858
Cost of capital 11.35%
Assuming no taxes, a $0.00 salvage value at the end of each projects' life, and the ability for each
project to be replicated identically, identify the superior project according to the Replacement Chain
approach. Additionally, what is the NPV and IRR of the superior project over the common life?

A. Project B, NPV $25,577.90, IRR 13.30%


B. Project A, NPV $18,954.46, IRR 10.33%
C. Project B, NPV $35, 417.16, IRR 15.67%
D. Project B, NPV $35,417.16, IRR 13.30%
E. The Replacement Chain approach cannot be applied to these projects.
F. Project A, NPV $22,256.14, IRR 12.22%

Answer: D

Explanation:
The Replacement Chain, or "Common Life" approach, is a useful analysis method which allows two
or more projects with unequal lives to be examined. In the Replacement Chain approach, the
lifespans of each project being examined are multiplied in such a way that the resulting projects
share a "common life." In this example, Project A has a lifespan of 4 periods, whereas Project B has a
lifespan of 3. The common multiple of both is 12, and to transform each project into one which has a
twelve period lifespan, multiply project A by 3 and Project B by 4. Doing so will result in the following
series of cash flows for Project A:
Project A
t0 ($500,000)
t1: $125,000
t2: $125,000
t3: $155,000
t4: [$285,000 + ($500,000)]=($215,000)
t5: $125,000
t6: $125,000
t7: $155,000
t8: [$285,000 + ($500,000)]=($215,000)
t9: $125,000
t10 $125,000
t11: $155,000
t12: $285,000
Multiplying project B by 4 will result in the following cash flows: t0 ($395,000)
t1: $170,000
t2: $160,000
t3: [$175,000 + ($395,000)]=($220,000)
t4: $170,000
t5: $160,000
t6: [$175,000 + ($395,000)]=($220,000)
t7: $170,000
t8: $160,000
t9: [$175,000 + ($395,000)]=($220,000)
t10: $170,000
t11: $160,000
t12 $175,000
Solving for NPV and IRR will determine that Project B is superior on both figures, with an NPV of
$35,417.16, and an IRR of 13.301%. Project A has a NPV of $22,256.14 and an IRR of 12.22%.

Question: 2315

Page | 859
Suppose capital gains are taxed at 32% and realized income is taxed at 38%. The tax preference
theory implies that as the dividend pay-out ratio is increased, the cost of equity:

A. increases or decreases.
B. increases.
C. remains unaffected.
D. decreases.

Answer: B

Explanation:
Since the capital gains tax rate is lower than the realized income tax rate, investors would prefer to
defer the realization of this income through the capital gains component. Hence, increasing the
payout ratio will make the stock less attractive and depress the price, raising the cost of equity.

Question: 2316

Clay Industries, a large industrial firm, is examining the capital structure of one of its Lebanese
subsidiaries. The management of Clay Industries has identified the following information:
EBIT $1,000,000
EPS $1.88
Interest paid $121,590
Sales $1,940,000
Cost of debt 6.60%
Given this information, what is the Degree of Financial Leverage for this operating division?

A. 1.940
B. 1.138
C. 1.551
D. The Degree of Financial Leverage cannot be calculated from the information provided.
E. 1.197
F. 1.063

Answer: B

Explanation:
To calculate the DFL, the financial analyst needs to determine the EBIT and interest paid for a
predetermined time period. To calculate the Degree of Financial Leverage, the following equation is
used: {EBIT/[EBIT - interest paid]}. Incorporating the given information into this equation yields the
following: {EBIT $1,000,000 / [EBIT $1,000,000 - interest paid $121,590]}= 1.138.
The Degree of Financial Leverage measures the percentage change in EPS which results from a given
percentage change in EBIT. Remember that any preferred stock dividends must be incorporated into
the DFL calculation, and that the DFL can never be less than one.

Question: 2317

Which of the following factors in the discounted cash flow approach to estimating the cost of
common equity is the least difficult to estimate?

A. All of these answers are equally difficult to estimate.


B. Expected rate of return.
C. Required return.
D. Dividend yield.
E. Expected growth rate.

Page | 860
Answer: D

Explanation:
It is easy to determine the dividend yield since the dividend and the price of the stock are known. It
is more difficult to establish a proper growth rate or beta as required in the other factors.

Question: 2318

A firm has to pay 1.5% fee to underwriters when it issues new equity. The firm has a dividend payout
ratio of 37% and a return on equity of 13.9%. The firm has just announced earnings of $3.27 per
share. If the stock's cost of external equity is 14.9%, how much capital would the firm raise by issuing
6 million shares?

A. $111.12 million
B. none of these answers
C. $98.33 million
D. $130.55 million

Answer: D

Explanation:
IF F is the percentage flotation cost and P is the amount of new equity raised per new share, then
P = D1/[(Ke - g)(1-F)], where Ke is the cost of external equity and D1 is next year's expected dividend.
Also, g = ROE*retention ratio = ROE*(1-payout ratio) = 13.9%*(1-37%) = 8.76%. D1 =
3.27*0.37*(1+8.76%) = $1.316. Therefore, P = 1.316/[(1-1.5%)*(14.9% - 8.76%) = $21.76. By issuing 6
million shares, the firm will therefore raise 6*21.74 = $130.55 million.

Question: 2319

Which of the following statements is most correct?

A. None of the answers are correct.


B. The modified internal rate of return (MIRR) can never exceed the IRR.
C. All of the answers are correct.
D. If the IRR of Project A exceeds the IRR of Project B, then Project A must also have a higher NPV.
E. If a project with normal cash flows has an IRR which exceeds the cost of capital, then the project
must have a positive NPV.

Answer: E

Explanation:
The IRR is the discount rate at which a project's NPV is zero. If a project's IRR exceeds the firm's cost
of capital, then its NPV must be positive, since NPV is calculated using the firm's cost of capital to
discount project cash flows.

Question: 2320

Ace Consulting, a multinational corporate finance consulting firm, is performing an analysis of the
East Asian distribution network of Smith, Kleen, and Beetchnutty. Specifically, Ace Consulting is trying
to identify the effect of changes in specific variables on the overall efficiency of SKB's distribution
process. In their analysis, Ace Consulting identified a "base case" situation using the expected values
for each input. Then, Ace modified each variable a few points above and below the base case,
holding other variables constant. This was done in an effort to determine the effect of each variable

Page | 861
on the overall efficiency of SKB's distribution process. Which of the following choices correctly
describes this stand-alone risk measurement technique?

A. Monte Carlo simulation


B. Scenario analysis
C. Case study analysis
D. Regression analysis
E. Sensitivity analysis
F. Relational analysis

Answer: E

Explanation:
In this example, Ace consulting has been conducting a sensitivity analysis. This analysis begins with
the identification of a "base case" situation using expected values for each input. Then, a variable is
manipulated holding the other variables constant, in an effort to determine the sensitivity of the
output to manipulations in each variable. Sensitivity analysis is the most widely used technique for
measuring stand-alone risk, and can be performed relatively easily using a commercially available
spreadsheet package such as Microsoft Excel.

Question: 2321

The Seattle Corporation has been presented with an investment opportunity which will yield cash
flows of $30,000 per year in Years 1 through 4, $35,000 per year in Years 5 through 9, and $40,000 in
Year 10. This investment will cost the firm $150,000 today, and the firm's cost of capital is 10 percent.
Assume cash flows occur evenly during the year, 1/365th each day. What is the payback period for
this investment?

A. 4.35 years
B. 4.00 years
C. 5.23 years
D. 4.86 years
E. 6.12 years

Answer: D

Explanation:
Using the even cash flow distribution assumption, the project will completely recover initial
investment after 30/35 = 0.86 of Year 5:
Payback = 4 + 30/35 = 4.86 years.

Question: 2322

If debt financing is used, which of the following is correct?

A. The percentage change in net operating income is greater than a given percentage change in net
income.
B. The percentage change in net operating income is less than the percentage change in net income.
C. The percentage change in net operating income is equal to a given percentage change in net
income.
D. The percentage change in net operating income depends on the interest rate charged on debt.
E. The degree of operating leverage is greater than 1.

Answer: B

Page | 862
Explanation:
This is because the interest charges on debt are included in net income and not operating income.

Question: 2323

The stand-alone risk of a project is measured by:

A. the project's impact on the systematic risk of the firm's stock.


B. the project's impact on the unsystematic risk of the firm's stock.
C. the variability of the project's projected returns.
D. the project's impact on the uncertainty about the firm's future earnings.

Answer: C

Explanation:
Standalone risk evaluates the risk of a project ignoring all portfolio aspects by looking at the
variability of the project's projected returns.

Question: 2324

If a firm adheres strictly to the residual dividend policy, a sale of new common stock by the company
would suggest that ________.

A. the dividend payout ratio is decreasing


B. the dividend payout ratio has remained constant
C. the dollar amount of investments has decreased
D. the dividend payout ratio is increasing
E. no dividends were paid for the year

Answer: E

Explanation:
The residual dividend model is a model in which the dividend paid is set equal to the actual earnings
minus the amount of retained earnings necessary to finance the firm's optimal capital budget. A firm
follows 4 steps when using this model:
1. The optimal capital budget is determined.
2. The amount of equity needed to finance that budget, given its target capital structure, is
determined.
3. Retained earnings are used to meet equity requirements to the extent possible.
4. Dividends are paid only if more earnings are available than are needed to support the optimal
capital budget.
As long as the firm finances with the optimal mix of debt and equity, and provided it uses only
internally generated equity (retained earnings), then the marginal cost of each new dollar of capital
will be minimized. Internally generated equity is available for financing some new investment, but
beyond that amount, the firm must finance through more expensive common stock. At this point
where new stock must be sold, the cost of equity and the marginal cost of capital, increases.

Question: 2325

Which of the following statements is most correct?

A. Stock splits reduce the number of shares outstanding.


B. A key advantage of the residual dividend policy is that it usually results in a stable dividend policy,

Page | 863
which is attractive to investors.
C. A reduction in the capital gains rate should work to discourage corporations from repurchasing
their shares.
D. The bird-in-hand theory of dividends suggests that firms that increase their dividend payout
should expect to realize a higher share price and a lower cost of equity capital.

Answer: D

Explanation:
The main conclusion of MM's irrelevance theory is that dividend policy does not affect the required
rate of return on equity. Gordon-Lintner disagreed stating that k(s) decreases as the dividend payout
is increased because investors are less certain of receiving the capital gains which should result from
retaining earnings than they are of receiving dividends. They said that investors value expected
dividends more highly than expected capital gains because the dividend yield is less risky than the
growth component in the total expected return equation, k(s) = D1/Po + g. MM disagreed and
theorized that k(s) is independent of dividend policy, implying that investors are indifferent between
dividends and capital gains. MM called the Gordon-Lintner's study the bird-in-the- hand fallacy,
because MM thought the riskiness of the firm's cash flows to investors in the long run is determined
by the riskiness of the operating cash flows, not by dividend policy.

Question: 2326

Consider the following argument: "By selling predetermined amounts of stock in an environment of
no taxes or transaction costs, investors can create their own dividend policy. For example, a
shareholder that wants a 5% dividend can "create" it by selling 5% of her stock. Conversely, if a
company pays ahigher dividend than an investor desires, the investor can use the unwanted portion
of this dividend to purchase additional stock." This argument applies to which of the following
theories? Choose the best answer.

A. Dividend Relevance Theory


B. Tax Preference Theory
C. Trade-off Theory
D. Bird-in-hand Theory
E. Dividend Irrelevance Theory

Answer: E

Explanation:
Modigliani and Miller established the Dividend Irrelevance Theory, which stated that in an
environment of no taxes or transaction costs, dividend policy is irrelevant. Modigilani and Miller
grounded this assumption in the fact that in such an environment, investors could create their own
dividend policy by selling and buying shares of stock.

Question: 2327

Which of the following statements is most correct?

A. One advantage of adopting a residual dividend policy is that it makes it easier for corporations to
maintain dividend clienteles.
B. None of these answers are correct.
C. The clientele effect can explain why firms often change their dividend policies.
D. The bird-in-hand theory would predict that companies could decrease their cost of equity
financing by raising their dividend payout.
E. All of these answers are correct.

Page | 864
Answer: D

Explanation:
The main conclusion of MM's irrelevance theory is that dividend policy does not affect the required
rate of return on equity. Gordon-Lintner disagreed stating that k(s) decreases as the dividend payout
is increased because investors are less certain of receiving the capital gains which should result from
retaining earnings than they are of receiving dividends. They said that investors value expected
dividends more highly than expected capital gains because the dividend yield is less risky than the
growth component in the total expected return equation, k(s) = D1/Po + g. MM disagreed and
theorized that k(s) is independent of dividend policy, implying that investors are indifferent between
dividends and capital gains. MM called the Gordon-Lintner's study the bird-in-the- hand fallacy,
because MM thought the riskiness of the firm's cash flows to investors in the long run is determined
by the riskiness of the operating cash flows, not by dividend policy.

Question: 2328

Which of the following methods of evaluating capital projects incorporate an explicit discount rate
into the equation?

A. Net Present Value, Payback Period


B. Internal Rate of Return, Modified Internal Rate of Return
C. Discounted Payback Period, Net Present Value, Payback Period
D. Discounted Payback Period, Net Present Value, Modified Internal Rate of Return
E. Discounted Payback Period, Net Present Value, Internal Rate of Return

Answer: D

Explanation:
Of the methods for evaluating capital projects, the Net Present Value, Modified Internal Rate of
Return, and Discounted Payback Period Methods incorporate an explicit discount rate into their
equations. This discount rate is often referred to as the "cost of capital" for the project being
examined. Remember that the Internal Rate of Return equation does not involve the incorporation
of an explicit discount rate,rather solves to find that rate which equates the present value of a
project's cash inflows with that of its cash outflows. Additionally, the "Payback Period" method does
not involve an explicit discount rate, rather fails to incorporate any form of discounting into its
calculation. The Payback Period is an overtly simplistic method, and as such, the figures produced by
this method should be viewed with a degree of caution.

Question: 2329

Which of the following figures is not explicitly incorporated into the earnings per share (EPS)
calculation?

A. Interest Expense
B. Sales
C. Fixed Costs
D. Tax Rate
E. Weighted Average Cost of Capital
F. Variable Costs

Answer: E

Explanation:

Page | 865
The WACC is not incorporated into the EPS calculation. The EPS calculation is found by the following
equation:
{EPS = [(Sales - Fixed Costs - Variable Costs - Interest Expense)(1 - Tax Rate)] / [# of Common Shares
Outstanding]}
Additionally, the EPS figure can be found by:
{EPS = [(EBIT - Interest Expense)(1 - Tax Rate) / # of Common Shares Outstanding]}.

Question: 2330

The trade-off theory of capital structure implies that:

A. firms issue debt up to the level where the total value added by the debt tax shield is offset by
expected bankruptcy costs.
B. firms will use debt up to the level where the flotation cost of new debt equals that of issuing more
equity, thus minimizing the costs of raising capital.
C. managers are uncomfortable with either too much debt or too much equity and hence, tend to
choose debt ratios around 0.40 to 0.60.
D. none of these answers.

Answer: D

Explanation:
According to the trade-off theory of capital structure, firms issue debt up to the level where the
additional value added by the debt tax shield for another dollar of capital raised is offset by expected
bankruptcy costs. This ensures that with only these two effects, the firm's stock price is maximized.
Clearly, at this point, the total value added by the debt tax shield exceeds the expected bankruptcy
costs.

Question: 2331

PQR Manufacturing Corporation has $1,500,000 in debt outstanding. The company's before-tax cost
of debt is 10 percent. Sales for the year totaled $3,500,000 and variable costs were 60 percent of
sales. Net income was equal to $600,000 and the company's tax rate was 40 percent. If PQR's degree
of total leverage is equal to 1.40, what is its degree of operating leverage?

A. 1.15
B. 1.22
C. 2.68
D. 1.12
E. 1.00

Answer: B

Explanation:
First, calculate PQR's DFL as EBIT/(EBIT - I). Interest expense (I) on the debt is $1,500,000(10%) =
$150,000. We can work backwards from NI to find EBIT as follows: EBT = NI/(1 - T) or $600,000/0.6 =
$1,000,000. EBIT = EBT + I or $1,000,000 + $150,000 = $1,150,000. DFL is thus
$1,150,000/($1,150,000- $150,000) = 1.15. Recognizing DTL = DFL x DOL, we can solve 1.40 = 1.15 x
DOL for DOL = 1.22.

Question: 2332

Which of the following statements is most correct?

Page | 866
A. None of these answers.
B. All else equal, an increase in a company's stock price will increase the marginal cost of retained
earnings.
C. All of these answers.
D. All else equal, an increase in a company's stock price will increase the marginal cost of issuing new
common equity.
E. If a company's tax rate increases, but the yield to maturity of its noncallable bonds remains the
same, the company's marginal cost of debt capital will fall.

Answer: E

Explanation:
The debt cost used to calculate a firm's WACC is k(d)(1 - T).
If k(d) remains constant but T increases, then the term (1 - T) decreases and the value of the entire
equation, k(d)(1 - T), decreases.
k(d)(1 - T) = after-tax component cost of debt, where T is the firm's marginal tax rate.

Question: 2333

Which of the following projects is likely to produce multiple Internal Rates of Return.
Project A
Initial investment outlay: ($1,000,000)
t1: $0.00
t2: $0.00
t3: $0.00
t4: $0.00
t5: $0.00
t6: $10,000,000
Project B
Initial investment outlay: ($1,000,000)
t1: $500,000
t2: $500,000
t3: $500,000
t4: $0.01
Project C
Initial investment outlay: ($1,000,000)
t1: $800,000
t2: ($100,000)
t3: $550,000
Project D
Initial investment outlay: ($500,000)
t1: $400,000
t2: ($1,000)
t3: $230,000
t4: ($50,000)

A. Project D
B. Project A, C and D
C. Project A
D. Project B
E. Project C and D
F. Project C

Answer: F

Page | 867
Explanation:
In evaluating projects with "non-normal cash flows" the Internal Rate of Return method will often
produce multiple IRRs calculation which leads to an incorrect accept/reject decision. Non-normal
cash flows are defined as cash flows in which the sign changes more than once. Projects C and D
involve cash outflows superimposed within their cash inflows, resulting in a sign change from
positive to negative and negative to positive. In examining projects such as this, it is advisable to use
either the NPV or MIRR methods, which are not subject to the problem of multiple IRRs associated
with the traditional IRR method. From observation alone, we can determine that project C and D are
non-normal projects, and are thus likely to result in multiple IRRs. While project A is somewhat
unusual in the fact that the first five periods produce no cash flows at all, there is only one sign
change present in its cash flows, and thus is characterized as a "normal" project.

Question: 2334

While evaluating a project using net income figures, you must:

A. subtract all non-cash net expenses.


B. none of these answers.
C. add back all non-cash net expenses.
D. add back depreciation.

Answer: C

Explanation:
In capital budgeting, annual cash flows, not accounting income, are used to evaluate a project.
Hence, you must add back all non-cash net expenses, defined as total non-cash expenses - total non-
cash revenues. You must also ignore financing costs.

Question: 2335

Byron Corporation's present capital structure, which is also its target capital structure, is 40 percent
debt and 60 percent common equity. Next year's net income is projected to be $21,000, and Byron's
payout ratio is 30 percent. The company's earnings and dividends are growing at a constant rate of 5
percent; the last dividend was $2.00; and the current equilibrium stock price is $21.88. Byron can
raise all the debt financing it needs at 14 percent. If Byron issues new common stock, a 20 percent
flotation cost will be incurred. The firm's marginal tax rate is 40 percent. Assume that at one point
along the marginal cost of capital schedule the component cost of equity is 18 percent. What is the
Weighted Average Cost of Capital (WACC) at that point?

A. 16.4%
B. 14.2%
C. 10.8%
D. 13.6%
E. 18.0%

Answer: B

Explanation:
MCC (Marginal Cost of Capital) = 0.4(0.14)(1 - 0.4) + 0.6(0.18) = 0.142 = 14.2%.

Question: 2336

Consider the following information for a company.

Page | 868
Common Stock Price $53.25
Preferred Stock Par Price $100
Preferred Dividend $10
Debt Rating BB+
Owners Equity 25%
Preferred Stock Flotation Cost 2.5%
The Preferred Stock is issued at Par
Calculate the component cost of this newly issued preferred stock.

A. 10%
B. 2.5%
C. 18.78%
D. 12.5%
E. 10.26%

Answer: E

Explanation:
The component cost of preferred stock is the dividend divided by issue price minus floatation cost. In
this case the component cost of preferred stock = $10 / (100 - 2.5) = 10.26%.

Question: 2337

Industries which are cyclical and heavily oriented toward research tend to have high levels of
________. Industries which are subject to high levels of lawsuits tend to high levels of ________.

A. debt; equity
B. equity; equity
C. equity; debt
D. debt; debt

Answer: B

Explanation:
Cyclical business conditions, high dependence on research and development and higher expected
liabilities are all indicative of high business risks. This makes firms more risk-averse to debt.

Question: 2338

The WACC of a firm equals 10.67%. The pre-tax cost of debt equals 8.4%, the firm pays 38% taxes and
the firm's equity holders expect a rate of return of 17%. The firm's debt-to-equity ratio equals
________.

A. 1.41
B. 0.72
C. 1.16
D. 0.86

Answer: C

Explanation:
Let E/(D+E) =
A. Then,
WACC = (1-A)*(1-t)*RD + A*RE, where t is the tax rate. Therefore,

Page | 869
10.67% = (1-A)*(1-38%)*8.4% + A*17%. Hence, A = E/(D+E) = 0.463 and (D+E)/E = 1/0.463 = 2.16.
This gives D/E = 1.16.

Question: 2339

Which of the following are methods of estimating a company's Cost of Retained Earnings?
I. CAPM
II. CANSLIM
III. DCF Method
IV. Bond-Yield-plus-Risk-Premium
V. Least Cost Debt vs. Equity

A. I, III, IV, & V


B. I, II, III, IV, & V
C. I, III & IV
D. II only
E. I only
F. I & III

Answer: C

Explanation:
The Capital Asset Pricing Model (CAPM), Discounted Cash Flow (DCF method), and Bond-Yield-plus-
Risk-Premium methods may all be used to estimate a firm's Cost of Retained Earnings.

Question: 2340

Ameriscam, Inc. is considering the issuance of some junior subordinated debt. The Company's
combined state/federal corporate tax rate is 30%, and the coupon on its outstanding senior debt is
7.55%. The proposed debt would pay an annual coupon. Very recently, Ameriscam has met with a
corporate finance firm, who advised the Company that the pre-tax cost of a debt issuance would be
7.70%; Ameriscam's finance division mirrored these findings. A month earlier, a study in a popular
financial magazine found that shareholder's require a 7.95% rate of return on similar investments.
Which of the following represents the best answer for Ameriscam's estimated after-tax cost of debt
for this proposed debt issuance?

A. 5.425%
B. The after-tax cost of debt cannot be determined from the information provided.
C. 5.565%
D. 5.39%
E. 5.285%

Answer: D

Explanation:
Remember that in calculating the after-tax cost of debt for new debt issues, the MARGINAL cost of
issuing new debt is the most relevant measure. In this example, the marginal cost of debt is given as
7.70%, and the after-tax cost of debt capital is found by multiplying (1 - corporate tax rate) by this
figure. Specifically, the calculation of the after-tax cost of debt for Ameriscam is as follows: {pre-tax
cost of issuing new debt 7.70%
* [1-combined state/federal tax rate 30%]} = 5.39%. While the coupon rate on the firm's outstanding
senior debt is often used as a proxy, this number is significantly inferior to the use of marginal
figures. What is relevant in this example is the cost of new debt, and this can best be approximated
by using the marginal cost of a new debt issue. Often, in capital budgeting and related corporate

Page | 870
finance decisions, the only available figures are related to the cost and yield of outstanding debt, and
in these instances, using the cost of outstanding debt is very appropriate. However, in this example,
using the cost of existing debt does not yield the best possible answer.

Question: 2341

Suppose capital gains are taxed at 28% and realized income is taxed at 40%. The tax preference
theory implies that as the dividend pay-out ratio is increased, the stock price:

A. increases or decreases.
B. decreases.
C. increases.
D. remains unaffected.

Answer: B

Explanation:
Since the capital gains tax rate is lower than the realized income tax rate, investors would prefer to
defer the realization of this income through the capital gains component. Hence, increasing the
payout ratio will make the stock less attractive and depress the price.

Question: 2342

The Present Value of a project's cash flows when its cost of capital equals its internal rate of return :

A. equals zero.
B. is positive.
C. is negative.
D. could be all of these answers.

Answer: B

Explanation:
The IRR is by definition the discount rate at which the NPV = 0. Therefore, at this point, the PV is
greater than zero, since the initial outlay is always non-zero and NPV = PV - cash outlay.

Question: 2343

J. Ross and Sons Inc. has a target capital structure that calls for 40 percent debt, 10 percent preferred
stock, and 50 percent common equity. The firm's current after-tax cost of debt is 6 percent, and it can
sell as much debt as it wishes at this rate. The firm's preferred stock currently sells for $90 a share
and pays a dividend of $10 per share; however, the firm will net only $80 per share from the sale of
new preferred stock. Ross expects to retain $15,000 in earnings over the next year. Ross' common
stock currently sells for $40 per share, but the firm will net only $34 per share from the sale of new
common stock. The firm recently paid a dividend of $2 per share on its common stock, and investors
expect thedividend to grow indefinitely at a constant rate of 10 percent per year. What will be the
WACC above the break point?

A. 11.9%
B. 8.3%
C. 12.5%
D. 14.1%
E. 10.6%That answer is correct!

Page | 871
Answer: A

Explanation:
Preferred stock return: 10/80 = 12.5% WACC = 6%(0.40) + 12.5%(0.10) + 16.5%(0.50) = 11.90%.

Question: 2344

Polk Products is considering an investment project with the following cash flows:
tCash Flow
0-100,000
140,000
290,000
330,000
460,000
The company has a 10 percent cost of capital. What is the project's discounted payback?

A. 2.67 years
B. 1.86 years
C. 2.49 years
D. 1.67 years
E. 2.11 years

Answer: B

Explanation:
The PV of t = 1 CF is found as follows: N = 1, I = 10, PMT = 0, FV = 40,000, and, thus PV = - $36,363.64.
Similarly, find the PV of t = 2 CF which is $74,380.17. Since the sum of these PVs is greater than the t
= 0 CF of $100,000, we know the discounted payback is less than two years. We can now solve for the
discounted payback period as follows: DP = 1 + ($100,000 - $36,363.64)/$74,380.17 = 1.86.

Question: 2345

Your company is choosing between the following non-repeatable, equally risky, mutually exclusive
projects with the cash flows shown below. Your cost of capital is 10 percent. How much value will
your firm sacrifice if it selects the project with the higher IRR? Project S:
0123
-1,000500500500
Project L:
012345
-2,000668.76668.76668.76668.76668.76

A. $481.15
B. $291.70
C. $332.50
D. $243.43
E. $535.13

Answer: B

Explanation:
NPV(S) = $243.43; IRR(S) = 23.38%.
NPV(L) = $535.13; IRR(L) = 20.00%.
Value sacrificed: $535.13 - $243.43 = $291.70.

Page | 872
Question: 2346

Simmons Shoes is considering a project with the following cash flows:


TimeProject Cash Flows ($)
0-700
1400
2-200
3600
4500
Simmons' WACC is 10 percent. What is the project's modified internal rate of return (MIRR)?

A. 28.93%
B. 17.10%
C. 18.26%
D. 29.52%
E. 25.28%

Answer: C

Explanation:
There are three steps to getting an MIRR:
1. Find PV of outflows:
-$700 + -$200/(1.1)^2 = -$865.2893.
2. Find FV of inflows:
$400 (1.1)^3 + $600(1.1) + $500 = $1,692.40.
3. Find MIRR:
N=4
PV = -865.2893
PMT = 0
FV = 1,692.40
Solve for I = MIRR = 18.2593%.

Question: 2347

Consider the following information for Company XYZ:


Current Price of Stock $35.00
Expected dividend in 1 Year $1.20
Growth rate 7.2%
Beta 1.6
Risk Free Rate 4.5%
Expected Market Return 15%
Marginal Corporate Tax rate 34%
Bond Yield 12.34%
Calculate this company's cost of retained earnings using the Discounted Cash Flow (DCF) method.

A. 10.63%
B. 11.52%
C. 13.30%
D. 9.20%
E. 12.0%
F. 21.30%That answer is correct!

Answer: A

Page | 873
Explanation:
The DCF method for estimating the cost of retained earnings states: Cost of Retained Earnings =
(Dividend for period 1 / Current Price) + Growth Rate. In this case the estimated Cost of Retained
Earnings = (1.2 / 35.00) + 7.2% = 3.43 + 7.2 = 10.63%

Question: 2348

Sensitivity Analysis ignores:

A. the range of likely values that key variables can take.


B. changes in some of the key variables.
C. none of these answers.
D. effect on the IRR of changes in project variables.That answer is correct!

Answer: A

Explanation:
One of the drawbacks of Sensitivity Analysis is that it ignores the range of likely values that key
variables can take. This is rectified using Scenario Analysis.

Question: 2349

You have recently accepted a one-year employment term by a firm. The firm has given you the
option of receiving your salary as a lump sum value of $30,000 at the end of the year or as 12
monthly payments of $2,400 starting one month after you start work. If your relevant discount rate is
2 percent per month, then which salary options would you prefer? (Ignore taxes, risk, and
consumption needs.)

A. Monthly payments, since you do not have to wait so long to receive your money.
B. Monthly payments, since it has the larger present value.
C. The lump sum payment, since it has the larger present value.
D. Either one, since they have the same present value.
E. The lump sum payment, since it has the larger future value.

Answer: B

Explanation:
Monthly option PV = $2,400(PVIFA(2%,12)) = $2,400(10.5753) = $25,380.72.
Annual option PV = $30,000(PVIF(2%,12)) = $30,000(0.7885) = $23,655.

Question: 2350

Which of the following is most likely to encourage a company to use more debt in its capital
structure?

A. All of these answers are correct.


B. An increase in unit production.
C. An increase in the corporate tax rate.
D. An increase in the company's degree of operating leverage.
E. An increase in the personal tax rate.

Answer: C

Explanation:

Page | 874
A major reason for using debt is that interest is deductible, which lowers the effective cost of debt.

Question: 2351

A firm has a dividend growth rate of 2.8%. It typically pays out 48% of its earnings as dividends.
Recently, it paid out $2.4 per share dividend and the required rate of return on its stock is 13%. The
firm's return on equity equals ________.

A. 5.83%
B. 5.38%
C. insufficient information
D. 12.19%

Answer: B

Explanation:
g = ROE*(1 - dividend payout ratio). Therefore, ROE = 0.028/(1 - 0.48) = 5.38%.

Question: 2352

Which of the following statements is most correct?

A. None of the statements are correct.


B. The discounted payback method solves all the problems associated with the payback method.
C. The NPV method is appealing to some managers because it produces a dollar amount upon which
to base decisions rather than a IRR method.
D. All of the statements are correct.
E. For independent projects, the decision to accept or reject will always be the same using either the
IRR method or the NPV method.

Answer: E

Explanation:
For mutually exclusive projects, a conflict can exist if the cost of capital is less than the crossover rate.

Question: 2353

Adams Audio is considering whether to make an investment in a new type of technology. Which of
the following factors should the company consider when it decides whether to undertake the
investment?

A. None of these factors should be considered.


B. The installation costs for the new equipment for the new technology are very high.
C. The new technology will affect the cash flows produced by its other operations.
D. If the investment is not made, then the company will be able to sell one of its laboratories for $2
million.
E. All of these factors should be considered.

Answer: E

Explanation:
These are all incremental cash flows which change the firm's total cash flow that occurs as a direct
result of accepting the project, and should all be considered

Page | 875
Question: 2354

Which of the following statements about capital structure theory is most correct?

A. In general, an increase in the corporate tax rate would cause firms to use less debt in their capital
structures.
B. Signaling theory suggests firms should in normal times maintain reserve-borrowing capacity which
can be used if an especially good investment opportunity comes along.
C. All of the statements are correct.
D. None of the statements are correct.
E. According to the "trade-off theory," a decrease in the costs of debt would lead firms to increase
equity financing in their capital structures.

Answer: B

Explanation:
An increase in the corporate tax rate reduces the after-tax cost of debt making it more attractive
relative to equity. Thus, firms might be expected to use more debt. The trade-off theory of leverage
states a firm trades off the benefits of debt financing (favorable corporate tax treatment) against the
higher interest rates and bankruptcy costs.

Question: 2355

Moynihan Motors has a cost of capital of 10.25 percent. The firm has two normal projects of equal
risk. Project A has an internal rate of return of 14 percent, while Project B has an internal rate of
return of 12.25 percent. Which of the following statements is most correct?

A. If the crossover rate (i.e., the rate at which the Project's NPV profiles intersect) is 8 percent,
Project A will have a lower net present value than Project B.
B. All of these answers are correct.
C. If the projects are mutually exclusive, the firm should always select Project A.
D. None of these answers are correct.
E. Both projects have a positive net present value.

Answer: E

Explanation:
If the projects were independent, both should be accepted. They both have an IRR greater than the
cost of capital, so they have positive NPVs. If the cost of capital were above 14%, both projects
should be rejected. Project B will have a higher NPV at discount rates below 8% and Project A will
have a higher NPV at discount rates above 8%.

Question: 2356

Empirical testing has confirmed the validity of which of the following dividend theories?

A. Tax differential theory.


B. Empirical testing has not produced any definitive results.
C. Empirical testing has produced some evidence in support of each of these theories.
D. Dividend irrelevance, or Modigliani-Miller, theory.
E. Bird-in-the-hand theory.

Answer: C

Page | 876
Explanation:
These 3 theories have produced unclear empirical tests because of two reasons: 1. For a valid
statistical test, things other than dividend policy must be held constant, and 2. We must be able to
measure with a high degree of accuracy each sample firm's cost of equity. Neither of these two
conditions holds.

Question: 2357

A firm's earnings break point equals $98 million. Its net income is $58 million and it is committed to a
dividend payout ratio of 30%. It's after-tax cost of debt equals 9% and its shareholders demand an
expected rate of return of 15%. The firm's WACC equals ________.

A. 12.2%
B. 9.8%
C. 11.5
D. 10.3%

Answer: C

Explanation:
The retained earnings of the firm = $58*0.7 = $40.6 million. If the earnings breakpoint is $98 million
then the firm must issue $(98-40.6) = $57.4 million in debt to maintain constant D/E ratio. This
implies that the firm's D/E ratio equals 57.4/40.6 = 1.41. Debt comprises 57.4/98 = 58.57% of the
capital structure. Therefore, WACC = 0.5857*0.09 + 0.4143*15% = 11.49%.

Question: 2358

Assume a project has normal cash flows (that is, the initial cash flow is negative, and all other cash
flows are positive). Which of the following statements is most correct?

A. All else equal, a project's IRR increases as the cost of capital declines.
B. All else equal, a project's NPV decreases as the cost of capital declines.
C. All else equal, a project's NPV increases as the cost of capital declines.
D. All else equal, a project's MIRR is unaffected by changes in the cost of capital.
E. None of the answers are correct.

Answer: C

Explanation:
Since the cost of capital is in the denominator of the NPV formula, the lower the cost of capital the
higher the NPV. The more positive the NPV, the more cash the project is generating to service its
debt and to provide the required return to shareholders.

Question: 2359

Conrad Corp. has an investment project with the following cash flows:
TimeProject Cash Flows
0-$1,000
1200
2-300
3900
4-700
5600
The company's WACC is 12 percent. What is the project's modified internal rate of return (MIRR)?

Page | 877
A. 5.68%
B. 3.95%
C. 6.83%
D. 3.20%
E. 2.63%

Answer: B

Explanation:
Find the present value of the outflows:
t = 0: -1,000
t = 2: N = 2, I = 12, PMT = 0, FV = 300, and solve for PV = -$239.1582.
t = 4: N = 4, I = 12, PMT = 0, FV = 700, and solve for PV = -$444.8627.
Total PV(Costs) = -$1,000 - $239.1582 - $444.8627 = -$1,684.0209.
Find the future value of the inflows:
t = 1: N = 4, I = 12, PV = -200, PMT = 0, and solve for FV = $314.7039.
t = 3: N = 2, I = 12, PV = -900, PMT = 0, and solve for FV = $1,128.96.
t = 5: N = 0, I = 12, PV = -600, PMT = 0, and solve for FV = $600.
Total FV(Inflows) = $314.7039 + $1,128.96 + $600 = $2,043.6639.
Then find the MIRR:
N=5
PV = -1,684.0209
PMT = 0
FV = 2,043.6639
Solve for MIRR = I = 3.9471%.

Question: 2360

The management of Olively.com, an online research network, are considering becoming a public
company. At a lengthy meeting with the board of directors, the CEO of Olively.com details his idea as
to methods in which the firm should raise capital. In his discussion, the CEO states that "45% of new
capital should come from debt, leaving 55% to come from the issuance of common equity. We will
disregard issuing preferred stock at this point." In the simplistic sense, the CEO of Olively.com is
detailing which of the following?

A. Optimal capital structure


B. Target capital structure
C. Target asset base
D. Capital asset base
E. Optimal asset base

Answer: B

Explanation:
In this example, the CEO of Olively.com is detailing his idea of the company's target capital structure.
The target capital structure can best be thought of as the proportion of debt, common stock, and
preferred stock that the firm plans to issue in its effort to raise capital. The "optimal capital structure"
is defined as the capital structure that balances risk and return, thereby maximizing the firm's stock
price.

Question: 2361

The corporate finance division of Intelligent Semiconductor is examining the firm's recent sales in an

Page | 878
attempt to forecast future operating performance. In their investigation, the management of the
firm's corporate finance division have identified the following sales and EBIT information for the
previous two years:
Sales in year 1 $1,200,000
Sales in year 2 $1,500,000
EBIT in year 1 $400,000
EBIT in year 2 $550,000
Given this information, what is the degree of operating leverage for Intelligent Semiconductor for
this period?

A. .350
B. 1.25
C. .3667
D. .3333
E. 1.50

Answer: E

Explanation:
To calculate the degree of operating leverage (DOL), use the following equation: {% change in EBIT/%
change in sales}. Incorporating the given information into this equation yields the following: {[(year 2
EBIT $550,000 - year 1 EBIT $400,000)/year 1 EBIT $400,000]/[(year 2 sales $1,500,000- year 1 sales
$1,200,000)/year 1 sales $1,200,000] = 1.50

Question: 2362

Which of the following is/are true?


I. Discounted payback period and simple payback period can produce conflicting project rankings.
II. Independent projects are mutually exclusive.
III. The payback period rule ignores cash flows beyond the payback period.

A. I, II & III
B. II only
C. I & III
D. I & II
E. I only
F. II & III
G. III only

Answer: C

Explanation:
Independent projects are ones whose cash flows are not dependent on each other while mutually
exclusive projects are those which cannot be undertaken simultaneously. It is not necessary for
independent projects to be mutually exclusive or vice versa.

Question: 2363

Which of the following types of risk measures the variability of an asset's expected returns, assuming
that the asset is not the only asset of the company in question while at the same time not taking into
consideration the effects of shareholder diversification? Choose the best answer

A. Beta coefficient
B. Unsystematic risk

Page | 879
C. Market risk
D. More than one of these answers is correct
E. Corporate risk
F. Alpha risk

Answer: E

Explanation:
Corporate risk is defined as the variability of an assets expected returns without taking into
consideration the effects of shareholder diversification. This is one step away from Stand-alone Risk,
which measures the risk of an asset not only without taking into consideration the effect of
shareholder diversification, but of Company diversification as well. Stand-alone risk assumes that the
asset in question is the only asset of the firm and that the securities of the firm are the only assets in
investors' portfolios. Corporate risk takes into consideration that firms will diversify their asset bases.

Question: 2364

Which of the following factors affect(s) a firm's optimal pay-out ratio?


I. The availability and cost of external capital.
II. The investment opportunities available.
III. The firm's target debt-to-equity ratio.
IV. Investors' preference for dividends versus capital gains.

A. II only
B. I only
C. I, II, III & IV
D. IV only
E. I, II & III
F. I & III
G. III only
H. III & IV

Answer: C

Explanation:
A firm must consider all of these factors while determining what fraction of the earnings it should
pay out. It should be noted that another factor that must be considered is the capability of keeping
the dividends stable over time.

Question: 2365

A company is considering a project with the following cash flows:


TimeCash flow
0-$100,000
150,000
250,000
350,000
4-10,000
The project's cost of capital is estimated to be 10 percent. What is the modified internal rate of
return (MIRR)?

A. 11.25%
B. 20.34%
C. 14.25%

Page | 880
D. 11.56%
E. 13.28%

Answer: C

Explanation:
First, calculate the present value of costs:
N = 4, I/YR = 10, PMT = 0, FV = -10,000, and solve for PV = -$6,830.13.
Add -$100,000 + -6,830.13 = -$106,830.13.
Then, find the terminal value of inflows:
Shift to BEGIN MODE, N = 3, I/YR = 10, PV = 0, PMT = -50,000, and solve for FV = $182,050.
Finally, shift back to END mode, and solve for MIRR, where N = 4, PV = $-106,830, PMT = 0, FV =
182,050, and solve for I/YR = 14.25%.

Question: 2366

An entrepreneur has invested $2.2 million in project A with an NPV of $245,000 and an estimated
beta of 0.59. She has invested another $3.7 million in project B with an NPV of $320,000 and an
estimated beta of 1.23. The firm's estimated beta equals ________.

A. 1.11
B. 0.72
C. 1.23
D. 0.99

Answer: D

Explanation:
The market value of project A equals $2.2 million + $245,000 = $2.445 million.
The market value of project B equals $3.7 million + $320,000 = $4.02 million.
The firm can be considered a portfolio of 2 projects. The beta of a portfolio equals the weighted
average of the betas of the individual components. The weight of a component equals the fraction of
the market value it comprises. Therefore, the firm's market value equals 2.445 + 4.02 = $6.465
million and its beta equals 2.445/6.465*0.59 + 4.02/6.465*1.23 = 0.99.

Question: 2367

Consider the following information:


Borrowing Rate 10%
Marginal Tax Rate 40%
Preferred Stock Par Price $100
Preferred Dividend $10
Preferred Stock floatation cost 2.5%
Cost of common equity 12.0%
Preferred Stock issued at Par
The Optimal Capital Structure is 40% debt, 50% common equity, and 10% preferred stock. Credit
Rating BB+ What is the firm's Weighted Average Cost of Capital (WACC)?

A. 12.62%
B. 7.42%
C. 9.0%
D. 8.0%
E. 2.5%
F. 9.42%

Page | 881
Answer: F

Explanation:
The firm's Weighted Average Cost of Capital (WACC) is a weighted average of the component cost of
capital. In this case 10%(borrowing rate) x (1-.4)Tax savings = 6% is the component cost of debt. $10
(preferred dividend) / 97.5(Par minus floatation cost) = 10.25% is the component cost of preferred
stock. Thus the WACC = .4(6%) + .5(12%) + .1(10.25%) = 9.42%

Question: 2368

Which of the following statements is most correct?

A. None of these statements are correct.


B. If a firm finds that the cost of debt financing is currently less than the cost of equity financing, an
increase in its debt ratio will always reduce its cost of capital.
C. A firm can use retained earnings without paying a flotation cost. Therefore, while the cost of
retained earnings is not zero, the cost of retained earnings is generally lower than the after-tax cost
of debt financing.
D. The capital structure which minimizes the firm's cost of capital is also the capital structure which
maximizes the firm's stock price.
E. The capital structure which minimizes the firm's cost of capital is also the capital structure which
maximizes the firm's earnings per share.

Answer: D

Explanation:
The optimal capital structure is the one that maximizes the price of the firm's stock, and this
generally calls for a debt ratio which is lower than the one that maximized expected EPS.

Question: 2369

J. Ross and Sons Inc. has a target capital structure that calls for 40 percent debt, 10 percent preferred
stock, and 50 percent common equity. The firm's current after-tax cost of debt is 6 percent, and it can
sell as much debt as it wishes at this rate. The firm's preferred stock currently sells for $90 a share
and pays a dividend of $10 per share; however, the firm will net only $80 per share from the sale of
new preferred stock. Ross expects to retain $15,000 in earnings over the next year. Ross' common
stock currently sells for $40 per share, but the firm will net only $34 per share from the sale of new
common stock. The firm recently paid a dividend of $2 per share on its common stock, and investors
expect the dividend to grow indefinitely at a constant rate of 10 percent per year. What is the firm's
cost of retained earnings?

A. 15.5%
B. 12.5%
C. 10.0%
D. 18.0%
E. 16.5%That answer is correct!

Answer: A

Explanation:
k(s) = $2.20/$40 + 0.10 = 15.5%.

Question: 2370

Page | 882
In applying the CAPM (Capital Asset Pricing Model) to estimate the cost of equity capital, which of
the following elements is not subject to dispute or controversy?

A. Expected rate of return on the market.


B. All of these answers are subject to dispute.
C. The stock's beta coefficient.
D. Market risk premium.
E. Risk-free rate.

Answer: B

Explanation:
Under the CAPM approach, it is difficult at best, to obtain correct estimates of the inputs required to
make it operational:
1. there is controversy about whether to use long-term or short-term Treasury yields for the risk-free
rate.
2. it is difficult to estimate the beta that investors expect to firm to have in the future, and it is
difficult to estimate the market risk premium.

Question: 2371

Flavortech Inc. expects EBIT of $2,000,000 for the coming year. The firm's capital structure consists of
40 percent debt and 60 percent equity, and its marginal tax rate is 40 percent. The cost of equity is 14
percent, and the company pays a 10 percent rate on its $5,000,000 of long-term debt. One million
shares of common stock are outstanding. In its next capital budgeting cycle, the firm expects to fund
one large positive NPV project costing $1,200,000, and it will fund this project in accordance with its
target capital structure. If the firm follows a residual dividend policy and has no other projects, what
is its expected dividend payout ratio?

A. 0%
B. 40%
C. 60%
D. 100%
E. 20%

Answer: E

Explanation:
EBIT$2,000,000
- I500,000 ($5,000,000 debt x 10% coupon)
EBT1,500,000
- Taxes600,000 ($1,500,000 EBT x 40% tax rate)
NI900,000
Project funding $1,200,000 project funded:
Residual earnings 0.60 equity = $720,000
0.40 debt = $480,000
Payable as dividends: 900,000 - 720,000 = $180,000
Dividend payout ratio = $180,000/$900,000 = 20%.

Question: 2372

Myron Gordon and John Lintner believe that the required return on equity increases as the dividend
payout ratio is decreased. Their argument is based on the assumption that

Page | 883
A. investors require that the dividend yield and capital gains yield equal a constant.
B. investors are indifferent between dividends and capital gains.
C. investors value a dollar of expected capital gains more highly than a dollar of expected dividends
because of the lower tax rate on capital gains.
D. investors view dividends as being less risky than potential future capital gains.
E. capital gains are taxed at a higher rate than dividends.

Answer: D

Explanation:
The main conclusion of MM's irrelevance theory is that dividend policy does not affect the required
rate of return on equity. Gordon-Lintner disagreed stating that k(s) decreases as the dividend payout
isincreased because investors are less certain of receiving the capital gains which should result from
retaining earnings than they are of receiving dividends. They said that investors value expected
dividends more highly than expected capital gains because the dividend yield is less risky than the
growth component in the total expected return equation, k(s) = D1/Po + g.

Question: 2373

Zippy Pasta Corporation (ZPC) has a constant growth rate of 7 percent. The company retains 30
percent of its earnings to fund future growth. ZPC's expected EPS and k(s) for various capital
structures are given below. What is the optimal capital structure for ZPC? Debt/AssetsExpected EPS
($)k(s)
20%2.5015.0%
303.0015.5
403.2516.0
503.7517.0
704.0018.0

A. Debt/Total Assets = 20%


B. Debt/Total Assets = 30%
C. Debt/Total Assets = 70%
D. Debt/Total Assets = 40%
E. Debt/Total Assets = 50%

Answer: E

Explanation:
The optimal capital structure maximizes the firm's stock price. When the debt ratio is 20%, expected
EPS is $2.50. Given the firm's policy of retaining 30% of earnings, the expected dividend per share D1
is $2.50 x 0.70 = $1.75. The stock price Po is $1.75/(15% - 7%) or $21.88. When the debt ratio is 30%,
expected EPS is $3.00 and expected D1 is $3.00 x 0.70 = $2.10. The stock price Po is $2.10/(15.5% -
7%) =$24.71. Similarly, when the debt ratio is 40%, D1 = $2.275 and Po = $25.28. When the debt ratio
is 50%, D1 = $2.625 and Po = $26.25. When the debt ratio is 70%, D1 = $2.80 and Po = $25.45. The
stock price is highest when the debt ratio is 50%.

Question: 2374

A portfolio manager with Mally, Feasance, & Company is examining shares of Allcycles.com. Assume
the following information:
Annual Dividend: $0.45
EPS: $2.15
Tax Rate: 35%

Page | 884
Discount Rate: 12.25%
ROE: 18%
Using this information, and assuming that ROE is expected to remain stable, what is the dividend
growth rate for Clay Industries?

A. 14.64%
B. 15.51%
C. 3.77%
D. 12.67%
E. The answer cannot be determined from the information provided.
F. 14.23%

Answer: F

Explanation:
To calculate the growth rate, use the following equation: {g = ROE(1 - Dividend Payout Ratio)}.
While the ROE figure has been provided, the Dividend Payout Ratio must be calculated manually. To
find the Dividend Payout Ratio, divide the annual dividend by the EPS figure, giving the following:
{Dividend Payout Ratio = ($0.45/$2.15)}
From this equation, we determine that the Dividend Payout Ratio for this firm is 20.93%. Inputting
this figure into the growth rate equation will yield a dividend growth rate of 14.23% for this firm.
As you can see, tax rates and discount rates are not factored into the calculation.

Question: 2375

Which of the following statements is correct?

A. Due to the way the MCC (Marginal Cost of Capital) is constructed, the first break point in the MCC
schedule must be associated with using up all available retained earnings and having to issue
common stock.
B. Normally, the cost of external equity raised by issuing new common stock is above the cost of
retained earnings. Moreover, the higher the growth rate is relative to the dividend yield, the more
the cost of external equity will exceed the cost of retained earnings.
C. The lower a company's tax rate, the greater the advantage of using debt in terms of lowering its
WACC.
D. Because we often need to make comparisons among firms that are in different income tax
brackets, it is best to calculate the WACC (Weighted Average Cost of Capital) on a before-tax basis.
E. If a firm has been suffering accounting losses and is expected to continue suffering such losses,
and therefore its tax rate is zero. It is possible that its after-tax component cost of preferred stock as
used to calculate the WACC will be less than its after-tax component cost of debt.

Answer: E

Explanation:
Because corporations can exclude dividends for tax purposes, preferred stock often has a market
return that is less than the issuing company's cost of debt. Then, if the issuer's tax rate is zero, its
component cost of preferred would be less than its cost of debt.

Question: 2376

A financial analyst with Smith, Kleen, & Beetchnutty is examining shares of Clay Industries for
possible investment. Assume the following information:
EPS: $4.19
ROE: 11.25%

Page | 885
Growth rate of dividends: 6.75%
Discount rate: 11.50%
Tax Rate 35%
Using this information, what is the dividend payout ratio for Clay Industries? Further, what is the
annual dividend?

A. 35.87%, $1.50
B. 60.00%, $2.51
C. 60.00%, $1.68
D. The answer cannot be determined from the information provided.
E. 40.00%, $1.68
F. 40.00%, $2.51

Answer: E

Explanation:
To determine the dividend payout ratio, the equation used to determine the growth rate of dividends
must be manipulated. This equation is originally structured as follows:
{g = ROE (1 - Dividend Payout Ratio)}
In order to determine the Dividend Payout Ratio, the equation must be rearranged to the following:
{(1 - Dividend Payout Ratio) = Growth Rate of Dividends / ROE}.
Imputing the given information into this equation will yield:
{(1 - Dividend Payout Ratio) = 0.0675/0.1125)} = 0.60
Finally, subtracting this answer from 1 will yield the answer of 40%. We must subtract the first
answer from one because the first answer represents the retention rate, i.e. the percentage of
earnings that is retained and reinvested at the firm's ROE, and not the Dividend Payout Ratio. The
retention rate and the payout ratio will always combine to equal positive one. In order to determine
the annual dividend, take the Dividend Payout Ratio, which was found to be 40%, and multiply this
figure by the Earnings Per Share calculation, which is given as $4.19. This will yield an annual
dividend of $1.676. As you can see, neither the discount rate nor the tax rate is factored into the
equation

Question: 2377

Using the Security Market Line concept in capital budgeting, which of the following is correct?

A. If two mutually exclusive projects' expected returns are both above the SML, the project with the
lower risk should be accepted.
B. If a project's expected rate of return is greater than the expected rate of return on an average
project, it should be accepted.
C. If a project's return lies below the SML, it should be rejected if it has a beta greater than the firm's
existing beta but accepted if its beta is below the firm's beta.
D. If the expected rate of return on a given capital project lies above the SML, the project should be
accepted even if its beta is above the beta of the firm's average project.

Answer: D

Explanation:
If the expected rate of return on a given capital project lies above the SML, the expected rate of
return on the project is more than enough to compensate for its risk, and the project should be
accepted. Conversely, if the project's rate of return lies below the SML, it should be rejected.

Question: 2378

Page | 886
Bouchard Company's stock sells for $20 per share, its last dividend was $1.00. Its growth rate is a
constant 6 percent, and the company would incur a flotation cost of 20 percent if it sold new
common stock. Retained earnings for the coming year are expected to be $1,000,000, and the
amount of common equity in the capital structure is 60 percent. If Bouchard has a capital budget of
$2,000,000, what component cost of common equity will be built into the WACC for the last dollar of
capital the company raises?

A. 12.15%
B. 11.80%
C. 11.30%
D. 11.45%
E. 12.63%

Answer: E

Explanation:
BP(RE) = RE/Equity fraction = $1,000,000/0.6 = $1,666,667.
Since the capital budget will be $2 million, and since all equity in the WACC (Weighted Average Cost
of Capital) beyond $1,666,667 will be external equity, the WACC of the last dollar raised will include
equity at a cost of k(e) (component cost of external equity obtained by issuing new common stock as
opposed to retaining earnings):
k(e) = $1(1.06)/$20(1 - 0.2) + 0.06 = 0.0663 + 0.06 = 0.1263 = 12.63%.

Question: 2379

A company is analyzing two mutually exclusive projects, S and L, whose cash flows are shown below:
Years01234
S-1,1009003505010
L-1,1000300500850
The company's cost of capital is 12 percent, and it can get an unlimited amount of capital at that cost.
What is the regular IRR (not MIRR) of the better project?

A. 12.00%
B. 13.09%
C. 17.46%
D. 12.53%
E. 13.88%

Answer: B

Explanation:
Time line:
0 k = 12%1234 Years
Cash flows S -1,1009003505010
NPV(S) = ? IRR(S) = ?
Cash flows L -1,1000300500850
NPV(L) = ? IRR(L) = ?
Financial calculator solution:
Calculate the NPV and IRR of each project then select the IRR of the higher NPV project Project S;
Inputs: CF(0) = -1,100; CF(1) = 900; CF(2) = 350; CF(3) = 50; CF(4) = 10; I = 12
Output: NPV(S) = 24.53; IRR(S) = 13.88%.
Project L; Inputs: CF(0) = -1,100; CF(1) = 0; CF(2) = 300; CF(3) = 500; CF(4) = 850; I = 12
Output: NPVL = 35.24; IRR(L) = 13.09%.
Project L has the higher NPV and its IRR = 13.09%.

Page | 887
Question: 2380

The market risk of a project is measured by:

A. the project's impact on the systematic risk of the firm's stock.


B. the variability of the project's projected returns.
C. the project's impact on the uncertainty about the firm's future earnings.
D. the project's impact on the unsystematic risk of the firm's stock.That answer is correct!

Answer: A

Explanation:
Remember that it is the systematic risk that you must worry about.

Question: 2381

Pickles Corp. is a company which sells bottled iced tea. The company is thinking about expanding its
operations into the bottled lemonade business. Which of the following factors should the company
incorporate into its capital budgeting decision as it decides whether or not to enter the lemonade
business?

A. All of the statements are correct.


B. If the company doesn't produce lemonade, it can lease the building to another company and
receive after-tax cash flows of $500,000 a year.
C. The company will spend $3 million to renovate a building for the proposed project.
D. If the company enters the lemonade business, its iced tea sales are expected to fall 5 percent as
some consumers switch from iced tea to lemonade.
E. None of the statements are correct.That answer is correct!

Answer: A

Explanation:
These are all incremental cash flows and should be considered.

Question: 2382

Which of the following projects is likely to have multiple Internal Rates of Return? Project A
Initial investment outlay: ($1,000,000)
t1: $400,000
t2: $100
t3: $1,000,000
t4: $1,000,000
t5: $100
t6: $0.00
Project B
Initial investment outlay: ($1,000,000)
t1: $40,000
t2: $90,000
t3: $590,000
t4: ($105,000)
t5: ($10,000)
t6: $900,000
Project C

Page | 888
Initial investment outlay: ($500,000)
t1: $100,000
t2: $100,000
t3: $100,000
t4: $100,000
t5: $0.00
t6: $500,000
Project D
Initial investment outlay: ($500,000)
t1: $105,000
t2: ($40,000)
t3: $45,000)
t4: $400,000
t5: $400,000
t6: $65,000

A. None of these answers


B. Both Project B and D likely to have multiple IRRs
C. Project C
D. Project D
E. Project B
F. Project A

Answer: B

Explanation:
In evaluating projects with "non-normal cash flows" the Internal Rate of Return method will often
produce multiple IRRs which leads to an incorrect accept/reject decision. Non-normal cash flows are
defined as cash flows in which the sign changes more than once. Projects B and D involve cash
outflows superimposed within the cash inflows, resulting in a sign change from positive to negative
and negative to positive. In examining projects such as this, it is advisable to use either the NPV or
MIRR methods. From observation alone, we can determine that project B and D are non-normal
projects, and are thus likely to result in multiple IRR calculations. While projects A and C do involve
periods of zero cash flow, this will not interfere with the IRR calculation to the extent of producing
multiple IRRs.

Question: 2383

Your company's stock sells for $50 per share, its last dividend was $2.00, its growth rate is a constant
5 percent, and the company would incur a flotation cost of 15 percent if it sold new common stock.
Net income for the coming year is expected to be $500,000, the firm's payout ratio is 60 percent, and
its common equity ratio is 30 percent. If the firm has a capital budget of $1,000,000, what
component cost of common equity will be built into the WACC for the last dollar of capital the
company raises?

A. 12.30%
B. 11.75%
C. 10.50%
D. 9.94%
E. 9.20%

Answer: D

Explanation:

Page | 889
BP(RE) = RE/Equity fraction = $500,000(0.4)/0.3 = $666,667. BP
= break point; RE = retained earnings
Since the capital budget will be $1 million, and since all equity in the WACC beyond $666,667 will be
external equity, the WACC of the last dollar raised will include equity at a cost of k(e):
k(e) = $2(1.05)/$50(1 - 0.15) + .05 = 9.94%.

Question: 2384

Consider the following characteristics of firm XYZ:


Stock price $50
Annual dividend $2
Debt rate 10%
Equity floatation cost 7%
Tax rate 40%
Preferred Stock Par value $100
What is the firm's after tax cost of debt?

A. 60%
B. 4.3%
C. 10%
D. 40%
E. 6%
F. 4%
G. 5%

Answer: E

Explanation:
A firm's after tax cost of debt may be calculated using the following formula: After Tax Cost of Debt =
Cost of Debt x (1 - Tax Rate). In this case the After Tax Cost of Debt = 10% x (1 - 40%) = 10% x 60% =
6%.

Question: 2385

The Oneonta Chemical Company is evaluating two mutually exclusive pollution control systems.
Since the company's revenue stream will not be affected by the choice of control systems, the
projects are being evaluated by finding the PV of each set of costs. The firm's required rate of return
is 13 percent, and it adds or subtracts 3 percentage points to adjust for project risk differences.
System A is judged to be a high-risk project (it might end up costing much more to operate than is
expected). System A's risk-adjusted cost of capital is

A. 16 percent; since A is more risky, its cash flows should be discounted at a higher rate, because this
correctly penalizes the project for its high risk.
B. indeterminate, or, more accurately, irrelevant, because for such projects we would simply select
the process that meets the requirements with the lowest required investment.
C. 13 percent; the firms cost of capital should not be adjusted when evaluating outflow only projects.
D. somewhere between 10 percent and 16 percent, with the answer depending on the riskiness of
the relevant inflows.
E. 10 percent; this might seem illogical at first, but it correctly adjusts for risk where outflows, rather
than inflows, are being discounted.

Answer: E

Explanation:

Page | 890
k(A) = 13% - 3% = 10%. If the cash flows are cost only outflows, and the analyst wants to correctly
reflect their risk, the discount rate should be adjusted downward (in this case by subtracting 3
percentage points) to make the discounted flows comparatively larger.

Question: 2386

Two projects being considered are mutually exclusive and have the following projected cash flows:
YearProject AProject B
0-$50,000-$50,000
115,9900
215,9900
315,9900
415,9900
515,990100,560
At what rate (approximately) do the NPV profiles of Projects A and B cross?

A. The NPV profiles of these two projects do not cross.


B. 11.5%
C. 6.5%
D. 16.5%
E. 20.0%

Answer: B

Explanation:
Solve for numerical PVIFA and PVIF and obtain corresponding interest rates from table. Project
A:50,000 = 15,990(PVIFA(Irr,5))
3.12695 = PVIFA(IrrA,5)
IRR(A) = 18%
Project B:50,000 = 100,560(PVIF(IrrB,5)0
0.49722 = PVIF(IrrB,5)
IRR(B) = 15%
Solving for the crossover rate of 11.49% requires interpolation, which is not covered in the text.
However, by using trial and error and an NPV profile drawing, the student can select the correct
multiple choice answer, 11.5%. Drawing an NPV profile drawing using the calculated IRRs, and the
NPVs at k = 0%, shows that there is a crossover rate. Of the responses listed in the problem, 16.5%
and 20.0% are clearly too high, since the IRR(B) is 15%. At k = 6.5% the NPVs are not equal, thus
11.5% must be the correct response.

Question: 2387

Intelligent Semiconductor is considering the development of a new data storage medium, which will
allow tremendous increases in the efficiency of its customer's high-end server lines. The
development of the new system will take place in Intelligent's existing facilities, and the storage costs
for the additional equipment are expected to be residual in nature. The following information applies
to this project:
Rent expense for existing facilities ($10,500)
Initial cash outlay ($50,000)
t1: $15,000
t2: $11,000
t3: $11,000
t4: $15,000
t5 $25,000
Discount rate: 9%

Page | 891
Assuming no taxes or related charges, that the initial cash outlay does not include any sunk costs,
and a $0.00 salvage value at after the fifth year, which of the following choices best represents the
payback period for this investment?

A. 4 years
B. 3.75 years
C. 3.13 years
D. 3.87 years
E. 4.23 years

Answer: D

Explanation:
Remember that the rental expense of the firm's existing facilities is a sunk cost, and should not be
incorporated into the calculation. This is due to the fact that the rental expense is not incremental in
nature, and is unaffected by the acceptance of the project in question. In this example, the payback
period is approximately 3.87 years. After the third year, $37,000 of the initial $50,000 investment has
been recouped, leaving $13,000 to be recovered. The following period has a cash inflow of $15,000,
exceeding the $13,000 amount required to completely "pay back" the initial investment. To calculate
the period required, divide the $13,000 left to be recouped by the $15,000 cash inflow during period
4. This will yield an answer of 0.8667, which is added to the three-year period already passed, giving
an answer of 3.87 years. While somewhat appealing in a simplistic sense, the payback period is not
an advisable method for valuation and analysis of capital projects, primarily due to the fact that this
method completely ignores the time value of money principle which governs the field of finance.

Question: 2388

A financial analyst with Mally, Feasance & Company is examining shares of Microscam International.
Assume the following information:
Retention Rate: 72%
EPS: $2.16
Growth Rate: 21%
Discount Rate: 14.50%
Tax Rate: 35%
Using this information, what is the ROE for Microscam International?

A. 5.88%
B. 15.12%
C. 56.88%
D. 33.40%
E. The answer cannot be calculated from the information provided.
F. 29.17%

Answer: F

Explanation:
To determine the ROE for Intelligent, the equation used to determine the dividend growth rate must
be manipulated. The dividend growth rate equation to be used is originally structured as follows: {g =
ROE(1 - Dividend Payout Ratio)}.
The original equation must be rearranged using algebra, and will yield the following: {ROE = g /
Retention Rate of Dividends}. Imputing the given information into this equation will yield the
following:
{ROE = 0.21 / 0.72}.
Solving for ROE will yield a figure of 29.17%.

Page | 892
As you can see, neither the discount rate nor the tax rate is incorporated into the equation.
Additionally, remember that (1 - Dividend Payout Ratio) is the same thing as the Retention Rate of
Dividends.

Question: 2389

Congress considered a tax plan that would reduce capital gains tax rates from the existing levels. The
current maximum capital gains rate is 28 percent compared with a maximum rate of 31 percent for
ordinary personal income. With this tax bill, which of the following statements is least correct for an
investor in a high personal tax bracket?

A. A 2-for-1 stock split is announced for a stock that the investor currently holds. The company had
split the stock because the stock price had increased beyond the desired price range and is expected
to continue to grow. This is good news to the investor because it means that any gains from
increased stock value will be taxed at a new lower capital gains rate when the stock is sold.
B. None of these statements are correct.
C. One of the companies in the investor's portfolio recently announced that it will embark on a stock
repurchase plan. The lower capital gains tax rate will reduce the investor's taxes if he/she decides to
tender some shares of stock in the company.
D. All of these statements are correct.
E. The stock of a company that pays high cash dividends and has a dividend reinvestment plan (DRIP)
is a good investment for this individual because he/she will receive more money that can then be
reinvested in the company's stock.

Answer: E

Explanation:
In a dividend reinvestment plan, the stockholder must pay taxes on the dividend amount, even
though stock and not cash has been received.

Question: 2390

Which of the following is/are true about operating cash flows of a project?
I. The annual operating cash flow equals operating income minus net non-cash expenses.
II. Financing costs are excluded from the operating cash flows.
III. Project evaluation is based on net cash flows, not net income.

A. III only
B. I, II & III
C. I only
D. II & III
E. I & III
F. II only
G. I & II

Answer: D

Explanation:
The annual operating cash flow equals operating income plus net non-cash expenses. Financing costs
are excluded since they are accounted for in the discounting process through the use of WACC.

Question: 2391

Vanderheiden Inc. is considering two average-risk alternative ways of producing its patented polo

Page | 893
shirts. Process S has a cost of $8,000 and will produce net cash flows of $5,000 per year for 2 years.
Process L will cost $11,500 and will produce cash flows of $4,000 per year for 4 years. The company
has a contractthat requires it to produce the shirts for 4 years, but the patent will expire after 4 years,
so the shirts will not be produced after 4 years. Inflation is expected to be zero during the next 4
years. If cash inflows occur at the end of each year, and if Vanderheiden's cost of capital is 10 percent,
by what amount will the better project increase Vanderheiden's value?

A. $1,179.46
B. $1,237.76
C. $677.69
D. $1,312.31
E. $1,098.89

Answer: B

Explanation:
0 k = 10%1234
S
-8,0005,0005,0005,0005,000
-8,000
-3,000
IRR(S) = 16.26%.
NPV(S) = $1,237.76. (extended NPV)
0 k = 10%1234
L
-11,5004,0004,0004,0004,000
IRR(L) = 14.66%.
NPV(L) = $1,179.46.

Question: 2392

Intelligent Semiconductor, a diversified technology company, is evaluating the sales of its cadmium
silicon transistor coils, and has identified the following information:
Fixed production costs for these transistors are $800,000
Average sales price per unit is $505.50
Breakeven quantity of 4,084
Which of the following best describes the average variable cost for this product?

A. $424.16
B. $195.89
C. $20.84
D. $309.61
E. The average variable cost cannot be determined from the information provided.

Answer: D

Explanation:
To calculate the breakeven quantity for a product, use the following equation: {Fixed operating
costs/[avg. sales price per unit - variable cost per unit]}. To determine the average variable cost of
this product, we must rearrange the standard equation using algebra, in a manner such that the
resulting equation resembles the following: {[$800,000/4,084] + X = $505.50}. This equation is
further rearranged into the following: {$195.89 + X = $505.50}. Finally, the ending equation
becomes:{X = $505.50 - $195.89}. Solving for X yields an average variable cost per unit of $309.61.

Page | 894
Question: 2393

Martin Corporation currently sells 180,000 units per year at a price of $7.00 per unit; its variable cost
is $4.20 per unit; and fixed costs are $400,000. Martin is considering expanding into two additional
states which would increase its fixed costs to $650,000 and would increase its variable unit cost to an
averageof $4.48 per unit. If Martin expands it expects to sell 270,000 units at $7.00 per unit. By how
much will Martin's breakeven sales dollar level change?

A. $910,667
B. $183,333
C. $456,500
D. $1,200,000
E. $805,556

Answer: E

Explanation:
Calculate the initial breakeven volume in dollars:
Old S(BE) = FC/(1-(VC/S)) = $400,000/1-(4.20/7.00) = $1,000,000.
Calculate the new breakeven volume in sales dollars:
New S(BE) = FC/(1-(VC/S)) $650,000/1-(4.48/7.00) = $1,805,556.
The increase in SB = $1,805,556 - $1,000,000 = $805,556.

Question: 2394

When a firm uses no debt,

A. its financial risk equals its business risk.


B. its business risk equals the market risk.
C. all of these answers.
D. its ROA equals its ROE.

Answer: D

Explanation:
With no debt, total capital equals total equity, giving ROA = ROE. Note that without debt, financial
risk is zero since financial risk is defined as the additional risk caused due to debt in the capital
structure. Market risk is the systematic risk arising from the correlation of the firm's stock price with
the market and is different from business risk.

Question: 2395

It has been observed in the market that most of the increases in dividends are followed by an
increase in the stock price and vice-versa. This implies that:
I. at least one of the M&M assumptions must be false.
II. there must be signaling effects involved.
III. investors are behaving irrationally.

A. II only
B. III only
C. I, II & III
D. II & III
E. I & II
F. I only

Page | 895
Answer: F

Explanation:
If all of the M&M assumptions held, a change in dividend policy would not cause the stock price to
change; dividend policy would be irrelevant. However, one does not necessarily need signaling
effects to account for market behavior. Other theories like the Bird-in-the-Hand theories can also
explain the phenomenon in the presence of transaction costs. In itself, then, the phenomenon does
not imply that the market is behaving irrationally.

Question: 2396

Your assistant has just completed an analysis of two mutually exclusive projects. You must now take
her report to a board of directors meeting and present the alternatives for the board's consideration.
To help you with your presentation, your assistant also constructed a graph with NPV profiles for the
two projects. However, she forgot to label the profiles, so you do not know which line applies to
which project. Of the following statements regarding the profiles, which one is most reasonable?

A. If one of the projects has a NPV profile which crosses the X-axis twice, hence the project appears
to have two IRRs, your assistant must have made a mistake.
B. If the two projects' NPV profiles cross once, in the upper left quadrant, at a discount rate of minus
10 percent, then there will probably not be a NPV versus IRR conflict, irrespective of the relative sizes
of the two projects, in any meaningful, practical sense (that is, a conflict which will affect the actual
investment decision).
C. If the two projects both have a single outlay at t = 0, followed by a series of positive cash inflows,
and if their NPV profiles cross in the lower left quadrant, then one of the projects should be
accepted. Both would be accepted if they were not mutually exclusive.
D. Whenever a conflict between NPV and IRR exist, then, if the two projects have the same initial
cost, the one with the steeper NPV profile probably has less rapid cash flows. However, if they have
identical cash flow patterns, then the one with the steeper profile probably has the lower initial cost.
E. If the two projects have the same investment cost, and if their NPV profiles cross once in the upper
right quadrant, at a discount rate of 40 percent, this suggests that a NPV versus IRR conflict is not
likely to exist.

Answer: B

Explanation:
A conflict will exist if the cost of capital is less than the crossover rate. In this case the cost of capital
must be greater than minus 10 percent and, therefore, there will probably not be a NPV versus IRR
conflict.

Question: 2397

Consider the following information for Company ABC:


Current Price of Stock $25.5
Expected dividend in 1 Year $1.00
Growth rate 8.0%
Beta 1.2
Risk Free Rate 4.5%
Calculate this company's cost of retained earnings using the Discounted Cash Flow (DCF) method.

A. 13.30%
B. 8.0%
C. 12.0%

Page | 896
D. 11.92%
E. 12.2%

Answer: D

Explanation:
The DCF method for estimating the cost of retained earnings states: Cost of Retained Earnings =
(Dividend for period 1 / Current Price) + Growth Rate. In this case the estimated Cost of Retained
Earnings = (1 / 25.5) + 8.0% = 3.92 + 8.00 = 11.92%

Question: 2398

If you know that your firm is facing relatively poor prospects but needs new capital, and you know
that investors do not have this information, signaling theory would predict that you would

A. postpone going into capital markets until your firm's prospects improve.
B. be indifferent between issuing debt and equity.
C. issue debt to maintain the returns of equity holders.
D. issue equity to share the burden of decreased equity returns between old and new shareholders.
E. convey your inside information to investors using the media to eliminate the information
asymmetry.

Answer: D

Explanation:
The announcement of a stock offering is generally taken as a signal that the firm's prospects as seen
by its management are not bright.

Question: 2399

Which of the following is/are advantages of stock repurchases?


I. Stock repurchases increase the price per share by reducing the number of shares.
II. Stock repurchases are often viewed as a positive signal by investors, raising the intrinsic value of
each share and increasing shareholder value.
III. Stock repurchases allow firms to distribute funds to shareholders without raising "sticky"
dividends.

A. II only
B. II & III
C. I only
D. I & III
E. I, II & III
F. III only
G. I & II

Answer: B

Explanation:
The value of a stock repurchase does not come from a simple reduction in the number of shares
outstanding. Remember that for this reduction, the firm must pay out part of its assets in the buy-
back process. If such a buyback takes place at a fair price, the shareholder value is completely
unaffected, for all you have done is exchanged cash for an equivalent value of common stock. The
real wealth increase through a repurchase program arises from real and tangible effects like the
interpretation of the program as a positive signal about future prospects. Further, if the management

Page | 897
thinks that excess cash reserves are only temporary, then they would be reluctant to raise dividends
and add instability todividend policy. Repurchase programs allow them to distribute excess funds
without paying them as dividends.

Question: 2400

Martin Fillmore is a big football star who has been offered contracts by two different teams. The
payments (in millions of dollars) he receives under the two contracts are listed below:
Team ATeam B
TimeCash FlowsCash Flows
08.02.5
14.04.0
24.04.0
34.08.0
44.08.0
Fillmore is committed to accepting the contract that provides him with the highest net present value
(NPV). At what discount rate would he be indifferent between the two contracts?

A. 16.49%
B. 10.85%
C. 11.35%
D. 19.67%
E. 21.03%

Answer: C

Explanation:
First, find the differential CFs by subtracting Team A CFs from Team B CFs (or vice versa). Enter these
into the cash flow register; then solve to find IRR/YR to get the discount rate for which he is
indifferent between the two contracts, 11.35%.

Question: 2401

Which of the following statements is incorrect?

A. NPV can be negative if the IRR is positive.


B. Assuming a project has normal cash flows, the NPV will be positive if the IRR is less than the cost
of capital.
C. If IRR = k (the cost of capital), then NPV = 0.
D. If the multiple IRR problem does not exist, any independent project acceptable by the NPV
method will also be acceptable by the IRR method.
E. The NPV method is not affected by the multiple IRR problem.

Answer: B

Explanation:
NPV is positive if IRR is greater than the cost of capital.

Question: 2402

Anderson Company has four investment opportunities with the following costs (all costs are paid at
t=0) and estimated internal rates of return (IRR):
ProjectCostIRR
A$2,00016.0%

Page | 898
B$3,00014.5
C$5,00011.5
D$3,0009.5
The company has a target capital structure, which consists of 40 percent common equity, 40 percent
debt, and 20 percent preferred stock. The company has $1,000 in retained earnings. The company
expects its year-end dividend to be $3.00 per share. The dividend is expected to grow at a constant
rate of 5 percent a year. The company's stock price is currently $42.75. If the company issues new
common stock, the company will pay its investment bankers a 10 percent flotation cost. The
company can issue corporate bonds with a yield to maturity of 10 percent. The company is in the 35
percent tax bracket. How large can the cost of preferred stock be (including flotation costs) and it still
be profitable for the company to invest in all four projects?

A. 7.75%
B. 12.68%
C. 10.46%
D. 8.90%
E. 11.54%

Answer: D

Explanation:
We need to find k(ps) at the point where all 4 projects are accepted. In other words, the capital
budget = $2,000 + $3,000 + $5,000 + $3,000 = $13,000. The WACC at that point is equal to IRR(D) =
9.5%.
Step 1 Find the retained earnings break point to determine whether k(s) or k(e) is used in the WACC
calculation:
BP(RE) = $2,500.
Since the capital budget > the retained earnings break point, k(e) is used in the WACC calculation.
Step 2 Calculate k(e):
k(e) = [3.00/$42.75(1-.10)] + 5% = 12.80%.
Step 3 Find k(ps):
9.5% = 0.4(10%)(0.65) + 0.2(kps) + 0.4(12.80%)
9.5% = 2.60% + 0.2(kps) + 5.12%
1.78% = 0.2k(ps)
8.90% = k(ps).

Question: 2403

Assume that all the assumptions of Modigliani and Miller hold. In particular, there are no taxes and
transaction costs. A firm has a policy of paying out 5% of the stock price as dividends. However, an
investor would like to receive a 7% dividend. For this, he should:

A. liquidate 7% of his stock holding after receiving the dividend.


B. liquidate 2% of his stock holding after receiving the dividend.
C. use the dividend to buy 2% of the stock after receiving the dividend.
D. none of these answers.

Answer: D

Explanation:
Suppose the investor is holding stocks worth $100. The company then pays $5 as dividends. When
the dividend is paid out, the stock price falls to $95 (in the absence of tax effects, as assumed). To
increase his income to $7, the investor must sell off stocks worth $2. This corresponds to 2/95 =
2.105% of the post dividend stock holding.

Page | 899
Question: 2404

Which of the following statements is correct?

A. The cost of debt used in calculating the WACC is an average of the after-tax cost of new debt and
of outstanding debt.
B. Preferred stock does not involve any adjustment for flotation cost since the dividend and price are
fixed.
C. The cost of new common equity includes an adjustment for flotation costs which is expressed as a
fixed percentage of the current stock price. The flotation percentage is determined jointly by the
current price of the firm's stock and its growth rate.
D. The opportunity cost principle implies that if the firm cannot invest retained earnings and earn at
least k(s) (component cost of retained earnings or internal equity), it should pay these funds to its
stockholders and let them invest directly in other assets that do provide this return.
E. Capital components are the types of capital used by firms to raise money. All capital comes from
one of three types: long-term debt, preferred stock, and equity.

Answer: D

Explanation:
The firm's after-tax earnings belong to its stockholders. All earnings remaining after interest and
preferred dividends belong to them, and these earnings serve to compensate stockholders for the
use of their capital. The firm may either pay out earnings as dividends or retain them and reinvest
them in the firm. If the firm retains earnings, there is an opportunity cost involved - stockholders
could have received the earnings as dividends and invested the money in other investments. Thus,
the firm should earn on its retained earnings at least as much as the stockholders themselves could
have earned on alternative investments of comparable risk.

Question: 2405

Foxglove Corp. is faced with an investment project. The following information is associated with this
project:
Allowable
Depreciation
YearNet Income*for 3-Yr. MACRS class
1$50,0000.33
260,0000.45
370,0000.15
460,0000.07
*Assume no interest expenses and a zero tax rate. The project involves an initial investment of
$100,000 in equipment that falls in the 3-year MACRS class and has an estimated salvage value of
$15,000. In addition, the company expects an initial increase in net working capital of $5,000, which
will be recovered in year 4. The cost of capital for the project is 12 percent. What is the project's net
present value? (Round your final answer to the nearest whole dollar.)

A. $153,840
B. $168,604
C. $162,409
D. $159,071
E. $182,344

Answer: B

Page | 900
Explanation:
Step 1 Calculate depreciation:
Dep 1 = 100,000(0.33) = 33,000.
Dep 2 = 100,000(0.45) = 45,000.
Dep 3 = 100,000(0.15) = 15,000.
Dep 4 = 100,000(0.07) = 7,000.
Step 2 Calculate cash flows:
CF 0 = -100,000 - 5,000 = -105,000.
CF 1 = 50,000 + 33,000 = 83,000.
CF 2 = 60,000 + 45,000 = 105,000.
CF 3 = 70,000 + 15,000 = 85,000.
CF 4 = 60,000 + 7,000 + 5,000 + 15,000 = 87,000.
Step 3 Calculate NPV:
Use CF key on calculator. Enter cash flows shown above. Enter I/YR = 12%. Solve for NPV = $168,604.

Question: 2406

Chandler Communications' CFO has provided the following information:


* The company's capital budget is expected to be $5,000,000.
* The company's target capital structure is 70 percent debt and 30 percent equity.
* The company's net income is $4,500,000. If the company follows a residual dividend policy, what
portion of its net income should it pay out as dividends this year?

A. 66.67%
B. 40.00%
C. 33.33%
D. 60.00%
E. 50.00%That answer is correct!

Answer: A

Explanation:
$5,000,000 x 0.3 = $1,500,000 of retained earnings needed to fund the capital budget. $4,500,000 -
$1,500,000 = $3,000,000 of net income available for dividends. Dividend payout ratio =
$3,000,000/$4,500,000 = 0.6667, or 66.67%.

Question: 2407

If the MM hypothesis about dividends is correct, and if one found a group of companies which
differed only with respect to dividend policy, which of the following statements would be most
correct?

A. All of these statements are true.


B. The residual dividend model should not be used, because it is inconsistent with the MM dividend
hypothesis.
C. The total expected return, which in equilibrium is also equal to the required return, would be
higher for those companies with lower payout ratios because of the greater risk associated with
capital gains versus dividends.
D. None of these statements are true.
E. If the expected total return of each of the sample companies were divided into a dividend yield
and a growth rate, and then a scatter diagram (or regression) analysis were undertaken, then the
slope of the regression line (or b in the equation D1/Po = a + b(g)) would be equal to +1.0.

Answer: D

Page | 901
Explanation:
None of these statements are correct. Miller and Modigliani argued that the value of the firm
depends only on the income produced by its assets, not on how this income is divided between
dividends and retained earnings.

Question: 2408

Which of the following terms describes the risk of individual projects to a corporation, taking into
consideration that each project represents only one of the firm's portfolio of assets?

A. Alpha coefficient
B. Corporate risk
C. Stand-alone risk
D. Systematic risk
E. Market risk

Answer: B

Explanation:
Corporate, or "within-firm" risk, is defined as the risk of individual projects to a corporation, taking
into consideration the fact that each project represents only one of the firm's assets. In examining
corporaterisk, there is an implicit assumption that some of the total risk to the firm's profits from the
addition of new projects will be partially diversified away. Corporate risk is measured by the project's
impact on uncertainty about the firm's future earnings.

Question: 2409

Which of the following statements best describes the optimal capital structure?

A. All of these answers are correct.


B. None of these answers are correct.
C. The optimal capital structure is the mix of debt, equity, and preferred stock, which maximizes the
company's stock price.
D. The optimal capital structure is the mix of debt, equity, and preferred stock which minimizes the
company's cost of debt.
E. The optimal capital structure is the mix of debt, equity, and preferred stock which maximizes the
company's earnings per share (EPS).

Answer: C

Explanation:
The optimal capital structure is the one that maximizes the price of the firm's stock, and minimizes
the firm's WACC.

Question: 2410

Consider the following information:


30-day treasury rate (Risk Free rate) 6.4%
Company XYZ Bond yield 11.2%
Beta 1.1
Risk Premium 3.5%
Credit Rating B-
Marginal Tax Rate 40% Calculate Company XYZ's cost of retained earnings using the Bond-Yield-plus-

Page | 902
Risk-Premium approach.

A. 17.6%
B. 15.2%
C. 14.7%
D. 8.82%
E. 16.17%
F. 11.36%

Answer: C

Explanation:
To estimate a firm's cost of retained earnings using the Bond-Yield-plus-Risk-Premium approach,
simply take the company's bond yield and add the risk premium. In this case the cost of retained
earnings = 11.2% + 3.5% = 14.7%.

Question: 2411

An increase in which of the following, holding everything else equal, will cause a decrease in the
theoretical growth rate of common stock dividends according to the Growth Rate of Dividends
Model?
I. Return on equity
II. Tax rate
III. Dividend payout ratio
IV. Annual dividend
V. Discount rate
VI. Beta coefficient
VII. Retention rate

A. I, II, IV
B. II, III, VI, V
C. I, II, VI, V
D. III
E. III, VI, VII
F. VII

Answer: D

Explanation:
III The equation used to determine the theoretical growth rate of common stock dividends is as
follows: {Annual growth rate = [ROE * (1 - dividend payout ratio)]} As you can see, of the choices
listed, only an increase in the dividend payout ratio will cause a decrease in the theoretical growth
rate of common stock dividends. Remember that the retention rate is equal to (1 - dividend payout
ratio). Additionally, neither the discount rate nor the tax rate is explicitly factored into the dividend
growth rate equation.

Question: 2412

A stock has a beta of 0.85 and the risk-free rate is 6.95%. Its dividend growth rate is 5.2% and its P/E
ratio is 11.6. If the firm has a dividend payout ratio of 63%, the market risk premium equals
________.

A. 5.91%
B. 6.54%

Page | 903
C. 5.15%
D. 4.33%

Answer: D

Explanation:
P0/E1 = dividend payout/(k - g) Therefore, 11.6 = 0.63/(k - 0.052), giving expected return = k =
10.63%. Now, the CAPM expected return on the stock is given by k = Rf + beta*(Rm - Rf). Therefore,
10.63% = 6.95% + 0.85*market premium, giving market premium = (10.63 - 6.95)%/0.85 = 4.33%.

Question: 2413

Which of the following statements is most correct?

A. Generally, we do not need to adjust project cash flows to take into account the effects of inflation,
since inflation is not accounted for in the cost of capital.
B. All of these statements are correct.
C. For comparing mutually exclusive projects with unequal lives, replacement chain analysis leads to
the same decision as obtained by calculating the equivalent annual annuity (EAA).
D. In comparing mutually exclusive projects with unequal lives, you should always choose the project
which has the highest NPV.
E. The rate of depreciation affects accounting statements, but has no effect on the firm's capital
budgeting decisions.

Answer: C

Explanation:
The replacement chain analysis and the equivalent annual annuity methods lead to the same
decision. Depreciation affects a firm's cash flow, which affects its capital budgeting decisions. The
project with the highest extended NPV should be chosen. The cost of capital reflects inflation;
therefore, project cash flows must be adjusted for inflation.

Question: 2414

Which of the following figures is not explicitly incorporated into the earnings per share (EPS)
calculation?

A. Variable costs
B. Common shares outstanding
C. Interest expense
D. None of these answers
E. Fixed costs
F. Sales

Answer: D

Explanation:
The EPS figure can be found using two principal equations. The first is illustrated as follows:
{EPS = [(Sales - Fixed Costs - Variable Costs - Interest Expense)(1 - Tax Rate)] / [# of Common Shares
Outstanding]}
Additionally, the EPS figure can be found by:
{EPS = [(EBIT - Interest Expense)(1 - Tax Rate) / # of Common Shares Outstanding]} As you can see, all
of the choices listed are expressly incorporated into the EPS calculation.

Page | 904
Question: 2415

A firm's capital structure has a debt-to-equity ratio of 1.2. The pretax cost of debt is 6.8% and the
weighted average cost of capital of the firm equals 9.8%. The risk-free rate in the economy is 6.2%
the expected rate of return on the market is 14%. The firm must pay 35% of its gross income in taxes.
The beta of the stock equals ________.

A. 1.02
B. 1.64
C. 1.3
D. 0.91

Answer: C

Explanation:
Since the debt interest is tax deductible, the after-tax cost of debt equals 6.8%*(1-0.35) = 4.42%.
Since the D/E ratio = 1.2, (D+E)/E = 2.2, giving E/(D+E) = 0.455. Thus, equity forms 45.5% of the
capital whiledebt forms 54.5%. The WACC is then equal to 0.455*RE + 0.545*4.42% = 9.8% (given).
Solving gives RE = 16.24%. If B is the beta of the stock, then using CAPM, 16.24% = 6.2% + B*(14% -
6.2%), giving B = 1.29.

Question: 2416

Which of the following equations correctly illustrates the calculation of the Weighted Average Cost of
Capital (WACC)?

A. None of these answers


B. {{Percentage of debt* [coupon on outstanding debt * (1 - combined state/federal tax rate)]} +
{percentage of preferred stock * [annual preferred dividend/(required rate of return)]} + {percentage
of common equity * cost of common equity}}
C. {{Percentage of debt * [coupon on outstanding debt * (1 + combined state/federal tax rate)]} +
{percentage of preferred stock * [annual preferred dividend/(offering price - flotation costs)]} +
{percentage of common equity * cost of common equity}}
D. {{Percentage of debt* [yield to maturity of outstanding debt * (1 - combined state/federal tax
rate)]} + {percentage of preferred stock * [annual preferred dividend/(offering price - flotation
costs)]} + {percentage of common equity * cost of common equity}}
E. {Average cost of equity + average cost of debt + average cost of preferred stock}* subjective divisor
F. {Percentage of debt * [coupon on outstanding debt * (1 + combined state/federal tax rate)]} +
{percentage of preferred stock * [annual preferred dividend/(offering price + flotation costs)]} +
{percentage of common equity * cost of common equity}}

Answer: D

Explanation:
The Weighted Average Cost of Capital is a fundamentally important concept within the field of
corporate finance, and the CFA candidate should have a complete understanding of both the
mechanics of the WACC figure, as well as the relationships between its components. In calculating
the cost of outstanding common equity, there exist three distinct methods, the Dividend-Yield-plus-
Growth-Rate, or Discounted Cash Flow approach, the Capital Asset Pricing Model (CAPM), and the
Bond-Yield-plus-Risk-Premium approach. It is important for the CFA candidate to have a complete
understanding of each method, along with their weaknesses and advantages.

Question: 2417

Page | 905
Lincoln Lodging Inc. estimates that if its sales increase 10 percent then its net income will increase 18
percent. The company's EBIT equals $2.4 million, and its interest expense is $400,000. The company's
operating costs include fixed and variable costs. What is the level of the company's fixed operating
costs?

A. $2,125,000
B. $2,000,000
C. $1,200,000
D. $666,667
E. $450,000

Answer: C

Explanation:
We're given enough information to find both DFL and DTL.
DTL = .18/.10 = 1.8.
DFL = $2,400,000/($2,400,000 - $400,000) = 1.2
Given DTL = DFL x DOL, we can calculate DOL = 1.5. Recognizing S - VC - FC = EBIT, 1.5 = (S -
VC)/$2,400,000 or S - VC = $3,600,000. The difference between (S - VC) and EBIT must represent
fixed operating costs. Thus, FC = $3,600,000 - $2,400,000 = $1,200,000.

Question: 2418

The Altman Company has a debt ratio of 33.33 percent, and it needs to raise $100,000 to expand.
Management feels that an optimal debt ratio would be 16.67 percent. Sales are currently $750,000,
and the total assets turnover is 7.5. How should the expansion be financed so as to produce the
desired debt ratio?

A. Finance 20 percent debt, 80 percent equity.


B. Finance 40 percent debt, 60 percent equity.
C. Finance it all with debt.
D. Finance 50 percent debt, 50 percent equity.
E. Finance it all with equity.

Answer: E

Explanation:
Old debt ratio = 0.3333; New debt ratio = 0.1667.
$750,000/TA= 7.5.
TA = $100,000.
Debt = 0.3333($100,000) = $33,330.
New TA = $100,000 + $100,000 = $200,000.
New Debt = $200,000(0.1667) = $33,340.
Altman's current debt of $33,330 represents approximately 16.67% of total assets following the
expansion, thus the firm should finance with 100 percent equity.

Question: 2419

Which of the following is a key determinant of operating leverage?

A. Cost of debt.
B. Technology.
C. Level of debt.
D. Capital structure.

Page | 906
E. Physical location of production facilities.

Answer: B

Explanation:
If a high percentage of a firm's total costs are fixed, the firm has high operating leverage. Higher fixed
costs are generally associated with more highly automated, capital-intensive firms and industries.

Question: 2420

Which of the following statements is most correct?

A. All of these answers are correct.


B. An increase in the risk-free rate is likely to decrease the marginal cost of both debt and equity
financing.
C. The WACC is calculated on a before-tax basis.
D. None of these answers are correct.
E. The WACC (Weighted Average Cost of Capital) for a given capital budget level is a weighted
average of the marginal cost of each relevant component which makes up the firm's target capital
structure.

Answer: E

Explanation:
A value-maximizing firm will establish a target (optimal) capital structure and then raise new capital
in a manner that will keep the actual capital structure on target over time. The target proportions of
debt, preferred stock, and common equity, along with the component cost of capital, are used to
calculate the firm's WACC.

Question: 2421

Which of the following statements is most correct?

A. The factors which affect a firm's business risk are determined partly by industry characteristics and
partly by economic conditions. Unfortunately, these and other factors, which affect a firm's business
risk, are not subject to any degree of managerial control.
B. The firm's financial risk may have both market risk and diversifiable risk components.
C. One of the benefits to a firm of being at or near its target capital structure are that financial
flexibility becomes much less important.
D. A firm's business risk is solely determined by the financial characteristics of its industry.
E. All of these statements are false.

Answer: B

Explanation:
Financial risk is an increase in stockholders' risk, over and above the firm's basic business risk,
resulting from the use of financial leverage, which refers to the use of fixed-income securities.

Question: 2422

Jackson Jets is considering two mutually exclusive projects. The projects have the following cash
flows:
Project AProject B
TimeCash FlowsCash Flows

Page | 907
0-$10,000-$8,000
11,0007,000
22,0001,000
36,0001,000
46,0001,000
At what cost of capital, do the two projects have the same net present value?

A. 13.03%
B. 12.26%
C. 11.20%
D. 12.84%
E. 14.15%That answer is correct!

Answer: A

Explanation:
Find the differential cash flows by subtracting B's cash flows from A's cash flows for each year.
CF(0) = -2,000
CF(1) = -6,000
CF(2) = 1,000
CF(3) = 5,000
CF(4) = 5,000
Enter these cash flows and solve for the IRR = crossover rate = 13.03%.

Question: 2423

Which of the following types of risk cannot be eliminated through diversification? Choose the best
answer

A. Unsystematic risk
B. Market risk
C. Corporate risk
D. Alpha risk
E. Gamma risk

Answer: B

Explanation:
Market risk is defined as that part of an asset's risk which cannot be eliminated through
diversification. Market risk is measured by the Beta coefficient, and is commonly referred to as
"systematic" or "nondiversifiable" risk. Additionally, market risk is referred to as "beta risk."
Corporate risk is defined as the variability of assets expected returns without taking into
consideration the effects of shareholder diversification. This is one step away from Stand-alone Risk,
which measures the risk of an asset, not only without taking into consideration the effect of
shareholder diversification, but of company diversification as well. Stand-alone risk assumes that the
asset in question is the only asset of the firm and that the securities of the firm are the only assets in
investors' portfolios. Corporate risk takes into consideration that firms will diversify their asset bases.

Question: 2424

Dumb & Dumber Development Company has two mutually exclusive investment projects to
evaluate. Assume both projects can be repeated indefinitely. The following cash flows are associated
with each project:
TimeProject AProject B

Page | 908
0-$100,000-$70,000
130,00030,000
250,00030,000
370,00030,000
4-30,000
5-10,000
The project types are equally risky and the firm's cost of capital is 12 percent. What is the EAA of the
higher valued project? (Round your final answer to the nearest whole dollar.)

A. $16,470
B. $7,433
C. $11,325
D. $6,857
E. None of the answers

Answer: B

Explanation:
Step 1 Calculate the NPV for both projects:
A:CF(0) = -100,000; CF(1) = 30,000; CF(2) = 50,000; CF(3) = 70,000; I/YR = 12; solve for NPV = $16,470.
B:CF(0) = -70,000; CF(1) = 30,000; CF(2) = 30,000; CF(3) = 30,000;CF(4) = 30,000; CF(5) = 10,000; I/YR
= 12; NPV = $26,795.
Step 2 Calculate the EAA for both projects:
A: N = 3; I/YR = 12; PV = -16,470; FV = 0; solve for PMT = $6,857.
B: N = 5; I/YR = 12; PV = -26,795; FV = 0; solve for PMT = $7,433.
The PMT is the EAA for each project. Since Project B has the larger EAA $7,433 is the correct answer.

Question: 2425

Consider the following information for a company.


Common Stock Price $53.25
Preferred Stock Par Price $50
Preferred Dividend $3.5
Debt Rating BB+
Owners Equity 25%
Preferred Stock Flotation Cost 2.0%
The Preferred Stock is issued at Par
Calculate the component cost of this newly issued preferred stock. (Note that for existing preferred
stock, flotation costs may be ignored.)

A. 18.78%
B. 7.14%
C. 6.57%
D. 7.0%
E. 10%
F. 3.5%
G. 12.5%

Answer: B

Explanation:
The component cost of preferred stock is the dividend divided by issue price minus floatation cost. In
this case the component cost of preferred stock = $3.50 / (50 - 1) = 7.14%.

Page | 909
Question: 2426

A project has the following cash flows over the next 5 years: $1,000, $600, $300, $1,200 and $1,400.
Assume all cash flows occur at the end of a year. The project requires an initial cash outlay of $2,900.
The project's cost of capital is 8%. The NPV of the project equals ________.

A. $1,194
B. $1,735
C. $3,513
D. $613

Answer: D

Explanation:
The discounted cash flow at the end of year N is obtained by dividing that year's cash flow by 1.08N,
since the project's cost of capital is 8%. Using this, the discounted cash flows are:
$926, $514, $238, $882 and $953.
The present value of all the future cash flows = $(926 + 514 + 238 + 882 + 953) = $3,513. The NPV is
then equal to $(3,513 - 2,900) = $613.

Question: 2427

Which of the following statements is most correct?

A. An increase in the flotation cost incurred in selling new stock will reduce the retained earnings
break point, as long as the dollar level of retained earnings and the fraction of capital which is equity
financed remains constant.
B. None of these answers are correct.
C. All of these answers are correct.
D. An increase in a firm's corporate tax rate, will increase the firm s cost of debt capital, as long as the
yield to maturity on the company's bonds remains constant or falls.
E. An increase in the flotation cost incurred in selling new stock will increase the cost of retained
earnings.

Answer: B

Explanation:
An increase in the flotation cost has no effect on the cost of retained earnings or the retained
earnings break point. An increase in the firm's corporate tax rate reduces the after-tax component
cost of debt.

Question: 2428

Ranking conflicts occur between the NPV and IRR methods because I NPV profiles have differing
slopes. II IRR assumes reinvestment of intermediate cash flows at the IRR rate. III IRR doesn't take
into account cash flows occurring far in the future. IV NPV incorrectly uses the same discount rate for
all cash flows.

A. I, II and III only


B. II only
C. I only
D. II, III and IV only
E. All of these are correct
F. I and II only

Page | 910
Answer: F

Explanation:
Project with differing initial investments and cash flow patterns may have NPV profiles with differing
slopes. The point at which the NPV profiles cross represents the discount rate at which the NPV of
both projects is the same. There will be ranking conflicts between NPV and IRR at discount rates
below this point. The NPV method implicitly assumes that the rate at which cash flows can be
reinvested is the cost of capital, whereas the IRR method assumes that the firm can reinvest at the
IRR. IRR does take all cash flows into account, even those far in the future. It is correct to use the
same discount rate for all project cash flows, unless you know project risk will change in the future.

Question: 2429

Which of the following statements is/are true about observed capital structures?
I. Firms with higher stability in sales tend to use higher debt ratios.
II. Firms with a higher ratio of general-purpose assets to special-purpose assets tend to have higher
debt ratios.
III. Young firms with higher growth rates tend to have higher debt ratios.

A. II only
B. I & II
C. I only
D. I, II & III
E. I & III
F. III only

Answer: B

Explanation:
Stability of sales implies a stability in profitability. This allows firms to service higher levels of debt
without worrying about bankruptcy. Similarly, since general-purpose assets are more valuable as
collateral than firm-specific assets, firms with a higher ratio of general-purpose assets to special-
purpose assets tend to have higher debt ratios. However, young firms which face higher uncertainty
in their project payoffs tend to favor equity financing.

Question: 2430

Clay Industries, a large industrial firm, is examining the operating leverage of its tooling division
during the last year. Consider the following information:
% change in EBIT during the last year 28%
Sales for period 1$435,000
Sales for period 2$578,000
Cost of debt 7.75%
Expected return on the market 15%
Risk-free rate 4.55%
Beta 1.05
Given this information, what is the operating leverage of this division during the time period in
question?

A. 0.8518
B. 1.1741
C. 0.9178
D. The degree of operating leverage for this firm cannot be calculated from the information provided.

Page | 911
E. 0.1322
F. 0.98402That answer is correct!

Answer: A

Explanation:
To calculate the degree of operating leverage, use the following equation: {% change in EBIT/%
change in sales}. In this example, the percentage change in EBIT is provided. However, the
percentage change in sales must be calculated, and can be found by the following: {[sales in period 2
- sales in period 1]/sales in period 1}. Incorporating the given information into this equation yields a
percentage change in sales of 32.87%. From this point on, the calculation of the DOL is
straightforward, and can be found as follows: {% change in EBIT 28%/% change in sales 32.87%}=
0.8518. In attempt to trick you, the cost of debt, along with the components of the Capital Asset
Pricing Model have been provided. None of these figures are relevant in the calculation of the
Degree of Operating Leverage.

Question: 2431

Which of the following statements is most correct?

A. All of these answers are correct.


B. None of these answers are correct.
C. The capital structure that maximizes stock price is also the capital structure, which maximizes
earnings per share.
D. The capital structure that maximizes stock price is also the capital structure, which maximizes the
firm's times-interest-earned (TIE) ratio.
E. The capital structure that maximizes stock price is also the capital structure, which minimizes the
weighted average cost of capital (WACC).

Answer: E

Explanation:
The capital structure which maximizes the firm's stock price generally calls for a debt ratio which is
lower than the one that maximizes EPS. The firm could maximize its TIE by having no debt (i.e. zero
interest payments). But, this capital structure would probably not maximize the firm's stock price.

Question: 2432

The management of Clay Industries have adhered to the following capital structure: 50% debt, 45%
common equity, and 5% perpetual preferred equity. The following information applies to the firm:
Before-tax cost of debt = 7.5% Combined state/federal tax rate = 35% Expected return on the market
= 14.5% Annual risk-free rate of return = 5.25% Historical Beta coefficient of Clay Industries Common
Stock = 1.15 Annual preferred dividend = $1.35 Preferred stock net offering price = $17.70 Expected
annual common dividend = $0.45 Common stock price = $30.90 Expected growth rate = 11.75%
Subjective risk premium = 3.8% Given this information, and using the Capital Asset Pricing Model to
calculate the component cost of common equity, what is the Weighted Average Cost of Capital for
Clay Industries?

A. 15.31%
B. 11.30%
C. 9.92%
D. 9.968%
E. The WACC for Clay Industries cannot be calculated from the information.
F. 10.05%

Page | 912
Answer: D

Explanation:
The calculation of the Weighted Average Cost of Capital is as follows: {fraction of debt * [yield to
maturity on outstanding long-term debt][1-combined state/federal income tax rate]} + {fraction of
preferred stock * [annual dividend/net offering price]} + {fraction of common stock * cost of equity}.
The cost of common equity can be calculated using three methods, the Capital Asset Pricing Model
(CAPM), the Dividend-Yield-plus-Growth-Rate (or Discounted Cash Flow) approach, and the Bond-
Yield-plus-Risk-Premium approach. In this example, you are asked to calculate the cost of common
equity using the Capital Asset Pricing Model (CAPM). This approach involves the following equation:
{risk-free rate +beta[expected return on the market - risk-free rate]}. Specifically, the calculation of
the component cost of common equity using the CAPM is as follows: {[5.25% + 1.15(14.5%-5.25%]} =
15.888%. The after-tax cost of debt can be found by multiplying the yield to maturity on the firm's
outstanding long-term debt (7.5%) by (1-tax rate). Using this method, the after-tax cost of debt is
found as 4.875%. The calculation of the cost of perpetual preferred stock is relatively straightforward,
simply divide the annual preferred dividend by the net offering price. Using this method, the cost of
preferred stock is found as 7.627%. Incorporating these figures into the WACC equation gives the
answer of 9.968%.

Question: 2433

The president of Real Time Inc. has asked you to evaluate the proposed acquisition of a new
computer. The computer's price is $40,000, and it falls into the MACRS 3-year class. Purchase of the
computer would require an increase in net working capital of $2,000. The computer would increase
the firm's before-tax revenues by $20,000 per year but would also increase operating costs by $5,000
per year. The computer is expected to be used for 3 years and then be sold for $25,000. The firm's
marginal tax rate is 40 percent, and the project's cost of capital is 14 percent. What is the net
investment required at t = 0?

A. -$36,600
B. -$40,000
C. -$38,600
D. -$42,000
E. -$37,600

Answer: D

Explanation:
Initial investment:
Cost($40,000)
Change in NWC(2,000)
($42,000)

Question: 2434

Buchanan Brothers anticipates that its net income at the end of the year will be $3.6 million (before
any recapitalization). The company currently has 900,000 shares of common stock outstanding and
has no debt. The company's stock trades at $40 a share. The company is considering a
recapitalization where it will issue $10 million worth of debt at a yield to maturity of 10 percent, and
use the proceeds to repurchase common stock. Assume the stock price remains unchanged by the
transaction, and the company's tax rate is 34 percent. What will be the company's earnings per share
if it proceeds with the recapitalization?

Page | 913
A. $4.52
B. $5.54
C. $2.23
D. $3.26
E. $2.45That answer is correct!

Answer: A

Explanation:
After issuing the debt, the company can repurchase $10,000,000/$40 = 250,000 shares leaving
650,000 shares outstanding. We still need to find the expected NI after issuing the debt. We're given
the anticipated NI is $3.6 million. Thus, the EBIT (before the debt issue) can be found as follows:
$3,600,000 = EBIT(1 - 0.34) or EBIT = $5,454,545.45. The company will pay $1,000,000 in interest
after issuing the debt so the new EBT will be $5,454,545.45 - $1,000,000 = $4,454,545.45. Also, the
new NI figure will be: $4,454,545.45(1 - 0.34) = $2,940,000. Finally, $2,940,000/650,000 = $4.52 is
the EPS after the recapitalization.

Question: 2435

Which of the following choices correctly describes a project which will direct the operations of a
company into a new market or functional niche, is primarily enacted to expand revenues, and one in
which the firm does not have an existing proxy?

A. Marginal project
B. Extraordinary item
C. Retrenchment project
D. Replacement project
E. Expansion project

Answer: E

Explanation:
In an analysis of an expansion project, the relevant cash flows are those which apply wholly to the
proposed project. There is no existing project whose cash flows must be incorporated into the
analysis, as in the examination of a replacement project. "Marginal project" and "retrenchment
project," are fictitious terms, and "extraordinary item" is an accounting classification referring to an
event that is both unusual and nonrecurring.

Question: 2436

According to the Signaling Theory of capital structure, an increase in bankruptcy costs:

A. increases the debt ratio of a firm.


B. none of these answers.
C. may or may not affect the debt ratio of a firm.
D. decreases the debt ratio of a firm.

Answer: C

Explanation:
The Signaling Theory of capital structure considers the decisions of a firm's manager to raise debt or
equity capital as a function of the relative profitability prospects of the firm's projects. It does not use
bankruptcy costs as an explanation of the debt ratios prevalent in various industries. Bankruptcy
costs are used by the Trade-off Theory of capital structure.

Page | 914
Question: 2437

Projects A and B are mutually exclusive and will be repeated. The company's cost of capital is 12.5
percent.
tProj. A-Cash FlowsProj. B-Cash Flows
0- 10,000- 10,000
1+ 40,000+ 30,000
2+ 50,000+ 30,000
3+ 30,000+ 30,000
4+ 20,000+ 30,000
5+ 30,000
What is the equivalent annual annuity (EAA) of the best project?

A. $24,227
B. $27,192
C. $32,811
D. $23,243
E. $35,000

Answer: C

Explanation:
First find the NPV of each project, using the cash flow register:
Project A NPV = $98,617.59.
Project B NPV = $96,817.05.
Then find EAA:
Project A:
N = 4; I = 12.5; PV = -98,617.59; FV = 0; solve for PMT = $32,810.85.
Project B:
N = 5; I = 12.5; PV = -96,817.05; FV = 0; solve for PMT = $27,191.46.
Project A has the higher EA
A.

Question: 2438

Project A has an internal rate of return of 18 percent, while Project B has an internal rate of return of
16 percent. However, if the company's cost of capital (WACC) is 12 percent, Project B has a higher net
present value. Which of the following statements is most correct?

A. All of these answers are correct.


B. The crossover rate for the two project is less than 12 percent.
C. None of these answers are correct.
D. Assuming that the two projects have the same scale, Project A probably has a faster payback than
Project B.
E. Assuming the timing of the two projects is the same, Project A is probably of larger scale than
Project B.

Answer: D

Explanation:
Draw out the NPV profiles of these two projects. As B's NPV declines more rapidly with an increase in
discount rates, this implies that more of the cash flows are coming later on. Therefore, Project A has
a faster payback than Project B.

Page | 915
Question: 2439

Dorrie James, a financial analyst with consulting firm Brown, Ketchuppe & Company, is trying to
determine the earnings per share (EPS) figure for Floweration.com. Assume the following
information:
Sales: $4,500,000
Fixed costs: $2,000,000
Variable costs: $1,200,000
Interest expense: $40,000
Tax rate: 35%
Weighted Average Cost of Capital: 12.25%
Beta coefficient: 1.25
Common shares outstanding: 4,755,000
Using this information, what are the EPS for Floweration.com?

A. $0.1492
B. $0.1722
C. $0.1676
D. $0.2440
E. $0.2360
F. The answer cannot be calculated from the information provided.

Answer: B

Explanation:
The EPS figure is perhaps the single most popular term in the field of conventional equity
investments. Any glance into financial media and business periodicals will undoubtedly uncover
numerous instances in which the EPS figure is cited. While quite popular and useful, many individuals
do not understand themechanics behind the EPS calculation, and an investigation into the
components of EPS is a valuable learning experience. The EPS calculation is found by the following
equation: {EPS = [(Sales - Fixed Costs - Variable Costs - Interest Expense)(1 - Tax Rate)] / [# of
Common Shares Outstanding]} Additionally, the EPS figure can be found by:
{EPS = [(EBIT - Interest Expense)(1 - Tax Rate) / # of Common Shares Outstanding]} Incorporating the
given information into the first EPS equation will yield the following: {EPS = [($4,500,000 -
$2,000,000 - $1,200,000 - $40,000)(1 - .35)] / 4,755,000}= $0.1722

Question: 2440

The following information applies to a company's preferred stock:


Current price $101.00 per share
Par value $100.00 per share
Annual dividend $6.50 per share
The company issued the preferred stock at par and incurred a 10% floatation cost. If the company's
marginal corporate tax rate is 34%, what is the after-tax cost of preferred stock?

A. 4.3%
B. 6.4%
C. 13%
D. 4.2%
E. 6.5%
F. 7.2%

Answer: F

Page | 916
Explanation:
The cost of preferred stock is calculated as the preferred stock dividend divided by the net issuing
price. The dividend for this preferred stock is $6.50, and the net issuing price was $90.00. Thus the
cost of preferred stock is 6.5 divided by 90 or 7.2%. There are no tax savings associated with the use
of preferred stock, therefore no tax adjustments are made when calculating the cost.

Question: 2441

Consider the following information:


30-day treasury rate (Risk Free rate) 5.2%
Company XYZ Bond yield 12.2%
Beta 1.2
Risk Premium 4.5%
Credit Rating BBB
Calculate Company XYZ's cost of retained earnings using the Bond-Yield-plus-Risk-Premium
approach.

A. 21.9%
B. 16.7%
C. 12.2%
D. 5.2%
E. 20.4%
F. 9.7%

Answer: B

Explanation:
To estimate a firm's cost of retained earnings using the Bond-Yield-plus-Risk-Premium approach,
simply take the company's bond yield and add the risk premium. In this case the cost of retained
earnings = 12.2% + 4.5% = 16.7%.

Question: 2442

Which of the following is not a cash flow that results from the decision to accept a project?

A. Externalities.
B. Shipping and installation costs.
C. Opportunity costs.
D. Changes in working capital.
E. Sunk costs.

Answer: E

Explanation:
Sunk cost is not a relevant cash flow.

Question: 2443

A stock split will cause a change in the total dollar amounts shown in which of the following balance
sheet accounts?

A. Retained earnings
B. Cash

Page | 917
C. None of these will change
D. Paid-in capital
E. Common stock

Answer: C

Explanation:
If a stock rises above a specific amount, management may declare, for example, a two-for-one stock
split, where the number of shares outstanding doubles and the stock price is halved. Each
stockholder would have more shares, but each share is worth less. Theoretically, a stock split should
not affect the value of the firm. They are generally used after a sharp price run-up to produce a large
price reduction.

Question: 2444

Consider the following information:


30 day T-Bill rate (Risk free rate) 7.2%
Common Stock Beta 0.8
Expected Rate of return for the market 15.0%
Net Worth to Total Asset Multiple .25
Calculate this firm's cost of retained earnings using the CAPM approach.

A. 7.2%
B. 13.44%
C. 12.0%
D. 9.6%
E. 22.2%
F. 10.2%

Answer: B

Explanation:
To calculate the cost of retained earnings for a firm using CAPM, one may use the following formula:
Cost of retained earnings = risk free rate + ((expected rate of return on the market - risk free rate) x
Beta). In this case the cost of retained earnings = 7.2% + ((15.0% - 7.2%) x 0.8 = 13.44%.

Question: 2445

Lloyd Enterprises has a project, which has the following cash flows:
YearCash Flows
0-$200,000
150,000
2100,000
3150,000
440,000
525,000
The cost of capital is 10 percent. What is the project's discounted payback?

A. 2.3333 years
B. 1.8763 years
C. 2.4793 years
D. 2.0000 years
E. 2.6380 years

Page | 918
Answer: E

Explanation:
Discounted CFCumulative CF
0-200,000.00-200,000.00
145,454.55-154,545.45
282,644.63-71,900.82
Payback
3112,697.22+40,796.40
427,320.54+68,116.94
515,523.03+83,639.97
Payback period = 2 years + (71,900.82/112,697.22) = 2.638 years.

Question: 2446

A firm initially has no debt in its capital structure. As it starts increasing its debt, the stock price
begins to rise because of ________. After a threshold, an increase in debt reduces the stock price
due to ________.

A. none of these answers


B. higher leverage; higher probability of default
C. tax deductions; expected default and bankruptcy costs
D. lower cost of retained earnings; higher cost of debt

Answer: C

Explanation:
Debt offers tax shelter for income that equity does not since interest paid on debt is tax deductible.
Therefore, as the firm starts adding debt to its capital structure in lieu of equity, the stock price
startsrising. However, at a threshold debt-to-equity level, the higher probability of default offsets the
value of the debt shield. Beyond this, addition of more debt reduces the stock price.

Question: 2447

Ace Consulting, a multinational corporate finance consulting firm, is examining the operations of
Minishabby Farms, an Irish conglomerate who is considering the development of a new distilling
process for their specialty spirits division. In order to determine the feasibility of the new distilling
process, Ace Consulting is trying to determine the beta of the proposed project. In their analysis, Ace
Consulting begins by identifying publicly-traded companies whose operations are solely within the
distilling business. Ace identifies four such firms, determines the beta of each Company, and
averages them together. This figure is used as the beta of the proposed project. Which of the
following techniques most correctly describes this method of identifying individual project betas?

A. Monte Carlo simulation


B. Empirical smoothing
C. Accounting beta method
D. Relational method
E. Pure play method
F. Case study analysis

Answer: E

Explanation:
In this example, Ace Consulting has used the beta coefficient of four firms whose business is solely

Page | 919
within the distilling process. These firms are "pure plays" in the distilling field. The Pure Play method
of determining project betas is a popular technique in corporate finance, most likely due to its ease
of use. However, the identification of "pure play" firms is often difficult for certain projects. In these
cases, the second method of determining project betas is employed, the Accounting Beta method.
"Monte Carlo simulation" is a method for evaluating stand-alone risk, and "empirical smoothing" is a
fictitious term.

Question: 2448

If the firm is being operated so as to maximize shareholder wealth, and if our basic assumptions
concerning the relationship between risk and return are true, then which of the following should be
true?

A. If the beta of the asset is greater than the corporate beta prior to the addition of that asset, then
the corporate beta after the purchase of the asset will be smaller than the original corporate beta.
B. If the beta of the asset is smaller than the firm's beta, then the required return on the asset is
greater than the required return on the firm.
C. None of these answers are true.
D. If the beta of an asset is larger than the corporate beta prior to the addition of that asset, then the
required return on the firm will be greater after the purchase of that asset than prior to its purchase.
E. If the beta of the asset is larger than the firm's beta, then the required return on the asset is less
than the required return on the firm.

Answer: D

Explanation:
An increase in a project's beta will cause its stock price to decrease unless the increased beta were
offset by a higher expected rate of return. Therefore, high-risk investments require higher rates of
return, whereas low-risk investments require lower rates of return.

Question: 2449

A project has the following cash flows over the next 5 years: $1,000, $600, $300, $1,200 and $1,400.
Assume all cash flows occur at the end of a year. The project requires an initial cash outlay of $2,900.
The project's cost of capital is 8%. The discounted payback period for the project equals ________.

A. 4.11 years
B. 3.84 years
C. 4.81 years
D. 4.36 years

Answer: D

Explanation:
The discounted payback period is defined as the expected number of years that would be required to
recover the original investment using discounted cash flows. The discounted cash flow at the end of
year N is obtained by dividing that year's cash flow by 1.08^N, since the project's cost of capital is 8%.
Using this, the discounted cash flows are: $926, $514, $238, $882 and $953. Recovery occurs in the
5th year. At the beginning of the 5th year, the outstanding balance equals 2,900-926-514-238-882 =
$340. Therefore, the discounted payback period = 4 + 340/953 = 4.36 years.

Question: 2450

Which of the following statements is most correct?

Page | 920
A. All else being equal, an increase in a firm's fixed costs will decrease its degree of operating
leverage.
B. All of these statements are correct.
C. Firms that have large fixed costs and low variable costs have a higher degree of financial leverage
than do firms with low fixed costs and high variable costs.
D. If a firm's net income rises 10 percent every time its EBIT rises 10 percent, this implies the firm has
no debt outstanding.
E. None of these statements are correct.

Answer: D

Explanation:
If no debt were used, there will be no interest charges, which is included in net income but not EBIT.

Question: 2451

Grateway Inc. has a weighted average cost of capital of 11.5 percent. Its target capital structure is 55
percent equity and 45 percent debt. The company has sufficient retained earnings to fund the equity
portion of its capital budget. The before-tax cost of debt is 9 percent, and the company's tax rate is 30
percent. If the expected dividend next period and current stock price are $5 and $45, respectively,
what is the company's growth rate?

A. 3.44%
B. 8.16%
C. 2.68%
D. 4.64%
E. 6.75%

Answer: D

Explanation:
Solve for k(s) (component cost of retained earnings or internal equity):
WACC (Weighted Average Cost of Capital) = 11.5% = w(s)k(s) + w(d)k(d)(1 - T)
11.5% = 0.55k(s) + 0.45(0.09)(0.70)
k(s) = 15.75%.
Solve for g: k(s) = 15.75% = D1/P0 + g
15.75% = $5/$45 + g
g = 4.64%.

Question: 2452

A project has the following cash flows over the next 5 years: $1,000, $600, $300, $1,200 and $1,400.
Assume all cash flows occur at the end of a year. The project requires an initial cash outlay of $2,900.
The project's cost of capital is 8%. The MIRR of the project equals ________.

A. 11.54%
B. 12.22%
C. 9.92%
D. 14.19%

Answer: B

Explanation:

Page | 921
The MIRR is defined as that rate which discounts the terminal value of the cash inflows to equate to
the present value of a project's costs (using the project's cost of capital). The present value of the
costs = 2,900. The terminal value (future value at the end of year 5) of the project equals 1,000*1.084
+ 600*1.083 + 300*1.082 + 1,200*1.08 + 1,400 = 5,162. Note that this is calculated using the project's
cost of capital. Then, MIRR satisfies 2,900 = 5,162/(1+MIrr)5. Solving gives MIRR = 12.22%.

Question: 2453

Which of the following statements is most correct?

A. The MIRR method will always arrive at the same conclusion as the NPV method.
B. All of the statements are correct.
C. The MIRR method can overcome the multiple IRR problem, while the NPV method cannot.
D. None of the statements are correct.
E. The MIRR method uses a more reasonable assumption about reinvestment rates than the IRR
method.

Answer: E

Explanation:
MIRR and NPV can conflict for mutually exclusive projects if the projects differ in size. NPV does not
suffer from the multiple IRR problem.

Question: 2454

Braun Industries is considering an investment project, which has the following cash flows:
tProject Cash Flows
0-$1,000
1400
2300
3500
4400
The company's WACC is 10 percent. What is the project's payback, internal rate of return and net
present value?

A. Payback = 2.6, IRR = 24.12%, NPV = $300.


B. Payback = 2.6, IRR = 21.22%, NPV = $260.
C. Payback = 2.4, IRR = 21.22%, NPV = $260.
D. Payback = 2.6, IRR = 21.22%, NPV = $300.
E. Payback = 2.4, IRR = 10.00%, NPV = $600.

Answer: B

Explanation:
Payback = 2 + 300/500 = 2.6 years.
IRR: -1000 + 400/(1+Irr)^1 + 300/(1+Irr)^2 + 500/(1+Irr)^3 + 400/(1+Irr)^4 = 0: IRR = 21.22%.
NPV = -1000 + 400/(1+.10)^1 + 300/(1+.10)^2 + 500/(1+.10)^3 + 400/(1+.10)^4 = $260.46.

Question: 2455

The date on which if you are listed by the company as an owner, you will receive a dividend is known
as the:

A. Holder-of-Record Date

Page | 922
B. Declaration Date
C. Beneficiary Date
D. Payment Date
E. Ex-Dividend DateThat answer is correct!

Answer: A

Explanation:
The "Holder-of-Record Date" is the date on which if you are listed by the company as an owner, you
will receive the dividend.

Question: 2456

Which of the following statements is most correct?

A. The component cost of preferred stock is expressed as k(ps) (1 - T), because preferred stock
dividends are treated as fixed charges, similar to the treatment of debt interest.
B. Due to the way the Marginal Cost of Capital (MCC) schedule is constructed, the first breakpoint in
the MCC schedule must be associated with using up all available retained earnings and having to
issue common stock.
C. The cost of equity obtained by using retained earnings is generally regarded as being the rate of
return stockholders require on the firm's outstanding common stock.
D. The higher the firm's flotation cost for new common equity, the more likely the firm is to use
preferred stock, which has no flotation cost.
E. The bond-yield-plus-risk-premium approach to estimating a firm's cost of common equity involves
adding a subjectively determined risk-premium to the market risk-free bond rate.

Answer: C

Explanation:
Preferred stock dividends are not tax deductible; therefore, the component cost of preferred stock is
only k(ps). The risk premium in the bond-yield-plus-risk premium approach would be added to the
firm's cost of debt, not the risk-free rate. The first break point does not have to be associated with
retained earnings. It could be from other sources of funds such as debt. The choice between
preferred stock or common equity financing depends on a number of factors, including required
return, flotation costs, management's desired capital structure, etc.

Question: 2457

Regarding the net present value of a replacement decision, which of the following statements is
false?

A. The present value of the after-tax cost reduction benefits resulting from the new investment is
treated as an inflow.
B. The present value of depreciation expenses on the new equipment, multiplied by the tax rate, is
treated as an inflow.
C. An increase in net working capital is treated as an outflow when the project begins and as an
inflow when the project ends.
D. Any loss on the sale of the old equipment is multiplied by the tax rate and is treated as an outflow
at t = 0.
E. The after-tax market value of the old equipment is treated as an inflow at t = 0.

Answer: D

Page | 923
Explanation:
Since the old equipment is sold at a loss which reduces taxable income, a tax savings is realized and is
deducted from the investment outlay.

Question: 2458

The Global Advertising Company had net income after interest but before taxes of $40,000 this year.
The marginal tax rate is 40 percent, and the dividend payout ratio is 30 percent. The company can
raise debt at a 12 percent interest rate. The last dividend paid by Global was $0.90. Global's common
stock is selling for $8.59 per share, and its expected growth rate in earnings and dividends is 5
percent. If Global issues new common stock, the flotation cost incurred will be 10 percent. Global
plans to finance all capital expenditures with 30 percent debt and 70 percent equity. What is Global's
cost of retained earnings?

A. 10.33%
B. 9.66%
C. 12.22%
D. 17.22%
E. 16.00%

Answer: E

Explanation:
k(s) (component cost of retained earnings) = $.945/$8.59 + 0.05 = 0.1600 = 16.00%.

Question: 2459

Marcus Corporation currently sells 150,000 units a year at a price of $4.00 a unit. Its variable costs
are approximately 30 percent of sales, and its fixed costs amount to 50 percent of revenues at its
current output level. Although fixed costs are based on revenues at the current output level, the cost
level is fixed. What is Marcus's degree of operating leverage in sales dollars?

A. 2.2
B. 3.5
C. 1.0
D. 5.0
E. 4.0

Answer: B

Explanation:
Use the information provided and the formula for DOL in sales dollars: DOL(s) = [($150,000 x $4.00)-
180,000]/[($150,000 x $4.00)-180,000-300,000] = 3.5

Question: 2460

The Global Advertising Company had net income after interest but before taxes of $40,000 this year.
The marginal tax rate is 40 percent, and the dividend payout ratio is 30 percent. The company can
raise debt at a 12 percent interest rate. The last dividend paid by Global was $0.90. Global's common
stock is selling for $8.59 per share, and its expected growth rate in earnings and dividends is 5
percent. If Global issues new common stock, the flotation cost incurred will be 10 percent. Global
plans to finance all capital expenditures with 30 percent debt and 70 percent equity. What is the
break point due to retained earnings being used up?

Page | 924
A. $17,000
B. $24,000
C. $10,000
D. $30,000
E. $56,000

Answer: B

Explanation:
Calculate net income and retained earnings
EBT$40,000
Less: Taxes$16,000
NI$24,000
RE = 0.70($24,000) = $16,800.
Break point retained earnings:
BP(RE) = $16,800/0.70 = $24,000.

Question: 2461

A firm's capital structure consists of 25% debt with a pre-tax cost of 7% and an after-tax cost of 4.9%.
Common equity makes up 45% of the structure and the rest is made up of preferred equity.
Thepreferred stock has a coupon of 8% and is currently trading at 84% of its par value. The required
rate of return on the common stock is 16.2%. The firm's WACC is ________.

A. 11.5%
B. 10.92%
C. 11.37%
D. 11.90%

Answer: C

Explanation:
To get the WACC in this case, you need to have information on the cost of preferred stock. This is not
necessarily equal to the coupon rate on the preferred equity. Rather, it is the discount rate, R, that
equates the present value of the perpetual payments on the preferred equity to its current price. The
price of a perpetuity that pays C per year, at a discount rate of R, equals C/R. In this problem, since
the preferred stock is at 84% and pays 8% coupon, we have 84% = 8%/R, giving R = 9.52%. Now, the
interest payments on debt are tax-deductible but those on preferred equity are not. Hence, no tax
adjustment is necessary for preferred stock but you must use after-tax cost of debt in WACC
calculations. With this in mind, WACC = 0.25*4.9% + 0.45*16.2% + 0.3*9.52% = 11.37%.

Question: 2462

A project requires an initial outlay of 650. It also needs capital spending of 700 at the end of year 1
and 900 at the end of year 2. It has no revenues for the first 2 years but receives 1,200 in year 3,
1,600 in year 4 and 2,300 in year 5. The project's cost of capital is 10%. The discounted payback
period equals ________.

A. 2.26 years
B. 4.02 years
C. 3.19 years
D. 3.46 years

Answer: B

Page | 925
Explanation:
The cash flows of the project starting at the end of year 1 are:
-700, -900, +1,200, +1,600, +2,300
The discounted payback period is defined as the expected number of years that would be required to
recover the original investment using discounted cash flows. The discounted cash flow at the end of
year N is obtained by dividing that year's cash flow by 1.1N, since the project's cost of capital is 10%.
Using this, the discounted cash flows are:
-636, -744, +902, +1,093, +1,428.
Recovery occurs in the 5th year. At the beginning of the 5th year, the outstanding balance equals 650
+ 636 + 744 - 902 - 1093 = 35. Therefore, the discounted payback period = 4 + 35/1428 = 4.02 years.

Question: 2463

O'Donnell Inc. has a cost of capital of 11.5 percent. The company has a project with the following
cash flows:
YearCash flow
0-$200
1235
2-65
3300
What is the project's modified internal rate of return (MIRR)?

A. 28.15%
B. 39.87%
C. 40.15%
D. 32.90%
E. 36.27%

Answer: D

Explanation:
Step 1 Find the PV of the outflows:
CF(0) = -200
CF(1) = 0
CF(2) = -65
I = 11.5
Solve for NPV = -$252.28.
Step 2 Find the TV of the inflows by first finding their NPV and then finding the FV of the NPV:
CF(0) = 0 N = 3
CF(1) = 235 I = 11.5
CF(2) = 0 PV = -427.18
CF(3) = 300 PMT = 0
I = 11.5 Solve for FV = $592.16.
Solve for NPV = $427.18.
Step 3 Calculate the MIRR by equating the PV of the outflows with the TV of the inflows:
N=3
PV = -252.28
PMT = 0
FV = 592.16
Solve for I = 32.90%.

Question: 2464

Page | 926
A financial analyst is examining shares of Floweration.com, a sprawling Internet "dot com" company,
in an attempt to evaluate the firm for possible investment. In her analysis, the financial analyst has
determined the following information for the most recent fiscal year:
Sales $2,250,000
Total fixed cost $1,300,000
Total variable cost $305,000
Interest expense $5,750
EBIT $645,000
Amortization expense $4,550
Given this information, what is the Degree of Total Leverage for Floweration.com?

A. 2.843
B. 3.064
C. 1.731
D. 3.488
E. 3.043

Answer: E

Explanation:
The Degree of Total Leverage (DTL) demonstrates how a given change in sales will impact a firm's
EPS. The equation used for calculating the DTL is as follows: {[Sales - variable costs] / [sales - variable
costs - fixed costs - interest expense]}. Incorporating the given values for these components into the
DTL equation yields the following: {[Sales $2,250,000 - variable costs $305,000] / [sales $2,250,000 -
variable costs $305,000 - fixed costs $1,300,000 - interest expense $5,750]}=3.043. While somewhat
intuitively appealing, "EBIT" and "amortization expense" are not explicitly incorporated into the DTL
equation.

Question: 2465

Davis Corporation is faced with two independent investment opportunities. The Corporation has an
investment policy, which requires acceptable projects to recover all costs within 3 years. The
Corporation uses the discounted payback method to assess potential projects and utilizes a discount
rate of 10 percent. The cash flows for the two projects are:
TimeProject AProject B
0-100,000-$80,000
140,00050,000
240,00020,000
340,00030,000
430,0000
Which investment project(s) does the company invest in?

A. Project B only.
B. Project A only.
C. Neither Project A or Project B.
D. Project A and Project B.That answer is correct!

Answer: A

Explanation:
The sum of the PVs of the t = 1, t = 2, and t = 3 cash flows at t = 0 for Project A is $99,474.08. Thus,
the discounted payback period of Project A exceeds 3 years and Project A is not acceptable. The PVs
of the t = 1, t = 2, and t = 3 cash flows at t = 0 for Project B are $45,454.55, $16,528.93, and
$22,539.44, respectively. These PVs sum to $84,522.92, which is greater than the cost of the project,

Page | 927
indicating that the discounted payback period is less than 3 years. Thus, Project B will be undertaken.

Question: 2466

Rollins Corporation is constructing its MCC schedule. Its target capital structure is 20 percent debt, 20
percent preferred stock, and 60 percent common equity. Its bonds have a 12 percent coupon, paid
semiannually, a current maturity of 20 years, and sell for $1,000. The firm could sell, at par, $100
preferred stock, which pays a 12 percent annual dividend, but flotation costs of 5 percent would be
incurred. Rollins' beta is 1.2, the risk-free rate is 10 percent, and the market risk premium is 5
percent. Rollins is a constant growth firm, which just paid a dividend of $2.00, sells for $27.00 per
share, and has a growth rate of 8 percent. The firm's policy is to use a risk premium of 4 percentage
points when using the bond-yield-plus-risk- premium method to find k(s). The firm's net income is
expected to be $1 million, and its dividend payout ratio is 40 percent. Flotation costs on new
common stock total 10 percent, and the firm's marginal tax rate is 40 percent. What is Rollins' cost of
retained earnings using the bond-yield-plus-risk-premium approach?

A. 16.6%
B. 16.0%
C. 16.9%
D. 14.1%
E. 13.6%

Answer: B

Explanation:
Cost of retained earnings (Bond yield plus risk premium approach): k(s) = 12.0% + 4.0% = 16.0%.

Question: 2467

The opportunity costs of a project refer to:

A. the costs already incurred in developing the project.


B. none of these answers.
C. the costs that would be incurred in one or more of mutually exclusive projects that are rejected in
favor of the project selected.
D. the costs incurred due to the effects of the project on the firm's other projects at hand.

Answer: B

Explanation:
The cash flows that could be earned if the assets were not used for the project but elsewhere are
known as "opportunity costs." None of the given choices fits this definition. Note that the costs
already incurred in developing the project are called "sunk costs," the costs incurred due to the
effects of the project on the firm's other projects at hand are "externality" or "spill-over" costs. "The
costs that would be incurred in one or more of mutually exclusive projects that are rejected in favor
of the project selected" looks tempting but is not an opportunity cost.

Question: 2468

Green Grocers is deciding among two mutually exclusive projects. The two projects have the
following cash flows:
Project AProject B
TimeCash FlowsCash Flows
0-$50,000-$30,000

Page | 928
110,0006,000
215,00012,000
340,00018,000
420,00012,000
The company's cost of capital is 10 percent (WACC = 10%). What is the net present value (NPV) of the
project with the highest internal rate of return (IRR)?

A. $7,090
B. $12,510
C. $8,360
D. $11,450
E. $15,200

Answer: E

Explanation:
NPV(A) = $15,200; IRR(A) = 21.3811%.
NPV(B) = $7,092; IRR(B) = 19.2783%.
Project A has the highest IRR, so the answer is $15,200.

Question: 2469

Dandy Product's overall weighted average required rate of return is 10 percent. Its yogurt division is
riskier than average, its fresh produce division has average risk, and its institutional foods division
has below-average risk. Dandy adjusts for both divisional and project risk by adding or subtracting 2
percentage points. Thus, the maximum adjustment is 4 percentage points. What is the risk-adjusted
required rate of return for a low-risk project in the yogurt division?

A. 8%
B. 10%
C. 14%
D. 12%
E. 6%

Answer: B

Explanation:
k(YD) = 10% + 2% = 12%.
However, for a low-risk project, Dandy Product subtracts 2 percentage points. Therefore, the
required rate of return is 10 percent.
k(YD,Low risk project) = 10% + 2% - 2% = 10%.

Question: 2470

Alvarez Technologies has sales of $3,000,000. The company's fixed operating costs total $500,000 and
its variable costs equal 60 percent of sales, so the company's current operating income is $700,000.
The company's interest expense is $500,000. What is the company's degree of total leverage (DTL)?

A. 3.500
B. 6.000
C. 1.714
D. 3.100
E. 3.250

Page | 929
Answer: B

Explanation:
DTL = (S - VC)/(EBT - I)
= ($3,000,000 - $1,800,000)/($700,000 - $500,000)
= 6.

Question: 2471

Intelligent Semiconductor is considering issuing additional common stock. The current yield to
maturity on the firm's outstanding senior long-term debt is 13%. The company's combined
federal/state income tax is 35%. The risk-free rate of return is 5.6%, and the annual return on the
broadest market index is expected to be 13.5%. Shares of Intelligent Semiconductor have a historical
beta of 1.6. In the past, the firm has assumed a 265 basis point risk premium when calculating the
cost of equity. What is the cost of equity for this proposed common stock issue using the Bond-Yield-
plus-Risk-Premium approach?

A. 15.65%
B. 16.15%
C. 18.24%
D. 11.20%
E. The cost of equity using the Bond-Yield-plus-Risk-Premium approach cannot be calculated from
the information provided.That answer is correct!

Answer: A

Explanation:
The cost of issuing common stock can be calculated using several methods, including the Bond-Yield-
Plus-Risk-Premium approach, Discounted Cash Flow method, or by using the Capital Asset Pricing
Model. In this example, you have been asked to calculate the cost of equity using the Bond-Yield-
plus- Risk-Premium approach. This method is a rather ad hoc procedure used by finance
professionals, and involves adding a subjective "risk premium" to the yield to maturity of the firm's
outstanding debt. This approach should be used with a degree of caution, as the subjective nature of
the procedure leaves significant room for variation in estimates. In using the Bond-Yield-Plus-Risk-
Premium approach, a subjective risk premium is added to the yield to maturity of the firm's
outstanding long-term debt, and the calculation of the correct answer in this case is as follows: {yield
to maturity of the outstanding senior debt 13% + subjective risk premium 2.65%}=15.65%.

Question: 2472

A firm has just issued 6%, 10-year coupon bonds, which have a yield-to-maturity of 7.1%. The firm
has old debt, which pays a coupon of 8%. The firm is in the 45% tax bracket. Its marginal after-tax
cost of debt equals ________.

A. 3.3%
B. 3.2%
C. 4.4%
D. 3.9%

Answer: D

Explanation:
Since debt interest is tax-deductible, the after-tax cost of debt equals 7.1%*(1-45%) = 3.9%. Note that
you must use the yield-to-maturity of the new bonds while calculating the cost of debt since this is

Page | 930
the rate at which interest expense is computed in maintaining the accounts. The coupon rate does
not enter directly in this calculation (though it does matter since it affects the yield-to-maturity).

Question: 2473

Mid-State Electric Company must clean up the water released from its generating plant. The
company's cost of capital is 10 percent for average projects, and that rate is normally adjusted up or
down by 2 percentage points for high- and low-risk projects. Clean-up Plan A, which is of average
risk, has an initial cost of -$1,000 at Time 0, and its operating cost will be -$100 per year for its 10-
year life. Plan B, which is a high-risk project, has an initial cost of -$300, and its annual operating cost
over Years 1 to 10 will be -$200. What is the proper PV of costs for the better project?

A. -$1,642.02
B. -$1,430.04
C. -$1,728.19
D. -$1,525.88
E. -$1,614.46

Answer: E

Explanation:
The first thing to note is that risky cash outflows should be discounted at a lower discount rate, so in
this case we would discount the riskier Project B's cash flows at 10% - 2% = 8%. Project A's cash flows
would be discounted at 10%.
Now we would find the PV of the costs as follows:
Project AProject B
CF(0) = -1,000CF(0) =-300
CF(1-10) = -100CF(1-10)=-200
I =10.0I= 8.0
Solve for NPV = -$1,614.46. Solve for NPV = -$1,642.02.
Project A has the lower PV of costs. If Project B had been evaluated with a 12% cost of capital, its PV
of costs would have been -$1,430.04, but that would have been wrong.

Question: 2474

After getting her degree in marketing and working for 5 years for a large department store, Sally
started her own specialty shop in a regional mall. Sally's current lease calls for payments of $1,000 at
the end of each month for the next 60 months. Now the landlord offers Sally a new 5-year lease
which calls for zero rent for 6 months, then rental payments of $1,050 at the end of each month for
the next 54 months. Sally's cost of capital is 11 percent. By what absolute dollar amount would
accepting the new lease change Sally's theoretical net worth?

A. $4,681.76
B. $3,803.06
C. $4,299.87
D. $3,243.24
E. $2,810.09

Answer: B

Explanation:
CF(0) = 0
CF(1-6) = 0
CF(7-60) = 1,050

Page | 931
I = 11/12 = 0.9167
Solve for NPV = -$42,189.97. Therefore, the PV of payments under the proposed lease would be less
than the PV of payments under the old lease by $45,993.03 - $42,189.97 = $3,803.06. Sally should
accept the new lease because it would raise her theoretical net worth by $3,803.06.

Question: 2475

In comparing two mutually exclusive projects of equal size and equal life, which of the following
statements is most correct?

A. None of these answers are correct.


B. The project with the higher NPV may not always be the project with the higher IRR.
C. The project with the higher NPV may not always be the project with the higher MIRR.
D. The project with the higher IRR will always be the project with the higher MIRR.
E. All of the answers are correct.

Answer: B

Explanation:
Due to reinvestment rate assumptions, NPV and IRR can lead to conflicts; however, there will be no
conflict between NPV and MIRR if the projects are equal in size (which is one of the assumptions in
this question).

Question: 2476

The Ace Company is considering investing in a piece of property, which costs $105,000. The property
will return a constant cash flow forever. If the firm's cost of capital is 9 percent and the corporate tax
rate is 40 percent, what is the minimum after-tax cash flow that would make the investment
acceptable to Ace?

A. $2,375
B. $15,942
C. $5,000
D. $10,831
E. $9,450

Answer: E

Explanation:
$105,000 = CF(AT)/0.09; CF(AT) = $9,450.

Question: 2477

The percentage mix of debt, preferred stock, and common equity that will maximize a firm's stock
price is known as:

A. Marginal cost of capital


B. Weighted average cost of capital (WACC)
C. After tax cost of capital
D. Marked to market value of equity
E. Target (Optimal) Capital Structure

Answer: E

Page | 932
Explanation:
The target or optimal capital structure of a firm is that percentage mix of debt, preferred stock, and
common equity that will maximize the firm's stock price.

Question: 2478

Which of the following is false?

A. The IRR rule is not dependable when applied to projects with non-normal cash flows.
B. For independent projects with normal cash flows, the IRR and NPV rules give the same
accept/reject results.
C. For mutually exclusive projects, the IRR and NPV rules can give conflicting results.
D. None of these answers.

Answer: D

Explanation:
Remember to always use the NPV result. There are quite a few problems with using other decision
rules like IRR and payback period.

Question: 2479

Calculate the cost of debt for the following firm:


Borrowing Rate 9.5%
Historical Beta .97
Marginal Tax Rate 40%
Credit Rating BB+
Owner's Equity 15%
Quick Ratio 1.7
EPS $1.70
P/E ratio 12
Estimated Dividends $.30

A. 8.075%
B. 6.27%
C. 1.43%
D. 5.7%
E. 1.5%
F. 9.5%

Answer: D

Explanation:
The cost of debt is simply the rate of borrowing less the tax savings. Due to the fact that interest
expense is tax deductible, the cost of debt in this case is 9.5%(1 - .4) = 9.5%(.6) = 5.7%.

Question: 2480

Bell Brothers has $3,000,000 in sales. Its fixed costs are estimated to be $100,000, and its variable
costs are equal to fifty cents for every dollar of sales. The company has $1,000,000 in debt
outstanding at a before-tax cost of 10 percent. If Bell Brothers' sales were to increase by 20 percent,
how much of a percentage increase would you expect in the company's net income?

A. 15.66%

Page | 933
B. 18.33%
C. 19.24%
D. 23.08%
E. 21.50%

Answer: D

Explanation:
Step 1 Find Degree of Total Leverage (DTL)
DTL = (S-VC)/(S-VC-F-I) = ($3,000,000-$1,500,000)/($3,000,000-$1,500,000-$100,000-$100,000 =
1.1538.
Step 2 Find percentage increase in net income:
%Change NI = (0.20)(DTL) = (0.20)(1.1538) = 0.2308 = 23.08%.

Question: 2481

A firm is considering a project whose estimated cash flows have indicated a payback period of 3.68
years. It requires an initial outlay of $1,000 and has end-of-year cash flows of $350, $270 and $225 in
the first 3 years. The firm's marginal discount rate is 9%. The project's projected cash flow for year 4
equals ________.

A. 373
B. 514
C. 495
D. 228

Answer: D

Explanation:
The payback period is defined as the expected number of years that would be required to recover the
original investment. In particular, Payback period = Years before full recovery + (unrecovered cost at
the start of payback year)/(net cash flow in the payback year) In this case, the recovery occurs in the
3rd year. At the beginning of the 3rd year, the unrecovered cost equals 1,000 - 350 - 270 - 225 = 155.
If total cash flow in the 4th year equals C, then payback period = 3 + 155/C = 3.68 years. Solving for C
gives C = 228. Note that the discount rate does not figure in the calculation of payback period.

Question: 2482

A firm has a target dividend payout ratio of 36% and net income of $1.7 million. It is committed to
maintaining an optimal capital structure consisting of 63% debt and 37% equity. The firm is in the
40% tax bracket. Its retained earnings breakpoint equals ________.

A. $1.89 million
B. $2.58 million
C. $1.31 million
D. $2.75 million
E. $2.94 million
F. $3.41 million
G. $4.64 million

Answer: E

Explanation:
The retained earnings breakpoint refers to the maximum amount of capital that can be raised using

Page | 934
retained earnings, assuming a constant capital structure and dividend payout ratio. In other words, it
is the maximum amount of capital that can be raised without increasing the marginal cost of capital.
In the present example, the maximum internal equity capital equals 1.7 * 0.64 = $1.088 million. To
maintain a D/E ratio of 63/37 = 1.7, the amount of debt to be issued equals 1.088 * 1.7 = $1.85
million. Therefore, earnings break-point equals (1.088 + 1.85) million = $2.938 million.

Question: 2483

A project has a high correlation with the firm's other projects. It also has a low CAPM beta. The
project will have ________ corporate risk and ________ market risk.

A. high; high
B. high; low
C. low; low
D. low; high

Answer: B

Explanation:
Corporate risk measures the impact of a project on the corporate earnings while market risk
measures its effect on the systematic risk of the stock. Since the project's returns are highly
correlated with those of the firm's other projects, it will increase the earnings volatility. At the same
time, due to low beta, the project will have a low market risk.

Question: 2484

Which of the following statements is most correct?

A. If it could be demonstrated that a clientele effect exists, this would suggest that firms could alter
their dividend payment policies from year to year to take advantage of investment opportunities
without having to worry about the effects of changing dividends on capital costs.
B. Each of these statements are false.
C. If a company raises its dividend by an unexpectedly large amount, the announcement of this new
and higher dividend is generally accompanied by an increase in the stock price. This is consistent
with the bird-in-the-hand theory, and Modigliani and Miller used these findings to support their
position on dividend theory.
D. If the dividend irrelevance theory (which is associated with the names Modigliani and Miller) were
exactly correct, and if this theory could be tested with "clean" data, then we would find, in a
regression of dividend yield and capital gains, a line with a slope of -1.0.
E. The tax preference and bird-in-the-hand theories lead to identical conclusions as to the optimal
dividend policy.

Answer: D

Explanation:
The main conclusion of MM's irrelevance theory is that dividend policy does not affect the required
rate of return on equity. MM theorized that k(s) is independent of dividend policy, implying that
investors are indifferent between dividends and capital gains.

Question: 2485

Which of the following statements is most correct?

A. The optimal capital structure simultaneously maximizes EPS and minimizes the WACC.

Page | 935
B. As a rule, the optimal capital structure is found by determining the debt-equity mix that maximizes
expected EPS.
C. The optimal capital structure minimizes the cost of equity, which is a necessary condition for
maximizing the stock price.
D. All of these statements are false.
E. The optimal capital structure simultaneously minimizes the cost of debt, the cost of equity, and the
WACC.

Answer: D

Explanation:
The optimal capital structure is the one that maximizes the price of the firm's stock, and minimizes
the firm's WACC.

Question: 2486

Foundation Systems, a software engineering company, is considering the acceptance of two mutually
exclusive projects. Assume the following information:
Project A
Initial cash outlay ($40,000)
t1: $8,000
t2: $14,000
t3: $13,000
t4: $12,000
t5: $11,000
t6: $10,000
cost of capital is 11.5%
Project B
Initial cash outlay ($20,000)
t1: $7,000
t2: $13,000
t3 $12,000
cost of capital is 11.5%
Assuming no taxes, a $0.00 salvage value at the end of each project, and the fact that both projects
can be replicated identically at the end of their lives, which is the superior project according to the
Common Life approach? Additionally, what is the NPV of the superior project over the common life?

A. Project A, NPV $7,165.11


B. Project A, NPV $9,280.90
C. The Common Life approach cannot be applied to these two projects, due to the fact that the
projects share unequal lives.
D. Project B, NPV $9,280.90
E. Project B, NPV $5,391.49

Answer: D

Explanation:
The Common Life, or "Replacement Chain" approach is a method which allows two projects with
differing lives to be compared on the basis of NPV or IRR. In analyzing two or more projects using
theCommon Life approach, the two projects are multiplied in such a way that each comprises the
same amount of time periods. In this example, you are provided with one project that is three
periods long, and another which is six periods. By multiplying the three-period project by two, then
we are able to formulate a situation in which both projects share a "common life." The calculation of
the NPV for Project B assumes the following series of cash flows:

Page | 936
t0: ($20,000)
t1: $7,000
t2: $13,000
t3: [$12,000 + ($20,000)]= ($8,000)
t4: $7,000
t5: $13,000
t6: $12,000
During t3, the project is "replaced," and the $12,000 inflow is offset by the ($20,000) required to
implement the project once again. Incorporating this series of cash flows into your calculator will
yield a NPV of $9,280.90 for project B, and this is higher than the NPV of $7,165.11 for project
A.

Question: 2487

Which of the following are objectives of conducting a post audit? Choose the best answer.
I. Identifying arbitrage opportunities
II. Improving forecasts
III. Identifying expansion opportunities
IV. Improve operations
V. Adhering to governmental guidelines for performance presentation

A. I, II, V
B. I, II, III, IV
C. I, II, VII
D. II, IV
E. I, II, III, IV, V
F. I, III, IV

Answer: D

Explanation:
The post-audit is an important aspect of the capital budgeting process, and involves two steps.
Specifically, the post audit involves comparing actual results with forecasted results and determining
why any differences exist. There are numerous specific reasons why companies conduct post-audits,
but these reasons can be assimilated into two main objectives. Specifically, the objectives of the
postaudit are to improve forecasting capacity and improve operations.

Question: 2488

In an investigation of Clay Industries, Marcus Litton, a financial analyst, has determined the following
information:
Sales: $300,000,000
Fixed costs: $100,000,000
Variable costs: $115,200,000
Interest expense: $1,800,000
Tax rate: 35%
Weighted Average Cost of Capital: 10.15%
Beta coefficient: 0.80
Common shares outstanding: 10,000,000
Mr. Litton has asked for your assistance in determining the earnings per share (EPS) of Clay
Industries. Using this information, which of the following answers correctly illustrates this EPS
calculation?

A. $2.91

Page | 937
B. $6.17
C. $6.80
D. The EPS figure cannot be completely determined from the information provided.
E. $4.90
F. $5.40

Answer: F

Explanation:
The EPS figure is perhaps the single most popular term in the field of conventional equity
investments. Any glance into financial media and business periodicals will undoubtedly uncover
numerous instances in which the EPS figure is cited. While quite popular and useful, many individuals
do not understand the mechanics behind the EPS calculation, and an investigation into the
components of EPS is a valuable learning experience. The EPS calculation is found by the following
equation: {EPS = [(Sales - Fixed Costs - Variable Costs - Interest Expense)(1 - Tax Rate)] / [# of
Common Shares Outstanding]} Additionally, the EPS figure can be found by: {EPS = [(EBIT - Interest
Expense)(1 - Tax Rate) / # of Common Shares Outstanding]} Incorporating the given information into
the first EPS equation will yield the following: {EPS = {[($300,000,000 - $100,000,000 - $115,200,000 -
$1,800,000)(1 - 0.35)] / 10,000,000]} = $5.40.

Question: 2489

Howell Enterprises is forecasting EPS of $4.00 per share for next year. The firm has 10,000 shares
outstanding, it pays 12 percent interest on its debt, and it faces a 40 percent marginal tax rate. Its
estimated fixed costs are $80,000 while its variable costs are estimated at 40 percent of revenue. The
firm's target capital structure is 40 percent equity and 60 percent debt and it has total assets of
$400,000. On what level of sales is Howell basing its EPS forecast?

A. $105,280
B. $292,444
C. $1,000,000
D. $480,400
E. $316,722

Answer: B

Explanation:
EPS = (Sales - Fixed costs- Variable costs - Interest)(1 - T)/Shares outstanding.
Step 1 Calculate interest expense
Debt = 0.60 x $400,000 = $240,000.
Interest = 0.12 x $240,000 = $28,800.
Step 2 Solve for Sales (S)
EPS = $4.00 = (S - 0.40S - $80,000 - $28,800) x (1 - 0.40)/10,000
= (0.60S - $108,800)(0.6)/10,000
$4.00 = (0.36S - $65,280)/10,000
$105,280 = 0.36S
Sales = $292,444.44.
Alternative method
EPS = (EBIT - Interest)(1 - T)/Shares outstanding.
Solve for EBIT
Net Income = EPS x Shares outstanding = $4.00 x 10,000 = $40,000.
EBT = NI/(1 - T) = $40,000/(0.6) = $66,667.
Interest (from above) = $28,800.
EBIT = EBT + Interest = $66,667 + $28,800 = $95,467.

Page | 938
S = 0.40S + $95,467 + $80,000
0.6S = $175,467
S = $175,467/0.6 = $292,445.

Question: 2490

An increase in the tax rate ________ the optimal debt-to-equity ratio. It ________ the after-tax cost
of debt. Assume all else equal.

A. this answer cannot be generated because "tax" is not a factor in the optimal debt-to-equity ratio
B. increases; increases
C. this answer cannot be generated because "tax" is not a factor in after-tax cost of debt
D. decreases; increases
E. increases; decreases
F. decreases; decreases

Answer: E

Explanation:
Since interest payments are tax deductible, higher tax rates make debt more attractive relative to
equity, increasing the optimal D/E ratio. The after-tax cost of debt decrease, assuming that the pre-
tax cost of debt is not affected by the change in the tax rate (in reality, it could increase due to
decreased profitability of the firm and the resultant decrease in its solvency).

Question: 2491

The internal rate of return of a capital investment

A. changes when the cost of capital changes.


B. must exceed the cost of capital in order for the firm to accept the investment.
C. is equal to the annual net cash flows divided by one half of the project's cost when the cash flows
are an annuity.
D. all of the answers are correct.

Answer: B

Explanation:
The IRR is calculated by finding the discount rate that equates the present value of future cash
inflows to the project's cost. The IRR is the project's expected rate of return. If the IRR exceeds the
cost of the funds used to finance the project, a surplus accrues. Thus, accepting a project whose IRR
exceeds its cost of capital increases shareholder wealth.

Question: 2492

A stock has a beta of 1.1 and the risk-free rate is 5.5%. Its dividend growth rate is 4.1% and the
dividend payout ratio is 38%. If the market risk premium is 7.3%, the P/E ratio of the stock equals
________.

A. 7.19
B. 6.73
C. 3.86
D. 4.03

Answer: D

Page | 939
Explanation:
The required rate of return on the stock can be found using CAPM, which gives Rstock = k = Rf +
beta*(Rm - Rf) = 5.5% + 1.1*7.3% = 13.53%. Therefore, P0/E1 = dividend payout/(k - g) =
0.38/(0.1353 - 0.041) = 4.03.

Question: 2493

Projects L and S each have an initial cost of $10,000, followed by a series of positive cash inflows.
Project L has total, undiscounted cash inflows of $16,000, while S has total undiscounted inflows of
$15,000. Further, at a discount rate of 10 percent, the two projects have identical NPVs. Which
project's NPV will be more sensitive to changes in the discount rate?

A. Both projects are equally sensitive to changes in the discount rate since their NPVs are equal at all
costs of capital.
B. Neither project is sensitive to changes in the discount rate, since both have NPV profiles which are
horizontal.
C. Project S.
D. The solution cannot be determined unless the timing of the cash flows is known.
E. Project L.

Answer: E

Explanation:
The NPV profile plots a project's NPV against the discount rates. The higher the undiscounted
inflows, the steeper the NPV profile, and the steeper the NPV profiles, the more sensitive the project
is to discount rate changes.

Question: 2494

Alabama Pulp Company (APC) can control its environmental pollution using either "Project Old Tech"
or "Project New Tech." Both will do the job, but the actual costs involved with Project New Tech,
which uses unproved, new state-of-the-art technology, could be much higher than the expected cost
levels. The cash outflows associated with Project Old Tech, which uses standard proven technology,
are less risky--they are about as uncertain as the cash flows associated with an average project. APC's
cost of capital for average risk projects is normally set at 12 percent, and the company adds 3 percent
for high risk projects but subtracts 3 percent for low risk projects. The two projects in question meet
the criteriafor high and average risk, but the financial manager is concerned about applying the
normal rule to such cost-only projects. You must decide which project to recommend, and you should
recommend the one with the lower PV of costs. What is the PV of costs of the better project?
Cash Outflows
Years:01234
Project New Tech1,500315315315315
Project Old Tech600600600600600

A. 2,399
B. 2,521
C. 2,457
D. 2,422
E. 2,543

Answer: D

Explanation:

Page | 940
Recognize that (1) risky outflows must be discounted at lower rates, and (2) since Project New Tech is
risky, it must be discounted at a rate of 12% - 3% = 9%. Project Old Tech must be discounted at 12%.
Tabular solution:
PV(New Tech) = -$1,500 - $315(PVIFA9%,4) = -$1,500 - $315(3.2397) = -$2,520.51. PV(Old Tech) = -
$600 - $600(PVIFA12%,4) = -$600 - $600(3.0373) = -$2,422.38. PV(Old Tech) is a smaller outflow than
NPV(New Tech), thus, Project Old Tech is the better project.

Question: 2495

Intelligent Semiconductor is considering the development of a new data storage medium, which will
allow tremendous increases in the efficiency of its customer's high-end server lines. The
development of the new system will take place in the firm's existing facilities, and the storage costs
for the additional equipment are expected to be residual in nature. The following information applies
to this project:
Rent expense for the firm's existing facilities ($10,500)
Initial cash outlay ($50,000)
t1: $15,000
t2: $11,000
t3: $11,000
t4: $15,000
t5 ($10,000)
t6 ($10,000)
t7 $25,000
Discount rate: 9%
Assuming no taxes or related charges, that the initial cash outlay does not include any sunk costs,
and a $0.00 salvage value at t7, what is the MIRR of this project?

A. 7.262%
B. 6.231%
C. None of these answers
D. 12.461%
E. This problem has more than one MIRR
F. 14.606%That answer is correct!

Answer: A

Explanation:
In this example, you are asked to calculate the Modified Internal Rate of Return for a project. In
calculating the MIRR for this project, the rent cost of $10,500 is ignored because this expense
represents a sunk cost. Remember that sunk costs are irrelevant in capital budgeting decisions, and
should not beincorporated into the calculation. This is due to the fact that sunk costs are not
incremental in nature, and are not directly related to the acceptance of the project in question, i.e.
these costs have already been incurred or have been earmarked for payment. The following
illustration details the calculation of the Terminal Value (TV) for the MIRR calculation in this case:
{[$15,000 *1.6771] + [11,000 * 1.53862] + [11,000 * 1.41158] + [$15,000 * 1.29503] + [$25,000 * 1]}=
TV $102,034.15. The calculation of the present value of the cash outflows is found by discounting the
cash outflows of periods 5 and 6 and adding them to the initial cash outlay as follows: {-$50,000 + [-
$10,000/1.53862] + [-$10,000/1.6771]= PV of cash outflows ($62,462.00). Incorporating these figures
into your calculator's cash flow worksheet will yield the MIRR of 7.2623%. The calculation is found by
the following: PV=($62,462.00), FV= $102,034.15, PMT=0, N=7, CPT I/Y ---7.2623%. The modified IRR
has a significant advantage over the regular IRR, in the fact that MIRR assumes cash flows from all
projects are reinvested at some explicit rate, while the regular IRR assumes that all cash flows are
reinvested at the project's IRR. This allows for much greater flexibility.

Page | 941
Question: 2496

In examining the beta for its machine tools division, the management of Clay Industries has
regressed the division's ROA against that of the S&P 500. Which of the following best characterize
this method of calculating project beta?

A. None of these answers


B. Project Beta Method
C. Monte Carlo Regression
D. Pure Play Method
E. Regression of the Poisson Distribution
F. Accounting Beta Method

Answer: F

Explanation:
In this example, the management of Clay Industries has determined the Beta coefficient for its
machine tools division using the Accounting Beta Method. In using the Accounting Beta Method, the
monthly (or quarterly) ROE or ROA of a specific division/project is compared to that of a large group
of firms. This"large group of firms" is frequently characterized by a major market index, such as the
S&P 500 or the Wilshire 5000. The Accounting Beta Method is used more frequently than the Pure
Play Method, primarily because it is often difficult to find "pure-play" firms.

Question: 2497

Ludicrous Telecom, an international telecommunications company, has recently announced its plans
to issue additional common stock. The company has been publicly traded for over 25 years, and
currently has a capital structure consisting of 35% debt, 55% equity, and 10% preferred stock. This is
the first time since its initial public offering that the company has announced its intention to issue
common stock. According to the Signaling Theory, this announcement should be viewed as which of
the following? Choose the best answer.

A. Bullish, because the company will be provided with additional capital from the share offering.
B. Bullish, because it indicates superior investment prospects for the firm.
C. Bullish, because it indicates a shift toward a more conservative capital structure.
D. Bearish, because it indicates poor investment prospects for the firm.
E. Bearish, because it indicates an shift toward a more radical capital structure.
F. The signaling theory would not apply to this announcement.

Answer: D

Explanation:
According to the Signaling Theory, the management of companies send implicit signals to investors
by their capital budgeting decisions. Believers of this theory feel that corporate managers have
access to superior information, and are allowed to exploit this information asymmetry through their
capital budgeting decisions. According to the signaling theory, when investment prospects are good,
companies will prefer to raise capital first by using internally generated funds, i.e. retained earnings
and marketable securities investments. If this source of capital is unavailable, then companies will
prefer to issue debt rather than common or preferred equity. The reasoning behind this is the fact
that by raising debt, the company will not dilute the ROE figure, which is expected to be high due to
favorable investment prospects. In contrast, when investment prospects are poor, the Signaling
Theory states that companies will prefer to raise funds first by issuing common equity. The reasoning
behind this is the fact that byissuing additional equity when investment prospects are poor,
companies will be able to "spread the losses" amongst a greater pool of investors, thereby lessening

Page | 942
the overall negative effect. In this example, the management of Ludicrous Telecom have announced
their intention on issuing additional common equity, and the Signaling Theory would state that this is
sending a bearish signal as to the investment prospects of the firm.

Question: 2498

Which of the following conditions are necessary for the IRR and NPV calculations to produce similar
ranking decisions. Choose the best possible answer.

A. Projects must have equal lifespans, projects must be of equal scale


B. Projects must be independent, have equal lifespans, and must be equal in scale
C. Projects must be of equal size and have equal life, and must have normal cash flows
D. Projects must have equal lifespans, projects must have normal cash flows
E. Projects must be mutually exclusive, have equal lifespans, and must be of equal scale

Answer: C

Explanation:
When examining mutually-exclusive projects with equal lifespans and of equal size, the IRR and NPV
calculations will produce similar ranking results as long as the projects under examination have
"normal" cash flows. It is when the projects under examination have "non-normal" cash flows that
IRR calculations can experience difficulty. Non-normal cash flows are defined as cash flows in which
negative flows are juxtaposed within a series of positive cash inflows, creating a situation in which
the sign will change more than once. When examining these "non-normal" projects, the Internal
Rate of Return method will often produce multiple answers which leads to an incorrect accept/reject
decision.

Question: 2499

Which of the following statements about the cost of capital is incorrect?

A. The cost of retained earnings is equal to the return stockholders could earn on alternative
investments of equal risk.
B. WACC calculations should be based on after-tax costs of capital.
C. Flotation costs can increase the WACC.
D. If a company's tax rate increases, then, all else equal, its weighted average cost of capital will
increase.
E. A company's target capital structure affects its WACC (Weighted Average Cost of Capital).

Answer: D

Explanation:
A tax rate increase would lead to a decrease in the after-tax cost of debt and, consequently, the firm's
WACC would decrease.

Question: 2500

Which of the following statements best describes the theories of investors' preferences for
dividends?

A. The tax preference theory suggests that a company can increase its stock price by increasing its
dividend payout ratio.
B. One key advantage of a residual dividend policy is that it enables a company to follow a stable
dividend policy.

Page | 943
C. The clientele effect suggests that companies should follow a stable dividend policy.
D. Modigliani and Miller argue that investors prefer dividends to capital gains.
E. The bird-in-hand theory suggests that a company can reduce its cost of equity capital by reducing
its dividend payout ratio.

Answer: C

Explanation:
Different groups, or clientele, of stockholders prefer different dividend payout policies. Stockholders
in a low or tax-free tax bracket generally prefer cash income, so a payout would be their preference.
On the other hand, stockholders in a high tax bracket might prefer reinvestment of earnings because
they have little need for current investment income. To the extent that stockholders can switch firms,
a firm can change from one dividend payout policy to another to let stockholder who do not like the
new policy sell to other investors who do. Yet this would be costly because of brokerage costs, the
capital gains taxes that would have to be paid by the selling stockholders, and the chance that there
will be a net loss of investors who like the firm's new dividend policy. Management should therefore,
probably not change its policy. Several studies show that there is a clientele effect, which is the
tendency of a firm to attract a set of investors who like its dividend policy. The existence of the
clientele effect does not necessarily imply that one dividend policy is better than another.

Question: 2501

Two mutually exclusive projects each have a cost of $10,000. The total, undiscounted cash flows from
Project L are $15,000, while the undiscounted cash flows from Project S total $13,000. Their NPV
profiles cross at a discount rate of 10 percent. Which of the following statements best describes this
situation?

A. To determine if a ranking conflict will occur between the two projects the cost of capital is needed
as well as an additional piece of information.
B. Project L should be selected at any cost of capital, because it has a higher IRR.
C. The NPV and IRR methods will select the same project if the cost of capital is greater than 10
percent; for example, 18 percent.
D. The NPV and IRR methods will select the same project if the cost of capital is less than 10 percent;
for example, 8 percent.
E. Project S should be selected at any cost of capital, because it has a higher IRR.

Answer: C

Explanation:
The crossover rate is the discount rate at which the NPV profiles of the two projects cross and, thus,
at which the projects' NPVs are equal. As long as the discount rate is greater than the crossover rate,
both the NPV and IRR methods will lead to the same conclusion.

Question: 2502

Bricks, Inc. has just installed a factory for producing titanium-strengthened bricks. The fixed costs
equal $1.25 million. The bricks can be sold at $2.25 per unit and cost $1.9 per unit in variable
expenses. How many bricks must be sold by Bricks, Inc. for it to break even?

A. 1.34 million
B. 3.57 million
C. 4.19 million
D. 2.19 million

Page | 944
Answer: B

Explanation:
The break-even quantity, Q, is given by Q = FC/(P - V), where FC = total fixed costs, P = average sale
price per unit and V = average variable cost per unit. In this case, Q = 1.25/(2.25 - 1.9) million = 3.57
million bricks.

Question: 2503

The management of Clay Industries have adhered to the following capital structure: 40% debt, 45%
common equity, and 15% perpetual preferred equity. The following information applies to the firm:
Yield to maturity of outstanding long-term debt = 9.5%
Expected return on the market = 14.5%
Annual risk-free rate of return = 6.25%
Historical Beta coefficient of Clay Industries Common Stock = 1.24 Annual preferred dividend = $1.75
Preferred stock net offering price = $28.50
Combined state/federal corporate tax rate = 35%
Given this information, and using the Capital Asset Pricing Model to calculate the component cost of
common equity, what is the Weighted Average Cost of Capital for Clay Industries?

A. The WACC for Clay Industries cannot be determined from the information provided.
B. 14.45%
C. 10.00%
D. 12.14%
E. 8.54%
F. 10.81%

Answer: F

Explanation:
The calculation of the Weighted Average Cost of Capital is as follows: {fraction of debt * [yield to
maturity on outstanding long-term debt][1-combined state/federal income tax rate]} + {fraction of
preferred stock * [annual dividend/net offering price]} + {fraction of common stock * cost of equity}.
The cost of common equity can be calculated using three methods, the Capital Asset Pricing Model
(CAPM), the Dividend-Yield-plus-Growth-Rate (or Discounted Cash Flow) approach, and the Bond-
Yield-plus-Risk-Premium approach. In this example, you are required to calculate the cost of equity
using the Capital Asset Pricing Model (CAPM), which is illustrated as follows: {risk-free rate +
beta(expected return on the market - risk-free rate)}. Using this model, the cost of common equity
can be found as 16.48%. The cost of perpetual preferred stock can be found by dividing the annual
dividend by the net offering price, which is illustrated in this case as follows: {$1.75/28.50) = 6.14%.
The after-tax cost of debt can be found by taking the yield to maturity on the firm's outstanding long-
term debt (9.5%), and multiplying this figure by (1 - annual tax rate 35%) = 6.175%. Incorporating all
of these figures into the WACC equation gives a Weighted Average Cost of Capital of 10.807%

Question: 2504

Which of the following statements is correct?

A. If you are choosing between two projects which have the same cost, and if their NPV profiles
cross, then the project with the higher IRR probably has more of its cash flows coming in the later
years.
B. The NPV and IRR methods both assume that cash flows are reinvested at the cost of capital.
However, the MIRR method assumes reinvestment at the MIRR itself.
C. There can never be a conflict between NPV and IRR decisions if the decision is related to a normal,

Page | 945
independent project, i.e., NPV will never indicate acceptance if IRR indicates rejection.
D. A change in the cost of capital would normally change both a project's NPV and its IRR.
E. To find the MIRR, we first compound CFs at the regular IRR to find the TV, and then we discount the
TV at the cost of capital to find the PV.

Answer: C

Explanation:
To see this, sketch out a NPV profile for a normal, independent project, which means that only one
NPV profile will appear on the graph. If WACC < IRR, then IRR says accept. But in that case, NPV > 0,
so NPV will also say accept.

Question: 2505

Ace Consulting, a corporate finance consulting firm, is examining the operations of Intelligent
Semiconductor and has determined the following information:
Sales $1,000,000
Total variable costs $270,000
Total fixed costs $400,000
Interest expense $75,000
EBIT $325,000
Given this information, what is the degree of total leverage for Intelligent Semiconductor?

A. 1.342
B. 2.863
C. 1.4925
D. 3.077
E. 2.292

Answer: B

Explanation:
The Degree of Total Leverage (DTL) demonstrates how a given change in sales will impact a firm's
EPS. The equation used for calculating the DTL is as follows: {[Sales - variable costs] / [sales - variable
costs - fixed costs - interest expense]}. Incorporating the given values for these components into the
DTL equation yields the following: {[Sales $1,000,000 - variable costs $270,000] / [sales $1,000,000 -
variable costs $270,000 - fixed costs $400,000 - interest expense $75,000]}=2.863. The EBIT figure is
not explicitly incorporated into the DTL equation.

Question: 2506

Which of the following are necessary conditions for the NPV and IRR methods to produce similar
rankings? Choose the best possible answer.

A. Projects must be mutually exclusive and of equal scale


B. Projects must be independent and have normal cash flows
C. Projects must be mutually exclusive and have normal cash flows
D. Projects must have normal cash flows, and must be equal in scale and lifespan
E. Projects must be of equal scale and have equal lifespans

Answer: D

Explanation:
When examining mutually exclusive projects with equal lifespans and of equal size, the IRR and NPV

Page | 946
calculations will produce similar ranking results as long as the projects under examination have
"normal" cash flows. It is when the projects under examination have "non-normal" cash flows that
the IRR method can experience some difficulty. Non-normal cash flows are defined as cash flows in
which negative cash inflows are juxtaposed within a series of positive cash inflows, creating a
situation in which the sign will change more than once. When examining these "non-normal"
projects, the Internal Rate of Return calculation will often produce multiple answers which leads to
an incorrect accept/reject decision. In any examination in which the IRR and NPV produce conflicting
rankings, the NPV calculation should be used.

Question: 2507

Byron Corporation's present capital structure, which is also its target capital structure, is 40 percent
debt and 60 percent common equity. Next year's net income is projected to be $21,000, and Byron's
payout ratio is 30 percent. The company's earnings and dividends are growing at a constant rate of 5
percent; the last dividend was $2.00; and the current equilibrium stock price is $21.88. Byron can
raise all the debt financing it needs at 14 percent. If Byron issues new common stock, a 20 percent
flotation cost will be incurred. The firm's marginal tax rate is 40 percent. What is the maximum
amount of new capital that can be raised at the lowest component cost of equity?

A. $14,700
B. $21,000
C. $17,400
D. $24,500
E. $12,600

Answer: D

Explanation:
BP(RE) = $21,000 x .70/ .60 = $24,500.

Question: 2508

Which of the following statements is most correct?

A. If a firm repurchases its stock in the open market, the shareholders that tender are subject to
capital gains taxes.
B. All of the statements are correct.
C. None of these statements are correct.
D. If you own 100 shares in a company's stock, and the company does a 2 for 1 stock split, you will
own 200 shares in the company following the split.
E. Some dividend reinvestment plans increase the amount of equity capital available to the firm.

Answer: B

Explanation:
Tendering (selling) shares in the open market is a taxable event. With the stock split, you will own
twice the shares but the stock price will be halved. Dividend reinvestment plans permit stockholders
to automatically reinvest their dividends in the stock of the paying firm. The new stock type of DRIPs
invests the dividends in newly issued stock, hence these plans raise new capital for the firm.

Question: 2509

Intelligent Semiconductor is considering issuing additional common stock. The firm has an after-tax
cost of debt of 8.55%, and the company's combined federal/state income tax is 35%. The risk-free

Page | 947
rate of return is 5.6%, and the annual return on the broadest market index is expected to be 13.5%.
Shares of Intelligent Semiconductor have a historical beta of 1.6. What is the cost of equity for this
proposed common stock issue using the Capital Asset Pricing Model?

A. 18.24%
B. 4.09%
C. 12.64%
D. 7.04%
E. 5.56%That answer is correct!

Answer: A

Explanation:
The cost of issuing common stock can be calculated using several methods, including the Bond-Yield-
Plus-Risk-Premium approach, Discounted Cash Flow method, or by using the Capital Asset Pricing
Model. The latter is illustrated in this example. To calculate the cost of common equity using the
CAPM, use the following formula: {cost of common equity = [risk free rate of return + beta(expected
return on the market - risk free rate of return)}. Incorporating the appropriate figures into this
example will yield a cost of common equity at 18.24%.

Question: 2510

Given the following information, what is the required cash outflow associated with the acquisition of
a new machine; that is, in a project analysis, what is the cash outflow at t = 0?
Purchase price of new machine$8,000
Installation charge2,000
Market value of old machine2,000
Book value of old machine1,000
Inventory decrease if new machine
is installed1,000
Accounts payable increase if new
machine is installed500
Tax rate35%
Cost of capital15%

A. -$6,460
B. -$8,980
C. -$12,020
D. -$5,200
E. -$6,850

Answer: E

Explanation:
Cost plus installation($10,000)
Sale of old machine+2,000
Tax effect of sale ($1,000 x 0.34)(350)
Decrease in working capital1,500
Total investment at t = 0($6,850)

Question: 2511

Los Angeles Lumber Company (LALC) is considering a project with a cost of $1,000 at time = 0 and
inflows of $300 at the end of Years 1 - 5. LALC's cost of capital is 10 percent. What is the project's

Page | 948
modified IRR (MIRR)?

A. 15.2%
B. 12.9%
C. 20.7%
D. 10.0%
E. 18.3%

Answer: B

Explanation:
Tabular/Numerical solution:
TV = $300(FVIFA(10%,5)) = $300(6.1051) = $1,831.53.
$1,000 = TV/(1 + MIRR)^5
$1,000 = $1,831.53/(1 + MIRR)^5
(1 + MIRR)^5 = 1.83153
MIRR = 12.866%.

Question: 2512

The degree of financial leverage is defined as:

A. the change in EPS for a unit change in EBIT.


B. the percentage change in EBIT for a 1% change in EPS.
C. none of these answers.
D. the percentage change in EBIT for a 1% change in the quantity sold.

Answer: C

Explanation:
The degree of financial leverage is defined as the percentage change in EPS for a 1% change in EBIT.

Question: 2513

Grant Grocers is considering the following investment projects:


ProjectSize of ProjectIRR of Project
V1.0 million12.0%
W1.2 million11.5%
X1.2 million11.0%
Y1.2 million10.5%
Z1.0 million10.0%
The company has a target capital structure, which is 50 percent debt and 50 percent equity. The
after- tax cost of debt is 8 percent. The cost of retained earnings is estimated to be 13.5 percent. The
cost of equity is estimated to be 14.5 percent if the company issues new common stock. The
company's net income is $2.5 million. If the company follows a residual dividend policy, what will be
its payout ratio?

A. 66%
B. 12%
C. 32%
D. 54%
E. 100%

Answer: C

Page | 949
Explanation:
The company's WACC (provided no new equity is issued) is 8%(0.5) + 13.5%(0.5) = 10.75%.
Comparing the WACC with the project IRRs reveals that the company will undertake projects V, W,
and X. Total financing costs for these projects is $3,400,000. Of this amount, 0.5($3,400,000) =
$1,700,000 will be financed from retained earnings. Thus, $2,500,000 - $1,700,000 = $800,000 will be
available for dividends. The payout ratio is then $800,000/$2,500,000 = 32%.

Question: 2514

Rollins Corporation is constructing its MCC (Marginal Cost of Capital) schedule. Its target capital
structure is 20 percent debt, 20 percent preferred stock, and 60 percent common equity. Its bonds
have a 12 percent coupon, paid semiannually, a current maturity of 20 years, and sell for $1,000. The
firm could sell, at par, $100 preferred stock, which pays a 12 percent annual dividend, but flotation
costs of 5 percent would be incurred. Rollins' beta is 1.2, the risk-free rate is 10 percent, and the
market risk premium is 5 percent. Rollins is a constant growth firm, which just paid a dividend of
$2.00, sells for $27.00 per share, and has a growth rate of 8 percent. The firm's policy is to use a risk
premium of 4 percentage points when using the bond-yield-plus-risk- premium method to find k(s)
(component cost of retained earnings). The firm's net income is expected to be $1 million, and its
dividend payout ratio is 40 percent. Flotation costs on new common stock total 10 percent, and the
firm's marginal tax rate is 40 percent. What is Rollins' component cost of debt?

A. 8.6%
B. 7.2%
C. 10.0%
D. 8.0%
E. 9.1%

Answer: B

Explanation:
Since the bond sells at par of $1,000, its YTM and coupon rate (12 percent) are equal. Thus, the
before-tax cost of debt to Rollins is 12.0 percent. The after-tax cost of debt equals = 12.0%(1 - 0.40) =
7.2%.

Question: 2515

A firm needs to raise $123 million for a proposed capital expansion project. It's earnings breakpoint is
$178 million and it is committed to maintaining a debt-to-equity ratio of 1.2. Its after-tax cost of debt
is6.2% and the required rate of return on its equity is 13.2%. The firm's marginal cost of capital for
the project equals ________.

A. 7.12%
B. 6.89%
C. 9.38%
D. 12.19%

Answer: C

Explanation:
Since the proposed capital requirement of $123 million is less than the earnings breakpoint, the
firm's marginal cost of capital for the project equals its WACC. With D/E = 1.2, E/(D+E) = 1/(1+1.2) =
0.455. The WACC then equals 0.455*13.2% + 0.545*6.2% = 9.38%.

Page | 950
Question: 2516

Clay Industries, a diversified industrial firm, is considering investing into a new manufacturing facility
which would allow the Company to expand its operations into a promising new market for industrial
motors, specifically the High Temperature Superconducting, or HTS motors. This project is one of
many currently under consideration for Clay Industries, and the amount of R&D expense allocated
toward researching this new manufacturing facility is residual in nature. The following information
applies to this new project.
R&D expense for the quarter $15,000
Initial cash outlay $45,000
t1: ($40,000)
t2: ($10,000)
t3: $40,000
t4: $40,000
t5: $16,000
t6: $25,000
Assuming no taxes and a $0.00 salvage value at t6, what is the MIRR of this project?

A. This project will have multiple MIRR at any discount rate


B. 7.038%
C. The MIRR cannot be calculated due to the fact that no discount rate has been provided
D. The MIRR cannot be calculated due to the fact that the project has uneven cash flows
E. 2.639%

Answer: C

Explanation:
In order to calculate the Modified Internal Rate of Return, a explicit discount rate must be given. In
this example, the MIRR cannot be calculated due to the fact that no discount rate has been provided.
Remember that while the Internal Rate of Return is calculated without the use of an explicit discount
rate, the Modified Internal Rate of Return requires some figure for the cost of capital.

Question: 2517

Which of the following methods involves calculating an average beta for firms in a similar business
and then applying that beta to determine the beta of its own project?

A. Risk premium method.


B. CAPM method.
C. Accounting beta method.
D. Pure play method.
E. All of these answers are correct.

Answer: D

Explanation:
The pure play method is used for estimating the beta of a project in which a firm identifies several
companies whose only business is the product in question, then calculates the beta for each firm,
and finally, averages the betas to find an approximation to its own project's beta.

Question: 2518

In his determination of a project's NPV and IRR, a financial analyst with Smith, Kleen, & Beetchnutty
indexes the project's anticipated cash flows for the expected effects of inflation. However, the

Page | 951
discount rate applied to these cash flows does not factor an adjustment for inflation. Assuming a
positive inflation figure for the every period in the project's lifespan, which of the following correctly
describes the effects of the omission on the NPV and IRR calculations?

A. NPV and IRR will be biased upward


B. NPV and IRR will be biased downward
C. NPV will be biased downward, IRR will be biased upward
D. NPV will be biased upward, IRR will be unaffected
E. NPV will remain unaffected, IRR will be biased downward
F. NPV will be biased downward, IRR will be unaffected

Answer: D

Explanation:
By failing to include an the effects of anticipated inflation in the discount rate applied to the project's
cash inflows, this analyst is creating a situation in which the NPV calculations will be biased upward.
This is due to the fact that the project's inflows have been adjusted for the anticipated effects of
POSITIVE inflation, i.e. these cash flows have been indexed upward, while at the same time the rate
at which these cash flows are being discounted has not increased. In effect, the cash inflows of the
project are being overstated, and this will lead to an upward bias in the NPV calculation. Remember
that the Internal Rate of Return calculation does not specify an explicit discount rate, rather
calculates the discount rate that equates the cash inflows of a project with its cash outflows. The fact
that this analysthas not incorporated the effects of inflation into the discount rate has no bearing on
IRR, because the IRR equation does not call for a discount rate.

Question: 2519

Brock Brothers wants to maintain its capital structure, which is 30 percent debt, and 70 percent
equity. The company forecasts that its net income this year will be $1,000,000. The company follows
a residual dividend policy, and anticipates a dividend payout ratio of 40 percent. What is the size of
the company's capital budget?

A. $857,143
B. $1,428,571
C. $1,000,000
D. $600,000
E. $2,000,000That answer is correct!

Answer: A

Explanation:
Since the company expects to pay out 40% of net income or $400,000, it must expect to have
$600,000 of retained earnings available for capital investment. Given that the firm will finance new
investment with 70% equity and 30% debt, $600,000 must represent 70% of the firm's capital
budget, that is, $600,000 = (0.7)CB or CB = $857,143.

Question: 2520

Which of the following statements is most correct?

A. Investors can interpret a stock repurchase by a firm as a signal that the firm's managers believe
the stock is underpriced.
B. None of these statements are correct.
C. After a 3-for-1 stock split, a company's price per share will fall and it's number of shares

Page | 952
outstanding will rise.
D. Stock repurchases can be used by firms to defend against hostile takeovers since they increase the
proportion of debt in a firm's capital structure.
E. All of these statements are correct.

Answer: E

Explanation:
These are all correct.

Question: 2521

Stargell Industries follows a strict residual dividend policy. The company has a capital budget of
$3,000,000. It has a target capital structure, which consists of 30 percent debt and 70 percent equity.
The company forecasts that its net income will be $3,500,000. What will be the company's expected
dividend payout ratio this year?

A. 40%
B. 45%
C. 30%
D. 25%
E. 35%That answer is correct!

Answer: A

Explanation:
Step 1 Find equity required to maintain capital budget:
Capital budget$3,000,000
% of budget financed with equityx 0.70
$2,100,000
Step 2 Calculate dividend:
Earnings$3,500,000
Less equity retained(2,100,000)
Dividend$1,400,000
Step 3 Find payout ratio:
Dividend/Earnings = $1,400,000/$3,500,000 = 0.4000 = 40%.

Question: 2522

Projects whose cash flows are not affected by the acceptance or rejection of other projects are
known as ________.

A. independent projects
B. project net worth optimization
C. optimal capital budgeting
D. equity enhancement
E. mutually exclusive projectsThat answer is correct!

Answer: A

Explanation:
Independent Projects are defined as projects whose cash flows are not affected by the acceptance or
rejection of other projects.

Page | 953
Question: 2523

Which of the following statements is correct?

A. The rent referred to in the other statement is a sunk cost, and as such it should be ignored.
B. The preceding statement would be true if "upward" were replaced with "downward."
C. The existence of "externalities" reduces the NPV to a level below the value that would exist in the
absence of externalities.
D. If one of the assets that would be used by a potential project is already owned by the firm, and if
that asset could be leased to another firm if the project is not undertaken, then the net rent that
could be obtained should be charged as a cost to the project under consideration.
E. In a capital budgeting analysis where part of the funds used to finance the project are raised as
debt, failure to include interest expense as a cost in the cash flow statement when determining the
project's cash flows will lead to an upward bias in the NPV.

Answer: D

Explanation:
The foregone rent is an "opportunity cost" which should be charged to the project under
consideration. The cash flows should not take account of interest, because financial costs are dealt
with by discounting at the WACC. If interest were deducted to find cash flows, then this cost would
be "double counted," and the NPV would be downward biased. Ignoring interest when determining
cash flows produces no bias in the NPV whatever. Note also that externalities can be either positive
or negative--they tend to be negative if the new project is a substitute for existing products, but
positive if the new project is complementary to the firm's other products.

Question: 2524

Which of the following equations correctly illustrates the calculation of the cost of equity using the
Dividend-Yield-plus-Growth-Rate approach?

A. Annual dividend/current stock price * (1-tax rate)


B. (Next annual dividend/current stock price) + expected growth rate
C. (Retention rate)*(ROE)
D. Risk-free rate of return + beta(expected return on the market - risk-free rate of return)
E. Payout ratio * (ROE/[expected return-required rate of return])
F. (Last annual dividend/[expected return - required return]) * expected growth rate

Answer: C

Explanation:
The Dividend-Yield-plus-Growth-Rate approach calls for the following components: next annual
dividend, current stock price, and expected growth rate. This approach, also known as the
Discounted Cash Flow (DCF) method, is a flexible and very adept tool in the hands of the financial
analyst, and is it is imperative that the CFA candidate fully understand both the applications and the
methodology of this approach. The first choice illustrates the Capital Asset Pricing Model, while the
second represents an approach for calculating sustainable growth rate. The remaining answers are
somewhat fictitious.

Question: 2525

Which of the following statement completions is most correct? If investors prefer dividends to capital
gains, then

Page | 954
A. dividend policy as determined by the residual dividend policy is the only dividend policy which will
maximize the price per share of common stock.
B. k(s) will increase as dividends are reduced.
C. k(s) will decrease as dividends are reduced.
D. the equilibrium return, k(s), will not be affected by a change in dividend policy because tax effects
will offset these preferences.
E. k(s) will decrease as the retention rate increases.

Answer: B

Explanation:
The main conclusion of MM's irrelevance theory is that dividend policy does not affect the required
rate of return on equity. Gordon-Lintner disagreed stating that k(s) decreases as the dividend payout
is increased because investors are less certain of receiving the capital gains which should result from
retaining earnings than they are of receiving dividends. They said that investors value expected
dividends more highly than expected capital gains because the dividend yield is less risky than the
growth component in the total expected return equation, k(s) = D1/Po + g. MM disagreed and
theorized that k(s) is independent of dividend policy, implying that investors are indifferent between
dividends and capital gains.

Question: 2526

Gibson Inc. is considering the following five independent projects:


Project RequiredAmount of CapitalIRR
A$200,00020%
B600,00015
C400,00012
D400,00011
E400,00010
The company has a target capital structure, which is 40 percent debt and 60 percent equity. The
company can issue bonds with a yield to maturity of 11 percent. The company has $600,000 in
retained earnings, and the current stock price is $42 per share. The flotation costs associated with
issuing new equity are $2 per share. Gibson's earnings are expected to continue to grow at 6 percent
per year. Next year's dividend is forecasted to be $4.00. The firm faces a 40 percent tax rate. What is
the size of Gibson's capital budget?

A. $200,000
B. $1,200,000
C. $800,000
D. $1,600,000
E. $2,000,000

Answer: C

Explanation:
The size of Gibson's capital budget will be determined by the number of projects it can profitably
undertake, i.e., those projects for which IRR > applicable WACC. First, find the costs of each type of
financing: cost of retained earnings = k(s) = $4/$42 + 0.06 = 15.52% and cost of debt = k(d) = 11%. To
calculate the cost of new equity, we solve for k(e) = $4/($42-$2) + 0.06 = 0.16 = 16%. Given the firm's
target capital structure and its retained earnings balance of $600,000, the firm can raise $1,000,000
with debt and retained earnings before it must use outside equity. Therefore, the WACC for 0 -
$1,000,000 of financing = 0.4(0.11)(1 - 0.4) + 0.6(0.1552) = 11.95%. Above $1,000,000, the firm must
issue some new equity, so the WACC = 0.4(0.11)(1 - 0.4) + 0.6(0.16) = 12.24%. Obviously, Projects A
and B will be undertaken. You must then determine whether Project C will be profitable. Since in

Page | 955
taking A and B we will need financing of $800,000, the $400,000 needed for Project C would involve
financing $200,000 with debt and retained earnings and $200,000 with debt and new equity. Thus,
the WACC for Project C is ($200,000/$400,000) x 0.1195 + ($200,000/$400,000) x 0.1224 = 12.095%
which is greater than Project C's IRR. Clearly, only Projects A and B should be accepted, and the firm's
capital budget is $800,000.

Question: 2527

Ace Consulting, a multinational corporate finance consulting firm, is examining the sales potential for
a new line of industrial motors developed by Clay Industries, a large industrial firm. In their analysis,
Ace Consulting meets with the management of Clay Industries, and asks these individuals to specify
the worst "reasonable" set of circumstances, along with the best "reasonable set." These figures are
measured against the predetermined "base case" situation. Which of the following choices best
describes this technique for measuring stand-alone risk?

A. Case study analysis


B. Sensitivity analysis
C. Monte Carlo simulation
D. Relational analysis
E. Scenario analysis
F. Regression analysis

Answer: E

Explanation:
Scenario analysis is a risk analysis technique that considers both the sensitivity of NPV to changes in
key variables and the likely range of key variable values. In a scenario analysis, a financial analyst
asks operating or other managers to identify the best and worst "reasonable" situations, and these
situations are examined against the predetermined "base case."

Question: 2528

Doering Computers is considering two mutually exclusive projects. Their cash flows are shown
below:
tProj. A Cash FlowsProj. B Cash Flows
0-$500-$700
1200250
2400475
3100125
4----225
The company's cost of capital (WACC) is 10 percent. Each of the projects can be repeated. What is the
equivalent annual annuity (EAA) of the project, which adds the most to shareholder value?

A. $61.64
B. $52.82
C. $63.45
D. $35.20
E. $25.41

Answer: B

Explanation:
Find NPV of each project.
NPV(A) = $87.5282.

Page | 956
NPV(B) = $167.4271.
Find EAA:
For Project A:
N = 3; I = 10; PV = -87.5282; FV = 0; solve for PMT = EAA = $35.1964.
For Project B:
N = 4; I = 10; PV = -167.4271; FV = 0; solve for PMT = EAA = $52.8184.

Question: 2529

TCH Corporation is considering two alternative capital structures with the following characteristics.
AB
Debt/Assets ratio0.30.7
kd10%14%
The firm will have total assets of $500,000, a tax rate of 40 percent, and book value per share of $10,
regardless of capital structure. EBIT is expected to be $200,000 for the coming year. What is the
difference in earnings per share (EPS) between the two alternatives?

A. $4.78
B. $2.87
C. $7.62
D. $1.19
E. $3.03

Answer: B

Explanation:
Capital structure A: The firm will have debt of $500,000(0.3) = $150,000 and equity of $350,000.
We're told the shares have a book value of $10 so the number of shares outstanding is $350,000/$10
= 35,000. Interest expense will be $150,000(10%) = $15,000. We can compute EBT as EBIT - I or
$200,000 - $15,000 = $185,000. Also, we can compute NI as EBT(1 - T) or $185,000(1 - 0.4) =
$111,000. Finally, EPS = $111,000/35,000 = $3.17. Capital structure B: The firm will have debt of
$500,000(0.7) = $350,000 and equity of $150,000. The number of shares outstanding is
$150,000/$10 = 15,000. Interest expense will be $350,000(14%) = $49,000. We can compute EBT as
$200,000 - $49,000 = $151,000. Also, we can compute NI as $151,000 (1 - 0.4) = $90,600. Finally, EPS
= $90,600/15,000 = $6.04. The difference in EPS between capital structure A and capital structure B is
$6.04 - $3.17 = $2.87.

Question: 2530

Longstreet Corporation has a target capital structure of 30 percent debt, 50 percent common equity,
and 20 percent preferred stock. The tax rate is 30 percent. The company has an optimal capital
budget of $1,500,000. Longstreet will retain $500,000 of after-tax earnings this year. The last
dividend was $5, the current stock price is $75, and the growth rate of the company is 10 percent. If
the company raises capital through a new equity issuance, then the flotation costs are 10 percent for
the first $500,000. If the company issues more than $500,000 in new equity the flotation cost
increases to 15 percent. The cost of preferred stock is 9 percent and the cost of debt is 7 percent.
(Assume debt and preferred stock have no flotation costs.) What is the weighted average cost of
capital at the firm's optimal capital budget?

A. 12.18%
B. 18.15%
C. 12.34%
D. 11.94%
E. 12.58%

Page | 957
Answer: C

Explanation:
First, calculate the after-tax component cost of debt as 7%(1 - 0.3) = 4.9%. Next, calculate the
retained earnings breakpoint as $500,000/0.5 = $1,000,000. Thus, to finance its optimal capital
budget, Longstreet must issue some new equity. Note, Longstreet needs $500,000 in financing
beyond that which can be supported by retained earnings alone. However, of this additional
$500,000, 50% will be new equity and the remaining 50% will represent preferred stock and debt.
Thus, Longstreet will issue $250,000 in new equity and flotation costs of 10% will be incurred. The
cost of new equity is then[$5(1.10%)/$75(1 - 0.1)] + 10% = 8.15% + 10% = 18.15%. Finally, the WACC
= 4.9%(0.3) + 9%(0.2) + 18.15%(0.5) = 12.34%.

Question: 2531

Seasons, Inc. has just decided to issue 1 million shares of new equity. The firm has had a steady
dividend growth of 3% and is expected to continue along this path, having just paid a $3.23 per share
dividend. The flotation costs for the new equity amount to 2.2% of the total capital raised and the
firm receives $31.4 million before flotation costs, calculate the cost of external equity.

A. 13.52%
B. 14.19%
C. 13.23%
D. 13.83%

Answer: D

Explanation:
IF F is the percentage flotation cost and P is the amount of new equity raised per new share, then Ke
= D1/[P(1-F)] + g, where Ke is the cost of external equity. Here, g = 3%, D1 = 3.23*(1+3%) = $3.32, P =
$31.4 and F = 2.2%. Therefore, Ke = 3.32/(31.4*(1-0.022)) + 3% = 13.83%.

Question: 2532

A 5-year project requires an initial outlay of 650. It also needs capital spending of 700 at the end of
year 1 and 900 at the end of year 2. It has no revenues for the first 2 years but receives 1,200 in year
3, 1,600 in year 4 and 2,300 in year 5. If the project's cost of capital is 7.5%, the project's MIRR equals
________.

A. 21%
B. 17%
C. 14%
D. 7.5%That answer is correct!

Answer: A

Explanation:
The MIRR is defined as that rate which discounts the terminal value of the cash inflows to equate to
the present value of a project's costs (using the project's cost of capital). This can be better
understood using actual numbers. The present value of the costs = 650 + 700/1.075 + 900/1.075^2 =
2,080. The terminal value (future value at the end of year 5) of the project equals 1,200*1.075^2 +
1,600*1.075 + 2,300 = 5406.75. Note that both these are calculated using the project's cost of capital.
Then, MIRR satisfies 2,080 = 5406.75/(1+MIrr)^5. Solving gives MIRR = 21%.

Page | 958
Question: 2533

Which of the following statements is most correct?

A. When equipment is sold, companies receive a tax credit as long as the salvage value is less than
the initial cost of the equipment.
B. None of the answers are correct.
C. In estimating net cash flows for the purpose of capital budgeting, interest and dividend payments
should not be included since the effects of these items are already included in the weighted average
cost of capital.
D. Capital budgeting analysis for expansion and replacement projects is essentially the same because
the types of cash flows involved are the same.
E. All of the answers are correct.

Answer: C

Explanation:
Interest payments should not be included in the estimated cash flows because the effects of debt
financing are reflected in the cost of capital used to discount the cash flows. If interest was
subtracted from the cash flows, and then the remaining cash flows were discounted, the cost of debt
would be double-counted.

Question: 2534

Which of the following affects a firm's business risk?

A. The degree of operating leverage.


B. The risk of adjusting sales prices.
C. The level of uncertainty about future sales.
D. All of these answers are correct.

Answer: D

Explanation:
Business risk depends on: (1) unit sales variability, (2) sales price variability, (3) input price variability,
(4) ability to adjust output prices for changes in input prices and, (5) the extent to which costs are
fixed (operating leverage).

Question: 2535

Which of the following statements is correct?

A. "Business risk" is differentiated from "financial risk" by the fact that financial risk reflects only the
use of debt, while business risk reflects both the use of debt and such factors as sales variability, cost
variability, and operating leverage.
B. If corporate tax rates were decreased while other things were held constant, and if the
ModiglianiMiller tax-adjusted tradeoff theory of capital structure were correct, this would tend to
cause corporations to increase their use of debt.
C. The optimal capital structure is the one which simultaneously (1) maximizes the price of the firm's
stock, (2) minimizes its WACC, and (3) maximizes its EPS.
D. None of these statements are true.
E. If corporate tax rates were decreased while other things were held constant, and if the
ModiglianiMiller tax-adjusted tradeoff theory of capital structure were correct, this would tend to
cause corporations to decrease their use of debt.

Page | 959
Answer: E

Explanation:
If corporate tax rates were decreased while other things were held constant, and if the MM tax-
adjusted tradeoff theory of capital structure were correct, corporations would decrease their use of
debt because the tax shelter benefit would not be as great as when tax rates are high. Business risk is
the riskiness of the firm's operations if it uses no debt. The optimal capital structure does not
maximize EPS, and the degree of total leverage shows how a given change in sales will affect
earnings per share.

Question: 2536

Which of the following are practical difficulties associated with capital structure and degree of
leverage analyses?

A. All of these statements are correct.


B. None of the statements represent a serious impediment to the practical application of leverage
analysis in capital structure determination.
C. Managers' attitudes toward risk differ and some managers may set a target capital structure other
than the one that would maximize stock price.
D. Managers often have a responsibility to provide continuous service; they must preserve the long-
run viability of the enterprise. Thus, the goal of employing leverage to maximize short-run stock
price and minimize capital cost may conflict with long-run viability.
E. It is nearly impossible to determine exactly how P/E ratios or equity capitalization rates are
affected by different degrees of financial leverage.That answer is correct!

Answer: A

Explanation:
Two projects being considered are mutually exclusive and have the following projected cash flows:
Year Project AProject B
0-$50,000-$50,000
115,6250
215,6250
315,6250
415,6250
515,62599,500

Question: 2537

If the required rate of return on these projects is 10 percent, which would be chosen and why?

A. Project B because it has the higher IRR.


B. Neither, because both have IRRs less than the cost of capital.
C. Project B because it has the higher NPV.
D. Project A because it has the higher IRR.
E. Project A because it has the higher NPV.

Answer: C

Explanation:
NPV(A) = $15,625(PVIFA(10%,5)) - $50,000 = $15,625(3.7908) - $50,000 = $59,231.25 - $50,000 =
$9,231.25.

Page | 960
NPVB = $99,500(PVIF(10%,5)) - $50,000 = $99,500(0.6209) - $50,000 = $61,779.55 - $50,000 =
$11,779.55.
NPV(B) > NPV(A); $11,779.55 > $9,231.25; Choose Project B.

Question: 2538

A financial analyst with Smith, Kleen, and Beetchnutty is examining shares of Clever Industries, for
possible investment. Clever Industries is involved in textile manufacturing, and the firm has been
growing at a steady rate for much of the last nine decades. The analyst is trying to determine the
appropriate current price range for Clever shares, and has ascertained the following information:
Expected annual dividend = $0.35
Expected sustainable annual growth rate = 15%
Investors required rate of return = 18.6%
Given this information, what is the appropriate current price for Clever Industries common stock?

A. $10.28
B. The current price of Clever Industries cannot be determined from the given information.
C. $1.88
D. $2.33
E. $9.72

Answer: E

Explanation:
To calculate the appropriate stock price for Clever using the given information, the appropriate
equation is as follows: {expected annual dividend/[investor's required rate of return - expected
growth rate]}. Incorporating the given information into this equation yields a stock price of $9.722.
Remember that this model to stock valuations is most appropriate for firms who are in the constant
growth stage. Another important note to remember is that this model will yield realistic results only
in those instances in which the investor's required rate of return exceeds the expected growth rate.

Question: 2539

A set of projects where only one can be accepted is known as ________.

A. Project Net Worth Optimization


B. Equity Enhancement
C. Independent Projects
D. Optimal Capital Budgeting
E. Mutually Exclusive Projects

Answer: E

Explanation:
Mutually Exclusive Projects are defined as a set of projects where only one can be accepted.

Question: 2540

Which of the following statements is most correct?

A. None of these answers are correct.


B. An increase in fixed costs, (holding sales and variable costs constant) will reduce the company's
degree of operating leverage.
C. If the company has no debt outstanding, then its degree of total leverage equals its degree of

Page | 961
operating leverage.
D. All of these answers are correct.
E. An increase in interest expense will reduce the company's degree of financial leverage.

Answer: C

Explanation:
The degree of financial leverage is the percentage change in EPS that results form a given percentage
change in earnings before interest and taxes. If a firm has no debt outstanding the degree of financial
leverage would be 1.0.

Question: 2541

Which of the following firm's earnings per share (EPS) figure would be least sensitive to a percentage
change in Earnings Before Interest and Taxes (EBIT)?
Firm A
EBIT: $6,800,000
Interest Paid: $505,000
Total Operating Expenses: $80,000,000
Fixed Operating Expenses: $50,250,000
Firm B
EBIT: $20,000,000
Interest Paid: $600,000
Total Operating Expenses: $40,000,000
Fixed Operating Expenses: $30,250,000
Firm C
EBIT: $50,500,000
Interest Paid: $3,500,000
Total Operating Expenses: $66,000,000
Fixed Operating Expenses: $30,750,000
Firm D
EBIT: $49,700,000
Interest Paid: $7,750,000
Total Operating Expenses: $90,000,000
Fixed Operating Expenses: $75,000,000
Firm E
EBIT: $43,000,000
Interest Paid: $7,000,000
Total Operating Expenses: $85,000,000
Fixed Operating Expenses: $60,500,000

A. The answer cannot be determined from the information provided.


B. Firm B
C. Firm A
D. Firm D
E. Firm C
F. Firm E

Answer: B

Explanation:
This question is asking you to calculate the Degree of Financial Leverage for each company. The
Degree of Financial Leverage (DFL) measures the percentage change in EPS that results from a given
percentage change in EBIT. Financial Leverage is the second component of total leverage, along with

Page | 962
Operating Leverage. The equation used to calculate the Degree of Financial Leverage is as follows:
{DFL = [EBIT/(EBIT - Interest Paid)]}. As companies incorporate more debt in their capital structure,
their EPS figure will become more sensitive to fluctuations occurring from interest payments, and
this is evidenced by an increase in the Degree of Financial Leverage. In this example, Firm B has the
lowest DFL with a figure of 1.031. In light of this information, it can be concluded that firm B has an
EPS figure which is the least sensitive to a given change in EBIT. When calculating the DFL figure,
remember that the answer can never be less than one, and can never be negative. In a situation
where the company under examination has zero debt, and no preferred stock dividends (and
therefore no interest expense for purposes of the DFL equation), the DFL would be equal to one. It is
important note to remember is that in calculating the Degree of Financial Leverage, dividend
payments to preferred stockholders should be included in the interest expense figure. Operating
expenses are not factored into the DFL calculation, rather are used in the determination of Operating
Leverage.

Question: 2542

The length of time required for an investment's net revenues to cover its cost is known as ________.

A. Optimal Capital Structure


B. Net Present Valuing
C. Capital Budgeting
D. Payback Period
E. Weighted Average Cost of Capital (WACC)

Answer: D

Explanation:
Payback Period is defined as the length of time required for an investment's net revenues to cover its
cost.

Question: 2543

Which of the following is/are true about project risk analysis?


I. Stand-alone risk is measured by the variability of the projects expected returns.
II. Corporate risk measures the impact of the project's risk on the company's stock price variability.
III. Market risk measures the impact of the project on the stock's unsystematic risk.

A. I & II
B. II & III
C. III only
D. I, II & III
E. II only
F. I only

Answer: F

Explanation:
Standalone risk evaluates the risk of a project ignoring all portfolio aspects by looking at the
variability of the project's projected returns. The corporate risk of a project is measured by the
project's impact on the uncertainty about the firm's future earnings. The market risk of a project is
measured by the project's impact on the systematic risk of the firm's stock.

Question: 2544

Page | 963
Scenario analysis ignores:

A. the range of likely values that key variables can take.


B. changes in some of the key variables.
C. effect on the NPV of changes in project variables.
D. none of these answers.

Answer: D

Explanation:
It is the Sensitivity Analysis that ignores the range of likely values that key variables can take. This is
rectified using Scenario Analysis.

Question: 2545

All else equal, which of the following is/are true?


I. Firms with higher business risk tend to have lower debt ratios.
II. The higher the tax rate imposed on a firm, the lower its optimal debt ratio.
III. The lower a firm's future capital requirements, the lower its current debt ratio.

A. II & III
B. III only
C. I only
D. II only
E. I, II & III
F. I & II

Answer: C

Explanation:
The higher the business risk and the future capital requirement, the stronger the balance sheet must
be. This is accomplished through a lower reliance on debt. The higher the tax rate, the higher is the
attractiveness of the tax-deductibility of the interest payments on debt. This lowers the after-tax cost
of debt, raising the optimal debt ratio.

Question: 2546

A company is analyzing two mutually exclusive projects, S and L, whose cash flows are shown below:
Years0123
S-1,1001,00035050
L-1,10003001,500
The company's cost of capital is 12 percent, and it can get an unlimited amount of capital at that cost.
What is the regular IRR (not MIRR) of the better project, i.e., the project which the company should
choose if it wants to maximize its stock price?

A. 12.00%
B. 18.62%
C. 20.46%
D. 19.08%
E. 15.53%

Answer: D

Explanation:

Page | 964
Because the two projects are mutually exclusive, the project with the higher positive NPV is the
"better" project.
0123
S-1,1001,00035050
NPV(S) = $107.46
IRR(S) = 20.46%
0123
L-1,10003001,500
NPV(L) = $206.83
IRR(L) = 19.08%
Project L is the "better" project: its IRR = 19.08%.

Question: 2547

Incremental cash flows are

A. cash flows that can be attributed to specific tax deductions like depreciation and interest expense.
B. cash flows from a project that occur after the initial capital expense.
C. cash flows that occur only if a project under consideration is accepted.
D. the additional cash flows from a project for a given increase in capital invested.

Answer: C

Explanation:
Incremental cash flows of a project are the cash flows that occur if and only if the project is
undertaken.

Question: 2548

The management of Intelligent Semiconductor have adhered to the following capital structure: 40%
debt, 45% common equity, and 15% perpetual preferred equity. The following information applies to
the firm:
Before-tax cost of debt = 8.25%
Combined state/federal tax rate = 33%
Expected return on the market = 16.5%
Annual risk-free rate of return = 6.25%
Historical Beta coefficient of Intelligent Semiconductor's Common Stock = 1.34 Annual preferred
dividend = $1.05
Preferred stock net offering price = $18.90
Expected annual common dividend = $0.20
Common stock price = $100.90
Expected growth rate = 9.75%
Subjective risk premium = 5.3%
Given this information, and using the Capital Asset Pricing Model (CAPM) to calculate the
component cost of common equity, what is the Weighted Average Cost of Capital for Clay Industries?

A. The WACC for Clay Industries cannot be calculated from the information.
B. 12.94%
C. 13.55%
D. 12.03%
E. 15.60%
F. 11.92%

Answer: D

Page | 965
Explanation:
The calculation of the Weighted Average Cost of Capital is as follows: {fraction of debt * [yield to
maturity on outstanding long-term debt][1-combined state/federal income tax rate]} + {fraction of
preferred stock * [annual dividend/net offering price]} + {fraction of common stock * cost of equity}.
The cost of common equity can be calculated using three methods, the Capital Asset Pricing Model
(CAPM), the Dividend-Yield-plus-Growth-Rate (or Discounted Cash Flow) approach, and the Bond-
Yield-plus-Risk-Premium approach. In this example, you are asked to calculate the cost of common
equity using the Capital Asset Pricing Model. To calculate the cost of equity using this approach, use
the following equation: {risk-free rate + beta(expected return on the market - risk-free rate).
Incorporating the given information into this equation gives a cost of equity of 19.989% The after-tax
cost of debt can be found by multiplying the yield to maturity on the firm's outstanding long-term
debt (8.25%) by (1-tax rate). Using this method, the after-tax cost of debt is found as 5.50%. The
calculation of the cost of perpetual preferred stock is relatively straightforward, simply divide the
annual preferred dividend ($1.05) by the net offering price ($18.90). Using this method, the cost of
preferred stock is found as 5.556%. Incorporating these figures into the WACC equation gives the
answer of 12.027%.

Question: 2549

Woodson Inc. has two possible projects, Project A and Project B with the following cash flows:
Year Project AProject B
0-150,000-100,000
1100,00045,000
2105,00065,000
340,00080,000
At what cost of capital do the two projects have the same net present value (NPV)?

A. 34.8%
B. 10.3%
C. 13.5%
D. 15.8%
E. 21.7%

Answer: E

Explanation:
To determine the crossover rate, find the differential cash flows between the 2 projects and then
calculate the IRR of those differential cash flows.
tProject change, A - B
0-50,000
155,000
240,000
3-40,000
IRR = 21.7%.

Question: 2550

A project requires an initial outlay of $600. Over the next 5 years, it expects to have cash outflows of
$200, $300, $175, $350 and $150. The revenues over the same period equal $100, $400, $700, $550
and $800. Assume that these cash flows occur at year-end. The project's payback period equals
________.

A. 2.91 years

Page | 966
B. 3.38 years
C. 4.11 years
D. 3.82 years

Answer: B

Explanation:
The payback period is defined as the expected number of years that would be required to recover the
original investment. In particular, Payback period = Years before full recovery + (unrecovered cost at
the start of payback year)/(net cash flow in the payback year) To calculate the payback period, you
must have the stream of net cash flows = Revenues - out flows. The net cash flows over the next 5
years are $(100-200), $(400-300), $(700-175), $(550-350) and $(800-150) i.e. the net cash flows are: -
$100, $100, $525, $200 and $650. The complete recovery of the total outlay of $600 + $100 (year 1
net outflow) occurs in the 4th year. At the beginning of the 4th year, the outstanding balance equals
600+100-100-525 = $75. Therefore, payback period = 3 + 75/200 = 3.375 years.

Question: 2551

Hensley Corporation uses breakeven analysis to study the effects of expansion projects it considers.
Currently, the firm's plastic bag business segment has fixed costs of $120,000, while its unit price per
carton is $1.20 and its variable unit cost is $0.60. The firm is considering a new bag machine and an
automatic carton folder as modifications to its existing production lines. With the expansion, fixed
costs would rise to $240,000, but variable cost would drop to $0.41 per unit. One key benefit is that
Hensley can lower its wholesale price to its distributors to $1.05 per carton (i.e., its selling price), and
this would likely more than double its market share, as it will become the lowest cost producer. What
is the change in the breakeven volume with the proposed project?

A. 100,000 units
B. 75,000 units
C. 0 units
D. 175,000 units
E. 200,000 units

Answer: D

Explanation:
Calculate the old and new breakeven volumes using the old data and new projections: Old Q(BE) =
$120,000/($1.20 - $0.60) = $120,000/$0.60 = 200,000 units.
New Q(BE) = $240,000/($1.05 - $0.41) = $240,000/$0.64 = 375,000 units.
Change in breakeven volume = 375,000 - 200,000 = 175,000 units.

Question: 2552

If a firm uses debt financing (Debt ratio = 0.40) and sales change from the current level, which of the
following statements is most correct?

A. The percentage change in net income relative to the percentage change in sales (and in EBIT) will
not depend on the interest rate paid on the debt.
B. The percentage change in EBIT will equal the percentage change in net income.
C. The percentage change in net operating income (EBIT) resulting from the change in sales will
exceed the percentage change in net income (NI).
D. Since debt is used, the degree of operating leverage must be greater than 1.
E. The percentage change in net operating income will be less than the percentage change in net
income.

Page | 967
Answer: E

Explanation:
The greater the use of fixed assets, the more sensitive EBIT will be to changes in sales. Interest
charges on debt are included in net income and not operating income, as the use of debt financing
will have an impact on net income when sales change.

Question: 2553

Intelligent Semiconductor is considering issuing additional common stock. The firm has an after-tax
cost of debt of 8.55%, with the yield to maturity on the firm's outstanding senior long-term debt at
13%. The company's combined federal/state income tax is 35%. The risk-free rate of return is 5.6%,
and the annual return on the broadest market index is expected to be 13.5%. Shares of Intelligent
Semiconductor have a historical beta of 1.6, and in the past, the firm has assumed a 265 basis point
risk premium when calculating the cost of equity. The firm's next dividend is expected to be $0.50
per share, and the dividend has been growing at a 12% annual rate. Finally, the firm's common stock
is priced at $24.78. What is the cost of equity for this firm using the Dividend-Yield-plus-Growth-
Rate, or Discounted Cash Flow (DCF) approach?

A. 18.24%
B. The cost of equity using the DCF approach cannot be calculated from the information provided.
C. 16.15%
D. 14.02%
E. 15.65%
F. 11.20%

Answer: D

Explanation:
The cost of issuing common stock can be calculated using several methods, including the Bond-Yield-
Plus-Risk-Premium approach, Discounted Cash Flow method, or by using the Capital Asset Pricing
Model. In this example, you have been asked to calculate the cost of equity using the Discounted
Cash Flow method, which is commonly referred to as the Dividend-Yield-plus-Growth-Rate approach.
In calculating the cost of equity using this approach, the following components are necessary: next
expected annual dividend, growth rate of dividends, and the current stock price. Everything else
provided in this example is largely irrelevant. The calculation of the cost of equity using the DCF
approach is as follows: {[next annual dividend $0.50 / common stock $24.78] + expected dividend
growth rate 12%} = 14.018%.

Question: 2554

Clay Industries, a diversified industrial firm, is considering investing into a new manufacturing facility
which would allow the Company to expand its operations into a promising new market for
industrialmotors, specifically the High Temperature Superconducting, or HTS motors. This project is
one of many currently under consideration for Clay Industries, and the amount of R&D expense
allocated toward researching this new manufacturing facility is residual in nature. The following
information applies to this new project.
R&D expense for the quarter $15,000
Initial cash outlay ($45,000)
t1: ($40,000)
t2: ($10,000)
t3: $40,000
t4: $40,000

Page | 968
t5: $16,000
t6: $25,000
Assuming no taxes and a $0.00 salvage value at t6, which of the following best represent the IRR for
his project?

A. 7.039%
B. This project will have multiple IRR at any discount rate
C. The IRR cannot be calculated due to the fact that no discount rate has been provided
D. The IRR cannot be calculated due to the fact that the project has uneven cash flows
E. 2.639%That answer is correct!

Answer: A

Explanation:
Remember that the quarterly R&D expense is a sunk cost, and one which cannot be directly
attributable to this project. Because this quarterly R&D expense is not incremental in nature, it
should be omitted from the IRR calculation. Additionally, the fact that this project has uneven cash
flows is irrelevant for our calculation of IRR. To determine the IRR for this project, the following
information is necessary: the initial investment outlay, the amount of each period's cash inflow, and
the number of periods. Incalculating IRR, no discount rate is necessary, so the last answer is
incorrect. The calculation of the IRR is found as follows: incorporate the initial investment outlay of
($45,000) as Cfo, for CO1=($40,000), CO2=($10,000), CO3=$40,000, CO4=$40,000, CO5=$16,000,
CO6=$25,000, CPT IRR. This yields an IRR of 7.039%.

Question: 2555

Allison Engines Corporation has established a target capital structure of 40 percent debt and 60
percent common equity. The firm expects to earn $600 in after-tax income during the coming year,
and it will retain 40 percent of those earnings. The current market price of the firm's stock is $28; its
last dividend was $2.20, and its expected dividend growth rate is 6 percent. Allison can issue new
common stock at a 15 percent flotation cost. What will Allison's marginal cost of equity capital (not
the WACC) be if it must fund a capital budget requiring $600 in total new capital?

A. 13.9%
B. 14.3%
C. 9.7%
D. 15.8%
E. 7.9%

Answer: D

Explanation:
Calculate the retained earnings break point:
Given:
Net income = $600; Debt = 0.4; Equity = 0.6; Dividend payout = 0.6.
Break point(RE) = $600(1 - 0.6)/0.6 = $400.
Allison will need new equity capital; capital budget exceeds Break point(RE).
Use the dividend growth model to calculate k(s):
k(s) = D1/Po + g = 2.2(1.06)/28(1-.15) + .06 = 0.0979 + 0.06 = 0.1579 = 15.8%.
k(s) = component cost of retained earnings or internal equity.

Question: 2556

As the director of capital budgeting for Raleigh/Durham Company, you are evaluating two mutually

Page | 969
exclusive projects with the following net cash flows:
Year Project XProject Z
0-$100-$100
15010
24030
33040
41060
Is there a crossover point in the relevant part of the NPV profile graph (the northeast, or upper right,
quadrant)?

A. Yes, at k = 13%
B. Yes, at k = 9%
C. No
D. Yes, at k = 7%
E. Yes, at k = 11%

Answer: D

Explanation:
Financial calculator solution:
Project X
Inputs: CF(0) = -100; CF(1) = 50; CF(2) = 40; CF(3) = 30; CF(4) = 10.
Output: IRR = 14.489%.
Project Y
Inputs: CF(0) = -100; CF(1) = 10; CF(2) = 30; CF(3) = 40; CF(4) = 60.
Output: IRR = 11.79%.
Calculate the NPVs of the projects at k = 0 discount rate.
NPV(X,k = 0%) = -100 + 50+ 40 + 30 + 10 = 30.
NPV(Y,k = 0%0 = -100 + 10 + 30 + 40 + 60 = 40.
Calculate the IRR of the differential project, i.e., Project(X - Y)
IRR(X - Y)Inputs: CF(0) = 0; CF(1) = 40; CF(2) = 10; CF(3) = -10; CF(4) = -50.
Output: IRR = 7.167%.
Solely using the calculator we can determine that there is a crossover point in the relevant part of an
NPV profile graph. Project X has the higher IRR. Project Y has the higher NPV at k = 0. The crossover
rate is 7.17% and occurs in the upper right quadrant.

Question: 2557

The capital budgeting director of Sparrow Corporation is evaluating a project, which costs $200,000,
is expected to last for 10 years and produce after-tax cash flows, including depreciation, of $44,503
per year. If the firm's cost of capital is 14 percent and its tax rate is 40 percent, what is the project's
IRR?

A. 18%
B. 8%
C. 12%
D. -5%
E. 14%That answer is correct!

Answer: A

Explanation:
$200,000 = $44,503(PVIFA(Irr,10))
PVIFA(Irr,10) = 4.49408 ;IRR = 18%.

Page | 970
Question: 2558

The management of Clay Industries have adhered to the following capital structure: 50% debt, 35%
common equity, and 15% perpetual preferred equity. The following information applies to the firm:
Before-tax cost of debt = 9.5%
Combined state/federal tax rate = 35%
Expected return on the market = 14.5%
Annual risk-free rate of return = 6.25%
Historical Beta coefficient of Clay Industries Common Stock = 1.24
Annual preferred dividend = $1.55
Preferred stock net offering price = $24.50
Expected annual common dividend = $0.80
Common stock price = $30.90
Expected growth rate = 9.75%
Given this information, and using the Dividend-Yield-plus-Growth-Rate approach to calculate the
component cost of common equity, what is the Weighted Average Cost of Capital for Clay Industries?

A. 9.82%
B. 6.93%
C. 8.36%
D. 10.02%
E. The WACC for Clay Industries cannot be calculated from the information provided.
F. 9.79%

Answer: C

Explanation:
The calculation of the Weighted Average Cost of Capital is as follows: {fraction of debt * [yield to
maturity on outstanding long-term debt][1-combined state/federal income tax rate]} + {fraction of
preferred stock * [annual dividend/net offering price]} + {fraction of common stock * cost of equity}.
The cost of common equity can be calculated using three methods, the Capital Asset Pricing Model
(CAPM), the Dividend-Yield-plus-Growth-Rate (or Discounted Cash Flow) approach, and the Bond-
Yield-plus-Risk-Premium approach. In this example, you are asked to calculate the cost of common
equity using the Dividend-Yield-plus-Growth-Rate, or Discounted Cash Flow, approach. To calculate
the cost of common equity using this approach, divide the expected annual dividend by the selling
price of the outstanding common stock, and add the expected growth rate. Using the DCF method,
the cost of common equity can be found as follows: {[$0.80/$30.90] + 9.75%} = 12.34%. The after-tax
cost of debt can be found by multiplying the yield to maturity of the firm's outstanding long-term
debt (9.5%) by (1-tax rate). Using this method, the after-tax cost of debt is found as 6.175%. The
calculation of the cost of perpetual preferred stock is relatively straightforward, simply divide the
annual preferred dividend by the net offering price. Using this method, the cost of preferred stock is
found as 6.327%. Incorporating these figures into the WACC equation gives the answer of 8.355%.

Question: 2559

Under the Residual Dividend Policy, a firm pays out:

A. none of these answers.


B. only net earnings left over after financing the current optimal capital budget requirements,
consistent with the target capital structure.
C. all of its earnings left over after taxes and expenses as dividends.
D. only net earnings from new projects as dividends, using the rest to finance current capital
requirements.

Page | 971
Answer: B

Explanation:
Under the Residual Dividend Policy, a firm first determines the amount of capital it requires for
sufficiently profitable projects. It then uses retained earnings to supply equity capital and raises debt
in the proper amount to maintain the target capital structure. If any earnings are left over after this,
they are paid out as dividends. If not, the firm will not only not pay any dividends but also issues new
equity for financing.

Question: 2560

In the calculation of WACC, which of the following should be ignored?

A. none of these answers.


B. long-term debt.
C. current liabilities.
D. preferred equity.

Answer: C

Explanation:
Short-term debt is not a part of the capital structure. The capital budgeting process allocates
resources to long-term asset investments and as such is financed by liabilities/capital of similar
maturity. Hence, short-term/current liabilities do not enter into WACC calculations.

Question: 2561

Which of the following is/are true about the MACRS?


I. MACRS does not use economic life of an asset while calculating depreciation.
II. Under the MACRS system, the depreciation expense is larger in the early years, leading to lower
taxes.
III. Depreciation under MACRS must be calculated using the accelerated depreciation method.

A. III only
B. II only
C. I, II & III
D. II & III
E. I only
F. I & II

Answer: F

Explanation:
MACRS classifies assets into several classes based on a pre-determined length of time called the
"recovery period." The recovery period, while positively correlated with actual economic life, does
not track the economic lives of individual assets precisely. In particular, recovery periods are shorter
than actual economic lives, leading to higher depreciation expenses and lower taxes. For long-
recovery period classes (>27 years), straight-line depreciation must be used while accelerated
methods may be used for shorter life assets.

Question: 2562

Which of the following statements is false?

Page | 972
A. When IRR = k (the cost of capital), NPV = 0.
B. If the multiple IRR problem does not exist, any independent project acceptable by the NPV
method will also be acceptable by the IRR method.
C. The IRR can be positive even if the NPV is negative.
D. The NPV will be positive if the IRR is less than the cost of capital.
E. The NPV method is not affected by the multiple IRR problem.

Answer: D

Explanation:
If the IRR is less than the cost of capital, then taking on the project imposes a cost on current
stockholders. If the cost of capital is greater than the IRR, the NPV will be negative.

Question: 2563

A firm's capital structure has a debt-to-equity ratio of 0.8. The pretax cost of debt is 7%. The beta of
the stock is 1.3 in an environment with risk-free rate of 5.5% and an expected market return of 16%.
The firm is in the 45% tax bracket. The weighted average cost of capital of the firm equals ________.

A. 12.35%
B. 9.43%
C. 6.91%
D. 13.81%That answer is correct!

Answer: A

Explanation:
Using CAPM, the cost of equity equals 5.5% + 1.3*(16% - 5.5%) = 19.15%. Since the debt interest is
tax deductible, the after-tax cost of debt equals 7%*(1-0.45) = 3.85%. Now, the D/E ratio = 0.8.
Hence, (D+E)/E = 1.8, giving E/(D+E) = 0.556. Thus, equity forms 55.6% of the capital while debt
forms 44.4%. The WACC is then equal to 0.556*19.15% + 0.444*3.85% = 12.35%.

Question: 2564

A project has the following cash flows over the next 5 years: $800, $300, $400, $900 and $1,200.
Assume all cash flows occur at the end of a year. The project requires an initial cash outlay of $1,750.
The firm faces a marginal borrowing rate of 8%. The payback period for the project equals ________.

A. 3.86 year
B. 4.19 years
C. 4 years
D. 3.28 years

Answer: D

Explanation:
The payback period is defined as the expected number of years that would be required to recover the
original investment. In particular,
Payback period = Years before full recovery +
(unrecovered cost at the start of payback year)/(net cash flow in the payback year) In this case, the
recovery occurs in the 4th year. At the beginning of the 4th year, the unrecovered cost equals 1,750 -
800 - 300 - 400 = 250. Total cash flow in the 4th year equals 900. Therefore, payback period = 3 +
250/900 = 3.28 years. Note that the discount rate does not figure in the calculation of payback

Page | 973
period.

Question: 2565

While calculating the weights of various components of the capital structure, one must use:

A. minimum of book or market values.


B. book values.
C. liquidation values.
D. market values.

Answer: D

Explanation:
WACC calculations are based on current market values, not historical cost.

Question: 2566

Assume the following information about two individual projects.


Project A
Initial cash outflow: $175,000
Expected cash inflows
t1: $75,000
t2: $65,000
t3: $35,000
t4: $35,000
t5: $15,000
Project B
Initial cash outflow: $100,000
Expected cash inflows
t1: $15,000
t2: $15,000
t3: $18,000
t4: $45,000
t5: $45,000
Assuming these projects are not mutually exclusive, and the cost of capital is 10%, which of the two
should be undertaken according to NPV? Additionally, which of the two projects has the steeper NPV
profile?

A. Project B should be accepted, project A has a steeper NPV profile


B. Project A should be accepted, project A has a steeper NPV profile
C. Project B should be accepted, project B has a steeper NPV profile
D. Both projects should be accepted, project A has a steeper NPV profile
E. Project A should be accepted, project B has a steeper NPV profile
F. Neither project should be accepted, project B has a steeper NPV profile

Answer: E

Explanation:
The NPV of project B is found to be ($1,766.21), and thus should not be accepted. However, project A
has a positive NPV of $6,416.14, and should be accepted. Project B is characterized as having the
majority of its cash inflows occurring in later time periods, and thus is more sensitive to changes in
the cost of capital. This is exemplified by a steeper NPV profile for project B.

Page | 974
Question: 2567

Which of the following equations correctly illustrates the calculation of the cost of equity using the
Discounted Cash Flow approach?

A. (Retention rate)*(ROE)
B. Last annual dividend/(1 + required rate of return)
C. Next annual dividend/current stock price
D. (1-tax rate)
E. (Next annual dividend/current stock price) + expected growth rate
F. (Last annual dividend/[expected return - required return])
G. expected growth rate
H. Risk-free rate of return + beta(expected return on the market - risk-free rate of return)

Answer: E

Explanation:
The Dividend-Yield-plus-Growth-Rate approach calls for the following components: next annual
dividend, current stock price, and expected growth rate. This approach, also known as the
Discounted Cash Flow (DCF) method, is a flexible and very adept tool in the hands of the financial
analyst, and is it is imperative that the CFA candidate fully understand both the applications and the
methodology of this approach. The fourth choice illustrates the Capital Asset Pricing Model, while
the fifth represents an approach for calculating sustainable growth rate. The remaining answers are
somewhat fictitious.

Question: 2568

Rollins Corporation is constructing its MCC schedule. Its target capital structure is 20 percent debt, 20
percent preferred stock, and 60 percent common equity. Its bonds have a 12 percent coupon, paid
semiannually, a current maturity of 20 years, and sell for $1,000. The firm could sell, at par, $100
preferred stock, which pays a 12 percent annual dividend, but flotation costs of 5 percent would be
incurred. Rollins' beta is 1.2, the risk-free rate is 10 percent, and the market risk premium is 5
percent. Rollins is a constant growth firm, which just paid a dividend of $2.00, sells for $27.00 per
share, and has a growth rate of 8 percent. The firm's policy is to use a risk premium of 4 percentage
points when using the bond-yield-plus-risk- premium method to find k(s). The firm's net income is
expected to be $1 million, and its dividend payout ratio is 40 percent. Flotation costs on new
common stock total 10 percent, and the firm's marginal tax rate is 40 percent. What is Rollins' WACC
once it starts using new common stock financing?

A. 16.6%
B. 13.6%
C. 14.1%
D. 16.9%
E. 16.0%

Answer: C

Explanation:
k(e) = $2.16/ $27.00(1-.10) + 0.08 = 0.08889 + 0.08 = 0.169 = 16.9%.
WACC = 0.2(12.0%)(0.6) + 0.2(12.6%) + 0.6(16.9%) = 14.1%.

Question: 2569

Intelligent Semiconductor, a diversified technology company, is evaluating the sales of its cadmium

Page | 975
silicon transistor coils, and has identified the following information:
Fixed production costs for these transistors: $750,000
Average sales price per unit: $405.00
Variable cost per unit: $313.60
Which of the following best describes the breakeven quantity for this product?

A. The breakeven quantity for this product cannot be determined from the information provided.
B. 8,206 units
C. 1,044 units
D. 5,397 units
E. 7,397 units

Answer: B

Explanation:
To calculate the breakeven quantity for a product, use the following equation: {Fixed operating
costs/[avg. sales price per unit - variable cost per unit]}. Incorporating the given information into this
equation yields the following: {$750,000/[$405 - $313.60]}=8,206 units.

Question: 2570

The optimal debt ratio is the debt ratio that:

A. minimizes the firm's bankruptcy costs.


B. maximizes the firm's earnings per share and maximizes the firm's stock price.
C. maximizes the firm's stock price.
D. maximizes the firm's earnings per share.

Answer: C

Explanation:
The optimal debt ratio is defined as the debt level that maximizes the firm's stock price.

Question: 2571

A cash outlay that has already been incurred and which cannot be recovered regardless of whether
the project is accepted or rejected is known as which of the following terms?

A. Incremental Cash Flow


B. Externality
C. Sunk Cost
D. Opportunity Cost
E. Cannibalization

Answer: C

Explanation:
Sunk cost is defined as a cash outlay that has already been incurred and which cannot be recovered
regardless of whether a project is accepted or rejected.

Question: 2572

The management of Intelligent Semiconductor is examining the asset structure of its superconductor
division, and has ascertained the following annual financial information: Invoiced sales $6,850.000

Page | 976
EBITA $3,525,000
Interest expense $150,000
Amortization expense $245,000
Given this information, which of the following best characterizes the Degree of Financial Leverage for
this division?

A. 1.048
B. None of these answers is correct.
C. 1.044
D. .0.488
E. 2.03That answer is correct!

Answer: A

Explanation:
To calculate the DFL, the financial analyst needs to determine the EBIT and interest paid for a
predetermined time period. To calculate the Degree of Financial Leverage, the following equation is
used: {EBIT/[EBIT - interest paid]}. In this example, EBITA is provided rather than EBIT. Thankfully,
however, a figure is given for amortization expense. To determine the EBIT, subtract the amortization
expense from then EBITA figure, which gives a figure of $3,280,000 for the EBIT. The next step is to
incorporate the given information into the DFL equation as follows: {EBIT $3,280,000 / [EBIT
$3,280,000 - interest expense $150,000]}=1.048 The Degree of Financial Leverage measures the
percentage change in EPS which results from a given percentage change in EBIT. Remember that any
preferred stock dividends must be incorporated into the DFL calculation, and that the DFL can never
be less than one.

Question: 2573

Sunk costs:

A. should be ignored while evaluating a project.


B. affect a project's desirability adversely.
C. are incremental cash flows of the project under consideration.
D. lower a project's NPV.That answer is correct!

Answer: A

Explanation:
Sunk costs represent expenses that have already been incurred or committed to. Therefore, they are
not pertinent to future decisions.

Question: 2574

Becker Glass Corporation expects to have earnings before interest and taxes during the coming year
of $1,000,000, and it expects its earnings and dividends to grow indefinitely at a constant annual rate
of 12.5 percent. The firm has $5,000,000 of debt outstanding bearing a coupon interest rate of 8
percent, and it has 100,000 shares of common stock outstanding. Historically, Becker has paid 50
percent of net earnings to common shareholders in the form of dividends. The current price of
Becker's common stock is $40, but it would incur a 10 percent flotation cost if it were to sell new
stock. The firm's tax rate is 40 percent. What is Becker's cost of newly issued stock?

A. 17.5%
B. 16.5%
C. 16.0%

Page | 977
D. 17.0%
E. 18.0%That answer is correct!

Answer: A

Explanation:
Cost of new common equity:
k(e) = $1.80/$40.00(1-.10) + 0.125 = 17.5%.
The dividend of $1.80 was derived by:
EBIT$1,000,000
Interest 400,000
EBT$600,000
Taxes (40%)240,000
Net income$360,000
EPS(1) = $360,000/100,000 = $3.60.
D(1) = $3.60(0.5) = $1.80.

Question: 2575

Which of the following statements is most correct?

A. None of the answers are correct.


B. All is these answers are correct.
C. If Project A has a higher IRR than Project B, then Project A must also have a higher NPV.
D. The IRR calculation implicitly assumes that all cash flows are reinvested at a rate of return equal to
the cost of capital.
E. If a project's internal rate of return (IRR) exceeds the cost of capital, then the project's net present
value (NPV) must be positive.

Answer: E

Explanation:
The IRR on a project is its expected rate of return. If the return exceeds the cost of the funds used to
finance the project, a surplus remains after paying for the capital, and this surplus accrues to the
firm's stockholders. Therefore, a project whose IRR exceeds its cost of capital increase shareholders'
wealth, just as a positive NPV does.

Question: 2576

The management of Microscam International, a large software manufacturer, is examining its capital
structure. The firm is financed according to the following schedule based on market values:
40% debt
50% common stock
10% perpetual preferred stock
Additionally, consider the following information:
Yield on outstanding debt: 9.25%
Tax rate: 35%
Annual preferred dividend: $2.02
Preferred stock price: $17.44
Return on equity: 22%
Dividend payout ratio: 15%
Cost of common stock: 15.40%
Using this information, what is the Weighted Average Cost of Capital for Microscam?

Page | 978
A. 11.08%
B. 11.12%
C. None of these answers.
D. 10.88%
E. 11.26%
F. The answer cannot be completely calculated from the given information.

Answer: E

Explanation:
In order to calculate the WACC, it is necessary to first calculate the component after-tax cost of debt,
common equity, and preferred equity. Once the cost of these components is determined, they are
imputed into the WACC equation, which is as follows:
{WACC = [(% weight of debt securities * cost of debt) + (% weight of common stock * cost of common
stock) + (% weight of preferred stock * cost of preferred stock)]}
To calculate the component cost of debt, use the following equation: {After-tax cost of debt = [yield
on outstanding debt securities * (1 - tax rate)}
Factoring in the given information into this equation would yield the following:
{After-tax cost of debt = [9.25% * (1 - 0.35%)]} = 6.013%
To calculate the component cost of outstanding preferred stock, the following equation must be
used:
{Cost of preferred stock = [annual dividend / preferred stock price]}
{Cost of preferred stock - = [$2.02 / $17.44]} = 11.58%.
The final component of the WACC calculation, the cost of common equity, has been provided as
15.40%.
Now that the after-tax cost of debt, preferred stock, and common stock have been determined, the
WACC calculation can be found. The calculation of the WACC is as follows: {[0.40 * 0.06013] + [0.50 *
0.1540] + [0.10 * 0.1158]} = 11.263%.

Question: 2577

Firm A has just paid a cash dividend of $6.2 per share. If the growth rate is expected to be 6% and the
price of the stock is $24.90, the expected return on the stock is:

A. 32.39%
B. 26.39%
C. 20.39%
D. 24.90%That answer is correct!

Answer: A

Explanation:
Po = D1/(k-g). In this case, g = 6%, D1 = Do*(1+g) = 6.2*1.06 = $6.572 and Po = $24.90. Therefore, k =
32.39%. Note that Brigham & Houston refer to k as the cost of retained earnings; this is the same as
the expected rate of return demanded by shareholders, which is the same as the rate of return on
common equity.

Question: 2578

When examining a capital project with non-normal cash flows from the perspective of IRR, some
distinct problems can arise, which of the following choices best describes the problems that can
occur when examining projects with non-normal cash flows using IRR?

A. No IRR, an IRR which leads to an incorrect accept reject decision, concave MIRR profile

Page | 979
B. Multiple IRRs, an IRR which leads to an incorrect accept/reject decision, convex MIRR profile
C. Multiple IRRs, no IRR
D. Multiple IRRs, no IRR, an IRR which leads to an incorrect accept/reject decision
E. Timing differences, project scale differences, multiple IRRs

Answer: D

Explanation:
When examining capital projects with non-normal cash flows, three distinct problems can occur: the
project can have multiple IRRs, no IRR, or the calculation can lead to the production of an IRR figure
which leads to an incorrect accept/reject decision. Thus, the second choice represents the most
correct answer. "Timing differences" and "project scale differences" are two reasons for a conflict in
the NPV and IRR calculations, and "MIRR profile" is a fictitious term.

Question: 2579

Consider the following two projects:


Project A
Initial cash outflow:$1,000,000
Cash inflows as follows
t1: $500,000
t2: $450,000
t3: $250,000
t4: $150,000
t5: $150,000
Project B
Initial cash outflow: $1,000,000
Cash inflows as follows
t1: $150,000
t2: $150,000
t3: $250,000
t4: $450,000
t5: $500,000
Assuming a cost of capital of 9%, no taxes, and a $0.00 salvage value for each project at the end of
year 5, what is the NPV of each project? Additionally, which of the two projects has the steeper NPV
profile?

A. Project A NPV: $88,596.13, Project B NPV: $110,900.51, Project A has a steeper NPV profile
B. Project A NPV: $114,078.88, Project B NPV: $100,669.59, Project has B has a steeper NPV profile
C. Project A NPV: $234,270.95, Project B NPV: $100,669.59 , Project A has a steeper NPV profile
D. Project A NPV: $234,270.95, Project B NPV: $100,669.59, Project B has a steeper NPV profile
E. Project A NPV: $234,270.95, Project B NPV: $100,669.59, Project A has a steeper NPV profile

Answer: D

Explanation:
Due to the fact that project B has the majority of its cash inflows coming in later periods, it is more
sensitive to changes in the cost of capital than is project A, which has the majority of its cash flows
coming in earlier periods. This is exemplified by a steeper NPV profile.

Question: 2580

Clay Industries, a large industrial firm, has begun the development of an underwater drilling system
which will greatly increase the efficiency of deep-sea petroleum extraction. In their analysis of the

Page | 980
project's cash-flow potential, the corporate finance division of Clay Industries does not factor in the
initial R&D costs for the quarter, rather examines only the initial cash outlay and expected cash
inflows specific to the underwater drilling system. The R&D costs involved for this quarter could best
be described as which of the following?

A. Externality
B. None of these answers
C. Opportunity cost
D. Implicit cost
E. Sunk cost
F. Incremental cost

Answer: E

Explanation:
In this example, the R&D expenditures are an example of a sunk cost. In an analysis of any project,
sunk costs are not included. This is because sunk costs represent outlays which have already occurred
or have already been committed. These costs are not incremental, and hence are not affected by the
decision under consideration.

Question: 2581

Which of the following projects would likely result in multiple Internal Rates of Return?
Project A
Initial investment outlay: ($450,000)
t1: $400,000
t2: ($40,000)
t3: $190,000
Project B
Initial investment outlay: ($50,000)
t1: $0.00
t2: $0.00
t3: $75,000
Project C
Initial investment outlay: ($300,000)
t1: $15,000
t2: ($34,000)
t3: $0.00
t4: $400,000
Project D
Initial investment outlay: ($100,000)
t1: $150,000
t2: $380,000
t3: $45,000
t4: $45,000
Project E
Initial investment outlay: ($1,000,000)
t1: $1,500,000
t2: $1,300
t3: $0.00
t4: $60,000

A. None of these choices


B. Project B, Project D

Page | 981
C. The answer cannot be determined from the information provided
D. Project C, Project E
E. Project A, Project C,
F. Project D, Project E

Answer: E

Explanation:
In evaluating projects with "non-normal cash flows" the Internal Rate of Return method will often
produce multiple IRRs which leads to an incorrect accept/reject decision. Non-normal cash flows are
defined as cash flows in which the sign changes more than once. Projects A, and C involve cash
outflows superimposed within their cash inflows, resulting in a sign change from positive to negative
and negative to positive. In examining projects such as these, it is advisable to use either the NPV or
MIRR methods, which are not subject to the problem of multiple IRRs. From observation alone, we
can determine that project A and C are non-normal projects, and are thus likely to result in multiple
IRR calculations. While project B, D, and E have periods of zero cash flow, they have only one change
of sign in the overall cash flow process, and therefore should be characterized as "normal." While the
cost of capital has been provided, it is not necessary for the determination of the correct answer in
this case. What you should look for are projects with non-normal cash flows, and this should not
involve any computational analysis. Besides, the cost of capital is not incorporated into the Internal
Rate of Return calculation, rather is a component of the NPV and MIRR.

Question: 2582

The following information applies to a company's preferred stock:


Current price $105.00 per share
Par value $100.00 per share
Annual dividend $5.00 per share
The company issued the preferred stock at par and incurred a 10% floatation cost. If the company's
marginal corporate tax rate is 40%, what is the after-tax cost of preferred stock?

A. 5.0%
B. 10.0%
C. 3.0%
D. 4.8%
E. 5.6%
F. 2.9%

Answer: E

Explanation:
The cost of preferred stock is calculated as the preferred stock dividend divided by the net issuing
price. The dividend for this preferred stock is $5.00, and the net issuing price was $90.00. Thus the
cost of preferred stock is 5 divided by 90 or 5.6%. There are no tax savings associated with the use of
preferred stock, therefore no tax adjustments are made when calculating the cost.

Question: 2583

Consider the following information:


Borrowing Rate 10%
Marginal Tax Rate 40%
Preferred Stock Par Price $50
Preferred Dividend $5
Preferred Stock floatation cost 2.0%

Page | 982
Cost of common equity 15.0%
Preferred Stock issued at Par
The Optimal Capital Structure is 45% debt, 50% common equity, and 5% preferred stock. Credit
Rating BB+
What is the firm's Weighted Average Cost of Capital (WACC)?

A. 9.0%
B. 7.14%
C. 9.06%
D. 10.71%
E. 2.5%
F. 28.00%

Answer: D

Explanation:
The firm's Weighted Average Cost of Capital (WACC) is a weighted average of the component cost of
capital. In this case 10%(borrowing rate) x (1-.4)Tax savings = 6% is the component cost of debt. $5
(preferred dividend) / 49(Par minus floatation cost) = 10.2% is the component cost of preferred stock.
Thus the WACC = .45(6%) + .5(15%) + .05(10.2%) = 10.71%

Question: 2584

Which of the following is/are true for a project which needs only an initial outlay and no further
expenses?
I. The shorter the payback period, the greater the liquidity of the project.
II. The discounted payback period is always more than the simple payback period.
III. The payback period rule considers all the cash flows involved in a project.

A. II & III
B. II only
C. I & II
D. III only
E. I & III
F. I only
G. I, II & III

Answer: C

Explanation:
The payback period measures how quickly you recover your initial investment. The shorter this
period, the greater the liquidity in terms of cash recovery. The payback rule ignores cash flows
beyond the payback period. The discounted payback period is defined as the expected number of
years that would be required to recover the original investment using discounted cash flows. Hence,
(II) is true if there are no negative cash flows after the initial investment since discounting reduces
the present value of the future cash flows.

Question: 2585

Which of the following statements is correct?

A. When the MCC (Marginal Cost of Capital) schedule is developed, the first break point always
occurs as a result of using up retained earnings.
B. Flotation costs must be included in the component cost of preferred stock.

Page | 983
C. If a company with a debt ratio of 50 percent were suddenly exempted from all future income
taxes, then, all other things held constant, this would cause its WACC to increase.
D. The WACC (Weighted Average Cost of Capital) should include only after-tax component costs.
Therefore, the required rates of return on debt, preferred, and common equity must be adjusted to
an after-tax basis before they are used in the WACC equation.
E. The cost of retained earnings is generally higher than the cost of new common stock.

Answer: C

Explanation:
If a firm paid no income taxes, its cost of debt would not be adjusted downward, hence the
component cost of debt would be higher than if T (the firm's marginal tax rate) were greater than 0.
With a higher component cost of debt, the WACC would be increased. Of course, the company would
have higher earnings, and its cash flows from a given project would be high, so the higher WACC
would not impede its investments, i.e., its capital budget would be larger than if it were taxed.

Question: 2586

Your company is planning to open a new gold mine which will cost $3 million to build, with the
expenditure occurring at the end of the year. The mine will bring year-end after-tax cash inflows of
$2 million at the end of the two succeeding years, and then it will cost $0.5 million to close down the
mine at the end of the third year of operation. What is this project's IRR?

A. 12.70%
B. 14.36%
C. 10.17%
D. 21.53%
E. 17.42%That answer is correct!

Answer: A

Explanation:
Time line:
0123
-3,000,0002,000,0002,000,000-500,000
Financial calculator solution: (In millions)
Inputs: CF(0) = -3; CF(1) = 2; N(j) = 2; CF(2) = -.5.
Output: IRR% = 12.699%.

Question: 2587

Firmica recently bought a fleet of trucks which fall in the 5-year MACRS class for $135,000, with an
additional $10,000 for shipping, minor taxes and paperwork. The trucks are expected to last for 7
years and have a total salvage value of $25,620. The recovery allowance for year 1 in the 5-year
MACRS class is 20% and in the second year, it is 32%. In the second year, the depreciation expense
arising from the fleet equals ________.

A. $46,400
B. $34,560
C. $43,200
D. $37,120That answer is correct!

Answer: A

Page | 984
Explanation:
You should remember four things about MACRS:
1. Salvage value is never considered in calculating the depreciable basis.
2. The depreciable basis equals the purchase price plus all shipping and installation costs. 3. The
depreciable basis does not change over the life of the asset in question.
4. Depreciation expense for any given year equals the allowed recovery percentage in that MACRS
class times the depreciable basis.
Therefore, in this case, the depreciable basis equals $135,000 + $10,000 = $145,000 and the year 2
depreciation equals $145,000*32% = $46,400.

Question: 2588

Jackson Corporation is evaluating the following four independent, investment opportunities:


Project CostRate of Return
A$300,00014%
B$150,00010
C$200,00013
D$400,00011
Jackson's target capital structure is 60 percent debt and 40 percent equity. The yield to maturity on
the company's debt is 10 percent. Jackson will incur flotation costs for a new equity issuance of 12
percent. The growth rate is a constant 6 percent. The stock price is currently $35 per share for each of
the 10,000 shares outstanding. Jackson expects to earn net income of $100,000 this coming year and
the dividend payout ratio will be 50 percent. If the company's tax rate is 30 percent, then which of
the projects will be accepted?

A. All of the investment projects will be taken.


B. Projects A, C, and D.
C. Projects A and C.
D. None of the investment projects will be taken.
E. Project A.

Answer: E

Explanation:
Calculate the after-tax component cost of debt as 10%(1 - 0.3) = 7%. If the company has earnings of
$100,000 and pays out 50% or $50,000 in dividends, then it will retain earnings of $50,000. The
retained earnings breakpoint is $50,000/0.4 = $125,000. Since it will require financing in excess of
$125,000 to undertake any of the alternatives, we can conclude the firm must issue new equity.
Therefore, the pertinent component cost of equity is the cost of new equity. Calculate the expected
dividend per share as $50,000/10,000 = $5. Thus, the cost of new equity is $5/[($35(1 - 0.12)] + 6% =
22.23%. Jackson's WACC is 7%(0.6) + 22.23%(0.4) = 13.09%. Only the return on Project A exceeds the
WACC, so only Project A will be undertaken.

Question: 2589

Helms Aircraft has a capital structure, which consists of 60 percent debt and 40 percent common
stock. The company's equity financing will come from issuing new common stock. The company
recently issued bonds with a yield to maturity of 9 percent. The company's stock is currently trading
at $40 a share. The year-end dividend is expected to be $4 a share (that is, D(1) = $4.00), and the
dividend is expected to grow at a constant rate of 5 percent. The flotation costs associated with
issuing new common stock are estimated to be 10 percent. The company's tax rate is 35 percent.
What is the company's weighted average cost of capital?

A. 11.84%

Page | 985
B. 10.98%
C. 8.33%
D. 9.51%
E. 9.95%

Answer: E

Explanation:
Calculate the after-tax cost of debt = (1 - 0.35) 9% = 5.85%.
Calculate the cost of new equity: k(e) = $4/[$40(1 - 0.1)] + 0.05 = 0.1611 or 16.11%. Compute WACC
(Weighted Average Cost of Capital): 0.6(5.85%) + 0.4(16.11%) = 9.95%.

Question: 2590

Copybold Corporation is a start-up firm considering two alternative capital structures--one is


conservative and the other aggressive. The conservative capital structure calls for a D/A ratio = 0.25,
while the aggressive strategy call for D/A = 0.75. Once the firm selects its target capital structure it
envisions two possible scenarios for its operations: Feast or Famine. The Feast scenario has a 60
percent probability of occurring and forecast EBIT in this state is $60,000. The Famine state has a 40
percent chance of occurring and the EBIT is expected to be $20,000. Further, if the firm selects the
conservative capital structure its cost of debt will be 10 percent, while with the aggressive capital
structure its debt cost will be 12 percent. The firm will have $400,000 in total assets, it will face a 40
percent marginal tax rate, and the book value of equity per share under either scenario is $10.00 per
share. What is the coefficient of variation of expected EPS under the aggressive capital structure
plan?

A. 2.45
B. 1.00
C. 1.18
D. 3.76
E. 2.88That answer is correct!

Answer: A

Explanation:
Calculate coefficient of variation
Expected EPS(Aggressive):
E(EPS) = 0.6 EPS(Feast) + 0.4 EPS(Famine) = (0.6)($1.44) + 0.4(-$0.96) = $0.48. Standard deviation
SD(EPS-aggressive) = [0.6($1.44 - $0.48)^2 + 0.4(-$0.96 - $0.48)^2]^1/2
= [0.5530 + 0.8294]^1/2 = 1.176.
CV(Aggressive) = 1.176/0.48 = 2.45.

Question: 2591

Clay Industries, a large industrial firm, is evaluating the sales of its existing line of coiled machine
tubing. In their analysis, the operating managers of Clay Industries have identified the following
information related to the coiled machine tubing division and its product:
Average variable cost of $100.50
Average sales price of $167.75
Breakeven quantity of 15,985 units
Which of the following best describes the total fixed cost for this product?

A. $1,114,800
B. The calculation of the total fixed production costs for this product cannot be calculated from the

Page | 986
information given.
C. $875,000
D. None of these answers is correct.
E. $925,000
F. $1,075,000

Answer: F

Explanation:
To calculate the breakeven quantity for a product, use the following equation: {Fixed operating
costs/[avg. sales price per unit - variable cost per unit]}. To determine the total fixed production cost
of this product, we must rearrange the standard equation using algebra, in a manner such that the
resulting equation resembles the following: [$167.75 - $100.50]
* 15,985 units = X. Solving for X, which represents the total fixed production costs, yields an answer
of $1,075,000.

Question: 2592

Santorum Co. has a capital structure which consists of 50 percent debt, 30 percent common stock,
and 20 percent preferred stock. The company's net income was just reported to be $1,000,000. The
company pays out 40 percent of its net income as dividends. How large of a capital budget can the
company have, without having to issue additional common stock or change its capital structure?

A. $600,000
B. $2,000,000
C. $1,200,000
D. $200,000
E. $180,000

Answer: B

Explanation:
The retained earnings break point indicates the size of the capital budget when not issuing additional
common stock. BP(RE) = ($1,000,000(1 - 0.4))/0.30 = $2,000,000.
BP = break point; RE = retained earnings

Question: 2593

The management of Intelligent Semiconductor is considering two mutually exclusive projects, which
are detailed below:
Project A
Electron looping apparatus
Initial investment outlay ($6,000,000)
t1: $2,750,000
t2: $1,250,000
t3: $1,250,000
t4: $2,750,000
Cost of capital of 10.55%
Project B
Optical switching apparatus
Initial investment outlay ($5,040,000)
t1: $1,000,000
t2: $1,000,000
t3: $1,500,000

Page | 987
t4: $1,500,000
t5: $1,500,000
t6: $750,000
t7: $300,000
t8: $50,000
Cost of capital of 10.55%
Assuming no taxes, a $0.00 salvage value at the end of the each project's life, as well as the ability to
replicate each project identically at the end of its lifespan, which is the superior investment
according to the Common Life approach? Additionally, what are the NPV and IRR of the superior
project over the common life?

A. Project B, NPV $305,221; IRR 13.65%


B. None of these answers
C. Project B, NPV $287,725.32; IRR 12.38%
D. Project A, NPV $465,515; IRR 12.78%
E. Project A, NPV $462,038; IRR 12.72%

Answer: E

Explanation:
The Replacement Chain, or "Common Life" approach, is a useful method which allows two or more
projects with unequal lives to be examined. In the Replacement Chain approach, the lifespans of
each project being examined are multiplied in such a way that the resulting projects share a
"common life." In this example, the Optical Switching apparatus has a lifespan of eight periods, while
the electron looping apparatus has a four-period lifespan. The common multiple of both projects is 8,
and by replicating the cash flows of the electron looping project through period 4, i.e. by carrying out
the project for an additional cycle, we can arrive at a "common life" for both projects. Carrying out
the electron-looping project through eight periods will yield the following series of cash flows:
Electron looping apparatus
t0: ($6,000,000)
t1: $2,750,000
t2: $1,250,000
t3: $1,250,000
t4: [$2,750,000 + ($6,000,000)]=($3,250,000)
t5: $2,750,000
t6: $1,250,000
t7: $1,250,000
t8: $2,750,000
By incorporating these cash flows into your calculator, you will find a NPV of $462,038 for this
project, as well as an IRR of 12.72%. The Optical Switching apparatus has a NPV of $287,725 and an
IRR of 12.38%.

Question: 2594

Which of the following statements is most correct?

A. Sunk costs should be ignored in capital budgeting.


B. None of these answers are correct.
C. Externalities should be ignored in capital budgeting.
D. All of these answers are correct.
E. Opportunity costs should be ignored in capital budgeting.That answer is correct!

Answer: A

Page | 988
Explanation:
Sunk costs should be ignored. The other statements are false.

Question: 2595

The management of Intelligent Semiconductor is considering the creation of a new manufacturing


facility. The following information applies to the new facility:
Initial investment outlay: ($50,200,000)
t1: ($3,000,000)
t2: ($1,500,000)
t3: $12,000,000
t4: $20,000,000
t5: $25,000,000
t6: $25,000,000
t7: $20,000,000
t8: ($1,500,000)
t9: ($3,000,000)
t10: $500,000
Assuming a 15% discount rate, along with a $0.00 salvage value at the end of year 10, what is the
Modified Internal Rate of Return for this project?

A. 13.19%
B. 9.88%
C. 12.66%
D. 14.61%
E. 13.90%
F. Because this is a non-normal project, the Modified Internal Rate of Return cannot be calculated.

Answer: E

Explanation:
Remember that the Modified Internal Rate of Return escapes many of the pitfalls associated with the
traditional Internal Rate of Return calculation. One such pitfall is the fact that the traditional IRR
cannot produce reliable calculations for "non-normal" projects, such as the project illustrated in this
example. The Modified Internal Rate of Return, however, escapes this basic flaw and can be used to
evaluate virtually any project. The calculation of the answer in this example is as follows:
Step 1: Determine the Future Value of the cash inflows by compounding each positive inflow by the
cost of capital. This value is often referred to as the "Terminal Value." Remember that in this
example, the positive cash inflows begin at period 3.
Step 2: Determine the Present Value of the cash outflows by discounting each negative inflow by the
cost of capital. The cash inflows to be discounted occur in periods 1, 2, 8, and 9.
Step 3: Determine the rate that equates the PV of the cash outflows to the FV of the cash inflows.
The calculation of the FV of the cash inflows is shown as follows:
FV of the cash inflows = {[$12,000,000 * 2.660] + [$20,000,000 * 2.313] + [$25,000,000 * 2.011] +
[$25,000,000 * 1.749] + [$20,000,000 * 1.521] + [$500,000 * 1]} = $203,100,000.
This is the terminal value.
The calculation of the PV of the cash outflows is calculated as follows:
PV of the cash outflows = {$50,200,000 + [$3,000,000 / 1.15] + [$1,500,000 /1.323] + [$1,500,000 /
3.059] + [$3,000,000 / 3.518]} = $55,285,596.07
Now that the present and future (terminal) values of the cash flows have been determined, the
Modified Internal Rate of Return can take place. The following values are imputed into the Present
Value worksheet on your calculator:
PV =($55,285,596), FV = $203,100,000, N = 10, PMT = $0.00, Compute I. Imputing these values will
yield an answer of 13.896% for the Modified Internal Rate of Return.

Page | 989
Question: 2596

A firm has issued a perpetuity with a total face value of 100 million dollars and a coupon rate of 5.8%.
If the risk free rate equals 5.8% and investors require a rate of return of 10.6% from the perpetuity,
what's the amount the firm raised through the issue?

A. $55.28 million
B. none of these answers
C. $100 million
D. $54.72 million

Answer: D

Explanation:
The price of a perpetuity that pays C per year, at a discount rate of R, equals C/R. Hence, the price of
the perpetuity issue = $(100*5.8%/10.6% ) million = $54.72 million.

Question: 2597

Merryweather, a manufacturer of summer casual wear, has a return on equity of about 10.6%. It
typically pays out about 27% of its earnings as dividends. The firm's stock has a beta of +0.23. The
market has an expected return of 16.2% and the prevailing risk-free rate is 6.9%. Merryweather
recently announced that last year's EPS was $4.3 per share. Given these data, Merryweather's share
price should be:

A. $91.19
B. $84.84
C. $96.07
D. $78.29

Answer: C

Explanation:
The dividend growth rate, g = ROE*(1-payout ratio) = 0.106*(1-0.27) = 7.738%.
The dividend this year was 4.3*0.27 = $1.16. Therefore, expected dividend next year = D1 = (1+g)*Do
= 1.07738*1.16 = $1.25.
The required rate of return on the stock can be found using CAPM, which gives Rstock = Rf +
beta*(Rm - Rf) = 6.9% + 0.23*(16.2% - 6.9%) = 9.039%.
Therefore, Po = D1/(k-g) = 1.25/(9.039% - 7.738%) = $96.07.

Question: 2598

Which of the following statements is correct?

A. It is unrealistic to expect that increases in net working capital that are required at the start of an
expansion project are simply recovered at the project's completion. Thus, these cash flows are
included only at the start of a project.
B. Equipment sold for more than its book value at the end of a project's life will increase income and,
despite increasing taxes, will generate a greater cash flow than if the same asset is sold at book
value.
C. All of these statements are false.
D. An asset that is sold for less than book value at the end of a project's life will generate a loss for
the firm and will cause an actual cash outflow attributable to the project.

Page | 990
E. Only incremental cash flows are relevant in project analysis and the proper incremental cash flows
are the reported accounting profits because they form the true basis for investor and managerial
decisions.

Answer: B

Explanation:
A gain on the sale occurs when equipment is sold for more than its book value. This increases profit
and cash flow.

Question: 2599

Which of the following is/are true about the DOL?


I. The DOL measures the change in EBIT for a given change in the quantity sold.
II. The DOL is zero at the break-even level.
III. The DOL decreases as the level of sales increases.

A. I only
B. II & III
C. I, II & III
D. III only
E. II only

Answer: D

Explanation:
The DOL measures the percentage change in EBIT for a given percentage change in the quantity sold.
At a sales level of Q units, DOL = Q(P-V)/[Q(P-V) - FC] Hence, at the break-even level, the DOL is
infinite but decreases as the sales increase.

Question: 2600

Javier Corporation is considering a project with the following cash flows:


Time Cash Flow
0-$13,000
112,000
28,000
37,000
4-1,500
The firm's cost of capital is 11 percent. What is the project's modified internal rate of return (MIRR)?

A. 21.68%
B. 23.78%
C. 24.90%
D. 25.93%
E. 16.82%

Answer: C

Explanation:
First, find PV of all cash outflows:
PV of CF(0) is -$13,000. PV of CF(4) is -1,500 discounted at 11% for 4 periods or -$988.10. Thus, the
PV of all cash outflows is -$13,988.10.
Second, find the FV at t = 4 of all cash inflows:

Page | 991
The sum of these cash inflows is the project's terminal value. FV of CF(1) at t = 4 is found by entering
N = 3, I = 11, PV = -12,000, and PMT = 0. Then solve for FV = $16,411.57. Similarly, the FVs at t = 4 of
CF(2) and CF(3) are found to be $9,856.80 and $7,770.00, respectively. Thus, the projects TV =
$16,411.57 + $9,856.80 + $7,770.00 = $34,038.37.
To find the MIRR, enter N = 4, PV = -13,988.10, PMT = 0, and FV = 34,038.37, which yields I/YR =
MIRR = 24.90%.

Question: 2601

Elephant Books sells paperback books for $7 each. The variable cost per book is $5. At current annual
sales of 200,000 books, the publisher is just breaking even. It is estimated that if the authors'
royalties are reduced, the variable cost per book will drop by $1. Assume authors' royalties are
reduced and sales remain constant; how much more money can the publisher put into advertising (a
fixed cost) and still break even?

A. $175,225
B. $200,000
C. $600,000
D. $333,333
E. $466,667

Answer: B

Explanation:
$7(200,000) - $5(200,000) - F = 0
F = $400,000.
$7(200,000) - $4(200,000) - F = 0
F = $600,000.
$600,000 - $400,000 = $200,000.

Question: 2602

Which of the following statements is correct?

A. To find a firm's marginal cost of capital for capital budgeting purposes, we would develop an MCC
and an IOS schedule, find the WACC at the intersection of the two curves, and define that WACC to
be the corporate cost of capital. However, this procedure will not lead to a meaningful answer if the
firm uses debt.
B. If a project has only costs (no revenues) as would certain environmental projects, then the project
is likely to have two regular IRRs but only one MIRR.
C. In general, the PVs of riskier cash flows should be found using relatively high discount rates.
However, if a cash flow is non-normal (cash inflows followed by cash outflows, a lower discount rate
should be used to evaluate risky projects.
D. It is better to use the NPV method to evaluate independent projects, but for mutually exclusive
projects, especially if projects vary greatly in size, the MIRR method is better.
E. Firms A and B have identical balance sheets and income statements, pay the same rate of interest,
have the same cost of retained earnings, k(s), and have the same very good set of investment
opportunities. However, Firm A pays out only 20 percent of its earnings versus an 80 percent payout
for Firm B. Firm A will probably have the higher marginal cost of capital schedule.

Answer: C

Explanation:
The other statements are incorrect because of the following: A project that has only costs has no IRR.

Page | 992
The WACC incorporates the cost of all capital: debt, preferred stock, and equity. The NPV method is
best used to evaluate mutually exclusive projects of varying size because the MIRR method may
produce a conflicting result from the NPV in this situation. Firm B will have the higher cost of capital
because it has the higher payout forcing it to go to more costly equity capital before Firm
A.

Question: 2603

Mooradian Corporation estimates that its cost of capital is 11 percent. The company is considering
two mutually exclusive projects whose after-tax cash flows are as follows:
Year Project SProject L
0-$3,000 -$9,000
12,500-1,000
21,5005,000
31,5005,000
4-5005,000
What is the modified internal rate of return (MIRR) of the project with the highest NPV?

A. 18.25%
B. 11.89%
C. 20.12%
D. 16.01%
E. 13.66%

Answer: C

Explanation:
Use cash flow registers to determine the NPV of each project:
NPV(S) = $1,237.11; NPV(L) = $1,106.82.
Since NPV(S) > NPV(L) we need to calculate MIRR(S).
Calculate the PV of cash outflows:
CF(0) = -3,000; CF(1-3) = 0; CF(4) = -500; I = 11. Solve for NPV = $3,329.37.
Calculate the TV of cash inflows:
First find the cumulative PV, then take forward as a lump sum to find the TV.
Solve for NPV = $4,566.47.
Calculate TV or FV: N = 4; I = 11; PV = -4,566.47; PMT = 0.
Solve for FV = $6,932.22.
Calculate MIRR: N = 4; PV = -3,329.37; PMT = 0; FV = 6,932.22.
Solve for MIRR = I = 20.12%.

Question: 2604

Calculate the weighted average cost of capital (WACC) for a firm with the following capital structure:
10% Preferred stock
50% Common equity
40% Debt
Tax rate 40%
Before tax cost of debt 12%
The cost of common equity is 15%
Cost of preferred stock 10%

A. 7.2%
B. 12.33%
C. 11.38%

Page | 993
D. 7.98%
E. 13.3%

Answer: C

Explanation:
A firm's weighted average cost of capital (WACC) = the cost of each component of capital weighted
by the proportion of that component in the firm's capital structure. In this case WACC = (after tax cost
of debt x 40%) + (cost of common equity x 50%) + (cost of preferred stock x 10%) = (12% x 60% x 40%)
+ (15% x 50%) + (10% x 10%) = 2.88% + 7.5% + 1% = 11.38%.

Question: 2605

You have been asked by the president of your company to evaluate the proposed acquisition of a
new special-purpose truck. The truck's basic price is $50,000, and it will cost another $10,000 to
modify it for special use by your firm. The truck falls into the MACRS three-year class, and it will be
sold after three years for $20,000. Use of the truck will require an increase in net working capital
(spare parts inventory) of $2,000. The truck will have no effect on revenues, but it is expected to save
the firm $20,000 per year in before-tax operating costs, mainly labor. The firm's marginal tax rate is
40 percent. What is the net investment in the truck?

A. -$65,000
B. -$50,000
C. -$52,600
D. -$55,800
E. -$62,000

Answer: E

Explanation:
Initial investment:
Cost($50,000)
Modification(10,000)
Change in NWC(2,000)
Total net investment =($62,000)

Question: 2606

Which of the following characteristics is not necessary for the NPV and MIRR calculations to
consistently produce similar results?

A. Projects must have cash flows


B. Projects must have equal lifespans
C. Project must be of equal scale
D. Projects must be of equal size
E. Projects must be independent

Answer: E

Explanation:
When examining mutually-exclusive projects with normal cash flows, the MIRR and NPV methods
will ALWAYS produce similar results as long as the projects being examined are equal in size and have
the same life. It is not necessary for projects to be independent in order for the NPV and MIRR
methods to produce similar results.

Page | 994
Question: 2607

Which of the following events is likely to encourage a corporation to increase its debt ratio?

A. An increase in the personal tax rate.


B. An increase in the expected cost of bankruptcy.
C. Increased uncertainty about the level of sales and output prices.
D. An increase in the corporate tax rate.
E. An increase in the company's degree of operating leverage.

Answer: D

Explanation:
A major reason for using debt is that interest is deductible, which lowers the effective cost of debt.
An increase in the corporate tax rate will increase the tax savings from using debt. An interest
increase in the personal tax rate will make interest income less attractive. An increase in operating
leverage, bankruptcy costs, and uncertainty about sales and output prices will encourage the firm to
decrease financial leverage.

Question: 2608

Of the commonly-employed methods for evaluating capital projects, which of the following offers
the most useful, reliable, and germane results for the financial analyst?

A. Internal Rate of Return


B. Discounted Payback Period
C. Net Present Value
D. Payback Period

Answer: C

Explanation:
In analyzing capital projects, particular weight should be given to Net Present Value (NPV)
calculations, as this method is viewed as the most reliable and realistic of the four major capital
budgeting analysis methods. Net Present Value calculations are superior to Internal Rate of Return
calculations in that NPV works regardless of the size or timing of cash flows, and has a more flexible
incorporation of the appropriate discount rate. Payback period should be viewed as the most inferior
of the four methods, asit does not incorporate the "time-value of money" principle into its
calculation. The Discounted Payback Period is only a slightly improved version of the basic Payback
Period.

Question: 2609

Which of the following statements is most correct?

A. Corporations should fully account for sunk costs when making investment decisions.
B. All of the answers are correct.
C. The rate of depreciation will not affect operating cash flows, because depreciation is not a cash
expense.
D. Corporations should fully account for opportunity costs when making investment decisions.
E. None of the answers are correct.

Answer: D

Page | 995
Explanation:
Cash flow = Net income + depreciation; therefore, depreciation affects operating cash flows. Sunk
costs should be disregarded when making investment decisions, while opportunity costs should be
considered when making investment decisions, as they represent the best alternative use of an
asset.

Question: 2610

Which of the following is false?

A. All of these answers.


B. The IRR and NPV rules do not always give the same project rankings.
C. A project with a higher IRR is always preferable to a project with a lower IRR.
D. Both IRR and NPV rules are based on cash flow discounting.

Answer: C

Explanation:
You should always use the NPV criterion for selecting projects. The IRR method can give project
rankings different from the NPV criterion depending on the type of cash flows of the project as well
as the cost of capital involved. Further, a project with a higher NPV at the project's cost of capital can
have a lower IRR than another project with lower NPV. Therefore, (III) is false.

Question: 2611

Which of the following cannot be eliminated through diversification?


I. Stand-alone risk
II. Unsystematic risk
III. Systematic risk
IV. Market risk
V. Beta risk
VI. Corporate risk
VII. Alpha risk
VIII. Gamma risk

A. I, II, V, VII, VIII


B. I, III, IV, VI, VII, VIII
C. I, II, V, VI
D. II, III, VI
E. III, IV, V

Answer: E

Explanation:
Of the various components of asset risk, only systematic risk cannot be diversified away. Systematic
risk measures that part of asset risk that is inherent regardless of the level of diversification, and is
measured by the Beta coefficient. Systematic risk is also referred to as "market risk" and "beta risk."
Corporate risk is defined as the variability of an asset's expected returns without taking into
consideration the effects of shareholder diversification. This is one step away from Stand-alone Risk,
which measures the risk of an asset, not only without taking into consideration the effect of
shareholder diversification, but of company diversification as well. Stand-alone risk assumes that the
asset in question is the only asset of the firm and that the securities of the firm are the only assets in
investors' portfolios. Corporate risk takes into consideration that firms will diversify their asset bases.

Page | 996
Stand-alone risk is defined as the variability of an asset's expected returns if it were the only asset of
a firm and the stock of that firm was the only security in an investor's portfolio. This type of risk is
definitively reduced through diversification, and is commonly referred to as "unsystematic risk."

Question: 2612

Which of the following firms has the highest degree of financial leverage?
Firm A
EBIT: $1,000,000
Interest Paid: $50,000
Total Operating Expenses: $900,000
Fixed Operating Expenses: $350,000
Firm B
EBIT: $490,000
Interest Paid: $15,000
Total Operating Expenses: $300,000
Fixed Operating Expenses: $180,000
Firm C
EBIT: $1,500,000
Interest Paid: $75,000
Total Operating Expenses: $3,000,000
Fixed Operating Expenses: $2,250,000
Firm D
EBIT: $875,000
Interest Paid: $75,000
Total Operating Expenses: $3,000,000
Fixed Operating Expenses: $2,000,000
Firm E
EBIT: $1,250,000
Interest Paid: $90,000
Total Operating Expenses: $2,900,000
Fixed Operating Expenses: $1,750,000

A. Firm E
B. Firm C
C. Firm D
D. Firm B
E. Firm A

Answer: C

Explanation:
The Degree of Financial Leverage (DFL) measures the percentage change in EPS that results from a
given percentage change in EBIT. Financial Leverage is the second component of total leverage, along
with Operating Leverage. The equation used to calculate the Degree of Financial Leverage is as
follows: {DFL = [EBIT/(EBIT - Interest Paid)]}. In this example, Firm D has the highest DFL, with a figure
of 1.09375. Remember that the Degree of Financial Leverage can never be less than one, and can
never be negative In a situation where the company under examination has zero interest expense,
the DFL would be equal to one, i.e. the EBIT is equal to the EBIT minus the interest expense. Another
important note to remember is that in calculating the Degree of Financial Leverage, dividend
payments to preferred stockholders should be included in the interest expense figure. Operating
expenses are not factored into the DFL calculation, rather are used in the determination of Operating
Leverage.

Page | 997
Question: 2613

Which of the following factors affect a firm's cost of capital?

A. Tax rates
B. Investment Policy
C. All of these answers
D. Dividend Policy
E. The level of interest rates
F. Capital Structure Policy

Answer: C

Explanation:
Each of these factors may affect a firm's cost of capital. As interest rates rise, the cost of debt will also
rise forcing firms to pay a higher rate of interest on debt capital (bonds). Tax rates also affect the cost
of debt. Furthermore, a lower capital gains tax rate relative to ordinary income tax rates will affect
the cost of equity capital relative to the cost of debt capital. Capital structure policy will affect the
weighted average cost of capital (WACC), as well as affect the riskiness of both equity and debt
capital. A change in the level of capital risk will in turn also affect the WACC. Dividend policy
including the level of dividends and stability of dividends will have a direct affect on the cost of
equity capital. Finally, a firm's WACC is affected by its investment policy. The types of investments
that a firm undertakes and the riskiness of those investments are reflected in the WACC.

Question: 2614

The length of time required for an investment's cash flows, discounted at the investment's cost of
capital, to cover its cost is known as ________.

A. Weighted Average Cost of Capital (WACC)


B. Payback Period
C. Discounted Payback Period
D. Net Present Valuing
E. Optimal Capital Structure
F. Capital Budgeting

Answer: C

Explanation:
Discounted Payback Period is defined as the length of time required for an investment's cash flows,
discounted at the investment's cost of capital, to cover its cost.

Question: 2615

In an examination of several capital projects, the management of a large international conglomerate


attempts to calculate the Weighted Average Cost of Capital for the firm. The Company is capitalized
according to the following schedule based on market values: 55% debt
36% common stock
9% perpetual preferred stock
Additionally, assume the following information:
Yield on outstanding debt: 8.95%
Tax rate: 35%
Annual preferred dividend: $0.70
Preferred stock price: $8.90

Page | 998
Return on equity: 17.36%
Dividend payout ratio: 45%
Cost of common stock: 15.10%
Using this information, what is the WACC for this large multinational conglomerate?

A. 10.11%
B. 9.34%
C. None of these answers.
D. 9.29%
E. 9.78%
F. The answer cannot be completely calculated from the information provided.

Answer: B

Explanation:
In order to calculate the WACC, it is necessary to first calculate the component cost of debt, common
equity, and preferred equity. Once the cost of these components is determined, they are imputed
into the WACC equation, which is as follows: {WACC = [(% weight of debt securities * cost of debt) +
(% weight of common stock * cost of common stock) + (% weight of preferred stock * cost of
preferred stock)]}
To calculate the component cost of debt, use the following equation:
{Cost of debt = [yield on outstanding debt securities * (1 - tax rate)}
Factoring in the given information into this equation would yield the following:
{After-tax cost of debt = [8.95% * (1 - 0.35%)]} = 5.818%
To calculate the component cost of outstanding preferred stock, the following equation must be
used:
{Cost of preferred stock = [annual dividend / preferred stock price]}
{Cost of preferred stock - = [$0.70 / $8.90]} = 7.865%.
The final component of the WACC calculation, the cost of common equity, has been provided as
15.10%.
Now that the after-tax costs of debt, preferred stock, and common stock have been determined, the
WACC calculation can be found. The calculation of the WACC is as follows:
{[0.55 * 0.05818] + [0.36 * 0.1510] + [0.09 * 0.07865]} = 9.344%.

Question: 2616

Pierce Products is deciding whether it makes sense to purchase a new piece of equipment. The
equipment costs $100,000 (payable at t = 0). The equipment will provide before-tax cash inflows of
$45,000 a year at the end of each of the next four years (t = 1, 2, 3, 4). The equipment can be
depreciated according to the following schedule:
t = 1: 0.33
t = 2: 0.45
t = 3: 0.15
t = 4: 0.07 At the end of four years the company expects to be able to sell the equipment for a
salvage value of $10,000 (after-tax). The company is in the 40 percent tax bracket. The company has
an after-tax cost of capital of 11 percent. Since there is more uncertainty about the salvage value, the
company has chosen to discount the salvage value at 12 percent. What is the net present value of
purchasing the equipment?

A. $22,853.90
B. $9,140.78
C. $28.982.64
D. $20,564.23
E. $16,498.72That answer is correct!

Page | 999
Answer: A

Explanation:
First, find the after-tax CFs associated with the project. This is accomplished by subtracting the
depreciation expense from the raw CF, reducing this net CF by taxes and then adding back the
depreciation expense.
For t = 1: ($45,000 - $33,000)(1 - 0.4) + $33,000 = $40,200.
Similarly, the after-tax CFs for t = 2, t = 3, and t = 4 are $45,000, $33,000, and $29,800, respectively.
Now, enter these CFs along with the cost of the equipment to find the pre-salvage NPV (note that the
salvage value is not yet accounted for in these CFs). The appropriate discount rate for these CFs is
11%. This yields a pre-salvage NPV of $16,498.72. Finally, the salvage value must be discounted. The
PV of the salvage value is: N = 4, I = 12, PMT = 0, FV = -10,000, and PV = $6,355.18. Adding the PV of
the salvage amount to the pre-salvage NPV yields the project NPV of $22,853.90.

Question: 2617

The most commonly held view of capital structure, according to the text, is that the weighted
average cost of capital ________.

A. increases proportionately with increases in leverage


B. does not change with leverage
C. none of these answers
D. increases with moderate amounts of leverage and then falls
E. first falls with moderate levels of leverage and then increases

Answer: E

Explanation:
The optimal capital structure must strike a balance between risk and return which maximizes the
firm's stock price. Using more debt raises the risk borne by stockholders, however, using more debt
leads to a higher expected rate of return.

Question: 2618

Interstate Transport has a target capital structure of 50 percent debt and 50 percent common equity.
The firm is considering a new independent project which has an IRR of 13 percent and which is not
related to transportation. However, a pure play proxy firm has been identified that is exclusively
engaged in the new line of business. The proxy firm has a beta of 1.38. Both firms have a marginal tax
rate of 40 percent, and Interstate's before-tax cost of debt is 12 percent. The risk-free rate is 10
percent, and the market risk premium is 5 percent. The firm should

A. Be indifferent between accepting or rejecting; the firm's required rate of return on the project
equals its expected return.
B. Accept the project; its IRR exceeds the risk-free rate and the before-tax cost of debt.
C. Accept the project; its IRR is greater than the firm's required rate of return on the project of 12.05
percent.
D. Reject the project; its IRR is less than the firm's required rate of return on the project of 16.9
percent.
E. Reject the project; its IRR is only 13 percent.

Answer: C

Explanation:

Page | 1000
Calculate the required return, k(s), and use to calculate the WACC:
k(s) = 10% + 1.38(5%) = 16.9%.
WACC = 0.5(12.0%)(0.6) + 0.5(16.9%) = 12.05%.
Compare expected project return, to WACC:
Accept the project since IRR (13%) is more than the WACC (12.05%).

Question: 2619

Consider the following information for a company.


Common Stock Price $75.50
Preferred Stock Par Price $100
Preferred Dividend $4.0
Debt Rating BB+
Owners Equity 12.27%
Preferred Stock Flotation Cost 2.0%
The Preferred Stock is issued at Par
Calculate the component cost of this newly issued preferred stock.

A. 6.57%
B. 4.0%
C. 7.0%
D. 3.92%
E. 18.78%
F. 4.08%
G. 12.27%

Answer: F

Explanation:
The component cost of preferred stock is the dividend divided by issue price minus floatation cost. In
this case the component cost of preferred stock = $4.00 / (100 - 2) = 4.08%.

Question: 2620

Which of the following statements is most correct?

A. All these statements are false.


B. The degree of total leverage (DTL) is equal to the DOL plus the degree of financial leverage (DFL).
C. Arithmetically, financial leverage and operating leverage offset one another so as to keep the
degree of total leverage constant. Therefore, the formula shows that the greater the degree of
financial leverage, the smaller the degree of operating leverage.
D. All these statements are true.
E. The degree of operating leverage (DOL) depends on a company's fixed costs, variable costs, and
sales. The DOL formula assumes (1) that fixed costs are constant and (2) that variable costs are a
constant proportion of sales.

Answer: E

Explanation:
DOL is the % change in EBIT that results from a given % change in sales, while DFL is the % change in
EPS that results from a given % change in EBIT. If no debt were used, the DFL would be 1.0.

Question: 2621

Page | 1001
Which of the following statements is most correct?

A. All of these answers are correct.


B. Increasing the amount of debt in a firm's capital structure is likely to increase the cost of both debt
and equity financing.
C. The optimal capital structure maximizes EPS.
D. If the after-tax cost of equity financing exceeds the after-tax cost of debt financing, firms are
always able to reduce their WACC by increasing the amount of debt in their capital structure.
E. The optimal capital structure minimizes the cost of equity.

Answer: B

Explanation:
Increases in the debt ratio increase the cost of both debt and equity. Bondholders recognize that
firms with higher debt levels are more likely to experience financial problems, which will explain why
increases in the debt ratio raise the cost of debt. Concerning the cost of equity, the business risk
premium, in the required rate of return equation, does not depend on the debt level. It will remain
constant at all debt levels. However, the financial risk premium does vary depending on the debt
level; the higher the debt level, the greater the risk premium, and therefore the higher the cost of
equity.

Question: 2622

Delphinium Inc.'s target capital structure has a debt ratio of 60 percent. The firm can raise up to
$100,000 in new debt at a before-tax cost of 8.5 percent. If more than $100,000 of debt is required,
the cost will be 9 percent. Net income last year was $100,000, and it is expected to continue to grow
each year at a rate of 10 percent indefinitely. The firm expects to maintain its dividend payout ratio of
40 percent on the 10,000 shares of common stock outstanding. If it must sell new common stock, it
would encounter a 15 percent flotation cost on the first $400,000, and a 20 percent cost if more than
$400,000 of new outside equity is required. Delphinium's tax rate is 30 percent, and its current stock
price is $88 per share. The firm has an unlimited number of projects, which will earn a 10.25 percent
return. What is this year's capital budget if the firm invests to the point where the Marginal Cost of
Capital (MCC) intersects the Investment Opportunity Schedule (IOS)?

A. The company has an infinite capital budget.


B. $1,000,000
C. $1,165,000
D. $400,000
E. $1,150,000

Answer: C

Explanation:
Step 1 Find the break points for the problem. There will be three break points: one for retained
earnings, one for debt, and one for the change in flotation costs of new stock.
Step 2 Calculate the component costs of capital:
We know NI = $100,000 and there are 10,000 shares, therefore E(0) = $100,000/10,000 = $10. Also,
the payout ratio is 40% so D(0) is $4. D(1) = D(0)(1 + g) = $4(1.1) = $4.40.
k(s) = D1/P0 + g = $4.40/$88 + 10% = 15%.
ke(1) = D1/P0(1 - F1) + g = $4.40/$88(1 - 0.15) + 10% = 15.88%.
ke(2) = D1/P0(1 - F2) + g = $4.40/$88(1 - 0.20) + 10% = 16.25%.
kd(1) = 8.5%(1 - T) = 8.5%(0.7) = 5.95%.
kd(2) = 9%(1 - T) = 9%(0.7) = 6.30%.
Step 3 Calculate the marginal costs of capital:

Page | 1002
MCC(1) = w(ce)k(s) + w(d)k(d1) = 0.4(15.00%) + 0.6(5.95%) = 9.57%.
MCC(2) = w(ce)k(e1) + w(d)k(d1) = 0.4(15.88%) + 0.6(5.95%) = 9.92%.
MCC(3) = w(ce)k(e1) + w(d)k(d2) = 0.4(15.88%) + 0.6(6.30%) = 10.13%.
MCC(4) = w(ce)k(e2) + w(d)k(d2) = 0.4(16.25%) + 0.6(6.30%) = 10.28%.
MCC Applicable Range
1$0 through $165,000
2$165,000 - $166,667
3$166,667 - $1,165,000
4over $1,165,000
Since the IRR of all projects is given as 10.25% the point where the MCC intersects the IOS is
$1,165,000, where the MCC jumps from 10.13% to 10.28%.

Question: 2623

Which of the following statements is most correct?

A. The NPV method assumes that cash flows will be reinvested at the risk-free rate while the IRR
method assumes reinvestment at the IRR.
B. The NPV method assumes that cash flows will be reinvested at the cost of capital while the IRR
method assumes reinvestment at the risk-free rate.
C. The NPV method does not consider the inflation premium.
D. The IRR method does not consider all relevant cash flows, and particularly cash flows beyond the
payback period.
E. The NPV method assumes that cash flows will be reinvested at the cost of capital while the IRR
method assumes reinvestment at the IRR.

Answer: E

Explanation:
The NPV method implicitly assumes that the rate at which cash flows can be reinvested is the cost of
capital, whereas the IRR method assumes that the firm can reinvest at the IRR.

Question: 2624

Byron Corporation's present capital structure, which is also its target capital structure, is 40 percent
debt and 60 percent common equity. Next year's net income is projected to be $21,000, and Byron's
payout ratio is 30 percent. The company's earnings and dividends are growing at a constant rate of 5
percent; the last dividend was $2.00; and the current equilibrium stock price is $21.88. Byron can
raise all the debt financing it needs at 14 percent. If Byron issues new common stock, a 20 percent
flotation cost will be incurred. The firm's marginal tax rate is 40 percent. What is the component cost
of the equity raised by selling new common stock?

A. 12.0%
B. 16.4%
C. 14.6%
D. 15.0%
E. 17.0%

Answer: E

Explanation:
k(e) (component cost of external equity) = ($2.10/$21.88(1-.20)) + 0.05 = 17%.

Question: 2625

Page | 1003
Which of the following is/are true about dividend policies?
I. Under the Bird-in-the-Hand theory, stocks with lower pay-out ratios have higher required rates of
return.
II. Under the Tax Preference theory, stocks with lower pay-out ratios have lower required rates of
return.
III. Under the Modigliani-Miller theory, the price of a stock does not change with a change in the
dividend policy.

A. II only
B. I & II
C. I, II & III
D. I & III
E. I only
F. III only
G. II & III

Answer: D

Explanation:
(II) is not necessarily true when the capital gains tax is higher than realized income tax.

Question: 2626

The management of Allcycles.com, a motorcycle supply chain, is examining several capital projects.
The firm is financed according to the following schedule based on market values:
60% debt
35% common stock
5% perpetual preferred stock
Additionally, consider the following information:
Yield on outstanding debt: 10.12%
Tax rate: 35%
Annual preferred dividend: $0.64
Preferred stock price: $7.36
Return on equity: 18%
Dividend payout ratio: 25%
Cost of common stock: 16.33%
Using this information, what is the Weighted Average Cost of Capital for Allcycles.com?

A. 10.55%
B. The answer cannot be completely calculated from the given information.
C. 9.98%
D. 10.04%
E. None of these answers.
F. 10.10%

Answer: F

Explanation:
In order to calculate the WACC, it is necessary to first calculate the component after-tax cost of debt,
common equity, and preferred equity. Once the cost of these components is determined, they are
imputed into the WACC equation, which is as follows:
{WACC = [(% weight of debt securities * cost of debt) + (% weight of common stock * cost of common
stock) + (% weight of preferred stock * cost of preferred stock)]}

Page | 1004
To calculate the component after-tax cost of debt, use the following equation:
{Cost of debt = [yield on outstanding debt securities * (1 - tax rate)}
Factoring in the given information into this equation would yield the following:
{After-tax cost of debt = [10.12% * (1 - 0.35%)]} = 6.578%
To calculate the component cost of outstanding preferred stock, the following equation must be
used:
{Cost of preferred stock = [annual dividend / preferred stock price]}
{Cost of preferred stock - = [$0.64 / $7.36]} = 8.70%.
The final component of the WACC calculation, the cost of common equity, has been provided as
16.33%.
Now that the after-tax cost of debt, preferred stock, and common stock have been determined, the
WACC calculation can be found. The calculation of the WACC is as follows:
{[0.60 * 0.06578] + [0.35 * 0.1633] + [0.05 * 0.0870]} = 10.10%.

Question: 2627

Which of the following rules are essential to successful cash flow estimates, and ultimately, to
successful capital budgeting?

A. All of the statements are correct.


B. Only incremental cash flows are relevant to the accept/reject decision.
C. The return on invested capital is the only relevant cash flow.
D. None of the statements are correct.
E. Total cash flows are relevant to capital budgeting analysis and the accept/reject decision.

Answer: B

Explanation:
Incremental cash flows are the net cash flows attributable to an investment project and are the only
cash flows relevant in capital budgeting.

Question: 2628

Becker Glass Corporation expects to have earnings before interest and taxes during the coming year
of $1,000,000, and it expects its earnings and dividends to grow indefinitely at a constant annual rate
of 12.5 percent. The firm has $5,000,000 of debt outstanding bearing a coupon interest rate of 8
percent, and it has 100,000 shares of common stock outstanding. Historically, Becker has paid 50
percent of net earnings to common shareholders in the form of dividends. The current price of
Becker's common stock is $40, but it would incur a 10 percent flotation cost if it were to sell new
stock. The firm's tax rate is 40 percent. What is the firm's cost of retained earnings?

A. 15.0%
B. 15.5%
C. 16.5%
D. 16.0%
E. 17.0%

Answer: E

Explanation:
EBIT$1,000,000
Interest400,000
EBT$600,000
Taxes (40%)240,000

Page | 1005
Net income$360,000
EPS(1) = $360,000/100,000 = $3.60.
D(1) = $3.60(0.5)= $1.80.
k(s) = ($1.80/$40.00) + 0.125 = 17.0%.

Question: 2629

Which of the following methods for examining a project's stand-alone risk cannot be effectively
conducted without the use of a random number generator? Choose the best answer.

A. Probability Analysis
B. More than one of these answers is correct
C. Monte Carlo Regression
D. Monte Carlo Simulation
E. Scenario Analysis
F. Sensitivity Analysis

Answer: D

Explanation:
Monte Carlo Simulation seeks to examine the NPV of a project throughout a large series of possible
values for input variables. When measuring stand-alone risk using Monte Carlo Simulation, the use
of a computer and rather complex software package is necessary. In Monte Carlo Simulation, the
expected ranges of input variables are specified, and random numbers are incorporated into the
analysis function, producing a NPV value across a wide range of possible situations. Monte Carlo
Simulation is so named because the technique evolved largely from methods used to analyze
probabilities in casino gambling. Scenario analysis involves the establishment of a "best case" and
"worst case" scenario, which is compared to a predetermined "base case." Sensitivity Analysis seeks
to determine the sensitivity of a project's NPV to changes in specific input variables, and "Probability
Analysis" can be conducted using simple statistical formulas. None of these three methods require
the use of a random number generator.

Question: 2630

Michigan Mattress Company is considering the purchase of land and the construction of a new plant.
The land, which would be bought immediately (at t = 0), has a cost of $100,000 and the building,
which would be erected at the end of the first year (t = 1), would cost $500,000. It is estimated that
the firm's after-tax cash flow will be increased by $100,000 starting at the end of the second year,
and that this incremental flow would increase at a 10 percent rate annually over the next 10 years.
What is the approximate payback period?

A. 4 years
B. 6 years
C. 2 years
D. 10 years
E. 8 years

Answer: B

Explanation:
Payback = 5 + 135.9/146.41 = 5.928 years = 6 years.

Question: 2631

Page | 1006
Doherty Industries wants to invest in a new computer system. The company only wants to invest in
one system, and has narrowed the choice down to System A and System B. System A requires an up-
front cost of $100,000 and then generates positive after-tax cash flows of $60,000 at the end of each
of the next two years. The system can be replaced every two years with the cash inflows and
outflows remaining the same. System B also requires an up-front cost of $100,000 and then
generates positive after-tax cash flows of $48,000 at the end of each of the next three years. System
B can be replaced every three years, but each time the system is replaced, both the cash inflows and
outflows increase by 10 percent. The company needs a computer system for the six years, after
which time the current owners plan on retiring and liquidating the firm. The company's cost of
capital is 11 percent. What is the NPV (on a six-year extended basis) of the system, which creates the
most value to the company?

A. $31,211.52
B. $103,065.82
C. $17,298.30
D. $38,523.43
E. $22,634.77That answer is correct!

Answer: A

Explanation:
To find the NPV of the system we must use the replacement chain approach.
Time System ASystem B
0-100,000-100,000
160,00048,000
260,000 - 100,000 = -40,00048,000
360,00048,000-110,000 =-62,000
460,000 - 100,000 = -40,00052,800
560,00052,800
660,00052,800
Use the CF key to enter the cash flows for each period. I/YR = 11. This should give the following NPVs:
NPV(A) = $6,796.93.
NPV(B) = $31,211.52.
Computer system B creates the most value for the firm.

Question: 2632

Rollins Corporation is constructing its MCC (marginal cost of capital) schedule. Its target capital
structure is 20 percent debt, 20 percent preferred stock, and 60 percent common equity. Its bonds
have a 12percent coupon, paid semiannually, a current maturity of 20 years, and sell for $1,000. The
firm could sell, at par, $100 preferred stock, which pays a 12 percent annual dividend, but flotation
costs of 5 percent would be incurred. Rollins' beta is 1.2, the risk-free rate is 10 percent, and the
market risk premium is 5 percent. Rollins is a constant growth firm, which just paid a dividend of
$2.00, sells for $27.00 per share, and has a growth rate of 8 percent. The firm's policy is to use a risk
premium of 4 percentage points when using the bond-yield-plus-risk-premium method to find k(s)
(component cost of retained earnings). The firm's net income is expected to be $1 million, and its
dividend payout ratio is 40 percent. Flotation costs on new common stock total 10 percent, and the
firm's marginal tax rate is 40 percent. What is Rollins' cost of retained earnings using the CAPM
(Capital Asset Pricing Model) approach?

A. 16.0%
B. 14.1%
C. 16.9%
D. 16.6%

Page | 1007
E. 13.6%That answer is correct!

Answer: A

Explanation:
Cost of retained earnings (CAPM approach): k(s) (component cost of retained earnings) = 10% +
1.2(5%) = 16.0%.

Question: 2633

Copybold Corporation is a start-up firm considering two alternative capital structures--one is


conservative and the other aggressive. The conservative capital structure calls for a D/A ratio = 0.25,
while the aggressive strategy call for D/A = 0.75. Once the firm selects its target capital structure it
envisions two possible scenarios for its operations: Feast or Famine. The Feast scenario has a 60
percent probability of occurring and forecast EBIT in this state is $60,000. The Famine state has a 40
percent chance of occurring and the EBIT is expected to be $20,000. Further, if the firm selects the
conservative capital structure its cost of debt will be 10 percent, while with the aggressive capital
structure its debtcost will be 12 percent. The firm will have $400,000 in total assets, it will face a 40
percent marginal tax rate, and the book value of equity per share under either scenario is $10.00 per
share. What is the difference between the EPS forecasts for Feast and Famine under the conservative
capital structure?

A. $2.20
B. $0.80
C. $0
D. $0.44
E. $1.00

Answer: B

Explanation:
Debt = 25% = $100,000; Equity = 75% = $300,000; Total assets = $400,000.
Feast Famine
Probability0.60.4
EBIT$60,000$20,000
Less: Interest10,00010,000
EBT$50,000$10,000
Less: Taxes20,0004,000
NI$30,000$6,000
# shares30,00030,000
EPS$1.00$0.20
Difference in EPS for conservative capital structure:
EPS(Feast) - EPS(Famine) = $1.00 - $0.20 = $0.80.

Question: 2634

Shannon Industries is considering a project, which has the following cash flows:
Time Cash Flow
0?
1$2,000
23,000
33,000
41,500
The project has a payback of 2.5 years. The firm's cost of capital is 12 percent. What is the project's

Page | 1008
net present value NPV?

A. $3,765.91
B. $765.91
C. $577.68
D. $1,049.80
E. $2,761.32

Answer: B

Explanation:
First, find the missing t = 0 cash flow. If payback = 2.5 years, this implies t = 0 cash flow must be -
$2,000 - $3,000 + (0.5) $3,000 = -$6,500. Enter the cash flows and the firm's cost of capital of 12%,
then NPV = $765.91.

Question: 2635

Which of the following statements is most correct? The modified IRR (MIRR) method:

A. All of these answers are correct.


B. Calculates a return that is always less than the regular IRR.
C. Overcomes the problem of multiple rates of return.
D. Always leads to the same ranking decision as NPV for independent projects.

Answer: C

Explanation:
MIRR assumes that cash flows from all projects are reinvested at the cost of capital, while the regular
IRR assumes that the cash flows from each project are reinvested at the project's own IRR. The MIRR
is a better indicator of profitability because reinvestment at the cost of capital is generally more
correct.

Question: 2636

Which of the following statements is likely to encourage a firm to increase its debt ratio?

A. Management believes that the firm's stock is overvalued.


B. Its corporate tax rate declines.
C. Its sales become less stable over time.
D. None of these answers are correct.
E. All of these answers are correct.

Answer: D

Explanation:
None of these would promote an increase in the debt ratio of a firm. Less stable sales would lead a
firm to lower its debt ratio. A lower corporate tax rate reduces the tax advantage of the deductibility
of interest expense. This reduction in the tax shield provided by debt would encourage less use of
debt. If management believes the firm's stock is overvalued, then it would want to issue equity,
rather than debt.

Question: 2637

The following information applies to a company's preferred stock:

Page | 1009
Current price $48.00 per share
Par value $50.00 per share
Annual dividend $3.50 per share
The company issued the preferred stock at par and incurred a 10% floatation cost. If the company's
marginal corporate tax rate is 34%, what is the after-tax cost of preferred stock at the time of issue?

A. 4.6%
B. 5.1%
C. 7%
D. 7.8%
E. 7.3%
F. 3.5%

Answer: D

Explanation:
The cost of preferred stock is calculated as the preferred stock dividend divided by the net issuing
price. The dividend for this preferred stock is $3.50, and the net issuing price was $45.00. Thus the
cost of preferred stock is 3.5 divided by 45 or 7.8%. There are no tax savings associated with the use
of preferred stock, therefore no tax adjustments are made when calculating the cost.

Question: 2638

Makeover Inc. believes that at its current stock price of $16.00 the firm is undervalued in the market.
Makeover plans to repurchase 2.4 million of its 20 million shares outstanding. The firm's managers
expect that they can repurchase the entire 2.4 million shares at the expected equilibrium price after
repurchase. The firm's current earnings are $44 million. If management's assumptions hold, what is
the expected market price after repurchase?

A. $16.00
B. $17.26
C. $20.00
D. $18.18
E. $24.40

Answer: D

Explanation:
Step 1: Current EPS = $44 million/20 million = $2.20 per share.
Step 2: P/E ratio = $16.00/$2.20 = 7.27x.
Step 3: EPS after repurchase = $44 million/17.6 million = $2.50.
Step 4:Expected market price after repurchase: 7.27 x $2.50 = $18.18 per share.

Question: 2639

If you were to argue that the firm's cost of equity increases as the dividend payout decreases, you
would be making an argument ________ with MM's dividend irrelevance theory, and ________ with
Gordon and Lintner's "bird-in-the-hand" theory.

A. consistent; consistent
B. consistent; inconsistent
C. the argument does not make sense; neither theory involves the cost of equity capital
D. inconsistent; consistent
E. inconsistent; inconsistent

Page | 1010
Answer: D

Explanation:
The main conclusion of MM's irrelevance theory is that dividend policy does not affect the required
rate of return on equity. Gordon-Lintner disagreed stating that k(s) decreases as the dividend payout
is increased because investors are less certain of receiving the capital gains which should result from
retaining earnings than they are of receiving dividends. They said that investors value expected
dividends more highly than expected capital gains because the dividend yield is less risky than the
growth component in the total expected return equation, k(s) = D1/Po + g. MM disagreed and
theorized that k(s) is independent of dividend policy, implying that investors are indifferent between
dividends and capital gains. MM called the Gordon-Lintner's study the bird-in-the- hand fallacy,
because MM thought the riskiness of the firm's cash flows to investors in the long run is determined
by the riskiness of the operating cash flows, not by dividend policy.

Question: 2640

In a recent press release, the management of Intelligent Semiconductor have announced their
intention on a engaging a rather liberal debt offering, which will bring the proportion of debt within
their capital structure from 35% to 50%. According to the Signaling Theory, his decision should be
viewed as which of the following? Choose the best answer.

A. The Signaling Theory would not apply to this announcement.


B. Bullish, because it indicates superior investment prospects for the firm.
C. Bearish, because it is indicative of a shift toward a more liberal capital structure.
D. Bearish, because this will increase the financial risk of the firm.
E. Bullish, because it is indicative of a shift toward a more conservative capital structure.
F. Bearish, because it indicates poor investment prospects for the firm.

Answer: B

Explanation:
According to the Signaling Theory, the management of companies send implicit signals to investors
by their capital budgeting decisions. Believers of this theory feel that corporate managers have
access tosuperior information, and are allowed to exploit this information asymmetry through their
capital budgeting decisions. According to the signaling theory, when investment prospects are good,
companies will prefer to raise capital first by using internally generated funds, i.e. retained earnings
and marketable securities investments. If this source of capital is unavailable, then companies will
prefer to issue debt rather than common or preferred equity. The reasoning behind this is the fact
that by raising debt, the company will not dilute the ROE figure, which is expected to be high due to
favorable investment prospects. In contrast, when investment prospects are poor, the Signaling
Theory states that companies will prefer to raise funds first by issuing common equity. The reasoning
behind this is the fact that by issuing additional equity when investment prospects are poor,
companies will be able to "spread the losses" amongst a greater pool of investors, thereby lessening
the overall negative effect of the poor investment prospects. In this example, the management of
Intelligent Semiconductor has announced its intention on issuing more debt. According to the
Signaling Theory, this should be viewed as an indication that the firm has superior investment
prospects. While the decision to issue more debt is indicative of a shift toward a more liberal capital
structure, and the issuance of this debt will increase the financial risk of the firm, both of which are
somewhat bearish, neither represent the best answer.

Question: 2641

"Operating leverage" refers to:

Page | 1011
A. the degree of reliance on debt capital.
B. none of these answers.
C. the extent to which changes in sales revenues affect operating profits.
D. the extent to which operating profits are affected by variable costs.

Answer: C

Explanation:
Operating leverage is the extent to which changes in sales revenues affect operating profits.

Question: 2642

A decrease in a firm's willingness to pay dividends is likely to result from an increase in its ________.

A. collection of accounts receivable


B. earnings stability
C. access to capital markets
D. stock price
E. profitable investment opportunities

Answer: E

Explanation:
Two points must be kept in mind when deciding how much cash should be distributed to
stockholders:
1. The main objective is to maximize shareholder value.
2. The cash flows generated by the firm belong to its shareholders.
The optimal payout ratio is a function of 4 factors:
1. Investors' preferences for dividends versus capital gains.
2. The firm's investment opportunities.
3. The firm's target capital structure.
4. The availability and cost of external capital.
The last 3 steps are combined in the residual dividend model, which is a model in which the dividend
paid is set equal to the actual earnings minus the amount of retained earnings necessary to finance
the firm's optimal capital budget.

Question: 2643

A company currently sells 75,000 units annually. At this sales level, its EBIT is $4 million, and its
degree of total leverage is 2.0. The firm's debt consists of $15 million in bonds with a 9.5 percent
coupon. The company is considering a new production method which will entail an increase in fixed
costs but adecrease in variable costs, and will result in a degree of operating leverage of 1.6. The
president, who is concerned about the stand-alone risk of the firm, wants to keep the degree of total
leverage at 2.0. If EBIT remains at $4 million, what amount of bonds must be retired to accomplish
this?

A. $9.19 million
B. $8.42 million
C. $6.58 million
D. $4.44 million
E. $7.63 million

Answer: C

Page | 1012
Explanation:
DTL = (DOL)(DFL)
2.0 = 1.6(DFL)
1.25 = DFL.
1.25 = $4,000,000/($4,000,000-I)
$5,000,000 - 1.25(I) = $4,000,000
I = $800,000.
Debt = $800,000/.095 = $8,421,053.
Must retire = $15,000,000 - $8,421,053 = $6.58 million of debt.

Question: 2644

A firm pays an annual preferred dividend of $1.9 per share and investors expect a rate of return of
7.8% from this equity issue. The firm is in the 35% tax bracket. The preferred stock should be trading
at:

A. $37.47
B. $42.19
C. $69.60
D. $24.35

Answer: D

Explanation:
Preferred dividends are not tax-deductible. Hence, no tax adjustment is made while calculating the
cost of preferred equity. The price of a perpetuity that pays C per year, at a discount rate of R, equals
C/R. Hence, P = 1.9/0.078 = $24.35.

Question: 2645

The Price Company will produce 55,000 widgets next year. Variable costs will equal 40 percent of
sales, while fixed costs will total $110,000. At what price must each widget be sold for the company
to achieve an EBIT of $95,000?

A. $5.37
B. $5.00
C. $4.45
D. $2.00
E. $6.21

Answer: E

Explanation:
EBIT = PQ - VQ - FC
$95,000 = P(55,000) - (0.4)P(55,000) - $110,000
$205,000 = (0.6)(55,000)P
$205,000 = 33,000P
P = $6.21.

Question: 2646

Rollins Corporation is constructing its MCC schedule. Its target capital structure is 20 percent debt, 20
percent preferred stock, and 60 percent common equity. Its bonds have a 12 percent coupon, paid

Page | 1013
semiannually, a current maturity of 20 years, and sell for $1,000. The firm could sell, at par, $100
preferred stock, which pays a 12 percent annual dividend, but flotation costs of 5 percent would be
incurred. Rollins' beta is 1.2, the risk-free rate is 10 percent, and the market risk premium is 5
percent. Rollins is a constant growth firm, which just paid a dividend of $2.00, sells for $27.00 per
share, and has a growth rate of 8 percent. The firm's policy is to use a risk premium of 4 percentage
points when using the bond-yield-plus-risk- premium method to find k(s) (component cost of
retained earnings). The firm's net income is expected to be $1 million, and its dividend payout ratio is
40 percent. Flotation costs on new common stock total 10 percent, and the firm's marginal tax rate is
40 percent. What is the firm's cost of retained earnings using the DCF approach?

A. 14.1%
B. 16.9%
C. 16.0%
D. 16.6%
E. 13.6%

Answer: C

Explanation:
Cost of retained earnings (DCF approach): k(s) = $2.16/`$27.00 + 8% = 16.0%.

Question: 2647

Which of the following conditions must be satisfied for a stable dividend policy to result from the
Residual Dividend Policy?
I. The earnings of a firm must be stable.
II. Investor preference for dividends must be stable.
III. The investment opportunities available to the firm must be stable.
IV. There should be no signaling effects involved.

A. III & IV
B. I, III & IV
C. I only
D. I & III
E. I & II
F. III only
G. II only

Answer: D

Explanation:
Under the Residual Dividend Policy, a firm first determines the amount of capital it requires for
sufficiently profitable projects. It then uses retained earnings to supply equity capital and raises debt
in the proper amount to maintain the target capital structure. If any earnings are left over after this,
they are paid out as dividends. If not, the firm will not only not pay any dividends but also issues new
equity for financing. Thus, for this to lead to stable dividends, one must have stability in earnings and
available investment opportunities. Note that for positive dividends, earnings must exceed capital
requirements under the Residual Dividend Policy.

Question: 2648

Ace Consulting, a corporate finance consulting firm, is examining the operating performance and
asset structure of Clay Industries. In their analysis, Ace has identified the following information for
the most recent reporting period:

Page | 1014
EBIT $500,590
Sales $988,000
Interest paid $40,800
Given this information, what is the Degree of Financial Leverage for Clay Industries?

A. 0.567
B. 0.465
C. 1.974
D. None of these answers is correct.
E. The Degree of Financial Leverage cannot be calculated from the information provided.
F. 2.149

Answer: D

Explanation:
To calculate the DFL, the financial analyst needs to determine the EBIT and interest paid for a
predetermined time period. To calculate the Degree of Financial Leverage, the following equation is
used: {EBIT/[EBIT - interest paid]}. Incorporating the given information into this equation yields the
following: {$500,590/[$500,590 - $40,800]}= 1.089. The annual sales figure is not necessary in the
calculation of DFL. Additionally, remember that the DFL figure is always greater than one, therefore
the first and fourth answers can be eliminated as possibilities by observation alone.

Question: 2649

As a general rule, the capital structure that

A. Minimizes the required rate on equity also maximizes the stock price.
B. Maximizes the price per share of common stock also minimizes the weighted average cost of
capital.
C. Maximizes expected EPS also maximize the price per share of common stock.
D. None of these are correct.
E. Minimizes the interest rate on debt also maximizes the expected EPS.

Answer: B

Explanation:
The optimal capital structure is the one that maximizes the price of the firm's stock, and this
generally calls for a debt ratio which is lower than the one that maximizes expected EPS.

Question: 2650

The Congress Company has identified two methods for producing playing cards. One method
involves using a machine having a fixed cost of $10,000 and variable costs of $1.00 per deck of cards.
The other method would use a less expensive machine (fixed cost = $5,000), but it would require
greater variable costs ($1.50 per deck of cards). If the selling price per deck of cards will be the same
under each method, at what level of output will the two methods produce the same net operating
income?

A. 15,000 decks
B. 20,000 decks
C. 25,000 decks
D. 10,000 decks
E. 5,000 decks

Page | 1015
Answer: D

Explanation:
Total cost(Method 1) = $1.00(Q) + $10,000.
Total cost(Method 2) = $1.50(Q) + $5,000.
Set equal and solve for Q: Q + $10,000 = $1.50(Q) + $5,000
$5,000 = $0.5(Q)
10,000 = Q.

Question: 2651

If the calculated NPV is negative, then which of the following must be true? The discount rate used is
________.

A. equal to the internal rate of return


B. too high
C. greater than the internal rate of return
D. too low
E. less than the internal rate of return

Answer: C

Explanation:
If a project has a positive NPV, then it is generating more cash than is needed to service its debt and
to provide the required return to shareholders, and this excess cash accrues solely to the firm
stockholders. On the other hand is a project has a negative NPV, the required rate of return is not
being met and the discount rate used must be greater.

Question: 2652

A financial analyst with Mally, Feasance & Company is examining shares of a large specialty retailer.
Assume the following information:
EPS: $2.30
ROE: 16.25%
Growth rate of dividends: 12.00%
Discount rate: 13.33%
Tax Rate 35%
Using this information, what is the dividend payout ratio for this specialty retailer? Further, what is
the annual dividend?

A. 73.85% $1.70
B. 23.07%, $0.53
C. 26.15%, $0.60
D. 26.15%, $1.70
E. 65.16%, $1.50
F. 73.85%, $0.60

Answer: C

Explanation:
To determine the dividend payout ratio, the equation used to determine the growth rate of dividends
must be manipulated. This equation is originally structured as follows:
{g = ROE (1 - Dividend Payout Ratio)}
In order to determine the Dividend Payout Ratio, the equation must be rearranged to the following:

Page | 1016
{(1 - Dividend Payout Ratio) = Growth Rate of Dividends / ROE}.
Imputing the given information into this equation will yield:
{(1 - Dividend Payout Ratio) = 0.12/0.1625)} = 0.73846
Finally, subtracting this figure from 1 will yield the answer of 26.15%. We must subtract the first
answer from one because the first answer represents the retention rate, i.e. the percentage of
earnings that is retained and reinvested at the firm's ROE. The retention rate and the payout ratio
will always combine to equal positive one. In order to determine the annual dividend, take the
Dividend Payout Ratio, which was found to be 26.15%, and multiply this figure by the Earnings Per
Share calculation, which is given as $2.30. This will yield an annual dividend of $0.60154. As you can
see, neither the discount rate nor the tax rate is factored into the equation.

Question: 2653

Clay Industries, a large industrial firm, is examining its capital structure. The firm is financed
according to the following schedule based on market values:
50% debt
40% common stock
10% perpetual preferred stock
Additionally, consider the following information:
Yield on outstanding debt: 8.50%
Tax rate: 35%
Annual preferred dividend: $2.55
Preferred stock price: $25.97
Return on equity: 16.75%
Dividend payout ratio: 50%
Cost of common stock: 14.25%
Using this information, what is the Weighted Average Cost of Capital for Clay Industries?

A. 8.97%
B. 9.37%
C. 9.45%
D. 9.25%
E. 9.37%
F. None of these answers

Answer: C

Explanation:
In order to calculate the WACC, it is necessary to first calculate the component after-tax cost of debt,
common equity, and preferred equity. Once the cost of these components is determined, they are
imputed into the WACC equation, which is as follows:
{WACC = [(% weight of debt securities * cost of debt) + (% weight of common stock * cost of common
stock) + (% weight of preferred stock * cost of preferred stock)]}
To calculate the component cost of debt, use the following equation:
{After-tax cost of debt = [yield on outstanding debt securities * (1 - tax rate)}
Factoring in the given information into this equation would yield the following:
{After-tax cost of debt = [8.50% * (1 - 0.35%)]} = 5.525%
To calculate the component cost of outstanding preferred stock, the following equation must be
used:
{Cost of preferred stock = [annual dividend / preferred stock price]}
{Cost of preferred stock - = [$2.55 / $25.97]} = 9.82%.
The final component of the WACC calculation, the cost of common equity, has been provided as
14.25%.
Now that the after-tax cost of debt, preferred stock, and common stock have been determined, the

Page | 1017
WACC calculation can be found. The calculation of the WACC is as follows:
{[0.50 * 0.05525] + [0.40 * 0.1425] + [0.10 * 0.0982]} = 9.445%

Question: 2654

Effects of a project on cash flows in other parts of the firm is known as which of the following terms?

A. Cannibalization
B. Sunk Cost
C. Incremental Cash Flow
D. Opportunity Cost
E. Externality

Answer: E

Explanation:
Externalities are defined as effects of a project on cash flows in other parts of the firm

Question: 2655

Which of the following are factors in the optimal dividend payout ratio?
I. Investor's preference for dividends versus capital gains
II. The target capital structure
III. The investment opportunities available to the firm
IV. The cost and availability of external financing
V. Beta Coefficient

A. I, II, III
B. I, III, III, V
C. II, III, IV
D. None of these answers
E. I, II, III, IV
F. I, II, III, IV, V

Answer: E

Explanation:
The optimal payout ratio of a firm represents the ideal amount of earnings that should be distributed
to shareholders as dividends. This figure is comprised of four components, namely: the cost and
availability of external financing, the investment opportunities available to the firm, the firm's target
capital structure, and investor preferences. The Beta coefficient is not expressly incorporated into the
determination of the Optimal Dividend Payout Ratio.

Question: 2656

Interest payments should be ________ the project cash inflows.

A. added to
B. subtracted from
C. ignored while estimating
D. subtracted from or ignored while estimating

Answer: C

Page | 1018
Explanation:
The effects of debt financing are taken into account through the discount rate used to discount the
project cash flows. Hence, interest payments must be ignored while estimating a project's cash flows.

Question: 2657

Which of the following statements is most correct?

A. All of these statements are true.


B. As a firm's debt ratio approaches 100 percent, the after-tax cost of debt, k(d)(1 - T) (after-tax
component cost of debt, where T is the firm's marginal tax rate), will be at its lowest level.
C. All of these statements are false.
D. An increase in the corporate tax rate would lower the weighted average cost of capital for an
average firm, other things held constant.
E. Depreciation-generated funds have a cost equal to the firm's lowest WACC (Weighted Average
Cost of Capital), and hence they have no impact on the MCC (Marginal Cost of Capital) schedule.

Answer: D

Explanation:
Increasing the corporate tax rate would lower the after-tax component cost of debt, thereby
lowering the WACC, with all other things held constant.

Question: 2658

Which of the following methods for measuring "stand-alone" risk is characterized by the formulation
of a "best case" and "worst case" scenario?

A. Monte Carlo Simulation


B. None of these answers
C. Miller and Thorn Simulation
D. Sensitivity Analysis
E. Tributary Leads Analysis
F. Probability Analysis

Answer: B

Explanation:
The answer prompted in this question is "Scenario Analysis." Scenario Analysis is a method of
measuring a project's stand-alone risk. This method is considered as superior to Sensitivity Analysis,
and this is primarily because Scenario Analysis considers a range of possible values for the input
values whereas Sensitivity Analysis considers only the sensitivity of the project's NPV to fluctuations
in the underlying input variable(s). In Scenario Analysis, the financial analyst establishes a "worst
case" and "best case" situation, in addition to the "base case." The base case scenario sets all the
input variables at their most likely values. Often, the values used for the base case scenario are the
current values for input variables or their expected values into the near future. In Scenario Analysis,
the best and worst case scenarios are compared to the base case, and the sensitivity of the project's
NPV is examined. "Tributary Leads Analysis," along with "Miller and Thorn Simulation," are
completely fictitious answers.

Question: 2659

Assume a project has normal cash flows (i.e., the initial cash flow is negative, and all other cash flows
are positive). Which of the following statements is most correct?

Page | 1019
A. All else equal, a project's IRR increases as the cost of capital declines.
B. None of the answers are correct.
C. All of the answers are correct.
D. All else equal, a project's NPV increases as the cost of capital declines.
E. All else equal, a project's MIRR is unaffected by changes in the cost of capital.

Answer: D

Explanation:
Since the present value of each cash flow is discounted at the project's cost of capital, the NPV will
increase as the cost of capital declines, and the NPV will decline as the cost of capital increases. This
relationship is plotted on the net present value profile.

Question: 2660

For a typical firm with a given capital structure, which of the following is correct? (Note: All rates are
after taxes.)

A. None of these answers.


B. k(e) > k(s) > WACC > k(d).
C. WACC > k(e) > k(s) > k(d).
D. k(d) > k(e) > k(s) > WACC.
E. k(s) > k(e) > k(d) > WACC.

Answer: B

Explanation:
k(d) = interest rate on the firm's new debt; k(e) = component cost of external equity, or equity
obtained by issuing new common stock as opposed to retaining earnings; k(s) = component cost of
retainedearnings (or internal equity). Typically, the cost of equity is higher than the cost of retained
earnings because of flotation costs involved in issuing new common stock. The cost of debt is the
lowest because it is the relevant cost of new debt, taking into account the tax deductibility of
interest, and because of this deductibility of interest, it is the lowest component. The WACC is a
weighted average of all the components, so it would be somewhere in the middle.

Question: 2661

The date on which the right to the current dividend no longer accompanies a stock is known as the:

A. Payment Date
B. Declaration Date
C. Expiration Date
D. Ex-Dividend Date
E. Holder-of-Record Date

Answer: D

Explanation:
The Ex-Dividend date is the date on which the right to the current dividend no longer accompanies
the stock. This date is usually four days prior to the holder-of-record date.

Question: 2662

Page | 1020
A firm currently has 3 million dollars worth of 6% debt outstanding. It can currently borrow in the
capital markets at the rate of 7.2%. The firm faces a 40% tax rate. Its marginal after-tax cost of debt is
about:

A. 7.2%
B. 6%
C. 3.6%
D. 4.3%

Answer: D

Explanation:
Since debt interest is tax-deductible, the after-tax cost of debt equals 7.2%*(1-40%) = 4.32%. Note
that bonds issued in the past are of no relevance since it is the current cost of borrowing that is of
concern.

Question: 2663

A company is considering an expansion project. The company's CFO plans to calculate the project's
NPV by discounting the relevant cash flows (which include the initial up-front costs, the operating
cash flows, and the terminal cash flows) at the company's cost of capital (WACC). Which of the
following factors should the CFO include when estimating the relevant cash flows?

A. All of the answers are correct.


B. Any interest expenses associated with the project.
C. None of the answers are correct.
D. Any opportunity costs associated with the project.
E. Any sunk costs associated with the project.

Answer: D

Explanation:
Sunk costs should be excluded from the analysis, and interest expense is incorporated in the WACC
and not the cash flows.

Question: 2664

In the real world, dividends ________.

A. are usually changed every year to reflect earnings changes


B. fluctuate more widely than earnings
C. tend to be a lower percentage of earnings for mature firms
D. are usually set as a fixed percentage of earnings
E. usually exhibit greater stability than earnings

Answer: E

Explanation:
Most firms and stockholders expect earnings to grow over time with dividends growing virtually the
same as earnings. In the past, a "stable dividend policy" meant a company paid the same dollar
dividend for several years in a row, but today it means increasing the dividend at a reasonably steady
rate. From an investor's viewpoint, the most stable policy is that whose dividend growth rate is
predictable. The second most stable policy is where stockholders can reasonably be sure that the
current dividend will not be reduced. The least stable is where earnings and cash flows are so volatile

Page | 1021
that investors cannot count on the company to maintain the current dividend. Since profits and cash
flow vary over time for a firm, one would suggest that firms should vary their dividends over time,
increasing them when cash flows are large and lowering them when cash is low relative to
investment opportunities. However, many stockholders rely on dividends and reducing dividends
may send incorrect signals, which could drive the stock price. Thus, maximizing a firm's stock price
requires a balance of its internal fund requirements against the desires of the stockholders

Question: 2665

Which of the following statements is most correct?

A. None of these answers are correct.


B. All of these answers are correct.
C. Opportunity costs should not be incorporated into capital budgeting decisions.
D. Relevant externalities should be incorporated into capital budgeting decisions.
E. Sunk costs should be incorporated into capital budgeting decisions.

Answer: D

Explanation:
Sunk costs should not be taken into consideration. Opportunity costs should be taken into
consideration.

Question: 2666

Which of the following Companies has the highest degree of financial leverage?
Firm A
EBIT: $1,500,000
Interest Paid: $130,000
Total Operating Expenses: $600,000
Fixed Operating Expenses: $350,000
Firm B
EBIT: $400,000
Interest Paid: $55,000
Total Operating Expenses: $1,300,000
Fixed Operating Expenses: $1,000,000
Firm C
EBIT: $500,000
Interest Paid: $45,000
Total Operating Expenses: $6,000,000
Fixed Operating Expenses: $4,750,000
Firm D
EBIT: $995,000

Interest Paid: $105,000


Total Operating Expenses: $5,000,000
Fixed Operating Expenses: $3,000,000
Firm E
EBIT: $995,000
Interest Paid: $120,000
Total Operating Expenses: $5,900,000
Fixed Operating Expenses: $2,000,000

A. Firm D

Page | 1022
B. Firm A
C. Firm B
D. Firm C
E. Firm E

Answer: C

Explanation:
The Degree of Financial Leverage (DFL) measures the percentage change in EPS that results from a
given percentage change in EBIT. Financial Leverage is the second component of total leverage, along
with Operating Leverage. The equation used to calculate the Degree of Financial Leverage is as
follows: {DFL = [EBIT/(EBIT - Interest Paid)]}.
In this example, Firm B has the highest DFL, with a figure of 1.15942. Remember that the Degree of
Financial Leverage can never be less than one, and can never be negative In a situation where
thecompany under examination has zero interest expense, the DFL would be equal to one, i.e. the
EBIT is equal to the EBIT minus the interest expense. Another important note to remember is that in
calculating the Degree of Financial Leverage, dividend payments to preferred stockholders should be
included in the interest expense figure. Operating expenses are not factored into the DFL calculation,
rather are used in the determination of Operating Leverage.

Question: 2667

A financial analyst with Smith, Kleen & Beetchnutty is examining shares of a publicly-traded specialty
brewer. Assume the following information:
EPS: $1.83
ROE: 19.00%
Growth rate of dividends: 10.85%
Discount rate: 12.50%
Tax Rate 35%
Using this information, what is the dividend payout ratio for this specialty brewer? Further, what is
the annual dividend?

A. 50.76%, $0.93
B. The answer cannot be determined from the information provided.
C. 42.90%, $0.79
D. 38.13%, $0.70
E. 57.11%, $1.05
F. 42.90%, $1.05

Answer: C

Explanation:
To determine the dividend payout ratio, the equation used to determine the growth rate of dividends
must be manipulated. This equation is originally structured as follows:
{g = ROE (1 - Dividend Payout Ratio)}
In order to determine the Dividend Payout Ratio, the equation must be rearranged to the following:
{(1 - Dividend Payout Ratio) = Growth Rate of Dividends / ROE}.
Imputing the given information into this equation will yield:
{(1 - Dividend Payout Ratio) = 0.1085/0.19)} = 0.57105
Finally, subtracting this answer from 1 will yield the answer of 42.90% for the dividend payout ratio.
(Remember that the original answer, 0.57105, is the retention rate, not the dividend payout ratio).
In order to determine the annual dividend, take the Dividend Payout Ratio, which was found to be
42.90%, and multiply this figure by the Earnings Per Share calculation, which is given as $1.83. This
will yield an annual dividend of $0.785

Page | 1023
Question: 2668

Which of the following methods of measuring a project's stand-alone risk is characterized by using a
computer to analyze a project's NPV across a wide range of values for numerous input variables?
(Hint: This style of analysis evolved from methods used to examine probabilities in casino gambling.)

A. Sensitivity Analysis
B. Monte Carlo Simulation
C. Scenario Analysis
D. Probability Analysis
E. None of these answers

Answer: B

Explanation:
When measuring stand-alone risk using Monte Carlo Simulation, the use of a computer and a rather
complex software package is necessary. In Monte Carlo Simulation, the expected ranges of
inputvariables are specified, and random numbers are incorporated into the analysis function,
producing a NPV value across a wide range of possible situations. Monte Carlo Simulation is so
named because the technique evolved largely from methods used to analyze probabilities in casino
gambling. Scenario analysis involves the establishment of a "best case" and "worst case" scenario,
which is compared to a predetermined "base case." Sensitivity Analysis seeks to determine the
sensitivity of a project's NPV to changes in specific input variables, and "Probability Analysis" can be
conducted using simple statistical formulas. None of these three methods absolutely require the use
of a computer or complex statistical software package.

Question: 2669

A project requires an initial outlay of 650. It also needs capital spending of 700 at the end of year 1
and 900 at the end of year 2. It has no revenues for the first 2 years but receives 1,200 in year 3,
1,600 in year 4 and 2,300 in year 5. The project's payback period equals ________.

A. 4.54 years
B. 2.26 years
C. 3.66 years
D. 4.91 years

Answer: D

Explanation:
The cash flows of the project starting at the end of year 1 are:
-700, -900, +1,200, +1,600, +2,300
The payback period is defined as the expected number of years that would be required to recover the
original investment. In particular, Payback period = Years before full recovery + (unrecovered cost at
the start of payback year)/(net cash flow in the payback year) The net account balance goes positive
in the 4th year. At the beginning of the 4th year, the outstanding balance equals 650+700+900-1,200
= $1,050. Therefore, payback period = 3 + 1,050/1,600 = 3.66 years.

Question: 2670

Which of the following are substantive purposes for conducting post-audit procedures in capital
budgeting situations? Choose the best answer.
I. Improving forecasts

Page | 1024
II. Shifts in the Security Market Line
III. Improving operations
IV. Controlling management
V. Adhering to Bond Covenants
VII. Ensuring adherence to governmental guidelines for performance presentation

A. I, III, IV
B. I, II, III, IV, VII, VIII
C. I, III, IV, VIII
D. I, III
E. II, IV, VIII
F. I, V, VII, VIII

Answer: D

Explanation:
An important aspect of the capital budgeting process is the post-audit, which involves comparing
actual results with those forecasted and determining why any discrepancies exist. The post-audit
process has two main purposes. First, the post-audit process is used to improve the forecasting
capacity of the firm. Second, this process is used to improve the operations of the firm. "Shifts in the
Security Market Line" has little to do with the post-audit process. Additionally, remember that aside
from AIMR PPS and other suggested performance presentation guidelines, there are no real
"guidelines for research and performance presentation." There are certainly no "governmental
guidelines" for performance presentation; therefore, answer VII is incorrect.

Question: 2671

Which of the following is/are true?


I. The IRR method assumes that future cash flows are reinvested at the project's cost of capital.
II. The NPV method assumes that future cash flows are reinvested at the project's cost of capital.
III. The MIRR method assumes that future cash flows are reinvested at the project's cost of capital.
IV. MIRR and NPV methods always lead to the same decisions for projects of similar sizes.

A. II, III & IV


B. I only
C. II & III
D. II only
E. I, II, III & IV
F. III only
G. II & IV
H. IV onlyThat answer is correct!

Answer: A

Explanation:
The IRR method assumes that future cash flows are reinvested at the internal rate of return of the
project, not the project's cost of capital. Statements (II), (III) and (IV) are true.

Question: 2672

Which of the following terms describes what happens when the introduction of a new product
causes the sales of existing products to decrease?

A. Opportunity Cost

Page | 1025
B. Sunk Cost
C. Externality
D. Incremental Cash Flow
E. Cannibalization

Answer: E

Explanation:
When the introduction of a new product causes the sales of existing products to decrease, it is called
cannibalization.

Question: 2673

Which of the following statements is most correct?

A. If a company's stock price increases, this increases its cost of equity capital.
B. The before-tax cost of preferred stock may be lower than the before-tax cost of debt, even though
preferred stock is riskier than debt.
C. None of these statements are correct.
D. If the cost of equity capital is low enough, it may be cheaper to issue common stock than it is to
finance projects with retained earnings.
E. All of these statements are correct.

Answer: B

Explanation:
Corporations, which receive a 70 percent exclusion of dividend income from their taxable income,
own most preferred stock. Consequently, the before-tax coupons on preferred stock may be lower
than the before-tax coupons on debt, despite the fact that preferred stock is riskier than debt. All the
other statements are false.

Question: 2674

Musgrave Corporation has fixed costs of $46,000 and variable costs that are 30 percent of the current
sales price of $2.15. At a price of $2.15, Musgrave sells 40,000 units. Musgrave can increase sales by
10,000 units by cutting its unit price from $2.15 to $1.95, but variable cost per unit won't change.
Should it cut its price?

A. No, EBIT decreases by $250.


B. No, EBIT decreases by $6,000.
C. Yes, EBIT increases by $8,050.
D. Yes, EBIT increases by $11,500.
E. Yes, EBIT increases by $5,050.

Answer: C

Explanation:
Calculate EBIT(1) at 40,000 units using the current sales price:
EBIT(1) = S - VC - FC = 40,000($2.15) - 0.30(40,000)($2.15) - $46,000 = $86,000 - $25,800 - $46,000 =
$14,200.
Calculate EBIT(2) at 50,000 units using the lower price of $1.95:
EBIT(2) = 50,000($1.95) - 0.30(50,000)($1.95) - $46,000 = $97,500 - $29,250 - $46,000 = $22,250.
The change in EBIT = $22,250 - $14,200 = +$8,050.
Yes, Musgrave should cut its price, EBIT increases by $8,050.

Page | 1026
Question: 2675

Consider the following information:


30 day T-Bill rate (Risk free rate) 8.0%
Common stock Beta 1.2
Expected rate of return for the market 12.0%
Asset turnover ratio 3.4x
Calculate this firm's cost of retained earnings using the CAPM approach.

A. 43.52%
B. 12.8%
C. 9.6%
D. 8.0%
E. 27.2%
F. 12.0%

Answer: B

Explanation:
To calculate the cost of retained earnings for a firm using CAPM, one may use the following formula:
Cost of retained earnings = risk free rate + [(expected rate of return on the market - risk free rate) x
Beta]. In this case, the cost of retained earnings = 8.0% + [(12.0% - 8.0%) x 1.2] = 12.8%.

Question: 2676

Assume that a firm currently has EBIT of $2,000,000, a degree of total leverage of 7.5, and a degree
of financial leverage of 1.875. If sales decline by 20 percent next year, then what will be the firm's
expected EBIT in one year?

A. $1,600,000
B. $3,600,000
C. $1,350,000
D. $400,000
E. $2,400,000

Answer: D

Explanation:
DOL = DTL/DFL = 7.5/1.875 = 4.0.
Change EBIT = (-0.20)(4.0)($2,000,000) = -$1,600,000.
EBIT = $2,000,000 - $1,600,000 = $400,000.

Question: 2677

Coats Corp. generates $10,000,000 in sales. Its variable costs equal 85 percent of sales and its fixed
costs are $500,000. Therefore, the company's operating income (EBIT) equals $1,000,000. The
company estimates that if its sales were to increase 10 percent, its net income and EPS would
increase 17.5 percent. What is the company's interest expense? (Assume that the change in sales
would have no effect on the company's tax rate.)

A. $142,857
B. $100,000
C. $857,142

Page | 1027
D. $105,874
E. $111,584That answer is correct!

Answer: A

Explanation:
Recall that DTL = % change in NI/% change in sales = 0.175/0.10 = 1.75.
DTL = (S - VC)/(S - VC - FC - I)
1.75 = ($10,000,000 - $8,500,000)/($10,000,000 - $8,500,000 - $500,000 - I)
1.75 = $1,500,000/($1,000,000 - I)
$1,500,000 = $1,750,000 - 1.75I
I = $142,857.14.

Question: 2678

The Bird-in-the-Hand theory implies that as the dividend pay-out ratio is increased, the stock price:

A. remains unaffected.
B. decreases.
C. increases.
D. increases or decreases.

Answer: C

Explanation:
The Bird-in-the-Hand theory implies that as the dividend pay-out ratio is increased, the stock price
increases.

Question: 2679

A fundamental analyst for Smith, Kleen, & Beetchnutty is examining the financial information of
Ludicrous Bubble Semiconductor to determine whether it is an appropriate investment for a hedge
fund. In his analysis, the fundamental analyst has identified the following financial information:
Sales $5,900,000
Total fixed cost $2,300,000
Total variable cost $1,665,000
Interest expense $75,750
EBIT $1,885,000
Amortization expense $47,550
Given this information, what is the Degree of Total Leverage for Ludicrous Bubble Semiconductor?

A. 2.338
B. 3.544
C. 2.311
D. 2.278
E. 2.565

Answer: D

Explanation:
The Degree of Total Leverage (DTL) demonstrates how a given change in sales will impact a firm's
EPS. The equation used for calculating the DTL is as follows: {[Sales - variable costs] / [sales - variable
costs - fixed costs - interest expense]}. Incorporating the given values for these components into the
DTL equation yields the following: {[Sales $5,900,000 - variable costs $1,665,000] / [sales $5,900,000

Page | 1028
- variable costs $1,665,000 - fixed costs $2,300,000 - interest expense $75,750]}=2.278. "EBIT" and
"amortization expense" are not explicitly incorporated into the DTL equation.

Question: 2680

Given the following choices, what is the optimal capital structure for Chip Co.? (Assume that the
company's growth rate is 2 percent.)

Debt Ratio
Dividends
Per Share ($)
Cost of Equity
0%5.5011.5%
256.0012.0
406.5013.0
507.0014.0
757.5015.0

A. 50% debt; 50% equity


B. 40% debt; 60% equity
C. 25% debt; 75% equity
D. 75% debt; 25% equity
E. 0% debt; 100% equity

Answer: C

Explanation:
First, calculate the stock price for each debt level using the dividend growth model, Po = D1/(ks - g).
Debt Div/share ksPo
0%$5.5011.5% $5.50/(0.115 - 0.02) = $57.89.
25%6.0012%$6.00/(0.12 - 0.02) = $60.00.
40%6.5013%$6.50/(0.13 - 0.02) = $59.09.
50%7.0014%$7.00/(0.14 - 0.02) = $58.33.
75%7.5015%$7.50/(0.15 - 0.02) = $57.69.
Clearly, $60.00 is the highest price, so 25% debt and 75% equity is the optimal capital structure.

Question: 2681

The date on which a company actually distributes a dividend is known as the:

A. Ex-Dividend Date
B. Holder-of-Record Date
C. Declaration Date
D. Expiration Date
E. Payment Date

Answer: E

Explanation:
The "Payment Date" is the date that a company actually mails the dividend checks.

Question: 2682

Phoenix Products Inc. requires a new machine to produce a part for a solar air conditioner. Two

Page | 1029
companies have submitted bids, and you have been assigned the task of choosing one of the
machines. Cash flow analysis indicates the following:
Year Machine AMachine B
0-$1,000-$1,000
10417
20417
30417
41,938417
If the cost of capital for Phoenix Products is 5 percent, which of the following is the most valid
statement?

A. The IRR(A) > IRR(B), therefore accept Machine A.


B. The NPV(A) < NPV(B), therefore accept Machine B.
C. Take neither A nor B since the cost of capital is greater than the internal rate of return.
D. The NPV(A) > NPV(B), therefore accept Machine A.
E. The IRR(A) < IRR(B), therefore accept Machine B.

Answer: D

Explanation:
NPV(A) = $1,938(PVIF(5%,4)) - $1,000
= $1,938(0.8227) - $1,000 = $1,594.39 - $1,000 = $594.39.
NPV(B0 = $417(PVIFA(5%,4)) - $1,000
= $417(3.5460) - $1,000 = $1,478.68 - $1,000 = $478.68.
Therefore, accept Machine A since NPV(A) > NPV(B).

Question: 2683

A project with normal (or conventional) cash flows has a single IRR of 10%. If a project's hurdle rate is
8%, the project NPV:

A. is positive.
B. could be all of these answers.
C. equals zero.
D. is negative.That answer is correct!

Answer: A

Explanation:
"Normal cash flows" implies that there is an up front cost when the project is set up but all cash flows
after that are cash inflows. For normal cash flows, the NPV is positive for discount rates lower than
the IRR and negative for discount rates higher than the IRR. Since the project's hurdle rate is less than
the IRR, the project's NPV is positive. Note: if there are cash outflows after the project is put into
operation, one can get multiple IRRs.

Question: 2684

A firm has fixed costs of $13,000, variable costs of $15 and sale price per unit of $22. The firm has an
interest expense of $800. The degree of financial leverage of the firm at an output level of 2,000
units is:

A. 4.2
B. 2.3
C. 4.0

Page | 1030
D. 5.0

Answer: D

Explanation:
DFL = EBIT/(EBIT - I). Also, EBIT = Q(P-V) - FC. Therefore, EBIT = 2000*(22-15) - 13,000 = 1,000 and
DFL = 1000/(1000 - 800) = 5.

Question: 2685

Normal projects C and D are mutually exclusive. Project C has a higher net present value if the WACC
is less than 12 percent, whereas Project D has a higher net present value if the WACC exceeds 12
percent. Which of the following statements is most correct?

A. All of the statements are incorrect.


B. Project D has a higher internal rate of return.
C. All of the statements are correct.
D. Project D is probably larger in scale than Project C.
E. Project C probably has a faster payback.

Answer: B

Explanation:
From the information given, D has the higher IRR. The project's scale cannot be determined from the
information given. As C's NPV declines more rapidly with an increase in rates, this implies that more
of the cash flows are coming later on. So C would have a slower payback than D.

Question: 2686

The use of financial leverage by the firm has a potential impact on which of the following?
1. The risk associated with the firm.
2. The return experienced by the shareholder.
3. The variability of net income.
4. The degree of operating leverage.
5. The degree of financial leverage.

A. 1, 3, 5
B. 1, 2, 5
C. 2, 3, 5
D. 2, 3, 4, 5
E. 1, 2, 3, 5

Answer: E

Explanation:
Since financial leverage is the extent to which fixed-income securities are used in a firm's capital
structure all of these would impact except the degree of operating leverage, which is the percentage
change in EBIT that results from a given percentage change in sales.

Question: 2687

A company estimates that its weighted average cost of capital (WACC) is 10 percent. Which of the
following independent projects should the company accept?

Page | 1031
A. Project C requires an up-front expenditure of $1,000,000 and generates a positive internal rate of
return of 9.7 percent.
B. Project D has an internal rate of return of 9.5 percent.
C. None of the projects should be accepted.
D. Project B has a modified internal rate of return of 9.5 percent.
E. Project A requires an up-front expenditure of $1,000,000 and generates a net present value of
$3,200.

Answer: E

Explanation:
This is the only project with either a positive NPV or an IRR which exceeds the cost of capital.

Question: 2688

The following information applies to Lott Enterprises:


Operating Income (EBIT)$300,000
Debt$100,000
Interest Expense$10,000
Tax Rate40%
Shares Outstanding120,000
EPS$1.45
Stock Price$17.40
The company is considering a recapitalization where it would issue $348,000 worth of new debt and
use the proceeds to buyback $348,000 worth of common stock. The buyback will be undertaken at
the pre-recapitalization share price ($17.40). The recapitalization is not expected to have an effect on
operating income or the tax rate. After the recapitalization, the company's interest expense will be
$50,000. Assume that the recapitalization has no effect on the company's price earnings ratio. What
is the expected price of the company's stock following the recapitalization?

A. $15.30
B. $19.03
C. $20.48
D. $18.00
E. $17.75

Answer: D

Explanation:
We can do this problem by using the P/E before and after the recap. Recall that P/E = Price/EPS.

Before the recap


After recap
EBIT$300,000$300,000
Interest-10,000-50,000
EBT$290,000$250,000
Tax116,000100,000
NI$174,000$150,000
Shares120,000100,000*
EPS$174,000/120,000$150,000/100,000
= $1.45= $1.50
P/E$17.40/1.45 = 12x
*120,000 - ($348,000/$17.40)
As P/E = 12 after the recapitalization (recall the question states that it does not change), we know 12

Page | 1032
= Price/$1.50 Price = 12 x $1.50 = $18.00.

Question: 2689

Projects A and B both have normal (conventional) cash flows. A's IRR is 7% and B's IRR is 8%. If
projects A and B are mutually exclusive, you should select:

A. Project B.
B. Neither A not B.
C. Insufficient information.
D. Project A.

Answer: C

Explanation:
If you were to use the IRR rule, you would select project B if the project's cost of capital were greater
than 8%. However, you should always use the NPV criterion for selecting projects. Even though B has
a higher IRR, it could have a lower NPV at the project's cost of capital. Since this information is
missing, you cannot say for sure whether project A is preferable or project B. Indeed, you don't even
know if either of the projects has a positive NPV!

Question: 2690

Project A has an IRR of 10% and project B has an IRR of 12%. The crossover rate for these projects is
7.4%. You use the NPV rule for making project selections. If both the projects have a cost of capital of
6.9%, and are mutually exclusive with normal cash flows, you should:

A. insufficient information.
B. select project A.
C. select neither A nor B.
D. select project B.

Answer: B

Explanation:
The crossover rate is the discount rate at which the graphs of NPV versus discount rate for the two
projects cross. Since the projects have normal cash flows, they will have a single crossover rate.
Further, the project with the higher IRR has a "flatter" NPV profile. Therefore, if the cost of capital is
smaller than the crossover rate, the project with the flatter profile will have a smaller NPV. Therefore,
project A has a higher NPV at the cost of capital of 6.9% and should be selected over project B.

Question: 2691

Genuine Products Inc. requires a new machine. Two companies have submitted bids, and you have
been assigned the task of choosing one of the machines. Cash flow analysis indicates the following:
Year Machine AMachine B
0-$2,000-$2,000
10832
20832
30832
43,877832
What is the internal rate of return for each machine?

A. IRR(A) = 16%; IRR(B) = 20%

Page | 1033
B. IRR(A) = 24%; IRR(B) = 20%
C. IRR(A) = 18%; IRR(B) = 24%
D. IRR(A) = 18%; IRR(B) = 16%
E. IRR(A) = 24%; IRR(B) = 26%

Answer: C

Explanation:
Solve for numerical PVIF and PVIFA then obtain corresponding interest rate from table Machine A
$2,000 = $3,877(PVIF(IrrA,4))
0.51586 = PVIFA(IrrA,4)
IRR(A) = 18%.
Machine B $2,000 = $832(PVIFA(IrrB,4))
2.40385 = PVIFA(IrrB,4)
IRR(B) = 24%.

Question: 2692

The Target (optimal) Capital Structure will be the mix of debt, preferred stock, and common equity
that will accomplish which of the following items?

A. Ensures that all debt and preferred dividend payments are met
B. Maximize the firm's profits
C. Provides funding for any project that a firm wishes to undertake
D. Provide the lowest debt cost
E. Maximize the firm's stock price
F. Provide the smallest chance of bankruptcy

Answer: E

Explanation:
The Target (Optimal) Capital Structure is defined as the percentages of debt, preferred stock, and
common equity that will maximize the firm's stock price.

Question: 2693

Which of the following constitutes an example of a cost which is not incremental, and therefore not
relevant in an accept/reject decision?

A. A firm has a parcel of land that can be used for a new plant site or, alternatively, can be used to
grow watermelons.
B. All of these are examples of incremental cash flows, and therefore, relevant cash flows.
C. A firm orders and receives a piece of new equipment, which is shipped across the country and
requires $25,000 in installation and set-up costs.
D. A firm can produce a new cleaning product that will generate new sales, but some of the new
sales will be from customers who switch from another product the company currently produces.
E. All of these are not examples of incremental cash flows.

Answer: B

Explanation:
These are examples of opportunity costs, externalities or installation costs and are all incremental
cash flows.

Page | 1034
Question: 2694

Van Slyke Inc. has $5,000,000 in assets, and currently has no debt--it is financed entirely with 200,000
shares of common stock, each of which trades at $25 per share. The firm's EBIT is expected to be
$1,250,000 at year-end (i.e., at t=1). The corporate tax rate is 40 percent. Van Slyke expects to pay
out a dividend at year-end which is 50 percent of its net income. The company estimates that its
earnings and dividends grow at a constant rate of 3 percent a year. The company is considering a
recapitalization where they would issue $1,000,000 of debt at a before- tax cost of 10 percent. The
proceeds from thedebt issued would be used to repurchase shares of the company's stock at $25 per
share. The company's investment bankers estimate that the cost of equity capital would be 16
percent after the recapitalization. What would you expect the company's stock price to be
immediately following the recapitalization? Assume that the dividend has not yet been paid.

A. $27.25
B. $33.17
C. $12.15
D. $16.59
E. $20.98

Answer: D

Explanation:
Step 1 Find EPS:
EBIT$1,250,000
Interest expense100,000 = 0.10($1,000,000)
Earnings before taxes$1,150,000
Taxes (40%)460,000
Net income$690,000
Shares purchased = $1,000,000/$25 = 40,000.
Shares remaining = 200,000 - 40,000 = 160,000.
EPS = $690,000/160,000 = $4.3125.
Step 2 Find D1:
EPS(1)P/o = D1
$4.3125 (0.50) = $2.1563.
Step 3 Find Po:
ks = D1/Po + g
0.16 = $2.1563/Po + 0.03
Po = $16.59.

Question: 2695

Which of the following is/are true?


I. A project's sunk costs are irrelevant to the decision of accepting or rejecting it.
II. A project's incremental cash flows are not affected by interest expenses.
III. Project rankings using incremental net income and incremental net cash flows can be different.

A. II only
B. I & II
C. II & III
D. I, II & III
E. I & III
F. I only
G. III only

Page | 1035
Answer: D

Explanation:
Incremental cash flows of a project are the cash flows that occur if and only if the project is
undertaken. Since the effects of debt financing are taken into account through the discount rate used
to discount the project cash flows, interest payments are ignored while estimating the project's cash
flows. Therefore, a project's incremental cash flows are not affected by interest expense. Sunk costs
represent expenses that have already been incurred or committed to. Therefore, they should not be
allowed to affect future decisions. Finally, since income includes non-cash items, the discounting of
income numbers can distort the project rankings based on cash flows. In capital budgeting, annual
cash flows, not accounting income, are used to evaluate a project.

Question: 2696

Which of the following actions will enable a company to raise additional equity capital (that is, which
of the following will raise the total book value of equity)?

A. A stock repurchase.
B. The establishment of a new-stock dividend reinvestment plan.
C. A stock split.
D. All of these answers are correct.
E. The establishment of an open-market purchase dividend reinvestment plan.

Answer: B

Explanation:
The new stock type of dividend reinvestment plan invests the dividends in newly issued stock, hence
these plans raise new capital for the firm.

Question: 2697

Externalities:
I. are spill-over effects of a project.
II. are not necessarily harmful and can actually be beneficial.
III. should be ignored in project evaluation, just as sunk costs are.

A. I, II & III
B. II & III
C. I & II
D. I & III
E. I only
F. III only
G. II only

Answer: C

Explanation:
You should remember three things about externalities:
1. They are spill-over effects from the project under consideration and affect the rest of the firm.
2. They are not necessarily harmful. Many of them can be beneficial ("positive externalities").
3. Unlike sunk costs, which have no future cash flow effects, externalities cannot be ignored in
project evaluation since their effects on the cash flows are incremental cash flows themselves.

Question: 2698

Page | 1036
Which of the following is a key benefit of using the degree of leverage concept in financial analysis?

A. It shows how a given change in leverage will affect sales.


B. None of these statements are correct.
C. It establishes the optimal capital structure for the firm.
D. It allows decision-makers a relatively clear assessment of the consequences of alternative actions.
E. All of these statements are correct.

Answer: D

Explanation:
The degree of leverage concept is useful primarily for the insights it provides regarding the joint
effects of operating and financial leverage on EPS. DOL concepts provide alternatives to decision-
makers, giving a better idea of the ramifications of alternative actions.

Question: 2699

Copybold Corporation is a start-up firm considering two alternative capital structures--one is


conservative and the other aggressive. The conservative capital structure calls for a D/A ratio = 0.25,
while the aggressive strategy call for D/A = 0.75. Once the firm selects its target capital structure it
envisions two possible scenarios for its operations: Feast or Famine. The Feast scenario has a 60
percent probability of occurring and forecast EBIT in this state is $60,000. The Famine state has a 40
percent chance of occurring and the EBIT is expected to be $20,000. Further, if the firm selects the
conservative capital structure its cost of debt will be 10 percent, while with the aggressive capital
structure its debt cost will be 12 percent. The firm will have $400,000 in total assets, it will face a 40
percent marginal tax rate, and the book value of equity per share under either scenario is $10.00 per
share. What is the coefficient of variation of expected EPS under the conservative capital structure
plan?

A. 0.58
B. 0.15
C. 0.23
D. 0.39
E. 1.00That answer is correct!

Answer: A

Explanation:
Calculate coefficient of variation
Expected EPS conservative:
E(EPS) = 0.6($1.00) + 0.4($0.20) = $0.68.
Standard deviation
SD(EPS-Conservative) = [0.6($1.00 - $0.68)^2 + 0.4($0.20 - $0.68)^2]^1/2 = [0.0614 + 0.0922]^1/2 =
0.3919.
CV(Conservative) = 0.3919/0.68 = 0.576.

Question: 2700

Which of the following statements is most correct?

A. An increase in the cost of equity capital when a company announces an increase in its dividend per
share, would be consistent with the bird-in-the-hand theory.
B. All of these statements are correct.

Page | 1037
C. The tax preference theory states that, all else equal, investors prefer stocks that pay low dividends
because retained earnings can lead to capital gains that are taxed at a lower rate.
D. A dividend policy that involves paying a consistent percentage of net income is the best policy if
the "clientele effect" is correct.
E. An increase in the stock price when a company decreases its dividend is consistent with the
signaling theory.

Answer: C

Explanation:
The reasons investors might prefer a low dividend payout to a high payout are:
1. Long-term capital gains are taxed at a maximum 28% rate, whereas dividend income is taxed at
rates up to 39.6%. Obviously, high tax-bracket investors might prefer to have firms plow back
earnings into the firm, presumably leading to a higher stock price, and resulting in a lower-taxed
capital gain.
2. Capital gain taxes are not paid until the stock is sold and taxes paid in the future has a lower
effective cost than taxes paid today.
3. If stock is held until one dies, no capital gains tax is due because the cost basis of the stock of the
beneficiaries who inherit the stock, is the stock's value on the death day, thus escaping any capital
gains tax.

Question: 2701

A firm is considering undertaking a project requiring $7 million of new capital. The managers of the
firm consider the prospects of the project having a 36% rate of return extremely likely, with a small
probability that the project will not recover anything invested. The firm's current debt ratio is 70%
and market analysts have estimated that the tax benefits from an increase in the debt level will be
far smaller than the increase in bankruptcy costs. The Signaling Theory implies that the firm will try
to raise _______ capital. The Trade-off Theory implies that the firm will try to raise _______ capital.

A. equity; equity
B. equity; debt
C. debt; debt
D. debt; equity

Answer: D

Explanation:
Since additional debt does not create more benefits from tax shield than the losses anticipated due
to increased bankruptcy costs, the Trade-off theory implies that the firm will try to raise equity
capital for the project. On the other hand, since the prospects of the project are extremely favorable,
Signaling Theory says that the firm would rather raise the capital through debt so that the profits
from the project are shared on an equity basis only with the current shareholders.

Question: 2702

Assume the following information for Bearstone Concrete and Manufacturing, Inc.
EPS: $6.25
ROE: 10.16%
Growth rate of dividends: 5.25%
Discount rate: 12.80%
Tax Rate 35%
Common shares outstanding 2,000,000
Using this information, what is the retention rate for this firm? Further, what is the annual dividend?

Page | 1038
A. 48.33%, $3.02
B. 51.67%, $3.23
C. 45.81%, $2.86
D. The answer cannot be determined from the information provided.
E. 48.33%, $3.23
F. 51.67%, $3.02

Answer: F

Explanation:
To determine the retention rate of dividends, the equation used to determine the growth rate of
dividends must be manipulated. This equation is originally structured as follows: {g = ROE (1 -
Dividend Payout Ratio)}
In order to determine the retention rate, the equation must be rearranged to the following: {(1 -
Dividend Payout Ratio) = Growth Rate of Dividends / ROE}.
Imputing the given information into this equation will yield as follows: {(1 - Dividend Payout Ratio) =
0.0525/0.1016)} = 0.51673.
Remember that the retention rate is equal to (1 - Dividend Payout Ratio). Therefore, no further
calculation is necessary to determine the retention rate.
In order to determine the annual dividend, take the Dividend Payout Ratio, which is found by (1 -
Retention Rate), and multiply this figure by the Earnings Per Share calculation, which is given as
$6.25. This will yield an annual dividend of $3.02.
As you can see, neither the discount rate, tax rate, nor the number of common shares outstanding is
factored into the equation.

Question: 2703

Which of the following would not have an influence on the optimal dividend policy?

A. The costs associated with selling new common stock.


B. All of these statements can have an effect on dividend policy.
C. Bond indenture constraints.
D. A strong shareholders' preference for current income versus capital gains.
E. The possibility of accelerating or delaying investment projects.

Answer: B

Explanation:
The optimal payout ratio is a function of 4 factors:
1. Investors' preferences for dividends versus capital gains.
2. The firm's investment opportunities.
3. The firm's target capital structure.
4. The availability and cost of external capital.

Question: 2704

The management of Clay Industries have adhered to the following capital structure: 50% debt, 45%
common equity, and 5% perpetual preferred equity. The following information applies to the firm:
Before-tax cost of debt = 9.5%
Combined state/federal tax rate = 35%
Expected return on the market = 14.5%
Annual risk-free rate of return = 6.25%
Historical Beta coefficient of Clay Industries Common Stock = 1.24 Expected annual preferred

Page | 1039
dividend = $1.55
Preferred stock net offering price = $24.50
Annual common dividend = $0.80
Common stock price = $30.90
Expected growth rate = 9.75%
Subjective risk premium = 3.3%
Given this information, and using the Discounted Cash Flow (DCF) approach, what is the Weighted
Average Cost of Capital for Clay Industries?

A. The WACC for Clay Industries cannot be calculated from the information.
B. 8.96%
C. 13.05%
D. 12.34%
E. 7.70%
F. 9.97%

Answer: B

Explanation:
The calculation of the Weighted Average Cost of Capital is as follows: {fraction of debt * [yield to
maturity on outstanding long-term debt][1-combined state/federal income tax rate]} + {fraction of
preferred stock * [annual dividend/net offering price]} + {fraction of common stock * cost of equity}.
The cost of common equity can be calculated using three methods, the Capital Asset Pricing Model
(CAPM), the Dividend-Yield-plus-Growth-Rate (or Discounted Cash Flow) approach, and the Bond-
Yield-plus-Risk-Premium approach. In this example, you are asked to calculate the cost of common
equity using the Discounted Cash Flow, or Dividend-Yield-plus-Growth-Rate approach. To calculate
the cost of equityusing this approach, take the expected annual dividend on common equity ($0.80)
divided by the market price of common stock ($30.90), and add the expected growth rate (9.75%) to
this figure. Using this method, the cost of common equity is found to be 12.34%. The after-tax cost of
debt can be found by multiplying the yield to maturity on the firm's outstanding long-term debt
(9.5%) by (1-tax rate). Using this method, the after-tax cost of debt is found as 6.175%. The
calculation of the cost of perpetual preferred stock is relatively straightforward, simply divide the
annual preferred dividend by the net offering price. Using this method, the cost of preferred stock is
found as 6.327%. Incorporating these figures into the WACC equation gives the answer of 8.957%.

Question: 2705

Project A has an IRR of 12% and project B has an IRR of 9%. The crossover rate for these projects is
6.9%. You use the NPV rule for making project selections. If both the projects have a cost of capital of
10.4% and the projects are independent with normal cash flows, you should:

A. select neither A nor B.


B. select project B.
C. insufficient information.
D. select project A.

Answer: D

Explanation:
Note that the crossover rate is unimportant here since the projects are independent, not mutually
exclusive. Hence, you should select all projects with positive NPV. In this case, since A has an IRR
greater than the cost of capital and it has normal cash flows, it has a positive NPV. Project B's IRR
exceeds the cost of capital and hence has negative NPV. Therefore, you should select A and reject B.

Page | 1040
Question: 2706

Consider the following information for Company XYZ:


30 day T-Bill rate (Risk free rate) 5.2%
Common Stock Beta 1.1
Expected Rate of return for the market 12.0%
Debt Credit Rating BBB
Calculate this firm's cost of retained earnings using the CAPM approach.

A. 5.72%
B. 17.2%
C. 10.2%
D. 12.0%
E. 12.68%
F. 5.2%

Answer: E

Explanation:
To calculate the cost of retained earnings for a firm using CAPM, one may use the following formula:
Cost of retained earnings = risk free rate + ((expected rate of return on the market - risk free rate) x
Beta). In this case the cost of retained earnings = 5.2% + ((12.0% - 5.2%) x 1.1 = 12.68%.

Question: 2707

Capitol City Transfer Company is considering building a new terminal in Salt Lake City. If the company
goes ahead with the project, it must spend $1 million immediately (at t = 0) and another $1 million at
the end of Year 1 (t = 1). It will then receive net cash flows of $0.5 million at the end of Years 2 - 5,
and it expects to sell the property and net $1 million at the end of Year 6. All cash inflows and
outflows are after taxes. The company's cost of capital is 12 percent, and it uses the modified IRR
criterion for capital budgeting decisions. What is the project's modified IRR?

A. 11.5%
B. 11.9%
C. 11.4%
D. 12.0%
E. 11.7%

Answer: E

Explanation:
Time line: (In millions)
k = 12%
MIRR = ?
0123456 Yrs
-1-1.5.5.5.51.0
Tabular/Numerical solution:
PV(Outflows) = -$1,000,000 - ($1,000,000/1.12) = $1,892,857.
TV(Inflows) = $500,000(FVIFA(12%,4))(FVIF(12%,1)) + $1,000,000
= $500,000(4.7793)(1.12) + $1,000,000 = $3,676.408.
$1,892,857 = $3,676,408/(PVIF(MIrr,6)0
PVIF(MIrr,6) = 1.94225 (Take 6th root of both sides)
1 + MIRR = 1.11699
MIRR = 11.699%.

Page | 1041
Question: 2708

According to the "Tax Preference Theory," which factor(s) would lead investors to desire a lower
payout of dividends over a relatively higher payout of dividends?
I. Capital gains may be taxed at a lower marginal rate than ordinary income
II. The cost of retained earning equity capital is usually lower than debt capital
III. Capital gains are not taxed until the stock is sold and the gain is realized
IV. If the stock is held until the owner dies, the beneficiary may use the stock price at the time of
inheritance as the basis, thus any capital gains up until that point are not taxed
V. Investors prefer a stable dividend policy

A. None of these answers


B. I only
C. I, III and IV
D. I, II, III, IV & V
E. II and III

Answer: C

Explanation:
The "Tax Preference Theory" states that there may be three reasons that investors would prefer
lower dividend payments along with higher capital gains as opposed to high dividend payments.
Long term capital gains may be taxed at a lower marginal rate than ordinary income (dividends).
Also, investors have more control over when the taxable event occurs with capital gains. They are not
taxed until the stock is sold and the gain is realized. Finally, if the stock is held until death, the
beneficiary may claim the value at that time as the basis, thus avoiding taxes on any gains that
previously accrued.

Question: 2709

Smith Company has no retained earnings. The company uses the CAPM to calculate the cost of
equity capital. The company's capital structure consists of common stock, preferred stock, and debt.
Which of the following events will reduce the company's WACC?

A. An increase in the flotation costs associated with issuing preferred stock.


B. An increase in the flotation costs associated with issuing common equity.
C. A reduction in the market risk premium.
D. An increase in the company's beta.
E. An increase in expected inflation.

Answer: C

Explanation:
If the risk premium decreases, the required return on common equity will be reduced. All of the
other answers will increase the firm's WACC.

Question: 2710

A sudden and unexpected decrease in the tax rate would be expected to have the most significant
impact on the carrying cost of which of the following components of a firm's capital structure?

A. Retained earnings
B. Common stock

Page | 1042
C. None of these answers
D. Preferred stock
E. Debt

Answer: E

Explanation:
After a sudden and unexpected decrease in the tax rate, corporations will find themselves in an
environment where the tax-shelter benefits of debt financing have become compromised. Currently,
firms can deduct coupon payments on their debt securities from their annual income, and this
taxdeductibility feature is an important consideration in any capital budgeting decision. A decrease in
the tax rate will increase the cost of debt, and therefore will decrease the attractiveness of debt
securities as an alternative to other methods of financing.

Question: 2711

Your firm's EPS last year was $1.00. You expect sales to increase by 15 percent during the coming
year. If your firm has a degree of operating leverage equal to 1.25 and a degree of financial leverage
equal to 3.50, then what is its expected EPS?

A. $1.6563
B. $2.5843
C. $2.2427
D. $1.3481
E. $1.9813That answer is correct!

Answer: A

Explanation:
EPS(0) = $1.00. DOL = 1.25. EPS(1) = ?
%Change S = 15%. DFL = 3.50.
DTL = DOL(DFL) = 1.25(3.50) = 4.375.
EPS(1) = EPS(0)[1.0 + (DTL)(%Change Sales)] = $1.00[1.0 + (4.375)(0.15)] = $1.00[1.6563] = $1.6563.

Question: 2712

Gulf Electric Company (GEC) uses only debt and equity in its capital structure. It can borrow
unlimited amounts at an interest rate of 10 percent so long as it finances at its target capital
structure, which calls for 55 percent debt and 45 percent common equity. Its last dividend was $2.20;
its expected constant growth rate is 6 percent; its stock sells on the NYSE at a price of $35; and new
stock would net the company $30 per share after flotation costs. GEC's tax rate is 40 percent, and it
expects to have $100 million of retained earnings this year. GEC has two projects available: Project A
has a cost of $200 million and a rate of return of 13 percent, while Project B has a cost of $125 million
and a rate of return of 10 percent. All of the company's potential projects are equally risky. Assume
now that GEC needs to raise $300 million in new capital. What is GEC's marginal cost of capital for
evaluating the $300 million in capital projects and any others that might arise during the year?

A. 12.66%
B. 6.00%
C. 9.50%
D. 13.77%
E. 9.00%

Answer: C

Page | 1043
Explanation:
k(d) (interest rate on the firm's new debt) = 10%;
k(d)(1 - T) (after-tax component cost of debt) = 10%(0.6) = 6%.
D/A = 55%
D(0) = $2.20
g = 6%
P(0) = $35
P(N) = $35
T (The firm's marginal tax rate) = 40%
Retained earnings = $100M; BP(RE) = $100M/ .45 = $222.22M
k(s) (component cost of retained earnings) = $2.33/$35 + 6% = 12.66%
k(e) (component cost of external equity) = $2.33/$30 + 6% = 13.77%.
WACC (Weighted Average Cost of Capital) (1) = 0.55(6%) + 0.45(12.66%) = 9.0% WACC(2) = 0.55(6%)
+ 0.45(13.77%) = 9.5%

Question: 2713

Assume that all the assumptions of Modigliani and Miller hold. In particular, there are no taxes and
transaction costs. A firm has a policy of paying out 8% of the stock price as dividends. However, an
investor would like to receive only a 4% dividend. For this, he should:

A. none of these answers.


B. liquidate 8% of his stock holding after receiving the dividend.
C. liquidate 4% of his stock holding after receiving the dividend.
D. use half of the dividend amount to buy stock after receiving the dividend.

Answer: D

Explanation:
Suppose the investor is holding stocks worth $100. The company then pays $8 as dividends. To
reduce his dividend income to $4, the investor must buy stocks worth $4.

Question: 2714

Which of the following equations correctly illustrates the calculation of the cost of perpetual
preferred equity?

A. None of these examples


B. Net offering price/(required rate of return) + expected growth
C. Offering price/(expected rate of return- required rate of return) + expected growth
D. Annual dividend/(offering price + flotation costs)
E. (Annual dividend/current preferred stock price) + expected growth rate
F. Annual dividend/(offering price - flotation costs)

Answer: F

Explanation:
The cost of perpetual preferred equity can be found by dividing the annual dividend by the net
offering price, or gross offering price less flotation costs. Of the six answers listed, only #2 represents
a recognized financial equation; the rest are largely fictitious.

Question: 2715

Page | 1044
Which of the following choices correctly describes an investment in which the cash flows from an
existing project must be considered along with the expected cash flows of a proposed project?

A. Expansion project
B. Retrenchment project
C. Replacement project
D. Non-normal project
E. Marginal project

Answer: C

Explanation:
In an analysis of a replacement project, the cash flows associated with the existing project must be
examined along with the expected cash flows from a new project. Expansion project is defined as one
for which the firm in question does not have an existing proxy, or a project which will expand the
operations of the Company into a new market or functional niche. Expansion projects are for the
expansion of revenues. "Marginal project" and "retrenchment project," are fictitious terms.

Question: 2716

Consider the following information:


Borrowing Rate 10%
Marginal Tax Rate 40%
Preferred Stock Par Price $100
Preferred Dividend $10
Preferred Stock floatation cost 2.5%
Cost of common equity 12.0%
Preferred Stock issued at Par
The Optimal Capital Structure is 45% debt, 50% common equity, and 5% preferred stock. Credit
Rating BB+
What is the firm's Weighted Average Cost of Capital (WACC)?

A. 7.21%
B. 9.21%
C. 9.0%
D. 8.0%
E. 2.5%
F. 28.00%

Answer: B

Explanation:
The firm's Weighted Average Cost of Capital (WACC) is a weighted average of the component cost of
capital. In this case 10%(borrowing rate) x (1-.4)Tax savings = 6% is the component cost of debt. $10
(preferred dividend) / 97.5(Par minus floatation cost) = 10.25% is the component cost of preferred
stock. Thus the WACC = .45(6%) + .5(12%) + .05(10.25%) = 9.21%

Question: 2717

Firm A has a higher operating leverage than firm B. All else equal, firm A has a _______ business risk
and a ________ variability of ROE.

A. lower, higher
B. higher, lower

Page | 1045
C. higher, higher
D. lower, lower

Answer: C

Explanation:
"Operating leverage" refers to the extent to which changes in sales revenues affect operating profits.
The greater the leverage, the higher the business risk and the variability of ROE.

Question: 2718

Strategic Systems Inc. expects to have net income of $800,000 during the next year. Its target, and
current, capital structure is 40 percent debt and 60 percent common equity. The Director of Capital
Budgeting has determined that the optimal capital budget for next year is $1.2 million. If Strategic
uses the residual dividend model to determine next year's dividend payout, what is the expected
dividend payout ratio?

A. 0%
B. 56%
C. 28%
D. 42%
E. 10%

Answer: E

Explanation:
Equity requirement = 0.6($1,200,000) = $720,000.
Expected NI$800,000
Equity requirement720,000
Available for dividends$80,000
Payout ratio = $80,000/$800,000 = 0.10 = 10%.

Question: 2719

If a company uses the same discount rate for evaluating all projects, which of the following results is
likely?

A. Accepting no projects.
B. Accepting poor, high-risk projects.
C. Accepting only poor, high-risk projects.
D. Accepting only good, low risk projects.
E. Accepting all projects.

Answer: B

Explanation:
Under the risk-adjusted discount rate, differential project risk is dealt with by changing the discount
rate. High-risk projects are discounted at a higher cost of capital; lower-risk projects are discounted
at a rate below the firm's average cost of capital, etc. This approach incorporates project risk into
capital budgeting.

Question: 2720

In the absence of bankruptcy and agency costs and signaling actions, tax deductibility of interest

Page | 1046
payments implies that the optimal debt ratio should be:

A. none of these answers


B. 50%
C. 100%
D. zero

Answer: C

Explanation:
If there are no costs to increased debt, such as bankruptcy costs, and no signaling effects in a firm's
decisions about the capital structure, then a firm would try to maximize the benefits of the debt
shield i.e. the deductibility of interest payments. This implies a debt ratio of 100%.

Question: 2721

Which of the following methods of estimating the cost of common equity for a firm treats risk
explicitly?

A. Composite method.
B. Bond-yield-plus-risk-premium method.
C. CAPM and Bond-yield-plus-risk-premium methods.
D. DCF (Discounted Cash Flow) method and CAPM (Capital Asset Pricing Model) methods.
E. CAPM method.

Answer: C

Explanation:
In the CAPM approach, the stock's beta coefficient must be estimated, which is used as an index of
the stock's risk. In the bond-yield-plus-risk-premium method the risk premium is judgmentally added
to the interest rate on the firm's own long-term debt.

Question: 2722

The Residual Dividend Policy leads to:

A. the firm paying out a constant fraction of earnings as dividends.


B. the firm maintaining a constant absolute amount of dividends.
C. the firm maintaining a steady growth of dividends.
D. a highly unstable dividend policy.

Answer: D

Explanation:
Under the Residual Dividend Policy, a firm first determines the amount of capital it requires for
sufficiently profitable projects. It then uses retained earnings to supply equity capital and raises debt
in the proper amount to maintain the target capital structure. If any earnings are left over after this,
they are paid out as dividends. If not, the firm will not only not pay any dividends but also issues new
equity for financing. Thus, the amount of dividends paid out can swing wildly from one year to the
next, based on the current business conditions and future growth prospects.

Question: 2723

Business risk is concerned with the operations of the firm. Which of the following is not associated

Page | 1047
with (or not a part of) business risk?

A. The ability to change prices as costs change.


B. Changes in required returns due to financing decisions.
C. The extent to which operating costs are fixed.
D. Demand variability.
E. Sales price variability.

Answer: B

Explanation:
Business risk depends on:
(1) unit sales variability,
(2) sales price variability,
(3) input price variability,
(4) ability to adjust output prices for changes in input prices and,
(5) the extent to which costs are fixed (operating leverage).

Question: 2724

Holding all else equal, an decrease in which of the following will cause an increase in the theoretical
growth rate of common stock dividends according to the Growth Rate of Dividends Model?
I. ROE
II. Payout ratio
III. Tax rate IV Preferred stock price
V. Discount rate
VI. Retention rate

A. II, V
B. None of these answers
C. II, IV
D. III, II
E. I, III, IV
F. V, VI

Answer: B

Explanation:
The equation used to determine the theoretical growth rate of common stock dividends is as follows:
{Annual growth rate = [ROE * (1 - dividend payout ratio)]}
As you can see, of the choices listed, only a decrease in the dividend payout ratio will cause an
increase in the theoretical growth rate of common stock dividends.

Question: 2725

Ecodevelopment Company has two mutually exclusive construction projects to evaluate. Each type of
project can be duplicated repeatedly in different geographic locations around the country, but each
requires a very different set of assets to construct. Thus, Ecodevelopment uses the equivalent annual
annuity method to evaluate such projects. Project type A costs $6 million initially and generates
expected end-of-year cash flows of $3 million, $5 million, and then $10 million when it is sold after 3
years. Project type B costs $9 million initially and has projected end-of-year cash flows of $3, $3, $6,
and $6 million in Year 1 through Year 4, and then $10 million in Year 5. The project types are equally
risky and the firm's cost of capital is 12 percent. What is the EAA of the higher valued project type?

Page | 1048
A. $2.726 million
B. $4.389 million
C. $3.240 million
D. $6.000 million
E. $5.600 million

Answer: C

Explanation:
Calculate each project's original life NPV:
NPV(A) = $3.0(PVIF(12%,1)) + $5.0(PVIF(12%,2)) + $10.0(PVIF(12%,3)) - $6.0
= $3.0(0.8929) + $5.0(0.7972) + $10.0(0.7118) - $6.0
= $2.6787 + $3.9860 + $7.1180 - $6.0
= $7.7827 million = $7,782,700.
NPV(B) = $3.0(PVIFA(12%,2)) + $6.0(PVIFA(12%,2))(PVIF(12%,2))
+ $10.0(PVIF(12%,5)) - $9.0
= $3.0(1.6901) + $6.0(1.6901)(0.7972) + $10.0(0.5674) - $9.0 =
$5.0703 + $8.0841 + $5.6740 - $9.0
= $9.8284 million = $9,828,400.
Calculate the equivalent annual annuity:
NPV(A) = $7,782,700 = EAA(A)(PVIFA(12%,3))
EAA(A) = $7,782,700/2.4018 = $3,240,361.40.
NPV(B) = $9,828,400 = EAA(B)(PVIFA(12%,5))
EAA(B) = $9,828,400/3.6048 = $2,726,475.81.

Question: 2726

A firm's reliance on debt ________ as bankruptcy costs increase. The debt ratio _______ as the
probability of default increases.

A. increases; decreases.
B. decreases; decreases.
C. increases; increases.
D. decreases; increases.

Answer: B

Explanation:
Expected bankruptcy costs are a deterrent to high debt levels. These costs increase if the probability
of default increases or the costs associated with the bankruptcy increase. In either of these cases, a
firm will reduce its reliance on debt.

Question: 2727

A consulting firm is currently under contract with a Busy Bus and Van Lines, Inc., and has agreed to
formulate various financial reports and trend projections for the Company. During the most recent
month, the consulting firm has been able to determine the following; Busy Bus and Van Lines
currently pays out 30% of its net income as dividends, and this rate is expected to remain stable.
Additionally, Busy has maintained a steady ROE of 15% for the last ten years, and this is also expected
to remain stable. The risk-free rate of return is 5.35%, and the firm is in a 35% combined
state/federal income tax rate. Finally, Busy Bus and Van Lines has informed the consulting firm that
its shareholders require a 12.5% or greater rate of return, and the firm's common stock is priced at
$9.65. Using the Retention Growth Rate method, which of the following most closely resembles the
growth rate of Busy Bus and Van Lines, Inc.?

Page | 1049
A. 12.5%
B. 184%
C. 19.5%
D. The growth rate of this firm cannot be determined from the information provided.
E. 15.85%
F. 10.5%

Answer: F

Explanation:
When security analysts estimate growth rates, the Retention Growth Rate method is often the
method employed. In estimating growth using this model, the retention rate (defined as the
percentage of net income that is retained within the firm and not distributed as dividends), is
multiplied by the figure for ROE. The calculation of the answer in this example is as follows: {[1 -
payout ratio 30%]
* ROE 15%}= 10.5%. This method is most useful when the growth rate or the components of the
equation are expected to remain stable.

Question: 2728

Consider the following characteristics of a firm:


Stock price $60
Annual dividend $1
Debt rate 12%
Equity floatation cost 5%
Tax rate 40%
Preferred Stock Par value $100
What is the firm's after tax cost of debt?

A. 1.8%
B. 5%
C. 7.2%
D. 12%
E. 1.7%
F. 60%

Answer: C

Explanation:
A firm's after tax cost of debt may be calculated using the following formula: After Tax Cost of Debt =
Cost of Debt x (1 - Tax Rate). In this case the After Tax Cost of Debt = 12% x (1 - 40%) = 12% x 60% =
7.2%.

Topic 6, Portfolio Management

Question: 2729

Milra Choudra is considering how to invest her $100,000 investment portfolio- Choudra's investment
advisor has recommended that she invest 60% in the S&P 500 stock market index and 40% in the
risk-free asset. The advisor has derived the following forecasts for the S&P 500:

Page | 1050
Assuming a risk-free rate of 5%, the expected return on Choudra's portfolio is closest to:

A. 6.6%.
B. 7.4%.
C. 9.0%.

Answer: B

Question: 2730

Carl Vandenberg has been asked to explain the security market line (SML), including its slope. Which
of the following is equal to the slope of the SML?

A. Beta.
B. Alpha.
C. Market risk premium.

Answer: C

Question: 2731

An analyst is interested in the relationship between the stock prices of two companies. After
downloading a time series of stock prices for each company, the analyst concludes that Company A
has a variance equal to 0.25 and Company B has a variance equal to 0.20, and the covariance
between the two stocks is -0.10. Calculate the correlation coefficient between the two stocks.

A. -0.45
B. -0.50
C. -1.00.

Answer: A

Question: 2732

The standard deviation of a two-stock portfolio least likely:

A. must be less than or equal to the weighted-average standard deviation.


B. can be reduced by increasing the relative weight of the stock with lower standard deviation.
C. will be the lowest when the correlation between the two stocks equals zero.

Answer: C

Question: 2733

Page | 1051
An analyst predicts that the return on Royal Company stock will be 15%. The analyst is provided with
the following data for Royal and the broad market:
Royal Company beta 1.5
Risk-free rate 5%
Expected market return 11 %
From the data, determine if Royal Company stock is undervalued, overvalued, or correctly valued.

A. Overvalued.
B. Undervalued.
C. Correctly valued.

Answer: B

Question: 2734

Consider two individuals, David Lywie and Julio Stromek who have requested advice on their
investment policy statements. Lywie is mid-career, has very little insurance coverage, and has fairly
low job-related income expectations. In contrast, Stromek has fairly high net worth and long-term
job-related earning potential. Which of the following statements is most likely correct?

A. Lywie has higher risk tolerance than Stromek.


B. Stromek has higher risk tolerance than Lywie.
C. Stromek and Lywie have equivalent levels of risk tolerance.

Answer: B

Question: 2735

Danielle Paftee, age 55, has an investment account designed to ftind her granddaughter's college
education. Paftee's granddaughter is two years old. Paftee also will use the account for intermittent
health care expenses for her elderly parents, whose health plans and retirement plans do not
adequately cover their expenses. Which of the following statements regarding Paftee's investment
objectives and constraints is least likely correct?

A. Paftee has a long-term time horizon.


B. Paftee has an significant liquidity requirement.
C. Paftee should focus on total return with very low current income requirements.

Answer: C

Question: 2736

Steve McCool is estimating the expected return and standard deviation of his equity portfolio. Steve
has estimated a 20% chance that the portfolio will provide an 8% rate of return, a 40% chance that
theportfolio will provide a 10% return, and a 40% chance that the portfolio will provide a 12% rate of
return. Calculate the standard deviation of McCool's portfolio.

A. 0.00022
B. 0.01497
C. 0.02240

Answer: B

Page | 1052
Question: 2737

Chuck Hill, CFA, is explaining an efficient frontier analysis to one of his clients. Which of Hill's
following statements regarding the efficient frontier is correct?

A. The left endpoint of the efficient frontier is represented by the portfolio with the lowest level of
risk.
B. Portfolios that are further to the right on the efficient frontier dominate portfolios that are to the
left.
C. Only efficient assets are on the efficient frontier.

Answer: A

Question: 2738

Adding the risk-free asset to a portfolio of risky assets will:

A. decrease portfolio standard deviation because it is uncorrelated with risky assets


B. not affect portfolio standard deviation because it is uncorrelated with risky assets.
C. decrease portfolio standard deviation due to its negative correlation with risky assets

Answer: A

Question: 2739

Bruce Johansen, CFA, is currently fully invested in the market portfolio that lies on the capita! market
line (CML). Johansen desires to increase the expected return from his portfolio. Johansen is risk
aversebut willing to accept higher risk if he can increase the expected return from his portfolio.
According to capital market theory, Johansen can meet his risk and return objectives best by:

A. allocating a higher proportion of the portfolio to higher risk assets


B. borrowing at the risk-free rate to invest in the risky market portfolio
C. owning the risky market portfolio and lending at the risk-free rate

Answer: B

Question: 2740

Penny Linn, CFA, predicts that both Stock X and Y will return 20% next year. The Treasury bill rate is
5% and the market risk premium is 8%. The beta for Stock X is 1.5 and for Stock Y is 2. The standard
deviation for Stock X is 20% and for Stock Y is 30%. Determine if Linn's predictions lie above or below
the security market line.

A. Only Stock X lies below the SML.


B. Only Stock Y lies below the SML.
C. Both Stock X and Stock Y lie below the SML.

Answer: B

Question: 2741

GBM stock currently trades at S54 per share and is expected to trade at $62 per share in one year,
The required return on the market over the same period is 12%. Which of the following statements
about GBM stock is most likely correct?

Page | 1053
A. GBM stock has greater than average systematic risk and is undervalued.
B. GBM stock has less than average systematic risk and is overvalued.
C. GBM stock has average systematic risk and is properly valued.

Answer: A

Question: 2742

Which of the following statements about security market line (SML) is least likely to be true?

A. The SML must be graphed using the standard deviation of the market portfolio.
B. The SML measures risk using the standardized covariance of the stock with the market.
C. Securities plotting above the SML are undervalued.

Answer: A

Question: 2743

The probability distribution of stock returns for Kokomo Beach Tours, Inc., is provided below.

From the data provided, the expected standard deviation of returns for Kokomo is closest to:

A. 1.3%.
B. 2.5%.
C. 7.1%.

Answer: C

Question: 2744

Derek Bonney, CFA, is writing an investment policy statement for one of his high net worth clients,
Joey Rook. Rook is a retiree who receives Social Security benefits but because he was self-employed,
has no pension income. Rook's social security benefits cover all but $1,000 of his monthly living
expenses. He has a portfolio of SI.2 million, an effective tax rate of 30%, and recently purchased a
vacation cabin with mortgage and maintenance expenses of $6,000 per month. After meeting with
his client, Bonney writes the following policy statement: "The total return objective is to earn 7%
after-tax. At no time should the principal amount decline in value by more than 15%." The most valid
criticism of this return objective statement is that:

A. it considers only the after-tax return.


B. the return objective is too conservative.
C. it fails to consider Rook's current income needs.

Answer: C

Page | 1054
Question: 2745

WSX Capital management is considering changing the allocation of its clients' portfolios to include
commodities. The portfolios are currently invested in stocks, bonds, real estate, and hedge funds.
Which of the following is the most important investment attribute to evaluate when adding
commodities as an asset class?

A. The return volatility of commodities.


B. The relationship between commodity returns and other asset class returns.
C. The beta of the portfolio after adding commodities.

Answer: B

Question: 2746

Bill Turner is a security analyst for Secure-Invest Inc. The firm has concerns about the equal
borrowing and lending rate assumption made by the traditional capital asset pricing model (CAPM),
and, instead, tells Turner to use the zero-beta CAPM when selecting assets. Turner finds that the
return on the zero-beta portfolio exceeds the risk-free rate. Which of the following most accurately
describes the effect of relaxing the equal borrowing and lending assumption?

A. The slope of the security market line will increase.


B. The slope of the security market line will decrease.
C. The slope of the security market line will stay the same.

Answer: B

Question: 2747

Two stocks, Rich Shaw Inc., and Melon Inc., have identical total risk. The Rich Shaw stock risk is
comprised of 60% systematic risk and 40% unsystematic risk, while the Melon stock risk is comprised
of 40% systematic risk and 60% unsystematic risk. Relative to the Melon stock, the Rich Shaw stock
has:

A. a higher required return.


B. a lower required return.
C. the same required return.

Answer: A

Question: 2748

Bill Smythe and Katherine Banning want to invest 100% of their available funds in the optimal risky
portfolio. Smythe invests his money in a portfolio with an expected return of 14% and a standard
deviation of 10%. Banning invests her funds in a portfolio with an expected return of 19% and a
standard deviation of 12%. Which of the two investors has invested his/her funds in the optimal
portfolio?

A. Smythe, since his portfolio has minimized total risk.


B. Banning, since the expected return per unit of risk is higher for her investment.
C. Both, since the optimal portfolio depends on an investor's individual utility function.

Answer: C

Page | 1055
Question: 2749

Greg Burns, CFA, manages a portfolio, P, with expected return equal to 10% and standard deviation
equal to 20%. The risk-free rate is 5%. Burns advises Victoria Hull to invest 40% in portfolio P and the
remainder in the risk-free asset. The standard deviation for Hull's overall investment will be:

A. 7%.
B. 8%.
C. 12%.

Answer: B

Question: 2750

Thomas Reid is planning his $1 million retirement fund and decides to invest $300,000 in stocks,
$500,000 in bonds, and $200,000 in Treasury bills. Donna Craig decides to invest all of her $500,000
retirement fund in bonds. The expected return on stocks equals 12% and the expected return on
Treasury bills equals 4%. Both investors compare their performance against a benchmark portfolio
that equally weights stocks, bonds, and Treasury bills. The following data are provided for bonds:

Which portfolio has the highest expected return?

A. Craig's portfolio.
B. Reid's portfolio.
C. The equally weighted benchmark.

Answer: B

Question: 2751

Omar Henry is a firm believer in capital market theory and the capita! asset pricing model. Henry has
developed a model to select overpriced stocks as indicated by the security market line. The model
identifies the overpriced securities and then executes a short position in the overpriced stocks.
Which of the following practical conditions would prevent Henry from using his model to explain
capital market behavior?

A. All investors use exactly the same two-stage dividend discount model to evaluate stocks.
B. All investors pay the same commission rate of $0.03/share on all equity trades.
C. All changes in Federal Reserve policy are perfectly anticipated by investors.

Answer: B

Page | 1056
Question: 2752

William Moore is explaining the attributes and importance of asset allocation for investment
portfolios to a group of wealthy individual investors. Which of Moore's following statements is least
likely correct?

A. Asset allocation involves assigning policy weights to relevant asset classes.


B. Asset allocation is the process of selecting specific securities to include in the portfolio.
C. 85-95% of a typical portfolio's return can be explained by the target asset allocation.

Answer: B

Question: 2753

Colin Pollard currently owns a portfolio lying on the Markowitz efficient frontier that has an expected
return equal to 15% and a standard deviation equal to 15%. Pollard tells his adviser he would prefer a
portfolio tying on the Markowitz efficient frontier with a standard deviation equal to 10%. Which of
the following most likely describes the expected return on Pollard's new portfolio?

A. The expected return will be equal to 10%.


B. The expected return will be less than 10%.
C. The expected return will be greater than 10%.

Answer: B

Question: 2754

All portfolios that lie on the capital market line:

A. contain the same mix of risky assets unless only the risk-free asset is held.
B. have some unsystematic risk unless only the risk-free asset is held.
C. contain at least some positive allocation to the risk-free asset.

Answer: A

Question: 2755

Donald Northerland forecasts the stock return, beta, and standard deviation for three stocks:
Cayman, Bonaire, and Lucia. The expected return and standard deviation for the broad market equal
12% and 20%, respectively. The risk-free rate equals 5%.

Using the capital asset pricing model, determine which of the following statements is least likely
correct.

Page | 1057
A. Cayman is overvalued.
B. Bonaire is undervalued.
C. Lucia is overvalued.

Answer: A

Question: 2756

Stephanie Dell is evaluating two stocks (X and Y) using the capital asset pricing model. Dell predicts
that the betas for the two stocks will be identical, but that the unsystematic risk for Stock X will be
much higher than for Stock Y. Using the capital asset pricing model, determine which of the following
statements is correct.

A. Stock X will have a higher expected return than Stock Y, but a standard deviation less than or equal
to Stock Y.
B. Stock X will have a higher standard deviation than Stock Y, but an expected return less than or
equal to Stock Y.
C. Both the expected return and standard deviation for Stock X will be higher than Stock Y.

Answer: B

Question: 2757

Joe Finn is a highly paid corporate executive who will retire in two years. Over 25 years ago, Finn
invested in a portfolio of growth stocks that has performed quite well. Finn has asked his
financialadviser to consider switching from stocks to high-yielding bonds. The investment issue of
greatest concern in implementing this strategy will be the client's:

A. liquidity needs.
B. time horizon
C. tax considerations

Answer: C

Question: 2758

An analyst makes the following two statements about the assumptions underlying the use of the
efficient frontier to construct an optimal portfolio of assets.
Statement 1:Investors believe all investments are represented by a probability distribution of
expected returns.
Statement 2:Investors base investment decisions solely on the expected risk of the investment-
Determine whether each statement correctly describes one of the assumptions.

A. Only Statement 1 is correct.


B. Only Statement 2 is correct.
C. Both Statement 1 and 2 are correct.

Answer: A

Question: 2759

MNB Capital manages money for high net worth individuals located within the United States. MNB's
disciplined investment approach selects a small number of domestic equities selling at significant
discounts to book value. If MNB wanted to implement the capital asset pricing theory to evaluate

Page | 1058
equity investments, which of the following is least appropriate as an input to construct the market
portfolio?

A. U.S. Treasury bills.


B. Noninvestment grade bonds.
C. Collectible art work sold in Europe.

Answer: A

Question: 2760

Juliet Kaufman manages a large portfolio of risky assets for a family of investors- The portfolio
consists primarily of stocks but also includes a small allocation of fixed-income investments.
Currently, the expected return and standard deviation of the portfolio are 14.0% and 12.0%,
respectively. Kaufman is considering adding one of three stocks to the portfolio. Data on each stock's
expected return, beta, standard deviation, and covariance with the existing portfolio are presented
below. Which stock should Kaufman add to the portfolio?

A. Stock A
B. Stock B
C. Stock C

Answer: A

Question: 2761

Martin Philips evaluates stocks using the security market line while also considering the transaction
costs of each buy and sell decision. Philips assumes that both high and low beta stocks incur the
same positive percentage transactions costs on all stock trades. Which of the following is least likely
an effect of Philips' assumptions?

A. The intercept of the security market line will increase for buy signals.
B. The intercept of the security market line will decrease for sell signals.
C. The slope of the security market line will increase for both buy and sell signals.

Answer: C

Question: 2762

Patrick Manning owns stock in Lumber Providers with a return variance equal to 16%. Manning is
considering the addition of Smithson Homebuilders to his portfolio. The variance of returns for
Smithson Homebuilders equals 25%, and its correlation of returns with Lumber Providers equals -
0.60. The covariance of returns between Lumber Providers and Smithson Homebuilders is closest to:

Page | 1059
A. -15.0.
B. -0.024
C. -0.120

Answer: C

Question: 2763

A portfolio manager for Klein Capital Management has been slowly increasing the number of stocks
in his portfolio randomly over the last five years. Currently, the portfolio contains 20 stocks. Over
time, what has most likely happened to the risk of the portfolio if macroeconomic variables have
remained steady?

A. Unsystematic risk has been decreasing.


B. Systematic risk has been decreasing.
C. Both systematic and unsystematic risk remain at average levels.

Answer: A

Question: 2764

When assets in a defined-benefit plan are worth less than the present value of the promised
benefits,the fund is considered to be:

A. underfunded.
B. unfunded.
C. overfunded.
D. fully funded.

Answer: A

Question: 2765

You currently own Cavanaugh Inc. and are thinking of adding either Coe Co. or Firm Co. to your
holdings. All three stocks offer the same expected return and total risk. The covariance of returns
between Cavanaugh and Coe is +0.5 and the covariance between Cavanaugh and Firm Co. is – 0.5.
Portfolio’s risk would:

A. decline more if you bought Coe Co.


B. decline more if you bought Firm Co.
C. decrease if you bought Coe Co. but increase if you bought Firm Co.
D. would decrease most if you put half your money in Coe Co. and half in Firm Co.

Answer: B

Explanation:
In portfolio composition questions return and standard deviation are the key variables. Here you are
told that both returns and standard deviations are equal. Thus, you just want to pick the companies
with the lowest covariance, because that would mean you picked the ones with the lowest
correlation coefficient.= [++ 2where==so you want to pick the lowest covariance which is between
Cavanaugh and Firm.

Question: 2766

Page | 1060
The covariance of the market’s returns with the stock’s returns is .008. The standard deviation of the
market’s returns is .1 and the standard deviation of the stock’s returns is .2. What is the correlation
coefficient between the stock and market returns?

A. .4
B. .91
C. 1.0
D. 1.25

Answer: A

Explanation:
Covariance = (standard deviation 1)(standard deviation 2)(correlation coefficient 1,2) Correlation
coefficient = cov/(st’d1)(st’d2) = .008/(.1)(.2) = .4

Question: 2767

Cavanaugh Inc. has a beta of 1.2. Next year you project the market will earn 12% and the risk free
rate will be 5%. If you buy Cavanaugh you project your return will be 13%. Thus you think:

A. Cavanaugh will underperformed the market on a risk-adjusted basis.


B. Cavanaugh’s performance will just matched the market return on a risk-adjusted basis.
C. Cavanaugh will outperform the market on a risk-adjusted basis.
D. Cavanaugh’s performance has nothing to do with the market return.

Answer: A

Explanation:
Rs = Rf + B(Rm - Rf)Rs = .05 + 1.2(.12-.05)Rs = .134 or 13.4%You project Cavanaugh will out perform
the market.

Question: 2768

A(n) _____________ return requirements must be balanced between the need for current income
and the need for the long-term protection of capital.

A. pension fund’s
B. life insurance company’s
C. endowment fund’s
D. investment company’s

Answer: C

Question: 2769

Which of the following statements about portfolio theory is false:

A. Total risk equals systematic risk plus unsystematic risk.


B. If a security plots above the SML it is undervalued.
C. The risk measure associated with the CML is standard deviation (total risk).
D. If a security plots above the theoretical CML it is undervalued.

Answer: D

Page | 1061
Explanation:
No security can plot above the theoretical CML. All risky securities must be contained within or lie
upon the efficient frontier and the CML is tangent to the efficient frontier.

Question: 2770

Each of the following statements regarding the optimal portfolio is true except:

A. is the efficient portfolio that has the highest utility for a given investor.
B. lies at the point of tangency between the efficient frontier and the investor’s indifference curve.
C. is the portfolio that gives the investor the maximum level of return.
D. will be fully diversified.

Answer: C

Explanation:
C is wrong because it does not specify the fact that risk must also be considered.

Question: 2771

Which one of the following statements about portfolio diversification is false?

A. As more securities are added to a portfolio total risk falls, but at a decreasing rate.
B. In a well diversified portfolio of over 25 stocks market risk will account for over 85% of the
portfolio's total risk.
C. The lower the correlation coefficient between the portfolio and a stock, the lower the
diversification effect from adding that stock to the portfolio.
D. International diversification can further reduce the total risk of a portfolio.

Answer: C

Question: 2772

An investor has two stocks, Stock A and Stock B in her portfolio. What is the variance of theportfolio
given the following information about the two stocks?

A. .1472.
B. .1832.
C. .0217.
D. .0096.

Answer: C

Explanation:
Variance of two-stock portfolio =[++ 2=[(.7(.2+ (.3(.15+ (2)(.7)(.3)(.2)(.15)(.0032)] = .0217

Question: 2773

You combine three assets (A, B, and C) into a portfolio. What is the expected return and standard
deviation of the portfolio? You will put 25% of your funds into Asset A with an expected return of 5%
and a standard deviation of zero, 50% of your funds into Asset B with an expected return of 10% and
a standard deviation of 5%, and 25% of your funds into Asset C with an expected return of 5% with a
standard deviation of zero.

Page | 1062
A. .075 .025
B. .025 .05
C. .075 .075
D. .05 .10

Answer: A

Explanation:
Return = (.25)(.05) + (.50)(.10) + (.25)(.05) = .075Trick, the zero standard deviation causes everything
to go to zero.
σp = √[AA+BB+CC+ 2wAwBσAσBρAB+ 2wAwCσAσCρAC+2wCwBσCσBρCB]
σp = √[.2(0+ .5(.05+ .2(0+ 2(.25)(.5)(0)(.05)(?) + 2(.25)(.25)(0)(0)(?) +2(.25)(.5)(0)(.5)(?)]
σp = √[5(.05= 50(.05) = .025 with risk free assets in the portfolio the only risk is associated with the
portion of funds invested in the risky asset.

Question: 2774

Which of the following statements about the security market line (SML) is FALSE?

A. Movement along the SML indicates a change in risk characteristics of the individual investment.
B. The SML will rotate in response to changes in investor attitude toward risk.
C. The SML intersects the y-axis at the real risk-free rate.
D. Changes in market conditions will cause the SML to shift up or down in a parallel manner.

Answer: C

Explanation:
The SML intersects the y-axis at the nominal risk-free rate. The other choices are true.

Question: 2775

According to the fundamental view of risk, an individual’s required rate of return is determined at
least in part by all of the following factors EXCEPT:

A. the preference consumers have for current consumption.


B. relative ease or tightness in the capital markets.
C. business risk.
D. beta risk.

Answer: D

Explanation:
Beta risk, or the riskiness of the stock as viewed by a well-diversified stockholder, is used in the
systematic view of risk. The fundamental view of risk views required return as (1 + real rate)(1 +
expected inflation rate)(1 + risk premium) - 1. The risk premium is a function of total risk. (Total risk =
business risk + financial risk + liquidity risk + exchange rate risk + country risk.)
The preference consumers have for current consumption is a component of the real risk-free rate.
The relative ease or tightness in the capital markets is a component of the inflation rate. Both the
real-risk free rate and the inflation rate are components of the total return. Under the fundamental
view of risk, business risk is part of the risk premium.

Question: 2776

Page | 1063
Which of the following statements about portfolio management is FALSE?

A. Interest and dividends are typically taxed at a higher rate than capital gains.
B. Deferred tax investments may be appropriate for investors with tax concerns.
C. Mutual funds face two basic constraints: time horizon and liquidity concerns.
D. The purpose of the investment policy includes imposing discipline on the client.

Answer: C

Explanation:
The two constraints faced by mutual funds are: laws created to protect the investors and investment
choices as defined in the prospectus.
The other statements are true. Interest and dividends are typically taxed at a higher rate than capital
gains. Deferred tax investments such as 401(k) plans allow the investor to delay paying taxes until a
later date and are an important tool for an invest with current tax concerns. The investment policy
imposes discipline on both the client and portfolio manager.

Question: 2777

Dhana Urbanek is a new CFA Charterholder and is considering multiple job offers. She is most
interested in managing a fund for an institutional client with high liquidity needs and a short time
horizon that also must manage tax consequences. Which of the following institutional investors is
she most likely to select?

A. Satori University Endowment Fund.


B. Northern State Bank.
C. Caliber Life Insurance.
D. Inka Point Defined Benefit Pension Plan.

Answer: B

Explanation:
Urbanek is most likely to select the bank because it meets all her criteria. Banks have high liquidity
needs and short time horizons due to the need to maintain a positive spread and the need to payout
deposits and fund loans. In addition, banks are subject to taxation at the state and Federal levels.
Endowment funds and pension funds do not meet any of her criteria – they generally have low
liquidity needs, long time-horizons, and are usually tax-exempt. The liquidity needs and time
horizons of life insurance companies depend on the specific products offered. However, they
generally have lower liquidity needs and longer time horizons than banks, and usually only a portion
of income is taxable.

Question: 2778

The correlation between the capital market returns of Germany and Japan depend on all the
following factors EXCEPT:

A. international trading patterns.


B. economic growth rate.
C. fiscal and monetary policy.
D. geographical separation.

Answer: D

Explanation:

Page | 1064
Geographical separation has not been identified as a factor that affects correlations. The other three
choices impact the capital market returns and correlations of the respective countries.

Question: 2779

An internationally diverse group of students attending the London School of Economics has formed a
Level 1 CFA Study Group. Three of the students (from Germany, Japan, and the U.S) are assigned the
Portfolio Management Study Session reading on selecting global investments. As part of determining
how changes in currency rates can affect returns on foreign investments, the group analyzes the
return in the U.S. bond market from the late 1980s to the mid 1990s and ranks the U.S. bond market
returns in the following three ways: 1- measured in each individual’s home currency, 2- adjusted for
exchange rate changes and expressed in U.S. dollar terms, and 3 – on a risk adjusted basis. They
summarized their results in the following table:

* Rank is out of the set: Germany, Japan, & U.S.


Following is a set of possible conclusions drawn from their results. Which of the statements is
INCORRECT?

A. Investors in different countries may earn different local currency returns even if they invest in the
same investments.
B. Even though the non-U.S. markets experienced higher returns over this period, the increased risk
of the non-U.S. securities offset the higher return and lowered the return/risk ratio.
C. The U.S. dollar strengthened during the time period observed.
D. Foreign exchange rate effects can have negative impacts on investment returns.

Answer: C

Explanation:
Since the U.S. Bond rank decreased when expressed in U.S. dollar terms, the U.S. dollar weakened
over the time period observed. As the dollar weakens, a U.S. investor would earn higher returns on
foreign investments because each unit of foreign currency buys more dollars. A foreign investor
would also be able to earn higher returns in the U.S., as it would take less of a foreign currency to
buy dollars.

Question: 2780

Gregg Goebel and Mason Erikson are studying for the Level 1 CFA examination. They have just
started the section on Portfolio Management and Erikson is having difficulty with the equations for
the covariance (cov1,2) and the correlation coefficient (r1,2) for two-stock portfolios. Goebel is
confident with the material and creates the following quiz for Erikson. Using the information in the
table below, he asks Erickson to fill in the question marks.

Page | 1065
Which of the following choices correctly gives the covariance for Portfolio J and the correlation
coefficients for Portfolios K and L, respectively?

A. 1.680, 0.002, 0.000.


B. 0.011, 0.833, 0.056.
C. 0.110, 0.224, 0.076.
D. 0.083, 0.011, 0.417.

Answer: B

Explanation:
The calculations are as follows:
Portfolio J covariance= cov1,2= (r1,2) * (1) * (2) = 0.75 * 0.08 * 0.18 = 0.0108, or 0.011.
Portfolio K correlation coefficient= (r1,2) = cov1,2/ [ (1) * (2) ] = 0.02 / (0.20 * 0.12) = 0.833
Portfolio L correlation coefficient= (r1,2) = cov1,2/ [ (1) * (2)1/2] = 0.003 / (0.18 * 0.091/2) = 0.003 /
(0.18 * 0.30) = 0.056Remember:The standard deviation is the square root of the variance. Read
carefully on the exam!

Question: 2781

Which of the following statements about portfolio theory is FALSE?

A. For a two-stock portfolio, the lowest risk occurs when the correlation coefficient is close to
negative one.
B. Assuming that the correlation coefficient is less than one, the risk of the portfolio will always be
less than the simple weighted average of individual stock risks.
C. Risk aversion results in an upward sloping security market line (SML).
D. When the return on an asset added to a portfolio has a correlation coefficient of less than one
with the other portfolio asset returns but has the same risk, adding the asset will not decrease the
overall portfolio standard deviation.

Answer: D

Explanation:
This statement misstates the principle of diversification and should read, “When the return on an
asset added to a portfolio has a correlation coefficient of less than one with the other portfolio asset
returns but has the same risk, adding the asset will decrease the overall portfolio standard
deviation.” Anytime the correlation coefficient is less than one, there are benefits from
diversification. The other choices are true.

Question: 2782

Consider the following graph of the Security Market Line (SML). The letters X, Y, and Z represent risky
asset portfolios. The SML crosses the y-axis at the point 0.07. The expected market return equals 13.0
percent. Note: The graph is NOT drawn to scale.

Page | 1066
Using the graph above and the information provided, determine which of the following statements is
TRUE.

A. Portfolio Y is undervalued.
B. The correct label for the x-axis is total risk.
C. Portfolio X's required return is greater than the market expected return.
D. The expected return (or holding period return) for Portfolio Z equals 14.8%.

Answer: D

Explanation:
At first, it appears that we are not given the information needed to calculate the holding period, or
expected return (beginning price, ending price, or annual dividend). However, we are given the
information required to calculate the required return (CAPM) and since Portfolio Z is on the SML, we
know that the required return (RR) equals the expected return (ER). So, ER = RR = Rf+ (ERM– Rf) *
Beta = 7.0% + (13.0% - 7.0%) * 1.3 = 14.8%.
The SML plots beta (orsystematic risk) versus expected return, the CML plots total risk (systematic
plus unsystematic risk) versus expected return. Portfolio Y is overvalued – any portfolio located
below the SML has an RR > ER and is thus overpriced. Since Portfolio X plots above the SML, it is
undervalued and the statement should read, “Portfolio X’s required return islessthan the market
expected return.”

Question: 2783

Karissa Grossklaus recently joined an investment banking firm as a research analyst. One of the
partners asks her to determine whether a certain stock, Park Street Holdings, is overvalued or
undervalued, and by how much (expressed as percentage return). Grossklaus runs a regression and
finds the following information on the stock:
Grossklaus reports that Park Street Holdings stock is:

A. undervalued by 1.1%.
B. overvalued by 3.7%.
C. overvalued by 1.1%.
D. undervalued by 3.7%.

Answer: C

Explanation:
To determine whether a stock is overvalued or undervalued, we need to compare the expected
return (or holding period return) and the required return (from Capital Asset Pricing Model, or

Page | 1067
CAPM).
Step 1: Calculate Expected Return (Holding period return):
The formula for the (one-year) holding period return is:
HPR = (D1+ S1– S0) / S0, where D = dividend and S = stock price.
Here, HPR = (0 + 55 – 45) / 45 = 22.2%
Step 2: Calculate Required Return:
The formula for the required return is from the CAPM:
RR = Rf+ (ERM– Rf) * Beta
Here, we are given the information we need except for Beta. Remember that Beta can be calculated
with: Betastock= [covS,M] / [2M]. Here we are given the standard deviation of the market, so the
calculation is: 1.30 / 0.752= 2.31.
RR = 4.25% + (12.5 – 4.25%) * 2.31 = 23.3%.
Step 3: Determine over/under valuation:
The required return is greater than the expected return, so the security is overvalued. The amount =
23.3% - 22.2% = 1.1%.

Question: 2784

While working abroad, United States citizen Alex Beggs purchases a foreign bond with an annual
coupon of 7.0 percent for 93.0. He holds the bond for one year and then sells it for 95.5 before he
leaves. During the year, the dollar appreciated 2.5% relative to the foreign currency.
Which of the following is closest to Begg's Total Dollar Return?

A. 7.460%.
B. 12.970%.
C. 10.215%.
D. 0.121%.

Answer: A

Explanation:
TheTotal Dollar Returnis calculated as follows:
Step 1: Calculate the value change in the foreign currency:
The dollarappreciated0.025, so the foreign currencydepreciatedby 0.025, or 2.5%.
Step 2: Use the Total Dollar return formula to calculate the return:
R$= { [ 1+($coupon + VEND- VBEG)/VBEG] * (1 + g) } - 1,
WhereR$=Total dollar return,VEND= Bond value at end of period,VBEG=Bond value at end of period,
andg= % change in thedollarvalue of the foreign currency.
Here,R$= { [ 1+(7.0 + 95.5 – 93.0)/93.0 ] * (1 - 0.025) } - 1
= { [1.102151 ] * (0.975) } - 1
= 0.07460, or7.460%

Question: 2785

Which of the following statements about international portfolio investing is TRUE?

A. U.S. investors are internationally diversified.


B. Emerging markets offer the greatest degree of diversification and the highest expected returns
because of their industrial composition and currency behavior.
C. The national beta measures the volatility of a nation's index relative to the referenced stock index,
usually the U.S. index or the world index.
D. The benefits of diversification increase when the weights used for international and domestic
countries/assets conform to relative market capitalization weights.

Page | 1068
Answer: C

Explanation:
U.S. investors are not internationally diversified, because of the following barriers: regulatory
accounting and disclosure requirements that keep many foreign firms from registering and selling
their shares in the U.S. capital markets, higher international trading costs, complications of foreign
tax laws, and foreign exchange transaction cost and risk. Emerging markets offer the greatest degree
of diversification and the highest expected returns because of their low correlations (due to volatile
economic and political natures). Industrial composition and currency movements are not considered
factors that impact correlations. The benefits of diversification increase when the weights used do
not conform to relative market capitalization weights.

Question: 2786

Clair Boschart is preparing for her CFA study group’s discussion of the security market line (SML). She
asks her co-worker, a fellow CFA candidate, to review her summary of points. Which of the following
statements does the co-worker identify as INCORRECT? If:

A. risk perception increases, the SML will rotate counterclockwise.


B. inflation expectations increase, the SML will experience an upward parallel shift.
C. economic growth decreases, the SML will experience an upward parallel shift.
D. the capital markets tighten, the SML will experience an upward parallel shift.

Answer: C

Explanation:
If economic growth decreases, the SML will experience a downward parallel shift. Investors who
supply capital demand a lower rate.

Question: 2787

Which of the following equations is INCORRECT?

A. Real Risk-Free Rate = [(1 + nominal Risk-Free rate) * (1 + inflation rate)] - 1.


B. Expected Return (SML) =Rnominal Risk-Free+ (RMarket- Rnominal Risk-Free) * Beta.
C. RequiredReturnnominal= [(1 + Risk-FreeRatereal) * (1 + Expected Inflation) * (1 + Risk Premium)] -
1.
D. RiskpremiumFundamental view= total risk = business risk + financial risk + liquidity risk + exchange
rate risk + country risk.

Answer: A

Explanation:
The real Risk-Free Rate = [(1 + nominal Risk-free Rate) divided by (1 + inflation rate)] – 1. The other
equations are correct.

Question: 2788

Isabelle Santana and Marat Loring are studying for the Level 1 CFA examination. Loring is having
difficulty determining the objectives and constraints of defined contribution and defined benefit
pension plans. To help Loring study, Santana creates the following list of characteristics and asks
Loring to select the one that is FALSE. Which statement should Loring select?

A. For a defined benefit plan, the most important factors that affect long-term fund performance are

Page | 1069
the individual asset selection process and the degree of market timing allowed.
B. Both plans are tax-exempt.
C. Both plans are federally regulated under the Employee Retirement Income Security Act (ERISA).
D. The employee bears all the investment risk in a defined contribution plan.

Answer: A

Explanation:
Even though these two investment decisions (timing and individual security selection) add value to
the portfolio, the real foundation of returns comes from the original asset allocation policy decisions.
Empirical studies have shown that 85 percent to 95 percent of a portfolio’s total return comes from
the asset allocation policy decision (determining allowable asset classes and weighting these
classes).
The other statements are correct.

Question: 2789

Which of the following statements about institutional investors is TRUE?

A. Banks have high liquidity needs and short time horizons.


B. Pension funds and endowment funds have low liquidity needs and long time horizons, carry tax
exempt status, and both are regulated at the state and federal level.
C. In general, Life Insurance companies have lower liquidity needs and shorter time horizons than
Property/Casualty Insurance Companies.
D. The liquidity and time horizon parameters for defined contribution pension plans are determined
by employee age and turnover rate.

Answer: A

Explanation:
Banks have high liquidity needs and short time horizons due to the need to maintain a positive
spread and the need to pay out deposits and fund loans.
Pension funds are regulated at the Federal level under ERISA and endowments are regulated at the
state level. Life Insurance companies have longer time horizons than Property/Casualty Insurance
Companies. (Lower liquidity needs usually translate to longer time horizons.) The liquidity and time
horizon parameters for defined benefit pension plans are determined by employee age and turnover
rate. These are plans where the employer promises the employee a specific income stream upon
retirement.

Question: 2790

The graph below combines the efficient frontier with the indifference curves for two different
investors, X and Y (represented by U(X) and U(Y)). The letters A, B, C, and D represent four distinct
portfolios.

Page | 1070
Which of the following statements about the above graph is CORRECT?

A. The backward bend in the efficient frontier is due to less than perfect correlation between
portfolio assets.
B. Investor X would be better off moving to indifference curve U(X)1 and Portfolio C because of the
higher return on that portfolio.
C. Investor X is less risk-averse than Investor Y.
D. Portfolio B is an optimal portfolio, Portfolio A is suboptimal.

Answer: A

Explanation:
This statement is true. Markowitz was the first person to recognize that there are no perfectly
positively correlated assets or perfectly negatively correlated assets. Thus, the efficient frontier has
the shape noted above.
The other choices are incorrect. The optimal portfolio for each investor is on the highest indifference
curve that is tangent to the efficient frontier. Thus, portfolios A and B are both optimal portfolios, but
for different investors. In addition, any portfolio on the efficient frontier is superior to one that is not.
Thus, Investor X would not be better off with Portfolio C (this portfolio is on a lower indifference
curve and has more risk.) Investor X has a steep indifference curve, indicating that he is risk-averse.
Flatter curves, such as those for investor Y, indicate a less risk-averse investor.

Question: 2791

Kendra Jackson, CFA, is given the following information on two stocks, Rockaway and Bridgeport.
Assuming that Jackson must construct a portfolio using only these two stocks, which of the following
combinations will result in the minimum variance portfolio?

A. 100% in Rockaway.
B. 50% in Bridgeport, 50% in Rockaway.
C. 80% in Bridgeport, 20% in Rockaway.
D. 100% in Bridgeport.

Answer: D

Explanation:
First, calculate the correlation coefficient to check whether diversification will provide any benefit.
rBridgeport, Rockaway= covBridgeport, Rockaway/ [ (Bridgeport) * (Rockaway) ] = 0.0325 / (0.13 *

Page | 1071
0.25) = 1.00
Since the stocks are perfectly positively correlated, there are no diversification benefits and we select
the stock with the lowest risk (as measured by variance or standard deviation), which is Bridgeport.

Question: 2792

Fabrice Miro and Victoria Leete are studying for the Level 1 CFA examination. Miro wants to test
Leete’s understanding of the graph of the capital market line (CML) and the efficient frontier. He
develops the following statements and asks her to identify the one that is FALSE. Assuming that
Leete answers correctly, which statement does she select?

A. The market portfolio lies on the CML and has only unsystematic risk.
B. The CML is not always straight.
C. Investors move up and down the CML by varying the weightings of the risk-free asset and portfolio
M by either lending or borrowing the risk-free asset.
D. One weakness of the CML graph is that it measures standard deviation against returns.

Answer: A

Explanation:
The first part of this statement is true - the market portfolio does lie on the CML. However, the
market portfolio is completely diversified and thus has no unsystematic risk. The risk that remains is
market portfolio risk, or nondiversifiable, or systematic risk.
The other choices are true. The CML will “kink” if the borrowing rate and lending rate are not equal.
The CML does measure standard deviation (or total risk) against returns, and this is considered a
weakness since systematic, or undiversifiable, risk is what investors are compensated for
undertaking. The securitymarket line (SML) better represents this because it measures systematic, or
beta, risk against expected returns.

Question: 2793

Turi Teigen, CFA candidate, prepares the following question for her weekly Level 1 study program.

Using the graph (along with the list of assumptions), determine which of the following statements is
CORRECT.

A. The expected return on Portfolio Y could be 15.00%.


B. The expected return on Portfolio Z is greater than the required return.
C. The required return on Portfolio X is 10.25%.
D. Portfolio X is overvalued.

Answer: C

Explanation:

Page | 1072
Note: RR = required return, ER = expected return.Remember that the SML graph plots systematic, or
beta, risk versus expected return. Thus, the numbers on the x-axis represent beta. Using the Capital
Asset Pricing Model (CAPM) equation, RR = Rf+ (ERM– Rf) * Beta = 5.0% + (7.5%) * 0.7 = 10.25%.
Portfolio Y lies below the SML and is thus overvalued and the expected return must be less than the
required return. Using the CAPM, RR = Rf+ (ERM– Rf) * Beta = 5.0% + (7.5%) * 1.0 = 12.50%. (On the
exam, you can quickly determine the RR for a portfolio/asset with a beta of 1.0 by adding the risk-
free rate and the market premium.) Since the ER must be less than the RR, the ER must be less than
12.50% and cannot be 15.00%. Since Portfolio Z is on the SML, it is fairly valued and RR = ER. Since
Portfolio X lies above the SML, it isundervalued.

Question: 2794

Which of the following statements about portfolio management is TRUE?

A. As an investor diversifies away the unsystematic portion of risk, the correlation between his
portfolio return and that of the market approaches negative one.
B. The security market line (SML) measures systematic and unsystematic risk versus expected return;
the CML measures total risk.
C. Combining the capital market line (CML) (risk-free rate and efficient frontier) with an investor's
indifference curve map separates out the decision to invest from the decision of what to invest in.
D. The expected return on a 0 beta security is the expected market return.

Answer: C

Explanation:
Combining the CML (risk-free rate and efficient frontier) with an investor’s indifference curve map
separates out the decision to invest from what to invest in and is called the separation theorem. The
investment selection process is thus simplified from stock picking to efficient portfolio construction
through diversification.
The other statements are false. As an investor diversifies away the unsystematic portion of risk, the
correlation between his portfolio return and that of the market approaches positive one. (Remember
that the market portfolio has no unsystematic risk). The SML measures systematic risk, or beta risk.
The expected return on a 0 beta security is the risk free rate (the market premium term has a value
of 0).

Question: 2795

Which of the following statements about Arbitrage Pricing Theory (APT) and the Capital Asset Pricing
Model (CAPM) is FALSE?

A. APT can equal CAPM.


B. In both the APT and the CAPM, the risk-free rate is added to a premium for risk factor (X) and the
responsiveness of the asset's returns to factor (X).
C. If zero-investment arbitrage does not hold, the APT does not hold.
D. APT is a multi-factored model with restrictive assumptions.

Answer: D

Explanation:
Arbitrage Pricing Theory is a multifactored model with few limiting assumptions. More than one risk
factor is able to influence security prices.
The other statements are true. Arbitrage Pricing Theory can equal the Capital Asset Pricing Model
(CAPM) if there is only one risk factor – market risk. Zero-investment arbitrage is an assumption of
the APT. More specifically, zero-investment arbitrage means that if an investor buys an overpriced

Page | 1073
security, the investor has access to the short-sale money needed to buy an underpriced security.

Question: 2796

While in the managerial training program for a large multinational corporation, Daniel Waite is
assigned a one-year rotation in the Mediterranean. Upon arriving at the assignment, he purchases a
local (foreign currency) bond with an annual coupon of 8.5 percent for 96.5. He holds the bond for
one year and then sells it for 98.0. Waite is pleased with his return, which he calculates at 10.4%.
On the plane ride home, Waite is seated next to his fellow coworker, Penny King, who begins to talk
about the depressed local economy and the negative returns she had experienced on her local bond
investments over the same period as Waite. She states that her total dollar return on an 8.0 percent
annual coupon bond purchased at the same time as Waite's for 95.0 and sold for 98.0 (at the same
time as Waite's) was a disappointing negative 10.737%.
Assume that King’s calculation is correct and that Waite made some calculation error. Which of the
following is closest to Waite’s actual total dollar return?

A. -32.435%.
B. -10.363%.
C. -18.756%.
D. -11.712%.

Answer: D

Explanation:
Waite forgot to take into account the impact of the percentage change in the dollar value of the
foreign currency.
Using the information provided by King, we can calculate the percentage change in the value of the
foreign currency and then calculate Waite's total dollar return. Using the formula for total dollar
return of:
R$= { [ 1+($coupon + VEND- VBEG)/VBEG] * (1 + g) } - 1,
-0.10737= { [ 1+(8.0 + 98.0 – 95.0)/95.0 ] * (1 + FChange) } - 1
-0.10737 = { [1.115789] * (1 + FChange) } - 1
-0.2000 = FChange, or 20.0% depreciation.
Now, we can calculate Waite’s total dollar return.
R$ Waite= { [ 1+(8.5 + 98.0 - 96.5)/96.5 ] * (1 - 0.20) } - 1
R$ Waite=-11.712%

Question: 2797

Which of the following statements about international portfolio investing is FALSE?

A. The majority of risk for individual foreign country markets is unsystematic.


B. When markets are volatile, global diversification is of increased value.
C. Barriers to international diversification include lack of liquidity, currency controls, and exchange
rate risk.
D. While the introduction of foreign stocks ("second layer") into a domestically-only diversified
portfolio shifts the efficient frontier up and left, adding a "third layer" of foreign bonds shifts the
efficient frontier even more up and left.

Answer: B

Explanation:
When markets are volatile, global diversification is of limited value. Studies show that correlations
between markets increase as market volatility increases, which limits the benefits of diversification.

Page | 1074
In particular, studies show that correlations increase when markets are falling.
The other statements are true. Although foreign exchange risk is not considered a factor that impacts
correlations, it is a significant barrier to international investing.

Question: 2798

An investor has two stocks, Stock R and Stock S in her portfolio. What is the standard deviation of the
portfolio given the following information about the two stocks?

A. 0.0056.
B. 0.0867.
C. 0.2944.
D. 0.0208.

Answer: C

Explanation:
Standard deviation of a two-stock portfolio = [W12σ12+ W22σ22+
2W1W2σ1σ2r1,2]1/2=[(.8)2(.34)2+ (.2)2(.16)2+ 2(.8)(.2)(.34)(.16)(.67)]1/2= 0.29439

Question: 2799

An overvalued stock is one whose:

A. estimated return is more than the required return.


B. return is expected to be less than the expected market return.
C. estimated return is less than the required return.
D. return is expected to be greater than the expected market return.

Answer: C

Question: 2800

Nicki Tobin, CFA candidate, has just started studying portfolio management. She notes the following
similarities between the graphical representations of the fundamental and systematic views of risk.
Which one of her observations is INCORRECT? For both views of risk:

A. the y-axis represents expected return.


B. investors require a higher expected return for riskier investments.
C. investors demand a risk premium above the nominal risk-free rate.
D. the graphs plot the market's risk premium against expected return.

Answer: D

Explanation:
The systematic view of risk is that the security’s required risk premium is a function of its systematic,
or non diversifiable risk, while the fundamental view of risk is that the security’s risk premium is a
function of its total risk. However, the market’s risk premium is not on either axis for either view. The
fundamental view plots total risk versus expected return, and the systematic view plots beta
(systematic risk) versus expected return. Although both views use the concept of a risk premium over
the risk-free rate, the calculation differs. The risk premiumFundamental view= total risk = business
risk + financial risk + liquidity risk + exchange rate risk + country risk. The risk premiumsystematic
view= Expected ReturnMarket– Nominal ReturnRisk-Free. The equation for the stock’s risk premium
is: [Expected ReturnMarket– Nominal ReturnRisk-Free] * Beta.

Page | 1075
Question: 2801

In the context of the security market line (SML), the expected (or required) return will decrease in
the following situations EXCEPT:

A. The U.S. Federal Reserve takes action that tightens the capital markets.
B. Leading market indicators decrease, causing consumers to expect lower inflation levels.
C. A firm announces that it has won a major product-liability lawsuit.
D. The growth in the U.S. economy is expected to slow from 2.7% last year to 1.5% this year.

Answer: A

Explanation:
If the capital markets tighten, the SML will shift upward in a parallel manner, resulting in a higher
required return. If the capital markets loosen, the SML will shift downward in a parallel fashion,
resulting in a decrease in expected return.
All other choices result in a downward shift in the SML. After the lawsuit is “won,” investors would
likely view a firm as less risky and would move down along the SML to require a lower return. If
economic growth decreases or if inflation decreases, the SML will shift downward in a parallel
fashion. Investment growth opportunities are determined by the economy’s long-term growth rate
and factor into the real risk-free rate, which is a component of required return. When the growth
rate decreases, investors who supply capital require a lower rate of return.

Question: 2802

Maxime Rivela, CFA, is interviewing for a portfolio manager position with a medium-sized
investment firm. At the interview, the hiring manager provides the following list of actions taken by
the former portfolio manager. The hiring manager asks Rivela to identify which action most likely was
the reason that the previous portfolio manager was asked to resign. The former portfolio manager
likely:

A. managed a 40 year old attorney's portfolio with a strategy of a long time horizon and moderate
risk.
B. used the following four-step portfolio management process - write a policy statement, develop the
investment strategy, implement the plan, and monitor and update (rebalance) as needed.
C. set the objectives for a well-funded, local private college's endowment fund as total return
focused primarily in long-term, taxable investments.
D. focused on timing and security selection when constructing a client investment strategy.

Answer: D

Explanation:
Even though these two investment decisions (timing and individual security selection) add value to
the portfolio, the real foundation of returns comes from the original asset allocation policy decisions
(determining allowable asset classes and weighting these classes). Empirical studies have shown that
85 percent to 95 percent of a portfolio’s total return comes from the asset allocation policy decision.
The other choices describe appropriate actions. The 40 year old attorney is most likely in the
consolidation phase of the investing life cycle, and a long time horizon and moderate risk strategy are
appropriate. The steps in the four-step portfolio management process are correct and are listed in
the correct order. The objective and investments for the endowment fund appear appropriate given
the information in the question. Since endowments are tax-exempt, it would not necessarily benefit
from tax-exempt securities.

Page | 1076
Question: 2803

Assume that a U.S. investor can invest in two asset classes: Domestic and Foreign Bonds. The data in
the following table are risk and return data calculated in U.S. Dollar terms. Given this data, which of
the following choices is most likely the correct risk/return combination for a portfolio weighted 50
percent in domestic bonds and 50 percent in foreign bonds?

A. Risk of 7.50%, Return of 8.25%.


B. Risk of 8.00%, Return of 10.90%.
C. Risk of 6.00%, Return of 9.25%.
D. Risk of 10.00%, Return of 13.50%.

Answer: C

Explanation:
According to portfolio theory and the principle of diversification, we would expect that adding more
of a security that has a low correlation to an existing portfolio would lower the portfolio risk. Since
the correlation between foreign and domestic bonds is low at 0.50, we would expect the risk of the
portfolio to decline below 8.00% when we replace domestic bonds with foreign bonds. This narrows
the possible correct responses to the two with risk less than 8.00%. In addition, we would expect the
return to increase, as the return on foreign bonds is higher than that on domestic bonds, butremain
slightly lower than that for the portfolio that is 20% domestic and 80% foreign. We can calculate the
estimated portfolio return as the weighted average of returns for the individual asset classes. (0.50 *
12) + (0.50 * 6.5) = 9.25%

Question: 2804

Which of the following statements about portfolio theory is TRUE?

A. According to Markowitz, the appropriate measures of risk include variance, semivariance, and
covariance.
B. There are no perfectly positively or perfectly negatively correlated stocks.
C. A correlation coefficient of zero means that combining the securities eliminates risk.
D. Risk decreases as the correlation coefficient goes from -1 to +1.

Answer: B

Explanation:
Markowitz was the first to identify this. The lack of perfect positive or negative correlation gives rise
to the efficient frontier shape.
Covariance is a measure of movement, not of risk. A correlation coefficient of zero means that the
returns are uncorrelated. A correlation coefficient of negative one means that risk can go to zero.
Adding a security with a correlation coefficient of zero may reduce risk, but risk will not be
eliminated. Risk increases as the correlation coefficient goes from –1 to +1. Risk decreases as the

Page | 1077
correlation coefficient goes from +1 to -1.

Question: 2805

Duncan Manz believes that he has found an error in a sample CFA Study Program question. Prior to
e-mailing the provider about the error, he discusses his logic with Julia Cook, a fellow finance student
at the Hess School of Business. Manz does not believe that the following question provides enough
information to completely answer the question. Cook disagrees. Who is correct ?Manz or Cook? And,
if Cook is correct, what is the correct answer?
Question: An investor
抯portfolio currently consists of 100% of stocks that have a mean return of 18 percent and an
expected variance of 0.0625. The investor plans to diversify slightly by replacing 30 percent of her
portfolio with U.S. Treasury bills that earn 4.25 percent. Assuming the investor diversifies, what are
the expected return and expected standard deviation of the portfolio?

A. Cook is correct. The portfolio's expected return is 13.875%and the expected standard deviation is
4.375%.
B. Cook is correct. The portfolio's expected return is 18.000% and the expected standard deviation is
15.250%.
C. Manz is correct. There is not enough information to completely answer the question.
D. Cook is correct. The portfolio's expected return is 13.875% and the expected standard deviation is
17.500%.

Answer: D

Explanation:
Cook is correct. Since Treasury bills (T-bills) are consideredrisk-free, we know that the standard
deviation of this asset and the correlation between T-bills and the other stocks is 0. Thus, we can
calculate the portfolio expected return and standard deviation.
Step 1: Calculate the expected return:
Expected ReturnPortfolio= (wT-bills* ERT-bills) + (wStocks* ERStocks)= (0.30) * (0.0425) + (1.00 -
0.30) * (0.18) = 0.13875, or 13.875%.
Step 2: Calculate the expected standard deviation:
When combining a risk-free asset and a risky asset (or portfolio of risky assets), the equation for the
standard deviation of1,2= [(w12)(12) + (w22)(22) + 2w1w212r1,2]1/2reduces to:Portfolio=
[(wStocks)(Stocks)] = 0.70 * 0.06251/2= 0.17500, or 17.500%. (Remember to convert variance to
standard deviation.)

Question: 2806

Consider the following graph of the risk-free asset Rf and the efficient frontier. The letters K, W, X Y,
and Z represent risky portfolios. Portfolio M is the market portfolio. The linesRfXandRfYrepresent the
combination of the risk-free asset and the risky portfolio.

Page | 1078
Which of the following statements about the above graph is FALSE?

A. The area to the right of point S represents unsystematic risk, or risk that is not paid for.
B. Assets W and Z are perfectly positively correlated with each other.
C. By definition of the capital market line (CML), portfolio K is possible, but not the most efficient
because it does not fall on the efficient frontier and is overvalued.
D. Investors on the capital market line (CML) to the right of M are leveraged and hold more than
100% of portfolio M.

Answer: C

Explanation:
By definition, all stocks and portfolios other than M, the market portfolio, fall on or below the CML.
Thus, K is not a possible portfolio on this graph. Overvaluation/undervaluation is usually expressed in
relation to the SML (security market line). The other statements are true.

Question: 2807

Kira Trace, research analyst at an investment banking firm, took the Level 1 CFA examination in 2001,
but did not pass. Last year, she studied alone, and only for one-month before the exam. This year, she
is starting earlier and is working with a mentor, Anton Park, CFA. While discussing asset pricing
models with Park, Trace makes the following statements. Park can see that Trace still needs to study
this area because only one of her statements is correct. Which statement is CORRECT?

A. According to the Capital Asset Pricing Model (CAPM), the rate of return of a portfolio with a beta
of 1.0 and an alpha of 0 is the market expected return.
B. Assuming assets are not perfectly positively correlated, the systematic risk of a portfolio decreases
as more assets are added.
C. Adding the risk-free asset to a portfolio will reduce return and total risk.
D. It is difficult for the individual investor to achieve the benefits from diversification because
significantly reducing risk requires the purchase of approximately 1,000 securities.

Answer: A

Explanation:
Diversification reduces unsystematic or unique risk. With the risk-free asset and a portfolio of risky
assets, the equation for the expected standard deviation is linear:wA.A combination of the risk free
asset and a portfolio always gives more return for a given level of risk. Risk tends to be reduced, but

Page | 1079
assuming that assets are not perfectly positively correlated, an investor can achieve the benefits of
diversification by adding just one security (Markowitz). Studies have shown that approximately 18-30
stocks are needed for proper diversification. The main point is that the number ofstocks required is
small and is significantly less than all securities (and significantly less than 1,000 securities).

Question: 2808

While working abroad, United States citizen Elpida Costa purchases a foreign bond with an annual
coupon of 10.0 percent for par. She holds the bond for one year and then sells it for 103.6 before she
leaves. During the year, the dollar depreciated 2.0% relative to the foreign currency.
Which of the following is closest to Costa's Total Dollar Return?

A. 11.328%.
B. 13.600%.
C. 15.872%.
D. 5.672%.

Answer: C

Explanation:
TheTotal Dollar Returnis calculated as follows:
Step 1: Calculate the value change in the foreign currency:
The dollardepreciated0.02, so the foreign currencyappreciatedby 0.02, or 2.0%.
Step 2: Use the Total Dollar return formula to calculate the return:
R$= { [ 1+($coupon + VEND- VBEG)/VBEG] * (1 + g) } - 1,
WhereR$=Total dollar return,VEND= Bond value at end of period,VBEG =Bond value at end of period,
andg= % change in thedollarvalue of the foreign currency.
Here,R$= { [ 1+(10.0 + 103.6 – 100.0)/100.0 ] * (1 + 0.02) } - 1
= { [1.13600 ] * (1.02) } - 1
= 0.15782, or15.782%

Question: 2809

Using the data in the following table, calculate the national beta for Country A and Country B and
determine which country is most likely an emerging market. Note: rx, U.S. represents the correlation
coefficient between Country X and the U.S. index.

Which of the following statements about Country A and Country B is FALSE?

A. Country B is most likely an emerging market.


B. The national beta for Country A is less than the beta for Country B.
C. The national beta for Country B equals 0.82.
D. Adding either country to a domestically-only diversified portfolio will reduce overall risk.

Answer: A

Explanation:
The equation for the national beta is as follows:

Page | 1080
As shown above, the beta for Country A is less than the beta for Country B. Country A is most likely
the emerging market because of its lower correlation with the U.S, the lower Beta, and the higher
risk (standard deviation).
Because of the low correlations with the U.S. market, adding either asset to a domestically-only
diversified portfolio will reduce risk. The principle of diversification holds even for risky assets.

Question: 2810

An investor believes Stock M will rise from a current price of $20 per share to a price of $26 per share
over the next year. The following information pertains:
Should the investor purchase the stock?

A. No, because it is undervalued.


B. Yes, because it is overvalued.
C. No, because it is overvalued.
D. Yes, because it is undervalued.

Answer: C

Explanation:
Expected return of Stock M = 8 + 1.7(16 - 8) = 21.6. Estimated return of Stock M = 26 - 20/20 = 30%.
Since the estimated return is greater than the expected return, the investor should purchase the
stock.

Question: 2811

A drastic decrease in the United States market would:

A. cause an increase in the world market index.


B. not affect the world market index.
C. cause a decrease in the world market index.
D. cannot predict the impact.

Answer: C

Question: 2812

When investor inflation expectations increase, all else equal, the security market line (SML) will:

A. shift downward in a parallel manner.


B. rotate counterclockwise.
C. shift upward in a parallel manner.
D. rotate clockwise.

Page | 1081
Answer: C

Explanation:
Parallel shifts in the SML result from changing market conditions (such as an increase in expected
inflation). When inflation expectations increase, investors want to be compensated for the increased
risk and to insure that their purchasing power is not eroded. Thus, the SML shifts upward, increasing
the expected return at each level of risk.
The SML will shift downward in response to decreasing inflation expectations, decreased economic
growth, and expanding capital markets. Changes in slope cause the SML to rotate. The slope of the
SML will change in response to changes in attitudes of investors toward risk that affects the market
risk premium. If there is an increase in the market risk premium, the SML will rotate
counterclockwise about the risk free rate. If there is a decrease in the market risk premium, the SML
will rotate clockwise about the risk free rate.

Question: 2813

Ryan Konkol, CFA, is helping Gabriel Miskowiak study for the Level 1 CFA examination. Konkol asks
Miskowiak to calculate the market risk premium based on the following assumptions:
Which of the following choices is closest to the correct answer?

A. 8.0%.
B. 4.4%.
C. 8.5%.
D. 5.2%.

Answer: B

Explanation:
The Market Risk Premium = Expected Market Return – Nominal Risk Free Rate. Here, we are given
the expected market return, but need to calculate the nominal risk free rate. Then, we can
determine the Market Risk Premium.
Step 1: Calculate the Nominal Risk-Free Rate:
Nominal Risk-Free Rate = [(1 + Risk FreeRatereal)*(1 + Inflation Premium)] - 1
= [(1 + 0.04)*(1 + 0.035)] – 1 = 1.076 – 1 = 0.076 or 7.6%
Step 1: Calculate the Risk Premium:
Market Risk Premium = Expected Market Return – Nominal Risk Free Rate
= 12% - 7.6% = 4.4%

Question: 2814

The nominal required rate is determined by the inflation premium, the risk premium, and the:

A. nominal risk-free rate.


B. real risk-free rate.
C. exchange rate impact.
D. liquidity risk.

Answer: B

Explanation:
The equation for the nominal risk free rate is:
RequiredReturnnominal= [(1 + Risk-FreeRatereal) * (1 + Inflation Premium) * (1 + Risk Premium)] - 1
The market risk premium is used when determining the security market line (SML) and equals the
required market return minus the nominal risk-free rate. Liquidity risk is a part of total risk which is

Page | 1082
measured by the risk premium. The exchange rate is not a factor in this calculation.

Question: 2815

Which of the following statements about the investment life cycle is FALSE?

A. In general, a recent college graduate working at her first job is in the accumulation phase, has a
long time horizon, and should invest in high-risk securities to earn above average nominal returns.
B. A working couple both in the midpoint of their careers has a fairly long time horizon and is
beginning to move to investments with moderate risk to provide asset protection.
C. A retired financial planner in the spending phase is most concerned with estate planning and tax
minimization.
D. Risk capacity generally declines though the investment life cycle.

Answer: C

Explanation:
The gifting state is associated with estate planning and tax minimization. The spending phase is
associated with protection for assets. Although these two states usually run concurrently, the life
cycle distinguishes them in that the gifting phase is concerned with giving away assets while in the
spending phase, there is still some accumulation from low-risk investments.
The other choices are true.

Question: 2816

Which of the following statements about portfolio management is TRUE?

A. Investment constraints include liquidity, time horizon, taxes, legal and regulatory concerns, and
unique characteristics such as concern for social issues.
B. The order of the four step investment process is: analyze current financial and economic
conditions, specify goals and constraints, allocate assets across asset classes, and monitor and
rebalance the portfolio as needed.
C. An investor's portfolio (in the spending and gifting phase) should contain a sizable portion of
equities to preserve capital over the long term.
D. The investment performance should be judged based on the raw portfolio return.

Answer: A

Explanation:
This statement is true.
The steps of the investment process are listed in the incorrect order. The order should be as follows:
specify goals and constraints (write a policy statement), analyze current financial and economic
conditions (develop investment strategy), allocate assets across asset classes (implement the plan),
monitor and rebalance the portfolio as needed (monitor and update). In the spending and gifting
phases, investor portfolios generally less in equities as investors seek lower risk securities.
Performance should be judged using an objective, benchmark standard – not the raw portfolio
return.

Question: 2817

Keenan Omarik, a portfolio manager in the private banking division of GSK Bank, is trying to convince
a conservative client to diversify using international bonds and equities. Currently, the client holds
only U.S. investments (but is well diversified) and is reluctant to change his allocation. All of the
following statements support Omarik’s recommendation for international diversification EXCEPT:

Page | 1083
A. By replacing domestic investments with foreign investments in a portfolio, investors can reduce
the portfolio risk while holding the number of investments constant.
B. Foreign securities usually have low correlations with U.S. investments and offer more
diversification opportunities than domestic investments alone.
C. Historically, non-U.S. securities have outperformed U.S. investments on a return basis, at least in
part because the non-U.S. economies have experienced higher growth rates.
D. By diversifying both domestically and internationally, investors eliminate the risk from worldwide
systematic risk factors.

Answer: D

Explanation:
This statement should read, “By diversifying both domestically and internationally, investors limit risk
to worldwide systematic risk factors only." (Much like domestic diversification reduces unsystematic
risk from individual investments, but systematic risk still exists.) The other choices are valid
arguments for international diversification.

Question: 2818

The graph below combines the efficient frontier with the indifference curves for two different
investors, X and Y.

Which of the following statements about the above graph is INCORRECT?

A. Investor X is less risk-averse than Investor Y.


B. Investor X's return will always be less than that of Investor Y.
C. The portfolios indicated by the points X and Y represent the optimal portfolio for each investor.
D. The efficient frontier line represents the portfolios that provide the highest return at each risk
level.

Answer: A

Explanation:
Investor X has a steep indifference curve, indicating that he is risk-averse. Flatter indifference curves,
such as those for Investor Y, indicate a less risk-averse investor. The other choices are true. A more
risk-averse investor will likely obtain lower returns than a less risk-averse investor.

Question: 2819

Page | 1084
Jeret Behr is a member of a Level 1 CFA Study Group. Last week, he was assigned the reading on
asset pricing models and is to prepare a “tough” question on the capital market line (CML). He
constructs the following graphical representation of the CML and drafts the following four
statements about the graph. The letters W, X Y, and Z represent risky portfolios. Portfolio M is the
market portfolio. The linesRfXandRfYrepresent the combination of the risk-free asset and the risky
portfolio.

Which of the following statements about the above graph did Behr make TRUE?

A. Investors are not compensated for the risk represented by the area to the left of point S.
B. The correct label for the x-axis is beta risk.
C. Portfolio Z was created by borrowing at the risk free rate.
D. Portfolio Y is inefficient.

Answer: D

Explanation:
This selection is true (or not false). By definition, all portfolios below the CML are inefficient in that
they do not give enough return for their risk. Investors could do better by combining the risk-free
asset with the portfolio.
The area to the left of point S represents systematic or undiversifiable risk. Investors are
compensated for this risk. The correct label for the x-axis is total risk, or standard deviation. The CML
plots total risk (systematic plus unsystematic risk) versus expected return; the security market line
(SML) plots beta (or systematic risk) versus expected return. Portfolio Z is created by lending at the
risk-free rate and holding less than 100% of Portfolio M. Investors in portfolios to the left of the
market portfolio, M, are invested in less than 100% of M.

Question: 2820

Which of the following statements about Beta is TRUE?

A. In the characteristic line, the termEResidualsequals the portion of the asset's returns that are
correlated to the market portfolio's returns.
B. The calculation of Beta assumes that future variability will equal past variability.
C. Beta <1, ERstock>ERmarket.
D. Beta and standard deviation differ as risk measures in that beta measures only systematic risk,
whereas standard deviation measures only unsystematic risk.

Answer: B

Explanation:
Standard deviation measures total risk. When Beta < 1, ERstock< ERmarket. The characteristic line is
the equation for Beta. The error term, EResiduals, equals the portion of the asset’s returns that
areuncorrelatedto the market portfolio’s returns, or unsystematic risk. This error term has an
expected value of 0 and the standard deviation is called the residual standard error.

Question: 2821

While studying abroad, United States citizen David Rees purchases a foreign bond with an annual
coupon of 8.0 percent for 94.0. He holds the bond for one year and then sells it for 93.0 before he
leaves. During the year, foreign currency appreciated 5.0% relative to the U.S. dollar.
Which of the following is closest to Rees's Total Dollar Return?

A. 2.074%.

Page | 1085
B. 7.447%.
C. 3.883%.
D. 12.819%.

Answer: D

Explanation:
TheTotal Dollar Returnis calculated as follows:Step 1: Calculate the value change in the foreign
currency:The foreign currencyappreciatedby 0.05, or 5.0%.
Step 2: Use the Total Dollar return formula to calculate the return:
R$= { [ 1+($coupon + VEND- VBEG)/VBEG] * (1 + g) } - 1,
WhereR$=Total dollar return,VEND= Bond value at end of period,VBEG =Bond value at end of period,
andg= % change in thedollarvalue of the foreign currency.
Here,R$= { [ 1+(8.0 + 93.0 – 94.0)/94.0 ] * (1 + 0.05) } - 1
= { [1.074468 ] * (1.05) } - 1
= 0.12819, or12.819%

Topic 7, Alternative Investments

Question: 2822

Open-end funds

A. have shares that trade on secondary exchanges.


B. never charge redemption fees for share redemptions.
C. sometimes charge sales fees for share sales.
D. never charge sales fees for share sales.

Answer: C

Explanation:
Open-end funds (also known as mutual funds) continue to sell and repurchase shares at their NAVs
after their initial public offerings. Some funds (load funds) charge a sales fee for share sales that is
typically 7.5 to 8% of the NAV. Load funds generally do not charge a redemption fee, but some
noload funds do charge a small one of about half a percent of NAV.

Question: 2823

The variability among venture capital investments is exhibited by the fact that

A. approximately 75 percent of investments lose money.


B. about 50 percent of all gains come from about 7 percent of the investments.
C. about 75 percent of all gains come from about 5 percent of the investments.
D. about one-third of the investments are responsible for half of all gains.

Answer: B

Explanation:
The returns of venture capital are not uniformly realized across investments, i.e., about 1 in 15
investments is responsible for half of all gains. Approximately one-third of investments lose money.

Question: 2824

Money market funds

Page | 1086
A. are typically load funds.
B. attempt to provide significant capital gains.
C. tend to be illiquid.
D. invest in short-term securities.
E. tend to impose a penalty for early withdrawal.

Answer: D

Explanation:
Money market funds attempt to provide current income, liquidity, and principal safety though
investing in short-term securities such as T-bills, banker certificates of deposits, and commercial
paper. They generally allow holders to write checks against their accounts.

Question: 2825

Most mutual funds compared to the (Dow Jones Industrial Average) DJIA _______.

A. are less risky


B. achieved lower net returns
C. show similar returns
D. achieved greater net returns

Answer: B

Explanation:

Most mutual funds compared to the (Dow Jones Industrial Average) DJIA achieved lower net returns,
after deducting operating expenses.

Question: 2826

Which of the following is categorized under speculative property and not income property?

A. Land
B. Office buildings
C. Factories
D. Single family properties
E. Multi-family properties
F. Warehouses

Answer: A

Explanation:
Income property includes single and multi-family properties, office buildings, factories, and
warehouses. Speculative property includes raw land.

Question: 2827

Which type of investment company may charge a redemption fee?

A. none of these answers


B. both of these answers
C. open-end investment company

Page | 1087
D. closed-end investment company

Answer: C

Explanation:

An open-end investment company (mutual fund) sells shares with or without a sales charge (load)
and redeems shares with or without redemption fees.

Question: 2828

Low-load open-end funds

A. never charge sales or redemption fees.


B. never charge sales fees.
C. charge sales fees of about 0.5 to 1% of NAV.
D. charge sales fees of about 3% of NAV.
E. charge sales fees of about 5 to 6% of NAV

Answer: D

Explanation:

Low-load open-end funds typically charge front-end sales fees in the 3% range (rather than 7 to 8%)
when the fund is bought. These funds typically are used for bond funds or equity funds offered by
management companies that also offer no-load funds.

Question: 2829

The majority of empirical evidence indicates that

A. the gross returns on mutual funds tend to as good as or slightly better than those on buy-and-hold
strategies, while the net returns on mutual funds tend to be slightly lower than those on buy-and-
hold strategies.
B. the gross returns on mutual funds tend to as good as or slightly worse than those on buy-and-hold
strategies, while the net returns on mutual funds tend to be significantly lower than those on buy-
and-hold strategies.
C. the gross returns on mutual funds tend to be significantly better than those on buy-and-hold
strategies, while the net returns on mutual funds tend to be about equal to those on buy-and-hold
strategies.
D. the gross and net returns on mutual funds tend to be significantly higher than those on buy-and-
hold strategies.
E. the gross and net returns on mutual funds tend to be significantly lower than those on buy-and-
hold strategies.

Answer: E

Explanation:
Net returns (which take account for research and trading costs) have been about 1% lower on mutual
funds than on buy-and-hold strategies. Because of these studies, index funds have grown in
popularity. Because index funds only imitate a broad market index, they have no research costs and
minimal trading expenses.

Question: 2830

Page | 1088
What is not a characteristic of an investment company?

A. per share value is the NAV


B. invests in a pool of funds
C. investment management company is voted and approved by the individual investors
D. can be classified as closed-end or open-end
E. an investment management company is hired by a board of directors

Answer: C

Explanation:

An investment advisory firm starts an investment company and selects a board of directors for the
fund. Subsequently this board of directors hires the investment advisory firm as the fund's portfolio
manager.

Question: 2831

________ permits funds to deduct as much as 1.25 percent of average net assets per year to cover
distribution costs.

A. The 12b-1 Act


B. Rule 144a
C. The New Funds plan
D. The 12b-1 plan

Answer: D

Explanation:

The 12b-1 plan permits funds to deduct as much as 1.25 percent of average net assets per year to
cover distribution costs.

Question: 2832

Name the type of fee charged by a fund when it is sold if is held for less than a specified time period?

A. 12b-1
B. no-load
C. deferred sales load
D. management fees
E. commissions
F. low-load

Answer: C

Explanation:

Also known as back-end charges when the fund is sold as opposed to front-end charges when the
fund is purchased.

Question: 2833

Page | 1089
________ financing is an example of expansion financing.

A. Turnaround
B. Leveraged-buyout
C. Third-stage
D. First-stage
E. Start-up

Answer: C

Explanation:

There are three stages of "expansion financing":


1. Second-stage financing. This capital is used for initial expansion of a company that has already
been selling a product.
2. Third-stage financing. Capital provided to fund major expansion.
3. Mezzanine (bridge) financing. Capital provided for a company that expects to go public within a
year or so.

Question: 2834

The venture capital industry now funds investments such as

A. companies seeking leveraged buyouts.


B. none of these answers.
C. hedged funds.
D. companies in the mature stage of their life cycle.
E. all of these answers are correct.
F. companies emerging from bankruptcy.

Answer: A,D,F

Explanation:
Recently, the venture capital industry has begun to cover a broader range of investments including
funding for:

1. Established businesses that are in the process of expanding, just prior to the IPO stage.
2. Companies in "turnaround" situations.
3. Management seeking LBOs, also referred to MBOs of their companies.

Question: 2835

________ financing is used to restructure a financially troubled company.

A. None of these answers


B. Leveraged-buyout
C. Bridge
D. Management-buyout
E. Turnaround

Answer: E

Explanation:

Page | 1090
Turnaround financing is capital is used to restructure a troubled company that is at a more
established stage of development.

Question: 2836

Studies have found that mutual fund managers tend to have

A. great market timing ability.


B. ignored the potential of market timing.
C. moderate market timing ability.
D. little market timing ability.

Answer: D

Explanation:

Most studies in this area have found that funds could not consistently generate superior returns
through attempts at market timing. In fact, many have found that mutual funds cannot outperform
passive investment strategies.

Question: 2837

The primary mechanism for motivating venture capitalists is

A. allowing a high ownership stake in the venture.


B. permitting certain types of self-dealing.
C. to have finite life funds.
D. the carried interest paid to them.

Answer: D

Explanation:

Carried interest is calculated over and above some assumed base level of return to investors and the
amount paid is usually approximately 20 percent of the portfolio gains.

Question: 2838

The per-share value of an investment company is called ________.

A. the net share price


B. the net start-up cost
C. the net asset value
D. the net unit portfolio investment

Answer: C

Explanation:

The per-share value of an investment company is called the Net Asset Value or NAV.

Question: 2839

Which of these funds typically charge a sales fee when the fund is sold?

Page | 1091
I. Funds with contingent, deferred sales loads
II. Funds under 12b-1 plans
III. Low-load funds
IV. No-load funds
V. Closed-end funds

A. II only
B. I, II & V
C. I & III
D. I only
E. I, II & III
F. III only

Answer: D

Explanation:

Funds with contingent, deferred sales load charges sales fee if the shares are sold before a certain
duration, low-funds charge a small sales fee. Not all funds under the 12b-1 plan do necessary charge
sales fee. Closed-end funds trade at a discount usually to the NAV.

Question: 2840

The management fee of an investment management firm typically ranges from ________.

A. one to two percent of the total value of the fund


B. none of these answers
C. one-quarter to one-half percent of the total value of the fund
D. one-half to one percent of the total value of the fund

Answer: C

Explanation:

The management fee of an investment management firm typically ranges from one-quarter to one-
half percent of the total value of the fund, with a sliding scale as the size of the fund increases.

Question: 2841

Which type of investment company may charge a load?

A. both of these answers


B. none of these answers
C. closed-end investment company
D. open-end investment company

Answer: D

Explanation:

An open-end investment company (mutual fund) sells shares with or without a sales charge (load)
and redeems shares with or without redemption fees.

Page | 1092
Question: 2842

Venture capital can create value to the firm by

A. venture capitalists figuring out how to manage a firm in a poorly managed sector and then
acquiring other firms in the same sector at low prices.
B. providing current management with new and additional funding.
C. paying a premium for the firm and then proceeding to sell the firm in "pieces."
D. investing in additional company assets.

Answer: A

Explanation:
The venture capitalist believes that the managements of these acquired firms can be improved by
using the same techniques that were used for the first company. The values of the acquired firms will
therefore increase because of the improvement in the management.

Question: 2843

Venture capitalists expect to actually realize their returns

A. up front, before the firm has any operating profit.


B. spread out evenly over the length of the investment.
C. only if the company becomes profitable on a consistent basis.
D. only when they are able to cash out.
E. during the first round of financing.

Answer: D

Explanation:

The objective of the venture capitalist is to earn a 40%-60% annual return on an investment, net of
inflation. Venture capitalists expect to actually realize their returns only when they are able to cash
out, which hopefully is less than ten years, and preferably in three to seven.

Question: 2844

Which is a true statement concerning the empirical evidence of mutual fund performance?

A. more aggressive funds did not outperform more conservative funds


B. over 50 percent of the funds outperform the DJIA
C. risk measures increase as fund objectives become more aggressive
D. successful market forecasting was accomplished overall by fund managers
E. none of these answers
F. good performance is associated with high expense ratios

Answer: C

Explanation:

There is a positive relationship between stated objectives and measure of risk, with risk measures
increasing as objectives become more aggressive.

Question: 2845

Page | 1093
Which best describes venture capital?

A. Venture capital is typically a secondary form of financing, after bank financing.


B. Venture capital is in the form of equity.
C. Venture capital is short-term financing, usually at higher-than-market rates.
D. Venture capital is financing for privately held companies, typically in the form of equity and/or
longterm debt.
E. Venture capital is equity and debt financing available to privately and publicly held companies.

Answer: D

Explanation:

Venture capital becomes available when financing from banks and public debt or equity markets is
either unavailable or inappropriate.

Question: 2846

The ________ of an investment company depends on the company's portfolio of stocks.

A. bond-rating
B. liquidity of the shares
C. none of these answers
D. value of the shares

Answer: D

Explanation:

Share value derives from future dividends and capital gains on each stock within the portfolio. The
type, quality and relative quantity of stocks within a portfolio affects the value of the shares of the
portfolio.Portfolio composition also affects liquidity and creditworthiness (i.e. bond rating) but much
less significantly.

Question: 2847

________ financing is capital provided for a company that expects to go public within a year or so.

A. Third-stage
B. Second-stage
C. Mezzanine
D. First-stage
E. Leveraged-buyout

Answer: C

Explanation:

There are three stages of "expansion financing":


1. Second-stage financing. This capital is used for initial expansion of a company that has already
been selling a product.
2. Third-stage financing. Capital provided to fund major expansion.
3. Mezzanine (bridge) financing. Capital provided for a company that expects to go public within a

Page | 1094
year or so.

Question: 2848

What stage of venture capital investments has provided the highest mean return?

A. later
B. seed
C. balance
D. early
E. later and balance are identical

Answer: A

Explanation:
The later stage has provided a 17.7% mean return from 1981-1996, followed by the early stage
(16.0%), balance stage (13.7%), and seed stage (8.3%).

Question: 2849

When valuing real estate, ________ is more difficult to apply to older properties.

A. none of these answers


B. the comparative sales approach
C. the cost approach
D. the income approach

Answer: C

Explanation:

The cost approach works best with new or relatively new properties.

Question: 2850

________ financing is used for product development and initial marketing for firms in business under
one year and has not sold their product commercially.

A. Seed
B. First-stage
C. Start-up
D. Third-stage
E. Second-stage

Answer: C

Explanation:

There are three early stages of financing:


1. Seed financing. This is capital (typically less than $50,000) that is provided at the "idea" stage,
which goes for product development and market research.
2. Start-up financing. This capital is used in product development and initial marketing for firms in
business under one year and has not sold their product commercially.
3. First-stage financing. This is capital provided to initiate commercial manufacturing and sales.

Page | 1095
Question: 2851

No-load funds have traditionally been

A. rare.
B. distributed by a sales force.
C. distributed by direct marketing.
D. distributed by brokers.

Answer: C

Explanation:

No-load funds have traditionally been distributed by direct marketing because brokers had little
incentive to sell them. Distribution has been done through mail, telephone, bank wire, or office of
the fund. More recently, some no-load funds have developed agreements with some brokerage firms
to sell the funds in return for a fee.

Question: 2852

Net returns on mutual funds have tended to be

A. much higher than the market return.


B. slightly higher than the market return.
C. slightly lower than the market return.
D. much lower than the market return.

Answer: C

Explanation:

Most funds have not provided net returns superior to a buy-and-hold strategy that tries to emulate
the aggregate market. The shortfall of performance has been about 1 percent a year, which is roughly
equal to the average cost of research and trading commissions for mutual funds.

Question: 2853

The largest obstacle to development of venture capital in emerging markets is

A. the lack of capital to develop young companies.


B. the lack of investor interest.
C. the lack of a strong IPO market for fast-growing and high technology firms.
D. government interference.
E. the lack of accurate financial information of the young companies.

Answer: C

Explanation:

The largest obstacle to development in emerging markets is the lack of a strong IPO market for
fastgrowing and high technology firms. Emerging markets lack an active IPO market, which will
create problems for exit strategies.

Page | 1096
Question: 2854

________ indicates the price at which a property would sell under current market conditions.

A. Cost approach value


B. Income approach value
C. Market value
D. Appraisal
E. Market approach value

Answer: C

Explanation:

Remember though, that many properties sell for prices significantly above or below their estimated
market values.

Question: 2855

Which of the following describes an investment company?

A. provides complete investment advice to clients for a fee


B. tracks the market price performance of a sample of U.S. mutual funds
C. invests in a pool of funds belonging to many individuals in a portfolio of investments
D. all of these answers are correct
E. market value of all its assets divided by shares outstanding

Answer: C

Explanation:

For example, an investment company sells 20 million shares to individuals at $10 per share for a total
of $200 million. The manager then invests this money in securities (according to fund objectives).
Each investor owns a percentage of the investment company's total portfolio.

Question: 2856

Venture capital differs from investments from publicly held firms in that they are

A. illiquid in the short-term.


B. none of these are correct.
C. for newer firms with very little operating history.
D. difficult to value.
E. all of these are correct.
F. for small firms in which the venture capital firm and the subject company have personal
involvement.

Answer: E

Explanation:

Venture capital differs from investments from publicly held firms in that they are generally:
1. For newer companies with little operating history.
2. For smaller companies in which the venture capital firm and the entrepreneurial company have a

Page | 1097
high degree of personal involvement.
3. Illiquid in the short-term, until the company goes public or is acquired by another company, which
usually takes three to seven years.
4. Difficult to value because there is no public market for these securities.
5. Going to require future rounds of financing.

Question: 2857

When looking at demographics and psychographics, which one of the following four major
determinant of real estate value categories are you in?

A. the property
B. none of these answers
C. demand
D. supply
E. the property transfer process

Answer: C

Explanation:

Demographics and psychographics are both subsets of analyzing demand.

Question: 2858

Most market value appraisals, when estimating the value of real estate, are ________.

A. looking forward in time


B. both looking backward and forward in time
C. looking backward in time
D. neither looking forward nor backward in time

Answer: C

Explanation:

Most information that is gathered by an appraiser is based on events that have already taken place,
for example, sales prices of comparable properties that have closed in the past.

Question: 2859

A/An ________ sells shares at their NAV.

A. initial offer fund


B. par-value fund
C. no-load fund
D. closed-end fund

Answer: C

Explanation:

A no-load fund imposes no sales charges so it sells shares at their NAV, however some charge a small
redemption fee of about half-percent.

Page | 1098
Question: 2860

Aggressive (growth) funds have tended to

A. outperform conservative funds.


B. have portfolio compositions very similar to conservative funds.
C. have returns about equal to those of conservative funds.
D. underperform conservative funds.

Answer: A

Explanation:
A study by McDonald found a positive relationship between stated objectives (growth vs. income, for
example) and measures of risk. The higher risk was also found to have been positively related to
higher return. The objectives of a fund were thus found to have influenced its performance.

Question: 2861

Name the fee charged by a fund when the fund is bought and is typically in the 3 percent range of
the NAV?

A. low-load
B. management fees
C. deferred sales load
D. no-load
E. commissions
F. 12b-1 plan

Answer: D

Explanation:
Between the full-load and no-load funds charges, low-load funds are used for bond or equity funds
offered by management companies that also offer no-load funds.

Question: 2862

Which statement is correct concerning the correlation of venture capital?

A. It has a high correlation with the other assets in a portfolio.


B. There is a positive correlation with treasury bills.
C. It is negatively correlated to the large-cap stock market.
D. There is a small correlation with corporate and treasury bonds.
E. It is highly correlated with the small-cap stock market.

Answer: E

Explanation:

The high correlation with small-cap stocks primarily results from the implications of exit strategies,
which are fundamentally connected to the value of the market.

Question: 2863

Page | 1099
What is a characteristic of an open-end investment company?

A. NAV is based on current market prices


B. its stock trades on the regular secondary market
C. its market price is determined by supply and demand
D. the NAV is computed twice daily
E. it continues to repurchase and sell shares

Answer: E

Explanation:

Open-end investment companies (mutual funds) sell additional shares of the fund at the NAV, with or
without sales charge, or redeem shares at the NAV, with or without redemption fees.

Question: 2864

The portfolio of securities of an investment company is typically managed by

A. the company's board of directors.


B. the company's analysts.
C. the company's CEO.
D. a legally separate investment management company.

Answer: D

Explanation:

The portfolio and most other administrative duties of the investment company are typically managed
by a separate investment management company. In fact, that investment management company
usually starts the investment company and picks its board of directors, who then hire the investment
management company to manage the new investment company's portfolio.

Question: 2865

Which one of the following is not one of the four major determinants of real estate value?

A. the property
B. demand
C. supply
D. the property transfer process
E. none of these answers

Answer: E

Explanation:

Demand, supply, the property and the property transfer process all are determinants of real estate
value.

Question: 2866

Which is not a true statement concerning the empirical evidence of mutual fund performance?

Page | 1100
A. less than 32 percent of the funds outperform the DJIA
B. risk measures increase as fund objectives become more aggressive
C. none of these answers are true
D. more aggressive funds outperformed more conservative funds
E. good performance is associated with low expense ratios
F. successful market forecasting was not accomplished overall by fund managers
G. all of these answers are true

Answer: G

Explanation:

All of these are true as a result of research.

Question: 2867

What would the offering price be if the NAV of a fund with a 7.5% load is $10.25?

A. 10.25
B. 10.90
C. 10.45
D. 11.31
E. 11.08

Answer: E

Explanation:

The offering (ask) price = NAV / (1.0 - percent load) $10.25 / (1 - .075) = 11.08

Question: 2868

________ refers to measurable characteristics, such as household size, age structure, occupation,
gender, and marital status.

A. Characgraphics
B. Socialgraphics
C. Psychographics
D. Demographics

Answer: D

Explanation:

By comparing demographic and psychographic trends to the features of a property, you can judge
whether it is likely to gain or lose favor among potential buyers or tenants.

Question: 2869

According to the framework for real estate analysis, one determinant of value is supply. Which of
following is not a subset of "supply"?

A. Inventorying competitors
B. Market structure

Page | 1101
C. Sources of competition
D. All of these answers
E. None of these answers

Answer: E

Explanation:

All of these answers are demand determinants of value.

Question: 2870

The major duties of an investment management company are:


I. handling of redemptions and dividends
II. investment research and portfolio management
III. recommending stocks and bonds to investors
IV. arranging bank loans for fund investors

A. II only
B. I, II, III & IV
C. I & II
D. I, II & III

Answer: C

Explanation:

The major duties of an investment management company are investment research and portfolio
management, handling of redemptions and dividends and issuing securities.

Question: 2871

Open-end investment companies

A. will repurchase shares at their NAVs.


B. do not typically sell shares after their initial public offerings.
C. have shares that tend to be priced at a discount to their NAVs.
D. do not typically repurchase shares.

Answer: A

Explanation:
Open-end investment companies (also known as mutual funds) continue to sell and repurchase
shares after their initial public offerings. They do so at the current NAVs, sometimes with sales or
redemption fees.

Question: 2872

The stock of a closed-end investment company ________.

A. trades through private brokerage firms


B. trades on the regular secondary market
C. trades via the investment management company
D. trades only between existing stock-holders

Page | 1102
Answer: B

Explanation:

A close-end investment company operates like any other public firm. Its stock trades on the regular
secondary market and the market price is determined by supply and demand. Closed-end typically
means the company offers no further shares, thus shares can only be bought over from existing
shareholders.

Question: 2873

When valuing real estate, ________ should rarely be used exclusively as an approach to value.

A. none of these answers


B. the comparative sales approach
C. the income approach
D. the cost approach

Answer: D

Explanation:

The cost approach works best with new or relatively new properties and is a good method to use as a
check of the other approaches to value.

Question: 2874

The NAV of a closed-end investment company is computed ________.

A. twice every day of trading


B. once every day of trading
C. once every financial year
D. once every financial quarter

Answer: A

Explanation:
The NAV of a closed-end investment company is computed twice every day of trading when there are
prevailing market prices for the portfolio securities.

Question: 2875

Money market funds

A. were first developed in 1973 when short-term interest rates were at record levels. These funds
attempt to provide current income, safety of principal, and liquidity by investing in diversified
portfolios of short-term securities including Treasury bills, certificates of deposit, bank acceptances,
and commercial paper.
B. were first developed in 1981 when short-term interest rates were at record levels. These funds
attempt to provide current income, safety of principal, and liquidity by investing in diversified
portfolios of short and medium-term securities including Treasury bills, government agency bonds,
and commercial paper.
C. were first developed in 1981. These funds attempt to provide current income, safety of principal,

Page | 1103
high returns, and liquidity by investing in diversified portfolios of short and medium-term securities
including Treasury bills and notes, certificates of deposit, bank acceptances, and commercial paper.
D. were first developed in 1973 when interest rates were at record levels. These funds attempt to
provide safety of principal and liquidity by investing in diversified portfolios of short-term securities
including Treasury bills, municipal bonds, certificates of deposit, and commercial paper.

Answer: A

Explanation:
Money market accounts generally allow investors to write checks against them. Their growth rate is
often associated with investor attitudes toward the stock market. When investors are bullish, they
move funds from their money market accounts and invest in stocks. When they are bearish, they sell
off stocks and put some of the proceeds in money market accounts. Municipal bonds are not
included in this type of investment.

Question: 2876

Which best describes venture capitalists?

A. They typically exert control over a firm so it will generate required returns.
B. They usually will fire the management team of the entrepreneurial firm and replace with their
own management team.
C. They are typically passive investors and invest in a company because they like the management
team.
D. They will only invest in a firm that will provide a return of 100%+ when they cash out.
E. They will structure financing to the liking of the entrepreneur so that the company has a better
opportunity to grow.

Answer: C

Explanation:

Most venture capitalists, however, have no desire to control a venture. The main reason they invest
in a company in the first place is because they like the management team. Venture capitalists are
typically passive investors and experts in financing companies, structuring deals and advising
management on important long-term decisions.

Question: 2877

Mutual funds are distributed by

A. three major methods: through brokers, through dedicated sales forces, or through direct purchase
from the fund or direct marketing. Most no-load funds have been distributed through brokers, while
most load funds have been distributed through direct marketing.
B. two major methods: through a sales forces, or through direct purchase from the fund or direct
marketing. Although most funds are currently distributed through direct marketing, there has
recently been a trend toward distribution through a sales force.
C. three major methods: through brokers, through dedicated sales forces, or through direct purchase
from the fund or direct marketing. Most no-load funds have been distributed through a sales force,
while most load funds have been distributed through brokers.
D. two major methods: through a sales forces, or through direct purchase from the fund or direct
marketing. Although most funds are currently distributed through a sales force, there has recently
been a trend toward distribution through direct marketing.

Page | 1104
Answer: D

Explanation:

In the past, no-load funds were sold only through direct marketing because the lack of a sales fee
made it difficult to compensate brokers or a dedicated sales force. More recently, some brokers have
started distributing no-load funds. Currently, about 54% of funds are distributed through a sales
force, while 34% are distributed through direct marketing.

Question: 2878

The average discount from NAV for a closed-end investment company shares ________ over time.

A. remains constant
B. falls
C. rises and falls
D. rises

Answer: C

Explanation:

The average discount from NAV for a closed-end investment company shares rises and falls over time
depending on investors' sentiments.

Question: 2879

A major goal of the venture capital industry has been to resolve the conflicts that arise between

A. investors and entrepreneurs.


B. venture capitalists and investment bankers.
C. investors and venture capitalists and between venture capitalists and entrepreneurs.
D. venture capitalists and stockbrokers.

Answer: C

Explanation:

When selecting venture capitalists, investors must understand exactly how both sets relationships
are to be handled and whether the incentive structure is competitive.

Question: 2880

________ financing is one of the early stages of financing in venture capital.

A. all of these answers are correct.


B. Seed
C. First-stage
D. Start-up

Answer: A

Explanation:
There are three early stages of financing:

Page | 1105
1. Seed financing. This is capital (typically less than $50,000) that is provided at the "idea" stage,
which goes for product development and market research.
2. Start-up financing. This capital is used in product development and initial marketing for firms in
business under one year and has not sold their product commercially.
3. First-stage financing. This is capital provided to initiate commercial manufacturing and sales.

Question: 2881

An investment company

A. invests only in stocks.


B. borrows money from individuals and invests that money in securities.
C. lends money to other companies or individuals.
D. invests a pool of funds belonging to many individuals.

Answer: D

Explanation:

An investment company sells shares in itself, and uses the proceeds to invest in a portfolio of
individual investments such as stocks and bonds. If such a company sold 1 million shares for $50
dollars each, it would invest $50 million (minus commissions) into things like government bonds or
speculative stocks.

Question: 2882

International funds

A. typically have one-half of their portfolios invested in the U.S.


B. typically have one-fifth of their portfolios invested in the U.S.
C. do not invest any portion of their portfolios in the U.S.
D. typically have two-thirds of their portfolios invested in the U.S.
E. typically have one-third of their portfolios invested in the U.S.

Answer: C

Explanation:

An International fund is a type of foreign fund that only invests in non-U.S. securities. The
international investment opportunities provided by this type of fund give American investors an
added level of diversification.

Question: 2883

A no-load open-end fund

A. never charges sales or redemption fees.


B. usually charges sales fees.
C. sometimes charges sales fees.
D. sometimes charges redemption fees.

Answer: D

Explanation:

Page | 1106
A no-load open-end fund does not charge sales fees, but rather sells shares at their NAVs. Some
noload funds do charge redemption fees of about half a percent of NAV. The popularity of no-load
funds has increased substantially in recent years.

Question: 2884

When valuing real estate, ________ utilizes selling prices of properties that are similar to the subject
property.

A. the comparative sales approach


B. the income approach
C. the cost approach
D. none of these answers

Answer: A

Question: 2885

Which of the following is not classified as an income property? A leased:

A. office building.
B. none of these answers.
C. condominium.
D. shopping center.
E. apartment complex.

Answer: B

Explanation:

All of the answers are income properties.

Question: 2886

Global funds

A. invest only in established countries.


B. invest in any country in the world.
C. invest only in developing countries.
D. invest only outside of the U.S.

Answer: B

Explanation:

A global fund is a type of foreign fund that invests in both U.S. and non-U.S. securities. The
international investment opportunities provided by this type of fund give American investors an
added level of diversification.

Question: 2887

Name the type of fee charged by a fund to cover distribution costs?

Page | 1107
A. management fees
B. 12b-1
C. commissions
D. low-load
E. deferred sales load
F. no-load

Answer: B

Explanation:

The 12b-1 plan allows a fund to deduct as much as 1.25 percent of average net assets per year to
cover distribution costs such as advertising, brokers' commissions and marketing expenses.

Question: 2888

Which of the following approaches determines the value by finding the amount it would take to
rebuild it at today's prices for land, labor and construction materials?

A. Cost approach
B. Income approach
C. Current pricing approach
D. Comparative sales approach

Answer: A

Explanation:
Cost approach determines value by assessing how much it would cost to rebuild it at today's prices
for land, labor and construction methods.

Question: 2889

The market price of closed-end fund shares are historically ________ the NAV.

A. about 20 percent premium above


B. 5 to 10 percent discount to
C. very close to
D. none of these answers

Answer: D

Explanation:

The NAV and the market price of a closed-end fund are almost never the same, because closed-end
funds typically have market prices that has historically been a 5 to 20 percent discount to NAV.

Question: 2890

Which of the following statements is true?

A. The shares of an open-end investment company usually trade at a 10 to 25% discount to its NAV,
but in some relatively rare instances, the shares trade exactly at NAV.
B. The shares of a closed-end mutual fund almost never trade exactly at its NAV. Rather, shares
typically trade at a 5 to 20% discount to the NAV.

Page | 1108
C. The shares of a closed-end investment company usually trade at exactly its NAV, but in some
relatively rare instances, the shares trade at a small discount or premium to the NAV.
D. The shares of an open-end investment company usually trade at exactly its net asset value (NAV),
but in some relatively rare instances, the shares trade at a small discount or premium to the NAV.
E. The shares of a closed-end investment company almost never trade at exactly its NAV. Rather,
shares typically trade at a 5 to 20% discount to the NAV.

Answer: E

Explanation:

Open-end investment companies continue to sell and repurchase shares after their initial public
offerings. They sell shares at the net asset value with or without sales charges, and repurchase shares
at the net asset value with or without redemption fees. The shares of closed end investment
companies,however, are rarely sold or redeemed by the funds, but typically trade on a stock market.
Over the long-run, the shares tend to trade at a 5 to 20% discount to their net asset value (NAV).
There is no such thing as a closed-end mutual fund. Only open-end investment companies are
referred to as mutual funds.

Question: 2891

Investment companies often start numerous funds to ________.

A. justify investment in more securities


B. attract many investors with different risk-return preferences
C. diversify the aggregate portfolio
D. all of these answers

Answer: B

Explanation:

Investment companies often start numerous funds to attract many investors with different risk-return
preferences, which increases the total capital managed by the investment company.

Question: 2892

The share price of a closed-end investment company is

A. generally lower than its NAV.


B. generally higher than its NAV.
C. typically equal to its NAV.
D. always equal to its NAV.

Answer: A

Explanation:
The share price of a closed-end investment company is determined by the market supply and
demand for its stock. Over the long run, shares of closed-end investment companies have traded 5 to
20 percent below their NAVs. The reason for this disparity is still open to question.

Question: 2893

Many different types of investment companies offer ________.

Page | 1109
A. only one investment fund that pools the risks and return characteristics of the Fortune 500
companies
B. a wide variety of alternative instruments with a range of risk and return characteristics
C. arbitrage opportunities
D. risk-free investments

Answer: B

Explanation:

Investment companies typically offer investments that are portfolios of various instruments that
would take an individual a lot of effort to compose via direct investment. Such fund-portfolios are
created based on target risks and return characteristics.

Question: 2894

Each individual owning shares in an investment company's fund ________.

A. owns the right to change the fund composition


B. owns a percentage of the investment company's total portfolio
C. owns a few individual investments within the fund
D. owns a percentage of the investment company

Answer: B

Explanation:

Because the fund is financed and owned by a pool of investors, each investor will be entitled to a
portion of the earnings generated by the fund, although no one completely owns the fund. The
investment company usually manages the fund composition.

Question: 2895

Management fees are charged

A. only by low-load and 12b-1 plan open-end investment companies.


B. only be load open-end investment companies.
C. by all investment companies.
D. only by no-load open-end investment companies.

Answer: C

Explanation:

All investment companies charge annual management fees to compensate the managers of the fund.
These fees are typically 0.25 to 1 percent of average net assets of the fund and tend to decline with
the size of the fund.

Question: 2896

The majority of venture capital funds are managed by

A. investment companies.

Page | 1110
B. corporations.
C. independent and private concerns.
D. small business investment companies.

Answer: C

Explanation:

In 1994, independent firms managed approximately 70 percent of the venture capital market.

Question: 2897

The principle of ________ says that people do not really buy or rent real estate per se but, instead,
judge properties as different sets of benefits and costs.

A. substitution
B. replacement
C. equivalency
D. surrogation

Answer: A

Explanation:
Because of the principle of substitution, an analysis of supply should not limit potential competitors
to geographically and physically similar properties.

Question: 2898

About three-quarters of mutual funds are more than

A. 70 percent diversified.
B. 30 percent diversified.
C. 60 percent diversified.
D. 90 percent diversified.
E. 50 percent diversified.

Answer: D

Explanation:

One of the great advantages of mutual funds is the diversification they provide. Small investors do
not have the resources to acquire 100 shares of 10 or 15 different shares, but can instead receive the
same diversification by purchasing a mutual fund. Typically about three-quarters of mutual funds
have a correlation above 0.90 with the market, meaning that they are more than 90 percent
diversified.

Question: 2899

Which best describes venture capital?

A. Venture capitalists exert control over the entrepreneur in order to achieve high returns.
B. Venture capital is an extremely risky investment, whose returns are usually negative.
C. Venture capital is method of equity financing whereby the entrepreneur loses majority ownership
of his company.

Page | 1111
D. Venture capital is a method to take control over a young company.
E. Venture capital is more than just capital, it is a process that fuels the growth of the venture.

Answer: E

Explanation:

Venture capital is the process forces the entrepreneur to hire a management team. The venture
capital firms have significant representation on the board of directors and play a major role in
shaping the
company's growth strategy.
The process also forces entrepreneurs to prepare a business plan to describe their objectives and to
specify their financial projections. Venture capitalists bring in a broad amount of experience to the
venture, thereby improving management's available resources. Ideally, the venture capital process
should develop better entrepreneurs and better managers.

Question: 2900

The NAV and the market price of ________ are almost never the same.

A. a low-load fund
B. an open-end fund
C. a no-load fund
D. a closed-end fund

Answer: D

Explanation:

The NAV and the market price of a closed-end fund are almost never the same, because closed-end
funds typically have market prices that has historically been a 5 to 20 percent discount to NAV.

Question: 2901

________ in real estate are the additions to a site, such as building, sidewalks, and various on-site
amenities.

A. Improvements
B. All of these answers.
C. Enhancements
D. Betterments
E. None of these answers

Answer: A

Explanation:
Also, building size is usually measured and expressed in terms of square footage.

Question: 2902

Which of the following is not an approach to real estate value, when performing an appraisal?

A. Market approach
B. Cost approach

Page | 1112
C. Income approach
D. Property transfer approach

Answer: D

Explanation:

Real estate appraisers look at each of these approaches, cost, comparative sales and income to arrive
at the best estimate of value.

Question: 2903

The primary use of venture capital is

A. to restructure a private firm's balance sheet debt.


B. to help private firms out of possible bankruptcy proceedings.
C. to finance production expansion.
D. all of these answers are correct.
E. for emerging growth business.

Answer: E

Explanation:

Venture capital is financing for privately held companies, typically in the form of equity and/or
longterm debt. Venture capital becomes available when financing from banks and public debt or
equity markets is either unavailable or inappropriate.
Much like mutual fund managers, venture capitalists raise capital from investors in order to manage
a portfolio of privately held investments.
The primary use of venture capital is for emerging growth business.

Question: 2904

A/An ________ imposes a front-end sales charge when buying the fund typically in the range of 3
percent.

A. low-load fund
B. no-load fund
C. oversubscribed fund
D. closed-end fund

Answer: A

Explanation:
A low-load fund imposes a front-end sales charge when buying the fund typically in the range of 3
percent instead of the typical load fund charge of 7 to 8 percent.

Question: 2905

Venture capital has proved to impact economic development in the following ways:

A. all of these are correct


B. job development
C. competitive adaptation

Page | 1113
D. formation of capital
E. none of these is correct
F. technological innovation

Answer: B,C,D,F

Explanation:
Several researchers have concluded that venture capital investments are critical to the economy in
terms of capital formation, local development, economic expansion, technological growth and
innovation, employment growth, and competitive adaptation in global markets.
Besides adding jobs, venture capital investments have resulted in development of medical diagnostic
and treatment equipment, and scientific improvements.
Venture capital investments have developed entrepreneurs and have contributed to innovation and
competitiveness.

Question: 2906

How will the dividend distribution and share prices react when a property's rent goes up in equity
REIT?

A. They will both stay the same.


B. Dividend distribution will go down, share prices go up.
C. They both will go up.
D. They both will go down.
E. Dividend distribution will go up, share prices go down.

Answer: C

Explanation:

With an equity REIT, when the property's rent goes up, so will the dividend distribution and normally
the share prices as well.

Question: 2907

The major difference between venture capital and conventional financing is that

A. the interest rate of venture capital is much higher than conventional financing.
B. venture capital is more than investing, it builds companies.
C. All of these answers are correct.
D. None of these answers.
E. venture capital will provide equity ownership, conventional financing will not.

Answer: B

Explanation:

The difference between venture capital and conventional financing is that venture capital is more
than investing and more than building personal wealth, it builds companies.

Question: 2908

Which of the following was not a guideline put forth by George Doriot?

Page | 1114
A. All of these were Doriot's guidelines.
B. Be realistic.
C. Treat the companies like you own children.
D. Run the business because the entrepreneur does not have the skills.
E. Don't pull the plug too soon.

Answer: D

Explanation:

George Doriot had guidelines for venture capitalists, which were:


1. Don't try to run the business. The entrepreneur knows best.
2. Treat the portfolio companies like your own children. Kick them every now and then if they need
it, but be fair.
3. Be a help. Be there when needed. Be an ear, a sounding board, an alter ego, and a crying towel.
Do what is necessary to allow the portfolio company the opportunity to achieve the potential you
originally thought you saw was there. Don't pull the plug too soon.
4. Be realistic. Don't get swept away. It is very easy to listen to an entrepreneur and start getting fired
up. Entrepreneurs are persuasive people.

Question: 2909

In valuing real estate, the most popular income approach is called ________.

A. retrospective value
B. the cost approach
C. none of these answers
D. direct capitalization
E. the comparative sales approach
F. depreciation method

Answer: D

Explanation:

The direct capitalization approach =


Market Value = Annual NOI / Market Capitalization Rate

Question: 2910

The NAV of an investment company equals the ________ of all its assets ________ the number of
fund shares outstanding.

A. total market value; multiplied by


B. total market value; divided by
C. the per-share value; divided by
D. the per-share value; multiplied by

Answer: B

Explanation:
Net Asset Value (NAV) = (Total market value of all assets/Number of outstanding fund shares)

Question: 2911

Page | 1115
Which is a most correct statement concerning companies that receive venture capital?

A. The company has little control over its own future.


B. The success rate is about the same for companies that receive venture capital funding and for
those that do not.
C. None of these statements are correct.
D. The success rate is much higher for companies that do not receive funding because entrepreneurs
do not have to deal with undue control of venture capitalists.
E. These companies are much more likely to succeed than companies that do not receive such
funding.

Answer: E

Explanation:

Companies that receive venture capital are far more likely to succeed than companies that do not
receive such funding. Studies show that there is only a 10% to 20% failure rate for investments in a
venture capitalist's portfolio, as compared to a 10%-20% success rate for all new companies.

Question: 2912

Mutual funds have

A. sometimes maintained the stability of their correlation with the market.


B. generally maintained the stability of their correlation with the market.
C. rarely maintained the stability of their correlation with the market.
D. tended to decrease their level of diversification over time.

Answer: B

Explanation:

Mutual fund managers rarely change the makeup of reasonably well diversified portfolios
substantially. Studies have also found that funds tend to remain consistent in their risk class. This
consistency allows investors to maintain their diversification and desired risk class.

Question: 2913

Which of the following statements is true?

A. One-half of all mutual funds are 95% or more diversified, providing one of their most important
benefits. Studies have also found that most mutual funds have maintained the stability of their
correlation with the market, and of their risk-adjusted returns.
B. Three-quarters of all mutual funds are 90% or more diversified, providing one of their most
important benefits. Studies also have found that most mutual funds have maintained the stability of
their correlation with the market, and of their risk class.
C. Three-quarters of all mutual funds are 95% or more diversified, providing one of their most
important benefits. Studies also have found that most mutual funds have maintained the stability of
their correlation with the market, but not of their risk class.
D. Three-quarters of all mutual funds are 80% or more diversified, providing one of their most
important benefits. But studies also have found that most mutual funds have not maintained the
stability of their correlation with the market, or of their risk class.

Page | 1116
Answer: B

Explanation:

Diversification is one of the main benefits of mutual funds. Most funds are highly diversified, and
tend to keep their level of diversification with the market. They also tend to maintain the same risk
class, if not consistent risk-adjusted returns.

Question: 2914

The management of the portfolio of securities held by an investment company is usually handled by
________.

A. the board of directors


B. the CEO or the CFO
C. a separate investment management company
D. a group of elected shareholders

Answer: C

Explanation:

The management of the portfolio of securities held by an investment company is usually handled by
a separate investment management company which is hired by the board of directors.

Question: 2915

What is the major difference between venture capitalists and other types of portfolio managers?

A. Venture capitalists only invest in start-up companies, while other types of portfolio managers do
not.
B. Portfolio managers do not become too involved with the companies they invest in, while venture
capitalists do.
C. Portfolio managers cannot exert control over a company in the way a venture capitalist can.
D. There are no major differences between the two.
E. Venture capitalists require a much higher rate of return over the life of the investment.

Answer: B

Explanation:
The major difference between venture capitalists and other types of portfolio managers is that
portfolio managers do not become too involved with the companies they invest in because they may
have to liquidate positions quickly. This is the direct opposite of venture capitalists who must
establish more personal relationships with the entrepreneurs in whom they invested over much
longer time periods.

Question: 2916

If the discount relative to the NAV of a closed-end investment company ________ during the holding
period, the investor will likely experience ________ excess returns.

A. does not change; zero


B. increases; positive
C. declines; positive

Page | 1117
D. declines; negative

Answer: C

Explanation:

It is intuitive that if the discount increases, the share price has decreased relative to the NAV and the
investor loses because he had originally bought the shares at a higher price. The analysis of these
discounts remains a major question in modern finance.

Question: 2917

The "family of funds" approach of investment companies ________.

A. promotes flexibility for the investor


B. improves the net risk-return exposure
C. all of these answers
D. discourages investors from switching from one fund to the other
E. consistently outperforms the market

Answer: A

Explanation:
The "family of funds" approach of investment companies promotes flexibility and increases the total
capital managed by the investment firm. It allows investors to switch among funds as economic or
personal conditions change.

Question: 2918

________ financing is one of the early stages of financing in venture capital.

A. Mezzanine
B. Asset-backed
C. Turnaround
D. Bridge
E. none of these answers.
F. Seed

Answer: F

Explanation:

There are three early stages of financing:


1. Seed financing. This is capital (typically less than $50,000) that is provided at the "idea" stage,
which goes for product development and market research.
2. Start-up financing. This capital is used in product development and initial marketing for firms in
business under one year and has not sold their product commercially.
3. First-stage financing. This is capital provided to initiate commercial manufacturing and sales.

Question: 2919

________ are funds that continue to sell and repurchase shares after their initial public offerings.

A. Unrestricted public funds

Page | 1118
B. Open-end investment funds
C. Closed-end investment funds
D. No-load closed-end funds

Answer: B

Explanation:

Open-end investment funds or mutual funds continue to sell and repurchase shares after their initial
public offering. The total number of outstanding fund shares can change with time.

Question: 2920

Annual management fees compensating the professional managers typically vary from ________ of
the average net assets of the fund.

A. two to five percent


B. one quarter to one percent
C. one to two percent
D. one half to one percent

Answer: B

Explanation:

Annual management fees compensating the professional managers typically vary from one quarter
to one percent of the average net assets of the fund; with a larger (dollar value) size fund charging a
lower percentage.

Question: 2921

The investment management company which handles the portfolio of an investment company is
hired by ________.

A. the investment company's board of directors


B. the shareholders in the investment company
C. none of these answers
D. an advisory firm

Answer: A

Explanation:
The investment management company which handles the portfolio of an investment company is
hired by the investment company's board of directors.

Question: 2922

The majority of venture capital investors are

A. endowments.
B. insurance companies.
C. individuals.
D. corporations.
E. pension funds.

Page | 1119
Answer: E

Explanation:

In 1994, pension funds made up approximately 45 percent of the venture capital investors.

Question: 2923

Money market funds attempt to ________.

I. provide current income and safety of principal


II. provide liquidity
III. hedge the investor's risk
IV. provide long-term capital gain

A. I & III
B. I & IV
C. I & II
D. II only

Answer: C

Explanation:

Money market funds attempt to provide current income with some safety of principal and some
liquidity by investing in a diversified portfolio of short-term securities.

Question: 2924

Name the fee charged by a fund that is typically .25 to 1.00 percent of the average net assets?

A. low-load
B. commissions
C. 12b-1 plan
D. deferred sales load
E. no-load
F. management fees

Answer: F

Explanation:

The fee, to compensate professional managers, is on a sliding scale that declines with the size of the
fund.

Question: 2925

Which of the following type of REIT's make construction loans to real estate investors?

A. Equity REIT's
B. Equity REIT's & Hybrid REIT's
C. Hybrid REIT's
D. Mortgage REIT's & Hybrid REIT's

Page | 1120
E. Mortgage REIT's

Answer: D

Explanation:

Mortgage - these make construction loans to real estate investors.

Hybrid - these invest in both properties, construction and real estate mortgage loans.

Question: 2926

Which is a characteristic of an open-end investment company?

A. sells shares at a discount to NAV


B. its stock trades on the secondary market
C. sells no further shares
D. market price of its shares is determined by supply and demand
E. you receive the NAV price of any share sales

Answer: E

Explanation:

When buying or selling shares of an open-end fund, an investor pays or receives the net asset value
price of the fund plus or minus any possible loads.

Question: 2927

The total market value of stocks in an investment company's portfolio divided by total outstanding
shares (with no further transactions) is ________.

A. 12b-1 amount
B. forward earnings per share
C. market value of an open-end investment company
D. market value of a closed-end fund
E. net asset value

Answer: E

Explanation:

The net asset value (NAV) equals the total market value of all the funds assets divided by the number
of fund shares outstanding.

Question: 2928

________ is a position in which, if a property's return is in excess of its debt cost, the investor's
return will be increased to a level well above what could have been earned from an all-cash real
estate deal.

A. Positive leverage
B. None of these answers
C. Excess financing

Page | 1121
D. Negative leverage

Answer: A

Explanation:
Remember that, in real estate, the use of debt financing to purchase a piece of property will affect its
risk-return parameters.

Question: 2929

The direct capitalization approach equals:


Market Value = ________ / Market Capitalization Rate

A. Monthly Gross Operating Income


B. Annual NOI
C. Annual Gross Operating Income
D. Monthly NOI

Answer: B

Explanation:

The direct capitalization approach =


Market Value = Annual NOI / Market Capitalization Rate

Question: 2930

A closed-end investment company

A. operates like any other public firm; its stock trades on the secondary market, and the price of the
stock is determined by supply and demand. Most closed-end investment companies do not offer
more shares or repurchase shares after the initial public offerings.
B. continues to sell and repurchase shares after their initial public offerings. It sells shares at the net
asset value with or without sales charges, and repurchases shares at the net asset value plus a
redemption fee.
C. is a type of mutual fund. Most closed-end mutual funds continue to sell and repurchase shares
after their initial public offerings. They sell shares at the net asset value with or without sales
charges, and repurchase shares at the net asset value with or without redemption fees.
D. is a type of mutual fund. It operates like any other public firm; its stock trades on the secondary
market, and the price of the stock is determined by supply and demand. Most closed-end investment
companies do not offer more shares or repurchase shares after the initial public offerings.

Answer: A

Explanation:
The only way to invest in a typical closed-end investment company after its initial public offering is to
purchase its shares on the secondary market. Many such companies trade their shares on the NYSE.
Over the long-run, shares of closed-end investment companies tend to trade at a 5 to 20% discount
to their net asset value (NAV).

Question: 2931

A closed-end investment company

Page | 1122
A. is a type of mutual fund.
B. has stock that trades on a secondary market.
C. typically repurchases shares on demand.
D. will typically offer more share issues after the initial share offering.

Answer: B

Explanation:

A mutual fund is an open-end investment company. A closed-end investment company is the other
type of investment company. Its stock trades on a secondary exchange, and the stock price is
determined by market supply and demand. The company's asset is the portfolio that it invests in
securities.

Question: 2932

If the average annual after tax cash flow was $5,101, the after tax net proceeds from sales was
$89,514, the initial equity was $60,000 and n was 5, then what is the approximate yield?

A. 15.7%
B. 15.2%
C. 16.9%
D. 14.7%

Answer: D

Explanation:

Approximate yield =
[($5,101 + (($89,514 - $60,000) / 5)] / (($89,514 + $60,000) / 2) = 14.7%

Question: 2933

The ________ for a share of a load fund equals the NAV of the share plus a sales charge.

A. expected price
B. redemption price
C. bid price
D. offering price

Answer: D

Explanation:

A load fund is typically quoted by its NAV and an offering price, which is the NAV plus a sales charge.

Question: 2934

Open-end mutual funds

A. sell their shares at their NAV. In contrast, closed-end investment companies typically charge a
sales fee in addition to the NAV for shares at their initial public offerings. The sales fee tends to be
about 10 to 11% of NAV.
B. sometimes sell their shares at their NAV plus a sales fee. Mutual funds that do charge sales fees

Page | 1123
are known as load open-end funds. The sales fee tends can range from 3.0 to 8.0% of NAV.
C. sell their shares at their net asset value (NAV). In contrast, closed-end investment companies
typically charge a sales fee in addition to the NAV for shares at their initial public offerings. The sales
fee tends to be about 4 to 4.5% of NAV.
D. almost always sell their shares at their NAV plus a sales fee. Mutual funds that do charge sales fees
are known as load open-end funds. The sales fee tends to be 4 to 4.5% of NAV.
E. sell their shares at their NAV. In contrast, closed-end investment companies typically charge a sales
fee in addition to the NAV for shares at their initial public offerings. The sales fee tends to be about
7.5 to 8.0% of NAV.

Answer: B

Explanation:

Some open-end funds charge sales fees for sales shares. The fees are typically 3.0 to 8.0% of NAV.
These load funds generally do not charge redemption fees. No-load funds sell shares at their NAV,
but some of them have small redemption fees of about 0.5% of NAV.

Question: 2935

When investors are bullish about the stock market,

A. money market funds tend to loose popularity.


B. money market funds tend to grow in popularity.
C. money market fund volatility decreases.
D. returns on money market funds tend to decrease.

Answer: A

Explanation:
When investors are bullish about the stock market, they tend to withdraw funds from money market
funds in order to invest in stocks or stock funds. When investors are cautious about the stock market,
they tend to shift funds from stocks into safer money market funds.

Question: 2936

What stage of venture capital investments has provided the highest standard deviation of mean
returns?

A. balance
B. later and balance are identical
C. seed
D. early
E. later

Answer: D

Explanation:

The early stage has provided a 24.3% standard deviation from 1981-1996, followed by the later stage
(17.7%), seed stage (17.6%), and balance stage (14.4%).

Question: 2937

Page | 1124
Foreign funds are those that invest

A. only outside of the developed world.


B. completely or partially outside of the U.S.
C. only outside of the U.S.
D. only in emerging markets.

Answer: B

Explanation:

The two kinds of foreign funds are international funds and global funds. International funds invest
only in non-U.S. securities. Global funds invest in both U.S. and non-U.S. securities. Investment in
foreign funds by American investors provides them with an added degree of diversification.

Question: 2938

________ invest a pool of funds belonging to many individuals in a portfolio of individual


investments such as stocks and bonds.

A. Public-listed companies
B. Partnerships
C. Stock exchanges
D. Investment companies

Answer: D

Explanation:

Investment companies create a portfolio of individual market instruments such as stocks and bonds
and market the shares of this portfolio to the individual investor.

Question: 2939

Investment companies tend to be

A. corporations that are managed by separate investment management companies. About 20% of
funds, however, are managed by the internal management, rather than by outside companies.
B. corporations that are self-managed. About 30% of funds, however, are managed by separate
investment management companies that are chosen by their board of directors based on previous
management experience.
C. corporations that are managed by separate investment management companies. The board of
directors of an investment company typically choose an investment management company to
manage the fund. That company is very often the owner of the investment company.
D. corporations that are managed by separate investment management companies. The board of
directors of an investment company typically choose an independent investment management
company to manage the fund.

Answer: C

Explanation:

Investment management companies are corporations that are managed by investment management
companies that tend to be their owners. A typical scenario would involve an investment

Page | 1125
management company forming an investment company, and the board of directors of that
investment company proceeding to hire the parent investment management company to manage
the new firm.

Question: 2940

________ is a position in which, if a property's return below its debt cost, the investor's return will be
less than from an all-cash real estate deal.

A. Negative leverage
B. Positive leverage
C. Excess financing
D. None of these answers

Answer: A

Explanation:
Remember that, in real estate, the use of debt financing to purchase a piece of property will affect its
risk-return parameters.

Question: 2941

Load open-end funds charge

A. sales fees of about 7.5 to 8% of NAV.


B. sales and redemption fees of about 1 to 2% of NAV.
C. sales and redemption fees of about 7.5 to 8% of NAV.
D. sales fees of about 10 to 15% of NAV.
E. sales fees of about 1 to 2% of NAV.

Answer: A

Explanation:
Load funds charge a sales fee for share sales that is typically 7.5 to 8% of the NAV. Share prices are
quoted with a NAV and an offering price. Load funds generally do not charge a redemption fee, but
some no-load funds do charge a small one of about half a percent of NAV.

Question: 2942

If an investment company originally issued 0.5 million shares for $30, then issued a further 0.1
million shares for $40, and its portfolio of investments now has a market value of $27 million and a
book value of $19 million, what is its net asset value (NAV)?

A. Not enough information


B. $31.67
C. $27 million
D. $45
E. $19 million

Answer: D

Explanation:

An investment company sells shares in itself, and uses the proceeds to invest in a portfolio of

Page | 1126
individual investments such as stocks and bonds. The NAV is equal to the net value of the investment
company's assets divided by the number of its shares outstanding. In this question the NAV is equal
to $27 million / (500,000 + 100,000) = $45.

Question: 2943

The direct capitalization approach equals:


Market Value = Annual NOI / ________

A. Market Capitalization Rate


B. Market Discount Rate
C. None of these answers
D. Subject Property Capitalization Rate
E. Subject Property Discount Rate

Answer: A

Explanation:
The direct capitalization approach =
Market Value = Annual NOI / Market Capitalization Rate

Question: 2944

Which is not a true statement concerning the empirical evidence of mutual fund performance?

A. sub-par performance is associated with low expense ratios


B. managers can forecast large changes in the market
C. all of these answers are false
D. most fund managers achieved gross returns lower than the DJIA
E. risk is not consistent with fund objectives
F. all of these answers are true

Answer: C

Explanation:

All of these are false. The opposite is true in all answers.

Question: 2945

Given the following:


Return on Investor's Equity with 80% Financing = 50%
Return on Investor's Equity with 0% Financing = 10%
The investor is experiencing the effects of ________.

A. negative leverage
B. none of these answers
C. cannot be determined by the information given
D. positive leverage

Answer: D

Explanation:

Page | 1127
Remember that, in real estate, the use of debt financing to purchase a piece of property will affect its
risk-return parameters. In this case, leverage has a positive effect.

Question: 2946

According to the framework for real estate analysis, one determinant of value is demand. Which of
following is not a subset of "demand"?

A. Economic base
B. Target market potential
C. Mortgage financing conditions
D. Sources of competition
E. Tenant preferences
F. None of these answers

Answer: D

Explanation:

Sources of competition, along with market structure and inventorying competitors are SUPPLY
determinants of value.

Question: 2947

What is not a characteristic of a closed-end investment company?

A. its stock trades on the regular secondary market


B. the NAV is computed twice daily
C. NAV is based on current market prices
D. the NAV and market price are usually the same
E. its market price is determined by supply and demand

Answer: D

Explanation:

The market price is determined by the relative supply and demand for the investment company stock
in the public secondary market. When trading these shares an investor will pay or receive this market
price plus or minus a commission. The market price has historically been 5 to 20 percent below the
NAV.

Question: 2948

Market index funds

A. attempt to diversify out of securities.


B. have management costs similar to those of regular mutual funds.
C. attempt to outperform the market.
D. None of these answers.
E. have very high correlations with market indexes.

Answer: E

Explanation:

Page | 1128
Market index funds are closely matched in composition to specified market indexes such as the S&P
500. For this reason, a market index fund typically has correlation rates in excess of 0.99 with its
chosen index. Because it only attempts to replicate the index, its management costs are very low.

Question: 2949

Which of the following is not demographic figure?

A. Person's lifestyle
B. Person's age structure
C. Person's occupation
D. Person's marital status
E. Person's gender

Answer: A

Explanation:
Demographics - Age structure, occupation, gender, marital status. A person's lifestyle would be part
of psychographics.

Question: 2950

If an investment company issued 1 million shares for $20, and its portfolio of investments is now
worth $38 million, what is its net asset value (NAV)?

A. Not enough information


B. $20 million
C. $38
D. $38 million
E. $20

Answer: C

Explanation:

An investment company sells shares in itself, and uses the proceeds to invest in a portfolio of
individual investments such as stocks and bonds. The NAV is equal to the net value of the investment
company's assets divided by the number of its shares outstanding. In this question the NAV is equal
to $38 million / 1 million = $38.

Question: 2951

An examination of the relationship between performance and the expense ratio indicated that
________.

A. good performance was associated with low expense ratios


B. bad performance was associated with low expense ratios
C. good performance was associated with high expense ratios
D. there was no association between performance and expense ratios

Answer: A

Explanation:

Page | 1129
An examination of the relationship between performance and the expense ratio indicated that good
performance was associated with low expense ratios; in one way, it is because management expense
is money not invested in the portfolio of shares and/or bonds. Expense is necessary however when
one needs to shift to better performing stocks according to market changes.

Question: 2952

Raw land is an example of a(n)

A. income property.
B. all of these answers.
C. commercial property.
D. speculative property.

Answer: D

Explanation:

A speculative property is generally raw land and real estate investment properties that are expected
to provide returns primarily from appreciation in value.

Question: 2953

The following has its stock traded on the regular secondary market:

A. investment management company


B. load fund
C. closed-end investment company
D. no-load fund
E. open-end investment company

Answer: C

Explanation:

The market price of its shares is determined by supply and demand and typically offers no further
shares and does not repurchase the shares on demand.

Question: 2954

________ pertains to those characteristics that describe people's mental dispositions, such as
personality, lifestyle, and self-concept.

A. Characgraphics
B. Socialgraphics
C. Psychographics
D. Demographics

Answer: C

Explanation:

By comparing demographic and psychographic trends to the features of a property, you can judge
whether it is likely to gain or lose favor among potential buyers or tenants.

Page | 1130
Question: 2955

Given the following:


Return on Investor's Equity with 80% Financing = -75%
Return on Investor's Equity with 0% Financing = -15%
The investor is experiencing the effects of ________.

A. cannot be determined by the information given


B. negative leverage
C. positive leverage
D. none of these answers

Answer: B

Explanation:

Remember that, in real estate, the use of debt financing to purchase a piece of property will affect its
risk-return parameters. In this case, leverage has a negative effect.

Question: 2956

Balanced funds invest

A. in a broad market index.


B. in a wide variety of common stocks.
C. in both common stock and fixed-income securities.
D. primarily in bonds.

Answer: C

Explanation:

Balanced funds diversify outside the stock market by combining investments in common stock with
those in fixed-income securities such as government and corporate bonds. The ratio of stocks to
fixed-income securities that the fund will invest in will be stated in its prospectus.

Question: 2957

The offering price for a share of a ________ equals the NAV of the share plus a sales charge.

A. closed-end investment company


B. 12b-1 plan fund
C. no-load fund
D. load fund

Answer: D

Explanation:

The offering price for a share of a load fund equals the NAV of the share plus a sales charge, which
typically is 7.5 to 8.0 percent of the NAV.

Question: 2958

Page | 1131
Which type of environment relates to the restrictions on use that apply to nearby properties?

A. Socioeconomic
B. Fiscal
C. Legal
D. Aesthetic

Answer: C

Explanation:

Legal environment relates to the restrictions on use that apply to nearby properties.

Question: 2959

Managed investment companies are appealing alternatives to individual investing because:

A. few individual investors outperform the aggregate market averages


B. many investors are not well informed
C. the risk-return ratio is higher in mutual funds
D. they offer a risk-free rate of return

Answer: A

Explanation:
Efficient market studies have indicated that few investors outperform the aggregate market
averages. This makes Managed investment companies a good alternative to direct investment
because most of these companies offer a wide range of investment possibilities with a range of risk
and return characteristics.

Question: 2960

Management fees for open-end mutual funds tend to be

A. about 0.5 to 1.5% of NAV. The management fees tend to be larger for large funds than for small
funds because of the quickly increasing bureaucratic costs associated with larger funds. This has
encouraged the existence of thousands of small mutual funds.
B. about 1 to 2% of net asset value (NAV). The management fees are on a sliding percentage scale
that increases with the size of the fund.
C. about 1 to 1.5% of NAV. The management fees tend to be larger for large funds than for small
funds because of the quickly increasing bureaucratic costs associated with larger funds. This has
encouraged the existence of thousands of small mutual funds.
D. about 0.4 to 1.2% of NAV. The management fees tend to be smaller for large funds than for small
funds because of the substantial economies of scale that exist in managing financial assets.
E. about 0.25 to 1% of NAV. The management fees are on a sliding percentage scale that decline with
the size of the fund.

Answer: E

Explanation:

Most management fees decline with the size of a fund. For example, a fund with assets of $500
million might charge management fees of 1%, a fund with assets of $1 billion might charge 0.75%,

Page | 1132
and a fund with assets of $6 billion might charge 0.25%.

Question: 2961

One of the major benefits of mutual funds is ________.

A. all of these answers are major benefits


B. that they provide instant diversification
C. that they are easily bought and sold
D. that they have predictable returns
E. that they are low-risk

Answer: B

Explanation:

One of the major benefits of mutual funds is that they provide instant diversification. They are not
necessarily low-risk, as there are many aggressive, speculative funds. Not all funds have high
liquidity.

Question: 2962

Funds distributed by a sales force are

A. less common than those distributed through direct marketing.


B. typically no-load funds.
C. typically load funds.
D. rare.

Answer: C

Explanation:

Most funds are either distributed by a sales force (such as Merrill Lynch brokers, commission-based
financial planners, or dedicated sales forces) or by direct marketing, with those distributed by a sales
force being the more common. Funds distributed by a sales force typically charge a sales fee in order
to compensate the salespeople.

Question: 2963

If 50 units are sold at $7,500 to buy a piece of property for $1,000,000, how much money will be
coming from the partners?

A. $625,000
B. $1,000,000
C. $375,000
D. $7,500

Answer: C

Explanation:

Partners will supply (50 x 7,500) = $375,000.

Page | 1133
Question: 2964

________ attempt to match the composition of the market.

A. Balanced funds
B. Growth funds
C. Composite funds
D. Market index funds

Answer: D

Explanation:

They are funds that attempt to match a benchmark index of the market.

Question: 2965

Compute the Net Asset Value given the following total portfolio:

Stock A - 1000 shares - market value = $ 12,000


Stock B - 5000 shares - market value = $ 25,000
Stock C - 1000 shares - market value = $ 2,000

A. 6.1667
B. 6.5841
C. 5.5714
D. 5.3241
E. 7.4456

Answer: C

Explanation:

NAV equals the total market value of all assets divided by the number of fund shares outstanding.
($12,000 + $25,000 + $2,000) / (1000 + 5000 + 1000) = $5.5714

Question: 2966

________ diversify outside the stock market by combining common stock with fixed-income
securities, including government bonds, corporate bonds, convertible bonds, or preferred stock.

A. Balanced funds
B. None of these answers
C. Fixed income funds
D. Bond funds

Answer: A

Explanation:
Balanced funds diversify outside the stock market by combining common stock with fixed-income
securities.

Question: 2967

Page | 1134
Net asset value (NAV) of an investment company is equal to

A. the net value of its assets.


B. the net value of its assets divided by the number of its shares outstanding.
C. the net value of its assets divided by the number of shares issued.
D. the net value of its assets plus depreciation expense.
E. the net value of its assets plus depreciation expense, divided by the number of shares outstanding.

Answer: B

Explanation:

An investment company sells shares in itself, and uses the proceeds to invest in a portfolio of
individual investments such as stocks and bonds. If such a company sold 1 million shares for $50
dollars each, it would invest $50 million (minus commissions) into things like government bonds or
speculative stocks. If the value of this portfolio increased to $60 million, the NAV would increase to
$60.

Question: 2968

A fund that invests exclusively in Russian securities would be

A. a global fund. Like most international and global funds, this one would probably be a load open-
end fund.
B. a global fund. Like most international or global funds, this one would probably be a load open-end
fund. There has, however, been a recent increase in the number of international funds that closed-
end.
C. an international fund. Like most single country and emerging market funds, this one would
probably be a closed-end fund. Most international and global funds, however, are load open-end
funds.
D. an international fund. Like most international and global funds, this one would probably be a no-
load open-end fund. There has, however, been a recent increase in the number of international
funds that are closed-end.
E. an international fund. Like most single country and emerging market funds, this one would
probably be a closed-end fund. Most international and global funds, however, are no-load open-end
funds.
F. an international fund. Like most international and global funds, this one would probably be a load
open-end fund. There has, however, been a recent increase in the number of international funds that
are closed-end.
G. a global fund. Like most international or global funds, this one would probably be a closed-end
fund.

Answer: E

Explanation:

Funds that invest in non-U.S. securities are referred to as foreign funds. Foreign funds can be divided
into two categories: (1) international funds, which invest only in non-U.S. stocks, (2) and global
funds, which invest in both U.S. and non-U.S. securities. Most global and international funds are no-
load open-end funds, although there are still a significant number of closed-end funds. Closed-end
funds have the advantage of not being subject to major investor liquidations that would require the
sale of stocks on an illiquid foreign stock exchange.

Topic 8, Equity Investment

Page | 1135
Question: 2969

Name the first step in estimating the expected value of an industry.

A. Estimate the expected payout ratio for the industry.


B. Estimate the earnings per share for the industry.
C. Estimate the sales per share for the industry.
D. Estimate the expected industry P/E ratio.
E. Estimate the expected growth in dividends for the industry.

Answer: B

Explanation:
The second step of the two-step process is to estimate the expected industry P/E ratio. Multiplying
the expected earnings per share by the expected earnings multiplier gives the expected ending value
for the industry.

Question: 2970

Assume the following information about a stock market series:

Observed beginning value: 1677


Anticipated ending value: 1890
Expected dividends during the period: $16.36
Required rate of return: 19.50%

Using this information, what is the expected rate of return for this index? (Assume a one-year
holding period.)

A. 10.40%
B. 14.79%
C. None of these answers is correct.
D. 12.14%
E. 11.73%

Answer: C

Explanation:
The anticipated rate of return for this stock market series is found as 13.68%. Thus, none of these
answers is correct.

To calculate the expected rate of return for a stock market series, the following information must be
known: The beginning value for the series, the anticipated ending value for the series, and the
amount of any dividends and/or distributions during the period.
Once this information has been determined, the expected return on a stock market index can be
found by employing the following equation: {E(R) = [(EV - BV + Div) / BV]}. Where E(R) = the expected
return on the stock market series, EV = the anticipated ending value for the series, BV = the observed
beginning value for the series, and Div = the amount of any dividends paid during the period.

In this example, all of the necessary information has been provided and the calculation of the
expected return on this stock market series is found as follows: {E(R) = [$1890 - $1677 + $16.36] /
1677} = 13.68%.

Page | 1136
This is significantly less than the required rate of return. Assuming that both the ending value and
dividend figure is accurate, investment in this stock market series is likely not warranted.

Question: 2971

Which of the following are beliefs espoused by technical analysts?

A. Popularity of trading rules will eventually eliminate the value of the technique.
B. No one can consistently get new information and process it correctly and quickly.
C. All of these answers.
D. Fundamental analysts can only achieve superior returns if they obtain information before other
investors.

Answer: C

Explanation:
Under the heading 'advantages of technical analysis': there is a discussion on how technical analysis
relies more on tracking market movement than assimilating intrinsic market data, which is
advantageous perhaps because there is too much data accumulating too fast too be any help in
proficient forecasting. But technicians also recognize that new market tracking techniques have short
shelf lives.

Question: 2972

Milton Manufacturing has an outstanding issue of preferred stock that pays a $1.15 annual dividend.
This dividend is not assumed to change in the future, and similar investments are currently
warranting a13.75% per year return. What is the value of Milton Manufacturing's preferred stock?
Further, does this value represent a perpetuity or a finite series of cash flows?

A. $33.33; finite series of cash flows


B. $10.99; perpetuity
C. $8.36; perpetuity
D. $10.99; finite series of cash flows
E. $8.36; finite series of cash flows

Answer: C

Explanation:
Preferred stock is commonly valued as a perpetuity using the following equation: {P0 = [d1 / k]}

Where: P0 = the price of the preferred stock at time 0, d1 = the annual dividend at t = 1, and k = the
required rate of return.

In this example, the dividend is provided as an annual figure, so all of the necessary information has
been given. The calculation of the value of this preferred stock is found as follows:

{P0 = [$1.15 / 0.1375] = $8.36.

Preferred stock is commonly valued as a perpetuity because there is no finite conclusion to the
projected series of cash flows for a preferred stock. Unlike a bond, whose cash flows are
characterized by a finite lifespan (i.e. the cash flows of a bond cease at maturity), the cash flows
(dividends) produced by a preferred stock could theoretically last forever.

Question: 2973

Page | 1137
When estimating change in sales for a market series, change in sales is regressed against change in
________.

A. Earnings
B. Nominal GNP
C. None of these answers
D. Revenues

Answer: B

Explanation:
Regressing change in sales with change in GNP basically to find a relationship between GNP and
sales, so that GNP growth rate can be used to estimate future sales changes.

Question: 2974

Composite Software, Inc. is anticipated to experience temporary supernormal growth of 40% per
year for the next two years. After this supernormal growth period has passed, the growth rate of
Composite Software is anticipated to experience a two-year transition phase of 25% per year growth.
Following this transition phase, the growth rate of Composite Software is expected to stabilize at 15%
annually. The Company currently pays a dividend of $0.10 per share, and the required rate of return
is 18% per year. What is the value of Composite Software common stock?

That answer is correct!


A. $6.62
B. $23.83
C. $14.15
D. $6.03
E. None of these answers is correct.

Answer: A

Explanation:
To determine the value of a common stock experiencing temporary supernormal growth, use the
following equation:

{V = {[d0 * (1 + gs)^1] / k} + {[d1 * (1 + gs)^2} + ... {dn * (1 + gs)^n} + {[dn * (1 + gs)^n * (1 + gn] / (k -
g)}/ (1 + k)^n}}

Where: V = the value of common stock at t0, d0 = the dividend at t0, d1 = the dividend at t1, dn = the
dividend at tn, gs = the supernormal rate of growth, gn = the normal rate of growth, n = the time
period "n", and k = the required rate of return.

In this example, there is a transitional growth period of two years, during which the growth rate of
Composite Software is expected to grow at 25% annually. This period will follow the two-year
supernormal growth period, and would be denoted as g subset t. The calculation of the value of this
common stock is illustrated as follows:

{V = {[$0.10 * (1.40)^1] / (1.18)} + {[$0.10 * (1.40)^2] / (1.18)^2} + {[$0.10 * (1.40)^2 * (1.25)^1] /


(1.18)^3} + {[$0.10 * (1.40)^2 * (1.25)^2] / (1.18)^4} + {{[$0.10 * (1.40)^2 * (1.25)^2 * (1.15)^1]/
(0.18 - 0.15)}/ (1.18)^4}

Which can be deduced to the following series of discounted cash flows:

Page | 1138
{V = [$0.118644 + $0.140764 + $0.149115 + $0.15796 + $6.055146] = $6.62}

Question: 2975

You have invested in a stock that has a dividend growth rate of 4%. It is expected to pay a dividend of
$4 per share next year. You expect to sell the stock after 3 years, for a capital gain of about $6 per
share. If your required rate of return is 8%, what price would you be ready to pay for the stock?

That answer is correct!


A. $75.04
B. $18.98
C. $14.18
D. $15.47

Answer: A

Explanation:
Be careful about the fact that $6 represents the capital gain on the stock, not the selling price. If you
buy the stock for a price, P, then the problem has in effect told you that the cash flows from the stock
are expected to be: $4 next year, $4 * 1.04 = $4.16 in year 2 and (P + $6 + $4 * 1.04^2) = $(P + 10.33)
in year 3. The present value of these cash flows at a discount rate of 8% per year is equal to P and this
equals

P = 4/1.08 + 4.16/1.08^2 + (P + 10.33)/1.08^3 = P/1.08^3 + 15.47. Solving for P gives P = 15.47/(1 -


1/1.08^3) = $75.04

Question: 2976

Which of the following is the correct order of the steps (from first to last) of the top-down, three-step
approach to valuation?

That answer is correct!


A. Analysis of the economy and security markets, analysis of alternative industries, and analysis of
individual firms and stocks
B. Analysis of alternative countries and regions, analysis of the economy and security markets, and
analysis of individual firms and stocks.
C. Analysis of the economy and security markets, analysis of alternative countries, and analysis of
individual firms and stocks
D. Analysis of alternative industries, analysis of the economy and security markets, and analysis of
individual firms and stocks

Answer: A

Explanation:
Analysis of alternative countries and regions is considered part of analysis of the economy and
security markets. The steps follow a top-down approach, with the most general category (the
economy) followed by more specific areas (industries and then individual firms).

Question: 2977

Consider the following annual growth forecasts for a common stock:

Growth in years 1-2 = 30%

Page | 1139
Growth in years 3-4 = 20%
Growth after year 4 = 15%

Assuming that the last dividend was $0.80 per share, and the required rate of return is 17.5% per
year, what is the value of this common stock?

That answer is correct!


A. $50.87
B. $43.59
C. $58.12
D. $47.05
E. None of these answers is correct.
F. $61.78

Answer: A

Explanation:
To determine the value of a common stock experiencing temporary supernormal growth, use the
following equation:

{V = {[d0 * (1 + gs)^1] / k} + {[d1 * (1 + gs)^2} + ... {dn * (1 + gs)^n} + {[dn * (1 + gs)^n * (1 + gn] / (k -
g)}/ (1 + k)^n}}

Where: V = the value of common stock at t0, d0 = the dividend at t0, d1 = the dividend at t1, dn = the
dividend at tn, gs = the supernormal rate of growth, gn = the normal rate of growth, n = the time
period "n", and k = the required rate of return.

In this example, there is a transitional growth period of two years, during which the growth rate of
Composite Software is expected to grow at 25% annually. This period will follow the two-year
supernormal growth period, and would be denoted as g subset t. The calculation of the value of this
common stock is illustrated as follows:

{V = {[$0.80 * (1.30)^1] / (1.175)^1} + {[$0.80 * (1.30)^2] / (1.175)^2} + {[$0.80 * (1.30)^2 * (1.20)^1]


/ (1.175)^3} + {[$0.80 * (1.30)^2 * (1.20)^2] / (1.175)^4} + {{[$0.80 * (1.30)^2 * (1.20)^2 * (1.15)^1]/
(0.175 - 0.15)}/ (1.175)^4}

Which can be deduced to the following:

{V = [$0.885106 + $0.979267 + $1.000102 + $1.021381 + $46.98352] = $50.87}

Question: 2978

Technicians believe that a high confidence index is

That answer is correct!


A. a bullish sign.
B. indicative of an approaching trough.
C. an unimportant sign.
D. indicative of an approaching peak.
E. a bearish sign.

Answer: A

Explanation:

Page | 1140
The confidence index measures the yield spread between high-grade bonds and a large cross section
of bonds. Some technical analysts believe that during periods of high confidence, investors are more
willing to invest in lower-quality bonds, thereby pushing down their yields, and increasing the
confidence index. A high index value is thus viewed as a bullish sign.

Question: 2979

The resistance level of a stock is the price at which the technician expects:

A. the stock price to break out of the falling trend channel.


B. a substantial supply of the stock.
C. a substantial demand for the stock.
D. an unstable trading volume.

Answer: B

Explanation:
The resistance level can be thought of as a temporary ceiling on the stock price in the sense that if
the price rises above this level, additional supply from investors trying to cash in will depress the
price below the resistance level.

Question: 2980

You have a 15-year bond that pays $500 every 6 months. The face value is $10,000. The required rate
of return is 10. What is the bond's value?

That answer is correct!


A. $10,000
B. $8,435
C. $12,000
D. none of these answers

Answer: A

Explanation:
Present value of interest payments at 10%: $500 x 15.3725 = $7,686
Present value of principal payment at 10%: $10,000 x .2314 = 2,314
$7,686 + 2,314 = 10,000

Question: 2981

Which of the following is an assumption of technical analysis?

A. Supply and demand is governed by both rational and irrational factors.


B. All of these choices are assumptions of technical analysis.
C. Changes in the market value of any good are determined solely by supply and demand
fluctuations.
D. Securities markets are weak-form inefficient.
E. Securities prices move in identifiable trends.

Answer: B

Explanation:
Technical analysis is primarily grounded on three major assumptions - (1.) the market value of any

Page | 1141
good is determined solely by supply and demand, (2.) supply and demand fluctuations are caused by
both rational and irrational factors, and (3.) the prices of individual securities and securities markets
move in trends, i.e. securities markets "price-in" information slowly. The third assumption of
technical analysis negates any belief in the weak form efficient market hypothesis, which assumes
that past performance data cannot be used to predict securities prices, and that the securities
markets "price-in" information instantly. This "incongruence" with all forms of the EMH is cited as
the greatest criticism of technical analysis.

Question: 2982

You are going to hold a stock for an infinite amount of time. The current dividend is $1 per share and
is expected to grow at 10% a year. Your long run required rate of return is 13%. Using the infinite
period dividend discount model calculate the value of the stock.

A. $40.01
B. $27.25
C. none of these answers
D. $38.89

Answer: C

Explanation:
g = .10 k = .13 Dividend = 1.10 x $1.00
V = 1.10/(.13 - .10) = $36.67

Question: 2983

Given that the beginning value on a stock is $530, expected earnings are $50, the dividend payout
ratio is 40%, and the required rate of return is 14%, what is the minimum expected ending value of
the stock that makes it a profitable investment?

A. $611.20
B. $584.20
C. $591.20
D. $604.20
E. Not enough information

Answer: B

Explanation:
Expected dividends equal 50 x 0.4 = $20. In order for a stock to be a good investment, its rate of
return should be equal to or greater than the required rate of return. The minimum ending value that
would make the stock investment in this question profitable is given by the equation (P2 + D) / P1 = 1
+ k, where P2 is the ending value, P1 is the beginning value, D is the expected dividend, and k is the
required rate return. Rearranging this yield P2 = ((k + 1) x P1) - D. In this question, the minimum
ending value is (1.14 x 530) - 20 = $584.20.

Question: 2984

Assume the following information about a publicly traded automobile manufacturer:

Revenue: $16,000,000
Cash flow: $1,700,000
Net worth per share: $14.55

Page | 1142
Current stock price: $30.25 per share
Number of common shares outstanding: 1,300,000

Using this information, what are the price-to-sales, price-to-book, and price-to-cash flow ratios,
respectively?

A. The answer cannot entirely be calculated from the information provided.


B. None of these answers is correct.
C. 1.18, 2.08, 23.13
D. 2.46, 0.48, 23.21
E. 2.46, 2.08, 23.13
F. 1.18, 2.08, 23.13

Answer: E

Explanation:
To calculate the price-to-sales ratio, divide the market price of a common stock by its sales-per share
figure. The equation for the price-to-sales ratio is as follows:

Price-to-sales ratio = [P0 / sales per share].

Incorporating the given information into this equation will yield the following:

Price-to-sales ratio = [$30.25 / ($16,000,000 / 1,300,000)] = 2.4578

The calculation of the price-to-book ratio involves dividing the market price of a common stock by its
net worth per share. The equation for the price-to-book ratio is as follows:

Price-to-book ratio = [P0 / net worth per share].

Where: net worth per share = (total assets - total liabilities) / # of common shares outstanding.

In this example, the net worth figure has been converted to a per-share basis, and the calculation of
the price-to-book ratio is straightforward:

Price-to-book ratio = ($30.35 / $14.55) = 2.0859

The calculation of the price-to-cash flow ratio involves dividing the market price of a common stock
by the cash-flow-per-share figure. The calculation of the price-to-cash flow ratio is as follows:

Price-to-cash flow = (P0 / cash flow per share)

Incorporating the given information into this equation will yield the following:

Price-to-cash flow = [$30.35 / (1,700,000 / 1,300,000)] = 23.132

Question: 2985

A firm issues debt to repurchase equity and at the same time, experiences an increase in its profit
margin. All else equal, using the Dividend Discount Model, its stock price ________.

A. is not affected
B. increases
C. decreases

Page | 1143
D. insufficient information given

Answer: B

Explanation:
Intuitively, it should be clear that the stock price rises since the profit margin has gone up and at the
same time, the percentage of equity holders has gone down. To see this mathematically, note that
the duPont system gives ROE = Profit margin * Total Asset Turnover * Financial Leverage. In the
present case, the ROE increases since profit margin and Financial Leverage have both increased. The
dividend growth rate, g, equals ROE*(1 - payout ratio). Hence, as ROE increases, the dividend growth
rate increases with a constant payout ratio. In the Dividend Discount Model, the stock price increases
as the dividend growth rate increases, all else equal.

Question: 2986

A junior financial analyst with Churn Brothers brokerage has been instructed to value shares of
Intelligent Semiconductor, a diversified technology company. A senior analyst at Churn Brothers has
provided the following information:

The required rate of return on equity is 15% per year

The senior analyst has predicted that shares of Intelligent will sell at a multiple of 25 times predicted
free cash flow to equity in four years.

The estimated free cash flows for each of the next four years are:

Year 1: $15,000,000
Year 2: $18,500,000
Year 3: $21,000,000
Year 4: $35,000,000

Intelligent Semiconductor has 1,000,000 shares of common stock outstanding.

Using this information, what is the value per share of Intelligent Semiconductor according to the free
cash flow to equity model?

A. $65.81
B. $561.13
C. $56.67
D. $650.34
E. The answer cannot be calculated from the information provided.
F. None of these answers is correct.

Answer: B

Explanation:

When determining the value of a common stock using the free cash flow to equity model, it is
necessary to determine three things:

1. The required rate of return on equity investments.


2. The estimated free cash flow to equity multiple at time "k."
3. The estimated free cash flows figures for the time periods leading up to "k."

Page | 1144
In this example, the calculation must begin with the discounting the free cash flow to equity figures
for each of the four years provided. These figures are discounted each period by the required return
on equity investments, and the final answer is converted to a per-share basis. This process is
illustrated below:

Year 1: ($15,000,000 / 1.15) / 1,000,000 shares outstanding = $13.04


Year 2: [$18,500,000 / (1.15)(1.15)] / 1,000,000 shares outstanding = $13.99
Year 3: [$21,000,000 / (1.15)(1.15)(1.15)] / 1,000,000 shares outstanding = $13.81
Year 4: [$35,000,000 / (1.15)(1.15)(1.15)(1.15)] / 1,000,000 shares outstanding = $20.01

Now that the free cash-flow-to-equity figures have been discounted and converted to a per-share
basis, the next step in the valuation process is to determine the value of the final cash flow, which is
defined as:

[(Free cash flow to equity multiple * Final free cash flow) / (1 + r)(1+r)...(1 + k)]

In the body of this question, we were given the anticipated multiple of free cash to equity that shares
of Intelligent Semiconductor will sell for at time period 4: 25 times. Imputing this information into
the terminal cash flow equation will yield the following:

{[25 * ($35,000,000 / 1,000,000 shares outstanding)] / [(1.15) (1.15)(1.15)(1.15)]} = $500.28.

Adding the answers from step 1 to the final year cash flow will yield the following:

Value of Intelligent Semiconductor = [$13.04 + $13.99 + $13.81 + $20.01 + $500.28] = $561.13 per
share.

Question: 2987

The Dow Theory

A. is widely used today.


B. was invented in the 1960s.
C. was invented in the early 19th century.
D. was one of the first theories of technical analysis.

Answer: D

Explanation:
The Dow Theory was invented in the late 19th century by Charles Dow, publisher of the Wall Street
Journal. It postulated that there were three types of price movements over time: major trends,
intermediate trends, and short-term movements. These three types of trends interacted with one
another, and identifying and isolating them individually would lead to profit opportunities.

Question: 2988

The P/E ratio is not determined by

A. the required rate of return.


B. the expected dividend payout ratio.
C. the expected growth rate of dividends for the stock.
D. the financial leverage ratio.

Answer: D

Page | 1145
Explanation:
Using the dividend discount model, the price of a stock is equal to D / (k - g), where D is the expected
dividend, k is the required rate of return, and g is the expected growth rate of dividends for the stock.
Dividing both sides of the equation by expected earnings (E) yields P/E = (D/E) / (k - g), where D/E is
the expected dividend payout ratio. Thus, the P/E ratio is determined by the expected dividend
payout ratio, the required rate of return, and the expected growth rate of dividends for the stock.
Financial leverage does not help determine P/E ratio.

Question: 2989

The Dividend Discount Model:

I. is primarily used to price mature stocks.


II. assumes constant dividends.
III. assumes a constant dividend payout ratio.
IV. works only if the growth rate is higher than the expected rate of return.

A. II only
B. III only
C. I only
D. III & IV
E. I & IV
F. I & II
G. IV only

Answer: C

Explanation:
Dividend Discount Model assumes a constant growth in dividends but does not require the payout
ratio to be constant. Also, it is applicable only when the constant growth rate is lower than the
expected rate of return (It does not preclude the growth rate from exceeding the expected rate of
return over some periods. However, in that case, the formula that arises from the assumption of
constant growth rate cannot be used).

Question: 2990

The confidence index is equal to the ratio of

That answer is correct!


A. the yield on 10 top-grade corporate bonds divided by the yield on the Dow Jones average of 40
bonds, multiplied by 100.
B. the yield on the Dow Jones average of 40 bonds divided by the average yield on 10 top-grade
corporate bonds, multiplied by 100.
C. the expected return on the S&P 500 divided by the yield on 10 top-grade corporate bonds,
multiplied by 100.
D. the yield on the Dow Jones average of 40 stocks divided by the expected return on the S&P 500.
E. the expected return on the S&P 500 divided by the yield on the Dow Jones average of 40 stocks.

Answer: A

Explanation:
The confidence index measures the yield spread between high-grade bonds and a large cross section
of bonds. Some technical analysts believe that during periods of high confidence, investors are more

Page | 1146
willing to invest in lower-quality bonds, thereby pushing down their yields, and increasing the
confidence index. An increase in the index is viewed as a bullish sign.

Question: 2991

Which of the following is the preferred method of return calculation in the investment management
industry?

That answer is correct!


A. Time-weighted rate of return
B. Dollar-weighted rate of return
C. Internal rate of return
D. None of these answers is correct.
E. Asset-weighted rate of return

Answer: A

Explanation:
The time-weighted rate of return is the preferred method of return calculation in the investment
management industry, and this is precisely because the time-weighted rate of return is not sensitive
to significant portfolio additions or withdrawals, unlike the dollar weighted rate of return.

Remember that the dollar-weighted rate of return is another name for the Internal Rate of Return.
Knowing this fact allows you to narrow the answer down to three possible choices. The asset-
weighted rate of return is by definition sensitive to additions and withdrawals of portfolio assets.

Question: 2992

What would be indicative of a high-growth industry?

That answer is correct!


A. A relatively low payout ratio.
B. A relatively high debt-to-equity ratio.
C. All of these answers are correct.
D. A relatively low return on assets ratio.
E. A relatively low return on equity ratio.
F. None of these answers.

Answer: A

Explanation:
In order to sustain a high rate of growth in earnings and dividends, a firm will needfinancing and if a
large cash dividend is paid, it gives up one source of financing. Theresult being low payout of
dividends for these high-growth companies.

Question: 2993

If the spread between the required rate of return on a stock and the dividend growth rate decreases,
the price of the stock:

A. is not affected.
B. decreases.
C. is not affected or increases.
D. increases.

Page | 1147
Answer: D

Explanation:
In the usual notation, the Dividend Discount Model gives Po = D1/(k-g). When k - g decreases, all else
equal, the stock price rises.

Question: 2994

Which is a measure of a stockholder's return?

A. Return on Equity
B. Debt to Equity Ratio
C. Dividend Yield
D. Return on Assets
E. Payout Ratio

Answer: C

Explanation:
The dividend yield = Dividends for the Year/Stock Price at the Beginning of the Year; It is one of the
two important measures of a stock investor's return, with the other being capital gain.

Question: 2995

According to the contrarians, if a large number of investment advisory services are bullish, it implies:

I. the market is reaching a peak.


II. the onset of a market decline.
III. the beginning of a bull market.

That answer is correct!


A. I & II
B. I only
C. III only
D. II only

Answer: A

Explanation:
Contrarians believe that most market participants make wrong investment decisions as the market
approaches the peak or trough in a cycle. Therefore, if a large number of advisory services are
bullish, they believe that the market is about to turn bearish after having reached a peak.

Question: 2996

A mature firm, in the face of a new product introduced by its competition, has suddenly seen its
profit margins fall by 50%. The market expects the management to streamline its sales force in a very
short time and increase the sales-to-assets ratio by 30%. The dividend growth rate due to these
changes will:

A. decrease by 50%.
B. decrease by 15%.
C. increase by 30%.

Page | 1148
D. decrease by 35%.

Answer: D

Explanation:
Use g = ROE * retention ratio and

ROE = profit margin * asset turnover * financial leverage.

If profit margin falls by 50% and asset turnover increases by 30%, the change in ROE is (1-
0.5)*(1+0.3) - 1 = -0.35. With retention ratio constant, a 35% fall in ROE translates into a 35% fall in
dividend growth rate.

Question: 2997

Given that the expected growth rate for a firm is 5%, the expected total asset turnover is 0.87, the
expected financial leverage multiplier is 0.81, the expected return on capital is 1.4, and expected
retention rate is 60%, what is the expected net profit margin of the firm?

That answer is correct!


A. 11.8%
B. 12.3%
C. 14.8%
D. Not enough information
E. 6.9%
F. 9.3%

Answer: A

Explanation:
The growth rate of a firm is equal to the expected retention rate multiplied by the expected return on
equity. Rearranging this yields that the expected ROE is equal to the growth rate divided by the
retention rate (ROE = 0.05/0.60 = 0.0833). The expected ROE is itself equal to the expected profit
margin multiplied by the expected asset turnover multiplied by the expected financial leverage
multiplier. Rearranging this yields that the expected net profit margin is equal to the ROE divided by
the total asset turnover and the financial leverage multiplier (0.0833 / (0.87 x 0.81) = 0.118 = 11.8%

Question: 2998

An analyst is attempting to value shares of a regional bank, and has solicited the help of a senior
financial analyst. During their conversation, the senior financial analyst provides the following
information about the regional bank under examination:

Required rate of return on the bank's equity: 12.75% per year


Free cash flow to equity multiple at t4: 20
1,500,000 shares outstanding

Additionally, the analyst has obtained the following estimates of free cash flow to equity over the
next four years:

Year 1: $1,750,000
Year 2: $2,225,000
Year 3: $2,500,000
Year 4: $2,650,000

Page | 1149
Using this information, what is the value per share of this regional bank according to the free cash
flow to equity model?

A. $18.99
B. $45.54
C. The answer cannot be calculated from the information provided.
D. None of these answers is correct.
E. $31.26
F. $26.31

Answer: F

Explanation:
When determining the value of a common stock using the free cash flow to equity model, it is
necessary to determine three things:

1. The required rate of return on equity investments.


2. The estimated free cash flow to equity multiple at time "k."
3. The estimated free cash flows figures for the time periods leading up to "k."

In this example, the calculation must begin with the discounting the free cash flow to equity figures
for each of the four years provided. These figures are discounted each period by the required return
on equity investments, and the final answer is converted to a per-share basis. This process is
illustrated below:

Year 1: ($1,750,000 / 1.1275) / 1,500,000 shares outstanding = $1.03


Year 2: [$2,225,000 / (1.1275)(1.1275)] / 1,500,000 shares outstanding = $1.17
Year 3: [$2,500,000 / (1.1275)(1.1275)(1.1275)] / 1,500,000 shares outstanding = $1.16
Year 4: [$2,650,000 / (1.1275)(1.1275)(1.1275)(1.1275)] / 1,500,000 shares outstanding = $1.09

Now that the free cash-flow-to-equity figures have been discounted and converted to a per-share
basis, the next step in the valuation process is to determine the value of the final cash flow, which is
defined as:

[(Free cash flow to equity multiple * Final free cash flow) / (1 + r)(1+r)...(1 + k)]

In the body of this question, we were given the anticipated multiple of free cash to equity that shares
of Intelligent Semiconductor will sell for at time period: specifically, 20 times. Imputing this
information into the terminal cash flow equation will yield the following:

{[20 * ($2,650,000 / 1,500,000 shares outstanding)] / [(1.1275) (1.1275)(1.1275)(1.1275)]} = $21.86

Adding the answers from step 1 to the final year cash flow will yield the following:

Value of Intelligent Semiconductor = [$1.03 + $1.17 + $1.16 + $1.09 + $21.86] = $26.31 per share.

Question: 2999

Which of the following factors is an underlying assumption of technical analysis?

That answer is correct!


A. Supply and demand is driven by rational and irrational behavior.
B. The actual shifts in supply and demand cannot be observed in market behavior.

Page | 1150
C. Prices move randomly.
D. Prices are not determined by supply and demand.

Answer: A

Explanation:
Supply and demand is driven by rational and irrational behavior.

Question: 3000

Mary Short is a retail investor. During the course of the last several weeks, Ms. Short has been
examining shares of Tellcorr Industries, a large telecommunications firm. In her examination, Mary
has determined that Tellcorr's $1.05 per share dividend is anticipated to grow 20% annually.
Assuming that Mary can sell her shares of Tellcorr for $70 per share at the end of three years, and
that her required rate of return is 22% per year, what is the value of Tellcorr's common stock?

That answer is correct!


A. $41.60
B. $39.98
C. None of these answers is correct.
D. $59.23
E. $63.44

Answer: A

Explanation:
The Multiple Holding Period form of the Dividend Discount Model takes the following form: {V = {[d1
/ (1 + k)] + [d2 / (1 + k)^2] + ... .[dn / (1 + k)^n] + [Pn / (1 + k)^n]}

Where: V = the price of the common stock at t0, d1 = the annual dividend at t1 (this is found by
multiplying the annual dividend at t0 by (1 + the anticipated growth rate), d2 = the annual dividend
at t2 (this is found by multiplying the dividend at t1 by (1 + the anticipated growth rate), k = the
required rate of return, n = period "n", and Pn = the sale price of the common stock at time "n".

In this example, time "n" is the third year, as this is the end horizon for Mary's holding period. Had
the investor in this example forecasted selling the shares at the end of the 10th year, then "n" would
be the tenth year.

Now that the formality of expressing the equation for this form of the DDM has been carried
through, we can move toward a calculation of the value of this common stock. In this example, all of
the necessary information has been provided, and the calculation of the value of this retail stock is as
follows:

{V = [($1.05 * 1.20) / (1 + 0.22)^] + [($1.26 * 1.20) / (1 + 0.22)^2] + [($1.512 * 1.20) / (1 + 0.22)^3] +


[$70 / (1 + 0.22)^3]}

Which can be further broken down into the following:

{V = [$1.032787 + $1.015856 + $0.999203+ $38.549482] = $41.60}

Question: 3001

Which of the following represents a "contrary opinion" technical indicator?

Page | 1151
That answer is correct!
A. Mutual fund cash position.
B. Diffusion Index.
C. T-Bill-Eurodollar Yield Spread.
D. None of these answers is correct.
E. Short sales by specialists.
F. The Confidence Index.

Answer: A

Explanation:
Of the choices listed, only "mutual fund cash positions" represents a contrarian technical indicator.
Technical analysts often believe that the majority of market participants are incorrect in their
opinions about market direction and valuation, especially during periods preceding market peaks
and troughs. This style of thinking is often referred to as "contrary opinion" technical analysis.

The mutual fund cash position is a contrary opinion technical indicator because contrarian technical
analysts believe that mutual funds will be wrong in their forecasts of market direction. Technical
analysts often use mutual funds as a proxy for institutional investors, and believe that the mutual
fund cash position provides important insight into the sentiment of institutional investors.
Specifically, these technical analysts believe that a high mutual fund cash position, which indicates
that mutual funds are bearish on the market, is actually a signal of an impending upward move in
stock prices. Conversely, a low mutual fund cash position is viewed as bearish by contrarian technical
analysts. The intuition behind this opinion is relatively straightforward - a low mutual fund cash
position indicates a low amount of buying power within mutual funds, i.e. there is less money
available to support stock prices.

Several studies have examined the cash ratio's ability to predict market cycles and have determined
its usefulness is less than that implied by the technical analyst community.
The Confidence Index is a measure of yield spreads between high-grade corporate bonds and the
yields on average corporate bonds. The Diffusion Index measures the breadth of the market, and is
found by taking the total volume of advancing shares plus one-half of the issues unchanged, divided
by the total number of issues traded. The ratio of short sales by specialists is used by technical
analysts to track the opinions of the "smart money," and the yield spread between T-Bills and
Eurodollars is used to measureinternational sentiment and confidence. The T-Bill-Eurodollar Yield
Spread is another example of a "smart money" technical indicator.

Question: 3002

Which of the following does not affect the growth rate of earnings and dividends of a stock?

A. Changes in the dividend payout ratio


B. Changes in the return on equity (ROE)
C. Changes in the risk premium
D. Changes in the earnings retention rate

Answer: C

Explanation:
The growth rate is equal to the retention rate multiplied by the ROE. The retention rate is itself equal
to one minus the dividend payout ratio. Changes in the risk premium affect the required rate of
return on a stock. Changes in all these variables are used in the direction of change approach to
estimate the earnings multiplier.

Page | 1152
Question: 3003

The ratio of upside-downside volume is equal to

A. the number of downticks in the stock market divided by the number of upticks.
B. the total volume of increasing stocks plus half the volume of unchanged stocks, divided by the
total volume of decreasing stocks.
C. the number of stocks increasing divided by the number of stocks decreasing.
D. the total volume of increasing stocks divided by the total volume of decreasing stocks.

Answer: D

Explanation:
Technical analysts may use the ratio of upside-downside volume as an indicator of short-term
momentum for the market. They feel that a ratio value of 1.50 or more indicates that the market is
overbought, while a ratio of 0.70 or less indicates that the market is oversold.

Question: 3004

If the moving average of past stock prices has been above the current price, this indicates

That answer is correct!


A. that the stock price has been declining.
B. that the stock price has been increasing.
C. an approaching market peak.
D. probable market instability.

Answer: A

Explanation:
A 50-day moving average, for example, would be equal to the average stock price for the past 50
days. In order for this average to be consistently above the current price, the current price must be
going through a general decline that is dragging it below the moving average, which takes longer to
adjust to falling prices.

Question: 3005

Which of the following are assumptions of the dividend discount model?

A. No inflation
B. All of these answers
C. The required rate of return is greater than the growth rate
D. Earnings will not be negative

Answer: C

Explanation:
The assumptions of the Dividend Discount Model are: (1) Dividends grow at a constant rate; (2) The
constant growth rate will continue for an infinite period; (3) The required rate of return is greater
than the growth rate.

Question: 3006

The P/E ratio is not determined by

Page | 1153
That answer is correct!
A. the ROE.
B. the expected dividend growth rate for the stock.
C. the expected dividend payout ratio.
D. the required rate of return on the stock.

Answer: A

Explanation:
The infinite period Dividend Discount Model claims that the current price of a common stock is equal
to D1 / (k - g), where D1 is next period's (most often next year's) dividend, k is the required rate of
return, and g is the growth rate of dividends. If we divide both sides of the infinite period Dividend
Discount Model equation by expected earnings during the next 12 months, we get P/E = (D1/E) / (k -
g). This equation shows that the P/E ratio is determined by the expected dividend payout ratio
(D1/E), k, and g. ROE does not help determine the P/E ratio.

Question: 3007

Closed end funds sell

A. none of these answers.


B. at a price equal to its NAV.
C. at a premium over its NAV.
D. at a discount from its NAV.

Answer: D

Explanation:
Historically, the market price of a closed end fund has been 5 to 20 percent below the NAV of the
fund.

Question: 3008

At which stage in an industry life cycle would profit margins most likely be at their highest?

A. mature growth
B. pioneering development
C. rapid accelerating growth
D. deceleration of growth and decline
E. stabilization and market maturity

Answer: C

Explanation:
In this stage, there are a limited of number of firms for the product/service and demand is very high
enabling the firm to experience high markups.

Question: 3009

Which of the following is an advantage of technical analysis?

A. It explains why investors are buying and selling.


B. It involves adjusting for accounting problems.

Page | 1154
C. It only incorporates economic reasoning.
D. It is quick and easy.

Answer: D

Explanation:
Technical Analysis is viewed as less vigorous than fundamental analysis.

Question: 3010

Historically, the earnings per share (EPS) figure for a stock market series has been less volatile than
the earnings multiplier for the same series. Which of the following best characterizes the primary
reason for the greater volatility experienced by the earnings multiplier? Choose the best answer.

A. The price/earnings figure experiences a tax leveraging effect that is not passed on to the EPS
figure.
B. The EPS figure is less volatile due to accounting manipulations and the malleability of
international and domestic accounting standards including GAAP.
C. The earnings multiplier is more sensitive to changes in dividend policies than is the EPS figure.
D. None of these answers is correct.
E. The price/earnings ratio is more sensitive to changes in the spread between the required rate of
return and the anticipated future growth rate.
F. The earnings multiplier is more sensitive to fluctuations in the equity markets than is the EPS
figure; i.e. the earnings multiplier is "forward looking."

Answer: E

Explanation:
The greater relative volatility of the earnings multiplier versus the EPS figure is primarily attributable
to an increased sensitivity to changes in the spread between the required rate of return "k" and the
anticipated growth rate "g." Remember that the equation used to determine the appropriate
earnings multiplier for a stock market series is the following:

P/E = [D/E / (k - g)]

Where: P/E = the earnings multiplier, or Price-to-Earnings ratio, D/E = the dividend payout ratio at t1,
k = the required rate of return, and g = the anticipated growth rate of dividends.

As you can see, changes in the spread between the required rate of return and the anticipated
growth rate can have a dramatic effect on the earnings multiplier for a stock market series. While the
earnings multiplier is sensitive to changes in the dividend payout ratio, volatility in this figure is not
cause for the increased volatility of the earnings multiplier versus the EPS figure.

Question: 3011

If a stock has an expected dividend payout ratio of 50%, a required rate of return of 13% and an
expected dividend growth rate of 10%, what is the P/E ratio?

A. 10
B. 12.5
C. None of these answers
D. 8.5

Answer: C

Page | 1155
Explanation:
The price/earnings ratio can be computed by dividing the expected dividend payout ratio (dividends
divided by earnings) by the required rate of return (k) minus the expected growth rate of dividends
(g).

In this case, P/E = .50/(.13-.10) = 16.7

Question: 3012

Given that the beginning value of a stock is $120, the ending value is $110, earnings are $40, and the
retention rate of earnings is 0.6, what is the rate of return on the stock over this period?

A. 6%
B. 5.7%
C. 5%
D. -8.3%
E. Not enough information.

Answer: C

Explanation:
The dividend payout ratio is equal to one minus the retention rate (1 - 0.6 = 0.4). Dividends are equal
to the dividend payout ratio multiplied by earnings (0.4 x 40 = $16). The rate of return is equal to the
ending price plus the dividend payments, divided by the beginning price, minus one. In this question,
the rate of return is [(110 + 16)/ 120] - 1 = 0.05 = 5%.

Question: 3013

An fundamental analyst with Street Brothers asset management is considering shares of Polynomial
Software Solutions, Inc., for possible investment. In her analysis, this investor has determined the
following information:

The Company currently pays a $2.20 per share dividend, and this dividend is anticipated to grow at
13% annually. Additionally, the investor has assumed that she will be able to sell the stock for $125
per share at the end of four years. Similar investments have warranted a 15.25% per year required
rate of return. What is the value of Polynomial Software Solutions?

That answer is correct!


A. $79.23
B. $110.64
C. $71.89
D. $122.16
E. The Multiple Holding Period DDM will produce a nonsensical answer for this stock.

Answer: A

Explanation:
The Multiple Holding Period form of the Dividend Discount Model takes the following form: {V = {[d1
/ (1 + k)] + [d2 / (1 + k)^2] + ... .[dn / (1 + k)^n] + [Pn / (1 + k)^n]}

Where: V = the price of the common stock at t0, d1 = the annual dividend at t1 (this is found by
multiplying the annual dividend at t0 by (1 + the anticipated growth rate), d2 = the annual dividend
at t2 (this is found by multiplying the dividend at t1 by (1 + the anticipated growth rate), k = the

Page | 1156
required rate of return, n = period "n", and Pn = the sale price of the common stock at time "n".

In this example, time "n" is the fourth year, as this is the end horizon for this investors holding period.
Had the investor in this example forecasted selling the shares at the end of the 10th year, then "n"
would be the tenth year.

Now that the formality of expressing the equation for this form of the DDM has been carried
through, we can move toward a calculation of the value of this common stock. In this example, all of
the necessary information has been provided, and the calculation of the value of this retail stock is as
follows:

{V = [($2.20 * 1.13) / (1 + 0.1525)^1] + [($2.486 * 1.13) / (1 + 0.1525)^2] + [($2.80918 * 1.13) / (1 +


0.1525)^3] + [($3.174373 * 1.13) / (1 + 0.1525)^4] + [$125 / (1 + 0.1525)^4]}

Which breaks down into the following:

{V = [$2.15705 + $2.114938 + $2.073649 + $2.033165 + $70.851048] = $79.23

Question: 3014

Which statement is not true?

A. Within industries, firms tend to have similar capital structures.


B. Most ratios vary across time within a given industry.
C. The higher proportion of debt, the higher the return on equity ratio will be.
D. The lower the dividend yield, the greater the anticipated price appreciation.
E. High P/E ratios tend to go with high payout ratios.
F. The higher the payout ratio in a given industry, the more important dividends are to shareholders.

Answer: E

Explanation:
High P/E ratios tend to go with low payout ratios as both of these measures are associated with
higher growth rates. Remember that low payout ratios are common for high-growth companies as
they give up a paying large dividends to finance their firm.

Question: 3015

Joe Wellworth, an oil analyst with Smith, Kleen and Beetchnutty institutional brokerage, is trying to
determine an appropriate earnings multiplier for the natural gas industry. In his research, Mr.
Wellworth has examined the relationship between the earnings multiplier of the natural gas industry
and the Price-to-Earnings ratio of the Standard & Poors 500. Using a time series analysis, Joe
examines the trend in the relationship between the natural gas industry and the overall market and
uses this information to estimate the appropriate earnings multiplier for the natural gas industry.
Which of the following best characterizes this method of estimating the earnings multiplier of an
industry? Choose the best answer.

A. Correlation analysis
B. Microanalysis
C. The bottom-up approach
D. Macroanalysis
E. Time series analysis

Answer: D

Page | 1157
Explanation:
The answer called for in this example is macroanalysis. This method involves an examination of the
relationship between the earnings multiplier of a stock market series and the earnings multiplier of
the overall market. For example, an individual projecting an earnings multiplier for a software index
using macroanalysis would begin by examining the relationship between the P/E ratio of the
software index and the P/E ratio of a broad market index such as the Standard & Poors 500. Both
historical trends and point estimates would be examined, and from this information, a projection of
the earnings multiplier for the stock market series is deduced. This is precisely the process illustrated
in this example.

This is contrasted by microanalysis, which involves an examination of the components of the


earnings multiplier, including the anticipated growth rate of dividends, the required rate of return,
and the dividend payout ratio. Once these variables have been examined, both from the perspective
of trend analysis and point estimation, a value for the earnings multiplier is deduced.

The bottom-up approach is used in the investment selection process, and involves identifying
superior investments by first examining companies, rather than beginning with an examination of
macroeconomic cycles and influence. Time series analysis, while materially correct, does not
represent the best possible answer.

Question: 3016

A technical analyst with Bullfighter.com, a noted investment research firm, has been examining the
U.S. securities markets, and believes that the market is technically "overbought." Which of the
following technical indicators would this analyst likely use to support his opinion? Choose the best
answer.

A. The Block Uptick-Downtick Ratio has declined below 0.70.


B. All of these choices indicate an "overbought" condition.
C. The Diffusion Index has increased significantly.
D. The Block Uptick-Downtick Ratio has advanced beyond 1.1.
E. The CBOE Put/Call Ratio has declined to 0.50.

Answer: D

Explanation:
The Block Uptick-Downtick Ratio is used by technical analysts to gauge institutional investment
activity by measuring the percentage of block trades which result in an uptick versus the block trades
which are executed on a downtick. The idea behind this ratio is the belief that a block buyer would
initiate an "uptick", or a bid up in the securities' price, and a block seller would initiate a "downtick,"
or a bid down in the securities' price. Technical analysts view a decline in the Block Uptick-Downtick
Ratio below 0.70 as
an indication of an oversold condition, and an increase in the Block Uptick-Downtick Ratio above
1.10 as indicative of an overbought condition.

The "Diffusion Index" is a measure of market breadth, and is defined as [(# of advancing issues + 1/2
# of issues unchanged) / # of issues traded]. An increase in the diffusion index is indicative of an
increase in advancing issues relative to declining issues. The CBOE Put/Call Ratio is a contrarian
technical indicator used to gauge the sentiment of investment professionals, and a ratio greater than
50% is viewed by contrarian technical analysts as overtly bullish. Finally, contrarian technical analysts
would view a large increase in the amount of futures traders who express bullish sentiment on stock
index futures as a bearish signal.

Page | 1158
The % of issues trading below their 200-day moving average is frequently cited by technical analysts
as a measure of oversold and overbought market conditions. Specifically, technical analysts see the
market as "overbought" when 80% of issues are trading above their 200-day moving average, and
consider a market "oversold" when 80% of issues are trading under their 200-day moving average.

Question: 3017

Technical analysts may view a decline in credit balances as

A. a bullish sign.
B. a bearish sign.
C. the result of low mutual fund cash positions.
D. a sign indicating a short-term, transient bull market.

Answer: B

Explanation:
A credit balance results when an investor sells stocks and deposits the proceeds with his broker.
Technical analysts view a decline in credit balances as a decrease in a pool of potential buying power.
For this reason, such a decline is viewed as a bearish sign.

Question: 3018

A market strategist with Churn Brothers Brokerage is trying to determine the earnings multiplier of
an equity index comprised of grocery stores, and has gathered the following information:

g: 6.00% per year


k: 8.50% per year
EPS: $3.35
D0: $1.20

Using this information, what is earnings multiplier for this equity index? Further, assuming that the
grocery business is a mature industry, and that the economy is experiencing stable growth, is this
earnings multiple realistic?

A. None of these answers is correct.


B. 4.21, this multiple is too low
C. 5.97, this multiple is too low
D. 130, this multiple is much too high
E. 15.67, this multiple is likely realistic
F. 14.33, this multiple is likely realistic

Answer: F

Explanation:
To determine the earnings multiplier, or "P/E ratio," of a stock market series, use the following
equation: P/E = [(D1 / E1) / (k-g)

Where: D1 = the annual per-share dividend at t1, E1 = the EPS figure at t1, k = the required rate of
return on common stock, and g = the expected growth rate of dividends.

In this example, all of the necessary information has been provided, and putting it into the equation
above will yield the following:

Page | 1159
P/E of a stock market series = [($1.20*1.06 / ($3.35*1.06)) / (0.085 - 0.06)] = 14.33

This earnings multiple is appropriate for lower growth industries, such as the grocery business, which
has historically grown in the mid-to-high single digits for much of the last decade. Consider the fact
that the P/E ratio is a proxy for future growth. Firms in the automobile, basic materials, or other
mature industries, which are expected to grow slowly, are characterized by lower earnings multiples
and higher dividend payout ratios. Firms in the software, networking, biotechnology, and other high-
growth industries, are typically characterized by high earnings multiples and low dividend payout
ratios. What is happening here is that investors are giving up current income (i.e. dividends) in the
hopes of rapid earnings growth (i.e. greatly increased EPS in the future).

A complete understanding of this relationship is absolutely crucial, and as a Level 1 candidate, I


encourage you to examine this relationship further if you are not completely comfortable with the
P/E ratio, its components and the relationships between them, and the implications of the earnings
multiplier across different industries. Indeed, the P/E ratio is a valuable tool, one which can provide
significant information about the growth prospects priced into a common stock.

Question: 3019

Analytics Software Incorporated currently pays 20% of its earnings in dividends and the Company has
a steady growth rate of 25% per year. Assuming a 27.5% per year required rate of return, what is the
appropriate earnings multiplier for Analytics Software?

A. The answer cannot be determined from the information provided.


B. 16
C. 18
D. 22
E. 8
F. 9

Answer: E

Explanation:
To determine the earnings multiplier (i.e. the price-to-earnings ratio) for an individual company, use
the following formula:

P/E = [(d1 / e1) / (k - g)]

Where: P/E = the earnings multiplier, d1 / e1 = the dividend payout ratio at t1, k = the required rate
of return, and g = the anticipated future growth rate.

In this example, all of the necessary information has been provided, and the calculation of the
earnings multiplier is as follows:

P/E = [0.20 / (0.275 - 0.25)] = 8

This multiplier is likely low for a software firm growing at 25% annually. An analyst examining shares
of Analytics Software would likely take this disparity into account in his or her analysis. Perhaps the
use of the Infinite Period DDM to determine an earnings multiplier for this company is unrealistic.
Perhaps valuing this company in accordance to anticipated free cash flows would provide a more
realistic measure of value.

Question: 3020

Page | 1160
Which of the following is not an advantage of technical analysis?

A. It does not involve adjusting for accounting problems.


B. It tells when to buy and sell not why investors are buying and selling.
C. It only incorporates economic reasoning.
D. It is quick and easy.

Answer: C

Explanation:
It incorporates economic and psychological reasoning.

Question: 3021

Which of the following is NOT typically true for a firm which has adopted the low-cost competitive
strategy?

A. The firm will enjoy above-average rates of return only if its price premium based on its
differentiation exceeds the extra cost of being unique.
B. None of these answers.
C. All of these answers.
D. The firm seeks to differentiate itself based on its distribution system, through some unique
marketing approach, or by providing an important service to its customers.
E. The firm seeks to identify itself as unique in its industry in an area that is important to buyers.

Answer: C

Explanation:
The above are all true for a company that has adopted the differentiation strategy. With the low-cost
strategy, the firm is determined to become the low-cost producer and, hence, the cost leader in the
industry. Cost advantage might include economies of scale, proprietary technology, or preferential
access to raw materials. The objective is to charge less for the product or service but still enjoy higher
profit margins and returns on capital.

Question: 3022

If the spread between the required rate of return and the anticipated dividend growth rate were to
decrease significantly and suddenly while the remaining components of the P/E ratio were to remain
unchanged, which of the following would likely occur? Further, a decrease in the retention rate
would lead to what effect on the earnings multiplier, holding both k and g constant?

That answer is correct!


A. The earnings multiplier would increase; the earnings multiplier would increase.
B. The earnings multiplier would decrease; the earnings multiplier would decrease.
C. The earnings multiplier would increase; the earnings multiplier would either increase or decrease
depending on the firm's ROE compared to its cost of capital.
D. The earnings multiplier would increase; the earnings multiplier would decrease.
E. The earnings multiplier would decrease; the earnings multiplier would increase.

Answer: A

Explanation:
Remember that the equation used to determine the appropriate earnings multiplier for a stock
market series is the following:

Page | 1161
{P/E = [D/E / (k - g)]}

Where: P/E = the earnings multiplier, or Price-to-Earnings ratio, D/E = the dividend payout ratio at t1,
k = the required rate of return, and g = the anticipated growth rate of dividends.

As the spread between k and g narrows, the earnings multiplier figure will increase. Indeed, the
earnings multiplier is very sensitive to changes in the spread between k and g, and this is the primary
reason for the greater relative volatility of the P/E ratio versus the EPS figure for a stock market
series.

A decrease in the retention rate will lead to an increase in the earnings multiplier figure, holding
everything else equal. Remember that the retention rate is simply (1 - dividend payout ratio). So, a
decline in the retention rate is analogous to an increase in the dividend payout ratio. An increase in
the dividend payout ratio, holding everything else equal, will lead to an increase in the earnings
multiplier figure.

Question: 3023

Jim, an investment manager with Smith, Kleen & Associates, is in the process of determining the
annualized return for a client portfolio. Jim uses a specific three-step process to determine this
annualized return, which is detailed as follows:

Step 1:
Jim prices the portfolio immediately prior to any significant addition or withdrawal of funds. The
portfolio is broken into specific subperiods based on the dates of cash inflows and outflows. The
product of the subperiods is 10 years.

Step 2:
Jim calculates the holding period return of the portfolio for each subperiod.

Step 3:
Jim calculates the geometric mean of the annual returns. This calculation is used as the annual
portfolio return measure.

Which of the following best describes the final calculation produced by Jim?

A. Dollar-weighted rate of return


B. Time-weighted rate of return
C. Modified Internal Rate of Return
D. Annualized holding period return
E. Geometrical rate of return
F. Asset-weighted rate of return

Answer: B

Explanation:
The process illustrated in this example is the calculation of the time-weighted rate of return. This
method of calculating portfolio returns is superior in many respects to the dollar-weighted rate of
return, which is also known as the IRR. The reason for this relative superiority is the fact that the
time-weighted rate of return is not sensitive to portfolio additions or withdrawals. So said, the time-
weighted rate of return is more popular in the field of investment management.

While "annualized holding period return" is a tempting choice, it does not represent the best

Page | 1162
possible answer. "Geometrical rate of return" could refer to several return measures, but not the
time-weighted rate of return.

Question: 3024

Which of the following is not an assumption of technical analysis?

That answer is correct!


A. Securities markets are weak-form efficient.
B. Changes in the market value of any good are determined solely by supply and demand
fluctuations.
C. Securities prices exhibit identifiable patterns.
D. Supply and demand is governed by both rational and irrational factors.
E. More than one of these answers is correct.
F. Securities prices move in identifiable trends.

Answer: A

Explanation:
Technical analysis is primarily grounded on three major assumptions - (1.) the market value of any
good is determined solely by supply and demand, (2.) supply and demand fluctuations are caused by
both rational and irrational factors, and (3.) the prices of individual securities and securities markets
move in trends, i.e. securities markets "price-in" information slowly. The third assumption of
technical analysis negates any belief in the Weak Form Efficient Market Hypothesis, which assumes
that past performance data cannot be used to predict securities prices, and that the securities
markets "price-in" information instantly. This "incongruence" with all forms of the EMH is cited as
the greatest criticism of technical analysis.

Question: 3025

When studying industry analysis, which would be of most importance when concentrating on
financial performance?

That answer is correct!


A. all of these would be important
B. financial leverage
C. components of return on equity
D. return on total capital
E. return on foreign investments

Answer: A

Explanation:
These are all factors in financial performance when studying industry analysis.

Question: 3026

The current dividend yield on a stock A is 3.2%. The stock has a required rate of return of 9%. If the
firm just paid a dividend of $1.65, what's the expected dividend for next year, assuming a constant
growth rate?

A. $2.01
B. $1.89
C. $1.75

Page | 1163
D. $1.83

Answer: C

Explanation:
The dividend yield, defined as the expected dividend next period divided by current price, equals
D1/Po, in standard notation. Using the Dividend Discount Model, this is equal to k - g. With k = 9%,
we get 3.2% = 9% - g. Therefore, g = 5.8%. The expected dividend next year is then equal to 1.65 *
1.058 = $1.75.

Question: 3027

The estimate of ________ combined with the firm's retention rate will indicate its growth potential.

That answer is correct!


A. ROE
B. assets
C. ROA
D. current ratio
E. equity

Answer: A

Explanation:
The estimate of ROE combined with the firm's retention rate will indicate its growth potential.

Question: 3028

Consider the following information about a natural gas driller:

Next annual dividend: $1.18


Earnings per share next year: $4.60
Anticipated growth rate: 12.5% per year
Required rate of return: 15% per year

What is the expected earnings multiplier for this utility company?

A. 12.46
B. 10.26
C. 53.10
D. 15.59
E. 47.20
F. None of these answers is correct.

Answer: B

Explanation:
To determine the earnings multiplier (i.e. the price-to-earnings ratio) for an individual company, use
the following formula:

P/E = [(d1 / e1) / (k - g)]

Where: P/E = the earnings multiplier, d1 / e1 = the dividend payout ratio at t1, k = the required rate
of return, and g = the anticipated future growth rate.

Page | 1164
In this example, all of the necessary information has been provided, but some rearranging is
necessary.

Specifically, the dividend payout ratio must be determined. This figure is found as follows:

Dividend payout ratio = [$1.18 / $4.60] = 0.256522, or 25.65%

Now that the dividend payout ratio has been determined, we can solve for the appropriate earnings
multiplier. The calculation of this figure is found as follows:

P/E = [0.2565 / (0.15 - 0.125)] = 10.26

In generally favorable economic conditions, this is a pretty realistic multiple for an average natural
gas drilling company.

Question: 3029

The ________ gives an estimate for the spread between the required rate of return and the expected
growth of dividends.

A. none of these answers


B. dividend yield
C. P/E ratio
D. yield-to-maturity

Answer: B

Explanation:
The dividend yield gives an estimate for the spread between the required rate of return and the
expected growth of dividends.

Question: 3030

A high short interest ratio would be interpreted by technical analysts as

A. a sign of coming market instability.


B. irrelevant.
C. bearish.
D. bullish.
E. indicative of an approaching market peak.

Answer: D

Explanation:
The short interest ratio is equal to the outstanding short interest divided by the average daily volume
of trading on the exchange. A high ratio is interpreted as bullish because a larger relative short
interest is indicative of a high potential demand for stock from those who have sold short but not yet
covered their sales.

Question: 3031

What is the value of a preferred stock with a par value of $150, an annual dividend equal to 15% of
par value, and a required rate of return of 12%?

Page | 1165
A. $150.00
B. $187.50
C. Not enough information
D. $123.49
E. 230.54

Answer: B

Explanation:
The value of a preferred stock is the present value of its dividends, which is equal to the annual
dividend divided by the required rate of return. In this question, the annual dividend is equal to 150 x
.15 = $22.5, and the preferred stock is worth $22.5/0.12 = $187.50

Question: 3032

The risk premium

A. causes differences in the required rate of return among alternate types of investments.
B. causes differences in required rates of return among alternate types of investments and among
investments of the same type.
C. can be less than zero.
D. causes differences in the required rate of return among investments of the same type.

Answer: B

Explanation:
Every investment has its own risk premium, which may change over time. The required rate of return
on any security is equal to the risk-free rate of return plus the risk premium on that security. Because
the lowest possible required rate of return on any investment is the risk-free rate of return, the risk
premium cannot be less than zero.

Question: 3033

Using the microanalysis approach to estimating a company's earnings multiplier, the multiplier is
based on:

I. the dividend payout ratio


II. the required rate of return
III. the company's relationship to the industry
IV. the rate of growth
V. the estimated earnings per share
VI. the company's relationship to the market

A. III, VI
B. I, II, III, IV, V
C. I, II, IV
D. I, II, III
E. II, III, IV

Answer: C

Explanation:
The earnings multiplier, under the microanalysis approach, is estimated based on its three

Page | 1166
components: the dividend payout ratio, the required rate of return, and the rate of growth. Under
the Macroanalysis approach, it is estimated from the relationships among the firm, its industry and
the market. The estimates derived from each approach are resolved to settle on one estimate.

Question: 3034

There are two popular methods for estimating the earnings multiplier for an industry or a stock
market series, which of the following correctly lists these two techniques?

A. Discounted cash flow estimation, regression analysis.


B. The direction of change and the specific estimate approach.
C. Microanalysis and Macroanalysis.
D. The top-down and the bottom-up approach.
E. Scenario Analysis, Monte Carlo Simulation.
F. More than one of these answers is correct.

Answer: F

Explanation:
More than one of these answers is correct. Specifically, the "direction of change" and the "specific
estimate" are methods of forecasting the earnings multiplier of a stock market series and
"microanalysis" and "macroanalysis" are used to forecast the earnings multiplier for an industry.

When estimating the earnings multiplier for an industry, there are two common methods - the
"direction of change" and the "specific estimate" approaches. The direction of change approach
begins with the present earnings multiplier for a stock market series and seeks to estimate both the
amount and direction of any change based upon changes in the components of the P/E ratio - the
required rate of return, the anticipated growth rate of dividends, and the dividend payout ratio.

This approach is contrasted by the specific estimate approach, which involves the estimation of a
specific value for the earnings multiplier based on a series of projections for values of the
components of the earnings multiplier components. When using this approach, analysts commonly
confine their estimates to a series of scenarios, typically a best case, worst case, and base case
scenario.

There are two common methods for estimating the earnings multiplier for an industry -
macroanalysis and microanalysis. Macroanalysis involves an examination of the relationship
between the earnings multiplier for an industry and the earnings multiplier for the market.
Microanalysis involves an estimation of the specific variables that influence the earnings multiplier,
including the required rate of return, the estimated growth rate, and the dividend payout ratio.
While the top down and bottom up method are similar to Macroanalysis and Microanalysis,
respectively, they do not represent the best possible answer. Specifically, the top-down and bottom
up approaches are typically used to identify investment opportunities, not for the estimation of an
industry earnings multiplier.

Question: 3035

The short-interest ratio is equal to

That answer is correct!


A. the outstanding short interest on an exchange divided by the daily volume of trading on the
exchange. A low short-interest ratio would be interpreted by technical analysts as a bearish sign,
because of the small potential demand for stocks by those who have sold short by not yet covered
their sales.

Page | 1167
B. the outstanding short interest on an exchange divided by the daily volume of trading on the
exchange. A high short-interest ratio would be interpreted by technical analysts as a bearish sign,
because a large amount of short sales indicates investors' expectations of declining share prices.
C. the outstanding short interest on an exchange divided by the market capitalization of shares on
the exchange. A low short-interest ratio would be interpreted by technical analysts as a bullish sign,
because a small amount of short sales indicates investors' expectations of rising share prices.
D. the outstanding short interest on an exchange divided by the number of long positions held. A
high short-interest ratio would be interpreted by technical analysts as a bullish sign, because of the
large potential demand for stocks by those who have sold short but not yet covered their sales.

Answer: A

Explanation:
Technical analysts interpret the short-interest ratio contrary to initial intuition. Because short sales
reflect investors' expectations of declining share prices, a large amount of short sales might be taken
as a bearish sign. But technical analysts prefer to focus on the potential demand for shares that short
sales create. Once an investor has sold short a stock, he will eventually have to repurchase it to cover
his position.

Question: 3036

You have a stock that you are holding for one year. It has an estimated dividend payout of $1.10 and
an expected sale price of $22. Using the dividend discount model, calculate the value of the stock if
your required rate of return is 14%.

A. $23.10
B. $20.26
C. $22
D. not enough information to calculate it

Answer: B

Explanation:
Bring both dividend and expected sale price to present value and sum the two: V = $1.10/1.14 +
$22.00/1.14 = $20.26

Question: 3037

A market strategist for Churn Brothers Brokerage is trying to determine an EPS figure for a stock
market series. In her analysis, this portfolio manager has determined the following:

1. Regressing sales for the series against Nominal GDP, the sales figure for the index has been
estimated at: $26.44.

2. Analyzing capacity utilization rates, foreign competition, rates of inflation and unit labor costs, the
operating profit margin for the series has been determined to be 28%.

3. Creating a time series based upon inputs such as levels of capital expenditures and PP&E turnover,
next year's depreciation-per-share has been determined to be: $1.89.

4. Creating a time series based upon levels of debt outstanding and prevailing debt yields, the
interest expense for next year is determined to be: $0.94 per share.

5. Coordinating his research with a legislative consultant, the corporate tax rate for this series has

Page | 1168
been estimated at: 36%

Using this information, what is the EPS figure for this stock market series?

A. The answer cannot be determined from the information provided.


B. $3.87
C. None of these answers is correct.
D. $3.18
E. $4.82
F. $2.93

Answer: F

Explanation:
All of the necessary information has been provided in this example. To determine the EPS for a stock
market series, the following steps are necessary:

Step 1: Estimate sales-per-share for the series:


Step 2: Estimate operating profit margin for the series
Step 3: Estimate the depreciation-per-share for next year
Step 4: Estimate the interest expense-per-share for the next year
Step 5: Estimate next year's corporate tax rate

Once these five steps have been completed, the calculation of EPS for a stock market series is found
by the following:

EPS = [(Sales per share * Operating profit margin) - Depreciation-per-share - Interest Expense] * (1 -
Corporate Tax Rate)

The calculation of EPS for this stock market series is shown as follows:

EPS = [($26.44 * 0.28) - $1.89 - $0.94] * (1 - 0.36) = $2.93

Question: 3038

The estimated ________ is applied to the estimated ________ to arrive at estimated future values of
a company's share.

That answer is correct!


A. earnings multiplier, earnings per share
B. dividend payout ratio, expected growth rate less the required rate of return
C. dividend payout ratio, required rated of return less the expected growth rate

Answer: A

Explanation:
Estimated earnings multiplier x Estimated earnings per share = Estimated future value of the share.

Question: 3039

A fundamental analyst is examining the perpetual preferred stock of a large telecom company. The
preferred stock is expected to pay a quarterly dividend of $0.55, and the required rate of return is
11.75% per year. At what price would this preferred stock be fairly valued?

Page | 1169
A. $16.44
B. The answer cannot be calculated from the information provided.
C. $18.20
D. None of these answers is correct.
E. $18.72
F. $21.14

Answer: E

Explanation:
Assuming that the quarterly dividend is to remain unchanged forever allows us to use the standard
perpetuity model, which is illustrated as follows:

Value of preferred stock = {Annual dividend / required rate of return}

In this example, we are given the quarterly dividend, which must be multiplied be annualized in
order to be imputed into the perpetuity valuation equation.

So said, a quarterly dividend of $0.55 translates into a yearly dividend of $2.20. Incorporating this
yearly dividend into the perpetuity valuation model will result in the following:

Value of preferred stock = {$2.20 / 0.1175} = $18.72

Question: 3040

A large net advance on an advance-decline series in a rising market would be viewed by technicians
as

That answer is correct!


A. a bullish signal.
B. a sign of caution.
C. indicative of an uneven market.
D. irrelevant.

Answer: A

Explanation:
The large net advance means that considerably more stocks advanced than declined in the series,
meaning that the general market rise is broadly based and extends to most of the stocks. This would
be viewed as a bullish sign.

Question: 3041

Which of the following is a method of assessing country risk?

That answer is correct!


A. Delphi technique
B. Simulation analysis
C. Monte Carlo simulation
D. Scenario Analysis
E. None of these answers is correct.
F. Darden case method
G. More than one of these answers is correct.

Page | 1170
Answer: A

Explanation:
The Delphi technique is a popular method of assessing country risk, and involves the collection of
several independent opinions the appropriate countries risk premium to be applied to the country
under examination. In the Delphi technique, a group of experts are asked to quantify the country risk
of a particular nation, without any input from other experts. By limiting any group discussion, the
Delphi technique seeks to provide a realistic quantification of the country risk premium.

"Simulation analysis," "Monte Carlo simulation," and "scenario analysis," are techniques designed to
measure stand-alone risk. The "Darden case method" is largely a fictitious term.

Question: 3042

Given that the expected dividend payout ratio is 0.34, the expected net profit margin is 0.16, the
expected total asset turnover is 0.94, the expected return on capital is 0.24, and the expected
financial leverage multiplier is 1.13, what is the expected growth rate of the firm?

A. 19%
B. 11%
C. 6%
D. 4%
E. Not enough information
F. 13%

Answer: B

Explanation:
The expected growth rate of the firm is equal to the expected retention rate multiplied by the
expected return on equity. The return on equity is equal to the expected net profit margin multiplied
by theexpected total asset turnover multiplied by the expected financial leverage multiplier (0.16 x
0.94 x 1.13 = 0.17). The expected retention rate is equal to 1 minus the expected dividend payout
ratio (1 - 0.34 = 0.66). In this question, the expected growth rate is equal to 0.66 x 0.17 = 0.11 = 11%

Question: 3043

When the relative strength of a stock with respect to an index is increasing, the stock is

A. keeping pace with the chosen index.


B. all of these answers are possible.
C. doing better than the chosen index.
D. worse than the chosen index.

Answer: C

Explanation:
The Relative Strength of a stock relative to an index equals the ratio of the stock price to the index
price. Hence, if the relative strength is increasing, it indicates that the stock is doing better in price
appreciation than the index.

Question: 3044

Given the following information, what would the expected industry rate of return equal?

Page | 1171
Retention rate= 80%
Net earnings estimate= $15.00/share
Multiple estimate= 22
Current earnings= $13.95/share
Current multiple= 21

A. 12.8%
B. 12.2%
C. 13.7%
D. 14.0%
E. 11.9%

Answer: C

Explanation:
Expected industry return = (Index estimate - Current index + Dividend) / Current index = (330 - 292.95
+ $3.00) / 292.95 = 13.7%

Index estimate = $15.00 x 22 = 330


Current index = $13.95 x 21 = 292.95
Dividend = (1 - .80) x $15.00 = $3.00

Question: 3045

Given that the risk-free rate of return is 6%, what is the value of a riskless zero-coupon bond with
which the principal payment is $10,000 in 15 years?

A. $5,733
B. $4,173
C. $5,929
D. $6,841
E. $7,126
F. Not enough information

Answer: B

Explanation:
The value of a zero-coupon bond is equal to the present value of its principal payment. The required
rate of return on a riskless bond is the risk-free rate of return. Usingappendix C in the book by Reilly
& Brown, the present value of the bond is $10,000 x0.4173 = $4,173, or $10,000/(1.06^15).

Question: 3046

Given the following information, what would the expected industry rate of return equal?

Dividend payout= 30%


Net earnings estimate= $12.62/share
Multiple estimate= 19
Current earnings index= 225.50

A. 8.5%
B. 9.0%
C. 8.0%
D. 7.5%

Page | 1172
E. 10.0%

Answer: C

Explanation:
Expected industry return = (Index estimate - Current index + Dividend) / Current index = (239.78 -
225.50 + $3.79) / 225.50 = 8.0%

Index estimate = $12.62 x 19 = 239.78


Dividend = .30 x $12.62 = $3.79

Question: 3047

Which of the following best describes the primary reason for the greater volatility of the earnings
multiplier of a stock market series compared to the earnings per share (EPS) for the same series?
Choose the best answer.

A. The price/earnings ratio is less insulated from accounting distortions than is the EPS figure.; i.e. it
is harder to "normalize."
B. None of these answers is correct.
C. The EPS figure is subject to a deleveraging effect caused by changes in the capital structure.
D. The earnings multiplier is more sensitive to changes in the spread between the required return
and growth.
E. The earnings multiplier is more sensitive to fluctuations in the equity markets than is the EPS
figure; i.e. the earnings multiplier is "forward looking."
F. The price/earnings ratio is more sensitive to increases in a companies dividends.

Answer: D

Explanation:
The greater relative volatility of the earnings multiplier versus the EPS figure is primarily attributable
to an increased sensitivity to changes in the spread between the required rate of return "k" and the
anticipated growth rate "g." Remember that the equation used to determine the appropriate
earnings multiplier for a stock market series is the following:

P/E = [D/E / (k - g)]

Where: P/E = the earnings multiplier, or Price-to-Earnings ratio, D/E = the dividend payout ratio at t1,
k = the required rate of return, and g = the anticipated growth rate of dividends.

As you can see, changes in the spread between the required rate of return and the anticipated
growth rate can have a dramatic effect on the earnings multiplier figure for a stock market series.
While the earnings multiplier is sensitive to changes in the dividend payout ratio, volatility in this
figure is not cause for the increased volatility of the earnings multiplier versus the EPS figure.

Question: 3048

Given that the P/E ratio on a common stock is 15, the expected dividend payout ratio is 0.6, and the
required rate of return is 19%, what is the dividend growth rate?

A. 13.4%
B. 9%
C. 12.8%
D. Not enough information

Page | 1173
E. 15%

Answer: E

Explanation:
The infinite period Dividend Discount Model claims that the current price of a common stock is equal
to D1 / (k - g), where D1 is next period's (most often next year's) dividend, k is the required rate of
return, and g is the growth rate of dividends. The earnings multiplier model goes a step further by
dividing both sides of the infinite period Dividend Discount Model equation by expected earnings
during the next 12 months, yielding P/E = (D1/E) / (k - g). Rearranging this results in g = k - (D1/E) /
(P/E). In this question, the dividend growth rate is equal to 0.19 - 0.6/15 = 0.15 = 15%

Question: 3049

The situation of monopsony is most closely affiliated with which of Porter's Five Forces of industry
competition?

A. Bargaining power of suppliers


B. Rivalry among existing firms
C. Threat of substitute products
D. Bargaining power of buyers
E. Threat of new entrants

Answer: D

Explanation:
A monopsony exists when there is only one buyer for a product or service. This is contrasted by a
monopoly, where there are many buyers and only one seller. Monopsony is most closely
characterized by the bargaining power of buyers, as the sole buyer has much power in influencing
prices. If there exists only one buyer for an industry or company's product, the bargaining power of
suppliers will be small. This will lead to increased competition in the industry.

While monopsony is a rare situation, it nonetheless occurs. For example, certain automobile parts
are purchased only by one of the large auto manufacturers. These automobile manufacturers have
tremendous pricing power from their suppliers, to many of which they are the only customer.

Question: 3050

A portfolio manager with an independent money management firm has been examining a stock
market series and is trying to determine an appropriate EPS figure for the series. In her research, this
portfolio manager has determined the following information:

1. Regressing sales for the series against Nominal GDP, the sales figure for the index has been
estimated at: $19.85.

2. Analyzing capacity utilization rates, foreign competition, rates of inflation and unit labor costs, the
operating profit margin for the series has been determined to be 31%.

3. Creating a time series based upon inputs such as levels of capital expenditures and PP&E turnover,
next year's depreciation-per-share has been determined to be: $1.18.

4. Creating a time series based upon levels of debt outstanding and prevailing debt yields, the
interest expense for next year is determined to be: $0.61 per share.

Page | 1174
5. Coordinating his research with a legislative consultant, the corporate tax rate for this series has
been estimated at: 34.77%.

Using this information, what is the EPS figure for this stock market series?

A. None of these answers is correct.


B. $2.85
C. $2.46
D. $4.03
E. The answer cannot be determined from the information provided. * $1.52

Answer: B

Explanation:
All of the necessary information has been provided in this example. To determine the EPS for a stock
market series, the following steps are necessary:

Step 1: Estimate sales-per-share for the series:


Step 2: Estimate operating profit margin for the series
Step 3: Estimate the depreciation-per-share for next year
Step 4: Estimate the interest expense-per-share for the next year
Step 5: Estimate next year's corporate tax rate

Once these five steps have been completed, the calculation of EPS for a stock market series is found
by the following:

EPS = [(Sales per share * Operating profit margin) - Depreciation-per-share - Interest Expense] * (1 -
Corporate Tax Rate)

The calculation of EPS for this stock market series is shown as follows: EPS = [($19.85 * 0.31) - $1.18 -
$0.61] * (1 - 0.3477) = $2.85

Question: 3051

Given that the required rate of return on a common stock is 17%, the dividend growth rate is 13%,
and the P/E ratio is 17, what is the expected dividend payout ratio?

A. 0.43
B. Not enough information
C. 0.46
D. 0.79
E. 0.68

Answer: E

Explanation:
The infinite period Dividend Discount Model claims that the current price of a common stock is equal
to D1 / (k - g), where D1 is next period's (most often next year's) dividend, k is the required rate of
return, and g is the growth rate of dividends. The earnings multiplier model goes a step further by
dividing both sides of the infinite period Dividend Discount Model equation by expected earnings
during the next 12 months,
yielding P/E = (D1/E) / (k - g). Rearranging this results in D1/E = (P/E) x (k - g). In this question, the
expected dividend payout ratio is equal to 17 x (0.17 - 0.13) = 0.68.

Page | 1175
Question: 3052

Volume considerations are

A. largely irrelevant to technical analysts.


B. the primary tool of technical analysts.
C. important to technical analysts.
D. not had by technical analysts.

Answer: C

Explanation:
Most technical trading rules consider both stock price and volume movements. The volume of
trading is an important, yet not dominant, variable in technical analysis.

Question: 3053

Given that a stock is correctly priced at $29.34, has an expected dividend payment of $1.30 in one
year, and has a required rate of return of 18%, what should its price be right after that dividend
payment in one year?

A. $35.68
B. Not enough information
C. $37.74
D. $29.41
E. $33.32
F. $34.95

Answer: E

Explanation:
In order for the stock to be correctly priced, the present value of next year's dividend plus the
present value of the stock price right after the dividend must equal the current price. In this question,
29.34 = (1.3 / 1.18) + (next year's price / 1.18). Rearranging this yields next year's price = [29.34 -
(1.3/1.18)] x 1.18 = $33.32.

Question: 3054

An uptick occurs when

A. the current day's closing price is higher than the previous day's closing price.
B. the current day's opening price is higher than the previous day's opening price.
C. the current day's closing price is higher than or equal to the previous day's closing price. * the
current day's opening price is higher than or equal to the previous day's opening price.
D. the current transaction price is higher than the last transaction price.

Answer: D

Explanation:
An uptick is simply when the current transaction price is higher than the last transaction price.

Question: 3055

In estimating a firm's earnings multiplier, the expected growth rate is determined by the firm's:

Page | 1176
That answer is correct!
A. retention rate and expected return on equity
B. estimated required rate of return and dividend payout ratio
C. estimated return on equity (ROE)
D. relationship to its industry and market

Answer: A

Explanation:
Higher expected growth rates would indicate that a firm should have a higher multiple than its
industry and the market.

Question: 3056

Given that the beginning value on a stock is $640, expected earnings are $80, the retention rate of
earnings is 40%, and the required rate of return is 21%, what is the minimum expected ending value
of the stock that makes it a profitable investment?

A. $725.40
B. $742.40
C. Not enough information
D. $726.40
E. $758.40

Answer: D

Explanation:
The dividend payout ratio is equal to one minus the retention rate (1 - 0.4 = 0.6). Expected dividends
are equal to the dividend payout ratio multiplied by expected earnings (0.6 x 80 = $48). In order for a
stock to be a good investment, its rate of return should be equal or greater than the required rate of
return.The minimum ending value that would make the stock investment in this question profitable is
given by the equation (P2 + D) / P1 = 1 + k, where P2 is the ending value, P1 is the beginning value, D
is the expected dividend, and k is the required rate return. Rearranging this yield P2 = ((k + 1) x P1) -
D. In this question, the minimum ending value is (1.21 x 640) - 48 = $726.40.

Question: 3057

Assuming that a preferred stock is fairly priced, is worth $23, and has annual dividends of $6, what is
its required rate of return?

A. 19%
B. Not enough information
C. 26%
D. 31%
E. 14%

Answer: C

Explanation:
The value of a preferred stock is the present value of its dividends, which is equal to the annual
dividend divided by the required rate of return. Rearranging this, the required rate of return is equal
to the dividend divided by the stock price. In this question, the required rate of return is equal to
6/23 = 0.26 = 26%.

Page | 1177
Question: 3058

Which of the following represents a "contrary opinion" technical indicator? Choose the best answer.

A. More than one of these answers are correct.


B. The Diffusion Index
C. T-Bill-Eurodollar Yield Spread
D. Debit balances in brokerage accounts (margin debt)
E. The Chicago Board Options Exchange (CBOE) Put/Call Ratio
F. The Confidence Index

Answer: E

Explanation:
Of the choices listed, only the "CBOE Put/Call Ratio" represents a contrarian technical indicator.
While "debit balances in brokerage accounts (margin debt)" appears to be an appealing choice, this
indicator is actually a measure of "smart money" because investors who leverage their portfolios
through margin loans are viewed by technical analysts as being sophisticated.

Technical analysts often examine the opinions of market participants, including future and options
traders, for indications of the overall sentiment of securities prices and the direction of impending
market movements. Believing that the majority of these market participants will be inaccurate in
their forecasts, particularly near market peaks and troughs, contrary opinion technical analysts will
take a contra approach.

When the ratio of puts to calls is high, indicating a high degree of bearishness amongst options
traders, contrary opinion technical analysts would look for an upward move in securities prices.
Conversely, contrary opinion technical analysts would view a low put/call ratio as a bearish signal.

The Confidence Index, a "smart money" technical indicator, is a measure of yield spreads between
highgrade corporate bonds and the yields on average corporate bonds. The Diffusion Index measures
the breadth of the market, and is found by taking the total volume of advancing shares plus one-half
of the issues unchanged, divided by the total number of issues traded. The T-Bill-Eurodollar Yield
Spread is another example of a "smart money" technical indicator.

Question: 3059

Joe Wellworth, an oil analyst with Smith, Kleen and Beetchnutty institutional brokerage, is trying to
determine an appropriate earnings multiplier for the natural gas industry. In his research, Mr.
Wellworth has examined the relationship between the earnings multiplier of the natural gas industry
and the Price-to-Earnings ratio of the Standard & Poors 500. Using a time series analysis, Joe
examines the trend in the relationship between the natural gas industry and the overall market and
uses this information to estimate the appropriate earnings multiplier for the natural gas industry.

Which of the following best characterizes this method of estimating the earnings multiplier of an
industry? Choose the best answer.

A. Correlation analysis
B. Microanalysis
C. The bottom-up approach
D. Time series analysis
E. Macroanalysis

Page | 1178
Answer: E

Explanation:
The answer called for in this example is macroanalysis. This method involves an examination of the
relationship between the earnings multiplier of a stock market series and the earnings multiplier of
the overall market. For example, an individual projecting an earnings multiplier for a software index
using macroanalysis would begin by examining the relationship between the P/E ratio of the
software index and the P/E ratio of a broad market index such as the Standard & Poors 500. Both
historical trends and point estimates would be examined, and from this information, a projection of
the earnings multiplier for the stock market series is deduced. This is precisely the process illustrated
in this example.

This is contrasted by microanalysis, which involves an examination of the components of the


earnings multiplier, including the anticipated growth rate of dividends, the required rate of return,
and the dividend payout ratio. Once these variables have been examined, both from the perspective
of trend analysis and point estimation, a value for the earnings multiplier is deduced.

The bottom-up approach is used in the investment selection process, and involves identifying
superior investments by first examining companies, rather than beginning with an examination of
macroeconomic cycles and influence. Time series analysis, while materially correct, does not
represent the best possible answer.
L1 SS 13

Question: 3060

An economist with Smith, Kleen & Beetchnutty Institutional Brokerage has been examining a stock
market series and is trying to determine the anticipated rate of return for the series. In her research,
this economist has determined the following information:

Anticipated ending value: 11,800


Expected dividends during the period: $521
Observed beginning value: 10,050.14
Required rate of return: 17.50%

Using this information, what is the anticipated rate of return for this stock market series? (Assume a
oneyear holding period).

A. 19.24%
B. None of these answers is correct.
C. 12.23%
D. 24.41%
E. 22.60%

Answer: E

Explanation:
To calculate the expected rate of return for a stock market series, the following information must be
known:

The beginning value for the series, the anticipated ending value for the series, and the amount of any
dividends and/or distributions during the period.

Once this information has been determined, the expected return on a stock market index can be
found by employing the following equation: {E(R) = [(EV - BV + Div) / BV]}. Where: E(R) = the

Page | 1179
expected return on the stock market series, EV = the anticipated ending value for the series BV = the
observed beginning value for the series, and Div = the amount of any dividends paid during the
period.

In this example, all of the necessary information has been provided and the calculation of the
expected return on this stock market series is found as follows: {E(R) = [$11,800 - $10,050.14 +
$521]/10,050} = 22.60%. This figure is significantly higher than the required rate of return. Assuming
that the anticipated ending value and expected dividends prove accurate, investment in this stock
market series is likely advisable.

Question: 3061

Alpha is a 9% load-fund, which you expect to have an annual rate of return of about 19% over the
next 2 years. Beta is a no-load fund, which is expected to have a rate of return of around 13%. If your
investment horizon is 2 years, which fund should you invest in and what is your expected net rate of
return per year?

That answer is correct!


A. Alpha; 13.5%
B. none of these answers
C. Beta; 13.0%
D. Alpha; 17.3%

Answer: A

Explanation:
With fund Alpha, a deposit of $100 will give you shares worth $91 after the load charge is taken into
account. This amount is expected to grow to 91*(1+0.19)^2 = $128.87. Thus, the net return with
Alpha is expected to be (128.87/100)^0.5 - 1 = 13.52% per year, annually compounded. Hence, for a
2 year horizon, you should select Alpha, since Beta has an annual rate of 13%.

Question: 3062

Which statement is true concerning Americans living longer?

That answer is correct!


A. All of these statements are true.
B. None of these statements are true.
C. The elderly have more disposable income than the younger population.
D. The income group that has advanced the most is that of the elderly.
E. The health care industry should expect to grow.
F. Youth-oriented industries should expect to shrink.

Answer: A

Explanation:
These are all trends that have happened or should expect to happen in the future given the fact that
there will be more older people in the population.

Question: 3063

A firm has a dividend growth rate of 4.3%. It typically pays out 45% of its earnings as dividends.
Recently, it paid out $1.2 per share dividend and the required rate of return on its stock is 12.6%. The
firm's return on equity equals ________.

Page | 1180
That answer is correct!
A. 7.82%
B. none of these answers
C. 9.56%
D. 12.60%

Answer: A

Explanation:
The dividend growth rate equals the product of ROE and the earnings retention ratio. The earnings
retention in this case equals 1 - 0.45 = 0.55. Hence, the ROE equals 4.3%/0.55 = 7.82%.

Question: 3064

What is the value of a bond with coupon payments of $150 every six months, a final payment of
$5,500 in 12 years, and a risk-premium of 8%?

That answer is correct!


A. Not enough information
B. $3,864
C. $2,239
D. $3,046
E. $1,240

Answer: A

Explanation:
In order to take the present value of the coupon and principal payments, one must know the
required rate of return on the bond. The required rate of return is equal to the risk-free rate plus the
risk premium. The risk-free rate is not given, so there is not enough information to answer the
question.

Question: 3065

Growth companies are:

A. all of these answers.


B. companies whose shares generate returns higher than stocks with similar risks.
C. companies that have consistently above-average sales and earnings.
D. companies that have management abilities and investment opportunities that yield rates of return
higher than the required rate of return.

Answer: D

Explanation:
Growth companies need not have stocks that generate higher than required rates of return. They
have rates of return on their investments which are higher than that required by the risks in the
investments since the market impounds all available information in the stock price. Further, growth
companies are measured in terms of the growth and not the level of sales and earnings.

Question: 3066

An analyst with Churn Brothers Brokerage is examining shares of the common stock of Nexis

Page | 1181
Pharmaceuticals. Consider the following information about Nexis' common stock:

Price per share: $98.73


Last dividend per share: $1.30
Expected growth rate: 12% per year
Required return: 11% per year

What is the value of Nexis' common stock? Choose the best answer.

A. $130
B. The Infinite Period DDM will produce a nonsensical answer in this case.
C. $146
D. $110
E. $89

Answer: B

Explanation:
The Infinite Period DDM will produce a nonsensical answer in this case because the required rate of
return is less than the growth rate. This will lead to a negative answer for the value of Nexis
Pharmaceutical common stock! The following equation illustrates the Infinite Period DDM:

{P0 = [D1 / (k - g)]}

Where: P0 = the price of the common stock at t0, D1 = the annual dividend at t1 (this is found by
multiplying the dividend at t0 by (1 + expected growth rate), k = the required rate of return, and g =
the anticipated growth rate.

As you can see, the Infinite Period DDM should not be used in this case, because the annual growth
rate is in excess of the required rate of return. In this instance, the analyst should first reexamine his
figures for the growth rate and required rate of return. If these figures are indeed accurate, then
another method should be used to value the stock.

Question: 3067

If the ratio of specialists' short sales to total short sales is 25%, then technicians would view this as

A. neither particularly bullish nor bearish.


B. a bearish sign.
C. a sign of an approaching flat market trend.
D. a sign of a approaching market peak.
E. a bullish sign.

Answer: E

Explanation:
Technical analysts trying to follow the "smart money" sometimes use the proportion of specialists'
short sales to total short sales as a market indicator. A decline of the ratio below 30% is viewed as a
bullish sign, indicating that specialists are trying to minimize participation in short sales in
expectation of a rising market. A increase in the ratio above 50%, conversely, is viewed as a bearish
sign.

Question: 3068

Page | 1182
The real risk-free rate of return depends most on

That answer is correct!


A. the real growth rate of the economy.
B. money supply.
C. interest rates.
D. inflation.

Answer: A

Explanation:
The real risk-free rate of return does not take inflation into account (in contrast to the nominal risk-
free rate). It should depend on the real growth rate of the economy because the capital invested
should grow at least as fast as the economy. The rate can also be temporarily affected by tightness or
ease in the capital markets.

Question: 3069

Which of the following have a clear positive relationship with operating profit margin?

A. Inflation
B. Unit labor costs
C. Capacity utilization rates
D. Foreign competition

Answer: C

Explanation:
Higher capacity utilization rates intuitively improves operating margin because of economies of
scale, though up to certain volume whereby typically there will be some diminishing returns.

Question: 3070

The government decides to change its fiscal stance by raising levels of general taxation. What layer of
the top-down equity valuation would the change impact the most?

A. It impacts all steps equally.


B. General economic forecast.
C. Security selection.
D. Projected economic outlook for the industry.

Answer: B

Explanation:
Government economic activity primarily impacts the first stage of the top down approach.

Question: 3071

Projecting a firm's net profit margin should include an analysis of:

That answer is correct!


A. all of these answers
B. the firm's relationship with its industry, which should indicate whether the company's past
performance is attributable to its industry or if it is unique to the firm

Page | 1183
C. the firm's specific competitive strategy, either low cost or differentiation
D. the firm's internal performance, including general company trends and consideration of any
problems that might affect its future performance

Answer: A

Explanation:
These three examinations help in understanding the firm's past performance and provide the
background to make meaningful estimates for the future.

Question: 3072

Which of the following is not a stage in the industrial life cycle?

A. Market deceleration and decline.


B. All of these are stages in the industrial life cycle.
C. Mature growth.
D. Horizon growth.
E. Accelerating growth.
F. Development.

Answer: D

Explanation:
The industry life cycle is divided into five distinct stages. Specifically, the industrial life cycle
progresses from the development stage to an accelerating growth stage to a mature growth stage to
a market maturation and stabilization stage. Finally, the fifth stage of the industrial life cycle is
characterized by decelerating growth and sales decline. As you can see, "horizon growth" is not a
stage in the industrial life cycle.

As an industry or company progresses through the industrial life cycle, sales begin to grow rapidly
(accelerating growth) then slow considerably as the product or service begins to reach critical mass
(mature growth). During the mature growth stage, the industry or company grows at a slower pace,
until eventually growth begins to slow considerably (market maturity and stabilization) as the market
for the industry or company's products becomes more completely defined. An industry or company
typically reaches the fifth stage of the industrial life cycle, growth deceleration and decline, only after
many years of stabilization. For example, the automobile became widely available in the early 20th
century, and the automobile industry has experienced a period of stabilization for nearly 50 years.
The market for automobiles is still experiencing single digit annual growth, and has arguably not
progressed into the decline stage.

Question: 3073

Studies have found that the confidence index

A. has been moderately useful for predicting stock price movements.


B. has not been very useful for predicting stock price movements.
C. has been very useful for predicting stock price movements.
D. has varied more or less logarithmically with stock index values.

Answer: B

Explanation:
Many technical analysts believe that the confidence index accurately reflects investors' attitudes

Page | 1184
toward financial markets. They view it as a useful way to follow the "smart money" on Wall Street.
Yet several studies have found that the index is not very useful for predicting stock price movements.

Question: 3074

A telecommunications analyst is attempting to value shares of Ludicrous Telecom, a multinational


fiberoptics and networking company. Assume the following information for Ludicrous shares:

Required rate of return: 14.50% per year


Free cash flow to equity multiple at t4: 30
1,000,000 shares outstanding

Additionally, the analyst has obtained the following estimates of free cash flow to equity for Ludicros
Telecom over the next four years:

Year 1: $4,000,000
Year 2: $4,500,000
Year 3: $3,500,000
Year 4: $4,800,000

Using this information, what is the value per share of Ludicrous Telecom according to the free cash
flow to equity model?

A. $24.64
B. $83.44
C. None of these answers is correct.
D. $54.60
E. The answer cannot be calculated from the information provided.
F. $99.34

Answer: C

Explanation:
When determining the value of a common stock using the free cash flow to equity model, it is
necessary to determine three things:

1. The required rate of return on equity investments.


2. The estimated free cash flow to equity multiple at time "k."
3. The estimated free cash flows figures for the time periods leading up to "k."

In this example, the calculation must begin with the discounting the free cash flow to equity figures
for each of the four years provided. These figures are discounted each period by the required return
on equity investments, and the final answer is converted to a per-share basis. This process is
illustrated below:

Year 1: ($4,000,000 / 1.145) / 1,000,000 shares outstanding = $3.49


Year 2: [$4,500,000 / (1.145)(1.145)] / 1,000,000 shares outstanding = $3.43
Year 3: [$3,500,000 / (1.145)(1.145)(1.145)] / 1,000,000 shares outstanding = $2.33
Year 4: [$4,800,000 / (1.145)(1.145)(1.145)(1.145)] / 1,000,000 shares outstanding = $2.79

Now that the free cash-flow-to-equity figures have been discounted and converted to a per-share
basis, the next step in the valuation process is to determine the value of the final cash flow, which is
defined as:

Page | 1185
[(Free cash flow to equity multiple * Final free cash flow) / (1 + r)(1+r)...(1 + k)]

In the body of this question, we were given the anticipated multiple of free cash to equity that shares
of Intelligent Semiconductor will sell for at time period: specifically, 30 times. Imputing this
information into the terminal cash flow equation will yield the following:

{[30 * ($4,800,000 / 1,000,000 shares outstanding)] / [(1.145) (1.145)(1.145)(1.145)]} = $83.78.


Adding the answers from step 1 to the final year cash flow will yield the following:

Value of Intelligent Semiconductor = [$3.49 + $3.43 + $2.33 + $2.79 + $83.78] = $95.82 per share.
This is illustrated by none of the answers provided.

Question: 3075

Which would have the most impact on an economy?

A. The increasing amount of older people.


B. The trend of less manufacturing jobs and more service industries.
C. More disparity between the rich and poor.
D. Population shifts to the Sun Belt, away from the Northeast.
E. None of these answers.
F. All of these answers are correct.

Answer: F

Explanation:
All of these important demographic and social changes have profound affects on the economy.

Question: 3076

Technical analysis

A. holds that superior returns can be gained from the use of fundamental economic and company
variables.
B. holds that past stock performance has no influence on future performance.
C. holds that past market price and volume data can be used to predict future performance.
D. is supported by the efficient market hypothesis.

Answer: C

Explanation:
Technical analysis, in contrast to the efficient market hypothesis and fundamental analysis, holds that
past price and volume data can be used to discover market trends that can predict future market
behavior. Technical analysts believe that "the market is its own best predictor."

Question: 3077

A stock that you are considering for purchase has the following characteristics:

Current dividend $2.00


Expected dividend in 1 year $2.20
Long term growth rate of dividends 10%
Required rate of return 14%

Page | 1186
Using the infinite period Dividend Discount Model, what is the maximum price that you would pay
for this stock?

A. $10.00
B. $12.00
C. $120.00
D. $15.00
E. $55.00
F. $150.00

Answer: E

Explanation:
The infinite period Dividend Discount Model indicates that:

Value = (Dividend for period 1)/(k-g) where k is the required rate of return and g is the growth rate. In
this case Value = ($2.20)/(.16-.10) = $2.20/.04 = $55.

Question: 3078

Which of the following best describes the primary reason for the greater volatility of the earnings
multiplier of a stock market series compared to the EPS for the same series? Choose the best answer.

A. The price-to-earnings ratio is less insulated from accounting distortions than is the EPS figure; i.e.
it is harder to "normalize."
B. The earnings multiplier is more sensitive to fluctuations in the equity markets than is the EPS
figure; i.e. the earnings multiplier is "forward looking."
C. The EPS figure is subject to a deleveraging effect caused by changes in the capital structure. *
None of these answers is correct.
D. The P/E ratio is more sensitive to increases in a company's dividends.
E. The earnings multiplier is more sensitive to changes in the spread between "k" and "g."

Answer: E

Explanation:
The greater relative volatility of the earnings multiplier versus the EPS figure is primarily attributable
to an increased sensitivity to changes in the spread between the required rate of return "k" and the
anticipated growth rate "g." Remember that the equation used to determine the appropriate
earnings multiplier for a stock market series is the following:

{P/E = [D/E / (k - g)]}

Where: P/E = the earnings multiplier, or Price-to-Earnings ratio, D/E = the dividend payout ratio at t1,
k = the required rate of return, and g = the anticipated growth rate of dividends.

As you can see, changes in the spread between the required rate of return and the anticipated
growth rate can have a dramatic effect on the earnings multiplier figure for a stock market series.
While the earnings multiplier is sensitive to changes in the dividend payout ratio, volatility in this
figure is not cause for the increased volatility of the earnings multiplier versus the EPS figure.

Question: 3079

Which of the following events would a technical analyst interpret as bullish?

Page | 1187
A. a decline in the ratio of specialist's short sales to total short sales
B. all of these answers
C. an increase in debit balances in brokerage accounts
D. when the proportion of bullish speculators declines to 30 percent

Answer: B

Explanation:
(1) Decline of specialist's short sale ratio to below 30%;
(2) Increase in debit balance means that investors have to post increasing margin to for more
investments;
(3) Percentage of bullish stock futures speculators decline (because if market is bullish then
speculation is irrelevant) - are all signs that the market may be turning bullish.

Question: 3080

Which of the following statements is true?

A. Given a small decrease in the real risk-free rate, a large increase in the risk premium for a stock,
and a large decrease in the return on equity, one would expect the required rate of return on the
stock to increase, the growth rate to decrease, and the earnings multiplier to increase.
B. Given a large decrease in the rate of inflation and the risk premium for a stock, and a large
increase in the return on equity, one would expect the required rate of return on the stock to
decrease, the growth rate to increase, and the earnings multiplier to increase.
C. Given a large increase in the rate of inflation, a small decrease in the risk premium for a stock, and
a large decrease in the return on equity, one would expect the required rate of return on the stock to
decrease, the growth rate to decrease, and the earnings multiplier to decrease.
D. Given a small increase in the rate of inflation, a small increase in risk premium for a stock, and a
larger decrease in the return on equity, one would expect the required rate of return on the stock to
increase, the growth rate to decrease, and the earnings multiplier to increase.

Answer: B

Explanation:
The direction of change approach to estimating an earnings multiplier begins with the current
earnings multiplier and estimates the direction of change for the dividend payout ratio and the
variables that influence the required rate of return and growth rate. The required rate of return is
positively correlated with the real risk-free rate, the rate of inflation, and the risk premium. The
growth rate is positively correlated with the return on equity and earnings retention rate. The
earnings multiplier is positively correlated with the growth rate, and negatively correlated with the
required rate of return.

Question: 3081

A firm follows the simple growth model and can invest $1000 every year into new projects which
have a rate of return of 17%. Each project generates a constant stream of earnings year after year.
The firm's stock has a required rate of return of 13%. If the firm is founded today, the firm value
equals ________.

A. $3,165
B. $1,895
C. $2,675
D. $2,440

Page | 1188
Answer: C

Explanation:
You should be careful in solving such a problem.

Suppose the firm invests $1,000 today in a project. This first project generates a constant stream of
perpetuity with an annual payment of $1,000 * 0.17 = $170. The present value of this stream is
170/0.13 = $1,307.6. The NPV of the project is then $1,307.6 - $1,000 = $307.6. Since the firm does
this every year, it has a perpetuity of $307.6 (the NPVs of projects 2 and on, at the time the projects
are started).The present value of these NPVs is $307.6/0.13 = $2,367. Thus, the total value of the firm
is $2,367 + $307.6 (first project) = $2,675.

Question: 3082

A mutual fund started last year by issuing 1,000 shares at $100 per share. This amount was invested
in 3 different stocks,

A - 500 shares at $50 per share


B - 600 shares at $70 per share
C - 1,000 shares at $33 per share

The current stock prices are $58 for A, $63 for B and $37 for stock C. The fund's NAV equals
________.

A. $100
B. $98.3
C. none of these answers
D. $102.9

Answer: C

Explanation:
The fund's total assets are worth 500 * 58 + 600 * 63 + 1,000 * 37 = 103,800. Hence, the NAV equals
103,800/1,000 = $103.8

Question: 3083

Which of the following is not a component of Porter's Five Force Model of industry competition?

A. All of these are components of Porter's Five Force Model.


B. Threat of new entrants.
C. Threat of substitute products.
D. Bargaining power of buyers.
E. Bargaining power of suppliers.
F. Threat of technological obsolescence.

Answer: F

Explanation:
Porter's Five Force Model of industry competition, named after its creator, famed Harvard professor
Micheal Porter, is used to measure the amount of competition within industries. The five forces
within this model are as follows:

Threat of new entrants

Page | 1189
Threat of substitute products
Bargaining buyer of buyers
Bargaining power of suppliers
Rivalry of existing firms

A shift in any of these threats will lead to a change in industry competition. As you can see, "threat of
technological obsolescence" is not named as an explicit threat within the Five Forces Model.

Question: 3084

Dewpoint Systems is a renowned firm, with a total asset turnover of 1.3. A conservative firm, it has a
low debt-to-equity ratio of 0.2. It manages to keep 15% of its sales as net profit and pays out 63% of
the earnings as dividends. It has a beta of 0.79, at a time when the market risk premium is 6.3%.
Assuming a risk-free investment rate of 6.4%, Dewpoint's price-to-earnings multiple equals
________.

A. 18.22
B. 23.16
C. 11.59
D. 13.62

Answer: B

Explanation:
The Dividend Discount Model implies that the firm's price-to-earnings ratio is given by Po/E1 =
(D1/E1)/(k-g) = (dividend payout ratio)/(k-g), using standard notation.

The dividend growth rate is given by g = ROE*(1-dividend payout ratio).

ROE = net income/common equity = (net income/sales)*(sales/assets)*(assets/equity) = profit


margin * asset turnover*(1 + debt / equity) = 0.15 * 1.3*(1+0.2) = 23.4%.

Therefore, g = 23.4%*(1-63%) = 8.66%.

The CAPM expected rate of return on the stock is equal to the risk-free rate plus beta times the
market premium. So the expected return on the stock is 6.4% + 0.79 * 6.3% = 11.38%. Therefore, the
price-to-earnings ratio equals 0.63/(11.38 - 8.66)% = 23.16.

Question: 3085

Which of the following factors determine the growth rate of dividends?

That answer is correct!


A. Growth rate of earnings and the payout ratio
B. Growth rate of earnings
C. Depends on the individual firm
D. Growth rate of earnings and cash flow

Answer: A

Explanation:
While growth rate of earnings gives a sense of future opportunities, the payout ratio determines how
much reinvestment is made to capture these opportunities. The better the firm performs, the higher
the chances of higher future dividends.

Page | 1190
Question: 3086

If the earnings per share on the Dow Jones were to fall, the Dow Jones:

A. the question is based on a false premise. Dow Jones is an index and does not have an associated
"earnings per share."
B. will increase in value.
C. will decrease in value.
D. may increase or decrease in value.

Answer: D

Explanation:
The index value = earnings multiplier * earnings per share.
Even if the earnings per share fall, the earnings multiplier can increase if the outlook for future
earnings improves. Hence, a decrease in the earnings itself does not automatically imply that the
Dow Jones will fall.

Question: 3087

Assume the following information for the common stock of Clay Industries, a large industrial firm:

Required rate of return on equity: 14.5% per year


Expected growth rate: 12.50% per year
Dividend at t0: $0.70

Assuming that the growth rate will remain constant, what is the projected value of Clay Industries
common stock?

That answer is correct!


A. $39.38
B. None of these answers
C. $32.78
D. The answer cannot be calculated from the information provided
E. $30.76
F. $35.34

Answer: A

Explanation:
In this example, the growth rate of dividends is assumed to remain stable, allowing the use of the
Gordon Model. The Gordon Model is also known as the "constant growth dividend discount model"
and takes the following form:

P0 = [D1 / (r - g)]

Where
P0 = the price of common stock X at time 0
D1 = the expected dividend at t1
r = the required rate of return on equity investments and g = the expected growth rate of dividends.

Since the dividend at t1 is not provided, we must calculate it manually by multiplying the dividend at
t0 by (1 + g). This will produce an answer of $0.7875 at t1.

Page | 1191
Now that the dividend at t1 has been determined, the given information can be put into the
equation provided, leading to the following series of calculations:

P0 = [$0.7875 / (.145 - .125)] = $39.38.

When using the Gordon model, remember that the required rate of return "r" must be greater than
the expected growth rate "g." Otherwise, this equation will produce a nonsensical answer.

Question: 3088

The NAV of a closed end fund _______ the market price of the fund.

That answer is correct!

A. can be less than, equal to or greater than


B. is greater than
C. is equal to
D. is less than

Answer: A

Explanation:
Historically, the market price of a closed end fund has been 5 to 20 percent below the NAV of the
fund. Closed end funds also can and do at times trade at a premium. Therefore the NAV can be less
than, equal to, or greater than the market price.

Question: 3089

The T-bill - Eurodollar yield spread should increase during periods of

A. high inflation.
B. low inflation.
C. international stability.
D. high consumption levels.
E. international crisis.

Answer: E

Explanation:
The T-bill - Eurodollar yield spread can be used as a measure of global investor confidence. During
periods of international crisis, it is believed that the spread will widen as money flows back into safe
U.S. T-bills.

Question: 3090

A firm has an expected dividend payout ratio of 50%, and an expected dividend growth rate of 6%
per year. What is the firm's Price/Earnings ratio if the appropriate discount rate is 10% per year?

A. 50
B. Not able to compute with the above data.
C. 12.5
D. 125

Page | 1192
Answer: C

Explanation:
Value = 0.50/(0.10-0.06) = 12.5.

Question: 3091

Assume the following information about a common stock:

Last annual dividend per share: $0.25


Price per share: $18.90
Required return: 15% per year
Expected growth rate: 11% per year

What is the value of this common stock?

A. $16.43
B. The answer cannot be determined from the information provided.
C. $6.25
D. $17.03
E. $6.94

Answer: E

Explanation:
To determine the value of a common stock using the Infinite Period Dividend Discount Model, use
the following equation:

{V = [d1 / (k - g)]}

Where: V = the value of the common stock at t0, d1 = the annual dividend at t1 (which is found by
multiplying d0 by (1 + g), k = the investor's required rate of return, and g = the anticipated annual
growth rate.

In this example, all of the necessary information has been provided, and incorporating this
information will lead to the following:

{V = [($0.25 * 1.11) / (0.15 - 0.11)] = $6.94

This value is significantly less than the price of the shares in the open market. While at first it may be
appealing to assume that the common stock is overvalued, this may be a dangerous assumption.
Equally likely is the possibility that the Infinite Period DDM is not the ideal valuation model for this
common stock. Perhaps the price of this common stock is reflecting other sources of potential cash
flows, ratherthan the summation of the present value of future dividends. This is an important point
to consider, and one with which you should become familiar.

Question: 3092

The upside-downside volume ratio is equal to

A. the total number of share prices on an exchange increasing divided by the number of share prices
decreasing. Technical analysts consider ratio values of 2.00 or greater to be indicative of an oversold
market.
B. the total volume of shares increasing on an exchange divided by the total volume of shares

Page | 1193
decreasing. Technical analysts consider ratio values of 1.25 or greater to be bullish.
C. the total volume of shares increasing on an exchange divided by the total volume of shares
decreasing. Technical analysts consider ratio values of 0.70 or less to be indicative of an oversold
market.
D. the total number of share prices on an exchange increasing divided by the number of share prices
decreasing. Technical analysts consider ratio values of 1.25 or greater to be indicative of excessive
market speculation.
E. the total number of share prices on an exchange increasing divided by the number of share prices
decreasing. Technical analysts consider ratio values of 2.00 or greater to be indicative of excessive
market speculation.

Answer: C

Explanation:
Technicians use the upside-downside volume ratio as an indicator of short-term momentum for the
market. The ratio typically ranges between 0.50 and 2.00. Ratio values of 1.25 or greater are viewed
as bearish, while values of 0.70 or less are viewed as bullish.

Question: 3093

Assume the following information about the common stock of a mid-sized regional bank.

Required rate of return on equity: 13.75% per year


Expected growth rate: 10.20% per year
Dividend at t0: $0.35

Assuming that the growth rate will remain stable, what is the value of this regional bank's common
stock?

A. $13.93
B. The answer cannot be calculated from the information provided.
C. $15.23
D. $10.86
E. $10.02
F. None of these answers is correct.

Answer: D

Explanation:
In this example, the growth rate of dividends is assumed to remain stable, allowing the use of the
Gordon Model. The Gordon Model is also known as the "constant growth dividend discount model"
and takes the following form:

P0 = [D1 / (r - g)]

Where
P0 = the price of common stock X at time 0
D1 = the expected dividend at t1
r = the required rate of return on equity investments and g = the expected growth rate of dividends.

Since the dividend at t1 is not provided, we must calculate it manually by multiplying the dividend at
t0 by (1 + g). This will produce an answer of $0.3857 at t1.

Now that the dividend at t1 has been determined, the given information can be put into the

Page | 1194
equation provided, leading to the following series of calculations:

P0 = [$0.3857 / (.1375 - .1020)] = $10.86.

When using the Gordon model, remember that the required rate of return "r" must be greater than
the expected growth rate "g." Otherwise, the equation will produce a nonsensical answer.

Question: 3094

An analyst with Churn Brothers Brokerage is attempting to value shares of Intelligent Semiconductor
using the Multi-stage Dividend Discount Model. Intelligent Semiconductor is expected to grow at a
rate of 35% per year for the next two years, grow at to 25% per year in years 3 and 4, and then grows
at 12% per year forever. Similar investments have warranted a 14.50% per year rate of return, and
Intelligent
Semiconductor paid a dividend of $0.70 at t0.

Using the information provided, determine the value of Intelligent Semiconductor shares according
to the Multi-stage Dividend Discount Model.

That answer is correct!


A. $55.98
B. None of these answers is correct.
C. $88.23
D. The answer cannot be calculated from the information provided.
E. $87.83
F. $91.87

Answer: A

Explanation:
The multi-stage dividend discount model is a more realistic way of valuing fast-growing companies
that pay dividends. With this model, it is necessary to estimate the above-average, or
"supernormal," rate ofgrowth, as well as the long-term rate of growth. Once these growth rates have
been determined, they are used to calculate the dividends at various points in the future. In this
example, you are provided with two supernormal growth rates, in addition to the long-term rate of
growth. Since two supernormal growth rates exist for Intelligent Semiconductor, the calculation is
somewhat more complex than a situation characterized by a single supernormal growth rate.
Nonetheless, the multi-stage dividend discount model can be applied.

The multi-stage dividend discount model is often referred to as the "two-stage dividend discount
model," and these two titles should be considered interchangeable for all intents and purposes, i.e.
the "two stage" dividend discount model can be used to determine the value of a company that has
multiple growth rate changes.

Incorporating the given information into the multi-stage dividend discount model will yield the
following:

P = {[$0.70 * 1.35) / 1.145] + [($0.945 * 1.35) / 1.31103] + [($1.276 * 1.25) / 1.50112] + [($1.595 *
1.25) / 1.71879] + [($1.994 * 1.12) / (.145 - .12]/1.71879}

Which can further be developed into:

P = {$0.82533 + $0.97309 + $1.06254 + $1.15998 + $51.96} = $55.98

Page | 1195
Question: 3095

An independent investment advisor is examining shares of Claypool Manufacturing, Inc. for possible
investment. In her examination, this investment advisor has gathered the following information:

Market discount rate: 12.5% per year


Observed Price/Earnings ratio: 14.25

Given this information, what is the Franchise Price/Earnings ratio for Claypool Manufacturing?

A. None of these answers is correct.


B. 6.25
C. 24.25
D. The answer cannot be calculated from the information provided.
E. 12.67
F. 16.28

Answer: B

Explanation:
The Franchise Factor method of value measurement is in many respects similar to EVA and MVA
calculations. When examining a company using the franchise value approach, the observed price-to-
earnings ratio is broken down into its two components - (1) the "base P/E," which is based on the
Company's ongoing performance, and (2) a "franchise P/E" that is based on the expected value of
new and profitable business opportunities. This relationship is illustrated as follows:

Franchise P/E = Observed P/E - Base P/E

Where the Base P/E equals the reciprocal of the market discount rate. For example, if the market
discount rate is 12.5%, the base P/E would be equal to (1 / 12.5%) = 8.

In this example, all the necessary information has been provided, and the calculation of the
Franchise P/E is as follows:

Franchise P/E = (14.25 - 8) = 6.25

Question: 3096

Which of the following correctly lists the two techniques for estimating the earnings multiplier for an
industry? Choose the best answer.

A. Residual earnings method and the arbitrage pricing theory


B. None of these answers is completely correct
C. Macroanalysis and microanalysis
D. The time series method and regression analysis
E. The industry life cycle method and the free-cash flow method
F. The top-down approach and the bottom-up approach

Answer: C

Explanation:
There is two common methods for estimating the earnings multiplier for an industry - macroanalysis
and microanalysis. Macroanalysis involves an examination of the relationship between the earnings
multiplier for an industry and the earnings multiplier for the market. Microanalysis involves an

Page | 1196
estimation of the specific variables that influence the earnings multiplier, including the required rate
of return, the estimated growth rate, and the dividend payout ratio. While the top down and bottom
up method are similar to macroanalysis and microanalysis, respectively, they do not represent the
best possible answer.
Specifically, the top-down and bottom up approaches are typically used to identify investment
opportunities, not for the estimation of an industry earnings multiplier.

Question: 3097

Point-and-figure charts include

A. high, low, and ending prices for a given period.


B. all ending prices.
C. all daily volumes and ending prices.
D. only significant price changes.

Answer: D

Explanation:
With a point-and-figure chart, a technical analyst determines which price interval to record as
significant (two points, for example) and records price changes equal to or larger than this, regardless
of time interval. Given a graph, an analyst would try to discern price trends.

Question: 3098

In a changing economy, a firm has experienced a fall in profit margin by 25%, a fall in asset turnover
of 15% and an increase in financial leverage of 20%. If the firm increases its payout ratio from 30% to
40%, what's the change in its dividend growth rate?

A. +12.14%
B. + 18.98%
C. -34.43%
D. -23.5%

Answer: C

Explanation:
Use g = ROE * retention ratio

ROE = profit margin * asset turnover * financial leverage.

The change in ROE equals (1-25%)*(1-15%)*(1+20%) - 1 = -23.5%. The retention ratio decreases from
70% to 60%. Therefore, the change in the growth rate equals (1-23.5%)*60/70 - 1 = -34.43%. Thus,
the growth rate falls by 34.43%.

Question: 3099

Which is a measure of profitability?

A. Debt to Equity Ratio


B. Price/Earnings Ratio
C. Return on Assets
D. Dividend Yield
E. Payout Ratio

Page | 1197
Answer: C

Explanation:
Return on Assets = Net Income/Total Assets; With other things equal, firms with relatively large
amounts of assets on their books (such as those in capital-intensive industries) will have low ROA
figures.

By contrast, service oriented firms having small amounts of capital on their books will have higher
ROA figures.

Question: 3100

Mutual fund A is a 7% load-fund, which you expect to have a rate of return of about 17%. Mutual
fund B is a no-load fund, which is expected to have a rate of return of around 9%. If your investment
horizon is 1 year, which fund should you invest in and what is your expected net rate of return?

A. A; 8.8%
B. A; 9.35%
C. B; 9.0%
D. none of these answers

Answer: C

Explanation:
With fund A, a deposit of $100 will give you shares worth $93 after the load charge is taken into
account. This amount is expected to grow to 93*(1+0.17) = $108.81. Thus, the net return with fund A
is expected to be 8.8%. Hence, for a 1-year horizon, you should select fund B, which is expected to
return 9%.

Question: 3101

An analyst is attempting to value shares Kingdom Semiconductor, a large silicon distributor. Assume
the following information for Kingdom shares:

Required rate of return on equity: 16.25% per year


Free cash flow to equity multiple at t4: 32
2,500,000 shares outstanding

Additionally, the analyst has obtained the following estimates of free cash flow to equity for Kingdom
over the next four years:

Year 1: $2,000,000
Year 2: $3,500,000
Year 3: $4,500,000
Year 4: $5,000,000

Using this information, what is the value per share of Kingdom Semiconductor according to the Free
Cash Flow to Equity Model?

A. $44.74
B. $46.29
C. None of these answers is correct.
D. $35.69

Page | 1198
E. The answer cannot be calculated from the information provided.
F. $39.02

Answer: F

Explanation:
When determining the value of a common stock using the free cash flow to equity model, it is
necessary to determine three things:

1. The required rate of return on equity investments.


2. The estimated free cash flow to equity multiple at time "k."
3. The estimated free cash flows figures for the time periods leading up to "k."

In this example, the calculation must begin with the discounting the free cash flow to equity figures
for each of the four years provided. These figures are discounted each period by the required return
on equity investments, and the final answer is converted to a per-share basis. This process is
illustrated below:

Year 1: ($2,000,000 / 1.1625) / 2,500,000 shares outstanding = $0.69


Year 2: [$3,500,000 / (1.1625)(1.1625)] / 2,500,000 shares outstanding = $1.04
Year 3: [$4,500,000 / (1.1625)(1.1625)(1.1625)] / 2,500,000 shares outstanding = $1.15
Year 4: [$5,000,000 / (1.1625)(1.1625)(1.1625)(1.1625)] / 2,500,000 shares outstanding = $1.10

Now that the free cash-flow-to-equity figures have been discounted and converted to a per-share
basis, the next step in the valuation process is to determine the value of the final cash flow, which is
defined as:

[(Free cash flow to equity multiple * Final free cash flow) / (1 + r)(1+r)...(1 + k)]

In the body of this question, we were given the anticipated multiple of free cash to equity that shares
of Intelligent Semiconductor will sell for at time period: specifically 32 times. Imputing this
information into the terminal cash flow equation will yield the following:
{[32 * ($5,000,000 / 2,500,000 shares outstanding)] / [(1.1625) (1.1625)(1.1625)(1.1625)]} = $35.04.

Adding the answers from step 1 to the final year cash flow will yield the following:

Value of Intelligent Semiconductor = [$0.69 + $1.04 + $1.15 + $1.10 + $35.04] = $39.02 per share.

Question: 3102

A preferred stock has a $100 par value and a dividend payout of $8 per year. Your required return is 9.
What is the value of the preferred stock?

A. not enough information to calculate it


B. $88.89
C. $92.24
D. $85.67

Answer: B

Explanation:
The value of a preferred stock is the stated annual dividend divided by the required rate of return on
preferred stock.

Page | 1199
In this case, V = $8/.09 = $88.89

Question: 3103

Holding other things constant, a decrease in a firm's retention rate of earnings will

A. increase the ROE.


B. decrease the ROE.
C. increase the firm's expected growth rate.
D. decrease the earnings multiplier.

Answer: D

Explanation:
A decrease in a firm's retention rate will decrease its expected growth rate of earnings and dividends.
This, in turn, will decrease the earnings multiplier. Changes in a firm's retention rate should not affect
its return on equity (ROE).

Question: 3104

Earnings multipliers for series of stocks (such as the S&P 400) have been found to be

That answer is correct!


A. more volatile than the earnings for series of stocks.
B. less volatile than earnings for series of stocks.
C. stable.
D. approximately as volatile as earnings for series of stocks.

Answer: A

Explanation:
The standard deviation of annual changes of the earnings multiplier for the S&P 400 is 28.2, while
that for the annual earnings for the S&P 400 is 18.3. To value stock series, one needs to estimate
both the earnings multiplier and the earnings. Although some analysts have tended to ignore the
earnings multiplier on the assumption that it is stable over time, its high measured variability makes
it hard to ignore.

Question: 3105

A preferred stock has a $100 par value and a dividend payout of $5 every 6 months. Your required
rate of return is 10%. What is the value of the preferred stock?

A. $50
B. none of these answers
C. $99
D. $75

Answer: B

Explanation:
The value of a preferred stock is the stated annual dividend divided by the required rate of return on
preferred stock.

In this case, V = $10 a year/.10 = $100

Page | 1200
Question: 3106

Consider the following annual growth forecasts for a common stock:

Growth in years 1-2 = 25%


Growth in years 3-4 = 15%
Growth after year 4 = 10%

Assuming that the last dividend was $1.30 per share, and that the required rate of return is 14% per
year, what is the value of this common stock?

A. $34.54
B. $28.28
C. $46.90
D. The answer cannot be determined from the information provided.
E. $49.90
F. None of these answers is correct.

Answer: E

Explanation:
To determine the value of a common stock experiencing temporary supernormal growth, use the
following equation:

{V = {[d0 * (1 + gs)^1] / k} + {[d1 * (1 + gs)^2} + ... {dn * (1 + gs)^n} + {[dn * (1 + gs)^n * (1 + gn] / (k -
g)}/ (1 + k)^n}}

Where: V = the value of common stock at t0, d0 = the dividend at t0, d1 = the dividend at t1, dn = the
dividend at tn, gs = the supernormal rate of growth, gn = the normal rate of growth, n = the time
period "n", and k = the required rate of return.

In this example, there is a transitional growth period of two years, during which the growth rate of
this stock is expected to grow at 15% annually. This period will follow the two-year supernormal
growth period, and would be denoted as "g subset t" if we were to reproduce the equation
illustrated above. The calculation of the value of this common stock is illustrated as follows:

{V = {[$1.30 * (1.25)^1] / (1.14)} + {[$1.30 * (1.25)^2] / (1.14)^2} + {[$1.30. * (1.25)^2 * (1.15)^1] /


(1.14)^3} + {[$1.30 * (1.25)^2 * (1.15)^2] / (1.14)^4} + {{[$1.30 * (1.25)^2 * (1.15)^2 * (1.10)^1]/
(0.14 - 0.10)}/ (1.14)^4}

Which can be deduced to the following:

{V = [$1.425439 + $1.562981 + $1.576691 + $1.590522 + $43.739352] = $49.89}

Question: 3107

Consider the following transactional information for the investment account of a retail investor:

1st Quarter
Ending portfolio value: $65,000
Total amount invested: $59,000

2nd Quarter

Page | 1201
Ending portfolio value: $63,500
Total amount invested: $65,000
3rd Quarter
Ending portfolio value: $60,900
Total amount invested: $63,500

4th Quarter
Ending portfolio value: $57,200
Total amount invested: $60,900

Using this information, what is the annual time-weighted rate of return for this portfolio? Assume no
taxes or transaction charges.

A. (4.56%) per year


B. (3.05%) per year
C. 2.59% per year
D. The calculation of the time-weighted rate of return cannot be calculated from the information
provided
E. (3.27%) per year
F. None of these answers

Answer: B

Explanation:
The time-weighted rate of return is the preferred method of return calculation in the investment
management industry, primarily because this method is not sensitive to significant additions and
withdrawals of funds from portfolios under examination. The calculation of the time-weighted rate
of return involves three steps, which are illustrated as follows:

Step 1:
Price the portfolio immediately prior to any significant additions or withdrawals. Separate the
portfolio into a series of subperiods based on the dates of cash inflows and outflows.

Step 2:
Calculate the holding period return for each subperiod.

Step 3:
Determine the annualized holding period return by linking or compounding the holding period return
of each subperiod. If the investment is for more than one year, use the geometric mean of the annual
returns as the time-weighted rate of return. If the investment is for less than one year, compound the
subperiod returns to obtain an annualized measurement.

To begin the process of determining the time-weighted rate of return, we would break the portfolio
up into the following series of cash flows. However, in this example, the cash flows are already
aggregated for us and we can move on to the next step: determining the holding period return for
each subperiod. This process is detailed as follows:

Quarter 1 holding period return = [($65,000 ending value - $59,000 invested) / $59,000 invested] =
10.169%
Quarter 2 holding period return = [($63,500 ending value - $65,000 invested) / $65,000 invested] =
(2.308%)
Quarter 3 holding period return = [($60,900 ending value - $63,500 invested) / $63,500 invested] =
(4.094 %)
Quarter 4 holding period return = [($57,200 ending value - $60,900 invested) / $60,900 invested] =

Page | 1202
(6.076%)

Now that the holding period return for each subperiod has been determined, we must annualize the
return measure by taking the product of all four quarterly returns. This process is illustrated below:

[(1 + .10169) * (1 - .02308) * (1 - .04094) * (1 - .06076) - 1] = (.0305) or (3.05%)

When calculating the time-weighted rate of return, remember that the total amount invested is the
relevant figure, not the beginning portfolio value. So said, whenever possible you should use the
total amount invested rather than the beginning portfolio value in the calculation of the subperiod
holding period return.

Question: 3108

Assuming that the nominal risk-free rate of interest is currently at 4.90% per year, with the real
inflationfree rate of return at 2.05% per year, what is the inflation premium? Further, assuming that
the inflation premium is to fall by 130 basis points, what would be the adjusted nominal risk-free rate
of interest?

Assume that both the new inflation rate and the inflation-free rate of interest are considered small
by historical standards.

A. 2.85% per year, increase by 70 basis points


B. 2.05% per year, increase by 130 basis points
C. The answer cannot be calculated from the information provided.
D. 2.85% per year, increase by 130 basis points
E. None of these answers is correct.
F. 2.05% per year, decrease by 130 basis points

Answer: E

Explanation:
Remember that the nominal risk-free rate of interest is comprised of two components, the real
"inflation-free" rate of interest and an inflation premium. The inflation premium is equal to the
anticipated inflation rate.

The equation for the calculation of the nominal interest rate when the real inflation-free rate of
interest and/or the inflation premium are significantly small is as follows:

Risk-free rate of return = k* + IP

where: k* = the real inflation-free rate of return and IP = the inflation premium

In this example, the nominal risk-free interest rate is provided as 4.90%, and the inflation-free rate of
return as 2.05%. Incorporating these values into the equation illustrated above will yield a value of
2.85% for the inflation premium.

Increasing the inflation premium by 130 basis points will lead to a 130 basis point increase in the
nominal risk-free interest rate. None of the answers correctly illustrate these findings.

When either the real "inflation-free" interest rate or the expected inflation rate are significantly
large, the calculation of the nominal risk-free rate differs from the equation used when these factors
are significantly small. Specifically, the calculation of the nominal risk-free rate of interest when the
inflation-free rate of interest and/or the inflation premium are significantly high, the calculation of

Page | 1203
the nominal risk-free rate is as follows:

Nominal RFR = (1 + Real RFR)(1 + E(I)) - 1

Where: Real RFR = the real inflation-free rate of interest and E(I) = the anticipated inflation rate

Question: 3109

In investment management applications, the Internal Rate of Return is commonly referred to as


which of the following?

That answer is correct!


A. Dollar-weighted rate of return
B. Intrinsic rate of return
C. Time-weighted rate of return
D. Cost-averaged rate of return
E. None of these answers
F. Dollar-averaged rate of return

Answer: A

Explanation:
Performance presentation is an important consideration in the field of investment management, and
often involves a rather complex series of calculations. This is particularly true for portfolio's which
are characterized by significant additions and withdrawals, as the cash flow measurement is
periodically disrupted. In the Level I readings, two specific measurement tools are discussed: the
dollar-weighted rate of return and the time-weighted rate of return. In investment management
applications, the IRR equation is commonly referred to as the "dollar-weighted rate of return"
because it incorporates both the timing and size of all cash flows in the determination of the return
figure.

Question: 3110

Technical analysts tend to believe that

A. successful fundamental analysis is impossible.


B. financial statements contain all relevant information needed to price stocks.
C. successful fundamental analysis is difficult.
D. the majority of investors can earn above-average returns through fundamental analysis.

Answer: C

Explanation:
Technical analysts tend to believe that while successful fundamental analysis is possible, it is very
difficult to achieve. They contend that since most fundamental analysis is heavily dependent on
financial statements, which do not contain all relevant stock pricing information, most investors
cannot consistently earn above-average returns using that method.

Question: 3111

A market researcher with Churn Brothers Brokerage is attempting to estimate the earnings per share
(EPS) for a pharmaceutical index, and has gathered the following information:

Sales per share: $200

Page | 1204
Next year's operating profit margin: 30%
Next year's depreciation per share: $18
Next year's interest expense: $13
Next year's corporate tax rate: 35%

Using this information, what is the EPS figure for this stock market series?

A. $17.75
B. None of these answers is correct.
C. $30.55
D. $36.85
E. $18.85
F. The answer cannot be calculated from the information provided.

Answer: E

Explanation:
The estimation of EPS for a stock market series involves five steps. Specifically, to determine an
estimate of EPS for a stock market series, it is necessary to:

Estimate the sales per share


Estimate next year's operating profit (EBIDT), or operating profit margin Estimate next year's
depreciation per share
Estimate next year's interest expense per share
Estimate next year's corporate tax rate

Once estimates for these components have been determined, they are put into the following
equation:

EPS for a stock market series = {[(Sales per share * operating profit margin) - depreciation per share -
interest expense per share] * (1 - corporate tax rate).

Imputing the given information into this equation will yield the following:

EPS for a stock market series = {[($200 * 0.30) - $18 - $13] * (1 - 0.35)} = $18.85

If you chose $36.85, remember that the depreciation figure is not added back to the EPS calculation.
What we are looking at is an operating earnings after tax figure, not a cash-based figure.

Question: 3112

Assuming that the dividend payout ratio is 0.4, net income is 100, net sales are 400, and the equity is
500, what is the growth rate of earnings of the firm?

A. 8%
B. 12%
C. 11.2%
D. Not enough information
E. 10%

Answer: B

Explanation:
The growth rate of earnings of the firm is equal to the retention rate multiplied by the return on

Page | 1205
equity (ROE). The retention rate is equal to one minus the dividend payout ratio (1 - 0.4 = 0.6). ROE is
equal to net income divided by equity (100/500 = 0.2). The growth rate is 0.6 x 0.2 = 0.12 = 12%.

Question: 3113

Which is not a true statement concerning industry analysis?

That answer is correct!


A. Industries that perform well in one time period will continue to outperform the aggregate market
in subsequent time periods.
B. Risk measures for different industries remain fairly constant over time.
C. During any time period, the returns for different industries vary within a wide range.
D. Company analysis is a necessary follow-up to industry analysis.
E. During any time period, different industries' risk levels vary within wide ranges.

Answer: A

Explanation:
Researchers found almost no association in individual industry performance year to year or over
sequential rising or falling markets.

Question: 3114

High payout ratios are closely correlated with which of the following attributes?

A. The level of competition in the industry.


B. More than one of these answers is correct.
C. The growth rate of earnings.
D. Maturity of the industry.
E. All of these answers are correct.
F. The level of regulation in an industry.

Answer: B

Explanation:
All but one of these answers is correct. Specifically, the dividend payout ratio is very closely related
to the maturity of the industry, the level of competition in the industry, and the growth rate in
earnings.
Remember that all of these factors are interrelated. As an industry becomes more mature,
competition becomes maximized, margins shrink, and the growth potential of the industry declines.
This will lead to a decrease in positive NPV investment opportunities in the industry, forcing firms to
increase the proportion of earnings paid out to shareholders in the form of dividends.

The level of regulation in an industry is not necessarily highly correlated with the dividend payout
ratio of firms within the industry. For example, the financial services industry is very heavily
regulated, yet firms within this sector do not pay a disproportionate amount of their earnings as
dividends. However, he utility industry, also heavily regulated, pays a very high proportion of
earnings as dividends.

Question: 3115

A preferred stock has a $100 par value and a dividend payout of $8 per year. What is the value of the
preferred stock?

Page | 1206
That answer is correct!
A. not enough information to calculate it
B. $102.61
C. $93.24
D. $89.65

Answer: A

Explanation:
The value of a preferred stock is the stated annual dividend divided by the required rate of return on
preferred stock.

The required rate of return in this case is not given.

Question: 3116

Which of the following is not an assumption of the infinite period Dividend Discount Model?

That answer is correct!


A. Earnings grow at a constant rate.
B. The required rate of return is greater than the growth rate of dividends.
C. The constant dividend growth rate will continue for an infinite period.
D. Dividends grow at a constant rate.

Answer: A

Explanation:
The infinite period dividend discount model does not concern itself with earnings. It assumes that
dividends (which are the basis for its valuation of stock), rather than earnings, grow at a constant
rate. But implicit in this assumption is the further assumption that earnings remain high enough to
finance the constant and often growing dividend payments.

Question: 3117

Given that a firm will exist for two years and issue dividends of $20 per share at the end of each year,
and the required rate of return on shares is 10%, what is the value of its common stock according to
the Dividend Discount Model?

A. $28.32
B. $40.00
C. $35.93
D. Not enough information
E. $34.71

Answer: E

Explanation:
The dividend discount model (Dividend Discount Model) assumes that the value of a share of
common stock is the present value of all future dividends. In this question, the value of a share of
common stock is equal to 20/(1 +0.10) + 20 /[(1 + 0.10)^2] = $34.71.

Question: 3118

Most studies

Page | 1207
A. weakly support technical analysis.
B. strongly support technical analysis.
C. have not supported technical analysis.
D. have drawn mixed results with regard to technical analysis.

Answer: C

Explanation:
The vast majority of studies have found that prices do not move in trends based on statistical tests of
autocorrelation, runs, and trading rules. But many trading rules have not yet been tested.

Question: 3119

The stock of a zero growth firm has a beta of 1.3 at a time when the market premium equals 7.7%
and the risk-free rate equals 5%. The firm's earnings multiplier ratio equals ________.

A. 7.19
B. information about earnings and dividends is missing
C. 6.66
D. 4.32

Answer: C

Explanation:
According to the Dividend Discount Model, P/E = payout ratio/(k-g) in standard notation. A no growth
firm has a retention ratio of zero and a payout ratio of 1. Further, g = 0 for no growth. Therefore, the
P/Eratio of a no-growth firm is simply equal to the reciprocal of its required rate of return. the firm's
CAPM required rate = 5% + 1.3 * 7.7% = 15%. The earnings multiplier is therefore equal to 1/0.15 =
6.66.

Question: 3120

Which of the following best describes the first step in the Top Down Approach to security valuation?

A. None of these answers


B. Analysis of the growth potential of specific industries
C. Identification of geographical growth trends
D. Analysis of specific companies
E. Analysis of the Macroeconomic Environment

Answer: E

Explanation:
The "top down" approach to investment identification begins with an analysis of the macroeconomic
environment, then moving on to an analysis of specific regions and countries. The later stages of the
top-down approach involve the analysis of specific industries and end with the analysis of specific
companies.

While "identification of geographical growth trends" is an appealing choice, it does not represent the
best possible answer. Had the question asked for the first step in the Bottom Up approach, then
"analysis of specific companies" would have been the correct choice.

Question: 3121

Page | 1208
Technical analysts would consider the fact that 90% of stocks are trading above their 200-day moving
average as

A. a sign of a market trough.


B. a sign that the market is overbought.
C. a bullish indicator.
D. a sign that the market is oversold.

Answer: B

Explanation:
Technical analysts may compute the moving average of a stock market series in order to determine
its general trend. The 200-day moving average has been popular. When over 80% of stocks are
trading above their 200-day moving average, the market is considered overbought. When less than
20% of stocks are trading above their 200-day moving average, the market is considered oversold.

Question: 3122

The industry growth rate is a function of the

A. return on equity multiplied by profit margin.


B. earnings per share multiplied by the retention rate.
C. payout ratio multiplied by the return on total assets.
D. required rate of return multiplied by the return on equity.
E. retention rate multiplied by the return on equity.

Answer: E

Explanation:
The growth rate is a function of the retention rate multiplied by the return on equity.

Question: 3123

Two stocks, A and B, have the same price. However, stock A has a dividend growth twice that of stock
B. If the recent dividend on A was half that on B, the expected return on A equals 5% and B's
dividend growth rate is 2%, the expected return on B is:

A. 1.75%
B. 4.0%
C. 10%
D. 2.5%

Answer: B

Explanation:
In the usual notation, Po = D1/(k-g) and D1 = Do*(1+g). Hence, Po = Do*(1+g)/(k-g).

The dividend growth rate of A equals 2 * 2% = 4%. We have been given PoA = PoB. Therefore, DoA*(1
+ 4%)/(kA - 4%) = DoB*(1 + 2%)/(kB - 2%). Also, DoA = 0.5* DoB and kA = 5%. Thence, 0.5 * 1.04/(5%
- 4%) = 1.02/(kB - 2%). Solving for kB gives the expected return on B equal to 3.96%

Question: 3124

Page | 1209
Macro techniques are based on the relationship between the economy and ________ markets.

A. international
B. none of these answers
C. micro
D. security

Answer: D

Explanation:
Macro techniques are based on the strong relationship between the economy and security markets.
These models base their market projections on their outlook for the aggregate economy and certain
components. Micro techniques estimate future market values by applying basic valuation models to
equity markets.

Question: 3125

In practice, preferred stocks have

A. higher rates of return than junk bonds.


B. lower rates of return than corporate bonds.
C. higher rates of return than common stock.
D. higher rates of return than moderately rated bonds.

Answer: B

Explanation:
Although one would think that preferred stocks would have higher rates of return than many bonds
because their dividend payments are riskier than bond interest and principal payments, it turns out
that preferred stocks have a lower rate of return than corporate bonds because of the tax advantages
preferred stocks present corporations (thereby increasing their demand and decreasing their yield).

Question: 3126

A confidence index value of 105 is

A. a bullish sign.
B. impossible.
C. an unimportant sign.
D. a bearish sign.

Answer: B

Explanation:
The confidence index is equal to the average yield on 10 top-grade corporate bonds divided by the
yield on the Dow Jones average of 40 bonds, multiplied by 100. Since the average yield on high-grade
bonds will always be higher than the average yield on a large cross section of bonds, the index will
always have a value under 100.

Question: 3127

An investor purchases 1,000 shares of Clay Industries common stock for $40.00 per share at t0. At t1,
this investor receives a $0.78 per share dividend on the 1,000 shares and purchases an additional 400
shares for $49.75 per share. At t2, he receives another $0.78 per share dividend on 1,400 shares and

Page | 1210
purchases 400 more shares for $55.90 per share. At t3, sells 1,000 of the shares for $59.50 per share
and the remaining 800 shares at $60.25 per share. Assuming no commissions or taxes, what is the
dollar-weighted rate of return received on this investment?

A. 18.22%
B. None of these answers is correct.
C. 15.42%
D. The answer cannot be calculated from the information provided.
E. 15.07%
F. 17.70%

Answer: B

Explanation:
Remember that the dollar-weighted rate of return uses the IRR equation in the determination of its
answer. Further, the dollar-weighted rate of return is another name for the IRR equation, and this
name is commonly used within the field of investment management. In the determination of the
dollar-weighted rate of return calculation, the first step should be to identify the cash flows for each
period. This process is illustrated as follows:

t0: {-[1,000 shares purchased * $40 per share] = [$40,000.00]


t1: {-[ 400 shares purchased * $49.75 per share] + [$0.78 per share dividend * 1,000 shares] =
[$19,120]
t2: {-[400 shares purchased * $55.90 per share] + [$0.78 per share dividend * 1,400 shares]} =
[$21,268]
t3: {[1000 shares sold * $59.50 per share] + [800 shares sold * $60.25 per share] = $107,700.00

Now that the cash flows have been determined, incorporating this information into your calculator's
cash flow worksheet and solving for IRR will yield a dollar-weighted rate of return of 13.70% for this
investment, which is none of the answers provided.

Question: 3128

The third step of the three-step, top-down approach to valuation is to decide to allocate investment
funds

A. among the various types of securities.


B. among individual firms, and the various securities they offer.
C. among the various groups of firms.
D. among individual firms.

Answer: D

Explanation:
The third step of the three-step, top-down approach is the analysis (for purposes of allocating
investment funds) of individual companies and stocks. The decision of types of securities to invest in
is made in the first step, and the decision to focus on certain industries is made in the second step.

Question: 3129

Which of the following securities is commonly valued as a perpetuity? Further, which of the following
best describes the equation used to value this security?

A. Zero coupon bond; {P0 = [Face value / (1 + k)^n]}

Page | 1211
B. More than one of these answers is correct
C. Preferred stock; {P0 = [d1 / k]}
D. Common stock; {P0 = [d1 / g]}
E. Zero coupon bond; {P0 = [Face value / (1 + k)^n] + g}
F. Preferred stock; {P0 = [(d1 / k) +g]}

Answer: C

Explanation:
A "perpetuity" is an investment which is expected to last forever. Preferred stock is commonly valued
as a perpetuity, using this equation:

{P0 = [d1 / k]}

Where: , P0 = the price of the preferred stock at time 0, d1 = the dividend at t = 1 and k = the
required rate of return.

A zero coupon bond is not an example of a perpetuity, because the duration of the cash flows
produced by a zero coupon bond has a finite and measurable life. Common stock, on the other hand,
is sometimes valued as a perpetuity, but the equation provided in this example is incorrect.

Question: 3130

Which of the following firms would likely have the lowest dividend payout ratio? Further, the capital
structure of this firm would likely be weighted more heavily with debt or equity? Choose the best
answer.

A. A large computer manufacturer; debt


B. A nationwide grocery chain; debt
C. A specialty retailer; equity
D. A healthcare company specializing in home care; debt
E. A fledgling biotechnology company; equity

Answer: E

Explanation:
Of the firms listed, the biotechnology company would be expected to have the lowest dividend
payout ratio. The nascent biotechnology industry is characterized by a high degree of investment in
research and development, along with a high growth rate and high level of uncertainty. All of these
factors are conducive to a high retention rate, i.e. a low dividend payout ratio.

Further, firms within industries associated with high growth rates and high levels of uncertainty are
likely to be financed primarily with equity. This is reasoned by several factors, some of which include
the following:

Firms in new industries typically have a balance sheet weighted heavily toward intangible assets,
which can neither be easily liquidated nor pledged as collateral for a loan.

Firms in new industries are characterized by a high degree of research and development expenses.

Equity provides firms with a higher degree of agility than debt financing.

The cash flows of firms in developing industries are characterized by a high degree of uncertainty.

Page | 1212
Question: 3131

The dividend payout ratio

A. varies positively with the retention rate of earnings.


B. varies inversely with the retention rate of earnings.
C. varies convexly with the retention rate of earnings.
D. does not vary with the retention rate of earnings.

Answer: B

Explanation:
The dividend payout ratio is simply one minus the retention rate (what is not retained is paid out in
dividends). An increase in the dividend payout ratio would increase the earnings multiplier. But this
would be partially offset by the decrease in the retention rate, which would decrease the growth rate
in earnings and dividends, which would, in turn, decrease the earnings multiplier.

Question: 3132

Given that the correct value of a common stock is $29, the dividend growth rate is 6%, and next
period's dividend is $2, using the infinite period Dividend Discount Model, what is the required rate
of return on the common stock?

A. 6.9%
B. Not enough information
C. 18.3%
D. 14.8%
E. 12.9%

Answer: E

Explanation:
The infinite period Dividend Discount Model postulates that the current value of a common stock is
equal to D1 / (k - g), where D1 is next period's dividend, k is the required rate of return, and g is the
growth rate of dividends. Rearranging this yields k = D1/(current value) + g. In this question, the
required rate of return is (2 / 29) + 0.06 = 0.129 = 12.9%.

Question: 3133

Given that the expected dividend payout ratio for a firm is 0.25, its expected profit margin is 0.12, its
expected financial leverage is 0.95, its expected return on capital is 1.26, its expected EBIT is $403,
and its expected growth rate is 8%, what is the firm's expected total asset turnover?

A. 1.39
B. 0.939
C. 1.02
D. Not enough information
E. 0.839

Answer: B

Explanation:
The growth rate of a firm is equal to expected retention rate of earnings multiplied by its expected
ROE. The retention rate is equal to 1 - dividend payout ratio (1 - 0.25 = 0.75). The ROE is thereby

Page | 1213
equal to the growth rate divided by the retention rate (0.08 / 0.75 = 0.107). The ROE can be set equal
the profit margin multiplied by the total asset turnover multiplied by financial leverage. Rearranging
this yields that the total asset turnover is equal to the ROE divided by the profit margin and the
financial leverage: 0.107 / (0.12 x
0.95) = 0.939.

Question: 3134

Interest rate expense per share

A. has consistently increased from 1977 to the present.


B. increased throughout the 1980s and decreased in the early 1990s.
C. has fluctuated widely from year to year.
D. has consistently decreased from 1977 to the present.

Answer: B

Explanation:
U.S. firms in the 1980s assumed increased debt financing and financial risk. As a result, interest rate
expenses increased consistently during that period. This growth was reversed after the 1989-1990
recession because of a decrease in debt levels and interest rates.

Question: 3135

Which of the following represents a "contrary opinion" technical indicator?

A. The Diffusion Index.


B. Investment advisor opinions.
C. T-Bill-Eurodollar spread.
D. The Confidence Index.
E. Short sales by specialists.
F. More than one of these answers is correct.

Answer: B

Explanation:
Of the choices listed, only "investment advisor opinions" represents a contrary opinion technical
indicator. Technical analysts often believe that the majority of market participants are incorrect in
their opinions about market direction and valuation, especially preceding market peaks and troughs.
This style of thinking is often referred to as "contrary opinion" technical analysis.

Contrarian technical analysts often examine the opinions of market participants, including
investment advisors and futures speculators, for insight into the overall sentiment of securities
professionals.

Believing these individuals to be inaccurate in their forecasts, contrary opinion technical analysts
assume the contra-side opinion. So said, a high degree of bullishness amongst investment advisors is
viewed as bearish by contrary opinion technical analysts. Conversely, a high degree of bearishness
amongst investment advisors is viewed as a bullish signal by contrary opinion technical analysts.

The Confidence Index is a measure of yield spreads between high-grade corporate bonds and the
yields on average corporate bonds. The Diffusion Index measures the breadth of the market, and is
found by taking the total volume of advancing shares plus one-half of the issues unchanged, divided
by the total number of issues traded.

Page | 1214
Question: 3136

Given current earnings of $2 per share, an expected dividend growth rate of 10% and a P/E of 12.5,
what is the value of the stock?

A. $30.50
B. $22.50
C. None of these answers
D. $27.50

Answer: D

Explanation:
The next period earnings is $2.00 x 1.10 or $2.20. The value of the stock is thus the P/E ratio times
the earnings = 12.5 x $2.20 or $27.50

Question: 3137

A portfolio manager is examining shares of Ottawa Industries to determine if the shares are fairly
valued. Ottawa Industries common stock is currently trading at $70 per share on the New York Stock
Exchange. In her analysis, the portfolio manager determines the value of Ottawa Industries common
stock using the two-stage dividend growth rate model. Due to the release of several important new
products, the Company is anticipated to grow at a rate of 16% per year for the next three years. After
this supernormal growth rate period, Ottawa Industries is anticipated to return to its long-term
growth rate of 12% per year. The Company recently paid an annual dividend of $0.90 per share, and
investments of a similar nature have warranted a 13.5% per year rate of return.

Given this information, what is the value of Ottawa Industries common stock? Use the dividend
discount model. Additionally, are shares of Ottawa Industries currently overvalued, undervalued, or
fairly valued in the stock market?

A. None of these answers is correct.


B. $74.56, undervalued
C. $103, undervalued
D. $98, overvalued
E. The answer cannot completely be calculated from the information provided.
F. $100, fairly valued

Answer: B

Explanation:
The multi-stage dividend discount model is a more realistic way of valuing fast-growing companies
that pay dividends. With this model, it is necessary to estimate the above-average, or
"supernormal," rate of growth, as well as the long-term rate of growth. Once these growth rates
have been determined, they are used to calculate the anticipated annual dividends leading up to the
"horizon point" at which the growth rate decelerates to the long-run rate of growth.

Incorporating the given information into the two-stage dividend discount model will yield the
following

P = {[$0.90 * 1.16) / 1.135] + [($1.044 * 1.16 / 1.28823] + [($1.21104 * 1.16) / 1.46214] + [($1.40481 *
1.12) / (.135 - .12)]/1.46214}

Page | 1215
Which can further be developed into:

P = {$0.91982 + $0.94008 + $0.96079 + $71.74} = $74.56

Assuming that the results produced by the multi-stage dividend discount model are accurate, Ottawa
industries common stock appears to be undervalued.

Question: 3138

Which of the following variables have a positive relationship with the P/E ratio?

A. Dividend payout ratio


B. Both the growth rate of dividends and the dividend payout ratio
C. Growth rate of dividends
D. Required rate of return

Answer: B

Explanation:
If larger dividends are paid out, one is willing to pay a higher price for the stock. However, if a high
rate of return is required, one should be paying less for the dividend streams in general.

Question: 3139

The two most important variables affecting aggregate profit margin are

A. the capacity utilization rate and rate of inflation.


B. foreign competition and unit labor costs.
C. the capacity utilization rate and foreign competition.
D. the rate of inflation and unit labor costs.
E. the capacity utilization rate and unit labor costs.

Answer: E

Explanation:
Analysis of annual data for the period of 1977 to 1994 has shown that there is a significant positive
relationship between the capacity utilization rate and operating profit, and a significant negative
relationship between unit labor costs and operating profit. The rate of inflation and foreign
competition variables were not found to be significant in a multiple regression on operating profit.

Question: 3140

Which of the following factors is not an underlying assumption of technical analysis?

A. Supply and demand is driven by rational and irrational behavior.


B. Prices move in trends that persist for long periods of time.
C. The actual shifts in supply and demand cannot be observed in market behavior.
D. Prices are determined by supply and demand.

Answer: C

Explanation:
Shifts in supply and demand CAN be observed in market behavior.

Page | 1216
Question: 3141

In general, the earnings multiplier for a stock market series is a more volatile figure than the
earnings-per-share for the same series. The greater relative volatility of the earnings multiplier is
mostly attributable to which of the following?

That answer is correct!


A. The earnings multiplier is more sensitive to changes in the spread between the required return
and growth.
B. The earnings per share (EPS) figure is subject to cash flow adjustments, which "normalize" the EPS
figure over time.
C. The earnings multiplier is more sensitive to changes in the payout ratio.
D. The earnings multiplier is subject to a tax-deleveraging effect.
E. None of these answers is correct.

Answer: A

Explanation:
The greater relative volatility of the earnings multiplier versus the EPS figure is primarily attributable
to an increased sensitivity to changes in the spread between the required rate of return "k" and the
anticipated growth rate "g." Remember that the equation used to determine the appropriate
earnings multiplier for a stock market series is the following:

P/E = [D/E / (k - g)]

Where: P/E = the earnings multiplier, or Price-to-Earnings ratio, D/E = the dividend payout ratio at t1,
k = the required rate of return, and g = the anticipated growth rate of dividends.

As you can see, changes in the spread between the required rate of return and the anticipated
growth rate can have a dramatic effect on the earnings multiplier for a stock market series. While the
earnings multiplier is sensitive to changes in the dividend payout ratio, volatility in this figure is not
cause for the increased volatility of the earnings multiplier versus the EPS figure.

Question: 3142

The ________ is responsible for differences in the required rates of return among alternative
investments.

A. real premium
B. risk rate
C. bond rate
D. risk premium

Answer: D

Explanation:
Investors demand higher return for riskier investments and thus require a higher rate of return on
them.

Question: 3143

Firms in which of the following industries would likely have the highest earnings retention rate?
Further, are firms in this industry are likely to be financed primarily through debt or equity?

Page | 1217
That answer is correct!
A. Fiber optics networking; equity
B. Computer manufacturing; debt
C. Media; equity
D. Media; debt
E. Computer manufacturing; equity
F. Fiber optics networking; debt

Answer: A

Explanation:
Of the industries listed, fiber optics networking would be expected to have the highest earnings
retention rate. The fiber optics networking industry is characterized by a high degree of investment
in research and development, along with a high growth rate and level of uncertainty. All of these
factors are conducive to a high retention rate, i.e. a low dividend payout ratio. Further, firms within
industries associated with high growth rates and high levels of uncertainty are likely to be financed
primarily with equity. This is reasoned by several factors, some of which include the following:

Firms in new industries typically have a balance sheet weighted heavily toward intangible assets,
which can neither be easily liquidated nor pledged as collateral for a loan.

Firms in new industries are characterized by a high degree of research and development expenses.

Equity provides firms in emerging industries with a higher degree of agility than debt financing.

The cash flows of firms in developing industries are characterized by a high degree of uncertainty,
which often prevents the reliable payment of debt obligations.

Question: 3144

Estimate the dividend growth for a company that exhibits the following characteristics:

Retention rate 75%


Payout rate 25%
Return on Equity (ROE) 20%

That answer is correct!


A. 15%
B. 12%
C. 10.0%
D. 9.6%
E. 6.4%
F. 25%

Answer: A

Explanation:
The estimated growth rate of dividends = (Retention Rate) x (Return on Equity). In this case the
estimated growth rate of dividends = 75% x 20% = 15%.

Question: 3145

An increase in margin debt would by viewed by technical analysts as

Page | 1218
A. signaling a market peak.
B. neither particularly bullish nor bearish.
C. a bearish sign.
D. a bullish sign.
E. irrelevant.

Answer: D

Explanation:
Those who purchase securities on margin tend to be relatively sophisticated investors. Technical
analysts trying to follow the "smart money" would view an increase in margin debt (debit balances in
brokerage accounts) as indicating that those sophisticated investors are bullish on the market, which
would lead the analysts to also be bullish.

Question: 3146

The situation of monopoly is most closely affiliated with which of Porter's Five Forces of industry
competition? Choose the best answer.

A. Threat of substitute products


B. None of these answers is correct.
C. Rivalry among existing firms
D. Bargaining power of buyers
E. Threat of new entrants

Answer: B

Explanation:
None of these answers is correct. The situation of monopoly is most closely associated with the
bargaining power of suppliers. When there exists only one supplier of a product, that supplier
possesses a great deal of influence over prices and delivery terms.

Monopolies are thus discouraged and regulated in many instances.

The bargaining power of buyers refers to the influence that customer's have on suppliers. This
bargaining power is greatest in the situation of monopsony, which is characterized by only one buyer.

While "rivalry among existing firms" is an appealing choice, it does not represent the best possible
answer.

Question: 3147

The earnings multiplier is also known as

A. the operating leverage ratio.


B. the profit margin multiplied by the total asset turnover.
C. the P/E ratio.
D. the ROE ratio.

Answer: C

Explanation:
The earnings multiplier, which is the basis for a method of valuing stocks, is also known as the price /
earnings (P/E) ratio. It is usually set equal to the current market price divided by expected 12-month

Page | 1219
earnings.

Question: 3148

Given that the expected dividend payout ratio on a common stock is 0.77, the required rate of return
is 23%, the dividend growth rate is 18%, using the earnings multiplier model, what is the estimated
value of the stock?

A. $28.51
B. $23.94
C. $34.91
D. $41.84
E. Not enough information

Answer: E

Explanation:
In order to estimate the value of the stock using the earnings multiplier model, one will eventually
have to multiply the P/E ratio by next year's earnings (E). The P/E ratio can be determined with the
information presented in this problem, but E cannot be.

Question: 3149

The confidence index is:

That answer is correct!


A. the ratio of the average yield on 10 top grade bonds to Dow Jones average of 40 bonds.
B. the ratio of the volume of the 5 top grade bonds to the volume of on-the-run T-bills.
C. the ratio of the total NYSE capitalization to the outstanding short interest.
D. The ratio of long-term bond spread to aggregate market dividend yield.

Answer: A

Explanation:
The Confidence index, published by Barron's, is the ratio of the average yield on 10 top grade bonds
to the Dow Jones average of 40 bonds. The index measures the difference in yield spreads between
high-grade bonds and a large cross-section of available bonds. Since high-grade bonds always have
lower yields, the index value never exceeds 100. An increase in this index implies that yields on high-
grade bonds and those on ordinary bonds have converged i.e. the yield spread between the bonds
has narrowed. From a demand perspective, this will happen when the demand for riskier bonds rises
i.e. when investors are bullish.

Question: 3150

Which of the following is not one of the three direct variables that affect an economy's real growth
rate?

That answer is correct!


A. Technological progress
B. Growth rate of the labor force
C. Growth rate of labor productivity
D. Growth rate of the average number of hours worked

Answer: A

Page | 1220
Explanation:
While technological progress is exceedingly important to an economy's real growth rate, its effect is
always indirect. Technological progress most often increases labor productivity, which in turn
increases the real growth rate.

Question: 3151

The earnings multiplier of a stock

A. is determined by the expected dividend payout ratio, the return on capital, and the expected
growth rate of dividends for that stock. The earnings multiplier tends to be rather stable between
stocks and industries, and does not vary significantly over time.
B. is determined by the expected dividend payout ratio, the required rate of return, and the expected
growth rate of dividends for that stock. The earnings multiplier tends to vary considerably between
stocks and industries, and over time.
C. is determined by the expected earnings, the required rate of return, and the expected growth rate
of dividends for that stock. The earnings multiplier tends to vary between stocks and industries, but
is rather stable over time.
D. is determined by the expected dividends, the required rate of return, and the expected growth
rate of dividends for that stock. The earnings multiplier tends to vary considerably between stocks
and industries, and over time.

Answer: B

Explanation:
The earnings multiplier (or price/earnings ratio) is equal to the expected dividend payout ratio,
divided by the spread between the required rate of return and growth rate of dividends on the stock.
Estimation of the earnings multiplier is important because it varies between stocks and industries as
well as over time. It, along with future earnings, is used to estimate the future market value of stocks
or stock market series.

Question: 3152

If a stock has an expected dividend payout of 50 percent, a required rate of return of 14 percent and
an expected dividend growth rate of 9 percent, what is the P/E ratio?

A. 12.5
B. None of these answers
C. 8.5
D. 10

Answer: D

Explanation:
The price/earnings ratio can be computed by dividing the expected dividend payout ratio (dividends
divided by earnings) by the required rate of return (k) minus the expected growth rate of dividends
(g).

In this case, P/E = .50/(.14-.09) = 10

Question: 3153

Consider the following preferred stock:

Page | 1221
Price per share: $70.10
Semiannual dividend: $3.75 per share
Required rate of return: 12% per year

Is the preferred stock realistically overvalued, undervalued, or correctly valued? Further, should this
preferred stock be valued as a perpetuity or a finite series of cash flows? (Assume a long-term
holding period).

A. Correctly valued; perpetuity


B. Undervalued; perpetuity
C. Overvalued; finite series of cash flows
D. Undervalued; finite series of cash flows
E. Correctly valued; finite series of cash flows
F. Overvalued; perpetuity

Answer: F

Explanation:
The preferred stock profiled in this example is trading above its theoretical value, which is found to
be $62.50. Assuming that this value is accurate, then the stock appears to be overvalued in the
marketplace.

To determine the value of a preferred stock, use the following equation: {P0 = [d1 / k]}
Where: P0 = the price of the preferred stock at time 0, d1 = the annual dividend at t = 1, k = the
required rate of return.

In this example, the dividend is provided as a semiannual figure, which must be doubled to show the
annual dividend. After this adjustment has been made, the value of the preferred stock can be found
as follows.

{P0 = [$7.50 / 0.12] = $62.50.

Preferred stock is commonly valued as a perpetuity because there is no finite conclusion to the
projected series of cash flows for a preferred stock. Unlike a bond, whose cash flows are
characterized by a finite lifespan (i.e. the cash flows of a bond cease at maturity), the cash flows
(dividends) produced by a preferred stock could theoretically last forever.

Question: 3154

Which of the following are assumptions of the dividend discount model?

That answer is correct!


A. The required rate of return is greater than the growth rate
B. Earnings will not be negative
C. Dividends will be paid on stocks
D. All of these answers

Answer: A

Explanation:
The infinite period Dividend Discount Model has the following assumptions:

1. Dividends grow at a constant rate

Page | 1222
2. The constant growth rate will continue for an infinite period
3. The required rate of return (k) is greater than the infinite growth rate (g). If it is not, the model
gives meaningless results because the denominator becomes negative.

Question: 3155

In order to value an asset, one needs

A. the stream of expected future cash flows.


B. the stream of expected future cash flows and the ROE of the asset.
C. the stream of expected future cash flows, and the book value of the asset.
D. the book value of the asset, and its ROE.
E. the stream of expected returns and the required rate of return on the asset.

Answer: E

Explanation:
In order to value an asset, one needs to find the present value of its expected future cash flows. In
order to do this, one needs the stream of expected future cash flows, and the required rate of return
to discount that stream in order to arrive at its present value.

Question: 3156

According to Porter, which is not a competitive force that determines the intensity of competition?

A. none of these are competitive forces


B. bargaining power of suppliers
C. bargaining power of buyers
D. substitute products
E. rivalry among competitors
F. all of these are competitive forces

Answer: F

Explanation:
The remaining force is the threat of new entrants.

Question: 3157

Net profit margin for a market series is difficult to estimate because it is very ________.

A. stable
B. costly
C. volatile
D. inaccurate

Answer: C

Explanation:
Net profit margin for a market series is difficult to estimate because it is highly volatile, a result of
changes in depreciation, interest and tax rate over time.

Question: 3158

Page | 1223
Given that the retention rate of earnings of a firm is 0.30, the required rate of return on its stock is
17%, the expected growth rate is 12%, and expected 12 month earnings per share are $2.32, what
should the current market price of the stock be?

A. $43.00
B. Not enough information
C. $32.48
D. $39.18
E. $29.66
F. $14.12

Answer: C

Explanation:
According to the earnings multiplier model, the price/earnings ratio is equal to the dividend payout
ratio divided by the spread between the required rate of return and the expected growth rate. The
dividend payout ratio is equal to 1 - the retention rate of earnings (everything the firm does not
retain is paid out in dividends). In this question, it is 1 - 0.30 = 0.70. The P/E ratio is thus equal to 0.70
/ (0.17 - 0.12) = 14. Multiplying the P/E ratio by earnings per share yields the current market price. In
this question, it is 14 x
2.32 = $32.48.

Question: 3159

By what factor will earnings per share will have to change for a 5% change in earnings multiplier to
induce a change of 9% in the price of the stock?

A. -12.62%
B. +14.45%
C. -3.67%
D. +3.81%

Answer: D

Explanation:
Stock price = earnings multiplier * earnings per share. Therefore, the earnings multiplier will have to
increase by 1.09/1.05 - 1 = 3.81%.

Question: 3160

A finance major works evenings as an intern at Churn Brothers Brokerage. As a test of her abilities,
this intern has been asked to calculate the earnings multiplier of a software index. The firms that
comprise this index are all high-technology names for whose products there exists great demand.
Further, this demand is expected to be explosive in the future and the earnings visibility of these
software firms is clear, reliable, and concise.

In her research of the software index, the intern has gathered the following information:

EPS: 0.39
k: 62% per year
g: 60% per year
D1: 0.02

Using this information, what is the earnings multiplier of this software index? Further, is this earnings

Page | 1224
multiplier realistic given the demand for the firms' products and the visibility of future earnings?

A. 19.5, multiple is unrealistic


B. The answer cannot be calculated from the information provided.
C. None of these answers is correct.
D. 1, multiple is unrealistic
E. 2.56, multiple is unrealistic
F. 2.56, multiple is realistic

Answer: E

Explanation:
To determine the earnings multiplier, or "P/E ratio," of a stock market series, use the following
equation: P/E = [(D1 / E1) / (k-g)

Where: D1 = the annual per-share dividend at t1, E1 = the EPS figure at t1, k = the required rate of
return on common stock, and g = the expected growth rate of dividends.

In this example, all of the necessary information has been provided, and putting it into the equation
above will yield the following:

P/E of a stock market series = [($0.02*1.6 / ($0.39*1.6)) / (0.62 - 0.60)] = 2.56

This is an extremely low multiple, appropriate for virtually no-growth industries. The fact that the
index under examination is a compilation of firms in high-tech software business, for whose products
there exists great demand and promise of future growth, this low multiple seems very unrealistic.
Consider the fact that the P/E ratio is a proxy for future growth. Firms in the automobile, basic
materials, or other mature industries, which are expected to grow slowly, are characterized by lower
earnings multiples and higher
dividend payout ratios. Firms in the software, networking, biotechnology, and other high-growth
industries, are typically characterized by high earnings multiples and low dividend payout ratios.
What is happening here is that investors are giving up current income (i.e. dividends) in the hopes of
rapid earnings growth (i.e. greatly increased EPS in the future).

Question: 3161

If the OTC-to-NYSE volume is low after a substantial market decline, the contrarians interpret it as

A. a bearish signal.
B. none of these answers.
C. a hold signal.
D. a bullish signal.

Answer: D

Explanation:
Since the OTC market is dominated by small stocks which are riskier than the large stocks on NYSE, a
low OTC volume relative to NYSE (less than 87-90%) implies that investors are bearish. Since the
contrarians bet against the market at market turning points, they will consider this behavior by
investors as a bullish signal if the market is at a low.

Question: 3162

According to the infinite period Dividend Discount Model, an increase in the spread between the

Page | 1225
required rate of return on a common stock and the expected growth rate in its dividends will
probably cause

That answer is correct!


A. the stock price to decrease.
B. price effects that cannot be predicted with the given information.
C. earnings to fall.
D. the stock price to increase.

Answer: A

Explanation:
According to the infinite period Dividend Discount Model, the value of a common stock is equal to a
fraction with next period's dividend in the numerator, and the spread between the required rate of
return and the expected growth rate in dividends in the denominator. An increase in the spread will
increase the value of the denominator, thereby decreasing the value of the stock.

Question: 3163

Assume the following information about a large pharmaceutical company.

Net income / sales = 0.22


Total assets / common equity = 1.43
Sales / total assets = 0.65
Dividend payout ratio = 0.20

What is the expected annual growth rate of this firm's dividends?

That answer is correct!


A. 16.36%
B. None of these answers is correct.
C. The answer cannot be determined from the information provided.
D. 4.09%
E. 8.00%
F. 38.72%

Answer: A

Explanation:
A popular model for determining the growth rate of dividends is the following: g = RR * ROE

Where: g = the expected growth rate of dividends, RR = the retention rate (this is equal to 1 -
dividend payout ratio), and ROE = the return on equity.

Although it may at first appear otherwise, all of the necessary information has been provided.
Remember the Du Pont decomposition process for ROE, which breaks down the ROE figure into the
following:

ROE = (Net Income / Sales) * (Sales / Total Assets) * (Total Assets * Common Equity)

Mathematically, this will break down into (Net Income / Common Equity) --- the ROE figure. The
calculation of the return on equity for this company is as follows:

ROE = [0.22 * 0.65 * 1.43] = 0.204490, or 20.45%.

Page | 1226
Now that the ROE figure has been determined, the calculation of the growth rate of dividends is as
follows:

g = [(1 - 0.20) * 0.2045] = 16.36%

Question: 3164

The ________ approach derives specific estimates for the earnings multiplier based on a range of
estimates for the dividend payout, required rate of return and dividend growth rate.

A. Gauss-Markowitz
B. market driven
C. specific estimate
D. direction of change

Answer: C

Explanation:
There are two ways to estimate the earnings multiplier: (1) Direction of change approach and (2)
Specific estimate approach. The first focuses on change and change direction, while the other focuses
on scenario-based estimates.

Question: 3165

This shows the number of stocks advancing plus one-half the number unchanged, divided by the
total number of issues traded.

A. Odd-Lot, Short-Sales Theory


B. Relative Trend
C. Mutual Fund Cash Positions
D. Diffusion Index
E. Margin Debt
F. Dow Theory
G. Block Uptick-Downtick Ratio
H. Short Sales by Specialists

Answer: D

Explanation:
The diffusion index shows the daily total of stocks advancing plus one-half the number unchanged,
divided by the total number of issues traded.

Question: 3166

Using the dividend discount model, the spread between the required rate of return on a stock and
the expected growth rate of dividends on that stock is equal to

That answer is correct!


A. the expected dividend payment divided by its price.
B. the dividend payout ratio divided by the price.
C. its price divided by the dividend payout ratio.
D. its price divided by the expected dividend payment.

Page | 1227
Answer: A

Explanation:
Using the dividend discount model, the price of a stock is equal to D / (k - g), where D is the expected
dividend, k is the required rate of return, and g is the expected growth rate of dividends for the stock.
Rearranging this equation yields D/P = k - g.

Question: 3167

Contrary-opinion rules hold that

A. imitation of the trading patterns of astute investors will lead to superior returns.
B. technical analysis is false.
C. the majority of investors are wrong as the market approaches peaks and troughs.
D. it is always best to trade against the general market sentiment.

Answer: C

Explanation:
Contrary-opinion rules are technical trading rules that work on the assumption that the majority of
investors are wrong as the market approaches peaks and troughs. Technical analysts try to determine
when the majority of investors are either very bullish or bearish, and then trade in the opposite
direction.

Question: 3168

Jones Rutherford, a portfolio manager with Churn Brothers Brokerage, has been examining a stock
market series and is trying to determine the anticipated rate of return for the series. In his analysis,
Jones has amassed the following information:

Anticipated ending series value: 1475


Expected dividends during the period: $35
Observed beginning series value: 1310
Required rate of return: 19% per year

What is the anticipated annual rate of return for this stock market series? (Assume a one-year
holding period).

That answer is correct!


A. 15.27%
B. 13.56%
C. 8.81%
D. 9.92%
E. None of these answers is correct.

Answer: A

Explanation:
To calculate the expected rate of return for a stock market series, the following information must be
known:

The beginning value for the series

The anticipated ending value for the series, and

Page | 1228
The amount of any dividends and/or distributions during the period

Once this information has been determined, the expected return on a stock market index can be
found by employing the following equation:

E(R) = [(EV - BV + Div) / BV]

Where: E(R) = the expected return on the stock market series, EV = the anticipated ending value for
the series, BV = the observed beginning value for the series, and Div = the amount of any dividends
paid during the period.

In this example, all of the necessary information has been provided and the calculation of the
expected return on this stock market series is found as follows:

E(R) = [$1475 - $1310 + $35] / $1310 = 15.27%

This is less than the required rate of return. Assuming that the figures for the ending value and the
expected dividends are accurate, then investment in this stock market series is not likely warranted.

Question: 3169

Assume the following information about a stock market series:

Retention rate at t1 = 70%


Expected growth rate of dividends at t1 = 8%
Expected growth rate of earnings at t1 = 20%
Required rate of return = 15%

Given this information, what is the appropriate earnings multiplier for this stock market series?
Further, what is the value of this series?

A. 4.29; $21.43
B. 6.00; $30.00
C. 6.00; $120.00
D. The answer cannot completely be determined from the information provided.
E. None of these answers is correct.
F. 4.29; $53.63

Answer: D

Explanation:
The answer cannot completely be determined from the information provided. Specifically, while the
appropriate earnings multiplier can be found from the given information (and it is found as 4.29), the
correct value for the series cannot be determined without an estimation of EPS at t1. The following
explains the derivation of the earnings multiplier for a stock market series.

Estimating the earnings multiplier for a stock market series requires the estimation of each of the
following components:

1. The dividend payout ratio.

2. The required rate of return on common stock in the country/region/industry/sector being


analyzed.

Page | 1229
3. The expected growth rate of dividends for the stocks in the country/region/industry/sector being
analyzed.

Once values for each of these components have been determined, they are imputed into the
following formula:

{P/E = [D/E / (k - g)]}. Where: P/E = the earnings multiplier, or Price-to-Earnings ratio, D/E = the
dividend payout ratio at t1 [remember this is equal to (1 - the retention rate)], k = the required rate
of return, and g = the anticipated growth rate of dividends.

Had this example provided a figure for the EPS at t1, then the value of the index could be found by
multiplying the EPS at t1 by the earnings multiplier. The earnings multiplier of 4.29 is calculated as
(1-Payout ratio)/(required rate of return-growth rate) = (1-.7)/(.15-.08) = 4.29.

Question: 3170

A ________ level is the price range at which the technician would expect a substantial increase in
demand for the stock. A ________ level is the price range at which the technician would expect an
increase in the supply of the stock to cause any price increase to reverse abruptly.

A. base; ceiling
B. support; resistance
C. sell; buy
D. bottom; top

Answer: B

Explanation:
Increase demand will resist price falls, so when a price falls to a level which triggers strong demand -
the price will not fall below this support level. Resistance level is a price range where trade activity in
that stock will tend to reverse the price trend - this is usually due to investors having set a target
price range for trade execution.

Question: 3171

Economic Value Added (EVA) equals

That answer is correct!


A. Operating profits after adjusted taxes less the cost of total capital.
B. Operating profits minus the cost of borrowed capital.
C. Net income minus the cost of equity capital.
D. Gross profits minus the cost of capital, including equity capital.

Answer: A

Explanation:
The EVA measures the net value added by the firm's operations to the firm once operating costs are
taken into account. It should be remembered that the EVA is designed to evaluate the value added
by the operating activities over and above that required by the equity holders.

Question: 3172

Technical analysts may view a buildup in credit balances as

Page | 1230
That answer is correct!
A. an increase in market buying power.
B. a decrease in market buying power.
C. an exit of investors out of the market.
D. a rush of investors into the market.

Answer: A

Explanation:
A credit balance results when an investor sells stocks and deposits the proceeds with his broker.
Technical analysts view a buildup of credit balances as an increase in a pool of potential buying
power that will soon be reinvested in the market. For this reason, such a buildup is viewed as a
bullish sign.

Question: 3173

A company pays a dividend of $6 per share to the holders of its perpetual preferred stock. The firm's
bonds are currently yielding 8% per year and the firm's preferred stock are selling to yield 100 basis
points below the firm's bond yield. What is the value of the preferred stock?

A. $8.64
B. $75.45
C. $85.71
D. Not able to compute with the above data.

Answer: C

Explanation:
Value = dividend/discount rate = 6/0.07= $85.71. The discount rate on the preferred stock is below
that of the bonds.

Question: 3174

To determine whether to make an investment you must estimate the value of the investment based
on your ________ and compare that to the market price.

A. opportunity costs
B. required rate of return
C. all of these answers
D. risk-return preferences

Answer: B

Explanation:
Typically one invests in order to get a series of returns in the future, to discount these future returns
to net present value (NPV), one must use a discount rate, which is the rate one requires the
investment to yield.
Using this discount rate, one can then compare the NPV with the market price to determine if the
investment is attractive.

Question: 3175

The real risk free rate is 5% per year and the expected inflation rate is 3% per year. What is the

Page | 1231
annual nominal rate of interest?

That answer is correct!


A. 8.2%
B. Not able to compute with the above data.
C. 8%
D. 2%

Answer: A

Explanation:
Nominal rate = (1.05)*(1.03)=8.2%.

Question: 3176

Which would not be considered an important characteristic when studying industry analysis?

A. sales growth comparisons


B. evaluation of stage in industrial life cycle
C. effective tax rate comparisons
D. common stock yields
E. regulatory changes

Answer: C

Explanation:
Tax issues would not be of overall importance in industry analysis, yet are to be considered when
forecasting industry profit margins.

Question: 3177

If a stock has an expected dividend payout ratio of 50 percent, a required rate of return of 13 percent
and an expected growth rate for dividends of 9 percent, what is the P/E ratio?

A. 8.5
B. 12.5
C. 10
D. None of these answers

Answer: B

Explanation:
The P/E ratio is calculated as: P/E = (D/E)/(k - g). In this case P/E = 0.5/(0.13 - 0.09) = 12.5

Question: 3178

Which of the following correctly lists the two techniques for estimating the earnings multiplier for an
industry? Choose the best answer.

A. Residual earnings method and the arbitrage pricing theory.


B. The top-down approach and the bottom-up approach.
C. The industry life cycle method and the free-cash flow method.
D. Macroanalysis and Microanalysis.
E. The time series method and regression analysis.

Page | 1232
F. None of these answers is completely correct.

Answer: D

Explanation:
There is two common methods for estimating the earnings multiplier for an industry - macroanalysis
and microanalysis. Macroanalysis involves an examination of the relationship between the earnings
multiplier for an industry and the earnings multiplier for the market. Microanalysis involves an
estimation of the specific variables that influence the earnings multiplier, including the required rate
of return, the estimated growth rate, and the dividend payout ratio. While the top down and bottom
up method are similar to Macroanalysis and Microanalysis, respectively, they do not represent the
best possible answer. Specifically, the top-down and bottom up approaches are typically used to
identify investment opportunities, not for the estimation of an industry earnings multiplier.

Question: 3179

Given that this period's dividend is $3, the required rate of return is 16%, and the dividend growth
rate is 4%, what is the current value of the common stock (using the infinite period Dividend
Discount Model)?

A. Not enough information


B. $18.75
C. $25.00
D. $26.00
E. $19.50

Answer: D

Explanation:
The infinite period Dividend Discount Model postulates that the current value of a common stock is
equal to D1 / (k - g), where D1 is next period's dividend, k is the required rate of return, and g is the
growth rate of dividends. Next period's dividend is equal to this period's dividend multiplied by the
dividend growth rate. In this question, next period's dividend is 3 x 1.04 = 3.12. The common stock is
worth 3.12 / (0.16 - 0.04) = $26.

Question: 3180

A technical analyst with Smith, Kleen & Beetchnutty is examining the automobile parts industry to
determine whether its current earnings multiple is justified. In his analysis, this technical analyst
examines the underlying components of the earnings multiplier, the dividend payout ratio, the
required rate of return, and the expected growth rate, and the relationship between the values of
these components for the automobile parts industry and the overall market. Which of the following
best characterizes this method of estimating an earnings multiplier for an industry? Choose the best
answer.

A. Porter method
B. Microanalysis
C. Macroanalysis
D. Sensitivity analysis
E. Simulation analysis
F. None of these answer is correct.

Answer: B

Page | 1233
Explanation:
The method profiled in this example is "microanalysis," which is one of two methods for estimating
the earnings multiplier of an industry. Microanalysis involves examining the variables underlying
theearnings multiplier - the required rate of return, the growth forecast, and the dividend payout
ratio. In microanalysis, these variables are examined for the industry and then compare them with
the values of these variables for the entire market.

The microanalysis method is contrasted by the macroanalysis method, which involves examining the
historical relationship between the earnings multiplier of an industry with that of the overall market.
Macroanalysis forecasts often use a time series.

When examining and forecasting an industry earnings multiplier, it is recommended that both
macroanalysis and microanalysis be used. This should produce a more reliable answer. "Simulation
analysis," and "scenario analysis," are methods of measuring stand-alone risk, and the "Porter
Method" is used to measure industry competition.

Question: 3181

A stock has an expected dividend growth rate of 3.5%. The firm has just paid a dividend of $1.5 per
share. With a required rate of return of 8%, the stock is trading at $36.40. The stock is ________.

That answer is correct!


A. overpriced
B. underpriced
C. fairly priced
D. insufficient information

Answer: A

Explanation:
In the usual notation, the Dividend Discount Model gives Po = D1/(k-g). In this case, g = 3.5%, D1 =
Do*(1+g) = 1.5 * 1.035 = $1.5525. The price that will give k = 8% equals P = 1.5525/(8% - 3.5%) =
$34.5. Since the stock is trading at a price higher than this, it is overpriced (by $1.90).

Question: 3182

Technical analysts using contrary-opinion rules would consider

That answer is correct!


A. a low cash position by mutual funds to be indicative of an upcoming market peak. When mutual
funds invest a relatively low proportion of their portfolios into cash, this is usually indicative of
bullish sentiment among the mutual fund managers. Technical analysts believe that the managers
are wrong, and that there will be a market peak.
B. a low cash position by mutual funds to be indicative of an upcoming market peak. When mutual
funds invest a relatively low proportion of their portfolios into cash, this is usually indicative of a
bearish sentiment among the mutual fund managers.
C. a high cash position by mutual funds to be indicative of an upcoming market peak. When mutual
funds invest a relatively high portion of their portfolios into cash, this is usually indicative of a bearish
sentiment among the mutual fund managers.
D. a high cash position by mutual funds to be indicative of a continuing bear market. When mutual
funds invest a relatively high portion of their portfolios into cash, this is usually indicative of a bullish
sentiment among the mutual fund managers. Technical analysts believe that the managers are
wrong, and that there will be a continuing bear market.

Page | 1234
Answer: A

Explanation:
Contrary-opinion rules hold that the majority of investors (including mutual fund managers) are
wrong as the market approached peaks or troughs. When mutual funds have a low cash position (7%,
for example), technical analysts would take this as a sign that mutual fund managers are too bullish
about the market, and that it will instead soon reach a peak, and fall after that.

Question: 3183

________ interest is the cumulative number of shares that have been sold short by investors and not
covered.

A. Minority
B. Selling
C. Short
D. None of these answers

Answer: C

Explanation:
The short interest is the cumulative number of shares that have been sold short by investors and not
covered. Technicians consider a high short-interest ratio bullish because it indicates potential
demand when those who previously sold short must cover their sales.

Question: 3184

Capacity utilization rate is

A. logarithmically related to profit margin.


B. not related to profit margin.
C. positively related to profit margin.
D. inversely related to profit margin.

Answer: C

Explanation:
Finkel and Tuttle found that the capacity utilization rate, unit labor costs, rate of inflation, and foreign
competition were the four major variables that affected the aggregate profit margin. If production
increases as a proportion of the total capacity (raising the capacity utilization rate), per-unit fixed
production costs and fixed financial costs decrease, thereby raising the profit margin.

Question: 3185

A portfolio manager for Smith, Kleen & Beetchnutty is examining the performance of several of her
institutional customers. The following transactions apply to the account of Dotcominous, a fledgling
"new media" advertising conglomerate:

t0: Purchase five Kansas City District 14 General Obligation Bonds for $5,545.00 t1: Purchase three
Kansas City District 14 General Obligation Bonds for $3,229.50
Receive coupon payment of $300.00
t2: Purchase three Kansas City District 14 General Obligation Bonds for $3,185.00
Receive coupon payment of $480.00
t3: Receive coupon payment of $660.00

Page | 1235
t4: Sell eleven Kansas City District 14 General Obligation Bonds for $11,780.00

Investments with similar characteristics merit an 11.75% required rate of return. Assuming that taxes
and commissions have already been incorporated into the cash flows provided, what is the dollar-
weighted rate of return for this series of transactions?

A. 1.68%
B. 3.44%
C. None of these answers is correct.
D. 5.44%
E. The answer cannot be calculated from the information provided.
F. (3.34%)

Answer: B

Explanation:
Remember that the dollar-weighted rate of return uses the IRR equation in the determination of its
answer. Further, the dollar-weighted rate of return is another name for the IRR equation, and this
nomenclature is commonly used within the field of investment management. So said, the required
rate of return is not explicitly factored into the calculation of the dollar-weighted rate of return,
rather what is being determined
is the rate which equates the present value of the cash inflows to the present value of the cash
outflows. In the determination of the dollar-weighted rate of return calculation, the first step should
be to identify the cash flows for each period. In this example, the cash flows have been stated, and
little calculation is necessary. The cash flows associated with this series of investments are illustrated
as follows:

t0: ($5,545.00)
t1: [($3,229.50 paid for the bonds) + $300.00 coupon payment] = ($2,929.50)
t2: [($3,185.00 paid for the bonds) + $480.00 coupon payment] = ($2,705.00)
t3: $660 coupon payment
t4: $11,780

Now that the cash flows have been determined, incorporating this information into your calculator's
cash flow worksheet and solving for IRR will yield a dollar-weighted rate of return of 3.44% for this
investment

Question: 3186

Which of the following defines the investment decision?

A. Compute the expected value of the stock discounted to the present and compare it to the
prevailing market price. If the present value of the expected price exceeds the market price, the
particular stock is an attractive investment.
B. All of these answers
C. Compute an expected long-run rate of return based on the expected dividend yield plus the
expected growth rate. If this expected return exceeds the required rate of return, the particular stock
is an attractive investment.
D. Compute the expected rate of return during the holding period on the basis of the expected value
of the stock and the expected dividend. If this expected rate of return exceeds the required rate of
return, the particular stock is an attractive investment.
E. None of these answers

Answer: B

Page | 1236
Explanation:
The investment decision, i.e., whether or not a particular stock is an attractive investment, is based
on all three of these comparisons.

Question: 3187

Which of the following is a reason for why valuation of bonds is difficult?

That answer is correct!


A. Uncertainty of the required rate of return
B. Uncertainty of time pattern of returns
C. Uncertainty of size of returns
D. Uncertainty of the market interest rate

Answer: A

Explanation:
The size and time pattern of returns is known with bonds. Payments follow a regular schedule. But
the required rate of return, which is needed to value bonds, is generally not known.

Question: 3188

Marlene Gooseberry, an institutional money manager with Middle Road Brokerage, has been
examining a stock market series and has determined the following information:

Anticipated ending series value: 2060


Expected dividends during the period: $41.20
Observed beginning series value: 1579.81
Required rate of return: 21% per year

What is the anticipated annual rate of return for this stock market series? (Assume a one-year
holding period.)

A. 27.79%
B. 33.00%
C. None of these answers is correct.
D. 25.31%
E. 21.31%

Answer: B

Explanation:
To calculate the expected rate of return for a stock market series, the following information must be
known:

The beginning value for the series

The anticipated ending value for the series, and

The amount of any dividends and/or shareholder distributions during the period

Once this information has been determined, the expected return on a stock market index can be
found by employing the following equation:

Page | 1237
E(R) = [(EV - BV + Div) / BV]

Where: E(R) = the expected return on the stock market series, EV = the anticipated ending value for
the series, BV = the observed beginning value for the series, and Div = the amount of any dividends
paid during the period.

In this example, all of the necessary information has been provided and the calculation of the
expected return on this stock market series is found as follows:

E(R) = [$2060 - $1579.81 + $41.20] / $1579.81 = 33.00%

This is significantly greater than the required rate of return (21%). If the assumptions behind the
expected ending value and dividends are both realistic and accurate, investment in this stock market
series is likely advisable.

Question: 3189

Consider the following transactional information for the investment account of an individual
investor:

1st Quarter
Ending portfolio value: $100,000
Total amount invested: $95,000

2nd Quarter
Ending portfolio value: $110,000
Total amount invested: $90,000

3rd Quarter
Ending portfolio value: $114,000
Total amount invested: $100,000

4th Quarter
Ending portfolio value: $105,000
Total amount invested: $114,000

Using this information, what is the annual time-weighted rate of return for this portfolio? Assume no
taxes or transaction charges.

A. 30.05% per year


B. 26.67% per year
C. The time-weighted rate of return cannot be calculated from the information provided.
D. 25.22% per year
E. None of these answers is correct.

Answer: E

Explanation:
The time-weighted rate of return is the preferred method of return calculation in the investment
management industry, primarily because this method is not sensitive to significant additions and
withdrawals of funds from portfolios under examination. The calculation of the time-weighted rate
of return involves three steps, which are illustrated as follows:

Page | 1238
Step 1:
Price the portfolio immediately prior to any significant additions or withdrawals. Separate the
portfolio into a series of subperiods based on the dates of cash inflows and outflows.

Step 2:
Calculate the holding period return for each subperiod.

Step 3:
Determine the annualized holding period return by linking or compounding the holding period return
of each subperiod. If the investment is for more than one year, use the geometric mean of the annual
returns as the time-weighted rate of return. If the investment is for less than one year, compound the
subperiod returns to obtain an annualized measurement.

To begin the process of determining the time-weighted rate of return, we would break the portfolio
up into the following series of cash flows. However, in this example, the cash flows are already
aggregated for us and we can move on to the next step: determining the holding period return for
each subperiod. This process is detailed as follows:

Quarter 1 holding period return = [($100,000 ending value - $95,000 invested) / $95,000 invested] =
5.263%

Quarter 2 holding period return = [($110,000 ending value - $90,000 invested) / $90,000 invested] =
22.22%

Quarter 3 holding period return = [($114,000 ending value - $100,000 invested) / $100,000 invested]
= 14.00%

Quarter 4 holding period return = [($105,000 ending value - $114,000 invested) / $114,000 invested]
= (7.895%)

Now that the holding period return for each subperiod has been determined, we must annualize the
return measure by taking the product of all four quarterly returns. This process is illustrated below:

[(1 + .0526) * (1 + .2222) * (1 + .14) * (1 - .07895) - 1] = .3508 or 35.08 %. This is characterized by


none of the answers provided.

When calculating the time-weighted rate of return, remember that the total amount invested is the
relevant figure, not the beginning portfolio value. Notice that during the third quarter, the total
amount invested does not equal the ending amount for the second quarter. This differential could be
explained by numerous phenomena. Perhaps the difference is due to a cash withdrawal from the
account. Maybe it was used to pay expenses or meet an outstanding margin call. What is important
to note is the fact that this money (the difference between the ending value of quarter 2 and the
amount invested in quarter 3) was not invested, and should not be included in the holding period
return for the third quarter. So said, whenever possible you should use the total amount invested
rather than the beginning portfolio value in the calculation of the subperiod holding period return.

Question: 3190

Assume the following information about a stock market series:

Retention rate = 70%


Expected growth rate of dividends = 8% per year
Expected growth rate of earnings = 20% per year
Required rate of return = 15% per year

Page | 1239
What is the appropriate earnings multiplier for this stock market series? Further, what is the value of
this series?

A. None of these answers is correct.


B. 4.29; $53.63
C. The answer cannot completely be determined from the information provided.
D. 6.00; $30.00
E. 6.00; $120.00
F. 4.29; $21.43

Answer: C

Explanation:
The answer cannot completely be determined from the information provided. Specifically, while the
appropriate earnings multiplier can be found from the given information (and it is 4.29), the correct
value for the series cannot be determined without an estimation of EPS at t1. The following explains
the derivation of the earnings multiplier for a stock market series.

Estimating the earnings multiplier for a stock market series requires the estimation of each of the
following components:

The dividend payout ratio.

The required rate of return on common stock in the country/region/industry/sector being analyzed.

The expected growth rate of dividends for the stocks in the country/region/industry/sector being
analyzed.

Once values for each of these components have been determined, they are imputed into the
following formula:

P/E = [D/E / (k - g)]

Where: P/E = the earnings multiplier, or Price-to-Earnings ratio, D/E = the dividend payout ratio at t1,
k = the required rate of return, and g = the anticipated growth rate of dividends.

Had this example provided a figure for the EPS at t1, then the value of the index could be found by
multiplying the EPS at t1 by the earnings multiplier.

Question: 3191

Stock ABC has the following characteristics:

Current dividend $1.00


Expected dividend in 1 year $1.10
Long term growth rate of dividends 10%
Required rate of return 12%

Using the infinite period Dividend Discount Model, what is the maximum price that you would pay
for stock ABC?

A. $10.00
B. $120.00

Page | 1240
C. $55.00
D. $550.00
E. $15.00
F. $12.00

Answer: C

Explanation:
The infinite period Dividend Discount Model indicates that:

Value = (Dividend for period 1)/(k-g) where k is the required rate of return and g is the growth rate. In
this case, Value = ($1.10)/(.12-.10) = $1.10/.02 = $55.

Question: 3192

The ________ ratio has been used extensively in the valuation of bank stocks because bank assets
often have similar book values and market values.

That answer is correct!


A. price/book value
B. economic value added
C. market value-added
D. franchise factor
E. price/cash flow

Answer: A

Explanation:
The P/BV ratio has become important as a measure of relative value for stocks. The relationship
between the market price of a stock and its book value per share can be used as a relative measure
of valuation because, under theoretically ideal conditions, the market value of a firm should reflect
its book value. However, particularly with industrial firms, an increase in the estimates of breakup
value has caused the average P/BV ratio to experience a volatile increase over time.

Question: 3193

Given that the correct value of a common stock is $25, the required rate of return on the stock is
15%, and the dividend growth rate is 5%, using the infinite period Dividend Discount Model, what
will next period's dividend be?

A. $2.28
B. $2.79
C. Not enough information
D. $2.50
E. $3.16

Answer: D

Explanation:
The infinite period Dividend Discount Model postulates that the current value of a common stock is
equal to D1 / (k - g), where D1 is next period's dividend, k is the required rate of return, and g is the
growth rate of dividends. Rearranging this yields D1 = current value x (k - g). In this question, next
period's dividend is equal to 25 x (0.15 - 0.05) = $2.50.

Page | 1241
Question: 3194

Which of the following are challenges to technical analysis? Choose the best answer.

I.The majority of studies have concluded that securities prices do not move in trends.
II. Technical analysis assumes that supply-demand fluctuations lead to changes in securities prices.
III. Technical analysis is heavily reliant on subjective judgement.
IV. Technical analysis assumes that supply and demand are partially influenced by irrational factors,
and that supply and demand fluctuations are the only determinant of shifts in securities prices.
V. The majority of studies have supported the weak-form Efficient Market Hypothesis. VI. The
standard rules that signal investment decisions can change over time.
VII. Technical analysis is heavily reliant on financial statements.

A. I, II, III, IV, V, VI, VII


B. None of these choices represents the best possible answer.
C. I, III, IV, V, VII
D. II, III, IV, V, VI, VII
E. I, II, III, V, VI

Answer: B

Explanation:
None of these answers are completely correct. Of the choices listed, only II and VII are incorrect.
While II appears correct, it is not. In fact, both fundamental and technical analyses recognize the
importance of supply and demand fluctuations. The difference is that technical analysis assumes that
supply and demand are influenced by both rational and irrational factors, whereas the EMH assumes
that investors are rational. Further, technical analysis assumes that supply and demand fluctuations
are the sole cause of shifts in securities prices.

Question: 3195

Marlene Gooseberry, an institutional money manager with Middle Road Brokerage, has been
examining a stock market series and has determined the following information:

Anticipated ending value: 2060


Expected dividends during the period: $41.20
Observed beginning value: 1579.81
Required rate of return: 21%

Using this information, what is the anticipated rate of return for this stock market series? (Assume a
oneyear holding period).

That answer is correct!


A. 33.00%
B. 27.79%
C. 25.31%
D. 21.31%
E. None of these answers is correct.

Answer: A

Explanation:
To calculate the expected rate of return for a stock market series, the following information must be
known:

Page | 1242
The beginning value for the series, the anticipated ending value for the series, and the amount of any
dividends and/or shareholder distributions during the period.

Once this information has been determined, the expected return on a stock market index can be
found by employing the following equation:{E(R) = [(EV - BV + Div) / BV]}. Where: E(R) = the expected
return on the stock market series, EV = the anticipated ending value for the series, BV = the observed
beginning value for the series, and Div = the amount of any dividends paid during the period.

In this example, all of the necessary information has been provided and the calculation of the
expected return on this stock market series is found as follows: {E(R) = [$2060 - $1579.81 + $41.20] /
$1579.81} = 33.00%.

This is significantly greater than the required rate of return (21%). If the assumptions behind the
expected ending value and dividends are both realistic and accurate, investment in this stock market
series is likely advisable.

Question: 3196

Which of the following statements is true?

That answer is correct!


A. There are two general approaches to the security valuation process: the top-down, three-step
approach, and the bottom-up stock valuation approach. Advocates of the top-down, three-
stepapproach believe that the economy and the industry effect have a significant impact on total
returns for individual stocks.
B. There are two general approaches to the security valuation process: the bottom-up stock valuation
approach, and the horizontal, two-step approach. Advocates of the horizontal, two-step approach
believe that the economy and the industry effect have a significant impact on total returns for
individual stocks.
C. There are two general approaches to the security valuation process: the top-down, two-step
approach, and the horizontal, four-step approach. Advocates of the top-down, two-step approach
believe that the economy and the industry effect have a significant impact on total returns for
individual stocks.
D. There are three general approaches to the security valuation process: the top-down, four-step
approach, the bottom-up stock valuation approach, and the horizontal, three-step approach.
Advocates of the horizontal, three-step approach believe that the economy and the industry effect
have a significant impact on total returns for individual stocks.

Answer: A

Explanation:
While advocates of the top-down, three-step approach believe that the industry effect and the
economy have a significant impact on total returns for individual stocks, advocates of the bottom-up
stock valuation approach believe that is possible to find stocks that are undervalued relative to their
market prices, and that these stocks will provide superior returns regardless of the market or
industry outlook.

Question: 3197

The stock of Constagrow pays dividends that are expected to grow at a steady rate of 3.2% per year.
Investors expect a rate of return of 11.5% from Constagrow stock. If the spread between this required
rate of return and the dividend growth rate were to increase by 50 basis points, the percentage
change in the stock price would be:

Page | 1243
A. -4.73%
B. +3.94%
C. -5.68%
D. +4.22%

Answer: C

Explanation:
In the usual notation, the Dividend Discount Model gives Po = D1/(k-g). When k - g = 11.5% - 3.2% =
8.3%, the price is given by Po = D1/0.083. When the spread increases by 50 basis points and all else
stays constant, the price becomes P1 = D1/(0.083 + 0.005) = D1/0.088. The percentage change in the
price equals (P1 - Po)/Po = (1/0.088 - 1/0.083)*0.083 = 8.3/8.8 - 1 = -5.68%. Thus, the stock price falls
by 5.68% when the spread between k and g increases by 50 basis points, all else equal.

Question: 3198

Assume the following information about an international textile company:

Next annual dividend: $1.10


Earnings per share next year: $2.45
Anticipated growth rate: 9% per year
Required rate of return: 11% per year

What is the expected earnings multiplier for this utility company?

A. 50
B. 31
C. 26
D. 19
E. 16
F. None of these answers is correct.

Answer: F

Explanation:
The earnings multiplier for this company is found as 22.45, thus none of the answers is correct.

To determine the earnings multiplier (i.e. the price-to-earnings ratio) for an individual company, use
the following formula:

P/E = [(d1 / e1) / (k - g)]

Where: P/E = the earnings multiplier,d1 / e1 = the dividend payout ratio at t1, k = the required rate of
return, and g = the anticipated future growth rate.

In this example, all of the necessary information has been provided, and the calculation of the
earnings multiplier is as follows:

In this example, all of the necessary information has been provided, but some rearranging is
necessary. Specifically, the dividend payout ratio must be determined. This figure is found as follows:

Dividend payout ratio = [$1.10 / $2.45] = 0.44898, or 44.90%

Page | 1244
Now that the dividend payout ratio has been determined, we can solve for the appropriate earnings
multiplier. The calculation of this figure is found as follows:

P/E = [0.4490 / (0.11 - 0.09] = 22.45

This is likely a realistic multiple for a company within the international textile sector.

Question: 3199

A portfolio manager with Smith, Kleen, & Beetchnutty is trying to determine the earnings per share
(EPS) for a software index, and has gathered the following information:

Sales per share: $340


Next year's operating profit margin: 50%
Next year's depreciation per share: $50
Next year's interest expense: $68
Next year's common stock dividend: $14
Next year's corporate tax rate: 35%

Using this information, what is the EPS figure for this stock market series?

A. $72.15
B. The answer cannot be calculated from the information provided.
C. $24.70
D. $33.80
E. $83.80
F. None of these answers is correct.

Answer: D

Explanation:
The estimation of EPS for a stock market series involves five steps. Specifically, to determine an
estimate of EPS for a stock market series, it is necessary to:

Estimate the sales per share


Estimate next year's operating profit (EBIDT), or operating profit margin Estimate next year's
depreciation per share
Estimate next year's interest expense per share
Estimate next year's corporate tax rate

Once estimates for these components have been determined, they are put into the following
equation:

EPS for a stock market series = {[(Sales per share * operating profit margin) - depreciation per share -
interest expense per share] * (1 - corporate tax rate).

Imputing the given information into this equation will yield the following:

EPS for a stock market series = {[($340 * 0.50) - $50 - $68] * (1 - 0.35)} = $33.80

If you chose $83.80, remember that the depreciation figure is not added back to the EPS calculation.
What we are looking at is an operating earnings after tax figure, not a cash-based figure.

If you chose $24.70, remember that common stock dividends are not incorporated into the EPS

Page | 1245
figure, only interest payments on debt and preferred securities.

Question: 3200

An intern at Smith, Kleen & Beetchnutty has recently been asked to value shares of General
Manufacturing, a multinational conglomerate, using the Gordon Model. General Manufacturing is a
stable firm which has experienced steady growth for much of the last twenty years, and this growth
rate is not anticipated to change. In his assignment, the intern has been provided with the following
information:

Expected dividend at t3: $1.05


Required rate of return on equity: 11.75% per year
Growth rate of dividends: 9.00% per year

Using the information provided, what is the value of General Manufacturing shares?

A. None of these answers is correct.


B. The answer cannot be calculated from the information provided.
C. $32.14
D. $34.45
E. $37.63
F. $28.81

Answer: C

Explanation:
Since the growth rate of dividends is not expected to change in the future, the Gordon Model can be
applied. The Gordon Model is commonly referred to as the "constant growth dividend discount
model." This model of common stock valuation is illustrated as follows:

P0 = [D1 / (r - g)]

Where
P0 = the price of common stock X at time 0
D1 = the expected dividend at t1
r = the required rate of return on equity investments and g = the expected growth rate of dividends.

Since we are not provided the dividend at t1, we must determine it manually be discounting the
dividend at t3 by the growth rate of dividends. The determination of D1 is as follows:

D1 = {D3 / [(1 + g)(1 + g)]}


D1 = {$1.05 / [1 + 0.09)(1 + 0.09)]}
D1 = {$1.05 / 1.1881} = $0.8838

Now that the dividend at t1 has been determined, the given information can be put into the
equation provided, leading to the following series of calculations:

P0 = [$0.8838 / (.1175 - .09)] = $32.14

When using the Gordon model, remember that the required rate of return "r" must be greater than
the expected growth rate "g." Otherwise, the equation will produce a nonsensical answer.

Question: 3201

Page | 1246
A diffusion index with a value of over 70 is considered by technical analysts to be

A. not particularly bullish or bearish.


B. a sign of caution.
C. a sign of an approaching trough.
D. a bullish sign.

Answer: B

Explanation:
The diffusion index is an alternative specification of advance-decline series, and is used as an
indicator of the direction of the market. If the index reaches up to levels of 56 to 60, the market is
considered overbought. If it comes down to levels of 40 to 44, the market is considered over
oversold.

Question: 3202

If a stock has an expected dividend payout ratio of 30 percent, a required rate of return of 10 percent
and an expected growth rate for dividends of 5 percent, what is the P/E ratio?

A. 12
B. 6.5
C. None of these answers
D. 10

Answer: B

Explanation:
The price/earnings ratio can be computed by dividing the expected dividend payout ratio (dividends
divided by earnings) by the required rate of return (k) minus the expected growth rate of dividends
(g).

In this case, P/E = .3/(.1-.05) = 6.5

Question: 3203

An increase in the ratio of short sales by specialists to total short sales is interpreted by smart-money
technicians as:

A. a bullish signal.
B. none of these answers.
C. a hold signal.
D. a bearish signal.

Answer: D

Explanation:
As part of their market-making activities, specialists regular indulge in short-selling activity. However,
since they have some discretionary authority over short selling, smart-money technicians (who track
the behavior of "smart" investors) consider a low participation rate by specialists in the short-selling
activity as a bullish signal. This participation rate is characterized by the ratio of specialist short sales
to total short sales. When this ratio falls below 30%, smart money says it is a bullish signal and when
it rises above 50%, it is considered a bearish signal.

Page | 1247
Question: 3204

High payout ratios are closely correlated with which of the following attributes?

A. The level of regulation in an industry.


B. The growth rate of earnings.
C. The level of competition in the industry.
D. More than one of these answers is correct.
E. All of these answers are correct.
F. Maturity of the industry.

Answer: D

Explanation:
All but one of these answers is correct. Specifically, the dividend payout ratio is very closely related
to the maturity of the industry, the level of competition in the industry, and the growth rate in
earnings. Remember that all of these factors are interrelated. As an industry becomes more mature,
competition becomes maximized, margins shrink, and the growth potential of the industry declines.
This will lead to a decrease in positive NPV investment opportunities in the industry, forcing firms to
increase the proportion of earnings paid out to shareholders in the form of dividends.

The level of regulation in an industry is not necessarily highly correlated with the dividend payout
ratio of firms within the industry. For example, the financial services industry is very heavily
regulated, yet firms within this sector do not pay a disproportionate amount of their earnings as
dividends. However, he utility industry, also heavily regulated, pays a very high proportion of
earnings as dividends.

Question: 3205

All else equal, an increase in the expected inflation rate ________ the value of a stock.

A. does not affect


B. decreases
C. increases
D. can be all of these answers

Answer: B

Explanation:
When expected inflation rises, the expected rate of return demanded by the investors increases. All
else equal, this can happen only if the price of the stock drops.

Question: 3206

An investor is examining shares of Intelligent Semiconductor to determine if they are trading at an


appropriate multiple. Assume the following information:

k = 14.25% per year


g = 13.4% per year
D0 = $2.15
P0 = $110.80

Using the information provided, what is the appropriate P/E ratio for Intelligent Semiconductor?

Page | 1248
A. 25.89
B. The answer cannot be determined from the information provided.
C. 51.53
D. 2.59
E. 45.11
F. None of these answers is correct.

Answer: B

Explanation:
In this example, not all of the necessary information has been provided. Specifically, the EPS figure
must be known in order for the P/E ratio to be determined.

If the EPS figure is known, however, the infinite period dividend discount equation can be
manipulated to solve for the P/E multiple. Specifically, by dividing each side of the infinite period
dividend discount equation by the EPS figure, it is possible to determine the P/E ratio. This is
illustrated as follows:

P0/EPS = (D1 / EPS)/(k-g)

Where: P0 = the price of the common stock at t0, D1 = the dividend at t1, EPS = the earnings per
share calculation for t1, k = the required rate of return, and g = the expected growth rate.

Manipulating the infinite period dividend discount model to solve for the PE is a rather intuitive
process.

Consider the fact that an investment's value is truly nothing more than the present value of all future
returns. So said, dividing both sides of the infinite period dividend discount model equation by the
EPS figure should yield the appropriate multiple, or "earnings multiplier." This is the price-to-
earnings ratio.

Question: 3207

Consider the following series of cash flows for an institutional investment account:

1st Quarter
Ending portfolio value: $10,340,000
Total amount invested: $10,000,000

2nd Quarter
Ending portfolio value: $10,660,000
Total amount invested: $10,340,000
3rd Quarter
Ending portfolio value: $11,110,000
Total amount invested: $10,660,000

4th Quarter
Ending portfolio value: $11,400,000
Total amount invested: $11,000,000

Using this information, what is the annual time-weighted rate of return for this portfolio? Assume no
taxes or transaction charges.

A. None of these answers is correct.

Page | 1249
B. The time-weighted rate of return cannot be calculated from the information provided.
C. 9.89% per year
D. 18.13% per year
E. 12.67% per year
F. 15.14% per year

Answer: F

Explanation:
The time-weighted rate of return is the preferred method of return calculation in the investment
management industry, primarily because this method is not sensitive to significant additions and
withdrawals of funds from portfolios under examination. The calculation of the time-weighted rate
of return involves three steps, which are illustrated as follows:

Step 1:
Price the portfolio immediately prior to any significant additions or withdrawals. Separate the
portfolio into a series of subperiods based on the dates of cash inflows and outflows.

Step 2:
Calculate the holding period return for each subperiod.

Step 3:
Determine the annualized holding period return by linking or compounding the holding period return
of each subperiod. If the investment is for more than one year, use the geometric mean of the annual
returns as the time-weighted rate of return. If the investment is for less than one year, compound the
subperiod returns to obtain an annualized measurement.

To begin the process of determining the time-weighted rate of return, we would break the portfolio
up into the following series of cash flows. However, in this example, the cash flows are already
aggregated for us and we can move on to the next step: determining the holding period return for
each subperiod. This process is detailed as follows:

Quarter 1 holding period return = [($10,340,000 ending value - $10,000,000 invested) / $10,000,000
invested] = 3.40%
Quarter 2 holding period return = [($10,660,000 ending value - $10,340,000 invested) / $10,340,000
invested] = 3.095%
Quarter 3 holding period return = [($11,110,000 ending value - $10,660,000 invested) / $10,660,000
invested] = 4.221%
Quarter 4 holding period return = [($11,400,000 ending value - $11,000,000 invested) / $11,000,000
invested] = 3.636%

Now that the holding period return for each subperiod has been determined, we must annualize the
return measure by taking the product of all four quarterly returns. This process is illustrated below:

[(1 + .034) * (1 + .03095) * (1 + .04221) * (1 + .03636) - 1] = .015139 or 15.139%

When calculating the time-weighted rate of return, remember that the total amount invested is the
relevant figure, not the beginning portfolio value. Notice that during the fourth quarter, the total
amount invested does not equal the ending amount for quarter 3. This differential could be
explained by numerous phenomena. Perhaps the difference is due to a cash withdrawal from the
account. Maybe it was used to pay expenses or meet an outstanding margin call. What is important
to note is the fact that this money (the difference between the beginning portfolio value and the
amount invested) was not invested in securities, and should not be included in the holding period
return for the fourth quarter. So said, whenever possible you should use the total amount invested

Page | 1250
rather than the beginning portfolio value in the calculation of the subperiod holding period return.

Question: 3208

Which of the following is not an advantage of technical analysis?

A. Technical analysis is not as time-consuming as fundamental analysis.


B. All of these answers are advantages.
C. Technical analysis assumes that securities prices move in trends, and identifying changes in these
trends can lead to the discovery of superior investment opportunities more closely to their price
moves than fundamental analysis.
D. Technical analysis is not heavily dependent on financial statements.

Answer: B

Explanation:
All of these reasons are cited by technical analysts as advantages of their method. One of the major
advantages cited by technical analysts is the fact that technical analysis is not heavily dependent on
financial statements. Technical analysts, as a community, are suspicious of financial statements, and
believe that few fundamental analysts can identify relevant information in financial statements, and
analyze the impact of this information on securities prices in time for superior investment returns to
be realized.

Additionally, technical analysts believe that securities markets "price in" material information
gradually, and that securities prices move in identifiable trends and patterns. An astute technical
analyst, it is argued, can identify superior investment opportunities by recognizing the implicit trends
exhibited by securities prices and quantifiable past performance data. Assuming that fundamental
analysis can identify whether a security is over or under-valued long before the security experiences
a significant price realization, the fundamental analyst is left with the daunting decision of when to
buy of sell. Ideally, the fundamental analyst would like to invest his money immediately before a
security experiences a profitable increase (or, for short sellers, a profitable decrease) in price,
preferring to hold the funds to invest in other opportunities until the change in price direction is
initiated. Technical analysts argue that because they do not invest until the move to the new
"equilibrium price" is underway, they are more likely to experience ideal timing compared to the
fundamental analyst.

Question: 3209

An investor had purchased 66 shares of firm X on January 1, at a price of $236 per share. On May
16th, the stock underwent a 2-for-1 split. On December 31, the stock price stood at $156. If the
investor realized a return of 35% during the year, his total dividend income during the year was:

That answer is correct!


A. $435.60
B. $250.80
C. $501.60
D. $156.00

Answer: A

Explanation:
If the stock had not undergone the split, the year-end price would be 156 * 2 = $312, assuming no
informational effects conveyed by the split. The per share income on each stock has been given to be
236 * 0.35 = $82.60. Of this, $(312-236) = $76 is in capital gains. Thus, dividend income per share

Page | 1251
equals $(82.6-76) = $6.6. Since the investor held 66 of the original shares, his dividend income equals
$6.6 * 66 = $435.6.

Question: 3210

A stock paid a $10 per share dividend this year. Dividends are expected to grow at 5% per year,
forever. What is the value of the stock if the appropriate discount rate is 9% per year?

A. Not able to compute with the above data.


B. $10.23
C. $26.30
D. $262.50

Answer: D

Explanation:
Value = $10.5/(0.09-0.05)=$262.50.

Question: 3211

Given that the real risk-free rate is 5%, the rate of inflation is 7%, and the growth in money supply is
9%, what is the exact nominal risk-free rate?

A. 14.5%
B. 14%
C. 12%
D. Not enough information.
E. 12.4%

Answer: E

Explanation:
The nominal risk-free rate is equal to (1 + RFR) x (1 + I) - I, where RFR is the real risk-free rate and I is
the inflation rate. In this question, the nominal risk-free rate is (1.05) x (1.07) - 1 = 0.124 = 12.4%.

Question: 3212

The value of an asset is

That answer is correct!


A. the present value of its expected future cash flows.
B. the sum of its expected future cash flows.
C. its replacement cost.
D. its book value.

Answer: A

Explanation:
One expects an asset to provide a stream of future cash flows during its lifetime. Future returns are
worth less than current ones. One adjusts for this by taking the present value of all its expected cash
flows.

Question: 3213

Page | 1252
Which of the following is/are true?

I. Proponents of efficient markets believe that new information is impounded correctly and gradually
in stock prices.
II. Technical analysts believe that new information spreads through the market rapidly and gets
reflected in the prices, thus moving the markets.
III. Fundamental analysts believe that stock prices are determined by supply and demand in the
security markets.

A. I & III
B. II only
C. III only
D. I only

Answer: C

Explanation:
Technicians believe that new information seeps into prices only gradually whereas efficient market
theorists believe that this adjustment is extremely rapid; too rapid to allow consistent and systematic
exploitation of particular kinds of information like dividend changes.

As in any other market, prices in security markets are determined by supply and demand; this is not
in dispute between technicians and non-technicians. What is in dispute is the source and nature of
demand for securities; technicians believe that demand adjusts slowly to new information while non-
technicians consider the process to be almost instantaneous.

Question: 3214

Assuming that the inflation rate and risk-free rate of interest are relatively high, which of the
following correctly illustrates the calculation of the nominal risk-free rate?

That answer is correct!


A. Nominal RFR = (1+ RFR)(1 + inflation premium) - 1
B. Nominal RFR = (1 + RFR)(1 + inflation premium)
C. Nominal RFR = RFR + E(I)
D. None of these answers is correct.
E. Nominal RFR = Real RFR * (1 + k)
F. Nominal RFR = Real RFR * E(I)

Answer: A

Explanation:
When either the real "inflation-free" interest rate or the expected inflation rate are significantly
large, the calculation of the nominal risk-free rate differs from the equation used when these factors
are significantly small. Specifically, the calculation of the nominal risk-free rate of interest when the
inflation-free rate of interest and/or the inflation premium are significantly high is as follows:

Nominal RFR = (1 + Real RFR)(1 + E(I)) - 1

Where: Real RFR = the real inflation-free rate of interest and E(I) = the anticipated inflation rate.

When the inflation-free rate of interest and/or the inflation premium are low, then the equation
above can be approximated by the following:

Page | 1253
Nominal RFR = Real RFR + Inflation premium.

Question: 3215

Which of the following must be estimated to determine the value of a stock that is to be held for one
year?

That answer is correct!


A. all of these answers
B. required rate of return
C. dividends
D. expected sale price

Answer: A

Explanation:
In order to determine the value of a stock that is to be held for one year, dividends, the required rate
of return and the expected sale price must be estimated.

Question: 3216

Which of the following is/are true about open-ended funds?

I. Shares of the fund trade on an exchange.


II. Shares of the fund can be redeemed at NAV, with or without redemption fees.
III. It issues shares as and when demanded by investors bringing in additional funds.

A. II only
B. I & II
C. I & III
D. II & III
E. III only
F. I only
G. I, II & III

Answer: D

Explanation:
Open ended funds are mutual funds which buy and sell shares as and when demanded by investors.
These funds may or may not have sales and redemption charges.

Question: 3217

Which of the following firms would likely have the lowest dividend payout ratio? Further, the capital
structure of this firm would likely be weighted more heavily with debt or equity? Choose the best
answer.

That answer is correct!


A. A fledgling biotechnology company; equity
B. A nationwide grocery chain; debt
C. A healthcare company specializing in home care; debt
D. A specialty retailer; equity
E. A large computer manufacturer; debt

Page | 1254
Answer: A

Explanation:
Of the firms listed, the biotechnology company would be expected to have the lowest dividend
payout ratio. The nascent biotechnology industry is characterized by a high degree of investment in
research and development, along with a high growth rate and high level of uncertainty. All of these
factors are conducive to a high retention rate, i.e. a low dividend payout ratio.

Further, firms within industries associated with high growth rates and high levels of uncertainty are
likely to be financed primarily with equity. This is reasoned by several factors, some of which include
the following:

Firms in new industries typically have a balance sheet weighted heavily toward intangible assets,
which can neither be easily liquidated nor pledged as collateral for a loan.
Firms in new industries are characterized by a high degree of research and development expenses.
Equity provides firms with a higher degree of agility than debt financing.
The cash flows of firms in developing industries are characterized by a high degree of uncertainty.

Question: 3218

If the 50-day moving average line is above the 200-day moving average line but not by a large
amount, technical analysts would consider this ________.

A. a bearish indicator
B. an under-bought market
C. a bullish indicator
D. a trend reversal

Answer: C

Explanation:
Since a 50-day average reflects more recent events than the 200-day average, its rise above the 200-
day average is interpreted as a bullish signal. However, if the gap gets too large, then the technician
might consider the stock to be over-bought and infer that the stock price is about to fall.

Question: 3219

A company pays a dividend of $6 per share to the holders of its perpetual preferred stock. The
appropriate discount rate is 6.5% per year. What is the value of the preferred stock?

A. Not able to compute with the above data.


B. $0.92
C. $9.28
D. $92.31

Answer: D

Explanation:
Value = dividend/discount rate = 6/0.065= $92.31.

Question: 3220

If investors become more risk averse, the yield spread between corporate bonds and Treasury bonds
will:

Page | 1255
That answer is correct!
A. widen.
B. not necessarily be affected.
C. narrow.
D. not be affected, since the spread reflects a constant risk premium.

Answer: A

Explanation:
As investors become more risk averse, they will demand more risk-free assets. Hence, thedemand
for Treasury bonds will increase and that for the corporate bonds decrease. Thiswill increase the
yields on corporate bonds and decrease those on Treasury bonds, thus widening the spread.

Question: 3221

Which of the following statements about relative strength ratios is correct?

A. In an increasing market, a stock price has to increase faster than the general market for its relative
strength ratio to increase. Relative strength ratios, however, do not work during stagnant or declining
markets.
B. In an increasing market, a stock price has to increase faster than the general market for its relative
strength ratio to increase. In a declining market, a stock price only has to increase in order for that
ratio to increase.
C. A high relative strength index that has not changed much over time would be interpreted as a
bullish sign by technical analysts. A high ratio value that has started declining would be viewed with
caution, however.
D. In an increasing market, a stock price has to increase faster than the general market for its relative
strength ratio to increase. In a declining market, a stock price only has to decline slower than the
general market in order for that ratio to increase.

Answer: D

Explanation:
Technical analysts believe that a stock or industry that is outperforming the market will continue to
do so. Relative strength ratios are computed to find such trends. The ratios are equal to the price of
the stock, or stocks in the industry group, relative to the value of some stock market series. If the
ratio increases over time, the stock or industry has beenoutperforming the market.

Question: 3222

Consider the following preferred stock:

Price per share: $12.55


Semiannual dividend per share: $0.725
Required return: 11.50% per year

Is the preferred stock realistically overvalued, undervalued, or correctly valued? Further, should this
preferred stock be valued as a perpetuity or a finite series of cash flows? (Assume a long-term
holding period).

That answer is correct!


A. Correctly valued; perpetuity
B. Undervalued; finite series of cash flows

Page | 1256
C. Correctly valued; finite series of cash flows
D. The answer cannot completely be determined from the information provided.
E. Overvalued; perpetuity
F. Undervalued; finite series of cash flows

Answer: A

Explanation:
The preferred stock profiled in this example is trading very close to its theoretical value, which is
found as $12.61.

To determine the value of a preferred stock, use the following equation; {P0 = [d1 / k]}
Where:P0 = the price of the preferred stock at time 0, d1 = the annual dividend at t = 1, and k = the
required rate of return.

In this example, the dividend is provided as a semiannual figure, which must be doubled to show the
annual dividend. After this adjustment has been made, the value of the preferred stock can be found
as follows.

{P0 = [$1.45 / 0.115] = $12.61.

Preferred stock is commonly valued as a perpetuity because there is no finite conclusion to the
projected series of cash flows for a preferred stock. Unlike a bond, whose cash flows are
characterized by a finite lifespan (i.e. the cash flows of a bond cease at maturity), the cash flows
(dividends) produced by a preferred stock could theoretically last forever.

Question: 3223

XYZ (a firm the produces consumer goods) is a stable company reporting the following financial
information:

Earnings per share $1.50


Dividends per share $0.30
Net Income $5 million
Equity $50 million

Given the above information, calculate the company's expected dividend growth rate.

A. 80%
B. 20%
C. 1.6%
D. 33%
E. 8%
F. 16%

Answer: E

Explanation:
The expected dividend growth rate = (Retention Rate) x (Return on Equity). Retention Rate = 1 -
Payout Rate. Return on Equity = NI/E. Thus in this case the expected dividend growth rate = (1 -
(.3/1.5)) x ($5 million/$50 million) = (1 - .20) x (.10) = .08 or 8%.

Question: 3224

Page | 1257
Contrarians assume that:

A. you should bet against "smart money."


B. Past winners will lose in the future.
C. the majority of investors are wrong.
D. the market moves against the trend.

Answer: C

Explanation:
According to the contrarians, most market participants make wrong investment decisions as the
market approaches the peak or trough in a cycle. Hence, their basic trading rule is to determine if the
general investing populace is strongly bullish or bearish and then bet against that trend (hence the
name, "contrarian").

Question: 3225

Short interest is

That answer is correct!


A. the cumulative number of shares that have been sold short by investors and not covered. * the
cumulative number of shares that have been sold short.
B. the cumulative number of shares that have been sold short and covered.
C. the cumulative number of shares that have been sold short, divided by daily NYSE volume.
D. the cumulative number of shares that have been sold short, divided by daily NYSE + OTC volume.

Answer: A

Explanation:
Technical analysts measure short interest through the short interest ratio, which is equal to the
outstanding short interest divided by the average daily volume of trading on the exchange. A higher
ratio is interpreted as bullish, because a larger relative short interest is indicative of high potential
demand for stock from those that have sold short but have not yet covered.

Question: 3226

Lynn Burns, CFA, is examining the performance of Intelligent Semiconductor, and has gathered the
following information:

Market discount rate: 14.5% per year


Observed Price/Earnings ratio: 26.50

Given this information, what is the Franchise Price/Earnings ratio for Intelligent Semiconductor?

A. The answer cannot be calculated from the information provided.


B. 30.99
C. 19.60
D. 33.40
E. None of these answers is correct.
F. 23.14

Answer: C

Explanation:

Page | 1258
The Franchise Factor method of value measurement is in many respects similar to EVA and MVA
calculations. When examining a company using the franchise value approach, the observed price-to-
earnings ratio is broken down into its two components - (1) the "base P/E," which is based on the
Company's ongoing performance, and (2) a "franchise P/E" that is based on the expected value of
new and profitable business opportunities. This relationship is illustrated as follows:

Franchise P/E = Observed P/E - Base P/E

Where the Base P/E equals the reciprocal of the market discount rate. For example, if the market
discount rate is 14.5%, the base P/E would be equal to 6.89655.

In this example, all the necessary information has been provided, and the calculation of the
Franchise P/E is as follows:

Franchise P/E = (26.5 - 6.89655) = 19.60345

Question: 3227

Technicians often feel that a large proportion of investment advisory services with bearish attitudes
is indicative of

A. a flat trend channel.


B. the approach of a peak and the onset of a bear market.
C. the approach of a trough and the onset of a bull market.
D. a low cash position by mutual funds.

Answer: C

Explanation:
Many technicians feel that investment advisory services tend to be trend followers, so that by the
time most services are bearish, the bear market is nearing a trough, which will be followed by a new
bull market.

Question: 3228

Growth companies are those that can

A. experience dividend growth that is consistently higher than their required rates of return.
B. experience sales growth that is consistently higher than their required rates of return.
C. earn rates of return on their investments that are consistently above their required rates of return.
D. keep their leverage ratios low.

Answer: C

Explanation:
Growth companies have the opportunities and abilities to earn rates of return on the investments
they make that are consistently higher than their required rates of return. Such companies have
trouble holding their high returns and growth forever, though. Almost all of them see a slowdown in
growth over time.

Question: 3229

An intern at Smith, Kleen, & Beetchnutty has recently been asked to value shares of Dynamic Impact,
a multinational media conglomerate, using the Gordon Model. Dynamic Impact is a stable firm which

Page | 1259
has experienced steady growth for much of the last twenty years, and this growth rate is not
anticipated to change. In his assignment, the intern has been provided with the following
information:

Expected dividend at t5: $1.45


Required rate of return on equity: 14.25% per year
Growth rate of dividends: 10.00% per year

Using the information provided, what is the value of Dynamic Impact's common shares?

A. $18.90
B. None of these answers is correct.
C. $21.12
D. $26.65
E. $31.34
F. The answer cannot be calculated from the information provided.

Answer: B

Explanation:
Since the growth rate of dividends is not expected to change in the future, the Gordon Model can be
applied. The Gordon Model is commonly referred to as the "constant growth dividend discount
model." This model of common stock valuation is illustrated as follows:

P0 = [D1 / (r - g)]

Where
P0 = the price of common stock X at time 0
D1 = the expected dividend at t1
r = the required rate of return on equity investments and g = the expected growth rate of dividends.

Since we are not provided the dividend at t1, we must determine it manually be discounting the
dividend at t5 by the growth rate of dividends. The determination of D1 is as follows:

D1 = {D5 / [(1 + g)(1 + g)(1 + g)(1 + g)]}


D1 = {$1.45 / [1 + 0.10)(1 + 0.10)(1 + 0.10)(1 + 0.10)]}
D1 = {$1.45 / 1.1461} = $0.9904

Now that the dividend at t1 has been determined, the given information can be put into the
equation provided, leading to the following series of calculations:

P0 = [$0.9904 / (0.1425 - 0.10)] = $23.30

This is illustrated by none of the answers provided.

When using the Gordon model, remember that the required rate of return "r" must be greater than
the expected growth rate "g." Otherwise, this equation will produce a nonsensical answer.

Question: 3230

Fund A is a no-load fund but it charges a 2% redemption fee. Fund B is a 5% load fund which charges
no redemption fee. Fund A is expected to have a return of 13% while fund B is expected to have a
return of 17%. If your investment horizon is 1 year, which fund should you invest in and what is your
expected net rate of return per year?

Page | 1260
A. A; 11.00%
B. B; 10.9%
C. A; 10.75%
D. B; 11.15%

Answer: D

Explanation:
Fund A's rate of return over 1 year equals 1.13 *0.98 - 1 = 10.74%. Fund B's return equals 0.95 * 1.17
- 1 = 11.15%. Hence, select fund B for a 1-year horizon.

Question: 3231

If the OTC-to-NYSE volume is high, the contrarians interpret it as implying that investors are:

That answer is correct!


A. bullish.
B. about to enter the market with more cash.
C. aggressive.
D. bearish.

Answer: A

Explanation:
Since the OTC market is dominated by small stocks which are riskier than the large stocks on NYSE, a
high OTC volume relative to NYSE (above 110-112%) implies that investors are bullish. Of course,
since the contrarians bet against the market at market turning points, they will consider this behavior
by investors as a bearish signal if the market is at a high.

Question: 3232

Given that the P/E ratio on a common stock is 12, the expected dividend payout ratio is 0.7, and the
dividend growth rate is 6%, what is the required rate of return?

A. 9.3%
B. Not enough information
C. 17.4%
D. 12.5%
E. 11.8%

Answer: E

Explanation:
The infinite period Dividend Discount Model claims that the current price of a common stock is equal
to D1 / (k - g), where D1 is next period's (most often next year's) dividend, k is the required rate of
return, and g is the growth rate of dividends. The earnings multiplier model goes a step further by
dividing both sides of the infinite period Dividend Discount Model equation by expected earnings
during the next 12 months,
yielding P/E = (D1/E) / (k - g). Rearranging this results in k = (D1/E) / (P/E) + g. In this question the
required rate of return is equal to 0.7/12 + 0.06 = 0.118 = 11.8%.

Question: 3233

Page | 1261
The objective of analysis of alternative economies and security markets in the three-step, top-down
approach is to decide how to allocate investment funds

A. among countries and industries.


B. among countries.
C. among countries and industries, and between bonds stock and cash.
D. between bonds, stock and cash.
E. among countries and between bonds, stock and cash.

Answer: E

Explanation:
The analysis of alternative economies and security markets is the first step in the three step process,
and involves the decision to allocate funds at a general level between countries and types of
securities. Making the decision to allocate between different industries is made in the second step.

Question: 3234

Marlene Gooseberry, an institutional money manager with Middle Road Brokerage, has been
examining a stock market series and has determined the following information:

The dividend payout ratio at t1 has been estimated at: 31% The required rate of return is 16%
The anticipated future growth rate of dividends is 13.75%
The anticipated future growth rate of earnings is 14.25%
The corporate tax rate is 35%

Using this information, what is the earnings multiplier for this stock market series? Choose the best
answer.

A. 8.96
B. 7.75
C. 13.78
D. 17.71
E. None of these answers is correct.
F. The answer cannot be determined from the information provided.

Answer: C

Explanation:
Estimating the earnings multiplier for a stock market series requires the estimation of each of the
following components:

1. The dividend payout ratio.

2. The required rate of return on common stock in the country/region/industry/sector being


analyzed.

3. The expected growth rate of dividends for the stocks in the country/region/industry/sector being
analyzed.

Once values for each of these components have been determined, they are imputed into the
following formula: {P/E = [D/E / (k - g)]}. Where: P/E = the earnings multiplier, or Price-to-Earnings
ratio, D/E = the dividend payout ratio at t1, k = the required rate of return, and g = the anticipated
growth rate of dividends.

Page | 1262
In this example, all of the necessary information has been provided, and the calculation of the
earnings multiplier is shown as follows: {P/E = [0.31 / 0.16 - 0.1375] = 13.78}.

Notice that it was the anticipated growth rate of dividends, not the anticipated growth rate of
earnings, which was used in determining the earnings multiplier. Additionally, note that the tax rate
was not explicitly factored in to the equation, as the earnings figure used in the dividend payout ratio
is already an after-tax figure.

Question: 3235

The exact estimate of future dividends depends on the outlook for ________ and the firm's
________.

A. earnings; sales
B. earnings growth; dividend policy
C. stock price; dividend policy
D. sales price; earnings

Answer: B

Explanation:
The exact estimate of future dividends depends on the outlook for earnings growth and the firm's
dividend policy.

Question: 3236

The NAV of an open-ended fund is $31.22. The fund charges a 6.3% sales charge and no redemption
charges. The price at which you can sell a share of the fund equals ________.

A. $29.37
B. $29.25
C. $31.22
D. none of these answers

Answer: C

Explanation:
The price at which you can redeem an open-ended fund equals its NAV (in the absence of any
redemption charges).

Question: 3237

Total assets/equity measures ________.

That answer is correct!


A. financial leverage
B. net profit margin
C. retention rate
D. operating performance

Answer: A

Explanation:

Page | 1263
Total assets divided by equity measures financial leverage.

Question: 3238

Consider the following annual growth forecasts for a common stock:

Growth in years 1-2 = 20%


Growth in year 3 = 15%
Growth after year 3 = 12%

Assuming that the last dividend was $1.80 per share, and the required rate of return is 17% per year,
what is the value of this common stock?

A. $22.27
B. $25.34
C. None of these answers is correct.
D. $35.82
E. $17.89
F. $31.92

Answer: C

Explanation:
None of these answers is correct.

To determine the value of a common stock experiencing temporary supernormal growth, use the
following equation:

{V = {[d0 * (1 + gs)^1] / k} + {[d1 * (1 + gs)^2} + ... {dn * (1 + gs)^n} + {[dn * (1 + gs)^n * (1 + gn] / (k -
g)}/ (1 + k)^n}}

Where: V = the value of common stock at t0, d0 = the dividend at t0, d1 = the dividend at t1, dn = the
dividend at tn, gs = the supernormal rate of growth, gn = the normal rate of growth, n = the time
period "n", and k = the required rate of return.
In this example, the supernormal growth period is followed by a transitional growth period of one
year, during which the growth rate of this stock is expected to grow at 15% annually. This period will
follow the two-year supernormal growth period, and would be denoted as "g subset t" if we were to
rewrite the basic equation listed above. The calculation of the value of this common stock is
illustrated as follows:

{V = {[$1.80 * (1.20)^1] / (1.17)} + {[$1.80 * (1.20)^2] / (1.17)^2} + {[$1.80 * (1.20)^2 * (1.15)^1] /


(1.17)^3} + {{[$1.80 * (1.20)^2 * (1.15)^1 * (1.12)^1]/ (0.17 - 0.12)}/ (1.17)^3}

Which can be deduced to the following:

{V = [$1.846154 + $1.893491 + $1.861124 + $24.814983] = $30.415752}

Question: 3239

The empirical evidence on mutual fund performance indicates which of the following?

I. Better performance is associated with higher expense ratios.


II. On a net basis, mutual funds under-perform the market on average.
III. On average, mutual funds do better than the buy-and-hold policy by about 65 basis points.

Page | 1264
A. II & III
B. II only
C. I only
D. I, II & III
E. I & III
F. III only
G. I & II

Answer: G

Explanation:
Most of the studies on mutual fund performance show I and II. On average, mutual funds have been
shown to be unable to outperform a buy-and-hold policy after fund expenses are accounted for.

Question: 3240

An decrease in the required rate of return will have what effect on the earnings multipliers of
common stocks? Further, what effect could be expected from a decrease in the dividend payout
ratio?

A. An decrease in the earnings multiplier; a decrease in the earnings multiplier


B. An decrease in the earnings multiplier; a increase in the earnings multiplier
C. No change in the earnings multiplier, a decrease in the earnings multiplier
D. An increase in the earnings multiplier; an increase in the earnings multiplier
E. An increase in the earnings multiplier; a decrease in the earnings multiplier

Answer: E

Explanation:
A simple method of determining the appropriate earnings multiplier for a common stock can be
found by manipulating the Infinite Period Dividend Discount Model such that it resembles the
following:

P/E = [(d1/e1) / (k - g)]

Where: P/E = the earnings multiplier expressed as the Price-to-Earnings Ratio, d1/e1 = the dividend
payout ratio at t1, k = the required rate of return, and g = the anticipated annual growth rate.

As you can see, a decrease in the required rate of return, holding everything else equal, will lead to
an increase in the earnings multiplier. Opposite this effect, a decrease in the dividend payout ratio
will lead to a decrease in earnings multiplier.

Question: 3241

The current earnings per share on a value-weighted index is $3.7. If the growth rate in the index is
expected to be 3%, its average payout ratio is 35% and the index value is $9.3 per share, the expected
return on the index is:

A. 16.9%
B. 15.4%
C. 17.3%
D. 16.6%

Page | 1265
Answer: C

Explanation:
You should remember that the Dividend Discount Model is applicable to individual stocks as well as
stock indices. Indeed, that and estimation of the variables involved in the Dividend Discount Model
are the thrust of Reilly & Brown, chapter 18.

In standard notation, Po = D1/(k-g). In this case, g = 3%. The expected dividend per share next year =
$3.7 * 35%*(1+3%) = $1.33. Therefore, 9.3 = 1.33/(k-3%). Solving for k gives k = 3% + 1.33/9.3 =
17.3%. This is the expected return on the index.

Question: 3242

Which of the following represents a "smart money" technical indicator? Choose the best answer.

A. Block Uptick/Downtick Ratio.


B. Diffusion Index.
C. More than one of these answers is correct.
D. Breadth of market.
E. Short sales by specialists.
F. Percentage of futures traders bullish on stock index futures.

Answer: E

Explanation:
Of the choices listed, only "short sales by specialists" represents a "smart money" technical indicator.
Technical analysts view a large increase in short sales by specialists as a bearish near-term indicator.
Conversely, a decline in the ratio of short sales by specialists is viewed as a near-term bullish
indicator. Data for short sales by NYSE specialists is published weekly in Barron's. The normal ratio of
short sales by specialists has been approximately 40%, and technicians view a decline to 30% or
below as a bullish sign. Conversely, technicians would view a ratio of 50% or greater as a bearish
signal.

"Breadth of market" refers to the measure of advancing versus declining issues. The "Diffusion Index"
is a measure of market breadth and is defined as the volume of advancing issues plus one-half of the
volume of unchanged issues, divided by the total number of issues traded. Short interest measures
the total volume of outstanding short positions, and the sentiment of futures traders is used by
contrarian technical analysts, who take a contra approach. The "Block Uptick/Downtick Ratio" is used
to measure the near-term sentiment of institutions.

Question: 3243

Which of the following statements is true?

A. Short sales by specialists making up more than 50 percent of total short sales on an exchange is
viewed as a bearish sign by technical analysts who try to follow the "smart money." Such an
occurrence would be viewed as a long-range indicator of market performance.
B. Short sales by specialists making up more than 50 percent of total short sales on an exchange is
viewed as a sign of an approaching market peak by technical analysts who try to follow the "smart
money." Such an occurrence would be viewed only as a short-range indicator of market
performance.
C. Short sales by specialists making up less than 30 percent of total short sales on an exchange is
viewed as a sign of an approaching market trough by technical analysts who try to follow the "smart
money." Such an occurrence would be viewed as a long-range indicator of market performance.

Page | 1266
D. Short sales by specialists making up less than 30 percent of total short sales on an exchange is
viewed as a bullish sign by technical analysts who try to follow the "smart money." Such an
occurrence would be viewed only as a short-range indicator of market performance.

Answer: D

Explanation:
Specialists, who have access to the limit-order books for the stocks in their charge, are often studied
by technical analysts trying to follow the "smart money." When their short sales make up over 50%
of total short sales, this is viewed as bearish sign because of the implicit specialist expectation of
declining share prices. When their short sales make up under 30% of total short sales, this is viewed
as a bullish sign because of their implicit attempts to stay away from short sales. Specialist short sales
typically make up 40% of total short sales.

Question: 3244

In the U.S. economy, suppose it is found that every 5% increase in GNP is associated with an 8%
increase in S&P sales, with a negligible intercept term. Further, the net profit margin on S&P 500 is
8.2%. If the earnings multiplier on S&P 500 changes from 11.3 to 11.9 and the GNP declines by 2%,
the change in the S&P 500 index value equals ________.

A. -8.08%
B. -5.29%
C. +5.03%
D. +1.94%

Answer: D

Explanation:
The 2% decline in GNP will cause a decline of 2/5 * 8 = 3.2% S&P sales. This will cause a decline in
earnings per share of 3.2%, too (NOT 8.2% * 3.2%). Since index value = earnings multiplier * earnings
per share, the change in S&P 500 equals (1-0.032)*11.9/11.3 - 1 = +1.94%.

Question: 3245

Which of the following is the correct formula for the breakdown of ROE?

A. Profit Margin x Total Assets x Financial Leverage


B. Profit Margin x Total Asset Turnover x Financial Leverage
C. P/E x Earnings per share x Number of Shares Outstanding
D. COGS x Total Asset Turnover x Current Liabilities

Answer: B

Explanation:
By definition: ROE = Profit Margin x Total Asset Turnover x Financial Leverage

Question: 3246

A current measure of speculative trading activity on the market is the ratio of

A. LSE volume to OTC volume.


B. OTC volume to NYSE volume.
C. mutual fund volume to NYSE volume.

Page | 1267
D. TSE volume to NYSE volume.
E. AMEX volume to NYSE volume.

Answer: B

Explanation:
Technicians consider speculative activity high when the OTC volume reaches some high level (such as
112%, for example) relative to NYSE volume. Conversely, they consider speculative activity low when
OTC volume reaches some low level (such as 87%, for example) relative to NYSE volume.

Question: 3247

The short interest ratio is

A. the cumulative number of shares that have been sold short, divided by outstanding short interest.
B. outstanding short interest divided by total daily volume on the exchange.
C. total daily volume on the exchange divided by outstanding short interest.
D. the cumulative number of shares that have been sold short by investors and not covered.

Answer: B

Explanation:
Technical analysts measure short interest through the short interest ratio, which is equal to the
outstanding short interest divided by the average daily volume of trading on the exchange. A higher
ratio is interpreted as bullish because a larger relative short interest is indicative of high potential
demand for stock from those who have sold short but have not yet covered their sales.

Question: 3248

The investment decision hypothesis maintains that the most important portfolio decision is that of

A. all of these answers


B. diversification
C. financing
D. asset allocation

Answer: D

Explanation:
A good portfolio achieves the investor's desired risk/return profile through asset allocation. Typically
a more diversified portfolio has lower risk.

Question: 3249

Consider the following transactional information for the inventory account of an institutional bond
house:

1st Quarter
Ending portfolio value: $400,500,000
Total amount invested: $396,000,000

2nd Quarter
Ending portfolio value: $401,900,000
Total amount invested: $400,500,000

Page | 1268
3rd Quarter
Ending portfolio value: $406,500,000
Total amount invested: $400,000,000

4th Quarter
Ending portfolio value: $409,800,000
Total amount invested: $400,000,000

Using this information, what is the annual time-weighted rate of return for this portfolio? Assume no
taxes or transaction charges.

A. None of these answers is correct.


B. 5.91% per year
C. The time-weighted rate of return cannot be calculated from the information provided.
D. 5.79% per year
E. 6.22% per year
F. 5.09% per year

Answer: B

Explanation:
The time-weighted rate of return is the preferred method of return calculation in the investment
management industry, primarily because this method is not sensitive to significant additions and
withdrawals of funds from portfolios under examination. The calculation of the time-weighted rate
of return involves three steps, which are illustrated as follows:

Step 1:
Price the portfolio immediately prior to any significant additions or withdrawals. Separate the
portfolio into a series of subperiods based on the dates of cash inflows and outflows.

Step 2:
Calculate the holding period return for each subperiod.

Step 3:
Determine the annualized holding period return by linking or compounding the holding period return
of each subperiod. If the investment is for more than one year, use the geometric mean of the annual
returns as the time-weighted rate of return. If the investment is for less than one year, compound the
subperiod returns to obtain an annualized measurement.

To begin the process of determining the time-weighted rate of return, we would break the portfolio
up into the subperiod series of cash flows. However, in this example, the cash flows are already
aggregated for us and we can move on to the next step:
determining the holding period return for each subperiod. This process is detailed as follows:

Quarter 1 holding period return = [($400,500,000 ending value - $396,000,000 invested) /


$396,000,000 invested] = 1.13636%
Quarter 2 holding period return = [($401,900,000 ending value - $400,500,000 invested) /
$400,500,000 invested] = 0.34956%
Quarter 3 holding period return = [($406,500,000 ending value - $400,000,000 invested) /
$400,000,000 invested] = 1.625%
Quarter 4 holding period return = [($409,800,000 ending value - $400,000,000 invested) /
$400,000,000 invested] = 2.450%

Page | 1269
Now that the holding period return for each subperiod has been determined, we must annualize the
return measure by taking the product of all four quarterly returns. This process is illustrated below:

[(1 + .0136) * (1 + .0035) * (1 + .0163) * (1 + .0245) - 1] = .05905 or 5.905%


When calculating the time-weighted rate of return, remember that the total amount invested is the
relevant figure, not the beginning portfolio value. Notice that during the third quarter, the total
amount invested does not equal the ending amount for the second quarter. (A similar situation exists
in the fourth quarter).

This differential could be explained by numerous phenomena. Perhaps the difference is due to a cash
withdrawal from the account. Maybe it was used to pay expenses or meet an outstanding margin
call.

What is important to note is the fact that this money (the difference between the total amount
invested and the ending portfolio value for the previous subperiod) was not invested, and should not
be included in the holding period return for the fourth quarter. So said, whenever possible you
should use the total amount invested rather than the beginning portfolio value in the calculation of
the subperiod holding period return.
If you chose 5.79%, remember that in the calculation of the time-weighted rate of return, it is the
geometric average that is used, not the arithmetic average.

Question: 3250

Calculate the dividend growth rate for a company that consistently pays out 30% of its earnings in
dividends and has a Return on Equity (ROE) of 10%.

A. 6.4%
B. 12%
C. 15%
D. 10.0%
E. 7.0%
F. 9.6%

Answer: E

Explanation:
The estimated growth rate of dividends = (Retention Rate) x (Return on Equity). In this case the
estimated growth rate of dividends = (1 - Payout Ratio) x (ROE) = 70% x 10% = 7%.

Question: 3251

Technical analysts would feel that an upside-downside volume ratio with a value of 1.04

A. is bearish.
B. indicates that the market is overbought.
C. indicates that the market is oversold.
D. is neither particularly bullish nor bearish.

Answer: D

Explanation:
Technical analysts may use the ratio of upside-downside volume as an indicator of short-term
momentum for the market. They feel that a ratio value of 1.50 or more indicates that the market is
overbought, while a ratio of 0.70 or less indicates that the market is oversold.

Page | 1270
Question: 3252

Jim Williams, a financial analyst with Churn Brothers Brokerage, is attempting to value share of
Intelligent Semiconductor. In his calculation, Jim calculates the following for each subperiod:

[Net income + Depreciation - Capital Expenditures - Increases in Working Capital - Principal


Repayments + New Debt Issues]. The results of this calculation will be referred to as "X."

The second step in the process is the determination of the "multiple" of X that shares of Intelligent
will trade for in three years. This figure will be referred to as "M."

Finally, Jim determines the required equity rate of return for shares of Intelligent Semiconductor.
This figure will be referred to as "r."

These calculations are inputted into the following equation:

Price of Intelligent Semiconductor = {[X during year 1 / (1 + r)] + [X during year 2 / (1 + r)(1 + r)] + [X in
year 3 / (1 + r)(1 + r)(1 + r)] + [M * X in year 3/ (1 + r)(1 + r)(1 + r)]}

The results of this calculation are used as the value of Intelligent shares.

Which of the following best characterized the valuation method employed by Jim Williams?

That answer is correct!


A. Free cash flow to equity method
B. Arbitrage-pricing model
C. Supernormal growth model
D. None of these answers
E. Multi-period dividend discount model

Answer: A

Explanation:
The method illustrated in this example is the free cash flow to equity method.

Question: 3253

During years of temporary supernormal growth, where growth exceeds the required rate of return,
the analyst must use the ________ version of the dividend discount model to value a stock.

A. expected
B. current
C. interim
D. finite

Answer: D

Explanation:
During years of temporary supernormal growth, where growth exceeds the required rate of return,
the analyst must use the finite version of the dividend discount model to value a stock. This is also
known as the variable growth version of the dividend discount model.

Question: 3254

Page | 1271
The problem with using the confidence index as an indicator of investor sentiment is that:

A. it ignores the equity market in its calculation.


B. it ignores the supply side of the bond market.
C. it is difficult to calculate precisely due to incomplete bond data.
D. it is difficult to calculate precisely due to incomplete bond data and it ignores the supply side of
the bond market.

Answer: B

Explanation:
The Confidence index is the ratio of the average yield on 10 top grade bonds to the Dow Jones
average of 40 bonds. The index measures the difference in yield spreads between high-grade bonds
and a large cross-section of available bonds. An increase in this index implies that the yield spread
between the bonds has narrowed. Now, from a demand perspective, this will happen when the
demand for riskier bonds rises. However, the spread could also decrease if the supply of the risky
bonds decreases relative to the highgrade bonds. In that case, the change in the confidence index
simply reflects the credit needs of the economy and not a change in the risk appetite of investors.

Question: 3255

Contrary-opinion traders expect mutual funds to have a

That answer is correct!


A. low percentage of cash near the peak of the market.
B. high percentage of cash near the peak of the market.
C. high percentage of cash during a rising market trend.
D. low percentage of cash during a rising market trend.

Answer: A

Explanation:
Contrary-opinion traders believe that mutual fund managers are usually wrong in their market
expectations near peaks or troughs. Mutual fund managers would thus be expected to believe the
market would continue to increase when it is near a peak. The managers would have a very low
position in cash in order to invest as much as possible in the rising market. When their cash positions
are very low, contrary-opinion traders would see this as a sign that the market is approaching a peak
and sell shares in anticipation of the upcoming bear market.

Question: 3256

Which of the following factors account for differences among required rates of return for different
countries?

A. Different risk premiums due to different risk environments


B. Wide variance in inflation rates
C. All of these answers
D. Exchange-rate volatility

Answer: C

Explanation:
All of the mentioned variables affect the RFR because they affect an investment's real return.

Page | 1272
Question: 3257

A net advance on an advance-decline series for the NYSE, coupled with a declining S&P 500 would,
according to technical analysts,

That answer is correct!


A. signal a market trough.
B. signal a period of market unpredictability.
C. signal a market peak.
D. signal a temporary reprieve from the bear market.

Answer: A

Explanation:
Technical analysts predict that when a market index is declining, but most stocks post price increases,
the market is near its trough. Conversely, they predict that when a market index is increasing, but
most stocks post price declines, the market is near its peak.

Question: 3258

When estimating the industry net profit margin, it is suggested that you

A. begin with profit before interest and taxes and then estimate interest.
B. begin with profit after taxes and deduct dividends paid.
C. begin with the operating profit margin and then estimate depreciation expense, interest expense,
and the tax rate.
D. calculate the average earnings per share.

Answer: C

Question: 3259

Which of the following is not one of the three main determinants of the required rate of return on an
asset?

A. The risk premium on the asset


B. The expected rate of inflation during the holding period
C. The economy's real risk-free rate of return
D. The Markowitz efficient frontier

Answer: D

Explanation:
Although the Markowitz efficient frontier may be used in the process of deriving the risk premium on
the asset, it is not in itself one of the three main determinants of the required rate of return on an
asset. All investments are affected by the real risk-free rate of return and the expected rate of
inflation, because those two factors determine the nominal risk-free rate of return. The nominal risk-
free rate of return plus the risk premium on the asset is equal to the asset's required rate of return.

Question: 3260

Assuming estimates of .30, .15 and 0.135 for the dividend payout, required rate of return, and
expected growth rate, respectively, calculate the earnings multiplier.

Page | 1273
A. 50X
B. 14X
C. 5X
D. 13X
E. 20X

Answer: E

Explanation:
The earnings multiplier or P/E ratio = Dividend payout ratio/Required rate of return-Expected growth
rate. Analyzing these inputs and the underlying factors, and comparing these to the indications of
the macroanalysis, the analyst forecasts an earnings multiplier for the firm. In this example, P/E =
.3/(.15-.135) = 20 times.

Question: 3261

For the past 30 years or so, the retention rate of earnings for firms in the S&P 400 has fluctuated
largely between

That answer is correct!


A. 45 and 60%.
B. 20 and 30%.
C. 10 and 20%.
D. 75 and 90%.

Answer: A

Explanation:
Between 1977 and 1994, the retention rate has fluctuated largely between 45 and 60%, although
there was a period in the early 1990s when it fell to a low of 25%. The retention rate is used to derive
the growth rate of earnings and dividends, which is used to derive the earnings multiplier.

Question: 3262

A preferred stock has a $100 par value and a dividend payout of $8 per year. Your required return is
10%. What is the value of the preferred stock?

A. $90
B. $180
C. not enough information to calculate it
D. $80

Answer: D

Explanation:
Value of preferred stock is Dividend/Required return or $8/0.1 = $80.

Question: 3263

Given that the correct value of a common stock is $43, the required rate of return on the stock is
17%, and next period's dividend will be $4.50, using the infinite period Dividend Discount Model,
what is the growth rate of dividends?

Page | 1274
A. Not enough information
B. 7.1%
C. 6.5%
D. 8.4%
E. 4.8%

Answer: C

Explanation:
The infinite period Dividend Discount Model postulates that the current value of a common stock is
equal to D1 / (k - g), where D1 is next period's dividend, k is the required rate of return, and g is the
growth rate of dividends. Rearranging this yields g = k - D1/(current value). In this question, the
dividend growth rate is equal to 0.17 - (4.5 / 43) = 0.065 = 6.5%.

Question: 3264

A portfolio manager with Old School Securities is trying to determine whether shares of Ludicrous
Telecom are undervalued. In his analysis, this portfolio manager has determined that the firm's
current dividend of $0.40 per share is anticipated to grow 16% annually. Additionally, this portfolio
manager has forecasted that she will be able to sell shares of Ludicrous Telecom for $27 per share in
four years. Assuming a 18.75% per year required rate of return, what is the value of Ludicrous
Telecom's shares?

That answer is correct!


A. $15.09
B. $17.34
C. $19.19
D. None of these answers is correct.
E. $14.10

Answer: A

Explanation:
The Multiple Holding Period form of the Dividend Discount Model takes the following form: {V = {[d1
/ (1 + k)] + [d2 / (1 + k)^2] + ... .[dn / (1 + k)^n] + [Pn / (1 + k)^n]}
Where: V = the price of the common stock at t0, d1 = the annual dividend at t1 (this is found by
multiplying the annual dividend at t0 by (1 + the anticipated growth rate), d2 = the annual dividend
at t2 (this is found by multiplying the dividend at t1 by (1 + the anticipated growth rate), k = the
required rate of return, n = period "n", and Pn = the sale price of the common stock at time "n".

In this example, time "n" is the fourth year, as this is the end horizon for this investor's holding
period. Had the investor in this example forecasted selling the shares at the end of the 10th year,
then "n" would be the tenth year.

Now that the formality of expressing the equation for this form of the DDM has been carried
through, we can move toward a calculation of the value of this common stock. In this example, all of
the necessary information has been provided, and the calculation of the value of this retail stock is as
follows:

{V = [($0.40 * 1.16) / (1 + 0.1875)^1] + [($0.464* 1.16) / (1 + 0.1875)^2] + [($0.53824 * 1.16) / (1 +


0.1875)^3] + [($0.624358 * 1.16) / (1 + 0.1875)^4] + [$27 / (1 + 1.1875)^4]}

Which can be further broker down into the following:

Page | 1275
{V = [$0.390737 + $0.381688 + $0.372849 + $0.364214 + $13.577796] = $15.09}

Question: 3265

Which of the following represents a "contrary opinion" technical indicator? Choose the best answer.

A. Futures traders bullish on stock index futures.


B. Investment Advisor Opinions.
C. Diffusion Index.
D. Confidence Index.

Answer: B

Explanation:
Contrarian technical analysts often examine the opinions of market participants, including
investment advisors and futures speculators, for insight into the overall sentiment of securities
professionals. Believing these individuals to be inaccurate in their forecasts, contrary opinion
technical analysts assume the contra-side opinion. So said, a high degree of bullishness amongst
investment advisors is viewed as bearish by contrary opinion technical analyst. Conversely, a high
degree of bearishness amongst investment advisors is viewed as a bullish signal by contrary opinion
technical analysts. In this same light, a high degree of bullishness amongst stock index futures traders
is viewed as a bearish signal by contrarian technical analysts, and a low degree of bullishness
amongst stock index futures traders is viewed as a bullish signal.

The Confidence Index, a "smart money" technical indicator, is a measure of yield spreads between
high-grade corporate bonds and the yields on average corporate bonds. The Diffusion Index
measures the breadth of the market, and is found by taking the total volume of advancing shares
plus one-half of the issues unchanged, divided by the total number of issues traded.

Question: 3266

Assume the following information about a stock market series:

Retention rate = 63%


Expected growth rate of dividends = 10% per year
Expected growth rate of earnings = 12% per year
Required rate of return = 13% per year
Earnings per share next year = $3.65

What is the appropriate earnings multiplier for this stock market series? Further, what is the value of
this series?

That answer is correct!


A. 12.33; $45
B. None of these answers is correct.
C. 21; $76.65
D. 63; $229.95
E. The answer cannot be determined from the information provided.
F. 37; $135.05

Answer: A

Explanation:
The earnings multiplier is found as 12.33 for this series, and the value of the series is computed as

Page | 1276
$45.

Estimating the earnings multiplier for a stock market series requires the estimation of each of the
following components:

The dividend payout ratio.

The required rate of return on common stock in the country/region/industry/sector being analyzed.

The expected growth rate of dividends for the stocks in the country/region/industry/sector being
analyzed.

Once values for each of these components have been determined, they are imputed into the
following formula:

P/E = [D/E / (k - g)]

Where: P/E = the earnings multiplier, or Price-to-Earnings ratio, D/E = the dividend payout ratio at t1,
k = the required rate of return, and g = the anticipated growth rate of dividends.

In this example, all of the necessary information has been provided. However, before the earnings
multiplier can be determined, the dividend payout ratio must be found by manipulating the
retention rate. Remember that the retention rate is equal to (1 - the dividend payout ratio), just as
the dividend payout ratio is found as (1 - the retention rate). By this logic, the dividend payout ratio
and the retention rate must always add up to one. The dividend payout ratio for this series is found
as (1 - 0.63) = 0.37.

Now that the dividend payout ratio has been determined, the calculation of the earnings multiplier is
shown below:

P/E = [0.37 / (0.13 - 0.10)] = 12.33

Notice that it is the anticipated growth rate of dividends that is used in this equation, not the
anticipated growth rate in earnings.

Once the earnings multiplier has been determined, the value of this stock market series can be found
by multiplying the EPS figure by the earnings multiplier. This will lead to a value of $45 for this stock
market series.

Question: 3267

Changes in nominal GNP have been found to have

A. little correlation with sales per share for stock series such as the S&P 400.
B. a relatively high correlation with EBIT for stock series such as the S&P 400.
C. a relatively high correlation with sales per share for stock series such as the S&P 400.
D. a relatively high correlation with net profits for stock series such as the S&P 400.

Answer: C

Explanation:
Studies have found that about 40% of the variance in percentage changes in S&P 400 sales can be
explained by percentage changes in nominal GNP. Estimating GNP is thus used to help estimate
earnings per share for stock series such as the S&P 400.

Page | 1277
Question: 3268

A market analyst is examining the financial performance of a large pharmaceutical firm, and has
assimilated the following information:

Adjusted operating profits before taxes: $26,700,000


Cash operating taxes: $9,000,000
Cost of capital: 14.5% per year
Total capital employed: $127,000,000

Using this information, what is the Economic Value Added for this large pharmaceutical firm?
Further, should the management of this Company be considered to have created value for
shareholders?

A. None of these answers is correct.


B. $611,354; management has created economic value
C. $611,354; management may have created economic value
D. ($715,000); management has not created economic value
E. ($715,000); management has created economic value
F. $715,000; management has created economic value

Answer: D

Explanation:
Economic Value Added, a value-based measure of economic profit, is a registered trademark of
Stern, Stewart & Company. The equation used to calculate EVA is as follows:

EVA = {Net Operating Profits Less Adjusted Taxes - [Total Capital Employed * (Cost of Capital)]}.

In this case, the NOPLAT figure must be calculated manually by subtracting the Cash Operating Taxes
from the Adjusted Operating Profit before Tax (AOPBT) figure. Doing so will produce an answer of
$17,700,000 for the Net Operating Profit Less Adjusted Taxes figure. Now that the necessary
information has been determined, the calculation of EVA is as follows:

EVA = {$17,7000,000 - ($127,000,000 * 0.145)} = ($715,000)

A negative EVA calculation indicates that management has failed to provide economic profits to
shareholders, as evidenced by the fact that the opportunity cost of capital employed is less than the
AOPAT figure. In this example, the management of Intelligent Semiconductor should be considered
as having failed to provide shareholder value. The calculation of Adjusted Operating Profit Before
Taxes (AOPBT), and Cash Operating Taxes are important. The calculation of AOPBT is as follows:
Operating profit (after depreciation and amortization) + Implied interest on operating leases + any
increase in the LIFO reserve + goodwill amortization = Adjusted Operating Profit Before Taxes.

Cash Operating Taxes, another important component of EVA, is calculated as follows:


Cash Operating Taxes = Income Tax Expense + tax benefit from interest expenses + tax benefit from
interest on leases + taxes on non-operating income

Adjusted Operating Profit Before Taxes minus Cash Operating Taxes = Net Operating Profit Less
Adjusted Taxes (NOPLAT). Subtracting the dollar cost of capital from the NOPLAT figure will yield the
EVA.

Question: 3269

Page | 1278
A technical analyst with Bullfighter.com, a noted investment research firm, has been examining the
U.S. securities markets, and believes that the market is technically "overbought." Which of the
following technical indicators would this analyst likely use to support his opinion? Choose the best
answer.

A. The CBOE Put/Call ratio is greater than 50%.


B. The % of futures traders bullish on stock index futures has increased significantly.
C. All of these choices are indicative of an "overbought" condition.
D. The ratio of short sales by specialists is less than 50%.
E. The diffusion index has declined significantly.
F. The % of issues trading above their 200-day moving averages is greater than 80%.

Answer: F

Explanation:
The % of issues trading below their 200-day moving average is frequently cited by technical analysts
as a measure of oversold and overbought market conditions. Specifically, technical analysts see the
market as "overbought" when 80% of issues are trading above their 200-day moving average, and
consider a market "oversold" when 80% of issues are trading under their 200-day moving average.
The "Diffusion Index" is a measure of market breadth, and is defined as [(# of advancing issues + 1/2
# of issues unchanged) / # of issues traded]. A decline in the diffusion index is indicative of an
increase in declining issues relative to advancing issues. The CBOE Put/Call Ratio is a contrarian
technical indicator used to gauge the sentiment of investment professionals, and a ratio greater than
50% would be viewed by technical analysts as bullish. Finally, contrarian technical analysts would
view a large increase in the amount of futures traders who express bullish sentiment on stock index
futures as a bearish signal.

Question: 3270

Which of the following represents a "smart money" technical indicator? Choose the best answer.

A. Percentage of futures traders bullish on stock index futures.


B. Diffusion Index.
C. T-Bill-Eurodollar Yield Spread.
D. More than one of these answers is correct.
E. Breadth of market.
F. Block Uptick/Downtick Ratio.

Answer: C

Explanation:
Of the choices listed, only the "T-Bill-Eurodollar Yield Spread" represents a "smart money" technical
indicator. The T-Bill-Eurodollar Yield Spread is used by technical analysts to measure investor
sentiment (i.e. confidence) on a global scale. Specifically, during times of international crisis, there is
usually a "flight to quality" personified by a bidding up on U.S. Treasury issues. This will cause a
widening of the spread between U.S. T-Bills and Eurodollars, causing a decline in this ratio. Technical
analysts would view a sharp increase in the spread between T-Bills and Eurodollars as indicative of
an upcoming market trough, and historical evidence has tended to support this belief.
"Breadth of market" refers to the measure of advancing versus declining issues. The "Diffusion Index"
is a measure of market breadth and is defined as the volume of advancing issues plus one-half of the
volume of unchanged issues, divided by the total number of issues traded. Short interest measures
the total volume of outstanding short positions, and the sentiment of futures traders is used by
contrarian technical analysts, who take a contra approach. The "Block Uptick/Downtick Ratio" is used

Page | 1279
to measure the near-term sentiment of institutions.

Question: 3271

Contrary-opinion technicians would view the fact that 85% of stock index futures speculators are
bullish as

A. an unimportant sign.
B. indicative of an approaching trough.
C. a bullish sign.
D. a bearish sign.
E. indicative of a flat trend channel.

Answer: D

Explanation:
The percentage of stock index futures speculators who are bullish is a measure used by contrary-
opinion technical analysts. If a majority over some decision rule (70%, for example) were bullish, this
would be interpreted as a bearish sign.

Question: 3272

Which of the following correctly illustrates the infinite period dividend discount model?

A. D1 = V * (k - g)
B. None of these answers is correct.
C. V = (D1 / k) + g
D. V = D1 / (1 + r)
E. V = D0 / (k - g)
F. (k-g) = V * r / D1

Answer: A

Explanation:
The infinite dividend discount model assumes can be used to value a common stock when dividends
are expected to grow at a constant rate. The equation which characterizes the infinite period
dividend discount model is as follows:

Value of common stock = D1 / (k - g)

Where: D1 = the dividend at t1, k = the required rate of return, and g = the assumed growth rate of
dividends.

The equation illustrated in choice D is simply an algebraic rearrangement of this equation, solving for
D1.

Question: 3273

Most technical analysts believe that

A. successful fundamental analysis is possible, and is practiced by the majority of fundamental


analysts. They believe that technical analysis is simpler to implement, however, and more reliable
because of its greater use of financial statements.
B. successful fundamental analysis is very difficult. They believe that while success is possible, most

Page | 1280
fundamental analysts are unable to consistently earn above-average returns. They believe that
fundamental analysts rely too heavily on financial accounting statements, which do not contain a
great deal of information useful to security analysts.
C. successful fundamental analysis is very difficult. They believe that while success is possible, most
fundamental analysts are unable to consistently earn above-average returns. Technical analysis is a
better investment guide because of its greater use of financial accounting statements.
D. successful fundamental analysis is very difficult. They believe that while success is possible, most
fundamental analysts are unable to consistently earn above-average returns. They believe that
fundamental analysts rely too heavily on financial accounting statements, which often have incorrect
information.

Answer: B

Explanation:
Technical analysts tend to believe that successful fundamental analysis is contingent on obtaining
new information before others, and processing it quickly and correctly. But they feel that this is very
difficult, and that fundamental analysis is held back by its heavy reliance on financial accounting
statements. They believe that those statements do not contain a great deal of information useful to
security analysts, that different accounting standards make comparisons difficult, and that many
important psychological factors and other nonquantifiable variables (such as customer goodwill) do
not appear in those statements.

Question: 3274

If the current stock price breaks through its moving average from below on heavy volume, most
technical analysts would consider this a to be a

A. bearish sign.
B. sign of an approaching period of instability.
C. sign of an approaching market peak.
D. bullish sign.

Answer: D

Explanation:
Technical analysts use moving averages of past stock prices as indicators of long-term trends. When
the current stock price breaks through its moving average from below on heavy volume, this is
viewed as a bullish sign possibly signaling a reversal in the declining price trend.

Question: 3275

Which of the following best describes the relationship between the relative maturation of an
industry and the retention ratio of companies within the industry? Further, what is the proposed
relationship between expected growth and the relative maturity of an industry?

A. Negative relationship; no correlation


B. Negative relationship; negative relationship
C. Positive relationship; negative relationship
D. Positive relationship; positive relationship
E. Negative relationship; positive relationship

Answer: B

Explanation:

Page | 1281
As an industry advances in maturity, growth of the overall industry will decline. As growth
opportunities diminish, companies within the industry will be forced to pay out a larger proportion of
their earnings as dividends; i.e. the dividend payout ratio will increase. Remember that the retention
ratio is equal to (1 - the dividend payout ratio). Thus, the retention ratio of companies will likely
decline as the industry advances in maturity. The relationship between the dividend payout ratio and
the maturity of the industry is negative and loosely linear.

As an industry becomes more mature, growth opportunities decline. This relationship is also loosely
linear.

Question: 3276

The direction of change approach to estimating an earnings multiplier involves

A. inferring the direction of change in the multiplier based on derivations of the specific estimates for
its ten major components.
B. inferring the direction of change in the multiplier based on predictions for changes in its three
major components.
C. inferring the direction of change in the multiplier based on derivations of specific estimates for its
three major components.
D. inferring the direction of change in the multiplier based on predictions for change in its ten major
components.

Answer: B

Explanation:
The direction of change approach begins with the current earnings multiplier and estimates the
direction and extent of change for the dividend payout ratio and the variables that influence the
required rate of return and the growth rate of dividends and earnings.

Question: 3277

This relates to stock prices moving in trends, analogous to the movement of water.

A. Mutual Fund Cash Positions


B. Dow Theory
C. Short Sales' by Specialists
D. Diffusion Index
E. Relative Trend
F. Margin Debt
G. Block Uptick-Downtick Ratio
H. Odd-Lot, Short-Sales Theory

Answer: B

Explanation:
Charles Dow, who published the Wall Street Journal in the late 1800s, described stock prices as
moving in trends analogous to the movement of water. He postulated three types of price
movements over time:

1. major trends that resemble tides in the ocean.


2. intermediate trends that resemble waves.
3. short-run movements that resemble ripples.

Page | 1282
Question: 3278

What is the name of this technical indicator?

(Number of advancing issues + 1/2 volume of issues unchanged) / total number of issues traded

A. Confidence Index.
B. CBOE Put/Call Ratio.
C. None of these answers is correct.
D. Block Uptick/Downtick Ratio.
E. Advance/Decline Line.

Answer: C

Explanation:
The technical indicator profiled in this example is the diffusion index, which is a measure of market
breadth. While "Advance/Decline Line" is an esthetically appealing choice, it is nonetheless incorrect.
The Advance/Decline Line is found by subtracting the volume of declining issues by the volume of
advancing issues, with no regard to the volume of issues unchanged.

Question: 3279

The scotch whiskey distilling industry is most likely in which stage of the industry life cycle? Further,
what type of earnings multiple and payout ratio should be expected from firms in this industry?

A. Accelerating growth, high multiple and low payout


B. Mature growth, low multiple and high payout
C. Mature growth, high multiple and high payout
D. Sales decline, low multiple and high payout
E. Development, high multiple and low payout
F. Decelerating growth, low multiple and high payout

Answer: B

Explanation:
The industry life cycle is divided into five distinct stages. Specifically, the industrial life cycle
progresses from the development stage to an accelerating growth stage to a mature growth stage to
a market maturation and stabilization stage. Finally, the fifth stage of the industrial life cycle is
characterized by decelerating growth and sales decline. During the mature growth stage, sales are
still growing, albeit slowly. During the last stage of the industrial life cycle, however, is characterizes
by a decline in annual sales.

As an industry or company progresses through the industrial life cycle, sales begin to grow rapidly
(accelerating growth) then slow considerably as the product or service begins to reach critical mass
(mature growth). During the mature growth stage, the industry or company grows at a slower pace,
until eventually growth begins to slow considerably (market maturity and stabilization) as the market
for the industry or company's products becomes more completely defined. Finally, the industry will
begin to decline, as it advances to the last stage in the industry life cycle.

The market for scotch whiskey is mature, and very defined. Companies who are in the scotch whiskey
distilling industry should be expected to have slow to mediocre annual sales growth, and thus should
warrant a lower earnings multiple. In this same light, firms in the scotch whiskey industry are likely to
have inferior investment opportunities, holding everything else equal. So said, shareholders of these
firms will likely require high dividend payments to compensate for the slow anticipated earnings

Page | 1283
growth.

Question: 3280

A company pays a dividend of $8 per share to the holders of its perpetual preferred stock. The
appropriate discount rate is 7% per year. What is the value of the preferred stock?

That answer is correct!


A. $114.28
B. $11.73
C. Not able to compute with the above data.
D. $14.18

Answer: A

Explanation:
Value = dividend/discount rate = 8/0.07= $114.28.

Question: 3281

A support level

A. is the price range below the current price at which the technical analyst would expect the stock to
get an added boost of demand, keeping its price from falling below that range. The support level is
usually near the 12-week low.
B. is the price range below the current price at which the technical analyst would expect the stock to
get an added boost of demand, keeping its price from falling below that range. A support level
usually develops after the stock has had a meaningful price increase, and has begun to experience
some profit taking.
C. is the price range above the current price at which the technical analyst would expect the stock to
get an added boost of demand, pushing it to even higher prices. The support level is usually near the
52-week high. Investors tend to expect a stock that has broken through its 52-week high to continue
increasing.
D. is the price range below the current price at which the technical analyst would expect the stock
supply to increase, pushing down its price below that range. A stock nearing that range would be a
good candidate for short selling because of the negative price support that it would receive.

Answer: B

Explanation:
A support level is viewed as something of a safety net by technical analysts. They believe that at
some price range there will be an increase in demand by investors who did not purchase the stock
prior to its price increase and who have been waiting for a small reversal to invest. When the stock
price falls within that range, there will be a surge in demand as they purchase the stock.

Question: 3282

To estimate the risk-free rate for a country, estimate the country's expected ________ and adjust the
real risk-free rate for this expectation.

A. GDP
B. growth rate of labor productivity
C. average P/E ratio
D. rate of inflation

Page | 1284
Answer: D

Explanation:
Inflation is risky and needs to be discounted to obtain a true risk-free rate.

Question: 3283

Assume the following information about a common stock:

Price per share: $90.35


Last dividend per share: $1.50
Required return: 15% per year
Expected growth rate: 12% per year

What is the value of this common stock?

A. None of these answers is correct.


B. The answer cannot be determined from the information provided.
C. $79
D. $50
E. $44
F. $56

Answer: F

Explanation:
To determine the value of a common stock using the Infinite Period Dividend Discount Model, use
the following equation:

{V = [d1 / (k - g)]}

Where: V = the value of the common stock at t0, d1 = the annual dividend at t1 (which is found by
multiplying d0 by (1 + g), k = the investor's required rate of return, and g = the anticipated annual
growth rate.

In this example, all of the necessary information has been provided, and incorporating this
information into the Infinite Period DDM will lead to the following:

{V = [($1.50 * 1.12) / (0.15 - 0.12] = $56}

This value is significantly less than the price of the shares in the open market. While at first it may be
appealing to assume that the common stock is overvalued, this may be a dangerous assumption.
Equally likely is the possibility that the Infinite Period DDM is not the ideal valuation model for this
common stock. Perhaps the price of this common stock is reflecting other sources of potential cash
flows, ratherthan the summation of the present value of future dividends. This is an important point
to consider, and one with which you should become familiar.

Question: 3284

To estimate the expected earnings multiplier, it is necessary to estimate changes in the ________.

A. unit labor cost


B. required rate of return

Page | 1285
C. relative strength of foreign competition
D. capacity utilization rate

Answer: B

Explanation:
To estimate the expected earnings multiplier, it is necessary to estimate changes in the required rate
of return, the expected growth rate of dividends (earnings) (g) and the spread between k and g.

Question: 3285

Of the choices listed, which is an important difference between the assumptions underlying technical
and fundamental analysis? Choose the best answer.

A. Technical analysis is reliant on financial statements, whereas fundamental analysis is not heavily
reliant on financial statements.
B. All of these answers represent important differences between fundamental and technical analysis.
C. None of these answers is correct.
D. Technical analysis can be applied to any financial market or security, whereas fundamental
analysis is limited primarily to the equity markets.
E. Technical analysis assumes that securities prices move in identifiable patterns, whereas
fundamental analysts believe that past price data cannot be used to predict future price movements.

Answer: E

Explanation:
Of all the differences between technical analysis and fundamental analysis, perhaps the most
important is the fact that technical analysis assumes that securities markets are not weak form
efficient, whereas fundamental analysis assumes that the Weak Form of the EMH is correct.
Technical analysts assume that securities markets will "price-in" relevant information gradually, and
that securities prices follow observable trends and patterns. Fundamental analysts, on the other
hand, generally assume that past price information cannot be used to predict movements in
securities prices.

Question: 3286

Ryan Williams, a professional money manager with Smith, Kleen & Associates, purchased 200 shares
of Invertran Semiconductor at t0 for $42. At time t1, Invertran paid a $0.90 per-share dividend on the
200 shares owned and Mr. Williams purchased an additional 100 shares for $56.87 per share. At t2,
Invertran paid a dividend of $1.00 per share on the 300 shares and then Mr. Williams sold all 300
shares for $63.15 per share. Similar investments have merited a 12.25% discount rate. Calculate the
dollar-weighted rate of return for this investment.

A. None of these answers is correct.


B. 22.19%
C. 14.34%
D. 9.64%
E. 16.96%
F. 22.10%

Answer: F

Explanation:
Remember that the dollar-weighted rate of return uses the IRR equation in the determination of the

Page | 1286
answer. Further, the dollar-weighted rate of return is simply another name for the IRR equation. So
said, the discount rate is not incorporated in the determination of the dollar-weighted rate of return,
and has been included within this example largely as a distraction. In the determination of the dollar-
weighted rate of return calculation, the first step should be to identify the cash flows for each period.
This process is illustrated as follows:

t0: {-[200 shares purchased * $42 per share] = ($8,400)


t1: {-[100 shares purchased * $56.87 per share] + [$0.90 per share dividend * 200 shares] = ($5,507)
t2: {[300 shares sold * $63.15 per share] + [$1.00 per share dividend * 300 shares]} = $19,245

Now that the cash flows have been determined, incorporating this information into your calculator's
cash flow worksheet and solving for IRR will yield a dollar-weighted rate of return of 22.10% for this
investment.

Question: 3287

A firm has a dividend payout ratio of 70% and it earns a 10% per year return on its equity. Calculate
the expected annual growth rate of the firm's dividends?

A. Not able to compute with the above data.


B. 7%
C. 8%
D. 3%

Answer: D

Explanation:
g=(RR)(ROE)=0.3*0.10=3%.

Question: 3288

Jones Rutherford, a portfolio manager with Churn Brothers Brokerage, has been examining a stock
market series and is trying to determine an appropriate earnings multiplier for the series. In this
analysis, Jones has amassed the following information:

The estimated annual dividend at t1 = $2.30


The estimated EPS at t1 = $4.85
The anticipated growth rate of dividends is 10%
The anticipated growth rate of earnings is 9%
The required rate of return is 14%

Given this information, what is the appropriate earnings multiplier for this stock market series?

A. The answer cannot be determined from the information provided.


B. 9.48
C. 13.04
D. 10.51
E. 13.14
F. None of these answers is correct.

Answer: F

Explanation:
The appropriate earnings multiplier for this stock market series is found as 11.86, therefore none of

Page | 1287
these answers is correct.

Estimating the earnings multiplier for a stock market series requires the estimation of each of the
following components:

1. The dividend payout ratio.

2. The required rate of return on common stock in the country/region/industry/sector being


analyzed.

3. The expected growth rate of dividends for the stocks in the country/region/industry/sector being
analyzed.

Once values for each of these components have been determined, they are imputed into the
following formula: {P/E = [D/E / (k - g)]}. Where: P/E = the earnings multiplier, or Price-to-Earnings
ratio, D/E = thedividend payout ratio at t1, k = the required rate of return, and g = the anticipated
growth rate of dividends.
In this example all of the necessary information has been provided. However, the dividend payout
ratio must be calculated based on the anticipated dividend at t1 and the projected EPS figure for t1.
The calculation of the dividend payout ratio is as follows: {D/E = [$2.30 / $4.85] = 0.474227}. Now
that the dividend payout ratio has been determined, the appropriate earnings multiplier is found as
follows: {P/E =
[0.474227/ (0.14 - 0.10)] = 11.86. Notice that it is the anticipated growth rate of dividends, not the
anticipated growth rate of earnings, which is used in the determination of the earnings multiplier.

Question: 3289

Freelancers.com, a publishing agency, has an issue of preferred stock outstanding. The preferred
stock of Freelancers pays a semiannual dividend of $0.40, and this dividend is not expected to grow
in the foreseeable future. Similar investments are currently warranting a 12.5% per year required
rate of return.

What is the value of Freelancers' preferred stock? Further, is this preferred stock valued as a
perpetuity or as a finite series of cash flows?

A. $6.40; finite series of cash flows


B. $3.20; perpetuity
C. None of these answers is completely correct.
D. $6.40; perpetuity
E. The answer cannot completely be calculated from the information provided.
F. $3.20; finite series of cash flows

Answer: D

Explanation:
Preferred stock is commonly valued as a perpetuity using the following equation: {P0 = [d1 / k]}

Where: P0 = the price of the preferred stock at time 0, d1 = the annual dividend at t1, and k = the
required rate of return.

In this example, the dividend is provided as a semiannual figure, which must be doubled to show the
annual dividend. After this adjustment has been made, the value of the preferred stock can be found
as follows:

Page | 1288
{P0 = [$0.80 / 0.125] = $6.40}

Preferred stock is commonly valued as a perpetuity because there is no finite conclusion to the
projected series of cash flows for a preferred stock. Unlike a bond, whose cash flows are
characterized by a finite lifespan (i.e. the cash flows of a bond cease at maturity), the cash flows
(dividends) produced by a preferred stock could theoretically last forever.

Question: 3290

Based on the P/E equation, there is ________ relationship between the payout ratio and the P/E
ratio.

A. no
B. a negative
C. a positive

Answer: C

Explanation:
P/E ratio = (Payout ratio)/(k - g); i.e. the relationship is positive.

Question: 3291

Which is not a correct stage in a 5-stage industry life cycle?

A. pioneering development
B. rapid accelerating growth
C. rapid decline and market exit
D. mature growth
E. deceleration of growth and decline

Answer: C

Explanation:
Stabilization and market maturity would be the other correct stage.

Question: 3292

An analyst with Smith, Kleen & Beetchnutty Securities has been working on a determination of EPS
for a fiber optics index. In his research, this analyst has determined the following:

1. Regressing sales for the series against Nominal GDP, the sales figure for the index has been
estimated at: $16.50 per share.

2. Analyzing capacity utilization rates, foreign competition, rates of inflation and unit labor costs, the
operating profit margin for the series has been determined to be 21.00%.

3. Creating a time series based upon inputs such as levels of capital expenditures and PP&E turnover,
next year's depreciation-per-share has been determined to be: $2.45.

4. Creating a time series based upon levels of debt outstanding and prevailing debt yields, the
interest expense for next year is determined to be: $0.45 per share.

5. Coordinating his research with a legislative consultant, the corporate tax rate for this series has

Page | 1289
been estimated at: 36%.

Using this information, what is the EPS figure for this stock market series?

A. $3.50
B. $2.81
C. The answer cannot be determined from the information provided.
D. $0.20
E. None of these answers is correct.
F. $0.36

Answer: F

Explanation:
All of the necessary information has been provided in this example. To determine the EPS for a stock
market series, the following steps are necessary:

Step 1: Estimate sales-per-share for the series:


Step 2: Estimate operating profit margin for the series
Step 3: Estimate the depreciation-per-share for next year
Step 4: Estimate the interest-expense-per-share for the next year
Step 5: Estimate next year's corporate tax rate

Once these five steps have been completed, the calculation of EPS for a stock market series is found
by the following:

EPS = [(Sales per share * Operating profit margin) - Depreciation-per-share - Interest Expense] * (1 -
Corporate Tax Rate)

The calculation of EPS for this stock market series is shown as follows:

EPS = [($16.50 * 0.21) - $2.45 - $0.45] * (1 - 0.36) = $0.3616

Question: 3293

Assume the following series of transactions:

t0: Purchase 150 Intelligent Semiconductor common stock at $90.60 per share t1: Receive a $4.45
per share dividend on 150 shares
t1: Purchase an additional 100 shares at $100 per share t2: Receive $4.50 per share dividend on 250
shares
t2: Purchase an additional 50 shares at $105 per share t3: Receive a $4.55 per share dividend on 300
shares t3: Sell 300 shares for $107.84 per share
Assuming no taxes or commissions and that each dividend was not reinvested, what is the dollar-
weighted rate of return for this series of transactions?

A. None of these answers is correct.


B. The answer cannot be calculated from the information provided.
C. 9.73%
D. 11.18%
E. 8.18%
F. 9.96%

Answer: C

Page | 1290
Explanation:
Remember that the dollar-weighted rate of return uses the IRR equation in the determination of the
answer. Further, the dollar-weighted rate of return is simply another name for the IRR equation, and
this nomenclature is commonly used within the field of investment management. The reason behind
this classification is the fact that the IRR equation takes into account both the timing and scope of all
project cash flows. In the determination of the dollar-weighted rate of return calculation, the first
step should be to identify the cash flows for each period. This process is illustrated as follows:
t0: -[150 shares purchased * $90.60 per share] = [$13,590]
t1: -[100 shares purchased * $100 per share] + [$4.45 per share dividend * 150 shares] = [$9,332.50]
t2: -[50 shares purchased * $105 per share] + [$4.50 per share dividend * 250 shares]} = [$4,125] t3:
[300 shares sold * $107.84 per share] + [$4.55 per dividend * 300 shares] = $33,717
Now that the cash flows have been determined, incorporating this information into your calculator's
cash flow worksheet and solving for IRR will yield a dollar-weighted rate of return of 9.73% for this
investment.

Question: 3294

Assume the following information about a publicly traded regional bank:

Revenue: $25,000,000
Cash flow: $6,500,000
Total Assets: $68,000,000
Total Liabilities: $53,000,000
Number of common shares outstanding: 2,000,000
Current stock price: $16.75 per share

Using this information, what are the price-to-sales, price-to-book, and price-to-cash flow ratios,
respectively?

A. 0.75, 2.23, 5.15


B. None of these answers is correct.
C. 1.34, 0.45, 5.15
D. 1.34, 2.23, 0.19
E. 0.75, 0.45, 5.15

Answer: B

Explanation:
To calculate the price-to-sales ratio, divide the market price of a common stock by its sales-per share
figure. The equation for the price-to-sales ratio is as follows:

Price-to-sales ratio = [P0 / sales per share].


Incorporating the given information into this equation will yield the following: Price-to-sales ratio =
[$16.75 / ($25,000,000 / 2,000,000)] = 1.34
The calculation of the price-to-book ratio involves dividing the market price of a common stock by its
net worth per share. The equation for the price-to-book ratio is as follows:

Price-to-book ratio = [P0 / net worth per share].

Where: net worth per share = (total assets - total liabilities) / number of common shares outstanding.

In this example, the net-worth-per-share figure must be calculated manually. This process is
illustrated as follows:

Page | 1291
Net worth per-share = [($68,000,000 - $53,000,000) / 2,000,000} = 7.5

Now that the net-worth-per-share figure has been determined, the price-to-book ratio can be
calculated. This process is shown below:

Price-to-book ratio = ($16.75 / $7.5) = 2.23

The calculation of the price-to-cash flow ratio involves dividing the market price of a common stock
by the cash-flow-per-share figure. The calculation of the price-to-cash flow ratio is as follows:

Price-to-cash flow = (P0 / cash flow per share)


Incorporating the given information into this equation will yield the following: Price-to-cash flow =
[$16.75 / ($6,500,000 / 2,000,000)] = 5.15385

Question: 3295

Assume the following information about a publicly traded pharmaceutical firm:

Revenue: $25,000,000
Cash flow: $8,750,000
Net worth per share: $12.97
Number of common shares outstanding: 1,750,000
Current stock price per share: $41.32

Using this information, what are the price-to-sales, price-to-book, and price-to-cash flow ratios,
respectively?

A. The answer cannot be completely calculated from the information provided.


B. 0.35, 0.31, 0.024
C. 2.89, 3.19, 8.44
D. 0.35, 4.62, 0.024
E. 2.89, 3.19, 8.26

Answer: E

Explanation:
To calculate the price-to-sales ratio, divide the market price of a common stock by its sales-per share
figure. The equation for the price-to-sales ratio is as follows:

Price-to-sales ratio = [P0 / sales per share].


Incorporating the given information into this equation will yield the following: Price-to-sales ratio =
[$41.32 / ($25,000,000 / 1,750,000)] = 2.8924
The calculation of the price-to-book ratio involves dividing the market price of a common stock by its
net worth per share. The equation for the price-to-book ratio is as follows:

Price-to-book ratio = [P0 / net worth per share].

Where: net worth per share = (total assets - total liabilities) / # of common shares outstanding.

In this example, the net worth figure has been converted to a per-share basis, and the calculation of
the price-to-book ratio is straightforward:

Price-to-book ratio = ($41.32 / $12.97) = 3.18581

Page | 1292
The calculation of the price-to-cash flow ratio involves dividing the market price of a common stock
by the cash-flow-per-share figure. The calculation of the price-to-cash flow ratio is as follows:

Price-to-cash flow = (P0 / cash flow per share)


In this example, the cash flow figure must be converted to a per-share basis, as follows: Cash-flow-
per-share = ($8,750,000 / 1,750,000) = $5.00
Now that the cash-flow-per-share has been determined, the price-to-cash-flow ratio can be
calculated. Incorporating the given information into this equation will yield the following:

Price-to-cash flow = [$41.32 / $5} = 8.264

Question: 3296

A stock's expected return is estimated by estimating its future value. These future values are derived
by:

A. predicting the stock's beta


B. predicting the stock's earnings per share and dividend-payout ratio
C. predicting the stock's earnings per share and expected earnings multiplier
D. predicting the stock's earnings per share and rate of growth

Answer: C

Explanation:
A stock's expected return is estimated by estimating its future value. These future values are derived
by predicting the stock's earnings per share and expected earnings multiplier. The earnings per share
is a function of the sales forecast and the estimated profit margin while the earnings multiplier is a
function of the estimated P/E ratio (based on industry and market comparisons) or the estimated
dividend-payout ratio, the required rate of return and the rate of growth.

Question: 3297

Assume the following information about a manufacturing company.

Sales / total assets = 0.85


Net income / sales = 0.14
Total assets / common equity = 2.2
Dividend payout ratio = 0.50

What is the expected annual growth rate of this firm's dividends?

A. 6.55%
B. The answer cannot be determined from the information provided.
C. 18.07%
D. 13.09%
E. 12.15%

Answer: D

Explanation:
A popular model for determining the growth rate of dividends is the following: g = RR * ROE
Where: g = the expected growth rate of dividends, RR = the retention rate (this is equal to 1 -
dividend payout ratio), and ROE = the return on equity.

Page | 1293
Although it may at first appear otherwise, all of the necessary information has been provided.
Remember the Du Pont decomposition process for ROE, which breaks down the ROE figure into the
following:

ROE = (Net Income / Sales) * (Sales / Total Assets) * (Total Assets * Common Equity)
Mathematically, this will break down into (Net Income / Common Equity), the ROE figure. The
calculation of the return on equity for this company is as follows:

ROE = [0.14 * 0.85 * 2.2] = 0.2618, or 26.18%.

Now that the ROE figure has been determined, the calculation of the growth rate of dividends is as
follows:

g = [(1 - 0.50) * 0.2618] = 13.09%

Question: 3298

The value of an asset is the ________ of its expected future cash flows.

That answer is correct!


A. none of these answers
B. weighted sum
C. future value
D. sum

Answer: A

Explanation:
The value of an asset is the net present value or NPV of its expected future cash flows.

Question: 3299

Technical analysts would feel that an upside-downside volume ratio with a value of 1.7 indicates that
the market is

A. breaching a resistance level.


B. breaching a support level.
C. is overbought.
D. entering a significant market upswing.
E. is oversold.

Answer: C

Explanation:
Technical analysts may use the ratio of upside-downside volume as an indicator of short-term
momentum for the market. They feel that a ratio value of 1.50 or more indicates that the market is
overbought, while a ratio of 0.70 or less indicates that the market is oversold.

Question: 3300

Assume the following information about a publicly traded specialty retailer:

Revenue: $6,500,000
Cash flow: $1,500,000

Page | 1294
Net worth per share: $10.87
Number of common shares outstanding: 1,000,000
Current stock price per share: $28.37

Using this information, what are the price-to-sales, price-to-book, and price-to-cash flow ratios,
respectively?

A. 4.36, 0.38, 18.91


B. 4.36, 2.61, 18.91
C. None of these answers is correct.
D. The answer cannot be completely calculated from the information provided.
E. 0.60, 2.61, 18.91
F. 4.36, 2.61, 42.56

Answer: B

Explanation:
To calculate the price-to-sales ratio, divide the market price of a common stock by its sales-per share
figure. The equation for the price-to-sales ratio is as follows:

Price-to-sales ratio = [P0 / sales per share].


Incorporating the given information into this equation will yield the following: Price-to-sales ratio =
[$28.37 / ($6,500,000 / 1,000,000)] = 4.364
The calculation of the price-to-book ratio involves dividing the market price of a common stock by its
net worth per share. The equation for the price-to-book ratio is as follows:

Price-to-book ratio = [P0 / net worth per share].

Where: net worth per share = (total assets - total liabilities) / # of common shares outstanding.

In this example, the net worth figure has been converted to a per-share basis, and the calculation of
the price-to-book ratio is straightforward:

Price-to-book ratio = ($28.37 / $10.87) = 2.6099

The calculation of the price-to-cash flow ratio involves dividing the market price of a common stock
by the cash-flow-per-share figure. The calculation of the price-to-cash flow ratio is as follows:

Price-to-cash flow = (P0 / cash flow per share)


Incorporating the given information into this equation will yield the following: Price-to-cash flow =
[$28.37 / ($1,500,000 / 1,000,000)] = 18.91

Question: 3301

A stock has a beta of 0.27 and the risk-free rate is 6.15%. Its dividend growth rate is 4.11% and its P/E
ratio is 8.9. If the firm has a dividend payout ratio of 23%, the market risk premium equals ________.

A. 13.65%
B. 10.23%
C. 2.00%
D. 11.82%

Answer: C

Page | 1295
Explanation:
If the expected return on the stock is K, then the Dividend Discount Model gives
P/E = (dividend payout ratio)/(K - g). Therefore, K = g + (div. payout ratio)/(P/E) = 0.0411 + 0.23/8.9 =
6.69%. Now, CAPM implies that the expected return on a stock equals the risk-free rate plus beta
times the market premium. Hence, 6.69% = 6.15% + 0.27 * market premium, giving market premium
= (6.69% - 6.15%)/0.27 = 2.00%.

Question: 3302

A stock's P/E ratio is 6.4, with an expected return of 8% and a dividend growth rate of 4%. The firm's
earnings retention ratio equals ________.

A. 42.8%
B. 74.4%
C. 25.6%
D. 31.7%

Answer: B

Explanation:
In standard notation, the Dividend Discount Model gives P/E = (dividend payout ratio)/(K - g).
Therefore, dividend payout ratio = 6.4*(8% - 4%) = 25.6%. The earnings retention ratio equals 1
minus the dividend payout ratio. In this case, the earnings retention ratio equals 1 - 25.6% = 74.4%.

Question: 3303

An investor should purchase a stock if

That answer is correct!


A. its estimated value is greater than its market value.
B. the company that issued it is expected to be the industry leader.
C. its estimated value is greater than its book value.
D. the company that issued it is one of the best performing firms in one of the best performing
industries in the economy.

Answer: A

Explanation:
Book value has little relevance to purchasing decisions. A company that is expected to perform well
may have a stock that is overvalued because of the very positive attention surrounding it. Instead, an
investor should purchase a stock if its estimated value is greater than its market value. This implies
that the market is undervaluing the stock, the investor can buy it for less than it is worth, and receive
excess returns as the market eventually realizes its mistake.

Question: 3304

Holding other things constant, an increase in a firm's ROE will

A. have no effect on the firm's expected growth rate.


B. decrease the earnings multiplier.
C. increase the earnings multiplier.
D. decrease the firm's expected growth rate.

Answer: C

Page | 1296
Explanation:
An increase in a firm's return on equity (ROE) will increase its expected growth rate of earnings and
dividends. This, in turn, will increase the earnings multiplier.

Question: 3305

A portfolio manager is attempting to determine the earnings multiple for an index of technology
companies, and has gathered the following information:

D1: $0.20
EPS: $1.44
k: 28% per year
g: 26.5% per year

Using the information provided, what is the price-to-earnings ratio for this technology index?
Further, is this earnings multiple realistic assuming that the demand for technology is great, earnings
visibility is clear, and the industry is expected to grow rapidly? To solve for P/E, manipulate the
infinite period dividend discount model.

A. 13.33, this multiple is unrealistic


B. 9.26, this multiple is realistic
C. 9.26, this multiple is unrealistic
D. 480, this multiple is likely to high
E. None of these answers is correct
F. 13.33, this multiple is realistic

Answer: C

Explanation:
To determine the earnings multiplier, or "P/E ratio," of a stock market series, use the following
equation: P/E = [(D1 / E1) / (k-g)
Where: D1 = the annual per-share dividend at t1, E1 = the EPS figure at t1,k = the required rate of
return on common stock, and g = the expected growth rate of dividends.

In this example, all of the necessary information has been provided, and putting it into the equation
above will yield the following:

P/E of a stock market series = [($0.20 *1.265/ ($1.44*1.265)) / (0.28 - 0.265)] = 9.26

This is a rather low multiple, appropriate only for slow growth industries or specific situations such as
industries with high risk or uncertainty. The fact that the index under examination is a compilation of
firms in the high tech business, where demand is expected to be great, does not logically lead to this
low of an earnings multiple. The fact that earnings visibility is clear augments this. The reasoning
behind this is the fact that firms in the technology sector are expected to grow very rapidly, i.e. they
should merit higher earnings multiples. Think of a P/E ratio as a proxy for future earnings growth. In
this situation, a multiple of less than 10 seems unrealistic.

Question: 3306

A portfolio manager with Churn Brothers Brokerage Company is asked to provide an a registered
representative with a figure for the "risk-free interest rate." The portfolio manager references the
quoted rate on U.S. Treasury 10-year notes, currently at 4.75% per year, and uses this rate as the risk-
free interest rate.

Page | 1297
Which of the following best describes the rate referenced by this portfolio manager? Further, what
two components comprise this rate?

A. Real interest rate; inflation premium, real inflation-free rate of return


B. Nominal interest rate; risk premium, real inflation-free rate of return
C. Nominal interest rate; inflation premium, real inflation-free rate of return
D. Quoted intrinsic rate, inflation premium, real inflation-free rate of return
E. Quoted interest rate; nominal interest rate, inflation premium
F. None of these answers is correct.

Answer: C

Explanation:
The risk-free rate as measured by the rate on U.S. Treasury securities often referred to as the
"nominal," or "quoted," rate. This rate is comprised of two components, the real "inflation-free" rate
of interest, and an inflation premium that is equal to the anticipated rate of inflation. The equation
for the calculation of the nominal interest rate is as follows:

Risk-free rate of return = k* + IP

where: k* = the real inflation-free rate of return and IP = the inflation premium

An increase in anticipated inflation will cause a change in the nominal risk-free rate equal to the
change in expected inflation. For instance, if the inflation premium increases by 100 basis points,
then the nominal risk-free interest rate will increase by 100 basis points. Further, if the inflation-free
rate of interest experiences a change, that change will be identically mirrored in the nominal risk-
free interest rate. For example, suppose the inflation-free rate of interest decreases by 50 basis
points. In this situation, the
result would be a 50 basis point reduction in the nominal risk-free rate of interest.

When either the real "inflation-free" interest rate or the expected inflation rate are significantly
large, the calculation of the nominal risk-free rate differs from the equation used when these factors
are significantly small. Specifically, the calculation of the nominal risk-free rate of interest when
theinflation-free rate of interest and/or the inflation premium are significantly high, the calculation
of the nominal risk-free rate is as follows:

Nominal RFR = (1 + Real RFR)(1 + E(I)) - 1

Where: Real RFR = the real inflation-free rate of interest and E(I) = the anticipated inflation rate.

Question: 3307

A technical analyst with Bullfighter.com, a noted investment research firm, has been examining the
U.S. securities markets, and believes that the market is technically "oversold." Which of the following
technical indicators would this analyst likely use to support his opinion? Choose the best answer.

A. The CBOE Put/Call ratio is greater than 50%.


B. The Confidence Index has decreased substantially.
C. The ratio of short sales by specialists is at 30%.
D. The % of issues trading below their 200 day moving average is greater than 80%.
E. The diffusion index has increased substantially.
F. All of these are indicative of an oversold condition.

Answer: D

Page | 1298
Explanation:
The % of issues trading below their 200-day moving average is frequently cited by technical analysts
as a measure of oversold and overbought market conditions. Specifically, technical analysts see the
market as "oversold" when 80% of issues are trading below their 200-day moving average, and
consider a market "overbought" when 80% of issues are trading above their 200-day moving
average.
The "Diffusion Index" is a measure of market breadth, and is defined as [(# of advancing issues + 1/2
# of issues unchanged) / # of issues traded]. The Confidence Index is used as a measure of
international sentiment, and a decline in this index is indicative of widespread investor uncertainty.
The CBOE Put/Call Ratio is a contrarian technical indicator used to gauge the sentiment of
investment professionals, and a ratio greater than 50% would be viewed by contrarian technical
analysts as bullish.

Question: 3308

Assume the following series of financial transactions:

t0: Purchase 1500 shares of Allcycles.com for $18,555.00


t1: Purchase an additional 500 shares for $8,130.00
t2: Purchase an additional 1000 shares for $18,000.00
t3: Sell 1000 shares for $19,810
t4: Sell the remaining 2000 shares for $42,400

Investments of this type have typically justified a 12.50% rate of return. Assuming no taxes or
transaction costs, what is the dollar-weighted rate of return for this series of investments?

A. 11.15%
B. 14.60%
C. 12.89%
D. The answer cannot be calculated from the information provided.
E. None of these answers is correct.
F. 12.32%

Answer: C

Explanation:
Remember that the dollar-weighted rate of return uses the IRR equation in the determination of its
answer. Further, the dollar-weighted rate of return is another name for the IRR equation, and this
nomenclature is commonly used within the field of investment management. So said, the required
rate of return is not explicitly factored into the calculation of the dollar-weighted rate of return;
rather what is being determined is the rate which equates the present value of the cash inflows to
the present value of the cash outflows. In the determination of the dollar-weighted rate of return
calculation, the first step should be to identify the cash flows for each period. In this example, the
cash flows have been stated, and no preliminary calculation is necessary. The portfolio cash flows are
illustrated as follows:

t0: ($18,555.00)
t1: ($8,130.00)
t2: ($18,000.00)
t3: $19,810.00
t4: $42,400.00

Now that the cash flows have been determined, incorporating this information into your calculator's

Page | 1299
cash flow worksheet and solving for IRR will yield a dollar-weighted rate of return of 12.885% for this
investment

Question: 3309

Which of the following is/are true about the confidence index?

I. It measures the spread between high-grade bonds and a broad cross-section of bonds. II. It
indicates investor confidence and their appetite for risk.
III. A high value of the confidence index is interpreted by technicians as a bullish indicator.

That answer is correct!


A. I, II & III
B. II only
C. II & III
D. I & III
E. III only
F. I only

Answer: A

Explanation:
The Confidence index is the ratio of the average yield on 10 top grade bonds to the Dow Jones
average of 40 bonds. The index measures the difference in yield spreads between high-grade bonds
and a large cross-section of available bonds. An increase in this index implies that the yield spread
between the bonds has narrowed. From a demand perspective, this will happen when the demand
for riskier bonds rises i.e. when investors are bullish. Similarly, a fall in the index value indicates that
investors are bearish.

Question: 3310

Mermen, Inc., a manufacturer of male swimwear, has a return on equity of about 7.2%. It typically
pays out about 33% of its earnings as dividends. The firm's stock has a covariance with the market of
0.045. The market has an expected return of 12.9% and a standard deviation of 19.4%. The prevailing
risk-free rate is 4.9% and Mermen's stock return has a standard deviation of 23%. Most analysts in
the market are of the opinion that Mermen's EPS next year is likely to be around $9.20 per share.
Given these data,
Mermen's share price should be about:

A. $41.62
B. $29.28
C. $37.22
D. $38.94

Answer: C

Explanation:
The CAPM expected rate of return on the stock is equal to the risk-free rate plus beta times the
market premium. The beta of the stock equals 0.045/(0.194 * 0.23) = 1.01. So the expected return on
Mermen's stock is 4.9% + 1.01*(12.9 - 4.9)% = 12.98%.
The Dividend Discount Model implies that Mermen's share price is given by Po = D1/(k-g), using
standard notation. We have D1 = 0.33 * 9.2 = $3.036. The dividend growth rate is given by g =
ROE*(1-dividend payout ratio) = 7.2% * 0.67 = 4.824%. The price of the stock then equals
3.036/(12.98% - 4.824%) = $37.22.

Page | 1300
Question: 3311

In a research report, a securities analyst with Smith, Kleen & Beetchnutty claims that the newly
issued perpetual preferred stock of Ludicrous Telecom should be purchased because its current
market price does not reflect its "intrinsic value." The analyst cites a higher valuation as evidenced by
the results produced by the perpetuity valuation model. Assume the following information:

Market price of Ludicrous Telecom preferred stock: $20.75


Quarterly preferred dividend: $0.80
Expected return on the market: 14.75% per year
Risk-free rate of return: 5.00% per year

Given this information, are the claims of the analyst justified? If not, at what price is the preferred
stock of Ludicrous Telecom fairly valued?

A. None of these answers is correct.


B. $4.05
C. The answer cannot be calculated from the information provided.
D. $21.70
E. $16.00
F. $5.42

Answer: C

Explanation:
Assuming that the quarterly dividend is to remain unchanged forever allows us to use the standard
perpetuity model, which is illustrated as follows:

Value of preferred stock = {Annual dividend / required rate of return}

However, in this example, the required rate of return on preferred stock is not provided, and thus the
answer cannot be calculated.

Question: 3312

Which of the following is not involved in determining a company's profit margin?

A. Identifying and evaluating the firm's competitive strategy (low-cost-high-volume, differentiation).


B. Evaluating the firm's dependency on the overall conditions prevailing in its industry.
C. Identifying the firm's internal performance like past trends and potential future problems.
D. Identifying and evaluating the risks involved in the company's cash flows.

Answer: D

Explanation:
A company's profit margin is determined by many factors like its competitive strategy, the degree of
monopoly it enjoys, the relative dependency on external factors versus internally generated
performance, etc. However, while the risks involved in the cash flows are relevant considerations in
determining the total level of earnings of a firm, they do not affect the percentage of the sales which
translate into profits i.e. the profit margin. Of course, higher risks can and do lower the return on
capital.

Question: 3313

Page | 1301
Changes in unit labor costs

A. are the results of three factors: changes in wages per hour, changes in hours worked, and changes
in worker productivity. Increased wages and hours worked increase unit labor costs, while increased
productivity decreases unit labor costs. One would expect a logarithmically negative relationship
between unit labor costs and the aggregate profit margin.
B. are the results of three factors: changes in wages per hour, changes in hours worked, and changes
in worker productivity. Increased wages, hours worked, and productivity all increase unit labor costs.
One would expect a bell-shaped relationship between unit labor costs and the aggregate profit
margin.
C. are the results of two factors: changes in wages per hour, and changes in worker productivity.
Increased wages increase unit labor costs, while increased productivity decreases unit labor costs.
One would expect a negative relationship between unit labor costs and the aggregate profit margin.
D. are the results of two factors: changes in wages per hour, and changes in worker productivity.
Increased wages increase unit labor costs, while increased productivity decreases unit labor costs.
One would expect a negative relationship between unit labor costs and the aggregate profit margin.

Answer: C

Explanation:
Increases in the hourly wage increase unit labor costs, while productivity growth decreases unit labor
costs. Because unit labor costs are a major variable cost of a firm, one would expect a negative
relationship between unit labor costs and the aggregate profit margin.

Question: 3314

Given that next period's dividend is 4, the required rate of return is 14%, and the dividend growth
rate is 5%, what is the current value of the common stock (using the infinite period Dividend
Discount Model)?

A. $28.57
B. Not enough information
C. $44.44
D. $31.50
E. $29.84

Answer: C

Explanation:
The infinite period Dividend Discount Model postulates that the current value of a common stock is
equal to D1 / (k - g), where D1 is next period's dividend, k is the required rate of return, and g is the
growth rate of dividends. In this question, the common stock is worth 4 / (0.14 - 0.05) = $44.44.

Question: 3315

A value investor is examining shares of Clay Industries common stock for possible investor. This
investor has a history of investing in companies trading below their "intrinsic value" and believes that
ClayIndustries represents such a situation. In her research, this value investor has gathered the
following information about Clay Industries:

Total assets: $150,000,000


Total liabilities: $119,000,000
Number of common shares outstanding: 1,000,000

Page | 1302
Current stock price: $26.43 per share
Required return: 17% per year
Expected growth rate: 14.5% per year
Next dividend: $1.05 per share
Earnings per share: $2.85

Using this information, what is the price-to-book ratio for Clay Industries common stock? Further, are
the beliefs of this value investor justified, assuming that the book value of Clay Industries accurately
illustrates the liquidation value of the firm?

A. 1.17, no
B. 9.27, yes
C. The answer cannot be calculated from the information provided.
D. 0.85, no
E. 9.27, no
F. 0.85, yes

Answer: F

Explanation:
The book value of a common stock is found by dividing the net worth of a company by the number of
outstanding shares. While there exists several different methods for calculating net worth, the most
simplistic involves subtracting total liabilities from total assets, giving us the shareholder's equity
figure. Investment professionals often see the book value as a "floor" for common stock prices, and
the price-to-book ratio is quite popular in the field of investment management. The calculation of
the price-to-book ratio involves the following equation:

Price-to-book ratio = {Market price of common stock at t0 / [(total assets - total liabilities) / # of
common shares outstanding]}

While the # of common shares outstanding has been provided, the shareholder's equity figure must
be calculated manually. The calculation of the per share shareholders equity is illustrated as follows:

Shareholders equity = [($150,000,000 - $119,000,000) / 1,000,000] = $31.

Now that the shareholders equity has been calculated and transformed into a per-share figure, the
priceto-book value can be calculated as follows:

Price-to-book value = ($26.43 / $31) = 0.8526.

As you can see, the required rate of return, along with the expected growth rate, is not necessary in
the calculation of the price-to-book ratio.

Any time the price-to-book ratio is less than one, a stock is said to be "trading at a discount to book."
Frequently, investment professionals will use the term "intrinsic value" when referring to the book
value, and thus stocks trading below their book value are often referred to as being priced "below
their intrinsic value." Stocks trading below book value are often sought after by value investors, who
believe that these shares have been discounted below their real value.
A figure of more than one for the price-to-book ratio is commonly referred to as "trading at a
premium to book."

While the price-to-book value is a useful tool in the stock selection process, it possesses some
important flaws. One problem with the Price-To-Book Ratio is that one of the terms - Book Value - is
so easily manipulated. Valuation of inventory and real estate are easily adjusted on the books. Stock

Page | 1303
buy-backs and write-offs of exceptional items also deflate Book Value, making high priced stocks
seem overvalued.

Question: 3316

Given the following estimated information, compute the estimated earnings per share.

Net profit margin3.2%


Gross profit margin28.1%
Sales$11.44B
Common shares outstanding246,141M

A. $13.06 per share


B. none of these answers
C. cannot compute with the information given
D. $1.49 per share

Answer: D

Explanation:
Earnings per share in this example equals sales x the estimated net profit margin/common shares
outstanding = (.032 * 11.44)/.246141 = $1.49.

Question: 3317

Advocates of the top-down, three-step approach believe that

A. an underpriced stock can be found from technical analysis.


B. political considerations are not relevant to stock values.
C. the economy and industry have a significant effect on the returns of individual stocks.
D. the economy and industry have little effect on the returns of individual stocks.

Answer: C

Explanation:
Advocates of the top-down, three-step approach stress the importance of economy and industry-
wide conditions on stock values and returns.

Question: 3318

Consider the following annual growth forecasts for a common stock:

Growth in years 1-2 = 50%


Growth in years 3-4 = 25%
Growth after year 4 = 10%

Assuming that the last dividend was $0.45 per share, and the required rate of return is 20% per year,
what is the value of this common stock?

A. $13.81
B. $8.36
C. $19.05
D. $11.15
E. $7.25

Page | 1304
Answer: D

Explanation:
To determine the value of a common stock experiencing temporary supernormal growth, use the
following equation:
{V = {[d0 * (1 + gs)^1] / k} + {[d1 * (1 + gs)^2} + ... {dn * (1 + gs)^n} + {[dn * (1 + gs)^n * (1 + gn] / (k -
g)}/ (1 + k)^n}}

Where: V = the value of common stock at t0, d0 = the dividend at t0, d1 = the dividend at t1, dn = the
dividend at tn, gs = the supernormal rate of growth, gn = the normal rate of growth, n = the time
period "n", and k = the required rate of return.

where

In this example, there is a transitional growth period of two years, during which the growth rate is
expected to grow at 25% annually. This period will follow the two-year supernormal growth period,
and would be denoted as "g subset t" if we were to reproduce the equation illustrated above. The
calculation of the value of this common stock is illustrated as follows:

{V = {[$0.45 * (1.50)^1] / (1.20)} + {[$0.45 * (1.50)^2] / (1.20)^2} + {[$0.45 * (1.50)^2 * (1.25)^1] /


(1.20)^3} + {[$0.45 * (1.50)^2 * (1.25)^2] / (1.20)^4} + {{[$0.45 * (1.50)^2 * (1.25)^2 * (1.10)^1]/
(0.20 - 0.10)}/ (1.20)^4}

which can be deduced to the following:

{V = [$0.5625 + $0.703125 + $0.732422 + $0.762939 + $8.392334] = $11.15}

Question: 3319

A financial analyst is examining the performance of Intelligent Semiconductor, a diversified


technology company, and has assimilated the following information for the most recent fiscal year:

Adjusted operating profit before tax: $12,675,000


Cash operating taxes: $4,240,000
Cost of capital: 16% per year
Total capital employed: $60,700,000

Using this information, what is the economic value added for Intelligent Semiconductor? Further,
should the management of Intelligent be considered to have provided appropriate economic value
for shareholders?

A. $1,633,620; management has not created economic value


B. $1,633,620; management has created economic value
C. None of these answers is correct.
D. ($1,227,000), management has not created economic value
E. ($497,000), management has created economic value
F. $735,000; management has not created economic value
G. $1,227,000; management has created economic value

Answer: D

Explanation:
Economic Value Added, a value-based measure of economic profit, is a registered trademark of

Page | 1305
Stern, Stewart & Company. The equation used to calculate EVA is as follows:
EVA = {Net Operating Profits Less Adjusted Taxes - [Total Capital Employed * (Cost of Capital)]}.
In this case, the NOPLAT figure must be calculated manually by subtracting the Cash Operating Taxes
from the Adjusted Operating Profit before Tax (AOPBT) figure. Doing so will produce an answer of
$8,435,000 for the Net Operating Profit less Adjusted Taxes figure. Now that the necessary
information has been determined, the calculation of EVA is as follows:

EVA = {$8,435,000 - ($60,700,000 * 0.16)} = ($1,227,000)

A negative EVA calculation indicates that management has failed to provide economic profits to
shareholders, as evidenced by the fact that the opportunity cost of capital employed is more than the
NOPLAT figure. In this example, the management of Intelligent Semiconductor should be considered
as having failed to provide shareholder value.

The calculation of Adjusted Operating Profit before Taxes (AOPBT),and Cash Operating Taxes are
important. The calculation of AOPBT is as follows:
Operating profit (after depreciation and amortization) + Implied interest on operating leases + any
increase in the LIFO reserve + goodwill amortization = Adjusted Operating Profit Before Taxes.

Cash Operating Taxes, another important component of EVA, is calculated as follows:

Cash Operating Taxes = Income Tax Expense + tax benefit from interest expenses + tax benefit from
interest on leases + taxes on non-operating income

Adjusted Operating Profit Before Taxes minus Cash Operating Taxes = Net Operating Profit Less

Adjusted Taxes (NOPLAT). Subtracting the dollar cost of capital from the NOPLAT figure will yield the
EVA.

Question: 3320

Given that the expected dividend payout ratio on a common stock is 0.55, the required rate of return
is 17%, the dividend growth rate is 12%, and next year's earnings are $2.52, using the earnings
multiplier model, what is the estimated value of the stock?

A. $23.95
B. $30.88
C. Not enough information
D. $25.52
E. $27.72

Answer: E

Explanation:
The earnings multiplier model postulates that P/E = (D1/E)/(k -g), where P/E is the price to earnings
ratio, D1 is next year's expected dividends, E is next year's earnings, k is the required rate of return,
and g is the growth rate in dividends. D1/E is also known as the dividend payout ratio. In this
question, the P/E is 0.55 / (0.17 - 0.12) = 11. We can multiply P/E by next year's earnings to arrive at
our expected current stock value ((P/E) x E = P). In this question, the estimated value is 11 x 2.52 =
$27.72.

Question: 3321

The price range at which a technician would expect a substantial increase in the supply of a stock is

Page | 1306
A. the incentive price.
B. the trough price.
C. the support level.
D. the resistance level.

Answer: D

Explanation:
Incentive price is not a real term. Technicians believe that after a stock has fallen from some previous
higher price level, investors who bought at that higher level will wait for the price to come back up
again to unload the stock at their break-even level. When the stock does reach that price range,
selling by those investors will cause a substantial increase in supply.

Question: 3322

A block uptick-downtick ratio of 0.67 would be viewed by technical analysts as

A. a neutral sign.
B. a bearish sign.
C. a bullish sign.
D. a sign that the market is oversold.
E. a sign that the market is overbought.

Answer: B

Explanation:
A block uptick-downtick ratio is equal to the number of upticks for block trades divided by the
number of downticks for block trades in a given period. Upticks are assumed to be initiated by
buyers, while downticks are assumed to be initiated by sellers. Technicians view a low ratio value
(0.70 for example) as a sign of heavy selling and a bearish sentiment on the market. In contrast, they
view a high ratio value (1.2 for example) as a sign of heavy buying and a bullish sentiment on the
market.

Question: 3323

The asset allocation for a country within a ________ portfolio will be affected by its economic
outlook; countries approaching a recession will be ________.

A. local; overweighted
B. global; underweighted
C. local; underweighted
D. global; overweighted

Answer: B

Explanation:
Monetary and fiscal policies enacted by national governments influence the countries' aggregate
economies, which, in turn, influence all industries and companies within the economies. Some
examples are government spending's multiplier effect and the raising of firms' costs due to restrictive
monetary policies. Taking a global perspective, the asset allocation for a country within a global
portfolio will be affected by its economic outlook - countries approaching a recession will be
underweighted, while optimistic economic outlooks would cause an investor to overweight the
country.

Page | 1307
Question: 3324

Which of the following will lead to a decrease in the price of a stock, all else equal?

I. An increase in the systematic risk of the stock.


II. An increase in the total variance of the stock.
III. A decrease in its covariance with the market.
IV. An increase in the growth rate of its dividends.

A. II only
B. I only
C. IV only
D. III only
E. I & III

Answer: B

Explanation:
An increase in the stock's expected rate of return decreases the price of the stock. This will happen if
the stock's systematic risk increases so I is a correct choice. Note that the systematic risk decreases
when the security's covariance with the market decreases. Hence, III is excluded.

An increase in the variance of the stock does not necessarily change its expected return. Only if the
systematic component of the stock's variance changes will the expected return change, causing a
price change. Therefore II is not a correct choice.

Finally, an increase in the growth rate of dividends will increase the stock price, so IV is also not a
correct choice.

Question: 3325

Which of the following is not required to determine the value of a stock that is to be held for one
year?

A. required rate of return


B. expected sale price
C. original purchase price
D. dividends

Answer: C

Explanation:
A stock that is held for one year earns the investor some dividend and will be sold after one year for
some capital gain. Both the expected sale price and the dividend payout during the one-year holding
period need to be discounted to reflect the present value of the stock.

Question: 3326

An independent financial analyst has been investigating shares of Viirius Wavelength, a small fiber
optics firm, for possible investment. This financial analyst has a history of value investing, and
believes that shares of Viirius are not reflecting their "intrinsic value." In her analysis, this financial
analyst has assimilated the following information for Viirius Wavelength:

Page | 1308
Net Worth: $2,000,000
Number of common shares outstanding: 8,900,000
Current stock price: $11.63 per share
Earnings per share: $0.16

Using this information, what is the price-to-book ratio for Viirius Wavelength? Further, what is the
price-to-earnings ratio using trailing earnings figures?

A. 51.75, (72.69)
B. 0.225, 72.69
C. The answer cannot be calculated from the information provided.
D. None of these answers is correct.
E. 0.225, (72.69)
F. 51.75, 72.69

Answer: F

Explanation:
The book value of a common stock is found by dividing the net worth of a company by the number of
outstanding shares. While there exists several different methods for calculating net worth, the most
simplistic involves subtracting total liabilities from total assets, giving us the shareholder's equity
figure. The book value is often seen by investment professionals as a "floor" for common stock prices,
and the price-to-book ratio is quite popular in the field of investment management. The calculation
of the price-to-book ratio involves the following equation:

Price-to-book ratio = {P0 / (Net worth / # of common shares outstanding)}


In this example, all of the necessary information has been provided. Imputing these figures into the
priceto-book value equation will yield the following:

Price-to-book value = {$11.63 / ($2,000,000 / 8,900,000)} = 51.75.

Any time the price-to-book ratio is greater than one, a stock is said to be "trading at a premium to
book."
Frequently, investment professionals will use the term "intrinsic value" when referring to the book
value, as illustrated in this example. A figure of less than one for the price-to-book ratio is commonly
referred to as "trading at a discount to book." Stocks trading below book value are often sought after
by value investors, who believe the shares have been discounted below their liquidation value.
While the price-to-book value is a useful tool in the stock selection process, it possesses some
important flaws. One problem with the Price-To-Book Ratio is that one of the terms - Book Value - is
so easily manipulated. Valuation of inventory and real estate are easily adjusted on the books. Stock
buy-backs and write-offs of exceptional items also deflate Book Value, making high priced stocks
seem overvalued.

Since trailing numbers are to be used (i.e. earnings figures which have already been realized), the
calculation of the price-to-earnings ratio is simple in this case - simply divide the market price by the
EPS figure, which leads to a P/E of 72.69 for Viirius Wavelength.

Question: 3327

The P/E ratio is determined by the expected ________ and by k and g.

That answer is correct!


A. dividend payout ratio
B. stock ratio

Page | 1309
C. earnings ratio
D. g/k ratio

Answer: A

Explanation:
The price/earnings ratio can be computed by dividing the expected dividend payout ratio (dividends
divided by earnings) by the required rate of return (k) minus the expected growth rate of dividends
(g).

Question: 3328

A preferred stock has a dividend payout of $5 every 6 months. Your required rate of return is 20%
annually. What is the value of the preferred stock?

A. $99
B. none of these answers
C. $75
D. $50

Answer: D

Explanation:
First recognize that the appropriate discount rate to use is half of 20% or 10% because the dividend is
paid out twice a year. Next calculate the value of the stock as Dividend/Required rate or $5/0.1 = $50.

Question: 3329

Contrary-opinion technicians would view a high put/call ratio as

A. a bearish sign.
B. a sign of a market peak.
C. a bullish sign.
D. an unimportant statistic.
E. a sign of a flat market.

Answer: C

Explanation:
A high put/call ratio indicates that a relatively large number of investors are betting that the market
will go down by holding put options. Contrary-opinion traders believe that such sentiment is the
opposite of the truth; they view it as a bullish sign.

Question: 3330

A technical analyst with Bullfighter.com, a noted investment research firm, has been examining the
U.S. securities markets, and believes that the market is technically "oversold." Which of the following
technical indicators would this analyst likely use to support his opinion? Choose the best answer.

A. There has been a decline in the Confidence Index.


B. The Block Uptick-Downtick Ratio has declined below 0.70.
C. The % of issues trading above their 200-day moving averages is greater than 80%.
D. The Diffusion Index has increased significantly.
E. The CBOE Put/Call Ratio is at 0.35.

Page | 1310
F. None of these answers is correct.

Answer: B

Explanation:
The Block Uptick-Downtick Ratio is used by technical analysts to gauge institutional investment
activity by measuring the percentage of block trades which result in an uptick versus the block trades
which are executed on a downtick. The idea behind this ratio is the belief that a block buyer would
initiate an "uptick", or a bid up in the securities' price, and a block seller would initiate a "downtick,"
or a bid down in the securities' price. Technical analysts view a decline in the Block Uptick-Downtick
Ratio below 0.70 as
an indication of an oversold condition, and an increase in the Block Uptick-Downtick Ratio above
1.10 as indicative of an overbought condition.
The "Diffusion Index" is a measure of market breadth, and is defined as [(# of advancing issues + 1/2
# of issues unchanged) / # of issues traded]. An increase in the diffusion index is indicative of an
increase in advancing issues relative to declining issues. The CBOE Put/Call Ratio is a contrarian
technical indicator used to gauge the sentiment of investment professionals, and a ratio greater than
50% is viewed by technical analysts as overtly bullish. Finally, contrarian technical analysts would
view a large increase in the amount of futures traders who express bullish sentiment on stock index
futures as a bearish signal.
The % of issues trading below their 200-day moving average is frequently cited by technical analysts
as a measure of oversold and overbought market conditions. Specifically, technical analysts see the
marketas "overbought" when 80% of issues are trading above their 200-day moving average, and
consider a market "oversold" when 80% of issues are trading under their 200-day moving average.

Question: 3331

Porter contends that there are two competitive strategies that dictate how a firm has decided to
cope with the five competitive conditions that define an industry's environment. The two
competitive strategies are known as ________ and ________.

That answer is correct!


A. low-cost strategy, differentiation strategy
B. lay low strategy, differentiation strategy
C. low-cost strategy, defensive leadership
D. low-cost strategy, defensive strategy
E. none of these answers

Answer: A

Explanation:
Within each industry, the strategies available and the ways of implementing them differ.

Question: 3332

In describing competition within industries, five conditions have been identified that could affect the
competitive structure and profits of an industry. Which of the following is not a condition identified
by Reilly & Brown?

A. Current rivalry
B. Potential substitutes
C. Bargaining power of buyers
D. Threat of new entrants
E. Bargaining power of suppliers

Page | 1311
F. Governmental regulations

Answer: F

Explanation:
Once the competitive structure of an industry has been determined, the specific competitive strategy
used by each firm in the industry should be determined and these strategies evaluated in terms of
the overall competitive structure for the industry.

Question: 3333

Assume the following information about a large food distributor.

Retention rate = 0.38


Net income / sales = 0.11
Total assets / common equity = 1.9
Sales / total assets = 0.69

What is the expected annual growth rate of this firm's dividends?

A. 8.94%
B. None of these answers is correct.
C. 5.48%
D. 11.51%
E. 6.14%
F. The answer cannot be determined from the information provided.

Answer: C

Explanation:
A popular model for determining the growth rate of dividends is the following: g = RR * ROE
Where: g = the expected growth rate of dividends, RR = the retention rate (this is equal to 1 -
dividend payout ratio), and ROE = the return on equity.
Although it may at first appear otherwise, all of the necessary information has been provided.
Remember the Du Pont decomposition process for ROE, which breaks down the ROE figure into the
following:

ROE = (Net Income / Sales) * (Sales / Total Assets) * (Total Assets * Common Equity)
Mathematically, this will reduce further into (Net Income / Common Equity)--- the ROE figure. The
calculation of the return on equity for this company is as follows:
ROE = [0.11 * 0.69 * 1.90] = 0.14421, or 14.42%.

Now that the ROE figure has been determined, the calculation of the growth rate of dividends is as
follows:

g = [(0.38) * 0.1442] = 0.054796, or 5.48%

Question: 3334

The ________ Index is the ratio of Barron's average yield on 10 top grade corporate bonds to the
yield on the Dow Jones average of 40 bonds.

A. Dow-Barron
B. Risk

Page | 1312
C. Confidence
D. Bond Rating

Answer: C

Explanation:
By definition, the Confidence Index is the ratio of Barron's average yield on 10 top grade corporate
bonds to the yield on the Dow Jones average of 40 bonds. This index measures the difference in yield
spread between premium grade bonds with a large cross section of bonds.

Question: 3335

The spread between k and g is the primary determinant of the size of the ________.

A. Risk-free rate
B. growth rate
C. firm
D. P/E

Answer: D

Explanation:
The spread between k and g is the primary determinant of the size of the P/E.

Question: 3336

A firm has a dividend payout ratio of 60%, and earns a 10% per year return on its equity. Calculate
the expected annual growth rate of the firm's dividends.

A. 8%
B. Not able to compute with the above data.
C. 7%
D. 4%

Answer: D

Explanation:
g=(RR)(ROE)=0.4*0.10.

Question: 3337

Technical analysts

A. use fundamental company information to find market trends.


B. do not believe in market trends.
C. try to take advantage of ongoing market trends.
D. try to predict future market trends.

Answer: C

Explanation:
Technical analysis, in contrast to the efficient market hypothesis and fundamental analysis, holds that
past price and volume data can be used to discover market trends that can predict future market
behavior. Technical analysts do not try to predict future market trends, but try to find and take

Page | 1313
advantage of current, ongoing ones.

Question: 3338

What is the value of a preferred stock with expected annual dividends of $20, and a required rate of
return of 11%?

A. $158.59
B. $45.32
C. Not enough information
D. $181.82
E. $20

Answer: D

Explanation:
The value of a preferred stock is the present value of its dividends, which is equal to the annual
dividend divided by the required rate of return. In this question, the preferred stock is worth 20/.11 =
$181.82

Question: 3339

A stock has an expected dividend growth rate of 2.4%. The firm has just announced a dividend of
$2.30 per share, with an ex dividend date 3 days from now. Investors expect a rate of return of 12%
from the stock and the stock is trading at $26.12. The stock is:

A. insufficient information.
B. overpriced.
C. underpriced.
D. fairly priced.

Answer: B

Explanation:
In the usual notation, the Dividend Discount Model gives Po = D1/(k-g). In this case, g = 2.4%, D1 =
Do*(1+g) = 2.3 *1.024 = $2.355, Po = $26.12. Therefore,
k = g + D1/Po = 2.4% + 2.355/26.12 = 11.42%. Thus, the rate of return built into the price is less than
that required by the investors, implying that the stock is overpriced. Another way of seeing this is to
find the price that will give k = 12%. This price equals P = 2.355/(12% - 2.4%) = $24.53. Since the stock
is trading at a price higher than this, it is overpriced (by $1.59).

Question: 3340

A resistance level

A. is the price range below the current price at which the technical analyst would expect the stock
supply to increase, pushing down its price below that range. A stock nearing that range would be a
good candidate for short selling because of the negative price support that it would receive.
B. is the price range above the current price at which the technical analyst would expect the stock
supply to increase, abruptly reversing any price increase. A resistance level tends to develop after the
stock has experienced a decline from a higher price level.
C. is the price range above the current price at which the technical analyst would expect the stock
price to get an added boost of demand, pushing its price even higher. If the stock price makes it to
the resistance level, future gains are expected.

Page | 1314
D. is the price range below the current price at which the technical analyst would expect the stock to
get an added boost of demand, keeping its price from falling below that range. The resistance level is
usually near the 12-week low.

Answer: B

Explanation:
Technical analysts believe that investors who have acquired a stock at a higher price will look for an
opportunity to sell it near their break-even points. A resistance level is the target price range that
these investors look for to sell their stock at minimum or no loss. Such a price range tends to develop
after the stock has experienced a decline from a higher price level.

Question: 3341

Which of the following are the two general approaches to the valuation process?

A. The top-down, three-step approach, and the bottom-down, two-step approach


B. The top-down, three-step approach, and the bottom-up stock picking approach
C. The bottom-up, three-step approach, and the Monte Carlo approach
D. The top-down, two-step approach, and the bottom-up, three-step approach

Answer: B

Explanation:
Advocates of the top-down, three-step approach believe that the economy and industry have an
important effect on total returns of individual stocks. Advocates of the bottom-up stock picking
approach believe that it is possible to find stocks that will have superior returns regardless of the
economy and industry.

Question: 3342

If a stock that you are holding for one year has an estimated dividend payout of $2.50 and an
expected sale price of $43, what is the value of the stock?

A. $22
B. $23.10
C. $20.26
D. not enough information to calculate it

Answer: D

Explanation:
To determine the value of a stock held for one year, it is necessary to estimate the dividend to be
received during the period, the expected sale price at the end of the period and its required rate of
return. The required rate of return in this case is not given.

Question: 3343

The second step of the three-step, top-down approach to valuation is to decide to allocate
investment funds

A. among corporations and types of securities.


B. among industries.
C. among industries and types of securities within those industries.

Page | 1315
D. among industries and corporations.

Answer: B

Explanation:
The second step of the three-step, top-down approach to valuation is the analysis of alternative
industries, which attempts only to determine which industries will prosper, and which will fail
(thereby determining investment strategy across industries). The security type decision is made in
the first step, and the decision of individual firm investments is made in the third step.

Question: 3344

What is the value of a stock that is expected to pay a $10 per share dividend in a year's time, and to
be selling for $30 per share at the end of the year? The appropriate discount rate is 10% per year.

A. $40.61
B. $36.36
C. $30.35
D. Not able to compute with the above data.

Answer: B

Explanation:
Value = $40/1.10=$36.36.

Question: 3345

An economist with Smith, Kleen & Beetchnutty Institutional Brokerage has been examining a stock
market series and is trying to determine the anticipated rate of return for the series. In her research,
this economist has determined the following information:

Anticipated ending series value: 11,800


Expected dividends during the period: $521
Observed beginning series value: 10,050.14
Required rate of return: 17.50% per year

What is the anticipated annual rate of return for this stock market series? (Assume a one-year
holding period.)

A. 12.23%
B. 22.60%
C. 24.41%
D. None of these answers is correct.
E. 19.24%

Answer: B

Explanation:
To calculate the expected rate of return for a stock market series, the following information must be
known:

The beginning value for the series

The anticipated ending value for the series, and

Page | 1316
The amount of any dividends and/or distributions during the period

Once this information has been determined, the expected return on a stock market index can be
found by employing the following equation:

E(R) = [(EV - BV + Div) / BV]

Where: E(R) = the expected return on the stock market series, EV = the anticipated ending value for
the series, BV = the observed beginning value for the series, and Div = the amount of any dividends
paid during the period.

In this example, all of the necessary information has been provided and the calculation of the
expected return on this stock market series is found as follows:

E(R) = [$11,800 - $10,050.14 + $521] = 22.60%

This figure is significantly higher than the required rate of return. Assuming that the anticipated
ending value and expected dividends prove accurate, investment in this stock market series is likely
advisable.

Question: 3346

A stock paid an $18 per share dividend this year. Dividends are expected to grow at 5% per year,
forever. What is the value of the stock is the appropriate discount rate is 10% per year?

A. $18.12
B. $37.80
C. Not able to compute with the above data.
D. $378.00

Answer: D

Explanation:
Value = $18.9/(0.10-0.05)=$378.00

Question: 3347

The returns on an investment are based on

A. earnings before taxes and interest.


B. net earnings.
C. dividends.
D. gross earnings.
E. a variety of different forms.

Answer: E

Explanation:
Earnings may not reflect real returns because of accounting methods, and other reasons. Returns
come in a number of different forms, including dividends, interest payments, and capital gains.

Question: 3348

Page | 1317
Assume the following information about a textile manufacturing company. Dividend retention rate =
0.20

Total assets / common equity = 2.01


Net income / sales = 0.09
Sales / total assets = 0.67

What is the expected annual growth rate of this firm's dividends?

That answer is correct!


A. None of these answers is correct.
B. 8.22%
C. 12.12%
D. 5.40%
E. The answer cannot be determined from the information provided.
F. 9.70%

Answer: A

Explanation:
The correct value for the growth rate of dividends for this textile-manufacturing firm is an abysmally
low 2.42%. Thus, none of the answers is correct.
A popular model for determining the growth rate of dividends is the following: g = RR * ROE
Where: g = the expected growth rate of dividends, RR = the retention rate (this is equal to 1 -
dividend payout ratio), and ROE = the return on equity.
Although it may at first appear otherwise, all of the necessary information has been provided.
Remember the Du Pont decomposition process for ROE, which breaks down the ROE figure into the
following:

ROE = (Net Income / Sales) * (Sales / Total Assets) * (Total Assets * Common Equity)

Mathematically, this will break down into (Net Income / Common Equity), the ROE figure. The
calculation of the return on equity for this company is as follows:

ROE = [0.09 * 0.67 * 2.01] = 0.121203, or 12.12%.

Now that the ROE figure has been determined, the calculation of the growth rate of dividends is as
follows:

g = [(0.20) * 0.1212] = 0.02424, or 2.42%

Question: 3349

The price/cash flow ratio is used to supplement the P/E ratio in company analysis because:

I. A firm's cash flow is typically subject to less accounting manipulation than reported earnings,
II. Cash flow has become an important measure of performance, value and financial strength because
numerous academic studies have shown that various cash flow measures can be used to predict both
success and future problems
III. Under theoretically ideal conditions, the market value of a firm should reflect its book value
IV. Cash flow data have become more accessible
V. It is easier to compute

A. II, III IV

Page | 1318
B. II, IV, V
C. All of these answers
D. I, II, III
E. I, II, IV

Answer: E

Explanation:
Developing benchmark values for these ratios is important because, similar to P/Es and P/BV ratios,
they are relative measures of value.

Question: 3350

What is the value of a stock that paid a $10 per share dividend this year, and is expected to pay a
dividend 20% higher in a year's time, the stock is expected to be selling for $40 per share at the end
of the year. The appropriate discount rate is 10% per year?

A. Not able to compute with the above data.


B. $45.49
C. $50.16
D. $47.27

Answer: D

Explanation:
Value = $52/1.10=$47.27

Question: 3351

Which of the following correctly illustrates the infinite period dividend discount model?

That answer is correct!


A. V = D0(1 + g) / (k - g)
B. None of these answers is correct.
C. (1 + g) = D0 * k / D1
D. (1 + k)(1 +g) / V = P0/(k-g)
E. V = {[D0 / (1 + r)] + [D1 / (1 + r)(1 + r)] ...+ [Dn / (k-g)]}
F. V = D1/k

Answer: A

Explanation:
The traditional equation for the infinite period dividend discount model is as follows: Value of
common stock = D1 / (k - g)
Where: D1 = the dividend at t1, k = the required rate of return, and g = the assumed growth rate of
dividends.

Remember that the dividend at t1, which is characterized as "D1" is calculated by multiplying the
dividend at t0 by (1 + growth rate of dividends). This equation will produce logical results regardless
of whether growth or contraction of dividends is expected in the future.

Question: 3352

A diffusion index is equal to

Page | 1319
That answer is correct!
A. the daily total of advancing stocks plus one-half the number of unchanged stocks, divided by the
total number of stocks traded.
B. the daily total of advancing stocks divided by the number of declining stocks.
C. the daily total of advancing stocks plus one-half the number of unchanged stocks, divided by the
number of declining stocks.
D. the daily total of advancing stocks divided by the total number of stocks traded.

Answer: A

Explanation:
The diffusion index is an alternative specification of advance-decline series, and is used as an
indicator of the direction of the market. If the index reaches up to levels of 56 to 60, the market is
considered overbought. If it comes down to levels of 40 to 44, the market is considered over
oversold.

Question: 3353

The assumptions underlying technical and fundamental analysis differ in many respects. Which of
the following is an important difference between technical analysis and fundamental analysis?
Choose the best answer.

A. None of these answers is correct.


B. All of these choices are important differences between technical analysis and fundamental
analysis.
C. Technical analysis assumes that securities "price in" information immediately, whereas
fundamental analysts assume that securities markets "price in" information gradually.
D. Technical analysis assumes that the Weak Form of the Efficient Market Hypothesis is correct,
whereas fundamental analysis assumes that it is incorrect.
E. Technical analysis is not reliant on financial statements whereas fundamental analysis is heavily
reliant on financial statements.

Answer: E

Explanation:
Of all the differences between technical analysis and fundamental analysis, perhaps the most
important is the fact that technical analysis assumes that securities markets are not weak form
efficient, whereas fundamental analysis assumes that the Weak Form of the EMH is correct.
Technical analysts assume that securities markets will "price-in" relevant information gradually, and
that securities prices follow observable trends and patterns. Fundamental analysts, on the other
hand, generally assume that past price information cannot be used to predict movements in
securities prices.

Question: 3354

A portfolio manager is in the process of forecasting an earnings multiplier for the specialty software
industry. In his analysis, this portfolio manager examines the historical payout ratio, required rate of
return, and growth forecasts for the software industry, and then compares these figures to those of
the overall market. Which of the following best characterizes this method of forecasting an industry
earnings multiple?

A. Microsimulation
B. Microanalysis

Page | 1320
C. Monte Carlo simulation
D. Macroanalysis
E. Macrosimulation
F. Porter Method

Answer: B

Explanation:
The method profiled in this example is "microanalysis," which is one of two methods for estimating
the earnings multiplier of an industry. Microanalysis involves examining the variables underlying the
earnings multiplier - the required rate of return, the growth forecast, and the dividend payout ratio.
In microanalysis, these variables are examined for the industry and then compared with the values of
these variables for the entire market.

The microanalysis method is contrasted by the macroanalysis method, which involves examining the
historical relationship between the earnings multiplier of an industry with that of the overall market.
Macroanalysis forecasts often use a time series.

The "Porter Method" is used to examine the level of competition within an industry, and "Monte
Carlo simulation" is a method of measuring stand-alone risk. "Microsimulation," and
"macrosimulation" are largely fictitious terms.

Question: 3355

Marlene Gooseberry, an institutional money manager with Middle Road Brokerage, has been
examining a stock market series and has determined the following information:
The dividend payout ratio has been estimated at: 31% The required rate of return is 16% per year
The anticipated future growth rate of dividends is 13.75% per year The anticipated future growth
rate of earnings is 14.25% per year The corporate tax rate is 35%

What is the earnings multiplier for this stock market series? Choose the best answer.

A. None of these answers is correct.


B. 7.75
C. 13.78
D. 17.71
E. 8.96
F. The answer cannot be determined from the information provided.

Answer: C

Explanation:
Estimating the earnings multiplier for a stock market series requires the estimation of each of the
following components:

The dividend payout ratio.

The required rate of return on common stock in the country/region/industry/sector being analyzed.

The expected growth rate of dividends for the stocks in the country/region/industry/sector being
analyzed.
Once values for each of these components have been determined, they are imputed into the
following formula:

Page | 1321
P/E = [D/E / (k - g)]

Where: P/E = the earnings multiplier, or Price-to-Earnings ratio, D/E = the dividend payout ratio at t1,
k = the required rate of return, and g = the anticipated growth rate of dividends.

In this example, all of the necessary information has been provided, and the calculation of the
earnings multiplier is shown as follows:

P/E = [0.31 / 0.16 - 0.1375] = 13.78

Notice that it was the anticipated growth rate of dividends, not the anticipated growth rate of
earnings, which was used in determining the earnings multiplier. Additionally, note that the tax rate
was notexplicitly factored in to the equation, as the earnings figure used in the dividend payout ratio
is already an after-tax figure.

Question: 3356

When an analyst would like to estimate the earnings-per-share for a stock market series, her analysis
would likely begin with which of the following activities?

A. Analyzing competitive pressures within the series


B. Formulating a composite of depreciation and amortization figures for the series
C. Estimating the profit margin of the series
D. Estimating the tax rate for the series
E. Estimating the sales-per-share of the series

Answer: E

Explanation:
When an analyst is trying to determine the EPS for a stock market series, the following series of steps
represent the preferred method:

Step 1: Estimate sales-per-share for the series


Step 2: Estimate the operating profit margin for the series
Step 3: Estimate the depreciation-per-share for the series for the next year
Step 4: Estimate the interest expense-per-share for the next year
Step 5: Estimate the corporate tax rate for the next year

These steps will lead to an EPS figure, which can be multiplied by the appropriate earnings multiplier
(i.e. the price-to-earnings ratio) to determine the ending price for the stock market series.

Question: 3357

Changes in the unit labor cost are caused by changes in

A. hours worked and worker productivity.


B. wages per hour and worker productivity.
C. hours worked.
D. wages per hour, hours worked, and worker productivity.
E. worker productivity.

Answer: B

Explanation:

Page | 1322
Finkel and Tuttle found that the capacity utilization rate, unit labor costs, rate of inflation, and foreign
competition were the four major variables that affected the aggregate profit margin. Per-unit labor
cost is a function of the percentage change in hourly wages minus the percentage change in
productivity during some period of time.

Question: 3358

If 80% of investment advisory services have bullish attitudes on the stock market, technicians may
view this as

That answer is correct!


A. a bearish sign.
B. a bullish sign.
C. inconsequential.
D. indicative of an unpredictable, volatile market.

Answer: A

Explanation:
Many technicians feel that investment advisory services tend to be trend followers, so that by the
time most services are bullish, the bull market is nearing its peak and will soon be followed by a new
bear market. For this reason, such a sign would be viewed as bearish.

Question: 3359

An economist with Smith, Kleen & Beetchnutty Institutional Brokerage has been examining a stock
market series and is trying to determine an appropriate earnings multiplier for the series. In her
research, this economist has determined the following information:

The annual dividend at t1 = $1.35


The EPS at t1 = $5.10
The anticipated growth rate of dividends is 12.5%
The anticipated growth rate of earnings is 14%
The required rate of return is 15.75%

Using this information, what is the appropriate earnings multiplier for this stock market series?
Further, what is the appropriate value for this stock market series? Choose the best answer.

A. None of these answers is correct.


B. 14.93; $76.13
C. The answer cannot be determined from the information provided.
D. 7.74; $39.46
E. 8.14; $41.51
F. 15.13; $77.16

Answer: E

Explanation:
The appropriate earnings multiplier is found to be 8.14. Estimating the earnings multiplier for a stock
market series requires the estimation of each of the following components:

1. The dividend payout ratio.

2. The required rate of return on common stock in the country/region/industry/sector being

Page | 1323
analyzed.

3. The expected growth rate of dividends for the stocks in the country/region/industry/sector being
analyzed.

Once values for each of these components have been determined, they are imputed into the
following formula: {P/E = [D/E / (k - g)]}. Where: P/E = the earnings multiplier, or Price-to-Earnings
ratio, D/E = the dividend payout ratio at t1, k = the required rate of return, and g = the anticipated
growth rate of dividends.

In this example all of the necessary information has been provided. However, the dividend payout
ratio must be calculated based on the anticipated dividend at t1 and the projected EPS figure for t1.
The calculation of the dividend payout ratio is as follows: {D/E = [$1.35 / $5.10] = 0.264706}. Now
that the dividend payout ratio has been determined, the appropriate earnings multiplier is found as
follows: {P/E =
[0.264706/ (0.1575 - 0.125)] = 8.14. Notice that it is the anticipated growth rate of dividends, not the
anticipated growth rate of earnings, which is used in the determination of the earnings multiplier.
Multiplying this figure by the projected EPS will lead to an appropriate value of $41.51 for this series.

Question: 3360

Which of the following variables has/have an inverse relationship with the P/E ratio?

A. Dividend payout ratio


B. Required rate of return
C. All of these answers
D. Growth rate of dividends

Answer: B

Explanation:
One can remember the formula P/E = (D/E)/(k - g); and note that k, which is the required rate of
return, is in the denominator, i.e. inverse relationship. Or one can remember that a high P/E ratio
means paying a high price for some earnings, i.e. the rate of return required is low.

Question: 3361

A value investor is considering investing in shares of Microscam International, believing that the
market price of Microscam is less than the "intrinsic value" of the shares. In his research, this value
investor has gathered the following information about Microscam International:

Total assets: $200,000,000


Total liabilities: $130,000,000
Number of common shares outstanding: 3,000,000
Current stock price: $22.15 per share
Required return: 18.75% per year
Expected growth rate: 17% per year
Next dividend: $0.30 per share
Earnings per share: $3.88

Using this information, what is the book value of Microscam International? Further, are the beliefs of
this value investor justified? Assume that the book value calculation accurately illustrates the
liquidation value of Microscam.

Page | 1324
A. 5.709, yes
B. 0.949, no
C. 0.949, yes
D. 5.709, no
E. 0.244, yes
F. None of these answers is correct.

Answer: C

Explanation:
The book value of a common stock is found by dividing the net worth of a company by the number of
outstanding shares. While there exists several different methods for calculating net worth, the most
simplistic involves subtracting total liabilities from total assets, giving us the shareholder's equity
figure.
Investment professionals often see the book value as a "floor" for common stock prices, and the
price-to-book ratio is quite popular in the field of investment management. The calculation of the
price-to-book ratio involves the following equation:

Price-to-book ratio = {Market price of common stock at t0 / [(total assets - total liabilities) / # of
common shares outstanding]}
While the # of common shares outstanding has been provided, the shareholder's equity figure must
be calculated manually. The calculation of the per share shareholders equity is illustrated as follows:

Shareholders equity = [($200,000,000 - $130,000,000) / 3,000,000] = $23.33.

Now that the shareholders equity has been calculated and transformed into a per-share figure, the
priceto-book value can be calculated as follows:

Price-to-book value = ($22.15 / $23.33) = 0.94942.

As you can see, the required rate of return, along with the expected growth rate, is not necessary in
the calculation of the price-to-book ratio.

Any time the price-to-book ratio is less than one, a stock is said to be "trading at a discount to book."
Frequently, investment professionals will use the term "intrinsic value" when referring to the book
value, and thus stocks trading below their book value are often referred to as being priced "below
their intrinsic value." Stocks trading below book value are often sought after by value investors, who
believe that these shares have been discounted below their real value.
A figure of more than one for the price-to-book ratio is commonly referred to as "trading at a
premium to book."

While the price-to-book value is a useful tool in the stock selection process, it possesses some
important flaws. One problem with the Price-To-Book Ratio is that one of the terms - Book Value - is
so easily manipulated. Valuation of inventory and real estate are easily adjusted on the books. Stock
buy-backs and write-offs of exceptional items also deflate Book Value, making high priced stocks
seem overvalued.

Question: 3362

Holding other things equal, an increase in net income will

A. decrease the ROE.


B. have no effect on the ROE.
C. increase the earnings multiplier.

Page | 1325
D. have no effect on the earnings multiplier.

Answer: C

Explanation:
An increase in net income increases the return on equity ratio (ROE), which would increase the
growth rate of earnings and dividends. This, in turn, would increase the earnings multiplier.

Question: 3363

A stock has a beta of 1.44 and the market risk premium is 7.4%. Its dividend growth rate is 5% and its
P/E ratio is 4.2. If the firm has a dividend payout ratio of 45%, the risk-free rate equals ________.

That answer is correct!


A. 5.06%
B. 4.64%
C. 5.33%
D. 4.89%

Answer: A

Explanation:
If the expected return on the stock is K, then the Dividend Discount Model gives
P/E = (dividend payout ratio)/(K - g). Therefore, K = g + (div. payout ratio)/(P/E) = 0.05 + 0.45/4.2 =
15.71%. Now, CAPM implies that the expected return on a stock equals the risk-free rate plus beta
times the market premium. Hence, 15.71% = Rf + 1.44 * 7.4%, giving Rf = 5.06%.

Question: 3364

If the debit balances in brokerage accounts decrease, the smart money technicians interpret it as:

That answer is correct!


A. a bearish signal.
B. none of these answers.
C. a bullish signal.
D. a hold signal.

Answer: A

Explanation:
Debit balances in brokerage accounts represent margin debt i.e. borrowings by investors from the
brokerages for investment in the market. Smart money technicians consider margin investors as a
group of astute investors and hence interpret large leveraged investments as a bullish signal.
Therefore, an large increase in the debit balances is considered a bullish sign while a decline is
considered a bearish sign.

Question: 3365

Assume the following information about a publicly traded pharmaceutical firm:

Revenue: $16,000,000
Cash flow: $1,700,000
Net worth per share: $14.55
Number of common shares outstanding: 1,000,000

Page | 1326
Required return: 21% per year
Expected growth rate: 19% per year
Next dividend: $1.05 per share

Using this information, what are the price-to-sales, price-to-book, and price-to-cash flow ratios,
respectively?

That answer is correct!


A. The answer cannot be completely calculated from the information provided.
B. 16, 52.5, 0.0324
C. 0.106, 38.4, 0.0324
D. None of these answers is correct.
E. 16, 52.5, 30.88
F. 0.106, 52.5, 30.88

Answer: A

Explanation:
To calculate the price-to-sales, price-to-book, and price-to-cash flow ratios, it is necessary to know
the price of the common stock in question at t0, which is not provided in this example.

Question: 3366

The NAV of an open-ended fund is $16.88. The fund charges a 7.2% sales charge and no redemption
charges. If you invested a $1,000 with the fund, how many shares would you receive?

A. 55.26
B. 54.98
C. none of these answers
D. 59.24

Answer: C

Explanation:
The offer price i.e. the price at which you can buy a share equals
NAV/(1-load charge) = 16.88/(1-0.072) = $18.19. Therefore, the number of shares you receive equals
1000/18.19 = 54.98
Be careful NOT to calculate the offer price as NAV*(1 + load charge). The sales charge is expressed as
an percentage of the amount you deposit. In this example, you deposit $1,000 and get shares worth
$1,000 * (1-0.072) = $928. Note that the 54.98 shares are worth 54.98 * 16.88 = $928 i.e. this is the
amount you would get if you were to sell the shares immediately.

Question: 3367

The P/E earnings multiplier can be set equal to

A. the required rate of return minus the dividend growth rate, divided by the expected dividend
payout ratio.
B. next year's dividend divided by the spread between the required rate of return and the dividend
growth rate.
C. the spread between the required rate of return and the dividend growth rate, divided by next
year's dividend.
D. the expected dividend payout ratio, divided by the required rate of return minus the dividend
growth rate.

Page | 1327
Answer: D

Explanation:
The earnings multiplier, which is the basis for a method of valuing stocks, is also known as the price /
earnings (P/E) ratio. The infinite period Dividend Discount Model claims that the current price of a
common stock is equal to D1 / (k - g), where D1 is next period's (most often next year's) dividend, k is
the required rate of return, and g is the growth rate of dividends. If we divide both sides of the
infinite period Dividend Discount Model equation by expected earnings during the next 12 months,
we get P/E = (D1/E) / (k - g).

Question: 3368

Jim Williams, a security analyst with Smith, Kleen & Beetchnutty, identifies promising investments by
first evaluating the global macroeconomic environment, then identifying the prospects of individual
regions and companies. Once promising areas have been identified, Mr. Williams then determines
which industries are expected to flourish within these areas. Finally, individual companies are
examined within the aforementioned industries. Which of the following best describes the process
used by Mr. Williams?

A. None of these answers is correct.


B. Macroeconomic Cycle Approach
C. Sensitivity Analysis Approach to security evaluation
D. Top down Approach
E. Bottom up Approach

Answer: D

Explanation:
The approach illustrated within this example is the Top Down, or "Macro" approach. When using this
method, an analyst first begins by evaluating the macroeconomic environment, and then moves on
to specific regions and countries. Next, individual industries are examined in an attempt to
determine their fundamentals. The final step in the Top Down Approach is to examine individual
companies.

The Top Down Approach is contrasted with the Bottom Up Approach, which begins by evaluating the
fundamentals of individual companies.

Question: 3369

________ = Retention Rate x Return on Equity

A. risk free rate, RFR


B. growth, g
C. required rate, k
D. period 1 earnings, E1

Answer: B

Explanation:
By definition: growth, g = retention rate x ROE

Question: 3370

Page | 1328
Given that next period's dividend will be $2.50, the growth rate of dividends is 7%, and the risk
premium on the common stock is 11%, using the infinite period Dividend Discount Model, what is
the current value of the common stock?

A. $38.52
B. Not enough information
C. $62.50
D. $22.73
E. $18.94

Answer: B

Explanation:
The infinite period Dividend Discount Model postulates that the current value of a common stock is
equal to D1 / (k - g), where D1 is next period's dividend, k is the required rate of return, and g is the
growth rate of dividends. In order to solve for the current value, the required rate of return on the
common stock must be known. But only the risk premium is known, which, without the risk-free
rate, cannot be used to derive the required rate of return

Question: 3371

Which of the following are challenges to technical analysis?

I. Technical analysis involves a great deal of subjective judgement.


II. Technical analysis is heavily reliant on financial statements.
III. The majority of studies have supported the Weak Form Efficient Market Hypothesis. IV. The
standard rules that signal investment decisions can change over time.
V. The majority of studies have concluded that securities prices do not move in trends.
VI. Technical analysis assumes that supply-demand fluctuations lead to changes in securities prices.

A. I, III, IV, V, VI
B. I, II, IV, V,
C. I, III, IV, V
D. I, III, IV
E. I, II, III, IV, V, VI

Answer: C

Explanation:
Of the choices listed, only II and VI are not criticisms of technical analysis. While VI appears correct, it
is not. In fact, both fundamental and technical analysis recognize the importance of supply and
demand fluctuations. The difference is that technical analysis assumes that supply and demand are
influenced by both rational and irrational factors, whereas the EMH assumes that investors are
rational. Had choice VI been phrased as "technical analysis assumes that supply-demand
fluctuations, which are caused in part
by irrational forces, are the only determinants of shifts in securities prices," then it would be correct
in this example.

Question: 3372

A market researcher with Churn Brothers Brokerage is attempting to estimate the earnings per share
(EPS) for a broad market index, and has assimilated the following information:

Sales per share: $700

Page | 1329
Next year's operating profit margin: 25%
Next year's depreciation per share: $80
Next year's interest expense: $45.50
Next year's corporate tax rate: 35%
Using this information, what is the EPS figure for this stock market series?

That answer is correct!


A. $32.17
B. $164.18
C. The answer cannot be calculated from the information provided.
D. $46.56
E. None of these answers is correct.
F. $112.67

Answer: A

Explanation:
The estimation of EPS for a stock market series involves five steps. Specifically, to determine an
estimate of EPS for a stock market series, it is necessary to:

Estimate the sales per share


Estimate next year's operating profit (EBIDT), or operating profit margin Estimate next year's
depreciation per share
Estimate next year's interest expense per share
Estimate next year's corporate tax rate

Once estimates for these components have been determined, they are put into the following
equation:

EPS for a stock market series = {[(Sales per share * operating profit margin) - depreciation per share -
interest expense per share] * (1 - corporate tax rate).

Putting the given information into this equation will yield the following:

EPS for a stock market series = {[($700 * 0.25) - $80 - $45.50] * (1 - 0.35)} = $32.17

If you chose $112.67, remember that the depreciation figure is not added back to the EPS figure in
the calculation of EPS for a stock market series. What we are looking at is operating earnings after
tax, not a cash-based figure.

Question: 3373

What is the value of a stock that is expected to pay a $15 per share dividend in a year's time? The
stock is expected to be selling for $40 per share at the end of the year. The appropriate discount rate
is 12% per year.

A. Not able to compute with the above data.


B. $4.91
C. $49.11
D. $55.34

Answer: C

Explanation:

Page | 1330
Value = $55/1.12=$49.11

Question: 3374

Which of the following is not involved in the estimation of the earnings per share (EPS) for a stock
market series?

A. Estimate next year's operating profit.


B. Estimate next year's interest expense.
C. Estimate next year's corporate tax rate.
D. Estimate the sales per share of the series.
E. Estimate next year's depreciation per share.
F. Estimate next year's dividend payout ratio.

Answer: F

Explanation:
While somewhat intuitively appealing, the estimation of next year's dividend payout ratio is not
necessary when determining the anticipated EPS for a stock market series. The other five choices
represent all the necessary steps in the estimation process.

Question: 3375

According to the infinite period dividend discount model, the value of a common stock is equal to

That answer is correct!


A. next period's dividend, divided by the required rate of return minus the growth rate of dividends.
B. this period's dividend, divided by the growth rate of dividends.
C. this period's dividend, divided by the required rate of return minus the growth rate of dividends.
D. next period's earnings, divided by the required rate of return minus the growth rate of dividends.
E. this period's earnings, divided by the required rate of return minus the growth rate of dividends.

Answer: A

Explanation:
With certain assumptions, including a constant rate of dividend growth, the infinite period Dividend
Discount Model can be simplified to D1 / (k - g), where D1 is next period's dividend, k is the required
rate of return, and g is the growth rate of dividends.

Question: 3376

Jones Rutherford, a portfolio manager with Churn Brothers Brokerage, has been examining a stock
market series and is trying to determine an appropriate earnings multiplier for the series. In this
analysis, Jones has amassed the following information:

The next estimated annual dividend per share = $2.30


The estimated earnings per share next year = $4.85
The anticipated growth rate of dividends is 10% per year
The anticipated growth rate of earnings is 9% per year
The required rate of return is 14% per year

Given this information, what is the appropriate earnings multiplier for this stock market series?

A. The answer cannot be determined from the information provided.

Page | 1331
B. None of these answers is correct.
C. 9.48
D. 10.51
E. 13.14
F. 13.04

Answer: B

Explanation:
The appropriate earnings multiplier for this stock market series is found as 11.86, therefore none of
these answers is correct.

Estimating the earnings multiplier for a stock market series requires the estimation of each of the
following components:

The dividend payout ratio.

The required rate of return on common stock in the country/region/industry/sector being analyzed.

The expected growth rate of dividends for the stocks in the country/region/industry/sector being
analyzed.

Once values for each of these components have been determined, they are imputed into the
following formula:

P/E = [D/E / (k - g)]

Where: P/E = the earnings multiplier, or Price-to-Earnings ratio, D/E = the dividend payout ratio at t1,
k = the required rate of return, and g = the anticipated growth rate of dividends.
In this example all of the necessary information has been provided. However, the dividend payout
ratio must be calculated based on the anticipated dividend at t1 and the projected EPS figure for t1.
The calculation of the dividend payout ratio is as follows:

D/E = [$2.30 / $4.85] = 0.474227

Now that the dividend payout ratio has been determined, the appropriate earnings multiplier is
found as follows:

P/E = [0.474227/ (0.14 - 0.10)] = 11.86

Notice that it is the anticipated growth rate of dividends, not the anticipated growth rate of earnings,
which is used in the determination of the earnings multiplier.

Question: 3377

This is the ratio of the price of a stock or an industry index to the value for some stock series.

A. Block Uptick-Downtick Ratio


B. Relative Trend
C. Odd-Lot, Short-Sales Theory
D. Mutual Fund Cash Positions
E. Short Sales by Specialists
F. Diffusion Index
G. Margin Debt

Page | 1332
H. Dow Theory

Answer: B

Explanation:
The relative trend (or relative-strength ratio) is the ratio of the price of a stock or an industry index to
the value for some stock market series like the DJIA or the S&P 400. If this ratio increases over time,
it shows that the stock or industry is outperforming the market and a technician would expect this
superior performance to continue.

Question: 3378

Technical analysts believe that

A. prices adjust instantaneously to new information.


B. prices adjust quickly to new information.
C. prices adjust slowly to new information.
D. prices never adjust to much new information.

Answer: C

Explanation:
Technical analysis, in contrast to the efficient market hypothesis and fundamental analysis, holds that
past price and volume data can be used to discover market trends that can predict future market
behavior. In order for the market to have these trends, prices must adjust slowly to new information,
rather than quickly or instantaneously. Adjustment is slow because of different sources of
information or because certain investors receive information before others.

Question: 3379

The NAV of an open-ended fund is $6.35. The fund charges a 9% sales charge and no redemption
charges. The price at which you can purchase a share of the fund equals ________.

A. none of these answers


B. $6.98
C. $6.92
D. $6.35

Answer: B

Explanation:
The offer price i.e. the price at which you can buy a share equals NAV/(1-load charge) = 6.35/(1-0.09)
= $6.98
Be careful NOT to calculate the offer price as NAV*(1 + load charge). The sales charge is expressed as
an percentage of the amount you deposit. In this example, you deposit $6.98 and get a share worth
$6.35 = $6.98*(1-0.09)

Question: 3380

An analyst with Churn Brothers is examining shares of Microscam International, a multinational


software firm, for possible investment. Shares of Microscam are anticipated to grow at a rate of 20%
per year for the next two years, then grow at 15% in the third year. Thereafter, the growth rate of
Microscam International is expected to be at 11% per year forever. Similar investments have
warranted a 15% per year rate of return, and Microscam paid a dividend of $0.50 at t0.

Page | 1333
Using this information, what is the value of Microscam International common stock? Use the two-
stage dividend discount model.

That answer is correct!


A. $16.72
B. $26.39
C. $21.39
D. $22.59
E. The answer cannot be calculated from the information provided.
F. None of these answers is correct.

Answer: A

Explanation:
The multi-stage dividend discount model is a more realistic way of valuing fast-growing companies
that pay dividends. With this model, it is necessary to estimate the above-average, or "supernormal"
rate of growth, as well as the long-term rate of growth. Once these growth rates have been
determined, they are used to calculate the dividends at various points in the future. In this example,
you are provided with two supernormal growth rates, in addition to the long-term rate of growth,
and the calculation is somewhat more complex than a situation characterized by a single
supernormal growth rate. Nonetheless, the multistage dividend discount model can be applied.

The multi-stage dividend discount model is often referred to as the "two-stage dividend discount
model," and these two titles should be considered interchangeable for all intents and purposes, i.e.
the "two stage" dividend discount model can be used to determine the value of a company that has
multiple growth rate changes.

Incorporating the given information into the multi-stage dividend discount model will yield the
following:

P = {[$0.50 * 1.20) / 1.15] + [($0.60 * 1.20) / 1.3225] + [($0.72 * 1.15) / 1.52087] + [($0.828 * 1.11) /
(.15 -
11)]/1.52087}

which can further be developed into:

P = {$0.52174 + $0.54442 + $0.54442 + $15.11} = $16.72

Question: 3381

If the spread between the required rate of return and the anticipated dividend growth rate were to
decrease significantly and suddenly while the remaining components of the P/E ratio were to remain
unchanged, which of the following would likely occur? Further, a decrease in the retention rate
wouldlead to what effect on the earnings multiplier, holding both the required return and expected
growth rate constant?

A. The earnings multiplier would decrease; the earnings multiplier would decrease.
B. The earnings multiplier would increase; the earnings multiplier would increase.
C. The earnings multiplier would increase; the earnings multiplier would decrease.
D. The earnings multiplier would decrease; the earnings multiplier would increase.
E. The earnings multiplier would increase; the earnings multiplier would either increase or decrease
depending on the firm's return on equity compared to its cost of capital.

Answer: B

Page | 1334
Explanation:
Remember that the equation used to determine the appropriate earnings multiplier for a stock
market series is the following:

P/E = [D/E / (k - g)]

Where: P/E = the earnings multiplier, or Price-to-Earnings ratio, D/E = the dividend payout ratio at t1,
k = the required rate of return, and g = the anticipated growth rate of dividends.

As the spread between k and g narrows, the earnings multiplier figure will increase. Indeed, the
earnings multiplier is very sensitive to changes in the spread between k and g, and this is the primary
reason for the greater relative volatility of the P/E ratio versus the EPS figure for a stock market
series.

A decrease in the retention rate will lead to an increase in the earnings multiplier figure, holding
everything else equal. Remember that the retention rate is simply (1 - dividend payout ratio). So
said, a decline in the retention rate is analogous to an increase in the dividend payout ratio. An
increase in the dividend payout ratio, holding everything else equal, will lead to an increase in the
earnings multiplier figure.

Question: 3382

According to the Dow Theory, a typical bullish price pattern

A. will have periodic sell-offs at high volume.


B. will not have troughs.
C. will have continuously increasing stock prices.
D. may have troughs.

Answer: D

Explanation:
According to the Dow Theory, there should be a general upward price movement during a bull
market, perhaps punctuated by occasional low-volume sell offs because of profit taking. Every
recovery from a sell off should reach a higher peak than that resulting from the previous recovery.

Question: 3383

You are going to hold a stock for an infinite amount of time. The current dividend is $1 per share and
is expected to grow at 9% a year. Your long run required return is 13%. Using the infinite period
dividend discount model calculate the value of the stock.

That answer is correct!


A. $27.25
B. none of these answers
C. $28.04
D. $26.45

Answer: A

Explanation:
First determine D1, which is the next period dividend. In this case it should be $1.09. Next use the
formula V = D1/(k - g) or V = $1.09/(0.13 - 0.09) = $27.25.

Page | 1335
Question: 3384

If the debit balances in brokerage accounts increase, the smart money technicians interpret it as:

A. none of these answers.


B. a bullish signal.
C. a hold signal.
D. a bearish signal.

Answer: B

Explanation:
Debit balances in brokerage accounts represent margin debt i.e. borrowings by investors from the
brokerages for investment in the market. Smart money technicians consider margin investors as a
group of astute investors and hence interpret large leveraged investments as a bullish signal.
Therefore, an large increase in the debit balances is considered a bullish sign while a decline is
considered a bearish sign.

Question: 3385

Firms in which of the following industries would likely have the highest earnings retention rate?
Further, are firms in this industry are likely to be financed primarily through debt or equity?

A. Media; debt
B. Fiber optics networking; equity
C. Fiber optics networking; debt
D. Media; equity
E. Computer manufacturing; debt
F. Computer manufacturing; equity

Answer: B

Explanation:
Of the industries listed, fiber optics networking would be expected to have the highest earnings
retention rate. The fiber optics networking industry is characterized by a high degree of investment
in research and development, along with a high growth rate and level of uncertainty. All of these
factorsare conducive to a high retention rate, i.e. a low dividend payout ratio. Further, firms within
industries associated with high growth rates and high levels of uncertainty are likely to be financed
primarily with equity. This is reasoned by several factors, some of which include the following:
Firms in new industries typically have a balance sheet weighted heavily toward intangible assets,
which can neither be easily liquidated nor pledged as collateral for a loan.
Firms in new industries are characterized by a high degree of research and development expenses.
Equity provides firms in emerging industries with a higher degree of agility than debt financing.
The cash flows of firms in developing industries are characterized by a high degree of uncertainty,
which often prevents the reliable payment of debt obligations.

Question: 3386

Preferred stock is

That answer is correct!


A. like a perpetuity. The holder of the stock receives a promise from the issuer to pay a stated
dividend, usually each quarter, for an infinite period. Because of tax advantages, the yields on

Page | 1336
preferred stocks tend to be lower than those on bonds.
B. like an annuity. The holder of the stock receives a promise from the issuer to pay a stated dividend,
usually once a year, for a long but finite period of time (usually 100 years). Because the dividend
payments on preferred stocks are not as certain as coupon payments on bonds, preferred stocks tend
to have higher required rates of return than most bonds.
C. like an annuity. The holder receives a promise from the issuer to pay a stated dividend, usually
semiannually, for an infinite period of time. Because the dividend payments on preferred stocks are
not as certain as coupon payments on bonds, preferred stocks tend to have higher required rates of
return than most bonds.
D. like a perpetuity. The holder of the stock receives a promise from the issuer to pay a stated
dividend, usually semiannually, for a long but finite period of time (usually 100 years). Because of tax
advantages, the yields on preferred stocks tend to be lower than those on bonds.

Answer: A

Explanation:
Payments of dividends for preferred stocks are only made after the firm meets its bond interest
payments. One would expect the required rates of return on preferred stocks to be higher than those
onbonds for this reason, but because of tax advantages, the preferred stock yields are actually lower
than those on even the highest-grade corporate bonds. 80% of intercompany preferred dividends are
tax-exempt, thereby increasing demand for preferred stocks and decreasing yields.

Question: 3387

S&P 500 sales have been estimated at $723 per share while the operating profit margin has been
estimated at 16.6%. The average tax rate on S&P 500 companies is about 40%, with an average
depreciation of 4.7% and an interest expense of 1.3%, all expressed as a percentage of sales. The
average payout ratio on S&P 500 is 17%. Assuming a 2.8% growth rate in the economy and an
investor demand of 12% return on equity, the per share S&P 500 must be valued at:

A. $123.22
B. $87.34
C. $79.11
D. $84.96

Answer: B

Explanation:
operating profits = Earnings before depreciation, interest & taxes = EBDIT and operating profit margin
= EBDIT/net sales. Therefore, net income/sales = (EBDIT/sales - depreciation/sales -
interest/sales)*(1-tax rate) = (16.6% - 4.7% - 1.3%)*(1-40%) = 6.36%.

Thus, earnings = 6.36% * 723 = $45.98 per share.

In the usual notation, the Dividend Discount Model gives Po = D1/(k-g). In this case, g = 2.8% and the
expected dividend per share next year = 45.98 * 17%*(1+2.8%) = $8.04 per share. Therefore, Po =
8.04/(12% - 2.8%) = $87.34 per share.

Question: 3388

ROE is equal to

A. net sales divided by equity.


B. net income divided by total capital.

Page | 1337
C. net income divided by equity.
D. Risk-free rate divided by equity.

Answer: C

Explanation:
Return on equity (ROE) multiplied by the retention rate is equal to the growth rate of earnings and
dividends. The growth rate, in turn, is used to derive the earnings multiplier, which is used to value
stocks.

Question: 3389

An intern with Churn Brothers Brokerage has been asked to calculate the Price-to-earnings ratio for
Clay Industries. She has been provided with the following information:

D0 = $1.25
g = 12% per year
k = 15.5% per year
Earnings per share: $2.78

Using this information, what is the price-to-earnings ratio for Clay Industries?

A. None of these answers is correct.


B. 17.05
C. 14.39
D. 21.27
E. 15.21
F. 12.85
G. The answer cannot be calculated from the information provided.

Answer: C

Explanation:
By dividing each side of the infinite period dividend discount equation by the EPS figure, it is possible
to determine the P/E ratio. This is illustrated as follows:

P/E = (D1 / EPS)/(k-g)

Where: D1 = the dividend at t1, EPS = the earnings per share calculation for t1, k = the required rate
of return, and g = the expected growth rate.

Manipulating the infinite period dividend discount model to solve for the PE is a rather intuitive
process.

Consider the fact that an investment's value is truly nothing more than the present value of all future
returns. So said, dividing both sides of the infinite period dividend discount model equation by the
EPS figure should yield the appropriate multiple, or "earnings multiplier." This is the price-to-
earnings ratio.
In this example, we are provided all of the necessary information. However, the dividend at t1 must
be calculated manually by multiplying D0 by (1 + growth rate). This will yield a figure of $1.40 for D1.
Now that D1 has been determined, we can solve for the P/E. Imputing all the given information into
the equation provided above will yield the following:

P/E = ($1.40 / $2.78) / (15.5% - 12%) = 14.39

Page | 1338
If you chose 12.85, remember that it is the dividend at t1 that is used in the determination of the
P/E, not the dividend at t0.

Question: 3390

The specific estimate approach to estimating an earnings multiplier involves

That answer is correct!


A. inferring the direction of change in the multiplier based on derivations of specific estimates for its
three major components.
B. inferring the direction of change in the multiplier based on predictions for change in its ten major
components.
C. inferring the direction of change in the multiplier based on predictions for changes in its three
major components.
D. inferring the direction of change in the multiplier based on derivations of the specific estimates for
its ten major components.

Answer: A

Explanation:
The specific estimate approach derives a specific estimate for the earnings multiplier based on a
range of estimates of the dividend payout ratio, the required rate of return, and the growth rate in
earnings and dividends.

Question: 3391

A preferred stock has a $100 par value and a dividend payout of $8 per year. What is the value of the
preferred stock?

A. $102.61
B. not enough information to calculate it
C. $89.65
D. $93.24

Answer: B

Explanation:
The missing information is the required rate of return in order to discount the dividend payout.

Question: 3392

Rhonda McWilliams, CFA, is examining the financial performance of a large manufacturing company,
and has assimilated the following information:

Adjusted operating profit before tax: $32,000,000


Cash operating taxes: $11,000,000
Cost of capital: 15% per year
Total capital employed: $130,670,000

Using this information, what is the Economic Value Added for this manufacturing firm? Further,
should the management of this Company be considered to have created value for shareholders?
Choose the best answer.

Page | 1339
A. $4,129,500; management has provided economic value
B. $5,504,350; management has provided economic value
C. $1,399,500; management has provided economic value
D. $6,330,000; management has not provided economic value
E. $6,330,000; management has provided economic value
F. None of these answers is correct.

Answer: C

Explanation:
Economic Value Added, a value-based measure of economic profit, is a registered trademark of
Stern, Stewart & Company. The equation used to calculate EVA is as follows:

EVA = {Net Operating Profits Less Adjusted Taxes - [Total Capital Employed * (Cost of Capital)]}.

In this case, the NOPLAT figure must be calculated manually by subtracting the Cash Operating Taxes
from the Adjusted Operating Profit before Tax (AOPBT) figure. Doing so will produce an answer
of$21,000,000 for the Net Operating Profit less Adjusted Taxes figure. Now that the necessary
information has been determined, the calculation of EVA is as follows:

EVA = {$21,000,000 - ($130,670,000 * 0.15)} = $1,399,500

A positive EVA calculation indicates that management has provided economic profits to
shareholders, as evidenced by the fact that the opportunity cost of capital employed is less than the
AOPAT figure. In this example, the management of Intelligent Semiconductor should be considered
as providing shareholder value, and due to the cash-based nature of the EVA calculation, this finding
is lithified.

The calculation of Adjusted Operating Profit Before Taxes (AOPBT), and Cash Operating Taxes are
important. The calculation of AOPBT is as follows:

Operating profit (after depreciation and amortization) + Implied interest on operating leases + any
increase in the LIFO reserve + goodwill amortization = Adjusted Operating Profit Before Taxes.

Cash Operating Taxes, another important component of EVA, is calculated as follows:

Cash Operating Taxes = Income Tax Expense + tax benefit from interest expenses + tax benefit from
interest on leases + taxes on non-operating income

Adjusted Operating Profit Before Taxes minus Cash Operating Taxes = Net Operating Profit Less
Adjusted Taxes (NOPLAT). Subtracting the dollar cost of capital from the NOPLAT figure will yield the
EVA.

Question: 3393

The "dollar-weighted" rate of return measure incorporates which of the following in its return
calculation?

A. None of these answers is correct.


B. Modified Internal Rate of Return
C. Net Present Value
D. More than one of these answers is correct.
E. Discounted Payback Period
F. Internal Rate of Return

Page | 1340
Answer: F

Explanation:
Performance measurement is an important consideration for the CFA candidate, and can be a
particularly difficult task in situations where the portfolio under consideration is subject to additions
and withdrawals. Two performance measurement tools discussed in the Level I readings are the
"dollar-weighted rate of return" and the "time-weighted rate of return." In fact, the dollar-weighted
rate of return is simply another name for the IRR equation. In investment management applications,
the IRR equation is commonly referred to as the "dollar-weighted rate of return" because it accounts
for the timing and scope of all cash flows.

Question: 3394

Assume the following information about a large-scale oil drilling company.

Net income / sales = 0.18


Total assets / common equity = 1.82
Sales / total assets = 0.70
Dividend payout ratio = 0.35

What is the expected annual growth rate of this firm's dividends?

A. 4.50%
B. The answer cannot be determined from the information provided.
C. 14.90%
D. 8.03%
E. None of these answers is correct.
F. 30.42%

Answer: C

Explanation:
A popular model for determining the growth rate of dividends is the following: g = RR * ROE

Where: g = the expected growth rate of dividends, RR = the retention rate (this is equal to 1 -
dividend payout ratio), and ROE = the return on equity.

Although it may at first appear otherwise, all of the necessary information has been provided.
Remember the Du Pont decomposition process for ROE, which breaks down the ROE figure into the
following:

ROE = (Net Income / Sales) * (Sales / Total Assets) * (Total Assets * Common Equity)
Mathematically, this will break down into (Net Income / Common Equity), the ROE figure. The
calculation of the return on equity for this company is as follows:

ROE = [0.18 * 0.70 * 1.82] = 0.22932, or 22.93%.

Now that the ROE figure has been determined, the calculation of the growth rate of dividends is as
follows:

g = [(1 - 0.35) * 0.2293] = 14.9045%

Question: 3395

Page | 1341
A price level at which the demand for a stock is expected to increase significantly is best
characterized by which of the following?

A. Reversal level.
B. Support level.
C. Resistance level.
D. More than one of these answers is correct.
E. Rotation price.

Answer: B

Explanation:
Technical analysts frequently examine historical price information to determine important "support"
and "resistance" levels. A support level is a price range at which the demand for a stock is expected
to increase, i.e. buying pressure will increase forcing the stock price upward, or will prevent it from
moving down. A resistance level is a price range at which the supply of a stock is expected to increase
significantly, forcing the price of the stock down, or preventing it from advancing.

"Rotation price" and "reversal level" are largely fictitious terms.

Question: 3396

An analyst with Smith, Kleen & Beetchnutty is examining shares of Mission Industries for possible
investment. Due to the development of several new products, the growth of Mission Industries is
expected to temporarily exceed its long-term rate of growth. Specifically, Mission Industries is
anticipated to grow at an 18% annual rate for the next three years, then return to its long-term rate
of growth of 13% per year.
The Company recently paid an annual dividend of $0.75, and similar investments have warranted a
14.5% per year rate of return.

Using the information provided, what is the value of Mission Industries common stock? Use the two-
stage dividend growth rate model.

A. $80.62
B. The answer cannot be determined from the information provided.
C. $81.44
D. $115.12
E. None of these answers is correct.
F. $64.16

Answer: F

Explanation:
The multi-stage dividend discount model is a more realistic way of valuing fast-growing companies
that pay dividends. With this model, it is necessary to estimate the above-average, or
"supernormal," rate of growth, as well as the long-term rate of growth. Once these growth rates
have been determined, they are used to calculate the anticipated annual dividends leading up to the
point at which the growth rate decelerates to the long-run rate of growth.

Incorporating the given information into the two-stage dividend discount model will yield the
following

P = {[$0.75 * 1.18) / 1.145] + [($0.885 * 1.18) / 1.31103] + [($1.04312 * 1.18) / 1.50112] + [($1.23088

Page | 1342
*
1.13) / (.145 - .13)]/1.50112} which can further be developed into:

P = {$0.77293 + $0.79655 + $0.81998 + $61.77} = $64.16

Question: 3397

Which of the following is not an advantage of technical analysis?

That answer is correct!


A. Requires the use of audited financial statements
B. Technical analysis is not time-consuming
C. Can catch market trends and reversals close to their occurrence
D. Does not rely heavily on financial figures and accounting statements
E. Does not assume that securities prices move in observable trends

Answer: A

Explanation:
One of the major advantages cited by technical analysts is that technical analysis does not rely
heavily on financial statements. Technical analysts believe that securities markets "price in" material
information gradually, and that securities prices move in identifiable trends and patterns. An astute
technical analyst, it is argued, can identify superior investment opportunities by recognizing the
implicit trends exhibited by securities prices and quantifiable past performance data. This belief is in
striking contrast with the Weak Form EMH, and is arguably the greatest criticism raised against
technical analysis.

Question: 3398

You have a stock that you are holding for one year. It has an estimated dividend payout of $2.50 and
an expected sale price of $43. Using the dividend discount model, calculate the value of the stock if
your required rate of return is 12%.

A. $45.50
B. not enough information to calculate it
C. $43
D. $40.63

Answer: D

Explanation:
Calculation = $2.50/(1+.12) + $43.00/(1+.12) = $40.63.

Question: 3399

A portfolio manager with Churn Brothers Brokerage is examining shares of a newly-issued perpetual
preferred stock for possible investment. The preferred stock is expected to pay a quarterly dividend
of $0.70, and the required rate of return is 12.50% per year. At what price would this preferred stock
be fairly valued?

A. None of these answers is correct.


B. $14.29
C. The answer cannot be calculated from the information provided.
D. $18.00

Page | 1343
E. $5.60
F. $22.40

Answer: F

Explanation:
Assuming that the quarterly dividend is to remain unchanged forever allows us to use the standard
perpetuity model for preferred stock, which is illustrated as follows:

Value of preferred stock = (Annual dividend / required rate of return)

In this example, we are given the quarterly dividend, which must be annualized before it can be
imputed into the perpetuity valuation equation.
So said, a quarterly dividend of $0.70 translates into a yearly dividend of $2.80. Incorporating this
yearly dividend into the perpetuity valuation model will result in the following:

Value of preferred stock = {$2.80 / 0.125} = $22.40

Question: 3400

________ means that the present value of expected cash inflows exceeds the asset's market price.

A. Expected value
B. Extra value
C. Excess present value
D. Past value

Answer: C

Explanation:
By definition: the amount the PV of expected cash flows exceeds the asset's market price is excess
present value. This is also called excess economic value or excess economic returns.

Question: 3401

A retail investor is considering the purchase of Intelligent Semiconductor common stock, believing
the shares have bottomed. In his analysis, the retail investor has gathered the following information:

Total assets: $640,000,000


Total liabilities: $510,000,000
Number of common shares outstanding:
1,500,000 Current stock price: $110.56 per share
Required return: 17.75% per year
Expected growth rate: 15.50% per year
Next dividend: $2.26 per share
Earnings per share: $7.89

Using this information, what is the price-to-book ratio for Intelligent Semiconductor? Further, should
Intelligent Semiconductor be considered as trading at a discount or a premium to its book value?

A. None of these answers is correct.


B. 1.27, discount
C. 14.01, premium
D. 34.84, premium

Page | 1344
E. 14.01, discount
F. 1.27, premium

Answer: F

Explanation:
The book value of a common stock is found by dividing the net worth of a company by the number of
outstanding shares. While there exists several different methods for calculating net worth, the most
simplistic involves subtracting total liabilities from total assets, giving us the shareholder's equity
figure.
Investment professionals often see the book value as a "floor" for common stock prices, and the
price-to-book ratio is quite popular in the field of investment management. The calculation of the
price-to-book ratio involves the following equation:

Price-to-book ratio = {Market price of common stock at t0 / [(total assets - total liabilities) / # of
common shares outstanding]}

While the # of common shares outstanding has been provided, the shareholder's equity figure must
be calculated manually. The calculation of the per share shareholders equity is illustrated as follows:

Shareholders equity = [($640,000,000 - $510,000,000) / 1,500,000] = $86.67.

Now that the shareholders equity has been calculated and transformed into a per-share figure, the
priceto-book value can be calculated as follows:

Price-to-book value = ($110.56 / $86.67) = 1.276.

As you can see, the required rate of return, along with the expected growth rate, is not necessary in
the calculation of the price-to-book ratio.

Any time the price-to-book ratio is greater than one, a stock is said to be "trading at a premium to
book." Frequently, investment professionals will use the term "intrinsic value" when referring to the
book value. A figure of less than one for the price-to-book ratio is commonly referred to as "trading
at a discount to book." Stocks trading below book value are often sought after by value investors,
who believe that these shares have been discounted below their real value.

While the price-to-book value is a useful tool in the stock selection process, it possesses some
important flaws. One problem with the Price-to-Book Ratio is that one of the terms - Book Value - is
so easilymanipulated. Valuation of inventory and real estate are easily adjusted on the books. Stock
buy-backs and write-offs of exceptional items also deflate Book Value, making high priced stocks
seem overvalued.

Question: 3402

James Clinton, a portfolio manager with Middle Road Investment Advisors, is trying to estimate the
appropriate earnings multiplier for the automobile industry. In his analysis, James examines the
expected growth rate of dividends for the industry, as well as the expected dividend payout ratio and
required rate of return. From this information, James proceeds toward an estimation of the earnings
multiplier for the series.

Which of the following best describes this method of estimating an earnings multiplier for an
industry?

A. The specific estimate approach

Page | 1345
B. Macroanalysis
C. Microanalysis
D. The rate of change approach
E. The arbitrage pricing method
F. Input-output analysis

Answer: C

Explanation:
The answer called for in this question is "microanalysis." This method involves an examination of the
components of the earnings multiplier, including the anticipated growth rate of dividends, the
required rate of return, and the dividend payout ratio. Once these variables have been examined,
both from the perspective of trend analysis and point estimation, a value for the earnings multiplier
is deduced.
Microanalysis is contrasted by macroanalysis, which involves an examination of the relationship
between the earnings multiplier of a stock market series and the earnings multiplier of the overall
market. For example, an individual projecting an earnings multiplier for a software index using
macroanalysis would begin by examining the relationship between the P/E ratio of the software
index and the P/E ratio of a broad market index such as the Standard & Poors 500. Both historical
trends and point estimates would be examined, and from this information, a projection of the
earnings multiplier for the stock market series is deduced.

Input-output analysis is used in the examination of competitive forces within an industry. The
arbitrage pricing model is a security valuation model which allows for several relevant "situation
specific" variables. Finally, the "specific estimate" and "rate of change" approaches are used to
estimate the earnings multiplier of a stock market series, rather than an industry.

Question: 3403

The real risk free rate is 6% per year and the expected inflation rate is 2% per year. What is the
annual nominal rate of interest?

A. Not able to compute with the above data.


B. 8.1%
C. 4%
D. 8%

Answer: B

Explanation:
Nominal rate = (1.06)*(1.02)=8.1%.

Question: 3404

An increase in debit balances would be a bullish sign. This relates to:

A. Block Uptick-Downtick Ratio


B. Mutual Fund Cash Positions
C. Relative Trend
D. Odd-Lot, Short-Sales Theory
E. Dow Theory
F. Short Sales by Specialists
G. Margin Debt
H. Diffusion Index

Page | 1346
Answer: G

Explanation:
Debit balances in brokerage accounts represent borrowing by knowledgeable investors from their
brokers. Such borrowing is called margin debt. In such cases, an increase in debit balances would be
a bullish sign.

Question: 3405

An analyst is attempting to value shares of a Clay Industries, and has solicited the help of a senior
financial analyst. During their conversation, the senior financial analyst provides the following
information about Clay Industries:

Required rate of return on the bank's equity: 13.50% per year Free cash flow to equity multiple at t4:
21

2,000,000 shares outstanding

Additionally, the analyst has obtained the following estimates of free cash flow to equity over the
next five years:

Year 1: $3,500,000
Year 2: $3,750,000
Year 3: $3,400,000
Year 4: $4,000,000
Year 5: $4,150,000

Using this information, what is the value per share of this Clay Industries' shares according to the
Free Cash Flow to Equity Model?

A. $34.02
B. None of these answers is correct.
C. $23.78
D. $29.60
E. $21.34
F. The answer cannot be calculated from the information provided.

Answer: D

Explanation:
When determining the value of a common stock using the free cash flow to equity model, it is
necessary to determine three things:

1. The required rate of return on equity investments.


2. The estimated free cash flow to equity multiple at time "k."
3. The estimated free cash flows figures for the time periods leading up to "k."

In this example, the calculation must begin with the discounting the free cash flow to equity figures
for each of the four years provided. These figures are discounted each period by the required return
on equity investments, and the final answer is converted to a per-share basis. This process is
illustrated below:

Year 1: ($3.500,000 / 1.135) / 2,000,000 shares outstanding = $1.54

Page | 1347
Year 2: [$3,750,000 / (1.135)(1.135)] / 2,000,000 shares outstanding = $1.46
Year 3: [$3,400,000 / (1.135)(1.135)(1.135)] / 2,000,000 shares outstanding = $1.16
Year 4: [$4,000,000 / (1.135)(1.135)(1.135)(1.135)] / 2,000,000 shares outstanding = $1.21
Year 5: [$4,150,000 / (1.135)(1.135)(1.135)(1.135)(1.135)] / 2,000,000 shares outstanding = $1.10

Now that the free cash-flow-to-equity figures have been discounted and converted to a per-share
basis, the next step in the valuation process is to determine the value of the final cash flow, which is
defined as:

[(Free cash flow to equity multiple * Final free cash flow) / (1 + r)(1+r)...(1 + k)]

In the body of this question, we were given the anticipated multiple of free cash to equity that shares
of Intelligent Semiconductor will sell for at time period: specifically, 21 times. Imputing this
information into the terminal cash flow equation will yield the following:

{[21 * ($4,150,000 / 2,000,000 shares outstanding)] / [(1.135) (1.135)(1.135)(1.135)(1.135)]} =


$23.13.

Adding the answers from step 1 to the final year cash flow will yield the following:
Value of Intelligent Semiconductor = [$1.54 + $1.46 + $1.16 + $1.21 + $1.10 + 23.13] = $29.60 per
share.

Question: 3406

Which of the following best describes the relationship between the relative maturation of an
industry and the retention ratio of companies within the industry? Further, what is the proposed
relationship between expected growth and the relative maturity of an industry?

That answer is correct!


A. Negative relationship; negative relationship
B. Positive relationship; negative relationship
C. Negative relationship; positive relationship
D. Negative relationship; no correlation
E. Positive relationship; positive relationship

Answer: A

Explanation:
As an industry advances in maturity, growth of the overall industry will decline. As growth
opportunities diminish, companies within the industry will be forced to pay out a larger proportion of
their earnings as dividends; i.e. the dividend payout ratio will increase. Remember that the retention
ratio is equal to (1 - the dividend payout ratio). Thus, the retention ratio of companies will likely
decline as the industryadvances in maturity. The relationship between the dividend payout ratio and
the maturity of the industry is negative and loosely linear.

As an industry becomes more mature, growth opportunities decline. This relationship is also loosely
linear.

Question: 3407

Which of the following calculations can be successfully performed without knowing the required rate
of return?

I.Net Present Value

Page | 1348
II.Internal Rate of Return
III.Modified Internal Rate of Return
IV.Time-Weighted Rate of Return
V.Dollar-Weighted Rate of Return
VI.Valuing a common stock using the Dividend Discount Model
VII.Valuing a common stock using the Free-Cash-Flow-to-Equity Model

A. II, V, VI
B. II, IV, V
C. I, II, VI
D. II, IV, V, VI, VII
E. II, IV, VI, VII

Answer: B

Explanation:
Of the financial figures listed, only the Internal Rate of Return, which is also known as the "Dollar-
weighted Rate of Return," and the Time-Weighted Rate of Return, can be calculated without
knowing the required rate of return.

Question: 3408

What is the value of coupon and principal payments on a 10-year bond with coupon payments of $50
every six months (for a total of 20 payments), a principal payment of $8,000 in 10 years, and a
required rate of return of 8%?

A. Coupons: $938.48, Principal: $4932.34


B. Coupons: $743.39, Principal: $5546.50
C. Coupons: $679.50, Principal: $3705.60
D. Coupons: $549.50, Principal: $2834.65
E. Not enough information

Answer: C

Explanation:
The value of coupon payments can be thought of as the value of an annuity with 20 periods, and with
a rate of return of 4% (half of 8%) for each period. Using appendix C in the book by Reilly & Brown,
the present value of the coupons is equal to 50 x 13.59 = $679.50. Using the same appendix, the
present value of the principal payment is equal to 8000 x 0.4632 = $3705.60. Note that many
textbooks recommend using the six-month interest rate and doubling the number of yearly periods
in making this calculation. Using 4% for 20 periods, the value of the final payment is
$8,000/(1.04^20) = $3,651.

Question: 3409

A portfolio manager with Churn Brothers Brokerage is trying to determine whether shares of Mile
High Airlines are fairly valued. In his analysis, the portfolio manager is using the two-stage dividend
discount model, and has ascertained the following information:

Mile High Airlines is expected to grow at a rate of 20% per year for the next four years.
At t5, Mile High Airlines is anticipated to return to its long-term growth rate of 10% per year. D0 =
$0.44

r = 15.75% per year

Page | 1349
Common shares outstanding = 1,500,000

Given this information, what is the value of Mile High Airlines? Use the two-stage dividend discount
model.

A. $20.77
B. The answer cannot completely be calculated from the information provided.
C. None of these answers is correct.
D. $16.91
E. $11.65
F. $21.23

Answer: E

Explanation:
The multi-stage dividend discount model is a more realistic way of valuing fast-growing companies
that pay dividends. With this model, it is necessary to estimate the above average, or "supernormal,"
rate of growth, as well as the long-term rate of growth. Once these growth rates have been
determined, they are used to calculate the anticipated annual dividends leading up to the point at
which the growth rate decelerates to the long-run rate of growth.

Incorporating the given information into the two-stage dividend discount model will yield the
following

P = {[$0.44 * 1.20) / 1.1575] + [($0.528 * 1.20 / 1.33981] + [($0.6336 * 1.20) / 1.55083] + [($0.76032 *
1.20) / 1.79508] + [($0.91238 * 1.10) / (.1575 - .1)]/1.55083} which can further be developed into:

P = {$0.45616 + $0.4729 + $0.49027 + $0.50827 + $9.72} = $11.65


As you can see, the number of common shares outstanding was not necessary in this instance, and
was provided largely as a distraction. However, in some cases knowing the number of common
shares outstanding will be necessary in order to convert a company's dividends paid to a per-share
basis.

Question: 3410

The rate of inflation

A. has been found to vary inversely with the aggregate profit margin.
B. has been found to vary positively with the aggregate profit margin.
C. has an unresolved affect on aggregate profit margin.
D. has been found not to affect aggregate profit margin.

Answer: C

Explanation:
Finkel and Tuttle found that the capacity utilization rate, unit labor costs, rate of inflation, and foreign
competition were the four major variables that affected the aggregate profit margin. They postulated
that the rate of inflation was positively related to profit, while others have postulated the opposite.

Question: 3411

This is the measure of the number of buyers versus sellers to indicate the institutional investor
sentiment.

Page | 1350
That answer is correct!
A. Block Uptick-Downtick Ratio
B. Mutual Fund Cash Positions
C. Margin Debt
D. Short Sales by Specialists

Answer: A

Explanation:
If a block trade (institutional trading) price is higher than prior transaction price, this is called uptick,
if otherwise then downtick: the development of block uptick (buyers) downtick (sellers) ratio
indicates investors' sentiment. A ratio bigger than one indicatesmore buyers, i.e. bullish sentiment.

Question: 3412

Consider the following transactional information for the investment account of an underwriting
syndicate:

1st Quarter
Ending portfolio value: $50,800,000
Total amount invested: $46,100,000

2nd Quarter
Ending portfolio value: $51,100,000
Total amount invested: $50,800,000

3rd Quarter
Ending portfolio value: $51,000,000
Total amount invested: $51,100,000

4th Quarter
Ending portfolio value: $54,500,000
Total amount invested: $50,000,000

Using this information, what is the annual time-weighted rate of return for this portfolio? Assume no
taxes or transaction charges.

A. None of these answers is correct.


B. 20.59% per year
C. 19.59% per year
D. 22.14% per year
E. 18.55% per year
F. The time-weighted rate of return cannot be calculated from the information provided.

Answer: B

Explanation:
The time-weighted rate of return is the preferred method of return calculation in the investment
management industry, primarily because this method is not sensitive to significant additions and
withdrawals of funds from portfolios under examination. The calculation of the time-weighted rate
of return involves three steps, which are illustrated as follows:

Step 1:

Page | 1351
Price the portfolio immediately prior to any significant additions or withdrawals. Separate the
portfolio into a series of subperiods based on the dates of cash inflows and outflows.

Step 2:
Calculate the holding period return for each subperiod.

Step 3:
Determine the annualized holding period return by linking or compounding the holding period return
of each subperiod. If the investment is for more than one year, use the geometric mean of the annual
returns as the time-weighted rate of return. If the investment is for less than one year, compound the
subperiod returns to obtain an annualized measurement.

To begin the process of determining the time-weighted rate of return, we would break the portfolio
up into the subsequent series of cash flows. However, in this example, the cash flows are already
aggregated for us and we can move on to the next step: determining the holding period return for
each subperiod. This process is detailed as follows:

Quarter 1 holding period return = [($50,800,000 ending value - $46,100,000 invested) / $46,100,000
invested] = 10.19523%
Quarter 2 holding period return = [($51,100,000 ending value - $50,800,000 invested) / $50,800,000
invested] = 0.59055%
Quarter 3 holding period return = [($51,000,000 ending value - $51,100,000 invested) / $51,100,000
invested] = (0.1957%)
Quarter 4 holding period return = [($54,500,000 ending value - $50,000,000 invested) / $50,000,000
invested] = 9.00%

Now that the holding period return for each subperiod has been determined, we must annualize the
return measure by taking the product of all four quarterly returns. This process is illustrated below:

[(1 + .10195) * (1 + .00591) * (1 - .00196) * (1 + .09) - 1] = .2059 or 20.59%

When calculating the time-weighted rate of return, remember that the total amount invested is the
relevant figure, not the beginning portfolio value. Notice that during the fourth quarter, the total
amount invested does not equal the ending amount for the third quarter. This differential could be
explained by numerous phenomena. Perhaps the difference is due to a cash withdrawal from the
account. Maybe it was used to pay expenses or meet an outstanding margin call. What is important
to note is the fact that this money was not invested, and should not be included in the holding period
return for the fourth quarter. So said, whenever possible you should use the total amount invested
rather than the beginning portfolio value in the calculation of the subperiod holding period return.
If you chose 19.59%, remember that in the calculation of the time-weighted rate of return, it is the
geometric mean that is used rather than the arithmetic mean.

Question: 3413

Given a required rate of return of 10%, what is the present value of the coupon payments on a 10-
year bond with coupon payments of $100 every six months (for a total of 20 payments)?

A. $2353.95
B. $1054.48
C. Not enough information
D. $1868.27
E. $1246.22

Answer: E

Page | 1352
Explanation:
The value of the coupon payments can be thought of as the value of an annuity for 20 periods with a
rate of return of 5% (half of 10%) for each period. Using appendix C in the book by Reilly & Brown,
the present value is equal to $100 x 12.4622 = $1,246.22.

Question: 3414

Interest rate expenses

A. have been increasing more or less continuously. That increase was especially pronounced during
the 1980s, when firms increased their debt financing and financial risk. Interest rate expenses as a
percentage of sales have also been very volatile.
B. have increased up to 1989, and have since been decreasing. That decrease is the result of an
interest rate decline and of corporations reducing their debt levels. Interest rate expenses as a
percentage of sales have also been very volatile.
C. have been increasing more or less continuously. That increase was especially pronounced during
the 1980s, when firms increased their debt financing and financial risk. Interest rate expenses as a
percentage of sales have increased from 2.44% in 1977 to 7.47% in 1996.
D. have increased up to 1989, and have since been decreasing. That decrease is the result of an
interest rate decline and of corporations reducing their debt levels. Interest rate expenses as a
percentage of sales have increased from 1.44% in 1977 to 3.47% in 1996.

Answer: B

Explanation:
Interest rate expenses had increased up to 1989, rising from 1.44% of sales in 1977 to 3.47% in 1989.
That increase was especially pronounced during the 1980s, when firms increased their debt financing
and financial risk. That trend started reversing during the 1989-1990 recession because of an interest
rate decline, and because of corporations reducing their debt levels. Interest rate expenses are
generally fixed, and have therefore varied considerably as a percentage of sales, which are quite
volatile.

Question: 3415

________ should be analyzed before individual industries and companies to determine whether the
economic and market outlook indicates an underweighting of asset classes.

A. Historical trends
B. Security markets
C. Economies and security markets
D. Economies

Answer: C

Explanation:
Economies and security markets should be analyzed before individual industries and companies to
determine whether the economic and market outlook indicates an underweighting of asset classes.

Question: 3416

A firm has an earnings retention rate of 35% and it earns a 12% per year return on its equity.
Calculate the expected annual growth rate of the firm's dividend?

Page | 1353
A. 5%
B. 4.2%
C. Not able to compute with the above data.
D. 6%

Answer: B

Explanation:
g=(RR)(ROE)=0.35*0.12=4.2%.

Question: 3417

A preferred stock has a $500 par value and a dividend payout of $45 per year. Your required rate of
return is 15%. What is the value of the preferred stock?

A. not enough information to calculate it


B. $400
C. $350
D. $300

Answer: D

Explanation:
The value of a preferred stock is the stated annual dividend divided by the required rate of return on
preferred stock.

In this case, V = $45/.15 = $300

Question: 3418

A technical analyst would sell a stock when

That answer is correct!


A. it leaves the bottom end of a flat trend channel.
B. it is in a rising trend channel.
C. it leaves the declining trend channel.
D. it is in a trough.

Answer: A

Explanation:
When the price of a stock falls below the bottom end a flat trend channel, a technical analyst would
probably predict the beginning of a declining trend, and would sell the stock.

Question: 3419

Assume the following information about a stock market series:

Observed beginning series value: 515.60


Anticipated ending series value: 609.15
Expected dividends during the period: $24
Required rate of return: 21% per year

What is the expected annual rate of return for this index? (Assume a one-year holding period.)

Page | 1354
A. None of these answers is correct.
B. 13.49%
C. 19.30%
D. 11.42%
E. 22.80%

Answer: E

Explanation:
To calculate the expected rate of return for a stock market series, the following information must be
known:

The beginning value for the series

The anticipated ending value for the series, and

The amount of any dividends and/or distributions during the period

Once this information has been determined, the expected return on a stock market index can be
found by employing the following equation:

E(R) = [(EV - BV + Div) / BV]

Where: E(R) = the expected return on the stock market series, EV = the anticipated ending value for
the series, BV = the observed beginning value for the series, and Div = the amount of any dividends
paid during the period.

In this example, all of the necessary information has been provided and the calculation of the
expected return on this stock market series is found as follows:

E(R) = [$609.15 - $515.60 + $24] / $515.60 = 22.80%


This is slightly higher than the required rate of return. Assuming that the ending value and dividends
prove accurate, investment in this stock market series is likely advisable.

Question: 3420

Given that the expected dividend payout ratio on a common stock is 0.65, the required rate of return
is 15%, and the dividend growth rate is 6%, using the earnings multiplier model, what is the P/E
ratio?

A. Not enough information


B. 8.20
C. 6.75
D. 7.22
E. 10.59

Answer: D

Explanation:
The infinite period Dividend Discount Model claims that the current price of a common stock is equal
to D1 / (k - g), where D1 is next period's (most often next year's) dividend, k is the required rate of
return, and g is the growth rate of dividends. The earnings multiplier model goes a step further by
dividing both sides of the infinite period Dividend Discount Model equation by expected earnings

Page | 1355
during the next 12 months, yielding P/E = (D1/E) / (k - g). In this question, the P/E ratio is equal to
0.65 / (0.15 - 0.06) = 7.22.

Question: 3421

In general, the earnings multiplier for a stock market series is a more volatile figure than the
earnings-per-share for the same series. The greater relative volatility of the earnings multiplier is
mostly attributable to which of the following?

A. The EPS figure is subject to cash flow adjustments, which "normalize" the EPS figure over time.
B. The earnings multiplier is subject to a tax-deleveraging effect.
C. The earnings multiplier is more sensitive to changes in the payout ratio.
D. The earnings multiplier is more sensitive to changes in the spread between k and g.
E. None of these answers is correct.

Answer: D

Explanation:
The greater relative volatility of the earnings multiplier versus the EPS figure is primarily attributable
to an increased sensitivity to changes in the spread between the required rate of return "k" and the
anticipated growth rate "g." Remember that the equation used to determine the appropriate
earnings multiplier for a stock market series is the following:

{P/E = [D/E / (k - g)]}

Where: P/E = the earnings multiplier, or Price-to-Earnings ratio, D/E = the dividend payout ratio at t1,
k = the required rate of return, and g = the anticipated growth rate of dividends.
As you can see, changes in the spread between the required rate of return and the anticipated
growth rate can have a dramatic effect on the earnings multiplier for a stock market series. While the
earningsmultiplier is sensitive to changes in the dividend payout ratio, volatility in this figure is not
cause for the increased volatility of the earnings multiplier versus the EPS figure.

Question: 3422

The infinite period Dividend Discount Model is likely to be more effective in the case of a ________.

That answer is correct!


A. mature firm
B. all of these answers
C. small firm
D. growth firm

Answer: A

Explanation:
The infinite period Dividend Discount Model assumes a constant dividend growth rate (which could
well be zero). This assumption is only reasonable for a mature firm that has settled in its industry. A
growth firm will have highly erratic dividends, depending on the available cash flows.

Question: 3423

The infinite period ________ model assumes that k is greater than g.

A. growth

Page | 1356
B. SP
C. Dividend Discount Model
D. valuation

Answer: C

Explanation:
The infinite period Dividend Discount Model has the following assumptions:

1. Dividends grow at a constant rate


2. The constant growth rate will continue for an infinite period
3. The required rate of return (k) is greater than the infinite growth rate (g). If it is not, the model
gives meaningless results because the denominator becomes negative.

Question: 3424

Given that the risk-free rate of return is 7%, what is the value of a bond with coupon payments of
$100 every six months, a final payment of $2,000 in 8 years, and a risk-premium of 5%?

A. $1,532.69
B. $2,320.56
C. $1,818.40
D. Not enough information
E. $1,285.38

Answer: C

Explanation:
The value of bond is equal to the present value of the stream of coupon payments (which can be
thought of as an annuity for a certain number of years) plus the present value of the final payment.
The required rate of return on the bond is equal to the risk-free rate of return plus the risk-premium
(7+5=12% for the year, 6% for six months). Using appendix C in the book by Reilly & Brown, the
present value of the coupons is $100 x 10.106 = $1,010.60. The present value of the final payment is
$2,000 x 0.4039 = $807.80, or $2,000/(1.06^16). The value of the bond is 1010.60 + 807.80 =
$1818.40. Note that many textbooks recommend using the six-month interest rate and doubling the
number of yearly periods in making this calculation. Using 6% for 16 periods, the value of the final
payment is $787.40 and the total value of the bond is 1010.60 + 787.40 = $1,798.

Question: 3425

A firm has an expected dividend payout ratio of 60%, and an expected dividend growth rate of 5%
per year. What is the firm's Price/Earnings ratio if the appropriate discount rate is 8% per year?

That answer is correct!


A. 20
B. Not able to compute with the above data.
C. 200
D. 2

Answer: A

Explanation:
Value = 0.60/(0.08-0.05) = 20.

Page | 1357
Question: 3426

The multiplier effect is caused by

A. government fiscal and monetary policy.


B. government fiscal policy.
C. government monetary policy.
D. inflation.
E. inflation and interest rates.

Answer: B

Explanation:
The multiplier effect is the effect government (fiscal) spending has on the broader economy. For
example, a certain amount of spending on roads by the government increases demand for
earthmoving equipment, concrete, and other items, causing firms that supply those items to
themselves require more materials and employees. This results in the government's spending being
amplified (or multiplied) beyond the amount that is directly spent.

Question: 3427

Using a time series, an economist with Churn Brothers Brokerage examines the relationship between
the historical earnings multiple of the banking industry with that of the S&P 500 index. The results of
this time series are used to project an estimate of the future earnings multiple for the banking
industry. Which of the following best characterizes this method of forecasting an industry earnings
multiple? Choose the best answer.

A. Monte Carlo simulation


B. Macroanalysis
C. Time series regression
D. Microanalysis
E. None of these answers is correct
F. Porter Method

Answer: B

Explanation:
The method profiled in this example is "macroanalysis," which is one of two methods for estimating
the earnings multiplier of an industry. Macroanalysis involves examining the historical relationship
between the earnings multiplier of an industry with that of the overall market. Macroanalysis
forecasts often use a time series.

The macroanalysis method is contrasted by microanalysis, which involves examining the variables
underlying the earnings multiplier - the required rate of return, the growth forecast, and the
dividend payout ratio. In microanalysis, these variables are examined for the industry and then
compare them with the values of these variables for the entire market.

The "Porter Method" is used to examine the level of competition in an industry, and "Monte Carlo
simulation" is a method of measuring stand-alone risk. While "time series regression" is an appealing
choice, it does not represent the best possible answer.

Question: 3428

When investors are pessimistic about the market, the confidence index should have

Page | 1358
A. a value over 100.
B. a low value.
C. a high value.
D. a value over 85.

Answer: B

Explanation:
The confidence index measures the yield spread between high-grade bonds and a large cross section
of bonds. Technical analysts believe that during periods of low confidence, investors are less willing
to invest in lower-quality bonds, thereby pushing up their yields, and decreasing the confidence
index. A low index value is thus viewed as a bearish sign.

Question: 3429

Which of the following is not necessary to determine the price/earnings ratio for a stock market
series?

That answer is correct!


A. All of these are necessary to determine the P/E ratio for a stock market series.
B. The dividend at D1.
C. The EPS figure at t1.
D. The expected growth rate of dividends.
E. The required rate of return.

Answer: A

Explanation:
All of the choices listed are components in the equation used to determine the earnings multiplier of
a stock market series, as shown below:

P/E = [(D1 / E1) / (k-g)

Where: D1 = the annual per-share dividend at t1, E1 = the EPS figure at t1, k = the required rate of
return on common stock, and g = the expected growth rate of dividends.

Question: 3430

Technical analysis assumes that shifts in trends can be detected sooner or later in the action of the
________.

A. consumer price index


B. investment managers
C. all of these answers
D. market

Answer: D

Explanation:
The four assumptions that support technical analysis are these:

1. The market value of any good or service is determined solely by the interaction of supply and
demand for it.

Page | 1359
2. Supply and demand are governed by numerous factors, both rational and irrational, including the
economic variables relied upon by the fundamental analyst, as well as opinions, models and guesses.
The market weights all of these factors continually and automatically.

3. Disregarding minor fluctuations, the prices for individual securities and the overall value of the
market tend to move in trends, which persist for appreciable lengths of time.

4. Prevailing trends change in reaction to shifts in supply and demand relationships. These shifts, no
matter why they occur, can be detected sooner or later in the action of the market itself.

Question: 3431

The real Risk-Free Rate in different countries will vary widely due to which of the following factors?

That answer is correct!


A. All of these answers
B. Growth rate of the labor force
C. Growth rate of the average number of hours worked
D. Growth rate of labor productivity

Answer: A

Explanation:
The real risk free rate is dependent on the growth rate, because typically an investor will want to
have a return larger than the projected growth rate of an economy. The growth rate of an economy is
affected by variables like the growth rates of the labor force, the work hours and productivity.

Question: 3432

Historically, the EPS figure for a stock market series has been less volatile than the earnings
multiplier for the same series. Which of the following best characterizes the primary reason for the
greater volatility experienced by the earnings multiplier? Choose the best answer.

A. None of these answers is correct.


B. The EPS figure is less volatile due to accounting manipulations and the malleability of
international and domestic accounting standards including GAAP.
C. The price-to-earnings figure experiences a tax leveraging effect that is not passed on to the EPS
figure.
D. The earnings multiplier is more sensitive to fluctuations in the equity markets than is the EPS
figure; i.e. the earnings multiplier is "forward looking."
E. The price-to-earnings ratio is more sensitive to changes in the spread between the required rate of
return and the anticipated future growth rate.
F. The earnings multiplier is more sensitive to changes in dividend policies than is the EPS figure.

Answer: E

Explanation:
The greater relative volatility of the earnings multiplier versus the EPS figure is primarily attributable
to an increased sensitivity to changes in the spread between the required rate of return "k" and the
anticipated growth rate "g." Remember that the equation used to determine the appropriate
earnings multiplier for a stock market series is the following:

{P/E = [D/E / (k - g)]}

Page | 1360
Where: P/E = the earnings multiplier, or Price-to-Earnings ratio, D/E = the dividend payout ratio at t1,
k = the required rate of return, and g = the anticipated growth rate of dividends.

As you can see, changes in the spread between the required rate of return and the anticipated
growth rate can have a dramatic effect on the earnings multiplier for a stock market series. While the
earnings multiplier is sensitive to changes in the dividend payout ratio, volatility in this figure is not
cause for the increased volatility of the earnings multiplier versus the EPS figure.

Question: 3433

A firm's stock has a coefficient of variation of 0.85. It has a beta of 1.23 and a firm specific variance of
0.06. The market portfolio has an expected return of 14% and a standard deviation of 19%. The firm
has a payout ratio of 0.3 and it obtains a return on equity equal to 10.3%. The firm's stock is trading
at $25.1 per share. The risk-free rate in the economy is 5.4%. What's the dividend that the market
expects the firm to pay out next year?

That answer is correct!


A. $2.20
B. $2.41
C. $2.05
D. $1.99

Answer: A

Explanation:
The Dividend Discount Model implies that the firm's share price is given by Po = D1/(k-g), using
standard notation. The dividend growth rate is given by g = ROE*(1-dividend payout ratio) = 10.3% *
0.7 = 7.21%. The CAPM expected rate of return on the stock is equal to the risk-free rate plus beta
times the market premium. So the expected return on the stock is 5.4% + 1.23*(14 - 5.4)% = 15.98%.
Therefore, the firm is expected to pay out $25.1*(15.98% - 7.21%) = $2.2.

Question: 3434

The confidence index

That answer is correct!


A. is equal to Barron's average yield on 10 top-grade bonds divided by the yield on the Dow Jones
average of 40 bonds, multiplied by 100. Technical analysts who try to follow the "smart money"
would view the confidence index as a bullish indicator; an increase in its value is a bullish sign.
B. is equal to Barron's average yield on 10 top-grade bonds divided by the yield on the Dow Jones
average of 100 bonds, multiplied by 100. Technical analysts who try to follow the "smart money"
would view the confidence index as a bearish indicator; an increase in its value is a bearish sign.
C. is equal to the yield on the Dow Jones average of 40 bonds divided by Barron's average yield on 40
topgrade bonds, multiplied by 100. Technical analysts who try to follow the "smart money" would
view index values of under 100 to be bearish, and index values over 100 to be bullish.
D. is equal to the yield on the Dow Jones average of 80 bonds divided by Barron's average yield on 80
topgrade bonds, multiplied by 100. Technical analysts who try to follow the "smart money" would
view the index as a bullish indicator; an increase in its value is a bullish sign.

Answer: A

Explanation:
The confidence index measures the yield spread between high-grade bonds (Barron's 10 top-grade

Page | 1361
bonds) and a large cross section of bonds (the Dow Jones average of 40 bonds). The yields on high-
grade bonds should always be lower than the average on a wide cross-section, so the index should
always have a value below 100. During periods of high confidence, investors are willing to invest
more in low-quality bonds, causing their yields to decrease relative to the yields on high-grade
bonds. So when confidence is high, the value of the index increases.

Question: 3435

Estimates of GNP

A. are used to estimate aggregate net sales for the firms in a stock market series such as the S&P 400.
About 90% of the variance in percentage changes in S&P 400 net profits can be attributed to
percentage changes in the GNP. Net sales are more volatile than GNP.
B. are used to estimate aggregate net profits for the firms in a stock market series such as the S&P
400. About 30% of the variance in percentage changes in S&P 400 net profits can be attributed to
percentage changes in the GNP. The GNP is more volatile than net profits.
C. are used to estimate aggregate net sales for the firms in a stock market series such as the S&P 400.
About 40% of the variance in percentage changes in S&P 400 net profits can be attributed to
percentage changes in the GNP. Net sales are more volatile than GNP.
D. are used to estimate aggregate net profits for the firms in a stock market series such as the S&P
400. About 80% of the variance in percentage changes in S&P 400 net profits can be attributed to
percentage changes in the GNP. Net profits are more volatile than GNP.

Answer: C

Explanation:
There is a strong relationship between GNP percentage changes and S&P 400 net sales percentage
changes; about 40% of the variance of S&P 400 net sales percentage changes is due to percentage
changes in the GNP. Estimating GNP changes allows one to then estimate net sales changes based on
a regression formula.

Question: 3436

The infinite period dividend discount model assumes that

That answer is correct!


A. dividends grow at a constant rate, that constant rate will continue for an infinite period of time,
and that the required rate of return on the stock is greater than the growth rate. Making these
assumptions, the model states that the value of a stock is equal to the current dividend payment
divided by the spread between the required rate of return and the growth rate of dividends.
B. dividends grow at a constant or increasing rate, and that the required rate of return on the stock is
greater than the growth rate. Making these assumptions, the model states that the value of a stock is
equal to the dividend payout ratio divided by the spread between the required rate of return and the
growth rate of dividends.
C. dividends grow at a constant or increasing rate, and that the growth rate of dividends is higher
than the required rate of return on the stock. Making these assumptions, the model states that the
valueof a stock is equal to the dividend payout ratio divided by the spread between the required rate
of return and the growth rate of dividends.
D. dividends grow at a constant rate, and that constant rate will continue for an infinite period of
time. Making these assumptions, the model states that the value of a stock is equal to the dividend
payout ratio divided by the spread between the required rate of return and the growth rate of
dividends.

Answer: A

Page | 1362
Explanation:
The dividend discount model's fundamental assumption is that the value of a share of common stock
is equal to the present value of all future dividends. The assumption that a firm's dividends grow at a
constant rate for an infinite period of time is seldom true for companies growing at above-average
rates. For example, although Intel and Wal-Mart have both grown at rates in excess of 30% a year for
several years, it is doubtful that they will be able to maintain such high growth for an infinite period.

Question: 3437

Which of the following variables are hypothesized to affect the aggregate profit margin?

That answer is correct!


A. All of these answers
B. Rate of inflation
C. Unit labor costs
D. Foreign competition

Answer: A

Explanation:
Economists Finkel and Tuttle hypothesized that 4 variables affect the aggregate profit margin. (1)
Capacity utilization rate (+); (2) Unit labor costs (-); (3) Rate of inflation (+); (4) Foreign competition.

Question: 3438

A portfolio manager with Smith, Kleen & Beetchnutty is examining shares of Microscam for possible
investment. In her research, this portfolio manager has assimilated the following information about
Microscam from the Company's most recent fiscal year:

Adjusted operating profit before tax: $30,040,000


Cash operating taxes: $10,500,000
Cost of capital: 18.25% per year
Total capital employed: $98,750,000

Using this information, what is the Economic Value Added for Microscam? Further, should the
management of Microscam be considered to have created value for shareholders?

A. $1,743,071; management has created economic value


B. None of these answers is correct.
C. ($1,150,000); management has not created economic value
D. ($1,518,125); management has created economic value
E. $1,518,125; management has created economic value
F. $1,518,125; management has not created economic value

Answer: E

Explanation:
Economic Value Added, a value-based measure of economic profit, is a registered trademark of
Stern, Stewart & Company. The equation used to calculate EVA is as follows:

EVA = {Net Operating Profits Less Adjusted Taxes - [Total Capital Employed * (Cost of Capital)]}.

In this case, the NOPLAT figure must be calculated manually by subtracting the Cash Operating Taxes

Page | 1363
from the Adjusted Operating Profit before Tax (AOPBT) figure. Doing so will produce an answer of
$19,540,000 for the Net Operating Profit Less Adjusted Taxes figure. Now that the necessary
information has been determined, the calculation of EVA is as follows:

EVA = {$19,540,000 - ($98,750,000 * 0.1825)} = $1,518,125

A positive EVA calculation indicates that management has provided economic profits to
shareholders, as evidenced by the fact that the opportunity cost of capital employed is less than the
AOPAT figure. In this example, the management of Intelligent Semiconductor should be considered
as having succeeded to provide shareholder value.
The calculation of Adjusted Operating Profit Before Taxes (AOPBT), and Cash Operating Taxes are
important. The calculation of AOPBT is as follows:

Operating profit (after depreciation and amortization) + Implied interest on operating leases + any
increase in the LIFO reserve + goodwill amortization = Adjusted Operating Profit Before Taxes.

Cash Operating Taxes, another important component of EVA, is calculated as follows:


Cash Operating Taxes = Income Tax Expense + tax benefit from interest expenses + tax benefit from
interest on leases + taxes on non-operating income Adjusted Operating Profit Before Taxes minus
Cash Operating Taxes = Net Operating Profit Less
Adjusted Taxes (NOPLAT). Subtracting the dollar cost of capital from the NOPLAT figure will yield the
EVA.

Question: 3439

An investor purchases 500 shares of Microscam common stock for $58.74 per share at t0. At t1,
dividends of $1.20 per share are received on the 500 shares owned and the investor purchases an
additional 340 shares at $60.12 per share, At t2, dividends of $1.20 are received on the 840 shares
owned and he purchases another 60 shares for $55.14 per share. At t3, the investor sells 500 shares
of Microscam for $61.25 per share and the other 400 shares for $62 per share. No dividend are
received at t3. Assuming no taxes or commissions, what is the dollar-weighted rate of return for this
investment?

A. The answer cannot be calculated from the information provided.


B. None of these answers is correct.
C. 6.03%
D. 4.55%
E. 2.94%
F. 5.23%

Answer: E

Explanation:
Remember that the dollar-weighted rate of return uses the IRR equation in the determination of the
answer. Further, the dollar-weighted rate of return is another name for the IRR equation, and this
nomenclature is commonly used within the field of investment management. In the determination
of the dollar-weighted rate of return calculation, the first step should be to identify the cash flows for
each period. This process is illustrated as follows:

t0: {-[500 shares purchased * $58.74 per share] = ($29,370.00)


t1: {-[340 shares purchased * $60.12 per share] + [$1.20 per share dividend * 500 shares] =
($19,840.80) t2: {-[60 shares purchased * $55.14 per share] + [$1.20 per share dividend * 840
shares]} = ($2,300.40) t3: {[500 shares sold * $61.25 per share] + [400 shares sold * $62 per share] =
$55,425.00

Page | 1364
Now that the cash flows have been determined, incorporating this information into your calculator's
cash flow worksheet and solving for IRR will yield a dollar-weighted rate of return of 2.94% for this
investment.

Question: 3440

Two firms, A and B, have identical operations set up (except for their relative sizes). A has a debt-to-
equity ratio of 0.4 while B has it at 0.3. A's profit margin is 1.2 times that of B but its sales force is not
as effective as that of B. Hence, A's asset turnover is 0.9, compared to B's ratio of 1.15. A pays out
45% of its earnings as dividends while B pays out only 20%. Given this, the ratio of A's growth rate to
B's growth rate must be:

A. 1.23
B. 0.70
C. 1.44
D. 0.88

Answer: B

Explanation:
Use g = ROE * retention ratio and
ROE = profit margin * asset turnover * financial leverage.
Financial leverage = assets/equity = 1 + debt/equity.
The ratio of A's growth rate to B's growth rate equals 1.2*(0.9/1.15)*[(1+0.4)/(1+0.3)]*[(1-0.45)/(1-
0.20)] = 0.70

Question: 3441

The value of a preferred stock is the stated ________ divided by the required rate of return on the
preferred stock.

A. monthly portfolio return


B. monthly value estimate
C. annual dividend
D. daily dividend

Answer: C

Explanation:
The value of preferred stock is the stated annual dividend divided by the required rate of return on
preferred stock.

Question: 3442

Which of the following is/are true about a firm's stock?

I. A decrease in the beta of a stock will raise its price.


II. An increase in the variance of a stock will raise its price.
III. An increase in the market's expected return will raise its price.
IV. An increase in the risk aversion of investors will raise its price.

A. III only
B. IV only

Page | 1365
C. II only
D. II only
E. II & III
F. I & III
G. I only

Answer: G

Explanation:
A decrease in the beta of a stock decreases its expected return, causing its price to rise. On the other
hand, an increase in the market's expected return or an increase in risk aversion raises the expected
return on the stock, decreasing its price. Finally, an increase in the variance of the stock does not
necessarily change its expected return. Only if the systematic component of the stock's variance
changes will the expected return change, causing a price change.

Question: 3443

A high-growth firm is expected to have a dividend growth of 10% for the next 2 years. It is then
expected to stabilize at 4%. The firm has just paid a dividend of $2 and investors require a rate of
return of 14%. The market price of the firm's stock is:

A. $17.98
B. $21.26
C. $19.68
D. $23.46

Answer: B

Explanation:
Since the dividends do not grow at a constant rate, you cannot directly apply the Dividend Discount
Model valuation formula. However, note that 2 years from now, looking into the future, you will see
a constant growth rate of 4% and the dividend 3 years from now will be $2 * 1.1^2 * 1.04 = $2.517.
Therefore, the stock price 2 years from now, using the required rate of return of 14%, will equal P =
2.517/(14% - 4%) =
$25.17. Thus, the current stock price equals 1.1/1.14 + (1.1/1.14)^2 + 25.17/1.14^2 = $21.26.

Note that you must be very careful about the time line. In the Dividend Discount Model valuation
formula, the price at time t uses the dividend paid at time (t+1). That's the reason we had to use the
dividend paid in year 3 to calculate the price at the end of year 2.

Question: 3444

Alice Treehorn, a portfolio manager for a private investment firm, uses a specific methodology for
identifying investment opportunities. Specifically, Ms. Treehorn begins with an examination of
macroeconomic cycles and the overall global investment environment. After promising areas have
been identified, Ms. Treehorn then progresses in her analysis to an examination of governmental and
regulatory influences in each geographical region, as well as the prospect for specific industries in
each region.
Finally, the analysis concludes with an examination of specific securities.

Which of the following best characterizes the investment approach taken by Alice Treehorn?

A. Security valuation approach


B. Top down approach

Page | 1366
C. Root analysis
D. Macroeconomic cycle approach
E. Bottom up approach
F. None of these answers is correct.

Answer: B

Explanation:
The approach detailed in this example is best characterized as the "top down" approach. Securities
industry professionals commonly use the top down approach to identify investment opportunities,
and this method has a strong following in both the academic and professional arenas. The process
involved in the top down approach begins with an examination of the macroeconomic environment,
then progresses to governmental, security market analysis and industry analysis, and finally to an
examination of specific securities.
The top down approach is contrasted by the bottom up approach, which begins with an examination
of specific companies and their securities.

Question: 3445

Recent ________ is the biggest external factor that affects the growth rate of the depreciation
expense series.

A. interest
B. change in current levels
C. taxes
D. capital expenditures

Answer: D

Explanation:
One way to understand this is: depreciation is most acute with new assets; if one has recently
invested in new assets (i.e. capital expenditures), one would expect depreciation expense to
increase. But know well that capital expenditure may not be only new asset purchase.

Question: 3446

A stock paid a $5 per share dividend this year. The company's dividends are expected to grow at 5%
per year, forever. What is the value of the stock if the appropriate discount rate is 8% per year?

A. $17.50
B. $167.12
C. $175.00
D. Not able to compute with the above data.

Answer: C

Explanation:
Value = $5.25/(0.08-0.05) = $175.00

Question: 3447

Estimates of earnings per share for a market series will consider which of the following factors?

A. Sources for an estimate GNP

Page | 1367
B. Operating profit margin for the series
C. All of these answers
D. Depreciation and interest expense per share

Answer: C

Explanation:
To estimate expected earnings per share considering an outlook for the aggregate economy and for
the corporate sector, it is necessary to estimate GNP and sales per share, the operating profit margin,
depreciation, interest expense and the corporate tax rate per share for the next year.

Question: 3448

The first thing one does using the top-down, three step approach to stock valuation is

A. analyze specific company performance. One undertakes such analysis to find the best performing
companies in a particular industry. After finding industry favorites, one would then examine which
industries look promising.
B. analyze the general economic influences affecting a national or regional economy. Such analysis
includes research on fiscal policy, monetary policy (with its multiplier effect), inflation, and other
events such as wars or political upheavals.
C. analyze the general economic influences affecting a national or regional economy. There is a
particular focus on inflation because of the important multiplier effect that inflation has on the rest
of the economy, particularly aggregate demand.
D. analyze the general economic influences affecting a national or regional economy. There is a
particular focus on government monetary policy because of the important multiplier effect that
monetary policy has on the rest of the economy.
E. analyze the general economic influences affecting a national or regional economy. Such analysis
includes research on fiscal policy (with its multiplier effect), monetary policy, inflation, and other
events such as wars or political upheavals.

Answer: E

Explanation:
The first step in the top-down, three-step approach to valuation is analysis of alternative economies
and securities markets in order to decide how to allocate investment among different countries and
securities. That includes analysis of fiscal and monetary policies, inflation, and other events such as
wars and political upheavals that might affect these decisions.

Question: 3449

An analyst with Smith, Kleen, & Beetchnutty is trying to determine an earnings per share (EPS)
estimate for a health-care index, and has gathered the following information:

Sales per share: $600


Next year's operating profit margin: 38%
Next year's depreciation per share: $95
Next year's interest expense: $81
Next year's common stock dividend: $1.50
Next year's corporate tax rate: 35%

Using this information, what is the EPS figure for this stock market series?

A. $104.73

Page | 1368
B. $32.83
C. $33.80
D. $128.80
E. The answer cannot be calculated from the information provided.
F. None of these answers is correct.

Answer: C

Explanation:
The estimation of EPS for a stock market series involves five steps. Specifically, to determine an
estimate of EPS for a stock market series, it is necessary to:

Estimate the sales per share


Estimate next year's operating profit (EBIDT), or operating profit margin
Estimate next year's depreciation per share
Estimate next year's interest expense per share
Estimate next year's corporate tax rate

Once estimates for these components have been determined, they are put into the following
equation:

EPS for a stock market series = {[(Sales per share * operating profit margin) - depreciation per share -
interest expense per share] * (1 - corporate tax rate).

Imputing the given information into this equation will yield the following:

EPS for a stock market series = {[($600 * 0.38) - $95 - $81] * (1 - 0.35)} = $33.80

If you chose $128.80, remember that the depreciation figure is not added back to the EPS
calculation. What we are looking at is an operating earnings after tax figure, not a cash-based figure.

If you chose $32.83, remember that common stock dividends are not incorporated into the EPS
figure, only interest payments on debt and preferred securities.

If you chose $104.73, remember to subtract the depreciation and interest expense figures from the
EBIDT figure, not from the sales figure itself. In other words, multiply the sales figure by the
operating profit margin, THEN subtract the depreciation and interest expense figures.

Question: 3450

The support level of a stock is the price at which the technician expects:

A. a substantial supply of the stock.


B. the stock price to break out of the rising trend channel.
C. a substantial demand for the stock.
D. a stable trading volume.

Answer: C

Explanation:
The support level can be thought of as a temporary floor on the stock price in the sense that if the
price falls below this level, additional demand will shore up the price.

Question: 3451

Page | 1369
A technical analyst with Churn Brothers brokerage is examining shares of Allcycles.com, believing
that the shares are overvalued. In his analysis, this technical analyst has gathered the following
information about Allcycles' common stock.

Net worth: $15,381,000


Number of common shares outstanding: 1,200,000
Current stock price: $20.30 per share
Required return: 15.35% per year
Expected growth rate: 12.80% per year
Next dividend: $0.15 per share
Earnings per share: $1.65

Using this information, what is the price-to-book ratio for Allcycles.com?

A. The answer cannot be calculated from the information provided.


B. 3.57
C. None of these answers is correct.
D. 15.81
E. 12.30
F. 1.584

Answer: F

Explanation:
The book value of a common stock is found by dividing the net worth of a company by the number of
outstanding shares. While there exist several different methods for calculating net worth, the most
simplistic involves subtracting total liabilities from total assets, giving us the shareholder's equity
figure.

Investment professionals often see the book value as a "floor" for common stock prices, and the
price-to-book ratio is quite popular in the field of investment management. The calculation of the
price-to-book ratio involves the following equation:

Price-to-book ratio = {P0 / (Net worth / # of common shares outstanding)}

In this example, all of the necessary information has been provided. Imputing these figures into the
priceto-book value equation will yield the following:

Price-to-book value = {$20.30 / ($15,381,000 / 1,200,000)} = 1.584.

As you can see, the required rate of return, along with the expected growth rate, is not necessary in
the calculation of the price-to-book ratio.
Any time the price-to-book ratio is greater than one, a stock is said to be "trading at a premium to
book." Frequently, investment professionals will use the term "intrinsic value" when referring to the
book value. A figure of less than one for the price-to-book ratio is commonly referred to as "trading
at a discount to book." Stocks trading below book value are often sought after by value investors,
who believe these shares have been discounted below their liquidation value.

While the price-to-book value is a useful tool in the stock selection process, it possesses some
important flaws. One problem with the Price-To-Book Ratio is that one of the terms - Book Value - is
so easily manipulated. Valuation of inventory and real estate are easily adjusted on the books. Stock
buy-backs and write-offs of exceptional items also deflate Book Value, making high priced stocks
seem overvalued.

Page | 1370
Question: 3452

Holding everything else equal, which of the following firms would likely have a high payout ratio?
Further, as time progresses (in the long run), would the retention ratio of similar firms be expected to
increase or decrease?

A. Automobile manufacturer; increase


B. Specialty retailer; decrease
C. Pharmaceutical firm; decrease
D. Specialty retailer; increase
E. Automobile manufacturer; decrease
F. Pharmaceutical firm; increase

Answer: E

Explanation:
Remember that a positive relationship exists between the maturity of an industry and the payout
ratio of firms within that industry. The automobile industry is a mature industry, more so than most
other industries including pharmaceuticals or specialty retailers. As an industry advances in maturity,
growth of the overall industry will decline. As growth opportunities diminish, companies within the
industry will be forced to pay out a larger proportion of their earnings as dividends; i.e. the dividend
payout ratio of firms within the
industry will increase. Remember that the retention ratio is equal to (1 - the dividend payout ratio).
Thus, the retention ratio of companies will likely decline as the industry advances in maturity. The
relationship between the dividend payout ratio and the maturity of the industry is negative and
loosely linear. As an industry becomes more mature, growth opportunities decline. This relationship
is also loosely linear.

Question: 3453

Which of the following is the formula for the value of preferred stock?

A. dividend multiplied by the required rate of return


B. dividend divided by the required rate of return
C. required rate of return divided by the dividend
D. the present value of an infinite stream of dividends

Answer: B

Explanation:
The value of preferred stock is the stated annual dividend divided by the required rate of return on
preferred stock.

Question: 3454

At which stage in an industry life cycle would growth most likely be able to be estimated most
accurately?

A. pioneering development
B. mature growth
C. deceleration of growth and decline
D. rapid accelerating growth
E. stabilization and market maturity

Page | 1371
Answer: E

Explanation:
During this stage, the industry growth rate matches the growth rate of the segment of the economy
of which the industry is a part. Sales are highly correlated with an economic series.

Question: 3455

Which of the following usually occurs at the beginning of an economic recovery (after a recession)?

A. Profit margin decreases.


B. Productivity decreases.
C. Unit labor costs increase slowly or decrease.
D. Unit labor costs increase greatly.

Answer: C

Explanation:
At the beginning of an economic recovery, capacity utilization increases from its low recession level.
Workers do not yet ask for much higher wages, and productivity increases, leading to a decrease or
only a slow increase in unit labor costs. Profit margin increases as a result of these trends.

Question: 3456

Which of the following measures can differ greatly among countries?

A. Specific components of ROE


B. Retention rates
C. Total asset/equity ratios
D. All of these answers

Answer: D

Explanation:
The differences in retention rate or the components of ROE in different countries result from
differences in accounting practices as well as alternative management performance or philosophy.

Question: 3457

Marie Vaszquez, a semiconductor analyst with Smith, Kleen & Beetchnutty, has been working on a
determination of an EPS figure for a semiconductor index. In her analysis, she has found the
following:

1. Regressing sales for the series against Nominal GDP, the sales figure for the index has been
estimated at: $22.14.

2. Analyzing capacity utilization rates, foreign competition, rates of inflation and unit labor costs, the
operating profit margin for the series has been determined to be: 40%.

3. Creating a time series based upon inputs such as levels of capital expenditures and PP&E turnover,
next year's depreciation-per-share has been determined to be: $1.95.

4. Creating a time series based upon levels of debt outstanding and prevailing debt yields, the

Page | 1372
interest expense for next year is determined to be: $0.23 per share.

5. Coordinating his research with a legislative consultant, the corporate tax rate for this series has
been estimated at: 37.3%.

Using this information, what is the EPS figure for this stock market series?

That answer is correct!


A. None of these answers is correct.
B. $1.82
C. $4.42
D. $6.14
E. $2.49
F. The answer cannot be determined from the information provided.

Answer: A

Explanation:
The EPS figure for this series is found as $4.19. Thus, none of the answers is correct.
All of the necessary information has been provided in this example. To determine the EPS for a stock
market series, the following steps are necessary:

Step 1: Estimate sales-per-share for the series:


Step 2: Estimate operating profit margin for the series
Step 3: Estimate the depreciation-per-share for next year
Step 4: Estimate the interest expense-per-share for the next year
Step 5: Estimate next year's corporate tax rate

Once these five steps have been completed, the calculation of EPS for a stock market series is found
by the following:

EPS = [(Sales per share * Operating profit margin) - Depreciation-per-share - Interest Expense] * (1 -
Corporate Tax Rate)

The calculation of EPS for this stock market series is shown as follows: EPS = [($22.14 * 0.40) - $1.95 -
$0.23] * (1 - 0.373) = $4.19

Question: 3458

What is the first step in a top-down equity valuation approach?

That answer is correct!


A. Forecast the direction of general economy.
B. Select the security you think will perform best based on your economic and industry forecast.
C. Project the economic outlook for each industry under review.
D. All steps would be taken simultaneously.

Answer: A

Explanation:
A projection of the general economic outlook is the first stage for a top-down approach.

Question: 3459

Page | 1373
The top-down, three-step approach to valuation has

A. been disproved by studies.


B. generally been supported by studies.
C. generally not been supported by studies.
D. been proven by studies.

Answer: B

Explanation:
The top-down, three-step approach to valuation has tended to be supported by various studies.
Studies have found that earnings and rates of return are significantly influenced by the economy,
industry, and market.

Question: 3460

Which measure may be interpreted as a measure of prospective growth rates in earnings and
dividends that are expected by the market?

That answer is correct!


A. Price/Earning Ratios
B. Dividend Yield
C. Return on Assets
D. Payout Ratio
E. Return on Equity
F. Debt to Equity

Answer: A

Explanation:
P/E ratios must be compared across an entire industry to view relative valuations. It is useful to
compare P/E ratios for individual firms with the industry average P/E ratio and P/E ratios of other
firms in the same industry.

Question: 3461

An analyst with Smith, Kleen, & Beetchnutty is attempting to value shares of an insurance company.
The insurance company has been growing at a very stable rate for much of the last decade, and is
expected to continue growing at a similar pace in the future. In determining the value of the
insurance company's common stock, assume the following information:

Required rate of return on equity: 12.75% per year


Expected dividend growth rate: 9.50% per year
Dividend at t0: $0.88

Using this information, determine the value per-share of this insurance company's common stock.

A. None of these answers is correct.


B. $29.65
C. $32.72
D. $28.55
E. The answer cannot be calculated from the information provided.
F. $24.12

Page | 1374
Answer: B

Explanation:
In this example, the growth rate of dividends is assumed to remain stable, allowing the use of the
Gordon Model. The Gordon Model is also known as the "constant growth dividend discount model"
and takes the following form:

P0 = [D1 / (r - g)]

Where

P0 = the price of common stock X at time 0

D1 = the expected dividend at t1

r = the required rate of return on equity investments and g = the expected growth rate of dividends.

Since the dividend at t1 is not provided, we must calculate it manually by multiplying the dividend at
t0 by (1 + g). This will produce an answer of $0.9636 at t1.
Now that the dividend at t1 has been determined, the given information can be put into the
equation provided, leading to the following series of calculations:

P0 = [$0.9636 / (.1275 - .095)] = $29.65.

When using the Gordon model, remember that the required rate of return "r" must be greater than
the expected growth rate "g." Otherwise, the equation will produce a nonsensical answer.

Question: 3462

If a stock that you are holding for one year has an estimated dividend payout of $1.10 and an
expected sale price of $22, what is the value of the stock?

That answer is correct!


A. not enough information to calculate it
B. $22
C. $23.10
D. $20.26

Answer: A

Explanation:
In order to value the stock, a discount rate must be provided.

Question: 3463

The stock market tends to reach a ________ shortly after times of international crisis or a ________
of the T-Bill-Eurodollar spread.

A. trough; narrowing
B. peak; widening
C. peak; narrowing
D. trough; widening

Answer: D

Page | 1375
Explanation:
An alternative measure of investor attitude or confidence on a global basis is the spread between T-
bill yields and Eurodollar rates. It is reasoned that, at times of international crisis, this spread widens
as money flows to safe-haven U.S. T-bills, which cause a decline in this ratio. The stock market tends
to reach a trough shortly thereafter.

Question: 3464

The technicians' interpretation of the confidence index assumes that changes in the yield spread are
caused

That answer is correct!


A. almost entirely by changes in investor demand.
B. in equal parts by changes in the supply of bonds and changes in investor demand.
C. mostly by changes in the supply of bonds.
D. almost entirely by changes in the supply of bonds.

Answer: A

Explanation:
Technicians interpret the confidence index as indicative of broader investment bullishness or
bearishness toward the market. But changes in the relative yields of bonds (which determine the
value of the index) can be caused by changes in the supply of bonds as well as by changes in investor
demand.

Question: 3465

Which of the following is not one of the reasons why the valuation of common stock is difficult?

That answer is correct!


A. Uncertainty of the risk-free rate of return
B. Uncertainty of size of returns
C. Uncertainty of the required rate of return
D. Uncertainty of time pattern of returns

Answer: A

Explanation:
The risk-free rate of return is widely known. It is simply the rate of return on risk-free U.S.
government bonds.

Question: 3466

The P/E ratio of a stock equals 7.1. The company has just released its earnings figures at $12.20 per
share. The firm's dividend payout ratio is 28%. If the current stock price is $100, what is its 1-year
expected return under the Dividend Discount Model?

That answer is correct!


A. 19.40%
B. none of these answers
C. 14.83%
D. 13.65%

Page | 1376
Answer: A

Explanation:
It is important to remember that the P/E ratio is the ratio of the current stock price and next year's
expected earnings. Therefore, the firm's expected earnings next year equal 100/7.1 = $14.09 per
share. Since the current earnings equal $12.2, the dividend growth rate equals (14.09/12.2 - 1) =
15.45%.

To get the 1-year expected return, first calculate the expected price next year. Under Dividend
Discount Model, the P/E ratio remains constant if the firm makes no policy changes and there are no
market disruptions. Therefore, price next year is expected to be $14.09*(1 + 15.45%)*7.1 = $115.45.
The dividend next year equals $14.09 * 0.28 = $3.95 per share. Thus, 1-year expected return (capital
gains + dividends) = (P1+D1)/Po = (115.45 + 3.95)/100 - 1 = 19.4%.

Question: 3467

Empire Builders is in need of capital to finance its current expansion plans. For this, it has decided not
to raise dividends for the next 4 years, maintaining them constant at $2 per share. Analysts expect
the growth rate after that to be about 3% per year. If the investors expect a 9% rate of return on the
stock, the market price of Empire Builders is ________.

A. $34.33
B. $27.61
C. $30.80
D. $32.23

Answer: C

Explanation:
Since the dividends do not grow at a constant rate, you cannot directly apply the Dividend Discount
Model valuation formula. However, note that 4 years from now, looking into the future, you will see
a constant growth rate of 3% and the dividend 5 years from now will be $2 * 1.03 = $2.06. Therefore,
the stock price 4 years from now, using the required rate of return of 9%, will equal P = 2.06/(9% -
3%) = $34.33. Thus, the current stock price equals $2*(1/1.09 + 1/1.09^2 + 1/1.09^3 + 1/1.09^4) +
34.33/1.09^4 = $30.80.

Note that you must be very careful about the time line. In the Dividend Discount Model valuation
formula, the price at time t uses the dividend paid at time (t+1). That's the reason we had to use the
dividend paid in year 5 to calculate the price at the end of year 4.

Question: 3468

Estimating company earnings per share is a function of the ________ and the ________.

A. firm's historical sales levels; relationship between sales and various relevant economic and
industry series
B. firm's internal performance; problems that might affect its future performance
C. firm's competitive strategy; the firm's relationship with the industry
D. sales forecast; estimated profit margin
E. historical earnings per share; historical earnings multiplier

Answer: D

Explanation:

Page | 1377
The future value of a stock is derived by predicting the stock's earnings per share and expected
earnings multiplier. Expected earnings per share is a function of the sales forecast and the estimated
profit margin.

Question: 3469

Which of the following is not a stage in the industry life cycle?

A. Mature growth
B. Mezzanine stage
C. Development
D. Market maturity and decline
E. Accelerating growth
F. Growth deceleration and decline

Answer: B

Explanation:
The industry life cycle is divided into five distinct stages. Specifically, the industrial life cycle
progresses from the development stage to an accelerating growth stage to a mature growth stage to
a market maturation and stabilization stage. Finally, the fifth stage of the industrial life cycle is
characterized by decelerating growth and sales decline.

As an industry or company progresses through the industrial life cycle, sales begin to grow rapidly
(accelerating growth) then slow considerably as the product or service begins to reach critical mass
(mature growth). During the mature growth stage, the industry or company grows at a slower pace,
until eventually growth begins to slow considerably (market maturity and stabilization) as the market
for the industry or company's products becomes more completely defined. An industry or company
typically reaches the fifth stage of the industrial life cycle, growth deceleration and decline, only after
many years of stabilization. For example, the automobile industry made automobiles widely available
in the early 20th century, and has experienced a period of stabilization for nearly 50 years. The
market for automobiles has arguably not progressed to the growth deceleration and decline stage, as
automobile sales are still experiencing single digit annual growth.

Question: 3470

Using the macroanalysis approach to estimating a company's earnings multiplier, the multiplier is
based on:

I. the dividend payout ratio


II. the required rate of return
III. the company's relationship to the industry
IV. the rate of growth
V. the estimated earnings per share
VI. the company's relationship to the market

A. II, III, IV
B. III, VI
C. I, II, IV
D. I, II, III, IV, V
E. III, VI

Answer: B

Page | 1378
Explanation:
The earnings multiplier, under the microanalysis approach, is estimated based on its three
components: the dividend payout ratio, the required rate of return, and the rate of growth. Under
the Macroanalysis approach, it is estimated from the relationships among the firm, its industry and
the market. The estimates derived from each approach are resolved to settle on one estimate.

Question: 3471

Dividends-R-Us has just paid a cash dividend of $3.10 per share. If the growth rate is expected to be
3% and the price of the stock is $12.45, the expected return on the stock is:

A. 24.90%
B. 28.65%
C. 25.65%
D. 27.90%

Answer: B

Explanation:
In the usual notation, the Dividend Discount Model gives Po = D1/(k-g). In this case, g = 3%, D1 =
Do*(1+g) = 3.1 * 1.03 = $3.193, Po = $12.45. Therefore, k = 28.65%

Question: 3472

A decline in this ratio below 30 percent is bullish, above 50 percent is bearish.

A. Mutual Fund Cash Positions


B. Short Sales by Specialists
C. Relative Trend
D. Dow Theory
E. Margin Debt
F. Block Uptick-Downtick Ratio
G. Odd-Lot, Short-Sales Theory
H. Diffusion Index

Answer: B

Explanation:
The normal ratio of specialists' short sales to the total amount of short sales on the NYSE has become
about 40 percent. Technicians view a decline in this ratio below 30 percent as a bullish sign because
it means that specialists are attempting to minimize their participation in short sales. In contrast and
increase in the proportion above 50 percent is a bearish sign.

Question: 3473

Given current earnings of $2.50 per share, an expected dividend growth rate of 12% and a P/E of
12.5, what is the value of the stock?

A. $28.62
B. $35.00
C. $37.45
D. None of these answers

Answer: B

Page | 1379
Explanation:
Value = 12.5 x ($2.50 * 1.12) = $35

Question: 3474

A portfolio manager with Churn Brothers Brokerage is examining the health care industry from the
perspective of relative valuation. In her analysis, the portfolio manager examines the relationship
between the historical earnings multiple of the health care industry and the historical earnings
multiple of the entire market using a time series forecast. Which of the following best characterizes
this method of estimating an earnings multiplier? Choose the best answer.

A. Microanalysis
B. Macroanalysis
C. Linear regression
D. Forbes method
E. Simulation analysis

Answer: B

Explanation:
The method profiled in this example is "macroanalysis," which is one of two methods for estimating
the earnings multiplier of an industry. Macroanalysis involves examining the historical relationship
between the earnings multiplier of an industry with that of the overall market. Macroanalysis
forecasts often use a time series.

The macroanalysis method is contrasted by microanalysis, which involves examining the variables
underlying the earnings multiplier - the required rate of return, the growth forecast, and the
dividend payout ratio. In microanalysis, these variables are examined for the industry and then
compare them with the values of these variables for the entire market.
"Simulation analysis" is a method of examining stand-alone risk, and the "Forbes method" is a
fictitious term. "Linear regression" does not represent the best choice.

Question: 3475

Which of the following affect the required rate of return on a stock?

That answer is correct!


A. All of these can affect the required rate of return.
B. Changes in the rate of inflation
C. Changes in the exchange rate
D. Changes in the real risk-free rate
E. Changes in the risk premium

Answer: A

Explanation:
The required rate of return is equal to the nominal risk-free rate (a function of the real risk-free rate
and the rate of inflation) plus the risk premium. Changes in the exchange rate usually are included in
the risk premium.

Question: 3476

Aggregate depreciation expense tends to

Page | 1380
A. decrease during recessions.
B. always decrease.
C. never decrease.
D. decrease during the peak of the business cycle.
E. decrease during the economic recoveries.

Answer: C

Explanation:
Because depreciation expense is an estimate of fixed cost expense related to total fixed assets, it
tends never to decrease because total fixed assets for a large number of firms (such as those in the
S&P 400) tend never to decrease. The relevant question is by how much depreciation expense
increases.

Question: 3477

Dwight and Clark (a large manufacturing firm) is a stable company reporting the following financial
information:

Earnings per share $1.50


Dividends per share $0.30
Net Income $10 million
Equity $50 million

Given the above information, calculate the company's expected dividend growth rate.

A. 80%
B. 20%
C. 1.6%
D. 16%
E. 33%
F. 8%

Answer: D

Explanation:
The expected dividend growth rate = (Retention Rate) x (Return on Equity). Retention Rate = 1 -
Payout Rate. Return on Equity = NI/E. Thus in this case the expected dividend growth rate = 1 -
(.3/1.5) x ($10 million/$50 million) = (1 - .2) x (.2) = .16 or 16%.

Question: 3478

Assume the following information about an international telecommunications company.

Retention rate = 0.91


Net income / sales = 0.21
Total assets / common equity = 3.125
Sales / total assets = 0.33

What is the expected annual growth rate of this firm's dividends?

A. 23.63%
B. 1.77%

Page | 1381
C. The answer cannot be determined from the information provided.
D. None of these answers is correct.
E. 28.71%
F. 19.71%

Answer: F

Explanation:
A popular model for determining the growth rate of dividends is the following: g = RR * ROE
Where: g = the expected growth rate of dividends, RR = the retention rate (this is equal to 1 -
dividend payout ratio), and ROE = the return on equity.

Although it may at first appear otherwise, all of the necessary information has been provided.
Remember the Du Pont decomposition process for ROE, which breaks down the ROE figure into the
following:

ROE = (Net Income / Sales) * (Sales / Total Assets) * (Total Assets * Common Equity)
Mathematically, this will break down into (Net Income / Common Equity), the ROE figure. The
calculation of the return on equity for this company is as follows:

ROE = [0.21 * 0.33 * 3.125] = 0.216563, or 21.66%.

Now that the ROE figure has been determined, the calculation of the growth rate of dividends is as
follows:

g = [(0.91) * 0.2166] = 0.197072, or 19.71%.

Question: 3479

If worker productivity increases by 4%, wages per hour increase by 4%, and hours worked increases
by 8%, what is the change in unit labor cost?

A. 4% increase
B. 8% increase
C. 4% decrease
D. 12% increase
E. There is no change

Answer: E

Explanation:
Per-unit labor cost is a function of the percentage change in hourly wages minus the percentage
change in productivity during some period of time. In this question, the increase in worker
productivity exactly offsets the increase in hourly wages.

Question: 3480

For an American investor, Japanese securities tend to have

That answer is correct!


A. higher financial, liquidity, and exchange rate risk than American securities.
B. lower financial and liquidity risk and higher exchanger rate risk than American securities.
C. higher financial and exchange rate risk and lower liquidity risk than American securities.
D. lower financial risk and higher liquidity and exchange rate risk than American securities.

Page | 1382
E. lower financial and exchange rate risk and higher liquidity risk than American securities.

Answer: A

Explanation:
Japanese firms use considerably more financial leverage than American firms, and therefore tend to
have higher financial risk. American capital markets are acknowledged to be the most liquid in the
world, followed closely by those in Japan and London. Japanese securities therefore tend have
higher liquidity risk than American securities. For an American investor, American securities have no
exchange rate risk because they are denominated in dollars, while Japanese securities are
denominated in a yen.

Question: 3481

An investor had invested in 50 shares of firm Z at the beginning of the year, when the stock price was
$56 per share. On September 18th, the stock underwent a 3-for-2 split. At year-end, the stock price
stood at $38. The investor realized a return of 17% during the year. His total dividend income during
the year was:

A. $284
B. $517
C. $426
D. $639

Answer: C

Explanation:
When the stock split, the investor was left with 50 * 3 / 2 = 75 shares. Hence, the year-end portfolio
value equaled 75 * 38 = $2,850. The original amount invested was equal to 50 * 56 = $2,800. Since
the investor realized a return of 17%, his total value inclusive of dividends at year-end was 2,800 *
1.17 = $3,276.
Therefore, the dividend income during the year was 3,276 - 2,850 = $426.

Question: 3482

Given that the risk-free rate is 7%, what is the value of a zero-coupon bond in which $10,000 will be
paid in 15 years?

A. $4,759
B. $4,522
C. $5,628
D. Not enough information
E. $3,624

Answer: D

Explanation:
In order to take the present value of the coupon and principal payments, one must know the
required rate of return on the bond. The required rate of return is equal to the risk-free rate plus the
risk premium. The risk premium is not given, so there is not enough information to answer the
question.

Question: 3483

Page | 1383
Firms in which of the following industries would likely have the highest earnings retention rates?
Further, would firms within this industry likely be financed primarily through debt or equity?

A. Automobile manufacturing; debt


B. Retail banking; equity
C. Pharmaceuticals; debt
D. Pharmaceuticals; equity
E. Retail banking; debt
F. Automobile manufacturing; equity

Answer: D

Explanation:
Firms in the pharmaceutical industry would likely retain a higher proportion of their earnings than
automobile manufacturers or retail banks. This is reasoned primarily by two factors. First,
pharmaceutical companies have high research and development costs, and their success depends
largely on the discovery of promising drugs. This requires that adequate funds be held within the
firm. Secondly, growth opportunities in the pharmaceuticals industry are much more abundant than
in either the automobile manufacturing or retail banking, and this is largely due to the maturity of
the industry as a whole.
Pharmaceutical companies typically have numerous positive NPV investment opportunities.
In terms of financing, pharmaceutical firms would be expected to have a capital structure weighted
more heavily toward equity than debt. As the cash flows of an industry become more certain and
stable, the level of debt financing should be expected to increase. Automobile manufacturers and
retail banks have very predictable cash flows, along with high degrees of leverage. Certainty of
revenues and earnings is a very important consideration in the financing decision, and firms whose
cash flows are both stable and easily forecasted will likely prefer a high degree of debt versus equity
financing. Another important note to consider is the fact that the balance sheet of a pharmaceutical
firm is more heavily weighted toward intangible assets than is a retail bank or an automobile
manufacturer. Intangible assets cannot easily be pledged as collateral for a loan, neither can
intangible assets be liquidated with any degree of certainty. A high proportion of intangible assets
within the asset structure of a firm is conducive to a low degree of debt financing.

Question: 3484

Which of the following usually occurs at the peak of the business cycle?

A. Wages decrease.
B. Profit margin increases.
C. Capacity utilization increases significantly.
D. Unit labor costs increase substantially.

Answer: D

Explanation:
During the peak of the business cycle, firms are already operating near full capacity, leading to very
small increases or declines in capacity utilization. Inflation tends to be high, resulting in workers'
demands for higher wages. Because firms now tend to be using marginal labor and production
facilities, productivity increases only slowly. These factors result in substantial increases in unit labor
costs, which along with the otherfactors, also help push profit margins lower.

Question: 3485

An financial analyst is in the process of measuring the annualized return of an investment portfolio.

Page | 1384
Consider the following information:

t0: purchase an initial 1 share of Microscam for $65.40


t1: purchase an additional 1 share of Microscam for $68.12 t1: receive a dividend of $0.75
t2: purchase an additional 1 share of Microscam for $75.95 t2: receive a dividend of $0.77
t3: sell 3 shares for $82.76 per share

Assuming no taxes or transaction costs, that dividends are not reinvested, and that each period
represents one year, what is the time-weighted rate of return per year on this portfolio?

A. 8.27% per year


B. The answer cannot be calculated from the information provided.
C. None of these answers is correct.
D. 10.73% per year
E. 14.43% per year
F. 8.92% per year

Answer: F

Explanation:
The time-weighted rate of return is the preferred method of return calculation in the investment
management industry, primarily because the return calculation it produces is not sensitive to
significant additions and withdrawals of funds from portfolios under examination. The calculation of
the time-weighted rate of return involves three steps, which are illustrated as follows:

Step 1:
Price the portfolio immediately prior to any significant additions or withdrawals. Separate the
portfolio into a series of subperiods based on the dates of cash inflows and outflows.

Step 2:
Calculate the holding period return for each subperiod.

Step 3:
Determine the annualized holding period return by linking or compounding the holding period return
of each subperiod. If the investment is for more than one year, use the geometric mean of the annual
returns as the time-weighted rate of return. If the investment is for less than one year, compound the
subperiod returns to obtain an annualized measurement.

To begin the process of determining the time-weighted rate of return, we would break the portfolio
up into the following series of cash flows. However, in this example, the cash flows are already
aggregated for us and we can move on to the next step: determining the holding period return for
each subperiod. This process is detailed as follows:

t1: [($68.12 ending price + $0.75 dividend received - $65.40 beginning price) / $65.40 beginning
price] = 5.306%.
t2: [($75.95 ending price + $0.77 dividend received - $68.12 beginning price) / $68.12 ending price] =
12.625%
t3: [$82.76 ending price - $75.95 beginning price / $75.95 beginning price] = 8.966%

Now that the holding period returns for each subperiod have been calculated, the determination of
the time-weighted rate of return can take place. Since the duration of these transactions exceeds one
year,we must take the geometric mean of the annual returns to obtain the time-weighted rate of
return. This is done by taking the cube root of [(1 + .05306) * (1 + .12625) * (1 + .08966], subtracting
1, and multiplying by 100%, which leads to a time-weighted rate of return of .0892 or 8.92%.

Page | 1385
If you chose 10.73%, remember that it is the geometric mean that is used in the time-weighted rate
of return calculation, not the arithmetic mean.

Question: 3486

The earnings multiple for next year's earnings must take into account the ________ of common
dividends.

A. country risk
B. expected growth rate
C. required return on equity
D. aggregate business risk

Answer: B

Explanation:
Assuming that the dividend payout ratio is to remain constant, growth in earnings can be related to
the firm's growth. It then logically follows that a faster growing firm, with faster increase in dividends
will be higher priced (i.e. high earnings multiple).

Question: 3487

There are two popular methods for estimating the earnings multiplier for an industry or a stock
market series, which of the following correctly lists these two techniques?

A. The top-down and the bottom-up approach


B. Discounted cash flow estimation, regression analysis
C. Scenario Analysis, Monte Carlo Simulation
D. More than one of these answers is correct
E. The direction of change and the specific estimate approach
F. Microanalysis and macroanalysis

Answer: D

Explanation:
More than one of these answers is correct. Specifically, the "direction of change" and the "specific
estimate" are methods of forecasting the earnings multiplier of a stock market series and
"microanalysis" and "macroanalysis" are used to forecast the earnings multiplier for an industry.
When estimating the earnings multiplier for an industry, there are two common methods - the
"direction of change" and the "specific estimate" approaches. The direction of change approach
begins with the present earnings multiplier for a stock market series and seeks to estimate both the
amount and direction of any change based upon changes in the components of the P/E ratio - the
required rate of return, the anticipated growth rate of dividends, and the dividend payout ratio.
This approach is contrasted by the specific estimate approach, which involves the estimation of a
specific value for the earnings multiplier based on a series of projections for values of the
components of the earnings multiplier components. When using this approach, analysts commonly
confine their estimates to a series of scenarios, typically a best case, worst case, and base case
scenario.
There are two common methods for estimating the earnings multiplier for an industry -
macroanalysis and microanalysis. Macroanalysis involves an examination of the relationship
between the earnings multiplier for an industry and the earnings multiplier for the market.
Microanalysis involves an estimation of the specific variables that influence the earnings multiplier,
including the required rate of return, the estimated growth rate, and the dividend payout ratio.

Page | 1386
While the top down and bottom up method are similar to macroanalysis and microanalysis,
respectively, they do not represent the best possible answer.
Specifically, the top-down and bottom up approaches are typically used to identify investment
opportunities, not for the estimation of an industry earnings multiplier.

Question: 3488

An economist with Smith, Kleen & Beetchnutty Institutional Brokerage has been examining a stock
market series and is trying to determine an appropriate earnings multiplier for the series. In her
research, this economist has determined the following information:

The annual dividend per share next year = $1.35


The earnings per share next year = $5.10
The anticipated growth rate of dividends is 12.5% per year
The anticipated growth rate of earnings is 14% per year
The required rate of return is 15.75% per year

What is the appropriate earnings multiplier for this stock market series? Further, what is the
appropriate value for this stock market series? Choose the best answer.

A. The answer cannot be determined from the information provided.


B. None of these answers is correct.
C. 14.93; $76.13
D. 7.74; $39.46
E. 15.13; $77.16
F. 8.14; $41.51

Answer: F

Explanation:
The appropriate earnings multiplier is found to be 8.14.

Estimating the earnings multiplier for a stock market series requires the estimation of each of the
following components:

The dividend payout ratio.

The required rate of return on common stock in the country/region/industry/sector bein analyzed.

The expected growth rate of dividends for the stocks in the country/region/industry/sector being
analyzed.

Once values for each of these components have been determined, they are imputed into the
following formula:

P/E = [D/E / (k - g)]

Where: P/E = the earnings multiplier, or Price-to-Earnings ratio, D/E = the dividend payout ratio at t1,
k = the required rate of return, and g = the anticipated growth rate of dividends.

In this example all of the necessary information has been provided. However, the dividend payout
ratio must be calculated based on the anticipated dividend at t1 and the projected EPS figure for t1.
The calculation of the dividend payout ratio is as follows:

Page | 1387
D/E = [$1.35 / $5.10] = 0.264706

Now that the dividend payout ratio has been determined, the appropriate earnings multiplier is
found as follows:

P/E = [0.264706/ (0.1575 - 0.125)] = 8.14

Notice that it is the anticipated growth rate of dividends, not the anticipated growth rate of earnings,
which is used in the determination of the earnings multiplier.

Multiplying this figure by the projected EPS will lead to an appropriate value of $41.51 for this series.

Question: 3489

An analyst with Smith, Kleen & Beetchnutty Securities is trying to determine the EPS for a stock
market series. Which of the following steps will not be involved in his EPS estimation?

A. Estimate next year's corporate tax rate for the series.


B. Estimate next year's interest-expense-per-share for the series.
C. Estimate the sales-per-share for the series.
D. Estimate the operating profit margin for the series.
E. All of these steps are required.
F. Estimate next year's depreciation-per-share for the series.

Answer: E

Explanation:
All of these answers are required steps in the estimation of the EPS figure for a stock market series.

When an analyst is trying to determine the EPS for a stock market series, the following steps
represent the preferred method:

Step 1: Estimate sales-per-share for the series


Step 2: Estimate the operating profit margin for the series
Step 3: Estimate the depreciation-per-share for the series for the next year
Step 4: Estimate the interest-expense-per-share for the next year
Step 5: Estimate the corporate tax rate for the next year

These steps will lead to an EPS figure, which can be multiplied by the appropriate earnings multiplier
(i.e. the price-to-earnings ratio) to determine the ending price for the stock market series.

Question: 3490

Given that an individual owns a common stock with a required rate of return of 15%, on which he
expects to receive a dividend of $5 after one year, after which time he will immediately sell it for an
expected price of $30, what is the value of the common stock to the individual?

A. $30.94
B. $30.43
C. $31.09
D. $35
E. Not enough information

Answer: B

Page | 1388
Explanation:
According to the dividend discount model with a finite horizon, the value of a common stock to the
owner is the present value of the future dividends that he will receive, and the present value of the
price for which he will sell the stock. In this question, the value of the stock will be 5/1.15 + 30/1.15 =
$30.43

Question: 3491

Which of the following represents a "smart money" technical indicator?

A. Futures traders bullish on stock index futures.


B. Breadth of market.
C. Short interest.
D. Diffusion Index.
E. None of these answers is correct.
F. Debit balances in brokerage accounts.

Answer: F

Explanation:
Of the choices listed, only "debit balances in brokerage accounts" is a smart money technical
indicator. Technical analysts view investors who leverage their portfolios through margin loans as
being sophisticated. So said, technical analysts see an increase in debit balances, i.e. an increase in
margin borrowing, as a bullish signal. Conversely, technical analysts would view a decline in margin
borrowingas a bearish sign. While this viewpoint is somewhat counter intuitive, the degree to which
technical analysts use aggregate margin debit balances to track the "smart money" is high.
"Breadth of market" refers to the measure of advancing versus declining issues. The Diffusion Index is
a measure of market breadth and is defined as the volume of advancing issues plus one-half of the
volume of unchanged issues, divided by the total number of issues traded. Short interest measures
the total volume of outstanding short positions, and the sentiment of futures traders is used by
contrarian technical analysts, who take a contra approach.

Question: 3492

A portfolio manager with Churn Brothers Brokerage is examining shares of a large industrial firm and
has gathered the following information:

Market discount rate: 13.75% per year


Observed Price/Earnings ratio: 15.43

Given this information, what is the Franchise Price/Earnings ratio for this large industrial firm?

A. The answer cannot be calculated from the information provided


B. 8.16
C. 22.70
D. 17.89
E. 13.56

Answer: B

Explanation:
The Franchise Factor method of value measurement is in many respects similar to EVA and MVA
calculations. When examining a company using the franchise value approach, the observed price-to-

Page | 1389
earnings ratio is broken down into its two components - (1) the "base P/E," which is based on the
Company's ongoing performance, and (2) a "franchise P/E" that is based on the expected value of
new and profitable business opportunities. This relationship is illustrated as follows:

Franchise P/E = Observed P/E - Base P/E

where the Base P/E equals the reciprocal of the market discount rate. For example, if the market
discount rate is 13.75%, the base P/E would be equal to 7.27273

In this example, all the necessary information has been provided, and the calculation of the
Franchise P/E is as follows:

Franchise P/E = (15.43 - 7.27273) = 8.15727

Question: 3493

The CBOE put/call ratio

A. is used by contrary-opinion technical analysts. They view a ratio of 1.00 or greater to be a bearish
sign. The value of the ratio has historically been in the 0.25 to 0.75 range, but has now moved
considerably higher.
B. is used by technical analysts who try to follow the "smart money." They view a ratio of .90 or
greater to be a bearish sign. The value of the ratio has historically been in the 0.30 to 0.85 range, but
has now moved considerably higher.
C. is used by contrary-opinion technical analysts. They view a ratio of 0.70 or less to be a bearish
sign. The value of the ratio has historically been in the 0.35 to 0.80 range, but has now moved
considerably higher.
D. is used by technical analysts who try to follow the "smart money." They view a ratio of 0.60 or less
to be a bullish sign. The value of the ratio has historically been in the 0.40 to 0.90 range, but has now
moved considerably lower.

Answer: C

Explanation:
The Chicago Board Options Exchange (CBOE) put/call ratio is used by contrary-opinion technical
analysts. A higher ratio of puts to calls indicates a bearish market sentiment, while a lower ratio
indicates the opposite. Contrary-opinion analysts believe that the sentiment is wrong, and assume
ahigher ratio value is bullish while a low value is bearish. Currently, ratio values of 0.70 or less are
viewed bearishly, while ratio values of 0.90 or greater are viewed bullishly.

Question: 3494

Common stocks that experience dividend growth than is consistently higher than their required rates
of return

A. are valued very highly using the infinite period Dividend Discount Model.
B. cannot be valued using the infinite period Dividend Discount Model.
C. are prime candidates for valuation using the infinite period Dividend Discount Model.
D. tend to underperform the market.

Answer: B

Explanation:
The infinite period Dividend Discount Model postulates that the current value of a common stock is

Page | 1390
equal to D1 / (k - g), where D1 is next period's dividend, k is the required rate of return, and g is the
growth rate of dividends. If the growth rate of dividends exceeds the required rate of return, the
value of the stock is shown to be negative, which is impossible. The infinite period Dividend Discount
Model cannot be used to value such stocks.

Question: 3495

If the 50-day moving average for the stock price crosses the 150-day moving average from below on
heavy volume, technical analysts would consider this to be

A. inconsequential.
B. a sign of a flat trend.
C. a bullish sign.
D. a bearish sign.

Answer: C

Explanation:
Technical analysts use moving averages of past stock prices as indicators of long-term trends. When
the 50-day moving average crosses the 150-day moving average from below on heavy volume, this is
viewed as a bullish sign possibly signaling a reversal in the declining price trend. In order for such a
crossing to occur, the stock price must be going though a recent uptrend, which is pulling up the 50-
day moving average faster than the 150-day moving average.

Question: 3496

An increase in the dividend payout ratio, a decrease in the required rate of return, and a slight
increase in the growth rate would

A. have an unpredictable affect on the earnings multiplier.


B. increase the earnings multiplier.
C. decrease the earnings multiplier.
D. slightly decrease the earnings multiplier.

Answer: B

Explanation:
The earnings multiplier is equal to Dp / (k - g), where Dp is the dividend payout ratio, k is the
required rate of return, and g is the growth rate. The changes specified in this question would
undoubtedly increase the earnings multiplier. This estimate is an example of the direction of change
approach, which begins with the current earnings multiplier and estimates the direction and extent
of change for the dividend payout ratio and the variables that influence the required rate of return
and the growth rate of dividends and earnings.

Question: 3497

As the covariance of a stock's return with the market portfolio increases, its price ______, all else
equal.

A. can be all of these answers


B. is not affected
C. decreases
D. increases

Page | 1391
Answer: C

Explanation:
As the covariance of a stock's return with the market portfolio increases, its betaincreases. This
increases its expected return in the CAPM framework, causing its price to decline.

Question: 3498

You are going to hold a stock for an infinite amount of time. The current dividend is $1 per share and
is expected to grow at 15% a year. Your long run required return is 20%. Using the infinite period
dividend discount model calculate the value of the stock.

A. none of these answers


B. $23.00
C. $25.60
D. $26.45

Answer: B

Explanation:
g = .15 k = .20 Dividend = 1.15 x $1.00 = $1.15
V = 1.15/(.20 - .15) = $23.

Question: 3499

A portfolio manager who uses the "top down" approach to investment evaluation would most likely
begin her examination with which of the following?

A. An examination of alternative industries.


B. None of these answers is correct.
C. An examination of governmental and regulatory influences.
D. An analysis of the macroeconomic environment.
E. More than one of these answers is correct.
F. An analysis of specific securities.

Answer: D

Explanation:
The top down approach typically begins with an examination of the macroeconomic environment,
then progresses to an examination of alternative economies and securities markets, then to an
analysis of specific industries, and finally to the examination of specific securities. The objective of
the top down approach is to first decide how to allocate investment funds among various economies
and styles of securities (bonds, equities, cash, etc.), then to decide which economies and industries
will prosper in the current environment. The objective of the last stage of the top down approach
(i.e. the examination of individual securities and companies) is to determine which companies within
the selected industries offer superior investment prospects (i.e. identify investments that are
undervalued).

Question: 3500

An analyst would buy when this ratio approaches 13 percent and sell when it approaches 7 percent.
This relates to:

A. Odd-Lot, Short-Sales Theory

Page | 1392
B. Mutual Fund Cash Positions
C. Block Uptick-Downtick Ratio
D. Margin Debt

Answer: B

Explanation:
Mutual funds tend to keep a portion of the investment portfolio in cash, specifically in the region of
13 to 7 percent, buying and selling cash positions to keep the cash portion in this band.

Question: 3501

If the earnings per share on the S&P 500 index increases by 16% and the earnings multiplier
decreases by 9%, the index will experience a change of:

A. +16%
B. -9%
C. +26.44%
D. +5.56%

Answer: D

Explanation:
The index value = earnings multiplier * earnings per share. Therefore, the change in index value
equals 1.16 * 0.91 - 1 = 5.56%

Question: 3502

An analyst with Smith, Kleen & Beetchnutty is trying to determine the earnings multiple of a stock
market series composed of firms in the basic materials business. In her research, the analyst has
gathered the following information:

D1: $1.10
EPS: $4.30
k: 13.75% per year
g: 10.50% per year

Using this information, what is the earnings multiplier of this stock market series? Further, is this
multiple realistic for firms in the basic materials business?

A. None of these answers is correct.


B. 7.88, no
C. 33.84, no
D. 7.88, yes
E. 10.54, no
F. 10.54, yes

Answer: D

Explanation:
To determine the earnings multiplier, or "P/E ratio," of a stock market series, use the following
equation: P/E = [(D1 / E1) / (k-g)

Where: D1 = the annual per-share dividend at t1, E1 = the EPS figure at t1, k = the required rate of

Page | 1393
return on common stock, and g = the expected growth rate of dividends.
In this example, all of the necessary information has been provided, and putting it into the equation
above will yield the following:

P/E of a stock market series = [($1.10*1.105 / ($4.30*1.105)) / (0.1375 - 0.105)] = 7.88

This is a rather low multiple, appropriate for slow growth industries. The fact that the index under
examination is a compilation of firms in the basic materials business substantiates this low multiple
because firms in this line of business are not expected to grow very rapidly. Further, firms in the basic
materials business are also likely to have a relatively high dividend payout ratio, as evidenced in this
example.

Question: 3503

Technical analysis

A. does not conflict with the efficient market hypothesis. Technical analysis claims that the market
moves in trends that can be predicted before they occur. The key to profiting from technical analysis
involves a consistent ability to use past and present market data to predict future trends.
B. does not conflict with the efficient market hypothesis. Technical analysts believe that information
is disseminated slowly, first to the professionals, and later to the great bulk of investors. This causes
stock prices to move in trends.
C. conflicts with the efficient market hypothesis. Technical analysis claims that the market moves in
trends that can be predicted before they occur. These trends are caused by the fact that information
is disseminated relatively quickly.
D. conflicts with the efficient market hypothesis. Technical analysts believe that information is
disseminated relatively slowly, first to the professionals, and later to the great bulk of investors. In
contrast, advocates of the efficient market hypothesis believe that information is spreads very
quickly and is reflected in stock prices soon after it is available.

Answer: D

Explanation:
Technical analysts believe that information is disseminated relatively slowly, causing the market to
move in trends. They key to profiting from technical analysis lies in finding existing trends and
exploiting them. The efficient market hypothesis holds that past market data (which is used to find
trends) is already reflected in current prices, making effective technical analysis impossible.

Question: 3504

If the overall price trend of the market has been down, the moving-average line would generally lie
________ current prices.

A. can't be determined without further information


B. below
C. on
D. above

Answer: D

Explanation:
Moving averages lag current trends because data used to calculate moving averages is from recent
past.

Page | 1394
Question: 3505

Assume the following information about a stock market series:

Observed beginning series value: 1677


Anticipated ending series value: 1890
Expected dividends during the period: $16.36
Required rate of return: 19.50% per year

What is the expected annual rate of return for this index? (Assume a one-year holding period.)

A. 12.14%
B. 14.79%
C. 10.40%
D. None of these answers is correct.
E. 11.73%

Answer: D

Explanation:
The anticipated rate of return for this stock market series is found as 13.68%. Thus, none of these
answers is correct.

To calculate the expected rate of return for a stock market series, the following information must be
known:

The beginning value for the series

The anticipated ending value for the series, and

The amount of any dividends and/or distributions during the period

Once this information has been determined, the expected return on a stock market index can be
found by employing the following equation:

E(R) = [(EV - BV + Div) / BV]

Where: E(R) = the expected return on the stock market series, EV = the anticipated ending value for
the series, BV = the observed beginning value for the series, and Div = the amount of any dividends
paid during the period.

In this example, all of the necessary information has been provided and the calculation of the
expected return on this stock market series is found as follows:

E(R) = [$1890 - $1677 + $16.36] / 1677 = 13.68%

This is significantly less than the required rate of return. Assuming that both the ending value and
dividend figure is accurate, investment in this stock market series is likely not warranted.

Question: 3506

If a stock price that was previously in a rising trend channel suddenly enters a flat trend channel,

A. most technical analysts would sell the stock on the presumption that it will next enter a declining

Page | 1395
trend channel. A declining trend channel is a general pattern of price decline, while a flat trend
channel is a pattern of unpredictable price fluctuation.
B. some technical analysts would sell the stock in case it will enter a declining trend channel. Most
analysts will keep the stock and see what occurs. A flat trend channel is a pattern of price stability.
C. most technical analysts would buy the stock on the presumption that it will soon re-enter the
rising trend channel. A rising trend channel is a general pattern of price increase, while a flat trend
channel is a pattern of unpredictable price fluctuation.
D. all technical analysts would sell the stock on the belief that it will almost certainly enter a
declining trend channel. A declining trend channel is a general pattern of price decline, while a flat
trend channel is a pattern of price stability.

Answer: B

Explanation:
A declining trend channel is a general pattern of price decline. A rising trend channel is a general
pattern of price increase, while a flat trend channel is a pattern of price stability. Most technical
analysts would keep a stock that has changed from being in a rising trend channel to a flat trend
channel because of the possibility of it re-entering the rising trend channel. If the stock price breaks
out on the downside, then the analysts would take that as a sign of an incipient declining trend
channel, and sell the stock.

Question: 3507

Consider the following annual growth forecasts for a common stock:

Growth in years 1-3 = 25%


Growth after year 3 = 15%

Assuming that the last dividend was $2.25 per share, and the required rate of return is 20% per year,
what is the value of this common stock?

A. $63.78
B. $58.23
C. $71.92
D. $65.82
E. None of these answers is correct.
F. $72.10

Answer: D

Explanation:
To determine the value of a common stock experiencing temporary supernormal growth, use the
following equation:

{V = {[d0 * (1 + gs)^1] / k} + {[d1 * (1 + gs)^2} + ... {dn * (1 + gs)^n} + {[dn * (1 + gs)^n * (1 + gn] / (k -
g)}/ (1 + k)^n}}

Where: V = the value of common stock at t0, d0 = the dividend at t0, d1 = the dividend at t1, dn = the
dividend at tn, gs = the supernormal rate of growth, gn = the normal rate of growth, n = the time
period "n", and k = the required rate of return.

In this example, there is a supernormal growth period of three years, during which the growth rate of
this common stock is expected to grow at 25% annually. After this period of supernormal growth, the
growth rate is anticipated to settle to a "normal" rate of 15%, and this rate is expected to remain

Page | 1396
stable indefinitely. The calculation of the value of this common stock is illustrated as follows:

{V = {[$2.25 * (1.25)^1] / (1.20)} + {[$2.25 * (1.25)^2] / (1.20)^2} + {[$2.25 * (1.25)^3] /


(1.20)^3}{{[$2.25 * (1.25)^3 * (1.15)^1]/ (0.20 - 0.15)}/ (1.20)^3}

which can be deduced to the following:

{V = [$2.34375 + $2.441406 + $2.543132 + $58.492025] = $65.82}

Question: 3508

Using the infinite period Dividend Discount Model, what is the value of a stock with the following
characteristics?

Current dividend $6
Long term dividend growth rate 8%
Required rate of return 12%

A. $96.00
B. $75.00
C. $14.25
D. $48.00
E. $162.00
F. $72.00

Answer: E

Explanation:
The infinite period DDM indicates that:

Value = (Dividend for period 1)/k-g where k is the required rate of return and g is the growth rate. In
this case, Value = ($6 x 1.08)/(.12-.08) = $6.48/.04 = $162.

Question: 3509

Genetree Labs, a small biotechnology company focused on human stem cell research, is best
characterized by which stage of the industrial life cycle? Further, what degree of earnings payout are
shareholders likely to require from Genetree Labs? Assume that Genetree Labs is exclusively a
research firm, and is in the process of developing its product line.

A. Pioneering and development stage, high payout ratio


B. Market stabilization stage, low payout ratio
C. Pioneering and development stage, low payout ratio
D. Accelerating growth stage, high payout ratio
E. None of these answers is correct.
F. Accelerating growth stage, low payout ratio

Answer: C

Explanation:
The industry life cycle is divided into five distinct stages. Specifically, the industrial life cycle
progresses from the development stage to an accelerating growth stage to a mature growth stage to
a market maturation and stabilization stage. Finally, the fifth stage of the industrial life cycle is
characterized by decelerating growth and sales decline. During the mature growth stage, sales are

Page | 1397
still growing, albeitslowly. During the last stage of the industrial life cycle, however, is characterizes
by a decline in annual sales.

As an industry or company progresses through the industrial life cycle, sales begin to grow rapidly
(accelerating growth) then slow considerably as the product or service begins to reach critical mass
(mature growth). During the mature growth stage, the industry or company grows at a slower pace,
until eventually growth begins to slow considerably (market maturity and stabilization) as the market
for the industry or company's products becomes more completely defined. An industry or company
typically reaches the fifth stage of the industrial life cycle, growth deceleration and decline, only after
many years of stabilization.

The firm profiled in this example is in the pioneering and development stage, as evidenced by the
Company's complete lack of sales or product line. Genetree Labs is in the process of developing a
product, and is exclusively a R&D firm. So said, this Company is an example of the pioneering and
development stage of the industrial life cycle. Shareholders of Genetree will likely expect a low
dividend payout ratio, giving up current income for the prospect of future earnings growth, i.e.
shareholders will prefer that earnings are "plowed back" into the operation of Genetree rather than
be distributed as dividends. This is augmented by the fact that firms in the pioneering and
development stage are likely to have superior investment opportunities, which leads directly to a
lower payout ratio.
If you chose "accelerating growth," remember that the firm profiled in this example has no sales or
existing product. Genetree Labs is in the development stage.

Question: 3510

If the dividend payout ratio for a stock market series is anticipated to increase as the industry
advances towards relative maturity, which of the following would occur assuming that both k and g
remain constant? Further, what would occur if the growth rate of dividends were to exceed the
required rate of return?

A. The earnings multiplier would decrease; the earnings multiplier would produce a nonsensical
(negative) answer.
B. The earnings multiplier would decrease; the earnings multiplier would increase.
C. The earnings multiplier would increase; the earnings multiplier would decrease.
D. The earnings multiplier would increase; the earnings multiplier would produce a nonsensical (very
large) answer.
E. The earnings multiplier would increase; the earnings multiplier would produce a nonsensical
(negative) answer.

Answer: E

Explanation:
Remember that the equation used to determine the appropriate earnings multiplier for a stock
market series is the following:

{P/E = [D/E / (k - g)]}

Where: P/E = the earnings multiplier, or Price-to-Earnings ratio, D/E = the dividend payout ratio at t1,
k = the required rate of return, and g = the anticipated growth rate of dividends.
According to this equation, an increase in the dividend payout ratio will lead to an increase in the
earnings multiplier, assuming that both the required rate of return and the anticipated growth rate
remain unchanged. In reality, this assumption is nonsensical in this case. Specifically, the dividend
payout ratio is anticipated to increase because the industry is becoming more mature. In other
words, companies comprising the series are expected to pay out more of their earnings as dividends

Page | 1398
due to diminishing positive NPV investment opportunities. By definition this would decrease the
anticipated growth rate, which would in turn lead to a decline in the earnings multiplier.
If the anticipated growth rate were to exceed the required rate of return, the resulting earnings
multiplier would be a negative number, a result that doesn't make sense.

Question: 3511

A high-growth firm is expected to have a dividend growth of 15% for the next 2 years. It is then
expected to stabilize at 5%. The firm has just paid a dividend of $1 and investors require a rate of
return of 12%. The market price of the firm's stock is ________.

A. $16.15
B. $14.22
C. $17.77
D. $19.86

Answer: C

Explanation:
Since the dividends do not grow at a constant rate, you cannot directly apply the Dividend Discount
Model valuation formula. However, note that 2 years from now, looking into the future, you will see
a constant growth rate of 5% and the dividend 3 years from now will be $1 * 1.15^2 * 1.05 = $1.39.
Therefore, the stock price 2 years from now, using the required rate of return of 12%, will equal P =
1.39/(12% - 5%) = $19.86. Thus, the current stock price equals 1.15/1.12 + (1.15/1.12)^2 +
19.86/1.12^2 = $17.77.
Note that you must be very careful about the time line. In the Dividend Discount Model valuation
formula, the price at time t uses the dividend paid at time (t+1). That's the reason we had to use the
dividend paid in year 3 to calculate the price at the end of year 2.

Question: 3512

Technical analysts believe that speculative trading peaks at market ________.

A. mid-points
B. troughs
C. none of these answers
D. peaks

Answer: D

Explanation:
Technicians consider speculative activity high when the ratio of OTC volume on the NASDAQ system
to NYSE volume gets to 90 percent or more. Speculative trading typically peaks at market peaks.

Question: 3513

The change in ________ is a compound effect of changes in wages per hour and changes in worker
productivity.

A. the rate of inflation


B. unit labor costs
C. foreign competition
D. the profit margin

Page | 1399
Answer: B

Explanation:
Unit labor will cost more if wages increase and cost less if worker productivity increases. Both these
effects are considered.

Question: 3514

ABC Company has consistently paid out 40% of its earnings in dividends. The company's return on
equity is 16%. Calculate ABC's estimated dividend growth rate.

A. 40%
B. 16%
C. 6.4%
D. 10.0%
E. 12%
F. 9.6%

Answer: F

Explanation:
The estimated growth rate of dividends = (Retention Rate) x (Return on Equity). In this case, the
estimated growth rate of dividends = 60% x 16% = 9.6%.

Question: 3515

If the dividend payout ratio for a stock market series is anticipated to increase as the industry
advances towards relative maturity, which of the following would occur assuming that both the
required return and expected growth rate remain constant? Further, what would occur if the growth
rate of dividends were to exceed the required rate of return?

That answer is correct!


A. The earnings multiplier would increase; the earnings multiplier would produce a nonsensical
(negative) answer.
B. The earnings multiplier would increase; the earnings multiplier would produce a nonsensical (very
large) answer.
C. The earnings multiplier would decrease; the earnings multiplier would increase.
D. The earnings multiplier would decrease; the earnings multiplier would produce a nonsensical
(negative) answer.
E. The earnings multiplier would increase; the earnings multiplier would decrease.

Answer: A

Explanation:
Remember that the equation used to determine the appropriate earnings multiplier for a stock
market series is the following:

P/E = [D/E / (k - g)]

Where: P/E = the earnings multiplier, or Price-to-Earnings ratio, D/E = the dividend payout ratio at t1,
k = the required rate of return, and g = the anticipated growth rate of dividends.

From observing this equation, we can determine what would happen if the dividend payout ratio
were to increase. The result is largely intuitive once an understanding of dividend discount

Page | 1400
methodology is gained. An increase in the dividend payout ratio will lead to an increase in the
earnings multiplier assuming that both the required rate of return and the anticipated growth rate
are assumed to remain unchanged. In reality, this assumption is nonsensical in this case. Specifically,
the dividend payout ratio is anticipated to increase because the industry is advancing toward relative
maturation. In other words, companies comprising the series are expected to pay out more of their
earnings as dividends due to diminishing positive NPV investment opportunities. By definition this
would decrease the anticipated growth rate, which would in turn lead to a decline in the earnings
multiplier.

If the anticipated growth rate were to exceed the required rate of return, the resulting earnings
multiplier would be nonsensical. Specifically, the earnings multiplier produced would be a negative
number.

Question: 3516

The common stock of Blooth, Incorporated currently pays a $0.35 per share dividend, and this
dividend is anticipated to grow 13% annually. Assuming that investors require a 16% per year rate of
return on their investment, what is the value of this common stock?

A. The answer cannot be determined from the information provided.


B. $13.18
C. $19.01
D. None of these answers is correct.
E. $24.72
F. $16.83

Answer: B

Explanation:
To determine the value of this common stock using the Infinite Period Dividend Discount Model, the
following equation is used:

{P0 = [D1 / (k - g)]}

Where: P0 = the price of the common stock at t0, D1 = the annual dividend at t1 (this is found by
multiplying the dividend at t0 by (1 + expected growth rate), k = the required rate of return, And g =
the anticipated growth rate.

In this equation, all of the necessary information has been provided, and the calculation of the price
of this common stock is as follows:

{P0 = [(0.35 * 1.13) / (0.16 - 0.13)] = $13.18.

While the Infinite Period DDM is a useful tool, it frequently inappropriate for valuation purposes
because of its generous assumptions. Specifically, the Infinite Period DDM assumes that dividends
will grow at a steady rate indefinitely, and that this rate is both known and measurable. For most
common stocks, the use of the Infinite Period DDM will produce a measure of value that is skewed
from objective reality.
Nonetheless, this valuation model is frequently cited and represents a powerful tool within specified
parameters.

Question: 3517

Which of the following is/are true about open-ended funds?

Page | 1401
I. The redemption price equals the NAV in the absence of a redemption charge.
II. The load charge, if any, typically is a constant percentage of the amount invested.
III. The NAV of a no-load fund equals its purchase price.

A. III only
B. I & III
C. II & III
D. I & II
E. II only
F. I only

Answer: B

Explanation:
The offer price i.e. the price at which you can buy a share equals

NAV/(1-load charge) while the redemption price, in the absence of a redemption fee, equals the NAV.
The load charge, if any, typically declines as the size of the order increases.

Question: 3518

Contrarians interpret a high cash ratio in mutual funds at market lows as an indication that the
mutual fund managers are:

A. about to enter the market with more cash.


B. aggressive.
C. bullish.
D. bearish.

Answer: D

Explanation:
According to the contrarians, most market participants make wrong investment decisions as the
market approaches the peak or trough in a cycle.
One category of market participants they consider is mutual funds. The cash ratio of mutual funds is
the fraction of total assets that mutual funds maintain in the form of cash. A high ratio (around 12-
13%) at a market low is considered by contrarians as a signal that the funds are bearish and a low
ratio (7-8%) at a market high is interpreted as an indication that the funds are bullish. The
Contrarians then take exactly the opposite position

Question: 3519

A stock has a beta of 0.9 and the risk-free rate is 5%. Its dividend growth rate is 2.2% and the
dividend payout ratio is 55%. If the market risk premium is 8%, the P/E ratio of the stock equals
________.

A. 5.2
B. 4.9
C. 6.1
D. 5.5

Answer: D

Page | 1402
Explanation:
Using CAPM, the expected return on the stock equals 5% + 0.9 * 8% = 12.2%. Using the Dividend
Discount Model, P/E = (dividend payout ratio)/(K - g). This gives P/E = 0.55/(12.2% - 2.2%) = 5.5

Question: 3520

ABC (a large manufacturer of farm equipment) is a stable company reporting the following financial
information:

Earnings per share $1.50


Dividends per share $0.50
Net Income $12 million
Equity $50 million

Given the above information, calculate the company's expected dividend growth rate.

A. 8%
B. 20%
C. 33%
D. 16%
E. 80%
F. 1.6%

Answer: D

Explanation:
The expected dividend growth rate = (Retention Rate) x (Return on Equity). Retention Rate = 1 -
Payout Rate. Return on Equity = NI/E. Thus in this case the expected dividend growth rate = 1 -
(.5/1.5) x ($12 million/$50 million) = (1 - .33) x (.24) = .16 or 16%.

Question: 3521

This valuation technique breaks a firm's observed P/E down into two components: The P/E, based on
the company's ongoing business (its base P/E), plus a franchise P/E the market assigns to the
expected value of new and profitable business opportunities. It is known as:

That answer is correct!


A. the franchise factor
B. market value-added
C. economic value-added
D. none of these answers

Answer: A

Explanation:
The franchise P/E is a function of the relative rate of return on new business opportunities (the
franchise factor) and the size of the superior return opportunities (the growth factor).

Question: 3522

A firm has an expected dividend payout ratio of 40%, and an expected dividend growth rate of 4%
per year. What is the firm's Price/Earnings ratio if the appropriate discount rate is 8% per year?

That answer is correct!

Page | 1403
A. 10
B. Not able to compute with the above data.
C. 1
D. 100

Answer: A

Explanation:
Value = 0.40/(0.08-0.04) = 10.

Question: 3523

Given that the risk-free rate of return is 5%, what is the value of a zero-coupon bond with a principal
payment of $15,000 in 15 years, and a risk-premium of 5%?

A. $7,864
B. $3,591
C. $6,415
D. $9,249
E. Not enough information
F. $11,358

Answer: B

Explanation:
The value of a zero-coupon bond is the present value of its principal payments. The required rate of
return is the risk-free rate of return plus the risk premium (5 + 5 = 10%). Using appendix C in the
book by Reilly & Brown, the present value of the bond is $15,000 x 0.2394 = $3,591, or
$15,000/(1.1^15).

Question: 3524

If a firm has historically had a lower earnings multiplier than similar firms in its industry, which of the
following factors could be responsible for this?

I. the firm has maintained a higher than average payout ratio.


II. the firm's profit margin is lower than average.
III. the firm's stock has a high financial risk.

A. III only
B. I, II & III
C. II only
D. II & III
E. I only

Answer: B

Explanation:
Recall the Dividend Discount Model formula: P/E = payout ratio/(k - g) in standard notation. From this
formula, you may be misled into believing that increasing the payout ratio increases P/E. However,
remember that g = ROE*(1-payout ratio). The more the firm pays out, the lower its growth rate is.
Thus, it is not always necessary that P/E ratio will increase with increasing payout ratio. Indeed, in
Walgreen's case, a low payout ratio was associated with low P/E.

Page | 1404
If a firm has high financial risk, its required rate of return (k) is higher, depressing P/E.

Question: 3525

Due to an impending recession in the industrial and high tech sectors, combined with dramatic
mismanagement of U.S. fiscal policies, the U.S. economy is expected to slip into a significant
recession.
Given this shift, the U.S. inflation rate is expected to decrease significantly from its current level.
Specifically, the inflation rate is expected to decrease from 4.0% to a deflationary (2)% per year, and
this decrease should be considered significantly large by historical standards. The current nominal
interest rate in the U.S., as measured by the quoted rate on U.S. 10-year notes, is 8.25% per year.
Further, the real inflation-free rate of interest is currently at 4.25%, and this rate is not anticipated to
change. Assuming this decrease in inflation has not been factored in, what is the appropriate value
for the nominal risk-free rate?

That answer is correct!


A. 2.165% per year
B. The answer cannot be calculated from the information given.
C. (0.225)% per year
D. None of these answers is correct.
E. 2.25% per year
F. (2.165)% per year

Answer: A

Explanation:
When either the real "inflation-free" interest rate or the expected inflation rate are significantly
large, the calculation of the nominal risk-free rate differs from the equation used when these factors
are significantly small. Specifically, the calculation of the nominal risk-free rate of interest when the
inflation-free rate of interest and/or the inflation premium are significantly high, the calculation of
the nominal risk-free rate is as follows:

Nominal RFR = (1 + Real RFR)(1 + E(I)) - 1

Where: Real RFR = the real inflation-free rate of interest and E(I) = the anticipated inflation rate.

In this example, all of the necessary inputs have been provided. Imputing these values into the
equation above will yield the following:

Nominal risk-free interest rate = {[(1 + 0.0425)(1 - 0.02) - 1] * 100} = 2.165%

When the inflation-free rate of interest and/or the inflation premium are low, then the equation
above can be simplified to the following:

Nominal RFR = Real RFR + Inflation premium.

If you chose 2.25%, remember that when the real inflation-free rate of interest and/or the inflation
premium are significantly large, the calculation of the nominal risk-free rate must involve a different
equation than when these rates are small.

Question: 3526

A recent graduate of Atlantis University has been debating whether to invest in a popular retail stock.
In this research, this graduate has determined that his required rate of return is 15% per year, and

Page | 1405
that thecompany's current $0.45 per share annual dividend is expected to grow by 12.5% annually.
Additionally, the investor anticipates that he will be able to sell the common stock for $28 per share
in four years. What is the value of this common stock?

A. $18.00
B. The answer cannot be determined from the information provided.
C. $17.70
D. $31.40
E. The DDM will produce a nonsensical answer in this case.
F. $21.23

Answer: C

Explanation:
The Multiple Holding Period form of the Dividend Discount Model takes the following form: {V = {[d1
/ (1 + k)] + [d2 / (1 + k)^2] + ... .[dn / (1 + k)^n] + [Pn / (1 + k)^n]}
Where: V = the price of the common stock at t0, d1 = the annual dividend at t1 (this is found by
multiplying the annual dividend at t0 by (1 + the anticipated growth rate), d2 = the annual dividend
at t2 (this is found by multiplying the dividend at t1 by (1 + the anticipated growth rate), k = the
required rate of return, n = period "n", and Pn = the sale price of the common stock at time "n".

In this example, time "n" is the fourth year, as this is the end horizon for the investor's holding
period. Had the investor in this example forecasted selling the shares at the end of the 10th year,
then "n" would be the tenth year.

Now that the formality of expressing the equation for this form of the DDM has been carried
through, we can move toward a calculation of the value of this common stock. In this example, all of
the necessary information has been provided, and the calculation of the value of this retail stock is as
follows:

{V = [( $0.45* 1.125) / (1 + 0.15)^1] + [($0.50625* 1.125) / (1 + 0.15)^2] + [($0.56953 * 1.125) / (1 +


0.15)^3] + [($0.640723 * 1.125) / (1 + 0.15)^4] + [ $28/ (1 + 0.15)^4]}

which becomes

{V = [$0.440217 + $0.430647 + $0.421285 + $0.412127 + $16.00909] = $17.71

Question: 3527

What is the best proxy for the risk-free rate?

A. The rate on T-bills.


B. The rate on 20-year Treasury bonds.
C. The rate on 5-year Treasury notes.
D. The rate on AAA corporate bonds.

Answer: C

Explanation:
Treasury securities are essentially risk-free because of the seeming impossibility of a U.S.
government default. In the recent past, the 5-year Treasury note rate has been used as a proxy for
the risk-free rate because it is said to reflect the investment horizon of most investors.

Question: 3528

Page | 1406
Which of the following best characterizes the last stage in the industrial life cycle?

That answer is correct!


A. Growth deceleration and decline
B. Saturation
C. Insolvency
D. Obsolescence
E. None of these answers is correct.
F. Mature growth

Answer: A

Explanation:
The industry life cycle is divided into five distinct stages. Specifically, the industrial life cycle
progresses from the development stage to an accelerating growth stage to a mature growth stage to
a market maturation and stabilization stage. Finally, the fifth stage of the industrial life cycle is
characterized by decelerating growth and sales decline. During the mature growth stage, sales are
still growing, albeit slowly. During the last stage of the industrial life cycle, however, is characterizes
by a decline in annual sales.

As an industry or company progresses through the industrial life cycle, sales begin to grow rapidly
(accelerating growth) then slow considerably as the product or service begins to reach critical mass
(mature growth). During the mature growth stage, the industry or company grows at a slower pace,
until eventually growth begins to slow considerably (market maturity and stabilization) as the market
for the industry or company's products becomes more completely defined. An industry or company
typically reaches the fifth stage of the industrial life cycle, growth deceleration and decline, only after
many years of stabilization.

Question: 3529

You are going to hold a stock for 3 years. It is estimated to pay dividends of $2.50, $2.60 and $2.80.
The estimated sale price at the end of the holding period is $68. Using the dividend discount model,
calculate the value of the stock if your required rate of return is 14%.

That answer is correct!


A. $51.98
B. $48.60
C. $55.60
D. $46.82

Answer: A

Explanation:
Calculation = $2.50/(1+.14) + 2.60/(1+.14)^2 + 2.80/(1+.14)^3 + 68.00/(1+.14)^3 = $51.98.

Question: 3530

Due to an overheated economy and dramatic monetary stimulus, the U.S. inflation rate is anticipated
to increase significantly from its current level. Specifically, the inflation rate is expected to increase
from 3.5% to 8% per year, and this increase should be considered significantly large by historical
standards. The current nominal interest rate in the U.S., as measured by the quoted rate on U.S. 10-
year notes, is 6.75%. Further, the real inflation-free rate of interest is currently at 3.25% per year, and
this rate is not anticipated to change. Assuming this increase in inflation has not been factored in,

Page | 1407
what is the appropriate value for the nominal risk-free rate?

A. 9.51% per year


B. 10.85% per year
C. 11.51% per year
D. The answer cannot be calculated from the information provided.
E. 11.25% per year
F. None of these answers is correct.

Answer: C

Explanation:
When either the real "inflation-free" interest rate or the expected inflation rate are significantly
large, the nominal risk free rate is calculated using a different equation than that which is used for
lower expected inflation rates. Specifically, the calculation of the nominal risk-free rate of interest
when the inflation-free rate of interest and/or the inflation premium are significantly high, the
calculation of the nominal risk-free rate is as follows:

Nominal RFR = (1 + Real RFR)(1 + E(I)) - 1

Where: Real RFR = the real inflation-free rate of interest and E(I) = the anticipated inflation rate,

In this example, all of the necessary inputs have been provided. Imputing these values into the
equation above will yield the following:

Nominal risk-free interest rate = {[(1 + 0.0325)(1 + 0.08) - 1] * 100} = 11.51%

When the inflation-free rate of interest and/or the inflation premium are low, then the equation
above can be simplified to the following:

Nominal RFR = Real RFR + Inflation premium.

If you chose 11.25%, remember that when the real inflation-free rate of interest and/or the inflation
premium are significantly large, the calculation of the nominal risk-free rate must involve a different
equation than when these rates are small.

Question: 3531

Holding everything else equal, which of the following firms would likely have a high payout ratio?
Further, as time progresses (in the long run), would the retention ratio of similar firms be expected to
increase or decrease?

A. Pharmaceutical firm; decrease


B. Specialty retailer; increase
C. Automobile manufacturer; increase
D. Pharmaceutical firm; increase
E. Automobile manufacturer; decrease
F. Specialty retailer; decrease

Answer: E

Explanation:
Remember that a positive relationship exists between the maturity of an industry and the payout
ratio of firms within that industry. The automobile industry is a mature industry, more so than most

Page | 1408
other industries including pharmaceuticals or specialty retailers. As an industry advances in maturity,
growth of the overall industry will decline. As growth opportunities diminish, companies within the
industry will be forced to pay out a larger proportion of their earnings as dividends; i.e. the dividend
payout ratio of firms within the industry will increase. Remember that the retention ratio is equal to
(1 - the dividend payout ratio). Thus, the retention ratio of companies will likely decline as the
industry advances inmaturity. The relationship between the dividend payout ratio and the maturity
of the industry is negative and loosely linear.

As an industry becomes more mature, growth opportunities decline. This relationship is also loosely
linear.

Question: 3532

How much would you pay for a 15 year bond with a semiannual coupon rate of 6%, and a par value
of $15,000 if you want a 14% percent annual return on your investment? What would be the value of
the coupons and principal to you?

A. The value of the bond would be $15,024. The present value of the coupons would be $10,833, and
that of the principal would be $4,191.
B. The value of the bond would be $13,860. The present value of the coupons would be $11,457, and
that of the principal would be $2,403.
C. The value of the bond would be $17,239. The present value of the coupons would be $14,828, and
that of the principal would be $2,411.
D. Not enough information.
E. The value of the bond would be $13,140. The present value of the coupons would be $11,169, and
that of the principal would be $1,971.

Answer: E

Explanation:
The semiannual coupon payment would be 0.06 x 15000 = $900. Using Appendix C of "Investment
Analysis and Portfolio Management," by Reilly and Brown, one sees that the present value of annuity
of $1 for 30 periods at a required rate of 7% (half of 14%) would be 12.410. The present value of the
coupon payments is therefore 900 x 12.410 = $11,169. Using the same appendix, one sees that the
present value of $1 after 30 periods at a required rate or 7% is 0.1314. The present value of the
principal payment is therefore 15000 x 0.1314 = $1,971. The total value of the bond to you would be
11169 + 1971 = $13,140.

Question: 3533

Suppose the inflation rate in the United States is expected to increase from 3% to 4.25% per year in
the next year. Assume the current quoted risk-free rate of interest, as measured by the nominal rate
on U.S. Treasury 10-year notes, is 5.25% per year. Further, assume that the news of an increase in
inflation has not been factored into the risk-free rate. Given this information, what is the expected
effect in the nominal risk-free rate? Assume that the inflation-free rate of interest and the inflation
premium are not significantly large.

A. Risk-free rate will decrease by 125 basis points


B. The answer cannot be calculated from the information provided.
C. Risk-free rate will increase by 42 basis points
D. None of these answers is correct.
E. Risk-free rate will increase by 417 basis points
F. Risk-free rate will increase by 125 basis points

Page | 1409
Answer: F

Explanation:
Remember that the nominal risk-free rate of interest is comprised of two components, the real
"inflation-free" rate of interest, and an inflation premium. The inflation premium is equal to the
anticipated inflation rate.

The equation for the calculation of the nominal interest rate in situations where the real inflation-
free rate of interest and/or the inflation premium are low is as follows:

Risk-free rate of return = k* + IP

where: k* = the real inflation-free rate of return and IP = the inflation premium

In this example, the anticipated inflation rate, IP, has increased by 125 basis points. The effect of this
increase will be mirrored by an equal increase in the real-risk free rate of interest.

When either the real "inflation-free" interest rate or the expected inflation rate are significantly
large, the calculation of the nominal risk-free rate differs from the equation used when these factors
are significantly small. Specifically, the calculation of the nominal risk-free rate of interest when
theinflation-free rate of interest and/or the inflation premium are significantly high, the calculation
of the nominal risk-free rate is as follows:

Nominal RFR = (1 + Real RFR)(1 + E(I)) - 1

Where: Real RFR = the real inflation-free rate of interest and E(I) = the anticipated inflation rate

Question: 3534

________ = (1 + Real Growth) (1 + Expected Inflation) - 1.

A. Break-even rate
B. Nominal risk-free rate
C. Projected discount rate, PDR
D. Real risk-free rate

Answer: B

Explanation:
By definition: Nominal risk-free rate = (1 + Real Growth) (1 + Expected Inflation) - 1.

Question: 3535

________ analysis should precede ________ analysis.

A. Industry; economic
B. Company; industry
C. Industry; company
D. Company; economic

Answer: C

Explanation:
In general, an industry's prospects within the global business environment determine how well or

Page | 1410
poorly an individual firm will fare, so industry analysis should precede company analysis.

Question: 3536

________ techniques are based on the strong relationship between the economy and security
markets. Market projections are based on the outlook for the aggregate economy.

A. Micro
B. Macro
C. Fundamental
D. Technical

Answer: B

Explanation:
Analysis of both economies and securities markets are macro-techniques to aid investment decision
which includes deciding if certain asset classes or industries could be performing better or worse
than they are actually weighted/valued at.

Question: 3537

Assume the following information about a stock market series:

Retention rate at t1 = 63%


Expected growth rate of dividends at t1 = 10%
Expected growth rate of earnings at t1 = 12%
Required rate of return = 13%
EPS at t1 = $3.65

Given this information, what is the appropriate earnings multiplier for this stock market series?
Further, what is the value of this series?

A. 63; $229.95
B. 21; $76.65
C. 12.33; $45
D. 37; $135.05
E. None of these answers is correct.
F. The answer cannot be determined from the information provided.

Answer: C

Explanation:
The earnings multiplier is found as 12.33 for this series, and the value of the series is computed as
$45.

Estimating the earnings multiplier for a stock market series requires the estimation of each of the
following components:

1. The dividend payout ratio.

2. The required rate of return on common stock in the country/region/industry/sector being


analyzed.

3. The expected growth rate of dividends for the stocks in the country/region/industry/sector being

Page | 1411
analyzed.

Once values for each of these components have been determined, they are imputed into the
following formula: {P/E = [D/E / (k - g)]}. Where: P/E = the earnings multiplier, or Price-to-Earnings
ratio, D/E = the dividend payout ratio at t1, k = the required rate of return, and g = the anticipated
growth rate of dividends.

In this example, all of the necessary information has been provided. However, before the earnings
multiplier can be determined, the dividend payout ratio must be found by manipulating the
retention rate. Remember that the retention rate is equal to (1 - the dividend payout ratio), just as
the dividend payout ratio is found as (1 - the retention rate). By this logic, the dividend payout ratio
and the retention rate must always add up to one. The dividend payout ratio for this series is found
as (1 - 0.63) = 0.37.
Now that the dividend payout ratio has been determined, the calculation of the earnings multiplier is
shown below: {P/E = [0.37 / (0.13 - 0.10)] = 12.33}. Notice that it is the anticipated growth rate of
dividends that is used in this equation, not the anticipated growth rate in earnings.
Once the earnings multiplier has been determined, the value of this stock market series can be found
by multiplying the EPS figure by the earnings multiplier. This will lead to a value of $45 for this stock
market series, or 12.33 x 3.65 = $45.02.

Question: 3538

The best stock for investment purposes

That answer is correct!


A. is the one that is the most undervalued.
B. has the highest expected rate of return.
C. is the one issued by the best company.
D. has the least risk.

Answer: A

Explanation:
The best stock for investment purposes is not necessarily issued by the best company, because the
best company's stock may be overvalued. The stock with the highest expected rate of return may
have excessive risk, while the stock with the lowest risk may have an excessively low expected rate of
return. Rather, the stock that is the most undervalued is the best investment.

Question: 3539

Technical analysts do not expect ________ to be as abrupt as do fundamental analysts.

A. income adjustments
B. none of these answers
C. return adjustments
D. price adjustments

Answer: D

Explanation:
Technicians do not expect the price adjustment to be as abrupt as fundamental analysts and efficient
market supporters do, but expect a gradual adjustment to reflect the gradual flow of information.

Question: 3540

Page | 1412
The nominal risk-free rate of return is equal to

A. the federal funds rate + last period's rate of inflation.


B. (1 + the real risk-free rate of return) x (1 + the expected rate of inflation) - 1.
C. (1 + the real risk-free rate of return) x (1 + the federal funds rate) - 1.
D. the real risk-free rate of return + the expected rate of inflation.

Answer: B

Explanation:
The nominal risk-free rate of return is simply the real risk-free rate adjusted for inflation. The real
risk-free rate is the absolute minimum rate that any investor would require.

Question: 3541

Which of the following is a component of the risk premium?

I. Business risk
II. Financial risk
III. Liquidity risk
IV. Exchange rate risk
V. Country risk

A. III & V
B. II & IV
C. I, II, III, IV & V
D. I, II, III & IV
E. III & IV
F. IV & V
G. I & II

Answer: C

Explanation:
The risk premium is comprised of each of these components which are influenced by differences in
the general economic and political environment as well differences in trade relations and operating
leverage employed within a country. After each of these components is analyzed, a unique risk
premium may be assigned to a country.

Question: 3542

An increase in the required rate of return will have what effect on the earnings multipliers of
common stocks? Further, what effect could be expected from an increase in the dividend payout
ratio?

That answer is correct!


A. Earnings multipliers will decrease; earnings multipliers will increase
B. Earnings multipliers will increase; earnings multipliers will decrease
C. Earnings multipliers will remain unchanged; earnings multipliers will increase
D. Earnings multipliers will increase; earnings multipliers will increase
E. Earnings multipliers will decrease; earnings multipliers will decrease

Answer: A

Page | 1413
Explanation:
A simple method of determining the appropriate earnings multiplier for a common stock can be
found by manipulating the Infinite Period Dividend Discount Model such that it resembles the
following:

P/E = [(d1/e1) / (k - g)]

Where: P/E = the earnings multiplier expressed as the Price-to-Earnings Ratio, d1/e1 = the dividend
payout ratio at t1, k = the required rate of return, and g = the anticipated annual growth rate.

As you can see, an increase in the required rate of return, holding everything else equal, will lead to
a decrease in the earnings multiplier. Opposite this effect, an increase in the dividend payout ratio
will lead to an increase in earnings multipliers.

Question: 3543

________ techniques estimate future market values by applying basic valuation models to equity
markets.

That answer is correct!


A. Micro
B. Technical
C. Fundamental
D. Macro

Answer: A

Explanation:
The prefix 'micro' suggests that such techniques uses formulaic or hypothetical models to predict
future market values rather than judging by broad-based economic indicators.

Question: 3544

A stock price approaching its resistance level should

A. trade at low volume.


B. trade at high volume.
C. soon rebound from its decline.
D. be speeded along in its increase.

Answer: B

Explanation:
The resistance level is the price range at which technicians expect a substantial increase in the supply
of a stock. Technicians believe that after a stock has fallen from some previous higher price level,
investors who bought at that higher level will wait for the price to come back up again to unload the
stock at their break-even level. The stock must thus approach its resistance level from below, and
once it reaches it, the extra selling will result in high volume.

Question: 3545

Which of the following is/are true about mutual funds?

Page | 1414
I. Mutual funds have significant market timing abilities.
II. Mutual funds show a clear persistence of performance.
III. Mutual funds can be used as instant diversification vehicles.

A. II only
B. III only
C. I only
D. II & III
E. III only
F. II only
G. I, II & III

Answer: B

Explanation:
Empirical evidence convincingly shows that mutual funds do not have a consistent ability to time the
market downturns. Further, based on most empirical research, it is unlikely that mutual funds can
consistently produce superior results, though evidence exists that consistency was achieved over
some historical periods.

Question: 3546

Unit labor cost varies

That answer is correct!


A. positively with aggregate profit margin.
B. logarithmically with aggregate profit margin.
C. inversely with aggregate profit margin.
D. convexly with aggregate profit margin.

Answer: A

Explanation:
Finkel and Tuttle found that the capacity utilization rate, unit labor costs, rate of inflation, and foreign
competition were the four major variables that affected the aggregate profit margin. Unit labor cost
is most often the major variable cost for a firm. If it increases (holding sales revenues constant), one
would expect the profit margin to decline.

Question: 3547

The breadth of the market indicates:

A. The traded volume by sectors.


B. The total number of issues traded.
C. The number of advances versus declines in a day.
D. The range of instruments traded in the market.

Answer: C

Explanation:
The "breadth" of the market is used to refer to the amount of activity spread across all the
instruments traded (as against the activity being concentrated on a few stocks or instruments). One
measure of this breadth is given by the number of price increases and decreases over the previous
day. The difference between the two not only serves as a partial explanation of the change in market

Page | 1415
direction but also indicates whether a rise or fall in the market is pervasive or influenced by a
concentrated group of traded securities.

Question: 3548

This measure of value-added is closely related to the NPV technique. The annual performance of
management is evaluated by comparing the firm's net operating profit less adjusted taxes to the
firm's total cost of capital in dollar terms, including the cost of equity. It is known as:

A. None of these answers


B. All of these answers
C. Economic value-added
D. Market value-added
E. The Franchise Factor

Answer: C

Explanation:
In economic value-added (EVA) analysis, if the firm's net operating profit less adjusted taxes
(NOPLAT) during a specific year exceeds its dollar cost of capital, it has a positive EVA for the year and
has added value for its stockholders. If the EVA is negative, the firm has not earned enough during
the year to cover its cost of capital and the value of the firm has declined.

Question: 3549

The required rate of return on a security is determined by

A. three factors: the real risk-free rate of return, the expected rate of inflation, and the risk premium.
The real risk-free rate of return and the expected rate of inflation are used to arrive at the nominal
risk-free rate. The risk premium of a security is a function of its variance, and may well fluctuate over
time.
B. two factors: the real risk-free rate of return and the risk premium. The risk-free rate of return may
be influenced in the short-term by tightness or ease in the capital markets. The risk premium of a
security is a function of its variance, and tends to be stable over time.
C. two factors: the real risk-free rate of return, and the risk premium. The risk-free rate of return
should depend on the real growth rate of the economy because capital should grow at least as fast as
the economy. The risk premium of a security is a function of its risk relative to the market, and tends
to be stable over time.
D. three factors: the real risk-free rate of return, the expected rate of inflation, and the risk premium.
The risk-free rate of return (which along with inflation determines the nominal risk-free rate) may be
influenced in the short-term by tightness or ease in the capital markets. The risk premium of a
security is a function of its risk relative to the market, and may well fluctuate over time.

Answer: D

Explanation:
Although the long-term risk-free rate of return should depend on the real growth rate of the
economy, it may be influenced in the short-term by tightness or ease in the capital markets. The
nominal risk-free rate is equal to (1 + real risk-free rate) x (1 + inflation rate) - 1. The risk premium is
the extra yield that a security requires over the nominal risk-free rate to be attractive to investors.

Question: 3550

According to the contrarians, if a substantial number of investment advisory services are bearish, it

Page | 1416
implies:

I. the market is reaching a peak.


II. the onset of a market decline.
III. the beginning of a bull market.

That answer is correct!


A. III only
B. I & II
C. II only
D. I only

Answer: A

Explanation:
Contrarians believe that most market participants make wrong investment decisions as the market
approaches the peak or trough in a cycle. Therefore, if a large number of advisory services are
bearish, they believe that the market is about to turn bullish after having reached a trough.

Question: 3551

A restrictive monetary policy will not

A. raise firms' costs by raising interest rates.


B. increase firms' costs by decreasing interest rates.
C. have an influence on individual firms.
D. reduce the supply of funds for working capital and expansion available to firms.

Answer: B

Explanation:
A restrictive monetary policy often raises interest rates (and never lowers them). Such increases in
rates would increase the cost of borrowing for firms. A restrictive monetary policy may also reduce
the growth rate of monetary supply, and thus reduce the supply of funds for working capital and
expansion available to businesses.

Question: 3552

In estimating a firm's earnings multiplier, finding the firm's required rate of return involves analyzing
the firm's fundamental risk characteristics. Which of the following is not considered a fundamental
risk?

A. Business risk
B. Exchange rate risk
C. Financial risk
D. Country risk
E. Liquidity risk
F. Government risk

Answer: F

Explanation:
In addition to the consideration of fundamental factors, one should consider market-determined risk
(beta) based on the CAPM. A low beta combined with below-average fundamental risk, for example,

Page | 1417
would indicate that the firm's risk is quite low compared to the aggregate market. This means that
the risk premium and the required rate of return for this firm should be lower than the market. By
itself, this lower required rate of return would suggest an earnings multiplier above the market
multiplier.

Question: 3553

A portfolio manager with Churn Brothers Brokerage has recently been approached by one of its
institutional accounts requesting that the dollar-weighted rate of return for the Microspeculative
investment be calculated.

Consider the following series of transactions:

t0: Purchase 20,000 shares for $0.90 per share


t1: Purchase 50,000 shares for $1.13 per share
t2: Purchase 50,000 shares for $1.20 per share
t3: Sell 20,000 shares of for $1.22 per share
Sell 80,000 shares of for $1.20 per share
Sell 20,000 shares of for $1.17 per share

Ignoring commissions, what is the dollar-weighted rate of return for this investment?

A. (3.77%)
B. (1.20%)
C. 4.02%
D. The answer cannot be calculated from the information provided.
E. 4.94%
F. None of these answers is correct.

Answer: C

Explanation:
Remember that the dollar-weighted rate of return uses the IRR equation in the determination of its
answer. Further, the dollar-weighted rate of return is another name for the IRR equation, and this
nomenclature is commonly used within the field of investment management. The logic behind this
characterization is the fact that the IRR equation takes into account both the timing and scale of all
project cash flows. In the determination of the dollar-weighted rate of return calculation, the first
step should be to identify the cash flows for each period. This process is illustrated as follows:

t0: -[20,000 shares purchased * $0.90 per share] = [$18,000]


t1: -[50,000 shares purchased * $1.13 per share] ] = [$56,500]
t2: -[50,000 shares purchased * $1.20 per share] = [$60,000]
t3: [(20,000 shares sold * $1.22 per share) + (80,000 shares sold * $1.20) + (20,000 shares sold *
$1.17 per share)] = $143,800

Now that the cash flows have been determined, incorporating this information into your calculator's
cash flow worksheet and solving for IRR will yield a dollar-weighted rate of return of 4.02% for this
investment.

Question: 3554

Charles Ray, a portfolio manager with Bay Side Brokerage, is examining shares of a large money
center bank. In his analysis, Mr. Ray has determined that the $1.20 per share dividend of this
company isanticipated to grow at 14.5% annually. Additionally, Charles Ray has calculated his

Page | 1418
required rate of return as 16% per year. Assuming that Charles Ray can sell his shares of this bank for
$85 at the end of three years, what is the value of this common stock?

That answer is correct!


A. $57.96
B. $54.82
C. $63.88
D. $92.45
E. The Multiple Period DDM will produce a nonsensical answer for this stock.

Answer: A

Explanation:
The Multiple Holding Period form of the Dividend Discount Model takes the following form: {V = {[d1
/ (1 + k)] + [d2 / (1 + k)^2] + ... .[dn / (1 + k)^n] + [Pn / (1 + k)^n]}
Where: V = the price of the common stock at t0, d1 = the annual dividend at t1 (this is found by
multiplying the annual dividend at t0 by (1 + the anticipated growth rate), d2 = the annual dividend
at t2 (this is found by multiplying the dividend at t1 by (1 + the anticipated growth rate), k = the
required rate of return, n = period "n", and Pn = the sale price of the common stock at time "n".
In this example, time "n" is the third year, as this is the end horizon for the investor's holding period.
Had the investor in this example forecasted selling the shares at the end of the 9th year, then "n"
would be the ninth year.
Now that the formality of expressing the equation for this form of the DDM has been carried
through, we can move toward a calculation of the value of this common stock. In this example, all of
the necessary information has been provided, and the calculation of the value of this retail stock is as
follows:
{V = [($1.20 * 1.145) / (1 + 0.16)^1] + [($1.374 * 1.145) / (1 + 0.16)^2] + [($1.57323 * 1.145) / (1 +
0.16)^3] + [$85 / (1 + 0.16)^3]}

which can be further reduced to the following:

{V = [$1.184483 + $1.169166 + $1.154048 + $54.455902] = $57.96}

Question: 3555

James Clinton, a portfolio manager with Middle Road Investment Advisors, is trying to estimate the
appropriate earnings multiplier for the automobile industry. In his analysis, James examines the
expected growth rate of dividends for the industry, as well as the expected dividend payout ratio and
required rate of return. From this information, James proceeds toward an estimation of the earnings
multiplier for the series.

Which of the following best describes this method of estimating an earnings multiplier for an
industry?

That answer is correct!


A. Microanalysis
B. The specific estimate approach
C. The rate of change approach
D. Input-output analysis
E. Macroanalysis
F. The arbitrage pricing method

Answer: A

Page | 1419
Explanation:
The answer called for in this question is "microanalysis." This method involves an examination of the
components of the earnings multiplier, including the anticipated growth rate of dividends, the
required rate of return, and the dividend payout ratio. Once these variables have been examined,
both from the perspective of trend analysis and point estimation, a value for the earnings multiplier
is deduced.

Microanalysis is contrasted by macroanalysis, which involves an examination of the relationship


between the earnings multiplier of a stock market series and the earnings multiplier of the overall
market. For example, an individual projecting an earnings multiplier for a software index using
macroanalysis would begin by examining the relationship between the P/E ratio of the software
index and the P/E ratio of a broad market index such as the Standard & Poors 500. Both historical
trends and point estimates would be examined, and from this information, a projection of the
earnings multiplier for the stock market series is deduced.

Input-output analysis is used in the examination of competitive forces within an industry. The
arbitrage pricing model is a security valuation model which allows for several relevant "situation
specific" variables. Finally, the "specific estimate" and "rate of change" approaches are used to
estimate the earnings multiplier of a stock market series, rather than an industry.

Question: 3556

Which of the following is/are true about a closed-end fund?

I. Shares of the fund trade on an exchange.


II. The market price of the fund is determined by supply and demand.
III. It issues shares only infrequently.

A. I only
B. II only
C. I & III
D. III only
E. I, II & III
F. II & III
G. I & II

Answer: E

Explanation:
Shares of a closed-end fund trade on an exchange like those of any other firm, with the share price
determined by the laws of supply and demand. The fund issues new shares only infrequently, when
it needs additional capital.

Question: 3557

An analyst is involved in the top-down equity valuation approach. He decides to compare an


individual firm's performance within the entire industry using financial ratios and cash flow values.
What stage of the top-down approach is he engaged in?

That answer is correct!


A. Security selection.
B. He is performing all three stages.
C. General economic forecast.
D. Projected economic outlook for the industry.

Page | 1420
Answer: A

Explanation:
This is a description of the final stage, security selection.

Question: 3558

In contrast to ________, which generally is an evaluation of internal performance, ________ is a


measure of external performance.

A. economic value-added, the franchise factor


B. the price/cash flow ratio, the price/book value ratio
C. market value-added, economic value-added
D. economic value-added, market value-added

Answer: D

Explanation:
To properly analyze performance using either economic value-added or market value-added, it is
necessary to look for changes over time, the percent change each year.

Question: 3559

If the premium on the market portfolio increases, the price of a firm's share ________, all else equal.

A. is not affected
B. decreases
C. increases
D. can be all of these answers.

Answer: B

Explanation:
If the premium on the market portfolio increases, the expected return on the stock goes up, too. All
else equal, this can happen only if the price of the stock decreases.

Question: 3560

An investment advisor with Churn Brothers Brokerage states that he uses the "bottom up" approach
to identify promising investment opportunities. When this investment advisor is evaluating an
investment using the bottom up approach, he would begin by doing which of the following?

That answer is correct!


A. Examining individual securities and companies.
B. None of these answers is correct.
C. Analyzing alternative economies and security markets.
D. Analyzing the macroeconomic environment.
E. Examining alternative industries.
F. More than one of these answers is correct.

Answer: A

Explanation:

Page | 1421
The bottom up approach begins with an examination of individual securities and companies.
Individuals who advocate the bottom up approach believe that it is possible to identify lucrative
investment opportunities within various sectors despite macroeconomic and industry influences. In
the field of security analysis, there is commonly a confluence of the bottom up approach with
"technical analysis,"an approach that is based on the belief that it is possible to predict securities
prices by using historical data.
The bottom up approach is contrasted by the top down approach, which begins with an examination
of the macroeconomic environment, then progresses to an examination of alternative economies
and securities markets, then moves toward industry analysis and the examination of specific
securities.

Question: 3561

Studies of short-interest trading rules

A. have tended to support their effectiveness.


B. have strongly supported their effectiveness.
C. have had mixed results.
D. have not supported their effectiveness.

Answer: C

Explanation:
Technical analysts believe that the short-interest ratio is a bullish indicator; the higher the ratio, the
more bullish an indication for the market. But studies that have examined short-interest trading rules
have had mixed results. For every study that has supported the rules, there is one that has not.

Question: 3562

Ferengi Profiteers is a high-growth mining firm, which prefers to plow back most of its earnings into
on-going operations. As a result, it maintains a low dividend payout ratio of 12%. Its permanent
earnings are expected to grow at 6%. It announced earnings of $7.22 per share yesterday and its
investors are expecting a return of 9% on its stock. What's the trading price of each stock of Ferengi
Profiteers?

A. $32.44
B. $30.60
C. $28.12
D. $25.83

Answer: B

Explanation:
The earnings next year are expected to be 7.22 * 1.06 = $7.65 per share. So the dividend expected
next year equals 0.12 * 7.65 = $0.918 per share. The growth rate of dividend also equals 6%.
Therefore the price of a stock using the Dividend Discount Model equals P = 0.918/(9% - 6%) =
$30.60.

Question: 3563

Contrary-opinion rule technical analysts would consider

A. a high AMEX volume to NYSE volume ratio to be a sign that the market is approaching a trough.
This ratio is a measure of investor confidence in the economy. A high value indicates that investors

Page | 1422
have enough confidence to invest in the relatively volatile AMEX. Recovering confidence indicates
the approach of a market trough.
B. a high AMEX volume to NYSE volume ratio to be a sign that the market is overbought. This ratio is
a measure of speculative trading activity. A high value indicates a high level of speculation, which
itself is thought to indicate a market peak.
C. a high NASDAQ volume to NYSE volume ratio to be a sign that the market is overbought. This ratio
is a measure of speculative trading activity. A high value indicates a high level of speculation, which
itself is thought to indicate a market peak.
D. a high NASDAQ volume to NYSE volume ratio to be a sign that the market is approaching a trough.
This ratio is a measure of investor confidence in the economy. A high value indicates that investors
have enough confidence to invest in the relatively volatile NASDAQ. Recovering confidence indicates
the approach of a market trough.

Answer: C

Explanation:
Prior to the 1970s, the measure of speculative activity was typically the ratio of AMEX volume to
NYSE volume. But because of the decline in importance of the AMEX, this ratio is no longer a useful
measure. The new measure of speculative activity is the ratio of NASDAQ volume to NYSE volume. A
high ratio is indicative of a high level of speculative activity, which is believed by contrary-opinion
rule technical analysts to be a sign of an overbought market, and of an approaching market peak.

Question: 3564

A price level at which the supply of stock is expected to increase significantly is best described as
which of the following?

That answer is correct!


A. Resistance level.
B. Rotation price.
C. Reversal level.
D. None of these answers is correct.
E. Support level.

Answer: A

Explanation:
Technical analysts frequently examine historical price information to determine important "support"
and "resistance" levels. A support level is a price range at which the demand for a stock is expected
to increase, i.e. buying pressure will increase forcing the stock price upward, or will prevent it from
moving down. A resistance level is a price range at which the supply of a stock is expected to increase
significantly, forcing the price of the stock down, or preventing it from advancing.

"Rotation price" and "reversal level" are largely fictitious terms.

Question: 3565

Assume the following information about a stock market series:

Observed beginning value: 515.60


Anticipated ending value: 609.15
Expected dividends during the period: $24
Required rate of return: 21%

Page | 1423
Using this information, what is the expected rate of return for this index? (Assume a one-year
holding period.)

A. None of these answers is correct.


B. 22.80%
C. 19.30%
D. 11.42%
E. 13.49%

Answer: B

Explanation:
To calculate the expected rate of return for a stock market series, the following information must be
known: The beginning value for the series, the anticipated ending value for the series, and the
amount of any dividends and/or distributions during the period.

Once this information has been determined, the expected return on a stock market index can be
found by employing the following equation: {E(R) = [(EV - BV + Div) / BV]}. Where: E(R) = the
expected return on the stock market series, EV = the anticipated ending value for the series, BV = the
observed beginning value for the series, and Div = the amount of any dividends paid during the
period.
In this example, all of the necessary information has been provided and the calculation of the
expected return on this stock market series is found as follows: {E(R) = [$609.15 - $515.60 + $24] /
$515.60} = 22.80%. This is slightly higher than the required rate of return. Assuming that the ending
value and dividends prove accurate, investment in this stock market series is likely advisable.

Question: 3566

________ analysis is the estimation of future security price movements based on past movements.

A. Transactional
B. Historical
C. Regression
D. Technical

Answer: D

Explanation:
As opposed to fundamental analysis, technical analysis focuses on market trends rather than
fundamental data, when estimating future price changes in the market.

Question: 3567

A decrease in the Eurodollar-T-bill spread indicates that:

A. there is a heavy demand for T-bills due to the economic strength of the U.S.
B. investors are ready to take on higher credit risks and there is reduced demand for t-bills relative to
Eurodollar futures.
C. none of these answers.
D. investors are bearish on a global basis.

Answer: B

Explanation:

Page | 1424
Eurodollar rate is the rate of interest paid on inter-banking U.S. Dollar loans and deposits outside the
U.S. Since the U.S. Treasury has the best credit in the world, the yield on U.S. Treasury bills is lower
than the Eurodollar rate. The "spread" or the difference between the two rates is a measure of the
additional credit risk involved in inter-bank loans compared to loans to the U.S. Treasury. If the
demand for Eurodollar lending increases relative to the purchase of t-bills, the ED-T-bill spread will
narrow. Thus, the behavior of the spread reflects the risk attitude of investors in the lending markets.

Question: 3568

Given that the expected dividend payout ratio on a common stock is 0.7, the required rate of return
is 19%, the dividend and earnings growth rate is 15%, and current earnings are $1.38, using the
earnings multiplier model, what is the estimated value of the stock?

A. Not enough information


B. $27.83
C. $43.71
D. $29.68
E. $25.53

Answer: B

Explanation:
The earnings multiplier model postulates that P/E = (D1/E)/(k -g), where P/E is the price to earnings
ratio, D1 is next year's expected dividends, E is next year's earnings, k is the required rate of return,
and g is the growth rate in dividends. D1/E is also known as the dividend payout ratio. In this
question, the P/E is (0.7) / (0.19 - 0.15) = 17.5. Next year's earnings are equal to current earnings
multiplied by the earnings (and dividend) growth rate (1.38 x 1.15 = $1.59). We can multiply P/E by
next year's earnings to arrive at our expected stock value ((P/E) x E = P) In this question, the
estimated value is 17.5 x 1.59 = $27.83.

Question: 3569

What three factors are examined in the three-step valuation process?

A. industry influences, company analysis, market analysis


B. economic influences, global analysis, company analysis
C. economic influences, industry influences, company analysis
D. industry influences, company analysis, technical analysis

Answer: C

Explanation:
Economic influences such as government spending and tax; Industry influences such as how
industries respond to the business cycle or economic changes; and Company analysis using cash flow
values and financial ratios - are examined in the three-step valuation process. Remember: the factors
are in increasing in scope.

Question: 3570

One of the assumptions of technical analysis is that the market value of any good or service is
determined solely by the interaction of ________.

A. consumers and investors


B. investment professionals and credit agencies

Page | 1425
C. supply and demand
D. none of these answers

Answer: C

Explanation:
Both technicians and non-technicians agree that, at any point in time, the price of a security (or good
or service) is determined by the interaction of supply and demand for it. The four assumptions that
support technical analysis are these:

1. The market value of any good or service is determined solely by the interaction of supply and
demand for it.

2. Supply and demand are governed by numerous factors, both rational and irrational, including the
economic variables relied upon by the fundamental analyst, as well as opinions, models and guesses.
The market weights all of these factors continually and automatically.

3. Disregarding minor fluctuations, the prices for individual securities and the overall value of the
market tend to move in trends, which persist for appreciable lengths of time.

4. Prevailing trends change in reaction to shifts in supply and demand relationships. These shifts, no
matter why they occur, can be detected sooner or later in the action of the market itself.

Question: 3571

A firm has an ROE of 13% and a required rate of return on the stock of 16%. The firm currently has a
dividend payout ratio of 26%. Investors pay no personal taxes on dividends. To increase value, it
must:

A. change its retention ratio to 1.


B. incomplete information to answer.
C. reduce its payout ratio to around 5%.
D. pay out all of its earnings as dividends.

Answer: D

Explanation:
If the firm cannot invest the funds at a rate of return at least equal to that which the investors
require, then the shareholders are better off if the firm pays out all it earns (provided these
distributions are not taxed) so that the investors can invest the funds on their own at higher rates.

Question: 3572

An increase in the tax rate, while holding everything else equal, would have what effect on the
earnings multiple of a stock market series? Further, what effect should be anticipated from an
increase in the depreciation per-share figure?

A. Earnings multiple would increase, earnings multiple would increase.


B. Earnings multiple would increase, earnings multiple would increase.
C. Earnings multiple would remain unchanged, earnings multiple would decrease.
D. Earnings multiple would decrease, earnings multiple would decrease.
E. Earnings multiple would decrease, earnings multiple would remain unchanged.
F. Earnings multiple would increase, earnings multiple would remain unchanged.

Page | 1426
Answer: D

Explanation:
The following equation is used to determine the earnings multiple of a stock market series:
EPS = {[(Sales per share * operating profit margin) - depreciation per share - interest expense per
share] * (1 - corporate tax rate).

As you can see, an increase in the tax rate, holding everything else equal, will lead to a decrease in
the earnings multiple. Likewise, an increase in the per-share depreciation figure will lead to a
decrease in the earnings multiplier.

Question: 3573

Consider the following information about a common stock:

Price per share: $115.88


Next dividend per share: $2.80
Required return: 15.25% per year
Expected growth rate: 12.75% per year

What is the value of this common stock?

A. $129
B. $112
C. $101
D. None of these answers is correct.
E. $103
F. The answer cannot be determined from the information provided.

Answer: B

Explanation:
To determine the value of a common stock using the Infinite Period Dividend Discount Model, use
the following equation:

{V = [d1 / (k - g)]}

Where: V = the value of the common stock at t0, d1 = the annual dividend at t1 (which is found by
multiplying d0 by (1 + g), k = the investor's required rate of return, and g = the anticipated annual
growth rate.

In this example, all of the necessary information has been provided, and incorporating this
information into the Infinite Period DDM will lead to the following:

{V = [$2.80 / (0.1525 - 0.1275] = $112}

This value is very close to the value of the common stock in the open market.

An important observation: notice that we have valued this common stock as a perpetuity, rather than
a finite series of cash flows. The reasoning behind this approach should be somewhat intuitive.
Specifically, unlike a bond, whose cash flows possess a finite lifespan, the cash flows (i.e. dividends)
produced by a common stock could theoretically last forever. Is this a realistic assumption for most
common stocks? What about a stock that pays little or no dividend?

Page | 1427
Question: 3574

The P/E ratio of a stock is 12.3. Its current earnings are $12.1 per share and a growth rate of 3.9%.
The current price of the stock is:

A. $159.22
B. $148.83
C. $143.24
D. $154.60

Answer: D

Explanation:
It is important to remember that the P/E ratio is the ratio of the current stock price and next year's
expected earnings. Therefore, current stock price = 12.3 * 12.1 * 1.039 = $154.6

Question: 3575

Which of the following should be taken into account when estimating country risk?

I.Political risk
II.Expropriation risk
III.Currency risk
IV.Economic environment
V.Inflation trends
VI.Trade policy
VII.Consumer tastes

A. I, III, IV, V, VII


B. I, II, III,
C. I, II, III, IV, V, VI, VII
D. II, IV, V
E. I, III, III, IV, V, VII
F. None of these choices are correct

Answer: C

Explanation:
In assessing country risk, one must consider various sources of risk and instability prevalent within
the country being examined. The two basic components of country risk are political risk and financial
risk. However, there are many considerations in assessing country risk that exist beyond the
boundaries of simple classification. For instance, attitudes of consumers must be taken into effect, as
well as such things as trends in consumer spending, unemployment rates, threats of ideological
instability, etc. So said, the determination of a finite "country risk premium" is often a complex and
daunting task. When assessing a country risk premium, the five sources of risk must be considered
(business risk, financial risk, liquidity risk, exchange rate risk, and country risk).

Question: 3576

Consider the following information about an automobile manufacturer:

Next annual dividend: $4.11


Earnings per share next year: $7.02
Anticipated growth rate: 6% per year

Page | 1428
Required rate of return: 11% per year

What is the expected earnings multiplier for this utility company?

A. 12.41
B. Manipulating the Infinite Period DDM will produce a nonsensical answer in this case.
C. 18.51
D. 11.71
E. 13.51

Answer: D

Explanation:
To determine the earnings multiplier (i.e. the price-to-earnings ratio) for an individual company, use
the following formula:

P/E = [(d1 / e1) / (k - g)]

Where: P/E = the earnings multiplier, d1 / e1 = the dividend payout ratio at t1, k = the required rate
of return, and g = the anticipated future growth rate.
In this example, all of the necessary information has been provided, but some rearranging is
necessary. Specifically, the dividend payout ratio must be determined. This figure is found as follows:

Dividend payout ratio = [$4.11 / $7.02] = 0.58547, or 58.55%

Now that the dividend payout ratio has been determined, we can solve for the appropriate earnings
multiplier. The calculation of this figure is found as follows:

P/E = [0.5855 / (0.11 - 0.06) = 11.71

In generally favorable economic conditions, this is a realistic earnings multiple for an average
automobile manufacturer.

Question: 3577

The trough is

That answer is correct!


A. the bottom price.
B. a trend indicating declining prices.
C. a trend indicating rising prices.
D. a period of low-volume trading.
E. the peak price.

Answer: A

Explanation:
Technical analysts would purchase a stock if they think that it has hit its trough, and is now in a rising
trend channel.

Question: 3578

Which of the following is/are growth stocks?

Page | 1429
I. The stock of Omega Corp., which is a highly successful firm that has had above average growth in
sales and earnings in the past 5 years.

II. The stock of InstyPrints, a paper company which has been poorly managed in the past, causing the
firm's stock price to plunge below what most analysts consider to be its fair value.

III. The stock of Zygotes, Etc., a biotech firm that has high business and financial risk.

A. I only
B. II only
C. I & III
D. III only
E. I & II
F. I, II & III

Answer: B

Explanation:
You have to be careful in distinguishing between a growth stock and a growth firm. A growth stock
need not represent a growth firm. Rather, it is defined as a stock that has consistently generated
returns higher than those justified by the risks. Such a situation arises either because the company
has surprise windfalls or because at some point in the past, the market underestimated the firm's
growth potential. Since InstyPrints has been identified and commonly tagged as being
"undervalued," one can reasonably expect
the stock to generate higher than required rates of return in the near future. (Again, keep in mind
that the return on the stock is different from the return on the firm's investments in various projects).
Therefore, InstyPrints' stock is a growth stock. On the other hand, Omega Corp. is a growth firm but
its stock need not necessarily be a growth stock. Indeed, if Omega's stock is "overvalued," it may be a
bad investment.

Question: 3579

When estimating an industry's risk premium, the following should be examined for the industry and
compared to the aggregate market

A. country risk
B. business risk
C. all of these are correct
D. financial risk
E. liquidity risk
F. the industry required rate of return
G. exchange rate risk

Answer: C

Explanation:
Alternatively, you can estimate the risk premium based on the CAPM, which implies that the risk
premium is a function of the systematic risk of the asset.

Question: 3580

Common stocks of good companies may not be good investments because:

I. The stocks may be overpriced compared to other stocks with similar risks.

Page | 1430
II. Empirically, stocks which have performed well in the past tend to produce lower-than-expected
returns in the future.

III. Good companies frequently happen to be large stocks which are low risk and do not provide very
high rates of return.

A. IV only
B. I & III
C. III only
D. I only
E. I, II & III
F. II only
G. I & II

Answer: D

Explanation:
Even if a company is good, the market could temporarily elevate the price above that consistent with
its risk. In that case, the stock is overpriced and it may not make sense to buy it, though that decision
still must be based on a portfolio diversification context.

Question: 3581

Which of the following is not involved in the estimation of the earnings per share (EPS) for a stock
market series?

A. Estimation of sales per share


B. Estimation of next year's interest expense
C. Estimation of next year's operating profit margin
D. Estimation of next year's corporate tax rate
E. All of these choices are involved in the estimation of EPS for a stock market series.
F. Estimation of next year's depreciation per share

Answer: E

Explanation:
All of the choices listed are used in the estimation of EPS for a stock market series. Further, the
estimation of EPS for a stock market series involves these five processes exclusively.

Question: 3582

Which of the following represents a "contrary opinion" technical indicator? Choose the best answer.

A. The Confidence Index


B. More than one of these answers is correct.
C. T-Bill-Eurodollar yield spread
D. The Diffusion Index
E. Debit balances in brokerage accounts
F. OTC versus NYSE volume

Answer: F

Explanation:

Page | 1431
Of all the choices listed, only "OTC versus NYSE volume" represents a contrarian technical indicator.
While "debit balances in brokerage accounts (margin debt)" appears to be an appealing choice, this
indicator is actually a measure of "smart money" because investors who leverage their portfolios
through margin loans are viewed by technical analysts as being sophisticated.

The measure of OTC to NYSE volume is used by contrarian technical analysts as a measure of
speculation in the U.S. securities markets. A high degree of OTC volume is viewed as a bearish signal
to contrarian technical analysts, who believe this is indicative of abounding speculation. Prior to the
collapse of the NASDAQ in 2000, the OTC to NYSE volume experienced a marked spike, and many
contrarian technical analysts were vocal in their opinion that the degree of speculation in the
securities markets was rampant. While for years the explicit level of OTC to NYSE volume was used,
recently it has been decided that the proper method is to examine trends in the OTC to NYSE volume,
rather than static figures.

The Confidence Index, a "smart money" technical indicator, is a measure of yield spreads between
highgrade corporate bonds and the yields on average corporate bonds. The Diffusion Index measures
the breadth of the market, and is found by taking the total volume of advancing shares plus one-half
of the issues unchanged, divided by the total number of issues traded. The T-Bill-Eurodollar Yield
Spread is another example of a "smart money" technical indicator.

Question: 3583

Consider the following preferred stock issued by Bluebook Pharmaceuticals:

Price per share: $12.65


Annual dividend per share: $1.30
Required rate of return: 9% per year

Is the preferred stock realistically overvalued, undervalued, or correctly valued? Further, should this
preferred stock be valued as a perpetuity or a finite series of cash flows?

That answer is correct!


A. Undervalued; perpetuity
B. Undervalued; finite series of cash flows
C. Correctly valued; finite series of cash flows
D. Correctly valued; perpetuity
E. Overvalued; finite series of cash flows
F. Overvalued; perpetuity

Answer: A

Explanation:
The preferred stock profiled in this example is trading above its theoretical value, which is found to
be $14.44.
To determine the value of a preferred stock, use the following equation: {P0 = [d1 / k]}
Where:P0 = the price of the preferred stock at time 0, d1 = the annual dividend at t = 1, and k = the
required rate of return.
In this example, the dividend is provided as an annual figure, so all of the necessary information has
been given. The calculation of the value of this preferred stock is as follows:

{P0 = [$1.30 / 0.09] = $14.44

Preferred stock is commonly valued as a perpetuity because there is no finite conclusion to the
projected series of cash flows for a preferred stock. Unlike a bond, whose cash flows are

Page | 1432
characterized by a finite lifespan (i.e. the cash flows of a bond cease at maturity), the cash flows
(dividends) produced by a preferred stock could theoretically last forever.

Question: 3584

George Black, CFA, is examining the financial performance of Clay Industries, a large industrial firm,
and has assimilated the following information from the most recent fiscal year:

Adjusted operating profit before tax: $13,900,000


Cash operating taxes: $4,350,000
Cost of capital: 15.60% per year
Total capital employed: $56,500,000

Using this information, what is the Economic Value Added for Clay Industries? Further, should the
management of Clay Industries be considered to have created economic value for shareholders?

A. ($736,000); management has not created economic value


B. $736,000; management has not created economic value
C. None of these answers is correct.
D. ($736,000); management has created economic value
E. $736,000; management has created economic value
F. $850,000; management has created economic value

Answer: E

Explanation:
Economic Value Added, a value-based measure of economic profit, is a registered trademark of
Stern, Stewart & Company. The equation used to calculate EVA is as follows:
EVA = {Net Operating Profits Less Adjusted Taxes - [Total Capital Employed * (Cost of Capital)]}.

In this case, the NOPLAT figure must be calculated manually by subtracting the Cash Operating Taxes
from the Adjusted Operating Profit before Tax (AOPBT) figure. Doing so will produce an answer of
$9,550,000 for the Net Operating Profit less Adjusted Taxes figure. Now that the necessary
information has been determined, the calculation of EVA is as follows:

EVA = {$9,550,000 - ($56,500,000 * 0.156)} = $736,000

A positive EVA calculation indicates that management has provided economic profits to
shareholders, as evidenced by the fact that the opportunity cost of capital employed is less than the
NOPLAT figure. In this example, the management of Intelligent Semiconductor should be considered
as having succeeded in providing shareholder value.
The calculation of Adjusted Operating Profit Before Taxes (AOPBT), and Cash Operating Taxes are
important. The calculation of AOPBT is as follows:

Operating profit (after depreciation and amortization) + Implied interest on operating leases + any
increase in the LIFO reserve + goodwill amortization = Adjusted Operating Profit Before Taxes.

Cash Operating Taxes, another important component of EVA, is calculated as follows:

Cash Operating Taxes = Income Tax Expense + tax benefit from interest expenses + tax benefit from
interest on leases + taxes on non-operating income

Adjusted Operating Profit Before Taxes minus Cash Operating Taxes = Net Operating Profit Less
Adjusted Taxes (NOPLAT). Subtracting the dollar cost of capital from the NOPLAT figure will yield the

Page | 1433
EVA.

Question: 3585

A high ratio of specialists' short sales to total short sales would be viewed

That answer is correct!


A. bearishly by technical analysts.
B. as unimportant by technical analysts.
C. bullishly by technical analysts.
D. as a signal of an approaching trough by technical analysts.

Answer: A

Explanation:
Technical analysts trying to follow the "smart money" sometimes use the proportion of specialists'
short sales to total short sales as a market indicator. If the ratio is high, specialists are seen as trying
to profit from upcoming market declines. If the ratio is low, specialists are seen as trying to minimize
their participation in short sales in expectation of a rising market.

Question: 3586

A net decline on an advance-decline series for the NYSE, coupled with a rising S&P 500 would,
according to technical analysts,

A. signal a market trough.


B. signal a market peak.
C. be neither particularly bullish nor bearish.
D. be a bullish sign.

Answer: B

Explanation:
Technical analysts predict that when a market index is going up, but most stocks post price declines,
the market is near its peak. Conversely, they predict that when a market index is going down, but
most stocks post price increases, the market is near its trough.

Question: 3587

Jones Rutherford, a portfolio manager with Churn Brothers Brokerage, has been examining a stock
market series and is trying to determine the anticipated rate of return for the series. In his analysis,
Jones has amassed the following information:

Anticipated ending value: 1475


Expected dividends during the period: $35
Observed beginning value: 1310
Required rate of return: 19%

Using this information, what is the anticipated rate of return for this stock market series? (Assume a
oneyear holding period).

A. None of these answers is correct.


B. 13.56%
C. 9.92%

Page | 1434
D. 8.81%
E. 15.27%

Answer: E

Explanation:
To calculate the expected rate of return for a stock market series, the following information must be
known:

The beginning value for the series, the anticipated ending value for the series, and the amount of any
dividends and/or distributions during the period.
Once this information has been determined, the expected return on a stock market index can be
found by employing the following equation: {E(R) = [(EV - BV + Div) / BV]}. Where: E(R) = the
expected return on the stock market series, EV = the anticipated ending value for the series, BV = the
observed beginning value for the series, and Div = the amount of any dividends paid during the
period.
In this example, all of the necessary information has been provided and the calculation of the
expected return on this stock market series is found as follows: {E(R) = [$1475 - $1310 + $35] /
$1310} = 15.27%.
This is less than the required rate of return. Assuming that the figures for the ending value and the
expected dividends are accurate, then investment in this stock market series is not likely warranted.

Question: 3588

The price range at which a technician would expect a substantial increase in demand for a stock is

That answer is correct!


A. the support level.
B. the resistance level.
C. the peak level.
D. the demand level.

Answer: A

Explanation:
Demand level is not a real term. Technicians believe that a support level on a stock will develop after
it has experienced substantial price gains. They reason that at some price range (the support level),
other investors who did not buy the stock the first time it went up will now buy it after a small price
reversal.

Question: 3589

Assume the following information about a publicly traded utility company:

Next annual dividend: $2.10


Earnings per share next year: $2.91
Anticipated growth rate: 7.5% per year
Required rate of return: 10.5% per year

What is the expected earnings multiplier for this utility company?

That answer is correct!


A. 24.05
B. None of these answers is correct.

Page | 1435
C. 13.30
D. 26.60
E. 46.20
F. 75.25

Answer: A

Explanation:
To determine the earnings multiplier (i.e. the price-to-earnings ratio) for an individual company, use
the following formula:

P/E = [(d1 / e1) / (k - g)]

Where: P/E = the earnings multiplier, d1 / e1 = the dividend payout ratio at t1, k = the required rate
of return, and g = the anticipated future growth rate.
In this example, all of the necessary information has been provided, but some rearranging is
necessary. Specifically, the dividend payout ratio must be determined. This figure is found as follows:

Dividend payout ratio = [$2.10 / $2.91] = 0.721649, or 72.16%

Now that the dividend payout ratio has been determined, we can solve for the appropriate earnings
multiplier. The calculation of this figure is found as follows:

P/E = [0.7216 / (0.105 - 0.075)] = 24.05

This is a relatively high multiple for a utility company growing by 7.5% per year. (Although this figure
is not completely beyond the realm of reason.) An analyst examining this stock would likely regard
this earnings multiplier with some degree of caution. Further analysis would probably be warranted.

Question: 3590

A junior analyst with Smith, Kleen & Beetchnutty is performing an analysis of Microscam
Incorporated. Assume the following information:

g = 20% per year


k = 21% per year
EPS = $3.10
D0 = $0.68

Using this information, what is the P/E ratio for Microscam shares?

A. None of these answers is correct.


B. 24.32
C. 26.32
D. 15.82
E. 18.78
F. The answer cannot be calculated from the information provided.

Answer: C

Explanation:
By dividing each side of the infinite period dividend discount equation by the EPS figure, it is possible
to determine the P/E ratio. This is illustrated as follows:

Page | 1436
P/E = (D1 / EPS)/(k-g)

Where: D1 = the dividend at t1, EPS = the earnings per share calculation for t1, k = the required rate
of return, and g = the expected growth rate.

Manipulating the infinite period dividend discount model to solve for the PE is a rather intuitive
process. Consider the fact that an investment's value is truly nothing more than the present value of
all future returns. So said, dividing both sides of the infinite period dividend discount model equation
should yield the appropriate multiple, or "earnings multiplier." This is the price-to-earnings ratio.

In this example, we are provided all of the necessary information. However, the dividend at t1 must
be calculated manually by multiplying D0 by (1 + growth rate). This will yield a figure of $0.816 for
D1.

Now that D1 has been determined, we can solve for the P/E. Imputing all the given information into
the equation provided above will yield the following:

P/E = ($0.816 / $3.10) / (21% - 20%) = 26.32

Question: 3591

Firms in which of the following industries would likely have the highest earnings retention rates?
Further, would firms within this industry likely be financed primarily through debt or equity?

A. Automobile manufacturing; debt


B. Retail banking; debt
C. Retail banking; equity
D. Automobile manufacturing; equity
E. Pharmaceuticals; debt
F. Pharmaceuticals; equity

Answer: F

Explanation:
Firms in the pharmaceutical industry would likely retain a higher proportion of their earnings than
automobile manufacturers or retail banks. This is reasoned primarily by two factors. First,
pharmaceutical companies have high research and development costs, and their success depends
largely on the discovery of promising drugs. This requires that adequate funds be held within the
firm. Secondly, growth opportunities in the pharmaceuticals industry are much more abundant than
in either the automobile manufacturing or retail banking, and this is largely due to the maturity of
the industry as a whole.
Pharmaceutical companies typically have numerous positive NPV investment opportunities.

In terms of financing, pharmaceutical firms would be expected to have a capital structure weighted
more heavily toward equity than debt. As the cash flows of an industry become more certain and
stable, the level of debt financing should be expected to increase. Automobile manufacturers and
retail banks have very predictable cash flows, along with high degrees of leverage. Certainty of
revenues and earnings is a very important consideration in the financing decision, and firms whose
cash flows are both stable and easily forecasted will likely prefer a high degree of debt versus equity
financing. Another important note to consider is the fact that the balance sheet of a pharmaceutical
firm is more heavily weighted toward intangible assets than is a retail bank or an automobile
manufacturer. Intangible assets cannot easily be pledged as collateral for a loan, neither can
intangible assets be liquidated with any degree of certainty. A high proportion of intangible assets
within the asset structure of a firm is conducive to a low degree of debt financing.

Page | 1437
Question: 3592

The divergence between the trend for the stock market series and the cumulative advance-decline
series signals a market ________.

That answer is correct!


A. peak
B. adjustment
C. crash
D. trough

Answer: A

Explanation:
The usefulness of the advance-decline series is supposedly greatest at market peaks and troughs.
The divergence between the trend for the stock market series and the cumulative advance-decline
series signals a market peak.

Question: 3593

Which of the following is not one of the three steps of the top-down, three-step approach to stock
valuation?

A. Analysis of alternative industries


B. Analysis of countries and regions
C. Analysis of the economy and security markets
D. Analysis of individual firms and stocks

Answer: B

Explanation:
Although analysis of alternative countries and regions may be undertaken, that analysis is done
under the more general step of analyzing the economy and security markets.

Question: 3594

The ________ approach begins with the current earnings multiplier and estimates the direction and
extent of a change in the specific factors affecting dividend payout, required rate of return and
growth rate of dividends.

A. specific estimate
B. current earnings
C. direction of change
D. gross magnitude

Answer: C

Explanation:
There are two ways to estimate the earnings multiplier: (1) Direction of change approach and (2)
Specific estimate approach. The first focuses on change and change direction, while the other focuses
on scenario-based estimates.

Question: 3595

Page | 1438
The ________ index shows the number of stocks advancing plus one-half the number unchanged,
divided by the total number of issues traded.

A. short
B. cumulative
C. contrarian
D. diffusion

Answer: D

Explanation:
By definition: The diffusion index shows the number of stocks advancing plus one-half the number
unchanged, divided by the total number of issues traded. Such an index gives an idea of how many
individual stocks are advancing as a percentage of all the individual stocks in a composite index.

Question: 3596

Which of the following events would a technical analyst interpret as bearish?

A. a low rate of odd-lot short sales as a percentage of total odd-lot sales


B. decline in credit balances
C. all of these answers
D. a low mutual fund cash position

Answer: B

Explanation:
Credit balances result when investors sell stocks and leave the proceeds with their brokers, expecting
to reinvest them shortly. Technicians view a decline of credit balances as a decrease in buying power
and a bearish sign.

Question: 3597

If the 200-day moving average for a stock is well above its current price, then

A. the stock price must have experienced an overall upturn. If the current price slide reverses itself,
and the stock price moves above its 200-day moving average on heavy volume, technical analysts
would view this as a very positive sign.
B. the stock price must have experienced an overall downturn. If the current price slide reverses
itself, and the stock price moves above its 200-day moving average on heavy volume, technical
analysts would consider this to be a sign that the stock is overbought.
C. the stock price must have experienced an overall downturn. If the 50-day moving average is also
below the 200-day moving average, but then moves up above the 200-day moving average, technical
analysts would consider this a sign that the market is overbought.
D. the stock price must have experienced an overall downturn. If the 50-day moving average is also
below the 200-day moving average, but then moves up above the 200-day moving average on heavy
volume, technical analysts would consider this to be a bullish sign.

Answer: D

Explanation:
In order for a moving average of a stock price to be below the current price, the stock price must
have experienced an overall decline. Technicians believe that the current price breaking through the

Page | 1439
moving average from below on heavy volume is a very positive sign, and may well signal the reversal
of the declining trend. If the current price breaks through the moving average from above on heavy
volume, this would be taken as a very negative sign. The same is true of changes in the relative
positions of a longer moving average and a shorter moving average.

Question: 3598

When using macroanalysis in estimating an industry earnings multiplier

A. you first estimate the industry growth rate (g) which is determined by the retention rate and the
return on equity.
B. you examine the relationship between the multiplier for the industry and the market.
C. you first estimate the industry required rate of return (k) because this is influenced by the risk-free
rate and the expected inflation rate.
D. you compute an average multiplier for the industry.
E. you examine the specific variables that influence the earnings multiplier.

Answer: B

Explanation:
The two alternative techniques to estimate an industry earnings multiplier aremacroanalysis, where
you examine the relationship between the multiplier for the industry and the market, and
microanalysis where you examine the specific variables that influence the earnings multiplier.

Question: 3599

A growth stock:

That answer is correct!


A. is a stock which generates rates of returns higher than stocks with similar risks.
B. all of these answers.
C. promises rates of return higher than those that can be obtained by investing in the market
portfolio.
D. represents a company that has management abilities and investment opportunities that yield
rates of return higher than the required rate of return.

Answer: A

Explanation:
You have to be careful in distinguishing between a growth stock and a growth firm. A growth stock
need not represent a growth firm. Rather, it is defined as a stock that has consistently generated
returns higher than those justified by the risks. Such a situation arises either because the company
has surprise windfalls or because at some point in the past, the market underestimated the firm's
growth potential. Note that this is not in violation of efficient markets theory, which says that on
average, investors correctly impound the available information in the stock price, though in any
individual case, the market might over- or underestimate the impact of the new information on the
value of the firm.

Question: 3600

Assume the following series of transactions

t0: Unknown
t1: Purchase 10,000 shares of Intelligent Semiconductor for $98.90 per share

Page | 1440
t2: Sell 10,000 shares of Intelligent Semiconductor for $105.30 per share
t3: Sell 5,000 shares of Intelligent Semiconductor for $111.65 per share
t4: Sell 5,000 shares of Intelligent Semiconductor for $140.00 per share

Similar investments have merited a 13.45% discount rate. Assuming no taxes or transaction charges,
what is the dollar-weighted rate of return for this series of investments?

A. 66.11%
B. None of these answers is correct.
C. The answer cannot be calculated from the information provided.
D. 46.76%
E. 58.27%

Answer: C

Explanation:
Remember that the dollar-weighted rate of return uses the IRR equation in the determination of the
answer. In fact, the dollar-weighted rate of return is another name for the IRR equation, and this
nomenclature is commonly used within the field of investment management. So said, in the
determination of the dollar-weighted rate of return calculation, the first step should be to identify
the cash flows for each period, beginning with t0: the initial investment outlay. In this example, the
initial cash outlay is not specified, and therefore the calculation of the dollar-weighted rate of return
cannot accurately be determined.

Question: 3601

A market strategist with East Coast Brokerage is examining a stock market series and trying to
determine an appropriate EPS figure. In her analysis, this market strategist has determined the
following information:

1. Regressing sales for the series against Nominal GDP, the sales figure for the index has been
estimated at: $21.

2. Analyzing capacity utilization rates, foreign competition, rates of inflation and unit labor costs, the
operating profit margin for the series has been determined to be 32%.

3. Creating a time series based upon inputs such as levels of capital expenditures and PP&E turnover,
next year's depreciation-per-share has been determined to be $1.10.

4. Creating a time series based upon levels of debt outstanding and prevailing debt yields, the
interest expense for next year is determined to be $1.03 per share.

5. Coordinating his research with a legislative consultant, the corporate tax rate for this series has
been estimated at: 35.15%.

Using this information, what is the EPS figure for this stock market series?

A. None of these answers is correct.


B. $2.23
C. $2.98
D. The answer cannot be determined from the information provided.
E. $4.01
F. $4.07

Page | 1441
Answer: C

Explanation:
All of the necessary information has been provided in this example. To determine the EPS for a stock
market series, the following steps are necessary:

Step 1: Estimate sales-per-share for the series:


Step 2: Estimate operating profit margin for the series
Step 3: Estimate the depreciation-per-share for next year
Step 4: Estimate the interest expense-per-share for the next year
Step 5: Estimate next year's corporate tax rate

Once these five steps have been completed, the calculation of EPS for a stock market series is found
by the following:

EPS = [(Sales per share * Operating profit margin) - Depreciation-per-share - Interest Expense] * (1 -
Corporate Tax Rate)

The calculation of EPS for this stock market series is shown as follows: EPS = [($21 * 0.32) - $1.10 -
$1.03] * (1 - 0.3515) = $2.98

Question: 3602

The Dow Theory postulates that there are

A. two types of price movements over time.


B. ten types of price movements over time.
C. four types of price movements over time.
D. five types of price movements over time.
E. three types of price movements over time.

Answer: E

Explanation:
The Dow Theory was invented in the late 19th century by Charles Dow, publisher of the Wall Street
Journal. It postulated that there were three types of price movements over time: major trends,
intermediate trends, and short-term movements. These three types of trends interacted with one
another, and identifying and isolating them individually would lead to profit opportunities.

Question: 3603

The dividend discount model assumes that

A. dividends must be discounted in favor of earnings to arrive at the correct valuation for common
stock.
B. the value of a share of common stock is the present value of all future earnings.
C. the value of a share of common stock is the present value of all future dividends.
D. dividends are worth more in the future than in the present.

Answer: C

Explanation:
The dividend discount model (Dividend Discount Model) assumes that the value of a share of
common stock is the present value of all future dividends, not earnings. The Dividend Discount

Page | 1442
Model assumes that a dollar today is worth more than a dollar tomorrow.

Question: 3604

Which of the following represents a "smart money" technical indicator? Choose the best answer.

A. More than one of these answers is correct.


B. Confidence Index.
C. Futures traders bullish on stock index futures.
D. Diffusion Index.
E. Breadth of market.
F. Block Uptick/Downtick Ratio.

Answer: B

Explanation:
Of the choices listed, only the Confidence Index represents a "smart money" technical indicator. The
Confidence Index, published weekly by Barron's, is used to measure the degree of confidence
amongst bond traders. The Confidence Index is constructed by measuring the ratio of Barron's
average yield on 10 top-grade corporate bonds to the yield on the Dow Jones average of 40 bonds.
The Confidence Index is designed to measure the difference in yield between high grade and more-
speculative bonds. As the yield spread between the two sets of bonds narrows, (i.e. high-grade
bonds are being bid down and low-grade bonds are being bid up) the ratio will approach one.
Remember that the yields on high-gradebonds is always less than that on lower-grade issues, so this
ratio will never exceed 1. Technicians would view an increase in the Confidence Index as a bullish
signal. The reasoning behind this opinion is relatively straightforward - more money is being directed
toward speculative bonds, indicating that investor confidence in the economy is high.
"Breadth of market" refers to the measure of advancing versus declining issues. The "Diffusion Index"
is a measure of market breadth and is defined as the volume of advancing issues plus one-half of the
volume of unchanged issues, divided by the total number of issues traded. Short interest measures
the total volume of outstanding short positions, and the sentiment of futures traders is used by
contrarian technical analysts, who take a contra approach. The "Block Uptick/Downtick Ratio" is used
to measure the near-term sentiment of institutions.

Question: 3605

All else equal, which of the following is/are true?

I. The stock price increases as the dividend growth rate increases.


II. The stock price increases as the expected rate of return increases. III. The stock price increases as
the current dividend increases.
IV. The payout ratio increases as current earnings increase.

A. III only
B. I, II & IV
C. I only
D. II & IV
E. IV only
F. I, III & IV
G. I & III
H. II only

Answer: G

Page | 1443
Explanation:
The Dividend Discount Model implies, in standard notation, that Po = D1/(k-g). This equation serves
to justify the choices. It should be noted that in this question, the phrase, "all else equal" is critical. In
III, for e.g., you cannot expect the stock price to rise solely because dividend increases (indeed, the
priceshould fall because part of the firm is being paid out as cash). The "all else equal" stipulates that
the growth rate of dividend and expected return remain constant, which is what causes the stock
price to rise.

Question: 3606

The increase in OTC trading relative to NYSE trading over time

A. is a sign of growing government regulation of the NYSE.


B. is a sign of the increased influence of institutional investors.
C. is a sign of the increased influence of individual investors.
D. demonstrates the decreasing level of speculative activity.

Answer: C

Explanation:
Irrespective of the level of speculative activity in the market, there has been a trend toward
relatively high volume on the OTC compared with the NYSE. This is partly an indication of the
growing importance of individual investors, who are more prone to invest in the OTC market, and of
the growing number of firms listed on the OTC relative to the NYSE.

Question: 3607

What is the value of a preferred stock with company earnings of $30 and a required rate of return of
10%?

A. $600
B. $283
C. $129
D. $300
E. Not enough information

Answer: E

Explanation:
The value of a preferred stock is the present value of its dividends, which is equal to the annual
dividend divided by the required rate of return. Since the annual dividend is not given, one cannot
answer this question.

Question: 3608

Contrarians consider an increase in credit balances in brokerage accounts as:

A. a hold signal.
B. a bearish signal.
C. none of these answers.
D. a bullish signal.

Answer: B

Page | 1444
Explanation:
Credit balances with brokerages result when investors sell their stock holdings and leave the cash
balances with their brokers, expecting to reinvest them shortly. Technical analysts view these
balances as pools of potential purchasing power overhanging the market and hence believe that the
market is bullish. Contrarians, on the other hand, believe that most market participants make wrong
investment decisions as the market approaches the peak or trough in a cycle. Hence, contrarians
consider a large increase in credit balances as a bearish signal.

Topic 4
4,Financial Reporting and Analysis

Question: 3609

Rocky Johnson, CFA, manages a large capitalization equity mutual fund. His superiors have requested
that he provide them the appropriate benchmarks to compare future performance against. Johnson
makes the following statements:
Statement 1:We should use an unweighted index because it would best reflect the large company
bias in the portfolio.
Statement 2:Stocks in the portfolio frequently split the number of shares outstanding. Therefore, in
the long run, the Dow Jones Industrial Average would best reflect these events.
Are Johnson's two statements correct?

A. No. Both statements are incorrect.


B. Statement 1 is correct and Statement 2 is incorrect.
C. Statement 1 is incorrect and Statement 2 is correct.

Answer: A

Question: 3610

Which of the following assumptions is least likely to be consistent with the concept of efficient
capital markets?

A. Expected returns implicitly include risk in the price of the security.


B. Market participants correctly adjust prices based on new information.
C. New information about securities comes to the market in a random fashion.

Answer: B

Question: 3611

Sal Nunn, CFA, is a portfolio manager at Walker Investments. Nunn sold 300,000 shares of a NASDAQ
listed stock on an electronic crossing network in after hours trading because the company announced
a significant negative earnings surprise. Indicate whether the third or fourth market best describes
the Nunn trade and state whether the NASDAQ market is a call or continuous market.

A. Nunn's trade is in the third market and NASDAQ is a call market.


B. Nunn's trade is in the fourth market and NASDAQ is a call market.
C. Nunn's trade is in the fourth market and NASDAQ is a continuous market.

Answer: C

Question: 3612

Page | 1445
An investor purchased 100 shares of a stock two years ago for $50 per share after deciding the stock
would be a good value investment. Since the initial purchase, the stock price has fallen to $35 per
share after several of the company's major customers canceled contracts. The investor has decided
to purchase another 50 shares at the lower price. Which of the following behavioral biases best
characterizes the investor's actions?

A. Escalation bias.
B. Momentum bias.
C. Overconfidence bias.

Answer: A

Question: 3613

Ian Lance, CFA, is discussing short selling with a client and states, "The short seller must pay any
dividend of the issuer to the lender of the stock. In addition, the short seller must provide some
collateral to the brokerage house." Is Lance correct about the short seller's obligations?

A. Yes.
B. Lance is correct about paying the dividend, and incorrect about providing collateral.
C. Lance is incorrect about paying the dividend, and correct about providing collateral.

Answer: A

Question: 3614

An analyst is using the following information to value AGF Company's common shares. AGF paid a
dividend of $1.90 per share last year. Dividends are expected to grow at 6% forever. The risk-free rate
is 5%, the market risk premium is 7%, and the beta of the common shares is 1.3. The value of the
AGF Company's common shares is closest to:

A. $23.46
B. $24.86
C. $33.57

Answer: B

Question: 3615

In valuing the stock of Evergreen Enterprises, an analyst compiles the following information about
the firm:
The value of the firm's stock today is closest to:

A. $31.80
B. $38.55
C. $53.00

Answer: A

Question: 3616

Radio Corp. owns a leading radio network with 200 million weekly listeners. To combat a declining
radio audience, the company has implemented a strategy of targeting the Hispanic audience. After
careful analysis, Radio Corp. makes the decision to acquire a Hispanic radio station, rather than

Page | 1446
convert existing radio stations. The acquisition strategy benefits from rules that allow clustering of
ownership of local radio stations. The basis of the strategic initiative and the basis of the acquisition
strategy, respectively, are:

A. market timing and geography


B. lifestyles and technology
C. demographics and regulation

Answer: C

Question: 3617

An analyst develops the following information to value a common stock.


The analyst estimates the required rate of return and uses a DDM to value the shares. The value per
share is closest to:

A. $14.39
B. $21.28
C. $31.39

Answer: A

Question: 3618

Jack Saunders is analyzing Barco Incorporated, an industrial conglomerate company. Saunders is


estimating the intrinsic value for Barco Incorporated by forecasting the company's earnings per share
and earnings multiplier. Which of the following attributes of Barco is least likely to increase the
company's earnings multiplier?

A. Barco Incorporated has never had a restructuring charge in its history.


B. Barco Incorporated^ earnings move in tandem with overall economic growth.
C. Barco Incorporated's dividend has been increasing for the last 30 years.

Answer: B

Question: 3619

Brad Kit, CFA, is analyzing the broadcasting industry. Kit has narrowed his analysis to Willow Corp.
and Vision Inc.
Willow Corp. is a media company with a diversified group of leading TV, newspaper, and cable news
operations. Revenues and earnings have grown slightly over the past ten years. The company's long-
term debt to capital ratio is 40%. During the last recession, the company's earnings remained flat
with the prior year. Still, Kit believes that Willow Corp. will have positive earnings surprises over the
next several quarters, due to several new programs that have been hugely successful.
Vision Inc.'s operations are located in emerging markets with a high degree of political and
regulatory risk. However, the TV, radio, and internet operations have the potential for extraordinary
returns. Vision's stock is trading at 30 times next year's earnings and five times book value.
A client asks Kit whether Willow Corp. is a growth company or growth stock and whether Vision Inc.
is a speculative company or speculative stock. Kit's most appropriate response would be that:

A. Willow Corp. is a growth stock and Vision Inc. is a speculative stock.


B. Willow Corp. is a growth company and Vision Inc. is a speculative stock.
C. Willow Corp. is a growth stock and Vision Inc. is a speculative company.

Page | 1447
Answer: C

Question: 3620

The following information is provided about Jacko Industries.


Using the method of average return on equity, Jacko's P/E ratio is closest to:

A. 10.0
B. 16.67
C. 20.0

Answer: A

Question: 3621

An analyst has stated that, holding all else constant, an increase in the maturity of a coupon bond will
increase its interest rate risk, and that a decrease in the coupon rate of a coupon bond will decrease
its interest rate risk. The analyst is correct:

A. only with respect to the effect of the increase in the maturity


B. only with respect to the effect of the decrease in the coupon rate
C. with respect to both the effect of the increase in maturity and the effect of the decrease in the
coupon rate

Answer: A

Question: 3622

An analyst has been researching a possible investment in collateralized debt obligations (CDOs).
Identify the statement which is most likely correct.

A. The underlying securities for a CDO are typically issued only by U.S.-based entities.
B. A CDO with corporate bonds as the underlying security is known as a collateralized loan obligation
(CLO).
C. A CDO is typically structured into tranches, similar to a collateralized mortgage obligation (CMO).

Answer: C

Question: 3623

Chris South owns $25,000 face value of Bradco bonds, which have a 7% coupon, pay interest
semiannually, and have six years remaining until maturity. The bonds are callable at par. The bonds
were rated A when Chris bought them at par two years ago, and they are currently worth $26,225,
with a rating of AA. Which of the following statements most accurately describes the change in the
risk of the Bradco bonds?

A. Call risk has decreased.


B. Liquidity risk has increased.
C. Credit risk has decreased.

Answer: C

Question: 3624

Page | 1448
Ned Jameson. CFA, is considering the purchase of a newly issued asset-backed security (ABS) for his
fixed income portfolio. According to the broker/dealer offering the bond, the OAS for the issue is 75
basis points (bps). Based on the OAS value, which of the following assumptions can Jameson make
about this particular ABS?

A. The OAS represents the investor's compensation for credit risk, liquidity risk, and option risk.
B. The bond is trading at a yield that is more than 75 bps higher than a Treasury security with a
comparable maturity.
C. The implied cost of an option embedded in the security is always equal to the difference between
the OAS and the Treasury spread.

Answer: B

Question: 3625

With regard to a theoretical Treasury yield curve constructed with the bootstrapping method:

A. every point on the curve is constructed by utilizing current on-the-run Treasury yields of various
maturities.
B. the yield for most maturities used to construct the Treasury yield curve are observed yields rather
than interpolated yields.
C. any yield on the Treasury yield curve that is not one of the on-the-run maturities is only an
approximation for that maturity.

Answer: C

Question: 3626

Bond X is a nonmailable corporate bond maturing in ten years. Bond Y is also a corporate bond
maturing in ten years, and Bond Y is callable at any time beginning three years from now. Both bonds
carry a credit rating of AA. Based on this information, identify the most accurate statement:

A. Bond Y will have a higher nominal spread over a 10-year U.S. Treasury security than Bond X.
B. The option adjusted spread (OAS) of Bond Y will be greater than the nominal spread of Bond Y.
C. The nominal spread of Bond X will be greater than the option adjusted spread of Bond X.

Answer: A

Question: 3627

A Treasury bond dealer observes the following Treasury spot rates from the spot rate curve: 1-year
7.40%, 2-year 7.00%, and 3-year 6.3%. The bond dealer also observes that the market price of a 3-
year 8% coupon, 100 par value bond is $103.95. Based on this information, the dealer should:

A. buy the 8% coupon bond in the open market, strip it, and sell the pieces.
B. sell the 8% coupon bond short, and buy the component cash flow strips with the proceeds.
C. do nothing since the 8% bond is selling for its arbitrage-free price.

Answer: A

Question: 3628

David Garcia, CFA, is analyzing two bonds. Bond X is an option tree corporate security with a 7%
annual coupon and ten years to maturity. Bond Y is a mortgage backed security that also matures in

Page | 1449
ten years. Garcia is considering two possible interest rate scenarios—one in which rates are flat over
the entire 10-year horizon, and one in which the yield curve is sloped steeply upwards. For each
bond, Garcia has calculated the nominal spread over the 10-year U.S. Treasury issue as well as the
zero-volatility spread. The zero-volatility spread would differ the most from the nominal spread:

A. for Bond X, when the yield curve is sloped steeply upwards


B. for Bond Y, when the yield curve is sloped steeply upwards
C. for Bond X, when the yield curve is flat

Answer: B

Question: 3629

The bonds of Joslin Corp. are currently callable at par value. The bonds mature in eight years and
have a coupon of 8%. The yield on the Joslin bonds is 175 basis points over 8-year U.S. Treasury
securities, and the Treasury spot yield curve has a normal, rising shape. As yields on bonds
comparable to the Joslin bonds decrease, the Joslin bonds will most likely exhibit:

A. negative convexity
B. increasing modified duration
C. increasing effective duration

Answer: A

Question: 3630

Donald McKay, CFA, is analyzing a client's fixed income portfolio. As of the end of the last quarter, the
portfolio had a market value of $7,545,000 and a portfolio duration of 6.24. McKay is predicting that
the yield for ali of the securities in the portfolio will decline by 25 basis points next quarter. Which of
the following statements regarding the portfolio's performance next quarter is most accurate?

A. For the expected change in portfolio yield next quarter, the market value of the portfolio will
change by approximately 6.24%.
B. If the yield curve has a 50 basis point downward parallel shift next quarter, the portfolio will
increase in value by approximately $235,404.
C. The portfolio's ending value after the expected decline in yields will be approximately $7,427,298.

Answer: B

Question: 3631

Kathy Hurst, CFA, is valuing a 4-year zero coupon security. She is provided the following information:
--------------------------------
6.0%
--------------------------------
7.3%
?
8.9%
-------------------------------
The 4-year spot rate is 7.5%.
Calculate the one-year forward rate two years from now ().

A. 7.3%.
B. 7.8%.

Page | 1450
C. 8.0%.

Answer: B

Question: 3632

ABC Corporation has just issued $200 million of 6.5% $1,000 par value bonds at face value. Which of
the following requirements in the indenture for these bonds would most likely be considered a
negative covenant? ABC must:

A. maintain its manufacturing equipment in good condition.


B. make timely semiannual payments of interest and principal when due.
C. have paid all bond coupon payments due before it can pay cash dividends.

Answer: C

Question: 3633

Ron Travis, CFA, manages a portfolio of long-term and short-term bonds. The portfolio is equally
weighted between 1-year, 2-year, 10-year, and 20-year maturities and currently has a portfolio
duration equal to 7.0. Travis is concerned that 1- and 2-year interest rates are going to increase by
100 basis points while 10- and 20-year rates decrease by 100 basis points. If his prediction is correct,
Travis' measure of duration will be ineffective at predicting interest rate risk since portfolio duration
is only accurate when the:

A. yield curve does not shift.


B. shift in the yield curve is parallel.
C. yield curve steepens.

Answer: B

Question: 3634

Which of the following best describes an option that gives the owner the right to sell 100 shares of
stock only on the expiration date three months from now at a strike price of $35, when the current
stock price is $25? This option is an:

A. out-of-the-money American put option.


B. in-the-money European put option.
C. out-of-the-raoney European put option.

Answer: B

Question: 3635

Roland Cad owns a portfolio of large capitalization stocks. He has a positive long term outlook for the
stock market, but Carl is worried about the possible effects of recent changes in monetary policy. Carl
would like to protect his portfolio from any sudden declines in the stock market, without selling his
holdings. The most likely way for Carl to achieve his objective of limiting the downside risk of his
portfolio is to:

A. sell put options on the S&P 500.


B. sell an S&P 500 futures contract.
C. buy an S&P 500 forward contract.

Page | 1451
Answer: B

Question: 3636

Julia Chen, a portfolio manager for U.S.-based Dane Investments, has just established a short
position in Swiss franc currency futures as part of a currency overlay strategy. The position consists of
100,000 contracts with an initial margin of $4,000, a maintenance margin of $2,500, and a contract
price of 0.9120 USD/CHF. If the futures price on the subsequent two days is 0.9300, and 0.8928,
respectively, what will be her margin account balance at the end of the second day?

A. $4,000
B. $6,200
C. $7,720

Answer: B

Question: 3637

Sue Wie, CFA, is the chief financial officer for Garth Company. The company will need to borrow S75
million in the near future to fund a plant expansion. Wie expects interest rates will rise and decides
to hedge against this risk using a 3 * 6 LIBOR based forward rate agreement (FRA). The underlying
rate for this FRA is:

A. 60-day LIBOR.
B. 90-day LIBOR.
C. 180-day LIBOR.

Answer: B

Question: 3638

Call options on the stock of Verdant, Inc., with a strike price of $45 are priced at $3.75. Put options
with a strike price of $45 are priced at $3.00. Which of the following most accurately describes the
potential payoffs for owners of these options (assuming no underlying positions in Verdant)?

A. The call writer has the maximum loss exposure.


B. The put buyer has the maximum loss exposure.
C. The put writer has the maximum potential gain.

Answer: A

Question: 3639

An analyst is considering buying a call option on ZXC stock, which is currently trading at $33.75 per
share. Three month call options with a strike price of $30 are trading at a premium of $4.50. Identify
which of the following statements is most likely true regarding the ZXC call options.

A. The ZXC call options are currently out of the money.


B. The breakeven underlying price for ZXC stock is $38.25 per share.
C. The potential upside of the ZXC call options is unlimited.

Answer: C

Page | 1452
Question: 3640

IRK Investments is actively engaged in various risk management strategies involving swaps. The
company currently has a position as the fixed rate payer in a quarterly fixed for equity swap with an
interest rate of 6.8%, a tenor of five years and notional principal of $10 million. Payments on the
swap are netted. The underlying equity return is based on the S&P 500 Index. IRK currently owes a
payment of $400,000. Which of the following is most likely correct?

A. The underlying equity index experienced a loss greater than 1.7% over the quarter.
B. The underlying equity index experienced a loss less than 1.7% over the quarter.
C. The underlying equity index experienced a loss equal to 1.7% over the quarter.

Answer: A

Question: 3641

An analyst is interested in determining the value of a real estate investment and has estimated the
following data for the property:
----------------------------------------------------------------------------------
Net operating income $50,480 Cost of debt 8.2%
Depreciation $3,550 Cost of equity 12.5%
Interest expense $2,720 WACC 9.6%
Tax rate 35% Cap rate 11.0%
----------------------------------------------------------------------------------
Which of the following is closest to the value of the property using the income approach?

A. $403,900
B. $458,900
C. $466,500.

Answer: B

Question: 3642

Bob Kramer, CFA, manages money for high net worth clients. Kramer creates an investment portfolio
tailored to his clients' specific needs using mutual funds. Kramer is considering the following
Emerging Market Fund and uses a live year time frame. Exhibit 1 details the Emerging Market Fund's
fees and expenses.
Exhibit 1 - Fees and expenses for Emerging Market Fund

Kramer expects the Emerging Market Fund to earn 12% per year.
Select the class of Emerging Market Fund shares that are most appropriate for Kramer's clients.

Page | 1453
A. C Class C

Answer: A

Question: 3643

Which of the following statements is least likely to be a unique risk associated with a hedge fund?

A. Cash needs arising from marking positions to market.


B. Unexpected absence of normal liquidity under extreme market conditions.
C. Higher volatility of returns as compared to traditional equity funds.

Answer: C

Question: 3644

A commodity market is in contango if:

A. the spot price is higher than the futures price.


B. the spot price is equal to the futures price.
C. the spot price is lower than the futures price.

Answer: C

Question: 3645

Jacques Fontenot wants to place an order to purchase 10,000 shares of BQ Inc. at a price of €75.00 or
below. The snares are currently trading for €82.1 bid and €.82.2 ask. What type of order should
Fontenot place?

A. Market order.
B. Stop loss order.
C. Limit order.

Answer: C

Question: 3646

Ned Jones, CFA, manages an endowment fund. The fund's asset allocation includes domestic stocks
and bonds, international stocks and bonds, as well as real estate investments. Jones wants to
establish an accurate benchmark to compare the fund's performance against. Which of Jones'
following statements concerning indexes to be used for benchmarking is least likely correct?

A. The creation of an investment bond index is difficult because of bond pricing issues, and because
the universe of bonds is constantly changing.
B. Correlations between bond index returns for different countries have tended to be lower than
correlations between different bond indexes within a single country.
C. Sovereign bond indexes outside the United States represent after tax returns.

Answer: C

Question: 3647

Two stocks have identical risk, but one of them offers a higher expected return than the other. This

Page | 1454
apparent inefficiency in the market:

A. indicates that arbitrageurs must be unaware of the mispricing.


B. may persist and even grow larger before any correction occurs.
C. can only arise when arbitrageurs lack the capital to exploit the situation.

Answer: B

Question: 3648

Three equity analysts at Schiler & Company are debating their supervisor's claim that significant
excess return can be generated by exploiting inefficiencies in the capital markets. Analyst A states,
"... the large number of profit maximizing investors researching investment opportunities creates an
efficient market." Analyst B rebuts by stating, "Over the past three years, my technical analysis
strategy has outperformed all the major benchmarks, which proves the markets are not efficient."
Analyst C states, "High transactions costs improve the information efficiency of capital markets." The
statement that is most likely to be correct was made by:

A. Analyst A.
B. Analyst B
C. Analyst C

Answer: A

Question: 3649

Rose Half, CFA, is analyzing EI Toro Electric Company. She has collected the following data:
What is El Toro Electric Company's expected long-run rate of return?

A. 10.5%.
B. 13.3%.
C. 15.0%.

Answer: B

Question: 3650

Peterson Manufacturing has earnings per share of $4.00 and paid a dividend of $1.00 per share.
Peterson's return on equity is 16.0%. Peterson is considering a debt issue that would increase its
financial leverage. Peterson is also considering increasing its dividend payout ratio. Assuming all
other factors are constant, Peterson's potential growth rate:

A. will increase due to the increased leverage, and increase further due to the higher payout ratio.
B. will increase due to the higher payout ratio, but this increase will be offset to some extent by the
increased leverage.
C. will increase due to the increased leverage, but this increase will be offset to some extent by the
higher payout ratio.

Answer: C

Question: 3651

Curzon Corp reported the following in its Shareholder's Equity account:


In calculating a Price to Book value for Curzon, the appropriate book value per common share is

Page | 1455
closest to:

A. $10.50
B. $12.50
C. $13.13

Answer: B

Question: 3652

Given the academic research supporting the efficiency of the stock market, which of the following is
the least accurate description of a portfolio manager's role in an efficient market?

A. Identifying and specifying a client's objectives and constraints.


B. Specifying an explicit investment strategy to meet the client's needs.
C. Diversifying the client's portfolio across all asset classes to eliminate systematic risk.

Answer: C

Question: 3653

Which of the following is the least accurate statement about the price-to-sales multiple?

A. Price-to-sales is a poor valuation technique for growth companies.


B. Sales growth drives all subsequent earnings and cash flows.
C. Revenue has minimal accounting manipulation concerns relative to other numbers.

Answer: A

Question: 3654

A U.S. investor is considering investing in a security of a company in a developing country. The


country's market is characterized by infrequent trading, high inflation, large market volatility, low
operating leverage, political unrest, low debt usage, and a depreciating exchange rate. In
determining the appropriate country risk premium for the developing country, the investor should
consider:

A. liquidity risk, exchange rate risk, financial risk, business risk, balance sheet risk
B. financial risk, liquidity risk, exchange rate risk, country risk, business risk
C. business risk, variability risk, country risk, exchange rate risk, financial risk

Answer: B

Question: 3655

An advantage of the duration/convexity approach over the full valuation approach is:

A. its superior accuracy for nonparallel shifts in the yield curve


B. it is not based on yield to maturity, which is a summary measure.
C. it saves considerable time when working with portfolios of bonds.

Answer: C

Question: 3656

Page | 1456
Six-month LIBOR is an interest rate which:

A. represents the interest rate paid on a CD that matures in 6 months


B. is the return available on the shortest term euro-denominated securities.
C. is determined by adding a small spread to the yield available on a UK government bond maturing
in 6 months.

Answer: A

Question: 3657

An analyst is evaluating an annual-pay bond with a yield to maturity of 7.0%. The bond-equivalent
yield of this bond is:

A. equal to 7.0%
B. less than 7.0%
C. greater than 7.0%

Answer: B

Question: 3658

An economist has forecast that the term structure of interest rates will remain perfectly flat.
According to the liquidity preference theory, the economist's forecast implies that future short-term
interest rates will:

A. decrease over time


B. increase over time
C. equal current short-term interest rates

Answer: A

Question: 3659

Identify the most accurate statement regarding collateralized borrowing transactions.

A. Repurchase agreements usually offer the lowest interest cost.


B. Margin buying usually allows for borrowing a higher percentage of the collateral value.
C. Margin buying is usually the preferred transaction structure for institutional bond investors.

Answer: A

Question: 3660

Eric Webb, an individual investor in a high tax bracket, would like to purchase a 5-year zero-coupon
security with no credit risk. His investment adviser has recommended U.S. Treasury STRIP securities,
and has told Webb that either coupon strips or principal strips would meet his needs. Which of the
following statements is TRUE regarding the investment adviser's recommendation?

A. While principal strips have no credit risk, there is credit risk in coupon strips.
B. The adviser should have informed Webb that the principal strips have higher reinvestment risk
than the coupon strips.
C. The adviser should have informed Webb that STRIP securities may have negative tax

Page | 1457
consequences related to accrued interest.

Answer: C

Question: 3661

Jeff Stone, CFA, is evaluating a newly issued mortgage backed security for his bond portfolio. Stone
expects interest rates to rise gradually over the next few years. If Stone's interest rate forecast is
correct, prepayment risk of the mortgage backed security:

A. will fall to zero, as borrowers will have no incentive to prepay their loans.
B. will increase, as curtailments become more likely.
C. will decrease, although prepayments will still occur.

Answer: C

Question: 3662

An analyst stated that the purpose of a collateralized mortgage obligation is to redistribute


prepayment risk among investors with different risk tolerances while at the same time reducing total
prepayment risk for all tranches in the structure. The analyst's statement is correct:

A. only with respect to redistribution of risk


B. only with respect to reducing total prepayment risk
C. with respect to both redistribution of risk and reducing total prepayment risk

Answer: A

Question: 3663

Jorge FuIIen is evaluating a 7% 10-year bond that is callable at par in 5 years. Coupon payments can
be reinvested at an annual rate of 7%, and the current price of the bond is $106.50. The bond pays
interest semiannually. Should Fullen consider the yield to first call (YTC) or the yield to maturity
(YTM) in making his purchase decision?

A. YTM, since YTM is greater than YTC.


B. YTC, since YTC is less than YTM
C. YTC, since YTC is greater than YTM.

Answer: B

Question: 3664

PRC International just completed a S234 million floating rate convertible bond offering. As stated in
the indenture, the interest rate on the bond is the lesser of 90-day LIBOR or 10%. The indenture also
requires PRC to retire $5.6 million per year with the option to retire as much as $10 million. Which of
the following embedded options is most likely to benefit the investor? The:

A. 10% cap on the floating interest rate.


B. accelerated sinking fund provision for principal repayment.
C. conversionoption on the convertible bonds

Answer: C

Page | 1458
Question: 3665

Richard Wallace manages a portfolio of fixed-income securities for a large multinational investment
firm. Wallace's portfolio is exposed to reinvestment risk, which he is attempting to reduce by adding
securities with low levels of reinvestment risk. Of the following bonds, Wallace should most
appropriately choose:

A. a mortgage-backed security with scheduledprincipal and interest payments


B. an 8%, 10-year Treasury bond with semiannual payments
C. a 15-year Treasury strip.

Answer: C

Question: 3666

Siegel, Inc. has issued bonds maturing in 15 years but callable at any time after the first 8 years. The
bonds have a coupon rate of 6%, and are currently trading at $992 per $ 1,000 par value. If interest
rates decline over the next few years:

A. the call option embedded in the bonds will increase in value, but the price of the bond will
decrease.
B. the price of the bond will increase, but probably by less than a comparable bond with no
embedded option.
C. the price of the bond will increase, primarily as a result of the increasing value of the call option.

Answer: B

Question: 3667

Bond X carries a rating of BBB-/Baa3. Bond Y has a rating of B/B2. Both bonds are callable after five
years, and both bonds mature in ten years. Identify the most accurate statement regarding the credit
risk of these bonds. Which bond's value would be most affected by a ratings downgrade, and which
bond has the higher default risk?

A. Bond X would be more affected by a ratings downgrade, but Bond Y has higher default risk.
B. Bond Y would be more affected by a ratings downgrade, but Bond X has higher default risk.
C. Bond X has higher default risk, but both bonds would feel equivalent effects of a ratings
downgrade.

Answer: A

Question: 3668

Karen Callaway is an investor in the 35% tax bracket. She is evaluating a tax-exempt municipal
security with a tax-exempt yield of 4.5%. What is the taxable equivalent yield (TEY) of the municipal
security?

A. 2.9%.
B. 6.9%.
C. 12.9%.

Answer: C

Question: 3669

Page | 1459
Ron Logan, CFA, is a bond manager. He purchased $50 million in 6.0% coupon Southwest
Manufacturing bonds at par three years ago. Today, the bonds are priced to yield 6.85%. The bonds
mature in nine years. Identify the most accurate statement regarding the pricing and yield of these
bonds.

A. The bonds are trading at a discount, and the yield to maturity (YTM) has increased since purchase.
B. The bonds are trading at a premium, and the yield to maturity (YTM) has decreased since
purchase.
C. The bonds are trading at a discount and the yield to maturity (YTM) has decreased.

Answer: A

Question: 3670

Steve Brown is questioned by his superior about the commonly cited criticisms and benefits of the
derivatives market. Which of Brown's statements regarding the criticisms and benefits of derivative
markets is most likely correct?

A. Derivatives markets are often criticized for being too risky and illiquid for all but the most
knowledgeable investors.
B. Derivatives benefit financial markets due to the price discovery and risk management functions
they provide.
C. Derivatives benefit financial markets by generating high fees for dealers wilting to make a market
in these securities.

Answer: B

Question: 3671

Two portfolio managers at an investment management firm are discussing option strategies for their
clients' portfolios. The first manager is considering a covered call strategy on Consolidated Steel Inc.
(CSI). The manager states that the strategy is attractive since it will increase the expected returns
from the anticipated appreciation in CSI, while reducing the downside risk. The second manager is
considering a protective put strategy on Millwood Lumber Company (MLC). The manager states that
the protective put strategy will allow his investors to retain an infinite profit potential while limiting
potential losses to an amount equal to the initial stock price minus the put premium. Determine
whether the comments made by the first and second manager are correct.

A. Only the first manager is incorrect.


B. Only the second manager is incorrect.
C. Both the first manager and the second manager are incorrect.

Answer: C

Question: 3672

An investor takes a long position in a corn futures contract. Initial margin on the contract is 10% of
the contract value and maintenance margin is half of the initial margin. If, at the beginning of the
second trading day for the contract, the investor receives a margin call, it is least likely that:

A. variation margin is greater than maintenance margin.


B. the final trade from the previous day is greater than the contract price.
C. the average of the last few trades from the previous day is less than the contract price.

Page | 1460
Answer: B

Question: 3673

Anne Quincy took the short side of a forward contract on the S&P 500 Index three months ago in an
attempt to hedge short-term changes in her index portfolio. The contract had a term of six months at
the purchase date, a contract price of $ 1,221 and Mason Inc. as the counterparty. Quincy is now
considering unwinding her short position using either a three-month Mason Inc. contract with a price
of $1,220, a three-month JonesCo contract with a price of S1,219, or a three-month Redding
Company contract with a price of $1,218. If Quincy wants to minimize credit risk, which of the
following should she do? Take the long position in the contract with:

A. JonesCo.
B. Mason Inc.
C. Redding Company.

Answer: B

Question: 3674

An analyst is evaluating a European call option with a strike price of 25 and 219 days to expiration.
The underlying stock is currently trading for $29, and the analyst thinks that by the option expiration
date the stock will be valued at $35. If the risk-free rate is 4.0%, what is the lower bound on the value
of this option?
A. $0
B. $4.00
C. $4.58.

Answer: C

Question: 67

Chris Kramer holds three options that expire on the same day. Option 1 is a call option on the stock of
Blintz Company with a strike price of $58. Option 2 is an interest rate put option on 90-day LIBOR
with a strike price of 5.4% and notional principal of $1,000. Option 3 is a put option on InstaCare
stock with a strike price of $23. On the expiration date of the options, if the price of Blintz stock is
$64, 90-day LIBOR is 3 0%, and the price of InstaCare stock is $29, which option will have a payoff
with the largest present value?

Answer: A

Question: 3675

Party A enters into a plain vanilla 1-year interest rate swap agreement with Bank B in which he will
make fixed-rate payments in exchange for receiving floating-rate payments based on LIBOR plus 100
basis points. Assume that payments are made quarterly in arrears based on a 360-day year. The fixed
rate on the swap is 6.5%. The current interest rates on 90, 180, 270, and 360-day LIBOR are 5.2%,
5.5%, 5.8%, and 6.0%, respectively. If the notional principal is SI00 million, what will Party A's net
cash flow at the end of the first quarter equal?

A. -$675,000
B. -$75,000
C. +$75,000

Page | 1461
Answer: B

Question: 3676

Krissy Steele, CFA, manages money for high net worth individuals. Steele develops unique
investment policies for all of her clients and uses various investment funds to construct portfolios.
However, Steelehas been reluctant to use hedge funds. Which of the following statements made by
Steele is least likely to be correct?

A. The volatility of historical returns associated with hedge fund indexes understates their true risk
level.
B. Hedge fund returns are normally distributed.
C. Published information on hedge fund returns is based on incomplete historical data.

Answer: B

Question: 3677

When compared to a traditional mutual fund, an ETF will most likely offer:

A. betterrisk management
B. less portfolio transparency
C. higher exposure to capital gains distribution taxes.

Answer: A

Question: 3678

An investor purchases oil commodity futures contracts worth $25 million and an equal amount of 10-
year Treasury notes with an interest rate of 3.5%. Assuming that oi! prices rise by 10% and the price
of the notes remains unchanged, the total return of the position after three months is closest to:

A. $2,500,000
B. $2,600,000
C. $2,700,000

Answer: C

Question: 3679

Archie Boone, CFA, is the managing director at Hoffman Advisors, an alternative investment
management company. Boone is reviewing the work of a real estate analyst and finds that in
calculatingnet operating income (NOI) for a property, the analyst has understated vacancy by $3,000,
overstated depreciation expense by $4,000, overstated insurance expense by $4,000, and
understated interest expense by $2,000. If Boone corrects the analyst's estimates of NOI for all these
items, the updated estimate will:

A. increase by $1,000 as the restatement of vacancy will be partially offset by the restatement of
insurance expense.
B. increase by $1,000 as the restatement of depreciation expense will be partially offset by the
restatement of vacancy.
C. decrease by $1,000 as the restatement of insurance expense will be more than offset by the
restatement of vacancy and interest expense.

Page | 1462
Answer: A

Question: 3680

An investor purchased a stock for $60 a share using margin from his broken If the initial margin
requirement is 40%, and the maintenance margin requirement is 20%, which of the following best
describes the price at which a margin call will initially be triggered?

A. Below $30.
B. Below $45.
C. Below $48.

Answer: B

Question: 3681

In a rebuttal to comments made by Dilbertico's fundamental analyst, Keith Howard states that future
changes in stock prices cannot be predicted based on a company's institutional ownership. State
which form of the efficient market hypothesis (EMH) Howard's statement supports and also state a
type of empirical study which tests that form of the EMH.

A. Weak form of the EMH andtrading rulestest.


B. Semistrong form of the EMH and trading rules test.
C. Semistrong form of the EMH and event study.

Answer: C

Question: 3682

Bart Wiggum believes the current level of the S&P 500 index reflects all public information. To
convince his supervisor of his hypothesis, Wiggum has downloaded a daily price series for the S&P
500 index for the period 1950 to 2007. Which of the following tests can be used to test Wiggum's
belief about public information?

A. Runs test.
B. Autocorrelation test.
C. Earnings surprise test.

Answer: C

Question: 3683

The constant-growth dividend discount valuation model states that the fair price of a share of
common equity is determined by dividing next period's forecasted dividend by the difference
between the cost of equity capital and the firm's long-term sustainable growth rate. Using this
relationship, the cost of equity capital can alternatively be stated as:

A. D/V + g.
B. RFR-(+ RFR).
C. expected growth rate of dividends minus required rate of return.

Answer: A

Page | 1463
Question: 3684

An analyst with Guffman Investments has developed a stock selection model based on earnings
announcements made by high P/E stocks. The model predicts that investing in companies with P/E
ratios twice that of their industry average that make positive earnings announcements will generate
significant excess return. If the analyst has consistently made superior risk-adjusted returns using this
strategy, which form of the efficient market hypothesis has been violated?

A. Weak form of market efficiency.


B. Semistrong and weak forms of market efficiency.
C. Strong, semistrong, and weak forms of market efficiency.

Answer: B

Question: 3685

Mark King, CFA, is valuing Nacho Inc., a food distributor. Nacho is currently selling for $28 per share
and has a 3.0% dividend yield. The risk-free rate is 4%, and the expected return on the market is 8%.
King has calculated Nacho's beta to be 1.25. Based on King's analysis, Nacho stock's intrinsic value is
$30 per share. King should:

A. invest in Nacho shares.


B. not invest in Nacho shares because the required rate of return is less than the expected rate of
return.
C. not invest in Nacho shares because the required rate of return is greater than the expected rate of
return.

Answer: A

Question: 3686

An analyst uses a temporary supernormal growth model to value a common stock. The company paid
a $2 dividend last year. The analyst expects dividends to grow at 15% each year for the next three
years and then to resume a normal rate of 7% per year indefinitely. The analyst estimates that
investors require a 12% return on the stock. The value of this common stock is closest to:

A. $48.
B. $53.
C. $71.

Answer: B

Question: 3687

Douglas Morin is discussing market efficiency with some college students who are visiting his firm.
Morin states that market efficiency would increase if the cost of trading decreases, if the cost of
information decreases, and if arbitrageurs had less capital. Morin is least likely to be correct in his
opinion about:

A. thecost of trading
B. the cost of information
C. arbitrageurs

Answer: C

Page | 1464
Question: 3688

James Fry, CFA, is evaluating the potential investment merit of Cushing Corporation. Cushing's most
recent year's earnings were S5.00 per share, and Cushing paid a dividend of $1.50 per share. Fry
forecasts that Cushing will cam $4.70 per share next year. Fry estimates Cushing's future growth rate
will be 10%, with a required rate of return of 12%. Based on the information provided, calculate
Cushing's leading price to earnings (P/E) ratio. If the required rate of return is increased, indicate
whether Cushing's P/E ratio will be higher or lower.

A. The P/E ratio is 15.0 and an increase in the required rate of return would results in a higher P/E
ratio.
B. The P/E ratio is 15.0 and an increase in the required rate of return would result in a lower P/E
ratio.
C. The P/E ratio is 17.6 and an increase in the required rate of return would result in a lower P/E
ratio.

Answer: C

Question: 3689

Berger Corporation has a profit margin of 10.0%, total asset turnover of 0.75, financial leverage of
1.6, and debt/equity ratio of 62.5%. Profit margin is defined as Net income/Sales, total asset
turnover is Sales/ Total assets, financial leverage is Total assets/Equity, and debt/equity ratio is Total
debt/Equity. Berger\s payout ratio is 60.0%. If these ratios are sustainable for the long term, the best
estimate of Berger's growth rate of earnings and dividends is:

A. 4.8%.
B. 7.2%.
C. 7.5%.

Answer: A

Question: 3690

Increasing which factor in the dividend discount model, without changing the other two, would be
least likely to increase a stock's price-to-earnings (P/E) ratio?

A. The expected dividend payout ratio.


B. The required rate of return on the stock.
C. The expected constant growth rate of dividends.

Answer: B

Question: 3691

A drawback of using the price-to-book value ratio as a valuation tool is that book value:

A. does not reflect human capital.


B. is not appropriate for valuing firms with large, highly liquid assets.
C. is only effective in valuing companies that are not expected to continue as a going concern.

Answer: A

Page | 1465
Question: 3692

Antun Blasevic manages a fixed-income mutual fund which holds a variety of high-yield corporate
bonds. His largest position is in Garjun Technologies, which currently trades to yield 8.75%, while the
equivalent maturity U.S. Treasury yields only 5.25%. Which of the following is the most accurate
description of the yield spread between Garjun Technologies and U.S. Treasuries?

A. The yield ratio is 1.67.


B. The absolute yield spread is 67%.
C. The relative yield spread is 350 basis points.

Answer: A

Question: 3693

Kelly Clark, CFA, is a fixed income analyst for Convex Capital. She is evaluating a 15-year bond with a
6.0% coupon. At the current interest rate of 5.5%, the bond is priced at $1,050.62. Clark calculates
that a 25 basis point drop in interest rates increases the bond's price to $1,077.20, while a 25 basis
point increase in interest rates reduces the bond's price to $1,024.90. Based on the information
provided, calculate the bond's effective duration.

A. 4.98
B. 5.06
C. 9.96

Answer: C

Question: 3694

Assume that there is a widely accepted belief in the U.S. that 1-year interest rates will remain stable
at their current level of 3.25%. A yield curve derived from spot rates on U.S. Treasury securities
shows the following data:
MaturitySpot Rate
1 year 3.25%
2 years 4.00%
5 years 6.80%
10 years 7.20%
The yield curve based on this data is least consistent with which theory of the term structure of
interest rates?

A. Pure expectations.
B. Liquidity preference.
C. Market segmentation.

Answer: A

Question: 3695

A 6% U.S. Treasury security maturing 9/30/10 is quoted at a price of 97.625 on July 1. The bond pays
interest semiannually on March 31 and September 30. On July 1, the clean price of this bond would
be closest to:

A. $976.25
B. $991.17

Page | 1466
C. $946.41

Answer: A

Question: 3696

An analyst is considering a bond for purchase. The bond has a coupon that resets semiannually and is
determined by the following formula:
coupon = 12% - (3.0 * 6-month Treasury bill rate)
Identify what type of bond this is, and calculate the coupon rate this bond would reset to if the 6-
month Treasury bill rate is 4.5%.

A. This bond is an inverse floater, and the coupon would reset to 1.50%.
B. This bond is an inverse floater, and the coupon would reset to 0.00%.
C. This bond is a step up note, and the coupon would reset to 4.50%.

Answer: B

Question: 3697

Which of the following statements best describes the relationship between a valuation factor and its
effect on the present value of a bond, holding all else constant?

A. Using a lower discount rate will generate a lower present value.


B. A bond with a lower coupon rate will have a higher present value.
C. Using a higher discount rate will generate a lower present value.

Answer: C

Question: 3698

Two newly hired fixed income analysts are debating the merits of federal agency backed mortgage
securities, specifically mortgage passthroughs and collateralized mortgage obligations (CMOs).
Analyst A and Analyst B make the following statements:
Analyst A:Investors in mortgage pass-through securities backed by one mortgage pool have equal
exposure to prepayment risk, whereas investors in the CMOs of one pool have different exposures to
prepayment risk.
Analyst B:Investors in CMOs have greater protection against default risk than investors in mortgage
pass-through securities due to additional credit enhancement-Identify whether the statements of
each analyst are correct or incorrect.

A. Only Analyst A is correct.


B. Only Analyst B is correct.
C. Neither analyst is correct.

Answer: A

Question: 3699

Sharon Foster owns a portfolio of two bonds. The first bond is a mortgage backed security (MBS) with
a coupon rate well above current market rates for securities with similar characteristics. Foster also
owns a callable corporate bond with five years to maturity and a coupon rate of 9%. The bond is
nonrefundable. Comparable corporate issues brought to market recently were priced to yield 6.5%.
The risks that Sharon faces by holding each of these securities could best be described as:

Page | 1467
A. interest rate risk for the MBS, and call risk for the corporate bond
B. price compression for the MBS, and reinvestment risk for the corporate bond
C. prepayment risk for the MBS, and call risk for the corporatebond

Answer: C

Question: 3700

Michelle Garcia, CFA, is analyzing two newly issued corporate debt securities for possible purchase
by a client. Bond X is a noncallable 10-year coupon bond currently trading at 102.50. Bond Y is a
noncallable 10-year coupon bond currently trading at 98.25. Garcia wants to ensure that her client is
fully aware of any probable changes in the bonds' values as they approach maturity. Holding interest
rates constant, which of the following best describes how each bond's price will change as it
approaches maturity?

A. The price of both bonds will decrease.


B. The price of Bond X will decrease, and the price of Bond Y will increase.
C. The price of Bond X will increase, and the price of Bond Y will decrease.

Answer: B

Question: 3701

The bonds of Grinder Corp. trade at a nominal spread of 150 basis points (bp) above comparable
maturity U.S. Treasury securities. The option adjusted spread (OAS) on the Grinder Corp. bonds is 75
bp. Using this information, and assuming that the Treasury yield curve is flat, which of the following
statements is most likely to be true?

A. The zero-volatility spread should be 75 bp.


B. The zero-volatility spread for these bonds is 225 bp.
C. The option cost component of these bonds should be 75 bp.

Answer: C

Question: 3702

Tony Horn, CFA, is evaluating two bonds. The first bond, issued by Kanon Corp., pays a 7.5% annual
coupon and is priced to yield 7.0%. The second bond, issued by Samuel Corp., pays a 7.0% annual
coupon and is priced to yield 8.0%. Both bonds mature in ten years. If Horn can reinvest the annual
coupon payments from either bond at 7.5%, what would his return be on each bond, assuming the
bond was held to maturity?

A. Greater than 7.0% on the Kanon bonds and less than 8.0% on the Samuel bonds.
B. Less than 7.0% on the Kanon bonds and less than 8.0% on the Samuel bonds.
C. Greater than 7.0% on the Kanon bonds and greater than 8.0% on the Samuel bonds.

Answer: A

Question: 3703

Maria Reyes, CFA, recently purchased a 10 year floating rate bond which is reset semiannually. The
bond's coupon is based on the six-month Treasury rate plus 200 basis points with a cap of 8.50%.
Identify the TRUE statement regarding these floating rate bonds.

Page | 1468
A. The maximum coupon rate on these bonds would occur when the six-month Treasury bill was at
8.50%.
B. These floating rate bonds have more interest rate risk than comparable floating rate bonds that
reset annually.
C. If the six-month Treasury rate has been greater than 7.00% for the past 12 months, these bonds
will be priced similar to comparable fixed rate securities.

Answer: C

Question: 3704

Chris Renburg owns the following portfolio of option-free bonds:

Calculate the duration of Renburg's bond portfolio.

A. 6.682
B. 6.753
C. 7.082

Answer: B

Question: 3705

A large silver mining corporation in Australia is expecting to have three large inflows of raw silver
resulting from a discovery of three silver seams that were previously undetectable. The firm expects
the first silver inflow to be ready for sale in nine months, followed by the second inflow three months
later and the final inflow six months later. The mining company is expecting the price of silver to
begin a downward trend for the next 18 months and wants to hedge the expected inflows without
exposing themselves to credit risks. The most appropriate instrument the company should use is a:

A. series of futures contracts expiring in 9, 12, and 15 months.


B. series of forward contracts expiring in 9, 12, and 15 months
C. swap contract with payments in 9, 12, and 15 months

Answer: A

Question: 3706

Consider two options, X and Y. Option X has a strike price of S40 and is selling in the marketplace for
$4. Option Y has a strike price of $32 and is selling in the market place for $3. The underlying assets
for the options, Stock X and Stock Y, have a current market price of $43 and $29, respectively. Which
of the following are most likely TRUE about Option X and Option Y?

Page | 1469
A. Option X is an expiring call, and option Y is an in-the-money put.
B. Option X is an in-the-money put, and option Y is an expiring call.
C. Option X is an in-the-money call, and option Y is an expiring put.

Answer: C

Question: 3707

Gus McCray, CFA, went long one oil futures contract at a price of SI 10 on Monday. Oil closed at $115
on Wednesday, and the contract expired on Thursday with oil at $117. To maximize his gain, McCray
should:

A. have closed out his position by selling one oil futures contract close to expiration.
B. have accepted cash settlement on his long position.
C. be indifferent between closing out his position by selling the contract and accepting cash
settlement.

Answer: C

Question: 3708

Pete Morris has written a deep out-of-the-money call option on the stock of Omacon, a small
capitalization technology company with a very promising medical software product. Omacon stock
had risen 365% for the 12 months ended just three weeks ago, but delays with release of the new
software have disappointed investors, and the stock has lost 50% of its market value in the past three
weeks. When he wrote the options yesterday, Morris received a premium of $3.00 each. Morris
would have risk only:

A. if the stock price rose above the option strike price.


B. if the stock price fell below the option strike price.
C. in the amount of the premium he received.

Answer: A

Question: 3709

KCE stock is currently selling for $51.13 per share in the market. Six-month American put options on
KCE with a strike price of $55 are available, and the risk-free rate of interest is 3.66%. Calculate the
lower bound for the KCE American put options.

A. $2.89
B. $3.75
C. $3.87.

Answer: C

Question: 3710

Mary Hames has bought a long FRAwith a notional principal of $10 million. The agreement expires in
30 days, and is based on 90-day LIBOR. The FRA is based upon an initial rate of 4.75%. Assume that at
expiration, 90-day LIBOR is 5.5%, and 60-day LIBOR is 5.25%. Calculate the payoff at expiration.

A. $12,338 paid to Hames.


B. $18,496 paid to Hames.

Page | 1470
C. $19,008 paid to Hames.

Answer: B

Question: 3711

The Pairagain mutual fund has entered into an equity swap with SingleSol, LLC, with a notional
principal of $50 million. Pairagain has agreed to pay the quarterly return on the NASDAQ 100 in
exchange for a fixed rate of 7.0%. The initial price of the NASDAQ 100 was 1825, and the value at the
end of the first quarter, 91 days later, was 1755. The swap uses a 365-day year convention. What is
the net payment to be made at the end of the first quarter?

A. SingleSol pays $2,790,411.


B. SingleSol pays $1,917,808.
C. SingleSol pays $872,603.

Answer: A

Question: 3712

The value of an existing single-family home used for residential purposes will most likely be
calculated using the:

A. cost approach.
B. income approach
C. sales comparison approach

Answer: C

Question: 3713

John Gavin, CFA, manages money for high net worth individuals. Gavin utilizes a combination of
open-end and closed-end mutual funds to meet each individual's investment objectives. Gavin is
evaluating a mutual fund that has assets of $233 millionand liabilities of $2 million. In addition, the
Fund has a sales charge of 4% and a redemption fee of 1%. The Fund has 16.8 million shares. Gavin
makes the following two statements.
Statement 1:The net asset value (NAV) of the fund is $13.75.
Statement 2:The primary difference between a closed-end and open-end fund is their method of
computing net asset value(NAV).
Indicate whether Statement 1 and Statement 2 are correct.

A. Only Statement 1 is correct.


B. Only Statement 2 is correct.
C. Statements 1 and 2 are both correct.

Answer: A

Question: 3714

An analyst valuing the non-controlling shares of a closely held company is using a similar firm quoted
on the NASDAQ with relatively high trading volume as his base for a comparable company analysis.
He is most likely to use the shares of the publicly traded comparable company and apply:

A. onlya marketability discount

Page | 1471
B. only a minority interest discount
C. both a marketabilityand minority interest discount

Answer: A

Question: 3715

Wireless Company received venture capital financing that allowed the company to begin commercial
manufacturing. This stage of financing is known as:

A. first-stage
B. second-stage
C. third-stage

Answer: A

Question: 3716

One of the functions of secondary markets is that they:

A. provide liquidity, and a financial futures contract is an example of a security trading on such a
market.
B. provide liquidity, and a private placement is an example of a security trading on such a market.
C. provide fees, and a financial futures contract is an example of a security trading on such a market.

Answer: A

Question: 3717

Mike Bowers has observed that during 2004 the S&P 500 index officially reported a return of 20%.
After recalculating the returns on an equally weighted basis, Bowers estimates that the index
returned 15%. The difference in the two calculations of return is best explained by:

A. large capitalization stocks outperforming small capitalization stocks.


B. small capitalization stocks outperforming large capitalization stocks.
C. the interest expenses on margin accounts.

Answer: A

Question: 3718

Which of the following is the least accurate description of behavioral finance and related investor
bias?

A. Avoiding or ignoring new information that would call a decision into question is an example of
overconfidence bias.
B. Behavioral finance may explain some of the anomalies that tend to refute the efficient markets
hypothesis.
C. Committing more funds to a position that has lost value is an example of escalation bias.

Answer: A

Question: 3719

Page | 1472
Ian Clark, CFA, is a technical analyst. Clark believes that information is incorporated gradually into
securities markets and that, as a technician, he can take advantage of this process. However, tests of
the efficient market hypothesis indicate security returns are random over time and new information
is processed rapidly. Clark makes the following statements:
Statement 1:Studies have reported that small capitalization stock returns are positive on a risk-
adjusted basis.
Statement 2:Although the academic research indicates that markets are weak form efficient, they are
not because many technical analysts beat the market.
Determine whether Clark's statements regarding tests of market efficiency are correct or incorrect.

A. Both statements arc correct.


B. Only Statement 1 is correct.
C. Only Statement 2 is correct.

Answer: B

Question: 3720

Augusta Sevilla has made the statements about company analysis and stock valuation:
Statement 1:A growth company is a growth stock. A growth company has opportunities to make
investments that yield returns above the firm's required rate of return. A growth company also offers
higher rates of return on investments in its shares they are undervalued and generate high returns
when complete information about the company arrives in the marketplace.
Statement 2:A defensive stock has a low, but not negative, beta. A stock with a negative beta is
actually pro-cyclical.
Are Sevilla's statements most likely correct?

A. Both statements are incorrect.


B. Only Statement 1 is correct.
C. Only Statement 2 is correct.

Answer: A

Question: 3721

Howard Keane is a strategist for Dove Investments. His models indicate that the expected inflation
rate will be 3.0%. The real rate of return on the S&P 500 index is expected to be 8.7%, while the real
rate of return on U.S. Treasury notes is expected to be 1.0%. Howard is interested in the current
equity risk premium. Based on the information above, the equity risk premium is closest to:

A. 4%.
B. 6%.
C. 8%.

Answer: C

Question: 3722

Tim Jan, CFA, relies on the earnings multiplier model in performing his fundamental analysis. His
model is based on the constant-growth DDM. Jan is evaluating two stocks, A and B, that have the
same 10% required rate of return and the same expected growth rate in dividends. Stock A has a
higher retention rate than stock B. Which stock should have the higher P/E ratio?

A. Stock A.

Page | 1473
B. Stock B
C. Both stocks should have the same P/E ratio

Answer: B

Question: 3723

Which of the following statements is the best description of the ability of arbitrageurs to correct
market anomalies?

A. Only the more significant mispricing may be exploited while others are allowed to persist.
B. There is a high degree of reliability that apparent mispricing will be corrected.
C. Investors supply largely unlimited amounts of capital to arbitrageurs because of the reliability of
the returns.

Answer: A

Question: 3724

As an analyst for Donavan Financial Advisors, Lou Marvin must estimate the appropriate inputs for
the firm's equity valuation models. Donavan's preferred valuation model is the single-stage dividend
discount model (DDM). Members of Marvin's valuation team have supplied him with several pieces
of data related to TMQ Utilities, including the company's earnings and dividends from the most
recent year, the expected real risk-free rale, and the expected nominal growth in net income. To
estimate the value of TMQ Utilities, additional inputs to the DDM that will be necessary include the:

A. price-to-cash flow ratio and the expected cash flow per share.
B. expected rate of inflation and the expected earnings retention rate.
C. historical growth rates in dividends and the required return on the Utility bond index.

Answer: B

Question: 3725

Fred Fleming is considering working as a security analyst for Sector Investments. In the past, the firm
has preferred to employ a top-down investment approach to analyzing potential investments.
However, Fred prefers to use a bottom-up approach. Which of the following statements regarding
the two analytical approaches is most accurate?

A. Fundamental analysts only employ the bottom-up approach to security selection.


B. Analysts using the top-down approach begin with forecasts of economic growth, interest rates,
and inflation.
C. The bottom-up approach emphasizes industry analysis for investment selection.

Answer: B

Question: 3726

Denver Savin, CFA, is an analyst for an investment boutique. Savin is considering investing in one of
the following two companies. Savin's evaluation is based on his estimation of price to cash flow.
Bell United is a producer of aluminum. The company earned record profits in the latest year. Delmar
is a major supplier to the worldwide auto industry. U.S. auto industry problems have reduced
Delmar's earnings in the latest year.

Page | 1474
Based on the price to cash flow multiple, state whether Delmar or Bell United is more attractive for
purchase

A. Delmar is more attractive.


B. Bell United is more attractive.
C. Delmar and Bell United are equally attractive.

Answer: A

Question: 3727

James Martindale, CFA, manages a small mutual fund specializing in defensive equity investments.
Martindale has purchased 10,000 shares of BLM stock for the portfolio after deciding that the stock
would contribute to meeting the fund's objectives. Which of the following characteristics would
make BLM stock suitable for Martindale's mutual fund?

A. A high beta.
B. Low systematic risk.
C. Higher rate of return than other stocks with similar risk characteristics.

Answer: B

Question: 3728

Martina Profis runs a fixed-income portfolio for the pension fund of Whether by Whittaker, Ltd. The
portfolio contains a $12 million position in the corporate bonds of Dewey Treadmills. Profis is
concerned that interest rates are likely to rise and has calculated that a 50-basis point increase in
rates would cause a 4% decline in the Dewey bonds. The dollar duration of the position in Dewey
Treadmills is closest to:

A. $96,000
B. $480,000
C. $960,000.

Answer: C

Page | 1475
Question: 3729

A 10-year 5% Treasury bond is issued at a price to yield 5.2%. Three months after issuance, market
rates for 10-year Treasuries decline by 100 basis points. The most likely price of this bond at issuance
and three months later is:

A. above par at issuance, but below par three months later


B. below par at issuance, but above par three months later
C. below par at issuance, and below par three months later

Answer: B

Question: 3730

The method used by the U.S. Treasury to issue debt is best described as a(n):

A. regular cycle auction—multiple price


B. regular cycle auction—single price
C. ad hoc auction system

Answer: B

Question: 3731

Peterson Investments has three bond portfolio managers. Manager X invests only in U.S. Treasury
STRIPS. Manager Y invests only in putable corporate bonds. Manager Z invests only in mortgage-
backed securities guaranteed by GNMA. Which of the following statements is most likely to be TRUE
regarding the risks of each manager's portfolio?

A. Manager X has more reinvestment risk than Manager Z.


B. Manager Z has more volatility risk than Manager X.
C. Manager Y has more interest rate risk than Manager X.

Answer: B

Question: 3732

Wendy Jones, CFA, is reviewing a current bond holding. The bond's duration is 10 and its convexity is
200. Jones believes that interest rates will fall by 100 basis points. Calculate the bond's percentage
price change based on a 100 basis point decline.
A. -8.0%.
B. +8.0%.
C. +12.0%.

Answer: C

Question: 3733

Two analysts have been asked to submit brief summaries to their supervisor on various risks related
to bond investing. Included in these summaries were the following statements from each analyst:
Analyst A:In a decreasing interest rate environment, both callable and amortizing securities will
experience the negative effects of price compression.
Analyst B:The reinvestment risk of a portfolio can be reduced by replacing zero coupon securities

Page | 1476
with shorter maturity, amortizing securities such as early tranches of a CMO.
Identify whether the statements of each analyst are correct or incorrect.

A. Only Analyst A is correct.


B. Only Analyst B is correct.
C. Both analysts are correct.

Answer: A

Question: 3734

Bartel Corp. has decided to build a new manufacturing facility in a foreign country where production
costs will be considerably less than costs at Bartel's aging domestic plant. Bartel expects the
increased profits from this off-shore facility will completely pay off the cost of construction within
seven years. Bartel hopes to finance the new facility with a single debt issue with the lowest possible
coupon rate. The form of borrowing best suited to this project would most likely be:

A. medium-term notes (MTN).


B. debentures with a negative pledge clause
C. secured mortgage bonds.

Answer: C

Question: 3735

Jane Higgins, CFA, is analyzing a corporate bond that she believes is a suitable addition for a client's
portfolio. The 10-year security has a 7.50% annual coupon and is non-callable by the issuer. The bond
is currently priced at 104.5 to yield 7.177%. According to Higgins* analysis, for a 25 basis point
decrease in yield, the bond's price will increase to 107.4166 and for a 25 basis point increase in yield,
the bond's price will decrease to 101.3834. Higgins' estimation of the bond's effective duration is
closest to:

A. 5.77.
B. 10.03.
C. 11.55.

Answer: C

Question: 3736

Samson Corp. needs to raise $100 million. Delilah Jones, CFA, the Treasurer of Samson, is considering
two alternative sources of financing:
Alternative 1:Selling a large portion of the company's accounts receivable to a separate entity
established solely for this purpose. This entity would then seek to obtain a higher credit rating than
Samson's own BB rating, to reduce the required coupon rate on the bond issue.
Alternative 2:Issuing bonds, but simultaneously entering into an equity swap so that coupon
payments can be covered by appreciation in the underlying equity index. Jones believes the equity
index returns will be high for several years.
Identify the common names for these alternatives.

A. Both alternatives are structured notes.


B. Alternative I is a structured note, and alternative 2 is a special purpose vehicle.
C. Alternative 1 is a special purpose vehicle, and alternative 2 is a structured note.

Page | 1477
Answer: C

Question: 3737

Two firms, Groening Inc. and Shearer Co., have just completed simultaneous bond issuances. Both
issues have a stated coupon rate of 5%, pay interest semiannually, and have a face value of $i,000 per
bond. The Groening and Shearer issues both have a maturity of 15 years and their duration is
approximately the same. If the Groening bond have a higher convexity measure than the Shearer
bonds, which issuance will sell for the higher price?

A. Groening, since the bonds will depreciate less in a period of rising interest rates.
B. Groening, since the bonds will depreciate less in a period of falling interest rates.
C. Shearer, since the bonds will depreciate less in a period of rising interest rates.

Answer: A

Question: 3738

Four years ago, the relative yield spread between ten-year A-rated corporate securities with no
embedded options, and ten-year on-1he-run U.S. Treasuries, was 27.5%. Currently, the nominal yield
on ten-year A-rated corporate securities is 5.45%, and the yield on ten-year on-the-run U.S.
Treasuries is 4.10%. Calculate the current relative yield spread, and, assuming that any change in the
yield spread isdue to changes in the credit spread, identify whether the economy has most likely
weakened or strengthened over the past four years.

A. The current yield spread is 32.9%, indicating that the economy has weakened.
B. The current yield spread is 32,9%, indicating that the economy has strengthened.
C. The current yield spread is 24.8%, indicating that the economy has strengthened.

Answer: A

Question: 3739

The following three bonds are available for purchase:


Bond X: Noncallable, accelerated sinking fund
Bond Y: Nonrefundable, callable, accelerated sinking fund
Bond Z: Noncallable, no sinking fund
Based only on the characteristics listed above and the most likely effect of those characteristics on
yield, identify the correct order for these three bonds, from highest yield to lowest yield.

A. Bond X; Bond Z; Bond Y.


B. Bond Y; Bond Z; Bond X.
C. Bond Y; Bond X; Dond Z.

Answer: C

Question: 3740

A 1-year U.S. Treasury bill is priced to yield 4.10%. A 2-year U.S. Treasury security is priced to yield
4.65%. The 1-year forward rate one year from now is closest to:

A. 3.55%.
B. 4.38%.
C. 5.20%.

Page | 1478
Answer: C

Question: 3741

The 8% McClintock bonds maturing in 10 years are currently trading at 97.55. These bonds are
option-free and pay coupons semiannually. Which of the following statements is most likely to be
TRUE?

A. The yield to maturity is greater than 8.0%.


B. The current yield is less than 8.0%.
C. The nominal yield is greater than 8.2%.

Answer: A

Question: 3742

JonesCorp just entered into a plain vanilla interest rate swap as the fixed-rate receiver. The swap has
a tenor of four years and makes payments quarterly on a netted basis. At the time the swap was
initiated the LIBOR term structure was flat causing LIBOR to be equal to the swap fixed rate. Under
which of the following circumstances would JonesCorp be required to make a future net payment to
the swap counterparty?

A. The LIBOR term structure becomes upward sloping.


B. The LIBOR term structure remains flat but shifts down.
C. The LIBOR term structure becomes downward sloping.

Answer: A

Question: 3743

Gretchen Miller has been analyzing options on the common stock of Spirit Electronics Group (SEG),
which last traded on the NASDAQ for $25.96. Miller has collected the following data on put options
for SEG stock that expire in three months:
_______________
StrikePut
_______________
22.500.25
25.000.65
27.502.00
______________
Miller has been asked by her supervisor to determine the profit on a protective put strategy using a
strike price of $25.00 if the stock price is $27.13 on the option expiration date. What figure should
Miller report to her supervisor?

A. $0.00
B. $0.52
C. S0.65

Answer: B

Question: 3744

Black Oil is an oil and gas exploration and production company. Black's management hedges its crude

Page | 1479
oil production using futures contracts. Which of the following would be the least likely method Black
would use to close out the futures position?

A. Holding the cash settled future until expiration.


B. Physically settling according to exchange rules.
C. Offsetting the transaction by shorting the oil futures contract on the same exchange.

Answer: C

Question: 3745

Coleman Industries' stock is currently trading in the market for a price of $21. Three months ago,
Myong Packard wrote a 6-month put option on 100 shares of Coleman stock for a premium of $3. The
exercise price on the put option is equal to S25. The put option is now trading in the market for
$5.25. Determine the moneyness of the put option.

A. Out-of-the money.
B. In-the-money.
C. At-the-money.

Answer: B

Question: 3746

Janice Grass, CFA, created a gourmet baby food line. To get the production lines up and running, she
must borrow $12 million. Grass is concerned that the US Federal Reserve (Fed) will raise interest
rates dramatically, so she enters into a 2 x 8 FRA agreement. The FRA is quoted at 6%. LIBOR interest
rates on the expiration day of the FRA are presented in the table below.

Based on the table, calculate the payoff on Grass’ transaction.

A. 0.
B. -$2,630.81
C. -$2,956.39

Answer: A

Question: 3747

Page | 1480
Two junior portfolio managers at ContraFunds, a hedge fund manager, have been asked to
summarize the mechanics of utilizing futures contracts for the firm's training manual. The first
manager, Tina Kent, submits a paragraph explaining that administering a futures position will require
bringing the margin account balance back to the initial margin level by posting maintenance margin
any time the balance falls below the variation margin level. The second manger, Martin Ramsey,
submits a paragraph explaining margin requirements are determined according to the daily
settlement price which is the average of the last few trades of the day. Are Kent and Ramsey correct
or incorrect with regard to their explanation of the mechanics of futures positions?

A. Only Kent is correct.


B. Only Ramsey is correct.
C. Neither is correct.

Answer: B

Question: 3748

Wilma Green has been following the stock price movements of Bakery Supply International (BSI) and
Hull Petrochemical Company (HPC). Green is convinced that the price of BSI stock is going to
dramatically increase from its current price of $53.60 and that the price of HPC stock is going to
dramatically decrease from its current price of $9.80. She has decided to use options to take
advantage of the situation and has thus gathered the following data on three-month put and call
options for the two stocks:

If after three months, the price of BSI stock is $54.60 and the price of HPC stock is $8.13, which of the
following strategies would have yielded Green the greatest profits?

A. Short BSI put with a $45.00 strike; Short HPC call with a S7.50 strike.
B. Long BSI put with a $45.00 strike; Long HPC call with a $7.50 strike.
C. Short BSI call with a $55.00 strike; Long HPC put with a $10.00 strike.

Answer: C

Question: 3749

Sandy Hart, CFA, is evaluating the possible purchase of an apartment building. As part of her
research, Hart found the following data on three comparable properties:

Page | 1481
The market cap rate that Hart would be willing to pay for the apartment building is closest to:

A. 0.125
B. 0.143
C. 0.145

Answer: C

Question: 3750

Which of the following would be the most likely reason to use ETFs instead of similar index funds?

A. Lower market risk.


B. Intraday valuation and trading.
C. ETFs do not experience tracking error.

Answer: B

Question: 3751

Harold Stone, CFA, is an analyst for Spartacus Venture Capital. Stone is considering investing $3
million in a project with a potential $150 million return over a ten year life. The current risk-free rate
is 5%, the equity risk premium is 5%, and the project's beta is 2.0. Stone believes that the project has
a 22% probability of failure in the first four years and 13% thereafter. The expected net present value
of the project is closest to:

A. SI million
B. $2 million
C. $3 million

Answer: C

Question: 3752

Giovanni DiPaglia is VP of strategic planning for Megaquistion Holdings, a global conglomerate active
in the mergers and acquisition market. DiPaglia is looking to invest in a new portfolio company for
Megaquistion, and has selected Temptytarg, Inc. as a potential candidate. He has gathered the
following information for Temptytarg, Inc.
Price/Book Value (P/BV) for Temptytarg is closest to:

A. 1.1.
B. 1.5.
C. 2.0.

Page | 1482
Answer: C

Question: 3753

Allen Jackson believes the stock of JMH Corporation is severely overvalued. JMH trades for 12.3
times annualized earnings, but Jackson thinks the multiple should be 8.1 times. In order to take
advantage of the expected 35% drop in the price, Jackson decides to establish a short position in JMH
stock. Which of the following is Jackson least likely to do in order to establish a short position?

A. Borrow the stock from another investor.


B. Reinvest any dividend payments.
C. Post margin with his brokerage firm.

Answer: B

Question: 3754

A buy side analyst is discussing the relative functionality of the Australian stock market with her
colleagues. She states three specific attributes of the Australian stock market that infer that the
market is a well-functioning securities market. She says, 'The Australian stock market is characterized
by (1) rapid adjustment of prices to reflect new information, and (2) price continuity in the absence
of significant new information." Are these accurate descriptions of attributes of a well functioning
market?

A. Both descriptions are accurate.


B. Neither description is accurate.
C. Only one of these descriptions is accurate.

Answer: A

Question: 3755

The creation of bond indexes is relatively new. Which of the following is the least likely reason a bond
index would be difficult to create?

A. Lack of continuous trading data.


B. Constantly changing duration for bonds.
C. The universe of bonds is limited.

Answer: C

Question: 3756

Gregory Johansson has collected the following data on Trilby & Tribble, Ltd:
The sustainable growth rate of the firm is closest to:

A. 6.0%.
B. 7.1%.
C. 8.4%.

Answer: C

Question: 3757

Page | 1483
In explaining the factors in the dividend discount model that affect a stock's expected price-to-
earnings (/) ratio, which of the following statements is least accurate? Holding other factors constant:

A. as the difference between k (required rate of return on the stock) and g (expected constant growth
rate of dividends) widens, the value of/decreases.
B. as g increases, the value of/increases.
C. as the expected dividend payout ratio decreases, the value of/increases.

Answer: C

Question: 3758

Jack George is evaluating Dunger Inc., a waste management firm. The company has been
experiencing a strong 15% growth rate, which is forecasted to continue over the next three years
before growth settles down to a sustainable level. Dunger's annual return on equity is expected to be
10%. The company recently paid a dividend of $0.50 per share from reported earnings of $2.50 per
share. George has calculated a 10% weighted average cost of capital for Dunger Inc. The firm has no
debt. The company's last reported trade on the New York Stock Exchange was $35 per share. Based
on the multi-stage dividend discount model, George should:

A. not buy the stock because its intrinsic value is $32 per share.
B. buy the stock because its intrinsic value is $38 per share.
C. buy the stock because its intrinsic value is $41 per share.

Answer: A

Question: 3759

Willa Dowd collected the following information for a small-cap firm that she is evaluating:
The Price/Cash Flow (P/CF) for the small-cap firm is closest to:

A. 7.1.
B. 8.5.
C. 9.1.

Answer: A

Question: 3760

Acquire Corp. has a business model based on making accretive acquisitions each year. The company
has historically been successful in implementing its strategy. Earnings per share have grown each of
the last five years at a 15% compounded rate. During the past year, Acquire Corp. acquired a services
company with large net operating losses, representing a third leg to its business model. The other
two business segments are engineering construction and mining. The purchase price was one-half
the company's current market value. The most appropriate technique to value Acquire Corp. is based
on its:

A. price-to-book value ratio


B. forward price-to-earnings ratio
C. trailing price-to-sales ratio

Answer: B

Page | 1484
Question: 3761

Lynn Smith, CFA, and a marketing associate have a disagreement over whether the stock market is
truly efficient. The marketing associate believes that the stock market is totally efficient based on
academic research that shows actively managed mutual funds do no better than a buy-and-hold
strategy. Thus, he invests solely in index funds. Smith counters that academic research indicates that
low P/E ratio stocks provide superior risk-adjusted returns. Based on this discussion, indicate which
of the following statements regarding the efficient markets hypothesis (EMH) is correct.

A. The strong form of the EMH is supported by academic research on low P/E stocks.
B. The semistrong form of the EMH is not supported by academic research on low P/E stocks.
C. The strong form of the EMH is not supported by academic research on mutual funds.

Answer: B

Question: 3762

James Larson, CFA, manages a large capitalization growth mutual fund. Larson's benchmark is the
Russell 1000 Growth index. Larson's colleague, Kevin Moore, CFA, manages an index fund which
mimics the Russell 1000 index. Moore believes that the capital markets are fully efficient, while
Larson disagrees. Larson defends his position with the following supporting statements.
Statement 1:Market participants must be adequately compensated for processing new information
to ensure the markets remain efficient. Yet a perfectly efficient market provides no incentive to
sufficiently reward investors for processing new information. Hence, markets cannot be fully
efficient.
Statement 2:Low trading costs have led to greater trading activity, which has had the unintended
consequence of greater securities mispricing.
Are Larson's statements correct?

A. Yes.
B. Only Statement 1 is correct.
C. Only Statement 2 is correct.

Answer: B

Question: 3763

Larry Rile is evaluating the investment merits of Bing Corp., a successful motorcycle manufacturer.
Rile is forecasting a dividend in year I of $1.50 per share, a dividend in year 2 of $3.00 per share, and
a dividend in year 3 of 4.50 per share. After year 3, Rile expects dividends to grow at the rate of 6%
per year. Rile calculated a beta of 1.3 for Bing Corp. Rile expects the S&P 500 index to return 8%. The
U.S. Treasury bill is yielding 2%. Using the multistage dividend discount model, what is Bing Corp.'s
intrinsic value to the nearest dollar?

A. $92 per share


B. $102 per share
C. $112 per share

Answer: B

Question: 3764

A bond does not pay initial coupon payments but instead accrues them over a pre-determined
period and then pays a lump sum at the end of that period. The bond subsequently pays regular

Page | 1485
coupon payments until maturity. Such a bond is best described as:

A. a step-up note
B. a zero-coupon bond
C. a deferred-coupon bond

Answer: C

Question: 3765

Sara Jones, CFA, recently purchased a U.S. government security that was issued on 6/1/2007 and will
mature on 6/1/2014. Jones purchased the security in the secondary market. This security is most
likely an:

A. on-the-run Treasury note


B. off-the-run Treasury bond
C. off-the-run Treasury note

Answer: C

Question: 3766

An entity desiring to issue a fixed-income security has placed $10 million worth of loan receivables in
a special purpose vehicle (SPV) that is completely independent of the company. Additionally, the
credit rating agencies have suggested the entity secure a third-party guarantee in order to have the
security rated AAA. After completing the transfer of assets to the SPV and obtaining a letter of credit
from a national bank, the entity issued the AAA-rated security. Which of the following securities did
the entity most likely issue?

A. Commercial paper.
B. International bonds.
C. Asset-backed securities.

Answer: C

Question: 3767

A fixed-income portfolio manager at Franken Investments is considering adding a security to his


existing portfolio. The bond, issued by KDJ Company, has an option adjusted spread (OAS) equal to
0.23% and a Z-spread equal to 0.15%. The manager is concerned that his portfolio is dominated by
callable bonds and will only accept new securities if they contain no call options. Should the portfolio
manager add the KDJ bond to his portfolio?

A. Yes, the negative option cost implies the bond is putable.

Page | 1486
B. No, the positive option cost implies the bond is callable.
C. No, the negative option cost implies the bond is callable.

Answer: A

Question: 3768

Leading economists have predicted that the Federal Reserve will continue to pursue a stable
monetary policy that has characterized the last five years, keeping the price level constant into the
future. Given the Fed's monetary policy, the pure expectations theory and the liquidity preference
theory would predict, respectively, the following yield curve shapes:

A. flat and flat


B. upwardsloping and flat
C. flat and upward sloping

Answer: C

Question: 3769

On November 15, 2006, the yield curve was upward sloping with yields of 3%, 4%, and 5.5% on 1-
year, 5-year, and 10-year Treasuries, respectively. The following day, the Treasury yield curve
experienced an upward parallel shift equal to 112 basis points. Which of the following noncallable
bonds would have experienced the least percentage change in price as a result of the yield curve
shift?

A. A 6% coupon corporate bond maturing in ten years.


B. A 6% coupon corporate bond maturing in five years.
C. A 0% coupon U.S. government bond maturing in ten years.

Answer: B

Question: 3770

A bond dealer determines that the present value of a particular Treasury note based on Treasury spot
rates is greater than its market price. The dealer can generate an arbitrage profit (assuming no
transactions costs) by:

A. buying the Treasury note and selling its cash flows as Treasury STRIPS
B. buying the equivalent Treasury STRIPS and selling them as a Treasury note.
C. buying the undervalued note and selling short the Treasury security with the nearest maturity.

Answer: A

Question: 3771

Bill Foley, CFA, manages an intermediate tax-exempt bond fund. Foley makes the following two
comments about securities in his portfolio.
Statement 1:Revenue bonds usually pay a higher coupon rate than general obligation bonds.
Statement 2:Double barreled bonds are municipal securities that are exempt from both federal and
state taxes.
Which of the following best evaluates Statement 1 and Statement 2?

A. Only Statement 1 is correct.

Page | 1487
B. Only Statement 2 is correct.
C. Both Statements are incorrect.

Answer: A

Question: 3772

A Japanese auto company announces a new plant to be constructed in San Antonio, Texas. The
company will partly finance the project with a dual currency bond offering. The $200 million offering
will have a 6.0% coupon payable in yen and mature in 2021 with the final principal payment in U.S.
dollars. Indicate whether a U.S.-based investor and/or the company will assume any potential
currency exchange risk related to these bonds.

A. Investor only.
B. Issuer only.
C. Both the issuer and the investor.

Answer: C

Question: 3773

Gerald Snow is a bond manager for Long Vision Investments. Snow is evaluating potential arbitrage
opportunities. He has the following list of bonds:
Bond X is a I-year zero coupon bond selling at 950.
All three bonds have a par value of SI,000. if no arbitrage opportunity exists, the price of bond Z is
closest to:

A. S975.
B. $995
C. $1,015

Answer: B

Question: 3774

Rob Ealey, CFA, has just purchased an option-free bond with a 6.50% coupon that is currently selling
at 94.73 to yield 7.25%. If yields increase by 50 bps, the new price of the bonds would be 91.41, and
if yields decrease by 50 bps the new price of the bond would be 98.20. Determine the approximate
new price of the bond if yields decrease by 75 basis points.

A. 89.64.
B. 99.82.
C. 104.92.

Answer: B

Question: 3775

A boad matures in ten years, pays a 7% semiannual coupon, and is currently priced to yield 6.25%.
The bond is callable at par beginning five years from now. Debbie Scott is planning to purchase this
bond. Scott can currently reinvest coupon income from the bond at 5.50%. Which of the following
statementsisleast accurate?

A. If the bond is called in six years, Scott's return will be less than 6.25%.

Page | 1488
B. If Scott's reinvestment rate was 6.25%, the bond's yield to worst would be less than 6.25%.
C. If the bond was priced to yield 5.50%, the current yield would be 7%.

Answer: C

Question: 3776

Sally Ferguson, CFA, is a hedge fund manager. Ferguson utilizes both futures and forward contracts in
the fund she manages. In speaking with a client, Ferguson makes the following statements to answer
their questions about futures and forward contracts:
Statement 1:A futures contract is an exchange traded instrument with standardized features.
Statement 2:Forward contracts are marked-to-market on a daily basis to reduce credit risk to both
counterparties.
Indicate whether Statement 1 and Statement 2 are most likely correct or incorrect.

A. Only Statement 1 is correct.


B. Only Statement 2 is correct.
C. Both statements are correct.

Answer: A

Question: 3777

Peter Ulrich runs a hedge fund which specializes in using option strategies to enhance the fund's
returns. In a training session for newly hired analysts, Ulrich explains option characteristics as
follows: "The maximum profit on a short call position is always less than the maximum profit on a
long call position and a long at-the-money put option position will break even as soon as the price of
the underlying stock decreases." Determine whether Ulrich is correct with regard to his statements
about call options and put options.

A. Ulrich is correct regarding call options only.


B. Ulrich is correct regarding put options only.
C. Ulrich is correct regarding both call and put options.

Answer: A

Question: 3778

Jack Cheney purchases an IBM October 80 put contract for a premium of $5. Cheney holds the option
until the expiration date when IBM stock sells for $78 per share. At expiration, what is the loss on the
contract?

A. -$2
B. -S3.
C. -$5

Answer: B

Question: 3779

Ken Willis is the portfolio manager of an aggressive growth fund. Ken is concerned about the future
performance of his high-beta portfolio in light of his belief that the stock market is currently
overvalued. Willis' firm requires that he maintain at least 80% of the portfolio's value in equities at all
times. Willis decided his best course of action is to buy put options to protect the portfolio from the

Page | 1489
potential loss resulting from a market decline. The profits and losses from an equity portfolio
combined with long puts would have risk characteristics similar to a:

A. long call option.


B. short put and long call position.
C. None

Answer: A

Question: 3780

Bill Turner, CFA, is short a futures contract on wheat. Turner entered into the futures position three
months ago at a contract price of $50. The contract expiration is tomorrow. The settlement prices for
the past four days (from oldest to most recent) were $56, $53, $49, and $52. If the settlement price
on the expiration day is $57, which of the following best describes a method Turner is most likely to
use to terminate his futures contract?

A. Sell a futures contract for $57 and receive a mark-to-market profit of $5.
B. Buy a futures contract for $57 and incur a mark-to-market loss of $5.
C. Leave the contract open, deliver the wheat to the long, and receive a price of $57.

Answer: B

Question: 3781

A call option on Hartco stock with an exercise price of S50 and an expiration date one year from now
is worth $4.00 today. A put option on Hartco stock with an exercise price of $50 and an expiration
date one year from now is worth $2.25 today. U.S. Treasury notes maturing in one year are yielding
2.0%. Hartco does not pay a dividend. The value of Hartco*s stock is closest to:

A. $43 per share


B. $47 per share
C. $51 per share.

Answer: C

Question: 3782

Pamela Burke is a cotton farmer in Texas. Her crop will be ready for harvest in three months, but
Burke does not believe prices will remain at their current level. Burke contacts Brooke Anderson, a
derivatives dealer, to negotiate a forward contract. Anderson agrees to be the counterparty to a
forward contract that will eliminate Burke's exposure to the price of cotton. The contract is
structured as a nondeliverable forward with a contract price of S47. If the price of cotton is $49 in
three months, which counterparty will be exposed to the greater amount of credit risk and which
counterparty will make a payment?

A. Burke will be exposed to greater credit risk, and Anderson will make a payment.
B. Anderson will be exposed to greater credit risk, and Burke will make a payment.
C. Burke will be exposed to greater credit risk, and Burke will make a payment.

Answer: B

Question: 3783

Page | 1490
The annual income and expense figures for a proposed property under consideration for purchase,
along with some recent sales data, are given below.

The appraised value for the proposed property using the income approach is closest to:

A. $1,268,125
B. $1,504,375
C. $1,623,200

Answer: B

Question: 3784

Jerry Paris, CFA, manages a high yield bond fund. 20% of Paris' portfolio is invested in distressed
securities. A colleague commented that investing in distressed securities is analogous to venture
capital investing. Which of the following statements concerning distressed securities and venture
capital is least likely to be true? An investment in distressed securities is similar to an investment in
venture capital because both:

A. are illiquid,
B. are normally priced efficiently.
C. require a long time horizon.

Answer: B

Question: 3785

Kerry Garrett, CFA, manages a hedge fund. The hedge fund industry has enjoyed strong growth over
the past ten years. Garrett states that the hedge fund industry has a goal of absolute returns. In
addition, Garrett states that the industry's high Sharpe ratio indicates that hedge funds are superior
investment vehicles. Is Garrett correct with regard to his statement on hedge fund returns and/or his
statement on hedge funds as superior investment vehicles?

Page | 1491
A. Only the statement on return is correct.
B. Only the statement on superiority is correct.
C. None

Answer: A

Question: 3786

Wiotech, LLC, a private company, is in the process of developing a revolutionary drug to fight
Alzheimer's disease. The drug is in stage 2 development. The management team consists of several
experienced clinical doctors. A reputable Wall Street firm has provided venture capital financing. The
company would like to go public in the next few years. Which of the following is least likely to be a
unique risk of this investment compared to other types of investment?

A. Inexperienced entrepreneurs.
B. Uncertain time to success.
C. Limited information.

Answer: C

Question: 3787

Suppose that stock prices are mean reverting over a three to five year period. Which form of the
efficient market hypothesis does this violate?

A. None.
B. Weak form only.
C. Semistrong and weak forms only.

Answer: B

Question: 3788

Brad Rich uses an investment strategy that assumes stock prices will not reflect quarterly earnings
surprises as quickly as suggested by the efficient market hypothesis. Rich believes stocks will earn
positive abnormal rates of return over the six months following an earnings surprise. Which form of
the efficient market hypothesis would this violate?

A. None.
B. Weak form only.
C. Semistrong and weak forms only.

Answer: C

Question: 3789

One year ago, Yong Kim bought a preferred stock that had a 6% dividend yield. Now, one year later,
Kim sells the stock which is how selling at a 5% dividend yield. The preferred stock pays a fixed
annual dividend, which Kim received right before selling. What rate of return did Kim realize on his
investment?

A. 14%.
B. 20%.
C. 26%.

Page | 1492
Answer: C

Question: 3790

Roger Templeton, an analyst for Bridgetown Capital Management, is studying past market data to
identify risk factors that produce anomalous returns. He tests monthly data on each of 60 financial
and economic variables over a 15-year period to find which ones are related to stock index returns.
Based on this research, Templeton identifies three variables that show statistically significant
relationships with equity returns. He presents his results to Bridgetown's managers and recommends
implementing a trading program based on changes in these three variables. What is the most likely
reason why Bridgetown's management should be skeptical of the anomalies Templeton has
identified? The results suffer from:

A. data mining bias


B. survivorship bias
C. small sample bias

Answer: A

Question: 3791

Horace Lance, CFA, states that the efficient market hypothesis and its rigorous testing have yielded
many benefits to investors. Lance makes the following statements concerning an efficient market.
Statement I:The Efficient Market Hypothesis (EMH) assumes that changes in security prices occur in a
random fashion.
Statement 2:Portfolio managers should reduce trading turnover of client accounts.
Statement 3:The EMH establishes that the expected rate of return is the risk-free rate plus a risk
premium that is the security beta times the market price of risk.
Which of Lance's statements is least likely to be correct?

A. Statement 1
B. Statement 2
C. Statement 3

Answer: C

Question: 3792

George Judas, CFA, manages a small capitalization mutual fund. Judas only invests in companies with
low price to earnings ratios. Judas states that research suggests that returns on both small
capitalization and low P/E companies are anomalous, in that they will provide investors with superior
risk-adjusted long-term returns. Judas' supervisor counters with the following two observations.
Observation 1:The research does not adequately account for the level of risk of small capitalization
and low price to earnings ratio companies.
Observation 2:The research on small capitalization and low price to earnings ratio companies suffers
from a small sample bias.
Are the supervisor's observations most likely correct?

A. Yes.
B. Only Observation 1 is correct.
C. Only Observation 2 is correct.

Answer: B

Page | 1493
Question: 3793

Al the end of the last 12-month period, Romano s Italian Foods had net income and ending equity for
the company of $16.68 million and $115 million, respectively. Romano's declared a $7.5 million
dividend for the year. Using internally generated funds, Romano's can grow its equity by
approximately:

A. 8.0% per year.


B. 10.0% per year.
C. 14.5% per year.

Answer: A

Question: 3794

Van Jeffery, CFA, utilizes price multiples to evaluate the attractiveness of potential investment
opportunities. However, Jeffery's supervisor does not support using price multiples exclusively in
making investment decisions. The supervisor points out the following:
Statement 1:P/S ratios are not able to capture the different cost structures of companies-Statement
2: The P/CF ratio is more stable than the P/E ratio.
Statement 3:An advantage of the P/B ratio (unlike the P/E ratio) is that the P/B ratio cannot be
negative.
Which of the supervisor's three statements is least likely to be correct?

A. Statement 1
B. Statement 2
C. Statement 3

Answer: C

Question: 3795

Rock Inc. maintains a policy of paying 30% of earnings to its investors in the form of dividends. Rock
is expected to generate a return on equity of 9.3%. Rock's beta is 1.5. The equity risk premium is 6%
and U.S. Treasury notes are yielding 3%. Rock's required rate of return is closest to:

A. 9.0%.
B. 9.3%.
C. 12.0%.

Answer: A

Question: 3796

Ben Click is analyzing an equity security using price-to-earnings, price-to-sales, price-to-book value,
and price-to-cash flow ratios. Which statement correctly identifies a drawback of using one of the
listed ratios?

A. Earning power is a chief driver of investment value.


B. Accounting effects may compromise the use of book value.
C. Sales are generally less subject to distortion or manipulation than are other fundamentals.

Answer: B

Page | 1494
Question: 3797

Darlenc Villanueva provides analytical support for portfolio managers at a small investment
management firm, Villanueva's latest report highlights two companies, Company X and Company Y.
Company X has consistently earned a higher rate of return on assets than their cost of capital, but the
stock price is substantially greater than the fair value. Company Y's earnings have been pulled down
by the recent economic slowdown, but its stock price has remained stable despite the negative
returns on the overall market. Which of the following statements correctly categorizes the two
companies?

A. Stock X is a value stock and Company Y is a defensive company.


B. Stock X is a growth stock and Company Y is a cyclical company.
C. Company X is a growth company and Stock Y is a defensive stock.

Answer: C

Question: 3798

Jim Boo, CFA, is analyzing Justin Corp., a maker of home appliances. Boo's research provides the
following facts:
Calculate Justin's expected price to earnings ratio (/).

A. 8.0X
B. 10.Ox
C. I2.0x

Answer: A

Question: 3799

A 10-year, semiannual-pay $1,000 bond with a 6% coupon is currently priced at $864.10, to yield 8%.
If yields increase by 50 basis points (bp), the new price of the bond would be $833.82. If yields
decrease by 50 bp, the new price of the bond would be $895.78. The expected percentage change in
the price of this bond for a 100 bp change in yield is closest to:

A. 3.6%.
B. 7.2%.
C. 14.4%.

Answer: B

Question: 3800

The current price of a $1,000 par value, 6-year, 4.2% semiannual coupon bond is $958.97. The bond's
PVBP is closest to:

A. $0.50
B. $4.20
C. $5.01

Answer: A

Question: 3801

Page | 1495
Jack Hare, CFA, is a fixed income analyst. Hare is evaluating a 15-year zero-coupon bond, which is
priced at $30.83. Determine the issue's approximate yield to maturity.

A. 6%.
B. 7%.
C. 8%.

Answer: C

Question: 3802

Bond X and Bond Y were issued at a premium to par value three years ago. Bond X matures in five
years, and Bond Y matures in ten years. Both bonds carry the same credit rating. Bond X has a
coupon of 7.25%, and Bond Y has a coupon of 8.00%. Currently the required yield for both bonds is
7.60%. Determine whether each bond is currently priced at a premium or discount to par value.

A. Both bonds are priced at a premium.


B. Bond X is priced at a premium, and Bond Y is priced at a discount.
C. Bond X is priced at a discount, and Bond Y is priced at a premium

Answer: C

Question: 3803

Charlotte Villa, CFA, is a portfolio manager analyzing two securities. The 10-year bonds of Zehmer
Corp. are callable beginning in two years. The 10-year bonds of Cavalier Inc. are not callable, but
have a floating coupon that adjusts annually based on a margin above comparable maturity U.S.
Treasury issues with no limits on the rate adjustment. Both bond issues are rated AA. Villa uses a
computer model to value individual bonds based on their zero-volatility spread and/or option-
adjusted spread (OAS). She decided to increase the interest rate volatility assumption in her model
without changing any of the other model inputs. Identify how this change in assumption will affect
the OAS for each bond.

A. The OAS for both bonds will increase.


B. The OAS for both bonds will decrease.
C. The OAS for the Zehmer bond will decrease, but the OAS for the Cavalier bond will be unchanged.

Answer: C

Question: 3804

Rob Pirate is considering investing in a subordinated tranche in a collateralized mortgage obligation


(CMO). If Pirate wishes to measure his interest rate risk for this debt security, which measure would
be most appropriate!

A. Modified duration
B. Effective duration
C. Effective convexity

Answer: B

Question: 3805

Page | 1496
Mark Davidson and James Case are bond traders at a large fixed-income investment firm. Both
Davidson and Case have developed bond valuation models for bonds with embedded options. Using
their respective valuation models, the traders have calculated the price of BMC Corp.'s callable and
putable bonds. Davidson uses a yield volatility assumption of 23%, while Case uses an assumption of
31%. Other than the volatility assumption, the traders use identical inputs for the valuation models.
Which of the following best summarizes the output of the two valuation models?

A. Davidson's model will calculate a lower value for the call option and a lower value for the putable
bond.
B. Case's model will calculate a higher value for the call option and a lower value for the putable
bond.
C. Davidson's model will calculate a lower value for the put option and a lower value for the callable
bond.

Answer: A

Question: 3806

James Volley, CFA, is evaluating a number of municipal bonds. One of the revenue bonds on his list is
marked as "prercfunded." When Volley asks his assistant about the bonds, the assistant tells him that
the collateral behind the bonds is a portfolio of U.S. government securities, and that, therefore,
these bonds have "less credit risk than insured municipal bonds." Which of the following statements
regarding these bonds is most accurate"?

A. The bonds do indeed have less credit risk than insured municipal bonds.
B. Only general obligation bonds can be prerefunded, not revenue bonds.
C. Prerefunded bonds are secured by a cash escrow account, not by US government securities.

Answer: A

Question: 3807

Laura Mack, is considering purchasing two Treasury securities. The first is the 7-year on-the-run
Treasury issued last week that has a coupon rate of 4.98%. The second is a 7-year off-the-run
Treasury that was issued two months ago and has a coupon rate of 4.74%. Which of the following
statements regarding the two issues under consideration is most accurate?

A. The on-the-run issue has higher reinvestment risk because of its higher coupon rate.
B. The on-the-run issue has higher interest rate risk because of its higher coupon rate.
C. Both the on-the-run and the off-the-run issues have equivalent interest rate risk.

Answer: A

Question: 3808

An investor wants to take advantage of the 5-year spot rate, currently at a level of 4.0%.
Unfortunately, the investor just invested all of his funds in a 2-year bond with a yield of 3.2%. The
investor contacts his broker, who tells him that in two years he can purchase a 3-year bond and end
up with the same return currently offered on the 5-year bond. What 3-year forward rate beginning
two years from now will allow the investor to earn a return equivalent to the 5-year spot rate?

A. 3.5%.
B. 4.5%.
C. 5.6%.

Page | 1497
Answer: B

Question: 3809

Eileen Hart, CFA, is a fixed income portfolio manager for MTY Investment Management. Hart's
portfolio includes US agency mortgage-backed securities. Which of the following statements about
U.S. agency mortgage-backed securities is least accurate?

A. Cash flows from mortgage loans include prepayments, which could affect the future performance
of Hart's portfolio.
B. Pass-through securities issued by Ginnie Mae guarantee the timely payment of interest and
principal.
C. Collateralized mortgage obligations (CMO) redirect cash flows from the underlying mortgage pool
to eliminate prepayment risk.

Answer: C

Question: 3810

Which of the following statements about debt retirement features is TRUE?

A. A bond issue must be retired in its entirety when exercising a call feature.
B. A make-whole premium provision and call price are identical terms.
C. A bond can be retired early even if it is nonrefundable.

Answer: C

Question: 3811

Allison Coleman, CFA, owns a bond portfolio that includes Bond X, a callable bond with ten years to
maturity that is callable at any time beginning one year from today. Coleman's portfolio also includes
Bond Y, a noncallable security with ten years to maturity that carries the same credit rating as Bond
X. Coleman expects interest rates to drift steadily lower over the next few years. Based on this
assumption, Coleman should expect that:

A. Bond Y will experience a larger decrease in value than Bond X.


B. Bond X will benefit from positive convexity as rates decline.
C. the option embedded in Bond X will increase in value.

Answer: C

Question: 3812

Jefferson Blake invests only in bonds and other fixed-income securities. Blake believes there is a good
opportunity to purchase an undervalued 4% annual pay corporate bond with three years left until
maturity and a par value of $1,000. Blake observes that 1-year, 2-year, and 3-year Treasury strip rates
are currently 4.0%, 4.5%, and 4.75%, respectively. What is the maximum price Blake should be
willing to pay for the bond?

A. $1,069.58.
B. $979.93.
C. $958.36.

Page | 1498
Answer: B

Question: 3813

Mark Waiters' risk aversion is relatively high compared to other individual investors. Waiters is
interested in generating some income on his equity portfolio. Walters decides to establish a covered
call position on CGF stock and simultaneously establish a protective put position on HSD stock. After
establishing the covered call and protective put positions, which of the following would least likely
describe Walters' portfolio, relative to the positions before adding the options?

A. The HSD position will have a higher breakeven price and less downside risk.
B. The CGF position will have a lower breakeven price and more upside potential.
C. The HSD position will have lower upside potential and less downside risk.

Answer: B

Question: 3814

In futures markets, the role of the clearinghouse is to:

A. prevent arbitrage and enforce federal regulations.


B. act as guarantor to both sides of a futures trade.
C. reduce transaction costs by making contract prices public.

Answer: B

Question: 3815

Morgan Dexter has been asked by his supervisor to present the features of interest rate swaps to a
group of newly hired risk managers. In his presentation, Dexter notes that in a plain-vanilla interest
rate swap, there is one floating rate-payer and one fixed-rate payer. Dexter points out that the
netting arrangements typical to plain vanilla swaps reduce the credit risk for both counterparties
Dexter also states that some interest rate swaps may have two floating rate payers. Are Dexter *s
statements regarding swaps correct or incorrect?

A. Only Dexter's statement about reduced credit risk from netting is correct.
B. Only Dexter's statement about two floating rate payers is correct.
C. Both statements are correct.

Answer: C

Question: 3816

Debbie Chon, CFA, is evaluating a put option on Lincoln Industrial. Lincoln's current stock price is $64
per share and the company will pay a $0.56 dividend. The 90-day U.S. Treasury bill is yielding 5.3%.
Lincoln's 3-month European call option with a strike price of $70 has a premium of $3.50. Based on
the put-call parity, calculate the value of the associated Lincoln put option.

A. $8.05
B. $8.60
C. $9.15

Answer: C

Page | 1499
Question: 3817

Peter Black is an options trader for HighSmith Investments. Black trades options on the U.S. and U.K.
stock exchanges. Over the past three weeks, Black has been following the price movements of
options on two companies: U.S.-based Pacific Chemicals Inc. (PCI), and U.K.-based Merchant Clothing
Co. (MCC). Black has observed that over the past few days, the price of put options on PCI stock have
suddenly increased, and the price of call options on MCC stock have suddenly increased. Which of
the following provides the most accurate explanation of Black's observations? Interest rates in:

A. the U.S. have risen and the volatility of MCC stock has risen.
B. the U.K. have fallen and the volatility of PCI stock has risen.
C. the U.S. have fallen and the volatility of MCC stock has risen.

Answer: C

Question: 3818

An investor holds a long position in a futures contract on the S&P 500 Index. The futures contract has
a term of three months, requires 10% margin, and has a futures price of 1,574. The investor posted
$37,500 into the margin account at contract initiation. After the contract initiation, the futures price
on the index experienced infrequent but dramatic drops. Two days ago, the investor received a
margin call and was required to post an additional $17,500 to the margin account. Which of the
following is most likely the maintenance margin on the contract?

A. $17,500.
B. $18,750.
C. $22,500.

Answer: C

Question: 3819

Frank Holmes, CFA, is reviewing Martha Inc, a distributor. Holmes is interested in the company's
European-style call option, which has a value of $5.90. Currently, Martha's stock is trading at $33 per
share and pays no dividend. The exercise price of both the call and put options is $30, with 80 days to
expiration. The current risk-free rate is 5.50%. Martha's put option sells for $2.75. Calculate the
synthetic call option value.

A. $3.35
B. $5.75
C. $6.10

Answer: C

Question: 3820

Kim Lee is valuing a closely held private shoe retailing company. She compares the company to other
shoe retailing competitors that are publicly traded and are highly liquid. Relative to the private
company, the shares of the publicly traded competitors most likely include a:

A. marketability discount.
B. minority interest discount.
C. control premium.

Page | 1500
Answer: B

Question: 3821

Nina Foch, CFA, is considering investing in an exchange traded fund (ETF). However, she is unsure
how the ETF compares to open-end and closed-end funds. Which of the following statements
comparing ETFs with open-end and closed-end funds is least likely to be true?

A. ETF shares can be sold short, while open-end fund shares cannot be shorted.
B. The legal structure of an ETF is similar to a closed-end fund.
C. ETF shares trade like closed-end fund shares.

Answer: B

Question: 3822

Ann Fowler, CFA, has a client that wants to invest in hedge funds. Fowler recommends the client
invest in a Fund of Funds (FOF), which will invest in a variety of hedge funds. Fowler makes the
following statements:
Statement 1:Investing in several different types of hedge funds will reduce risk compared to investing
in a single fund.
Statement 2:An important part of the selection process is due diligence to resolve any transparency
issues.
Which statements are correct?

A. Only statement 1 is correct.


B. Only statement 2 is correct.
C. Both statements 1 and 2 are correct.

Answer: A

Question: 3823

There are two stocks in an index:


Nothing has changed except now the price of Company A's stock is selling for $4 per share. What is
the price-weighted index and what is the value-weighted index?

A. Price-weighted = 8; Value-weighted =2.00


B. Price-weighted = 10; Value-weighted = 1.25
C. Price-weighted = 7; Value-weighted = 1.50
D. Price-weighted = 6; Value-weighted = 0.95

Answer: C

Explanation:
Price weight = [(4) + (10)] / 2 = 7

Question: 3824

An investor buys 200 shares of ABC at the market price of $100 on full margin. The initial
margin requirement is 40 percent and the maintenance margin requirement is 25 percent.
What is the leverage factor of the margin purchase?

A. 2.50.

Page | 1501
B. 0.40.
C. 0.60.
D. 4.00.

Answer: A

Explanation:
The leverage factor is 1/initial margin requirement, 1/0.4 = 2.50.

Question: 3825

Which of the following regarding bond market indexes is FALSE?

A. Unlike stocks, bonds lack continuous price trading data.


B. The bond universe is more stable than the stock universe.
C. There are more bond issues than stocks.
D. The price volatility of bonds is constantly changing due to the influence of maturity and market
yield on bond durations.

Answer: B

Question: 3826

The implications of stock market efficiency for fundamental analysis indicates that using the top-
down approach to analyze a firm will yield:

A. superior returns using past and current information.


B. superior returns compared to a randomly selected buy-and-hold portfolio of stocks.
C. returns that are not superior if the analysis only looks at past and current information.
D. superior returns using only past information.

Answer: C

Explanation:
The evidence is that fundamental analysis does not lead to superior returns using thetop-down
approach if the analyst uses only past and current information. The analyst'sjob has to bedirected
towards doing a superior job of estimating the variables thatcause long-run trends in realized
returns.

Question: 3827

Market efficiency is NOT based on which of the following assumptions?

A. A large number of profit maximizing participants are analyzing securities independently.


B. All of these choices are correct.
C. Market participants always correctly adjust prices when new information is received.
D. The expected returns implicitly include risk in the price of a security.

Answer: C

Explanation:
Market efficiency does not assume that market participants correctly adjust prices, just that their
price adjustments are unbiased.

Page | 1502
Question: 3828

An investor buys 200 shares of ABC at the market price of $100 on full margin. The initial margin
requirement is 40 percent and the maintenance margin requirement is 25 percent. If the shares of
stock later sold for $200 per share, what is the rate of return on the margin transaction?

A. 400%.
B. 100%.
C. 200%.
D. 500%.

Answer: A

Explanation:
200 shares at a cost of $100/share is $20,000 (i.e., 200x100). With a 40% initial margin requirement,
the cost of the investment would be $20,000x0.4=$8,000. When the shares are sold, the portfolio is
worth $40,000 ($200x200). Hence, the rate of return would be [(40,000/8,000)-1]x100 = 400%.

Question: 3829

All of the following statements are true about futures and options clearinghouses except:

A. The clearinghouse acts as the opposite side of all trades once they are initiated.
B. The clearinghouse guarantees that traders in the futures market will honor their obligations.
C. The clearinghouse requires the daily settlement of all margin accounts.
D. Clearinghouses have defaulted on less than one half of one percent of their trades.

Answer: D

Question: 3830

Which of the following statements about margin is false?

A. The initial margin must be posted within three days of the trade.
B. Each trader’s margin account is marked-to-market at the end of every day to reflect any gains and
losses they have experienced for that day.
C. If the margin account balance falls below the maintenance margin level, the trader must bring it
back up to the initial margin level.
D. The initial margin on a contract approximately equals the maximum daily price fluctuation of the
contract.

Answer: A

Explanation:
Margin must be posted before the trade.

Question: 3831

Which one of the following statements is true?

A. When the stock price is above the strike price, a put option is in-the-money.
B. When the stock price is below the strike price, a call option is in-the-money.
C. When the stock price is above the strike price, a put option is out-of-the-money.
D. When the stock price is below the strike price, a call option is at-the-money.

Page | 1503
Answer: C

Question: 3832

Which of the following statements about put options is false?

A. The most the buyer of a put can lose is the premium.


B. The most the buyer can gain is unlimited.
C. The most the writer can lose is the stock’s price less the premium.
D. The most the writer can gain is the put’s premium.

Answer: B

Question: 3833

Which of the following statements about call options at expiration are true?

A. The call buyer’s maximum loss is the call option’s premium.


B. The profit potential to the buyer of the option is unlimited.
C. The potential loss to the writer of the call option is unlimited.
D. The greatest profit the writer of a call option can make is the stock price minus the premium.

Answer: D

Question: 3834

Which of the following statements about interest rate swaps is false?

A. The parties agreeing to swap cash flows are call the counterparties.
B. Swap facilitators are the people who bring the counterparties together.
C. A plain vanilla interest rate swap is a fixed rate for variable rate swap. The variable rate is usually
set at LIBOR flat. The period of time involved in the swap is called the tenor.
D. Notional principal is exchanged at initiation and termination while only net interest rate payments
are exchanged on the settlement dates.

Answer: D

Question: 3835

You are analyzing a firm that has:


If you think next year’s earnings will be $4 per share, what value would you place on this stock?

A. $22.24
B. $26.67
C. $33.32
D. $45.45

Answer: C

Explanation:
Dividend payout = 1 - earnings retention rate = 1 - .4 = .6
=+ B(-) = .06 + 1.2(.11 - .06) = .12
g = (retention rate)(ROE) = (.4)(.12) = .048

Page | 1504
P/E = (div payout rate)/(k - g) = .6/(.12 - .048) = 8.33
Price = (E)(P/E) = (4)(8.33) = 33.32

Question: 3836

Cavanaugh’s stock will start paying a $2 per share dividend four years from today (D4). Analysts are
estimating at that time Cavanaugh’s dividend growth rate will stabilize at 7%. If investors want to
earn 12% on investments of this type, what value would they put on Cavanaugh shares today?

A. $28.47.
B. $31.89.
C. $40.00.
D. $50.18.

Answer: A

Explanation:
P3 = D4 / (k – g) = 2 / (.12 - .070 = $40P0 = [n = 3; i = 12; FV = 40] = $31.89

Question: 3837

Using time series analysis you project that the Widget Index’s sales per share will be $1,000. Youalso
project that:
Assuming a P/E ratio of 10X, project the Widget Index’s value at year-end?

A. $250
B. $325
C. $432
D. $490

Answer: D

Explanation:
= [($Sales)(EBDIT profit margin) - ($D) - ($I)](1 - T)= [($1,000)(.15) - ($50) - ($30)](1 - .30) = $49Price at
end = EPS(P/E) = (49)(10) = 490

Question: 3838

Assuming all other factors remain unchanged, which one of the following would reduce themarket
P/E ratio?

A. The market ROE is expected to increase.


B. The dividend payout ratio increases.
C. The dividend growth rate increases.
D. The required rate of return increases.

Answer: D

Explanation:
P/E = (dividend payout ratio) / (k - g)
k = real rate + inflation = risk premium
g = (ROE) (retention rate)
a. ROE up, g up, P/E up; Payout up, P/E up;
c. g up, k-g down, P/E up; k up, P/E down

Page | 1505
Question: 3839

Which of the following statements is false?

A. The price to book value ratio is seldom greater than one for industrial firms. This is caused by the
fact that due to accounting rules book value will exceed market value in most firms.
B. Since cash flows are more stable than earnings the price to cash flow ratio should be used in
conjunction with the P/E ratio.
C. Economic value added (EVA) is a present value technique used to measure management’s ability
over time to add value to the firm through their investment decisions.
D. Market Value Added equals the market value of the firm’s capital minus the adjusted book value
of the firm’s capital.

Answer: A

Question: 3840

When the relative strength ratio, the stock's price divided by the index's prices, is increasing
thismeans the stock is:

A. doing worse than the index.


B. doing the same as the index.
C. doing better than the index.
D. tracking the index.

Answer: C

Question: 3841

Weak form efficiency states that excess risk adjusted returns cannot be obtained by using:

A. insider information
B. technical analysis
C. fundamental analysis
D. portfolio theory

Answer: B

Explanation:
Weak form - you can’t make excess returns using technical analysis. Semi-Strong form - you can’t
make excess returns using fundamental analysis, which is the use of public information. Strong Form
- you can’t make excess returns using non-public information.

Question: 3842

If a bond sells at a discount:

A. its YTM will exceed its horizon yield.


B. its current yield is greater than its YTM.
C. its coupon rate is greater than its current yield.
D. its coupon rate is less than the market rate of interest.

Answer: D

Page | 1506
Explanation:
When a bond sells at a discount, the market rate goes above the coupon rate and the bond’s price
falls below par. The current yield is the coupon rate / price, so as price falls below 1000 the current
yield rises above the coupon rate. The YTM considers the current yield plus the capital gain
associated with the discount.

Question: 3843

You have a 3-year investment horizon. You can buy a 10% semi annual coupon, 10 year bond for
$1,000. You estimate you can reinvest the coupons at 12% and sell the bond in 3 years time for
$1,050. Based on this information, what is your horizon return?

A. 9.5%
B. 10.0%
C. 11.5%
D. 13.5%

Answer: C

Explanation:
1.Find the FV of the coupons and interest on interest:
n = 3(2)=6; i = 12/2 = 6; PMT = 50; compute FV =348.77
2. Determine the value of the bond at the end of 3 years: given = 1,050.00
1,398.77
3. Equate FV (1398.77) with PV (1000) over 3 years (n=6); compute i = 5.75(2) = 11.5%

Question: 3844

Consider a bond that pays an annual coupon of 5 percent and that has three years remaining until
maturity. Assume the term structure of interest rates is flat at 6 percent. How much does the bond
price change over the next twelve-month interval if the term structure of interest rates does not
change?

A. 0.84.
B. -0.84.
C. -0.56.
D. 0.00.

Answer: A

Explanation:
The bond price change is computed as follows: Bond Price Change =
New Price – Old Price = (5/1.06 + 105/1.062) - 5/1.06 + 5/1.062 + 105/1.063 = 0.84.

Question: 3845

A bond has a modified duration of 6 and a convexity of 62.5. What happens to the bond's price if
interest rates rise 25 basis points?

A. it goes up 1.46%
B. it goes down 1.46%
C. it goes up 4%
D. it goes down 15%

Page | 1507
Answer: B

Explanation:
ΔP/P = (-)(MD)(Δi) + (C)ΔP/P = (-)(6)(+.0025) + (62.5)=- .015 + .00039 = - .01461

Question: 3846

Which theory about the term structure of interest rates is correct?

A. The expectations hypothesis indicates that investors have varying opinions about future interest
rates.
B. The liquidity premium hypothesis assumes investors will give up yield to lock in longer-term
interest rates.
C. That the segmented markets hypothesis contends that borrowers and lenders prefer particular
segments of the yield curve.
D. The expectations hypothesis contends that the long-term rate is equal to the expected short-term
rate.

Answer: C

Question: 3847

You have a 1 year, 10% semi annual coupon bond with a price of $975. If the 6 month T-Bill rate is
6%, what is the one year theoretical spot rate?

A. 7.4%
B. 8.7%
C. 9.9%
D. 12.8%

Answer: D

Explanation:
975 = 50/1.06 + 1050/
975 - 47.17 = 1050/
= 1050/927.83 = 1.1317
r =- 1
r = 6.4%, note this rate is on a semi annual basis. If you annualized this rate by doubling
it you would get 12.8.

Question: 3848

The six-year spot rate is 7% and the five-year spot rate is 6%. What is the implied one-year
zerocoupon bond rate five years from now?

A. 5%
B. 6.5%
C. 7%
D. 12%

Answer: D

Explanation:

Page | 1508
5r1= [(1 +/ (1 +] - 1 = [(1.07/(1.06] – 1[1.5 / 1.338] - 1 = .12

Question: 3849

Which of the following definitions about appraisal is false?

A. The cost approach to valuation is based on what it would cost to rebuild the property at today’s
prices.
B. The comparative sales approach to valuation is based on the sales price of properties that are
similar to the subject property.
C. The income approach to valuation projects the property’s value as the present value of its future
annual after-tax net operating income.
D. The ‘capitalization rate’ equals the required rate of return minus the growth rate.

Answer: C

Question: 3850

David Bateman is contemplating the purchase of a shopping center. The average annual after tax
cash flow for the next ten years is expected to be $30,000. The property cost $750,000. Bateman will
put down 25 percent and borrow the rest. In ten years, the property will be sold netting $350,000
after taxes. What is the approximate yield on the shopping center?

A. 21.35.
B. 17.2%.
C. 12.3%.
D. 14.8%.

Answer: B

Explanation:
[30000 + (350,000-187,500)/10] / [(187500 + 350000) /2]

Question: 3851

Which of the following statements about investment companies is false?

A. The 12b-1 plan allows funds to deduct up to 1.25% of average assets per year to cover marketing
expenses.
B. Closed end investment companies trade at the net asset value of the shares.
C. The fund’s net asset value is the prevailing market value of all the fund’s assets divided by the
number of fund shares outstanding.
D. The typical management fees charged to compensate the Management Company for the expense
of running the fund are between ¼ and 1% of the fund’s net asset value.

Answer: B

Explanation:
Closed end funds sell for whatever people will pay for them. CE funds typically sell at premiums or
discounts from their NAV.

Question: 3852

Leveraged buyout financing is used by management to:

Page | 1509
A. take a private firm public.
B. buy additional product lines.
C. develop new lines to revitalize the firm.
D. take a public firm private.

Answer: D

Question: 3853

Tony Nguyen works in the investor relations department of a medium sized technology firm. He
recently received the following e-mail: “I am an investor concerned with agency problems between
managers and stockholders. What assurance do I have that the company works to align the interests
of these two groups?” Which of the following actions that the firm has taken does NOT address the e-
mail’s concerns?

A. Every employee receives performance shares and cash bonuses, based on his or her position and
company earnings per share results.
B. The company recently expanded its executive stock-option program to include middle-level
managers.
C. The company recently adopted a shareholder rights plan that allows existing shareholders
favorable terms over outside parties. The plan is triggered if a person or group acquires beneficial
ownership of 10 percent or more of the company's common stock.
D. The board has a reputation for aggressively monitoring current management and is quick to
remove poor-performing managers.

Answer: C

Explanation:
The shareholder rights agreement is an example of a poison pill, a device used to discourage hostile
takeovers. The threat of takeover is a mechanism designed to reduce conflicts between managers
and shareholders, so anything that weakens the threat would not align the interests of managers and
shareholders.
Performance shares, cash bonuses, and stock option plans are examples of managerial compensation
mechanisms designed to align the interests of stockholders and managers. The aggressive board is an
example of the threat of firing mechanism. The other mechanism noted in the reading is direct
intervention by shareholders. The goal of these mechanisms is to motivate managers to achieve a
higher stock price, which improves shareholder wealth.

Question: 3854

Thomas Otto is an associate in the strategic consulting group for a medium-sized manufacturing
company. Historically, the firm has financed projects using internal equity funds. The company is
approaching the retained earnings break-even point, and the group’s Executive Vice President asks
Otto to determine the change in the weighted average cost of capital (WACC) if the firm uses
external equity funds instead of internal funds. An analyst in the group provides the following
information:
And, the firm’s investment bank provides the following projections for a new common stock issue:
Which of the following choices best completes the following sentence? Using this information, Otto
reports that the WACC will:

A. increase by 1.82%.
B. increase by 1.66%.
C. decrease by 1.66%.

Page | 1510
D. remain the same.

Answer: A

Explanation:
Step 1: Calculate WACC using internal equity:
WACC = (wd)(kd) + (ws)(ks), where wd, wsare the weights used for debt and retained earnings.
WACC = (0.40 * 6.0%) + (0.60 * 13.0%) =10.20%.
Step 2: Calculate WACC using external equity (new common stock):
First, we need to determine the cost of new common equity.
Thecost of new common equityis given by:
ke= [D1/ (P0(1 – F))] + g
where
F = the percentage flotation cost incurred in selling new stock, or

(current stock price – funds going to company) / current stock price

D1= Dividend in next year

P0= Current stock price

g = Dividend growth rate


Here, D1and F are given, and ke= [ 2.33 / (30 * (1 – 0.035))] + 0.08 = 0.1604 or 16.04%
Then, calculate WACC as before:
WACC = (wd)(kd) + (we)(ke), where wd, weare the weights used for debt and common equity.
WACC = (0.40 * 6.0%) + (0.60 * 16.04%) =12.02%.
Step 3: Calculate the difference:
The change in WACC = 12.02% - 10.20% = 1.82%, or an increase of 1.82%.

Question: 3855

While studying for the Level 1 CFA examination, Leonard Hart draws the following graph of a firm’s
capital structure. Using his graph and the assumptions below, determine which of the following
statements A through D below is FALSE.

A. The value for X is $90 million.


B. The weighted average cost of capital (WACC) of 10.00% uses retained earnings, the WACC of
12.25% uses common stock.
C. The firm should use this graph to adjust projects for risk.
D. As the firm needs to raise more and more capital, the weighted average cost of capital (WACC) will
remain lower than the marginal cost of capital.

Page | 1511
Answer: A

Explanation:
This graph represents the marginal cost of capital (MCC).
BPRE= (Retained Earnings) / (Equity Fraction, we)
Here, RE = earnings * (1 – Payout) = $100 million (1 – 0.45) = $ 55.0 million,
And BPRE= $55.0 million / 0.50 = $110 million.
The other statements are true.
WACC using internal equity (retained earnings):
WACC = (wd)(kd) + (ws)(ks), where wd, wsare the weights used for debt and retained earnings.
WACC = (0.50 * 7.0%) + (0.50 * 13.0%) =10.00%.
WACC using external equity (common stock):
WACC = (wd)(kd) + (we)(ke), where wd, weare the weights used for debt and common equity.
WACC = (0.50 * 7.0%) + (0.50 * 17.5%) =12.25%.
Note:To save time on the exam, remember that internal equity cost is less than external equity cost
(or ks< ke) due to floatation costs.

Question: 3856

All of the following comments about the capital budget post auditing process are correct EXCEPT:

A. After the initial capital budgeting decision is made, the company should follow up and compare
the actual results to the projected results.
B. The project managers should explain large variances (projection versus actual).
C. The function of the post audit includes improving forecasting and operations.
D. One of the purposes of the post-audit process is to limit risky projects.

Answer: D

Explanation:
The role of the post-audit process is to follow-up on capital budgeting decisions and to track results.
The process is not meant to limit the capital projects; the company will still want to accept projects
based on the decision rules for net present value (NPV), internal rate of return (IRR), and other
valuation methods (using internal and external benchmarks).

Question: 3857

Erwin DeLavall, the Plant Manager of Patch Grove Cabinets, is trying to decide whether or not to
replace the old manual lathe machine with a new computerized lathe. He thinks the new machine
will add value, but is not sure how to quantify his opinion. He asks his colleague, Terri Wharten, for
advice. Wharten‘s son just happens to be a Level 2 CFA candidate. DeLavall and Wharten provide the
following information to Wharten’s son:
Company Assumptions:
New Machine Assumptions:
Which of the following choices is most correct? Patch Grove Cabinets should:

A. not replace the old lathe with the new lathe because the new one will decrease the firm's value by
$5,370.
B. replace the old lathe with the new lathe because the new one will add $10,316 to the firm's value.
C. not replace the old lathe with the new lathe because the new one will decrease the firm's value by
$3,132.
D. replace the old lathe with the new lathe because the new one will add $3,760 to the firm's value.

Answer: D

Page | 1512
Explanation:
The valuation method that shows the project’s impact on the value of the firm is net present value
(NPV). To calculate NPV, we need to determine the initial investment outlay, the operating cash
flows, and the terminal year cash flows. Then, we discount the cash flows at the WACC. The
calculations are as follows:
Step 1: Initial Investment Outlay:
= cost of new machine + proceeds/loss from old machine + change in net working capital (NWC)
= -$90,000 + $30,000 - $6,800 + $5,000 =-$61,800(cash outflow)
Details of calculation:
oSales price = $30,000inflow
oTax/tax credit: $6,800outflow
oNWC =current assets -current liabilities = 20,000 – 25,000 = -5,000 (a decrease in working
capital is a source of funds)
Step 2: Operating Cash Flows (years 1-4):Given as$16,800inflow
Step 3: Terminal Value:
=year 5 cash flow + return/use of NWC + proceeds/loss from disposal of new machine + tax/tax credit
=$16,800 - $5,000 + $15,000 + $1,920 =$28,720inflow
Details of calculation:
oSales price = $15,000inflow
oTax/tax credit: $1,920inflow
Step 4:Calculate NPV:
NPV = -$61,800 + ($16,800 / 1.131) + ($16,800 / 1.132) +($16,800 / 1.133) +($16,800 / 1.134)
+($28,720 / 1.135) =$3,759.
Since the NPV is positive, Patch Grove should replace the old lathe with the new one, because the
new lathe will increase the firm’s value by the amount of the NPV, or $3,759.
You may also solve this problem quickly by using the cash flow (CF) key on your calculator.

Page | 1513
Question: 3858

The management of Strings & All, Inc., a small, highly leveraged, electric guitar manufacturer, wants
to reduce the company’s degree of total leverage (DTL) to 2.0. Currently, the company’s expected
operating performance is as follows:
To obtain a DTL of 2.0, management must (all else constant):

A. increase variable expenses by 30%.


B. reduce variable expenses by 38.5%.
C. reduce variable expenses by 30%.
D. increase variable expenses by 38.5%.

Answer: C

Explanation:
To obtain this result, we need to calculate the current variable costs, determine the variable costs
that will result in a DTL ratio of 2.00, and calculate the percentage change.
Step 1: Calculate current variable costs (VC): VC = 0.6 * 500,000 = 300,000
Step 2: Calculate Variable costs needed to decrease the DTL to 2.0:
Rearranging the formula for DTL:
= (Sales – Variable Costs) / (Sales – Variable Costs – Fixed Costs – Interest Expense)
results in: Variable Costs (VC) = Sales – (2 * Fixed Costs) – (2 * Interest Expense)
= 500,000 – (2*120,000) – (2*25,000) = 210,000
Step 3: Calculate percentage change:
VC = (300,000 – 210,000) / 300,000 = 0.30, or 30%.

Question: 3859

Which of the following statements about dividend policy and capital structure is FALSE?

A. A company's growth rate equals the retention ratio multiplied by return on equity.
B. Companies should use the residual dividend model to set the long-run target dividend payout
ratio, but should not use it to set the dividend payment in any one year.
C. If the board of directors decreases the target payout ratio, the stock price may increase or
decrease.

Page | 1514
D. Assuming a world of taxes and bankruptcy, there is an optimal capital structure that maximizes
earnings per share (EPS) and minimizes the cost of debt.

Answer: D

Explanation:
The optimal capital structure is the one that maximizes the stock price and minimizes the cost of
capital.
The other statements are true. “A company’s growth rate equals the retention ratio multiplied by
return on equity,” is simply the word version of the equation g = (1 - Payout Ratio) * ROE. Know this
equation for the exam! A company that uses the residual dividend model to set the dividend
payment in any one year will not have a stable payout ratio, and investors will likely demand a higher
required return to offset this uncertainty. The higher required return will result in a lower stock price.
If the board of directors decreases the target payout ratio, the stock price may increase or decrease
because there will be two opposing effects. For example, this may result in a decrease in stock price
(due to lower dividend payout, D1), and an increase in the stock price due to increased funds
available for reinvestment. Remember: g = (1 – Payout) * ROE. The net effect partially depends on
investor’s preference for dividends over capital gains or vice versa.

Question: 3860

Using the following assumptions, calculate the rate of return on a margin transaction for an investor
who purchases the stock and the stock price at which the investor who shorts the stock will receive a
margin call.
What of the following choices is closest to the correct answer? The margin transaction return is:

A. -12.00%, and the investor will receive a margin call at a stock price of $16.67.
B. 24.00%, and the investor will receive a margin call at a stock price of $30.00.
C. 48.00%, and the investor will receive a margin call at a stock price of $20.83.
D. -24.00%, and the investor will receive a margin call at a stock price of $30.00.

Answer: D

Explanation:
To obtain the result:
Part 1: Calculate Margin Return:
Margin Return % = [((Ending Value - Loan Payoff) / Beginning Equity Position) – 1] * 100
= [(([$22 * 1,000] – [$25 * 1,000 * 0.50]) / ($25 * 0.50 * 1,000)) – 1] * 100
=-24.00%.
Alternative (Check):Calculate the all cash return and multiply by the margin leverage factor.
= [(22,000 – 25,000)/22,000] * [1 / 0.50] = -12.00% * 2.0 = -24.00%
Part 2:Calculate Margin Call Price:
Since the investor isshort(sold the stock), the formula for the margin call price is:
Margin Call = (original price) * (1 + initial margin) / (1 + maintenance margin)
= $25 * (1 + 0.50) / (1 + 0.25) =$30.00

Question: 3861

Which of the following statements about securities markets is FALSE?

A. Characteristics of a well-functioning securities market include: many buyers and sellers willing to
trade at below market price, low bid-ask spreads, timely information on price and volume of past
transactions, and accurate information on supply and demand.
B. Secondary markets, such as the over-the-counter (OTC) market, provide liquidity and price

Page | 1515
continuity.
C. A limit buy order and a stop buy order are both placed below the current market price.
D. When Conglomerate, Inc. trades directly with MultiNational, Ltd., it is using the fourth market.

Answer: C

Explanation:
A limit buy is placed below the current market price, but a stop buy order is placed above the current
market price (stop buy orders are often placed to protect a short sale from a rising market).
The other choices are true. A well-functioning securities market includes the following
characteristics:
The fourth market is the direct exchange of securities (no intermediaries and no transaction costs).

Question: 3862

The table below lists information on price per share and shares outstanding for three stocks –
Rocking, Payton, and Strand.

Using the information in the table, determine which of the following statements is FALSE?

A. The geometric return is less than 11.7%.


B. If the three stocks comprise an index, a change in Stock Payton would have the biggest impact if
the index was market-value weighted.
C. An investor creating a price-weighted index of these three stocks would need to change his
holdings at year-end to reflect the price changes.
D. If the three stocks comprise an index, a change in Stock Strand would have the biggest impact if
the index was price-weighted.

Answer: C

Explanation:
Aprice-weighted indexassumes that the investor holds an equal number of shares of each stock in
the index. Since the number of stocks did not change, the investor wouldnotneed to change his
holdings.
The other statements are true. A price-weighted index is most influenced by the stock with the
highest per-share price (Strand). A market-value weighted index is most influenced by the stock with
the largest market capitalization (Payton). The geometric return is always less than the arithmetic
return (see calculation).
The arithmetic and geometric return are calculated as follows:
Arithmetic return = sum of: (1 + stock return) divided by the number of stocks minus 1, or
[(1.5 + 1.0 + 0.85) / 3] – 1 = 0.117, or 11.7%. Thus, the geometric average must be less than 11.7%.
Check of geometric average: = product of (1+ stock return), all to the 1/n power (or nth root) minus
1.

Page | 1516
= [(1.5)*(1.0)*(0.85)]1/3- 1 = 0.084, or 8.4%

Question: 3863

Which of the following statements about the implications of tests for the efficient market hypothesis
(EMH) is FALSE?

A. By purchasing an index fund, an investor can match the market return and minimize costs.
B. Technical trading rules do not consistently provide excess returns after adjusting for trading costs
and taxes.
C. Other than corporate insiders and market specialists, most traders have monopolistic access to
information, which rejects the strong-form EMH.
D. The best way to measure the performance of investments professionals is against a randomly
selected buy-hold strategy of stocks (assuming the same risk level).

Answer: C

Explanation:
Other than corporate insiders and market specialists, no other group has monopolistic access to
information, which supports the strong-form EMH. The other statements are true.

Question: 3864

Kaylee Sumners, Level 1 CFA candidate, has just finished reviewing flash cards for the reading on the
efficient market hypothesis (EMH). Confused by the different tests for the different forms of the
EMH, she outlines the information (of which four summary points appear below) from memory. It
appears that Sumners should review the material because three of the points are incorrect. Which of
her summary points is CORRECT?

A. Early tests of the semi-strong form used the equation: ReturnAbnormal = ReturnActual - (RMarket
* BetaStock).
B. The superior historical performance of exchange specialists and corporate insiders rejects the
semi-strong form of the EMH.
C. Cross-sectional tests such as the price-earnings ratio, neglected firms tests, and book value to
market value tests support the semi-strong form of the EMH.
D. Statistical and trading rule tests support the weak-form of the EMH.

Answer: D

Explanation:
These two tests support the weak-form EMH contention that security prices reflect all historical
market information and that mechanical trading rules do not result in superior returns.
The other statements are false.Latertests of the semi-strong form EMH used the equation:
ReturnAbnormal= ReturnActual– (RMarket* BetaStock). Early tests omitted beta, using the formula:
ReturnAbnormal= ReturnActual– RMarket. The superior historical performance of exchange
specialists and corporate insiders rejects thestrongform of the EMH. Cross-sectional tests such as the
price-earnings ratio, neglected firms tests, and book value to market value testsrejectthe semi-strong
form of the EMH. These tests show that certain stocks have high realized returns (for example, low
P/E stocks and high book value to market value stocks).

Question: 3865

Marc Juneau, equity analyst, has just been assigned the task of valuing Avalon Games, Inc. The
company is expected to grow at 30 percent for the next two years. Beginning in the year 4, the

Page | 1517
growth rate is expected to reach seven percent and stabilize. The required return for this type of
company in the non-electronic games sector is estimated at 13 percent. The dividend in year 1 is
estimated at $3.00. Which of the following is closest to the value Juneau should calculate for the
stock of Avalon Games?

A. $71.88.
B. $64.68.
C. $73.01.
D. $45.41.

Answer: A

Explanation:
The high “supernormal” growth in the first three years and the decrease in growth thereafter signals
that we should use a combination of the multi-period and finite dividend growth models (DDM) to
value the stock of Avalon Games.
Step 1:Determine the Dividend stream through year 4
Step 2:Calculate the value of the stock at the end of year 3 (using D4)
Step 3:Calculate the PV of each cash flow stream at ke= 13%, and sum the cash flows.Note:We
suggest you clear the financial calculator memory registers before calculating the value.

Note: 1Future values are entered in a financial calculator as negatives to ensure that the PV result is
positive. It does not mean that the cash flows are negative.
Also, your calculations may differ slightly due to rounding. Remember that the question asks you to
select the closest answer.

Question: 3866

Following is a graph of the Industry Life Cycle with the names of the phases omitted.

Page | 1518
Using the graph above, which of the following choices is INCORRECT?

A. The return on equity (ROE) on new projects is likely greater than ke for firms in Phase B.
B. The infinite period dividend discount model (DDM) works well for valuing firms in Phases C and D.
C. In general, profit margins are lower in Phase A than in Phase B.
D. For most companies, Phase C lasts the longest.

Answer: D

Explanation:
For most companies, Phase D, the Stabilization and Market Maturity Phase, lasts the longest. Phase C
is the Mature Growth Phase.
The other statements are true. During Phase B, the Rapid Accelerating Growth Phase, it is likely that
the firm is earning a higher return on new projects than the required rate of return. During this
phase, investors likely prefer for the firm to reinvest rather than pay dividends. The infinite period
DDM works well for valuing firms in Phase C, the Mature Growth Phase, and Phase D, the
Stabilization and Market Maturity Phase. Remember that the infinite period DDM is most useful for
a company with the following assumptions:
Phase A, the Pioneering Phase, is the start-up phase. Here, the market is small and firms incur major
development costs. Sales growth is low and profit margins may be negative. In Phase B, the Rapid
Accelerating Growth Phase, markets develop and demand grows exponentially. Competition is low
and sales growth and profit margins are very high

Question: 3867

An analyst just received the following information for Mythical Interactions, Inc. A senior equity
trader in her group wants to know if he should purchase a large block of the stock.
Based on the assumptions above, which of the following recommendations is CORRECT? The analyst
should advise the trader to:

A. not purchase the stock. It is overvalued by approximately $10.00.


B. purchase the stock. It is undervalued by approximately $8.00.
C. purchase the stock. It is undervalued by approximately $14.20.
D. not purchase the stock. It is overvalued by approximately $14.20.

Answer: B

Explanation:

Page | 1519
To determine whether the trader should purchase the stock, we need to determine if the stock is
overvalued or undervalued. Given the information in this problem, we will use the price/earnings
(P/E) ratio and the earnings per share (EPS) to calculate an estimated value.
The P/E ratio = Dividend Payout Ratio / (ke– g),
EPS = [(Per share Sales Estimate) * (EBITDA%) – D (per share) – I (per share)] * (1 - t)
= [($175 * 0.22) - $20 - $12] * (1 – 0.40) = $3.90
Value of stock = EPS * P/E = 13.725 * $3.90 = approximately$53.50
Conclusion:The trader should purchase a block of the stock. It is undervalued by the difference
between the market price and the estimated value, $53.50 - $45.50, or approximately $8.00.

Question: 3868

Which of the following statements about asset valuation is FALSE?

A. The price to book value ratio can be used to value firms with negative cash flows.
B. Economic value added (EVA®) measures the economic profit generated per dollar of invested
capital.
C. When estimating the profit margin of a company, the higher the export/import level, the higher
the competition.
D. The purpose of top-down stock analysis is to determine the best company within the best
performing industry.

Answer: B

Explanation:
EVA measures management’s ability to add value to the firm. One of the problems with the EVA
calculation is that it is not measured relative to the capital invested in the firm. As a workaround, the
ratio of EVA to capital is used, and it is this ratio that measures the economic profit generated per
dollar of invested capital.
The other statements are true. Use Porter’s five forces to determine the level of competition within
an industry and to determine the company’s strategy for dealing with that competition. Do not forget
about the impact of foreign competitors!

Question: 3869

Myra Addison, Luz Bazo, and Erik Jenss, three equity traders, are having a quick lunch around the
corner from the exchange. Bazo’s cell phone beeps, letting him know that he has a text message. He
reads the message, then quietly tells Addison and Jenss that Badger Distributors, Inc. has just won
exclusive rights to supply all major league baseball parks with uniforms for hot dog/soda vendors.
Bazo stands up, gathers his unfinished lunch, and announces, “I’m going back to the exchange to
trade.” Jenss calmly eats his sandwich and says, “There’s plenty of time to trade.” Addison shakes
her head and mutters, “It’s too late already.” Based on their reactions to the news on Badger
Distributors, which statement best identifies the trading view of these three traders?

A. Addison uses fundamental analysis, Bazo is a technician, and Jenss supports the efficient market
hypothesis.
B. Addison and Jenss both use fundamental analysis and Bazo is a technician.
C. Addison and Jenss both use fundamental analysis and Bazo supports the efficient market
hypothesis.
D. Addison supports the efficient market hypothesis, Bazo uses fundamental analysis, and Jenss is a
technician.

Answer: D

Page | 1520
Explanation:
One major difference between technicians, fundamental analysis traders, and those who support the
efficient market hypothesis is the speed at which they believe the market reflects newinformation,
The order, from slowest to fastest, is: technical analysis (Jenss), fundamental analysis (Bazo), and
efficient market hypothesis (Addison).

Question: 3870

Which of the following statements about contrary-opinion and smart money technicians is
INCORRECT?

A. When margin balances in brokerages accounts increase, contrary-opinion technicians are bearish.
B. The investment advisory ratio is at 0.65. Contrary-opinion technicians are bullish.
C. The OTC volume is less than 87% of the NYSE volume. Investors are bearish.
D. A narrowing of the T-bill - Eurodollar futures spread is a signal for a smart-money technician to
buy.

Answer: A

Explanation:
Although an increase in margin (debit) balances in brokerages accounts means investors are bullish,
it is not anindicator used by contrary-opinion technicians. This would be a bullish sign to smart-
money technicians.
The other statements are correct. When the investment advisory ratio (bearish opinions/total
opinions) is equal to or greater than 0.60, it means that investors are bearish, and contrary-opinion
technicians are bullish. Investors are considered bullish if the OTC volume is greater than 112% of the
NYSE volume.
Summary of the indicators for contrary-opinion and smart money technicians:
Contrary-opinion technicians (trade the opposite of the mass of general investor):
Smart-money technicians (follow the professional investors):

Question: 3871

Which of the following statements about contrary-opinion and smart money technicians is
CORRECT?

A. A contrary-opinion technician is bearish when the specialist short sale ratio falls below 30%.
B. A smart-money technician buys when futures traders are bullish on stock index futures.
C. When investor credit balances are falling, contrary-opinion technicians are bearish.
D. A smart-money technician takes a bullish position when the yield spread on high quality versus
lower-quality bonds increases from 75 basis points to 150 basis points.

Answer: C

Explanation:
When investor credit balances are falling, investors are bullish, so contrary-opinion technicians are
bearish.
The other statements are incorrect. Although a specialist short sale ratio less than 30% indicates a
bull market, this indicator is followed bysmart money technicians. When the yield-differential
between high quality and lower-quality bonds widens, the confidence index decreases, and smart-
money technicians would bebearish. A widening yield spreadis a bearish signto smart-money
technicians because it means that the confidence index hasdecreased. When 75% or more of futures
traders are bullish on stock index futures,contrary-opinion techniciansbecome bearish and sell.
Summary of the indicators for contrary-opinion and smart money technicians:

Page | 1521
Contrary-opinion technicians (trade the opposite of the mass of general investors):
Smart-money technicians (follow the professional investors):

Question: 3872

Using the information below, value the stock of Meerkat Publishing, Inc. using the free cash flow
from equity (FCFE) valuation method.

Which of the following choices is closest to the correct answer? The per share value of Meerkat
Publishing is:

A. $31.52.
B. $45.39.
C. $32.86.
D. $24.06.

Answer: C

Explanation:
The FCFE is calculated as follows (all amounts in million of $ except per share price):
Step 1: Calculate Present Value of Each Year’s FCFE
Step 2: Calculate Present Value of final cash flow times FCFE multiple
Step 3: Calculate per share value
Add up PV of FCFE and end value and divide by number of shares outstanding = (10.67 + 11.85 +
15.45 + 339.93) / 11.5 =32.86

Question: 3873

Kelly Windsor and Joe Agosti, expatriates working in Yemen, are studying for the Level 1 CFA
examination. This week, they are focused on new concepts in the asset valuation material. While
sitting on his balcony overlooking the desert, Agosti creates the following true/false question to test
Windsor’s knowledge of dollar-weighted and time-weighted returns. Which of the following
statements did he make FALSE?

A. If a client adds funds to an investment during an unfavorable market, the time-weighted return
will be lower than actual.
B. The dollar-weighted return applies the concept of internal rate of return (IRR) to investment
portfolios.
C. The time-weighted return measures the compound rate of growth of $1 over a stated time period.
D. If the investment period is greater than one year, an analyst must use the geometric mean
approach when using the time-weighted return method.

Answer: A

Explanation:
The time-weighted method is preferred because it is not affected by the timing of cash flows. This
statement should read, "If a client adds funds to an investment during an unfavorable market, the
dollar-weighted return will be lower than actual.” The other statements are true.

Question: 3874

Page | 1522
Which bond has the greatest price volatility? A

A. 10% coupon bond with a 20-year life.


B. 5% coupon bond with a 20-year life.
C. 5% coupon bond with a 10-year life.
D. 10% coupon bond with a 10-year life.

Answer: B

Explanation:
This question is asking: given a change in yield, which of the bonds will exhibit the greatest price
change? Of the four choices, the bond with the longest maturity and lowest coupon will have the
greatest price volatility.
All else equal, the bond with the longer term to maturity is more sensitive to changes in interest
rates. Cash flows that are further into the future are discounted more than near-term cash flows.
Here, this means that one of the 20-year bonds will have the highest volatility. Similar reasoning
applies to the coupon rate. A lower coupon bond delivers more of its total cash flow (the bond's par
value) at maturity than a higher coupon bond. All else equal, a bond with a lower coupon than
another will exhibit greater price volatility. Here, this means that of the 20-year bonds, the one with
the 5% coupon rate will exhibit greater price volatility than the bond with the 10% coupon.

Question: 3875

Simone Girau holds a callable bond and Chi Rigazio holds a putable bond. Which of the following
statements about the two investors is TRUE?

A. As the yield volatility increases, the value of both Girau's bond and the underlying option
increases.
B. Both investors calculate the value of the bond held by adding the value of the option to the value
of a similar straight bond.
C. Girau's bond has less potential for price appreciation.
D. If yield volatility increases, the value of Rigazio's option will decrease.

Answer: C

Explanation:
When a bond has a call provision, the potential for price appreciation is reduced, because the call
caps the price of the bond near the call price. Even if interest rates fall considerably, it is unlikely that
investors would pay a price that exceeds the call price.
The other statements are false. To calculate the value of a putable bond, it is correct to add the
option value to the value of a similar straight bond. However, to calculate the callable bond value,
subtract the option value from that of a similar straight bond. As a result, when yield volatility
increases (thus increasing the option value), the value of a callable bond decreases and the value of a
putable bond increases. A call option does benefit the issuer, but a put option benefits the holder.
Embedded options (puts and calls) increase in value when volatility increases.

Question: 3876

Which of the following statements about refunding and redemption is TRUE?

A. A sinking fund is an example of refunding.


B. An investor concerned about premature redemption is indifferent between a noncallable bond
and a nonrefundable bond.
C. Callable bonds redeemed at a special redemption price are redeemed at par.

Page | 1523
D. A nonrefundable bond can be redeemed with funds from operations, from a new equity issue, or
from a lower coupon issue.

Answer: C

Explanation:
This statement is true. Bonds redeemed at par are redeemed at the special redemption price. When
bonds are called at a premium, the issuer redeems them at a price above par, or the regular or
general redemption price.
The other statements are false. A sinking fund is a type of redemption, which refers to the retirement
of bonds. An investor concerned about premature redemption would prefer a noncallable bond
because a noncallable bond cannot be called for any reason. A nonrefundable bond can be called for
any reason other than refunding. The term refunding specifically means redeeming a bond with
funds raised from a new bond issued at a lower coupon rate. A nonrefundable bond can be
redeemed with funds from operations or a new equity issue. Note: A bond can be both
nonrefundable and callable.

Question: 3877

Tony Ly is a Treasury Manager with Deeter Holdings, a large consumer products holding company.
The Assistant Treasurer has asked Ly to calculate the current yield (CY) and the Yield-to-first Call (YTC)
on a bond the company holds that has the following characteristics:
If Ly calculates correctly, the CY and YTC are approximately:

A. 7.80% and 15.82%, respectively.


B. 7.78% and 15.72%, respectively.
C. 3.89% and 15.72%, respectively.
D. 7.78% and 15.82%, respectively.

Answer: A

Explanation:
To calculate the CY and YTC, we first need to calculate the present value of the bond: FV =1,000, N
=14= 7*2, PMT =35=(1000*0.07)/2, I/Y =4.5(9 / 2), Compute PV = -897.77 (negative sign because we
entered the FV and payment as positive numbers).
Then, CY = (Face value * Coupon) / PV of bond = (1,000 * 0.07) / 897.77 =7.80%
And finally, YTC calculation: FV =1,060(price at first call), N =4(2*2), PMT =35(same as above), PV = -
897.77 (negative sign because we entered the FV and payment as positive numbers), ComputeI/Y =
7.91 (semi-annual rate, need to multiply by 2) =15.82%

Question: 3878

Given the following spot and forward rates, how much should an investor pay for a 3-year, annual
zero-coupon bond with a face value of $1,000?
The investor should pay approximately:

A. $720.
B. $724.
C. $884.
D. $886.

Answer: B

Explanation:

Page | 1524
The yield to maturity on an N-year zero coupon bond is equivalent to the N-yearspot rate.Thus, to
determine the present value of the zero-coupon bond, we need to calculate the 3-year spot rate.
Using the formula: (1 + Z3)3= (1 +1f0) + (1 +1f1) + (1 +1f2)
Where Z = spot rate andnfm= The n year rate m periods from today, (1f0= the 1 year spot rate now)
(1 + Z3)3= (1.035) * (1.115) * (1.1975)
Z3= 1.38191/3- 1 = 0.11386, or 11.39%
Then, the value of the zero coupon bond = 1,000 / (1.1139)3= 723.62, or approximately$724.
or, using a financial calculator, N = 3, I/Y = 11.39, FV = 1,000, PMT = 0, Compute PV = 723.54, or
approximately$724.

Question: 3879

Joel Franklin, CFA, has just been promoted to junior portfolio manager for a large equity portfolio at
Davidson-Sherman (DS), a large multinational investment banking firm. He is specifically responsible
for the development of a new investment strategy that DS wants all equity portfolios to implement.
DS has decided to begin overlaying option strategies on all equity portfolios. The reason for this
decision is the relatively poor performance of many of their equity portfolios. They look at the option
strategies as an opportunity to add value or reduce risk. Franklin recognizes that the behavior of an
options value is dependent on many variables and decides to spend information shown in Exhibit's 1
and 2 for European style options.

Joel recognizes that his software only includes the valuation information for European style options.
He wants to know how the premium of an American style option compares with its European
counterpart. Which of the following is TRUE? The premium of the American option:

A. the same or lower.


B. strictly higher.
C. the same or higher.
D. strictly lower.

Answer: C

Page | 1525
Explanation:
The owner of a European option may exercise it only at expiration whereas an American option can
be exercised at any time before or at expiration. Therefore, an American option cannot be worth less
than a European option.

Question: 3880

Kep Polznik is a financial consultant to an entrepreneur who owns a chain of check-cashing


establishments (individuals who meet certain criteria can cash post-dated checks and then repay the
money at a later date). The company’s Treasurer is considering using plain vanilla swaps. Which of
the following statements should Polznik NOT use to sell the Treasurer on the use of interest rate
swaps?

A. The Treasurer can use swaps to exploit market inefficiencies.


B. The swap market's operational efficiency results in reduced transaction costs.
C. Swap agreements allow companies a more flexible way to package cash flows.
D. By using swaps, the Treasurer can maintain the confidentiality of business strategy.

Answer: A

Explanation:
Exploiting market inefficiencies is no longer considered a motivation for entering into swap
agreements. Historically, there were two basic motivations for swaps, to exploit market inefficiencies
and to attempt to obtain cheaper financing. Both were based on the belief that financial markets are
inefficient. Today, the swap markets have matured and there are few arbitrage opportunities. The
swap markets are considered operationally efficient and flexible. Thus, the main reasons to enter
into swap agreements include: to reduce transaction costs, to avoid costly regulations, and to
maintain privacy.

Question: 3881

Which of the following statements about agency theory is TRUE?

A. A company that pays fixed salaries with no variable compensation schemes likely has little agency
conflict.
B. An agency relationship is created when the board of directors appoints a new Chief Financial
Officer.
C. Restrictive debt covenants reduce the conflict between stockholders and managers.
D. Encouraging managers to take on high-risk projects aligns their goals with that of bondholders.

Answer: B

Explanation:
An agency relationship is created when decision-making authority is delegated to an agent without
the agent being 100% responsible for the consequences of her decisions. Here, unless the Chief
Financial Officer owns 100% of the firm, she has an agency relationship with the company.
Stockholders and managers both benefit from risky projects that increase profitability and firm value.
However, bondholders bear the risk of bankruptcy. Thus, encouraging managers to take on high-risk
projects does not help to align their goals with that of creditors. A company that pays fixed salaries
with no variable compensation schemes is not set up to reduce agency conflict. Without managerial
compensation plans that motivate managers to act to increase stock price, stockholders and
managers may have disparate goals.

Question: 3882

Page | 1526
Calculate the weighted average cost of capital (WACC) for a company with the following capital
component information:
The firm can issue new common stock with a price of $40.00, floatation costs of 3.0%, and a dividend
in year 0 of D0= $3.00.
Which of the following is closest to the correct answer? The WACC equals:

A. 9.82%.
B. 9.49%.
C. 9.05%.
D. 10.05%.

Answer: B

Explanation:
Step 1: Determine the after-tax cost of debt:
Theafter-tax cost of debt[kd(1 – t)]is used to compute the weighted average cost of capital. It is the
interest rate on new debt (kd) less the tax savings due to the deductibility of interest (kdt).
Here, we are given the inputs needed to calculate kd: n = 15*2 = 30, PMT = (1000*0.07)/2 = 35, FV =
1000, PV = -1047.46, compute I = 3.25, multiply by 2 = 6.50%.
Thus, kd(1 – t) = 6.50% * (1 – 0.35) =4.22%
Step 2: Determine the cost of preferred stock:
Preferred stock is a perpetuity that pays a fixed dividend (Dps) forever. Thecost of preferred stock
(kps)= Dps/ Pnet
where
Dps= preferred dividends.

Pnet = net issuingprice after deducting flotation costs.


Here, Dps= 0.08 * $35.00 = $2.80, so kps= Dps/ Pnet =$2.80 / $33.60 = 0.0833, or8.33%.
Step 3: Determine the cost of retained earnings:Given as 10.5%.
Step 4: Determine the cost of common equity:
Thecost of new common equityis given by:
ke= [D1/ (P0(1 – F))] + g
where
F = the percentage flotation cost incurred in selling new stock, or

(current stock price – funds going to company) / current stock price

D1= Dividend in next year

P0= Current stock price

g = Dividend growth rate


Here, D1= D0* (1 + g) = $3.00 * ( 1 + 0.06) = $3.18. F is given.
ke= [ 3.18 / (40 * (1 – 0.03))] + 0.06 = 0.1420 or 14.20%.
Step 5: Calculate WACC:
WACC = (wd)(kd) + (wps)(kps) + (ws)(ks)+ (we)(ke)
where wd, wps, wsandweare the weights used for debt, preferred stock, retained earnings, and
common equity.
Here, WACC = (0.30 * 4.22%) + (0.20 * 8.33%) + (0.15 * 10.5%) + (0.35 * 14.2%) =9.49%.
Note:Your calculation may differ slightly, depending on whether you carry all calculations in your
calculator, or round to two decimals and then calculate.

Question: 3883

Page | 1527
Which of the following statements about the cost of capital is TRUE?

A. New common equity used to finance projects is usually dilutive.


B. The component cost of retained earnings equals the required rate of return on new stock.
C. Using the marginal cost of capital (MCC) is superior to the weighted average cost of capital (WACC)
because the MCC assumes different risks across projects.
D. A firm can shift its retained earnings breakpoint by changing the dividend policy.

Answer: D

Explanation:
The firm can shift the break point of the MCC schedule by changing its payout ratio through the
dividend policy. Remember that the dividend payout ratio is in the numerator of the calculation that
dictates the breakpoint between lower cost internally generated equity (retained earnings) and
higher cost externally generated equity (common stock).
The other choices are false. New common equity is not by definition dilutive. However, if the firm
does not earn the cost of new common equity on the portion of the investment that is financed with
the new issue, the firm’s earnings per share will fall. The MCC is the cost of the last dollar raised by
the company and assumes constant risk across projects. The component cost of retained earnings
equals the required rate of return on existing stock.

Question: 3884

Norine Benson is studying for the Level 1 CFA examination and is having difficulty with the broader
concepts of capital budgeting. Her study partner, Henri Manz, tests her understanding by asking her
to identify which of the following statements is TRUE?

A. For mutually exclusive projects, the decision rule is to pick the project that has the highest net
present value (NPV).
B. If the change in current liabilities is greater than the change in current assets, it means that
additional financing was needed and there is a cash outflow.
C. An analyst can ignore inflation since price level expectations are built into the weighted average
cost of capital (WACC).
D. Replacement decisions involve mutually exclusive projects.

Answer: D

Explanation:
Because replacement decisions involve either keeping the old asset or replacing the old asset, the
projects are mutually exclusive.
The decision rule for NPV is to pick the project with the highest positive NPV. Only projects with
positive NPV add to the company’s value. If a neither project has a positive NPV, neither project
should be chosen. The statement about net working capital (NWC) is stated in the reverse of how we
usually think of it:in NWC =Current Assets –Current Liabilities. Here, the change in current liabilities
exceeds the change in current assets and the result is negative, meaning the project frees up cash,
creating a cash inflow. Because the WACC is adjusted for inflation, the analyst must adjust project
cash flows upward to reflect inflation. If the cash flows are not adjusted for inflation, the NPV will be
biased downward. (Reverse the preceding logic for deflation.)

Question: 3885

KGraphix, a small, privately owned publishing company, plans to upgrade its printing process by
purchasing either a high-speed color laser printer or a webpress (a high speed color printing

Page | 1528
machine). Incremental cash flow information for each piece of equipment is as follows:
Assuming that the company’s weighted average cost of capital (WACC) is 13 percent, which of the
following statements is most correct? KGraphix should purchase the:

A. webpress because its net present value (NPV) of $9,939 is greater than the color laser printer's
NPV of $7,223.
B. color laser printer because its Internal rate of return (IRR) is higher than that of the webpress.
C. color laser printer because it has the highest equivalent annual annuity.
D. webpress because it has the longest life and the highest net present value (NPV).

Answer: C

Explanation:
Here, we have two projects with unequal lives, so we have to adjust the cash flow. We can either
adjust the cash flows of the shorter-term project to match the life of the longer-term project
(adjusted NPV), or we can use the equivalent annual annuity (EAA) approach. The answer given is
most correct because the project with the highest EAA will have the highest adjusted NPV, and the
color laser printer has the highest EAA (and adjusted NPV). (See table below.)
The statement, “KGraphix should purchase the webpress because its NPV of $9,939 is greater than
the color laser printer’s NPV of $7,223,” calculates NPV only using the given lives (or unadjusted
NPV). Unadjusted NPV is not the best criterion to use for deciding between mutually exclusive
projects with unequal lives. Although the color laser printer has an IRR higher than that of the
webpress, IRR is also not the best criterion to use for mutually exclusive projects with unequal lives
Quantitatively: (all amounts are in $ unless indicated otherwise):

Notes:1Repeats color laser project beginning in year 3; year 3 cash flow equals the net initial
investment cost of $40,000 and $20,000 inflow from the last year of the “first” printer.
Following are examples of how to use financial calculator to solve this problem. (Note, the example is
for the unadjusted NPV, IRR, and EAA of the color laser printer. Use a similar methodology to
determine the NPV, IRR, and EVA for the other cash flows streams.

Page | 1529
Note:1Negative sign is for calculator functionality, not to signify a negative amount

Question: 3886

Which of the following statements about dividend policy and capital structure is TRUE?

A. A person who believes in the clientele effect and a proponent of the "bird-in-hand" theory would
have similar views on dividend payout policy.
B. Investors view a stock repurchase as a positive signal and a stock issue as a negative signal.
C. A diversified shareholder is most concerned with stand-alone risk.
D. Monte Carlo simulation is used to estimate market risks; scenario analysis measures stand-alone
risk.

Answer: B

Explanation:
Investors view a stock repurchase as a positive signal and a stock issue as a negative signal. A

Page | 1530
repurchase may mean that management believes the stock is undervalued. To understand why a
stock issue is viewed negatively, consider the following circumstances: A biotech company has a new
blockbuster drug that will increase its profitability, but to produce and market the drug, the company
needs to raise capital. If the company sells new stock, then as sales (and thus profits) occur, the price
of the stock will rise. The current shareholders will do well but not as well as they would have had
the company not sold more stock before the share price increased. Thus, it is assumed that
management will prefer to finance growth with non-stock sources.
The other statements are false. A person who believes in the clientele effect and a proponent of the
“bird-in-hand” theory wouldnothave similar views on dividend policy. The clientele effect suggests
that different groups of investors want different dividend levels (often based on tax status), and
through the law of supply and demand, investors will select companies that meet their needs. Thus,
dividend payout policy does not matter. According to the “bird-in-hand” theory, investors prefer
dividends to capital appreciation because they view the former (D1/ P0) as less risky than the latter
(g, or growth rate). Stand-alone risk is the company’s individual, or unique, risk. Anundiversified
shareholder is concerned with stand-alone risk. A diversified shareholder is most concerned about
undiversifiable, or systematic, risk. Both Monte Carlo simulation and scenario analysis are used to
estimate stand-alone risk.

Question: 3887

Given the following information about a manufacturing firm, determine the optimal capital
structure.
The optimal capital structure for this firm is:

A. 10% Debt, 90% Equity.


B. 25% Debt, 40% Equity.
C. 40% Debt, 60% Equity.
D. 50% Debt, 50% Equity.

Answer: C

Explanation:
After calculating the missing data points in the table, we can determine that the capital structure of
40% debt and 60% equity minimizes the WACC and maximizes the stock price (see calculations
below), and is thus considered optimal.
To calculate WACC for the missing data points, we will use the formula:
WACC = (wd)(kd) + (we)(ke), where wd, weare the weights used for debt and common equity.
To calculate the Stock Price for the missing data points, we will use the formula:P0= D1/ ke
Here, we need to calculate D1using the EPS and payout information; g is given as 0.0%.

Question: 3888

Using the following assumptions, calculate the stock price at which investors Helen Alba, who shorts
the stock on margin, and Kobin Lubis, who purchases the stock on margin, will receive a margin call.
Which of the following choices is closest to the correct answer? Alba will receive a margin call at a
stock price of:

A. $33.00 and Lubis will receive a margin call at a stock price of $37.66.
B. $33.00 and Lubis will receive a margin call at a stock price of $53.45.
C. $46.85 and Lubis will receive a margin call at a stock price of $33.00.
D. $37.66 and Lubis will receive a margin call at a stock price of $53.45.

Answer: C

Page | 1531
Explanation:
Calculations are as follows:
Since Alba isshort(sold the stock), the formula for the margin call price is:
Margin Call = (original price) * (1 + initial margin) / (1 + maintenance margin)
= $42 * (1 + 0.45) / (1 + 0.30) =$46.85
Since Lubis islong(purchased the stock), the formula for the margin call price is:
Margin Call = (original price) * (1 – initial margin) / (1 – maintenance margin)
= $42 * (1 – 0.45) / (1 – 0.30) =$33.00

Question: 3889

Laleh Mali conducts a stock transaction with the following characteristics:


Which of the following statements about Mali’s trade is least likely to be correct? Mali placed her
order in:

A. a continuous market.
B. an order driven market.
C. conjunction with a short sale.
D. the secondary market.

Answer: D

Explanation:
Seasoned issues are new issues by a firm whose shares are already traded on the exchange and are
traded in the primary market. The other statements are likely true. Note that stop buy orders are
often placed to protect a short sale from a rising market.

Question: 3890

Which of the following statements about short selling is FALSE?

A. A short sale involves securities the investor does not own.


B. According to the uptick rule, a short sale can only trade at a price higher than previous trade.
C. A short seller loses if the price of the stock sold short falls.
D. A short seller is required to set up a margin account.

Answer: C

Explanation:
A short seller loses if stock prices rise. The other choices are true.

Question: 3891

Tamber Benz, CFA, recently joined Bay Area Investment Group as a personal financial planner. Today,
she has a meeting with a client interested in equity index funds, with a particular interest in learning
about the source and direction of biases. In preparation for this meeting, she makes some quick
notes (relying on her memory). These notes are listed below. She then finds her well-worn CFA study
notes and checks her memory. After reviewing her notes, which of the following choices does she
determine is INCORRECT?

A. The Dow Jones Industrial Index has a built-in downward bias.


B. An index such as the Valueline Composite Average is constructed by purchasing an equal number
of shares of each stock in the index, and will have a downward bias when geometric averaging is
used to compute the return.

Page | 1532
C. One problem with an index such as the S&P 500 is that firms with greater market capitalization
have more impact than other firms.
D. A market value-weighted index, such as the New York Stock Exchange Index, accurately reflects
the impact of price changes on wealth.

Answer: B

Explanation:
Although the latter part of this statement is correct, the first part is incorrect. The Valueline
Composite Average is an unweighted price indicator series, and is constructed by maintaining an
equal dollar investment in each stock in the index. The return of an unweighted index is usually
calculated using a geometric average. Assuming the existence of volatility, the geometric average
will always be lower than the arithmetic average.
The other statements are true. The Dow Jones Industrial Index is a price-weighted index and thus has
a built-in downward bias because of the impact of stock splits. After a stock split, the denominator is
adjusted downward to keep the index at the same level as before the split. Since high-growth
companies tend to announce stock splits more frequently than low-growth companies, the larger,
more successful firms lose influence on the index. The S&P 500 index is a market-value weighted
index. One problem with market-value weighted indexes is that firms with greater market
capitalization have more impact than other firms. If these firms also have higher returns, the firms
can dominate the index.

Question: 3892

Indie Carson, management consultant, wants to become a portfolio manager. While researching the
position, she learns that obtaining the CFA Charter is very important. She decides to take the Level 1
examination this June, and begins to study. During the reading on efficient markets, she rethinks her
new career choice. If markets are efficient, what is the role of a portfolio manager? Distraught, she e-
mails her mentor, LaMeda Durio. Durio wants to use the occasion to help Carson study, so she e-
mailsCarson the following reply (summarized in points A through D below) and asks her to identify
the INCORRECT statement.
Which of the following choices does Carson select as FALSE? Assuming an efficient market, portfolio
managers assist clients with:

A. minimizing transaction costs.


B. quantifying risk tolerances and return needs.
C. rebalancing the portfolio when necessary.
D. diversifying globally to reduce systematic risk.

Answer: D

Explanation:
This a “trick” question. Although portfolio managers can help clients diversify globally, they do so to
reduce unsystematic risk. Systematic, or market risk, is undiversifiable.
The other statements are true. There are three ways for a portfolio manager to minimize transaction
costs: reduce taxes, reduce trading volume (turnover) and minimize liquidity costs by trading
relatively liquid stocks.

Question: 3893

Kaylee Sumners, Level 1 CFA candidate, is having difficulty remembering the tests for the three forms
of the efficient market hypothesis (EMH). On her first attempt to outline the information from
memory, she made numerous mistakes. After reviewing the material, she tries again to summarize
the information. This time, three of her four points are correct. Which of her summary points is

Page | 1533
INCORRECT?

A. Results of trading rule tests, such as filter rules, support the semi-strong form of the EMH.
B. The historical performance of professional money managers supports the strong-form of the EMH.
C. Early tests of the semi-strong form used the formula:ReturnAbnormal= ReturnActual- RMarket
D. The tests for the semi-strong form EMH give mixed results. Time-series tests such as dividend yield
and default spread reject the semi-strong form EMH and event studies on stock splits and
announcements of accounting changes support it.

Answer: A

Explanation:
Results of trading rule tests, such as filter rules, support the weak form of the EMH.
The other choices are true. Tests show that professional money managers perform no better than a
random buy and hold strategy. This supports the strong form EMH contention that stock prices
reflectall information- public and private. (Aside from corporate insiders and specialists, no group has
monopolistic access to information that would result in superior returns.)

Question: 3894

Katrina Whittcomb, junior analyst, is trying to understand what variables impact the price/earnings
(P/E) ratio for a stock. Specifically, she wants to determine under what circumstances the P/E ratio
will increase or decrease. A senior analyst in the group, Clinton Dermont, devises the following
question to help her understand the impact of changes in P/E variables. To make the question less
theoretical, he provides the following assumptions:
Using the information above, determine which of the following statements is most likely FALSE. All
else equal, if the:

A. dividend payout increases, the P/E ratio will increase.


B. expected inflation rate decreases, the P/E ratio will rise above 12.5.
C. earnings retention ratio increases, the P/E ratio will increase.
D. risk free-rate increases, the P/E ratio will decrease.

Answer: A

Explanation:
Memo:The P/E ratio = Dividend Payout Ratio / (ke– g),
Impact of variables:

Question: 3895

Calculate the earnings per share (EPS) for the plastic bead industry using the information below.
The industry’s EPS is closest to:

A. $110.
B. $22.
C. $54.
D. $40.

Answer: D

Explanation:
The formula for the industry EPS is:
EPSindustry= [(Per share Sales Estimate) * (EBITDA%) – D – I] * (1 - t)

Page | 1534
= [($250 * 0.60) - $50 - $20] * (1 – 0.5) = $40.00

Question: 3896

Amie Minami recently graduated from the University of Rivendell School of Business and is now
studying for the Level 1 CFA examination. She thought she would never have to read about Porter’s
Five Forces again. However, while taking an online review quiz to help her focus her studies, she sees
a question on Porter. Luckily, she remembers all the points and correctly identifies that one of the
choices is incorrect. Which of the following choices is FALSE?

A. Porter details three strategies that are available to firms in a competitive environment: low-cost,
differentiation, and vertical integration.
B. Porter's five forces are the first step in identifying and evaluating a firm's specific competitive
strategy.
C. When the threat of substitution is highest, profit margins will be low, particularly for commodity-
like products.
D. Suppliers are more powerful if they are more concentrated than the firms in that industry.

Answer: A

Explanation:
While vertical integration may be a strategy, Porter identified two strategies that are available to
firms in a competitive environment: a low-cost strategy and a differentiation strategy. The other
statements are correct.

Question: 3897

Which of the following statements about asset valuation is TRUE?

A. The bottom-up stock picking approach is: first, stock analysis, second, industry analysis, and third,
economic analysis
B. The absolute value of economic value added (EVA®) is less important than the trend.
C. Earnings are considered the variable least likely to be manipulated.
D. A domestic steel firm and a foreign steel firm will have similar earnings per share (EPS) levels and
price/earnings (P/E) ratios.

Answer: B

Explanation:
Management’s performance over time is most important.
The other statements are false. The bottom-up stock picking approach consists of one step – pick the
stocks you believe are underpriced, regardless of the state of the economy or the industry. The
choices given here for picking stocks are the steps of the top-down approach in reverse order. Sales
are considered the variable least likely to be manipulated. Although the basic methodologies
forinternational stock valuation and domestic stock valuation (dividend discount model [DDM], EPS,
P/E) are the same, the analyst must be aware of and consider differences in accounting practices and
factor in the impact of exchange rate movements. It is also important to consider the absolute
performance of foreign firms and their relative performance against local firms.

Question: 3898

Which of the following statements about contrary-opinion and smart money technicians is
CORRECT?

Page | 1535
A. If mutual funds cash holdings are more than 13% of total fund assets, smart-money technicians
are bullish.
B. When the ratio of short sales by specialists to total NYSE short sales is at 0.20, smart-money
technicians are bearish.
C. The CBOE put call ratio is 0.75. Contrary-opinion technicians are bullish.
D. When the yield spread on high quality versus lower-quality bonds narrows, the confidence index
decreases and smart-money technicians become bullish.

Answer: C

Explanation:
When the CBOE put call ratio is equal to or greater than 0.50, this suggests that investors are bearish
and thus contrary-opinion technicians are bullish.
The other statements are incorrect. The mutual fund ratio is not an indicator for smart money
technicians. When the mutual fund ratio is equal to or greater than 0.13, it means that investors are
bearish andcontrary-opiniontechnicians are bullish. The statement beginning, “When the yield
spread on high quality versus lower-quality bonds narrows,” is partially true. A narrowing yield
spreadis a bullish signto smart-money technicians, but because it means that the confidence index
hasincreased.
Summary of the indicators for contrary-opinion and smart money technicians:
Contrary-opinion technicians (trade the opposite of the mass of general investors):
Smart-money technicians (follow the professional investors):

Question: 3899

Jay Crewson, equity analyst at a large investment bank, formerly worked with a group of contrary-
opinion technician traders who traded exclusively using contrary indicators. He was recently
transferred to support a group of smart-money technicians. Since he is still adjusting to the “new”
rules, he asks Richard Ruscoe, another analyst in the group, to review his work. Ruscoe reviews
Crewson’s latest recommendation list and points out that one of the statements is incorrect. Which
of the following is the INCORRECT statement? Buy:

A. debit balances in brokerage accounts increased.


B. the ratio of short sales by specialists to total NYSE short sales fell below 0.30.
C. investor credit balances in brokerage accounts increased.
D. the yield-differential between high quality and lower-quality bonds decreased to 90 basis points.

Answer: C

Explanation:
Increased investor credit balances in brokerage accounts (indicating a bearish trend) are a bullish sign
to contrary-opinion technicians. The other statements are true and are indicators used by smart-
money technicians. When the yield-differential between high quality and lower-quality bonds
narrows (or decreases), it indicates that the confidence index has increased and smart-money
technicians are bullish.

Question: 3900

Mikal Cosce uses technical analysis to determine his trading behavior. Cosce would be least likely to
agree with which of the following statements?

A. He supports the weak form of the efficient market hypothesis.


B. Stock prices move in trends, and these trends persist.
C. Technical analysis tells him when to buy.

Page | 1536
D. He does not have to rely on accounting information.

Answer: A

Explanation:
The weak form of the efficient market hypothesis (EMH) refutes technical trading. The tests for the
weak form of the EMH indicate that after incorporating trading costs, simple trading rules cannot
generate positive, consistent, abnormal returns. The other statements are true.

Question: 3901

Question: 101 -
29045
Assume an investor makes the following investments:
During year one, the stock paid a $5.00 per share dividend. In year two, the stock paid a $7.50 per
share dividend.
The time-weighted return is:

A. 51.4%.
B. 51.7%.
C. 23.2%.
D. 14.7%.

Answer: A

Explanation:
To calculate thetime-weightedreturn:
Step 1: Separate the time periods into holding periods and calculate the return over that period:
Holding period 1:P0= $50.00
D1= $5.00
P1= $75.00 (from information on second stock purchase)
HPR1 = (75 – 50 + 5) / 50 = 0.60, or 60%
Holding period 2:P1= $75.00
D2= $7.50
P2= $100.00
HPR2 = (100 – 75 + 7.50) / 75 = 0.433, or 43.3%.
Step 2: Use the geometric mean to calculate the return over both periods
Return = [(1 + HPR1) * (1 + HPR2)]1/2– 1 = [(1.60) * (1.433)]1/2– 1 = .5142, or51.4%.

Question: 3902

A bond has a yield of 10 percent and an effective duration of 7.5 years. If the market yield changes by
10 basis points, what is the change in the bond's price?

A. 0.375%.
B. 1.500%.
C. 2.000%.
D. 0.750%.

Answer: D

Explanation:
The formula for effective duration calculates the approximate change in price for a 100 basis point
change. Here, we are asked to provide the approximate percentage change in the bond's price for a

Page | 1537
10bp change. Ten basis points is 1/10th, or 0.10 of 100bp. Thus, the calculation is 0.10 * 7.50 =
0.750%.

Question: 3903

Scott Malooly recently paid $109.05 for a $1000 face value, semi-annual coupon bond with a quoted
price of 105 6/32. Assuming that transaction costs are zero, which of the following statements is
TRUE?

A. Malooly purchased the bond between coupon dates.


B. The price Malooly paid covers the amount of the next coupon payment not earned by the seller.
C. The bond was trading ex-coupon.
D. The price Malooly paid includes the discounted amount of accrued interest due to seller.

Answer: A

Explanation:
When a bond trades between two consecutive coupon dates, the seller is entitled to receive interest
earned from the previous coupon date until the date of the sale. The price paid includes accrued
interest and is referred to as the “dirty price.”
The other statements are false. The price Malooly paid covers the amount of the next coupon
payment not earned by the buyer. When a security trades ex-coupon, the buyer pays the clean price,
which is the quoted price without accrued interest. Accrued interest is not discounted when
calculating the dirty price of a bond.

Question: 3904

A 12-year, $1,000 face value zero-coupon bond is priced to yield a return of 7.00 percent on a semi-
annual basis. What is the price of the bond, and how much interest will the bond pay over its life,
respectively? (Select the choice that is closest to the correct answer.)

A. $562, $438.
B. $444, $556.
C. $840, $160.
D. $438, $562.

Answer: D

Explanation:
Using the equation:Pricezerocoupon= Face Value * [ 1 / ( 1 + i/n)n*2]
Here,Pricezerocoupon= 1000 * [ 1 / (1+ 0.070/2)12*2] = 1000 * 0.43796 = 437.95, or
approximately438.So, interest = Face Value – Price = 1000 – 438 =562.
Using thecalculator:N = (12*2) =24, I/Y = 7.00 / 2 =3.50, FV = 1000, PMT = 0. PV = -437.95, or
approximately $438

Question: 3905

Four years ago, at the advice of J.T. Lindseth, her financial planner, T.J. Ali purchased a $1,000 face,
5.70 percent, semi-annual coupon bond with four years to maturity priced to yield 8.50 percent for
$906.70. Now, the bond has matured, and Lindseth calls Ali and informs her that because he had
invested the coupons at an annual rate of 10.0 percent, her realized return was approximately:

A. 8.65%.
B. 8.50%.

Page | 1538
C. 10.00%.
D. 8.35%.

Answer: A

Explanation:
Time Saving Note:You can answer this question without doing any calculations! Because the coupon
reinvestment rate was greater than the original YTM, we would expect the YTM to increase. This
narrows down the choices to 8.65% and 10.00%. The rate of 10.00% is an unlikely choice because the
coupon payments do not comprise 100% of the return of the bond. Thus, the realized rate will be
greater than 8.50%, but less than 10.00%. The only choice that meets this criterion is 8.65%.
For those of you who want to work through the calculations: We first need to calculate the FVcoupon
annuity, then calculate the Total Future Value of the Bond, and finally calculate the realized return.
Step 1: Calculate theFVcoupon annuity

jjoachim_SS15_1_B_e_FVCoupon
Here, FVcoupon annuity= [ (1 + (0.10 / 2)4*2)-1] / [0.10 / 2] * [(1000 * 0.057) / 2]
= [ ( 1.05)8– 1] / 0.05 * 28.5 = 9.549 * 28.5 = $272.15
Alternatively, N = 8, I/Y = 5, PMT = 28.5, PV = 0, Compute FV = -272.15
Step 2: Calculate the Bond’s Total Future Value (TFV)
TFV = Face Value + FVcoupon annuity= $1,000 + $272.15 = $1,272.15.
Step 3: Calculate the Realized Annual Return

jjoachim_SS15_1_B_e_RealizedR
Here, Realized ReturnAnnualized Basis= [ (1,272.15 / 906.70)1/8–1] * 2 = 0.08648, or 8.65%.

Question: 3906

Kwagmyre Investments, Ltd., hold two bonds: a callable bond issued by Mudd Manufacturing Inc.
and a putable bond issued by Precarious Builders. Both bonds have option adjusted spreads (OAS) of
135 basis points (bp). Kevin Grisly, a junior analyst at the firm, makes the following statements (each
statement is independent). Apparently, Grisly could benefit from a CFA review course, because the
only statement that could be CORRECT is:

A. Given a nominal spread for Precarious Builders of 110 bp, the option cost is -25 bp.
B. The cost of the call option on the Mudd bond is -15bp.
C. The Z-spread for Mudd's bond is based on the YTM.
D. The spread over the spot rates for a Treasury security similar to Mudd's bond is 145 bp.

Answer: D

Explanation:
The “spread over the spot rates for a Treasury security similar to Mudd’s bond” refers to the Z-spread
on the bond.For a callable bond, the OAS < Z-spread, so this could be a true statement because

Page | 1539
135bp < 145 bp.
The other statements are false. The option cost is calculated using the OAS and theZ-spread,notthe
nominal spread. The cost of the call option should bepositive.(The issuer has to compensate for
increased uncertainty from the call option.) Thestatic spread (or Z-spread)is the spread over each of
the spot rates in a given Treasury term structure,notthe spreadover the Treasury’s YTM.
Following is a more detailed discussion:
Theoption-adjusted spread(OAS) is used when a bond has embedded options. The OAS can be
thought of as the difference between the static or Z-spread and the option cost. For the exam,
remember the following relationship between the static spread (Z-spread), the OAS, and the
embedded option cost:
Z Spread - OAS = Option Cost in % terms
Remember the following option value relationships:

Question: 3907

Consider an annual coupon bond with the following characteristics:


For a 75 basis point change in interest rates, the bond’s duration is:

A. 8.17 years.
B. 5.09 years.
C. 8.79 years.
D. 5.80 years.

Answer: B

Explanation:
Since the bond has an embedded option, we will use the formula foreffective duration.(This formula
is the same as the formula for modified duration, except that the “upper price bound” is replaced by
the call price.) Thus, we only need to calculate the price if the yieldincreases75 basis points, or
0.75%.
Price if yield increases 0.75%: FV =100, I/Y =7.25= 6.50 + 0.75, N =12, PMT =6.5, Compute PV = 94.12
The formula for effective

jjoachim_SS15_1_C_j_duration
Where:
V-
= Call price/price ceiling

V+
= estimated price if yield increases by a given amount,y

V0
= initial observed bond price

y
= change in required yield, in decimal form
Here,effective duration = (101.75 – 94.12) / (2 * 100 * 0.0075) = 7.63 / 1.5 =5.09 years.

Question: 3908

Collete Minogue holds stock in Bracken Entertainment. Although many of her associates still believe
that Bracken will be a high-performing stock, Minogue has lost faith and wants to conduct a covered

Page | 1540
call transaction. Current market conditions are as follows:
In assessing whether she should conduct the covered call strategy, Minogue sketches out the
following graph. Although her sketch is correct, she cannot remember all the labels.

Which of the following statements about the graph and the covered call strategy is INCORRECT?

A. If Minogue goes ahead with the covered call, she will limit her gain to $11.
B. The distance between points C and D is $5.
C. The call writer will have unlimited upside potential.
D. Line Y represents the covered call's profit line.

Answer: C

Explanation:
The call buyer has unlimited upside potential. If the stock price exceeds $39, the buyer will exercise
the option and will realize all gains (once the cost of the premium is recovered).
The other statements are true. Minogue is the call writer (a covered call consists of the stock and a
short call). Her gain is limited to $11 (the call premium of $5 plus the gain on the stock as long as the
market price is less than the strike price, or $39 - $33). The distance between points C and D
represents the call premium, or $5. Line Y represents the covered call profit line and Line X
represents the stock profit line.

Question: 3909

Which of the following projects would have multiple internal rates of return (IRRs)? The cost of
capital for all projects is 10.0%.

A. Project South only.


B. Projects South and West.
C. Project West only.
D. Projects South and East.

Page | 1541
Answer: A

Explanation:
The multiple IRR problem occurs if a project has non-normal cash flows, that is, the project has
negative (net) cash flows over its life or at the end of its life. In short, the sign of the net cash flows
changes from negative to positive to negative, or vice versa. For the exam, a shortcut to look for is
the project cash flows changing signs more than once. Only Project South has this cash flow pattern.
The 0 net cash flow in T2 for Project West and likely negative net present value (NPV) for Project East
would not necessarily result in multiple IRRs.

Question: 3910

Given the following information on the annual operating results for ArtFrames, a producer of quality
metal picture frames, calculate the degree of operating leverage (DOL) and the degree of financial
leverage (DFL).
Which of the following choices is closest to the correct answer? ArtFrame抯DOL and DFL are,
respectively:

A. 3.00 and 1.50.


B. 2.20 and 1.06.
C. 2.20 and 1.08.
D. 4.53 and 1.19.

Answer: C

Explanation:
The calculations are as follows:

1Variable costs = 0.45 * 3,500,000


2Interest Expense = 0.09 * 750,000
Second,calculate DOL:
DOL = (Sales – Variable Costs) / (Sales – Variable Costs – Fixed Costs)
= (3,500,000 – 1,575,000) / (3,500,000 – 1,575,000 – 1,050,000) =2.20
Third,calculate DFL:
DFL = EBIT / (EBIT – I) = 875,000 / 807,500 =1.08.

Question: 3911

Gerard Rouleau is studying for the Level 2 CFA examination. His friend, Sonia Fennell, is studying for
Level 1. One weekday morning they are sitting at the local Cafe drinking espresso when she asks him
for help with the section on margin trading. Rouleau creates the following scenario: Assume Fennell
purchases 1,000 shares of Xpressoh Inc. for $35 per share. The initial margin requirement is 50

Page | 1542
percent and the maintenance margin is 25 percent One year later, she sells the stock for $42 per
share. Rouleau asks Fennell to calculate the one-year return under two cases: first, assuming an all-
cash transaction,and second, assuming she buys on margin. She is to ignore transaction and
borrowing costs. The earnings retention rate is 100%.
Which of the following choices is closest to the correct answer? The return on an all cash transaction
is:

A. 20.00%, and the return on the margin transaction is 80.00%.


B. 20.00%, and the return on the margin transaction is 40.00%.
C. 40.00%, and the return on the margin transaction is 80.00%.
D. 20.00%, and the return on the margin transaction is -40.00%.

Answer: B

Explanation:
One quick (and less than intensive) way to calculate the answer to this on the examination (and is it
very important to save time on the examination) is to first calculate the return if all cash, then
calculate the margin leverage factor and then finally, multiply the leverage factor times the all cash
return to obtain the margin return.
Calculations:
Step 1: Calculate All Cash Return:
Cash Return % = [(Ending Value / Beginning Equity Position) – 1] * 100
= [(($42 * 1,000) / ($35 * 1,000)) – 1] * 100 = 20%
Step 2: Calculate Leverage Factor:
Leverage Factor = 1 / Initial Margin % = 1 / 0.50 = 2.00
Step 3: Calculate Margin Return:
Margin Transaction Return = All cash return * Leverage Factor = 20% * 2.00 = 40%
Note: You can verify the margin return as follows:
Margin Return % = [((Ending Value - Loan Payoff) / Beginning Equity Position) – 1] * 100
= [(([$42 * 1,000] – [$35 * 1,000 * 0.50]) / ($35 * 0.50 * 1,000)) – 1] * 100 = 40%

Question: 3912

The table below lists information on price per share and shares outstanding for three stocks
?Rocking, Payton, and Strand.

Using the information in the table, calculate the value of a price-weighted index at year-end and the
one- year return on the market weighted index. At the beginning of the year, the value of the market
weighted index was 100. (Note: The choices are listed in the order price-weighted index value,
market value-weighted index percent return, respectively.) Which of the following choices is closest
to the correct answer?

A. 50.0, 10.6.

Page | 1543
B. 50.0, 110.6.
C. -6.3, 10.6.
D. -6.3, 8.4.

Answer: A

Explanation:
Calculations are as follows:

First, we will calculate the value of the price-weighted index at year-end, and then we will calculate
the return on the market-weighted index.
Step 1: Calculate value of the price-weighted index at year-end:
Value of price-weighted index = (sum of stock per share prices) / (number of stocks)
= (15 + 50 + 85) / 3 =50.0
Step 2:Calculate the one-year return on the market-weighted index:
First, we will calculate the year-end market-weighted index value, then we will calculate the return
percentage.
Value of market-weighted index =
[(market capitalizationyear-end) / (market capitalizationbeginning of year)]* Beginning index value
= (442,500 / 400,000) * 100 = 110.625
One-Year Return = [(Index valueyear-end/ Index valuebeginning of year) -1]* 100
= [ (110.625 / 100) – 1] * 100 =10.625, or approximately 10.6%.

Question: 3913

Which of the following statements about bond indexes and international asset indexes is FALSE?

A. Global equity indexes were created to alleviate problems with local indexes (sample selection and
weighting).
B. Low correlation among monthly country indexes supports international diversification.
C. Some of the reasons that a bond index is more difficult to create than an equity index are that the
universe of bonds is less than the universe of stocks and it is difficult to price the bonds because of
the lack of continuous trade data.
D. Investment-grade bond indexes have a higher correlation than high-yield bond indexes.

Answer: C

Page | 1544
Explanation:
This statement contains correct and incorrect points. A bond index is more difficult to create than an
equity index because the universe of bonds is greater than the universe of stocks. The point about
pricing difficulties is correct.
The other statements are true.

Question: 3914

Which of the following choices is NOT a characteristic of an efficient market?

A. Market participants react quickly to news, and these reactions are quickly reflected in prices.
B. A large number of participants value securities independent of other parties.
C. Major news announcements are made on the second Friday of each month.
D. Expected returns implicitly include a risk component.

Answer: C

Explanation:
One assumption of efficient markets is that the timing of a news announcement is independent of
the timing of other news announcements. The other statements are examples of efficient markets.

Question: 3915

An investor is considering investing in Tawari Company for one year. He expects to receive $2 in
dividends over the year and feels he can sell the stock for $30 at the end of the year. What is the
maximum he can pay now to earn at least a 14 percent return on his investment?

A. $28.
B. $29.
C. $30.
D. $32.

Answer: A

Explanation:
HPR = [D + (End - Beginning]/Beginning
14 = [2 + (30 - P)]/P
1.14P = 32 so P = $28.07

Question: 3916

Using the one-year holding period and multiple-year holding period dividend discount model (DDM),
calculate the change in value of the stock of Monster Burger Place under the following scenarios.
First, assume that an investor holds the stock for only one year. Second, assume that the investor
intends to hold the stock for two years. Information on the stock is as follows:
The value of the stock if held for one year and the value if held for two years are, respectively:

A. $27.50 and $35.25.


B. $25.22 and $29.80.
C. $27.50 and $29.80.
D. $29.80 and $32.50.

Answer: B

Page | 1545
Explanation:
First, we need to calculate the required rate of return. When a stock’s beta equals 1, the required
return is equal to the market return, or 10.0%. Thus, ke= 0.10.Alternative:Using the capital asset
pricing model (CAPM), ke= Rf+ Beta * (Rm– Rf) = 4.5% + 1 * (10.0% - 4.5%) = 4.5% + 5.5% = 10.0%.
Next, we need to calculate the dividends for years 1 and 2.
Then, we use the one-year holding period DDM to calculate the present value of the expected stock
cash flows (assuming the one-year hold).
Finally, we use the multi-period DDM to calculate the return for the stock if held for two years.
P0= [D1/ (1 + ke)] + [D2/ (1 + ke)2]+[P2/ (1 + ke)2] = [$2.75 / (1.10)] + [$3.03 / (1.10)2] + [$30.0 /
(1.10)2] =$29.80.Note:since the growth rate in dividends, g, was equal to ke, the present value of
next year’s dividend is equal to last year’s dividend (for periods 1 and 2). Thus, a quick calculation
would be 2.5 * 2 + $30.00 / (1.10)2= 29.80.

Question: 3917

Following is a graph of the Industry Life Cycle with the names of the phases omitted.

Using the graph above, which of the following choices is CORRECT?

A. To value a firm in Phase B, an analyst should use the infinite period dividend discount model
(DDM).
B. Competition most likely intensifies in Phase B.
C. During Phase E, industry growth rates approach that of the economy.
D. The P/E ratio for a company in Phase D is approximated by 1/ke.

Answer: D

Explanation:
Phase D represents the Stabilization and Market Maturity Phase. Corporate finance shows that a
mature company has a return on equity (ROE) approximately equal to the required rate of return, ke.
Then, the formula:
P/E = (dividend payout) / (ke– retention rate * ROE) becomes
= (1 – retention rate) / (ke– retention rate * ke) = 1/ke
The other statements are incorrect. The infinite period dividend discount model is most
likelynotappropriate for a firm in Phase B, which represents the rapid accelerating growth phase (the
multi-stage model is best). Remember that the infinite period DDM is most useful for a company
with the following assumptions:
Phase B, the Rapid Accelerating Growth phase, is characterized bylimitedcompetition and very high
sales and profit margin growth. Companies in this phase likely have supernormal growth.

Page | 1546
Competition increases in Phase C, the mature growth phase. The industry growth rates approach that
of the economy in Phase D, Stabilization and Market Maturity. Phase E is the Deceleration of Growth
and Decline phase. During this phase, demand shifts away from the industry and growth of substitute
products may cause growth rates to be negative.

Question: 3918

Given the following assumptions about a company抯financial estimates, calculate the P/E ratio, and
determine whether the stock is undervalued or overvalued.
Which of the following statements is most correct? The P/E ratio is:

A. 7.41 and the stock is overpriced.


B. 7.41 and the stock is underpriced.
C. 6.78 and the stock is overpriced.
D. 6.78 and the stock is underpriced.

Answer: A

Explanation:
First, determine the value of the stock. Then, compare your calculation to the current market price.
The assumptions lead us to use the P/E (earnings multiplier) and EPS to calculate the value.
Step 1: Estimate the P/E ratio
Step 2: Estimate Earnings Per Share (given at $2.75)
Step 3: Calculate value
Step 4: Determine under/over valuation
Since the market value is greater than the estimated value, the stock is overpriced.

Question: 3919

Carlita Jefferson works in the treasury department of a manufacturing firm. The assistant treasurer e-
mails her the following message:
揟he new CFO has requested some information from our department. She wants our group to
recommend whether or not to purchase a target firm. Calculate the estimated share price using free
cash flow to equity (FCFE) (for three years) and compare the result to the current market price. I
referred this to you because I know you are studying for the Level 1 CFA examination, and I think this
calculation is new material. Here抯the latest estimates from our due diligence team. Don抰know if
you will need all of it. By the way, I need this ASAP. And be brief. I only want your conclusion.?/p>
Target Financial Information
Which of the following responses should Jefferson e-mail to the assistant treasurer?

A. Do not purchase target. Valuation using FCFE of $44.56 per share is less than market price.
B. Purchase target. Valuation using FCFE of $51.34 per share exceeds market price.
C. Do not purchase target. Valuation using FCFE of $43.29 per share is less than market price.
D. Inconclusive. Valuation using FCFE of $48.65 per share too close to market price.

Answer: A

Explanation:
The FCFE is calculated as follows (all amounts in million of $ except per share price):
Step 1: Calculate each year’s FCFE
Step 2: Calculate Present Value of Each Year’s FCFE
Step 3: Calculate Present Value of final cash flow times FCFE multiple
Step 4: Calculate per share value
Step 5: Make Decision

Page | 1547
Question: 3920

An investor buys a 15-year, 10 percent annual pay coupon bond for $1,000. He plans to hold the bond
for 5 years while reinvesting the coupons at 12 percent. At the end of the 5-year period he feels he
can sell the bond to yield 9 percent. What is the expected realized (horizon) yield?

A. 10.0%.
B. 11.8%.
C. 11.2%.
D. 12.2%.

Answer: C

Explanation:
The key to this problem is to find all future cash flows.N = 5, I/Y = 12, PMT = 100, PV = 0CPT FV =
635.28 = VALUE OF COUPONSN = 10, I/Y = 9, PMT = 100, FV= 1000CPT PV = 1,064.18 = VALUE OF
BOND 5 YEARS OUT
Realized Return =

Question-LI-SS14-LOS1-
A-a-answer-1

Question-LI-SS14-LOS1-A-a-answer-2

Question: 3921

All other things being equal, which one of the following bonds has the greatest duration?

A. 15-year, 8% coupon bond.


B. 5-year, 8% coupon bond.
C. 15-year, 12% coupon bond.
D. 5-year, 12% coupon bond.

Answer: A

Explanation:
If bonds are identical except for maturity and coupon, the one with the longest maturity and lowest
coupon will have the greatest duration. The rationale for this is similar to that for price volatility.
Duration is approximately equal to the point in years where the investor receives half of the present

Page | 1548
value of the bond's cash flows. Therefore, the later the cash flows are received, the greater the
duration.
The relationship of maturity to duration is direct - the longer the time to maturity, the greater the
duration. A longer-term bond pays its cash flows later than a shorter-term bond, increasing the
duration. Here, one of the 15-year bonds will have the greatest duration.
The relationship of coupon to duration is indirect - the lower the coupon rate, the greater the
duration. A lower coupon bond pays lower annual cash flows than a higher-coupon bond and thus
has less influence on duration. Here, the 15-year bond with the lowest coupon (8.00%) will have the
greatest duration.

Question: 3922

If a bond has a modified duration of 7 and convexity of 100 and interest rates fall 1 percent, what will
happen to the price of the bond?

A. Fall 7.5%.
B. Rise 6.5%.
C. Rise 7.5%.
D. Fall 6.5%.

Answer: C

Explanation:
∆V = -(7) (.01) + 1/2 (100) (.01)2.

Question: 3923

Calculating the price change in a bond caused by a 1 percent decline in interest rates using only the
modified duration equation will always result in an answer that is:

A. too high.
B. just right.
C. too low.
D. insignificant.

Answer: C

Question: 3924

Which of the following statements regarding daily cash settlement before contract maturity is TRUE?

A. A futures contract requires daily cash settlement, and a forward contract does not require daily
cash settlement.
B. Neither a futures contract nor a forward contract requires cash settlement before contract
maturity.
C. A forward contract requires cash settlement and a futures contract does not require cash
settlement.
D. Both a futures contract and a forward contract require cash settlement before contract maturity.

Answer: A

Question: 3925

Which of the following statements about the positions of the clearinghouse is CORRECT? The

Page | 1549
clearinghouse:

A. takes no position.
B. only takes short position to buyers.
C. only takes long positions to sellers.
D. takes short positions to buyers and long positions to sellers.

Answer: D

Question: 3926

The buyer of a call option has the:

A. obligation to sell the underlying asset in the future under certain conditions.
B. right to sell the underlying asset in the future under certain conditions.
C. obligation to buy the underlying asset in the future under certain conditions.
D. right to buy the underlying asset in the future under certain conditions.

Answer: D

Question: 3927

A real estate analysis estimates the market value of an income-producing property at $2,560,000.
The annual gross potential rental income is $596,000, the annual property operating expanses and
taxes are $178,800, and the annual vacancy and collection losses are $89,400. What capitalization
rate was used by the analysis to assess the property at $2,560,000.

A. 0.128.
B. 0.1275.
C. 0.129.
D. 0.127.

Answer: A

Explanation:

Question: 3928

Based upon the following information, what is the net operating income of the property?

Page | 1550
A. $104,000.
B. $98,600.
C. $114,600.
D. $120,000.

Answer: D

Explanation:

Question: 3929

What is the net operating income (NOI) of this property?

A. $113,124.
B. $108,599.
C. $92,886.
D. $97,410.

Answer: C

Explanation:
108,660 - 15,714 = 92,886

Question: 3930

What are the earnings before interest and taxes (EBIT) of the building?

A. $79,563.
B. $52,919.
C. $95,277.
D. $59,580.

Answer: A

Explanation:

Page | 1551
92,886 - 13,322 = 79,564

Question: 3931

What are the earnings before taxes (EBT)?

A. $52,919.
B. $66,241.
C. $62,629.
D. $59,580.

Answer: A

Explanation:
79,564 - 26,664 = 52,919

Question: 3932

What is the building's net income?

A. $42,646.
B. $49,307.
C. $35,985.
D. $29,324.

Answer: C

Explanation:
52,919 x .68 = 35,985

Question: 3933

What is the after tax cash flow of the investment?

A. $52,919.
B. $35,985.
C. $26,275.
D. $42,646.

Answer: D

Explanation:
NI + DEP EXP - PRIN PMT =
35,985 + 13,322 - 6,661 = 42,646

Question: 3934

The net asset value (NAV) of this fund would be:

Page | 1552
A. $17.30.
B. $26.00.
C. $23.12.
D. $15.80.

Answer: A

Explanation:
(Total Market Value/Total Number of Shares)=(432,400/25,000)=17.296=17.30

Question: 3935

If you are going to invest in a closed-end mutual fund and were told that the net asset value of the
fund is $11.20, and the share price was $11.80. What is the discount you would receive or the
premium that you would pay?

A. -0.0508.
B. 0.0508.
C. 0.0536.
D. -0.0536.

Answer: C

Explanation:
(SP-NAV)/NAV=
(11.80-11.20)/11.20=0.05357

Question: 3936

An individual investor approaches you and asks, "If I were to purchase a fund with a load of 6
percent, and I used $6,200 to purchase the fund, what dollar amount would the shares purchased
be?"

A. $6,200.
B. $5,828.
C. $6,572.
D. $372.

Answer: B

Page | 1553
Explanation:
6,200 x .94 = 5,828

Question: 3937

Jacobi Lefebre, CFA, recently accepted a position in the Real Estate Strategy group of a large retail
company. The Director of the group walks by Lefebre’s cubicle, hands him a report with the following
information, and asks Lefebre to determine how large a project the firm can undertake without
increasing the marginal cost of capital.
The firm can undertake a project costing up to approximately:

A. $54 million.
B. $81 million.
C. $83 million.
D. $29 million.

Answer: A

Explanation:
This question is asking you to calculate the retained earnings breakeven point BPRE(in relation to the
marginal cost of capital). Once the breakeven point is reached, the firm will have to use external
equity to fund projects, thus increasing the weighted average cost of capital (WACC).
The break point on the marginal cost curve is given by:
BPRE= (Retained Earnings) / (Equity Fraction, we)
Here, RE = earnings * (1 – Payout) = $50 million * (1 – 0.35) = $ 32.5 million,
And BPRE= $32.5 million / 0.60 = $54.17 million, or approximately $54 million.

Question: 3938

Helmut Humm, manager at a large U.S. firm, has just been assigned to the capital budgeting area to
replace a person who left suddenly. One of Humm’s first tasks is to calculate the company’s weighted
average cost of capital (WACC) – and fast! The CEO is scheduled to present to the board in half an
hour and needs the WACC – now! Luckily, Humm finds clear notes on the Target capital component
weights in the current workpapers. Unfortunately, all he can find for the cost of capital components
is some handwritten notes. He can make out the numbers, but not the corresponding capital
component. As time runs out, he has to guess.
Here is what Humm deciphered:
If Humm guesses correctly, the WACC is:

A. 10.1%.
B. 10.4%.
C. 9.7%.
D. 11.0%.

Answer: B

Explanation:
If Humm remembers to order the capital components from cheapest to most expensive, he can
calculate WACC. The order from cheapest to most expensive is: debt, preferred stock (which acts like
a hybrid of debt and equity), retained earnings, and common stock. (Remember that internal equity
– retained earnings is cheaper than external equity –common equity due to floatation costs.)
Then, using the formula for WACC = (wd)(kd) + (wps)(kps) + (ws)(ks)+ (we)(ke)
where wd, wps, wsandweare the weights used for debt, preferred stock, retained earnings, and

Page | 1554
common equity.
WACC = (0.30 * 6.0%) + (0.20 * 8.5%) + (0.15 * 11.00%) + (0.35 * 15.0%) =10.4%.

Question: 3939

Dana Leone, Level 2 candidate in the CFA Program, is a manager in the capital planning division of a
large consumer products company. Today, she must decide whether to recommend Project Forrest or
Project Trieste to the Division Director. Leone’s assistant has determined the following information
about the projects (which are of approximately the same size and life):

Note:NPV = Net Present Value


MIRR = Modified Internal Rate of Return
PBP = Pay Back Period
WACC = Weighted Average Cost of Capital
Based on the information in the above table, which of the following statements is FALSE? If company
management is most concerned with:

A. increasing the value of the firm, Leone should recommend Project Forrest.
B. liquidity, Leone should recommend Project Trieste.
C. return safety margin, Leone should recommend Project Forrest.
D. shareholder interests, Leone should recommend Project Trieste.

Answer: A

Explanation:
Since Leone can recommend either Forrest or Trieste, this question should be answered using the
decision rules for mutually exclusive projects. For a company most interested in shareholder wealth
and increasing the value of the firm (which are the same concept), Project Trieste is best because it
has the higher NPV. NPV is the measure that indicates the direct impact on shareholder wealth. The
other statements are true. The PBP measures liquidity, i.e., the shorter the PBP, the higher the
liquidity.The IRR method does provide a measure of safety margin (and Project Forrest does have the
highest IRR).

Question: 3940

Annah Korotkin is the sole proprietor of CoverMeUp, a business that designs and sews outdoor
clothing for dogs. Each year, she rents a booth at the regional Pet Expo and sells only blankets.
Korotkin views the Expo as primarily a marketing tool and is happy to breakeven (that is, cover her
booth rental). For the last 3 years, she has sold exactly enough blankets to cover the $750 booth
rental fee. This year, she decided to make all blankets for the Expo out of high-tech
waterproof/breathable material that is more expensive to produce, but that she believes she can sell
for a higher profit margin. Information on the two types of blankets is as follows:

Page | 1555
Assuming that Korotkin remains most interested in covering the booth cost (which has increased to
$840), how many more or fewer blankets (new style) does she need to sell to cover the booth cost?
To cover this year’s booth costs, Korotkin needs to sell:

A. 42 more blankets than last year.


B. 42 fewer blankets than last year.
C. 30 fewer blankets than last year.
D. the same amount of blankets as last year.

Answer: C

Explanation:
To obtain this result, we need to calculate Last Year’s Breakeven Quantity, This Year’s Breakeven
Quantity, and calculate the difference.
Step 1: Determine Last Year’s (Basic Blanket) breakeven quantity:
QBE= (Fixed Costs) / (Sales Price per unit – Variable Cost per unit) = 750 / (25 – 20) = 150
Step 2: Determine This Year’s (New Blanket) breakeven quantity:
QBE= (Fixed Costs) / (Sales Price per unit – Variable Cost per unit) = 840 / (40 – 33) = 120
Step 3: Determine Change in Units:
Q = QThis Year– QLast Year= 120 – 150 = -30. Korotkin needs to sell 30fewerblankets.

Question: 3941

Using the following assumptions, calculate the rate of return on a margin transaction and the stock
price at which the investor who purchases the stock will receive a margin call.
What of the following choices is closest to the correct answer? The margin transaction return is:

A. 83.33%, and the investor will receive a margin call at a stock price of $15.43.
B. 33.33%, and the investor will receive a margin call at a stock price of $15.43.
C. 111.11%, and the investor will receive a margin call at a stock price of $21.00.
D. 111.11%, and the investor will receive a margin call at a stock price of $15.43.

Answer: A

Explanation:
To obtain the result:
Part 1: Calculate Margin Return:
Margin Return % = [((Ending Value - Loan Payoff) / Beginning Equity Position) – 1] * 100 =
= [(([$24 * 1,000] – [$18 * 1,000 * 0.60]) / ($18 * 0.40 * 1,000)) – 1] * 100 =
= 83.33%
Alternative (Check): Calculate the all cash return and multiply by the margin leverage factor.
= [(24,000 – 18,000)/18,000] * [1 / 0.40] = 33.33% * 2.5 = 83.33%
Part 2: Calculate Margin Call Price:
Since the investor is long (purchased the stock), the formula for the margin call price is:
Margin Call = (original price) * (1 – initial margin) / (1 – maintenance margin)
= $18 * (1 – 0.40) / (1 – 0.30) = $15.43

Question: 3942

Page | 1556
The table below lists information on price per share and shares outstanding for three stocks –
Rocking, Payton, and Strand.

Using the information in the table, calculate the value of a market-value weighted index at year-end
and the one-year return on the price-weighted index. The beginning value for the market index is
100. (Note: The choices are listed in the order market-value weighted index value and price-
weighted index percent return, respectively). Which of the following choices is closest to the correct
answer?

A. 10.6, 6.3.
B. 110.6, -6.3.
C. 110.6, 50.0.
D. 10.6, 8.4.

Answer: B

Explanation:
Calculations are as follows:

First, we will calculate the value of the market-value weighted index at year-end, and then we will
calculate the return on the price-weighted index.
Step 1: Calculate value of the market-weighted index at year-end:
Value of market-weighted index =
[(market capitalizationyear-end) / (market capitalizationbeginning of year)]* Beginning index value
= (442,500 / 400,000) * 100 = 110.625, or approximately 110.6

Page | 1557
Step 2:Calculate the one-year return on the price-weighted index:
First, we will calculate the price-weighted index value for both the beginning of year and end of year,
then we will calculate the return percentage.
Value of price-weighted indexbeginning= (sum of stock per share pricesbeginning) / (number of
stocksbeginning)
= (10 + 50 + 100) / 3 =53.333
Value of price-weighted indexend= (sum of stock per share pricesend) / (number of stocksend)
= (15 + 50 + 85) / 3 =50.0
One-Year Return = [(Index valueyear-end/ Index valuebeginning of year) -1]* 100
= [ 50.0 / 53.333) – 1] * 100 =-6.3%
Note:Your calculation may differ slightly due to rounding. Remember that the question asks you to
select theclosestchoice.

Question: 3943

Caleb Gold is studying for the Level 1 CFA examination with a fellow group of first year MBA students
at the London School of Economics. During that night’s study session, Stephan LeMond, the self-
proclaimed group “leader,” gives a short presentation on the forms of the efficient market hypothesis
(EMH). As Gold listens, he hears LeMond make an obviously incorrect statement. He quickly speaks
up, and identifies which of the following statements as INCORRECT?

A. The weak-form EMH states that stock prices reflect current public market information and
expectations.
B. The semi-strong form EMH addresses market and non-market public information.
C. The strong-form EMH assumes perfect markets.
D. The weak-form EMH suggests that technical analysis will not provide excess returns while the
semi-strong form suggests that fundamental analysis cannot achieve excess returns.

Answer: A

Explanation:
The weak-form EMH assumes the price of a security reflects all currently available historical
information. Thus, the past price and volume of trading has no relationship with the future, hence
technical analysis is not useful in achieving superior returns.
The other statements are true. The strong-form EMH states that stock prices reflect all types of
information: market, non-public market, and private. No group has monopolistic access to relevant
information; thus no group can achieve excess returns. For these assumptions to hold, the strong-
form assumes perfect markets – information is free and available to all.

Question: 3944

Tamira Scott, CFA, manager of an index fund, needs to raise money soon (although not immediately)
to pay taxes. Although she believes in the efficient market hypothesis (EMH), she remembers that
there are a few anomalies she may take advantage of to earn higher returns. Which of the following
actions is most unlikely to provide excess returns? Scott should purchase stocks in:

A. companies with low price/earnings ratios and/or with high book to market ratios.
B. companies that announce stock splits.
C. companies not followed by analysts.
D. mid-December, with the intent to sell in early January.

Answer: B

Explanation:

Page | 1558
According to event studies of the semi-strong form of the EMH, stock splits do not have a short run
or long run impact on returns. This finding supports the EMH.
The other choices are considered anomalies. That is, they reject the semi-strong form of the EMH,
and suggest that investors can earn excess returns by exploiting these anomalies. The January
anomaly (from a time-series test of the semi-strong EMH) suggests that investors can earn excess
returns by buying stocks in December and selling them in the first week of January, due to tax-
induced trading at year-end. Cross-sectional tests of the semi-strong form EMH have shown that low
P/E stocks, stocks of firms neglected by analysts, and stocks of firms with high book to value ratios
can produce superior returns.

Question: 3945

Given the following estimated financial results, value the stock of Magic Holdings, Inc. using the
infinite period dividend discount model (DDM).
Which of the following choices is closest to the value of Magic Holding Inc. stock? (Note: Carry
calculations out to at least 3 decimals.)

A. $44.64.
B. $23.54.
C. Unable to calculate stock value because ke < g.
D. $109.27.

Answer: D

Explanation:
Here, we are given all the inputs we need. Use the following steps to calculate the value of the stock:
First, expand the infinite period DDM:
DDMformula: P0= D1/ (ke– g)
D1
= (Earnings * Payout ratio) / average number of shares outstanding

= ($200,000 * 0.625) / 50,000 = $2.50.

ke
= nominal risk free rate + [beta * (expected market return – nominal risk free rate)]
Note:Nominal risk-free rate = (1 + real risk free rate) * (1 + expected inflation) – 1

=(1.035)*(1.040) – 1 = 0.0764, or 7.64%.


ke
= 7.64% + [1.8 * (13.0% - 7.64%)] = 0.17288.

g
= (retention rate * ROE)

Retention = (1 – Payout) = 1 – 0.625 = 0.375.

ROE = (net income/sales)*(sales/total assets)*(total assets/equity)

= (200,000/1,000,000)*(1,000,000/750,000)*(750,000/500,000) = 0.40
g
= 0.375 * 0.40 = 0.15.
Then, calculate:P0= D1/ (ke– g) = $2.50 / (0.17288 - 0.15) = 109.27.

Question: 3946

Page | 1559
Ted McGovern works in the economics branch of a government bank regulator. When he arrives at
work this morning and checks his voicemail, he has a message from the Regional Director asking him
to calculate the expected rate of return for a stock market series. More detailed information will be
forthcoming in an e-mail. Fortunately, McGovern still has his CFA Program study guides in his office
and finds the correct formulas. McGovern logs on to the computer network and downloads an
attachment that contains the following estimates:
Overall Assumptions:
Index Estimates – Bull Market:
Index Estimates – Bear Market:
The expected return on the index is closest to:

A. 67.4%.
B. 39.4%.
C. 30.8%.
D. 98.2%.

Answer: C

Explanation:
To calculate the expected index return, we need to calculate the expected return for the bull market
and the expected return for the bear market. Then, we will use the given probabilities of each
scenario to calculate the expected index return.Note: All amounts are in millions of $ unless noted
otherwise.
BULL MARKET
BEAR MARKET
EXPECTED VALUE OF INDEX
= 0.35 * 110% + 0.65 * -11.8% =30.83%

Question: 3947

Daniel Tipton and Jesse Torrez are first-year MBA students at the Haas School of Business. Torrez has
an economics background, but Tipton’s background is in music. To help Tipton study one of the main
tenets of competition theory, Torrez creates the following question and asks Tipton to identify the
statement that is most inconsistent with Porter’s five forces. Which statement should Tipton select?

A. Supplier power is higher when there are only a few suppliers to an industry.
B. To sustain above average returns on invested capital, firms should strive for economies of scale.
C. Porter's five forces are: rivalry among current competitors, economies of scale, threat of
substitutes, bargaining power of suppliers, and bargaining power of buyers.
D. Rivalry increases when firms of equal size compete within an industry.

Answer: C

Explanation:
Porter’s five forces are: rivalry among current competitors, threat of new entrants, threat of
substitutes, bargaining power of suppliers, and bargaining power of buyers. Economies of scale are a
way to lessen the threat of new entrants, but are not the only way to discourage competition.
Companies can also have barriers to entry such as regulation or high start up capital. The other
choices are true.

Question: 3948

Consider the following information for Magical Interactions, Inc.

Page | 1560
Based on the assumptions above, which of the following statements is TRUE?

A. The stock is undervalued.


B. If the earnings retention rate increases, the value of the stock will increase (all else equal).
C. If management can increase the EBITDA ratio by only 1.0%, the stock will be properly priced (all
else equal).
D. If inflation expectations decrease, the value of the stock will increase (all else equal).

Answer: D

Explanation:
The expected inflation rateis a component of ke(through the nominal risk free rate). keis one
component of the P/E ratio and can be represented by the following: nominal risk free rate + stock
risk premium, where nominal risk free rate = [(1 + real risk free rate) * (1 + expected inflation rate)] –
1.
The other statements are false. To determine the stock over/under valuation, we need to calculate
both the P/E ratio and the EPS.
The P/E ratio = Dividend Payout Ratio / (ke– g),
EPS = [(Per share Sales Estimate) * (EBITDA%) – D (per share) – I (per share)] * (1 - t)
= [($150 * 0.18) - $15 - $10] * (1 – 0.35) = $1.30
Value of stock = EPS * P/E = 7.14 * $1.30 =$9.30
Since the market value of the stock is greater than the estimated value, the stock is overvalued.
An increase in earnings retentionwill likelydecreasethe P/E ratio. The logic is as follows: Because
earnings retention impacts both the numerator (dividend payout) and denominator (g) of the P/E
ratio, the impact of a change in earnings retention depends upon the relationship of keand ROE. If
the company is earning a lower rate on new projects than the rate required by the market (ROE < ke),
investors will likely prefer that the company pay dividends (absent tax concerns). Investors will likely
value the company lower if it retains a higher percentage of earnings.
If management increases EBITDA by 1.0%,the stock will beundervalued.
EPS = [($150 * 0.19) - $15 - $10] * (1 – 0.35) = $2.28
Value of stock = EPS * P/E = 7.14 * $2.28 = approximately$16.30, which is greater than the market
value.
Note: the EBITDA % that equates to the market price is approximately 18.5%, or a 0.5% increase.
Small changes in EBITDA% have a large impact on the EPS and thus on the estimated stock value.

Question: 3949

Assume an investor makes the following investments:


During year one, the stock paid a $5.00 per share dividend. In year 2, the stock paid a $7.50 per share
dividend. The investor’s required return is 35.0 percent.
The dollar-weighted return is:

A. 48.9%.
B. 16.1%.
C. 46.5%.
D. 102.4%.

Answer: A

Explanation:
To calculate the dollar-weighted return:
Step 1: Determine the timing and sign (inflow, outflow) of the cash flows
Purchase share 2, $75.00 outflow
Received dividend from share 2, $7.50 inflow

Page | 1561
Sell share 1, $100.00 inflow,
Sell share 2, $100.00 inflow.
Step 2: Calculate the net cash flows for each year (all amounts in $)
Step 3: Use your financial calculator to solve for IRR (or use trial and error)

Question: 3950

Duration of a bond normally increases with an increase in:

A. time to maturity.
B. coupon rate.
C. yield to maturity.
D. par value.

Answer: A

Explanation:
Duration is directly related to maturity and inversely related to the coupon rate and yield to maturity
(YTM). Duration is approximately equal to the point in years where the investor receives halfof the
present value of the bond's cash flows. Therefore, the later the cash flows are received, the greater
the duration.
The longer the time to maturity, the greater the duration (and vice versa). A longer-term bond pays
its cash flows later than a shorter-term bond, increasing the duration. The lower the coupon rate, the
greater the duration (and vice versa). A lower coupon bond pays lower annual cash flows than a
higher-coupon bond and thus has less influence on duration. The lower the YTM, the higher the
duration. This is because the bond's price (or present value) is inversely related to interest rates.
When market yields fall, the value (or cash flow) of a bond increases without increasing the time to
maturity.

Page | 1562
Consider the purchase of an existing bond selling for $1,150. This bond has 28 years to maturity, pays
a 12 percent annual coupon, and is callable in 8 years for $1,100.

Question: 3951

What is the bond's yield to call (YTC)?

A. 10.05%.
B. 9.26%.
C. 10.34%.
D. 10.55%.

Answer: A

Explanation:
N = 8, PMT = 120, PV = -1150, FV = 1100, CPT I/Y.

Question: 3952

What is the bond's yield to maturity (YTM)?

A. 9.26%.
B. 10.34%.
C. 10.05%.
D. 10.55%.

Answer: B

Explanation:
N = 28, PMT = 120, PV = -1150, FV = 1000, CPT I/Y.

Question: 3953

What rate should be used to estimate the potential return on this bond?

A. the YTM.
B. 12.00%.
C. 10.34%.
D. the YTC.

Answer: D

Question: 3954

An investor buys a 25-year, 10 percent annual pay bond for $900 planning to sell the bond in 5 years
when he estimates yields will be 9 percent. What is the estimate of the future price of this bond?

A. $964.
B. $1,000.
C. $1,122.
D. $1,091.

Answer: D

Page | 1563
Explanation:
This is a Present Value problem 5 years in the future. Input into your calculator:
N = 20, PMT = 100, FV = 1000, I/Y = 9CPT PV = 1,091.28The $900 purchase price is a distracter for this
problem.

Question: 3955

A trader has a long position in a wheat contract.


What is the price at which the trader will receive a maintenance margin call?

A. $1.90.
B. $2.05.
C. $2.25.
D. $1.75.

Answer: D

Explanation:
The trader would have to lose $1,250 or 5,000-3,750 before they get a margin call. 5,000(2.00-P) =
1,250. P = $1.75.

Question: 3956

Which of the following statements is TRUE about the profits and losses from buying a put:

A. potential losses are limited to the initial premium the buyer pays when he buys the put.
B. potential profits are theoretically unlimited.
C. potential losses are theoretically unlimited.
D. none of these choices are correct.

Answer: A

Question: 3957

XYZ company has entered into a "plain-vanilla" interest rate swap on $1,000,000 notional principal.
XYZ company pays a fixed rate of 8 percent on payments that occur at 90-day intervals. Six payments
remain with the next one due in exactly 90 days. On the other side of the swap, XYZ company
receives payments based on the LIBOR rate. Describe the transaction between XYZ company and the
dealer at the end of the sixth period if the appropriate LIBOR rate is 5 percent.

A. XYZ company receives $12,500.


B. Dealer pays XYZ company $7,500.
C. Dealer receives $20,000.
D. XYZ company pays dealer $7,500.

Answer: D

Explanation:
XYZ company owes the dealer ($1,000,000)(.08)(90/360) = $20,000. The dealer owes XYZ company
($1,000,000)(.05)(90/360) = $12,500. Net: XYZ company pays the dealer $7,500.

Question: 3958

Page | 1564
If a new option contract is listed on an option exchange and only one trader buys one contract in the
first day, the open interest after that day is:

A. two contract.
B. zero contract.
C. cannot be determined with this information.
D. one contract.

Answer: D

Question: 3959

A property has a potential gross rental income (PGRI) of $740,000. Operating expenses, excluding
insurance and property taxes, amount to 30 percent of gross rents. Insurance and property taxes
total $16,800. If the market capitalization rate is 22 percent, what is the value of this property?

A. $1,726,667.
B. $2,410,667.
C. $2,430,909.
D. $2,278,182.

Answer: D

Explanation:

Question: 3960

You need to estimate the market value of an income producing property located in your town.
Through research you have found that the property should have net operating income of $956,000,
taxes of $143,400, a capitalization rate of 16 percent, and an inflation rate of 3 percent. What is the
estimated property value?

A. $5,078,750.
B. $5,975,000.
C. $7,353,846.
D. $8,456,923.

Answer: B

Explanation:

Page | 1565
Question: 3961

Based on the following information, what is the net asset value (NAV) per share. There are currently
no expenses and no load.
Cap Stock Sold $109,000
Price per share $10

A. $13.26.
B. $27.03.
C. $15.96.
D. $27.03.

Answer: C

Explanation:

Question: 3962

You are going to invest in a closed-end mutual fund and are told that the net asset value of the fund
is $20.40, and the share price is $18.20. What is the discount you would receive or the premium that
you would pay?

A. 0.1209.
B. -0.1209.

Page | 1566
C. -0.1078.
D. 0.1078.

Answer: C

Explanation:

Question: 3963

A fund to be purchased had a net asset value (NAV) of $72.40 and a load of 5 percent. What is the
offering price per share?

A. $76.211.
B. $3.811.
C. $76.020.
D. $72.400.

Answer: A

Explanation:

Question: 3964

A mutual fund has a load of 4 percent and a net asset value (NAV) of $20 per share. What must an
investor pay to purchase 250 shares?

A. $5,200.
B. $4,800.
C. $5,013.
D. $5,208.

Answer: D

Page | 1567
Thank You for Purchasing CFA-LEVEL-I PDF

Test Your Preparation


Use Coupon “CD2016” for extra 15% discount on Practice Test
Software.

Prepare with PDF and examine your preparation with our multiple testing
modes practice test software.

Download Free Practice Test Demo Here:

https://www.testcollections.com/CFA-LEVEL-I.html

Page | 1568

You might also like